Você está na página 1de 442

Tribunal de Justiça do

Estado de São Paulo - TJ/SP


Assistente Social Judiciário

Língua Portuguesa
1. ortografia oficial; ..........................................................................................................................................................1
2. conjugação de verbos; .................................................................................................................................................5
3. flexão de gênero, número e grau; ........................................................................................................................... 10
4. regência e concordância; .......................................................................................................................................... 15
5. emprego de pronomes e crases; ............................................................................................................................. 21
6. formas de tratamento; .............................................................................................................................................. 28
7. pontuação; .................................................................................................................................................................. 29
8. figuras de sintaxe; ..................................................................................................................................................... 31
9. análise sintática; 10. orações e seus termos; 11. coordenação e subordinação; ............................................ 34
12. acentuação; .............................................................................................................................................................. 45
13. colocação pronominal e verbal; ............................................................................................................................ 46
14. emprego de preposição; ......................................................................................................................................... 47
15. conjunção e sinonímia; ........................................................................................................................................... 47
16. morfo-sintaxe. .......................................................................................................................................................... 51

Atualidades e Deveres dos Servidores Públicos


1. Questões relacionadas a fatos políticos, econômicos e sociais e culturais, nacionais e internacionais,
ocorridos a partir do 2º semestre de 2016, divulgados na mídia local e/ou nacional..........................................1
2. Estatuto dos Funcionários Públicos Civis do Estado de São Paulo (Lei n.º 10.261/68) - artigos 239 a 250;
com as alterações vigentes até a publicação deste Edital. ...................................................................................... 33
3. Lei Federal nº 8.429/92 (Lei de Improbidade Administrativa) artigos 1º ao 11º – com as alterações
vigentes até a publicação do Edital. ............................................................................................................................ 35

Informática
MS-Windows 10: conceito de pastas, diretórios, arquivos e atalhos, área de trabalho, área de transferência,
manipulação de arquivos e pastas, uso dos menus, programas e aplicativos, interação com o conjunto de
aplicativos MS-Office 2016, ............................................................................................................................................1
MS-Word 2016: estrutura básica dos documentos, edição e formatação de textos, cabeçalhos, parágrafos,
fontes, colunas, marcadores simbólicos e numéricos, tabelas, impressão, controle de quebras e numeração
de páginas, legendas, índices, inserção de objetos, campos predefinidos, caixas de texto. ............................. 10
MSExcel 2016: estrutura básica das planilhas, conceitos de células, linhas, colunas, pastas e gráficos,
elaboração de tabelas e gráficos, uso de fórmulas, funções e macros, impressão, inserção de objetos, campos
predefinidos, controle de quebras e numeração de páginas, obtenção de dados externos, classificação de
dados. ............................................................................................................................................................................... 18
Correio Eletrônico: uso de correio eletrônico, preparo e envio de mensagens, anexação de arquivos. ........ 28
Internet: navegação internet, conceitos de URL, links, sites, busca e impressão de páginas. .......................... 30

Apostila Digital Licenciada para Alice Caroline Guarino dos Santos - alice.guarino@hotmail.com (Proibida a Revenda)
Conhecimentos Específicos
1. fundamentos teórico-metodológicos, ético-políticos, técnico-operativos do Serviço Social; .........................1
2. políticas Públicas e Serviço Social; ............................................................................................................................7
3. seguridade Social e Assistência Social; .................................................................................................................. 17
4. prática profissional x prática social x prática institucional x avaliações; ........................................................ 21
5. laudos, relatórios e pareceres; ................................................................................................................................ 27
6. análise e fundamentação das relações sociais no âmbito das instituições e dos movimentos populares; 28
7. Serviço Social e interdisciplinaridade; .................................................................................................................. 30
8. Direitos Humanos; ..................................................................................................................................................... 32
9. Famílias - transformações e configurações; ......................................................................................................... 89
10. Relações de Gênero e Intergeracionais; .............................................................................................................. 93
11. casamento e separação; 12. Noções de Direito de Família e Sucessões sobre as áreas de atuação de Serviço
Social; ............................................................................................................................................................................... 97
13. direitos fundamentais da criança e do adolescente; ......................................................................................... 97
14. medidas específicas de proteção à criança e ao adolescente; ......................................................................... 99
15. colocação em família substituta - guarda-tutela-adoção; ..............................................................................101
16. Convivência Familiar e Comunitária; Acolhimento Institucional e Familiar; ............................................104
17. Adolescente em Conflito com a Lei; ...................................................................................................................124
18. Medidas Socioeducativas; ....................................................................................................................................126
19. Criança e Adolescente vitimizados; ...................................................................................................................127
20. Violências e Violência Doméstica; ......................................................................................................................128
21. Medidas de Proteção e a Tutela de idosos em situação de risco; .................................................................132
22. Tutela e Curatela; ..................................................................................................................................................132
23. Guarda; 24. Guarda Compartilhada; ..................................................................................................................132
25. Alienação Parental. ................................................................................................................................................134

Legislação
CONSTITUIÇÃO DA REPÚBLICA FEDERATIVA DO BRASIL TÍTULO II Dos Direitos e Garantias Fundamentais
CAPÍTULO I Dos Direitos e Deveres Individuais e Coletivos CAPÍTULO II Dos Direitos Sociais TÍTULO VIII Do
Ordem Social CAPÍTULO II Da Seguridade Social CAPÍTULO VII Da Família, da Criança, do Adolescente, do
Jovem e do Idoso. ...............................................................................................................................................................1
ESTATUTO DA CRIANÇA E DO ADOLESCENTE Atualizado com a Lei 12.010. .................................................... 10
CÓDIGO CIVIL Lei 10.406/2002 Arts: 1511 a 1638; 1694 a 1727 e 1728 a 1783. ............................................. 41
LEI MARIA DA PENHA LEI 11.340/2006. .................................................................................................................. 56
CÓDIGO DE PROCESSO CIVIL LEI 13.105/2015 Arts: 144 a 149, 156 a 158, 464 a 480, 693 a 699, 747 a
765. ................................................................................................................................................................................... 63
GUARDA COMPARTILHADA LEI 11.698/2014. ........................................................................................................ 70
NOVA GUARDA COMPARTILHADA LEI 13.058/2014. ............................................................................................ 71
ALIENAÇÃO PARENTAL LEI 12.318/2010. ............................................................................................................... 72
SISTEMA ÚNICO DE ASSISTÊNCIA SOCIAL – SUAS Lei 12.435/2011. .................................................................. 74
SISTEMA NACIONAL DE ATENDIMENTO SOCIOEDUCATIVO Lei 12.594/2012. ............................................... 78
PLANO NACIONAL DE PROMOÇÃO, PROTEÇÃO E DEFESA DO DIREITO DE CRIANÇAS E ADOLESCENTES À
CONVIVÊNCIA FAMILIAR E COMUNITÁRIA – 2006. ................................................................................................ 90
ESTATUTO DO IDOSO Lei 10.741/2003. ..................................................................................................................112
LEI BRASILEIRA DE INCLUSÃO DA PESSOA COM DEFICIÊNCIA LEI 13.146/2015. ........................................123
CÓDIGO DE ÉTICA DO/A ASSISTENTE SOCIAL LEI 8.662/93. .............................................................................141
CÓDIGO DE ÉTICA DO PSICÓLOGO............................................................................................................................143

Apostila Digital Licenciada para Alice Caroline Guarino dos Santos - alice.guarino@hotmail.com (Proibida a Revenda)
A apostila OPÇÃO não está vinculada a empresa organizadora do concurso público a que se destina,
assim como sua aquisição não garante a inscrição do candidato ou mesmo o seu ingresso na carreira
pública.

O conteúdo dessa apostila almeja abordar os tópicos do edital de forma prática e esquematizada,
porém, isso não impede que se utilize o manuseio de livros, sites, jornais, revistas, entre outros meios
que ampliem os conhecimentos do candidato, visando sua melhor preparação.

Atualizações legislativas, que não tenham sido colocadas à disposição até a data da elaboração da
apostila, poderão ser encontradas gratuitamente no site das apostilas opção, ou nos sites
governamentais.

Informamos que não são de nossa responsabilidade as alterações e retificações nos editais dos
concursos, assim como a distribuição gratuita do material retificado, na versão impressa, tendo em vista
que nossas apostilas são elaboradas de acordo com o edital inicial. Porém, quando isso ocorrer, inserimos
em nosso site, www.apostilasopcao.com.br, no link “erratas”, a matéria retificada, e disponibilizamos
gratuitamente o conteúdo na versão digital para nossos clientes.

Caso haja dúvidas quanto ao conteúdo desta apostila, o adquirente deve acessar o site
www.apostilasopcao.com.br, e enviar sua dúvida, que será respondida o mais breve possível, assim como
para consultar alterações legislativas e possíveis erratas.

Também ficam à disposição do adquirente o telefone (11) 2856-6066, dentro do horário comercial,
para eventuais consultas.

Eventuais reclamações deverão ser encaminhadas por escrito, respeitando os prazos instituídos no
Código de Defesa do Consumidor.

É proibida a reprodução total ou parcial desta apostila, de acordo com o Artigo 184 do Código
Penal.

Apostilas Opção, a opção certa para a sua realização.

Apostila Digital Licenciada para Alice Caroline Guarino dos Santos - alice.guarino@hotmail.com (Proibida a Revenda)
Apostila Digital Licenciada para Alice Caroline Guarino dos Santos - alice.guarino@hotmail.com (Proibida a Revenda)
LÍNGUA PORTUGUESA

Apostila Digital Licenciada para Alice Caroline Guarino dos Santos - alice.guarino@hotmail.com (Proibida a Revenda)
Apostila Digital Licenciada para Alice Caroline Guarino dos Santos - alice.guarino@hotmail.com (Proibida a Revenda)
APOSTILAS OPÇÃO
Exceção: mecha

4) Em vocábulos de origem indígena ou africana e nas palavras


inglesas aportuguesadas.
Exemplos: abacaxi, xavante, orixá, xará, xerife, xampu

5) Nas seguintes palavras:


1. ortografia oficial; bexiga, bruxa, coaxar, faxina, graxa, lagartixa, lixa, lixo, puxar,
rixa, oxalá, praxe, roxo, vexame, xadrez, xarope, xaxim, xícara, xale,
xingar, etc.
Ortografia
Emprega-se o dígrafo Ch:
1) Nos seguintes vocábulos:
A ortografia se caracteriza por estabelecer padrões para a
bochecha, bucha, cachimbo, chalé, charque, chimarrão,
forma escrita das palavras. Essa escrita está relacionada tanto
chuchu, chute, cochilo, debochar, fachada, fantoche, ficha, flecha,
a critérios etimológicos (ligados à origem das palavras) quanto
mochila, pechincha, salsicha, tchau, etc.
fonológicos (ligados aos fonemas representados). É importante
compreender que a ortografia é fruto de uma convenção. A
Para representar o fonema /j/ na forma escrita, a grafia
forma de grafar as palavras é produto de acordos ortográficos
considerada correta é aquela que ocorre de acordo com a origem
que envolvem os diversos países em que a língua portuguesa é
da palavra. Veja os exemplos:
oficial. A melhor maneira de treinar a ortografia é ler, escrever e
gesso: Origina-se do grego gypsos
consultar o dicionário sempre que houver dúvida.
jipe: Origina-se do inglês jeep.
O Alfabeto
Emprega-se o G:
O alfabeto da língua portuguesa é formado por 26 letras. Cada
1) Nos substantivos terminados em -agem, -igem, -ugem
letra apresenta uma forma minúscula e outra maiúscula. Veja:
Exemplos: barragem, miragem, viagem, origem, ferrugem
Exceção: pajem
a A (á) b B (bê)
c C (cê) d D (dê)
2) Nas palavras terminadas em -ágio, -égio, -ígio, -ógio, -úgio
e E (é) f F (efe)
Exemplos: estágio, privilégio, prestígio, relógio, refúgio
g G (gê ou guê) h H (agá)
i I (i) j J (jota)
3) Nas palavras derivadas de outras que se grafam com g
k K (cá) l L (ele)
Exemplos: engessar (de gesso), massagista (de massagem),
m M (eme) n N (ene)
vertiginoso (de vertigem)
o O (ó) p P (pê)
q Q (quê) r R (erre)
4) Nos seguintes vocábulos:
s S (esse) t T (tê)
algema, auge, bege, estrangeiro, geada, gengiva, gibi, gilete,
u U (u) v V (vê)
hegemonia, herege, megera, monge, rabugento, vagem.
w W (dáblio) x X (xis)
y Y (ípsilon) z Z (zê)
Emprega-se o J:
1) Nas formas dos verbos terminados em -jar ou -jear
Observação: emprega-se também o ç, que representa o
Exemplos:
fonema /s/ diante das letras: a, o, e u em determinadas palavras.
arranjar: arranjo, arranje, arranjem
despejar: despejo, despeje, despejem
Emprego das letras K, W e Y
gorjear: gorjeie, gorjeiam, gorjeando
Utilizam-se nos seguintes casos:
enferrujar: enferruje, enferrujem
a) Em antropônimos originários de outras línguas e seus
viajar: viajo, viaje, viajem
derivados.
Exemplos: Kant, kantismo; Darwin, darwinismo; Taylor,
2) Nas palavras de origem tupi, africana, árabe ou exótica
taylorista.
Exemplos: biju, jiboia, canjica, pajé, jerico, manjericão, Moji
b) Em topônimos originários de outras línguas e seus
3) Nas palavras derivadas de outras que já apresentam j
derivados.
Exemplos:
Exemplos: Kuwait, kuwaitiano.
laranja- laranjeira loja- lojista lisonja -
lisonjeador nojo- nojeira
c) Em siglas, símbolos, e mesmo em palavras adotadas como
cereja- cerejeira varejo- varejista rijo- enrijecer
unidades de medida de curso internacional.
jeito- ajeitar
Exemplos: K (Potássio), W (West), kg (quilograma), km
(quilômetro), Watt.
4) Nos seguintes vocábulos:
berinjela, cafajeste, jeca, jegue, majestade, jeito, jejum, laje,
Emprego de X e Ch
traje, pegajento
Emprega-se o X:
1) Após um ditongo.
Emprego das Letras S e Z
Exemplos: caixa, frouxo, peixe
Emprega-se o S:
Exceção: recauchutar e seus derivados
1) Nas palavras derivadas de outras que já apresentam s no
radical
2) Após a sílaba inicial “en”.
Exemplos: enxame, enxada, enxaqueca
Exemplos:
Exceção: palavras iniciadas por “ch” que recebem o prefixo
análise- analisar catálise- catalisador
“en-”
casa- casinha, casebre liso- alisar
Exemplos: encharcar (de charco), enchiqueirar (de chiqueiro),
encher e seus derivados (enchente, enchimento, preencher...)
2) Nos sufixos -ês e -esa, ao indicarem nacionalidade, título
ou origem
3) Após a sílaba inicial “me-”.
Exemplos:
Exemplos: mexer, mexerica, mexicano, mexilhão
burguês- burguesa inglês- inglesa

Língua Portuguesa 1
Apostila Digital Licenciada para Alice Caroline Guarino dos Santos - alice.guarino@hotmail.com (Proibida a Revenda)
APOSTILAS OPÇÃO
chinês- chinesamilanês- milanesa Existem diversas formas para a representação do fonema /S/.
Observe:
3) Nos sufixos formadores de adjetivos -ense, -oso e -osa
Exemplos: Emprega-se o S:
catarinense gostoso- gostosa amoroso- amorosa Nos substantivos derivados de verbos terminados em
palmeirense gasoso- gasosa teimoso- teimosa “andir”,”ender”, “verter” e “pelir”
Exemplos:
4) Nos sufixos gregos -ese, -isa, -osa expandir- expansão pretender- pretensão verter-
Exemplos: versão expelir- expulsão
catequese, diocese, poetisa, profetisa, sacerdotisa, glicose, estender- extensão suspender- suspensão
metamorfose, virose converter - conversão repelir- repulsão

5) Após ditongos Emprega-se Ç:


Exemplos: Nos substantivos derivados dos verbos “ter” e “torcer”
coisa, pouso, lousa, náusea Exemplos:
ater- atenção torcer- torção
6) Nas formas dos verbos pôr e querer, bem como em seus deter- detenção distorcer-distorção
derivados manter- manutenção contorcer- contorção
Exemplos:
pus, pôs, pusemos, puseram, pusera, pusesse, puséssemos Emprega-se o X:
quis, quisemos, quiseram, quiser, quisera, quiséssemos Em alguns casos, a letra X soa como Ss
repus, repusera, repusesse, repuséssemos Exemplos:
auxílio, expectativa, experto, extroversão, sexta, sintaxe, texto,
7) Nos seguintes nomes próprios personativos: trouxe
Baltasar, Heloísa, Inês, Isabel, Luís, Luísa, Resende, Sousa,
Teresa, Teresinha, Tomás Emprega-se Sc:
Nos termos eruditos
8) Nos seguintes vocábulos: Exemplos:
abuso, asilo, através, aviso, besouro, brasa, cortesia, acréscimo, ascensorista, consciência, descender, discente,
decisão,despesa, empresa, freguesia, fusível, maisena, mesada, fascículo, fascínio, imprescindível, miscigenação, miscível,
paisagem, paraíso, pêsames, presépio, presídio, querosene, plebiscito, rescisão, seiscentos, transcender, etc.
raposa, surpresa, tesoura, usura, vaso, vigésimo, visita, etc.
Emprega-se Sç:
Emprega-se o Z: Na conjugação de alguns verbos
1) Nas palavras derivadas de outras que já apresentam z no Exemplos:
radical nascer- nasço, nasça
Exemplos: crescer- cresço, cresça
deslize- deslizar razão- razoável vazio- esvaziar descer- desço, desça
raiz- enraizar cruz-cruzeiro
Emprega-se Ss:
2) Nos sufixos -ez, -eza, ao formarem substantivos abstratos a Nos substantivos derivados de verbos terminados em “gredir”,
partir de adjetivos “mitir”, “ceder” e “cutir”
Exemplos: Exemplos:
inválido- invalidez limpo-limpeza macio- maciez agredir- agressão demitir- demissão ceder- cessão
rígido- rigidez discutir- discussão
frio- frieza nobre- nobreza pobre-pobreza surdo- progredir- progressão t r a n s m i t i r - t r a n s m i s s ã o
surdez exceder- excesso repercutir- repercussão

3) Nos sufixos -izar, ao formar verbos e -ização, ao formar Emprega-se o Xc e o Xs:


substantivos
Exemplos: Em dígrafos que soam como Ss
civilizar- civilização hospitalizar- hospitalização Exemplos:
colonizar- colonização realizar- realização exceção, excêntrico, excedente, excepcional, exsudar

4) Nos derivados em -zal, -zeiro, -zinho, -zinha, -zito, -zita Observações sobre o uso da letra X
Exemplos: 1) O X pode representar os seguintes fonemas:
cafezal, cafezeiro, cafezinho, arvorezinha, cãozito, avezita /ch/ - xarope, vexame

5) Nos seguintes vocábulos: /cs/ - axila, nexo


azar, azeite, azedo, amizade, buzina, bazar, catequizar, chafariz,
cicatriz, coalizão, cuscuz, proeza, vizinho, xadrez, verniz, etc. /z/ - exame, exílio

6) Nos vocábulos homófonos, estabelecendo distinção no /ss/ - máximo, próximo


contraste entre o S e o Z
Exemplos: /s/ - texto, extenso
cozer (cozinhar) e coser (costurar)
prezar( ter em consideração) e presar (prender) 2) Não soa nos grupos internos -xce- e -xci-
traz (forma do verbo trazer) e trás (parte posterior) Exemplos: excelente, excitar

Observação: em muitas palavras, a letra X soa como Z. Veja os Emprego das letras E e I
exemplos: Na língua falada, a distinção entre as vogais átonas /e/ e /i /
exame exato exausto exemplo existir exótico pode não ser nítida. Observe:
inexorável
Emprega-se o E:
Emprego de S, Ç, X e dos Dígrafos Sc, Sç, Ss, Xc, Xs 1) Em sílabas finais dos verbos terminados em -oar, -uar

Língua Portuguesa 2
Apostila Digital Licenciada para Alice Caroline Guarino dos Santos - alice.guarino@hotmail.com (Proibida a Revenda)
APOSTILAS OPÇÃO
Exemplos: “Auriverde pendão de minha terra,
magoar - magoe, magoes Que a brisa do Brasil beija e balança,
continuar- continue, continues Estandarte que à luz do sol encerra
As promessas divinas da Esperança…”
2) Em palavras formadas com o prefixo ante- (antes, anterior) (Castro Alves)
Exemplos: antebraço, antecipar
Observações:
3) Nos seguintes vocábulos: - No início dos versos que não abrem período, é facultativo o
cadeado, confete, disenteria, empecilho, irrequieto, mexerico, uso da letra maiúscula.
orquídea, etc. Por Exemplo:
“Aqui, sim, no meu cantinho,
Emprega-se o I : vendo rir-me o candeeiro,
1) Em sílabas finais dos verbos terminados em -air, -oer, -uir gozo o bem de estar sozinho
Exemplos: e esquecer o mundo inteiro.”
cair- cai
doer- dói - Depois de dois pontos, não se tratando de citação direta, usa-
influir- influi se letra minúscula.
Por Exemplo:
2) Em palavras formadas com o prefixo anti- (contra) “Chegam os magos do Oriente, com suas dádivas: ouro,
Exemplos: incenso, mirra.” (Manuel Bandeira)
Anticristo, antitetânico
b) Nos antropônimos, reais ou fictícios.
3) Nos seguintes vocábulos: Exemplos:
aborígine, artimanha, chefiar, digladiar, penicilina, privilégio, Pedro Silva, Cinderela, D. Quixote.
etc.
c) Nos topônimos, reais ou fictícios.
Emprego das letras O e U Exemplos:
Emprega-se o O/U: Rio de Janeiro, Rússia, Macondo.
A oposição o/u é responsável pela diferença de significado de
algumas palavras. Veja os exemplos: d) Nos nomes mitológicos.
comprimento (extensão) e cumprimento (saudação, Exemplos:
realização) Dionísio, Netuno.
soar (emitir som) e suar (transpirar)
e) Nos nomes de festas e festividades.
Grafam-se com a letra O: bolacha, bússola, costume, Exemplos:
moleque. Natal, Páscoa, Ramadã.

Grafam-se com a letra U: camundongo, jabuti, Manuel, tábua f) Em siglas, símbolos ou abreviaturas internacionais.
Exemplos:
Emprego da letra H ONU, Sr., V. Ex.ª.
Esta letra, em início ou fim de palavras, não tem valor fonético.
Conservou-se apenas como símbolo, por força da etimologia e g) Nos nomes que designam altos conceitos religiosos,
da tradição escrita. A palavra hoje, por exemplo, grafa-se desta políticos ou nacionalistas.
forma devido a sua origem na forma latina hodie. Exemplos:
Igreja (Católica, Apostólica, Romana), Estado, Nação, Pátria,
Emprega-se o H: União, etc.
1) Inicial, quando etimológico
Exemplos: hábito, hesitar, homologar, Horácio Observação: esses nomes escrevem-se com inicial minúscula
quando são empregados em sentido geral ou indeterminado.
2) Medial, como integrante dos dígrafos ch, lh, nh Exemplo:
Exemplos: flecha, telha, companhia Todos amam sua pátria.

3) Final e inicial, em certas interjeições Emprego FACULTATIVO de letra maiúscula:


Exemplos: ah!, ih!, eh!, oh!, hem?, hum!, etc. a) Nos nomes de logradouros públicos, templos e edifícios.
Exemplos:
4) Em compostos unidos por hífen, no início do segundo Rua da Liberdade ou rua da Liberdade
elemento, se etimológico Igreja do Rosário ou igreja do Rosário
Exemplos: anti-higiênico, pré-histórico, super-homem, etc. Edifício Azevedo ou edifício Azevedo

Observações: 2) Utiliza-se inicial minúscula:


1) No substantivo Bahia, o “h” sobrevive por tradição. Note que a) Em todos os vocábulos da língua, nos usos correntes.
nos substantivos derivados como baiano, baianada ou baianinha Exemplos:
ele não é utilizado. carro, flor, boneca, menino, porta, etc.

2) Os vocábulos erva, Espanha e inverno não possuem a b) Nos nomes de meses, estações do ano e dias da semana.
letra “h” na sua composição. No entanto, seus derivados eruditos Exemplos:
sempre são grafados com h. Veja: janeiro, julho, dezembro, etc.
herbívoro, hispânico, hibernal. segunda, sexta, domingo, etc.
primavera, verão, outono, inverno
Emprego das Iniciais Maiúsculas e Minúsculas
1) Utiliza-se inicial maiúscula: c) Nos pontos cardeais.
a) No começo de um período, verso ou citação direta. Exemplos:
Exemplos: Percorri o país de norte a sul e de leste a oeste.
Disse o Padre Antonio Vieira: “Estar com Cristo em qualquer Estes são os pontos colaterais: nordeste, noroeste, sudeste,
lugar, ainda que seja no inferno, é estar no Paraíso.” sudoeste.

Língua Portuguesa 3
Apostila Digital Licenciada para Alice Caroline Guarino dos Santos - alice.guarino@hotmail.com (Proibida a Revenda)
APOSTILAS OPÇÃO
Observação: quando empregados em sua forma absoluta, os Função de
pontos cardeais são grafados com letra maiúscula. Exemplos:
substantivo
Exemplos: – vem
Nordeste (região do Brasil) Não é fácil encontrar o
acompanhado
Ocidente (europeu) Porquê porquê de toda confusão.
de artigo ou
Oriente (asiático) pronome
Lembre-se: Dê-me um porquê de sua
Depois de dois-pontos, não se tratando de citação direta, usa- saída.
se letra minúscula.
1. Por que (pergunta)
Exemplo:
2. Porque (resposta)
“Chegam os magos do Oriente, com suas dádivas: ouro,
3. Por quê (fim de frase: motivo)
incenso, mirra.” (Manuel Bandeira)
4. O Porquê (substantivo)
Emprego FACULTATIVO de letra minúscula:
Emprego de outras palavras
a) Nos vocábulos que compõem uma citação bibliográfica.
Exemplos:
Senão: equivale a “caso contrário”, “a não ser”: Não fazia coisa
Crime e Castigo ou Crime e castigo
nenhuma senão criticar.
Grande Sertão: Veredas ou Grande sertão: veredas
Se não: equivale a “se por acaso não”, em orações adverbiais
Em Busca do Tempo Perdido ou Em busca do tempo perdido
condicionais: Se não houver homens honestos, o país não sairá
desta situação crítica.
b) Nas formas de tratamento e reverência, bem como em
nomes sagrados e que designam crenças religiosas.
Tampouco: advérbio, equivale a “também não”: Não
Exemplos:
compareceu, tampouco apresentou qualquer justificativa.
Governador Mário Covas ou governador Mário Covas
Tão pouco: advérbio de intensidade: Encontramo-nos tão
Papa João Paulo II ou papa João Paulo II
pouco esta semana.
Excelentíssimo Senhor Reitor ou excelentíssimo senhor reitor
Santa Maria ou santa Maria.
Trás ou Atrás = indicam lugar, são advérbios.
Traz - do verbo trazer.
c) Nos nomes que designam domínios de saber, cursos e
disciplinas.
Vultoso: volumoso: Fizemos um trabalho vultoso aqui.
Exemplos:
Vultuoso: atacado de congestão no rosto: Sua face está
Português ou português
vultuosa e deformada.
Línguas e Literaturas Modernas ou línguas e literaturas
Questões
modernas
História do Brasil ou história do Brasil
01. Que mexer o esqueleto é bom para a saúde já virou
Arquitetura ou arquitetura
até sabedoria popular. Agora, estudo levanta hipóteses sobre
Fonte: http://www.soportugues.com.br/secoes/fono/
........................ praticar atividade física..........................benefícios
fono24.php
para a totalidade do corpo. Os resultados podem levar a novas
Emprego do Porquê
terapias para reabilitar músculos contundidos ou mesmo para
Orações .......................... e restaurar a perda muscular que ocorre com o
Interrogativas Exemplo: avanço da idade.
(Ciência Hoje, março de 2012)
(pode ser Por que devemos nos
substituído por: preocupar com o meio As lacunas do texto devem ser preenchidas, correta e
Por por qual motivo, ambiente? respectivamente, com:
Que por qual razão) (A) porque … trás … previnir
(B) porque … traz … previnir
Exemplo: (C) porquê … tras … previnir
Equivalendo (D) por que … traz … prevenir
a “pelo qual” Os motivos por que não (E) por quê … tráz … prevenir
respondeu são desconhecidos.
02. Assinale a opção que completa corretamente as lacunas
Exemplos: da frase abaixo: Não sei o _____ ela está com os olhos vermelhos,
talvez seja _____ chorou.
Você ainda tem coragem de (A) porquê / porque;
Final de
Por perguntar por quê? (B) por que / porque;
frases e seguidos
Quê (C) porque / por que;
de pontuação
Você não vai? Por quê? (D) porquê / por quê;
(E) por que / por quê.
Não sei por quê!
Exemplos: 03.
Conjunção
A situação agravou-se
que indica
porque ninguém reclamou.
explicação ou
causa
Ninguém mais o espera,
Porque porque ele sempre se atrasa.
Conjunção de
Exemplos:
Finalidade –
equivale a “para
Não julgues porque não te
que”, “a fim de Considerando a ortografia e a acentuação da norma-
julguem.
que”. padrão da língua portuguesa, as lacunas estão, correta e
respectivamente, preenchidas por:

Língua Portuguesa 4
Apostila Digital Licenciada para Alice Caroline Guarino dos Santos - alice.guarino@hotmail.com (Proibida a Revenda)
APOSTILAS OPÇÃO
(A) mal ... por que ... intuíto Por exemplo: faço     fiz      farei     fizesse
(B) mau ... por que ... intuito c) Defectivos: são aqueles que não apresentam conjugação
(C) mau ... porque ... intuíto completa. Classificam-se em impessoais, unipessoais e pessoais.
(D) mal ... porque ... intuito
(E) mal ... por quê ... intuito - Impessoais: são os verbos que não têm sujeito.
Normalmente, são usados na terceira pessoa do singular. Os
Respostas principais verbos impessoais são:
01. D/02. B/03. D a) haver, quando sinônimo de existir, acontecer, realizar-se
ou fazer (em orações temporais).
Havia poucos ingressos à venda. (Havia = Existiam)
2. conjugação de verbos; Houve duas guerras mundiais. (Houve = Aconteceram)
Haverá reuniões aqui. (Haverá = Realizar-se-ão)
Deixei de fumar há muitos anos. (há = faz)
Verbo
b) fazer, ser e estar (quando indicam tempo)
Verbo é a classe de palavras que se flexiona em pessoa, Faz invernos rigorosos no Sul do Brasil.
número, tempo, modo e voz. Pode indicar, entre outros Era primavera quando a conheci.
processos: ação (correr); estado (ficar); fenômeno (chover); Estava frio naquele dia.
ocorrência (nascer); desejo (querer).
O que caracteriza o verbo são as suas flexões, e não os seus c) Todos os verbos que indicam fenômenos da natureza
possíveis significados. Observe que palavras como corrida, são impessoais: chover, ventar, nevar, gear, trovejar, amanhecer,
chuva e nascimento têm conteúdo muito próximo ao de alguns escurecer, etc. Quando, porém, se constrói, “Amanheci mal-
verbos mencionados acima; não apresentam, porém, todas as humorado”, usa-se o verbo “amanhecer” em sentido figurado.
possibilidades de flexão que esses verbos possuem. Qualquer verbo impessoal, empregado em sentido figurado,
deixa de ser impessoal para ser pessoal.
Estrutura das Formas Verbais Amanheci mal-humorado. (Sujeito desinencial: eu)
Do ponto de vista estrutural, uma forma verbal pode Choveram candidatos ao cargo. (Sujeito: candidatos)
apresentar os seguintes elementos: Fiz quinze anos ontem. (Sujeito desinencial: eu)
a) Radical: é a parte invariável, que expressa o significado
essencial do verbo. Por exemplo: d) São impessoais, ainda:
fal-ei; fal-ava; fal-am. (radical fal-) 1. o verbo passar (seguido de preposição), indicando tempo.
Ex.: Já passa das seis.
b) Tema: é o radical seguido da vogal temática que indica a 2. os verbos bastar e chegar, seguidos da preposição de,
conjugação a que pertence o verbo. Por exemplo: fala-r indicando suficiência. Ex.:
São três as conjugações: Basta de tolices. Chega de blasfêmias.
1ª - Vogal Temática - A - (falar) 3. os verbos estar e ficar em orações tais como Está bem,
2ª - Vogal Temática - E - (vender) Está muito bem assim, Não fica bem, Fica mal, sem referência
3ª - Vogal Temática - I - (partir) a sujeito expresso anteriormente. Podemos, ainda, nesse caso,
classificar o sujeito como hipotético, tornando-se, tais verbos,
c) Desinência modo-temporal: é o elemento que designa o então, pessoais.
tempo e o modo do verbo. 4. o verbo deu + para da língua popular, equivalente de “ser
Por exemplo: possível”. Por exemplo:
falávamos ( indica o pretérito imperfeito do indicativo.) Não deu para chegar mais cedo.
falasse ( indica o pretérito imperfeito do subjuntivo.) Dá para me arrumar uns trocados?

d) Desinência número-pessoal: é o elemento que designa - Unipessoais: são aqueles que, tendo sujeito, conjugam-se
a pessoa do discurso ( 1ª, 2ª ou 3ª) e o número (singular ou apenas nas terceiras pessoas, do singular e do plural.
plural). A fruta amadureceu.
falamos (indica a 1ª pessoa do plural.) As frutas amadureceram.
falavam (indica a 3ª pessoa do plural.)
Obs.: os verbos unipessoais podem ser usados como verbos
Observação: o verbo pôr, assim como seus derivados pessoais na linguagem figurada:
(compor, repor, depor, etc.), pertencem à 2ª conjugação, pois a Teu irmão amadureceu bastante.
forma arcaica do verbo pôr era poer. A vogal “e”, apesar de haver Entre os unipessoais estão os verbos que significam vozes de
desaparecido do infinitivo, revela-se em algumas formas do animais; eis alguns:
verbo: põe, pões, põem, etc. bramar: tigre
bramir: crocodilo
Formas Rizotônicas e Arrizotônicas cacarejar: galinha
Ao combinarmos os conhecimentos sobre a estrutura dos coaxar: sapo
verbos com o conceito de acentuação tônica, percebemos com cricrilar: grilo
facilidade que nas formas rizotônicas, o acento tônico cai no
radical do verbo: opino, aprendam, nutro, por exemplo. Nas Os principais verbos unipessoais são:
formas arrizotônicas, o acento tônico não cai no radical, mas sim 1. cumprir, importar, convir, doer, aprazer, parecer,
na terminação verbal: opinei, aprenderão, nutriríamos. ser (preciso, necessário, etc.).
Cumpre trabalharmos bastante. (Sujeito: trabalharmos
Classificação dos Verbos bastante.)
Parece que vai chover. (Sujeito: que vai chover.)
Classificam-se em: É preciso que chova. (Sujeito: que chova.)
a) Regulares: são aqueles que possuem as desinências 2. fazer e ir, em orações que dão ideia de tempo, seguidos da
normais de sua conjugação e cuja flexão não provoca alterações conjunção que.
no radical.
Faz dez anos que deixei de fumar. (Sujeito: que deixei de
Por exemplo: canto     cantei      cantarei     cantava      cantasse fumar.)
b) Irregulares: são aqueles cuja flexão provoca alterações Vai para (ou Vai em ou Vai por) dez anos que não vejo Cláudia.
no radical ou nas desinências. (Sujeito: que não vejo Cláudia)

Língua Portuguesa 5
Apostila Digital Licenciada para Alice Caroline Guarino dos Santos - alice.guarino@hotmail.com (Proibida a Revenda)
APOSTILAS OPÇÃO
Obs.: todos os sujeitos apontados são oracionais. Pretérito Perfeito Simples: eu fui, tu foste, ele foi, nós
fomos, vós fostes, eles foram.
- Pessoais: não apresentam algumas flexões por motivos Pretérito Perfeito Composto: tenho sido.
morfológicos ou eufônicos. Por exemplo: Mais-que-perfeito simples: eu fora, tu foras, ele fora, nós
verbo falir. Este verbo teria como formas do presente do fôramos, vós fôreis, eles foram.
indicativo falo, fales, fale, idênticas às do verbo falar - o que Pretérito Mais-que-Perfeito Composto: tinha sido.
provavelmente causaria problemas de interpretação em certos Futuro do Pretérito simples: eu seria, tu serias, ele seria,
contextos. nós seríamos, vós seríeis, eles seriam.
verbo computar. Este verbo teria como formas do presente do Futuro do Pretérito Composto: terei sido.
indicativo computo, computas, computa - formas de sonoridade Futuro do Presente: eu serei, tu serás, ele será, nós seremos,
considerada ofensiva por alguns ouvidos gramaticais. Essas vós sereis, eles serão.
razões muitas vezes não impedem o uso efetivo de formas Futuro do Pretérito Composto: Teria sido.
verbais repudiadas por alguns gramáticos: exemplo disso é
o próprio verbo computar, que, com o desenvolvimento e a SER - Modo Subjuntivo
popularização da informática, tem sido conjugado em todos os
tempos, modos e pessoas. Presente: que eu seja, que tu sejas, que ele seja, que nós
sejamos, que vós sejais, que eles sejam.
d) Abundantes: são aqueles que possuem mais de uma Pretérito Imperfeito: se eu fosse, se tu fosses, se ele fosse,
forma com o mesmo valor. Geralmente, esse fenômeno costuma se nós fôssemos, se vós fôsseis, se eles fossem.
ocorrer no particípio, em que, além das formas regulares Pretérito Mais-que-Perfeito Composto: tivesse sido.
terminadas em -ado ou -ido, surgem as chamadas formas Futuro Simples: quando eu for, quando tu fores, quando ele
curtas (particípio irregular). Observe: for, quando nós formos, quando vós fordes, quando eles forem.
Futuro Composto: tiver sido.
Infinitivo Particípio regular Particípio irregular SER - Modo Imperativo

Anexar Anexado Anexo Imperativo Afirmativo: sê tu, seja ele, sejamos nós, sede
vós, sejam eles.
Dispersar Dispersado Disperso Imperativo Negativo: não sejas tu, não seja ele, não sejamos
Eleger Elegido Eleito nós, não sejais vós, não sejam eles.
Infinitivo Pessoal: por ser eu, por seres tu, por ser ele, por
Envolver Envolvido Envolto sermos nós, por serdes vós, por serem eles.
Imprimir Imprimido Impresso
SER - Formas Nominais
Matar Matado Morto
Morrer Morrido Morto Formas Nominais
Infinitivo: ser
Pegar Pegado Pego Gerúndio: sendo
Soltar Soltado Solto Particípio: sido

e) Anômalos: são aqueles que incluem mais de um radical Infinitivo Pessoal : ser eu, seres tu, ser ele, sermos
em sua conjugação. nós, serdes vós, serem eles.
Por exemplo:
ESTAR - Modo Indicativo
Ir Pôr Ser Saber Presente: eu estou, tu estás, ele está, nós estamos, vós estais,
vou ponho sou sei eles estão.
vais pus és sabes Pretérito Imperfeito: eu estava, tu estavas, ele estava, nós
ides pôs fui soube estávamos, vós estáveis, eles estavam.
fui punha foste saiba Pretérito Perfeito Simples: eu estive, tu estiveste, ele
esteve, nós estivemos, vós estivestes, eles estiveram.
foste seja
Pretérito Perfeito Composto: tenho estado.
Pretérito Mais-que-Perfeito Simples: eu estivera, tu
f) Auxiliares estiveras, ele estivera, nós estivéramos, vós estivéreis, eles
São aqueles que entram na formação dos tempos estiveram.
compostos e das locuções verbais. O verbo principal, quando Pretérito Mais-que-perfeito Composto: tinha estado
acompanhado de verbo auxiliar, é expresso numa das formas Futuro do Presente Simples: eu estarei, tu estarás, ele
nominais: infinitivo, gerúndio ou particípio. estará, nós estaremos, vós estareis, eles estarão.
                         Futuro do Presente Composto: terei estado.
Vou                       espantar           as          moscas. Futuro do Pretérito Simples: eu estaria, tu estarias, ele
(verbo auxiliar)       (verbo principal no infinitivo) estaria, nós estaríamos, vós estaríeis, eles estariam.
Futuro do Pretérito Composto: teria estado.
Está                    chegando            a         hora     do    debate.
(verbo auxiliar)      (verbo principal no gerúndio)                  ESTAR - Modo Subjuntivo e Imperativo
                   
Obs.: os verbos auxiliares mais usados são: ser, estar, ter e Presente: que eu esteja, que tu estejas, que ele esteja, que
haver. nós estejamos, que vós estejais, que eles estejam.
Pretérito Imperfeito: se eu estivesse, se tu estivesses, se
Conjugação dos Verbos Auxiliares ele estivesse, se nós estivéssemos, se vós estivésseis, se eles
estivessem.
SER - Modo Indicativo Pretérito Mais-que-Perfeito Composto: tivesse estado
Futuro Simples: quando eu estiver, quando tu estiveres,
Presente: eu sou, tu és, ele é, nós somos, vós sois, eles são. quando ele estiver, quando nós estivermos, quando vós
Pretérito Imperfeito: eu era, tu eras, ele era, nós éramos, estiverdes, quando eles estiverem.
vós éreis, eles eram. Futuro Composto: Tiver estado.

Língua Portuguesa 6
Apostila Digital Licenciada para Alice Caroline Guarino dos Santos - alice.guarino@hotmail.com (Proibida a Revenda)
APOSTILAS OPÇÃO
Imperativo Afirmativo: está tu, esteja ele, estejamos nós, tínhamos, vós tínheis, eles tinham.
estai vós, estejam eles. Pretérito Perfeito Simples: eu tive, tu tiveste, ele teve, nós
Imperativo Negativo: não estejas tu, não esteja ele, não tivemos, vós tivestes, eles tiveram.
estejamos nós, não estejais vós, não estejam eles. Pretérito Perfeito Composto: tenho tido.
Infinitivo Pessoal: por estar eu, por estares tu, por estar ele, Pretérito Mais-que-Perfeito Simples: eu tivera, tu tiveras,
por estarmos nós, por estardes vós, por estarem eles. ele tivera, nós tivéramos, vós tivéreis, eles tiveram.
Pretérito Mais-que-Perfeito Composto: tinha tido.
Formas Nominais Futuro do Presente Simples: eu terei, tu terás, ele terá, nós
Infinitivo: estar teremos, vós tereis, eles terão.
Gerúndio: estando Futuro do Presente: terei tido.
Particípio: estado Futuro do Pretérito Simples: eu teria, tu terias, ele teria,
nós teríamos, vós teríeis, eles teriam.
ESTAR - Formas Nominais Futuro do Pretérito composto: teria tido.

Infinitivo Impessoal: estar TER - Modo Subjuntivo e Imperativo


Infinitivo Pessoal: estar, estares, estar, estarmos, estardes,
estarem. Modo Subjuntivo
Gerúndio: estando Presente: que eu tenha, que tu tenhas, que ele tenha, que
Particípio: estado nós tenhamos, que vós tenhais, que eles tenham.
Pretérito Imperfeito: se eu tivesse, se tu tivesses, se ele
HAVER - Modo Indicativo tivesse, se nós tivéssemos, se vós tivésseis, se eles tivessem.
Pretérito Mais-que-Perfeito Composto: tivesse tido.
Presente: eu hei, tu hás, ele há, nós havemos, vós haveis, eles Futuro: quando eu tiver, quando tu tiveres, quando ele tiver,
hão. quando nós tivermos, quando vós tiverdes, quando eles tiverem.
Pretérito Imperfeito: eu havia, tu havias, ele havia, nós Futuro Composto: tiver tido.
havíamos, vós havíeis, eles haviam.
Pretérito Perfeito Simples: eu houve, tu houveste, ele Modo Imperativo
houve, nós houvemos, vós houvestes, eles houveram. Imperativo Afirmativo: tem tu, tenha ele, tenhamos nós,
Pretérito Perfeito Composto: tenho havido. tende vós, tenham eles.
Pretérito Mais-que-Perfeito Simples: eu houvera, tu Imperativo Negativo: não tenhas tu, não tenha ele, não
houveras, ele houvera, nós houvéramos, vós houvéreis, eles tenhamos nós, não tenhais vós, não tenham eles.
houveram. Infinitivo Pessoal: por ter eu, por teres tu, por ter ele, por
Pretérito Mais-que-Prefeito Composto: tinha havido. termos nós, por terdes vós, por terem eles.
Futuro do Presente Simples: eu haverei, tu haverás, ele
haverá, nós haveremos, vós havereis, eles haverão. g) Pronominais: São aqueles verbos que se conjugam com
Futuro do Presente Composto: terei havido. os pronomes oblíquos átonos me, te, se, nos, vos, se, na mesma
Futuro do Pretérito Simples: eu haveria, tu haverias, ele pessoa do sujeito, expressando reflexibilidade (pronominais
haveria, nós haveríamos, vós haveríeis, eles haveriam. acidentais) ou apenas reforçando a ideia já implícita no próprio
Futuro do Pretérito Composto: teria havido. sentido do verbo (reflexivos essenciais). Veja:
- 1. Essenciais: são aqueles que sempre se conjugam com os
HAVER - Modo Subjuntivo e Imperativo pronomes oblíquos me, te, se, nos, vos, se. São poucos: abster-se,
ater-se, apiedar-se, atrever-se, dignar-se, arrepender-se, etc. Nos
Modo Subjuntivo verbos pronominais essenciais a reflexibilidade já está implícita
Presente: que eu haja, que tu hajas, que ele haja, que nós no radical do verbo. Por exemplo:
hajamos, que vós hajais, que eles hajam. Arrependi-me de ter estado lá.
Pretérito Imperfeito: se eu houvesse, se tu houvesses, se A ideia é de que a pessoa representada pelo sujeito (eu) tem
ele houvesse, se nós houvéssemos, se vós houvésseis, se eles um sentimento (arrependimento) que recai sobre ela mesma,
houvessem. pois não recebe ação transitiva nenhuma vinda do verbo; o
Pretérito Mais-que-Perfeito Composto: tivesse havido. pronome oblíquo átono é apenas uma partícula integrante do
Futuro Simples: quando eu houver, quando tu houveres, verbo, já que, pelo uso, sempre é conjugada com o verbo. Diz-
quando ele houver, quando nós houvermos, quando vós se que o pronome apenas serve de reforço da ideia reflexiva
houverdes, quando eles houverem. expressa pelo radical do próprio verbo.  
Futuro Composto: tiver havido. Veja uma conjugação pronominal essencial (verbo e
respectivos pronomes):
Modo Imperativo Eu me arrependo
Imperativo Afirmativo: haja ele, hajamos nós, havei vós, Tu te arrependes
hajam eles. Ele se arrepende
Imperativo Negativo: não hajas tu, não haja ele, não Nós nos arrependemos
hajamos nós, não hajais vós, não hajam eles. Vós vos arrependeis
Infinitivo Pessoal: por haver eu, por haveres tu, por haver Eles se arrependem
ele, por havermos nós, por haverdes vós, por haverem eles.
- 2. Acidentais: são aqueles verbos transitivos diretos em que
HAVER - Formas Nominais a ação exercida pelo sujeito recai sobre o objeto representado por
pronome oblíquo da mesma pessoa do sujeito; assim, o sujeito
Infinitivo Impessoal: haver, haveres, haver, havermos, faz uma ação que recai sobre ele mesmo. Em geral, os verbos
haverdes, haverem. transitivos diretos ou transitivos diretos e indiretos podem ser
Infinitivo Pessoal: haver conjugados com os pronomes mencionados, formando o que se
Gerúndio: havendo chama voz reflexiva. Por exemplo: Maria se penteava.
Particípio: havido A reflexibilidade é acidental, pois a ação reflexiva pode
ser exercida também sobre outra pessoa. Por exemplo: Maria
TER - Modo Indicativo penteou-me.

Presente: eu tenho, tu tens, ele tem, nós temos, vós tendes, Observações:
eles têm. 1- Por fazerem parte integrante do verbo, os pronomes
Pretérito Imperfeito: eu tinha, tu tinhas, ele tinha, nós oblíquos átonos dos verbos pronominais não possuem função

Língua Portuguesa 7
Apostila Digital Licenciada para Alice Caroline Guarino dos Santos - alice.guarino@hotmail.com (Proibida a Revenda)
APOSTILAS OPÇÃO
sintática. 1. Tempos do Indicativo
2- Há verbos que também são acompanhados de pronomes
oblíquos átonos, mas que não são essencialmente pronominais, - Presente - Expressa um fato atual. Por exemplo:
são os verbos reflexivos. Nos verbos reflexivos, os pronomes, Eu estudo neste colégio.
apesar de se encontrarem na pessoa idêntica à do sujeito, - Pretérito Imperfeito - Expressa um fato ocorrido num
exercem funções sintáticas. momento anterior ao atual, mas que não foi completamente
Por exemplo: terminado. Por exemplo: Ele estudava as lições quando foi
Eu me feri. = Eu(sujeito) - 1ª pessoa do singular me (objeto interrompido.
direto) - 1ª pessoa do singular - Pretérito Perfeito (simples) - Expressa um fato ocorrido
num momento anterior ao atual e que foi totalmente terminado.
Modos Verbais Por exemplo: Ele estudou as lições ontem à noite.
- Pretérito Perfeito (composto) - Expressa um fato que teve
Dá-se o nome de modo às várias formas assumidas pelo início no passado e que pode se prolongar até o momento atual.
verbo na expressão de um fato. Em Português, existem três Por exemplo: Tenho estudado muito para os exames.
modos: - Pretérito-Mais-Que-Perfeito - Expressa um fato ocorrido
Indicativo - indica uma certeza, uma realidade. Por exemplo: antes de outro fato já terminado. Por exemplo: Ele já tinha
Eu sempre estudo. estudado as lições quando os amigos chegaram. (forma
Subjuntivo - indica uma dúvida, uma possibilidade. Por composta) Ele já estudara as lições quando os amigos chegaram.
exemplo: Talvez eu estude amanhã. (forma simples)
Imperativo - indica uma ordem, um pedido. Por - Futuro do Presente (simples) - Enuncia um fato que deve
exemplo: Estuda agora, menino. ocorrer num tempo vindouro com relação ao momento atual.
Por exemplo: Ele estudará as lições amanhã.
Formas Nominais - Futuro do Presente (composto) - Enuncia um fato que deve
ocorrer posteriormente a um momento atual, mas já terminado
Além desses três modos, o verbo apresenta ainda formas antes de outro fato futuro. Por exemplo: Antes de bater o sinal,
que podem exercer funções de nomes (substantivo, adjetivo, os alunos já terão terminado o teste.
advérbio), sendo por isso denominadas formas nominais. - Futuro do Pretérito (simples) - Enuncia um fato que pode
Observe: ocorrer posteriormente a um determinado fato passado. Por
- a) Infinitivo Impessoal: exprime a significação do verbo exemplo: Se eu tivesse dinheiro, viajaria nas férias.
de modo vago e indefinido, podendo ter valor e função de - Futuro do Pretérito (composto) - Enuncia um fato que
substantivo. Por exemplo: Viver é lutar. (= vida é luta) poderia ter ocorrido posteriormente a um determinado fato
É indispensável combater a corrupção. (= combate à) passado. Por exemplo: Se eu tivesse ganho esse dinheiro, teria
O infinitivo impessoal pode apresentar-se no presente viajado nas férias.
(forma simples) ou no passado (forma composta). Por exemplo:
É preciso ler este livro. Era preciso ter lido este livro. 2. Tempos do Subjuntivo

b) Infinitivo Pessoal: é o infinitivo relacionado às três - Presente - Enuncia um fato que pode ocorrer no momento
pessoas do discurso. Na 1ª e 3ª pessoas do singular, não atual. Por exemplo: É conveniente que estudes para o exame.
apresenta desinências, assumindo a mesma forma do impessoal; - Pretérito Imperfeito - Expressa um fato passado, mas
nas demais, flexiona- -se da seguinte maneira: posterior a outro já ocorrido. Por exemplo: Eu esperava que
ele vencesse o jogo.
2ª pessoa do singular: Radical + ES Ex.: teres(tu)
1ª pessoa do plural: Radical + MOS Ex.:termos (nós) Obs.: o pretérito imperfeito é também usado nas construções
2ª pessoa do plural: Radical + DES Ex.:terdes (vós) em que se expressa a ideia de condição ou desejo. Por exemplo:
3ª pessoa do plural: Radical + EM Ex.:terem (eles) Se ele viesse ao clube, participaria do campeonato.
- Pretérito Perfeito (composto) - Expressa um fato totalmente
Por exemplo: terminado num momento passado. Por exemplo: Embora tenha
Foste elogiado por teres alcançado uma boa colocação. estudado bastante, não passou no teste.
- Futuro do Presente (simples) - Enuncia um fato que pode
- c) Gerúndio: o gerúndio pode funcionar como adjetivo ou ocorrer num momento futuro em relação ao atual. Por exemplo:
advérbio. Por exemplo: Quando ele vier à loja, levará as encomendas.
Saindo de casa, encontrei alguns amigos. (função de Obs.: o futuro do presente é também usado em frases que
advérbio) indicam possibilidade ou desejo. Por exemplo: Se ele vier à loja,
Nas ruas, havia crianças vendendo doces. (função adjetivo) levará as encomendas.
Na forma simples, o gerúndio expressa uma ação em curso; - Futuro do Presente (composto) - Enuncia um fato posterior
na forma composta, uma ação concluída. Por exemplo: ao momento atual mas já terminado antes de outro fato
Trabalhando, aprenderás o valor do dinheiro. futuro. Por exemplo: Quando ele tiver saído do hospital, nós o
Tendo trabalhado, aprendeu o valor do dinheiro. visitaremos.

- d) Particípio: quando não é empregado na formação dos Presente do Indicativo


tempos compostos, o particípio indica geralmente o resultado
de uma ação terminada, flexionando-se em gênero, número e 1ª conjugação/2ª conjugação/3ª conjugação / Desinência
grau. Por exemplo: pessoal
Terminados os exames, os candidatos saíram. CANTAR VENDER PARTIR
Quando o particípio exprime somente estado, sem nenhuma cantO vendO partO O
relação temporal, assume verdadeiramente a função de adjetivo cantaS vendeS parteS S
(adjetivo verbal). Por exemplo: canta vende parte -
Ela foi a aluna escolhida para representar a escola. cantaMOS vendeMOS partiMOS MOS
cantaIS vendeIS partIS IS
Tempos Verbais cantaM vendeM parteM M

Tomando-se como referência o momento em que se fala, Pretérito Perfeito do Indicativo


a ação expressa pelo verbo pode ocorrer em diversos tempos.
Veja: 1ª conjugação/2ª conjugação/3ª conjugação/Desinência
pessoal

Língua Portuguesa 8
Apostila Digital Licenciada para Alice Caroline Guarino dos Santos - alice.guarino@hotmail.com (Proibida a Revenda)
APOSTILAS OPÇÃO
CANTAR VENDER PARTIR tema a desinência temporal -SSE mais a desinência de número
canteI vendI partI I e pessoa correspondente.
cantaSTE vendeSTE partISTE STE
cantoU vendeU partiU U 1ª conj. 2ª conj. 3ª conj. Des. temporal Desin. pessoal
cantaMOS vendeMOS partiMOS MOS 1ª /2ª e 3ª conj.
cantaSTES vendeSTES partISTES STES CANTAR VENDER PARTIR
cantaRAM vendeRAM partiRAM AM cantaSSE vendeSSE partiSSE SSE Ø
cantaSSES vendeSSES partiSSES SSE S
Pretérito mais-que-perfeito cantaSSE vendeSSE partiSSE SSE Ø
cantáSSEMOS vendêSSEMOS partíssemos SSE MOS
1ª conj. / 2ª conj. / 3ª conj. /Desin. Temp. /Desin. Pess. cantáSSEIS vendêSSEIS partíSSEIS SSE IS
1ª/2ª e 3ª conj. cantaSSE vendeSSEM partiSSEM SSE M
CANTAR VENDER PARTIR - -
cantaRA vendeRA partiRA RA Ø Futuro do Subjuntivo
cantaRAS vendeRAS partiRAS RA S
cantaRA vendeRA partiRA RA Ø Para formar o futuro do subjuntivo elimina-se a desinência
cantáRAMOS vendêRAMOS partíRAMOS RA MOS -STE da 2ª pessoa do singular do pretérito perfeito, obtendo-
cantáREIS vendêREIS partíREIS RE IS se, assim, o tema desse tempo. Acrescenta-se a esse tema a
cantaRAM vendeRAM partiRAM RA M desinência temporal -R mais a desinência de número e pessoa
correspondente.
Pretérito Imperfeito do Indicativo
1ª conj. / 2ª conj. / 3ª conj. / Des. temp. /Desin. pess.
1ª conjugação / 2ª conjugação / 3ª conjugação 1ª /2ª e 3ª conj.
CANTAR VENDER PARTIR CANTAR VENDER PARTIR
cantAVA vendIA partIA cantaR vendeR partiR Ø
cantAVAS vendIAS partAS cantaRES vendeRES partiRES R ES
CantAVA vendIA partIA cantaR vendeR partiR R Ø
cantÁVAMOS vendÍAMOS partÍAMOS cantaRMOS vendeRMOS partiRMOS R MOS
cantÁVEIS vendÍEIS partÍEIS cantaRDES vendeRDES partiRDES R DES
cantAVAM vendIAM partIAM cantaREM vendeREM PartiREM R EM

Futuro do Presente do Indicativo Imperativo

1ª conjugação 2ª conjugação 3ª conjugação Imperativo Afirmativo


CANTAR VENDER PARTIR
cantar ei vender ei partir ei Para se formar o imperativo afirmativo, toma-se do presente
cantar ás vender ás partir ás do indicativo a 2ª pessoa do singular (tu) e a segunda pessoa do
cantar á vender á partir á plural (vós) eliminando-se o “S” final. As demais pessoas vêm,
cantar emos vender emos partir emos sem alteração, do presente do subjuntivo. Veja:
cantar eis vender eis partir eis
cantar ão vender ão partir ão Pres. do Indicativo Imperativo Afirm. Pres. do Subjuntivo
Eu canto --- Que eu cante
Futuro do Pretérito do Indicativo Tu cantas CantA tu Que tu cantes
Ele canta Cante você Que ele cante
1ª conjugação 2ª conjugação 3ª conjugação Nós cantamos Cantemos nós Que nós cantemos
CANTAR VENDER PARTIR Vós cantais CantAI vós Que vós canteis
cantarIA venderIA partirIA Eles cantam Cantem vocês Que eles cantem
cantarIAS venderIAS partirIAS
cantarIA venderIA partirIA Imperativo Negativo
cantarÍAMOS venderÍAMOS partirÍAMOS
cantarÍEIS venderÍEIS partirÍEIS Para se formar o imperativo negativo, basta antecipar a
cantarIAM venderIAM partirIAM negação às formas do presente do subjuntivo.

Presente do Subjuntivo Presente do Subjuntivo Imperativo Negativo


Que eu cante ---
Para se formar o presente do subjuntivo, substitui-se a Que tu cantes Não cantes tu
desinência -o da primeira pessoa do singular do presente do Que ele cante Não cante você
indicativo pela desinência -E (nos verbos de 1ª conjugação) ou Que nós cantemos Não cantemos nós
pela desinência -A (nos verbos de 2ª e 3ª conjugação). Que vós canteis Não canteis vós
Que eles cantem Não cantem eles
1ª conj./2ª conj./3ª conju./Des.Temp./Des.temp./Des. pess
1ª conj. 2ª/3ª conj. Observações:
CANTAR VENDER PARTIR
cantE vendA partA E A Ø - No modo imperativo não faz sentido usar na 3ª pessoa
cantES vendAS partAS E A S (singular e plural) as formas ele/eles, pois uma ordem, pedido
cantE vendA partA E A Ø ou conselho só se aplicam diretamente à pessoa com quem se
cantEMOS vendAMOS partAMOS E A MOS fala. Por essa razão, utiliza-se você/vocês.
cantEIS vendAIS partAIS E A IS - O verbo SER, no imperativo, faz excepcionalmente: sê (tu),
cantEM vendAM partAM E A M sede (vós).

Pretérito Imperfeito do Subjuntivo Infinitivo Impessoal

Para formar o imperfeito do subjuntivo, elimina-se a 1ª conjugação 2ª conjugação 3ª conjugação


desinência -STE da 2ª pessoa do singular do pretérito perfeito, CANTAR VENDER PARTIR
obtendo-se, assim, o tema desse tempo. Acrescenta-se a esse

Língua Portuguesa 9
Apostila Digital Licenciada para Alice Caroline Guarino dos Santos - alice.guarino@hotmail.com (Proibida a Revenda)
APOSTILAS OPÇÃO
Infinitivo Pessoal a) átono: trocam IL por EIS.
1ª conjugação 2ª conjugação 3ª conjugação Ex.: fóssil − fósseis
CANTAR VENDER PARTIR
cantar vender partir b) tônico: trocam L por S.
cantarES venderES partirES Ex.: funil − funis
cantar vender partir
cantarMOS venderMOS partirMOS 5) Palavras terminadas em EL:
cantarDES venderDES partirDES a) átono: plural em EIS.
cantarEM venderEM partirEM Ex.: nível − níveis
Questões b) tônico: plural em ÉIS.
Ex.: carretel − carretéis
01. Considere o trecho a seguir. É comum que objetos
___ esquecidos em locais públicos. Mas muitos transtornos 6) Palavras terminadas em X são invariáveis.
poderiam ser evitados se as pessoas ______ a atenção voltada Ex.: o clímax − os clímax
para seus pertences, conservando-os junto ao corpo. Assinale a
alternativa que preenche, correta e respectivamente, as lacunas 7) Há palavras cuja sílaba tônica avança.
do texto. Ex.: júnior − juniores; caráter − caracteres
(A) sejam … mantesse Obs.: A palavra caracteres é plural tanto de caractere quanto
(B) sejam … mantivessem de caráter.
(C) sejam … mantém
(D) seja … mantivessem 8) Palavras terminadas em ÃO
(E) seja … mantêm Fazem o plural em ÃOS, ÃES e ÕES.
Veja alguns muito importantes.
02. Na frase –… os níveis de pessoas sem emprego estão a) Em ões: balões, corações, grilhões, melões, gaviões.
apresentando quedas sucessivas de 2005 para cá. –, a locução
verbal em destaque expressa ação b) Em ãos: pagãos, cristãos, cidadãos, bênçãos, órgãos.
(A) concluída. Obs.: Os paroxítonos, como os dois últimos, sempre fazem o
(B) atemporal. plural em ÃOS.
(C) contínua.
(D) hipotética. c) Em ães: escrivães, tabeliães, capelães, capitães, alemães
(E) futura.
d) Em ões ou ãos: corrimões/corrimãos, verões/verãos,
03. Sem querer estereotipar, mas já estereotipando: trata-- anões/anãos
se de um ser cujas interações sociais terminam, 99% das vezes,
diante da pergunta “débito ou crédito?”. Nesse contexto, o verbo e) Em ões ou ães: charlatões/charlatães, guardiões/
estereotipar tem sentido de guardiães, cirugiões/cirurgiães
(A) considerar ao acaso, sem premeditação.
(B) aceitar uma ideia mesmo sem estar convencido dela. f) Em ões, ãos ou ães: anciões/anciãos/anciães, ermitões/
(C) adotar como referência de qualidade. ermitãos/ermitães
(D) julgar de acordo com normas legais.
(E) classificar segundo ideias preconcebidas. 9) Plural dos diminutivos com a letra z
Coloca-se a palavra no plural, corta-se o s e acrescenta-se
Respostas zinhos (ou zinhas).
1-B / 2-C / 3-E
Ex.: coraçãozinho
3. flexão de gênero, número e corações → coraçõe → coraçõezinhos
grau;
azulzinha
azuis → azui → azuizinhas

Flexão nominal e verbal. 10) Plural com metafonia (ô → ó)

Flexão nominal Algumas palavras, quando vão ao plural, abrem o timbre da


vogal o; outras, não.
Flexão de número Veja a seguir.
Os nomes (substantivo, adjetivo etc.), de modo geral,
admitem a flexão de número: singular e plural. Com metafonia
Ex.: animal − animais
singular (ô) plural (ó)
Palavras simples coro - coros
1) Na maioria das vezes, acrescenta-se S. corvo - corvos
Ex.: ponte − pontes destroço - destroços
bonito − bonitos forno - fornos
fosso - fossos
2) Palavras terminadas em R ou Z: acrescenta-se ES. poço - poços
Ex.: éter − éteres rogo - rogos
avestruz − avestruzes
Obs.: O pronome qualquer faz o plural no meio: quaisquer. Sem metafonia

3) Palavras oxítonas terminadas em S: acrescenta-se ES. singular (ô) - plural (ô)


Ex.: ananás − ananases, adorno - adornos
Obs.: As paroxítonas e as proparoxítonas são invariáveis. bolso - bolsos
Ex.: o pires − os pires, o ônibus − os ônibus endosso - endossos
esgoto - esgotos
4) Palavras terminadas em IL: estojo - estojos

Língua Portuguesa 10
Apostila Digital Licenciada para Alice Caroline Guarino dos Santos - alice.guarino@hotmail.com (Proibida a Revenda)
APOSTILAS OPÇÃO
gosto - gostos Ex.: o perde-ganha − os perde-ganha

11) Casos especiais: c) Nas frases substantivas (frases que se transformam em


aval − avales e avais substantivos).
cal − cales e cais Ex.: O maria-vai-com-as-outras − os maria-vai-com-as-
cós − coses e cós outras
fel − feles e féis
mal e cônsul − males e cônsules Observações
a) São invariáveis arco-íris, louva-a-deus, sem-vergonha,
Palavras compostas sem-teto e sem-terra.
1) Os dois elementos variam. Ex.: Os sem-terra apreciavam os arco-íris.
Quando os compostos são formados por substantivo mais
palavra variável (adjetivo, substantivo, numeral, pronome). b) Admitem mais de um plural:
Ex.: amor-perfeito − amores-perfeitos pai-nosso − pais-nossos ou pai-nossos
couve-flor − couves-flores padre-nosso − padres-nossos ou padre-nossos
segunda-feira − segundas-feiras terra-nova − terras-novas ou terra-novas
salvo-conduto − salvos-condutos ou salvo-condutos
2) Só o primeiro elemento varia. xeque-mate − xeques-mates ou xeques-mate
a) Quando há preposição no composto, mesmo que oculta.
Ex.: pé-de-moleque − pés-de-moleque c) Casos especiais: palavras que não se encaixam nas regras.
cavalo-vapor − cavalos-vapor (de ou a vapor) o bem-me-quer − os bem-me-queres
o joão-ninguém − os joões-ninguém
b) Quando o segundo substantivo determina o primeiro (fim o lugar-tenente − os lugar-tenentes
ou semelhança). o mapa-múndi − os mapas-múndi
Ex.: banana-maçã − bananas-maçã (semelhante a maçã)
navio-escola − navios-escola (a finalidade é a escola) Flexão de gênero
Os substantivos e as palavras que o acompanham na frase
Observações admitem a flexão de gênero: masculino e feminino.
a) Alguns autores admitem a flexão dos dois elementos. É Ex.: Meu amigo diretor recebeu o primeiro salário.
uma situação polêmica. Minha amiga diretora recebeu a primeira prestação.
Ex.: mangas-espada (preferível) ou mangas-espadas A flexão de feminino pode ocorrer de duas maneiras.

b) Quando dizemos (e isso vai ocorrer outras vezes) que é 1) Com a troca de o ou e por a.
uma situação polêmica, discutível, convém ter em mente que a Ex.: lobo − loba
questão do concurso deve ser resolvida por eliminação, ou seja, mestre − mestra
analisando bem as outras opções.
2) Por meio de diferentes sufixos nominais de gênero, muitas
3) Apenas o último elemento varia. vezes com alterações do radical.
a) Quando os elementos são adjetivos.
Ex.: hispano-americano − hispano-americanos Veja alguns femininos importantes.
Obs.: A exceção é surdo-mudo, em que os dois adjetivos se ateu − atéia
flexionam: surdos-mudos. bispo − episcopisa
b) Nos compostos em que aparecem os adjetivos GRÃO, GRÃ conde − condessa
e BEL. duque − duquesa
Ex.: grão-duque − grão-duques frade − freira
grã-cruz − grã-cruzes ilhéu − ilhoa
bel-prazer − bel-prazeres judeu − judia
marajá − marani
c) Quando o composto é formado por verbo ou qualquer monje − monja
elemento invariável (advérbio, pigmeu − pigméia
interjeição, prefixo etc.) mais substantivo ou adjetivo.
Ex.: arranha-céu − arranha-céus Alguns substantivos são uniformes quanto ao gênero, ou
sempre-viva − sempre-vivas seja, possuem uma única forma para masculino e feminino.
super-homem − super-homens Podem ser:
1) Sobrecomuns: admitem apenas um artigo, podendo
d) Quando os elementos são repetidos ou onomatopaicos designar os dois sexos.
(representam sons). Ex.: a pessoa, o cônjuge, a testemunha
Ex.: reco-reco − reco-recos 2) Comuns de dois gêneros: admitem os dois artigos,
pingue-pongue − pingue-pongues podendo então ser masculinos ou femininos.
bem-te-vi − bem-te-vis Ex.: o estudante − a estudante, o cientista − a cientista, o
patriota − a patriota
Observações 3) Epicenos: admitem apenas um artigo, designando os
a) Como se vê pelo segundo exemplo, pode haver alguma animais.
alteração nos elementos, ou seja, não serem iguais. Ex.: O jacaré, a cobra, o polvo

b) Se forem verbos repetidos, admite-se também pôr os dois Observações


no plural. a) O feminino de elefante é elefanta, e não elefoa. Aliá é
Ex.: pisca-pisca − pisca-piscas ou piscas-piscas correto, mas designa apenas uma espécie de elefanta.
b) Mamão, para alguns gramáticos, deve ser considerado
4) Nenhum elemento varia. epiceno. É algo discutível.
c) Há substantivos de gênero duvidoso, que as pessoas
a) Quando há verbo mais palavra invariável. costumam trocar. Veja alguns que convém gravar.
Ex.: O cola-tudo − os cola-tudo Masculinos - Femininos
champanha - aguardente
b) Quando há dois verbos de sentido oposto. dó - alface

Língua Portuguesa 11
Apostila Digital Licenciada para Alice Caroline Guarino dos Santos - alice.guarino@hotmail.com (Proibida a Revenda)
APOSTILAS OPÇÃO
eclipse - cal humilde − humílimo
formicida - cataplasma magro − macérrimo
grama (peso) - grafite negro − nigérrimo
milhar - libido pobre − paupérrimo
plasma - omoplata sagrado − sacratíssimo
soprano - musse sério − seriíssimo
suéter - preá soberbo – superbíssimo
telefonema
Questões
d) Existem substantivos que admitem os dois gêneros.
Ex.: diabetes (ou diabete), laringe, usucapião etc. 1) Assinale a alternativa que apresenta erro de plural.
a) o balãozinho – os balõezinhos, o júnior – os juniores
Flexão de grau b) o lápis – os lápis, o projetil − os projéteis
c) o arroz – os arrozes, o éter – os éteres
Por razões meramente didáticas, incluo, aqui, o grau entre os d) o mel – os meles, o gol – os goles
processos de flexão.
Alguns autores também o fazem, talvez pelo mesmo motivo. 2) Está mal flexionada em número a palavra:
a) o paul − os pauis
Grau do substantivo b) o látex − os látex
c) a gravidez − as gravidezes
1) Normal ou positivo: sem nenhuma alteração. d) o caráter − os caráteres
Ex.: chapéu
3) Assinale o item em que todas as palavras são masculinas.
2) Aumentativo a) dinamite, pijama, eclipse
a) sintético: chapelão b) grafite, formicida, omoplata
b) analítico: chapéu grande, chapéu enorme etc. c) grama (peso), dó, telefonema
d) suéter, faringe, clã
3) Diminutivo
a) sintético: chapeuzinho 4) Marque a opção em que todas as palavras são femininas.
b) analítico: chapéu pequeno, chapéu reduzido etc. a) agravante, aguardente, libido
Obs.: Um grau é sintético quando formado por sufixo; b) milhar, alface, musse
analítico, por meio de outras palavras. c) cataplasma, lança-perfume, champanha
d) cal, soprano, laringe
Grau do adjetivo
1) Normal ou positivo: João é forte. Respostas
2) Comparativo 1–B/ 2–D /3–C /4–A
a) de superioridade: João é mais forte que André. (ou do que)
b) de inferioridade: João é menos forte que André. (ou do Flexão verbal
que)
c) de igualdade: João é tão forte quanto André. (ou como) 1) Número: singular ou plural
3) Superlativo Ex.: ando, andas, anda → singular
a) absoluto andamos, andais, andam → plural
sintético: João é fortíssimo.
analítico: João é muito forte. (bastante forte, forte demais 2) Pessoas: são três.
etc.) a) A primeira é aquela que fala; corresponde aos pronomes
eu (singular) e nós (plural).
b) relativo Ex.: escreverei, escreveremos
de superioridade: João é o mais forte da turma.
de inferioridade: João é o menos forte da turma. b) A segunda é aquela com quem se fala; corresponde aos
pronomes tu (singular) e vós (plural).
Observações Ex.: escreverás, escrevereis
a) O grau superlativo absoluto corresponde a um aumento
do adjetivo. Pode ser expresso por um sufixo (íssimo, érrimo c) A terceira é aquela acerca de quem se fala; corresponde
ou imo) ou uma palavra de apoio, como muito, bastante, aos pronomes ele ou ela (singular) e eles ou elas (plural).
demasiadamente, enorme etc. Ex.: escreverá, escreverão

b) As palavras maior, menor, melhor e pior constituem 3) Modos: são três.


sempre graus de superioridade. a) Indicativo: apresenta o fato verbal de maneira positiva,
Ex.: O carro é menor que o ônibus. indubitável.
menor (mais pequeno): comparativo de superioridade. Ex.: vendo
Ele é o pior do grupo.
pior (mais mau): superlativo relativo de superioridade. b) Subjuntivo: apresenta o fato verbal de maneira duvidosa,
hipotética.
c) Alguns superlativos absolutos sintéticos que podem Ex.: que eu venda
apresentar dúvidas.
acre − acérrimo c) Imperativo: apresenta o fato verbal como objeto de uma
amargo − amaríssimo ordem.
amigo − amicíssimo Ex.: venda!
antigo − antiquíssimo
cruel − crudelíssimo 4) Tempos: são três.
doce − dulcíssimo a) Presente: falo
fácil − facílimo
feroz − ferocíssimo b) Pretérito
fiel − fidelíssimo perfeito: falei
geral − generalíssimo imperfeito: falava

Língua Portuguesa 12
Apostila Digital Licenciada para Alice Caroline Guarino dos Santos - alice.guarino@hotmail.com (Proibida a Revenda)
APOSTILAS OPÇÃO
mais-que-perfeito: falara Ex.: faze (tu) ou faz (tu)
dize (tu) ou diz (tu)
Obs.: O pretérito perfeito indica uma ação extinta; o e) Procure ter “na ponta da língua” a formação e o emprego
imperfeito, uma ação que se prolongava num determinado do imperativo. É assunto muito cobrado em concursos públicos.
ponto do passado; o mais-que-perfeito, uma ação passada em
relação a outra ação, também passada. Tempos primitivos e tempos derivados
Ex.: Eu cantei aquela música. (perfeito)
Eu cantava aquela música. (imperfeito) 1) O presente do indicativo é tempo primitivo. Da primeira
Quando ele chegou, eu já cantara. (mais-que-perfeito) pessoa do singular sai todo o presente do subjuntivo.
Ex.: digo → que eu diga, que tu digas, que ele diga etc.
c) Futuro dizes
do presente: estudaremos diz
do pretérito: estudaríamos Obs.: Isso não ocorre apenas com os poucos verbos que não
apresentam a desinência o na primeira pessoa do singular.
Obs.: No modo subjuntivo, com relação aos tempos simples, Ex.: eu sou → que eu seja
temos apenas o presente, o pretérito imperfeito e o futuro (sem eu sei → que eu saiba
divisão). Os tempos compostos serão estudados mais adiante.
2) O pretérito perfeito é tempo primitivo. Da segunda pessoa
5) Vozes: são três do singular saem:

a) Ativa: o sujeito pratica a ação verbal. a) o mais-que-perfeito.


Ex.: O carro derrubou o poste. Ex.: coubeste → coubera, couberas, coubera, coubéramos,
coubéreis, couberam
b) Passiva: o sujeito sofre a ação verbal.
analítica ou verbal: com o particípio e um verbo auxiliar. b) o imperfeito do subjuntivo.
Ex.: O poste foi derrubado pelo carro. Ex.: coubeste → coubesse, coubesses, coubesse, coubéssemos,
sintética ou pronominal: com o pronome apassivador se. coubésseis, coubessem
Ex.: Derrubou-se o poste.
c) o futuro do subjuntivo.
Obs.: Estudaremos bem o pronome apassivador (ou partícula Ex.: coubeste → couber, couberes, couber, coubermos,
apassivadora) na sétima lição: concordância verbal. couberdes, couberem

c) Reflexiva: o sujeito pratica e sofre a ação verbal; aparece 3) Do infinitivo impessoal derivam:
um pronome reflexivo.
Ex.: O garoto se machucou. a) o imperfeito do indicativo.
Ex.: caber → cabia, cabias, cabia, cabíamos, cabíeis, cabiam
Formação do imperativo
1) Afirmativo: tu e vós saem do presente do indicativo b) o futuro do presente.
menos a letra s; você, nós e vocês, do presente do subjuntivo. Ex.: caber → caberei, caberás, caberá, caberemos, cabereis,
Ex.: Imperativo afirmativo do verbo beber caberão
Bebo → beba
bebes → bebe (tu) bebas c) o futuro do pretérito.
bebe beba → beba (você) Ex.: caber → caberia, caberias, caberia, caberíamos, caberíeis,
bebemos bebamos → bebamos (nós) caberiam
bebeis → bebei (vós) bebais
bebem bebam → bebam (vocês) d) o infinitivo pessoal.
Reunindo, temos: bebe, beba, bebamos, bebei, bebam. Ex.: caber → caber, caberes, caber, cabermos, caberdes,
caberem
2) Negativo: sai do presente do subjuntivo mais a palavra
não. e) o gerúndio.
Ex.: beba Ex.: caber → cabendo
bebas → não bebas (tu)
beba → não beba (você) f) o particípio.
bebamos → não bebamos (nós) Ex.: caber → cabido
bebais → não bebais (vós)
bebam → não bebam (vocês) Tempos compostos
Assim, temos: não bebas, não beba, não bebamos, não
bebais, não bebam. Formam-se os tempos compostos com o verbo auxiliar (ter
ou haver) mais o particípio do verbo que se quer conjugar.
Observações
a) No imperativo não existe a primeira pessoa do singular, 1) Perfeito composto: presente do verbo auxiliar mais
eu; a terceira pessoa é você. particípio do verbo principal.
b) O verbo ser não segue a regra nas pessoas que saem do Ex.: tenho falado ou hei falado → perfeito composto do
presente do indicativo. Eis o seu imperativo: indicativo tenha falado ou haja falado → perfeito composto do
afirmativo: sê, seja, sejamos, sede, sejam subjuntivo
negativo: não sejas, não seja, não sejamos, não sejais, não
sejam 2) Mais-que-perfeito composto: imperfeito do auxiliar mais
c) O tratamento dispensado a alguém numa frase não pode particípio do principal.
mudar. Se começamos a tratar a pessoa por você, não podemos Ex.: tinha falado → mais-que-perfeito composto do indicativo
passar para tu, e vice-versa. tivesse falado → mais-que-perfeito composto do subjuntivo
Ex.: Pede agora a tua comida. (tratamento: tu)
Peça agora a sua comida. (tratamento: você) 3) Demais tempos: basta classificar o verbo auxiliar.
d) Os verbos que têm z no radical podem, no imperativo Ex.: terei falado → futuro do presente composto (terei é
afirmativo, perder também a letra e que aparece antes da futuro do presente)
desinência s.

Língua Portuguesa 13
Apostila Digital Licenciada para Alice Caroline Guarino dos Santos - alice.guarino@hotmail.com (Proibida a Revenda)
APOSTILAS OPÇÃO
Verbos irregulares comuns em concursos b) presente do subjuntivo: mobílie, mobílies, mobílie,
mobiliemos, mobilieis, mobíliem
É importante saber a conjugação dos verbos que seguem.
Eles estão conjugados apenas nas pessoas, tempos e modos mais 12) Polir, no presente do indicativo: pulo, pules, pule,
problemáticos. polimos, polis, pulem
1) Compor, repor, impor, expor, depor etc.: seguem
integralmente o verbo pôr. 13) Passear, recear, pentear, ladear (e todos os outros
Ex.: ponho → componho, imponho, deponho etc. terminados em ear)
pus → compus, repus, expus etc.
a) presente do indicativo: passeio, passeias, passeia,
2) Deter, conter, reter, manter etc.: seguem integralmente o passeamos, passeais, passeiam
verbo ter.
Ex.: tivermos → contivermos, mantivermos etc. b) presente do subjuntivo: passeie, passeies, passeie,
tiveste → retiveste, mantiveste etc. passeemos, passeeis, passeiem

3) Intervir, advir, provir, convir etc.: seguem integralmente Observações


o verbo vir. a) Os verbos desse grupo (importantíssimo) apresentam
Ex.: vierem → intervierem, provierem etc. o ditongo ei nas formas risotônicas, mas apenas nos dois
vim → intervim, convim etc presentes.
b) Os verbos estrear e idear apresentam ditongo aberto.
4) Rever, prever, antever etc.: seguem integralmente o verbo Ex.: estreio, estreias, estreia; ideio, ideias, ideia
ver.
Ex.: vi → revi, previ etc. 14) Confiar, renunciar, afiar, arriar etc.: verbos regulares.
víssemos → prevíssemos, antevíssemos etc. Ex.: confio, confias, confia, confiamos, confiais, confiam

Observações Observações
a) Como se vê nesses quatro itens iniciais, o verbo derivado a) Esses verbos não têm o ditongo ei nas formas risotônicas.
segue a conjugação do seu primitivo. Basta conjugar o verbo
primitivo e recolocar o prefixo. Há outros verbos que dão origem b) Mediar, ansiar, remediar, incendiar, odiar e intermediar,
a verbos derivados. Por exemplo, dizer, haver e fazer. Para eles, apesar de terminarem em iar, apresentam o ditongo ei.
vale a mesma regra explicada acima. Ex.: medeio, medeias, medeia, mediamos, mediais, medeiam
Ex.: eu houve → eu reouve (e não reavi, como normalmente medeie, medeies, medeie, mediemos, medieis, medeiem
se fala por aí) 15) Requerer: só é irregular na 1ª pessoa do singular do
presente do indicativo e,
b) Requerer e prover não seguem integralmente os verbos consequentemente, em todo o presente do subjuntivo.
querer e ver. Eles serão mostrados mais adiante. Ex.: requeiro, requeres, requer
requeira, requeiras, requeira
5) Crer, no pretérito perfeito do indicativo: cri, creste, creu, requeri, requereste, requereu
cremos, crestes, creram.
16) Prover: conjuga-se como verbo regular no pretérito
6) Estourar, roubar, aleijar, inteirar etc.: mantém o ditongo perfeito, no mais-que-perfeito, no imperfeito do subjuntivo,
fechado em todos os tempos, inclusive o presente do indicativo. no futuro do subjuntivo e no particípio; nos demais tempos,
Ex.: A bomba estoura. (e não estóra, como normalmente se acompanha o verbo ver.
diz) Ex.: Provi, proveste, proveu; provera, proveras, provera;
Eu inteiro (e não intéro) provesse, provesses, provesse etc.
provejo, provês, provê; provia, provias, provia; proverei,
7) Aderir, competir, preterir, discernir, concernir, impelir, proverás, proverá etc.
expelir, repelir:
a) presente do indicativo: adiro, aderes, adere, aderimos, 17) Reaver, precaver-se, falir, adequar, remir, abolir, colorir,
aderimos, aderem. ressarcir, demolir,
acontecer, doer são verbos defectivos. Estude o que falamos
b) presente do subjuntivo: adira, adiras, adira, adiramos, sobre eles na lição anterior, no item sobre a classificação dos
adirais, adiram. verbos.
Ex.: Reaver, no presente do indicativo: reavemos, reaveis
Obs.: Esses verbos mudam o e do infinitivo para i na primeira
pessoa do singular do presente do indicativo e em todas do Questões
presente do subjuntivo.
1) Marque o erro de flexão verbal.
8) Aguar, desaguar, enxaguar, minguar: a) Teus amigos só veem problemas na empresa.
a) presente do indicativo: águo, águas, água; enxáguo, b) Eles vêm cedo para o trabalho.
enxáguas, enxágua c) Se nós virmos a solução, a brincadeira perderá a graça.
d) Viemos agora tentar um acordo.
b) presente do subjuntivo: águe, águes, águe; enxágue,
enxágues, enxágue 2) Assinale a única forma verbal correta.
a) Tudo que ele contradizer deve ser analisado.
9) Arguir, no presente do indicativo: arguo, argúis, argúi, b) Se o guarda retesse o trânsito, haveria enorme
arguimos, arguis, argúem engarrafamento.
c) Carlos preveu uma desgraça.
10) Apaziguar, averiguar, obliquar, no presente do d) Eu não intervinha no seu trabalho.
subjuntivo: apazigúe, apazigúes, apazigúe, apaziguemos,
apazigueis, apazigúem 3) Aponte a frase sem erro no que toca à flexão verbal.
a) Os funcionários reporam a mercadoria.
11) Mobiliar: b) Se ele manter a calma, poderá ser aprovado.
a) presente do indicativo: mobílio, mobílias, mobília, c) Quando eu revesse o processo, acharia o erro.
mobiliamos, mobiliais, mobíliam d) Àquela altura, já tínhamos intervindo na conversa.

Língua Portuguesa 14
Apostila Digital Licenciada para Alice Caroline Guarino dos Santos - alice.guarino@hotmail.com (Proibida a Revenda)
APOSTILAS OPÇÃO
Respostas Verbos Transitivos Diretos
1-D / 2-D / 3-B Os verbos transitivos diretos são complementados por
objetos diretos. Isso significa que não exigem preposição para
o estabelecimento da relação de regência. Ao empregar esses
4. regência e concordância; verbos, devemos lembrar que os pronomes oblíquos o, a, os,
as atuam como objetos diretos. Esses pronomes podem assumir
as formas lo, los, la, las (após formas verbais terminadas em -r,
-s ou -z) ou no, na, nos, nas (após formas verbais terminadas em
Regência Verbal e Nominal sons nasais), enquanto lhe e lhes são, quando complementos
verbais, objetos indiretos.
Dá-se o nome de regência à relação de subordinação que São verbos transitivos diretos, dentre outros: abandonar,
ocorre entre um verbo (ou um nome) e seus complementos. abençoar, aborrecer, abraçar, acompanhar, acusar, admirar,
Ocupa-se em estabelecer relações entre as palavras, criando adorar, alegrar, ameaçar, amolar, amparar, auxiliar, castigar,
frases não ambíguas, que expressem efetivamente o sentido condenar, conhecer, conservar,convidar, defender, eleger, estimar,
desejado, que sejam corretas e claras. humilhar, namorar, ouvir, prejudicar, prezar, proteger, respeitar,
socorrer, suportar, ver, visitar.
Regência Verbal Na língua culta, esses verbos funcionam exatamente como o
verbo amar:
Termo Regente:  VERBO Amo aquele rapaz. / Amo-o.
Amo aquela moça. / Amo-a.
A regência verbal estuda a relação que se estabelece entre Amam aquele rapaz. / Amam-no.
os verbos e os termos que os complementam (objetos diretos e Ele deve amar aquela mulher. / Ele deve amá-la.
objetos indiretos) ou caracterizam (adjuntos adverbiais).
O estudo da regência verbal permite-nos ampliar nossa Obs.: os pronomes lhe, lhes só acompanham esses verbos para
capacidade expressiva, pois oferece oportunidade de indicar posse (caso em que atuam como adjuntos adnominais).
conhecermos as diversas significações que um verbo pode Quero beijar-lhe o rosto. (= beijar seu rosto)
assumir com a simples mudança ou retirada de uma preposição. Prejudicaram-lhe a carreira. (= prejudicaram sua carreira)
Observe: Conheço-lhe o mau humor! (= conheço seu mau humor)
A mãe agrada o filho. -> agradar significa acariciar, contentar.
A mãe agrada ao filho. -> agradar significa “causar agrado ou Verbos Transitivos Indiretos
prazer”, satisfazer. Os verbos transitivos indiretos são complementados por
objetos indiretos. Isso significa que esses verbos exigem uma
Logo, conclui-se que “agradar alguém” é diferente de preposição para o estabelecimento da relação de regência.
“agradar a alguém”. Os pronomes pessoais do caso oblíquo de terceira pessoa que
podem atuar como objetos indiretos são o “lhe”, o “lhes”, para
Saiba que: substituir pessoas. Não se utilizam os pronomes o, os, a, as como
O conhecimento do uso adequado das preposições é um complementos de verbos transitivos indiretos. Com os objetos
dos aspectos fundamentais do estudo da regência verbal (e indiretos que não representam pessoas, usam-se pronomes
também nominal). As preposições são capazes de modificar oblíquos tônicos de terceira pessoa (ele, ela) em lugar dos
completamente o sentido do que se está sendo dito. Veja os pronomes átonos lhe, lhes.
exemplos:
Cheguei ao metrô. Os verbos transitivos indiretos são os seguintes:
Cheguei no metrô. a) Consistir - Tem complemento introduzido pela
preposição “em”.
No primeiro caso, o metrô é o lugar a que vou; no segundo A modernidade verdadeira consiste em direitos iguais para
caso, é o meio de transporte por mim utilizado. A oração “Cheguei todos.
no metrô”, popularmente usada a fim de indicar o lugar a que se b) Obedecer e Desobedecer - Possuem seus complementos
vai, possui, no padrão culto da língua, sentido diferente. Aliás, é introduzidos pela preposição “a”.
muito comum existirem divergências entre a regência coloquial, Devemos obedecer aos nossos princípios e ideais.
cotidiana de alguns verbos, e a regência culta. Eles desobedeceram às leis do trânsito.
c) Responder - Tem complemento introduzido pela
Para estudar a regência verbal, agruparemos os verbos de preposição “a”. Esse verbo pede objeto indireto para indicar “a
acordo com sua transitividade. A transitividade, porém, não é quem” ou “ao que” se responde.
um fato absoluto: um mesmo verbo pode atuar de diferentes Respondi ao meu patrão.
formas em frases distintas. Respondemos às perguntas.
Respondeu-lhe à altura.
Verbos Intransitivos Obs.: o verbo responder, apesar de transitivo indireto
Os verbos intransitivos não possuem complemento. É quando exprime aquilo a que se responde, admite voz passiva
importante, no entanto, destacar alguns detalhes relativos analítica. Veja:
aos adjuntos adverbiais que costumam acompanhá-los. O questionário foi respondido corretamente.
a) Chegar, Ir Todas as perguntas foram respondidas satisfatoriamente.
Normalmente vêm acompanhados de adjuntos adverbiais d) Simpatizar e Antipatizar - Possuem seus complementos
de lugar. Na língua culta, as preposições usadas para introduzidos pela preposição “com”.
indicar destino ou direção são: a, para. Antipatizo com aquela apresentadora.
Fui ao teatro. Simpatizo com os que condenam os políticos que governam
      Adjunto Adverbial de Lugar para uma minoria privilegiada.

Ricardo foi para a Espanha. Verbos Transitivos Diretos e Indiretos


                  Adjunto Adverbial de Lugar Os verbos transitivos diretos e indiretos são acompanhados
b) Comparecer de um objeto direto e um indireto. Merecem destaque, nesse
O adjunto adverbial de lugar pode ser introduzido grupo:
por em ou a.
Comparecemos ao estádio (ou no estádio) para ver o último Agradecer, Perdoar e Pagar
jogo. São verbos que apresentam objeto direto
relacionado a coisas e objeto indireto relacionado a pessoas.

Língua Portuguesa 15
Apostila Digital Licenciada para Alice Caroline Guarino dos Santos - alice.guarino@hotmail.com (Proibida a Revenda)
APOSTILAS OPÇÃO
Veja os exemplos: diferentes regências desses verbos é um recurso linguístico
Agradeço   aos ouvintes         a audiência. muito importante, pois além de permitir a correta interpretação
                   Objeto Indireto      Objeto Direto de passagens escritas, oferece possibilidades expressivas a
Cristo ensina que é preciso perdoar     o pecado       ao pecador. quem fala ou escreve. Dentre os principais, estão:
                                                                Obj. Direto       Objeto Indireto
Paguei     o débito        ao cobrador. AGRADAR
               Objeto Direto      Objeto Indireto 1) Agradar é transitivo direto no sentido de fazer carinhos,
acariciar.
- O uso dos pronomes oblíquos átonos deve ser feito com Sempre agrada o filho quando o revê. / Sempre o agrada
particular cuidado. Observe: quando o revê.
Agradeci o presente. / Agradeci-o. Cláudia não perde oportunidade de agradar o gato. / Cláudia
Agradeço a você. / Agradeço-lhe. não perde oportunidade de agradá-lo.
Perdoei a ofensa. / Perdoei-a.
Perdoei ao agressor. / Perdoei-lhe. 2) Agradar é transitivo indireto no sentido de causar agrado
Paguei minhas contas. / Paguei-as. a, satisfazer, ser agradável a. Rege complemento introduzido
Paguei aos meus credores. / Paguei-lhes. pela preposição “a”.
O cantor não agradou aos presentes.
Informar O cantor não lhes agradou.
- Apresenta objeto direto ao se referir a coisas e objeto
indireto ao se referir a pessoas, ou vice-versa. ASPIRAR
Informe os novos preços aos clientes. 1) Aspirar é transitivo direto no sentido de sorver, inspirar
Informe os clientes dos novos preços. (ou sobre os novos (o ar), inalar.
preços) Aspirava o suave aroma. (Aspirava-o)

- Na utilização de pronomes como complementos, veja as 2) Aspirar é transitivo indireto no sentido de desejar, ter
construções: como ambição.
Informei-os aos clientes. / Informei-lhes os novos preços. Aspirávamos a melhores condições de vida. (Aspirávamos a
Informe-os dos novos preços. / Informe-os deles. (ou sobre elas)
eles) Obs.: como o objeto direto do verbo “aspirar” não é pessoa,
Obs.: a mesma regência do verbo  informar é usada para os mas coisa, não se usam as formas pronominais átonas “lhe”
seguintes: avisar, certificar, notificar, cientificar, prevenir. e “lhes” e sim as formas tônicas “a ele (s)”, “ a ela (s)”. Veja o
exemplo:
Comparar Aspiravam a uma existência melhor. (= Aspiravam a ela)
Quando seguido de dois objetos, esse verbo admite as
preposições “a” ou “com” para introduzir o complemento ASSISTIR
indireto. 1) Assistir é transitivo direto no sentido de ajudar, prestar
Comparei seu comportamento ao (ou com o) de uma criança. assistência a, auxiliar. Por Exemplo:
As empresas de saúde negam-se a assistir os idosos.
Pedir As empresas de saúde negam-se a assisti-los.
Esse verbo pede objeto direto de coisa (geralmente na forma
de oração subordinada substantiva) e indireto de pessoa. 2) Assistir é transitivo indireto no sentido de ver, presenciar,
Pedi-lhe                 favores. estar presente, caber, pertencer.
Objeto Indireto    Objeto Direto
                                      Exemplos:
Pedi-lhe                     que mantivesse em silêncio. Assistimos ao documentário.
Objeto Indireto           Oração Subordinada Substantiva Não assisti às últimas sessões.
                                                           Objetiva Direta Essa lei assiste ao inquilino.
Obs.: no sentido de morar, residir, o verbo “assistir” é
Saiba que: intransitivo, sendo acompanhado de adjunto adverbial de lugar
1) A construção “pedir para”, muito comum na linguagem introduzido pela preposição “em”.
cotidiana, deve ter emprego muito limitado na língua culta. No Assistimos numa conturbada cidade.
entanto, é considerada correta quando a palavra licença estiver
subentendida. CHAMAR
Peço (licença) para ir entregar-lhe os catálogos em casa. 1) Chamar é transitivo direto no sentido de convocar,
Observe que, nesse caso, a preposição “para” introduz uma solicitar a atenção ou a presença de.
oração subordinada adverbial final reduzida de infinitivo (para Por gentileza, vá chamar sua prima. / Por favor, vá chamá-la.
ir entregar-lhe os catálogos em casa). Chamei você várias vezes. / Chamei-o várias vezes.
2) A construção “dizer para”, também muito usada
popularmente, é igualmente considerada incorreta. 2) Chamar no sentido de denominar, apelidar pode
apresentar objeto direto e indireto, ao qual se refere predicativo
Preferir preposicionado ou não.
Na língua culta, esse verbo deve apresentar objeto A torcida chamou o jogador mercenário.
indireto introduzido pela preposição “a”. Por Exemplo: A torcida chamou ao jogador mercenário.
Prefiro qualquer coisa a abrir mão de meus ideais. A torcida chamou o jogador de mercenário.
Prefiro trem a ônibus. A torcida chamou ao jogador de mercenário.
Obs.: na língua culta, o verbo “preferir” deve ser usado sem
termos intensificadores, tais como: muito, antes, mil vezes, um CUSTAR
milhão de vezes, mais. A ênfase já é dada pelo prefixo existente 1) Custar é intransitivo no sentido de ter determinado valor
no próprio verbo (pre). ou preço, sendo acompanhado de adjunto adverbial.
Frutas e verduras não deveriam custar muito.
Mudança de Transitividade versus Mudança de
Significado 2) No sentido de ser difícil, penoso, pode ser intransitivo ou
transitivo indireto.
Há verbos que, de acordo com a mudança de transitividade, Muito custa        viver tão longe da família.
apresentam mudança de significado. O conhecimento das             Verbo Oração Subordinada Substantiva Subjetiva

Língua Portuguesa 16
Apostila Digital Licenciada para Alice Caroline Guarino dos Santos - alice.guarino@hotmail.com (Proibida a Revenda)
APOSTILAS OPÇÃO
       Intransitivo                       Reduzida de Infinitivo (B) Eles assistem o espetáculo.
(C) João gosta de frutas.
Custa-me (a mim) crer que tomou realmente aquela atitude. (D) Ana reside em São Paulo.
        Objeto                 Oração Subordinada Substantiva Subjetiva (E) Pedro aspira ao cargo de chefe.
        Indireto                                     Reduzida de Infinitivo
02. Assinale a opção em que o verbo
Obs.: a Gramática Normativa condena as construções que chamar é empregado com o mesmo sentido que
atribuem ao verbo “custar” um sujeito representado por pessoa. apresenta em __ “No dia em que o chamaram de Ubirajara,
Observe o exemplo abaixo: Quaresma ficou reservado, taciturno e mudo”:
Custei para entender o problema. (A) pelos seus feitos, chamaram-lhe o salvador da pátria;
Forma correta: Custou-me entender o problema. (B) bateram à porta, chamando Rodrigo;
(C) naquele momento difícil, chamou por Deus e pelo Diabo;
IMPLICAR (D) o chefe chamou-os para um diálogo franco;
1) Como transitivo direto, esse verbo tem dois sentidos: (E) mandou chamar o médico com urgência.

a) dar a entender, fazer supor, pressupor 03. A regência verbal está correta na alternativa:
Suas atitudes implicavam um firme propósito. (A) Ela quer namorar com o meu irmão.
(B) Perdi a hora da entrevista porque fui à pé.
b) Ter como consequência, trazer como consequência, (C) Não pude fazer a prova do concurso porque era de menor.
acarretar, provocar (D) É preferível ir a pé a ir de carro.
Liberdade de escolha implica amadurecimento político de um
povo. Respostas
01. B\02. A\03. D
2) Como transitivo direto e indireto, significa comprometer,
envolver Regência Nominal
Implicaram aquele jornalista em questões econômicas.    
É o nome da relação existente entre um nome (substantivo,
Obs.: no sentido de antipatizar, ter implicância, é transitivo adjetivo ou advérbio) e os termos regidos por esse nome. Essa
indireto e rege com preposição “com”. relação é sempre intermediada por uma preposição. No estudo
Implicava com quem não trabalhasse arduamente. da regência nominal, é preciso levar em conta que vários nomes
apresentam exatamente o mesmo regime dos verbos de que
PROCEDER derivam. Conhecer o regime de um verbo significa, nesses casos,
1) Proceder é intransitivo no sentido de ser decisivo, conhecer o regime dos nomes cognatos. Observe o exemplo:
ter cabimento, ter fundamento ou portar-se, comportar-se, Verbo obedecer e os nomes correspondentes: todos regem
agir. Nessa segunda acepção, vem sempre acompanhado de complementos introduzidos pela preposição «a”.Veja:
adjunto adverbial de modo.
As afirmações da testemunha procediam, não havia como Obedecer a algo/ a alguém.
refutá-las. Obediente a algo/ a alguém.
Você procede muito mal.
Apresentamos a seguir vários nomes acompanhados
2) Nos sentidos de ter origem, derivar-se (rege a preposição” da preposição ou preposições que os regem. Observe-os
de”) e fazer, executar (rege complemento introduzido pela atentamente e procure, sempre que possível, associar esses
preposição “a”) é transitivo indireto. nomes entre si ou a algum verbo cuja regência você conhece.
O avião procede de Maceió.
Procedeu-se aos exames. Substantivos
O delegado procederá ao inquérito. Admiração a, por
Devoção a, para, com, por
QUERER Medo a, de
1) Querer é transitivo direto no sentido de desejar, ter Aversão a, para, por
vontade de, cobiçar. Doutor em
Querem melhor atendimento. Obediência a
Queremos um país melhor. Atentado a, contra
Dúvida acerca de, em, sobre
2) Querer é transitivo indireto no sentido de ter afeição, Ojeriza a, por
estimar, amar. Bacharel em
Quero muito aos meus amigos. Horror a
Ele quer bem à linda menina. Proeminência sobre
Despede-se o filho que muito lhe quer. Capacidade de, para
Impaciência com
VISAR Respeito a, com, para com, por
1) Como transitivo direto, apresenta os sentidos de mirar,
fazer pontaria e de pôr visto, rubricar. Adjetivos
O homem visou o alvo. Acessível a
O gerente não quis visar o cheque. Diferente de
Necessário a
2) No sentido de ter em vista, ter como meta, ter como Acostumado a, com
objetivo, é transitivo indireto e rege a preposição “a”. Entendido em
O ensino deve sempre visar ao progresso social. Nocivo a
Prometeram tomar medidas que visassem ao bem-estar Afável com, para com
público. Equivalente a
Questões Paralelo a
Agradável a
01. Todas as alternativas estão corretas quanto ao emprego Escasso de
correto da regência do verbo, EXCETO: Parco em, de
(A) Faço entrega em domicílio. Alheio a, de

Língua Portuguesa 17
Apostila Digital Licenciada para Alice Caroline Guarino dos Santos - alice.guarino@hotmail.com (Proibida a Revenda)
APOSTILAS OPÇÃO
Essencial a, para chegou
Passível de Temos que o verbo apresenta-se na terceira pessoa do
Análogo a singular, pois faz referência a um sujeito, assim também expresso
Fácil de (ele). Como poderíamos também dizer: os alunos chegaram
Preferível a atrasados.
Ansioso de, para, por Temos aí o que podemos chamar de princípio básico.
Fanático por Contudo, a intenção a que se presta o artigo em evidência é
Prejudicial a eleger as principais ocorrências voltadas para os casos de sujeito
Apto a, para simples e para os de sujeito composto. Dessa forma, vejamos:
Favorável a
Prestes a Casos referentes a sujeito simples
Ávido de
Generoso com 1) Em caso de sujeito simples, o verbo concorda com o
Propício a núcleo em número e pessoa: O aluno chegou atrasado.
Benéfico a
Grato a, por 2) Nos casos referentes a sujeito representado por
Próximo a substantivo coletivo, o verbo permanece na terceira pessoa do
Capaz de, para singular: A multidão, apavorada, saiu aos gritos.
Hábil em Observação:
Relacionado com - No caso de o coletivo aparecer seguido de adjunto adnominal
Compatível com no plural, o verbo permanecerá no singular ou poderá ir para o
Habituado a plural: Uma multidão de pessoas saiu aos gritos.
Relativo a Uma multidão de pessoas saíram aos gritos.
Contemporâneo a, de
Idêntico a 3) Quando o sujeito é representado por expressões partitivas,
representadas por “a maioria de, a maior parte de, a metade de,
Advérbios uma porção de, entre outras”, o verbo tanto pode concordar
Longe de Perto de com o núcleo dessas expressões quanto com o substantivo
que a segue: A maioria dos alunos resolveu ficar.   A maioria
Obs.: os advérbios terminados em -mente tendem a seguir dos alunos resolveram ficar.
o regime dos adjetivos de que são formados: paralela a;
paralelamente a; relativa a; relativamente a. 4) No caso de o sujeito ser representado por expressões
Fonte: http://www.soportugues.com.br/secoes/sint/sint61.php aproximativas, representadas por “cerca de, perto de”, o verbo
concorda com o substantivo determinado por elas: Cerca de
Questões vinte candidatos se inscreveram no concurso de piadas.

01. Assinale a alternativa em que a preposição “a” não deva 5) Em casos em que o sujeito é representado pela expressão
ser empregada, de acordo com a regência nominal. “mais de um”, o verbo permanece no singular: Mais de
(A) A confiança é necessária ____ qualquer relacionamento. um candidato se inscreveu no concurso de piadas.  
(B) Os pais de Pâmela estão alheios ____ qualquer decisão. Observação:
(C) Sirlene tem horror ____ aves. - No caso da referida expressão aparecer repetida ou
(D) O diretor está ávido ____ melhores metas. associada a um verbo que exprime reciprocidade, o verbo,
(E) É inegável que a tecnologia ficou acessível ____ toda necessariamente, deverá permanecer no plural: Mais de um
população. aluno, mais de um professor contribuíram na campanha de
doação de alimentos.
02. Quanto a amigos, prefiro João.....Paulo,.....quem sinto...... Mais de um formando se abraçaram durante as solenidades
simpatia. de formatura.
(A) a, por, menos
(B) do que, por, menos 6) Quando o sujeito for composto da expressão “um dos
(C) a, para, menos que”, o verbo permanecerá no plural: Esse jogador foi um dos
(D) do que, com, menos que atuaram na Copa América.
(E) do que, para, menos
7) Em casos relativos à concordância com locuções
03. Assinale a opção em que todos adjetivos podem ser pronominais, representadas por “algum de nós, qual de vós,
seguidos pela mesma preposição: quais de vós, alguns de nós”, entre outras, faz-se necessário nos
(A) ávido, bom, inconsequente atermos a duas questões básicas:
(B) indigno, odioso, perito - No caso de o primeiro pronome estar expresso no plural,
(C) leal, limpo, oneroso o verbo poderá com ele concordar, como poderá também
(D) orgulhoso, rico, sedento concordar com o pronome pessoal: Alguns de nós o receberemos.
(E) oposto, pálido, sábio / Alguns de nós o receberão.
- Quando o primeiro pronome da locução estiver expresso
Respostas no singular, o verbo permanecerá, também, no singular: Algum
01. D\02. A\03. D de nós o receberá.  

Concordância Verbal 8) No caso de o sujeito aparecer representado pelo pronome


“quem”, o verbo permanecerá na terceira pessoa do singular
Ao falarmos sobre a concordância verbal, estamos nos ou poderá concordar com o antecedente desse pronome:   
referindo à relação de dependência estabelecida entre um termo Fomos nós quem contou toda a verdade para ela. / Fomos
e outro mediante um contexto oracional. Desta feita, os agentes nós quem contamos toda a verdade para ela.
principais desse processo são representados pelo sujeito, que no
caso funciona como subordinante; e o verbo, o qual desempenha 9) Em casos nos quais o sujeito aparece realçado pela palavra
a função de subordinado. “que”, o verbo deverá concordar com o termo que antecede essa
Dessa forma, temos que a concordância verbal caracteriza- palavra: Nesta empresa somos nós que tomamos as decisões. /
se pela adaptação do verbo, tendo em vista os quesitos “número Em casa sou eu que decido tudo.   
e pessoa” em relação ao sujeito. Exemplificando, temos: O aluno

Língua Portuguesa 18
Apostila Digital Licenciada para Alice Caroline Guarino dos Santos - alice.guarino@hotmail.com (Proibida a Revenda)
APOSTILAS OPÇÃO
10) No caso de o sujeito aparecer representado por (C) Naquela barraca vendem-se tapiocas fresquinhas, pode
expressões que indicam porcentagens, o verbo concordará com o comê-las sem receio!
numeral ou com o substantivo a que se refere essa porcentagem:    (D) A multidão gritaram quando a cantora apareceu na
50% dos funcionários aprovaram a decisão da diretoria. / 50% janela do hotel!
do eleitorado apoiou a decisão.
Observações: 02. “Se os cachorros correm livremente, por que eu não
- Caso o verbo aparecer anteposto à expressão de posso fazer isso também?”, pergunta Bob Dylan em “New
porcentagem, esse deverá concordar com o numeral: Aprovaram Morning”. Bob Dylan verbaliza um anseio sentido por todos
a decisão da diretoria 50% dos funcionários.      nós, humanos supersocializados: o anseio de nos livrarmos
- Em casos relativos a 1%, o verbo permanecerá no singular: de todos os constrangimentos artificiais decorrentes do fato
1% dos funcionários não aprovou a decisão da diretoria.   de vivermos em uma sociedade civilizada em que às vezes nos
- Em casos em que o numeral estiver acompanhado de sentimos presos a uma correia. Um conjunto cultural de regras
determinantes no plural, o verbo permanecerá no plural: Os tácitas e inibições está sempre governando as nossas interações
50% dos funcionários apoiaram a decisão da diretoria. cotidianas com os outros.
Uma das razões pelas quais os cachorros nos atraem é o fato
11) Nos casos em que o sujeito estiver representado por de eles serem tão desinibidos e livres. Parece que eles jogam
pronomes de tratamento, o verbo deverá ser empregado na terceira com as suas próprias regras, com a sua própria lógica interna.
pessoa do singular ou do plural: Vossas Majestades gostaram das Eles vivem em um universo paralelo e diferente do nosso - um
homenagens. Vossa Majestade agradeceu o convite.   universo que lhes concede liberdade de espírito e paixão pela
vida enormemente atraentes para nós. Um cachorro latindo ao
12) Casos relativos a sujeito representado por substantivo vento ou uivando durante a noite faz agitar-se dentro de nós
próprio no plural se encontram relacionados a alguns aspectos alguma coisa que também quer se expressar.
que os determinam: Os cachorros são uma constante fonte de diversão para
- Diante de nomes de obras no plural, seguidos do verbo ser, nós porque não prestam atenção as nossas convenções sociais.
este permanece no singular, contanto que o predicativo também Metem o nariz onde não são convidados, pulam para cima
esteja no singular: Memórias póstumas de Brás Cubas é uma do sofá, devoram alegremente a comida que cai da mesa. Os
criação de Machado de Assis.    cachorros raramente se refreiam quando querem fazer alguma
- Nos casos de artigo expresso no plural, o verbo também coisa. Eles não compartilham conosco as nossas inibições. Suas
permanece no plural: Os Estados Unidos são uma potência emoções estão ã flor da pele e eles as manifestam sempre que
mundial. as sentem.
- Casos em que o artigo figura no singular ou em que ele nem (Adaptado de Matt Weistein e Luke Barber. Cão que
aparece, o verbo permanece no singular:  Estados Unidos é uma late não morde. Trad. de Cristina Cupertino. S.Paulo: Francis,
potência mundial. 2005. p 250)

Casos referentes a sujeito composto A frase em que se respeitam as normas de concordância


verbal é:
1) Nos casos relativos a sujeito composto de pessoas (A) Deve haver muitas razões pelas quais os cachorros nos
gramaticais diferentes, o verbo deverá ir para o plural, estando atraem.
relacionado a dois pressupostos básicos: (B) Várias razões haveriam pelas quais os cachorros nos
- Quando houver a 1ª pessoa, esta prevalecerá sobre as atraem.
demais: Eu, tu e ele faremos um lindo passeio. (C) Caberiam notar as muitas razões pelas quais os cachorros
- Quando houver a 2ª pessoa, o verbo poderá nos atraem.
flexionar na 2ª ou na 3ª pessoa: Tu e ele sois primos. (D) Há de ser diversas as razões pelas quais os cachorros nos
Tu e ele são primos. atraem.
(E) Existe mesmo muitas razões pelas quais os cachorros
2) Nos casos em que o sujeito composto aparecer anteposto nos atraem.
ao verbo, este permanecerá no plural: O pai e seus dois
filhos compareceram ao evento.   03. Uma pergunta

3) No caso em que o sujeito aparecer posposto ao verbo, este Frequentemente cabe aos detentores de cargos de
poderá concordar com o núcleo mais próximo ou permanecer responsabilidade tomar decisões difíceis, de graves
no plural: Compareceram ao evento o pai e seus dois filhos. consequências. Haveria algum critério básico, essencial, para
Compareceu ao evento o pai e seus dois filhos. amparar tais escolhas? Antonio Gramsci, notável pensador
e político italiano, propôs que se pergunte, antes de tomar a
4) Nos casos relacionados a sujeito simples, porém com decisão: - Quem sofrerá?
mais de um núcleo, o verbo deverá permanecer no singular: Para um humanista, a dor humana é sempre prioridade a se
Meu esposo e grande companheiro merece toda a felicidade do considerar.
mundo. (Salvador Nicola, inédito)

5) Casos relativos a sujeito composto de palavras sinônimas O verbo indicado entre parênteses deverá flexionar-se no
ou ordenado por elementos em gradação, o verbo poderá singular para preencher adequadamente a lacuna da frase:
permanecer no singular ou ir para o plural: Minha vitória, (A) A nenhuma de nossas escolhas ...... (poder) deixar de
minha conquista, minha premiação são frutos de meu esforço. corresponder nossos valores éticos mais rigorosos.
/ Minha vitória, minha conquista, minha premiação é fruto de (B) Não se ...... (poupar) os que governam de refletir sobre o
meu esforço. peso de suas mais graves decisões.
(C) Aos governantes mais responsáveis não ...... (ocorrer)
Questões tomar decisões sem medir suas consequências.
(D) A toda decisão tomada precipitadamente ...... (costumar)
01. A concordância realizou-se adequadamente em qual sobrevir consequências imprevistas e injustas.
alternativa? (E) Diante de uma escolha, ...... (ganhar) prioridade,
(A) Os Estados Unidos é considerado, hoje, a maior potência recomenda Gramsci, os critérios que levam em conta a dor
econômica do planeta, mas há quem aposte que a China, em humana.
breve, o ultrapassará.
(B) Em razão das fortes chuvas haverão muitos candidatos Respostas
que chegarão atrasados, tenho certeza disso. 01. C\02. A\03. C

Língua Portuguesa 19
Apostila Digital Licenciada para Alice Caroline Guarino dos Santos - alice.guarino@hotmail.com (Proibida a Revenda)
APOSTILAS OPÇÃO
Concordância Nominal Comi muitas frutas durante a viagem.
Pouco arroz é suficiente para mim.
Concordância nominal é que o ajuste que fazemos aos Os sapatos estavam caros.
demais termos da oração para que concordem em gênero e
número com o substantivo. Teremos que alterar, portanto, o 2- Como advérbios: são invariáveis.
artigo, o adjetivo, o numeral e o pronome. Além disso, temos Comi muito durante a viagem.
também o verbo, que se flexionará à sua maneira. Pouco lutei, por isso perdi a batalha.
Comprei caro os sapatos.
Regra geral: O artigo, o adjetivo, o numeral e o pronome
concordam em gênero e número com o substantivo. i) Mesmo, bastante
- A pequena criança é uma gracinha. 1- Como advérbios: invariáveis
- O garoto que encontrei era muito gentil e simpático. Preciso mesmo da sua ajuda.
Fiquei bastante contente com a proposta de emprego.
Casos especiais: Veremos alguns casos que fogem à regra 2- Como pronomes: seguem a regra geral.
geral mostrada acima. Seus argumentos foram bastantes para me convencer.
Os mesmos argumentos que eu usei, você copiou.
a) Um adjetivo após vários substantivos
1 - Substantivos de mesmo gênero: adjetivo vai para o plural j) Menos, alerta
ou concorda com o substantivo mais próximo. 1- Em todas as ocasiões são invariáveis.
- Irmão e primo recém-chegado estiveram aqui. Preciso de menos comida para perder peso.
- Irmão e primo recém-chegados estiveram aqui. Estamos alerta para com suas chamadas.

2 - Substantivos de gêneros diferentes: vai para o k) Tal Qual


plural masculino ou concorda com o substantivo mais próximo. 1- “Tal” concorda com o antecedente, “qual” concorda com o
- Ela tem pai e mãe louros. consequente.
- Ela tem pai e mãe loura. As garotas são vaidosas tais qual a tia.
Os pais vieram fantasiados tais quais os filhos.
3 - Adjetivo funciona como predicativo: vai obrigatoriamente
para o plural. l) Possível
- O homem e o menino estavam perdidos. 1- Quando vem acompanhado de “mais”, “menos”, “melhor”
- O homem e sua esposa estiveram hospedados aqui. ou “pior”, acompanha o artigo que precede as expressões.
A mais possível das alternativas é a que você expôs.
b) Um adjetivo anteposto a vários substantivos Os melhores cargos possíveis estão neste setor da empresa.
1 - Adjetivo anteposto normalmente concorda com o mais As piores situações possíveis são encontradas nas favelas da
próximo. cidade.
Comi delicioso almoço e sobremesa.
Provei deliciosa fruta e suco. m) Meio
2 - Adjetivo anteposto funcionando como predicativo: 1- Como advérbio: invariável.
concorda com o mais próximo ou vai para o plural. Estou meio (um pouco) insegura.
Estavam feridos o pai e os filhos. 2- Como numeral: segue a regra geral.
Estava ferido o pai e os filhos. Comi meia (metade) laranja pela manhã.

c) Um substantivo e mais de um adjetivo n) Só


1- antecede todos os adjetivos com um artigo. 1- apenas, somente (advérbio): invariável.
Falava fluentemente a língua inglesa e a espanhola. Só consegui comprar uma passagem.
2- coloca o substantivo no plural. 2- sozinho (adjetivo): variável.
Falava fluentemente as línguas inglesa e espanhola. Estiveram sós durante horas.

d) Pronomes de tratamento Questões


1 - sempre concordam com a 3ª pessoa.
Vossa Santidade esteve no Brasil. 01. Indique o uso INCORRETO da concordância verbal ou
nominal:
e) Anexo, incluso, próprio, obrigado (A) Será descontada em folha sua contribuição sindical.
1 - Concordam com o substantivo a que se referem. (B) Na última reunião, ficou acordado que se realizariam
As cartas estão anexas. encontros semanais com os diversos interessados no assunto.
A bebida está inclusa. (C) Alguma solução é necessária, e logo!
Precisamos de nomes próprios. (D) Embora tenha ficado demonstrado cabalmente a
Obrigado, disse o rapaz. ocorrência de simulação na transferência do imóvel, o pedido
não pode prosperar.
f) Um(a) e outro(a), num(a) e noutro(a) (E) A liberdade comercial da colônia, somada ao fato de D.
1 - Após essas expressões o substantivo fica sempre no João VI ter também elevado sua colônia americana à condição de
singular e o adjetivo no plural. Reino Unido a Portugal e Algarves, possibilitou ao Brasil obter
Renato advogou um e outro caso fáceis. certa autonomia econômica.
Pusemos numa e noutra bandeja rasas o peixe.
02. Aponte a alternativa em que NÃO ocorre silepse (de
g) É bom, é necessário, é proibido gênero, número ou pessoa):
1- Essas expressões não variam se o sujeito não vier (A) “A gente é feito daquele tipo de talento capaz de fazer a
precedido de artigo ou outro determinante. diferença.”
Canja é bom. / A canja é boa. (B) Todos sabemos que a solução não é fácil.
É necessário sua presença. / É necessária a sua presença. (C) Essa gente trabalhadora merecia mais, pois acordam às
É proibido entrada de pessoas não autorizadas. / A entrada cinco horas para chegar ao trabalho às oito da manhã.
é proibida. (D) Todos os brasileiros sabem que esse problema vem de
longe...
h) Muito, pouco, caro (E) Senhor diretor, espero que Vossa Senhoria seja mais
1- Como adjetivos: seguem a regra geral. compreensivo.

Língua Portuguesa 20
Apostila Digital Licenciada para Alice Caroline Guarino dos Santos - alice.guarino@hotmail.com (Proibida a Revenda)
APOSTILAS OPÇÃO
03. A concordância nominal está INCORRETA em: quem fala.
(A) A mídia julgou desnecessária a campanha e o Os pronomes pessoais variam de acordo com as funções
envolvimento da empresa. que exercem nas orações, podendo ser do caso reto ou do caso
(B) A mídia julgou a campanha e a atuação da empresa oblíquo.
desnecessária.
(C) A mídia julgou desnecessário o envolvimento da empresa Pronome Reto
e a campanha.
(D) A mídia julgou a campanha e a atuação da empresa Pronome pessoal do caso reto é aquele que, na sentença,
desnecessárias. exerce a função de sujeito ou predicativo do sujeito.
Respostas Nós lhe ofertamos flores.
01. D\02. D\03. B
Os pronomes retos apresentam flexão de número, gênero
5. emprego de pronomes e (apenas na 3ª pessoa) e pessoa, sendo essa última a principal
crases; flexão, uma vez que marca a pessoa do discurso. Dessa forma, o
quadro dos pronomes retos é assim configurado:
- 1ª pessoa do singular: eu
Pronome - 2ª pessoa do singular: tu
- 3ª pessoa do singular: ele, ela
Pronome é a palavra que se usa em lugar do nome, ou a ele - 1ª pessoa do plural: nós
se refere, ou ainda, que acompanha o nome qualificando-o de - 2ª pessoa do plural: vós
alguma forma. - 3ª pessoa do plural: eles, elas
A moça era mesmo bonita. Ela morava nos meus sonhos!
[substituição do nome] Atenção: esses pronomes não costumam ser usados como
complementos verbais na língua-padrão. Frases como “Vi
A moça que morava nos meus sonhos era mesmo bonita! ele na rua”, “Encontrei ela na praça”, “Trouxeram eu até aqui”,
[referência ao nome] comuns na língua oral cotidiana, devem ser evitadas na língua
formal escrita ou falada. Na língua formal, devem ser usados os
Essa moça morava nos meus sonhos! pronomes oblíquos correspondentes: “Vi-o na rua”, “Encontrei-a
[qualificação do nome] na praça”, “Trouxeram-me até aqui”.
Grande parte dos pronomes não possuem significados Obs.: frequentemente observamos a omissão do pronome
fixos, isto é, essas palavras só adquirem significação dentro de reto em Língua Portuguesa. Isso se dá porque as próprias formas
um contexto, o qual nos permite recuperar a referência exata verbais marcam, através de suas desinências, as pessoas do
daquilo que está sendo colocado por meio dos pronomes no verbo indicadas pelo pronome reto.
ato da comunicação. Com exceção dos pronomes interrogativos Fizemos boa viagem. (Nós)
e indefinidos, os demais pronomes têm por função principal
apontar para as pessoas do discurso ou a elas se relacionar, Pronome Oblíquo
indicando-lhes sua situação no tempo ou no espaço. Em virtude
dessa característica, os pronomes apresentam uma forma Pronome pessoal do caso oblíquo é aquele que, na sentença,
específica para cada pessoa do discurso. exerce a função de complemento verbal (objeto direto ou
indireto) ou complemento nominal.
Minha carteira estava vazia quando eu fui assaltada.
[minha/eu: pronomes de 1ª pessoa = aquele que fala] Ofertaram-nos flores. (objeto indireto)
Obs.: em verdade, o pronome oblíquo é uma forma variante
Tua carteira estava vazia quando tu foste assaltada? do pronome pessoal do caso reto. Essa variação indica a função
[tua/tu: pronomes de 2ª pessoa = aquele a quem se fala] diversa que eles desempenham na oração: pronome reto marca
o sujeito da oração; pronome oblíquo marca o complemento da
A carteira dela estava vazia quando ela foi assaltada. oração.
[dela/ela: pronomes de 3ª pessoa = aquele de quem se fala] Os pronomes oblíquos sofrem variação de acordo com
a acentuação tônica que possuem, podendo ser átonos ou tônicos.
Em termos morfológicos, os pronomes são palavras
variáveis em gênero (masculino ou feminino) e em número Pronome Oblíquo Átono
(singular ou plural). Assim, espera-se que a referência através
do pronome seja coerente em termos de gênero e número São chamados átonos os pronomes oblíquos que não são
(fenômeno da concordância) com o seu objeto, mesmo quando precedidos de preposição. Possuem acentuação tônica  fraca.
este se apresenta ausente no enunciado. Ele me deu um presente.

Fala-se de Roberta. Ele quer participar do desfile O quadro dos pronomes oblíquos átonos é assim configurado:
da nossa escola neste ano. - 1ª pessoa do singular (eu): me
[nossa: pronome que qualifica “escola” = concordância - 2ª pessoa do singular (tu): te
adequada] - 3ª pessoa do singular (ele, ela): o, a, lhe
[neste: pronome que determina “ano” = concordância - 1ª pessoa do plural (nós): nos
adequada] - 2ª pessoa do plural (vós): vos
[ele: pronome que faz referência à “Roberta” = concordância - 3ª pessoa do plural (eles, elas): os, as, lhes
inadequada]
Observações:
Existem seis tipos de pronomes: pessoais, possessivos, O “lhe” é o único pronome oblíquo átono que já se
demonstrativos, indefinidos, relativos e interrogativos. apresenta na forma contraída, ou seja, houve a união entre o
pronome “o” ou “a” e preposição “a” ou “para”. Por acompanhar
Pronomes Pessoais diretamente uma preposição, o pronome “lhe” exerce sempre a
função de objeto indireto na oração.
São aqueles que substituem os substantivos, indicando
diretamente as pessoas do discurso. Quem fala ou escreve Os pronomes me, te, nos e vos podem tanto ser objetos
assume os pronomes “eu” ou “nós”, usa os pronomes “tu”, “vós”, diretos como objetos indiretos.
“você” ou “vocês” para designar a quem se dirige e “ele”, “ela”, Os pronomes o, a, os e as atuam exclusivamente como
“eles” ou “elas” para fazer referência à pessoa ou às pessoas de objetos diretos.

Língua Portuguesa 21
Apostila Digital Licenciada para Alice Caroline Guarino dos Santos - alice.guarino@hotmail.com (Proibida a Revenda)
APOSTILAS OPÇÃO
Saiba que: Ele carregava o documento consigo.
Os pronomes me, te, lhe, nos, vos e lhes podem combinar-se
com os pronomes o, os, a, as, dando origem a formas como mo, - As formas “conosco” e “convosco” são substituídas por “com
mos, ma, mas; to, tos, ta, tas; lho, lhos, lha, lhas; no-lo, no-los, no- nós” e “com vós” quando os pronomes pessoais são reforçados
la, no-las, vo-lo, vo-los, vo-la, vo-las. Observe o uso dessas formas por palavras como outros, mesmos, próprios, todos, ambos ou
nos exemplos que seguem: algum numeral.

Você terá de viajar com nós todos.


- Trouxeste o pacote? - Não contaram a novidade a Estávamos com vós outros quando chegaram as más notícias.
vocês? Ele disse que iria com nós três.
- Sim, entreguei-to ainda há - Não, no-la contaram.
pouco. Pronome Reflexivo

No português do Brasil, essas combinações não são usadas; São pronomes pessoais oblíquos que, embora funcionem
até mesmo na língua literária atual, seu emprego é muito raro. como objetos direto ou indireto, referem-se ao sujeito da oração.
Indicam que o sujeito pratica e recebe a ação expressa pelo
Atenção: verbo.
Os pronomes o, os, a, as assumem formas especiais depois O quadro dos pronomes reflexivos é assim configurado:
de certas terminações verbais. Quando o verbo termina em -z,
-s ou -r, o pronome assume a forma lo, los, la ou las, ao mesmo - 1ª pessoa do singular (eu): me, mim.
tempo que a terminação verbal é suprimida. Eu não me vanglorio disso.
Por exemplo: fiz + o = fi-lo Olhei para mim no espelho e não gostei do que vi.
fazei + o = fazei-os
dizer + a = dizê-la - 2ª pessoa do singular (tu): te, ti.
Assim tu te prejudicas.
Quando o verbo termina em som nasal, o pronome assume Conhece a ti mesmo.
as formas no, nos, na, nas. Por exemplo:
viram + o: viram-no - 3ª pessoa do singular (ele, ela): se, si, consigo.
repõe + os = repõe-nos Guilherme já se preparou.
retém + a: retém-na Ela deu a si um presente.
tem + as = tem-nas Antônio conversou consigo mesmo.

Pronome Oblíquo Tônico - 1ª pessoa do plural (nós): nos.


Lavamo-nos no rio.
Os pronomes oblíquos tônicos são sempre
precedidos por preposições, em geral as preposições a, para, de - 2ª pessoa do plural (vós): vos.
e com. Por esse motivo, os pronomes tônicos exercem a função Vós vos beneficiastes com a esta conquista.
de objeto indireto da oração. Possuem acentuação tônica forte.
O quadro dos pronomes oblíquos tônicos é assim - 3ª pessoa do plural (eles, elas): se, si, consigo.
configurado: Eles se conheceram.
Elas deram a si um dia de folga.
- 1ª pessoa do singular (eu): mim, comigo
- 2ª pessoa do singular (tu): ti, contigo A Segunda Pessoa Indireta
- 3ª pessoa do singular (ele, ela): ele, ela
- 1ª pessoa do plural (nós): nós, conosco A chamada segunda pessoa indireta manifesta-se quando
- 2ª pessoa do plural (vós): vós, convosco utilizamos pronomes que, apesar de indicarem nosso
- 3ª pessoa do plural (eles, elas): eles, elas interlocutor ( portanto, a segunda pessoa), utilizam o verbo na
terceira pessoa. É o caso dos chamados pronomes de tratamento,
Observe que as únicas formas próprias do pronome tônico que podem ser observados no quadro seguinte:
são a primeira pessoa (mim) e segunda pessoa (ti). As demais
repetem a forma do pronome pessoal do caso reto. Pronomes de Tratamento
- As preposições essenciais introduzem sempre pronomes
pessoais do caso oblíquo e nunca pronome do caso reto. Nos Vossa Alteza V. A. príncipes, duques
contextos interlocutivos que exigem o uso da língua formal, os Vossa Eminência V. Ema.(s) cardeais
pronomes costumam ser usados desta forma: Vossa Reverendíssima V. Revma.(s) sacerdotes e bispos
Não há mais nada entre mim e ti. Vossa Excelência V. Ex.ª (s) altas autoridades e
Não se comprovou qualquer ligação entre ti e ela. oficiais-generais
Não há nenhuma acusação contra mim. Vossa Magnificência V. Mag.ª (s) reitores de
Não vá sem mim. universidades
Vossa Majestade V. M. reis e rainhas
Atenção: Vossa Majestade Imperial V. M. I. Imperadores
Há construções em que a preposição, apesar de surgir Vossa Santidade V. S. Papa
anteposta a um pronome, serve para introduzir uma oração cujo Vossa Senhoria V. S.ª (s) tratamento
verbo está no infinitivo. Nesses casos, o verbo pode ter sujeito cerimonioso
expresso; se esse sujeito for um pronome, deverá ser do caso Vossa Onipotência V. O. Deus
reto.
Também são pronomes de tratamento o senhor, a
Trouxeram vários vestidos para eu experimentar. senhora e você, vocês. “O senhor” e “a senhora” são empregados
Não vá sem eu mandar. no tratamento cerimonioso; “você” e “vocês”, no tratamento
familiar. Você e vocês são largamente empregados no português
- A combinação da preposição “com” e alguns pronomes do Brasil; em algumas regiões, a forma tu é de uso frequente;
originou as formas especiais comigo, contigo, consigo, em outras, pouco empregada. Já a forma vós tem uso restrito à
conosco e convosco. Tais pronomes oblíquos tônicos linguagem litúrgica, ultraformal ou literária.
frequentemente exercem a função de adjunto adverbial de
companhia. Observações:

Língua Portuguesa 22
Apostila Digital Licenciada para Alice Caroline Guarino dos Santos - alice.guarino@hotmail.com (Proibida a Revenda)
APOSTILAS OPÇÃO
a) Vossa Excelência X Sua Excelência: os pronomes de Marisa tem lá seus defeitos, mas eu gosto muito dela.
tratamento que possuem “Vossa (s)” são empregados em
relação à pessoa com quem falamos. 3- Em frases onde se usam pronomes de tratamento, o
Espero que V. Ex.ª, Senhor Ministro, compareça a este pronome possessivo fica na 3ª pessoa.
encontro. Vossa Excelência trouxe sua mensagem?
Emprega-se “Sua (s)” quando se fala a respeito da pessoa.
Todos os membros da C.P.I. afirmaram que Sua Excelência, o 4- Referindo-se a mais de um substantivo, o possessivo
Senhor Presidente da República, agiu com propriedade. concorda com o mais próximo.
Trouxe-me seus livros e anotações.
- Os pronomes de tratamento representam uma forma
indireta de nos dirigirmos aos nossos interlocutores. Ao 5- Em algumas construções, os pronomes pessoais oblíquos
tratarmos um deputado por Vossa Excelência, por exemplo, átonos assumem valor de possessivo.
estamos nos endereçando à excelência que esse deputado Vou seguir-lhe os passos. (= Vou seguir seus passos.)
supostamente tem para poder ocupar o cargo que ocupa.
Pronomes Demonstrativos
b) 3ª pessoa: embora os pronomes de tratamento dirijam-
se à 2ª pessoa, toda a concordância deve ser feita com a 3ª Os pronomes demonstrativos são utilizados para explicitar a
pessoa. Assim, os verbos, os pronomes possessivos e os posição de uma certa palavra em relação a outras ou ao contexto.
pronomes oblíquos empregados em relação a eles devem ficar Essa relação pode ocorrer em termos de espaço, no tempo ou
na 3ª pessoa. discurso.
Basta que V. Ex.ª cumpra a terça parte das suas promessas,
para que seus eleitores lhe fiquem reconhecidos. No espaço:
Compro este carro (aqui). O pronome este indica que o carro
c) Uniformidade de Tratamento: quando escrevemos ou está perto da pessoa que fala.
nos dirigimos a alguém, não é permitido mudar, ao longo do Compro esse carro (aí). O pronome esse indica que o carro
texto, a pessoa do tratamento escolhida inicialmente. Assim, está perto da pessoa com quem falo, ou afastado da pessoa que
por exemplo, se começamos a chamar alguém de “você”, não fala.
poderemos usar “te” ou “teu”. O uso correto exigirá, ainda, verbo Compro aquele carro (lá). O pronome aquele diz que o carro
na terceira pessoa. está afastado da pessoa que fala e daquela com quem falo.
Quando você vier, eu te abraçarei e enrolar-me-ei nos teus
cabelos. (errado) Atenção: em situações de fala direta (tanto ao vivo quanto
Quando você vier, eu a abraçarei e enrolar-me-ei nos seus por meio de correspondência, que é uma modalidade escrita de
cabelos. (correto) fala), são particularmente importantes o este e o esse - o primeiro
Quando tu vieres, eu te abraçarei e enrolar-me-ei nos teus localiza os seres em relação ao emissor; o segundo, em relação
cabelos. (correto) ao destinatário. Trocá-los pode causar ambiguidade.

Pronomes Possessivos Dirijo-me a essa universidade com o objetivo de solicitar


informações sobre o concurso vestibular. (trata-se da universidade
São palavras que, ao indicarem a pessoa gramatical destinatária).
(possuidor), acrescentam a ela a ideia de posse de algo (coisa Reafirmamos a disposição desta universidade em participar
possuída). no próximo Encontro de Jovens. (trata-se da universidade que
Este caderno é meu. (meu = possuidor: 1ª pessoa do singular) envia a mensagem).

Observe o quadro: No tempo:


Este ano está sendo bom para nós. O pronome este se refere
Número Pessoa Pronome ao ano presente.
singular primeira meu(s), minha(s) Esse ano que passou foi razoável. O pronome esse se refere a
um passado próximo.
singular segunda teu(s), tua(s) Aquele ano foi terrível para todos. O pronome aquele está se
singular terceira seu(s), sua(s) referindo a um passado distante.
plural primeira nosso(s), nossa(s) - Os pronomes demonstrativos podem ser variáveis ou
plural segunda vosso(s), vossa(s) invariáveis, observe:
plural terceira seu(s), sua(s) Variáveis: este(s), esta(s), esse(s), essa(s), aquele(s), aquela(s).
Invariáveis: isto, isso, aquilo.
Note que: A forma do possessivo depende da pessoa
gramatical a que se refere; o gênero e o número concordam com - Também aparecem como pronomes demonstrativos:
o objeto possuído. - o(s), a(s): quando estiverem antecedendo o “que” e puderem
Ele trouxe seu apoio e sua contribuição naquele momento ser substituídos por aquele(s), aquela(s), aquilo.
difícil. Não ouvi o que disseste. (Não ouvi aquilo que disseste.)
Essa rua não é a que te indiquei. (Esta rua não é aquela que
Observações: te indiquei.)
- mesmo(s), mesma(s):
1 - A forma “seu” não é um possessivo quando resultar da Estas são as mesmas pessoas que o procuraram ontem.
alteração fonética da palavra senhor. - próprio(s), própria(s):
- Muito obrigado, seu José. Os próprios alunos resolveram o problema.
2 - Os pronomes possessivos nem sempre indicam posse. - semelhante(s):
Podem ter outros empregos, como: Não compre semelhante livro.
a) indicar afetividade. - tal, tais:
- Não faça isso, minha filha. Tal era a solução para o problema.
b) indicar cálculo aproximado.
Ele já deve ter seus 40 anos. Note que:
c) atribuir valor indefinido ao substantivo.

Língua Portuguesa 23
Apostila Digital Licenciada para Alice Caroline Guarino dos Santos - alice.guarino@hotmail.com (Proibida a Revenda)
APOSTILAS OPÇÃO
a) Não raro os demonstrativos aparecem na frase, em São locuções pronominais indefinidas: cada qual, cada um,
construções redundantes, com finalidade expressiva, para qualquer um, quantos quer (que), quem quer (que), seja quem for,
salientar algum termo anterior. Por exemplo: seja qual for, todo aquele (que), tal qual (= certo), tal e qual, tal ou
Manuela, essa é que dera em cheio casando com o José Afonso. qual, um ou outro, uma ou outra, etc.
Desfrutar das belezas brasileiras, isso é que é sorte! Cada um escolheu o vinho desejado.
b) O pronome demonstrativo neutro ou pode representar
um termo ou o conteúdo de uma oração inteira, caso em que Indefinidos Sistemáticos
aparece, geralmente, como objeto direto, predicativo ou aposto.
O casamento seria um desastre. Todos o pressentiam. Ao observar atentamente os pronomes indefinidos,
c) Para evitar a repetição de um verbo anteriormente percebemos que existem alguns grupos que criam oposição
expresso, é comum empregar-se, em tais casos, o verbo fazer, de sentido. É o caso de: algum/alguém/algo, que têm sentido
chamado, então, verbo vicário (= que substitui, que faz as vezes afirmativo, e nenhum/ninguém/nada, que têm sentido negativo;
de). todo/tudo, que indicam uma totalidade afirmativa, e nenhum/
Ninguém teve coragem de falar antes que ela o fizesse. nada, que indicam uma totalidade negativa; alguém/ninguém,
d) Em frases como a seguinte, este se refere à pessoa que se referem à pessoa, e algo/nada, que se referem à coisa;
mencionada em último lugar; aquele, à mencionada em primeiro certo, que particulariza, e qualquer, que generaliza.
lugar. Essas oposições de sentido são muito importantes na
O referido deputado e o Dr. Alcides eram amigos íntimos; construção de frases e textos coerentes, pois delas muitas
aquele casado, solteiro este. [ou então: este solteiro, aquele casado] vezes dependem a solidez e a consistência dos argumentos
e) O pronome demonstrativo tal pode ter conotação irônica. expostos. Observe nas frases seguintes a força que os pronomes
A menina foi a tal que ameaçou o professor? indefinidos destacados imprimem às afirmações de que fazem
f) Pode ocorrer a contração das preposições a, de, em com parte:
pronome demonstrativo: àquele, àquela, deste, desta, disso, Nada do que tem sido feito produziu qualquer resultado
nisso, no, etc. prático.
Não acreditei no que estava vendo. (no = naquilo) Certas pessoas conseguem perceber sutilezas: não são
pessoas quaisquer.
Pronomes Indefinidos
Pronomes Relativos
São palavras que se referem à terceira pessoa do discurso,
dando-lhe sentido vago (impreciso) ou expressando quantidade São aqueles que representam nomes já mencionados
indeterminada. anteriormente e com os quais se relacionam. Introduzem as
Alguém entrou no jardim e destruiu as mudas recém- orações subordinadas adjetivas.
plantadas. O racismo é um sistema que afirma a superioridade de um
Não é difícil perceber que “alguém” indica uma pessoa grupo racial sobre outros.
de quem se fala (uma terceira pessoa, portanto) de forma (afirma a superioridade de um grupo racial sobre outros =
imprecisa, vaga. É uma palavra capaz de indicar um ser humano oração subordinada adjetiva).
que seguramente existe, mas cuja identidade é desconhecida ou O pronome relativo “que” refere-se à palavra “sistema” e
não se quer revelar. introduz uma oração subordinada. Diz-se que a palavra “sistema”
é antecedente do pronome relativo que.
Classificam-se em: O antecedente do pronome relativo pode ser o pronome
demonstrativo o, a, os, as.
- Pronomes Indefinidos Substantivos: assumem o lugar Não sei o que você está querendo dizer.
do ser ou da quantidade aproximada de seres na frase. São Às vezes, o antecedente do pronome relativo não vem
eles: algo, alguém, fulano, sicrano, beltrano, nada, ninguém, expresso.
outrem, quem, tudo. Quem casa, quer casa.
Algo o incomoda?
Quem avisa amigo é. Observe:
Pronomes relativos variáveis = o qual, cujo, quanto, os quais,
- Pronomes Indefinidos Adjetivos: qualificam um ser cujos, quantos, a qual, cuja, quanta, as quais, cujas, quantas.
expresso na frase, conferindo-lhe a noção de quantidade Pronomes relativos invariáveis = quem, que, onde.
aproximada. São eles: cada, certo(s), certa(s).
Cada povo tem seus costumes. Note que:
Certas pessoas exercem várias profissões. a) O pronome “que” é o relativo de mais largo emprego,
sendo por isso chamado relativo universal. Pode ser substituído
Note que: Ora são pronomes indefinidos substantivos, ora por o qual, a qual, os quais, as quais, quando seu antecedente for
pronomes indefinidos adjetivos: um substantivo.
algum, alguns, alguma(s), bastante(s) (= muito, muitos),
demais, mais, menos, muito(s), muita(s), nenhum, nenhuns, O trabalho que eu fiz refere-se à corrupção. (= o qual)
nenhuma(s), outro(s), outra(s), pouco(s), pouca(s), qualquer, A cantora que acabou de se apresentar é péssima. (= a qual)
quaisquer, qual, que, quanto(s), quanta(s), tal, tais, tanto(s), Os trabalhos que eu fiz referem-se à corrupção. (= os quais)
tanta(s), todo(s), toda(s), um, uns, uma(s), vários, várias. As cantoras que se apresentaram eram péssimas. (= as quais)

Menos palavras e mais ações. b) O qual, os quais, a qual e as quais são exclusivamente
Alguns se contentam pouco. pronomes relativos: por isso, são utilizados didaticamente para
verificar se palavras como “que”, “quem”, “onde” (que podem ter
Os pronomes indefinidos podem ser divididos várias classificações) são pronomes relativos. Todos eles são
em variáveis e invariáveis. Observe: usados com referência à pessoa ou coisa por motivo de clareza
ou depois de determinadas preposições:
Variáveis = algum, nenhum, todo, muito, pouco, vário, tanto,
outro, quanto, alguma, nenhuma, toda, muita, pouca, vária, Regressando de São Paulo, visitei o sítio de minha tia, o
tanta, outra, quanta, qualquer, quaisquer, alguns, nenhuns, qual me deixou encantado. (O uso de “que”, neste caso, geraria
todos, muitos, poucos, vários, tantos, outros, quantos, algumas, ambiguidade.)
nenhumas, todas, muitas, poucas, várias, tantas, outras, quantas.
Invariáveis = alguém, ninguém, outrem, tudo, nada, algo, Essas são as conclusões sobre as quais pairam muitas
cada. dúvidas? (Não se poderia usar “que” depois de sobre.)

Língua Portuguesa 24
Apostila Digital Licenciada para Alice Caroline Guarino dos Santos - alice.guarino@hotmail.com (Proibida a Revenda)
APOSTILAS OPÇÃO
c) O relativo “que” às vezes equivale a o que, coisa que, e se 1. Eu não sei essa matéria, mas ele irá me ajudar.
refere a uma oração. 2. Maria foi embora para casa, pois não sabia se devia ajudá-
lo.
Não chegou a ser padre, mas deixou de ser poeta, que era a
sua vocação natural. Na primeira oração os pronomes pessoais “eu” e “ele”
exercem função de sujeito, logo, são pertencentes ao caso reto.
d) O pronome “cujo” não concorda com o seu antecedente, Já na segunda oração, observamos o pronome “lhe” exercendo
mas com o consequente. Equivale a do qual, da qual, dos quais, função de complemento, e, consequentemente, é do caso oblíquo.
das quais. Os pronomes pessoais indicam as pessoas do discurso,
o pronome oblíquo “lhe”, da segunda oração, aponta para a
Este é o caderno cujas folhas estão rasgadas. segunda pessoa do singular (tu/você): Maria não sabia se devia
(antecedente) (consequente) ajudar.... Ajudar quem? Você (lhe).
Importante: Em observação à segunda oração, o emprego do
e) “Quanto” é pronome relativo quando tem por antecedente pronome oblíquo “lhe” é justificado antes do verbo intransitivo
um pronome indefinido: tanto (ou variações) e tudo: “ajudar” porque o pronome oblíquo pode estar antes, depois ou
entre locução verbal, caso o verbo principal (no caso “ajudar”)
Emprestei tantos quantos foram necessários. estiver no infinitivo ou gerúndio.
(antecedente) Eu desejo lhe perguntar algo.
Eu estou perguntando-lhe algo.
Ele fez tudo quanto havia falado.
(antecedente) Os pronomes pessoais oblíquos podem ser átonos ou tônicos:
os primeiros não são precedidos de preposição, diferentemente
f) O pronome “quem” se refere a pessoas e vem sempre dos segundos que são sempre precedidos de preposição.
precedido de preposição. - Pronome oblíquo átono: Joana me perguntou o que eu
estava fazendo.
É um professor a quem muito devemos. - Pronome oblíquo tônico: Joana perguntou para mim o que
(preposição) eu estava fazendo.

g) “Onde”, como pronome relativo, sempre possui Questões


antecedente e só pode ser utilizado na indicação de lugar.
A casa onde morava foi assaltada. 01. Observe as sentenças abaixo.
I. Esta é a professora de cuja aula todos os alunos gostam.
h) Na indicação de tempo, deve-se empregar quando ou em II. Aquela é a garota com cuja atitude discordei - tornamo-
que. nos inimigas desde aquele episódio.
Sinto saudades da época em que (quando) morávamos no III. A criança cuja a família não compareceu ficou inconsolável.
exterior.
O pronome ‘cuja’ foi empregado de acordo com a norma
i) Podem ser utilizadas como pronomes relativos as palavras: culta da língua portuguesa em:
- como (= pelo qual) (A) apenas uma das sentenças
Não me parece correto o modo como você agiu semana (B) apenas duas das sentenças.
passada. (C) nenhuma das sentenças.
- quando (= em que) (D) todas as sentenças.
Bons eram os tempos quando podíamos jogar videogame.
02. Um estudo feito pela Universidade de Michigan constatou
j) Os pronomes relativos permitem reunir duas orações que o que mais se faz no Facebook, depois de interagir com
numa só frase. amigos, é olhar os perfis de pessoas que acabamos de conhecer.
O futebol é um esporte. Se você gostar do perfil, adicionará aquela pessoa, e estará
O povo gosta muito deste esporte. formado um vínculo. No final, todo mundo vira amigo de todo
O futebol é um esporte de que o povo gosta muito. mundo. Mas, não é bem assim. As redes sociais têm o poder de
transformar os chamados elos latentes (pessoas que frequentam
k) Numa série de orações adjetivas coordenadas, pode o mesmo ambiente social, mas não são suas amigas) em elos
ocorrer a elipse do relativo “que”. fracos – uma forma superficial de amizade. Pois é, por mais
A sala estava cheia de gente que conversava, (que) ria, que existam exceções _______qualquer regra, todos os estudos
(que) fumava. mostram que amizades geradas com a ajuda da Internet são
mais fracas, sim, do que aquelas que nascem e se desenvolvem
Pronomes Interrogativos fora dela.
Isso não é inteiramente ruim. Os seus amigos do peito
São usados na formulação de perguntas, sejam elas diretas geralmente são parecidos com você: pertencem ao mesmo
ou indiretas. Assim como os pronomes indefinidos, referem- mundo e gostam das mesmas coisas. Os elos fracos, não. Eles
se à 3ª pessoa do discurso de modo impreciso. São pronomes transitam por grupos diferentes do seu e, por isso, podem lhe
interrogativos: que, quem, qual (e variações), quanto (e variações). apresentar novas pessoas e ampliar seus horizontes – gerando
uma renovação de ideias que faz bem a todos os relacionamentos,
Quem fez o almoço?/ Diga-me quem fez o almoço. inclusive às amizades antigas. O problema é que a maioria das
Qual das bonecas preferes? / Não sei qual das bonecas redes na Internet é simétrica: se você quiser ter acesso às
preferes. informações de uma pessoa ou mesmo falar reservadamente com
Quantos passageiros desembarcaram? / Pergunte quantos ela, é obrigado a pedir a amizade dela. Como é meio grosseiro
passageiros desembarcaram. dizer “não” ________ alguém que você conhece, todo mundo acaba
adicionando todo mundo. E isso vai levando ________ banalização
Sobre os pronomes: do conceito de amizade.
É verdade. Mas, com a chegada de sítios como o Twitter, ficou
O pronome pessoal é do caso reto quando tem função de diferente. Esse tipo de sítio é uma rede social completamente
sujeito na frase. O pronome pessoal é do caso oblíquo quando assimétrica. E isso faz com que as redes de “seguidores” e
desempenha função de complemento. Vamos entender, “seguidos” de alguém possam se comunicar de maneira muito
primeiramente, como o pronome pessoal surge na frase e que mais fluida. Ao estudar a sua própria rede no Twitter, o sociólogo
função exerce. Observe as orações: Nicholas Christakis, da Universidade de Harvard, percebeu

Língua Portuguesa 25
Apostila Digital Licenciada para Alice Caroline Guarino dos Santos - alice.guarino@hotmail.com (Proibida a Revenda)
APOSTILAS OPÇÃO
que seus amigos tinham começado a se comunicar entre si Observe:
independentemente da mediação dele. Pessoas cujo único ponto Vou a + a igreja.
em comum era o próprio Christakis acabaram ficando amigas. Vou à igreja.
No Twitter, eu posso me interessar pelo que você tem a dizer e
começar a te seguir. Nós não nos conhecemos. No exemplo acima, temos a ocorrência da
Mas você saberá quando eu o retuitar ou mencionar seu preposição “a”, exigida pelo verbo ir (ir a algum lugar) e a
nome no sítio, e poderá falar comigo. Meus seguidores também ocorrência do artigo “a” que está determinando o substantivo
podem se interessar pelos seus tuítes e começar a seguir você. feminino igreja. Quando ocorre esse encontro das duas vogais e
Em suma, nós continuaremos não nos conhecendo, mas as elas se unem, a união delas é indicada pelo acento grave. Observe
pessoas que estão ________ nossa volta podem virar amigas entre os outros exemplos:
si.
Adaptado de: COSTA, C. C.. Disponível em: Conheço a aluna.
<http://super.abril.com.br/cotidiano/como-internet- Refiro-me à aluna.
estamudando-amizade-619645.shtml>. No primeiro exemplo, o verbo é transitivo direto (conhecer
algo ou alguém), logo não exige preposição e a crase não pode
Considere as seguintes afirmações sobre a relação que se ocorrer. No segundo exemplo, o verbo é transitivo indireto
estabelece entre algumas palavras do texto e os elementos a que (referir-se a algo ou a alguém) e exige a preposição “a”.
se referem. Portanto, a crase é possível, desde que o termo seguinte seja
I. No segmento que nascem, a palavra que se refere a feminino e admita o artigo feminino “a” ou um dos pronomes já
amizades. especificados.
II. O segmento elos fracos retoma o segmento uma forma Veja os principais casos em que a crase NÃO ocorre:
superficial de amizade.
III. Na frase Nós não nos conhecemos, o pronome Nós refere- 1-) diante de substantivos masculinos:
se aos pronomes eu e você. Andamos a cavalo.
Quais estão corretas? Fomos a pé.
(A) Apenas I.
(B) Apenas II. 2-) diante de verbos no infinitivo:
(C) Apenas III. A criança começou a falar.
(D) Apenas I e II. Ela não tem nada a dizer.
(E) I, II e III.
Obs.: como os verbos não admitem artigos, o “a” dos
03. Observe a charge a seguir. exemplos acima é apenas preposição, logo não ocorrerá crase.

3-) diante da maioria dos pronomes e das expressões de


tratamento, com exceção das formas senhora, senhorita e dona:
Diga a ela que não estarei em casa amanhã.
Entreguei a todos os documentos necessários.
Ele fez referência a Vossa Excelência no discurso de ontem.

Os poucos casos em que ocorre crase diante dos pronomes


podem ser identificados pelo método: troque a palavra feminina
por uma masculina, caso na nova construção surgir a forma ao,
ocorrerá crase. Por exemplo:

Refiro-me à mesma pessoa. (Refiro-me ao mesmo indivíduo.)


Informei o ocorrido à senhora. (Informei o ocorrido ao senhor.)
Peça à própria Cláudia para sair mais cedo. (Peça ao próprio
Em relação à charge acima, assinale a afirmativa inadequada. Cláudio para sair mais cedo.)
(A) A fala do personagem é uma modificação intencional de
uma fala de Cristo. 4-) diante de numerais cardinais:
(B) As duas ocorrências do pronome “eles” referem-se a Chegou a duzentos o número de feridos
pessoas distintas. Daqui a uma semana começa o campeonato.
(C) A crítica da charge se dirige às autoridades políticas no
poder. Casos em que a crase SEMPRE ocorre:
(D) A posição dos braços do personagem na charge repete a
de Cristo na cruz. 1-) diante de palavras femininas:
(E) Os elementos imagísticos da charge estão distribuídos de Amanhã iremos à festa de aniversário de minha colega.
forma equilibrada. Sempre vamos à praia no verão.
Respostas Ela disse à irmã o que havia escutado pelos corredores.
01. A\02. E\03. B Sou grata à população.
Fumar é prejudicial à saúde.
Crase Este aparelho é posterior à invenção do telefone.

A palavra crase é de origem grega e significa «fusão», 2-) diante da palavra “moda”, com o sentido de “à moda de”
«mistura». Na língua portuguesa, é o nome que se dá à «junção» (mesmo que a expressão moda de fique subentendida):
de duas vogais idênticas. É de grande importância a crase da O jogador fez um gol à (moda de) Pelé.
preposição “a” com o artigo feminino “a” (s), com o “a” inicial dos Usava sapatos à (moda de) Luís XV.
pronomes aquele(s), aquela (s), aquilo e com o “a” do relativo a Estava com vontade de comer frango à (moda de) passarinho.
qual (as quais). Na escrita, utilizamos o acento grave ( ` ) para O menino resolveu vestir-se à (moda de) Fidel Castro.
indicar a crase. O uso apropriado do acento grave depende da
compreensão da fusão das duas vogais. É fundamental também, 3-) na indicação de horas:
para o entendimento da crase, dominar a regência dos verbos Acordei às sete horas da manhã.
e nomes que exigem a preposição “a”. Aprender a usar a Elas chegaram às dez horas.
crase, portanto, consiste em aprender a verificar a ocorrência Foram dormir à meia-noite.
simultânea de uma preposição e um artigo ou pronome.

Língua Portuguesa 26
Apostila Digital Licenciada para Alice Caroline Guarino dos Santos - alice.guarino@hotmail.com (Proibida a Revenda)
APOSTILAS OPÇÃO
4-) em locuções adverbiais, prepositivas e conjuntivas de quais depende do verbo. Se o verbo que rege esses pronomes
que participam palavras femininas. Por exemplo: exigir a preposição «a», haverá crase. É possível detectar a
ocorrência da crase nesses casos utilizando a substituição do
à tarde às ocultas às pressas à medida que termo regido feminino por um termo regido masculino.
à noite às claras às escondidas à força Por exemplo:
A igreja à qual me refiro fica no centro da cidade.
à vontade à beça à larga à escuta O monumento ao qual me refiro fica no centro da cidade
às avessas à revelia à exceção de à imitação de
Caso surja a forma ao com a troca do termo, ocorrerá a crase.
à esquerda às turras às vezes à chave Veja outros exemplos:
à direita à procura à deriva à toa São normas às quais todos os alunos devem obedecer.
Esta foi a conclusão à qual ele chegou.
à proporção Várias alunas às quais ele fez perguntas não souberam
à luz à sombra de à frente de
que responder nenhuma das questões.
à A sessão à qual assisti estava vazia.
semelhança às ordens à beira de
de Crase com o Pronome Demonstrativo “a”

Crase diante de Nomes de Lugar A ocorrência da crase com o pronome


demonstrativo “a” também pode ser detectada através da
Alguns nomes de lugar não admitem a anteposição do substituição do termo regente feminino por um termo regido
artigo “a”. Outros, entretanto, admitem o artigo, de modo que masculino.
diante deles haverá crase, desde que o termo regente exija a Veja:
preposição “a”. Para saber se um nome de lugar admite ou não Minha revolta é ligada à do meu país.
a anteposição do artigo feminino “a”, deve-se substituir o termo Meu luto é ligado ao do meu país.
regente por um verbo que peça a preposição “de” ou “em”. A As orações são semelhantes às de antes.
ocorrência da contração “da” ou “na” prova que esse nome de Os exemplos são semelhantes aos de antes.
lugar aceita o artigo e, por isso, haverá crase. Suas perguntas são superiores às dele.
Por exemplo: Seus argumentos são superiores aos dele.
Vou à França. (Vim da [de+a] França. Estou na [em+a] Sua blusa é idêntica à de minha colega.
França.) Seu casaco é idêntico ao de minha colega.
Cheguei à Grécia. (Vim da Grécia. Estou na Grécia.)
Retornarei à Itália. (Vim da Itália. Estou na Itália) A Palavra Distância
Vou a Porto Alegre. (Vim de Porto Alegre. Estou em Porto
Alegre.) Se a palavra distância estiver especificada, determinada, a
crase deve ocorrer.
- Minha dica: use a regrinha “Vou A volto DA, crase HÁ; vou A Por exemplo:
volto DE, crase PRA QUÊ?” Sua casa fica à distância de 100 Km daqui. (A palavra está
Ex: Vou a Campinas. = Volto de Campinas. determinada)
Vou à praia. = Volto da praia. Todos devem ficar à distância de 50 metros do palco. (A
palavra está especificada.)
- ATENÇÃO: quando o nome de lugar estiver especificado,
ocorrerá crase. Veja: Se a palavra distância não estiver especificada, a
Retornarei à São Paulo dos bandeirantes. = crase não pode ocorrer.
mesmo que, pela regrinha acima, seja a do “VOLTO DE” Por exemplo:
Irei à Salvador de Jorge Amado. Os militares ficaram a distância.
Gostava de fotografar a distância.
Crase diante dos Pronomes Demonstrativos Aquele (s), Ensinou a distância.
Aquela (s), Aquilo Dizem que aquele médico cura a distância.
Reconheci o menino a distância.
Haverá crase diante desses pronomes sempre que o termo
regente exigir a preposição “a”. Por exemplo: Observação: por motivo de clareza, para evitar ambiguidade,
pode-se usar a crase.
Refiro-me a + aquele atentado. Veja:
Preposição Pronome Gostava de fotografar à distância.
Ensinou à distância.
Refiro-me àquele atentado. Dizem que aquele médico cura à distância.

O termo regente do exemplo acima é o verbo transitivo Casos em que a ocorrência da crase é FACULTATIVA
indireto referir (referir-se a algo ou alguém) e exige preposição,
portanto, ocorre a crase. Observe este outro exemplo: 1-) diante de nomes próprios femininos:
Observação: é facultativo o uso da crase diante de nomes
Aluguei aquela casa. próprios femininos porque é facultativo o uso do artigo. Observe:
Paula é muito bonita. Laura é minha amiga.
O verbo “alugar” é transitivo direto (alugar algo) e não exige A Paula é muito bonita. A Laura é minha amiga.
preposição. Logo, a crase não ocorre nesse caso.
Veja outros exemplos: Como podemos constatar, é facultativo o uso do artigo
Dediquei àquela senhora todo o meu trabalho. feminino diante de nomes próprios femininos, então podemos
Quero agradecer àqueles que me socorreram. escrever as frases abaixo das seguintes formas:
Refiro-me àquilo que aconteceu com seu pai.
Não obedecerei àquele sujeito. Entreguei o cartão a Paula. Entreguei o cartão a
Roberto.
Crase com os Pronomes Relativos A Qual, As Quais Entreguei o cartão à Paula. Entreguei o cartão ao
Roberto.
A ocorrência da crase com os pronomes relativos a qual e as

Língua Portuguesa 27
Apostila Digital Licenciada para Alice Caroline Guarino dos Santos - alice.guarino@hotmail.com (Proibida a Revenda)
APOSTILAS OPÇÃO
2-) diante de pronome possessivo feminino:
Observação: é facultativo o uso da crase diante de
pronomes possessivos femininos porque é facultativo o uso do 6. formas de tratamento;
artigo. Observe:
Minha avó tem setenta anos. Minha irmã está
esperando por você. Pronome de Tratamento
A minha avó tem setenta anos. A minha irmã está
esperando por você. Quando nos dirigimos às pessoas do nosso convívio diário
utilizamos uma linguagem mais informal, mais íntima. Ao passo
Sendo facultativo o uso do artigo feminino diante de que, se formos nos dirigir a alguém que possui um prestígio social
pronomes possessivos femininos, então podemos escrever as mais alto ou um grau hierárquico mais elevado, necessariamente
frases abaixo das seguintes formas: temos que utilizar uma linguagem mais formal. Lembrando que
isto prevalece tanto para a escrita quanto para a fala.
Cedi o lugar a minha avó. Cedi o lugar a meu avô. Para isto, podemos usufruir de um completo aparato no
Cedi o lugar à minha avó. Cedi o lugar ao meu avô. que se refere às normas gramaticais e à maneira correta de
como e onde utilizá-las. E fazendo parte deste aparato, estão
3-) depois da preposição até: os pronomes, os quais pertencem às dez classes gramaticais e
Fui até a praia. ou Fui até à praia. possuem a função de acompanhar ou substituir o nome, ou seja,
Acompanhe-o até a porta. ou Acompanhe-o até à porta. o próprio substantivo, relacionando-o à pessoa do discurso.
A palestra vai até as cinco horas da tarde. ou É importante lembrarmos que eles representam a forma pela
A palestra vai até às cinco horas da tarde. qual nos atribuímos às pessoas, como já foi dito anteriormente.
São eles:
Questões
Vossa Alteza V. A. príncipes, duques
01. No Brasil, as discussões sobre drogas parecem limitar- Vossa Eminência V. Ema.(s) cardeais
se ______aspectos jurídicos ou policiais. É como se suas únicas Vossa Reverendíssima V. Revma.(s) sacerdotes e bispos
consequências estivessem em legalismos, tecnicalidades Vossa Excelência V. Ex.ª (s) altas autoridades e
e estatísticas criminais. Raro ler ____respeito envolvendo oficiais-generais
questões de saúde pública como programas de esclarecimento Vossa Magnificência V. Mag.ª (s) reitores de universidades
e prevenção, de tratamento para dependentes e de reintegração Vossa Majestade V. M. reis e rainhas
desses____ vida. Quantos de nós sabemos o nome de um médico Vossa Majestade Imperial V. M. I. Imperadores
ou clínica ____quem tentar encaminhar um drogado da nossa Vossa Santidade V. S. Papa
própria família? Vossa Senhoria V. S.ª (s) tratamento cerimonioso
Vossa Onipotência V. O. Deus
(Ruy Castro, Da nossa própria família. Folha de S.Paulo,
17.09.2012. Adaptado) Também são pronomes de tratamento o senhor, a senhora
e você, vocês. “O senhor” e “a senhora” são empregados no
As lacunas do texto devem ser preenchidas, correta e tratamento cerimonioso; “você” e “vocês”, no tratamento
respectivamente, com: familiar. Você e vocês são largamente empregados no português
do Brasil; em algumas regiões , a forma tu é de uso frequente,
(A) aos … à … a … a em outras, é muito pouco empregada. Já a forma vós tem uso
(B) aos … a … à … a restrito à linguagem litúrgica, ultraformal ou literária.
(C) a … a … à … à
(D) à … à … à … à Observações:
(E) a … a … a … a a) Vossa Excelência X Sua Excelência : os pronomes
de tratamento que possuem “Vossa (s)” são empregados em
02. Leia o texto a seguir. relação à pessoa com quem falamos.
Foi por esse tempo que Rita, desconfiada e medrosa, correu Por exemplo:
______ cartomante para consultá-la sobre a verdadeira causa do Espero que V. Ex.ª, Senhor Ministro, compareça a este
procedimento de Camilo. Vimos que ______ cartomante restituiu- encontro.
lhe ______ confiança, e que o rapaz repreendeu-a por ter feito o
que fez. Emprega-se “Sua (s)” quando se fala a respeito da pessoa.
Por Exemplo:
(Machado de Assis. A cartomante. In: Várias histórias. Rio de Todos os membros da C.P.I. afirmaram que Sua Excelência, o
Janeiro: Globo, 1997, p. 6) Senhor Presidente da República, agiu com propriedade.
- Os pronomes de tratamento representam uma forma
Preenchem corretamente as lacunas da frase acima, na indireta de nos dirigirmos aos nossos interlocutores. Ao
ordem dada: tratarmos um deputado por Vossa Excelência, por exemplo,
A) à – a – a estamos nos endereçando à excelência que esse deputado
B) a – a – à supostamente tem para poder ocupar o cargo que ocupa.
C) à – a – à
D) à – à – a b) 3ª pessoa: embora os pronomes de tratamento se dirijam
E) a – à – à à 2ª pessoa, toda a concordância deve ser feita com a 3ª pessoa.
Assim, os verbos, os pronomes possessivos e os pronomes
03 “Nesta oportunidade, volto ___ referir-me ___ problemas já oblíquos empregados em relação a eles devem ficar na 3ª pessoa.
expostos ___ V. Sª ___ alguns dias”. Por exemplo:
Basta que V. Ex.ª cumpra a terça parte das suas promessas,
a) à - àqueles - a - há para que seus eleitores lhe fiquem reconhecidos.
b) a - àqueles - a - há
c) a - aqueles - à - a c) Uniformidade de Tratamento: quando escrevemos ou
d) à - àqueles - a - a nos dirigimos a alguém, não é permitido mudar, ao longo do
e) a - aqueles - à - há texto, a pessoa do tratamento escolhida inicialmente. Assim,
por exemplo, se começamos a chamar alguém de “você”, não
Respostas poderemos usar “te” ou “teu”. O uso correto exigirá, ainda, verbo
1-B / 2-A / 3-B na terceira pessoa.

Língua Portuguesa 28
Apostila Digital Licenciada para Alice Caroline Guarino dos Santos - alice.guarino@hotmail.com (Proibida a Revenda)
APOSTILAS OPÇÃO
Por exemplo: Respostas
Quando você vier, eu te abraçarei e enrolar-me-ei nos teus
cabelos. (errado) 01.
Quando você vier, eu a abraçarei e enrolar-me-ei nos seus a – Encontrei-a passeando no shopping.
cabelos. (correto) b – Deixe-me sossegada, pois preciso descansar.
Quando tu vieres, eu te abraçarei e enrolar-me-ei nos teus c – Desejas ir conosco?
cabelos. (correto) d - De hoje em diante está tudo terminado entre nós.
Fontes: DUARTE, Vânia Maria Do Nascimento. “Pronomes de e - Entreguei o livro hoje, portanto poderás pegá-lo.
Tratamento”; Brasil Escola. Disponível em <http://brasilescola.uol.com.
br/gramatica/pronomes-tratamento.htm>. Acesso em 10 de fevereiro 02.
de 2016. a- Sim, o uso dos pronomes possessivos quando utilizados
http://www.soportugues.com.br/secoes/morf/morf46.php de forma inadequada incide de forma negativa na clareza
do discurso, muitas vezes demarcado pela ambiguidade. No
Questões caso em questão, o uso do pronome “sua” remete ao seguinte
questionamento: trata-se da irmã de Marcos ou de André?
01. Como sabemos, a língua escrita requer uma linguagem
que esteja de acordo com a norma padrão. Assim sendo, as b – No intuito de atribuir clareza ao enunciado, esse deve ser
frases a seguir pertencem a um nível mais coloquial. Reescreva- assim expresso: Marcos, o André saiu com a irmã dele.
as procurando adequá-las à forma correta:
a – Encontrei ela passeando no shopping. 03.
b – Deixa eu sossegada, pois preciso descansar. a - Esta é pessoa da qual lhe falei.
c – Desejas ir comigo e com minha irmã? b – Não conseguiremos encontrar aqueles estimados
d - De hoje em diante está tudo terminado entre eu e você. professores, pois eles já não trabalham mais aqui.
e - Entreguei o livro hoje, portanto poderás pegar ele. c – Empreste-me esse livro? Há muito tento encontrá-lo.
d- Olha! Quem são aqueles convidados que acabaram de
02. Da oração que segue, propõe-se que seja feita uma análise chegar?
e, posteriormente, responda às questões que a ela se refere: e – Este foi o aluno destaque deste ano, desejas entrevistá-
lo?
Marcos, o André saiu com sua irmã!
04. (C)
a – O uso do pronome possessivo implica em uma duplicidade
de sentido? Relate. 05.
b – Reescreva-a eliminando esta ocorrência de modo a torná- a – Vossa Santidade
la clara e objetiva. b – Vossa Magnificência
c - Você
03. Preencha as lacunas utilizando corretamente os d – Vossa Majestade
pronomes demonstrativos: e – Vossa Excelência
a - _________ é a pessoa da qual lhe falei.
b – Não conseguiremos encontrar ________________estimados
professores, pois eles já não trabalham mais aqui. 7. pontuação;
c – Empreste-me _____________livro? Há muito tento encontrá-
lo.
d- Olha! Quem são _____________________convidados que
acabaram de chegar? Pontuação
e - ___________ foi o aluno destaque deste ano, desejas
entrevistá-lo? Os sinais de pontuação são marcações gráficas que servem
para compor a coesão e a coerência textual além de ressaltar
04. Assinale o item em que há erro no emprego do pronome especificidades semânticas e pragmáticas. Vejamos as principais
demonstrativo: funções dos sinais de pontuação conhecidos pelo uso da língua
a – ( ) Paulo, que é isso que você leva? portuguesa.
b - ( ) “Amai vossos irmãos”! são essas as verdadeiras
palavras de amor. Ponto
c - ( ) Trinta de dezembro de 1977! Foi significativo para 1- Indica o término do discurso ou de parte dele.
mim esse dia. - Façamos o que for preciso para tirá-la da situação em que
d – ( ) Pedro, esse livro que está com José é meu. se encontra.
e – ( ) Não estou de acordo com aquelas palavras que José - Gostaria de comprar pão, queijo, manteiga e leite.
pronunciou.
- Acordei. Olhei em volta. Não reconheci onde estava.
05. Observe o trecho no qual encontra-se empregado o
pronome de tratamento de forma correta. Logo em seguida 2- Usa-se nas abreviações - V. Exª. - Sr.
atente-se para o que se pede:
Ponto e Vírgula ( ; )
“-Vossa Excelência, por obséquio, queira falar mais alto, que 1- Separa várias partes do discurso, que têm a mesma
não ouvi bem – e apontava agoniado, um dos deputados mais importância.
próximos.” (Fernando Sabino) - “Os pobres dão pelo pão o trabalho; os ricos dão pelo pão
a fazenda; os de espíritos generosos dão pelo pão a vida; os de
Qual o pronome de tratamento seria utilizado no caso de nos nenhum espírito dão pelo pão a alma...” (VIEIRA)
dirigirmos às seguintes pessoas:
a- Papa _____________________________ 2- Separa partes de frases que já estão separadas por
b – Ao reitor de uma universidade _________________________ vírgulas.
c – A um amigo mais íntimo, convidando-o para um passeio - Alguns quiseram verão, praia e calor; outros montanhas, frio
____________________ e cobertor.
d – A um rei ou uma rainha _____________________________
e – A uma autoridade ligada ao mundo da política 3- Separa itens de uma enumeração, exposição de motivos,
______________________________ decreto de lei, etc.

Língua Portuguesa 29
Apostila Digital Licenciada para Alice Caroline Guarino dos Santos - alice.guarino@hotmail.com (Proibida a Revenda)
APOSTILAS OPÇÃO
- Ir ao supermercado; - Para marcar inversão:
- Pegar as crianças na escola; a) do adjunto adverbial (colocado no início da oração):
- Caminhada na praia; Depois das sete horas, todo o comércio está de portas fechadas.
- Reunião com amigos. b) dos objetos pleonásticos antepostos ao verbo: Aos
pesquisadores, não lhes destinaram verba alguma.
Dois pontos c) do nome de lugar anteposto às datas: Recife, 15 de maio
1- Antes de uma citação de 1982.
- Vejamos como Afrânio Coutinho trata este assunto:
- Para separar entre si elementos coordenados (dispostos
2- Antes de um aposto em enumeração):
- Três coisas não me agradam: chuva pela manhã, frio à tarde Era um garoto de 15 anos, alto, magro.
e calor à noite. A ventania levou árvores, e telhados, e pontes, e animais.

3- Antes de uma explicação ou esclarecimento - Para marcar elipse (omissão) do verbo:


- Lá estava a deplorável família: triste, cabisbaixa, vivendo a Nós queremos comer pizza; e vocês, churrasco.
rotina de sempre.
- Para isolar:
4- Em frases de estilo direto
Maria perguntou: - o aposto:
- Por que você não toma uma decisão? São Paulo, considerada a metrópole brasileira, possui um
trânsito caótico.
Ponto de Exclamação
1- Usa-se para indicar entonação de surpresa, cólera, susto, - o vocativo:
súplica, etc. Ora, Thiago, não diga bobagem.
- Sim! Claro que eu quero me casar com você!
Questões
2- Depois de interjeições ou vocativos
- Ai! Que susto! 01. Assinale a alternativa em que a pontuação está
- João! Há quanto tempo! corretamente empregada, de acordo com a norma-padrão da
língua portuguesa.
Ponto de Interrogação (A) Diante da testemunha, o homem abriu a bolsa e, embora,
Usa-se nas interrogações diretas e indiretas livres. experimentasse, a sensação de violar uma intimidade, procurou
“- Então? Que é isso? Desertaram ambos?” (Artur Azevedo) a esmo entre as coisinhas, tentando encontrar algo que pudesse
Reticências ajudar a revelar quem era a sua dona.
1- Indica que palavras foram suprimidas. (B) Diante, da testemunha o homem abriu a bolsa e, embora
- Comprei lápis, canetas, cadernos... experimentasse a sensação, de violar uma intimidade, procurou
a esmo entre as coisinhas, tentando encontrar algo que pudesse
2- Indica interrupção violenta da frase. ajudar a revelar quem era a sua dona.
“- Não... quero dizer... é verdad... Ah!” (C) Diante da testemunha, o homem abriu a bolsa e, embora
experimentasse a sensação de violar uma intimidade, procurou
3- Indica interrupções de hesitação ou dúvida a esmo entre as coisinhas, tentando encontrar algo que pudesse
- Este mal... pega doutor? ajudar a revelar quem era a sua dona.
(D) Diante da testemunha, o homem, abriu a bolsa e, embora
4- Indica que o sentido vai além do que foi dito experimentasse a sensação de violar uma intimidade, procurou
- Deixa, depois, o coração falar... a esmo entre as coisinhas, tentando, encontrar algo que pudesse
ajudar a revelar quem era a sua dona.
Vírgula (E) Diante da testemunha, o homem abriu a bolsa e, embora,
Não se usa vírgula experimentasse a sensação de violar uma intimidade, procurou
*separando termos que, do ponto de vista sintático, ligam-se a esmo entre as coisinhas, tentando, encontrar algo que pudesse
diretamente entre si: ajudar a revelar quem era a sua dona.

a) entre sujeito e predicado. 02. Assinale a opção em que está corretamente indicada a
Todos os alunos da sala    foram advertidos. ordem dos sinais de pontuação que devem preencher as lacunas
Sujeito                            predicado da frase abaixo:
“Quando se trata de trabalho científico ___ duas coisas devem
b) entre o verbo e seus objetos. ser consideradas ____ uma é a contribuição teórica que o trabalho
O trabalho custou            sacrifício             aos realizadores. oferece ___ a outra é o valor prático que possa ter.
             V.T.D.I.              O.D.                      O.I. A) dois pontos, ponto e vírgula, ponto e vírgula
B) dois pontos, vírgula, ponto e vírgula;
c) entre nome e complemento nominal; entre nome e adjunto C) vírgula, dois pontos, ponto e vírgula;
adnominal. D) pontos vírgula, dois pontos, ponto e vírgula;
A surpreendente reação do governo contra os sonegadores E) ponto e vírgula, vírgula, vírgula.
despertou reações entre os empresários.
adj. adnominal nome adj. adn. complemento nominal 03. Os sinais de pontuação estão empregados corretamente
em:
Usa-se a vírgula: A) Duas explicações, do treinamento para consultores
iniciantes receberam destaque, o conceito de PPD e a construção
- Para marcar intercalação: de tabelas Price; mas por outro lado, faltou falar das metas de
a) do adjunto adverbial: O café, em razão da sua abundância, vendas associadas aos dois temas.
vem caindo de preço. B) Duas explicações do treinamento para consultores
b) da conjunção: Os cerrados são secos e áridos. Estão iniciantes receberam destaque: o conceito de PPD e a construção
produzindo, todavia, altas quantidades de alimentos. de tabelas Price; mas, por outro lado, faltou falar das metas de
c) das expressões explicativas ou corretivas: As indústrias vendas associadas aos dois temas.
não querem abrir mão de suas vantagens, isto é, não querem abrir C) Duas explicações do treinamento para consultores
mão dos lucros altos. iniciantes receberam destaque; o conceito de PPD e a construção

Língua Portuguesa 30
Apostila Digital Licenciada para Alice Caroline Guarino dos Santos - alice.guarino@hotmail.com (Proibida a Revenda)
APOSTILAS OPÇÃO
de tabelas Price, mas por outro lado, faltou falar das metas de “E assim o operário ia
vendas associadas aos dois temas. Com suor e com cimento*
D) Duas explicações do treinamento para consultores Erguendo uma casa aqui
iniciantes, receberam destaque: o conceito de PPD e a construção Adiante um apartamento.”
de tabelas Price, mas, por outro lado, faltou falar das metas de *Com trabalho.
vendas associadas aos dois temas.
E) Duas explicações, do treinamento para consultores - O lugar de origem ou de produção pelo produto:
iniciantes, receberam destaque; o conceito de PPD e a construção
de tabelas Price, mas por outro lado, faltou falar das metas, de Comprei uma garrafa do legítimo porto*.
vendas associadas aos dois temas. *O vinho da cidade do Porto.

Resposta - O autor pela obra:


1-C 2-C 3-B
Ela parecia ler Jorge Amado*.
*A obra de Jorge Amado.
8. figuras de sintaxe; - O abstrato pelo concreto e vice-versa:

Não devemos contar com o seu coração*.


Figuras de Linguagem *Sentimento, sensibilidade.

As figuras de linguagem ou de estilo, de acordo com Renan Sinédoque: Ocorre sinédoque quando há substituição de
Bardine, são empregadas para valorizar o texto, tornando um termo por outro, havendo ampliação ou redução do sentido
a linguagem mais expressiva. É um recurso linguístico para usual da palavra numa relação quantitativa. Encontramos
expressar experiências comuns de formas diferentes, conferindo sinédoque nos seguintes casos:
originalidade, emotividade ou poeticidade ao discurso.
- O todo pela parte e vice-versa:
As figuras revelam muito da sensibilidade de quem as
produz, traduzindo particularidades estilísticas do autor. A “A cidade inteira (1) viu assombrada, de queixo caído, o
palavra empregada em sentido figurado, não-denotativo, passa pistoleiro sumir de ladrão, fugindo nos cascos (2) de seu cavalo.”
a pertencer a outro campo de significação, mais amplo e criativo. *1 O povo. 2 Parte das patas.

As figuras de linguagem classificam-se em: - O singular pelo plural e vice-versa:


1) figuras de palavra;
2) figuras de harmonia; O paulista (3) é tímido; o carioca (4), atrevido.
3) figuras de pensamento; *3 Todos os paulistas. 4 Todos os cariocas.
4) figuras de construção ou sintaxe.
- O indivíduo pela espécie (nome próprio pelo nome comum):
1) FIGURAS DE PALAVRA
As figuras de palavra são figuras de linguagem que consistem Para os artistas ele foi um mecenas (5).
no emprego de um termo com sentido diferente daquele *5 Protetor.
convencionalmente empregado, a fim de se conseguir um efeito
mais expressivo na comunicação. Modernamente, a metonímia engloba a sinédoque.

São figuras de palavras: Catacrese: A catacrese é um tipo de especial de metáfora,


a) comparação e) catacrese “é uma espécie de metáfora desgastada, em que já não se sente
b) metáfora f) sinestesia nenhum vestígio de inovação, de criação individual e pitoresca.
c) metonímia g) antonomásia É a metáfora tornada hábito lingüístico, já fora do âmbito
d) sinédoque h) alegoria estilístico.” (Othon M. Garcia)

Comparação: Ocorre comparação quando se estabelece Exemplos: folhas de livro, pele de tomate, dente de alho,
aproximação entre dois elementos que se identificam, ligados montar em burro, céu da boca, cabeça de prego, mão de direção,
por conectivos comparativos explícitos – feito, assim como, ventre da terra, asa da xícara, sacar dinheiro no banco.
tal, como, tal qual, tal como, qual, que nem – e alguns verbos –
parecer, assemelhar-se e outros. Sinestesia: A sinestesia consiste na fusão de sensações
diferentes numa mesma expressão. Essas sensações podem ser
Exemplos: “Amou daquela vez como se fosse máquina. físicas (gustação, audição, visão, olfato e tato) ou psicológicas
Beijou sua mulher como se fosse lógico. (subjetivas).

Metáfora: Ocorre metáfora quando um termo substitui Exemplo: “A minha primeira recordação é um muro velho, no
outro através de uma relação de semelhança resultante da quintal de uma casa indefinível. Tinha várias feridas no reboco
subjetividade de quem a cria. A metáfora também pode ser e veludo de musgo. Milagrosa aquela mancha verde [sensação
entendida como uma comparação abreviada, em que o conectivo visual] e úmida, macia [sensações táteis], quase irreal.” (Augusto
não está expresso, mas subentendido. Meyer)

Exemplo: “Supondo o espírito humano uma vasta concha, o Antonomásia: Ocorre antonomásia quando designamos
meu fim, Sr. Soares, é ver se posso extrair pérolas, que é a razão.” uma pessoa por uma qualidade, característica ou fato que a
distingue.
Metonímia: Ocorre metonímia quando há substituição de
uma palavra por outra, havendo entre ambas algum grau de Na linguagem coloquial, antonomásia é o mesmo que apelido,
semelhança, relação, proximidade de sentido ou implicação alcunha ou cognome, cuja origem é um aposto (descritivo,
mútua. Tal substituição fundamenta-se numa relação objetiva, especificativo etc.) do nome próprio.
real, realizando-se de inúmeros modos:
Exemplos:
- A causa pelo efeito e vice-versa: “E ao rabi simples(1), que a igualdade prega,
Rasga e enlameia a túnica inconsútil;

Língua Portuguesa 31
Apostila Digital Licenciada para Alice Caroline Guarino dos Santos - alice.guarino@hotmail.com (Proibida a Revenda)
APOSTILAS OPÇÃO
*1 Cristo Exemplo: “Amigos ou inimigos estão, amiúde, em posições
Pelé (= Edson Arantes do Nascimento) trocadas. Uns nos querem mal, e fazem-nos bem. Outros nos
O poeta dos escravos (= Castro Alves) almejam o bem, e nos trazem o mal.” (Rui Barbosa)
O Dante Negro (= Cruz e Souza)
O Corso (= Napoleão) Apóstrofe: Ocorre apóstrofe quando há invocação de uma
pessoa ou algo, real ou imaginário, que pode estar presente
Alegoria: A alegoria é uma acumulação de metáforas ou ausente. Corresponde ao vocativo na análise sintática e é
referindo-se ao mesmo objeto; é uma figura poética que utilizada para dar ênfase à expressão.
consiste em expressar uma situação global por meio de outra
que a evoque e intensifique o seu significado. Na alegoria, todas Exemplo: “Deus! ó Deus! onde estás, que não respondes?”
as palavras estão transladadas para um plano que não lhes é (Castro Alves)
comum e oferecem dois sentidos completos e perfeitos – um
referencial e outro metafórico. Paradoxo: Ocorre paradoxo não apenas na aproximação
de palavras de sentido oposto, mas também na de idéias que
Exemplo: “A vida é uma ópera, é uma grande ópera. O tenor se contradizem referindo-se ao mesmo termo. É uma verdade
e o barítono lutam pelo soprano, em presença do baixo e dos enunciada com aparência de mentira. Oxímoro (ou oximoron) é
comprimários, quando não são o soprano e o contralto que outra designação para paradoxo.
lutam pelo tenor, em presença do mesmo baixo e dos mesmos
comprimários. Há coros numerosos, muitos bailados, e a Exemplo: “Amor é fogo que arde sem se ver;
orquestra é excelente… (Machado de Assis) É ferida que dói e não se sente;
É um contentamento descontente;
2) FIGURAS DE HARMONIA É dor que desatina sem doer;” (Camões)

Chamam-se figuras de som ou de harmonia os efeitos Eufemismo: Ocorre eufemismo quando uma palavra ou
produzidos na linguagem quando há repetição de sons ou, ainda, expressão é empregada para atenuar uma verdade tida como
quando se procura “imitar”sons produzidos por coisas ou seres. penosa, desagradável ou chocante.

As figuras de linguagem de harmonia ou de som são: Ex:“E pela paz derradeira(1) que enfim vai nos redimir
Deus lhe pague” (Chico Buarque)
a) aliteração c) assonância *1 paz derradeira: morte
b) paronomásia d) onomatopéia
Gradação: Ocorre gradação quando há uma seqüência de
Aliteração: Ocorre aliteração quando há repetição da palavras que intensificam uma mesma idéia.
mesma consoante ou de consoantes similares, geralmente em
posição inicial da palavra. Exemplo: “Aqui… além… mais longe por onde eu movo o
passo.” (Castro Alves)
Exemplo: “Toda gente homenageia Januária na janela.”
Hipérbole: Ocorre hipérbole quando há exagero de uma
Assonância: Ocorre assonância quando há repetição da idéia, a fim de proporcionar uma imagem emocionante e de
mesma vogal ao longo de um verso ou poema. impacto.

Exemplo: “Sou Ana, da cama Exemplo: “Rios te correrão dos olhos, se chorares!” (Olavo
da cana, fulana, bacana Bilac)
Sou Ana de Amsterdam.”
Ironia: Ocorre ironia quando, pelo contexto, pela entonação,
Paronomásia: Ocorre paronomásia quando há reprodução pela contradição de termos, sugere-se o contrário do que as
de sons semelhantes em palavras de significados diferentes. palavras ou orações parecem exprimir. A intenção é depreciativa
ou sarcástica.
Exemplo: “Berro pelo aterro pelo desterro
berro por seu berro pelo seu erro Exemplo: “Moça linda, bem tratada,
quero que você ganhe que você me apanhe três séculos de família,
sou o seu bezerro gritando mamãe.” burra como uma porta:
um amor.” (Mário de Andrade)
Onomatopeia: Ocorre quando uma palavra ou conjunto de
palavras imita um ruído ou som. Prosopopéia: Ocorre prosopopéia (ou animização ou
personificação) quando se atribui movimento, ação, fala,
Exemplo: “O silêncio fresco despenca das árvores. sentimento, enfim, caracteres próprios de seres animados a
Veio de longe, das planícies altas, seres inanimados ou imaginários.
Dos cerrados onde o guaxe passe rápido…
Vvvvvvvv… passou.” Também a atribuição de características humanas a seres
animados constitui prosopopéia o que é comum nas fábulas
3) FIGURAS DE PENSAMENTO e nos apólogos, como este exemplo de Mário de Quintana: “O
peixinho (…) silencioso e levemente melancólico…”
As figuras de pensamento são recursos de linguagem que se
referem ao significado das palavras, ao seu aspecto semântico. Exemplos: “… os rios vão carregando as queixas do caminho.”
(Raul Bopp)
São figuras de linguagem de pensamento:
Um frio inteligente (…) percorria o jardim…” (Clarice
a) antítese d) apóstrofe g) paradoxo Lispector)
b) eufemismo e) gradação h) hipérbole
c) ironia f) prosopopéia i) perífrase Perífrase: Ocorre perífrase quando se cria um torneio de
palavras para expressar algum objeto, acidente geográfico ou
Antítese: Ocorre antítese quando há aproximação de situação que não se quer nomear.
palavras ou expressões de sentidos opostos.
Exemplo: “Cidade maravilhosa

Língua Portuguesa 32
Apostila Digital Licenciada para Alice Caroline Guarino dos Santos - alice.guarino@hotmail.com (Proibida a Revenda)
APOSTILAS OPÇÃO
Cheia de encantos mil b) Pleonasmo vicioso: É o desdobramento de ideias que
Cidade maravilhosa já estavam implícitas em palavras anteriormente expressas.
Coração do meu Brasil.” (André Filho) Pleonasmos viciosos devem ser evitados, pois não têm valor de
reforço de uma idéia, sendo apenas fruto do descobrimento do
4) FIGURAS DE SINTAXE sentido real das palavras.

As figuras de sintaxe ou de construção dizem respeito a Exemplos: subir para cima, entrar para dentro, repetir de
desvios em relação à concordância entre os termos da oração, novo, ouvir com os ouvidos, hemorragia de sangue, monopólio
sua ordem, possíveis repetições ou omissões. exclusivo, breve alocução, principal protagonista
Elas podem ser construídas por:
a) omissão: assíndeto, elipse e zeugma; Polissíndeto: Ocorre polissíndeto quando há repetição
b) repetição: anáfora, pleonasmo e polissíndeto; enfática de uma conjunção coordenativa mais vezes do que exige
c) inversão: anástrofe, hipérbato, sínquise e hipálage; a norma gramatical ( geralmente a conjunção e). É um recurso
d) ruptura: anacoluto; que sugere movimentos ininterruptos ou vertiginosos.
e) concordância ideológica: silepse.
Exemplo: “Vão chegando as burguesinhas pobres,
Portanto, são figuras de linguagem de construção ou sintaxe: e as criadas das burguesinhas ricas
a) assíndeto e) elipse i) zeugma e as mulheres do povo, e as lavadeiras da redondeza.”
b) anáfora f) pleonasmo j) polissíndeto (Manuel Bandeira)
c) anástrofe g) hiperbato l) sínquise
d) hipálage h) anacoluto m) silepse Anástrofe: Ocorre anástrofe quando há uma simples
inversão de palavras vizinhas (determinante / determinado).
Assíndeto: Ocorre assíndeto quando orações ou palavras
deveriam vir ligadas por conjunções coordenativas, aparecem Exemplo: “Tão leve estou (1) que nem sombra tenho.” (Mário
justapostas ou separadas por vírgulas. Quintana)
*1 Estou tão leve…
Exigem do leitor atenção maior no exame de cada fato, por
exigência das pausas rítmicas (vírgulas). Hipérbato: Ocorre hipérbato quando há uma inversão
completa de membros da frase.
Exemplo: “Não nos movemos, as mãos é que se estenderam
pouco a pouco, todas quatro, pegando-se, apertando-se, Exemplo: “Passeiam à tarde, as belas na Avenida. ” 1 (Carlos
fundindo-se.” (Machado de Assis) Drummond de Andrade)
*1 As belas passeiam na Avenida à tarde.
Elipse: Ocorre elipse quando omitimos um termo ou
oração que facilmente podemos identificar ou subentender no Sínquise: Ocorre sínquise quando há uma inversão violenta
contexto. Pode ocorrer na supressão de pronomes, conjunções, de distantes partes da frase. É um hipérbato exagerado.
preposições ou verbos. É um poderoso recurso de concisão e
dinamismo. Exemplo: “A grita se alevanta ao Céu, da gente. ” 1 (Camões)
*1 A grita da gente se alevanta ao Céu.
Exemplo: “Veio sem pinturas, em vestido leve, sandálias
coloridas.” Hipálage: Ocorre hipálage quando há inversão da posição do
1 Elipse do pronome ela (Ela veio) e da preposição de (de adjetivo: uma qualidade que pertence a uma objeto é atribuída a
sandálias…) outro, na mesma frase.

Zeugma: Ocorre zeugma quando um termo já expresso na Exemplo: “… as lojas loquazes dos barbeiros.” 2 (Eça de
frase é suprimido, ficando subentendida sua repetição. Queiros)
*2 … as lojas dos barbeiros loquazes.
Exemplo: “Foi saqueada a vida, e assassinados os partidários
dos Felipes.” 1 Anacoluto: Ocorre anacoluto quando há interrupção
1 Zeugma do verbo: “e foram assassinados…” do plano sintático com que se inicia a frase, alterando-lhe a
seqüência lógica. A construção do período deixa um ou mais
Anáfora: Ocorre anáfora quando há repetição intencional de termos – que não apresentam função sintática definida –
palavras no início de um período, frase ou verso. desprendidos dos demais, geralmente depois de uma pausa
sensível.
Exemplo: “Depois o areal extenso…
Depois o oceano de pó… Exemplo: “Essas empregadas de hoje, não se pode confiar
Depois no horizonte imenso nelas.” (Alcântara Machado)
Desertos… desertos só…” (Castro Alves)
Silepse: Ocorre silepse quando a concordância não é feita
Pleonasmo: Ocorre pleonasmo quando há repetição da com as palavras, mas com a ideia a elas associada.
mesma ideia, isto é, redundância de significado.
a) Silepse de gênero: Ocorre quando há discordância entre
a) Pleonasmo literário: É o uso de palavras redundantes para os gêneros gramaticais (feminino ou masculino).
reforçar uma ideia, tanto do ponto de vista semântico quanto
do ponto de vista sintático. Usado como um recurso estilístico, Exemplo: “Quando a gente é novo, gosta de fazer bonito.”
enriquece a expressão, dando ênfase à mensagem. (Guimarães Rosa)

Exemplo: “Iam vinte anos desde aquele dia b) Silepse de número: Ocorre quando há discordância
Quando com os olhos eu quis ver de perto envolvendo o número gramatical (singular ou plural).
Quando em visão com os da saudade via.” (Alberto Exemplo: Corria gente de todos lados, e gritavam.” (Mário
de Oliveira) Barreto)

“Ó mar salgado, quando do teu sal c) Silepse de pessoa: Ocorre quando há discordância entre o
São lágrimas de Portugal” (Fernando Pessoa) sujeito expresso e a pessoa verbal: o sujeito que fala ou escreve
se inclui no sujeito enunciado.

Língua Portuguesa 33
Apostila Digital Licenciada para Alice Caroline Guarino dos Santos - alice.guarino@hotmail.com (Proibida a Revenda)
APOSTILAS OPÇÃO
Exemplo: “Na noite seguinte estávamos reunidas algumas Frase: é todo enunciado capaz de transmitir, a quem nos
pessoas.” (Machado de Assis) ouve ou lê, tudo o que pensamos, queremos ou sentimos. Pode
revestir as mais variadas formas, desde a simples palavra até
Questões o período mais complexo, elaborado segundo os padrões
sintáticos do idioma. São exemplos de frases:
01. Ao dizer que os shoppings são “cidades”, o autor do texto
faz uso de um tipo de linguagem figurada denominada Socorro!
(A) metonímia. Muito obrigado!
(B) eufemismo. Que horror!
(C) hipérbole. Sentinela, alerta!
(D) metáfora. Cada um por si e Deus por todos.
(E) catacrese. Grande nau, grande tormenta.
Por que agridem a natureza?
02. Identifique a figura de linguagem presente na tira “Tudo seco em redor.” (Graciliano Ramos)
seguinte: “Boa tarde, mãe Margarida!” (Graciliano Ramos)
“Fumaça nas chaminés, o céu tranquilo, limpo o terreiro.”
(Adonias Filho)
“As luzes da cidade estavam amortecidas.” (Érico Veríssimo)
“Tropas do exército regular do Sul, ajustadas pelos
seus aliados brancos de além mar, tinham sido levadas em
helicópteros para o lugar onde se presumia estivesse o inimigo,
mas este se havia sumido por completo.” (Érico Veríssimo)

As frases são proferidas com entoação e pausas especiais,


indicadas na escrita pelos sinais de pontuação. Muitas frases,
(A) metonímia principalmente as que se desviam do esquema sujeito +
(B) prosopopeia predicado, só podem ser entendidas dentro do contexto (=
(C) hipérbole o escrito em que figuram) e na situação (= o ambiente, as
(D) eufemismo circunstâncias) em que o falante se encontra. Chamam-se frases
(E) onomatopeia nominais as que se apresentam sem o verbo. Exemplo: Tudo
parado e morto.
03.
Está tão quente que dá para fritar um ovo no asfalto. Quanto ao sentido, as frases podem ser:
O dito popular é, na maioria das vezes, uma figura de Declarativas: aquela através da qual se enuncia algo,
linguagem. Entre as 14h30min e às 15h desta terça-feira, de forma afirmativa ou negativa. Encerram a declaração ou
horário do dia em que o calor é mais intenso, a temperatura enunciação de um juízo acerca de alguém ou de alguma coisa:
do asfalto, medida com um termômetro de contato, chegou a Paulo parece inteligente. (afirmativa)
65ºC. Para fritar um ovo, seria preciso que o local alcançasse Nunca te esquecerei. (negativa)
aproximadamente 90 ºC. Neli não quis montar o cavalo velho, de pêlo ruço. (negativa)
Disponível em: http://zerohora.clicrbs.com.br. Acesso em:
22 jan. 2014. Interrogativas: aquela da qual se pergunta algo, direta
(com ponto de interrogação) ou indiretamente (sem ponto de
O texto cita que o dito popular “está tão quente que dá para interrogação). São uma pergunta, uma interrogação:
fritar um ovo no asfalto” expressa uma figura de linguagem. O Por que chegaste tão tarde?
autor do texto refere-se a qual figura de linguagem? Gostaria de saber que horas são.
(A) Eufemismo. “Por que faço eu sempre o que não queria” (Fernando Pessoa)
(B) Hipérbole.
(C) Paradoxo. Imperativas: aquela através da qual expressamos uma
(D) Metonímia. ordem, pedido ou súplica, de forma afirmativa ou negativa.
(E) Hipérbato. Contêm uma ordem, proibição, exortação ou pedido:
Respostas “Cale-se! Respeite este templo.” (afirmativa)
01. D\02. D\03. B Não cometa imprudências. (negativa)
“Não me leves para o mar.” (negativa)
9. análise sintática; 10. orações e
Exclamativas: aquela através da qual externamos uma
seus termos; 11. coordenação e admiração. Traduzem admiração, surpresa, arrependimento,
subordinação; etc.:
Como eles são audaciosos!
Não voltaram mais!
Análise Sintática
Optativas: É aquela através da qual se exprime um desejo:
A Análise Sintática examina a estrutura do período, divide Bons ventos o levem!
e classifica as orações que o constituem e reconhece a função Oxalá não sejam vãos tantos sacrifícios!
sintática dos termos de cada oração. “E queira Deus que te não enganes, menino!” (Carlos de Laet)
Daremos uma ideia do que seja frase, oração, período, termo,
função sintática e núcleo de um termo da oração. Imprecativas: Encerram uma imprecação (praga, maldição):
As palavras, tanto na expressão escrita como na oral, são “Esta luz me falte, se eu minto, senhor!” (Camilo Castelo
reunidas e ordenadas em frases. Pela frase é que se alcança Branco)
o objetivo do discurso, ou seja, da atividade linguística: a “Não encontres amor nas mulheres!” (Gonçalves Dias)
comunicação com o ouvinte ou o leitor. “Maldito seja quem arme ciladas no seu caminho!”
Frase, Oração e Período são fatores constituintes de (Domingos Carvalho da Silva)
qualquer texto escrito em prosa, pois o mesmo compõe-se de
uma sequência lógica de ideias, todas organizadas e dispostas Como se vê dos exemplos citados, os diversos tipos de frase
em parágrafos minuciosamente construídos. podem encerrar uma afirmação ou uma negação. No primeiro

Língua Portuguesa 34
Apostila Digital Licenciada para Alice Caroline Guarino dos Santos - alice.guarino@hotmail.com (Proibida a Revenda)
APOSTILAS OPÇÃO
caso, a frase é afirmativa, no segundo, negativa. O que caracteriza Choveu durante a noite. (a oração toda predicado)
e distingue esses diferentes tipos de frase é a entoação, ora
ascendente ora descendente. O sujeito é o termo da frase que concorda com o verbo em
Muitas vezes, as frases assumem sentidos que só podem ser número e pessoa. É normalmente o «ser de quem se declara
integralmente captados se atentarmos para o contexto em que algo», «o tema do que se vai comunicar».
são empregadas. É o caso, por exemplo, das situações em que se O predicado é a parte da oração que contém “a informação
explora a ironia. Pense, por exemplo, na frase “Que educação!”, nova para o ouvinte”. Normalmente, ele se refere ao sujeito,
usada quando se vê alguém invadindo, com seu carro, a faixa de constituindo a declaração do que se atribui ao sujeito.
pedestres. Nesse caso, ela expressa exatamente o contrário do
que aparentemente diz. Observe: O amor é eterno. O tema, o ser de quem se declara
A entoação é um elemento muito importante da frase falada, algo, o sujeito, é “O amor”. A declaração referente a “o amor”, ou
pois nos dá uma ampla possibilidade de expressão. Dependendo seja, o predicado, é «é eterno».
de como é dita, uma frase simples como «É ela.» pode indicar
constatação, dúvida, surpresa, indignação, decepção, etc. Já na frase: Os rapazes jogam futebol. O sujeito é “Os rapazes”,
A mesma frase pode assumir sentidos diferentes, conforme o que identificamos por ser o termo que concorda em número e
tom com que a proferimos. Observe: pessoa com o verbo “jogam”. O predicado é “jogam futebol”.
Olavo esteve aqui.
Olavo esteve aqui? Núcleo de um termo é a palavra principal (geralmente um
Olavo esteve aqui?! substantivo, pronome ou verbo), que encerra a essência de
Olavo esteve aqui! sua significação. Nos exemplos seguintes, as palavras amigo e
revestiu são o núcleo do sujeito e do predicado, respectivamente:
Questões “O amigo retardatário do presidente prepara-se para
desembarcar.” (Aníbal Machado)
01. Marque apenas as frases nominais: A avezinha revestiu o interior do ninho com macias plumas.
(A) Que voz estranha!
(B) A lanterna produzia boa claridade. Os termos da oração da língua portuguesa são classificados
(C) As risadas não eram normais. em três grandes níveis:
(D) Luisinho, não! - Termos Essenciais da Oração: Sujeito e Predicado.

02. Classifique as frases em declarativa, interrogativa, - Termos Integrantes da Oração: Complemento Nominal e
exclamativa, optativa ou imperativa. Complementos Verbais (Objeto Direto, Objeto indireto e Agente
(A) Você está bem? da Passiva).
(B) Não olhe; não olhe, Luisinho!
(C) Que alívio! - Termos Acessórios da Oração: Adjunto Adnominal,
(D) Tomara que Luisinho não fique impressionado! Adjunto Adverbial, Aposto e Vocativo.
(E) Você se machucou?
(F) A luz jorrou na caverna. Termos Essenciais da Oração: São dois os termos essenciais
(G) Agora suma, seu monstro! (ou fundamentais) da oração: sujeito e predicado. Exemplos:
(H) O túnel ficava cada vez mais escuro.

Respostas Sujeito Predicado


Pobreza não é vileza.
01. “a” e “d”
Os sertanistas capturavam os índios.
02. a) interrogativa; b) imperativa; c) exclamativa; d) Um vento áspero sacudia as árvores.
optativa; e) interrogativa; f) declarativa; g) imperativa; h)
declarativa Sujeito: é equivocado dizer que o sujeito é aquele que pratica
Oração uma ação ou é aquele (ou aquilo) do qual se diz alguma coisa. Ao
fazer tal afirmação estamos considerando o aspecto semântico
Oração: é todo enunciado linguístico dotado de sentido, do sujeito (agente de uma ação) ou o seu aspecto estilístico
porém há, necessariamente, a presença do verbo. A oração (o tópico da sentença). Já que o sujeito é depreendido de uma
encerra uma frase (ou segmento de frase), várias frases ou um análise sintática, vamos restringir a definição apenas ao seu
período, completando um pensamento e concluindo o enunciado papel sintático na sentença: aquele que estabelece concordância
através de ponto final, interrogação, exclamação e, em alguns com o núcleo do predicado. Quando se trata de predicado verbal,
casos, através de reticências. o núcleo é sempre um verbo; sendo um predicado nominal, o
Em toda oração há um verbo ou locução verbal (às vezes núcleo é sempre um nome. Então têm por características básicas:
elípticos). Não têm estrutura sintática, portanto não são orações, - estabelecer concordância com o núcleo do predicado;
não podem ser analisadas sintaticamente frases como: - apresentar-se como elemento determinante em relação ao
predicado;
Socorro! - constituir-se de um substantivo, ou pronome substantivo
Com licença! ou, ainda, qualquer palavra substantivada.
Que rapaz impertinente!
Muito riso, pouco siso. Exemplo:

Na oração as palavras estão relacionadas entre si, como A padaria está fechada hoje.
partes de um conjunto harmônico: elas formam os termos está fechada hoje: predicado nominal
ou as unidades sintáticas da oração. Cada termo da oração fechada: nome adjetivo = núcleo do predicado
desempenha uma função sintática. Geralmente apresentam dois a padaria: sujeito
grupos de palavras: um grupo sobre o qual se declara alguma padaria: núcleo do sujeito - nome feminino singular
coisa (o sujeito), e um grupo que apresenta uma declaração (o
predicado), e, excepcionalmente, só o predicado. Exemplo: No interior de uma sentença, o sujeito é o termo determinante,
ao passo que o predicado é o termo determinado. Essa posição
A menina banhou-se na cachoeira. de determinante do sujeito em relação ao predicado adquire
A menina – sujeito sentido com o fato de ser possível, na língua portuguesa, uma
banhou-se na cachoeira – predicado sentença sem sujeito, mas nunca uma sentença sem predicado.

Língua Portuguesa 35
Apostila Digital Licenciada para Alice Caroline Guarino dos Santos - alice.guarino@hotmail.com (Proibida a Revenda)
APOSTILAS OPÇÃO
Exemplo: Indeterminado: quando não se indica o agente da ação
As formigas invadiram minha casa. verbal: Atropelaram uma senhora na esquina. (Quem atropelou
as formigas: sujeito = termo determinante a senhora? Não se diz, não se sabe quem a atropelou.); Come-se
invadiram minha casa: predicado = termo determinado bem naquele restaurante.
Há formigas na minha casa.
há formigas na minha casa: predicado = termo determinado Observações:
sujeito: inexistente - Não confundir sujeito indeterminado com sujeito oculto.
- Sujeito formado por pronome indefinido não é
O sujeito sempre se manifesta em termos de sintagma indeterminado, mas expresso: Alguém me ensinará o caminho.
nominal, isto é, seu núcleo é sempre um nome. Quando esse Ninguém lhe telefonou.
nome se refere a objetos das primeira e segunda pessoas, o - Assinala-se a indeterminação do sujeito usando-se o
sujeito é representado por um pronome pessoal do caso reto (eu, verbo na 3ª pessoa do plural, sem referência a qualquer agente
tu, ele, etc.). Se o sujeito se refere a um objeto da terceira pessoa, já expresso nas orações anteriores: Na rua olhavam-no com
sua representação pode ser feita através de um substantivo, de admiração; “Bateram palmas no portãozinho da frente.”; “De
um pronome substantivo ou de qualquer conjunto de palavras, qualquer modo, foi uma judiação matarem a moça.”
cujo núcleo funcione, na sentença, como um substantivo. - Assinala-se a indeterminação do sujeito com um verbo
Exemplos: ativo na 3ª pessoa do singular, acompanhado do pronome se. O
Eu acompanho você até o guichê. pronome se, neste caso, é índice de indeterminação do sujeito.
eu: sujeito = pronome pessoal de primeira pessoa Pode ser omitido junto de infinitivos.
Vocês disseram alguma coisa? Aqui vive-se bem.
vocês: sujeito = pronome pessoal de segunda pessoa Devagar se vai ao longe.
Marcos tem um fã-clube no seu bairro. Quando se é jovem, a memória é mais vivaz.
Marcos: sujeito = substantivo próprio Trata-se de fenômenos que nem a ciência sabe explicar.
Ninguém entra na sala agora.
ninguém: sujeito = pronome substantivo - Assinala-se a indeterminação do sujeito deixando-se o
O andar deve ser uma atividade diária. verbo no infinitivo impessoal: Era penoso carregar aqueles
o andar: sujeito = núcleo: verbo substantivado nessa oração fardos enormes; É triste assistir a estas cenas repulsivas.

Além dessas formas, o sujeito também pode se constituir Normalmente, o sujeito antecede o predicado; todavia, a
de uma oração inteira. Nesse caso, a oração recebe o nome de posposição do sujeito ao verbo é fato corriqueiro em nossa
oração substantiva subjetiva: língua.
Exemplos:
É difícil optar por esse ou aquele doce... É fácil este problema!
É difícil: oração principal Vão-se os anéis, fiquem os dedos.
optar por esse ou aquele doce: oração substantiva subjetiva “Breve desapareceram os dois guerreiros entre as árvores.”
(José de Alencar)
O sujeito é constituído por um substantivo ou pronome, ou
por uma palavra ou expressão substantivada. Exemplos: Sem Sujeito: constituem a enunciação pura e absoluta de um
fato, através do predicado; o conteúdo verbal não é atribuído a
O sino era grande. nenhum ser. São construídas com os verbos impessoais, na 3ª
Ela tem uma educação fina. pessoa do singular: Havia ratos no porão; Choveu durante o jogo.
Vossa Excelência agiu com imparcialidade. Observação: São verbos impessoais: Haver (nos sentidos
Isto não me agrada. de existir, acontecer, realizar-se, decorrer), Fazer, passar, ser
e estar, com referência ao tempo e Chover, ventar, nevar, gear,
O núcleo (isto é, a palavra base) do sujeito é, pois, um relampejar, amanhecer, anoitecer e outros que exprimem
substantivo ou pronome. Em torno do núcleo podem aparecer fenômenos meteorológicos.
palavras secundárias (artigos, adjetivos, locuções adjetivas, etc.).
Exemplo: “Todos os ligeiros rumores da mata tinham uma Predicado: assim como o sujeito, o predicado é um
voz para a selvagem filha do sertão.” (José de Alencar) segmento extraído da estrutura interna das orações ou das
frases, sendo, por isso, fruto de uma análise sintática. Nesse
O sujeito pode ser: sentido, o predicado é sintaticamente o segmento linguístico
que estabelece concordância com outro termo essencial
Simples: quando tem um só núcleo: As rosas têm espinhos; da oração, o sujeito, sendo este o termo determinante (ou
“Um bando de galinhas-d’angola atravessa a rua em fila indiana.” subordinado) e o predicado o termo determinado (ou principal).
Composto: quando tem mais de um núcleo: “O burro e o Não se trata, portanto, de definir o predicado como “aquilo
cavalo nadavam ao lado da canoa.” que se diz do sujeito” como fazem certas gramáticas da língua
Expresso: quando está explícito, enunciado: Eu viajarei portuguesa, mas sim estabelecer a importância do fenômeno
amanhã. da concordância entre esses dois termos essenciais da oração.
Oculto (ou elíptico): quando está implícito, isto é, quando Então têm por características básicas: apresentar-se como
não está expresso, mas se deduz do contexto: Viajarei amanhã. elemento determinado em relação ao sujeito; apontar um
(sujeito: eu, que se deduz da desinência do verbo); “Um soldado atributo ou acrescentar nova informação ao sujeito.
saltou para a calçada e aproximou-se.” (o sujeito, soldado, está
expresso na primeira oração e elíptico na segunda: e (ele) Exemplo:
aproximou-se.); Crianças, guardem os brinquedos. (sujeito:
vocês) Carolina conhece os índios da Amazônia.
Agente: se faz a ação expressa pelo verbo da voz ativa: O Nilo sujeito: Carolina = termo determinante
fertiliza o Egito. predicado: conhece os índios da Amazônia = termo
Paciente: quando sofre ou recebe os efeitos da ação expressa determinado
pelo verbo passivo: O criminoso é atormentado pelo remorso;
Muitos sertanistas foram mortos pelos índios; Construíram-se Nesses exemplos podemos observar que a concordância é
açudes. (= Açudes foram construídos.) estabelecida entre algumas poucas palavras dos dois termos
Agente e Paciente: quando o sujeito realiza a ação expressa essenciais. No primeiro exemplo, entre “Carolina” e “conhece”;
por um verbo reflexivo e ele mesmo sofre ou recebe os efeitos no segundo exemplo, entre “nós” e “fazemos”. Isso se dá porque
dessa ação: O operário feriu-se durante o trabalho; Regina a concordância é centrada nas palavras que são núcleos, isto
trancou-se no quarto. é, que são responsáveis pela principal informação naquele

Língua Portuguesa 36
Apostila Digital Licenciada para Alice Caroline Guarino dos Santos - alice.guarino@hotmail.com (Proibida a Revenda)
APOSTILAS OPÇÃO
segmento. No predicado o núcleo pode ser de dois tipos: um invejo, aspiro, etc., não transmitiriam informações completas:
nome, quase sempre um atributo que se refere ao sujeito da puxou o quê? Não invejo a quem? Não aspiro a quê?
oração, ou um verbo (ou locução verbal). No primeiro caso, Os verbos de predicação completa denominam-se
temos um predicado nominal (seu núcleo significativo é um intransitivos e os de predicação incompleta, transitivos. Os
nome, substantivo, adjetivo, pronome, ligado ao sujeito por verbos transitivos subdividem-se em: transitivos diretos,
um verbo de ligação) e no segundo um predicado verbal (seu transitivos indiretos e transitivos diretos e indiretos
núcleo é um verbo, seguido, ou não, de complemento(s) ou (bitransitivos).
termos acessórios). Quando, num mesmo segmento o nome e o Além dos verbos transitivos e intransitivos, quem encerram
verbo são de igual importância, ambos constituem o núcleo do uma noção definida, um conteúdo significativo, existem os de
predicado e resultam no tipo de predicado verbo-nominal (tem ligação, verbos que entram na formação do predicado nominal,
dois núcleos significativos: um verbo e um nome). Exemplos: relacionando o predicativo com o sujeito.
Quanto à predicação classificam-se, pois os verbos em:
Minha empregada é desastrada. Intransitivos: são os que não precisam de complemento,
predicado: é desastrada pois têm sentido completo.
núcleo do predicado: desastrada = atributo do sujeito “Três contos bastavam, insistiu ele.” (Machado de Assis)
tipo de predicado: nominal “Os guerreiros Tabajaras dormem.” (José de Alencar)
“A pobreza e a preguiça andam sempre em companhia.”
O núcleo do predicado nominal chama-se predicativo (Marquês de Maricá)
do sujeito, porque atribui ao sujeito uma qualidade ou
característica. Os verbos de ligação (ser, estar, parecer, etc.) Observações: Os verbos intransitivos podem vir
funcionam como um elo entre o sujeito e o predicado. acompanhados de um adjunto adverbial e mesmo de um
predicativo (qualidade, características): Fui cedo; Passeamos
A empreiteira demoliu nosso antigo prédio. pela cidade; Cheguei atrasado; Entrei em casa aborrecido.
predicado: demoliu nosso antigo prédio As orações formadas com verbos intransitivos não podem
núcleo do predicado: demoliu = nova informação sobre o “transitar” (= passar) para a voz passiva. Verbos intransitivos
sujeito passam, ocasionalmente, a transitivos quando construídos com
tipo de predicado: verbal o objeto direto ou indireto.
- “Inutilmente a minha alma o chora!” (Cabral do Nascimento)
Os manifestantes desciam a rua desesperados. - “Depois me deitei e dormi um sono pesado.” (Luís Jardim)
predicado: desciam a rua desesperados - “Morrerás morte vil da mão de um forte.” (Gonçalves Dias)
núcleos do predicado: desciam = nova informação sobre o - “Inútil tentativa de viajar o passado, penetrar no mundo
sujeito; desesperados = atributo do sujeito que já morreu...” (Ciro dos Anjos)
tipo de predicado: verbo-nominal
Alguns verbos essencialmente intransitivos: anoitecer,
Nos predicados verbais e verbo-nominais o verbo é crescer, brilhar, ir, agir, sair, nascer, latir, rir, tremer, brincar,
responsável também por definir os tipos de elementos que chegar, vir, mentir, suar, adoecer, etc.
aparecerão no segmento. Em alguns casos o verbo sozinho basta
para compor o predicado (verbo intransitivo). Em outros casos Transitivos Diretos: são os que pedem um objeto direto, isto
é necessário um complemento que, juntamente com o verbo, é, um complemento sem preposição. Pertencem a esse grupo:
constituem a nova informação sobre o sujeito. De qualquer julgar, chamar, nomear, eleger, proclamar, designar, considerar,
forma, esses complementos do verbo não interferem na tipologia declarar, adotar, ter, fazer, etc. Exemplos:
do predicado. Comprei um terreno e construí a casa.
Entretanto, é muito comum a elipse (ou omissão) do verbo, “Trabalho honesto produz riqueza honrada.” (Marquês de
quando este puder ser facilmente subentendido, em geral por Maricá)
estar expresso ou implícito na oração anterior. Exemplos: “Então, solenemente Maria acendia a lâmpada de sábado.”
(Guedes de Amorim)
“A fraqueza de Pilatos é enorme, a ferocidade dos algozes
inexcedível.” (Machado de Assis) (Está subentendido o verbo é Dentre os verbos transitivos diretos merecem destaque os
depois de algozes) que formam o predicado verbo nominal e se constrói com o
“Mas o sal está no Norte, o peixe, no Sul” (Paulo Moreira da complemento acompanhado de predicativo. Exemplos:
Silva) (Subentende-se o verbo está depois de peixe) Consideramos o caso extraordinário.
“A cidade parecia mais alegre; o povo, mais contente.” (Povina Inês trazia as mãos sempre limpas.
Cavalcante) (isto é: o povo parecia mais contente) O povo chamava-os de anarquistas.
Julgo Marcelo incapaz disso.
Chama-se predicação verbal o modo pelo qual o verbo
forma o predicado. Observações: Os verbos transitivos diretos, em geral, podem
Há verbos que, por natureza, tem sentido completo, ser usados também na voz passiva; Outra característica desses
podendo, por si mesmos, constituir o predicado: são os verbos verbos é a de poderem receber como objeto direto, os pronomes
de predicação completa denominados intransitivos. Exemplo: o, a, os, as: convido-o, encontro-os, incomodo-a, conheço-as; Os
verbos transitivos diretos podem ser construídos acidentalmente
As flores murcharam. com preposição, a qual lhes acrescenta novo matiz semântico:
Os animais correm. arrancar da espada; puxar da faca; pegar de uma ferramenta;
As folhas caem. tomar do lápis; cumprir com o dever; Alguns verbos transitivos
diretos: abençoar, achar, colher, avisar, abraçar, comprar,
Outros verbos há, pelo contrário, que para integrarem castigar, contrariar, convidar, desculpar, dizer, estimar, elogiar,
o predicado necessitam de outros termos: são os verbos de entristecer, encontrar, ferir, imitar, levar, perseguir, prejudicar,
predicação incompleta, denominados transitivos. Exemplos: receber, saldar, socorrer, ter, unir, ver, etc.

João puxou a rede. Transitivos Indiretos: são os que reclamam um


“Não invejo os ricos, nem aspiro à riqueza.” (Oto Lara complemento regido de preposição, chamado objeto indireto.
Resende) Exemplos:
“Não simpatizava com as pessoas investidas no poder.” “Ninguém perdoa ao quarentão que se apaixona por uma
(Camilo Castelo Branco) adolescente.” (Ciro dos Anjos)
“Populares assistiam à cena aparentemente apáticos e
Observe que, sem os seus complementos, os verbos puxou, neutros.” (Érico Veríssimo)

Língua Portuguesa 37
Apostila Digital Licenciada para Alice Caroline Guarino dos Santos - alice.guarino@hotmail.com (Proibida a Revenda)
APOSTILAS OPÇÃO
“Lúcio não atinava com essa mudança instantânea.” (José constituição do predicado verbo-nominal. Exemplos:
Américo) O trem chegou atrasado. (=O trem chegou e estava
“Do que eu mais gostava era do tempo do retiro espiritual.” atrasado.)
(José Geraldo Vieira) O menino abriu a porta ansioso.
Todos partiram alegres.
Observações: Entre os verbos transitivos indiretos importa
distinguir os que se constroem com os pronomes objetivos lhe, Observações: O predicativo subjetivo às vezes está
lhes. Em geral são verbos que exigem a preposição a: agradar-lhe, preposicionado; Pode o predicativo preceder o sujeito e até
agradeço-lhe, apraz-lhe, bate-lhe, desagrada-lhe, desobedecem- mesmo ao verbo: São horríveis essas coisas!; Que linda
lhe, etc. Entre os verbos transitivos indiretos importa distinguir estava Amélia!; Completamente feliz ninguém é.; Raros são os
os que não admitem para objeto indireto as formas oblíquas verdadeiros líderes.; Quem são esses homens?; Lentos e tristes,
lhe, lhes, construindo-se com os pronomes retos precedidos de os retirantes iam passando.; Novo ainda, eu não entendia certas
preposição: aludir a ele, anuir a ele, assistir a ela, atentar nele, coisas.; Onde está a criança que fui?
depender dele, investir contra ele, não ligar para ele, etc. Predicativo do Objeto: é o termo que se refere ao objeto de
Em princípio, verbos transitivos indiretos não comportam um verbo transitivo. Exemplos:
a forma passiva. Excetuam-se pagar, perdoar, obedecer, e O juiz declarou o réu inocente.
pouco mais, usados também como transitivos diretos: João O povo elegeu-o deputado.
paga (perdoa, obedece) o médico. O médico é pago (perdoado,
obedecido) por João. Há verbos transitivos indiretos, como Observações: O predicativo objetivo, como vemos dos
atirar, investir, contentar-se, etc., que admitem mais de uma exemplos acima, às vezes vem regido de preposição. Esta, em
preposição, sem mudança de sentido. Outros mudam de sentido certos casos, é facultativa; O predicativo objetivo geralmente
com a troca da preposição, como nestes exemplos: Trate de sua se refere ao objeto direto. Excepcionalmente, pode referir-se
vida. (tratar=cuidar). É desagradável tratar com gente grosseira. ao objeto indireto do verbo chamar. Chamavam-lhe poeta;
(tratar=lidar). Verbos como aspirar, assistir, dispor, servir, etc., Podemos antepor o predicativo a seu objeto: O advogado
variam de significação conforme sejam usados como transitivos considerava indiscutíveis os direitos da herdeira.; Julgo
diretos ou indiretos. inoportuna essa viagem.; “E até embriagado o vi muitas
vezes.”; “Tinha estendida a seus pés uma planta rústica da
Transitivos Diretos e Indiretos: são os que se usam com cidade.”; “Sentia ainda muito abertos os ferimentos que aquele
dois objetos: um direto, outro indireto, concomitantemente. choque com o mundo me causara.”
Exemplos:
No inverno, Dona Cléia dava roupas aos pobres. Termos Integrantes da Oração
A empresa fornece comida aos trabalhadores.
Oferecemos flores à noiva. Chamam-se termos integrantes da oração os que completam
Ceda o lugar aos mais velhos. a significação transitiva dos verbos e nomes. Integram (inteiram,
completam) o sentido da oração, sendo por isso indispensável à
De Ligação: Os que ligam ao sujeito uma palavra ou compreensão do enunciado. São os seguintes:
expressão chamada predicativo. Esses verbos, entram na - Complemento Verbais (Objeto Direto e Objeto Indireto);
formação do predicado nominal. Exemplos: - Complemento Nominal;
A Terra é móvel. - Agente da Passiva.
A água está fria.
O moço anda (=está) triste. Objeto Direto: é o complemento dos verbos de predicação
A Lua parecia um disco. incompleta, não regido, normalmente, de preposição. Exemplos:
As plantas purificaram o ar.
Observações: Os verbos de ligação não servem apenas de “Nunca mais ele arpoara um peixe-boi.” (Ferreira Castro)
anexo, mas exprimem ainda os diversos aspectos sob os quais Procurei o livro, mas não o encontrei.
se considera a qualidade atribuída ao sujeito. O verbo ser, por Ninguém me visitou.
exemplo, traduz aspecto permanente e o verbo estar, aspecto
transitório: Ele é doente. (aspecto permanente); Ele está doente. O objeto direto tem as seguintes características:
(aspecto transitório). Muito desses verbos passam à categoria - Completa a significação dos verbos transitivos diretos;
dos intransitivos em frases como: Era =existia) uma vez uma - Normalmente, não vem regido de preposição;
princesa.; Eu não estava em casa.; Fiquei à sombra.; Anda com - Traduz o ser sobre o qual recai a ação expressa por um
dificuldades.; Parece que vai chover. verbo ativo: Caim matou Abel.
- Torna-se sujeito da oração na voz passiva: Abel foi morto
Os verbos, relativamente à predicação, não têm classificação por Caim.
fixa, imutável. Conforme a regência e o sentido que apresentam
na frase, podem pertencer ora a um grupo, ora a outro. Exemplos: O objeto direto pode ser constituído:
O homem anda. (intransitivo) - Por um substantivo ou expressão substantivada: O lavrador
O homem anda triste. (de ligação) cultiva a terra.; Unimos o útil ao agradável.
- Pelos pronomes oblíquos o, a, os, as, me, te, se, nos, vos:
O cego não vê. (intransitivo) Espero-o na estação.; Estimo-os muito.; Sílvia olhou-se ao
O cego não vê o obstáculo. (transitivo direto) espelho.; Não me convidas?; Ela nos chama.; Avisamo-lo a
tempo.; Procuram-na em toda parte.; Meu Deus, eu vos amo.;
Não dei com a chave do enigma. (transitivo indireto) “Marchei resolutamente para a maluca e intimei-a a ficar
Os pais dão conselhos aos filhos. (transitivo direto e indireto) quieta.”; “Vós haveis de crescer, perder-vos-ei de vista.”
- Por qualquer pronome substantivo: Não vi ninguém na
Predicativo: Há o predicativo do sujeito e o predicativo do loja.; A árvore que plantei floresceu. (que: objeto direto de
objeto. plantei); Onde foi que você achou isso? Quando vira as folhas do
livro, ela o faz com cuidado.; “Que teria o homem percebido nos
Predicativo do Sujeito: é o termo que exprime um atributo, meus escritos?”
um estado ou modo de ser do sujeito, ao qual se prende por um
verbo de ligação, no predicado nominal. Exemplos: Frequentemente transitivam-se verbos intransitivos, dando-
A bandeira é o símbolo da Pátria. se-lhes por objeto direto uma palavra cognata ou da mesma
A mesa era de mármore. esfera semântica:
“Viveu José Joaquim Alves vida tranquila e patriarcal.”
Além desse tipo de predicativo, outro existe que entra na (Vivaldo Coaraci)

Língua Portuguesa 38
Apostila Digital Licenciada para Alice Caroline Guarino dos Santos - alice.guarino@hotmail.com (Proibida a Revenda)
APOSTILAS OPÇÃO
“Pela primeira vez chorou o choro da tristeza.” (Aníbal preposição necessária e sem valor circunstancial. Representa,
Machado) ordinariamente, o ser a que se destina ou se refere à ação verbal:
“Nenhum de nós pelejou a batalha de Salamina.” (Machado “Nunca desobedeci a meu pai”. O objeto indireto completa a
de Assis) significação dos verbos:
Em tais construções é de rigor que o objeto venha
acompanhado de um adjunto. - Transitivos Indiretos: Assisti ao jogo; Assistimos à missa e
à festa; Aludiu ao fato; Aspiro a uma vida calma.
Objeto Direto Preposicionado: Há casos em que o objeto - Transitivos Diretos e Indiretos (na voz ativa ou passiva):
direto, isto é, o complemento de verbos transitivos diretos, vem Dou graças a Deus; Ceda o lugar aos mais velhos; Dedicou sua
precedido de preposição, geralmente a preposição a. Isto ocorre vida aos doentes e aos pobres; Disse-lhe a verdade. (Disse a
principalmente: verdade ao moço.)
- Quando o objeto direto é um pronome pessoal tônico:
Deste modo, prejudicas a ti e a ela.; “Mas dona Carolina amava O objeto indireto pode ainda acompanhar verbos de outras
mais a ele do que aos outros filhos.”; “Pareceu-me que Roberto categorias, os quais, no caso, são considerados acidentalmente
hostilizava antes a mim do que à ideia.”; “Ricardina lastimava o transitivos indiretos: A bom entendedor meia palavra basta;
seu amigo como a si própria.”; “Amava-a tanto como a nós”. Sobram-lhe qualidades e recursos. (lhe=a ele); Isto não lhe
- Quando o objeto é o pronome relativo quem: “Pedro convém; A proposta pareceu-lhe aceitável.
Severiano tinha um filho a quem idolatrava.”; “Abraçou a todos;
deu um beijo em Adelaide, a quem felicitou pelo desenvolvimento Observações: Há verbos que podem construir-se com dois
das suas graças.”; “Agora sabia que podia manobrar com ele, com objetos indiretos, regidos de preposições diferentes: Rogue a
aquele homem a quem na realidade também temia, como todos Deus por nós.; Ela queixou-se de mim a seu pai.; Pedirei para
ali”. ti a meu senhor um rico presente; Não confundir o objeto direto
- Quando precisamos assegurar a clareza da frase, evitando com o complemento nominal nem com o adjunto adverbial; Em
que o objeto direto seja tomado como sujeito, impedindo frases como “Para mim tudo eram alegrias”, “Para ele nada é
construções ambíguas: Convence, enfim, ao pai o filho amado.; impossível”, os pronomes em destaque podem ser considerados
“Vence o mal ao remédio.”; “Tratava-me sem cerimônia, como a adjuntos adverbiais.
um irmão.”; A qual delas iria homenagear o cavaleiro?
- Em expressões de reciprocidade, para garantir a clareza e a O objeto indireto é sempre regido de preposição, expressa
eufonia da frase: “Os tigres despedaçam-se uns aos outros.”; “As ou implícita. A preposição está implícita nos pronomes objetivos
companheiras convidavam-se umas às outras.”; “Era o abraço de indiretos (átonos) me, te, se, lhe, nos, vos, lhes. Exemplos:
duas criaturas que só tinham uma à outra”. Obedece-me. (=Obedece a mim.); Isto te pertence. (=Isto
- Com nomes próprios ou comuns, referentes a pessoas, pertence a ti.); Rogo-lhe que fique. (=Rogo a você...); Peço-
principalmente na expressão dos sentimentos ou por amor da vos isto. (=Peço isto a vós.). Nos demais casos a preposição é
eufonia da frase: Judas traiu a Cristo.; Amemos a Deus sobre expressa, como característica do objeto indireto: Recorro a
todas as coisas. “Provavelmente, enganavam é a Pedro.”; “O Deus.; Dê isto a (ou para) ele.; Contenta-se com pouco.; Ele
estrangeiro foi quem ofendeu a Tupã”. só pensa em si.; Esperei por ti.; Falou contra nós.; Conto com
- Em construções enfáticas, nas quais antecipamos o objeto você.; Não preciso disto.; O filme a que assisti agradou ao
direto para dar-lhe realce: A você é que não enganam!; Ao público.; Assisti ao desenrolar da luta.; A coisa de que mais
médico, confessor e letrado nunca enganes.; “A este confrade gosto é pescar.; A pessoa a quem me refiro você a conhece.; Os
conheço desde os seus mais tenros anos”. obstáculos contra os quais luto são muitos.; As pessoas com
- Sendo objeto direto o numeral ambos(as): “O aguaceiro quem conto são poucas.
caiu, molhou a ambos.”; “Se eu previsse que os matava a
ambos...”. Como atestam os exemplos acima, o objeto indireto é
- Com certos pronomes indefinidos, sobretudo referentes a representado pelos substantivos (ou expressões substantivas)
pessoas: Se todos são teus irmãos, por que amas a uns e odeias a ou pelos pronomes. As preposições que o ligam ao verbo são: a,
outros?; Aumente a sua felicidade, tornando felizes também aos com, contra, de, em, para e por.
outros.; A quantos a vida ilude!.
- Em certas construções enfáticas, como puxar (ou arrancar) Objeto Indireto Pleonástico: à semelhança do objeto direto,
da espada, pegar da pena, cumprir com o dever, atirar com os o objeto indireto pode vir repetido ou reforçado, por ênfase.
livros sobre a mesa, etc.: “Arrancam das espadas de aço fino...”; Exemplos: “A mim o que me deu foi pena.”; “Que me importa
“Chegou a costureira, pegou do pano, pegou da agulha, pegou a mim o destino de uma mulher tísica...? “E, aos brigões,
da linha, enfiou a linha na agulha e entrou a coser.”; “Imagina-se incapazes de se moverem, basta-lhes xingarem-se a distância.”
a consternação de Itaguaí, quando soube do caso.”
Complemento Nominal: é o termo complementar reclamado
Observações: Nos quatro primeiros casos estudados a pela significação transitiva, incompleta, de certos substantivos,
preposição é de rigor, nos cinco outros, facultativa; A substituição adjetivos e advérbios. Vem sempre regido de preposição.
do objeto direto preposicionado pelo pronome oblíquo átono, Exemplos: A defesa da pátria; Assistência às aulas; “O ódio ao
quando possível, se faz com as formas o(s), a(s) e não lhe, mal é amor do bem, e a ira contra o mal, entusiasmo divino.”;
lhes: amar a Deus (amá-lo); convencer ao amigo (convencê- “Ah, não fosse ele surdo à minha voz!”
lo); O objeto direto preposicionado, é obvio, só ocorre com
verbo transitivo direto; Podem resumir-se em três as razões Observações: O complemento nominal representa o
ou finalidades do emprego do objeto direto preposicionado: recebedor, o paciente, o alvo da declaração expressa por um
a clareza da frase; a harmonia da frase; a ênfase ou a força da nome: amor a Deus, a condenação da violência, o medo de
expressão. assaltos, a remessa de cartas, útil ao homem, compositor
de músicas, etc. É regido pelas mesmas preposições usadas
Objeto Direto Pleonástico: Quando queremos dar destaque no objeto indireto. Difere deste apenas porque, em vez de
ou ênfase à ideia contida no objeto direto, colocamo-lo no complementar verbos, complementa nomes (substantivos,
início da frase e depois o repetimos ou reforçamos por meio do adjetivos) e alguns advérbios em –mente. Os nomes que
pronome oblíquo. A esse objeto repetido sob forma pronominal requerem complemento nominal correspondem, geralmente, a
chama-se pleonástico, enfático ou redundante. Exemplos: verbos de mesmo radical: amor ao próximo, amar o próximo;
O dinheiro, Jaime o trazia escondido nas mangas da camisa. perdão das injúrias, perdoar as injúrias; obediente aos pais,
O bem, muitos o louvam, mas poucos o seguem. obedecer aos pais; regresso à pátria, regressar à pátria; etc.
“Seus cavalos, ela os montava em pelo.” (Jorge Amado)
Agente da Passiva: é o complemento de um verbo na voz
Objeto Indireto: É o complemento verbal regido de passiva. Representa o ser que pratica a ação expressa pelo verbo

Língua Portuguesa 39
Apostila Digital Licenciada para Alice Caroline Guarino dos Santos - alice.guarino@hotmail.com (Proibida a Revenda)
APOSTILAS OPÇÃO
passivo. Vem regido comumente pela preposição por, e menos pai.; Júlio reside em Niterói.; Errei por distração.; Escureceu
frequentemente pela preposição de: Alfredo é estimado pelos de repente.
colegas; A cidade estava cercada pelo exército romano; “Era
conhecida de todo mundo a fama de suas riquezas.” Observações: Pode ocorrer a elipse da preposição antes
de adjuntos adverbiais de tempo e modo: Aquela noite, não
O agente da passiva pode ser expresso pelos substantivos ou dormi. (=Naquela noite...); Domingo que vem não sairei. (=No
pelos pronomes: domingo...); Ouvidos atentos, aproximei-me da porta. (=De
As flores são umedecidas pelo orvalho. ouvidos atentos...); Os adjuntos adverbiais classificam-se de
A carta foi cuidadosamente corrigida por mim. acordo com as circunstâncias que exprimem: adjunto adverbial
de lugar, modo, tempo, intensidade, causa, companhia, meio,
O agente da passiva corresponde ao sujeito da oração na voz assunto, negação, etc. É importante saber distinguir adjunto
ativa: adverbial de adjunto adnominal, de objeto indireto e de
A rainha era chamada pela multidão. (voz passiva) complemento nominal: sair do mar (ad.adv.); água do mar (adj.
A multidão aclamava a rainha. (voz ativa) adn.); gosta do mar (obj.indir.); ter medo do mar (compl.nom.).
Ele será acompanhado por ti. (voz passiva)
Aposto: É uma palavra ou expressão que explica ou esclarece,
Observações: desenvolve ou resume outro termo da oração. Exemplos:
Frase de forma passiva analítica sem complemento agente D. Pedro II, imperador do Brasil, foi um monarca sábio.
expresso, ao passar para a ativa, terá sujeito indeterminado “Nicanor, ascensorista, expôs-me seu caso de consciência.”
e o verbo na 3ª pessoa do plural: Ele foi expulso da cidade. (Carlos Drummond de Andrade)
(Expulsaram-no da cidade.); As florestas são devastadas.
(Devastam as florestas.); Na passiva pronominal não se declara O núcleo do aposto é um substantivo ou um pronome
o agente: Nas ruas assobiavam-se as canções dele pelos substantivo:
pedestres. (errado); Nas ruas eram assobiadas as canções dele Foram os dois, ele e ela.
pelos pedestres. (certo); Assobiavam-se as canções dele nas Só não tenho um retrato: o de minha irmã.
ruas. (certo)
O aposto não pode ser formado por adjetivos. Nas frases
Termos Acessórios da Oração seguintes, por exemplo, não há aposto, mas predicativo do
sujeito:
Termos acessórios são os que desempenham na oração Audaciosos, os dois surfistas atiraram-se às ondas.
uma função secundária, qual seja a de caracterizar um ser, As borboletas, leves e graciosas, esvoaçavam num balé de
determinar os substantivos, exprimir alguma circunstância. São cores.
três os termos acessórios da oração: adjunto adnominal, adjunto
adverbial e aposto. Os apostos, em geral, destacam-se por pausas, indicadas, na
escrita, por vírgulas, dois pontos ou travessões. Não havendo
Adjunto adnominal: É o termo que caracteriza ou determina pausa, não haverá vírgula, como nestes exemplos:
os substantivos. Exemplo: Meu irmão veste roupas vistosas. Minha irmã Beatriz; o escritor João Ribeiro; o romance Tóia;
(Meu determina o substantivo irmão: é um adjunto adnominal o rio Amazonas; a Rua Osvaldo Cruz; o Colégio Tiradentes, etc.
– vistosas caracteriza o substantivo roupas: é também adjunto “Onde estariam os descendentes de Amaro vaqueiro?”
adnominal). (Graciliano Ramos)
O adjunto adnominal pode ser expresso: Pelos adjetivos:
água fresca, terras férteis, animal feroz; Pelos artigos: o O aposto pode preceder o termo a que se refere, o qual, às
mundo, as ruas, um rapaz; Pelos pronomes adjetivos: nosso tio, vezes, está elíptico. Exemplos:
este lugar, pouco sal, muitas rãs, país cuja história conheço, Rapaz impulsivo, Mário não se conteve.
que rua?; Pelos numerais: dois pés, quinto ano, capítulo sexto; Mensageira da ideia, a palavra é a mais bela expressão da
Pelas locuções ou expressões adjetivas que exprimem qualidade, alma humana.
posse, origem, fim ou outra especificação:
- presente de rei (=régio): qualidade O aposto, às vezes, refere-se a toda uma oração. Exemplos:
- livro do mestre, as mãos dele: posse, pertença Nuvens escuras borravam os espaços silenciosos, sinal de
- água da fonte, filho de fazendeiros: origem tempestade iminente.
- fio de aço, casa de madeira: matéria O espaço é incomensurável, fato que me deixa atônito.
- casa de ensino, aulas de inglês: fim, especialidade
Um aposto pode referir-se a outro aposto:
Observações: Não confundir o adjunto adnominal formado “Serafim Gonçalves casou-se com Lígia Tavares, filha do
por locução adjetiva com complemento nominal. Este representa velho coronel Tavares, senhor de engenho.” (Ledo Ivo)
o alvo da ação expressa por um nome transitivo: a eleição do
presidente, aviso de perigo, declaração de guerra, empréstimo O aposto pode vir precedido das expressões explicativas isto
de dinheiro, plantio de árvores, colheita de trigo, destruidor é, a saber, ou da preposição acidental como:
de matas, descoberta de petróleo, amor ao próximo, etc. O
adjunto adnominal formado por locução adjetiva representa Dois países sul-americanos, isto é, a Bolívia e o Paraguai,
o agente da ação, ou a origem, pertença, qualidade de alguém não são banhados pelo mar.
ou de alguma coisa: o discurso do presidente, aviso de amigo, Este escritor, como romancista, nunca foi superado.
declaração do ministro, empréstimo do banco, a casa do
fazendeiro, folhas de árvores, farinha de trigo, beleza das O aposto que se refere a objeto indireto, complemento
matas, cheiro de petróleo, amor de mãe. nominal ou adjunto adverbial vem precedido de preposição:

Adjunto adverbial: É o termo que exprime uma circunstância O rei perdoou aos dois: ao fidalgo e ao criado.
(de tempo, lugar, modo, etc.) ou, em outras palavras, que modifica “Acho que adoeci disso, de beleza, da intensidade das
o sentido de um verbo, adjetivo ou advérbio. Exemplo: “Meninas coisas.” (Raquel Jardim)
numa tarde brincavam de roda na praça”. O adjunto adverbial De cobras, morcegos, bichos, de tudo ela tinha medo.
é expresso: Pelos advérbios: Cheguei cedo.; Ande devagar.;
Maria é mais alta.; Não durma ao volante.; Moramos aqui.; Vocativo: (do latim vocare = chamar) é o termo (nome, título,
Ele fala bem, fala corretamente.; Volte bem depressa.; Talvez apelido) usado para chamar ou interpelar a pessoa, o animal ou
esteja enganado.; Pelas locuções ou expressões adverbiais: Às a coisa personificada a que nos dirigimos:
vezes viajava de trem.; Compreendo sem esforço.; Saí com meu

Língua Portuguesa 40
Apostila Digital Licenciada para Alice Caroline Guarino dos Santos - alice.guarino@hotmail.com (Proibida a Revenda)
APOSTILAS OPÇÃO
“Elesbão? Ó Elesbão! Venha ajudar-nos, por favor!” (Maria Deves estudar para poderes vencer na vida. (duas locuções
de Lourdes Teixeira) verbais, duas orações)
“A ordem, meus amigos, é a base do governo.” (Machado de
Assis) Há três tipos de período composto: por coordenação, por
“Correi, correi, ó lágrimas saudosas!” (Fagundes Varela) subordinação e por coordenação e subordinação ao mesmo
tempo (também chamada de misto).
Observação: Profere-se o vocativo com entoação exclamativa.
Na escrita é separado por vírgula(s). No exemplo inicial, os Período Composto por Coordenação – Orações
pontos interrogativo e exclamativo indicam um chamado alto e Coordenadas
prolongado. O vocativo se refere sempre à 2ª pessoa do discurso,
que pode ser uma pessoa, um animal, uma coisa real ou entidade Considere, por exemplo, este período composto:
abstrata personificada. Podemos antepor-lhe uma interjeição de Passeamos pela praia, / brincamos, / recordamos os tempos
apelo (ó, olá, eh!): de infância.
1ª oração: Passeamos pela praia
“Tem compaixão de nós , ó Cristo!” (Alexandre Herculano) 2ª oração: brincamos
“Ó Dr. Nogueira, mande-me cá o Padilha, amanhã!” 3ª oração: recordamos os tempos de infância
(Graciliano Ramos) As três orações que compõem esse período têm sentido
“Esconde-te, ó sol de maio, ó alegria do mundo!” (Camilo próprio e não mantêm entre si nenhuma dependência sintática:
Castelo Branco) elas são independentes. Há entre elas, é claro, uma relação de
O vocativo é um tempo à parte. Não pertence à estrutura da sentido, mas, como já dissemos, uma não depende da outra
oração, por isso não se anexa ao sujeito nem ao predicado. sintaticamente.
As orações independentes de um período são chamadas
Questões de orações coordenadas (OC), e o período formado só de
orações coordenadas é chamado de período composto por
01. O termo em destaque é adjunto adverbial de intensidade coordenação.
em: As orações coordenadas são classificadas em assindéticas e
(A) pode aprender e assimilar MUITA coisa sindéticas.
(B) enfrentamos MUITAS novidades
(C) precisa de um parceiro com MUITO caráter - As orações coordenadas são assindéticas (OCA) quando
(D) não gostam de mulheres MUITO inteligentes não vêm introduzidas por conjunção. Exemplo:
(E) assumimos MUITO conflito e confusão Os torcedores gritaram, / sofreram, / vibraram.
OCA OCA OCA
02. Assinale a alternativa correta: “para todos os males, há
dois remédios: o tempo e o silêncio”, os termos grifados são “Inclinei-me, apanhei o embrulho e segui.” (Machado de
respectivamente: Assis)
(A) sujeito – objeto direto; “A noite avança, há uma paz profunda na casa deserta.”
(B) sujeito – aposto; (Antônio Olavo Pereira)
(C) objeto direto – aposto; “O ferro mata apenas; o ouro infama, avilta, desonra.”
(D) objeto direto – objeto direto; (Coelho Neto)
(E) objeto direto – complemento nominal.
- As orações coordenadas são sindéticas (OCS) quando vêm
03. Assinale a alternativa em que o termo destacado é objeto introduzidas por conjunção coordenativa. Exemplo:
indireto. O homem saiu do carro / e entrou na casa.
(A) “Quem faz um poema abre uma janela.” (Mário Quintana) OCA OCS
(B) “Toda gente que eu conheço e que fala comigo / Nunca
teve um ato ridículo / Nunca sofreu enxovalho (...)” (Fernando As orações coordenadas sindéticas são classificadas de
Pessoa) acordo com o sentido expresso pelas conjunções coordenativas
(C) “Quando Ismália enlouqueceu / Pôs-se na torre a sonhar que as introduzem. Pode ser:
/ Viu uma lua no céu, / Viu uma lua no mar.” (Alphonsus de
Guimarães) - Orações coordenadas sindéticas aditivas: e, nem, não só...
(D) “Mas, quando responderam a Nhô Augusto: ‘– É a mas também, não só... mas ainda.
jagunçada de seu Joãozinho Bem-Bem, que está descendo para Saí da escola / e fui à lanchonete.
a Bahia.’ – ele, de alegre, não se pôde conter.” (Guimarães Rosa) OCA OCS Aditiva

Respostas Observe que a 2ª oração vem introduzida por uma conjunção


01. D\02. C\03. D que expressa idéia de acréscimo ou adição com referência à
oração anterior, ou seja, por uma conjunção coordenativa aditiva.
Período
A doença vem a cavalo e volta a pé.
Período: Toda frase com uma ou mais orações constitui um As pessoas não se mexiam nem falavam.
período, que se encerra com ponto de exclamação, ponto de “Não só findaram as queixas contra o alienista, mas até
interrogação ou com reticências. nenhum ressentimento ficou dos atos que ele praticara.”
O período é simples quando só traz uma oração, chamada (Machado de Assis)
absoluta; o período é composto quando traz mais de uma - Orações coordenadas sindéticas adversativas: mas,
oração. Exemplo: Pegou fogo no prédio. (Período simples, oração porém, todavia, contudo, entretanto, no entanto.
absoluta.); Quero que você aprenda. (Período composto.)
Estudei bastante / mas não passei no teste.
Existe uma maneira prática de saber quantas orações há OCA OCS Adversativa
num período: é contar os verbos ou locuções verbais. Num
período haverá tantas orações quantos forem os verbos ou as Observe que a 2ª oração vem introduzida por uma conjunção
locuções verbais nele existentes. Exemplos: que expressa idéia de oposição à oração anterior, ou seja, por
Pegou fogo no prédio. (um verbo, uma oração) uma conjunção coordenativa adversativa.
Quero que você aprenda. (dois verbos, duas orações)
Está pegando fogo no prédio. (uma locução verbal, uma A espada vence, mas não convence.
oração) “É dura a vida, mas aceitam-na.” (Cecília Meireles)

Língua Portuguesa 41
Apostila Digital Licenciada para Alice Caroline Guarino dos Santos - alice.guarino@hotmail.com (Proibida a Revenda)
APOSTILAS OPÇÃO
- Orações coordenadas sindéticas conclusivas: portanto, 02. E\03. C
por isso, pois, logo.
Período Composto por Subordinação
Ele me ajudou muito, / portanto merece minha gratidão.
OCA OCS Conclusiva Observe os termos destacados em cada uma destas orações:
Vi uma cena triste. (adjunto adnominal)
Observe que a 2ª oração vem introduzida por uma conjunção Todos querem sua participação. (objeto direto)
que expressa ideia de conclusão de um fato enunciado na oração Não pude sair por causa da chuva. (adjunto adverbial de
anterior, ou seja, por uma conjunção coordenativa conclusiva. causa)

Vives mentindo; logo, não mereces fé. Veja, agora, como podemos transformar esses termos em
Ele é teu pai: respeita-lhe, pois, a vontade. orações com a mesma função sintática:
Vi uma cena / que me entristeceu. (oração subordinada
- Orações coordenadas sindéticas alternativas: ou,ou... ou, com função de adjunto adnominal)
ora... ora, seja... seja, quer... quer. Todos querem / que você participe. (oração subordinada
com função de objeto direto)
Seja mais educado / ou retire-se da reunião! Não pude sair / porque estava chovendo. (oração
OCA OCS Alternativa subordinada com função de adjunto adverbial de causa)

Observe que a 2ª oração vem introduzida por uma Em todos esses períodos, a segunda oração exerce uma
conjunção que estabelece uma relação de alternância ou escolha certa função sintática em relação à primeira, sendo, portanto,
com referência à oração anterior, ou seja, por uma conjunção subordinada a ela. Quando um período é constituído de pelo
coordenativa alternativa. menos um conjunto de duas orações em que uma delas (a
subordinada) depende sintaticamente da outra (principal), ele
Venha agora ou perderá a vez. é classificado como período composto por subordinação. As
“Jacinta não vinha à sala, ou retirava-se logo.” (Machado de orações subordinadas são classificadas de acordo com a função
Assis) que exercem: adverbiais, substantivas e adjetivas.
“Em aviação, tudo precisa ser bem feito ou custará preço
muito caro.” (Renato Inácio da Silva) Orações Subordinadas Adverbiais
“A louca ora o acariciava, ora o rasgava freneticamente.”
(Luís Jardim) As orações subordinadas adverbiais (OSA) são aquelas
que exercem a função de adjunto adverbial da oração principal
- Orações coordenadas sindéticas explicativas: que, (OP). São classificadas de acordo com a conjunção subordinativa
porque, pois, porquanto. que as introduz:
Vamos andar depressa / que estamos atrasados.
OCA OCS Explicativa - Causais: Expressam a causa do fato enunciado na oração
Observe que a 2ª oração é introduzida por uma conjunção principal. Conjunções: porque, que, como (= porque), pois que,
que expressa ideia de explicação, de justificativa em relação visto que.
à oração anterior, ou seja, por uma conjunção coordenativa Não fui à escola / porque fiquei doente.
explicativa. OP OSA Causal

Leve-lhe uma lembrança, que ela aniversaria amanhã. O tambor soa porque é oco.
“A mim ninguém engana, que não nasci ontem.” (Érico Como não me atendessem, repreendi-os severamente.
Veríssimo) Como ele estava armado, ninguém ousou reagir.
“Faltou à reunião, visto que esteve doente.” (Arlindo de
Questões Sousa)

01. Relacione as orações coordenadas por meio de - Condicionais: Expressam hipóteses ou condição para a
conjunções: ocorrência do que foi enunciado na principal. Conjunções: se,
(A) Ouviu-se o som da bateria. Os primeiros foliões surgiram. contanto que, a menos que, a não ser que, desde que.
(B) Não durma sem cobertor. A noite está fria. Irei à sua casa / se não chover.
(C) Quero desculpar-me. Não consigo encontrá-los. OP OSA Condicional
  
02. Em: “... ouviam-se amplos bocejos, fortes como o marulhar Deus só nos perdoará se perdoarmos aos nossos
das ondas...” a partícula como expressa uma ideia de: ofensores.
(A) causa Se o conhecesses, não o condenarias.
(B) explicação “Que diria o pai se soubesse disso?” (Carlos Drummond de
(C) conclusão Andrade)
(D) proporção A cápsula do satélite será recuperada, caso a experiência
(E) comparação tenha êxito.
- Concessivas: Expressam ideia ou fato contrário ao da
03. “Entrando na faculdade, procurarei emprego”, oração oração principal, sem, no entanto, impedir sua realização.
sublinhada pode indicar uma ideia de: Conjunções: embora, ainda que, apesar de, se bem que, por mais
(A) concessão que, mesmo que.
(B) oposição Ela saiu à noite / embora estivesse doente.
(C) condição OP OSA Concessiva
(D) lugar Admirava-o muito, embora (ou conquanto ou posto que
(E) consequência ou se bem que) não o conhecesse pessoalmente.
Respostas Embora não possuísse informações seguras, ainda assim
arriscou uma opinião.
01. Cumpriremos nosso dever, ainda que (ou mesmo quando
Ouviu-se o som da bateria e os primeiros foliões surgiram. ou ainda quando ou mesmo que) todos nos critiquem.
Não durma sem cobertor, pois a noite está fria.
Quero desculpar-me, mas consigo encontrá-los. Por mais que gritasse, não me ouviram.

Língua Portuguesa 42
Apostila Digital Licenciada para Alice Caroline Guarino dos Santos - alice.guarino@hotmail.com (Proibida a Revenda)
APOSTILAS OPÇÃO
- Conformativas: Expressam a conformidade de um fato À proporção que avançávamos, as casas iam rareando.
com outro. Conjunções: conforme, como (=conforme), segundo. O valor do salário, ao passo que os preços sobem, vai
O trabalho foi feito / conforme havíamos planejado. diminuindo.
OP OSA Conformativa
Orações Subordinadas Substantivas
O homem age conforme pensa.
Relatei os fatos como (ou conforme) os ouvi. As orações subordinadas substantivas (OSS) são aquelas
Como diz o povo, tristezas não pagam dívidas. que, num período, exercem funções sintáticas próprias de
O jornal, como sabemos, é um grande veículo de informação. substantivos, geralmente são introduzidas pelas conjunções
integrantes que e se. Elas podem ser:
- Temporais: Acrescentam uma circunstância de tempo ao
que foi expresso na oração principal. Conjunções: quando, assim - Oração Subordinada Substantiva Objetiva Direta: É
que, logo que, enquanto, sempre que, depois que, mal (=assim que). aquela que exerce a função de objeto direto do verbo da oração
Ele saiu da sala / assim que eu cheguei. principal. Observe: O grupo quer a sua ajuda. (objeto direto)
OP OSA Temporal O grupo quer / que você ajude.
OP OSS Objetiva Direta
Formiga, quando quer se perder, cria asas.
“Lá pelas sete da noite, quando escurecia, as casas se O mestre exigia que todos estivessem presentes. (= O
esvaziam.” (Carlos Povina Cavalcânti) mestre exigia a presença de todos.)
“Quando os tiranos caem, os povos se levantam.” (Marquês Mariana esperou que o marido voltasse.
de Maricá) Ninguém pode dizer: Desta água não beberei.
Enquanto foi rico, todos o procuravam. O fiscal verificou se tudo estava em ordem.
- Finais: Expressam a finalidade ou o objetivo do que foi
enunciado na oração principal. Conjunções: para que, a fim de - Oração Subordinada Substantiva Objetiva Indireta: É
que, porque (=para que), que. aquela que exerce a função de objeto indireto do verbo da oração
Abri a porta do salão / para que todos pudessem entrar. principal. Observe: Necessito de sua ajuda. (objeto indireto)
OP OSA Final Necessito / de que você me ajude.
OP OSS Objetiva Indireta
“O futuro se nos oculta para que nós o imaginemos.”
(Marquês de Maricá) Não me oponho a que você viaje. (= Não me oponho à sua
Aproximei-me dele a fim de que me ouvisse melhor. viagem.)
“Fiz-lhe sinal que se calasse.” (Machado de Assis) (que = Aconselha-o a que trabalhe mais.
para que) Daremos o prêmio a quem o merecer.
“Instara muito comigo não deixasse de frequentar as Lembre-se de que a vida é breve.
recepções da mulher.” (Machado de Assis) (não deixasse =
para que não deixasse) - Oração Subordinada Substantiva Subjetiva: É aquela
que exerce a função de sujeito do verbo da oração principal.
- Consecutivas: Expressam a consequência do que foi Observe: É importante sua colaboração. (sujeito)
enunciado na oração principal. Conjunções: porque, que, como (= É importante / que você colabore.
porque), pois que, visto que. OP OSS Subjetiva
A chuva foi tão forte / que inundou a cidade.
OP OSA Consecutiva A oração subjetiva geralmente vem:
- depois de um verbo de ligação + predicativo, em construções
Fazia tanto frio que meus dedos estavam endurecidos. do tipo é bom, é útil, é certo, é conveniente, etc. Ex.: É certo que
“A fumaça era tanta que eu mal podia abrir os olhos.” (José ele voltará amanhã.
J. Veiga) - depois de expressões na voz passiva, como sabe-se, conta-
De tal sorte a cidade crescera que não a reconhecia mais. se, diz-se, etc. Ex.: Sabe-se que ele saiu da cidade.
As notícias de casa eram boas, de maneira que pude - depois de verbos como convir, cumprir, constar, urgir,
prolongar minha viagem. ocorrer, quando empregados na 3ª pessoa do singular e seguidos
das conjunções que ou se. Ex.: Convém que todos participem
- Comparativas: Expressam ideia de comparação com da reunião.
referência à oração principal. Conjunções: como, assim como,
tal como, (tão)... como, tanto como, tal qual, que (combinado com É necessário que você colabore. (= Sua colaboração é
menos ou mais). necessária.)
Ela é bonita / como a mãe. Parece que a situação melhorou.
OP OSA Comparativa Aconteceu que não o encontrei em casa.
Importa que saibas isso bem.
A preguiça gasta a vida como a ferrugem consome o ferro.”
(Marquês de Maricá) - Oração Subordinada Substantiva Completiva Nominal:
Ela o atraía irresistivelmente, como o imã atrai o ferro. É aquela que exerce a função de complemento nominal de um
Os retirantes deixaram a cidade tão pobres como vieram. termo da oração principal. Observe: Estou convencido de sua
Como a flor se abre ao Sol, assim minha alma se abriu à luz inocência. (complemento nominal)
daquele olhar. Estou convencido / de que ele é inocente.
OP OSS Completiva Nominal
Obs.: As orações comparativas nem sempre apresentam
claramente o verbo, como no exemplo acima, em que está Sou favorável a que o prendam. (= Sou favorável à prisão
subentendido o verbo ser (como a mãe é). dele.)
- Proporcionais: Expressam uma ideia que se relaciona Estava ansioso por que voltasses.
proporcionalmente ao que foi enunciado na principal. Sê grato a quem te ensina.
Conjunções: à medida que, à proporção que, ao passo que, quanto “Fabiano tinha a certeza de que não se acabaria tão cedo.”
mais, quanto menos. (Graciliano Ramos)
Quanto mais reclamava / menos atenção recebia.
OSA Proporcional OP - Oração Subordinada Substantiva Predicativa: É aquela
que exerce a função de predicativo do sujeito da oração principal,
À medida que se vive, mais se aprende. vindo sempre depois do verbo ser. Observe: O importante é sua

Língua Portuguesa 43
Apostila Digital Licenciada para Alice Caroline Guarino dos Santos - alice.guarino@hotmail.com (Proibida a Revenda)
APOSTILAS OPÇÃO
felicidade. (predicativo) Valério, que nasceu rico, acabou na miséria.
O importante é / que você seja feliz. Ele tem amor às plantas, que cultiva com carinho.
OP OSS Predicativa Alguém, que passe por ali à noite, poderá ser assaltado.

Seu receio era que chovesse. (Seu receio era a chuva.) Orações Reduzidas
Minha esperança era que ele desistisse. Observe que as orações subordinadas eram sempre
Meu maior desejo agora é que me deixem em paz. introduzidas por uma conjunção ou pronome relativo e
Não sou quem você pensa. apresentavam o verbo numa forma do indicativo ou do
subjuntivo. Além desse tipo de orações subordinadas há outras
- Oração Subordinada Substantiva Apositiva: É aquela que se apresentam com o verbo numa das formas nominais
que exerce a função de aposto de um termo da oração principal. (infinitivo, gerúndio e particípio). Exemplos:
Observe: Ele tinha um sonho: a união de todos em benefício
do país. (aposto) - Ao entrar nas escola, encontrei o professor de inglês.
Ele tinha um sonho / que todos se unissem em benefício do (infinitivo)
país. - Precisando de ajuda, telefone-me. (gerúndio)
OP OSS Apositiva - Acabado o treino, os jogadores foram para o vestiário.
(particípio)
Só desejo uma coisa: que vivam felizes. (Só desejo uma
coisa: a sua felicidade) As orações subordinadas que apresentam o verbo numa das
Só lhe peço isto: honre o nosso nome. formas nominais são chamadas de reduzidas.
“Talvez o que eu houvesse sentido fosse o presságio disto: de Para classificar a oração que está sob a forma reduzida,
que virias a morrer...” (Osmã Lins) devemos procurar desenvolvê-la do seguinte modo: colocamos
“Mas diga-me uma cousa, essa proposta traz algum motivo a conjunção ou o pronome relativo adequado ao sentido e
oculto?” (Machado de Assis) passamos o verbo para uma forma do indicativo ou subjuntivo,
As orações apositivas vêm geralmente antecedidas de dois- conforme o caso. A oração reduzida terá a mesma classificação
pontos. Podem vir, também, entre vírgulas, intercaladas à oração da oração desenvolvida.
principal. Exemplo: Seu desejo, que o filho recuperasse a
saúde, tornou-se realidade. Ao entrar na escola, encontrei o professor de inglês.
Quando entrei na escola, / encontrei o professor de inglês.
Observação: Além das conjunções integrantes que e se, OSA Temporal
as orações substantivas podem ser introduzidas por outros Ao entrar na escola: oração subordinada adverbial temporal,
conectivos, tais como quando, como, quanto, etc. Exemplos: reduzida de infinitivo.
Não sei quando ele chegou.
Diga-me como resolver esse problema. Precisando de ajuda, telefone-me.
Se precisar de ajuda, / telefone-me.
Orações Subordinadas Adjetivas OSA Condicional
Precisando de ajuda: oração subordinada adverbial
As orações subordinadas Adjetivas (OSA) exercem condicional, reduzida de gerúndio.
a função de adjunto adnominal de algum termo da oração
principal. Observe como podemos transformar um adjunto Acabado o treino, os jogadores foram para o vestiário.
adnominal em oração subordinada adjetiva: Assim que acabou o treino, / os jogadores foram para o
Desejamos uma paz duradoura. (adjunto adnominal) vestiário.
Desejamos uma paz / que dure. (oração subordinada OSA Temporal
adjetiva) Acabado o treino: oração subordinada adverbial temporal,
reduzida de particípio.
As orações subordinadas adjetivas são sempre introduzidas
por um pronome relativo (que , qual, cujo, quem, etc.) e podem Observações:
ser classificadas em:
- Há orações reduzidas que permitem mais de um tipo de
- Subordinadas Adjetivas Restritivas: São restritivas desenvolvimento. Há casos também de orações reduzidas
quando restringem ou especificam o sentido da palavra a que se fixas, isto é, orações reduzidas que não são passíveis de
referem. Exemplo: desenvolvimento. Exemplo: Tenho vontade de visitar essa
O público aplaudiu o cantor / que ganhou o 1º lugar. cidade.
OP OSA Restritiva - O infinitivo, o gerúndio e o particípio não constituem
orações reduzidas quando fazem parte de uma locução verbal.
Nesse exemplo, a oração que ganhou o 1º lugar especifica Exemplos:
o sentido do substantivo cantor, indicando que o público não Preciso terminar este exercício.
aplaudiu qualquer cantor mas sim aquele que ganhou o 1º lugar. Ele está jantando na sala.
Essa casa foi construída por meu pai.
Pedra que rola não cria limo. - Uma oração coordenada também pode vir sob a forma
Os animais que se alimentam de carne chamam-se reduzida. Exemplo:
carnívoros. O homem fechou a porta, saindo depressa de casa.
Rubem Braga é um dos cronistas que mais belas páginas O homem fechou a porta e saiu depressa de casa. (oração
escreveram. coordenada sindética aditiva)
“Há saudades que a gente nunca esquece.” (Olegário Saindo depressa de casa: oração coordenada reduzida de
Mariano) gerúndio.
- Subordinadas Adjetivas Explicativas: São explicativas Qual é a diferença entre as orações coordenadas explicativas
quando apenas acrescentam uma qualidade à palavra a que se e as orações subordinadas causais, já que ambas podem ser
referem, esclarecendo um pouco mais seu sentido, mas sem iniciadas por que e porque? Às vezes não é fácil estabelecer a
restringi-lo ou especificá-lo. Exemplo: diferença entre explicativas e causais, mas como o próprio nome
O escritor Jorge Amado, / que mora na Bahia, / lançou um indica, as causais sempre trazem a causa de algo que se revela na
novo livro. oração principal, que traz o efeito.
OP OSA Explicativa OP Note-se também que há pausa (vírgula, na escrita) entre
a oração explicativa e a precedente e que esta é, muitas vezes,
Deus, que é nosso pai, nos salvará. imperativa, o que não acontece com a oração adverbial causal.

Língua Portuguesa 44
Apostila Digital Licenciada para Alice Caroline Guarino dos Santos - alice.guarino@hotmail.com (Proibida a Revenda)
APOSTILAS OPÇÃO
Essa noção de causa e efeito não existe no período composto por Paroxítonas – São aquelas em que a sílaba tônica se
coordenação. Exemplo: Rosa chorou porque levou uma surra. evidencia na penúltima sílaba.
Está claro que a oração iniciada pela conjunção é causal, visto Ex.: útil – tórax – táxi – leque – retrato – passível
que a surra foi sem dúvida a causa do choro, que é efeito.
Rosa chorou, porque seus olhos estão vermelhos. O Proparoxítonas - São aquelas em que a sílaba tônica se
período agora é composto por coordenação, pois a oração evidencia na antepenúltima sílaba.
iniciada pela conjunção traz a explicação daquilo que se revelou Ex.: lâmpada – câmara – tímpano – médico – ônibus
na coordena anterior. Não existe aí relação de causa e efeito: o
fato de os olhos de Elisa estarem vermelhos não é causa de ela Como podemos observar, mediante todos os exemplos
ter chorado. mencionados, os vocábulos possuem mais de uma sílaba, mas
em nossa língua existem aqueles com uma sílaba somente:
Ela fala / como falaria / se entendesse do assunto. são os chamados monossílabos, que, quando pronunciados,
OP OSA Comparativa OSA Condicional apresentam certa diferenciação quanto à intensidade.

Questões Tal diferenciação só é percebida quando os pronunciamos


em uma dada sequência de palavras. Assim como podemos
01. Na frase: “Maria do Carmo tinha a certeza de que estava observar no exemplo a seguir:
para ser mãe”, a oração destacada é:
(A) subordinada substantiva objetiva indireta “Sei que não vai dar em nada, seus segredos sei de cor”.
(B) subordinada substantiva completiva nominal
(C) subordinada substantiva predicativa Os monossílabos em destaque classificam-se como tônicos;
(D) coordenada sindética conclusiva os demais, como átonos (que, em, de).
(E) coordenada sindética explicativa
Os Acentos Gráficos
02. “Na ‘Partida Monção’, não há uma atitude inventada.
Há reconstituição de uma cena como ela devia ter sido na acento agudo (´) – Colocado sobre as letras “a”, “i”, “u” e
realidade.” A oração sublinhada é: sobre o “e” do grupo “em” - indica que estas letras representam
(A) adverbial conformativa as vogais tônicas de palavras como Amapá, caí, público, parabéns.
(B) adjetiva Sobre as letras “e” e “o” indica, além da tonicidade, timbre aberto.
(C) adverbial consecutiva Ex.: herói – médico – céu(ditongos abertos)
(D) adverbial proporcional
(E) adverbial causal acento circunflexo (^) – colocado sobre as letras “a”, “e” e
“o” indica, além da tonicidade, timbre fechado:
03.“Esses produtos podem ser encontrados nos Ex.: tâmara – Atlântico – pêssego – supôs
supermercados com rótulos como ‘sênior’ e com características
adaptadas às dificuldades para mastigar e para engolir dos acento grave (`) – indica a fusão da preposição “a” com
mais velhos, e preparados para se encaixar em seus hábitos de artigos e pronomes.
consumo”. O segmento “para se encaixar” pode ter sua forma Ex.: à – às – àquelas – àqueles
verbal reduzida adequadamente desenvolvida em
(A) para se encaixarem. trema (¨) – De acordo com a nova regra, foi totalmente
(B) para seu encaixotamento. abolido das palavras. Há uma exceção: é utilizado em palavras
(C) para que se encaixassem. derivadas de nomes próprios estrangeiros.
(D) para que se encaixem. Ex.: mülleriano (de Müller)
(E) para que se encaixariam.
til (~) – indica que as letras “a” e “o” representam vogais
Respostas nasais.
01. B\02. A\03. D Ex.: coração – melão – órgão – ímã
Regras fundamentais:

12. acentuação; Palavras oxítonas:


Acentuam-se todas as oxítonas terminadas em: “a”, “e”, “o”,
“em”, seguidas ou não do plural(s):
Acentuação Pará – café(s) – cipó(s) – armazém(s)

A acentuação é um dos requisitos que perfazem as regras Essa regra também é aplicada aos seguintes casos:
estabelecidas pela Gramática Normativa. Esta se compõe de
algumas particularidades, às quais devemos estar atentos, Monossílabos tônicos terminados em “a”, “e”, “o”, seguidos
procurando estabelecer uma relação de familiaridade e, ou não de “s”.
consequentemente, colocando-as em prática na linguagem Ex.: pá – pé – dó – há
escrita.
Formas verbais terminadas em “a”, “e”, “o” tônicos, seguidas
Regras básicas – Acentuação tônica de lo, la, los, las.
respeitá-lo – percebê-lo – compô-lo
A acentuação tônica implica na intensidade com que são
pronunciadas as sílabas das palavras. Aquela que se dá de Paroxítonas:
forma mais acentuada, conceitua-se como sílaba tônica. As Acentuam-se as palavras paroxítonas terminadas em:
demais, como são pronunciadas com menos intensidade, são - i, is
denominadas de átonas. táxi – lápis – júri
- us, um, uns
De acordo com a tonicidade, as palavras são classificadas vírus – álbuns – fórum
como: - l, n, r, x, ps
automóvel – elétron - cadáver – tórax – fórceps
Oxítonas – São aquelas cuja sílaba tônica recai sobre a - ã, ãs, ão, ãos
última sílaba. ímã – ímãs – órfão – órgãos
Ex.: café – coração – cajá – atum – caju – papel

Língua Portuguesa 45
Apostila Digital Licenciada para Alice Caroline Guarino dos Santos - alice.guarino@hotmail.com (Proibida a Revenda)
APOSTILAS OPÇÃO
- Dica: Memorize a palavra LINURXÃO. Para quê? Repare que As formas verbais que possuíam o acento tônico na raiz, com
essa palavra apresenta as terminações das paroxítonas que são “u” tônico precedido de “g” ou “q” e seguido de “e” ou “i” não
acentuadas: L, I N, U (aqui inclua UM =fórum), R, X, Ã, ÃO. Assim serão mais acentuadas. Ex.:
ficará mais fácil a memorização!
Antes Depois
- ditongo oral, crescente ou decrescente, seguido ou não de “s”. apazigúe (apaziguar) apazigue
argúi (arguir) argui
água – pônei – mágoa – jóquei
Acentuam-se os verbos pertencentes à terceira pessoa do
Regras especiais: plural de:

Os ditongos de pronúncia aberta “ei”, “oi” ( ditongos abertos), ele tem – eles têm
que antes eram acentuados, perderam o acento de acordo com ele vem – eles vêm (verbo vir)
a nova regra, mas desde que estejam em palavras paroxítonas.
A regra prevalece também para os verbos conter, obter, reter,
Cuidado: Se os ditongos abertos estiverem em uma deter, abster.
palavra oxítona (herói) ou monossílaba (céu) ainda são ele contém – eles contêm
acentuados. Mas caso não forem ditongos perdem o acento. ele obtém – eles obtêm
Ex.: ele retém – eles retêm
ele convém – eles convêm
Antes Agora
assembléia assembleia
Não se acentuam mais as palavras homógrafas que antes
idéia ideia
eram acentuadas para diferenciá-las de outras semelhantes
jibóia jiboia
(regra do acento diferencial). Apenas em algumas exceções,
apóia (verbo apoiar) apoia
como:
Quando a vogal do hiato for “i” ou “u” tônicos, acompanhados
A forma verbal pôde (terceira pessoa do singular do
ou não de “s”, haverá acento:
pretérito perfeito do modo indicativo) ainda continua
Ex.: saída – faísca – baú – país – Luís
sendo acentuada para diferenciar-se de pode (terceira
pessoa do singular do presente do indicativo). Ex:
Observação importante:
Não serão mais acentuados “i” e “u” tônicos, formando hiato
Ela pode fazer isso agora.
quando vierem depois de ditongo: Ex.:
Elvis não pôde participar porque sua mão não deixou...
Antes Agora
O mesmo ocorreu com o verbo pôr para diferenciar da
bocaiúva bocaiuva
preposição por.
feiúra feiura
- Quando, na frase, der para substituir o “por” por “colocar”,
O acento pertencente aos encontros “oo” e “ee” foi abolido.
então estaremos trabalhando com um verbo, portanto: “pôr”;
Ex.:
nos outros casos, “por” preposição. Ex:
Antes Agora
Faço isso por você.
crêem creem
Posso pôr (colocar) meus livros aqui?
vôo voo
Questões
- Agora memorize a palavra CREDELEVÊ. São os verbos que,
no plural, dobram o “e”, mas que não recebem mais acento
01. “Cadáver” é paroxítona, pois:
como antes: CRER, DAR, LER e VER.
A) Tem a última sílaba como tônica.
B) Tem a penúltima sílaba como tônica.
Repare:
C) Tem a antepenúltima sílaba como tônica.
1-) O menino crê em você
D) Não tem sílaba tônica.
Os meninos creem em você.
2-) Elza lê bem!
02. Assinale a alternativa correta.
Todas leem bem!
A palavra faliu contém um:
3-) Espero que ele dê o recado à sala.
A) hiato
Esperamos que os dados deem efeito!
B) dígrafo
4-) Rubens vê tudo!
C) ditongo decrescente
Eles veem tudo!
D) ditongo crescente
Respostas
- Cuidado! Há o verbo vir:
1-B / 2-C
Ele vem à tarde!
Eles vêm à tarde!
Não se acentuam o “i” e o “u” que formam hiato quando 13. colocação pronominal e
seguidos, na mesma sílaba, de l, m, n, r ou z: verbal;

Ra-ul, ru-im, con-tri-bu-in-te, sa-ir, ju-iz


Colocação dos Pronomes Oblíquos
Não se acentuam as letras “i” e “u” dos hiatos se estiverem
Átonos
seguidas do dígrafo nh:
ra-i-nha, ven-to-i-nha.
De acordo com as autoras Rose Jordão e Clenir Bellezi, a
colocação pronominal é a posição que os pronomes pessoais
Não se acentuam as letras “i” e “u” dos hiatos se vierem
oblíquos átonos ocupam na frase em relação ao verbo a que se
precedidas de vogal idêntica:
referem.
xi-i-ta, pa-ra-cu-u-ba
São pronomes oblíquos átonos: me, te, se, o, os, a, as, lhe,
lhes, nos e vos.

Língua Portuguesa 46
Apostila Digital Licenciada para Alice Caroline Guarino dos Santos - alice.guarino@hotmail.com (Proibida a Revenda)
APOSTILAS OPÇÃO
O pronome oblíquo átono pode assumir três posições na Questões
oração em relação ao verbo:
1. próclise: pronome antes do verbo 01. Considerada a norma culta escrita, há correta substituição
2. ênclise: pronome depois do verbo de estrutura nominal por pronome em:
3. mesóclise: pronome no meio do verbo (A) Agradeço antecipadamente sua Resposta // Agradeço-
lhes antecipadamente.
Próclise (B) do verbo fabricar se extraiu o substantivo fábrica. // do
verbo fabricar se extraiu-lhe.
A próclise é aplicada antes do verbo quando temos: (C) não faltam lexicógrafos // não faltam-os.
- Palavras com sentido negativo: (D) Gostaria de conhecer suas considerações // Gostaria de
Nada me faz querer sair dessa cama. conhecê-las.
Não se trata de nenhuma novidade. (E) incluindo a palavra ‘aguardo’ // incluindo ela.

- Advérbios: 02. Caso fosse necessário substituir o termo destacado em


Nesta casa se fala alemão. “Basta apresentar um documento” por um pronome, de acordo
Naquele dia me falaram que a professora não veio. com a norma-padrão, a nova redação deveria ser
(A) Basta apresenta-lo.
- Pronomes relativos: (B) Basta apresentar-lhe.
A aluna que me mostrou a tarefa não veio hoje. (C) Basta apresenta-lhe.
Não vou deixar de estudar os conteúdos que me falaram. (D) Basta apresentá-la.
(E) Basta apresentá-lo.
- Pronomes indefinidos:
Quem me disse isso? 03. Em qual período, o pronome átono que substitui o
Todos se comoveram durante o discurso de despedida. sintagma em destaque tem sua colocação de acordo com a
norma-padrão?
- Pronomes demonstrativos: (A) O porteiro não conhecia o portador do embrulho –
Isso me deixa muito feliz! conhecia-o
Aquilo me incentivou a mudar de atitude! (B) Meu pai tinha encontrado um marinheiro na praça Mauá
– tinha encontrado-o.
- Preposição seguida de gerúndio: (C) As pessoas relatarão as suas histórias para o registro no
Em se tratando de qualidade, o Brasil Escola é o site mais Museu – relatá-las-ão.
indicado à pesquisa escolar. (D) Quem explicou às crianças as histórias de seus
antepassados? – explicou-lhes.
- Conjunção subordinativa: (E) Vinham perguntando às pessoas se aceitavam a ideia de
Vamos estabelecer critérios, conforme lhe avisaram. um museu virtual – Lhes vinham perguntando.

Ênclise Respostas
01. D/02. E/03. C
A ênclise é empregada depois do verbo. A norma culta não
aceita orações iniciadas com pronomes oblíquos átonos. A
ênclise vai acontecer quando: 14. emprego de preposição;
- O verbo estiver no imperativo afirmativo:
Amem-se uns aos outros.
Sigam-me e não terão derrotas. Preposição

- O verbo iniciar a oração: Preposição é uma palavra invariável que serve para ligar
Diga-lhe que está tudo bem. termos ou orações. Quando esta ligação acontece, normalmente
Chamaram-me para ser sócio. há uma subordinação do segundo termo em relação ao
primeiro. As preposições são muito importantes na estrutura
- O verbo estiver no infinitivo impessoal regido da preposição da língua, pois estabelecem a coesão textual e possuem valores
“a”: semânticos indispensáveis para a compreensão do texto.
Naquele instante os dois passaram a odiar-se.
Passaram a cumprimentar-se mutuamente. Tipos de Preposição

- O verbo estiver no gerúndio: 1. Preposições essenciais: palavras que atuam exclusivamente


Não quis saber o que aconteceu, fazendo-se de como preposições.
despreocupada. A, ante, perante, após, até, com, contra, de, desde, em, entre,
Despediu-se, beijando-me a face. para, por, sem, sob, sobre, trás, atrás de, dentro de, para com.

- Houver vírgula ou pausa antes do verbo: 2. Preposições acidentais: palavras de outras classes
Se passar no vestibular em outra cidade, mudo-me no gramaticais que podem atuar como preposições.
mesmo instante. Como, durante, exceto, fora, mediante, salvo, segundo, senão,
Se não tiver outro jeito, alisto-me nas forças armadas. visto.
Mesóclise
3. Locuções prepositivas: duas ou mais palavras valendo
A mesóclise acontece quando o verbo está flexionado no como uma preposição, sendo que a última palavra é uma delas.
futuro do presente ou no futuro do pretérito: Abaixo de, acerca de, acima de, ao lado de, a respeito de, de
A prova realizar-se-á neste domingo pela manhã. (= ela se acordo com, em cima de, embaixo de, em frente a, ao redor de,
realizará) graças a, junto a, com, perto de, por causa de, por cima de, por
Far-lhe-ei uma proposta irrecusável. (= eu farei uma trás de.
proposta a você)
Fontes: A preposição, como já foi dito, é invariável. No entanto pode
http://www.soportugues.com.br/secoes/morf/morf42.php unir-se a outras palavras e assim estabelecer concordância em
http://www.brasilescola.com/gramatica/colocacao-pronominal. gênero ou em número. Ex: por + o = pelo por + a = pela
htm

Língua Portuguesa 47
Apostila Digital Licenciada para Alice Caroline Guarino dos Santos - alice.guarino@hotmail.com (Proibida a Revenda)
APOSTILAS OPÇÃO
Vale ressaltar que essa concordância não é característica da - Se for pronome pessoal oblíquo estará ocupando o lugar e/
preposição, mas das palavras às quais ela se une. ou a função de um substantivo.
Temos Maria como parte da família. / A temos como parte
Esse processo de junção de uma preposição com outra da família
palavra pode se dar a partir de dois processos: Creio que conhecemos nossa mãe melhor que ninguém. /
Creio que a conhecemos melhor que ninguém.
1. Combinação: A preposição não sofre alteração.
preposição a + artigos definidos o, os 2. Algumas relações semânticas estabelecidas por meio das
a + o = ao preposições:
preposição a + advérbio onde Destino = Irei para casa.
a + onde = aonde Modo = Chegou em casa aos gritos.
Lugar = Vou ficar em casa;
2. Contração: Quando a preposição sofre alteração. Assunto = Escrevi um artigo sobre adolescência.
Tempo = A prova vai começar em dois minutos.
Preposição + Artigos Causa = Ela faleceu de derrame cerebral.
De + o(s) = do(s) Fim ou finalidade = Vou ao médico para começar o
De + a(s) = da(s) tratamento.
De + um = dum Instrumento = Escreveu a lápis.
De + uns = duns Posse = Não posso doar as roupas da mamãe.
De + uma = duma Autoria = Esse livro de Machado de Assis é muito bom.
De + umas = dumas Companhia = Estarei com ele amanhã.
Em + o(s) = no(s) Matéria = Farei um cartão de papel reciclado.
Em + a(s) = na(s) Meio = Nós vamos fazer um passeio de barco.
Em + um = num Origem = Nós somos do Nordeste, e você?
Em + uma = numa Conteúdo = Quebrei dois frascos de perfume.
Em + uns = nuns Oposição = Esse movimento é contra o que eu penso.
Em + umas = numas Preço = Essa roupa sai por R$ 50 à vista.
A + à(s) = à(s)
Por + o = pelo(s) Questões
Por + a = pela(s)
01. Leia o texto a seguir.
Preposição + Pronomes
De + ele(s) = dele(s) “Xadrez que liberta”: estratégia, concentração e reeducação
De + ela(s) = dela(s)
De + este(s) = deste(s) João Carlos de Souza Luiz cumpre pena há três anos e dois
De + esta(s) = desta(s) meses por assalto. Fransley Lapavani Silva está há sete anos
De + esse(s) = desse(s) preso por homicídio. Os dois têm 30 anos. Além dos muros,
De + essa(s) = dessa(s) grades, cadeados e detectores de metal, eles têm outros pontos
De + aquele(s) = daquele(s) em comum: tabuleiros e peças de xadrez.
De + aquela(s) = daquela(s) O jogo, que eles aprenderam na cadeia, além de uma válvula
De + isto = disto de escape para as horas de tédio, tornou-se uma metáfora para o
De + isso = disso que pretendem fazer quando estiverem em liberdade.
De + aquilo = daquilo “Quando você vai jogar uma partida de xadrez, tem que pensar
De + aqui = daqui duas, três vezes antes. Se você movimenta uma peça errada,
De + aí = daí pode perder uma peça de muito valor ou tomar um xeque-mate,
De + ali = dali instantaneamente. Se eu for para a rua e movimentar a peça
De + outro = doutro(s) errada, eu posso perder uma peça muito importante na minha
De + outra = doutra(s) vida, como eu perdi três anos na cadeia. Mas, na rua, o problema
Em + este(s) = neste(s) maior é tomar o xeque-mate”, afirma João Carlos.
Em + esta(s) = nesta(s) O xadrez faz parte da rotina de cerca de dois mil internos
Em + esse(s) = nesse(s) em 22 unidades prisionais do Espírito Santo. É o projeto “Xadrez
Em + aquele(s) = naquele(s) que liberta”. Duas vezes por semana, os presos podem praticar
Em + aquela(s) = naquela(s) a atividade sob a orientação de servidores da Secretaria de
Em + isto = nisto Estado da Justiça (Sejus). Na próxima sexta-feira, será realizado
Em + isso = nisso o primeiro torneio fora dos presídios desde que o projeto foi
Em + aquilo = naquilo implantado. Vinte e oito internos de 14 unidades participam da
A + aquele(s) = àquele(s) disputa, inclusive João Carlos e Fransley, que diz que a vitória
A + aquela(s) = àquela(s) não é o mais importante.
A + aquilo = àquilo “Só de chegar até aqui já estou muito feliz, porque eu não
esperava. A vitória não é tudo. Eu espero alcançar outras coisas
Dicas sobre preposição devido ao xadrez, como ser olhado com outros olhos, como
1. O “a” pode funcionar como preposição, pronome pessoal estou sendo olhado de forma diferente aqui no presídio devido
oblíquo e artigo. Como distingui-los? ao bom comportamento”.
Segundo a coordenadora do projeto, Francyany Cândido
- Caso o “a” seja um artigo, virá precedendo a um substantivo. Venturin, o “Xadrez que liberta” tem provocado boas mudanças
Ele servirá para determiná-lo como um substantivo singular no comportamento dos presos. “Tem surtido um efeito positivo
e feminino. por eles se tornarem uma referência positiva dentro da unidade,
A dona da casa não quis nos atender. já que cumprem melhor as regras, respeitam o próximo e
Como posso fazer a Joana concordar comigo? pensam melhor nas suas ações, refletem antes de tomar uma
atitude”.
- Quando é preposição, além de ser invariável, liga dois Embora a Sejus não monitore os egressos que ganham a
termos e estabelece relação de subordinação entre eles. liberdade, para saber se mantêm o hábito do xadrez, João Carlos
Cheguei a sua casa ontem pela manhã. já faz planos. “Eu incentivo não só os colegas, mas também
Não queria, mas vou ter que ir à outra cidade para procurar minha família. Sou casado e tenho três filhos. Já passei para a
um tratamento adequado. minha família: xadrez, quando eu sair para a rua, todo mundo

Língua Portuguesa 48
Apostila Digital Licenciada para Alice Caroline Guarino dos Santos - alice.guarino@hotmail.com (Proibida a Revenda)
APOSTILAS OPÇÃO
vai ter que aprender porque vai rolar até o torneio familiar”. 1ª oração: A menina segurou a boneca 2ª oração: e mostrou
“Medidas de promoção de educação e que possibilitem que o 3ª oração: quando viu as amiguinhas.
egresso saia melhor do que entrou são muito importantes. Nós A segunda oração liga-se à primeira por meio do “e”, e a
não temos pena de morte ou prisão perpétua no Brasil. O preso terceira oração liga-se à segunda por meio do “quando”. As
tem data para entrar e data para sair, então ele tem que sair palavras “e” e “quando” ligam, portanto, orações.
sem retornar para o crime”, analisa o presidente do Conselho
Estadual de Direitos Humanos, Bruno Alves de Souza Toledo. Observe: Gosto de natação e de futebol.
(Disponível em: www.inapbrasil.com.br/en/noticias/xadrez-que- Nessa frase as expressões de natação, de futebol são partes
liberta-estrategia-concentracao-e-reeducacao/6/noticias. Adaptado) ou termos de uma mesma oração. Logo, a palavra “e” está
ligando termos de uma mesma oração.
No trecho –... xadrez, quando eu sair para a rua, todo mundo
vai ter que aprender porque vai rolar até o torneio familiar.– o Conjunção é a palavra invariável que liga duas orações
termo em destaque expressa relação de ou dois termos semelhantes de uma mesma oração.
A) espaço, como em – Nosso diretor foi até Brasília para falar
do projeto “Xadrez que liberta”. Morfossintaxe da Conjunção
B) inclusão, como em – O xadrez mudou até o nosso modo
de falar. As conjunções, a exemplo das preposições, não exercem
C) finalidade, como em – Precisamos treinar até junho para propriamente uma função sintática: são conectivos.
termos mais chances de vencer o torneio de xadrez.
D) movimento, como em – Só de chegar até aqui já estou Classificação - Conjunções Coordenativas- Conjunções
muito feliz, porque eu não esperava. Subordinativas
E) tempo, como em – Até o ano que vem, pretendo conseguir
a revisão da minha pena. Conjunções coordenativas
Dividem-se em:
02. Considere o trecho a seguir.
O metrô paulistano, ________quem a banda recebe apoio, - ADITIVAS: expressam a ideia de adição, soma.
garante o espaço para ensaios e os equipamentos; e a estabilidade Ex. Gosto de cantar e de dançar.
no emprego, vantagem________ que muitos trabalhadores sonham, Principais conjunções aditivas: e, nem, não só...mas também,
é o que leva os integrantes do grupo a permanecerem na não só...como também.
instituição.
- ADVERSATIVAS: Expressam ideias contrárias, de oposição,
As preposições que preenchem o trecho, correta, de compensação.
respectivamente e de acordo com a norma-padrão, são: Ex. Estudei, mas não entendi nada.
A) a ...com Principais conjunções adversativas: mas, porém, contudo,
B) de ...com todavia, no entanto, entretanto.
C) de ...a
D) com ...a - ALTERNATIVAS: Expressam ideia de alternância.
E) para ...de Ou você sai do telefone ou eu vendo o aparelho.
Principais conjunções alternativas: Ou...ou, ora...ora, quer...
03. Assinale a alternativa cuja preposição em destaque quer, já...já.
expressa ideia de finalidade.
A) Além disso, aumenta a punição administrativa, de R$ - CONCLUSIVAS: Servem para dar conclusões às orações. Ex.
957,70 para R$ 1.915,40. Estudei muito, por isso mereço passar.
B) ... o STJ (Superior Tribunal de Justiça) decidiu que Principais conjunções conclusivas: logo, por isso, pois
o bafômetro e o exame de sangue eram obrigatórios para (depois do verbo), portanto, por conseguinte, assim.
comprovar o crime.
C) “... Ele é encaminhado para a delegacia para o perito fazer - EXPLICATIVAS: Explicam, dão um motivo ou razão. Ex. É
o exame clínico”... melhor colocar o casaco porque está fazendo muito frio lá fora.
D) Já para o juiz criminal de São Paulo, Fábio Munhoz Principais conjunções explicativas: que, porque, pois (antes
Soares, um dos que devem julgar casos envolvendo pessoas do verbo), porquanto.
embriagadas ao volante, a mudança “é um avanço”.
E) Para advogados, a lei aumenta o poder da autoridade Conjunções subordinativas
policial de dizer quem está embriagado... - CAUSAIS
Principais conjunções causais: porque, visto que, já que, uma
Respostas vez que, como (= porque).
1-B / 2-B / 3-B Ele não fez o trabalho porque não tem livro.

- COMPARATIVAS
15. conjunção e sinonímia; Principais conjunções comparativas: que, do que, tão...como,
mais...do que, menos...do que.
Ela fala mais que um papagaio.
Conjunção
- CONCESSIVAS
Conjunção é a palavra invariável que liga duas orações ou Principais conjunções concessivas: embora, ainda que,
dois termos semelhantes de uma mesma oração. Por exemplo: mesmo que, apesar de, se bem que.
Indicam uma concessão, admitem uma contradição, um fato
A menina segurou a boneca e mostrou quando viu as inesperado. Traz em si uma ideia de “apesar de”.
amiguinhas.
Deste exemplo podem ser retiradas três informações: Embora estivesse cansada, fui ao shopping. (= apesar de estar
cansada)
1-) segurou a boneca 2-) a menina mostrou 3-) viu as Apesar de ter chovido fui ao cinema.
amiguinhas
- CONFORMATIVAS
Cada informação está estruturada em torno de um verbo: Principais conjunções conformativas: como, segundo,
segurou, mostrou, viu. Assim, há nessa frase três orações: conforme, consoante

Língua Portuguesa 49
Apostila Digital Licenciada para Alice Caroline Guarino dos Santos - alice.guarino@hotmail.com (Proibida a Revenda)
APOSTILAS OPÇÃO
Cada um colhe conforme semeia. em nossa língua, de numerosos pares de sinônimos. Exemplos:
Expressam uma ideia de acordo, concordância, conformidade. - Adversário e antagonista.
- Translúcido e diáfano.
- CONSECUTIVAS - Semicírculo e hemiciclo.
Expressam uma ideia de consequência. - Contraveneno e antídoto.
Principais conjunções consecutivas: que (após “tal”, “tanto”, - Moral e ética.
“tão”, “tamanho”). - Colóquio e diálogo.
Falou tanto que ficou rouco. - Transformação e metamorfose.
- Oposição e antítese.
- FINAIS O fato linguístico de existirem sinônimos chama-se sinonímia,
Expressam ideia de finalidade, objetivo. palavra que também designa o emprego de sinônimos.
Todos trabalham para que possam sobreviver.
Principais conjunções finais: para que, a fim de que, porque Questões
(=para que),
01. Leia o texto a seguir.
- PROPORCIONAIS A música alcançou uma onipresença avassaladora em nosso
Principais conjunções proporcionais: à medida que, quanto mundo: milhões de horas de sua história estão disponíveis em
mais, ao passo que, à proporção que. disco; rios de melodia digital correm na internet; aparelhos
À medida que as horas passavam, mais sono ele tinha. de mp3 com 40 mil canções podem ser colocados no bolso. No
entanto, a música não é mais algo que fazemos nós mesmos, ou
- TEMPORAIS até que observamos outras pessoas fazerem diante de nós.
Principais conjunções temporais: quando, enquanto, logo Ela se tornou um meio radicalmente virtual, uma arte sem
que. rosto. Quando caminhamos pela cidade num dia comum, nossos
Quando eu sair, vou passar na locadora. ouvidos registram música em quase todos os momentos − pedaços
de hip-hop vazando dos fones de ouvido de adolescentes no metrô,
Importante: o sinal do celular de um advogado tocando a “Ode à alegria”, de
Beethoven −, mas quase nada disso será resultado imediato de
Diferença entre orações causais e explicativas um trabalho físico de mãos ou vozes humanas, como se dava no
passado.
Quando estudamos Orações Subordinadas Adverbiais (OSA) Desde que Edison inventou o cilindro fonográfico, em1877,
e Coordenadas Sindéticas (CS), geralmente nos deparamos existe gente que avalia o que a gravação fez em favor e desfavor
com a dúvida de como distinguir uma oração causal de uma da arte da música. Inevitavelmente, a conversa descambou para
explicativa. Veja os exemplos: os extremos retóricos. No campo oposto ao dos que diziam que a
tecnologia acabaria com a música estão os utópicos, que alegam
1º) Na frase “Não atravesse a rua, porque você pode ser que a tecnologia não aprisionou a música, mas libertou-a, levando
atropelado”: a arte da elite às massas. Antes de Edison, diziam os utópicos,
a) Temos uma CS Explicativa, que indica uma justificativa ou as sinfonias de Beethoven só podiam ser ouvidas em salas de
uma explicação do fato expresso na oração anterior. concerto selecionadas. Agora, as gravações levam a mensagem
b) As orações são coordenadas e, por isso, independentes de Beethoven aos confins do planeta, convocando a multidão
uma da outra. Neste caso, há uma pausa entre as orações que saudada na “Ode à alegria”: “Abracem-se, milhões!”. Glenn Gould,
vêm marcadas por vírgula. depois de afastar-se das apresentações ao vivo em 1964, previu
Não atravesse a rua. Você pode ser atropelado. que dentro de um século o concerto público desapareceria no éter
b) Outra dica é, quando a oração que antecede a OC (Oração eletrônico, com grande efeito benéfico sobre a cultura musical.
Coordenada) vier com verbo no modo imperativo, ela será (Adaptado de Alex Ross. Escuta só. Tradução Pedro Maia
explicativa. Soares. São Paulo, Cia. das Letras, 2010, p. 76-77)
Façam silêncio, que estou falando. (façam= verbo imperativo)
No entanto, a música não é mais algo que fazemos nós mesmos,
2º) Na frase “Precisavam enterrar os mortos em outra cidade ou até que observamos outras pessoas fazerem diante de nós.
porque não havia cemitério no local.”
a) Temos uma OSA Causal, já que a oração subordinada Considerando-se o contexto, é INCORRETO afirmar que o
(parte destacada) mostra a causa da ação expressa pelo elemento grifado pode ser substituído por:
verbo da oração principal. Outra forma de reconhecê- A) Porém.
la é colocá-la no início do período, introduzida pela B) Contudo.
conjunção como - o que não ocorre com a CS Explicativa. C) Todavia.
Como não havia cemitério no local, precisavam enterrar os mortos D) Entretanto.
em outra cidade. E) Conquanto.
b) As orações são subordinadas e, por isso, totalmente
dependentes uma da outra. 02. Observando as ocorrências da palavra “como” em –
Como fomos programados para ver o mundo como um lugar
Sinônimos: são palavras de sentido igual ou aproximado. ameaçador… – é correto afirmar que se trata de conjunção
Exemplo: (A) comparativa nas duas ocorrências.
- Alfabeto, abecedário. (B) conformativa nas duas ocorrências.
- Brado, grito, clamor. (C) comparativa na primeira ocorrência.
- Extinguir, apagar, abolir, suprimir. (D) causal na segunda ocorrência.
- Justo, certo, exato, reto, íntegro, imparcial. (E) causal na primeira ocorrência.
Na maioria das vezes não é indiferente usar um sinônimo
pelo outro. Embora irmanados pelo sentido comum, os 03. Leia o texto a seguir.
sinônimos diferenciam-se, entretanto, uns dos outros, por
matizes de significação e certas propriedades que o escritor não Participação
pode desconhecer. Com efeito, estes têm sentido mais amplo,
aqueles, mais restrito (animal e quadrúpede); uns são próprios Num belo poema, intitulado “Traduzir-se”, Ferreira Gullar
da fala corrente, desataviada, vulgar, outros, ao invés, pertencem aborda o tema de uma divisão muito presente em cada um de
à esfera da linguagem culta, literária, científica ou poética nós: a que ocorre entre o nosso mundo interior e a nossa atuação
(orador e tribuno, oculista e oftalmologista, cinzento e cinéreo). junto aos outros, nosso papel na ordem coletiva. A divisão não é
A contribuição Greco-latina é responsável pela existência, simples: costuma-se ver como antagônicas essas duas “partes”

Língua Portuguesa 50
Apostila Digital Licenciada para Alice Caroline Guarino dos Santos - alice.guarino@hotmail.com (Proibida a Revenda)
APOSTILAS OPÇÃO
de nós, nas quais nos dividimos. De fato, em quantos momentos
por (per) pelo, pelos
da nossa vida precisamos escolher entre o atendimento de um
interesse pessoal e o cumprimento de um dever ético? Como poeta a, as um, uns uma, umas
e militante político, Ferreira Gullar deixou-se atrair tanto pela
à, às - -
expressão das paixões mais íntimas quanto pela atuação de um
convicto socialista. Em seu poema, o diálogo entre as duas partes da, das dum, duns duma, dumas
é desenvolvido de modo a nos fazer pensar que são incompatíveis.
na, nas num, nuns numa, numas
Mas no último momento do poema deparamo-nos com esta pela, pelas - -
estrofe:
“Traduzir uma parte na outra parte − que é uma questão de - As formas à e às indicam a fusão da preposição a com o
vida ou morte − será arte?” artigo definido a. Essa fusão de vogais idênticas é conhecida
por crase.
O poeta levanta a possibilidade da “tradução” de uma parte
na outra, ou seja, da interação de ambas, numa espécie de Constatemos as circunstâncias em que os artigos se
espelhamento. Isso ocorreria quando o indivíduo conciliasse manifestam:
verdadeiramente a instância pessoal e os interesses de uma
comunidade; quando deixasse de haver contradição entre a razão - Considera-se obrigatório o uso do artigo depois do numeral
particular e a coletiva. Pergunta-se o poeta se não seria arte esse “ambos”:
tipo de integração. Realmente, com muita frequência a arte se Ambos os garotos decidiram participar das olimpíadas.
mostra capaz de expressar tanto nossa subjetividade como nossa
identidade social. - Nomes próprios indicativos de lugar admitem o uso do
Nesse sentido, traduzir uma parte na outra parte significaria artigo, outros não:
vencer a parcialidade e chegar a uma autêntica participação, São Paulo, O Rio de Janeiro, Veneza, A Bahia...
de sentido altamente político. O poema de Gullar deixa-nos essa
hipótese provocadora, formulada com um ar de convicção. - Quando indicado no singular, o artigo definido pode indicar
(Belarmino Tavares, inédito) toda uma espécie:
O trabalho dignifica o homem.
Os seguintes fatos, referidos no texto, travam entre si uma
relação de causa e efeito: - No caso de nomes próprios personativos, denotando a ideia
A) ser poeta e militante político / confronto entre de familiaridade ou afetividade, é facultativo o uso do artigo:
subjetividade e atuação social O Pedro é o xodó da família.
B) ser poeta e militante político / divisão permanente em
cada um de nós - No caso de os nomes próprios personativos estarem no
C) ser movido pelas paixões / esposar teses socialistas plural, são determinados pelo uso do artigo:
D) fazer arte / obliterar uma questão de vida ou morte Os Maias, os Incas, Os Astecas...
E) participar ativamente da política / formular hipóteses
com ar de convicção - Usa-se o artigo depois do pronome indefinido todo(a) para
Respostas conferir uma ideia de totalidade. Sem o uso dele (o artigo), o
1-E / 2-E / 3-A pronome assume a noção de qualquer.
Toda a classe parabenizou o professor. (a sala toda)
Toda classe possui alunos interessados e desinteressados.
16. morfo-sintaxe. (qualquer classe)

- Antes de pronomes possessivos, o uso do artigo é facultativo:


Classes de Palavras Adoro o meu vestido longo. Adoro meu vestido longo.
- A utilização do artigo indefinido pode indicar uma ideia de
Artigo aproximação numérica:
O máximo que ele deve ter é uns vinte anos.
Artigo é a palavra que, vindo antes de um substantivo, indica
se ele está sendo empregado de maneira definida ou indefinida. - O artigo também é usado para substantivar palavras
Além disso, o artigo indica, ao mesmo tempo, o gênero e o oriundas de outras classes gramaticais:
número dos substantivos. Não sei o porquê de tudo isso.

Classificação dos Artigos - Nunca deve ser usado artigo depois do pronome relativo
cujo (e flexões).
Artigos Definidos: determinam os substantivos de maneira Este é o homem cujo amigo desapareceu.
precisa: o, a, os, as. Por exemplo: Eu matei o animal. Este é o autor cuja obra conheço.

Artigos Indefinidos: determinam os substantivos - Não se deve usar artigo antes das palavras casa (no sentido
de maneira vaga: um, uma, uns, umas. Por exemplo: Eu de lar, moradia) e terra (no sentido de chão firme), a menos que
matei um animal. venham especificadas.
Eles estavam em casa.
Combinação dos Artigos Eles estavam na casa dos amigos.
É muito presente a combinação dos artigos definidos e Os marinheiros permaneceram em terra.
indefinidos com preposições. Este quadro apresenta a forma Os marinheiros permanecem na terra dos anões.
assumida por essas combinações:
- Não se emprega artigo antes dos pronomes de tratamento,
Preposições Artigos
com exceção de senhor(a), senhorita e dona.
- o, os Vossa excelência resolverá os problemas de Sua Senhoria.
a ao, aos
- Não se une com preposição o artigo que faz parte do nome
de do, dos de revistas, jornais, obras literárias.
Li a notícia em O Estado de S. Paulo.
em no, nos

Língua Portuguesa 51
Apostila Digital Licenciada para Alice Caroline Guarino dos Santos - alice.guarino@hotmail.com (Proibida a Revenda)
APOSTILAS OPÇÃO
Morfossintaxe cidade, menino, homem, mulher, país, cachorro.
Para definir o que é artigo é preciso mencionar suas relações
com o substantivo. Assim, nas orações da língua portuguesa, Estamos voando para Barcelona.
o artigo exerce a função de adjunto adnominal do substantivo
a que se refere. Tal função independe da função exercida pelo O substantivo Barcelona designa apenas um ser da espécie
substantivo: cidade. Esse substantivo é próprio. Substantivo Próprio: é
aquele que designa os seres de uma mesma espécie de forma
A existência é uma poesia. particular.
Uma existência é a poesia.
Londres, Paulinho, Pedro, Tietê, Brasil.
Questões
2 - Substantivos Concretos e Abstratos
01. Determine o caso em que o artigo tem valor qualificativo:
A) Estes são os candidatos que lhe falei. LÂMPADA MALA
B) Procure-o, ele é o médico! Ninguém o supera.
C) Certeza e exatidão, estas qualidades não as tenho. Os substantivos lâmpada e mala designam seres com
D) Os problemas que o afligem não me deixam descuidado. existência própria, que são independentes de outros seres. São
E) Muito é a procura; pouca é a oferta. assim, substantivos concretos.
Substantivo Concreto: é aquele que designa o ser que existe,
02. Em qual dos casos o artigo denota familiaridade? independentemente de outros seres.
A) O Amazonas é um rio imenso.
B) D. Manuel, o Venturoso, era bastante esperto.
C) O Antônio comunicou-se com o João. Obs.: os substantivos concretos designam seres do mundo
D) O professor João Ribeiro está doente. real e do mundo imaginário.
E) Os Lusíadas são um poema épico
Seres do mundo real: homem, mulher, cadeira, cobra, Brasília,
03.Assinale a alternativa em que o uso do artigo está etc.
substantivando uma palavra. Seres do mundo imaginário: saci, mãe-d’água, fantasma, etc.
A) A liberdade vai marcar a poesia social de Castro Alves.
B) Leitor perspicaz é aquele que consegue ler as entrelinhas. Observe agora:
C) A navalha ia e vinha no couro esticado.
D) Haroldo ficou encantado com o andar de bailado de Joana. Beleza exposta
E) Bárbara dirigia os olhos para a lua encantada. Jovens atrizes veteranas destacam-se pelo visual.

Respostas O substantivo beleza designa uma qualidade.


1-B / 2-C / 3-D Substantivo Abstrato: é aquele que designa seres que
dependem de outros para se manifestar ou existir.
Substantivo Pense bem: a beleza não existe por si só, não pode ser
observada. Só podemos observar a beleza numa pessoa ou coisa
Tudo o que existe é ser e cada ser tem um nome. Substantivo é que seja bela. A beleza depende de outro ser para se manifestar.
a classe gramatical de palavras variáveis, as quais denominam Portanto, a palavra beleza é um substantivo abstrato.
os seres. Além de objetos, pessoas e fenômenos, os substantivos Os substantivos abstratos designam estados, qualidades,
também nomeiam: ações e sentimentos dos seres, dos quais podem ser abstraídos,
-lugares: Alemanha, Porto Alegre... e sem os quais não podem existir.
-sentimentos: raiva, amor... vida (estado), rapidez (qualidade), viagem (ação), saudade
-estados: alegria, tristeza... (sentimento).  
-qualidades: honestidade, sinceridade...
-ações: corrida, pescaria... 3 - Substantivos Coletivos
Ele vinha pela estrada e foi picado por uma abelha, outra
Morfossintaxe do substantivo abelha, mais outra abelha.
Ele vinha pela estrada e foi picado por várias abelhas.
Nas orações de língua portuguesa, o substantivo em geral Ele vinha pela estrada e foi picado por um enxame.
exerce funções diretamente relacionadas com o verbo: atua
como núcleo do sujeito, dos complementos verbais (objeto Note que, no primeiro caso, para indicar plural, foi necessário
direto ou indireto) e do agente da passiva. Pode ainda funcionar repetir o substantivo: uma abelha, outra abelha, mais outra
como núcleo do complemento nominal ou do aposto, como abelha...
núcleo do predicativo do sujeito ou do objeto ou como núcleo No segundo caso, utilizaram-se duas palavras no plural.
do vocativo. Também encontramos substantivos como núcleos No terceiro caso, empregou-se um substantivo no singular
de adjuntos adnominais e de adjuntos adverbiais - quando essas (enxame) para designar um conjunto de seres da mesma espécie
funções são desempenhadas por grupos de palavras. (abelhas).
O substantivo enxame é um substantivo coletivo.
Classificação dos Substantivos
Substantivo Coletivo: é o substantivo comum que, mesmo
1- Substantivos Comuns e Próprios estando no singular, designa um conjunto de seres da mesma
Observe a definição: espécie.
Formação dos Substantivos
s.f. 1: Povoação maior que vila, com muitas casas e edifícios, Substantivos Simples e Compostos
dispostos em ruas e avenidas (no Brasil, toda a sede de município
é cidade). 2. O centro de uma cidade (em oposição aos bairros). Chuva - subst. Fem. 1 - água caindo em gotas sobre a terra.

Qualquer “povoação maior que vila, com muitas casas e O substantivo chuva é formado por um único elemento ou
edifícios, dispostos em ruas e avenidas” será chamada cidade. radical. É um substantivo simples.
Isso significa que a palavra cidade é um substantivo comum. Substantivo Simples: é aquele formado por um único
Substantivo Comum é aquele que designa os seres de uma elemento.
mesma espécie de forma genérica. Outros substantivos simples: tempo, sol, sofá, etc. Veja agora:

Língua Portuguesa 52
Apostila Digital Licenciada para Alice Caroline Guarino dos Santos - alice.guarino@hotmail.com (Proibida a Revenda)
APOSTILAS OPÇÃO
O substantivo guarda-chuva é formado por dois elementos a) Regra geral: troca-se a terminação -o por -a.
(guarda + chuva). Esse substantivo é composto. aluno - aluna
Substantivo Composto: é aquele formado por dois ou mais
elementos. b) Substantivos terminados em -ês: acrescenta-se -a ao
Outros exemplos: beija-flor, passatempo. masculino.
freguês - freguesa
Substantivos Primitivos e Derivados
Meu limão meu limoeiro, c) Substantivos terminados em -ão: fazem o feminino de três
meu pé de jacarandá... formas:
- troca-se -ão por -oa. = patrão – patroa
O substantivo limão é primitivo, pois não se originou de - troca-se -ão por -ã. = campeão - campeã
nenhum outro dentro de língua portuguesa. - troca-se -ão por ona. = solteirão - solteirona
Substantivo Primitivo: é aquele que não deriva de nenhuma
outra palavra da própria língua portuguesa. Exceções: barão – baronesa ladrão- ladra sultão - sultana
O substantivo limoeiro é derivado, pois se originou a partir
da palavra limão. d) Substantivos terminados em -or:
Substantivo Derivado: é aquele que se origina de outra - acrescenta-se -a ao masculino = doutor – doutora
palavra. - troca-se -or por -triz: = imperador - imperatriz

Flexão dos substantivos e) Substantivos com feminino em -esa, -essa, -isa:


O substantivo é uma classe variável. A palavra é variável cônsul - consulesa abade - abadessa poeta - poetisa
quando sofre flexão (variação). A palavra menino, por exemplo, duque - duquesa conde - condessa profeta - profetisa
pode sofrer variações para indicar:
Plural: meninos f) Substantivos que formam o feminino trocando o -e final
Feminino: menina por -a:
Aumentativo: meninão elefante - elefanta
Diminutivo: menininho
g) Substantivos que têm radicais diferentes no masculino e
Flexão de Gênero no feminino:
Gênero é a propriedade que as palavras têm de indicar bode – cabra boi - vaca
sexo real ou fictício dos seres. Na língua portuguesa,
há dois gêneros: masculino e feminino. Pertencem ao h) Substantivos que formam o feminino de maneira especial,
gênero masculino os substantivos que podem vir precedidos dos isto é, não seguem nenhuma das regras anteriores:
artigos o, os, um, uns. Veja estes títulos de filmes: czar – czarina réu - ré
O velho e o mar
Um Natal inesquecível Formação do Feminino dos Substantivos Uniformes
Os reis da praia
- Epicenos:
Pertencem ao gênero feminino os substantivos que podem Novo jacaré escapa de policiais no rio Pinheiros.
vir precedidos dos artigos a, as, uma, umas: Não é possível saber o sexo do jacaré em questão. Isso ocorre
A história sem fim porque o substantivo jacaré tem apenas uma forma para indicar
Uma cidade sem passado o masculino e o feminino.
As tartarugas ninjas Alguns nomes de animais apresentam uma só forma para
designar os dois sexos. Esses substantivos são chamados de
Substantivos Biformes e Substantivos Uniformes epicenos. No caso dos epicenos, quando houver a necessidade
de especificar o sexo, utilizam-se palavras macho e fêmea.
Substantivos Biformes (= duas formas):  ao indicar nomes A cobra macho picou o marinheiro.
de seres vivos, geralmente o gênero da palavra está relacionado A cobra fêmea escondeu-se na bananeira.
ao sexo do ser, havendo, portanto, duas formas, uma para o
masculino e outra para o feminino. Observe: gato – gata, homem Sobrecomuns:
– mulher, poeta – poetisa, prefeito - prefeita
Entregue as crianças à natureza.
Substantivos Uniformes: são aqueles que apresentam uma A palavra crianças refere-se tanto a seres do sexo masculino,
única forma, que serve tanto para o masculino quanto para o quanto a seres do sexo feminino. Nesse caso, nem o artigo nem
feminino. Classificam-se em: um possível adjetivo permitem identificar o sexo dos seres a que
- Epicenos: têm um só gênero e nomeiam bichos. se refere a palavra. Veja:
a cobra macho e a cobra fêmea, o jacaré macho e o jacaré A criança chorona chamava-se João.
fêmea. A criança chorona chamava-se Maria.
- Sobrecomuns: têm um só gênero e nomeiam pessoas. Outros substantivos sobrecomuns:
a criança, a testemunha, a vítima, o cônjuge, o gênio, o ídolo, a criatura = João é uma boa criatura. Maria é uma boa
o indivíduo. criatura.
o cônjuge = O cônjuge de João faleceu. O
- Comuns de Dois Gêneros: indicam o sexo das pessoas por cônjuge de Marcela faleceu
meio do artigo.
o colega e a colega, o doente e a doente, o artista e a artista. Comuns de Dois Gêneros:
Saiba que:
- Substantivos de origem grega terminados em ema ou oma, Motorista tem acidente idêntico 23 anos depois.
são masculinos. Quem sofreu o acidente: um homem ou uma mulher?
o axioma, o fonema, o poema, o sistema, o sintoma, o teorema. É impossível saber apenas pelo título da notícia, uma vez
- Existem certos substantivos que, variando de gênero, que a palavra motorista é um substantivo uniforme. O restante
variam em seu significado. da notícia informa-nos de que se trata de um homem.
o rádio (aparelho receptor) e a rádio (estação emissora) o A distinção de gênero pode ser feita através da análise do
capital (dinheiro) e a capital (cidade) artigo ou adjetivo, quando acompanharem o substantivo.
o colega - a colega
Formação do Feminino dos Substantivos Biformes um jovem - uma jovem

Língua Portuguesa 53
Apostila Digital Licenciada para Alice Caroline Guarino dos Santos - alice.guarino@hotmail.com (Proibida a Revenda)
APOSTILAS OPÇÃO
artista famoso - artista famosa Uma Londres imensa e triste.

- A palavra personagem é usada indistintamente nos dois Exceções: o Rio de Janeiro, o Cairo, o Porto, o Havre.
gêneros.
a) Entre os escritores modernos nota-se acentuada Gênero e Significação:
preferência pelo masculino:
O menino descobriu nas nuvens os personagens dos contos de Muitos substantivos têm uma significação no masculino e
carochinha. outra no feminino.
b) Com referência a mulher, deve-se preferir o feminino: Observe:
O problema está nas mulheres de mais idade, que não aceitam
a personagem. o baliza (soldado que, que à frente da tropa, indica os
Não cheguei assim, nem era minha intenção, a criar uma movimentos que se deve realizar em conjunto; o que vai à frente
personagem. de um bloco carnavalesco, manejando um bastão)
- Diz-se: o (ou a) manequim Marcela, o (ou a) modelo a baliza (marco, estaca; sinal que marca um limite ou
fotográfico Ana Belmonte. proibição de trânsito)

Observe o gênero dos substantivos seguintes: o cabeça (chefe)


a cabeça (parte do corpo)
Masculinos
o tapa o cisma (separação religiosa, dissidência)
o eclipse a cisma (ato de cismar, desconfiança)
o lança-perfume
o dó (pena) o cinza (a cor cinzenta)
o sanduíche a cinza (resíduos de combustão)
o clarinete
o champanha o capital (dinheiro)
o sósia a capital (cidade)
o maracajá
o clã o coma (perda dos sentidos)
o hosana a coma (cabeleira)
o herpes
o pijama o coral (pólipo, a cor vermelha, canto em coro)
a coral (cobra venenosa)
Femininos
a dinamite o crisma (óleo sagrado, usado na administração da crisma e
a áspide de outros sacramentos)
a derme a crisma (sacramento da confirmação)
a hélice
a alcíone o cura (pároco)
a filoxera a cura (ato de curar)
a clâmide
a omoplata o estepe (pneu sobressalente)
a cataplasma a estepe (vasta planície de vegetação)
a pane
a mascote o guia (pessoa que guia outras)
a gênese a guia (documento, pena grande das asas das aves)
a entorse
a libido o grama (unidade de peso)
a grama (relva)
- São geralmente masculinos os substantivos de origem
grega terminados em -ma: o caixa (funcionário da caixa)
o grama (peso) a caixa (recipiente, setor de pagamentos)
o quilograma
o plasma o lente (professor)
o apostema a lente (vidro de aumento)
o diagrama
o epigrama o moral (ânimo)
o telefonema a moral (honestidade, bons costumes, ética)
o estratagema
o dilema o nascente (lado onde nasce o Sol)
o teorema a nascente (a fonte)
o apotegma
o trema Flexão de Número do Substantivo
o eczema
o edema Em português, há dois números gramaticais: o singular, que
o magma indica um ser ou um grupo de seres, e
o plural, que indica mais de um ser ou grupo de seres. A
Exceções: a cataplasma, a celeuma, a fleuma, etc. característica do plural é o “s” final.

Gênero dos Nomes de Cidades: Plural dos Substantivos Simples

Com raras exceções, nomes de cidades são femininos. a) Os substantivos terminados em vogal, ditongo oral e “n”
A histórica Ouro Preto. fazem o plural pelo acréscimo de “s”.
A dinâmica São Paulo. pai – pais ímã - ímãs hífen - hifens (sem acento, no
A acolhedora Porto Alegre. plural).

Língua Portuguesa 54
Apostila Digital Licenciada para Alice Caroline Guarino dos Santos - alice.guarino@hotmail.com (Proibida a Revenda)
APOSTILAS OPÇÃO
Exceção: cânon - cânones. notícia-bomba - notícias-bomba
homem-rã - homens-rã
b) Os substantivos terminados em “m” fazem o plural em
“ns”. d) Permanecem invariáveis, quando formados de:
homem - homens. verbo + advérbio = o bota-fora e os bota-fora
verbo + substantivo no plural = o saca-rolhas e os saca-rolhas
c) Os substantivos terminados em “r” e “z” fazem o plural
pelo acréscimo de “es”. e) Casos Especiais
revólver – revólveres raiz - raízes o louva-a-deus e os louva-a-deus
Atenção: O plural de caráter é caracteres. o bem-te-vi e os bem-te-vis
o bem-me-quer e os bem-me-queres
d) Os substantivos terminados em al, el, ol, ul flexionam-se o joão-ninguém e os joões-ninguém.
no plural, trocando o “l” por “is”.
quintal - quintais caracol – caracóis hotel - hotéis Plural das Palavras Substantivadas
Exceções: mal e males, cônsul e cônsules.
As palavras substantivadas, isto é, palavras de outras classes
e) Os substantivos terminados em “il” fazem o plural de duas gramaticais usadas como substantivo, apresentam, no plural, as
maneiras: flexões próprias dos substantivos.
- Quando oxítonos, em “is”: canil - canis Pese bem os prós e os contras.
- Quando paroxítonos, em “eis”: míssil - mísseis. O aluno errou na prova dos noves.
Obs.: a palavra réptil pode formar seu plural de duas Ouça com a mesma serenidade os sins e os nãos.
maneiras: répteis ou reptis (pouco usada). Obs.: numerais substantivados terminados em “s” ou “z” não
variam no plural.
f) Os substantivos terminados em “s” fazem o plural de duas Nas provas mensais consegui muitos seis e alguns dez.
maneiras:
- Quando monossilábicos ou oxítonos, mediante o acréscimo Plural dos Diminutivos
de “es”: ás – ases / retrós - retroses
- Quando paroxítonos ou proparoxítonos, ficam invariáveis: Flexiona-se o substantivo no plural, retira-se o “s” final e
o lápis - os lápis / o ônibus - os ônibus. acrescenta-se o sufixo diminutivo.
pãe(s) + zinhos = pãezinhos
g) Os substantivos terminados em “ao” fazem o plural de três animai(s) + zinhos = animaizinhos
maneiras. botõe(s) + zinhos = botõezinhos
- substituindo o -ão por -ões: ação - ações chapéu(s) + zinhos = chapeuzinhos
- substituindo o -ão por -ães: cão - cães farói(s) + zinhos = faroizinhos
- substituindo o -ão por -ãos: grão - grãos tren(s) + zinhos = trenzinhos
h) Os substantivos terminados em “x” ficam invariáveis: o colhere(s) + zinhas = colherezinhas
látex - os látex. flore(s) + zinhas = florezinhas
mão(s) + zinhas = mãozinhas
Plural dos Substantivos Compostos papéi(s) + zinhos = papeizinhos
A formação do plural dos substantivos compostos depende nuven(s) + zinhas = nuvenzinhas
da forma como são grafados, do tipo de palavras que formam funi(s) + zinhos = funizinhos
o composto e da relação que estabelecem entre si. Aqueles que pé(s) + zitos = pezitos
são grafados sem hífen comportam-se como os substantivos
simples: Plural dos Nomes Próprios Personativos
aguardente e aguardentes girassol e girassóis
pontapé e pontapés malmequer e malmequeres Devem-se pluralizar os nomes próprios de pessoas sempre
que a terminação preste-se à flexão.
O plural dos substantivos compostos cujos elementos são Os Napoleões também são derrotados.
ligados por hífen costuma provocar muitas dúvidas e discussões. As Raquéis e Esteres.
Algumas orientações são dadas a seguir:
Plural dos Substantivos Estrangeiros
a) Flexionam-se os dois elementos, quando formados de:
substantivo + substantivo = couve-flor e couves-flores Substantivos ainda não aportuguesados devem ser escritos
substantivo + adjetivo = amor-perfeito e amores-perfeitos como na língua original, acrescentando -se “s” (exceto quando
adjetivo + substantivo = gentil-homem e gentis-homens terminam em “s” ou “z”).
numeral + substantivo = quinta-feira e quintas-feiras os shows os shorts os jazz
Substantivos já aportuguesados flexionam-se de acordo com
b) Flexiona-se somente o segundo elemento, quando as regras de nossa língua:
formados de: os clubes os chopes
verbo + substantivo = guarda-roupa e guarda-roupas os jipes os esportes
palavra invariável + palavra variável = alto-falante e alto- as toaletes os bibelôs
falantes os garçons os réquiens
palavras repetidas ou imitativas = reco-reco e reco-recos
Observe o exemplo:
c) Flexiona-se somente o primeiro elemento, quando Este jogador faz gols toda vez que joga.
formados de: O plural correto seria gois (ô), mas não se usa.
substantivo + preposição clara + substantivo = água-de-
colônia e águas-de-colônia Plural com Mudança de Timbre
substantivo + preposição oculta + substantivo = cavalo-
vapor e cavalos-vapor Certos substantivos formam o plural com mudança de
substantivo + substantivo que funciona como determinante timbre da vogal tônica (o fechado / o aberto). É um fato fonético
do primeiro, ou seja, especifica a função ou o tipo do termo chamado metafonia (plural metafônico).
anterior.
palavra-chave - palavras-chave
bomba-relógio - bombas-relógio Singular Plural Singular Plural

Língua Portuguesa 55
Apostila Digital Licenciada para Alice Caroline Guarino dos Santos - alice.guarino@hotmail.com (Proibida a Revenda)
APOSTILAS OPÇÃO
03. Indique a alternativa em que a flexão do substantivo está
corpo (ô) corpos (ó) osso (ô) ossos (ó)
errada:
esforço esforços ovo ovos
A) Catalães.
fogo fogos poço poços
B) Cidadãos.
forno fornos porto portos
C) Vulcães.
fosso fossos posto postos
D) Corrimões.
imposto impostos rogo rogos
Respostas
olho olhos tijolo tijolos
1-D / 2-D / 3-C
Têm a vogal tônica fechada (ô): adornos, almoços, bolsos, Adjetivo
esposos, estojos, globos, gostos, polvos, rolos, soros, etc.
Obs.: distinga-se molho (ô) = caldo (molho de carne), de Adjetivo é a palavra que expressa uma qualidade ou
molho (ó) = feixe (molho de lenha). característica do ser e se relaciona com o substantivo.
Ao analisarmos a palavra bondoso, por exemplo, percebemos
Particularidades sobre o Número dos Substantivos que, além de expressar uma qualidade, ela pode ser colocada ao
lado de um substantivo: homem bondoso, moça bondosa, pessoa
a) Há substantivos que só se usam no singular: bondosa.
o sul, o norte, o leste, o oeste, a fé, etc. Já com a palavra bondade, embora expresse uma qualidade,
não acontece o mesmo; não faz sentido dizer: homem bondade,
b) Outros só no plural: moça bondade, pessoa bondade.
as núpcias, os víveres, os pêsames, as espadas/os paus Bondade, portanto, não é adjetivo, mas substantivo.
(naipes de baralho), as fezes.
Morfossintaxe do Adjetivo:
c) Outros, enfim, têm, no plural, sentido diferente do singular: O adjetivo exerce sempre funções sintáticas (função dentro
bem (virtude) e bens (riquezas) de uma oração) relativas aos substantivos, atuando como adjunto
honra (probidade, bom nome) e honras (homenagem, adnominal ou como predicativo (do sujeito ou do objeto).
títulos)
Adjetivo Pátrio
d) Usamos às vezes, os substantivos no singular, mas com Indica a nacionalidade ou o lugar de origem do ser. Observe
sentido de plural: alguns deles:
Aqui morreu muito negro. Estados e cidades brasileiros:
Celebraram o sacrifício divino muitas vezes em capelas
improvisadas.
Alagoas alagoano
Flexão de Grau do Substantivo Amapá amapaense
Grau é a propriedade que as palavras têm de exprimir as
variações de tamanho dos seres. Classifica-se em: Aracaju aracajuano ou aracajuense
Amazonas amazonense ou baré
- Grau Normal - Indica um ser de tamanho considerado
normal. Por exemplo: casa Belo Horizonte belo-horizontino
Brasília brasiliense
- Grau Aumentativo - Indica o aumento do tamanho do ser.
Classifica-se em: Cabo Frio cabo-friense
Analítico = o substantivo é acompanhado de um adjetivo que Campinas campineiro ou campinense
indica grandeza. Por exemplo: casa grande.
Sintético = é acrescido ao substantivo um sufixo indicador de Adjetivo Pátrio Composto
aumento. Por exemplo: casarão. Na formação do adjetivo pátrio composto, o primeiro
elemento aparece na forma reduzida e, normalmente, erudita.
- Grau Diminutivo - Indica a diminuição do tamanho do ser. Observe alguns exemplos:
Pode ser:
Analítico = substantivo acompanhado de um adjetivo que
indica pequenez. Por exemplo: casa pequena. África afro- / Por exemplo: Cultura afro-americana
Sintético = é acrescido ao substantivo um sufixo indicador de Alemanha germano- ou teuto- / Por exemplo:
diminuição. Por exemplo: casinha. Competições teuto-inglesas
Fonte: http://www.soportugues.com.br/secoes/morf/morf12.php América américo- / Por exemplo: Companhia
américo-africana
Questões Bélgica belgo- / Por exemplo: Acampamentos belgo-
franceses
01. A flexão de número do termo “preços-sombra” também
ocorre com o plural de China sino- / Por exemplo: Acordos sino-japoneses
(A) reco-reco. Espanha hispano- / Por exemplo: Mercado hispano-
(B) guarda-costa. português
(C) guarda-noturno.
(D) célula-tronco. Europa euro- / Por exemplo: Negociações euro-
(E) sem-vergonha. americanas
França franco- ou galo- / Por exemplo: Reuniões
02. Assinale a alternativa cujas palavras se apresentam franco-italianas
flexionadas de acordo com a norma-padrão.
(A) Os tabeliãos devem preparar o documento. Grécia greco- / Por exemplo: Filmes greco-romanos
(B) Esses cidadões tinham autorização para portar fuzis. Inglaterra anglo- / Por exemplo: Letras anglo-
(C) Para autenticar as certidãos, procure o cartório local. portuguesas
(D) Ao descer e subir escadas, segure-se nos corrimãos.
(E) Cuidado com os degrais, que são perigosos! Itália ítalo- / Por exemplo: Sociedade ítalo-
portuguesa

Língua Portuguesa 56
Apostila Digital Licenciada para Alice Caroline Guarino dos Santos - alice.guarino@hotmail.com (Proibida a Revenda)
APOSTILAS OPÇÃO
Observe
Japão nipo- / Por exemplo: Associações nipo-
- Azul-marinho, azul-celeste, ultravioleta e qualquer adjetivo
brasileiras
composto iniciado por cor-de-... são sempre invariáveis.
Portugal luso- / Por exemplo: Acordos luso-brasileiros - O adjetivo composto pele-vermelha têm os dois elementos
flexionados.
Flexão dos adjetivos
Grau do Adjetivo
O adjetivo varia em gênero, número e grau.
Os adjetivos flexionam-se em grau para indicar a
Gênero dos Adjetivos intensidade da qualidade do ser. São dois os graus do adjetivo:
o comparativo e o superlativo.
Os adjetivos concordam com o substantivo a que se referem
(masculino e feminino). De forma semelhante aos substantivos, Comparativo
classificam-se em:
Biformes - têm duas formas, sendo uma para o masculino e Nesse grau, comparam-se a mesma característica
outra para o feminino. atribuída a dois ou mais seres ou duas ou mais características
atribuídas ao mesmo ser. O comparativo pode ser de igualdade,
Por exemplo: ativo e ativa, mau e má, judeu e judia. de superioridade ou de inferioridade. Observe os exemplos
abaixo:
Se o adjetivo é composto e biforme, ele flexiona no feminino
somente o último elemento. 1) Sou tão alto como você. = Comparativo de Igualdade
Por exemplo: o moço norte-americano, a moça norte- No comparativo de igualdade, o segundo termo da
americana. comparação é introduzido pelas palavras como, quanto ou quão.

Uniformes - têm uma só forma tanto para o masculino como 2) Sou mais alto (do) que você. = Comparativo de
para o feminino. Por exemplo: homem feliz e mulher feliz. Superioridade Analítico
Se o adjetivo é composto e uniforme, fica invariável no No comparativo de superioridade analítico, entre os dois
feminino. Por exemplo: conflito político-social e desavença substantivos comparados, um tem qualidade superior. A forma é
político-social. analítica porque pedimos auxílio a “mais...do que” ou “mais...que”.

Número dos Adjetivos 3) O Sol é maior (do) que a Terra. = Comparativo de


Superioridade Sintético
Plural dos adjetivos simples
Os adjetivos simples flexionam-se no plural de acordo com Alguns adjetivos possuem, para o comparativo de
as regras estabelecidas para a flexão numérica dos substantivos superioridade, formas sintéticas, herdadas do latim.
simples.
Por exemplo: São eles:
mau e maus bom-melhor
feliz e felizes pequeno-menor
ruim e ruins mau-pior
boa e boas alto-superior
grande-maior
Caso o adjetivo seja uma palavra que também exerça função baixo-inferior
de substantivo, ficará invariável, ou seja, se a palavra que estiver
qualificando um elemento for, originalmente, um substantivo, Observe que:
ela manterá sua forma primitiva. Exemplo: a palavra cinza é a) As formas menor e pior são comparativos de superioridade,
originalmente um substantivo; porém, se estiver qualificando pois equivalem a mais pequeno e mais mau, respectivamente.
um elemento, funcionará como adjetivo. Ficará, então, invariável. b) Bom, mau, grande e pequeno têm formas sintéticas
Logo: camisas cinza, ternos cinza. (melhor, pior, maior e menor), porém, em comparações feitas
Veja outros exemplos: entre duas qualidades de um mesmo elemento, deve-se usar
as formas analíticas mais bom, mais mau, mais grande e mais
Motos vinho (mas: motos verdes) pequeno.
Paredes musgo (mas: paredes brancas). Por exemplo: Pedro é maior do que Paulo - Comparação de
Comícios monstro (mas: comícios grandiosos). dois elementos.
Pedro é mais grande que pequeno - comparação de duas
Adjetivo Composto qualidades de um mesmo elemento.

É aquele formado por dois ou mais elementos. Normalmente, 4) Sou menos alto (do) que você. = Comparativo de
esses elementos são ligados por hífen. Apenas o último elemento Inferioridade
concorda com o substantivo a que se refere; os demais ficam Sou menos passivo (do) que tolerante.
na forma masculina, singular. Caso um dos elementos que
formam o adjetivo composto seja um substantivo adjetivado, Superlativo
todo o adjetivo composto ficará invariável. Por exemplo: a
palavra rosa é originalmente um substantivo, porém, se estiver O superlativo expressa qualidades num grau muito
qualificando um elemento, funcionará como adjetivo. Caso se elevado ou em grau máximo. O grau superlativo pode ser
ligue a outra palavra por hífen, formará um adjetivo composto; absoluto ou relativo e apresenta as seguintes modalidades:
como é um substantivo adjetivado, o adjetivo composto inteiro Superlativo Absoluto: ocorre quando a qualidade de um
ficará invariável. Por exemplo: ser é intensificada, sem relação com outros seres. Apresenta-se
nas formas:
Camisas rosa-claro. Analítica: a intensificação se faz com o auxílio de palavras
Ternos rosa-claro. que dão ideia de intensidade (advérbios). Por exemplo: O
Olhos verde-claros. secretário é muito inteligente.
Calças azul-escuras e camisas verde-mar. Sintética: a intensificação se faz por meio do acréscimo de
Telhados marrom-café e paredes verde-claras. sufixos.
Por exemplo:

Língua Portuguesa 57
Apostila Digital Licenciada para Alice Caroline Guarino dos Santos - alice.guarino@hotmail.com (Proibida a Revenda)
APOSTILAS OPÇÃO
O secretário é inteligentíssimo. Na periferia das cidades brasileiras vivem milhões de crianças
que se enquadram nessas três condições de risco. Associados à
Observe alguns superlativos sintéticos: falta de acesso aos recursos materiais, à desigualdade social,
esses fatores de risco criam o caldo de cultura que alimenta a
violência crescente nas cidades.
benéfico beneficentíssimo Na falta de outra alternativa, damos à criminalidade a
bom boníssimo ou ótimo resposta do aprisionamento. Porém, seu efeito é passageiro: o
criminoso fica impedido de delinquir apenas enquanto estiver
comum comuníssimo preso.
cruel crudelíssimo Ao sair, estará mais pobre, terá rompido laços familiares
e sociais e dificilmente encontrará quem lhe dê emprego. Ao
difícil dificílimo mesmo tempo, na prisão, terá criado novas amizades e conexões
doce dulcíssimo mais sólidas com o mundo do crime.
Construir cadeias custa caro; administrá-las, mais ainda.
fácil facílimo Obrigados a optar por uma repressão policial mais ativa,
fiel fidelíssimo aumentaremos o número de prisioneiros. As cadeias continuarão
superlotadas.
Superlativo Relativo: ocorre quando a qualidade de um ser Seria mais sensato investir em educação, para prevenir a
é intensificada em relação a um conjunto de seres. Essa relação criminalidade e tratar os que ingressaram nela.
pode ser: Na verdade, não existe solução mágica a curto prazo.
De Superioridade: Clara é a mais bela da sala. Precisamos de uma divisão de renda menos brutal, motivar os
De Inferioridade: Clara é a menos bela da sala. policiais a executar sua função com dignidade, criar leis que
acabem com a impunidade dos criminosos bem-sucedidos e
Note bem: construir cadeias novas para substituir as velhas.
1) O superlativo absoluto analítico é expresso por meio Enquanto não aprendermos a educar e oferecer medidas
dos advérbios muito, extremamente, excepcionalmente, etc., preventivas para que os pais evitem ter filhos que não serão
antepostos ao adjetivo. capazes de criar, cabe a nós a responsabilidade de integrá-los
2) O superlativo absoluto sintético apresenta-se sob duas na sociedade por meio da educação formal de bom nível, das
formas : uma erudita, de origem latina, outra popular, de origem práticas esportivas e da oportunidade de desenvolvimento
vernácula. A forma erudita é constituída pelo radical do adjetivo artístico.
latino + um dos sufixos -íssimo, -imo ou érrimo. Por exemplo:
fidelíssimo, facílimo, paupérrimo. (Drauzio Varella. In Folha de S.Paulo, 9 mar.2002. Adaptado)
A forma popular é constituída do radical do adjetivo
português + o sufixo -íssimo: pobríssimo, agilíssimo. Em – características epidêmicas –, o adjetivo epidêmicas
3) Em vez dos superlativos normais seriíssimo, precariíssimo, corresponde a – características de epidemias.
necessariíssimo, preferem-se, na linguagem atual, as formas Assinale a alternativa em que, da mesma forma, o adjetivo
seríssimo, precaríssimo, necessaríssimo, sem o desagradável em destaque corresponde, corretamente, à expressão indicada.
hiato i-í. A) água fluvial – água da chuva.
B) produção aurífera – produção de ouro.
Questões C) vida rupestre – vida do campo.
D) notícias brasileiras – notícias de Brasília.
01. Leia o texto a seguir. E) costela bovina – costela de porco.

Violência epidêmica 02.Não se pluraliza os adjetivos compostos abaixo, exceto:


A) azul-celeste
A violência urbana é uma enfermidade contagiosa. Embora B) azul-pavão
possa acometer indivíduos vulneráveis em todas as classes C) surda-muda
sociais, é nos bairros pobres que ela adquire características D) branco-gelo
epidêmicas.
A prevalência varia de um país para outro e entre as cidades 03.Assinale a única alternativa em que os adjetivos não
de um mesmo país, mas, como regra, começa nos grandes estão no grau superlativo absoluto sintético:
centros urbanos e se dissemina pelo interior. A) Arquimilionário/ ultraconservador;
As estratégias que as sociedades adotam para combater a B) Supremo/ ínfimo;
violência variam muito e a prevenção das causas evoluiu muito C) Superamigo/ paupérrimo;
pouco no decorrer do século 20, ao contrário dos avanços D) Muito amigo/ Bastante pobre
ocorridos no campo das infecções, câncer, diabetes e outras
enfermidades. Respostas
A agressividade impulsiva é consequência de perturbações 1-B / 2-C / 3-D
nos mecanismos biológicos de controle emocional. Tendências
agressivas surgem em indivíduos com dificuldades adaptativas Advérbio
que os tornam despreparados para lidar com as frustrações de
seus desejos. O advérbio, assim como muitas outras palavras existentes
A violência é uma doença. Os mais vulneráveis são os que na Língua Portuguesa, advém de outras línguas. Assim sendo,
tiveram a personalidade formada num ambiente desfavorável ao tal qual o adjetivo, o prefixo “ad-” indica a ideia de proximidade,
desenvolvimento psicológico pleno. contiguidade.
A revisão de estudos científicos permite identificar três
fatores principais na formação das personalidades com maior Essa proximidade faz referência ao processo verbal, no
inclinação ao comportamento violento: sentido de caracterizá-lo, ou seja, indicando as circunstâncias
1) Crianças que apanharam, foram vítimas de abusos, em que esse processo se desenvolve.
humilhadas ou desprezadas nos primeiros anos de vida.
2) Adolescentes vivendo em famílias que não lhes O advérbio relaciona-se aos verbos da língua, no sentido de
transmitiram valores sociais altruísticos, formação moral e não caracterizar os processos expressos por ele. Contudo, ele não
lhes impuseram limites de disciplina. é modificador exclusivo desta classe (verbos), pois também
3) Associação com grupos de jovens portadores de modifica o adjetivo e até outro advérbio. Seguem alguns
comportamento antissocial. exemplos:

Língua Portuguesa 58
Apostila Digital Licenciada para Alice Caroline Guarino dos Santos - alice.guarino@hotmail.com (Proibida a Revenda)
APOSTILAS OPÇÃO
Para quem se diz distantemente alheio a esse assunto, Superlativo: aumenta a intensidade. Exemplos: longe
você está até bem informado. - longíssimo, pouco - pouquíssimo, inconstitucionalmente -
inconstitucionalissimamente, etc;
Temos o advérbio “distantemente” que modifica o adjetivo
Diminutivo: diminui a intensidade.
alheio, representando uma qualidade, característica.
Exemplos: perto - pertinho, pouco - pouquinho, devagar -
devagarinho,
O artista canta muito mal.
Questões
Nesse caso, o advérbio de intensidade “muito” modifica outro
advérbio de modo – “mal”. Em ambos os exemplos pudemos
01. Leia os quadrinhos para responder a questão.
verificar que se tratava de somente uma palavra funcionando
como advérbio. No entanto, ele pode estar demarcado por
mais de uma palavra, que mesmo assim não deixará de ocupar
tal função. Temos aí o que chamamos de locução adverbial,
representada por algumas expressões, tais como: às vezes, sem
dúvida, frente a frente, de modo algum, entre outras.

Mediante tais postulados, afirma-se que, dependendo das


circunstâncias expressas pelos advérbios, eles se classificam em
distintas categorias, uma vez expressas por:    
de modo: Bem, mal, assim, depressa, devagar, às pressas, às
claras, às cegas, à toa, à vontade, às escondidas, aos poucos, desse
jeito, desse modo, dessa maneira, em geral, frente a frente, lado
a lado, a pé, de cor, em vão, e a maior parte dos que terminam
em -mente: calmamente, tristemente, propositadamente,
pacientemente, amorosamente, docemente, escandalosamente,
bondosamente, generosamente
de intensidade: Muito, demais, pouco, tão, menos, em
excesso, bastante, pouco, mais, menos, demasiado, quanto, quão,
tanto, que(equivale a quão), tudo, nada, todo, quase, de todo, de
muito, por completo.
de tempo: Hoje, logo, primeiro, ontem, tarde outrora,
amanhã, cedo, dantes, depois, ainda, antigamente, antes,
doravante, nunca, então, ora, jamais, agora, sempre, já, enfim,
afinal, breve, constantemente, entrementes, imediatamente,
primeiramente, provisoriamente, sucessivamente, às vezes,
à tarde, à noite, de manhã, de repente, de vez em quando, de
quando em quando, a qualquer momento, de tempos em tempos,
em breve, hoje em dia (Leila Lauar Sarmento e Douglas Tufano. Português. Volume
de lugar: Aqui, antes, dentro, ali, adiante, fora, acolá, atrás, Único)
além, lá, detrás, aquém, cá, acima, onde, perto, aí, abaixo, aonde,
longe, debaixo, algures, defronte, nenhures, adentro, afora, No primeiro e segundo quadrinhos, estão em destaque dois
alhures, nenhures, aquém, embaixo, externamente, a distância, advérbios: AÍ e ainda.
à distância de, de longe, de perto, em cima, à direita, à esquerda, Considerando que advérbio é a palavra que modifica
ao lado, em volta um verbo, um outro advérbio ou um adjetivo, expressando
de negação : Não, nem, nunca, jamais, de modo algum, de a circunstância em que determinado fato ocorre, assinale
forma nenhuma, tampouco, de jeito nenhum a alternativa que classifica, correta e respectivamente, as
de dúvida: Acaso, porventura, possivelmente, circunstâncias expressas por eles.
provavelmente, quiçá, talvez, casualmente, por certo, quem sabe A) Lugar e negação.
de afirmação: Sim, certamente, realmente, decerto, B) Lugar e tempo.
efetivamente, certo, decididamente, realmente, deveras, C) Modo e afirmação.
indubitavelmente D) Tempo e tempo.
de exclusão: Apenas, exclusivamente, salvo, senão, somente, E) Intensidade e dúvida.
simplesmente, só, unicamente
de inclusão: Ainda, até, mesmo, inclusivamente, também 02. Leia o texto a seguir.
de ordem: Depois, primeiramente, ultimamente
de designação: Eis Impunidade é motor de nova onda de agressões
de interrogação: onde?(lugar), como?(modo),
quando?(tempo), por quê?(causa), quanto?(preço e intensidade), Repetidos episódios de violência têm sido noticiados nas
para quê?(finalidade) últimas semanas. Dois que chamam a atenção, pela banalidade
com que foram cometidos, estão gerando ainda uma série de
Locução adverbial repercussões.
É reunião de duas ou mais palavras com valor de advérbio. Em Natal, um garoto de 19 anos quebrou o braço da
Exemplo: estudante de direito R.D., 19, em plena balada, porque ela teria
Carlos saiu às pressas. (indicando modo) recusado um beijo. O suposto agressor já responde a uma ação
Maria saiu à tarde. (indicando tempo) penal, por agressão, movida por sua ex-mulher.
No mesmo final de semana, dois amigos que saíam de uma
Há locuções adverbiais que possuem advérbios boate em São Paulo também foram atacados por dois jovens
correspondentes. que estavam na mesma balada, e um dos agredidos teve a perna
Exemplo: fraturada. Esses dois jovens teriam tentado se aproximar, sem
Carlos saiu às pressas. = Carlos saiu apressadamente. sucesso, de duas garotas que eram amigas dos rapazes que
saíam da boate. Um dos suspeitos do ataque alega que tudo não
Apenas os advérbios de intensidade, de lugar e de modo são passou de um engano e que o rapaz teria fraturado a perna ao
flexionados, sendo que os demais são todos invariáveis. A única cair no chão.
flexão propriamente dita que existe na categoria dos advérbios Curiosamente, também é possível achar um blog que diz
é a de grau: que R.D., em Natal, foi quem atacou o jovem e que seu braço se
quebrou ao cair no chão.

Língua Portuguesa 59
Apostila Digital Licenciada para Alice Caroline Guarino dos Santos - alice.guarino@hotmail.com (Proibida a Revenda)
APOSTILAS OPÇÃO
Em ambos os casos, as câmeras dos estabelecimentos Os advérbios em destaque nos trechos expressam, correta e
felizmente comprovam os acontecimentos, e testemunhas vão respectivamente, circunstâncias de
ajudar a polícia na investigação. A) afirmação e de intensidade.
O fato é que é difícil acreditar que tanta gente ande se B) modo e de tempo.
quebrando por aí ao cair no chão, não é mesmo? As agressões C) modo e de lugar.
devem ser rigorosamente apuradas e, se houver culpados, que D) lugar e de tempo.
eles sejam julgados e condenados. E) intensidade e de negação.
A impunidade é um dos motores da onda de violência que
temos visto. O machismo e o preconceito são outros. O perfil Respostas
impulsivo de alguns jovens (amplificado pela bebida e por 1-B / 2-C / 3-B
outras substâncias) completa o mecanismo que gera agressões.
Sem interferir nesses elementos, a situação não vai mudar. Interjeição
Maior rigor da justiça, educação para a convivência com o outro,
aumento da tolerância à própria frustração e melhor controle Interjeição é a palavra invariável que exprime emoções,
sobre os impulsos (é normal levar um “não”, gente!) são alguns sensações, estados de espírito, ou que procura agir sobre o
dos caminhos. interlocutor, levando-o a adotar certo comportamento sem que,
(Jairo Bouer, Folha de S.Paulo, 24.10.2011. Adaptado) para isso, seja necessário fazer uso de estruturas linguísticas
mais elaboradas. Observe o exemplo:
Assinale a alternativa cuja expressão em destaque apresenta Droga! Preste atenção quando eu estou falando!
circunstância adverbial de modo. No exemplo acima, o interlocutor está muito bravo. Toda sua
A) Repetidos episódios de violência (...) estão gerando ainda raiva se traduz numa palavra: Droga!
uma série de repercussões.
B) ...quebrou o braço da estudante de direito R. D., 19, em Ele poderia ter dito: - Estou com muita raiva de você! Mas usou
plena balada… simplesmente uma palavra. Ele empregou a interjeição Droga!
C) Esses dois jovens teriam tentado se aproximar, sem As sentenças da língua costumam se organizar de forma
sucesso, de duas amigas… lógica: há uma sintaxe que estrutura seus elementos e os distribui
D) Um dos suspeitos do ataque alega que tudo não passou em posições adequadas a cada um deles. As interjeições, por
de um engano... outro lado, são uma espécie de “palavra-frase”, ou seja, há uma
E) O fato é que é difícil acreditar que tanta gente ande se ideia expressa por uma palavra (ou um conjunto de palavras -
quebrando por aí… locução interjetiva) que poderia ser colocada em termos de uma
sentença.
03. Leia o texto a seguir. Veja os exemplos:
Cultura matemática Bravo! Bis!
Hélio Schwartsman bravo e bis: interjeição / sentença (sugestão): «Foi muito
bom! Repitam!»
SÃO PAULO – Saiu mais um estudo mostrando que o ensino Ai! Ai! Ai! Machuquei meu pé...
de matemática no Brasil não anda bem. A pergunta é: podemos ai: interjeição / sentença (sugestão): “Isso está doendo!” ou
viver sem dominar o básico da matemática? Durante muito “Estou com dor!”
tempo, a resposta foi sim. Aqueles que não simpatizavam muito
com Pitágoras podiam simplesmente escolher carreiras nas A interjeição é um recurso da linguagem afetiva, em que
quais os números não encontravam muito espaço, como direito, não há uma ideia organizada de maneira lógica, como são as
jornalismo, as humanidades e até a medicina de antigamente. sentenças da língua, mas sim a manifestação de um suspiro,
Como observa Steven Pinker, ainda hoje, nos meios um estado da alma decorrente de uma situação particular, um
universitários, é considerado aceitável que um intelectual se momento ou um contexto específico. Exemplos:
vanglorie de ter passado raspando em física e de ignorar o beabá Ah, como eu queria voltar a ser criança!
da estatística. Mas ai de quem admitir nunca ter lido Joyce ou ah: expressão de um estado emotivo = interjeição
dizer que não gosta de Mozart. Sobre ele recairão olhares tão Hum! Esse pudim estava maravilhoso!
recriminadores quanto sobre o sujeito que assoa o nariz na hum: expressão de um pensamento súbito = interjeição
manga da camisa.
Joyce e Mozart são ótimos, mas eles, como quase toda a O significado das interjeições está vinculado à maneira
cultura humanística, têm pouca relevância para nossa vida como elas são proferidas. Desse modo, o tom da fala é que dita
prática. Já a cultura científica, que muitos ainda tratam com uma o sentido que a expressão vai adquirir em cada contexto de
ponta de desprezo, torna-se cada vez mais fundamental, mesmo enunciação. Exemplos:
para quem não pretende ser engenheiro ou seguir carreiras Psiu!
técnicas. contexto: alguém pronunciando essa expressão na rua;
Como sobreviver à era do crédito farto sem saber calcular as significado da interjeição (sugestão): “Estou te chamando! Ei,
armadilhas que uma taxa de juros pode esconder? Hoje, é difícil espere!”
até posicionar-se de forma racional sobre políticas públicas sem Psiu!
assimilar toda a numeralha que idealmente as informa. contexto: alguém pronunciando essa expressão em um
Conhecimentos rudimentares de estatística são pré-requisito hospital; significado da interjeição (sugestão): “Por favor, faça
para compreender as novas pesquisas que trazem informações silêncio!”
relevantes para nossa saúde e bem-estar. Puxa! Ganhei o maior prêmio do sorteio!
A matemática está no centro de algumas das mais intrigantes puxa: interjeição; tom da fala: euforia
especulações cosmológicas da atualidade. Se as equações da Puxa! Hoje não foi meu dia de sorte!
mecânica quântica indicam que existem universos paralelos, puxa: interjeição; tom da fala: decepção
isso basta para que acreditemos neles? Ou, no rastro de Eugene
Wigner, podemos nos perguntar por que a matemática é tão As interjeições cumprem, normalmente, duas funções:
eficaz para exprimir as leis da física. a) Sintetizar uma frase exclamativa, exprimindo alegria,
Releia os trechos apresentados a seguir. tristeza, dor, etc.
- Aqueles que não simpatizavam muito com Pitágoras Você faz o que no Brasil?
podiam simplesmente escolher carreiras nas quais os números Eu? Eu negocio com madeiras.
não encontravam muito espaço... (1.º parágrafo) Ah, deve ser muito interessante.
- Já a cultura científica, que muitos ainda tratam com uma b) Sintetizar uma frase apelativa
ponta de desprezo, torna-se cada vez mais fundamental...(3.º Cuidado! Saia da minha frente.
parágrafo) As interjeições podem ser formadas por:

Língua Portuguesa 60
Apostila Digital Licenciada para Alice Caroline Guarino dos Santos - alice.guarino@hotmail.com (Proibida a Revenda)
APOSTILAS OPÇÃO
a) simples sons vocálicos: Oh!, Ah!, Ó, Ô. Fora! Francamente! (Advérbios)
b) palavras: Oba!, Olá!, Claro!
c) grupos de palavras (locuções interjetivas): Meu Deus!, Ora 3) A interjeição pode ser considerada uma “palavra-frase”
bolas! porque sozinha pode constituir uma mensagem.
A ideia expressa pela interjeição depende muitas vezes Socorro!
da entonação com que é pronunciada; por isso, pode ocorrer que Ajudem-me!
uma interjeição tenha mais de um sentido. Por exemplo: Silêncio!
Oh! Que surpresa desagradável! (ideia de contrariedade) Fique quieto!
Oh! Que bom te encontrar. (ideia de alegria)
4) Há, também, as interjeições onomatopaicas ou imitativas,
Classificação das Interjeições
que exprimem ruídos e vozes.
Pum! Miau! Bumba! Zás! Plaft! Pof!
Comumente, as interjeições expressam sentido de:
Catapimba! Tique-taque! Quá-quá-quá!, etc.
- Advertência: Cuidado!, Devagar!, Calma!, Sentido!,
Atenção!, Olha!, Alerta!
5) Não se deve confundir a interjeição de apelo “ó” com a sua
- Afugentamento: Fora!, Passa!, Rua!, Xô!
homônima “oh!”, que exprime admiração, alegria, tristeza, etc.
- Alegria ou Satisfação: Oh!, Ah!,Eh!, Oba!, Viva!
Faz-se uma pausa depois do” oh!” exclamativo e não a fazemos
- Alívio: Arre!, Uf!, Ufa! Ah!
depois do “ó” vocativo.
- Animação ou Estímulo: Vamos!, Força!, Coragem!, Eia!,
Ânimo!, Adiante!, Firme!, Toca!
“Ó natureza! ó mãe piedosa e pura!» (Olavo Bilac)
- Aplauso ou Aprovação: Bravo!, Bis!, Apoiado!, Viva!, Boa!
Oh! a jornada negra!» (Olavo Bilac)
- Concordância: Claro!, Sim!, Pois não!, Tá!, Hã-hã!
6) Na linguagem afetiva, certas interjeições, originadas
- Repulsa ou Desaprovação: Credo!, Irra!, Ih!, Livra!, Safa!,
de palavras de outras classes, podem aparecer flexionadas no
Fora!, Abaixo!, Francamente!, Xi!, Chega!, Basta!, Ora!
diminutivo ou no superlativo.
- Desejo ou Intenção: Oh!, Pudera!, Tomara!, Oxalá!
Calminha! Adeusinho! Obrigadinho!
- Desculpa: Perdão!
Interjeições, leitura e produção de textos
- Dor ou Tristeza: Ai!, Ui!, Ai de mim!, Que pena!, Ah!, Oh!,
Eh!
Usadas com muita frequência na língua falada informal,
- Dúvida ou Incredulidade: Qual!, Qual o quê!, Hum!, Epa!,
quando empregadas na língua escrita, as interjeições costumam
Ora!
conferir-lhe certo tom inconfundível de coloquialidade. Além
- Espanto ou Admiração: Oh!, Ah!, Uai!, Puxa!, Céus!, Quê!,
disso, elas podem muitas vezes indicar traços pessoais do falante
Caramba!, Opa!, Virgem!, Vixe!, Nossa!, Hem?!, Hein?, Cruz!, Putz!
- como a escassez de vocabulário, o temperamento agressivo ou
- Impaciência ou Contrariedade: Hum!, Hem!, Irra!, Raios!,
dócil, até mesmo a origem geográfica. É nos textos narrativos -
Diabo!, Puxa!, Pô!, Ora!
particularmente nos diálogos - que comumente se faz uso
- Pedido de Auxílio: Socorro!, Aqui!, Piedade!
das interjeições com o objetivo de caracterizar personagens
- Saudação, Chamamento ou Invocação: Salve!, Viva!,
e, também, graças à sua natureza sintética, agilizar as falas.
Adeus!, Olá!, Alô!, Ei!, Tchau!, Ô, Ó, Psiu!, Socorro!, Valha-me,
Natureza sintética e conteúdo mais emocional do que
Deus!
racional fazem das interjeições presença constante nos textos
- Silêncio: Psiu!, Bico!, Silêncio!
publicitários.
- Terror ou Medo: Credo!, Cruzes!, Uh!, Ui!, Oh!
Fonte: http://www.soportugues.com.br/secoes/morf/
Saiba que: As interjeições são palavras invariáveis, isto é, morf89.php
não sofrem variação em gênero, número e grau como os nomes,
nem de número, pessoa, tempo, modo, aspecto e voz como os Numeral
verbos. No entanto, em uso específico, algumas interjeições
sofrem variação em grau. Deve-se ter claro, neste caso, que Numeral é a palavra que indica os seres em termos
não se trata de um processo natural dessa classe de palavra, numéricos, isto é, que atribui quantidade aos seres ou os situa
mas tão só uma variação que a linguagem afetiva permite. em determinada sequência.
Exemplos: oizinho, bravíssimo, até loguinho. Os quatro últimos ingressos foram vendidos há pouco.
[quatro: numeral = atributo numérico de “ingresso”]
Locução Interjetiva
Eu quero café duplo, e você?
[duplo: numeral = atributo numérico de “café”]
Ocorre quando duas ou mais palavras formam uma
A primeira pessoa da fila pode entrar, por favor!
expressão com sentido de interjeição. Por exemplo
[primeira: numeral = situa o ser “pessoa” na sequência de
Ora bolas!
“fila”]
Quem me dera!
Virgem Maria!
Note bem: os numerais traduzem, em palavras, o que
Meu Deus!
os números indicam em relação aos seres. Assim, quando a
Ai de mim!
expressão é colocada em números (1, 1°, 1/3, etc.) não se trata
Valha-me Deus!
de numerais, mas sim de algarismos.
Graças a Deus!
Além dos numerais mais conhecidos, já que refletem a
Alto lá!
ideia expressa pelos números, existem mais algumas palavras
Muito bem!
consideradas numerais porque denotam quantidade, proporção
ou ordenação. São alguns exemplos: década, dúzia, par,
Observações:
ambos(as), novena.
1) As interjeições são como frases resumidas, sintéticas. Por
Classificação dos Numerais
exemplo:
Ué! = Eu não esperava por essa!
Cardinais: indicam contagem, medida. É o número básico:
Perdão! = Peço-lhe que me desculpe.
um, dois, cem mil, etc.
Ordinais: indicam a ordem ou lugar do ser numa série dada:
2) Além do contexto, o que caracteriza a interjeição é o seu
primeiro, segundo, centésimo, etc.
tom exclamativo; por isso, palavras de outras classes gramaticais
Fracionários: indicam parte de um inteiro, ou seja, a divisão
podem aparecer como interjeições.
dos seres: meio, terço, dois quintos, etc.
Viva! Basta! (Verbos)
Multiplicativos: expressam ideia de multiplicação dos

Língua Portuguesa 61
Apostila Digital Licenciada para Alice Caroline Guarino dos Santos - alice.guarino@hotmail.com (Proibida a Revenda)
APOSTILAS OPÇÃO
seres, indicando quantas vezes a quantidade foi aumentada: da solidariedade. Ambos agora participam das atividades
dobro, triplo, quíntuplo, etc. comunitárias de seu bairro.
Obs.: a forma “ambos os dois” é considerada enfática.
Leitura dos Numerais Atualmente, seu uso indica afetação, artificialismo.

Separando os números em centenas, de trás para frente, Cardinais Ordinais Multiplicativos Fracionários
obtêm-se conjuntos numéricos, em forma de centenas e, no um primeiro - -
início, também de dezenas ou unidades. Entre esses conjuntos dois segundo dobro, duplo meio
usa-se vírgula; as unidades ligam-se pela conjunção “e”. três terceiro triplo, tríplice terço
quatro quarto quádruplo quarto
1.203.726 = um milhão, duzentos e três mil, setecentos e vinte cinco quinto quíntuplo quinto
e seis. seis sexto sêxtuplo sexto
sete sétimo sétuplo sétimo
45.520 = quarenta e cinco mil, quinhentos e vinte. oito oitavo óctuplo oitavo
nove nono nônuplo nono
Flexão dos numerais dez décimo décuplo décimo
onze décimo primeiro - onze avos
Os numerais cardinais que variam em gênero são um/uma, doze décimo segundo - doze avos
dois/duas e os que indicam centenas de duzentos/duzentas em treze décimo terceiro - treze avos
diante: trezentos/trezentas; quatrocentos/quatrocentas, etc. catorze décimo quarto - catorze avos
Cardinais como milhão, bilhão, trilhão, variam em número: quinze décimo quinto - quinze avos
milhões, bilhões, trilhões. Os demais cardinais são invariáveis. dezesseis décimo sexto - dezesseis avos
dezessete décimo sétimo - dezessete avos
Os numerais ordinais variam em gênero e número: dezoito décimo oitavo - dezoito avos
primeiro segundo milésimo dezenove décimo nono - dezenove avos
primeira segunda milésima vinte vigésimo - vinte avos
primeiros segundos milésimos trinta trigésimo - trinta avos
primeiras segundas milésimas quarenta quadragésimo - quarenta avos
cinquenta quinquagésimo - cinquenta avos
Os numerais multiplicativos são invariáveis quando atuam sessenta sexagésimo - sessenta avos
em funções substantivas: setenta septuagésimo - setenta avos
Fizeram o dobro do esforço e conseguiram o triplo de produção. oitenta octogésimo - oitenta avos
Quando atuam em funções adjetivas, esses numerais noventa nonagésimo - noventa avos
flexionam-se em gênero e número: cem centésimo cêntuplo centésimo
Teve de tomar doses triplas do medicamento. duzentos ducentésimo - ducentésimo
Os numerais fracionários flexionam-se em gênero e número. trezentos trecentésimo - trecentésimo
Observe: um terço/dois terços, uma terça parte/duas terças quatrocentos quadringentésimo - quadringentésimo
partes quinhentos quingentésimo - quingentésimo
Os numerais coletivos flexionam-se em número. Veja: uma seiscentos sexcentésimo - sexcentésimo
dúzia, um milheiro, duas dúzias, dois milheiros. setecentos septingentésimo - septingentésimo
oitocentos octingentésimo - octingentésimo
É comum na linguagem coloquial a indicação de grau nos novecentos nongentésimo
numerais, traduzindo afetividade ou especialização de sentido. ou noningentésimo - nongentésimo
É o que ocorre em frases como: mil milésimo - milésimo
milhão milionésimo - milionésimo
“Me empresta duzentinho...” bilhão bilionésimo - bilionésimo
É artigo de primeiríssima qualidade!
O time está arriscado por ter caído na segundona. (= segunda Questões
divisão de futebol)
01.Na frase “Nessa carteira só há duas notas de cinco reais”
Emprego dos Numerais temos exemplos de numerais:
A) ordinais;
*Para designar papas, reis, imperadores, séculos e partes em B) cardinais;
que se divide uma obra, utilizam-se os ordinais até décimo e a C) fracionários;
partir daí os cardinais, desde que o numeral venha depois do D) romanos;
substantivo: E) Nenhuma das alternativas.

Ordinais Cardinais 02.Aponte a alternativa em que os numerais estão bem


João Paulo II (segundo) Tomo XV (quinze) empregados.
D. Pedro II (segundo) Luís XVI (dezesseis) A) Ao papa Paulo Seis sucedeu João Paulo Primeiro.
Ato II (segundo) Capítulo XX (vinte) B) Após o parágrafo nono virá o parágrafo décimo.
Século VIII (oitavo) Século XX (vinte) C) Depois do capítulo sexto, li o capitulo décimo primeiro.
Canto IX (nono) João XXIII ( vinte e três) D) Antes do artigo dez vem o artigo nono.
E) O artigo vigésimo segundo foi revogado.
*Para designar leis, decretos e portarias, utiliza-se o ordinal
até nono e o cardinal de dez em diante: 03. Os ordinais referentes aos números 80, 300, 700 e 90
Artigo 1.° (primeiro) Artigo 10 (dez) são, respectivamente
Artigo 9.° (nono) Artigo 21 (vinte e um) A) octagésimo, trecentésimo, septingentésirno,
nongentésimo
*Ambos/ambas são considerados numerais. Significam “um B) octogésimo, trecentésimo, septingentésimo, nonagésimo
e outro”, “os dois” (ou “uma e outra”, “as duas”) e são largamente C) octingentésimo, tricentésimo, septuagésimo, nonagésimo
empregados para retomar pares de seres aos quais já se fez D) octogésimo, tricentésimo, septuagésimo, nongentésimo
referência.
Respostas
Pedro e João parecem ter finalmente percebido a importância 1-B / 2-D / 3-B

Língua Portuguesa 62
Apostila Digital Licenciada para Alice Caroline Guarino dos Santos - alice.guarino@hotmail.com (Proibida a Revenda)
ATUALIDADES E DEVERES DOS
SERVIDORES PÚBLICOS

Apostila Digital Licenciada para Alice Caroline Guarino dos Santos - alice.guarino@hotmail.com (Proibida a Revenda)
Apostila Digital Licenciada para Alice Caroline Guarino dos Santos - alice.guarino@hotmail.com (Proibida a Revenda)
APOSTILAS OPÇÃO

constituição federal. Nos estados da federação e no distrito


federal, o poder executivo é exercido pelos governadores e
seus secretários, com atribuições e responsabilidades
controlados pela constituição estadual. Nos municípios, os
representantes do poder executivo são os prefeitos e seus
secretários, que também possuem atribuições e
responsabilidades, definidas na lei orgânica de cada
município.
O presidente, governadores e prefeitos são eleitos através
de sufrágio (voto) universal. O eleitor tem o direito de escolher
aquele que melhor se encaixa em sua visão política. Todos os
candidatos devem ser filiados a um partido político e, quando
1. Questões relacionadas a fatos eleitos, possuem mandato com tempo determinado. No Brasil
políticos, econômicos e sociais e as funções de presidente, governador e prefeito possuem
culturais, nacionais e duração de 4 anos cada, com a possibilidade de reeleição.
internacionais, ocorridos a partir do Durante suas campanhas os candidatos discutem seus
programas de governo e os rumos que pretendem dar ao país.
2º semestre de 2016, divulgados na
Existem punições ao presidente da república em caso de
mídia local e/ou nacional. crime de responsabilidade, como previsto na constituição
federal, além de punição para infrações penais comuns. Para
ser submetido a julgamento o presidente precisa ter acusação
admitida por pelo menos dois terços da Câmara dos
O presente material tem por objetivo apresentar os Deputados. Nos casos de infrações penais ele é julgado pelo
principais fatos ocorridos e amplamente divulgados a partir Supremo Tribunal Federal e em caso de crimes de
segundo semestre de 2016 e primeiro semestre de 2017. responsabilidade é julgado pelo Senado Federal.
Tratam-se de assuntos relacionados as mais diversas áreas, em Entre as principais funções do presidente da república
conformidade com o edital. É importante lembrar que alguns estão a execução de leis e expedição de decretos e
dos temas aqui abordados ocorreram em 2016, porém tiveram regulamentos; prover cargos e funções públicas; promover a
início muito antes disso, e podem também não ter sido administração e a segurança públicas; emitir moeda; elaborar
concluídos, como é o caso da Operação Lava-Jato, que teve o orçamento e os planos de desenvolvimento econômico e
início no ano de 2014, continuou durante o ano de 2015 e social nos níveis nacional, regional e setoriais; exercer o
alcançou o ano de 2016, e este adentrando o ano de 2017 comando supremo das forças armadas; e manter relações com
É importante destacar que por conta do grande volume de estados estrangeiros.
informações diariamente produzidas pelos mais diversos Além das funções executivas, o presidente conta ainda, em
meios de comunicação (televisão, jornal, rádio, redes sociais), alguns casos, com poder legislativo. O poder pode ser aplicado
alguns temas acabam não sendo abordados ou são abordados em veto a leis aprovadas pelo Congresso Nacional e a edição
de maneira superficial. Diante da grande quantidade de de medidas provisórias com força de lei de aplicação e
conteúdos e temas, é importante atentar e acompanhar os execução imediatas.
meios de comunicação, como telejornais, programas de rádio, Os ministros de estado e auxiliares diretos do presidente
jornais online, sites, redes sociais, blogs, entre outros, para podem ser nomeados ou demitidos livremente por ele. Para
estar a par dos acontecimentos. assumir alguma das funções a pessoa deve ter no mínimo 21
anos de idade, brasileiros natos, e estar no exercício dos
POLÍTICA direitos políticos. Os ministros nomeados pelo presidente são
responsáveis por diversas políticas de governo, em diversos
Sistema Político Brasileiro campos de atuação, como educação, economia, cultura,
O sistema político brasileiro tem base nas ideias finanças e justiça, entre diversos outros. Os ministros podem
iluministas do pensador francês Montesquieu. O pensador ser convocados para justificar seus atos perante a Câmara dos
defendeu a divisão do poder político em Legislativo, Executivo Deputados, o Senado ou qualquer uma de suas comissões para
e Judiciário em sua obra “O Espírito das Leis”. Para ele o poder explicar atos ou programas.
concentrado na mão do rei leva à tirania, então o Estado
deveria dividi-lo em poder executivo (executa as leis, o Poder Legislativo
governo), legislativo (cria as leis, o congresso) e judiciário (que O Poder Legislativo é representado por pessoas que devem
julga e fiscaliza os poderes). elaborar as leis que regulamentam o Estado, conhecidos por
No Brasil o voto é universal, ou seja, todo cidadão com a legisladores. Na maioria das repúblicas e monarquias o poder
idade mínima de 16 anos pode participar do processo político legislativo é formado por um congresso, parlamento,
e eleger seus representantes. O país é uma república federativa assembleia ou câmara.
presidencialista, onde o Chefe de Estado, no caso o presidente, Seu objetivo é elaborar normas de abrangência geral ou em
é eleito através do voto direto da população e os estados raros casos individual, que são estabelecidas aos cidadãos ou
possuem autonomia política, com a possibilidade de criar leis às instituições públicas nas suas relações recíprocas.
específicas. Entre as principais funções do poder legislativo estão a de
Assim como na obra de Montesquieu o país possui a fiscalizar o Poder Executivo, votar leis orçamentárias e, em
divisão do poder entre Executivo, representado pelo situações específicas, julgar determinadas pessoas, como
presidente da república, Legislativo, que é representado pelo o Presidente da república ou os próprios membros do
congresso nacional e Judiciário que é representado pelo legislativo.
Supremo Tribunal Federal. No Brasil, o Poder legislativo é exercido em âmbito federal,
estadual e municipal. O Congresso Nacional é formados pela
Poder Executivo Câmara dos Deputados e o Senado Federal e é responsável
O poder executivo é compreendido pelo presidente da pelo Poder Legislativo federal. Possui a função de elaborar e
república e seus ministros de Estado no sistema federativo aprovar as leis do país, e também controlar os atos do
brasileiro, com atribuições e responsabilidades definidos pela executivo e impedir abusos pela fiscalização permanente. Nos

Atualidades e Deveres dos Servidores Públicos 1


Apostila Digital Licenciada para Alice Caroline Guarino dos Santos - alice.guarino@hotmail.com (Proibida a Revenda)
APOSTILAS OPÇÃO

estados é exercido pelas assembleias legislativas e nos Segundo a Infraero, a aeronave decolou às 13h01 do
municípios pelas câmaras municipais, ou de vereadores Campo de Marte, em São Paulo, com destino a Paraty, e caiu
próximo à Ilha Rasa, a 2 km de distância da cabeceira da pista
Poder Judiciário do aeroporto da cidade fluminense. O acidente ocorreu por
O Poder Judiciário é exercido pelos juízes e possui a volta das 13h45.
capacidade e a prerrogativa de julgar, de acordo com as regras Ainda não está totalmente claro o que ocorreu. Chovia
constitucionais e leis criadas pelo poder legislativo em bastante no momento do queda, segundo imagens de radar. O
determinado país. mau tempo é um fator que pode comprometer a aproximação
No Brasil, o judiciário não depende dos demais poderes do aeroporto de Paraty, em que as aterrissagens só podem
nem possui controles externos de fiscalização. Sua função é a acontecer em condição visual.
de aplicar a lei a fatos particulares e, por atribuição e Testemunhas disseram que não houve explosão. Uma delas
competência, declarar o direito e administrar justiça. Além afirmou ter visto o avião voando baixo ao fazer uma curva e
disso, pode resolver os conflitos que podem surgir na batendo uma das asas no mar.
sociedade e tomar decisões com base na constituição, nas leis, A Agência Nacional de Aviação Civil (Anac) informou que a
nas normas e nos costumes, que adapta a situações específicas. documentação da aeronave estava regular. O certificado era
O poder judiciário possui a divisão entre a União(Federal) válido até abril de 2022, e inspeção da manutenção (anual)
e os estados, com a denominação de justiça federal e justiça estava válida até abril de 2017.
estadual, respectivamente. O piloto Osmar Rodrigues, de 56 anos, era conhecido por
Entre os órgãos que formam o poder Judiciário estão o ser "muito cuidadoso" e chegou a dar palestra para outros
Supremo Tribunal Federal (STF), Superior Tribunal de Justiça pilotos sobre como fazer a rota São Paulo-Paraty, segundo
(STJ), além dos Tribunais Regionais Federais (TRF), Tribunais informações do Bom Dia Brasil.
e Juízes do Trabalho, Tribunais e Juízes Eleitorais, Tribunais e
Juízes Militares e os Tribunais e Juízes dos estados e do Distrito Investigações
Federal e Territórios. A apuração das razões técnicas que contribuíram para o
O STF é o órgão máximo do Judiciário brasileiro. Sua acidente, como a influência do mau tempo, da aeronave e do
principal função é zelar pelo cumprimento da Constituição e piloto, ficam a cargo do Cenipa, que esteve no local da queda
dar a palavra final nas questões que envolvam normas na quinta-feira. Ministério Público Federal (MPF) e Polícia
constitucionais. É composto por 11 ministros indicados pelo Federal (PF) irão apurar se houve eventual intenção
Presidente da República e nomeados por ele após aprovação deliberada de derrubar o avião.
pelo Senado Federal. O MPF de Angra dos Reis, no litoral sul do Rio de Janeiro,
Os juízes que atuam em tribunais superiores são abriu inquérito a respeito. A responsável é a procuradora da
nomeados pelo presidente da república, porem precisam de República Cristina Nascimento de Melo.
aprovação do Senado. Outros cargos são preenchidos através Na PF, o inquérito está sob responsabilidade do delegado
de concurso público. Os juízes têm cargo vitalício, não podem chefe da corporação em Angra, Adriano Antonio Soares. O
ser removidos e seus vencimentos não podem ser reduzidos. policial aguarda a chegada em Angra de um grupo da PF de
Brasília, especializado em acidentes aéreos.
Bombeiros retiram todos os corpos do local do
acidente que matou Teori1 Operação Lava Jato2
Foram resgatados os dois últimos corpos do acidente O nome do caso, “Lava Jato”, decorre do uso de uma rede
envolvendo o avião onde estava o ministro Teori Zavascki, em de postos de combustíveis e lava a jato de automóveis para
Paraty, Costa Verde do Rio de Janeiro. Trata-se do piloto do movimentar recursos ilícitos pertencentes a uma das
avião, Osmar Rodrigues, e de uma mulher que estava presa nas organizações criminosas inicialmente investigadas. Embora a
ferragens da aeronave. investigação tenha avançado para outras organizações
Cinco pessoas morreram na queda do avião no mar nesta criminosas, o nome inicial se consagrou.
quinta-feira (19): A operação Lava Jato é a maior investigação de corrupção
- Teori Zavascki, ministro do Supremo Tribunal Federal e lavagem de dinheiro que o Brasil já teve. Estima-se que o
- Carlos Alberto Fernandes Filgueiras, empresário volume de recursos desviados dos cofres da Petrobras, maior
- Osmar Rodrigues, piloto do avião estatal do país, esteja na casa de bilhões de reais. Soma-se a
- Maira Lidiane Panas Helatczuk, massoterapeuta de isso a expressão econômica e política dos suspeitos de
Filgueiras participar do esquema de corrupção que envolve a companhia.
- Maria Ilda Panas, mãe de Maira No primeiro momento da investigação, desenvolvido a
As identidades das duas mulheres foram confirmadas pelo partir de março de 2014, perante a Justiça Federal em Curitiba,
Hotel Emiliano, o proprietário do avião, nesta sexta-feira (20). foram investigadas e processadas quatro organizações
Equipes já tinham recuperado os corpos de Teori, do criminosas lideradas por doleiros, que são operadores do
empresário e de uma das mulheres. Eles chegaram no início mercado paralelo de câmbio. Depois, o Ministério Público
desta madrugada ao Instituto Médico Legal (IML) de Angra dos Federal recolheu provas de um imenso esquema criminoso de
Reis (RJ) para perícia. As duas últimas vítimas foram levadas corrupção envolvendo a Petrobras.
para um centro de apoio montado em uma marina às margens Nesse esquema, que dura pelo menos dez anos, grandes
da rodovia Rio-Santos e posteriormente encaminhadas ao IML. empreiteiras organizadas em cartel pagavam propina para
Homens da Marinha, Aeronáutica e Bombeiros altos executivos da estatal e outros agentes públicos. O valor
trabalharam nas buscas dos corpos. Os militares realizaram da propina variava de 1% a 5% do montante total de contratos
uma operação para estabilizar a aeronave, que tinha parte bilionários superfaturados. Esse suborno era distribuído por
submersa. meio de operadores financeiros do esquema, incluindo
O avião prefixo PR-SOM era um modelo Hawker Beechcraft doleiros investigados na primeira etapa.
King Air C90 e pertencia ao grupo Emiliano Empreendimentos. As empreiteiras - Em um cenário normal, empreiteiras
De pequeno porte, tinha capacidade para oito pessoas. concorreriam entre si, em licitações, para conseguir os
contratos da Petrobras, e a estatal contrataria a empresa que

1 20/01/2017. Fonte: http://g1.globo.com/rj/sul-do-rio-costa- 2 http://lavajato.mpf.mp.br/entenda-o-caso


verde/noticia/2017/01/bombeiros-retiram-todos-os-corpos-do-local-do-
acidente-que-matou-teori.html

Atualidades e Deveres dos Servidores Públicos 2


Apostila Digital Licenciada para Alice Caroline Guarino dos Santos - alice.guarino@hotmail.com (Proibida a Revenda)
APOSTILAS OPÇÃO

aceitasse fazer a obra pelo menor preço. Neste caso, as serão concedidos e as condições para que a cooperação seja
empreiteiras se cartelizaram em um “clube” para substituir premiada.
uma concorrência real por uma concorrência aparente. Os A lei brasileira que detalhou como funciona a colaboração
preços oferecidos à Petrobras eram calculados e ajustados em premiada é chamada Lei de Combate às Organizações
reuniões secretas nas quais se definia quem ganharia o Criminosas (Lei 12.850/2013). Embora não houvesse previsão
contrato e qual seria o preço, inflado em benefício privado e expressa de acordos de colaboração entre o criminoso e o
em prejuízo dos cofres da estatal. O cartel tinha até um Ministério Público antes da lei, eles já vinham sendo feitos
regulamento, que simulava regras de um campeonato de desde a força-tarefa do caso Banestado (entre 2003 e 2007).
futebol, para definir como as obras seriam distribuídas. Para Em cada acordo, muitas variáveis são consideradas, tais
disfarçar o crime, o registro escrito da distribuição de obras como informações novas sobre crimes e quem são os seus
era feito, por vezes, como se fosse a distribuição de prêmios de autores, provas que serão disponibilizadas, importância dos
um bingo. fatos e das provas prometidas no contexto da investigação,
Funcionários da Petrobras - As empresas precisavam recuperação do proveito econômico auferido com os crimes,
garantir que apenas aquelas do cartel fossem convidadas para perspectiva de resultado positivo dos processos e das
as licitações. Por isso, era conveniente cooptar agentes punições sem a colaboração, entre outras. Há uma criteriosa
públicos. Os funcionários não só se omitiam em relação ao análise de custos e benefícios sociais que decorrerão do acordo
cartel, do qual tinham conhecimento, mas o favoreciam, de colaboração sempre por um conjunto de procuradores da
restringindo convidados e incluindo a ganhadora dentre as República, ponderando-se diferentes pontos de vista. O acordo
participantes, em um jogo de cartas marcadas. Segundo é feito apenas quando há concordância de que os benefícios
levantamentos da Petrobras, eram feitas negociações diretas superarão significativamente os custos para a sociedade.
injustificadas, celebravam-se aditivos desnecessários e com
preços excessivos, aceleravam-se contratações com supressão PROCESSO DE IMPEACHMENT DE DILMA3
de etapas relevantes e vazavam informações sigilosas, dentre Às 13h34 desta quarta-feira (31/08/16), Dilma Rousseff
outras irregularidades. (PT) sofreu impeachment e encerrou seu mandato frente à
Operadores financeiros - Os operadores financeiros ou Presidência da República. Em discurso após a votação no
intermediários eram responsáveis não só por intermediar o Senado, Dilma disse que sofreu um segundo golpe e prometeu
pagamento da propina, mas especialmente por entregar a uma oposição “firme e incansável”. Às 16h49, Michel Temer
propina disfarçada de dinheiro limpo aos beneficiários. Em um (PMDB) deixou a vice-presidência oficialmente e foi
primeiro momento, o dinheiro ia das empreiteiras até o empossado presidente. Mais tarde, na primeira reunião
operador financeiro. Isso acontecia em espécie, por ministerial, respondeu aos opositores, prometendo não levar
movimentação no exterior e por meio de contratos simulados “desaforo para casa”: “golpista é você”.
com empresas de fachada. Num segundo momento, o dinheiro Após 73 horas, o julgamento do impeachment no Senado
ia do operador financeiro até o beneficiário em espécie, por terminou com o veredicto de condenação de Dilma por crime
transferência no exterior ou mediante pagamento de bens. de responsabilidade, pelas "pedaladas fiscais" no Plano Safra e
Agentes políticos - Outra linha da investigação – por ter editado decretos de crédito suplementar sem
correspondente à sua verticalização – começou em março de autorização do Congresso Nacional. Foram 61 a favor e 20
2015, quando o Procurador-Geral da República apresentou ao contrários ao impeachment, sem abstenções. Saiba como
Supremo Tribunal Federal 28 petições para a abertura de votou cada senador. Em uma segunda votação, os senadores
inquéritos criminais destinados a apurar fatos atribuídos a 55 decidiram manter a possibilidade de Dilma disputar novas
pessoas, das quais 49 são titulares de foro por prerrogativa de eleições e assumir cargos na administração pública.
função (“foro privilegiado”). São pessoas que integram ou
estão relacionadas a partidos políticos responsáveis por DISCURSO DE DILMA
indicar e manter os diretores da Petrobras. Elas foram citadas Em seu primeiro pronunciamento, a agora ex-presidente
em colaborações premiadas feitas na 1ª instância mediante Dilma Rousseff afirmou que a decisão é o segundo golpe de
delegação do Procurador-Geral. A primeira instância estado que enfrenta na vida e que os senadores que votaram
investigará os agentes políticos por improbidade, na área cível, pelo seu afastamento definitivo rasgaram a Constituição. Ao
e na área criminal aqueles sem prerrogativa de foro. lado de aliados, como o ex-presidente Luiz Inácio Lula da Silva,
foi enfática: "Ouçam bem: eles pensam que nos venceram, mas
Colaboração Premiada estão enganados. Sei que todos vamos lutar. Haverá contra eles
Acordos de colaboração com investigados e réus a mais firme, incansável e enérgica oposição que um governo
Para incentivar os criminosos a colaborar com a Justiça, golpista pode sofrer.
várias leis trouxeram a possibilidade de se conceder benefícios
àqueles acusados que cooperam com a investigação. Esses POSSE DE TEMER
benefícios podem ser a diminuição da pena, a alteração do Três horas após o afastamento de Dilma Rousseff, Michel
regime de seu cumprimento ou mesmo, em casos excepcionais, Temer foi empossado o novo presidente da República. A
isenção penal. Essa colaboração é extremamente relevante na cerimônia durou apenas 11 minutos. Ao apertar a mão de
investigação de alguns tipos de crime, como por exemplo: no Temer, o presidente do Senado, Renan Calheiros (PMDB-AL),
de organização criminosa, em que é comum a destruição de disse a ele: "Estamos juntos".
provas e ameaças a testemunhas; no de lavagem de dinheiro, Na primeira reunião ministerial do governo, Temer
o qual objetiva justamente ocultar crimes; e no de corrupção, afirmou que agora a cobrança sobre o governo será "muito
feito às escuras e com pacto de silêncio. maior" e rejeitou a acusação de que o impeachment foi um
Há duas formas de colaboração premiada. Na primeira, o golpe. "Golpista é você, que está contra a Constituição",
criminoso revela informações na expectativa de, no futuro, tal afirmou dirigindo-se a Dilma.
cooperação ser tomada em consideração pelo juiz quando da O novo presidente embarca para a China, onde participa,
aplicação da pena. Na segunda, o criminoso entra em acordo nos dias 4 e 5, em Hangzhou, da Cúpula de Líderes do G20,
com o Ministério Público, celebrando, após negociação, um grupo das 20 principais economias do mundo. Temer afirmou
contrato escrito. No contrato são estipulados os benefícios que que vai "revelar aos olhos do mundo que temos estabilidade

3 31/08/2016 – Fonte: http://especiais.g1.globo.com/politica/processo-

de-impeachment-de-dilma/2016/impeachment-de-dilma/

Atualidades e Deveres dos Servidores Públicos 3


Apostila Digital Licenciada para Alice Caroline Guarino dos Santos - alice.guarino@hotmail.com (Proibida a Revenda)
APOSTILAS OPÇÃO

política e segurança jurídica." Durante a ausência, assume dele disse que a prisão aconteceu no apartamento funcional de
provisoriamente a Presidência o deputado Rodrigo Maia Cunha.
(DEM-RJ), atual presidente da Câmara. O deputado cassado embarcou às 15h em um avião da
Polícia Federal (PF) no aeroporto de Brasília com destino a
REPERCUSSÃO E MANIFESTAÇÕES Curitiba, onde ficará preso. O avião chegou ao aeroporto, na
Após a votação final do impeachment, houve protestos a Região Metropolitana de Curitiba, às 16h45. De lá, Cunha
favor e contra Temer pelo país. Na Avenida Paulista, um grupo seguiu para a superintendência da PF.
protestava contra o impeachment, enquanto outro No despacho que determinou a prisão, Moro diz que o
comemorava com bolo e champagne. poder de Cunha para obstruir a Lava Jato "não se esvaziou". O
juiz havia autorizado a PF a entrar na casa de Cunha no Rio de
REPERCUSSÃO INTERNACIONAL Janeiro para prendê-lo.
A rede norte-americana CNN deu grande destaque à Moro é responsável pelas ações da operação Lava Jato na
notícia em seu site e afirmou que a decisão é “um grande revés” 1ª instância. Após Cunha perder o foro privilegiado com a
para Dilma, mas "pode não ser o fim de sua carreira política". cassação do mandato, ocorrida em setembro, o juiz retomou
O argentino “Clarín” afirma que o afastamento de Dilma marca na quinta-feira (13/10/2016) o processo que corria no
“o fim de uma era no Brasil”. O “El País”, da Espanha, chamou Supremo Tribunal Federal (STF).
a atenção para a resistência da ex-presidente, que decidiu Nesta segunda (17/10/2016), Moro havia intimado Cunha
enfrentar o processo até o final, apesar das previsões de que e dado 10 dias para que os advogados protocolassem defesa
seu afastamento seria concretizado. prévia.
Em nota divulgada por seus advogados, Cunha afirmou que
Rodrigo Maia: conheça o perfil do novo presidente da a decisão de Moro que resultou na prisão é "absurda" e "sem
Câmara4 nenhuma motivação".
O deputado federal Rodrigo Maia (DEM-RJ), 46 anos, foi De acordo com o Ministério Público Federal (MPF), em
eleito na madrugada de quinta-feira (14/07/2016), com 285 liberdade, Cunha representa risco à instrução do processo e à
votos, presidente da Câmara dos Deputados. Maia venceu em ordem pública. Além disso, os procuradores argumentaram
segundo turno o deputado Rogério Rosso (PSD-DF), que até que "há possibilidade concreta de fuga em virtude da
então era apontado como candidato favorito do Palácio do disponibilidade de recursos ocultos no exterior" e da dupla
Planalto para o cargo. Rosso somou 170 votos. Outros cinco cidadania. Cunha tem passaporte italiano e teria, segundo o
parlamentares votaram em branco. MPF, patrimônio oculto de cerca de US$ 13 milhões que podem
estar em contas no exterior.
Mandato tampão Para embasar o pedido de prisão do ex-presidente da
Rodrigo Maia ficará à frente da Câmara até fevereiro de Câmarax, a força-tarefa da Operação Lava Jato listou atitudes,
2017. Citando a crise econômica que atinge o país e o que conforme os procuradores, foram adotadas por Cunha
conturbado momento político pelo qual passa o Congresso, o para atrapalhar as investigações.
deputado afirmou que as repúblicas "nunca se consolidam sem Entre elas, a convocação pela CPI da Petrobras da
a força dos parlamentos advogada Beatriz Catta Preta, que atuou como defensora do
lobista e colaborador da Lava Jato Julio Camargo, responsável
Trajetória política pelo depoimento que acusou Cunha de ter recebido propina da
Formado em economia, Rodrigo Maia é deputado federal Petrobras.
pelo Rio de Janeiro há cinco legislaturas. Foi eleito para o
primeiro mandato em 1998. Tentou se eleger prefeito do Rio Aprovada na Câmara, PEC 241 passa a tramitar no
em 2012, tendo Clarissa Garotinho (PR-RJ) como vice. Senado como PEC 556
Maia também ocupou o cargo de secretário de Governo do Aprovada pela Câmara dos Deputados na madrugada desta
Rio de Janeiro (1997-1998) e de secretário de Governo do quarta-feira (26/10/2016), a Proposta de Emenda à
Município do Rio de Janeiro (1996). Antes de chegar ao Constituição (PEC) 241, que estabelece um limite para os
Democratas (DEM), o parlamentar foi filiado ao PFL e ao PTB. gastos públicos pelos próximos 20 anos, recebeu nova
Maia assumiu a presidência nacional do DEM, partido que numeração ao passar a tramitar no Senado, sob a numeração
ajudou a criar, em 2007. de PEC 55.
Filho do ex-prefeito do Rio de Janeiro Cesar Maia (DEM), A mudança na numeração da PEC não implica
Rodrigo Maia integra um bloco informal dos chamados necessariamente em uma mudança no conteúdo da proposta
governistas independentes. Além do DEM, compõem o grupo o (os senadores ainda analisarão o texto e poderão propor
PSDB, o PSB e o PPS. alterações). De acordo com a Secretaria-Geral da Mesa, a
modificação ocorre para organizar o sistema do Senado.
Eduardo Cunha é preso em Brasília por decisão de Tida como prioridade pelo governo do presidente Michel
Sérgio Moro5 Temer para reequilibrar as contas públicas, a PEC foi aprovada
O ex-presidente da Câmara e deputado cassado Eduardo pela Câmara em segundo turno, por 359 votos a 116 (e 2
Cunha (PMDB-RJ) foi preso nesta quarta-feira (19/10/2016), abstenções) e, agora, passará a ser analisada pelo Senado.
em Brasília. A prisão dele é preventiva, ou seja, por tempo Mais cedo, nesta quarta, o presidente da Câmara, Rodrigo
indeterminado. A decisão foi do juiz Sérgio Moro no processo Maia (DEM-RJ), entregou o texto da PEC ao presidente do
em que Cunha é acusado de receber propina de contrato de Senado, Renan Calheiros (PMDB-AL).
exploração de Petróleo no Benin, na África, e de usar contas na A proposta já foi remetida à Comissão de Constituição e
Suíça para lavar o dinheiro. Justiça do Senado e, posteriormente (veja o calendário
A Polícia Federal (PF) informou que o ex-presidente da completo ao final desta reportagem), será analisada, em dois
Câmara foi preso na garagem de um edifício. Já o advogado turnos, pelo plenário.

4 14/07/2016 - Fonte: 5 19/10/2016 Fonte:


http://agenciabrasil.ebc.com.br/politica/noticia/2016-07/rodrigo-maia- http://g1.globo.com/pr/parana/noticia/2016/10/juiz-federal-sergio-moro-
conheca-o-perfil-do-novo-presidente-da-camara determina-prisao-de-eduardo-cunha.html
6 http://g1.globo.com/politica/noticia/2016/10/aprovada-na-camara-
pec-241-vira-pec-55-no-senado.html

Atualidades e Deveres dos Servidores Públicos 4


Apostila Digital Licenciada para Alice Caroline Guarino dos Santos - alice.guarino@hotmail.com (Proibida a Revenda)
APOSTILAS OPÇÃO

Relatoria greve e para que a paralisação, que gera sacrifício à população


Ao fazer a leitura da PEC em plenário, nesta quarta, Renan não seja adotada pelos servidores sem maiores
Calheiros sugeriu o nome de Eunício Oliveira (CE), líder do consequências”, afirmou Roberto Barroso.
PMDB no Senado, para relatar da proposta. Cabe ao presidente O ministro Gilmar lembrou que, em quase todos os países,
da CCJ, José Maranhão (PMDB-PB), porém, indicar o relator. servidores com estabilidade no emprego não têm o direito
Segundo Renan Calheiros, Maranhão "demonstrou sequer de fazer greve.
disposição" em escolher o líder do PMDB para relatar o texto.
O nome de Eunício, mesmo sem um anúncio oficial pelo Oposição quer saída de Geddel; base sai em defesa de
Senado, já aparece como relator da matéria, no sistema ministro8
eletrônico. As acusações do ex-ministro da Cultura Marcelo Calero
Em plenário, Renan afirmou que a proposta seguirá o contra o titular da Secretaria de Governo, Geddel Vieira Lima,
calendário "natural" e que foi acordado entre líderes viraram assunto de conversas de deputados e senadores.
partidários. Parlamentares de oposição ao governo do presidente
Pelo cronograma acertado entre Renan Calheiros e os Michel Temer querem a demissão do ministro e prometem
líderes partidários, a votação da PEC em primeiro turno está acionar o Ministério Público pedindo a investigação do caso.
prevista para 29 de novembro e, em segundo turno, para 13 de Governistas preferem aguardar os desdobramentos da
dezembro. situação e saem em defesa de Geddel.
Por se tratar de uma emenda à Constituição, para entrar Calero disse em entrevista ao jornal "Folha de S.Paulo" e
em vigor, o texto precisa do apoio de, pelo menos, três quintos confirmou posteriormente em um evento com artistas no Rio
dos senadores (49 dos 81). Se os parlamentares aprovarem de Janeiro que o motivo principal de sua saída do governo foi
algum tipo de mudança no texto original, a PEC retornará à a pressão que sofreu do titular da Secretaria de Governo para
Câmara. liberar um empreendimento imobiliário de luxo em Salvador
no qual Geddel tinha comprado um apartamento.
Supremo admite corte de salário de servidores em O presidente da Comissão de Ética Pública da Presidência
greve7 da República, Mauro Menezes, afirmou que o colegiado vai
O Supremo Tribunal Federal (STF) considerou legítima analisar nesta segunda-feira (21/11/2016) se abre ou não
nesta quinta-feira (27/10/2016) a possibilidade de órgãos processo para investigar a conduta do ministro.
públicos cortarem o salário de servidores em greve desde o O líder do governo no Senado, Aloysio Nunes (PSDB-SP),
início da paralisação. disse ao G1 neste domingo (20) que prefere aguardar uma
Não poderá haver o corte nos casos em que a greve for posição do Planalto antes de comentar o assunto, mas elogiou
provocada por conduta ilegal do órgão público, como, por a atuação de Geddel como articulador político de Temer no
exemplo, o atraso no pagamento dos salários. Legislativo.
Com a decisão, a regra passa a ser o corte imediato do "Eu estou esperando a avaliação da Comissão de Ética da
salário, assim como na iniciativa privada, em que a greve Presidência da República. Ele [Geddel] tem sido um excelente
implica suspensão do contrato de trabalho. ministro no papel de interlocutor do governo no Congresso",
Mas os ministros abriram a possibilidade de haver acordo opinou Aloysio.
para reposição do pagamento se houver acordo para O líder do PTB, partido da base de Temer, Jovair Arantes
compensação das horas paradas. (GO), também teceu elogios a Geddel e disse que fazer
A decisão tem repercussão geral, devendo ser aplicada acusações no Brasil virou uma "doença".
pelas demais instâncias judiciais em processos semelhantes. "Esse vaievém dessas acusações virou uma doença de as
No julgamento, os ministros analisaram um recurso pessoas acusarem as outras, às vezes sem ter nenhuma
apresentado pela Fundação de Apoio à Escola Técnica do certeza, então não dá pra saber [o que é verdade]. Geddel é um
Estado do Rio de Janeiro (Faetec), que, em 2006, foi impedida excelente ministro, interage muito bem com a Câmara",
pela Justiça estadual de realizar o desconto na folha de declarou Jovair.
pagamento dos funcionários em greve. Aliado ao Planalto, o líder do PSD na Câmara, Rogério
Relator do caso e primeiro a votar quando começou o Rosso (PSD-DF), acredita que o assunto já "está superado".
julgamento, em 2015, o ministro Dias Toffoli afirmou que a "O ministro Geddel já se manifestou. O Brasil tem
decisão não derruba o direito de greve nem a possibilidade de problemas maiores que esse para resolver. Entendo ser um
os servidores recorrerem ao Judiciário. assunto superado. Não influenciará em nada o comportamento
“Qualquer decisão que nós tomarmos aqui não vai fechar da base do governo nas votações no Congresso", disse o
as portas do Judiciário, seja para os servidores seja para o deputado.
administrador público. O que estamos decidindo é se, havendo Já o líder do DEM no Senado, o governista Ronaldo Caiado
greve do servidor público, é legal o corte de ponto”, afirmou na (GO), criticou Calero por não ter falado antes sobre as supostas
sessão. pressões que sofreu do ex-colega. Caiado também afirmou que
Primeiro a se manifestar contra o desconto, Fachin Geddel "errou" ao procurar o ministro para tratar de um
defendeu que a suspensão do pagamento só ocorresse após assunto particular.
uma decisão judicial que reconhecesse a ilegalidade da greve. "Se alguém vem com uma proposta indecorosa,
“A suspensão do pagamento se dá no momento da própria incompatível com o cargo que a pessoa exerce, você deve dizer
gênese do movimento paredista. Está se interpretando que o isso na hora. Isso de falar depois dá margem para uma dupla
trabalhador deve ir a juízo para um obter direito que lhe é interpretação", afirmou Caiado.
assegurado constitucionalmente [salário]”, argumentou. "Mas se o fato foi esse, é lógico que o ministro Geddel errou.
Em vários momentos, ministros que defendem o corte na Ele não poderia ter misturado as coisas. Ele poderia tocar no
remuneração alertaram para os prejuízos causados à assunto se não tivesse nenhuma propriedade no prédio. No
população com a paralisação dos serviços. momento em que você tem propriedade no prédio, isso pode
“O administrador público não apenas pode, mas tem o parecer para a opinião pública que você está se valendo do
dever de cortar o ponto. O corte de ponto é necessário para a cargo para benefício próprio", completou.
adequada distribuição dos ônus inerentes à instauração da

727/10/2016 Fonte: 8 20/11/2016 Fonte:


http://g1.globo.com/politica/noticia/2016/10/supremo-admite-corte-de- http://g1.globo.com/politica/noticia/2016/11/oposicao-quer-saida-de-
salario-de-servidores-em-greve.html geddel-base-sai-em-defesa-de-ministro.html

Atualidades e Deveres dos Servidores Públicos 5


Apostila Digital Licenciada para Alice Caroline Guarino dos Santos - alice.guarino@hotmail.com (Proibida a Revenda)
APOSTILAS OPÇÃO

Caiado disse ainda que o assunto deve ser esclarecido A proposta


rapidamente e que é responsabilidade de Temer decidir se Pela proposta, o STJ não poderá admitir recurso especial
afasta Geddel do governo. sem que o recorrente demonstre a relevância das questões de
Oposição direito federal discutidas no caso. Quatro quintos dos
Por outro lado, a oposição já se movimenta para fechar o membros do órgão poderão rejeitar a relevância da questão.
cerco em torno do ministro da Secretaria de Governo. Geddel "Serão tidas como relevantes as questões de direito federal
foi um dos principais articuladores políticos do impeachment que tenham repercussão econômica, política, social ou
da ex-presidente Dilma Rousseff. jurídica", diz o texto.
Neste sábado, o deputado Jorge Solla (PT-BA) afirmou, por A PEC define ainda que não cabe recurso especial nas
meio de nota, que nesta segunda vai apresentar à Comissão de causas com valor inferior a 200 salários mínimos, a menos que
Fiscalização Financeira e Controle da Câmara um haja divergência entre a decisão recorrida e súmula do STJ.
requerimento para convocar Marcelo Calero a prestar O texto também estabelece que o STJ poderá aprovar
esclarecimentos sobre o episódio. súmula para impedir a interposição de recursos contra decisão
"É uma expressa acusação de crime de prevaricação [...]. Se que tiver aplicado. Essa medida será validada com a
Calero acusou outro ministro ao sair, é um caso muito grave e concordância de quatro quintos dos membros do órgão
precisa comprovar o que diz para que o caso tenha a competente.
consequência devida", diz trecho da nota divulgada pelo
deputado do PT. Temer se reúne com governadores para discutir crise
O líder do PT no Senado, Humberto Costa (PE), divulgou nos estados10
neste sábado um comunicado pedindo a demissão imediata de O Pesidente da República, Michel Temer, se reune na
Geddel do primeiro escalão e informando que vai pedir a manhã desta quinta-feira (1º), no Palácio do Planalto, com o
convocação de Geddel para que o ministro explique a denúncia ministro da Fazenda, Henrique Meirelles, a secretária do
no Senado. Além disso, o líder petista afirmou que irá solicitar Tesouro Nacional, Ana Paula Vescovi, e cinco governadores
que o Ministério Público Federal apure o caso. para discutir a crise financeira dos estados.
"É escandaloso que um ministro extremamente poderoso Participaram do encontro os governadores Luiz Fernando
dentro do governo, que trabalha na antessala de Temer, use do Pezão (RJ), Raimundo Colombo (SC), Rodrigo Rollemberg
próprio cargo para coagir e ameaçar colegas em favor de (DF), Simão Jatene (PA) e Wellington Dias (PI).
interesses pessoais", destacou o senador do PT. O rombo nas finanças tem afetado a maioria dos estados.
O líder do PT na Câmara, Afonso Florence (BA), também Alguns deles estão sem dinheiro até para pagar a folha do
quer a demissão de Geddel e disse que o governo de Temer funcionalismo. Os casos mais críticos são os do Rio de Janeiro
"derreteu" ao lembrar outros episódios desfavoráveis, como a e do Rio Grande do Sul.
exoneração do senador Romero Jucá (PMDB-RR) do Ministério Dos 27 estados brasileiros, quatro tiveram recuo no
do Planejamento, após desdobramentos da Lava Jato, e a saída Produto Interno Bruto (PIB) em 2014 na comparação com
de Fábio Medina Osório da Advocacia-Geral da União depois 2013, de acordo com a pesquisa Contas Regionais 2014
de desentendimentos como o chefe da Casa Civil, Eliseu divulgada no início desta semana pelo Instituto Brasileiro de
Padilha. Geografia e Estatística (IBGE).
"Não é só a história do Gedel, é a história do governo
Temer. O governo Temer derreteu. O ministro demissionário Pacto nacional
[Calero] disse que falou para Temer e ele não tomou nenhuma No último dia 22, o governo federal e estados anunciaram
providência [...]. Geddel não tem mais condições de continuar um "pacto nacional" pelo equilíbrio das contas públicas. O
no governo, ele tem que cair e o governo Temer também tem anúncio foi feito após reunião no Palácio do Planalto, em
que cair", disse o parlamentar baiano. Brasília, que também contou com a participação do presidente
Segundo o Blog do Camarotti, o Palácio do Planalto avalia Michel Temer, ministros da área econômica e governadores e
que se o assunto sair da pauta rapidamente, Geddel poderá que foi marcada para discutir a crise financeira nos estados.
permanecer no cargo. Mas se o tema seguir no noticiário, o Segundo o ministro da Fazenda, Henrique Meirelles, pelo
ministro será demitido. O receio é de que o caso acabe pacto o governo federal aceita dar aos estados uma fatia maior
atingindo o presidente Michel Temer dos recursos arrecadados com a chamada "repatriação". Em
contrapartida, os governadores se comprometem a fazer um
Câmara aprova em 1º turno PEC sobre recursos forte ajuste em suas contas, semelhante ao proposto pelo
especiais ao STJ9 próprio governo Temer, incluindo aumento da contribuição
A Câmara dos Deputados aprovou em primeiro turno a previdenciária paga por servidores públicos.
Proposta de Emenda à Constituição (PEC) que estabelece pré- Entretanto, o secretário de Fazenda do Rio Grande do
requisitos e restrições para a apresentação de recursos Norte, André Horta, disse nesta segunda (28), após reunião no
especiais ao Superior Tribunal de Justiça (STJ). Ministério da Fazenda, que os estados do Nordeste não
O recurso especial tem caráter excepcional e é apresentado concordam com algumas imposições do "pacto" pelo equilíbrio
ao STJ contra decisões de outros tribunais, em única ou última fiscal.
instância, quando houver ofensa à lei federal.
O texto foi aprovado em primeiro turno por 327 votos Reuniões com a Fazenda
favoráveis e 75 contrários. Cinco deputados se abstiveram de O Ministério da Fazenda informou que representantes da
votar. Por se tratar de uma emenda constitucional, a proposta pasta se reuniram com secretários estaduais de Fazenda para
precisava do apoio de, pelo menos, três quintos dos "detalhar o conjunto das medidas de ajuste fiscal que
parlamentares (308 dos 513). constarão do pacto nacional pelo equilíbrio das contas
Os deputados ainda precisarão votar a matéria em públicas, cujas linhas gerais foram apresentadas em reunião
segundo turno. Depois, a PEC, se novamente aprovada, seguirá com o presidente da República e o ministro da Fazenda na
para análise do Senado, onde precisará do dos votos de pelo última semana".
menos 49 senadores para entrar em vigor. O ministério informou ainda que o "resultado dessas
negociações será submetido pelos secretários de Fazenda aos

9 30/11/2016 Fonte: 10 01/12/2016 Fonte: http://g1.globo.com/economia/noticia/temer-se-

http://g1.globo.com/politica/noticia/2016/11/camara-aprova-em-1-turno- reune-com-governadores-para-discutir-crise-nos-estados.ghtml
pec-sobre-recursos-especiais-ao-stj.html

Atualidades e Deveres dos Servidores Públicos 6


Apostila Digital Licenciada para Alice Caroline Guarino dos Santos - alice.guarino@hotmail.com (Proibida a Revenda)
APOSTILAS OPÇÃO

respectivos governadores para posterior discussão com o Ministério Público. O STF, porém, ainda não analisou a
ministro da Fazenda". denúncia (veja todos os inquéritos sobre Renan ao final desta
reportagem).
Ministro do STF afasta Renan Calheiros da presidência No último domingo (4/12/2016), Renan foi o principal
do Senado11 alvo das manifestações de rua que reuniram milhares de
O ministro do Supremo Tribunal Federal (STF) Marco pessoas em todos os 26 estados mais o Distrito Federal. Os
Aurélio Mello concedeu liminar (decisão provisória) nesta atos, em apoio à Operação Lava Jato e ao projeto de dez
segunda-feira (5/12/2016) para afastar Renan Calheiros medidas de combate à corrupção, foram registrados em pelo
(PMDB-AL) da presidência do Senado. Ele, porém, mantém o menos 82 cidades.
mandato de senador.
O ministro atendeu a pedido do partido Rede Réu na linha de sucessão
Sustentabilidade e entendeu que, como Renan Calheiros virou Antes, em novembro, o Supremo começou a julgar ação
réu no Supremo, não pode continuar no cargo em razão de apresentada pela Rede sobre se um réu pode estar na linha
estar na linha sucessória da Presidência da República. sucessória da Presidência.
"Defiro a liminar pleiteada. Faço-o para afastar não do Para seis ministros, um parlamentar que é alvo de ação
exercício do mandato de Senador, outorgado pelo povo penal não pode ser presidente da Câmara ou presidente do
alagoano, mas do cargo de Presidente do Senado o senador Senado porque é inerente ao cargo deles eventualmente ter
Renan Calheiros. Com a urgência que o caso requer, deem que assumir a Presidência.
cumprimento, por mandado, sob as penas da Lei, a esta O julgamento, porém, não foi concluído porque o ministro
decisão", afirma o ministro no despacho. Dias Toffoli pediu vista, ou seja, mais tempo para analisar o
Em nota enviada pela assessoria, Renan Calheiros disse caso.
que só irá se manifestar sobre o afastamento após conhecer Apesar de o julgamento não ter sido concluído, a Rede
"oficialmente" o inteiro teor da decisão liminar. A nota diz argumentou no pedido de afastamento de Renan que isso não
ainda que o peemedebista consultará seus advogados sobre as impedia Marco Aurélio Mello de analisar a liminar. O partido
medidas adequadas a serem adotadas após a decisão de Marco lembrou que isso já aconteceu em outros casos, de um ministro
Aurélio que, segundo Renan, foi tomada "contra o Senado pedir vista sobre um tema e outro conceder liminar sobre o
Federal". mesmo tema.
Renan Calheiros ainda pode recorrer ao plenário do
Supremo. Além disso, a ação ainda terá que ser analisada pelo Decisão
plenário da Corte mesmo sem o recurso do peemedebista, mas O ministro Marco Aurélio destacou, em sua decisão, que a
isso ainda não tem data para ocorrer. permanência de Renan Calheiros na presidência do Senado
Com o afastamento do peemedebista da presidência, o após virar réu compromete a segurança jurídica. Ele lembrou
senador oposicionista Jorge Viana (PT-AC), primeiro-vice- que seis ministros do Supremo já decidiram que um réu não
presidente do Senado, assumirá o comando da Casa. Renan e o pode ocupar cargo que esteja na linha sucessória, mas frisou
petista se encontraram à noite, na casa do senador. que o julgamento ainda não foi retomado.
O afastamento efetivo de Renan Calheiros só ocorrerá após "O que não havia antes veio a surgir: o hoje Presidente do
a entrega de uma notificação, assinada pelo ministro Marco Senado da República, senador Renan Calheiros, por oito votos
Aurélio, no Senado. Até o início da noite desta segunda, o a três, tornou-se réu [...]. Mesmo diante da maioria absoluta já
documento ainda estava sendo produzido pela secretaria formada na arguição de descumprimento de preceito
judiciária do STF. Segundo apurou o G1, Marco Aurélio Mello fundamental e réu, o Senador continua na cadeira de
estaria disposto a assinar o documento ainda na noite desta Presidente do Senado, ensejando manifestações de toda
segunda, a fim de enviá-lo ao Senado. ordem, a comprometerem a segurança jurídica", disse o
Renan marcou de receber a notificação nesta terça, às 11h, ministro.
no Senado. Por volta das 21h30 desta segunda, um oficial de Segundo ele, a decisão liminar de afastamento visa
Justiça chegou à residência oficial do Senado, onde mora implementar provisoriamente decisão já tomada pela maioria
Renan Calheiros, dizendo: “Vim entregar a decisão do do Supremo.
ministro”. O homem passou pelo portão e ficou aguardando ser "Urge providência, não para concluir o julgamento de
recebido na porta da casa. Passados dois minutos, uma mulher fundo, atribuição do Plenário, mas para implementar medida
apareceu na porta da residência e conversou com o oficial de acauteladora, forte nas premissas do voto que prolatei, nos
Justiça, que foi embora sem falar com a imprensa. cinco votos no mesmo sentido, ou seja, na maioria absoluta já
formada, bem como no risco de continuar, na linha de
Réu no STF substituição do Presidente da República, réu, assim
Na semana passada, o plenário do Supremo decidiu, por qualificado por decisão do Supremo", argumenta Marco
oito votos a três, abrir ação penal e tornar Renan réu pelo Aurélio Mello.
crime de peculato (apropriação de verba pública).
Segundo o STF, há indícios de que Renan fraudou Delator da Odebrecht cita doações não declaradas a
recebimento de empréstimos de uma locadora de veículos mais de 30 políticos; veja lista12
para justificar movimentação financeira suficiente para pagar Em informações prestadas ao Ministério Público Federal
pensão à filha que teve com a jornalista Mônica Veloso. (MPF) para a assinatura de acordo de delação premiada, o ex-
A Corte também entendeu que há indícios de que Renan diretor de relações institucionais da Odebrecht Cláudio Melo
Calheiros usou dinheiro da verba indenizatória que deveria Filho apresentou valores repassados a políticos com a
ser usada no exercício do cargo de Senador para pagar a finalidade de obter vantagens para a empreiteira.
locadora, embora não haja nenhum indício de que o serviço foi O depoimento, que veio a público na sexta-feira
realmente prestado. (9/12/2016), traz nomes, valores, circunstâncias e motivação
Além da ação penal por peculato, Renan Calheiros é alvo de dos repasses. Parte dos recursos foi paga por meio de doações
outros 12 inquéritos no STF, sendo oito no âmbito da Operação eleitorais oficiais, mas também há registro de propina e de
Lava Jato. Em um dos inquéritos, ele já foi denunciado pelo caixa 2.

11 05/12/2016 Fonte: http://g1.globo.com/politica/noticia/ministro-do- 12 11/12/2016 Fonte: http://g1.globo.com/politica/operacao-lava-


stf-afasta-renan-da-presidencia-do-senado.ghtml jato/noticia/veja-valores-repassados-a-politicos-segundo-delator-da-
odebrecht.ghtml

Atualidades e Deveres dos Servidores Públicos 7


Apostila Digital Licenciada para Alice Caroline Guarino dos Santos - alice.guarino@hotmail.com (Proibida a Revenda)
APOSTILAS OPÇÃO

Em alguns casos, como o dos senadores Romero Jucá Duarte Nogueira, prefeito eleito de Ribeirão Preto, afirma
(PMDB-RR) e Renan Calheiros (PMDB-AL), o dinheiro era que todas as doações recebidas em suas campanhas eleitorais
entregue a uma pessoa, mas serviria para abastecer um grupo de 2010 e 2014 – por empresas ligadas ao grupo Odebrecht ou
dentro do partido. Em outros casos, não é possível identificar repassadas pelo Diretório Estadual do PSDB – foram
se a doação foi oficial. regularmente declaradas e aprovadas pelo TSE.
Cláudio atuava na relação da Odebrecht com o Congresso – Eduardo Cunha (PMDB-RJ), ex-deputado federal. Valor:
Nacional. Segundo ele, alguns pagamentos eram feitos para R$ 10,5 milhões, não determinado.
garantir a aprovação de projetos de interesse da empreiteira. A defesa de Eduardo Cunha disse que só vai se manifestar
Na pré-delação, ele citou 51 políticos de 11 partidos. depois que tiver acesso à delação e que ela deveria ter seu
Veja os nomes dos políticos que Cláudio Melo Filho disse sigilo resguardado.
que receberam doações não declaradas, os valores e a situação – Eliseu Padilha (PMDB-RS), ministro-chefe da Casa Civil.
do pagamento. Também confira o que cada um disse sobre as Valor: R$ 10 milhões a pedido de Temer, não determinado.
acusações: Eliseu Padilha afirmou que a acusação é “mentirosa” e que
– Adolfo Viana (PSDB-BA), deputado estadual. Valor: R$ nunca tratou de arrecadação para deputados ou para que quer
50 mil, não declarados. que seja.
O deputado estadual da Bahia Adolfo Viana disse que não – Eunício Oliveira (PMDB-CE), senador. Valor: R$ 2,1
conhece Cláudio Melo Filho e que todas as doações que milhões não declarados.
recebeu foram declaradas à Justiça Eleitoral. O líder do PMDB no Senado, Eunício Oliveira, disse "que
– Aécio Neves (PSDB-MG), senador. Pediu R$ 1 milhão todos os recursos arrecadados em suas campanhas foram
para o DEM, não declarados. recebidos de acordo com a lei e aprovados pela Justiça
O presidente do PSDB, senador Aécio Neves, afirmou que Eleitoral". Ele ainda disse que "nunca autorizou ninguém a
todos os pedidos de doações feitos pelo partido em 2014 se negociar recursos em seu nome em troca de favorecimento à
deram exclusivamente dentro da lei. qualquer empresa".
– Anderson Dornelles, ex-assessor de Dilma Rousseff. – Francisco Dornelles (PP-RJ), vice-governador do Rio de
Valor: R$ 350 mil, não declarados. Janeiro. Valor: R$ 200 mil não declarados.
O ex-assessor de Dilma, Anderson Dornelles, negou as O vice-governador do Rio, Francisco Dornelles, enviou uma
acusações. Ele disse que nunca se reuniu na sede da Odebrecht nota afirmando que todas as doações eleitorais recebidas pelo
nem solicitou ou recebeu qualquer ajuda financeira. Negou Partido Progressista foram feitas de acordo com a legislação
ainda que tenha autorizado qualquer pessoa a fazer isso em eleitoral e aprovadas pela Justiça.
nome dele. – Geddel Vieira Lima (PMDB-BA), ex-ministro da
– Antonio Brito (PSD-BA), deputado federal. Valor: R$ Secretaria de Governo. Valor: R$ 1 milhão não determinados,
100 mil não declarados, R$ 200 mil indefinidos e R$ 130 mil R$ 1,5 milhão não declarados e R$ 2,380 milhões declarados.
declarados. Geddel Vieira Lima disse que acha estranho o nome dele
O deputado federal Antonio Brito não atendeu as ligações estar na delação, já que todas as doações da Odebrecht à
do G1. campanha dele foram declaradas à Justiça Eleitoral.
– Arthur Maia (PPS-BA), deputado federal. Valor: R$ 250 – Gim Argello (PTB-DF), ex-senador. Valor: R$ 1,5 milhão
mil não declarados e R$ 350 mil declarados. não determinados, R$ 300 mil declarados e R$ 1 milhão não
"Todas as doações que recebi da Odebrecht constam na declarados.
minha prestação de contas e foram devidamente declaradas ao O G1 conversou com o responsável pela defesa do ex-
TSE e, portanto, aconteceram rigorosamente dentro dos senador Gim Argelo na manhã desta segunda-feira (12). O
parâmetros legais", afirmou o deputado federal Arthur Maia. advogado Marcelo Bessa preferiu não comentar as declarações
– Arthur Virgílio (PSDB-AM), prefeito eleito de Manaus. de Cláudio Melo Filho.
Valor: R$ 300 mil não declarados. – Heráclito Fortes (PSB-PI), deputado. Valor: R$ 200 mil
O prefeito eleito de Manaus, Arthur Virgílio, do PSDB, não declarados e R$ 50 mil declarados.
negou que tivesse recebido doação por caixa 2. Ele disse que A assessoria do deputado Heráclito Fortes disse que ele
todas as doações da Odebrecht foram declaradas à justiça recebeu contribuições legais e já aprovadas pela justiça
eleitoral. eleitoral, sem qualquer vinculação com propina.
– Ciro Nogueira (PP-PI), senador. Valor: R$ 300 mil não – Hugo Napoleão (PSD-PI), ex-senador. Valor: R$ 100 mil
declarados e R$ 1,8 milhão declarados. declarados e R$ 100 mil não declarados.
O senador Ciro Nogueira disse que as doações recebidas A assessoria do ex-senador Hugo Napoleão disse que
pelo partido foram legais e sempre devidamente declaradas à enviaria nota comentando as acusações de Cláudio Melo Filho.
justiça eleitoral. – Inaldo Leitão, ex-deputado. Valor: R$ 100 mil não
– Colbert Martins (PMDB-BA), ex-deputado federal. declarados.
Valor: R$ 150 mil não declarados e R$ 441 mil declarados. Segundo o jornal "O Globo", o ex-deputado Inaldo Leitão
O ex-deputado Colbert Martins negou que tenha recebido disse em um perfil nas redes sociais que nunca teve relação de
quantias não declaradas da Odebrecht ou de qualquer outra negócios com a Odebrecht e que não atuou em interesse da
empresa. “Tudo que tem nas minhas contas eleitorais foi empresa.
declarado. Nenhum tipo de omissão foi feita. As contas foram – Jaques Wagner (PT-BA), ex-ministro. Valor: R$ 3
apreciadas e aprovadas. Tudo que eu recebi eu declarei." milhões não determinados e R$ 7,5 milhões não declarados.
– Daniel Almeida (PCdoB-BA), deputado federal. Valor: Cláudio Melo diz acreditar que foram repassados R$ 10
R$ 100 mil não declarados. milhões para a candidatura do governador da Bahia, Rui Costa,
Daniel Almeida negou recebimento via caixa 2 e disse que em 2014, a pedido de Wagner.
todas as doações para suas campanhas foram legais. O ex-ministro Jaques Wagner não retornou os contatos da
– Delcídio do Amaral (sem partido-MS), ex-senador. TV Globo.
Valor: R$ 500 mil não declarados. – José Agripino (DEM-PI), senador. Valor: R$ 1 milhão, a
A defesa do ex-senador Delcídio do Amaral disse que as pedido de Aécio Neves, não determinados.
informações são especulativas, não oficiais e sem José Agripino disse que desconhece e repele as acusações.
credibilidade. – José Carlos Aleluia (DEM-BA), deputado federal. Valor:
– Duarte Nogueira (PSDB-SP), deputado. Valor: R$ 350 R$280 mil declarados e R$ 300 mil não declarados.
mil não declarados e R$ 300 mil declarados.

Atualidades e Deveres dos Servidores Públicos 8


Apostila Digital Licenciada para Alice Caroline Guarino dos Santos - alice.guarino@hotmail.com (Proibida a Revenda)
APOSTILAS OPÇÃO

O deputado José Carlos Aleluia disse ao G1 que "todos os – Paulo Skaf (PMDB-SP), presidente da Fiesp. Segundo o
valores recebidos estão declarados em minhas prestações de delator, a campanha de Skaf ao governo de São Paulo. Valor:
contas à Justiça Eleitoral”. Também há um comunicado no site R$ 6 milhões dos R$ 10 milhões negociados por Michel Temer.
do deputado, que diz que, em 2010, a legislação não exigia que O presidente da Fiesp, Paulo Skaf, disse que nunca pediu e
o candidato especificasse em sua prestação de contas o nome nem autorizou ninguém a pedir qualquer contribuição de
do doador original ao partido, como passou a ser feito a partir campanha que não fosse regularmente declarada em suas
de 2014. prestações de contas e afirmou que todas elas foram
– Jutahy Magalhães (PSDB-BA), deputado federal. Valor: aprovadas pela Justiça Eleitoral.
R$ 350 mil sem declaração e R$ 500 mil declarados. – Renan Calheiros (PMDB-AL), presidente do Senado.
O deputado federal Jutahy Magalhães negou acusações: Seria, segundo o delator, um dos beneficiários dos R$ 22
“Não tive doação de caixa 2 da Odebrecht em 2010 para minha milhões repassados ao grupo do PMDB na Casa (doações
campanha. Recebi oficialmente pelo partido e está registrado declaradas e não declaradas).
no TSE. Esta é a verdade” Renan Calheiros disse que "jamais autorizou ou consentiu
– Lídice da Mata (PSB-BA), senadora. Valor: R$ 200 mil que terceiros falassem em seu nome em qualquer
não declarados. circunstância". O senador afirmou ainda que "não há chance de
Em nota, a senadora Lídice da Mata negou as acusações. serem encontradas irregularidades em suas contas pessoais
"Não tenho e nunca tive qualquer tipo de negócios com a ou eleitorais". E que "essas contas já são investigadas há nove
Odebrecht ou qualquer outra empresa. Todas as doações de anos sem que nenhuma prova seja encontrada contra ele".
campanha que recebi foram dentro da legalidade e – Rodrigo Maia (DEM-RJ), presidente da Câmara. Valor:
devidamente declaradas conforme pode ser comprovado na R$ 600 mil, não determinados.
prestação de contas disponível no site do TSE / TRE-BA." O presidente da Câmara, Rodrigo Maia, negou com
– Lúcio Vieira Lima (PMDB-BA), deputado federal. Valor: veemência ter participado de qualquer negociação com a
R$ 400 mil declarados e teria recebido R$ 1,5 milhão não Odebrecht. Ele disse que nunca recebeu vantagem indevida em
declarados. troca de votações na Câmara e que todas as doações recebidas
O deputado Lúcio Vieira Lima afirmou que todas as por ele foram legais e declaradas ao TSE.
doações da sua campanha "estão declaradas na prestação de – Romero Jucá (PMDB-RR), senador. Seria, segundo o
contas do [Tribunal Superior Eleitoral] TSE”. delator, um dos beneficiários dos R$ 22 milhões repassados ao
– Marco Maia (PT-RS), deputado federal. Valor: R$ 1,35 grupo do PMDB do Senado (doações declaradas e não
milhão não declarados. declaradas).
O deputado Marco Maia refutou as declarações feitas pelo Romero Jucá, líder do governo no Congresso, negou que
delator, dizendo que foi relator da CPMI da Petrobras em 2014, recebesse recursos para o PMDB e disse que está à disposição
que indiciou 53 pessoas. "Não pedi e não autorizei ninguém a da Justiça.
pedir em meu nome nenhuma doação para minha campanha – Rui Costa (PT-BA), governador da Bahia. Teria recebido
eleitoral em 2014 de quaisquer empresas que estivessem na campanha, de acordo com o delator, R$ 10 milhões a pedido
sendo investigadas por tal CPMI. Todas as doações feitas à de Jaques Wagner. Delator não determina se foi doação legal
campanha estão devidamente registradas no TSE conforme ou caixa 2.
previa a legislação."
– Michel Temer (PMDB-SP), presidente da República. ECOMOMIA
Pediu, segundo o delator, repasse de R$ 10 milhões.
Em nota divulgada pelo Palácio do Planalto, o presidente Justiça de SP concede liminar que suspende cobrança
Michel Temer disse que repudia com veemência o que chamou extra por despacho de bagagem13
de "falsas acusações" de Cláudio Melo Filho. Segundo Temer, A Justiça Federal de São Paulo concedeu na tarde desta
as doações feitas pela construtora Odebrecht ao PMDB foram segunda-feira (13/03) liminar que suspende a cobrança extra
todas por transferência bancária e declaradas ao Tribunal pelo despacho de bagagem. A norma havia sido aprovada pela
Superior Eleitoral (TSE). A nota diz ainda que não houve caixa Agência Nacional de Aviação Civil (Anac), e entraria em vigor
dois nem entrega em dinheiro a pedido do presidente. no dia 14/03.
– Moreira Franco (PMDB-RJ), secretário do PPI. Segundo Pela decisão do juiz, ficam mantidas as regras atuais para
o relator, pediu recursos para o PMDB, mas o recebimento foi o despacho de bagagens. Pela regra atual, os passageiros
feito através de Eliseu Padilha. podem despachar um volume de até 23 kg nos voos nacionais
Moreira Franco disse que a acusação é mentirosa e afirmou e dois volumes de até 32 kg nos internacionais.
que jamais falou sobre política ou recursos para o PMDB com A Anac vai recorrer da decisão da Justiça. Em comunicado,
Cláudio Melo Filho. a agência informou que "respeita as instituições, mas adotará
– Paes Landim (PTB-PI), deputado federal. Valor: R$ 100 as providências necessárias para garantir os benefícios que
mil não declarados e R$ 80 mil declarados. acredita que as novas regras oferecem a toda a sociedade
O deputado Paes Landim disse que todas as doações que brasileira. As novas normas buscam aproximar o Brasil das
recebeu foram declaradas oficialmente e que jamais houve melhores práticas internacionais, trazendo novos estímulos
troca de favores entre ele e a Odebrecht. para a competição entre as empresas aéreas, com mais opções
– Paulo Henrique Lustosa (PP-CE), ex-deputado federal. de preços aos passageiros e seus diferentes perfis, como
Valor: R$ 100 mil não declarados e R$ 100 mil declarados. aqueles que pretendem transportar apenas os 10 kg na
Paulo Henrique Lustosa negou recebimento via caixa 2 e bagagem de mão."
disse que todas as doações para suas campanhas foram legais. O pedido de anulação havia sido feito pelo Ministério
– Paulo Magalhães Junior (PV-BA), vereador eleito de Público de São Paulo na quinta-feira (9/3). Segundo o MPF, a
Salvador. Valor: R$ 50 mil não declarados. nova regra contraria o Código Civil e o Código de Defesa do
O G1 não conseguiu contato com o vereador de Salvador Consumidor, além de ferir a Constituição por promover a
Paulo Magalhães Junior. perda de direitos já adquiridos pelos consumidores.
Pela regra aprovada pela Anac e suspensa nesta segunda, o
passageiro passaria a pagar à parte por bagagens despachadas

13 13/03/2017 – Fonte: http://g1.globo.com/economia/seu-


dinheiro/noticia/justica-de-sp-concede-liminar-que-suspende-cobranca-extra-
por-despacho-de-bagagem.ghtml

Atualidades e Deveres dos Servidores Públicos 9


Apostila Digital Licenciada para Alice Caroline Guarino dos Santos - alice.guarino@hotmail.com (Proibida a Revenda)
APOSTILAS OPÇÃO

em voos nacionais e internacionais. Hoje, esse serviço não tem


taxa extra. O limite de peso de bagagem de mão passaria de 5
para 10 quilos. Na prática, a medida permitiria que as
empresas criem suas próprias regras sobre o despacho de
bagagens.
A decisão, no entanto, suspende apenas as normas
relativas às bagagens. Outras medidas, como regras a respeito
de informações e cancelamento de voos, não foram atingidas.

Setor supermercadista pessimista com a Páscoa14


Passado o otimismo do Carnaval, a realidade: o setor
supermercadista está pessimista com a Páscoa.
Quase 88% dos lojistas esperam vendas iguais ou
inferiores ao ano passado. As encomendas dos supermercados
já apontam para uma redução nominal de -4,9% nas vendas.
Em valor real, a queda deverá ser de -7,7%.
Por outro lado, a expectativa é que as encomendas de
chocolate mais baratos cresçam, em detrimento aos populares
ovos. Como as caixas de bombons (4,0%), barras e tabletes
(4,0%) e bombom bola (2,1%).

FGTS: Agências da Caixa abrem sábado para saque de


conta inativa15
O saque das contas inativas do FGTS (Fundo de Garantia do
Tempo de Serviço) começa nesta sexta-feira (10) para
nascidos em janeiro e fevereiro. Para pagar quem tem direito
ao saque e dar informações, cerca de 1.800 agências abrirão
neste sábado (11).
O atendimento acontecerá das 9h às 15h. As agências
abrirão ainda em mais três sábados: 13 de maio, 17 de junho e
15 de julho.
O pagamento das contas inativas prossegue até 31 de julho.
Pelas estimativas da Caixa, 30,2 milhões de trabalhadores têm
direito ao saque de 43,6 bilhões de reais de 49,6 milhões de
contas inativas
A medida provisória 763/2016 prevê a liberação do saque
das contas que deixaram de receber depósitos a partir de 31
de dezembro de 2015. Podem sacar o dinheiro quem pediu
demissão ou foi demitido por justa causa até esta data. Quem
foi mandado embora sem justa causa tem o direito de sacar
imediatamente o dinheiro depositado no FGTS.

Pagamento
Os beneficiários têm quatro opções para recebimento dos
valores de contas inativas do Fundo de Garantia do Tempo de
Serviço:
Crédito em conta da Caixa: Quem tem conta corrente no
banco pode pedir o crédito no site das contas inativas.
Caixas eletrônicos: Valores de até 1.500 reais podem ser
sacados só com a senha do Cartão do Cidadão, mesmo que o
beneficiário tenha perdido o documento. Acima de R$ 3.000
reais é necessário sacar com cartão e senha.

1406/03/2017. Fonte: http://veja.abril.com.br/blog/radar-on-line/setor- 15 06/03/2017. Fonte: http://veja.abril.com.br/economia/fgts-agencias-

supermercadista-pessimista-com-a-pascoa/ da-caixa-abrem-sabado-para-saque-de-conta-inativa/

Atualidades e Deveres dos Servidores Públicos 10


Apostila Digital Licenciada para Alice Caroline Guarino dos Santos - alice.guarino@hotmail.com (Proibida a Revenda)
APOSTILAS OPÇÃO

Lotéricas e correspondentes: É preciso apresentar Cartão comparação com o 2º trimestre de 2015, o aumento foi de
do Cidadão, senha e documento de identidade para efetuar o 38,7%.
saque. Por outro lado, a população ocupada somou 90,8 milhões
Agências da Caixa: Os documentos necessários são o de pessoas e mostrou estabilidade em relação ao 1º trimestre
número de inscrição do PIS (Programa de Integração Social) e e queda de 1,5% sobre o período de abril a junho de 2015.
o documento de identificação do trabalhador. Segundo Cimar Azeredo, coordenador de Trabalho e
Divergência de informação Rendimento do IBGE, a ocupação se manteve estável em
Como há a possibilidade divergência entre no saldo da relação ao trimestre anterior, no entanto, foi menor do que em
conta inativa do FGTS, a Caixa recomenda que os beneficiários 2015. "Com o crescimento da população em idade para
também levem o comprovante da extinção do vínculo trabalhar (1,3%) e a redução da população ocupada, o nível da
empregatício. Pode ser a carteira de trabalho ou rescisão do ocupação no ano caiu de 56,2% para 54,6%." A Pnad entrevista
contrato. 211 mil domicílios em 3.464 municípios e 15.756 setores do
Essa divergência acontece, principalmente, porque a país.
empresa não deu baixa no vínculo empregatício do Também não houve alteração em relação à quantidade de
trabalhador. Nesses casos, a conta não aparece como inativa. trabalhadores com carteira assinada, que ficou em 34,4
milhões. Já na comparação com o mesmo trimestre do ano
Mercado eleva previsão de crescimento do PIB para anterior, esse número caiu 4,1%. De acordo com Azeredo, essa
2017 e 201816 retração gera "um movimento de pressão" ao mercado de
Economistas do mercado financeiro elevaram a previsão trabalho.
do crescimento econômico, medido pelo Produto Interno
Bruto, para este ano de 0,48% para 0,49%. De acordo com o Atividade extrativas puxam queda
Boletim Focus, do Banco Central, divulgado nesta segunda- Entre as atividades que registraram queda na produção no
feira, para 2018, a estimativa é de que a economia encerre o primeiro semestre do ano, estão as indústrias extrativas (-
ano com alta de 2,39%, contra 2,37% na última semana. 14,0%) e a de veículos automotores, reboques e carrocerias (-
A expectativa para o Índice Nacional de Preços ao 21,2%), que exerceram as principais pressões negativas.
Consumidor Amplo (IPCA), principal medidor da inflação Outras contribuições negativas partiram de máquinas e
brasileira, para 2017. se mantém em 4,36%. Para 2018, a equipamentos (-16%), coque, produtos derivados do petróleo
estimativa se mantém em 4,50%, pela 32ª semana seguida. e biocombustíveis (-5,9%), metalurgia (-11,9%) e
A expectativa para a Selic deste ano também se manteve equipamentos de informática, produtos eletrônicos e ópticos
em 9,25% nesta semana, de acordo com os especialistas do (-27%), entre outros.
mercado financeiro ouvidos pelo Focus. Para o fim de 2018,
também permaneceu inalterada, em 9% ao ano. A taxa básica Nº de novas empresas cresce no ano e cai no
de juros está atualmente em 12,25% ao ano. trimestre, diz Boa Vista20
A taxa de câmbio é outro indicador que permanece O número de novas empresas cresceu 1% no acumulado
inalterado. Para o fim de 2017, ela se manteve em 3,30 reais. do ano (de janeiro e setembro) em relação ao mesmo período
Para o fechamento de 2018, a estimativa dos economistas para do ano anterior, segundo levantamento da Boa Vista SCPC
o dólar se manteve em 3,40 reais. (Serviço Central de Proteção ao Crédito), com abrangência
nacional. Em relação ao 2º trimestre de 2016, houve queda de
Inflação17 2% no terceiro trimestre (de julho a setembro).
O IBGE divulga hoje(09/09/16) o IPCA de agosto, a Na análise por classificação de forma jurídica, as MEIs
inflação oficial do país, que em julho atingiu 0,52%, levando o (microempreendedor individual) continuam com papel de
acumulado no ano para 4,96%, menor que os 6,83% destaque. No acumulado do ano (janeiro a setembro de 2016),
registrados em igual período do ano anterior. as MEIs aumentaram 6% em relação ao mesmo período do ano
passado. Na mesma base de comparação, as MEs
Inflação do aluguel18 (microempresas) e as demais formas jurídicas recuaram 9,4%
A FGV divulga o IGP-M, conhecido como "inflação do e 14,3%, respectivamente.
aluguel" por ser usado para reajustado a maioria dos contratos De acordo com a Boa Vista, esse resultado refletiu
imobiliários, de agosto. Em julho, o índice desacelerou para diretamente na composição da abertura de empresas. As MEIs
0,32%, mas ainda acumula alta de 11,79% nos últimos 12 aumentaram 3,5 pontos percentuais de sua participação,
meses. totalizando 74,2% das novas empresas, enquanto as demais
categorias perderam representatividade.
Desemprego sobe para 11,3% no 2º trimestre, aponta
Pnad, do IBGE19 Setores
O desemprego subiu para 11,3% no trimestre encerrado Por setores, o levantamento da Boa Vista SCPC mostra que
em junho, segundo dados divulgados nesta sexta-feira o de serviços registrou ganho de representatividade. No
(29/08/16) pelo Instituto Brasileiro de Geografia e Estatística acumulado de 2016, este setor atingiu 55,9% das novas
(IBGE). A taxa é a maior já registrada pela série histórica da empresas, ante 54,4% no mesmo período do ano passado.
Pnad Contínua, que teve início em janeiro de 2012. Indústria e rural permaneceram estáveis, e o comércio
No trimestre encerrado em março, o índice de desemprego perdeu espaço, ao passar de 35% para 33,4% do total.
foi de 10,9% e no período de abril a junho de 2015, de 8,3%.
No trimestre de março a maio, a taxa bateu 11,2%. Regiões
A população desocupada cresceu 4,5% em relação ao O Sudeste e o Sul foram as regiões responsáveis pelo
primeiro trimestre e chegou a 11,6 milhões de pessoas. Já na crescimento das empresas. No acumulado do ano em
comparação com o mesmo período do ano anterior,

16 06/03/2017. Fonte: http://veja.abril.com.br/economia/mercado-eleva- 19 29/08/2016 – Fonte:


previsao-de-crescimento-do-pib-para-2017-e-2018/ http://g1.globo.com/economia/noticia/2016/07/desemprego-fica-em-113-
17 09/09/2016 – Fonte: http://g1.globo.com/agenda-do- no-2-trimestre-diz-ibge.html
dia/edicoes/2016/09/09.html 20 28/10/2016 Fonte:
18 30/08/2016 – Fonte: http://g1.globo.com/agenda-do- http://g1.globo.com/economia/pme/noticia/2016/10/n-de-novas-empresas-
dia/edicoes/2016/08/30.html cresce-no-ano-e-cai-no-trimestre-diz-boa-vista.html

Atualidades e Deveres dos Servidores Públicos 11


Apostila Digital Licenciada para Alice Caroline Guarino dos Santos - alice.guarino@hotmail.com (Proibida a Revenda)
APOSTILAS OPÇÃO

registraram alta de 2,8%. As demais regiões apresentaram Relatório


queda no período: Nordeste (2,5%), Norte (3,1%) e Centro- No relatório, Eunício Oliveira diz que não identificou
Oeste (3,4%). “qualquer violação” à Constituição na proposta apresentada
Com isso, Sudeste e Sul apresentaram ganho de pelo governo Temer. Na visão do parlamentar peemedebista,
representatividade, como apresentado no gráfico 5. Sul passou a PEC é constitucional e “não fere” a independência entre os
de 16,6% para 16,7% e o Sudeste aumentou de 50,7% para poderes.
51,8%. Em outubro, a Procuradoria Geral da República (PGR)
afirmou, por meio de nota, que a proposta do Executivo federal
Lucro da Eletrobras cai para R$ 875 milhões no 3º é “inconstitucional” porque “ofende” a independência dos
trimestre21 poderes. A nota da PGR foi rechaçada pelo Palácio do Planalto,
A estatal Eletrobras apresentou lucro de R$ 875 milhões que, à época, considerou a manifestação um ato corporativista.
no terceiro trimestre, revertendo prejuízo no mesmo período “A PEC não ofende quaisquer princípios ou regras
do ano passado, após contabilizar novas receitas relacionadas constitucionais, muito menos as chamadas cláusulas pétreas.
a indenizações devidas pela União à companhia desde a Ao contrário, [...] a proposta em análise se volta à realização de
renovação antecipada de seus contratos de concessão, vários objetivos da república, previstos na Constituição:
segundo dados divulgados nesta quinta-feira. garantir o desenvolvimento nacional, erradicar a pobreza e
As indenizações decorrem de um pacote de medidas do promover o bem de todos”, afirma Eunício no parecer.
governo para redução das tarifas de energia, no final de 2012. No relatório, Eunício também afirma que a PEC,
A companhia registrou um saldo contábil de R$ 1,499 diferentemente do que acusa a oposição, vai aumentar os
bilhão ao incluir uma atualização monetária referente a essas investimentos em saúde e educação.
indenizações, cujo recebimento foi reconhecido em seu “Em relação à saúde, o Novo Regime Fiscal elevará o piso
balanço do segundo trimestre. em 2017, de 13,7% da Receita Corrente Líquida para 15%.
No terceiro trimestre de 2015, o prejuízo da elétrica havia Considerando os valores atuais [...] esse aumento de
sido de R$ 4,225 bilhões. percentual implicará elevação do piso de gastos com saúde em
As indenizações renderam também mais R$ 1,931 bilhão à cerca de R$ 9 bilhões já em 2017”, escreveu Eunício em trecho
Eletrobras devido à participação detida pela estatal na do relatório.
transmissora de energia Cteep, que reconheceu o direito a Parlamentares da oposição, entretanto, argumentaram
compensações bilionárias em seu balanço do terceiro que a PEC representará o "congelamento" de investimentos
trimestre. sociais, nas áreas de saúde e educação, e reclamam que a
Com o resultado, a Eletrobras registra o segundo trimestre medida não deveria se estender por 20 anos. Os oposicionistas
consecutivo de lucros, após acumular mais de R$ 30 bilhões alegam que a regra deveria ser mais flexível para se adaptar as
em perdas desde 2012 mudanças econômicas no país.
O lucro antes de juros, impostos, depreciação e
amortização (Ebitda) do trimestre subiu para R$ 3,237 Emendas
bilhões, ante R$ 3,149 bilhões negativos no mesmo período de Senadores oposicionistas apresentaram uma série de
2015. emendas (sugestões de alteração ao texto) para, entre outros
pontos, retirar os investimentos na área de saúde, educação e
Comissão aprova relatório e PEC do teto de gastos ciência e tecnologia do teto de gastos.
avança no Senado22 Mas o relator Eunício Oliveira se posicionou
A Comissão de Constituição e Justiça (CCJ) do Senado contrariamente às possíveis modificações, alegando que tais
aprovou nesta quarta-feira (9/11/2016) o relatório do alterações iam de encontro aos objetivos da proposta.
senador Eunício Oliveira (PMDB-CE) favorável à Proposta de “Saúde e educação são áreas essenciais para o
Emenda à Constituição que estabelece um teto para os gastos desenvolvimento do país e bem-estar da população. A PEC
públicos. O placar da aprovação foi de 19 votos favoráveis e mantém a atual regra para o piso da Educação até 2017 e a
sete contrários – o presidente do colegiado, José Maranhão partir daí estabelece um crescimento pela inflação. Ao
(PMDB-PB), não vota. contrário de interpretações equivocadas que alardeiam que
Após aprovarem o texto-base, os senadores da comissão vamos tirar recursos da educação e da saúde, isso não é
rejeitaram um destaque ao texto e preservaram o conteúdo da verdade, não vamos tirar um centavo da saúde e da educação”,
proposta aprovada pela Câmara. O destaque queria afirmou Eunício.
estabelecer um referendo popular para determinar se a Senadores governistas são contrários a alterações do texto
proposta entraria ou não em vigor. porque mudanças no teor da PEC obrigariam que a medida
Depois de passar pela CCJ, a proposta seguirá para o voltasse para a Câmara. O Palácio do Planalto quer aprovar o
plenário principal do Senado, onde passará por dois turnos de texto ainda neste ano para que as regras já comecem a valer
votação. Para a medida, que já foi aprovada pela Câmara dos em 2017.
Deputados, entrar em vigor, será necessário o apoio de, pelo Eunício também deu parecer contrário a uma emenda da
menos, 49 senadores nas duas votações. senadora Vanessa Grazziotin (PC do B-AM) que sugeria a
A medida estabelece que as despesas da União só poderão realização de um referendo de consulta à população para que
crescer conforme a inflação do ano anterior e é considerada o teto de gastos entrasse em vigor. A parlamentar queria
pelo governo um dos principais mecanismos para tentar condicionar validade da PEC à aprovação da proposta por
reequilibrar as contas públicas do país. Criticada pela meio de referendo pela população.
oposição, a PEC foi apelidada de "PEC da maldade". O relator propôs ainda a rejeição de emendas dos
No início da manhã, alunos do Ensino Médio tentaram senadores Ângela Portela (PT-RR), José Pimentel (PT-CE) e
entrar no Senado para acompanhar a sessão. Impedidos pela Humberto Costa (PT-PE) que pretendiam estabelecer na
Polícia Legislativa, os manifestantes continuaram do lado de proposta um limite para o pagamento de juros e encargos da
fora e tentaram barrar o trânsito de pessoas e veículos. dívida pública. Eunício afirmou que a diminuição da dívida
No momento em que o texto era aprovado na CCJ, houve pública é “essencial” para a reorganização da economia do
protesto nos corredores do Senado (veja vídeo abaixo). país.

21 10/11/2016 Fonte: 22 09/11/2016 Fonte:


http://g1.globo.com/economia/negocios/noticia/2016/11/lucro-da- http://g1.globo.com/politica/noticia/2016/11/comissao-aprova-relatorio-e-
eletrobras-cai-para-r-875-milhoes-no-3-trimestre.html pec-do-teto-de-gastos-avanca-no-senado.html

Atualidades e Deveres dos Servidores Públicos 12


Apostila Digital Licenciada para Alice Caroline Guarino dos Santos - alice.guarino@hotmail.com (Proibida a Revenda)
APOSTILAS OPÇÃO

Voto em separado também fazer convergir a inflação para a meta central de 4,5%
Contrário à proposta, o senador Roberto Requião (PMDB- em 2017.
PR) apresentou, em nome de oposicionistas, um voto em
separado no qual recomendou a rejeição da PEC pela CCJ. Produto Interno Bruto
Para Requião, a proposta do teto de gastos apresenta Para o Produto Interno Bruto (PIB) de 2016, o mercado
diversas inconstitucionalidades. Uma delas está no fato de a financeiro prevê agora um encolhimento de 3,40%. Na
PEC retirar, na visão do senador paranaense, a prerrogativa pesquisa anterior, feita na semana retrasada, a previsão era de
que o presidente da República tem de elaborar a peça queda de 3,37%.
orçamentária com autonomia, uma vez que deverá obedecer O PIB é a soma de todos os bens e serviços feitos no país,
ao teto de gastos. independentemente da nacionalidade de quem os produz, e
“Eliminar, como pretende a PEC 55, a possibilidade do serve para medir o comportamento da economia brasileira.
chefe do poder Executivo legitimamente eleito definir o limite Essa será a primeira vez que o país registra dois anos
de despesas de governo, significa retirar-lhe uma das seguidos de retração no nível de atividade da economia – a
principais prerrogativas de orçamento e gestão. É o série histórica oficial, do IBGE, tem início em 1948. No ano
impeachment do próximo presidente da República”, afirmou passado, o recuo foi de 3,8%, o maior em 25 anos.
Requião. Os economistas das instituições financeiras também
O parlamentar também disse que a proposta fere a baixaram a previsão de alta do PIB em 2017, de 1,13% para
Constituição ao limitar, a partir de 2018, os investimentos em 1%, informou o BC.
educação e saúde à inflação acumulada no ano anterior, uma Nesta semana, o governo divulga sua nova previsão para o
vez que a Carta Magna prevê um piso, baseado na arrecadação, crescimento do PIB do ano que vem. Atualmente, a estimativa
de investimento nessas áreas. “O congelamento real significará é de uma alta de 1,6%.
um gigantesco retrocesso à educação básica”, acrescentou
Requião. Taxa de juros
Apesar do voto em separado da oposição, prevaleceu a O mercado financeiro manteve, na última semana, a
análise do relatório de Eunício Oliveira favorável à proposta. previsão para a taxa de juros no fim de 2016 em 13,75% ao
ano. Atualmente, os juros estão em 14% ao ano. Com isso, a
Bate-boca estimativa do mercado é de mais um corte nos juros até o fim
Antes de o colegiado concluir a análise da PEC, os ânimos de 2016.
se exaltaram no plenário da comissão. Em tom elevado, o Já para o fechamento de 2017, a estimativa para a taxa de
senador Aloysio Nunes (PSDB-SP) disse que o governo estava juros ficou estável em 10,75% ao ano - o que pressupõe
tentando recuperar a economia que o PT durante 13 anos continuidade do processo de corte dos juros no ano que vem.
“quebrou”. A taxa básica de juros é o principal instrumento do BC para
Também em tom elevado a senadora Gleisi Hoffmann (PT- tentar conter pressões inflacionárias. Pelo sistema de metas de
PR) defendeu a realização de um referendo popular para que inflação brasileiro, a instituição tem de calibrar os juros para
a PEC entre em vigor. atingir objetivos pré-determinados.
“Não é possível que nós, aqui nessa sala refrigerada, com As taxas mais altas tendem a reduzir o consumo e o crédito,
ar condicionado, vamos votar contra os direitos do povo o que pode contribuir para o controle dos preços. Quando julga
brasileiro. Como que vocês se sentem fazendo uma coisa que a inflação está compatível com as metas preestabelecidas,
dessas? Tenham coragem e decência de colocar essa proposta o BC pode baixar os juros.
nas urnas”, bradou Gleisi.
Lindbergh Farias (PT-RJ) também protestou contra a Câmbio, balança e investimentos
proposta, afirmando que o ajuste fiscal está sendo “feito em Nesta edição do relatório Focus, a projeção do mercado
cima dos mais pobres”. financeiro para a taxa de câmbio no fim de 2016 subiu de R$
3,22 para R$ 3,30. Para o fechamento de 2017, a previsão dos
Mercado estima menos inflação para este ano e economistas para o dólar ficou estável em R$ 3,40.
'tombo' maior do PIB23 A projeção para o resultado da balança comercial
Os economistas das instituições financeiras previram (resultado do total de exportações menos as importações) em
menos inflação para este ano e um "encolhimento" maior do 2016 recuou de US$ 47,6 bilhões para US$ 47,4 bilhões de
Produto Interno Bruto (PIB) em 2016, além de uma expansão resultado positivo. Para o próximo ano, a previsão de superávit
mais fraca da economia no próximo ano. permaneceu em US$ 45 bilhões.
As expectativas foram coletadas pelo Banco Central na Para 2016, a projeção de entrada de investimentos
semana passada e divulgadas nesta segunda-feira estrangeiros diretos no Brasil permaneceu inalterada em US$
(21/11/2016) por meio do relatório de mercado, também 65 bilhões e, para 2017, a estimativa dos analistas continuou
conhecido como Focus. Mais de cem instituições financeiras em US$ 70 bilhões.
foram ouvidas.
A estimativa do mercado para o Índice Nacional de Preços Prévia da inflação oficial é a menor para novembro
ao Consumidor Amplo (IPCA) deste ano recuou de 6,84% para desde 200724
6,80% na semana passada. Mesmo assim, permanece acima do O Índice de Preços ao Consumidor - Amplo 15 (IPCA-15),
teto de 6,5% do sistema de metas de inflação e bem distante considerado uma prévia da inflação oficial, ganhou força de
do objetivo central fixado para 2016, que é de inflação de 4,5%. outubro para novembro, ao passar de 0,19% para 0,26%,
Para 2017, a previsão do mercado financeiro para a segundo informou o Instituto Brasileiro de Geografia e
inflação permaneceu estável em 4,93%. O índice está abaixo do Estatística (IBGE) nesta quarta-feira (22/11/2016).
teto de 6% para o IPCA, fixado para o ano que vem, mas ainda Apesar de ter avançado de um mês para o outro, a taxa é a
acima da meta central, que de inflação de 4,5%. menor para novembro desde 2007, quando chegou a 0,23%.
O BC tem informado que buscará "circunscrever" o IPCA No ano, a prévia do IPCA acumula alta de 6,38%. No mesmo
aos limites estabelecidos pelo Conselho Monetário Nacional período de 2015, o avanço era de 9,42%. Nos últimos 12
(CMN) em 2016 (ou seja, trazer a taxa para até 6,5%), e meses, o índice ficou em 7,64%, abaixo dos 8,27% registrados

23 21/11/2016 Fonte: 24 23/11/2016 Fonte:


http://g1.globo.com/economia/mercados/noticia/2016/11/mercado-estima- http://g1.globo.com/economia/noticia/2016/11/previa-da-inflacao-oficial-
menos-inflacao-para-este-ano-e-tombo-maior-do-pib.html fica-em-026-novembro.html

Atualidades e Deveres dos Servidores Públicos 13


Apostila Digital Licenciada para Alice Caroline Guarino dos Santos - alice.guarino@hotmail.com (Proibida a Revenda)
APOSTILAS OPÇÃO

nos 12 meses imediatamente anteriores - mesmo assim, registraram em outubro economia recorde. A série histórica
permanece acima do teto de 6,5% do sistema de metas de do BC começa em dezembro de 2001.
inflação e bem distante do objetivo central fixado para 2016, O superávit primário no mês passado foi de R$ 39,58
que é de inflação de 4,5%. bilhões, informou o Banco Central. Até então, o maior saldo
O que mais impactou a alta do IPCA-15 em novembro foi o positivo, pelo conceito primário, havia acontecido em janeiro
etanol, que ficou 7,29% mais caro. O preço das multas de de 2013, com um superávit de R$ 30,25 bilhões.
trânsito, que foram reajustados no início do mês, também Nos últimos cinco meses, as contas estavam no vermelho,
pressionou o índice, ao subir, em média, 23,72%. até mesmo considerando essa metodologia (primário), por
Também influenciaram a prévia da inflação oficial os conta da recessão na economia, que tem diminuído as receitas
preços de seguro de veículo (2,61%), de plano de saúde com impostos não somente do governo, mas também dos
(1,07%), de empregado doméstico (0,87%), de mão de obra estados e municípios.
para pequenos reparos (0,87%), de artigos de higiene pessoal Os números do Banco Central revelam, entretanto, que
(0,87%), de emplacamento e licença (0,80%), de cabeleireiro sem os recursos da "repatriação" (R$ 45 bilhões), o resultado
(0,67%) e de gasolina (0,59%). do setor público consolidado teria registrado novo déficit, de
R$ 5,4 bilhões, em outubro deste ano. Seria o sexto mês
Grupos de gastos seguido com as contas no vermelho pelo conceito primário.
Na análise dos grupos de gastos, o maior aumento de
preços partiu de saúde e cuidados pessoais, cuja alta passou de Acumulado do ano e meta fiscal
0,28%, em outubro, para 0,68%, no mês seguinte. Apesar do bom resultado em outubro, fruto do ingresso
Os alimentos e bebidas também contribuíram com o dos recursos da "repatriação" de ativos, no acumulado dos dez
avanço do IPCA-15. Isso porque a queda de preços perdeu primeiros meses deste ano as contas públicas continuaram no
força em novembro, ao passar de -0,25% para - 0,06%. vermelho, quer seja pelo conceito primário (que não considera
Ficaram mais caros: açúcar cristal (3,73%), pescados (3,91%), os gastos com juros da dívida) ou nominal (que inclui os juros
batata-inglesa (3,26%), cerveja (2,36%) e carnes (1,43%). na conta).
Ficaram mais baratos: leite longa vida (-10,52%), feijão- De janeiro a outubro, foi registrado um déficit primário de
carioca (-11,84%), feijão-mulatinho (-7,82%), tomate (- R$ 45,91 bilhões. Foi disparado o maior rombo primário para
6,61%) e cenoura (-4,31%). este período desde o início da série histórica do Banco Central,
em dezembro de 2001. Até então, o maior rombo fiscal, para
Região este período, havia sido registrado no ano passado (-R$ 19,95
O IPCA-15 mais elevado partiu da região metropolitana do bilhões).
Recife (0,55%), puxado pelo preço do litro da gasolina Pelo conceito nominal, que considera os juros da dívida
(4,11%). Na outra ponta está Goiânia (-0,03%), sob influência pública no cálculo, o déficit das contas públicas somou
da queda de 4,15% da energia elétrica. expressivos R$ 377 bilhões de janeiro a outubro - com queda
frente ao mesmo período do ano pasado, quando totalizou R$
Previsão do mercado financeiro 446 bilhões. Em doze meses até outubro, o rombo fiscal pelo
A estimativa dos economistas para o IPCA deste ano está conceito nominal - que é olhado com atenção por investidores
em 6,80%, segundo o boletim Focus, do Banco Central, mais - somou R$ 544 bilhões, ou 8,8% do PIB.
recente. O bom resultado de outubro, obtido com a entrada dos
recursos da repatriação, ajuda no atingimento da meta fiscal
Após 'repatriação', contas públicas têm 1º superávit deste ano - que foi fixada em um déficit de até R$ 163,94
em 18 meses25 bilhões.
O setor público consolidado, o que inclui o governo federal, Esse número considera um superávit, ou seja, resultado
os estados, municípios e as empresas estatais, conseguiu positivo, de R$ 6,55 bilhões dos estados e municípios. Se
economizar, em outubro, o suficiente não só para pagar as confirmado este valor, também será o pior resultado da série
despesas correntes, mas também os gastos com juros da dívida histórica, que começa em dezembro de 2001.
pública, segundo informações divulgadas pelo Banco Central Em 2016, o Brasil registrará o terceiro ano seguido com as
nesta sexta-feira (28). contas no vermelho. Em 2014, houve um déficit de R$ 32,5
É a primeira vez em 18 meses que é registrado um bilhões e, em 2015, um rombo recorde de R$ 111 bilhões.
"superávit nominal" nas contas públicas - conceito que é mais
utilizado para comparações internacionais. O saldo positivo, Dívidas líquida e bruta
por essa metodologia, somou R$ 3,38 bilhões no mês passado. Segundo números do Banco Central, a dívida líquida do
A última vez que as contas públicas tinham ficado no azul, setor público (governo, estados, municípios e empresas
mesmo após contabilizar as despesas com juros, havia sido em estatais) subiu de R$ 2,69 trilhões em setembro, ou 44,1% do
abril de 2015, quando o saldo positivo somou R$ 11,23 bilhões, PIB, para R$ 2,72 trilhões em outubro deste ano – o
de acordo com dados oficiais. equivalente a 44,2% do PIB.
O bom resulado das contas públicas em outubro deste ano A dívida líquida considera os ativos do país como, por
só foi possível por conta do ingresso dos recursos da exemplo, as reservas internacionais – atualmente ao redor de
regularização de ativos no exterior, a chamada "repatriação", US$ 370 bilhões.
que gerou arrecadação de R$ 45 bilhões no mês passado. O No caso da dívida bruta do setor público, uma das
prazo para os sonegadores regularizarem sua situação, com principais formas de comparação internacional (que não
desconto de multa e imposto devido, terminou em 31 de considera os ativos dos países, como as reservas cambiais), o
outubro. endividamento brasileiro também cresceu. Esse indicador é
Sem contar juros, superávit bate recorde acompanhado mais atentamente pelas agências de
No cálculo que considera apenas as receitas e as despesas classificação de risco.
correntes, sem contar o que foi pago de juros da dívida pública, Em dezembro de 2015, a dívida estava em 66,5% do PIB
chamado de resultado "primário", as contas públicas (R$ 3,92 trilhões). Em setembro, já havia avançado para R$
4,32 trilhões, ou 70,7% do PIB mas, em outubro, recuou em

25 28/11/2016 Fonte:
http://g1.globo.com/economia/noticia/2016/11/contas-publicas-tem-1-
superavit-em-18-meses-apos-pagar-juros-da-divida.html

Atualidades e Deveres dos Servidores Públicos 14


Apostila Digital Licenciada para Alice Caroline Guarino dos Santos - alice.guarino@hotmail.com (Proibida a Revenda)
APOSTILAS OPÇÃO

termos percentuais do PIB - para 70,3%. Em reais, porém, A regra proposta pelo governo prevê, por exemplo, que
avançou para R$ 4,33 trilhões no mês passado. uma viúva poderá receber 60% do benefício se o casal tiver um
Se for considerado o conceito usado pelo Fundo Monetário filho. O INSS pagará 100% do benefício apenas aos
Internacional (FMI) - que leva em conta os títulos livres na pensionistas que tiverem cinco filhos.
carteira do BC – a dívida bruta estaria em 74,5% do PIB em Além disso, o valor extra pago por conta do número de
setembro deste ano. dependentes não será agregado à pensão no momento em que
os filhos completarem 18 anos. Também não será possível
Previdência: como é e como pode ficar26 acumular esse benefício com outra aposentadoria ou pensão.
O Governo anunciou nesta terça-feira (6/12/2016) os
detalhes da reforma da Previdência, que foi enviada ao Impacto na economia
Congresso Nacional. A proposta de emenda constitucional De acordo com estimativas do governo, os gastos com a
(PEC), que recebeu o número 287, é uma das principais Previdência passarão de 0,3% do Produto Interno Bruto em
medidas do governo para tentar reequilibrar as contas 1997 para estimados 2,7% do PIB em 2017. Atualmente,
públicas. representam 40% do gasto primário do governo.
Uma das principais mudanças é que a idade mínima para O pagamento de aposentadorias e outros benefícios do
se aposentar passaria para 65 anos, tanto para homens quanto INSS não pode sofrer cortes e abocanha mais de 40% dos
para mulheres. Para os homens com menos de 50 anos gastos primários (sem contar os juros da dívida). Estes gastos
atualmente e as mulheres com menos de 45, haverá uma regra obrigatórios crescem conforme a população envelhece e mais
de transição: o tempo que faltaria para que o trabalhador se pessoas se aposentam, mesmo que as contribuições não
aposentar pela regra atual seria acrescido de 50%. arrecadem na mesma proporção.
Por exemplo: para um homem de 50 anos e 34 de Em 10 anos, os gastos do sistema passaram de R$ 146
contribuição, será aplicado 50% sobre o tempo que restava bilhões para R$ 436 bilhões até 2015, um aumento de quase
para se aposentar. Nesse caso, seria acrescido 50% sobre um 200%, segundo dados da Previdência Social. Essas despesas
ano que faltava, que resultará em 1 ano e meio a mais de subiram 4,3% acima da inflação entre 2011 e 2015. Até 2037,
contribuição. os gastos com aposentadoria e pensões vão chegar a R$ 2,6
trilhões, apontam as projeções do próprio governo que
Veja abaixo as principais mudanças: consideram as regras atuais da Previdência Social.
Outros gastos sociais cresceram mais que a Previdência
nos últimos anos, mas eles têm pesos menores no Orçamento
não geram a mesma pressão sobre as contas públicas. Um
exemplo é o Bolsa Família, que nos últimos quatro anos teve
despesas 7,1% acima da inflação, mas representa apenas 2,3%
dos gastos primários do governo.

Regras da previdência rural


Atualmente, as regras de aposentadoria para quem
trabalha no campo são diferentes das do trabalhador da
cidade. Enquanto o trabalhador da cidade contribui com um
valor fixo no mês, o produtor rural paga um percentual sobre
a receita bruta de sua produção, que é variável. Pela regra
atual, ele não precisa cumprir um tempo mínimo de
contribuição e se aposenta por idade.
Para receber 100% do benefício, os urbanos precisam
obter a soma da idade e tempo de contribuição de 85, no caso
das mulheres, e 95, no caso dos homens. O tempo mínimo de
contribuição para elas é de 30 anos e, para eles, de 35 anos.
Os trabalhadores rurais continuam com o direito de se
aposentar aos 60 anos (homens) e 55 anos (mulheres), mesmo
sem ter cumprido a exigência por tempo de contribuição feita
ao trabalhador urbano. Em 2015, as contribuições rurais
representaram apenas 2% do total recolhido à Previdência,
mas os gastos chegaram a 22,4% dos pagamentos.
O resultado da previdência urbana é positivo desde 2009,
fruto do aumento do emprego e da menor informalidade. Em
2015, o superávit foi bem menor devido à queda na
arrecadação. A previdência rural sempre foi deficitária. O
rombo se intensificou a partir de 2007, passando de quase R$
20 bilhões, há dez anos, para mais de R$ 90 bilhões em 2015.

CULTURA
Pensão por morte
Portela é a campeã do carnaval 2017 do Rio após 33
Foram anunciadas também mudanças no valor pago à
anos de jejum27
viúva ou ao viúvo, que passaria a ser de 50% do valor do
A Portela quebrou um jejum de 33 anos e é a grande
benefício recebido pelo contribuinte que morreu com um
campeã do carnaval do Rio de 2017. No segundo ano com o
adicional de 10% para cada dependente do casal.
carnavalesco Paulo Barros, a escola de Madureira desfilou na
avenida as lendas dos rios. Agora com 22 títulos, a Portela é a

26 06/12/2016 Fonte: http://g1.globo.com/economia/noticia/veja-como- 27 01/03/2017 – Fonte: http://g1.globo.com/rio-de-


e-e-o-que-muda-na-previdencia.ghtml janeiro/carnaval/2017/noticia/portela-e-a-campea-do-carnaval-2017-do-
rio.ghtml

Atualidades e Deveres dos Servidores Públicos 15


Apostila Digital Licenciada para Alice Caroline Guarino dos Santos - alice.guarino@hotmail.com (Proibida a Revenda)
APOSTILAS OPÇÃO

escola que mais vezes foi campeã. A Mocidade Independente a revelação dos vencedores do Oscar há 83 anos, está "sob
de Padre Miguel ficou em segundo lugar. A Acadêmicos do revisão".
Salgueiro foi a terceira colocadas. De indicado a vencedor e apresentador
A presidente da Academia elogiou os atores Warren Beatty
e Faye Dunaway, e o apresentador Jimmy Kimmel, por terem
contornado a situação. E também louvou o produtor Jordan
Horowitz, de "La la land", que, em suas palavras, "foi de
indicado a vencedor e apresentador em questão de minutos".
Boone Isaacs lamentou ainda que os "últimos 90 segundos"
da transmissão acabaram ofuscando o que ela descreveu como
"o show mais brilhante e maravilhoso".

Centenas de obras de arte estão em risco após


terremoto na Itália30
Enquanto continuam as buscas por sobreviventes entre os
escombros do terremoto de magnitude 6.2 que atingiu a Itália,
um outro alerta veio à tona: o patrimônio histórico e cultural
da região corre perigo. Segundo um levantamento divulgado
nesta quinta-feira (25/08/16) pelo Ministério para os Bens
Culturais, 293 bens históricos e culturais foram destruídos ou
estão seriamente ameaçados em um raio de 20 km do
epicentro do terremoto. A maior parte dos danos aconteceu
em Amatrice, onde basílicas, igrejas, bibliotecas, portas
romanas, torres e pontes de grande valor histórico viraram
pó.Há anos ele pesquisa e escreve sobre a região, onde tinha
uma casa que também desabou no terremoto. Até o momento,
sabe-se que na cidade foram gravemente atingidas pelo
terremoto a basílica de São Francisco, a Igreja de Santo
Agostinho e o museu cívico. A cidade vizinha de Accumoli
também abriga portas medievais e palácios, e sua história
Nenhuma escola do Grupo Especial será rebaixada no remonta ao século 11. Em 1643, a vila foi comprada pela
carnaval desse ano28 família Medici (mecenas de grandes artistas como
Nenhuma escola do Grupo Especial será rebaixada para a Michelangelo e Rafael) e mais tarde vendida ao reino de
Série A no carnaval desse ano no Rio de Janeiro. A decisão foi Nápoles.
tomada pela Liga Independente das Escolas de Samba do Rio
(Liesa) após uma reunião de emergência na tarde desta Próximos passos
quarta-feira (1/3). O Ministério para Bens Culturais ativou quatro unidades
O que teria motivado a decisão foram os acidentes de crise para avaliar os danos e prevenir maiores estragos.
envolvendo os carros da Paraíso do Tuiuti e da Unidos da Outro risco também seriam possíveis roubos e danos,
Tijuca, que deixaram pelo menos 35 pessoas feridas nos principalmente a obras de arte dentro das igrejas. Em gesto de
desfiles do Grupo Especial entre a noite de domingo (26/02) e solidariedade, o governo convidou os italianos a visitarem
a madrugada de terça-feira (28/02) de carnaval. A decisão museus neste domingo, 28, quando toda a arrecadação dos
teria ocorrido em consenso com todos os presidentes das ingressos será doada aos esforços de reparação dos bens
agremiações. Em função disso, no carnaval de 2018 o Grupo artísticos atingidos pelo terremoto.
Especial terá 13 escolas desfilando, das quais duas serão
rebaixadas para a Série A do carnaval. Uma escola da Série A 'Aquarius', de Kleber Mendonça Filho, é selecionado
sobe para o Grupo Especial este ano. para festival de NY31
"Aquarius", filme do diretor pernambucano Kleber
Responsáveis por erro no Oscar 2017 nunca mais Mendonça Filho protagonizado por Sonia Braga, estará na 54ª
trabalharão na premiação, diz Academia de filmes29 edição do Festival de Cinema de Nova York, um dos mais
Os dois funcionários da empresa de auditoria PwC importantes dos Estados Unidos. A lista de 25 filmes da mostra
responsáveis pelo erro na entrega do prêmio de melhor filme principal foi anunciada nesta terça-feira (09/08/2016) e inclui
do Oscar 2017 nunca mais irão trabalhar na premiação, disse importantes nomes da indústria cinematográfica, entre eles
nesta quarta-feira (1º) a presidente da Academia de Artes e Pedro Almodóvar, que exibirá o seu "Julieta", e Ken Loach, com
Ciências Cinematográficas, Cheryl Boone Isaacs, à AP. "I, Daniel Blake", vencedor da Palma de Ouro em Cannes neste
Na entrevista, a presidente afirmou que Brian Cullinan, ano.
quem entregou o envelope errado que resultou no anúncio
equivocado de "La la land: Cantando estações" como melhor
filme, estava distraído nos bastidores. E que ele e sua parceira,
Martha Ruiz, foram permanentemente removidos de qualquer
participação com a Academia de filmes.
Cheryl Boone Isaacs disse também que a relação da
Academia com a empresa PwC, que faz a contagem de votos e

28 01/03/2017 – Fonte: http://g1.globo.com/rio-de- 30 29/08/2016 – Fonte:


janeiro/carnaval/2017/noticia/nenhuma-escola-do-grupo-especial-sera- http://g1.globo.com/mundo/noticia/2016/08/centenas-de-obras-de-arte-
rebaixada-no-carnaval-desse-ano.ghtml estao-em-risco-apos-terremoto-na-italia.html
29 01/03/2017. Fonte: http://g1.globo.com/pop- 31 10/08/2016 – Fonte: http://g1.globo.com/pop-
arte/oscar/2017/noticia/responsaveis-por-erro-no-oscar-2017-nunca-mais- arte/cinema/noticia/2016/08/aquarius-de-kleber-mendonca-filho-e-
trabalharao-na-premiacao-diz-academia-de-filmes.ghtml selecionado-para-festival-de-ny.html

Atualidades e Deveres dos Servidores Públicos 16


Apostila Digital Licenciada para Alice Caroline Guarino dos Santos - alice.guarino@hotmail.com (Proibida a Revenda)
APOSTILAS OPÇÃO

O abraço da serpente' é vencedor nos Prêmios Platino nenhum projeto aprovado por meio do incentivo fiscal: quem
com 7 estatuetas32 decide o financiamento são as empresas ou cidadãos que
O filme colombiano "O abraço da serpente" (2015), de Ciro patrocinam ou doam aos projetos. A decisão não é do governo.
Guerra, foi o grande vencedor da terceira edição dos Prêmios Todo projeto cultural, de qualquer artista, produtor e
Platino de Cinema Ibero-Americano, ao ganhar em sete das agente cultural brasileiro, pode se beneficiar desta lei e se
oito categorias às quais concorria, entre elas as de melhor candidatar à captação de recursos de renúncia fiscal. A análise
filme de ficção e direção (veja, abaixo, a lista de ganhadores). dos projetos submetidos atesta a adequação do projeto aos
A cerimônia de entrega aconteceu neste domingo parâmetros legais do mecanismo, sempre através de critérios
(24/07/2016) no Centro de Convenções de Punta del Este, no objetivos, o que impede quaisquer tipos de preferência ou
Uruguai. O filme brasileiro "Que horas ela volta?", de Anna censura, independentemente do governo que esteja em
Muylaert levou um prêmio fora de competição. atuação. O posicionamento político, artístico, estético ou
qualquer outro relacionado à liberdade de expressão não é
Nova regra da Lei Rouanet pode gerar prejuízos aos objeto de análise, sendo que a Lei veta expressamente
cofres públicos, diz TCU33 "apreciação subjetiva quanto ao seu valor artístico ou
Medida tomada pouco antes do afastamento da presidenta cultural".
Dilma Rousseff para acelerar a análise da prestação de contas O ciclo de aprovação de projetos inclui diversas etapas e se
de projetos da Lei Rouanet pode gerar prejuízos, conforme finaliza com a avaliação da Comissão Nacional de Incentivo à
entendimento do Tribunal de Contas da União (TCU). Pela Cultura (CNIC), que é formada com paridade de membros do
medida, são dispensados de análise financeira os projetos cujo poder público e da sociedade civil, todos de comprovada
valor captado seja menor ou igual a R$ 600 mil. De acordo com experiência em seus setores de atuação. Todas as decisões são
o TCU, tais projetos correspondem a 88,39% dos beneficiados públicas, tomadas com total transparência, estando os dados
pelo programa e a aproximadamente R$ 2,3 bilhões, ou de todos os projetos disponíveis para consulta pública através
41,68% do montante total dos recursos captados, do Sistema de Apoio às Leis de Incentivo à Cultura
considerando o período de 2007 a 2011. (Salic):http://sistemas.cultura.gov.br/salicnet.
A análise financeira é a segunda etapa da análise da Os servidores públicos e representações sociais envolvidas
prestação de contas de projetos financiados por meio de neste processo merecem respeito pela execução de seus
incentivos fiscais. Nessa etapa, é analisada a regularidade das trabalhos. Também merecem esse respeito as instituições,
demonstrações financeiras, dos documentos comprobatórios grupos, artistas e demais profissionais da cultura que
das despesas e do nexo causal com o objeto pactuado. A trabalham nos projetos realizados com o fomento do incentivo
primeira etapa é uma análise técnica da execução do projeto, fiscal. Os citados ataques recaem também sobre estas pessoas.
do alcance dos objetivos e da finalidade, proporcionais à Afirmar que o posicionamento político de artistas é afetado
captação de recursos para o projeto cultural. Para o TCU, a por "trocas de favores" é uma acusação grave, sem qualquer
dispensa da segunda etapa pode resultar em potencial risco de fundamento na história da Lei Rouanet. Esta calúnia resulta de
danos ao erário porque analisar apenas a execução do projeto uma compreensão que parte do pressuposto de que a
não comprova o adequado emprego de recursos federais cidadania das pessoas pode sempre estar à venda. Mais de 3
envolvidos. mil projetos são apoiados a cada ano pela Lei Rouanet.
A medida consta na Portaria 58, de 10 de maio de 2016 do Nenhum deles precisou ou precisará apoiar o atual governo ou
Ministério da Cultura, publicada ainda na gestão de Juca nenhum outro para utilizar deste direito garantido por
Ferreira. Ao assumir a pasta, o ministro Marcelo Calero enviou princípios republicanos
uma consulta ao TCU sobre a regularidade do normativo. A
intenção era rever, se necessário, a medida e eliminar o CIÊNCIA E TECNOLOGIA
"grande estoque de prestações de contas ainda pendentes
desde 2011", como consta no relatório do TCU. Dados de junho Introdução ao Conceito de Big Data35
do Ministério da Cultura mostram que havia 12.109 projetos Com a evolução das aplicações e a necessidade do ser
no passivo da Lei Rouanet, propostas aprovadas entre 1992 e humano pelas informações, mais e mais aplicações vem
2011, que não tiveram as contas examinadas. surgindo, deixando o homem cada vez mais refém da
informação. Para isso basta lembrar-nos da evolução da
Nota de esclarecimento34 relação homem-computador em que num passado existiam
O Ministério da Cultura (MinC) esclarece que são muitas pessoas para uma máquina (como o mainframe), em
infundados e caluniosos os ataques que têm sido feitos a seguida uma pessoa por máquina (o caso do computador
artistas que estão apresentando seus pontos de vista sobre a pessoal) e nos tempos atuais em que cada pessoa possui várias
atual situação política do país. A liberdade de expressão é máquinas (notebook, ultrabook, PC, tablet, smartphone, no
direito fundamental garantido pela Constituição Brasileira e futuro óculos, geladores, etc.). Com tantos aplicativos
tem especial relevância no campo artístico-cultural. A surgindo, as informações são geradas exponencialmente, com
concessão de incentivo fiscal a projetos culturais é uma isso a capacidade de gerenciar tantas informações se torna
possibilidade disponível a qualquer cidadão brasileiro que primordial para as aplicações atuais. Esse mesmo crescimento
atua na cultura. de dados acontece nos aplicativos empresariais com
Sancionada em 1991 e portanto já executada por diversos crescimento anual de 60%. Estima-se que uma empresa com
governos, a Lei Rouanet implementou o mecanismo de mil funcionários gera anualmente 1000 terabytes, sem falar
incentivo fiscal como principal fonte de financiamento à que essa quantidade tende a aumentar cinquenta vezes até
cultura no Brasil. Trata-se de estímulo à participação do 2020.
mercado e da sociedade no aporte de recursos a projetos Com o recém surgimento do bigdata, a primeira
culturais, podendo posteriormente deduzi-los de seu imposto dificuldade é encontrar o seu conceito. É possível vê-lo de
de renda. Não há, portanto, repasse direto de recursos para maneiras totalmente divergentes em cada blog que se lê, ou

32 25/07/2016 – Fonte: http://g1.globo.com/pop- 34 Fonte: http://www.cultura.gov.br/banner-2/-


arte/cinema/noticia/2016/07/o-abraco-da-serpente-e-vencedor-nos- /asset_publisher/0u320bDyUU6Y/content/nota-de-esclarecime-13/10883
premios-platino-com-7-estatuetas.html 35 Fonte: http://www.devmedia.com.br/introducao-ao-conceito-de-big-
33 02/08/2016 – Fonte: data/27066
http://agenciabrasil.ebc.com.br/educacao/noticia/2016-08/nova-regra-da-lei-
rouanet-pode-gerar-prejuizos-ao-governo-diz-tcu

Atualidades e Deveres dos Servidores Públicos 17


Apostila Digital Licenciada para Alice Caroline Guarino dos Santos - alice.guarino@hotmail.com (Proibida a Revenda)
APOSTILAS OPÇÃO

seja, se você ler 10 materiais sobre bigdata, provavelmente linguagem Java, conhecer o Java EE 6, muito em breve o Java 7,
cada um trará conceitos diferentes. Dentre as matérias, ao se além do JDK 7 trará velocidade de desenvolvimento,
tirar um mínimo comum se verá que o bigdata, na sua raiz, fala produtividade e um melhor gerenciamento de memória.
em tratar um grande volume de dados com grande velocidade. Assim pode-se concluir que o conceito do bigdata é
No entanto, se repara que essa definição é bastante abstrata relativamente muito fácil, mesmo sendo divergente em várias
pelo simples fato de que para uma pessoa A, por exemplo, um fontes, que é gerenciar um grande volume de memória em alta
grande volume seja um gigabyte e para uma pessoa B um velocidade. O mais difícil certamente é chegar nesse objetivo,
grande volume seja um terabyte e o mesmo pode acontecer ao já que para isso é necessário conhecer uma gama de
se referenciar a velocidade e o tempo de resposta de uma ferramentas, frameworks, metodologias, tipos de bancos como
requisição. NOSQL, SQL e NewSQL, cache, serviço de indexação de buscas,
Assim, o grande desafio do bigdata é estar administrando tipos de escalabilidades, etc. Apesar de ainda ser muito
um grande volume de dados e minerando informações em um discutida nas universidades, a normalização foi desenvolvida
menor tempo de requisição. Com o grande volume de dados, em 1970 (vale lembrar que os servidores da época possuíam
fazer com que a aplicação cresça à medida que é necessário é 16kb de memória principal e 800kb de armazenamento e os
uma ótima estratégia, assim, uma escalabilidade vertical (em celulares atuais são muitas vezes mais rápidos que estes
que se aumenta o poder do hardware, como aumento de servidores), ou seja, não é certo afirmar que os aplicativos, os
memória e de processamento de uma única máquina) ou hardwares, as necessidades de hoje são os mesmos daquela
horizontal (em que se aumenta a quantidade de máquinas) época, e com isso entender que esse padrão nem sempre é
deve ser analisada. Apesar de ser mais complexa, a válido.
escalabilidade horizontal acaba sendo muito barata, sem falar
de ser mais fácil de crescer ou diminuir os recursos por Após um ano isolado em vulcão, grupo diz ser
demanda. possível viver em Marte36
Para armazenar as informações com a escalabilidade Um grupo de seis pessoas completou uma missão da Nasa
horizontal, os bancos NOSQL são uma ótima estratégia. Vale na qual simularam como seria viver em Marte por um ano.
lembrar que o banco NOSQL significa não apenas not only e Eles conviveram em isolamento quase total, em um
não SQL. Uma diferença entre os bancos NOSQL e SQL é que o ambiente fechado no Havaí, nos Estados Unidos, sem acesso ao
primeiro possui uma grande variedade de bancos e cada um mundo exterior, comida fresca ou privacidade.
com características diferentes. Em termo de arquitetura, os Especialistas estimam que uma missão humana ao planeta
bancos NOSQL podem ser distribuídos ou não, embora sejam vermelho levaria de um a três anos. O grupo contava com um
mais populares do tipo distribuído. Sua forma de francês que é astrobiólogo (estuda a origem e o futuro da vida
armazenamento pode ser apenas em memória, apenas em no Universo), um físico alemão e quatro americanos: Carmel,
disco rígido ou configurável (vale apena lembrar que banco um piloto, um arquiteto e um jornalista. Durante a simulação,
apenas na memória são muito rápidos, no entanto são o grupo teve de viver com poucos recursos, usar roupas de
volúveis, já os somente no HD tem informações permanentes, astronauta ao sair e trabalhar para evitar conflitos pessoais.
porém o I/O é muito alto). Outra característica divergente Entre as comidas disponíveis estavam queijo ralado e atum
entre os bancos NOSQL está na forma do armazenamento que enlatado. Após os 12 meses nesta situação, os seis
são: chave-valor, documento, família de coluna e grafos. Os participantes afirmaram estar confiantes.
bancos NOSQL costumam ser muito rápidos na leitura e na "Posso dar minha impressão pessoal, que é a de que a
escrita, no entanto, possuem uma grande deficiência por parte missão para Marte no futuro próximo é realista", afirmou o
das buscas. Estas normalmente são feitas apenas pela chave, francês Cyprien Verseux, que integra o grupo. "Acredito que os
para isso usar um serviço para terceirizar o serviço pode ser obstáculos tecnológicos e psicológicos podem ser superados."
uma boa ideia, como o framework Lucene. Mas a comandante da missão, a cientista Carmel Johnston,
Mesmo realizando estratégias de buscas terceirizadas ou disse que não ter privacidade por um ano não foi fácil.
buscando pelo id, se a informação não estiver na memória "É como dividir o quarto com colegas que estão sempre
principal pode haver uma demora para recuperar essa presentes, e você nunca consegue se distanciar deles - tenho
informação. Assim, ter um dispositivo de acesso rápido pode certeza de que todo mundo consegue imaginar como é
ser uma melhor opção, entra aqui o uso do cache. Ao se optar conviver assim com qualquer pessoa." O americano Tristan
pelo cache deve-se levar em conta os dois maiores desafios: Bassingthwaghte, doutor em arquitetura pela Universidade do
Ter informação: o ato de esquentar a informação é Hawaí, elogiou a missão.
extremamente importante para esse mecanismo, já que não "Essa pesquisa é vital no momento em que for preciso
adianta existir um cache se o mesmo não possuir nenhuma escolher a equipe (que irá a Marte), descobrir como as pessoas
informação. Para isso ele pode ser esquentado de várias se comportam de verdade em diferentes tipos de missões e
formas (ao iniciar uma aplicação, por demanda, por sessão do lidar com o fator humano em viagens espaciais, colonizações
usuário) que deve ser definido com o aplicativo em questão. ou o que mais estiver sendo analisado", disse o arquiteto.
Matar o cache: para o cache é necessário que existam
apenas informações atuais, assim é importante que os dados Primeiro táxi sem motorista começa a ser testado em
antigos sejam mortos e trocados por informações mais atuais. Cingapura37
Um outro aspecto no bigdata, que não é muito tratado, está Os primeiros táxis que dispensam motoristas começaram
relacionado à velocidade da modelagem além da velocidade no a ser testados nas ruas de Cingapura nesta quinta-feira
desenvolvimento de software. Um exemplo muito interessante (25/08/16), informou a agência Reuters.
é o Twitter que viu muito usuários usando a hashtag (o '#' A empresa nuTonomy convidou um grupo de pessoas a
adicionado com uma palavra) e em pouco tempo teve que baixar o aplicativo no celular para chamar os "robô-táxis", que
realizar pesquisas através dos mesmos. são modelos elétricos da Mitsubishi equipados com sistema
Assim podemos verificar que o bigdata não apenas está capaz de guiar sozinho.
relacionado apenas à velocidade de requisição, mas também Por enquanto, um engenheiro acompanhará o trajeto no
em desenvolvimento, então conhecer bem o negócio além de banco do motorista, para coletar informações e reagir caso
várias ferramentas poderá ser extremamente importante. Na alguma coisa saia errada.

36 29/08/16 – Fonte: http://g1.globo.com/ciencia-e- 37 25/08/16 – Fonte: http://g1.globo.com/carros/noticia/2016/08/taxi-

saude/noticia/2016/08/apos-um-ano-isolado-em-vulcao-grupo-diz-ser- sem-motorista-comeca-ser-testado-em-cingapura.html
possivel-viver-em-marte.html

Atualidades e Deveres dos Servidores Públicos 18


Apostila Digital Licenciada para Alice Caroline Guarino dos Santos - alice.guarino@hotmail.com (Proibida a Revenda)
APOSTILAS OPÇÃO

A nuTonomy já testa o sistema de forma privada desde Cerca de mil anos mais tarde, de acordo com esta teoria, os
abril e espera ter pelo menos 100 "robô-táxis" operando primeiros seres humanos da era do gelo se moveram através
comercialmente na cidade até 2018. desta passagem interior para fundar novas culturas no sul.
"Este é verdadeiramente um momento na história que vai
mudar como as cidades são construídas, como olhamos ao Uma nova história
nosso redor", afirmou à Reuters o executivo Doug Parker, da Entre eles estava o povo de Clóvis, que apareceu pela
nuTonomy. A empresa é mais uma que pretende lançar um primeira vez nos registros arqueológicos mais de 13 mil anos
serviço de carros autonômos, em uma corrida que fez as atrás. Este enredo pressupõe, naturalmente, que estes
grandes montadoras de automóveis se aliarem a empresas de primeiros povos encontraram alimentos ao longo do caminho.
tecnologia. Na quarta-feira (24/08/16), a Delphi e a Mobileye E é aí que a teoria cai por terra, de acordo com Mikkel
anunciaram que devem lançar uma plataforma pronta para Pedersen, pesquisador do Centro de GeoGenetics da
carros autônomos em 2019. Já a Ford espera ter uma produção Universidade de Copenhague e autor principal do estudo.
em larga escala de carros sem volante e pedais em 2021. "O primeiro momento em que o corredor se abre para a
O Uber anunciou recentemente uma parceria com a Volvo migração humana é 12,6 mil anos atrás", disse à AFP.
e deve colocar uma frota de carros sem motorista para testes Embora a passagem possa ter estado livre, "não havia
nos Estados Unidos em breve - também com a supervisão de absolutamente nada antes desta data no ambiente em volta -
"pilotos" dentro. nem plantas, nem animais".
Nada, em outras palavras, que teria permitido aos seres
Estudo redesenha rota de passagem de humanos para humanos se alimentarem durante um longo e árduo caminho
as Américas38 entre penhascos de gelo.
As primeiras pessoas a chegar às Américas não poderiam Outras pesquisas que mostraram que os seres humanos
ter passado pelo corredor de camada de gelo - onde hoje fica o poderiam ter chegado às Américas pelo menos 14,5 mil anos
Estreito de Bering - que se pensava que tinha sido o ponto de atrás - e talvez alguns milhares de anos antes disso - já tinha
entrada de humanos para os continentes de acordo com um começado a enfraquecer a hipótese do corredor de camada de
estudo publicado quarta-feira (10/08/2016). gelo, forçando especialistas a olharem mais de perto a
possibilidade de uma rota costeira.
Pedersen e colegas agora parecem ter fechado
definitivamente a porta para a teoria da rota pelo interior.
Os métodos inovadores que eles utilizaram para
reconstruir o ecossistema no final da idade do gelo foram
cruciais para a descoberta. Em vez de procurar vestígios de
DNA de plantas ou animais específicos enterrados em
sedimentos - a abordagem padrão - a equipe de Pedersen usou
o que é chamado de método de "shotgun" (espingarda),
catalogando todas as formas de vida em uma determinada
amostra.
"Tradicionalmente, estivemos procurando genes
específicos de uma ou de várias espécies", explicou.
"Mas a abordagem 'shotgun' realmente nos deu uma visão
fantástica sobre todas as diferentes" cadeias alimentares.
"Camadas, desde bactérias e fungos até plantas superiores e
mamíferos".

Janela para mundos antigos


Os pesquisadores escolheram extrair núcleos de
Mapa mostra a abertura das rotas de migração para a sedimentos do que teria sido uma passagem estreita no
América do Norte, de acordo com o estudo (Foto: Mikkel corredor interior, uma área parcialmente coberta hoje pelo
Winther Pedersen/Divulgação) Charlie Lake, na província canadense de British Columbia.
A equipe fez datação por radiocarbono e recolheu
Mais provavelmente, os pioneiros humanos do Novo amostras enquanto estava na superfície do lago congelado no
Mundo - provavelmente cerca de 15 mil anos atrás - inverno. Até 12,6 mil anos atrás, o ambiente era quase
avançaram ao longo de uma costa do Pacífico, suficientemente inteiramente desprovido de vida, descobriram os cientistas.
livre de gelo para suportar a flora e a fauna necessárias para Mas o ecossistema evoluiu rapidamente, dando lugar
sustentar a vida. dentro de algumas centenas de anos a uma paisagem de grama
A rota e o período exatos desta migração inaugural e sálvia, logo povoada por bisões, mamutes, coelhos e
permanecem sendo conjecturas, disseram os pesquisadores. ratazanas. Mil anos depois, o ecossistema teve uma nova
Mas o que é certo, de acordo com as descobertas relatadas transição, está vez para um ambiente com árvores, alces e
na revista científica "Nature", é que a versão clássica sobre águias.
aquela passagem está errada. Durante décadas, os cientistas As descobertas abrem "uma janela para os mundos
trabalharam com a seguinte teoria. Cerca de 14,5 mil anos antigos" e são uma pedra angular em uma "grande
atrás, um corredor norte-sul de 1.500 km se abriu entre a reavaliação" de como os seres humanos chegaram à América,
camada de gelo Cordilleran - que cobria aproximadamente o disse Suzanne McGowan, da Universidade de Nottingham,
que é hoje a província canadense de Colúmbia Britânica - e a comentando o estudo na Nature. Elas também tornaram a
muito maior camada de gelo Laurentide, que ocupava o resto teoria da passagem costeira muito mais provável, acrescentou.
do Canadá. A era do gelo foi lentamente desaparecendo e
dando lugar a uma ponte de terra entre a Eurásia e o Alasca.

38 11/08/2016 – Fonte: http://g1.globo.com/ciencia-e-


saude/noticia/2016/08/estudo-redesenha-rota-de-passagem-de-humanos-
para-americas.html

Atualidades e Deveres dos Servidores Públicos 19


Apostila Digital Licenciada para Alice Caroline Guarino dos Santos - alice.guarino@hotmail.com (Proibida a Revenda)
APOSTILAS OPÇÃO

Outros estudiosos concordam. China finaliza o maior radiotelescópio do mundo41


"Se já houve um corredor livre de gelo durante o Último O maior radiotelescópio do mundo está prestes a entrar
Máximo Glacial, não era nas regiões do interior do norte da em funcionamento.
América do Norte, mas ao longo da costa noroeste", escreveu A China colocou a peça final do aparelho no último
James Dixon, da Universidade do Novo México, em um estudo domingo(03/07/2016) e, em setembro, ele começará a
recente. funcionar, segundo a agência estatal de notícias Xinhua.
O telescópio tem o tamanho de 30 campos de futebol (500
Cientistas descobrem objeto misterioso que se metros de diâmetro), é considerado o maior do mundo e sua
esconde atrás de Netuno39 construção custou US$ 180 milhões.
Um grupo internacional de pesquisadores descobriu um Com ele, a China espera conseguir explorar o espaço em
pequeno corpo celeste por trás de Netuno com uma órbita que busca de vida extraterrestre.
não é típica do sistema solar, informa a edição da revista New O subdiretor nacional de Observação Astronômica da
Scientist. Academia Chinesa de Ciências, Zheng Ziaonian, anunciou que,
Conforme estimativas dos especialistas, o achado a partir de agora, os cientistas começam o processo de testes
enigmático é 160 mil vezes menos brilhante que Netuno. O para detectar e corrigir problemas no telescópio.
diâmetro do corpo é cerca de 200 quilômetros. O Telescópio de Abertura Esférica de 500 metros (ou Fast,
Os cientistas assinalam que o corpo celeste possui uma na sigla em inglês) fica na Província de Guizhou, sudoeste da
órbita muito estranha: objeto gira em torno do Sol de modo China, uma área favorável para a atividade de pesquisa
quase perpendicular relativamente aos outros planetas. O astronômica pois o relevo dos arredores do radiotelescópio o
achado foi batizado de Niku, que, em chinês, significa protege naturalmente de perturbações eletromagnéticas.
"rebelde", "indócil".
O descobrimento foi feito com ajuda do sistema de Hidrogênio
telescópios Pan-Starrs, no Havaí. O Fast será usado para tentar descobrir a existência de
hidrogênio neutro em galáxias distantes e também pulsares
Tecnologia: uso de drones dispara e equipamentos distantes ("bolas" de nêutrons muito magnetizadas).
prometem mudar o mundo40 Além disso, o radiotelescópio também aumenta a
- Drones são veículos não tripulados, controlados à possibilidade de detectar ondas gravitacionais de baixa
distância. frequência.
- Esses equipamentos já são usados há muito tempo na O Fast vai tomar o lugar do Observatório de Arecibo, em
área militar, mas nos últimos anos, viraram uma febre para as Porto Rico, que mede 300 metros diâmetro, como o maior
mais diversas finalidades. telescópio do mundo.
- A segurança do espaço aéreo e a garantia de privacidade Sua construção ocorreu em tempo recorde: apenas cinco
da população são questões que preocupam a população em anos.
relação aos drones. E, apesar de as autoridades chinesas afirmarem que seu
Eles são leves, voam em uma altura que os helicópteros não programa espacial tem fins pacíficos, o Departamento de
alcançam, entram em espaços considerados pequenos e fazem Defesa dos Estados Unidos afirmou que o objetivo real da
manobras com facilidade. Mas não são aviões. As aeronaves China é "evitar que seus adversários usem dispositivos
remotamente pilotadas (RPAs), mais popularmente espaciais em uma crise".
conhecidas como drones, são máquinas voadoras operadas a O programa espacial do país se transformou em uma das
distância por controle remoto. prioridades do governo chinês. Em 2018 o país deve lançar um
Os drones aéreos surgiram durante a Segunda Guerra "módulo central" para sua primeira estação espacial.
Mundial e buscavam evitar mortes de soldados em operações
de risco. Durante a Guerra Fria, foram usados pelas Forças MEIO AMBIENTE
Armadas dos Estados Unidos em operações de espionagem. Na
Guerra do Iraque, em versões modernizadas, foram usados Estudo questiona capacidade das árvores de absorver
para bombardeios. gás carbônico42
Em 2014, o presidente Barack Obama anunciou que os EUA Não há quem não tenha aprendido na escola que uma das
passariam a utilizar drones em operações antiterroristas para funções mais importantes das árvores é ajudar a regular o gás
combater grupos jihadistas em países como a Síria, carbônico na atmosfera.
Afeganistão, Somália e Iêmen. Na última década, a frota de Mas um novo estudo realizado na Austrália questiona as
drones dos EUA aumentou de 50 para mais de 7 mil aeronaves, estimativas sobre a quantidade de dióxido de carbono (CO2)
incluindo minidrones e drones sem armas, que podem ser que árvores poderiam retirar da atmosfera.
pilotados a partir de bases norte-americanas. Segundo cientistas da Universidade da Western Sydney
Cientistas apostam em uma terceira revolução na University, árvores não conseguem armazenar tanto dióxido
tecnologia bélica (depois da pólvora e das armas atômicas) na de carbono (CO2) quando se pensava.
qual a guerra passa a se fazer por automação e inteligência Os experimentos põem em dúvida a tese de que níveis
artificial, com drones terrestres, marítimos e subterrâneos. crescentes de CO2 na atmosfera intensificariam o
Seu uso militar promete “uma guerra sem combate”. Segundo desenvolvimento (o chamado efeito de fertilização) das
David Deptula, um oficial da força aérea norte-americana, a árvores - que se "alimentam" do carbono retirado do ar através
verdadeira vantagem deles é “projetar poder sem projetar da fotossíntese - aumentando assim a absorção do gás.
vulnerabilidade”, ou seja, não é necessário enviar tropas e O estudo foi publicado na revista científica "Nature Climate
coloca-las em risco. Change".
Segundo os cientistas, a pesquisa também indica que, por
este motivo, levantamentos internacionais sobre o

39 12/08/2016 – Fonte: 41 05/07/2016


http://agenciabrasil.ebc.com.br/internacional/noticia/2016-08/cientistas- http://g1.globo.com/ciencia-e-saude/noticia/2016/07/china-finaliza-o-
descobrem-objeto-misterioso-que-se-esconde-atras-de-netuno maior-radiotelescopio-do-mundo.html
40 11/07/2016 – Fonte: http://vestibular.uol.com.br/resumo-das- 42 07/03/2017. Fonte: http://g1.globo.com/natureza/noticia/estudo-

disciplinas/atualidades/tecnologia-uso-de-drones-dispara-e-equipamentos- questiona-capacidade-das-arvores-de-absorver-gas-carbonico.ghtml
prometem-mudar-o-mundo.htm

Atualidades e Deveres dos Servidores Públicos 20


Apostila Digital Licenciada para Alice Caroline Guarino dos Santos - alice.guarino@hotmail.com (Proibida a Revenda)
APOSTILAS OPÇÃO

armazenamento de gás carbônico nas florestas podem estar Entenda como funciona o mercado de crédito de
superestimados. carbono44
Eles analisaram, por cinco anos, eucaliptos nos limites de A partir dos anos 2000, entrou em cena um mercado
Sydney que foram artificialmente bombardeados por altas voltado para a criação de projetos de redução da emissão dos
doses de dióxido de carbono lançadas por tubos a 28 metros gases que aceleram o processo de aquecimento do planeta.
de altura. Trata-se do mercado de créditos de carbono, que surgiu a
O objetivo do experimento era simular os níveis de CO2 na partir do Protocolo de Quioto, acordo internacional que
atmosfera previstos para 2050. estabeleceu que os países desenvolvidos deveriam reduzir,
A fotossíntese das árvores estudadas aumentou em 19% entre 2008 e 2012, suas emissões de Gases de Efeito Estufa
em três anos, mas – como elas crescem em uma região de solo (GEE) 5,2% em média, em relação aos níveis medidos em
pobre – a intensificação do processo não teve o impacto 1990. O Protocolo de Quioto criou o Mecanismo de
esperado no seu crescimento. Desenvolvimento Limpo (MDL), que prevê a redução
Como resultado, mais gás passou a ser absorvido, porém certificada das emissões. Uma vez conquistada essa
este aumento também não foi significativo, segundo os certificação, quem promove a redução da emissão de gases
cientistas, diante do intenso bombardeio de CO2 sofrido pelas poluentes tem direito a créditos de carbono e pode
plantas. comercializá-los com os países que têm metas a cumprir.
Os cientistas afirmaram que os resultados tendem a ser “O ecossistema não tem fronteira. Do ponto de vista
mais contundentes em solo de pior qualidade, com baixas ambiental, o que importa é que haja uma redução de emissões
taxas de fósforo ou nitrogênio. global”, ressalta o consultor de sustentabilidade e energia
Eles acrescentaram que isso poderia ter implicações renovável, Antonio Carlos Porto Araújo. Durante a última
importantes para outras regiões com florestas tropicais e Conferência do Clima (COP 17), realizada em 2011, na África
subtropicais. do Sul, as metas de Quioto foram atualizadas e ampliadas para
Este texto faz parte da série #SoICanBreathe, dedicada a cortes de 25% a 40% nas emissões, em 2020, sobre os níveis
problemas causados pela poluição de 1990 para os países desenvolvidos. “Isso pode significar um
fomento nas atividades de crédito de carbono que andavam
Desenvolvimento sustentável43 pouco atraentes”, disse Araújo, autor do livro “Como
Desenvolvimento sustentável é o modelo que prevê a comercializar créditos de carbono”. O Brasil ocupa a terceira
integração entre economia, sociedade e meio ambiente. Em posição mundial entre os países que participam desse
outras palavras, é a noção de que o crescimento econômico mercado, com cerca de 5% do total mundial e 268 projetos. A
deve levar em consideração a inclusão social e a proteção expectativa inicial era absorver 20%. O mecanismo incentivou
ambiental a criação de novas tecnologias para a redução das emissões de
gases poluentes no Brasil.
Gestão do Lixo
O lixo ainda é um dos principais desafios dos governos na Cálculo
área de gestão sustentável. No entanto, na última década, o A redução de emissões de Gases de Efeito Estufa (GEE) é
Brasil deu um salto importante no avanço para a gestão medida em toneladas de dióxido de carbono equivalente – t
correta dos resíduos sólidos. Segundo dados do Ministério do CO2e (equivalente). Cada tonelada de CO2e reduzida ou
Meio Ambiente, em 2000, apenas 35% dos resíduos eram removida da atmosfera corresponde a uma unidade emitida
destinados aos aterros. Em 2008, esse número subiu para pelo Conselho Executivo do MDL, denominada de Redução
58%. Além disso, o número de programas de coleta seletiva Certificada de Emissão (RCE). Cada tonelada de CO2e equivale
saltou de 451, em 2000, para 994, em 2008. Para regulamentar a 1 crédito de carbono. A ideia do MDL é que cada tonelada de
a coleta e tratamento de resíduos urbanos, perigosos e CO2 e não emitida ou retirada da atmosfera por um país em
industriais, além de determinar o destino final correto do lixo, desenvolvimento possa ser negociada no mercado mundial
o Governo brasileiro criou a Política Nacional de Resíduos por meio de Certificados de Emissões Reduzidas (CER). As
Sólidos (Lei n° 12.305/10), aprovada em agosto de 2010. nações que não conseguirem (ou não desejarem) reduzir suas
emissões poderão comprar os CER em países em
Créditos de Carbono desenvolvimento e usá-los para cumprir suas obrigações.
No mercado de carbono, cada tonelada de carbono que
deixa de ser emitida é transformada em crédito, que pode ser Consumo racional45
negociado livremente entre países ou empresas. É um modo de consumir capaz de garantir não só a
O sistema funciona como um mercado, só que ao invés das satisfação das necessidades das gerações atuais, como também
ações de compra e venda serem mensuradas em dinheiro, elas das futuras gerações. Isso significa optar pelo consumo de
valem créditos de carbono. bens produzidos com tecnologia e materiais menos ofensivos
Para isso é usado o Mecanismo de Desenvolvimento Limpo ao meio ambiente, utilização racional dos bens de consumo,
(MDL), que prevê a redução certificada das emissões de gases evitando-se o desperdício e o excesso e ainda, após o consumo,
de efeito estufa. Uma vez conquistada essa certificação, quem cuidar para que os eventuais resíduos não provoquem
promove a redução dos gases poluentes tem direito a degradação ao meio ambiente. Principalmente: ações no
comercializar os créditos. Por exemplo, um país que reduziu sentido de rever padrões insustentáveis de consumo e
suas emissões e acumulou muitos créditos pode vender este diminuir as desigualdades sociais. Adotar a prática dos três
excedente para outro que esteja emitindo muitos poluentes e 'erres': Redução, que recomenda evitar o consumo de
precise compensar suas emissões. O Brasil ocupa a terceira produtos desnecessários; Reutilização, que sugere que se
posição mundial entre os países que participam desse reaproveite diversos materiais; e Reciclagem, que orienta
mercado, com cerca de 5% do total mundial e 268 projetos. reaproveitar materiais, transformando-os e lhes dando nova
utilidade.

43 Fonte: 45Texto adaptado de http://www.wwf.org.br/natureza_


http://www.rio20.gov.br/sobre_a_rio_mais_20/desenvolvimento- brasileira/questoes_ambientais/desenvolvimento_sustentavel/
sustentavel.html
44 Fonte: http://www.brasil.gov.br/meio-ambiente/2012/04/entenda-
como-funciona-o-mercado-de-credito-de-carbono

Atualidades e Deveres dos Servidores Públicos 21


Apostila Digital Licenciada para Alice Caroline Guarino dos Santos - alice.guarino@hotmail.com (Proibida a Revenda)
APOSTILAS OPÇÃO

Aquecimento Global Conceito de desenvolvimento sustentável


O aquecimento global é uma consequência das alterações Usar os recursos naturais com respeito ao próximo e ao
climáticas ocorridas no planeta. Diversas pesquisas meio ambiente. Preservar os bens naturais e a dignidade
confirmam o aumento da temperatura média global. Conforme humana. É o desenvolvimento que não esgota os recursos,
cientistas do Painel Intergovernamental em Mudança do Clima conciliando crescimento econômico e preservação da
(IPCC), da Organização das Nações Unidas (ONU), o século XX natureza. Em Salvador, o TEDxPelourinho foi totalmente
foi o mais quente dos últimos cinco, com aumento de dedicado ao tema, e reuniu pensadores de diversas áreas e
temperatura média entre 0,3°C e 0,6°C. Esse aumento pode regiões do país para compartilhar suas experiências e mostrar
parecer insignificante, mas é suficiente para modificar todo como estão ajudando a transformar os centros urbanos em
clima de uma região e afetar profundamente a biodiversidade, locais planejados para serem ocupados por pessoas. As
desencadeando vários desastres ambientais. As causas do iniciativas incluem ciclovias, centros revitalizados,
aquecimento global são muito pesquisadas. Existe uma parcela instrumentos de participação coletiva e empoderamento dos
da comunidade científica que atribui esse fenômeno como um cidadãos, mais solidários, inclusivos, saudáveis, verdes e
processo natural, afirmando que o planeta Terra está numa humanas. Em relação a capital gaúcha, foi reconhecida pela
fase de transição natural, um processo longo e dinâmico, IBM com uma das 31 cidades do mundo merecedoras do
saindo da era glacial para a interglacial, sendo o aumento da prêmio Smarter Cities Challenge Summit. O reconhecimento
temperatura consequência desse fenômeno. No entanto, as veio graças ao projeto Cidade Cognitiva, que tem o objetivo de
principais atribuições para o aquecimento global são simular os impactos futuros sobre a vida do município, com as
relacionadas às atividades humanas, que intensificam o efeito obras e ações realizadas no presente demandadas pelo
de estufa através do aumento na queima de gases de orçamento participativo - sistema no qual a tomada de
combustíveis fósseis, como petróleo, carvão mineral e gás decisões sobre investimentos públicos é compartilhada entre
natural. A queima dessas substâncias produz gases como o sociedade e governo. Quem também fez progressos da área
dióxido de carbono (CO2), o metano (CH4) e óxido nitroso também foi o Rio de Janeiro. A sede das Olimpíadas de 2016
(N2O), que retêm o calor proveniente das radiações solares, tem investido em um moderno centro integrado de operações
como se funcionassem como o vidro de uma estufa de plantas, para antecipar e combater situações de calamidade. A
esse processo causa o aumento da temperatura. Outros fatores tecnologia, desenvolvida em parceria com a IBM, deve ser
que contribuem de forma significativa para as alterações aplicada nas demais cidades do país, segundo anunciou o
climáticas são os desmatamentos e a constante presidente da empresa no Brasil Rodrigo Kede. O prefeito da
impermeabilização do solo. Atualmente os principais cidade, Eduardo Paes, chegou a palestrar em uma Conferência
emissores dos gases do efeito de estufa são respectivamente: do TED explicando quatro grandes ideias que devem conduzir
China, Estados Unidos, Rússia, Índia, Brasil, Japão, Alemanha, o Rio (e todas as cidades) ao futuro, incluindo inovações
Canadá, Reino Unido e Coreia do Sul. Em busca de alternativas arrojadas e executáveis de infraestrutura.
para minimizar o aquecimento global, 162 países assinaram o Megacidades: Prefeitos das maiores cidades do mundo
Protocolo de Kyoto em 1997. Conforme o documento, as estiveram reunidos na Rio+20. Representantes das maiores
nações desenvolvidas comprometem-se a reduzir sua emissão metrópoles do mundo se reuniram para trocar experiências
de gases que provocam o efeito de estufa, em pelo menos 5% sobre desenvolvimento sustentável e traçar metas para
em relação aos níveis de 1990. Essa meta teve que ser reduzir os impactos dos grandes centros urbanos no planeta.
cumprida entre os anos de 2008 e 2012. Porém, vários países Prefeitos das 40 maiores cidades do mundo se encontraram
não fizeram nenhum esforço para que a meta fosse atingida, o em São Paulo para participar da C40 (Large Cities Climate
principal é os Estados Unidos. Leadership Group). Um dos destaques foi à assinatura de um
protocolo de intenções destinado a viabilizar suporte
Enfraquecido, Protocolo de Kyoto é estendido até financeiro a grandes cidades, no intuito de que elas
2020 desenvolvam ações de sustentabilidade. O documento foi
Quase 200 países concordaram em estender o Protocolo da assinado pelo presidente do Banco Mundial, Robert Zoellick, e
Kyoto até 2020. A decisão foi tomada durante a COP-18, pelo prefeito de Nova York, Michael Bloomberg, presidente da
Cúpula das Nações Unidas sobre Mudança Climática realizada cúpula. Outro encontro decisivo aconteceu durante a Rio+20,
em Doha, no Catar. Apesar do acordo, Rússia, Japão e Canadá quando os líderes das 59 maiores cidades do mundo se
abandonaram o Protocolo: assim, as nações que obedecerão comprometeram a reduzir em até 248 milhões de toneladas as
suas regras são responsáveis por apenas 15% das emissões emissões de gases do efeito estufa até 2020. Na mesma
globais de gases de efeito estufa. O acordo evita um novo ocasião, os prefeitos firmaram o compromisso de engajar 100
entrave nas negociações realizadas há duas décadas pela ONU. metrópoles no caminho do desenvolvimento sustentável até
Na oportunidade, não foi possível impedir o aumento das 2025.
emissões de gases do efeito estufa.
Sem o acordo, a vigência do Protocolo se encerraria no SOCIEDADE
começo de 2013. A extensão do Protocolo o mantém ativo
como único plano que gera obrigações legais com o objetivo de Previdência social: Entenda os custos da
enfrentar o aquecimento global. Rússia, Belarus e Ucrânia se aposentadoria para a sociedade46
opõem à decisão de estender o Protocolo para além de 2012. A Envelhecer é uma das grandes conquistas da sociedade. O
Rússia quer limites menos rígidos sobre as licenças de envelhecimento da população brasileira, assim como acontece
emissões de carbono que não foram utilizadas. Todos os lados no resto do mundo, se relaciona à melhoria do padrão de vida
dizem que as decisões tomadas em Doha ficaram aquém das e aos avanços da medicina, entre outros fatores.
recomendações de cientistas. Estes queriam medidas mais A Previdência Social é o sistema público que cuida da
duras para evitar mais ondas de calor, tempestades de areia, aposentadoria. É considerada como um direito social do
enchentes, secas e aumento do nível dos oceanos. cidadão, previsto no art. 6º da Constituição Federal de 1988. O
trabalhador que contribuir mensalmente para a Previdência
Social recebe uma aposentadoria depois de anos de trabalho e
contribuição ao Instituto Nacional do Seguro Social (INSS).

46 06/03/2017 - https://vestibular.uol.com.br/resumo-das-
disciplinas/atualidades/previdencia-social-entenda-os-custos-da-
aposentadoria-para-a-sociedade.htm

Atualidades e Deveres dos Servidores Públicos 22


Apostila Digital Licenciada para Alice Caroline Guarino dos Santos - alice.guarino@hotmail.com (Proibida a Revenda)
APOSTILAS OPÇÃO

Na prática, esse sistema também é um seguro social. Além Desvinculação de Receitas da União (DRU), um mecanismo
da aposentadoria, o trabalhador tem direito a benefícios que o criado em 1994 que permite ao governo federal usar
protegem em situações de doença ou vulnerabilidade: salário- livremente 20% de todos os tributos federais vinculados por
maternidade, salário-família, auxílio-acidente, auxílio-doença lei a fundos ou despesas.
e pensão. Na prática, esse mecanismo permite que o governo aplique
Pouco mais de cem anos atrás, não havia nenhum tipo de os recursos destinados a áreas como educação, saúde e
proteção para a pessoa que ficasse impedida de gerar renda previdência social em qualquer despesa considerada
por conta da idade avançada ou problemas de saúde. O prioritária e na formação de superávit primário. A DRU
primeiro registro de algum tipo de aposentadoria no Brasil também possibilita o manejo de recursos para o pagamento de
data de 1923, com a criação de uma pensão para os juros da dívida pública.
ferroviários, medida mais tarde estendida aos trabalhadores
marítimos e portuários. Nas décadas seguintes, esses Mudança demográfica
benefícios foram alcançando outras categorias de trabalho. Em O Brasil vive uma importante mudança demográfica. No
1988, a Constituição garantiu a previdência social como um passado, o número de pessoas que chegavam aos 60 ou 65
direito fundamental do cidadão. anos era pequeno.
Todos os empregados com carteira de trabalho assinada De 1999 a 2009, enquanto a população brasileira cresceu
são obrigados a recolher para esse sistema. Na estrutura da 12%, a parcela dos que tinham mais de 60 anos avançou 40%.
Previdência, os trabalhadores em idade ativa pagam a renda Nesse período, os idosos passaram de 9,1% para 11,3% da
de aposentados e pensionistas. Quem está trabalhando hoje população.
paga a previdência de quem já se aposentou, assim como a Em 2015, 14,3% da população brasileira tinha mais de 60
próxima geração pagará a de quem está trabalhando hoje. anos. A previsão é que por volta de 2050, as pessoas com mais
Para manter o sistema em equilíbrio, o país precisa ter um de 60 anos serão 25% da população total.
número maior de pessoas no mercado de trabalho em relação Quanto maior é a expectativa de vida de uma população,
ao número de beneficiados na previdência. É como se fosse maior é o gasto do governo com a aposentadoria dos
uma pirâmide, na qual a maior parte da base gera recursos trabalhadores.
para a ponta. Outra forma de equilibrar o sistema é alocar Enquanto isso, a taxa de fecundidade da mulher brasileira
recursos para a previdência oriundos de outras fontes de diminui cada vez mais, o que faz com que uma proporção
arrecadação do governo. menor de jovens trabalhe para ajudar a pagar a aposentadoria
Este ano, o Governo Federal quer votar a reforma da dos mais velhos.
Previdência no Congresso Nacional, que alteraria as regras O rápido envelhecimento da população esgotaria o atual
sobre quando as pessoas podem se aposentar e quanto elas modelo de previdência. Se o número dos que estão pagando a
vão receber. Para o Governo, a reforma seria fundamental para Previdência diminui e o número dos que recebem a
equilibrar as contas públicas. Previdência aumenta, as contas do Estado não fecham.
Em 2015, o Congresso já havia aprovado o mecanismo
85/95, que prevê mudanças no cálculo da aposentadoria. Com Trabalhos sem carteira
a nova regra, a soma da idade + o tempo de contribuição deve O trabalho sem registro na carteira diminui a arrecadação
ser de 85 anos para mulheres e 95 anos para homens. A partir da Previdência. A informalidade ainda é elevada no Brasil e
de 2017, o mecanismo será gradativamente acrescido em 1 contribui para o déficit do sistema. Segundo o Ipea, pouco mais
ponto até 2022. Recentemente, o Congresso também tornou da metade dos brasileiros (52%) trabalha hoje com carteira
mais rígidas regras para concessão de auxílio doença e assinada. A falta de contribuição ao INSS faz com que muitos
aposentadoria por invalidez. trabalhadores, principalmente os de baixa renda, sejam
Entenda alguns pontos fundamentais do debate sobre a excluídos do sistema de proteção social.
reforma da Previdência Social:
Falta de pagamento das empresas
Déficit das contas Segundo um levantamento realizado este ano pela
As novas gerações são ameaçadas por um rombo crescente Procuradoria-Geral da Fazenda Nacional (PGFN), R$ 426
no sistema previdenciário brasileiro. Segundo o Governo, a bilhões não foram repassados pelas empresas ao INSS. O valor
Previdência tem sido deficitária há mais de 20 anos, ou seja, o da dívida equivale a três vezes o chamado déficit da
valor dos pagamentos de benefícios supera a arrecadação Previdência em 2016. Entre elas, estão empresas ativas,
total. empresas falidas ou tradicionais sonegadoras. Uma
Em 2015, o déficit da previdência foi de 86 bilhões de reais. fiscalização mais eficiente contribuiria para o equilíbrio das
Em 2016, o país registrou um déficit recorde de R$ 149,73 contas.
bilhões. A participação da Previdência no gasto público total é
de 22%. A Reforma da Previdência proposta
O acúmulo de sucessivos déficits na Previdência é O projeto do governo deve ter um impacto positivo para os
resultado de um possível conjunto de fatores: a deterioração cofres públicos, mas por outro lado, ele dificultará o acesso à
dos postos de trabalho, as despesas indexadas ao salário aposentadoria para quem ainda não tem o direito ao benefício.
mínimo, o emprego informal (sem carteira assinada), a má Veja os principais pontos da reforma da Previdência proposta
gestão dos recursos do INSS e o envelhecimento da população. pelo governo:
Alguns economistas argumentam que o déficit na Ter a Idade mínima de 65 anos e 25 anos de
previdência social seria um mito. O argumento é feito com base contribuição. O governo busca aumentar o período
na Constituição Federal e na gestão de alocação de recursos. contributivo e estabelecer uma idade mínima para solicitar o
Segundo a Carta Magna, o financiamento da Seguridade Social benefício. Hoje, é possível obter o benefício com o mínimo de
(Saúde, Previdência e Assistência Social) é regido pelo artigo contribuição de 15 anos. Críticos dessa proposta avaliam que
195 da Constituição, que indica que os recursos para essas a idade mínima não leva em conta a realidade da expectativa
finalidades podem vir de outras fontes, não apenas da de vida de alguns estados. Por exemplo, enquanto a esperança
arrecadação dos trabalhadores. de vida de Santa Catarina é de 79 anos, no Maranhão é de 70,6
Outra crítica é que a Constituição estabelece que aquilo anos. Isso quer dizer que um maranhense pode morrer antes
que se arrecada para a Seguridade Social deve ser gasto de se aposentar.
exclusivamente com ela. Um dos problemas seria a

Atualidades e Deveres dos Servidores Públicos 23


Apostila Digital Licenciada para Alice Caroline Guarino dos Santos - alice.guarino@hotmail.com (Proibida a Revenda)
APOSTILAS OPÇÃO

Regras iguais para homens e mulheres. Atualmente, as Em uma década, o Brasil acumulou conhecimento e dados
mulheres podem se aposentar com um tempo de contribuição suficientes para deixar claro que sua política antidrogas vem
menor ou uma idade mínima mais baixa do que os homens. O promovendo um violento massacre às populações mais
governo quer equiparar as regras para os gêneros. Críticos da vulneráveis e tornado cada vez mais insustentável o sistema
igualdade da proposta de idade entre os gêneros avaliam que prisional. Existe uma demanda crescente dentro e fora do país
a maioria das mulheres possui uma dupla jornada de trabalho para a revisão da abordagem proibicionista e tratamento da
(no trabalho e em casa) e que esta regra prejudicaria a saúde questão dentro de seu devido lugar, que é a saúde pública.
da mulher.
Regra de transição. Mulher a partir de 45 anos e homens Uma de cada 7 crianças do mundo respira ar
a partir de 50 que ainda não tenham se aposentado pagarão altamente poluído, diz Unicef48
pedágio de metade do tempo que falta para a aposentadoria. Quase uma de cada sete crianças do mundo vive em áreas
Trabalhadores rurais. O governo quer mudanças na com níveis altos de poluição ambiental, a maioria no sul da
contribuição dos trabalhadores do campo, que, em geral, Ásia, e seus corpos em desenvolvimento são os mais
contribuem poucos para financiar suas aposentadorias. Hoje vulneráveis aos danos, disse o Fundo das Nações Unidas para
não é exigida contribuição dos trabalhadores rurais para ter a Infância (Unicef) nesta segunda-feira (31).
direito ao benefício. Eles podem se aposentar por idade, com O Unicef pediu a quase 200 governos, que irão se reunir no
um desconto de cinco anos (60 anos para homens e 55 para Marrocos entre 7 e 18 de novembro para discussões sobre o
mulheres). Basta comprovar que trabalharam na agricultura aquecimento global, que limitem o uso de combustíveis fósseis
por no mínimo 15 anos. Essa regra faz com que a previdência para que se obtenha o benefício duplo de uma saúde melhor e
rural tenha o maior déficit dentro do sistema. Em 2015, o uma desaceleração na mudança climática.
sistema rural arrecadou 7,3 bilhões de reais em contribuições, Cerca de 300 milhões de crianças, ou quase uma de cada
mas pagou 102 bilhões em aposentadorias, resultando num sete do planeta, moram em áreas onde a poluição ambiental é
déficit de 94,7 bilhões. Críticos apontam que o trabalhador mais alta, definida pelo Unicef como ao menos seis vezes as
rural possui uma realidade diferente do trabalhador urbano. O diretrizes internacionais estabelecidas pela Organização
desgaste físico é maior e a qualidade de vida menor. Isso Mundial da Saúde (OMS), disse o Unicef.
contribuir para que tenham uma expectativa de vida mais Do total, 220 milhões vivem no sul asiático. A agência
curta, de acordo com dados do IBGE e do IPEA, aproveitando identificou as regiões com imagens de satélite desenvolvidas
menos de sua aposentadoria. pela Agência Nacional Aeroespacial dos Estados Unidos (Nasa,
Militares. A proposta do governo não inclui a na sigla em inglês).
aposentadorias e pensões de militares, responsáveis por um O diretor-executivo do Unicef, Anthony Lake, disse que a
déficit de 32 bilhões de reais. Policiais militares e bombeiros poluição atmosférica é "um dos fatores que mais contribuem
serão incluídos. para as mortes de cerca de 600 mil crianças de menos de cinco
anos de idade todo ano" provocando doenças como a
Perdas e danos47 pneumonia.
Há dez anos o Brasil aprovava um novo marco legal para o "Os poluentes não prejudicam só os pulmões em
combate às drogas. A Lei 11.343/2006 nascia com a desenvolvimento das crianças – eles podem inclusive cruzar a
perspectiva de intensificar penas para o crime de tráfico e barreira hematoencefálica e danificar permanentemente seus
reduzir a criminalização dos usuários. Seu efeito, porém, cérebros em desenvolvimento – e, assim, seus futuros",
mostrou-se desastroso: cadeias superlotadas, mais mulheres afirmou em comunicado.
nas prisões e criminalização da população negra e pobre. Por "A poluição atmosférica afeta mais as crianças pobres",
outro lado, não há nenhum indicador de que as redes de tráfico disse à Reuters Nicholas Rees, especialista em clima e análise
tenham sido coibidas. econômica do Unicef que escreveu o relatório.
O grande responsável por essa desastrosa situação foi o No mundo inteiro, a OMS estima que a poluição ambiental
aumento da pena mínima de três para cinco anos, mesmo para matou 3,7 milhões de pessoas em 2012, entre elas 127 mil
pequenos traficantes. Soma-se a isso a relutância dos juízes em crianças de menos de cinco anos. Fábricas, usinas de energia e
aplicar a diminuição de pena para réus primários e a veículos que usam combustíveis fósseis, poeira e a queima de
insistência no encarceramento, muito embora o Supremo resíduos estão entre as fontes.
Tribunal Federal já tenha decidido que a equiparação a crime A poluição em ambientes fechados, muitas vezes causada
hediondo não impede a aplicação de penas alternativas, como por fornos que queimam carvão ou madeira usados em
ocorre para outros crimes não violentos como o furto. residências em nações em desenvolvimento, matou ainda
O resultado é uma distorção racista e classista, já enraizada mais: 4,3 milhões de pessoas, das quais 531 mil eram crianças
na cultura brasileira, mas bastante escancarada no sistema de menos de cinco anos, disse a entidade.
prisional: embora não existam dados sociodemográficos O Unicef fez um apelo à reunião liderada pela Organização
específicos dos presos por tráfico de drogas, o perfil geral da das Nações Unidas (ONU) no Marrocos para que acelere a troca
população prisional brasileira é composto majoritariamente de combustíveis fósseis por energias mais limpas, como solar
por negros (61,6%) e de baixa escolaridade (oito em cada dez ou eólica, para melhorar o acesso das crianças aos cuidados de
estudaram, no máximo, até o ensino fundamental). O foco da saúde, reduzir sua exposição à poluição e intensificar o
atuação policial no combate à venda de drogas no varejo e ao monitoramento do ar.
transporte feito por "mulas" faz com que um contínuo fluxo de
jovens desempregados sejam levados ao sistema prisional Em Fortaleza, Parada pela Diversidade Sexual
mesmo sem praticar qualquer ato violento, enquanto as reivindica cidadania plena49
grandes organizações têm seu complexo sistema de comércio A 17ª Parada pela Diversidade Sexual de Fortaleza reúne
e corrupção inalterado. neste domingo (13/11/2016) centenas de pessoas na Avenida
Beira Mar, em Fortaleza, para reivindicar cidadania plena para

47 23/08/16 – Fonte: 49 13/11/2016 Fonte: http://agenciabrasil.ebc.com.br/direitos-


http://brasil.elpais.com/brasil/2016/08/23/opinion/1471971725_335436.ht humanos/noticia/2016-11/em-fortaleza-parada-pela-diversidade-sexual-
ml reivindica-cidadania
48 31/10/2016 Fonte:
http://g1.globo.com/bemestar/noticia/2016/10/uma-de-cada-7-criancas-do-
mundo-respira-ar-altamente-poluido-diz-unicef.html

Atualidades e Deveres dos Servidores Públicos 24


Apostila Digital Licenciada para Alice Caroline Guarino dos Santos - alice.guarino@hotmail.com (Proibida a Revenda)
APOSTILAS OPÇÃO

lésbicas, gays, bissexuais e transexuais. Com o tema “Basta de "As mulheres jovens estão enfrentando uma ameaça
close errado”, o evento, organizado pelo Grupo de Resistência tripla", disse Sidibé. "Elas têm alto risco de infecção de HIV,
Asa Branca (Grab), enfatiza a exigência por igualdade de taxas baixas de exame de HIV e pouca adesão ao tratamento".
direitos. O documento, que afirma que o número de pessoas com
“Esse tema tem o objetivo de fazer a denúncia dos closes HIV recebendo remédios que salvam suas vidas é de 18,2
errados, que são a tentativa de diminuição dos direitos sociais, milhões, também mostrou que o progresso rápido na obtenção
incluindo o direito à homossexualidade plena, e o combate às de medicamentos para Aids para os necessitados está tendo
vulnerabilidades e às violências estruturais. A parada celebra um impacto significativo no prolongamento de suas vidas.
a diversidade, os direitos das comunidades LGBTs [Lésbicas, Em 2015 havia 5,8 milhões de pessoas acima de 50 anos
Gays, Bissexuais, Travestis, Transexuais e Transgêneros] e vivendo com HIV, mais do que nunca.
denuncia esse cenário, que torna cada vez mais importante A Unaids disse que se as metas de tratamento forem
que haja políticas efetivas e concretas de combate a essas alcançadas --a entidade pretende ter 30 milhões de pessoas em
vulnerabilidades”, explica o presidente do Grab, Francisco tratamento até 2020-- esse número irá disparar.
Pedrosa.
Na Avenida Beira Mar, um dos cartões-postais da capital Professora terá de pagar R$ 10 mil após chamar
cearense, os participantes da parada deram voz ao tema do aluno de 'negro burro'51
evento. A estudante Louisy Guardekey, junto a amigas, Uma professora e o Governo do Estado de São Paulo foram
levaram cartazes pedindo respeito ao público LGBT. Ela conta condenados pela Justiça a pagar indenização, de R$ 10 mil
que tomou a iniciativa de se expressar durante o evento pelo cada, para um aluno de uma escola pública em Guarujá, no
fato de o pai ser gay e sofrer preconceito com frequência. litoral paulista, e à mãe dele, por conta de injúrias raciais.
“As pessoas demonstram preconceito quando riem e fazem Em 2008, a professora da Escola Estadual Professora
piada com o modo dele de falar. A minha relação com amigos, Raquel de Castro Ferreira disse aos estudantes, durante uma
por conta disso, já foi bastante crítica, mas hoje eles já aula, que 'pessoas negras são burras e não conseguem
entenderam. Eu só espero da sociedade mais igualdade.” aprender'. O comentário considerado ofensivo foi gravado
pelo celular de um dos alunos.
Mais de 18 milhões recebem tratamento para HIV no Apesar do Estado ter entrado com recurso argumentando
mundo, diz Unaids50 que o caso não passou de um "mero aborrecimento", o
Mais de 18 milhões de pessoas estão recebendo desembargador Rebouças de Carvalho não acatou o pedido. Na
tratamento para Aids atualmente, 1,2 milhão a mais do que no última quinta-feira (17/11/2016), ele confirmou a
final do ano passado, informou o Programa Conjunto das condenação tanto da professora, quanto do governo.
Nações Unidas sobre HIV/Aids (Unaids) nesta segunda-feira “Os fatos ocorreram no interior de uma escola pública e
(21). motivado por comentário infeliz e impróprio, ainda que
Em um relatório sobre a pandemia de Aids, que infectou 78 episódico, e vindo de uma professora, ganha ainda contornos
milhões de pessoas e matou 35 milhões desde que teve início mais graves, isso porque a escola é o local da convivência, do
nos anos 1980, a Unaids disse que a intensificação consistente incentivo à liberdade da tolerância e do respeito e, ainda, da
dos tratamentos fez as mortes anuais relacionadas à Aids promoção da dignidade humana. Referido tipo de
diminuírem 45 por cento, ou para 1,1 milhão, em 2015 -- o pico comportamento de quem tem o dever de ensinar não pode ser
foi de cerca de 2 milhões em 2005. admitido, devendo ser coibido”, ponderou.
Mas agora que mais pessoas com o vírus vivem mais A decisão da 9ª Câmara de Direito Público do Tribunal de
tempo, os desafios de cuidar delas à medida que envelhecem, Justiça de São Paulo confirmou a sentença que já havia sido
de evitar que o vírus se dissemine e de reduzir novas infecções assinada pelo juiz Marcelo Machado da Silva, da 4ª Vara Cível
são duros, afirmou a Unaids, embora os remédios possam de Guarujá. Entre as provas colhidas durante o processo, está
diminuir os níveis de vírus no sangue dos pacientes para quase a gravação contida em um celular demonstrando que a
zero e reduzir significativamente o risco de transmiti-lo. professora se referiu "às pessoas de pele negra como sendo
"O progresso que fizemos é notável, particularmente nos pessoas 'burras' e que não conseguem aprender".
tratamentos, mas também é incrivelmente frágil", disse o
diretor-executivo da Unaids, Michel Sidibé, por ocasião da Avião da Chape cai, e autoridades informam: 71
publicação do relatório. mortos e 6 sobreviventes52
Com dados detalhados mostrando algumas das muitas Uma tragédia no futebol mundial e especialmente
complexidades da epidemia de HIV, o documento revelou que brasileiro. O avião que transportava a delegação da
as pessoas são especialmente vulneráveis ao vírus em certas Chapecoense para a primeira partida da final da Copa Sul-
épocas da vida. O estudo pediu uma abordagem de "ciclo de Americana contra o Atlético Nacional fez um pouso forçado na
vida" para oferecer ajuda e medidas preventivas a todas as madrugada desta terça-feira na região de Antióquia, em
pessoas em todas as fases da vida. gravíssimo acidente na Colômbia. Segundo informações do
À medida que as pessoas portadoras da HIV envelhecem, chefe da polícia colombiana, José Acevedo, 71 pessoas
correm o risco de desenvolver efeitos colaterais do tratamento morreram e seis sobreviveram. O zagueiro Neto, o lateral Alan
para HIV, criando resistências aos medicamentos e Ruschel e o goleiro Follmann estão entre os sobreviventes,
necessitando de cuidados para outras doenças, como sendo que Follmann teve uma perna amputada. Os outros três
tuberculose e hepatite C. que escaparam vivos da tragédia são o jornalista Rafael Henzel
O relatório também citou dados da África do Sul que e dois integrantes da tripulação: Ximena Suárez e Erwin
mostram que mulheres jovens que se infectam com o vírus Tumiri. O goleiro Danilo chegou a ser resgatado com vida, mas
com frequência o recebem de homens mais velhos, e disse que de acordo com informações do SporTV, não resistiu.
a prevenção é vital para acabar com a epidemia entre mulheres Inicialmente, as autoridades informaram que eram 75 mortos,
jovens e que o ciclo de infecção de HIV precisa ser mas quatro pessoas não chegaram a embarcar.
interrompido.

50 21/11/2016 Fonte: http://g1.globo.com/bemestar/noticia/mais-de-18- 52 29/11/2016 Fonte:


milhoes-recebem-tratamento-para-hiv-no-mundo-diz-unaids.ghtml http://globoesporte.globo.com/sc/futebol/times/chapecoense/noticia/2016/
51 22/11/2016 Fonte: http://g1.globo.com/sp/santos- 11/imprensa-colombiana-diz-que-aviao-que-transporta-chape-esta-
regiao/noticia/2016/11/professora-tera-de-pagar-r-10-mil-apos-chamar- desaparecido.html
aluno-de-negro-burro.html

Atualidades e Deveres dos Servidores Públicos 25


Apostila Digital Licenciada para Alice Caroline Guarino dos Santos - alice.guarino@hotmail.com (Proibida a Revenda)
APOSTILAS OPÇÃO

A delegação da Chapecoense saiu de Guarulhos para a Fisiologista - Cezinha


Bolívia em voo comercial com 72 passageiros e nove Médico - Dr. Marcio
tripulantes. Após escala técnica, deixou Santa Cruz de La Sierra Fisioterapeuta - Rafael Gobbato
em direção a Medellín. Quando sobrevoava as cidades Analista de Desempenho - Pipe Grohs
colombianas de La Ceja e Abejorral, perdeu contato com a
torre de controle do aeroporto José Maria Córdova. A causa do Veja abaixo a lista com os profissionais de imprensa
acidente teria sido uma pane elétrica. De acordo com a que estavam no voo:
imprensa local, o piloto teria liberado combustível para evitar Victorino Chermont (FOX), Lilacio Pereira Jr. (FOX),
explosão após o pouso forçado. Rodrigo Santana Gonçalves (FOX),Deva Pascovicci (FOX),
A delegação viajaria diretamente para Medellín em um voo Mário Sérgio (FOX), Paulo Julio Clement (FOX), Guilherme
fretado, o que foi desautorizado pela Agência Nacional de Marques (TV Globo), Guilherme Van der Laars (TV Globo), Ari
Aviação Civil (ANAC). Assim sendo, o time foi forçado a mudar de Araújo Jr. (TV Globo), Laion Espíndola (GloboEsporte.com),
seus planos e embarcar primeiro para São Paulo. De lá, pegou Giovane Klein Victória (RBS), André Podiacki (RBS), Bruno
o voo para a cidade colombiana. Mauri da Silva (RBS), Djalma Araújo Neto (RBS), Gelson
O local da queda do avião é de difícil acesso. Além disso, o Galiotto (Rádio Super Condá), Edson Luiz Ebeliny (Rádio
mau tempo na região metropolitana de Medellín, além da baixa Super Condá), Fernando Schardong, Douglas Dorneles (Rádio
temperatura - 5º C durante a madrugada, atrapalhou ainda Chapecó), Jacir Biavatti, Renan Agnolin (Rádio Oeste Capital).
mais o resgate. Dois helicópteros da força aérea do país
sobrevoam a área para auxiliar no trabalho das equipes de SAÚDE
resgate.
De acordo com informações fornecidas pelo aeroporto José Brasil vive o pior surto da história de febre amarela
Maria Córdova, a aeronave perdeu contato com a torre de silvestre; vacina é a melhor prevenção53
controle às 21h33 locais (0h33 de Brasília) e caiu às 22h15 Brasil vive o pior surto da doença que mata uma em cada
(01h15 de Brasília). cinco pessoas. No Bem Estar desta quinta-feira (2/2/17), o
"O Comitê Operativo de Emergência do aeroporto José infectologista e consultor do programa, Dr. Caio Rosenthal
Maria Córdova informa que às 10 horas da noite uma aeronave explica que a vacina é a melhor forma de proteção. Mas será
com matrícula CP2933 proveniente de Santa Cruz de la Sierra, que todo mundo tem que ir correndo se vacinar?
na Bolívia, da empresa Lamia Corporation, com matrícula Se você mora em área de risco ou pretende viajar nos
boliviana se declarou em emergência entre o município de Ceja próximos dias, faça a vacinação o quanto antes.
e La Union, a aeronave se declarou com falhas elétricas, Não há motivo para correria e antes de se vacinar, alguns
segundo foi informado a Torre de Controle da Aeronáutica". pontos devem ser observados, como mostra o quadro abaixo:
Devido ao difícil acesso e às condições climáticas de baixa
visibilidade, está sendo difícil o trabalho de resgate dos
feridos. Autoridades locais pedem aos moradores da região
que possuam camionetes 4x4 que auxiliem na busca das
vítimas. Residentes dos arredores dizem não ter ouvido
estrondos de queda.
Em nota oficial, a Conmebol suspendeu todas as atividades
envolvendo a Confederação, inclusive a partida, duelo de ida
pela final da Sul-Americana, que estava marcada para quarta-
feira às 21h45 (de Brasília) em Medellín.

“A Confederação Sul-Americana de Futebol confirma que


há sido notificada por autoridades colombianas que o avião em
que viajava a delegação do Atlético Chapecoense do Brasil
sofreu um acidente em sua chegada à Colômbia. Estamos em
contato com as autoridades e à espera de informações oficiais.
A família Conmebol lamenta enormemente o ocorrido. Todas
as atividades da Confederação ficam suspensas até novo
aviso.”

Confira a lista dos jogadores que viajaram para a


Colômbia
Jogadores
Goleiros: Danilo e Follmann;
Laterais: Gimenez, Dener, Alan Ruschel e Caramelo;
Zagueiros: Marcelo, Filipe Machado, Thiego e Neto;
Volantes: Josimar, Gil, Sérgio Manoel e Matheus Biteco;
Meias: Cleber Santana e Arthur Maia;
Atacantes: Kempes, Ananias, Lucas Gomes, Tiaguinho,
Bruno Rangel e Canela.

Comissão técnica
Treinador - Caio Júnior
Auxiliar técnico - Duca
Preparador Físico - Anderson Paixão
Preparador de Goleiros - Boião

53 02/02/2017 – Fonte: http://g1.globo.com/bemestar/noticia/brasil-

vive-o-pior-surto-de-febre-amarela-silvestre-vacina-e-a-melhor-forma-de-
prevencao.ghtml

Atualidades e Deveres dos Servidores Públicos 26


Apostila Digital Licenciada para Alice Caroline Guarino dos Santos - alice.guarino@hotmail.com (Proibida a Revenda)
APOSTILAS OPÇÃO

Idosos, gestantes e lactantes têm de falar com o médico meses entre elas. O fabricante garante proteção contra os
antes de tomar a vacina. Portadores de doenças autoimunes ou quatro tipos do vírus da dengue. Segundo os estudos, a
com histórico de doença do timo também. proteção é de 93% contra casos graves da doença, redução de
Não devem tomar a vacina menores de 6 meses e pessoas 80% das internações e eficácia global de pouco mais de 60%
que estejam com doença febril aguda, tenham histórico de contra todos os tipos do vírus. A capacidade de produção do
reações anafiláticas ao ovo de galinha e à gelatina e pessoas em laboratório é de 100 milhões de doses por ano.
tratamento com imunossupressores.
Os macacos devem ser protegidos e suas mortes SUS
comunicadas às autoridades de Saúde. Eles também são Apesar de poder ser comercializada em todo o Brasil, ainda
vítimas da doença e ao mesmo tempo sentinelas, cujas mortes não há determinação sobre se a Dengvaxia será utilizada na
alertam que um inimigo está próximo. rede pública. Para isso, o Ministério da Saúde deve fazer
estudos sobre o custo/benefício da compra e distribuição do
Mosquitos adultos podem passar zika para sua prole, produto e de qual seria a estratégia de aplicação para ter
diz estudo54 impacto em termos de saúde pública.
Mosquitos adultos fêmeas podem repassar o vírus da zika
para sua prole, disseram pesquisadores dos Estados Unidos Testes da “pílula do câncer” em humanos começam
nesta segunda-feira (29/08/16). A descoberta, publicada no nesta segunda em São Paulo56
periódico "American Journal of Tropical Medicine and Os testes clínicos para tratamento de câncer com a
Hygiene", mostra que o vírus da zika se comporta como vírus fosfoetanolamina sintética, que ficou conhecida como “pílula
relacionados, incluindo o da dengue e o da febre amarela. do câncer”, começam na próxima segunda-feira (25/07/2016)
Robert Tesh, do departamento de medicina da em São Paulo. A pesquisa será conduzida pelo Instituto do
Universidade do Texas em Galveston, um dos autores do Câncer do Estado de São Paulo (Icesp).
estudo, declarou que o fato de o vírus poder ser repassado O início dos testes em humanos será possível após
para a prole do mosquito torna o zika mais difícil de controlar. aprovação da Comissão Nacional de Ética em Pesquisa, do
O Aedes aegypti, o mosquito que transmite o vírus da zika, Ministério da Saúde. A Fundação para o Remédio Popular
coloca ovos em pequenos acúmulos de água. Cientistas (Furp), laboratório oficial da Secretaria de Saúde do estado,
queriam descobrir se alguma parte da prole desses mosquitos forneceu as cápsulas suficientes da substância para realização
tropicais poderia carregar o vírus, ajudando a perpetuar o da pesquisa.
surto durante as estações secas. Segundo a Secretaria de Saúde, o pesquisador aposentado
Para isso, eles injetaram o vírus da zika em mosquitos da Universidade de São Paulo (USP) de São Carlos Gilberto
Aedes aegypti fêmeas criados em laboratório. Eles foram Chierice tem acompanhado todo o processo. A
alimentados e, em uma semana, colocaram ovos. A equipe fosfoetanolamina sintética foi estudada por Chierice, enquanto
coletou e cuidou dos ovos até eles darem vida a mosquitos ele ainda estava ligado ao Grupo de Química Analítica e
adultos, quando ela contou o número deles que carregava o Tecnologia de Polímeros da universidade. Algumas pessoas
vírus. Em cada 290 mosquitos testados, eles encontraram o tiveram acesso às cápsulas contendo a substância, produzidas
vírus em um mosquito. pelo professor, que usaram como medicamento contra o
câncer. O estudo prevê avaliação de 10 pacientes na primeira
Vacina contra dengue já pode ser comercializada no fase, para determinar a segurança da dose que vem sendo
Brasil55 usada na comunidade.
Após sete meses registrada na Agência Nacional de Após a primeira etapa, caso a droga não apresente efeitos
Vigilância Sanitária, a vacina contra a dengue já pode ser colaterais graves, a pesquisa prosseguirá. No chamado Estágio
comercializada no Brasil. O Comitê Técnico Executivo da 1, está prevista a inclusão de mais 21 pacientes para cada um
Câmara de Regulação do Mercado de Medicamentos (Cmed) dos 10 grupos de tumor: cabeça e pescoço, pulmão, mama,
determinou que o preço da Dengvaxia, como é chamada a cólon e reto (intestino), colo uterino, próstata, melanoma,
vacina da Sanofi Pasteur, vai variar entre R$ 132,76 e R$ pâncreas, estômago e fígado.
138,53, dependendo do ICMS adotado em cada estado. Se o Icesp observar sinais de atividade da substância nessa
O valor estipulado é o que será pago ao fabricante por fase, o Estágio 2 começa com mais 20 participantes em cada
clínicas, hospitais e distribuidores e deve ser bem diferente do grupo. Progressivamente, desde que se comprove atividade
que será cobrado do consumidor final. De acordo com o relevante, a inclusão de novos pacientes continuará até atingir
infectologista, o produto é um avanço, considerando que o o máximo de mil pessoas, ou seja, 100 para cada tipo de câncer.
Brasil vivencia há muitos anos um grande problema de saúde
pública devido ao vírus da dengue. Em 2016, até 11 de junho, Alimentos passam a ter de listar ingredientes
mais de 1,3 milhão de pessoas tiveram dengue em todo o país alergênicos nos rótulos57
e 318 pessoas morreram em decorrência da infecção pelo Os rótulos dos alimentos passam a ter de sair da fábrica
vírus. com informação sobre ingredientes alergênicos a partir deste
Fabricada pela empresa francesa Sanofi Pasteur e domingo (03/07/2016). São 17 os itens a serem listados, como
registrada no Brasil desde dezembro de 2015, a Dengvaxia é a trigo, crustáceos, leite e nozes. A decisão partiu da Agência
primeira vacina desenvolvida contra a dengue no mundo e só Nacional de Vigilância Sanitária (Anvisa) ainda em 2015 e foi
precisava da determinação do valor de fábrica para poder ser reforçada no início de junho.
vendida. Segundo a Anvisa, a demora ocorreu devido ao Aprovada em junho do ano passado, a resolução obriga a
ineditismo do produto, já que normalmente a estipulação de indústria alimentícia a informar nas embalagens dos
preços leva em conta outros produtos semelhantes no produtos se há presença dos principais alimentos que causam
mercado. O imunizante é indicado para pessoas entre 9 e 45 alergias alimentares. O regulamento abrange tanto alimentos
anos, deve ser aplicado em três doses com intervalo de seis e quanto bebidas, ingredientes e aditivos.

54 29/08/16 – Fonte: 56 25/07/2016 – Fonte: http://agenciabrasil.ebc.com.br/pesquisa-e-

http://g1.globo.com/bemestar/noticia/2016/08/mosquitos-adultos-podem- inovacao/noticia/2016-07/testes-da-pilula-do-cancer-em-humanos-comecam-
passar-zika-para-sua-prole-diz-estudo.html nesta-segunda-em
55 26/07/2016 – Fonte: http://agenciabrasil.ebc.com.br/pesquisa-e- 57 03/07/2016 - Fonte:
inovacao/noticia/2016-07/vacina-contra-dengue-ja-pode-ser-comercializada- http://g1.globo.com/bemestar/noticia/2016/07/alimentos-passam-ter-de-
no-brasil listar-ingredientes-alergenicos-nos-rotulos.html

Atualidades e Deveres dos Servidores Públicos 27


Apostila Digital Licenciada para Alice Caroline Guarino dos Santos - alice.guarino@hotmail.com (Proibida a Revenda)
APOSTILAS OPÇÃO

Os rótulos dos produtos fabricados a partir de agora Nesta terça (8/11/2016), o ministério informou que após
deverão deverão informar se os alimentos possuem alguns dos selecionado, o médico brasileiro terá 15 dias para fazer
seguintes alimentos: trigo (centeio, cevada, aveia e suas permuta de vagas a fim de ficar em uma cidade onde tem
estirpes hibridizadas); crustáceos; ovos; peixes; amendoim; interesse.
soja; leite de todos os mamíferos; amêndoa; avelã; castanha de Quando o candidato se inscrever no programa, fará opção
caju; castanha do Pará; macadâmia; nozes; pecã; pistaches; por quatro cidades. Levando em conta critérios como
pinoli; castanhas, além de látex natural. experiência e especialização, ele será designado para uma
Os derivados desses produtos deverão trazer na localidade. Em um período de 15 dias, um sistema poderá
embalagem as seguintes informações: identificar candidatos insatisfeitos com sua cidade e oferecer
- Alérgicos: Contém (nomes comuns dos alimentos que trocas entre aqueles que estão concorrendo ao mesmo edital.
causam alergias alimentares); Editais semelhantes serão publicados de três em três
Alérgicos: Contém derivados de (nomes comuns dos meses com ofertas a brasileiros formados no Brasil ou em
alimentos que causam alergias); instituições de qualquer país.
Alérgicos: Contém (nomes comuns dos alimentos que A maioria das vagas disponíveis estão em capitais e em
causam alergias alimentares) e derivados. regiões metropolitanas. Com isso, o governo pretende
A Anvisa determinou também a forma de dispor esses aumentar o interesse de médicos brasileiros em concorrer aos
dados. Os detalhes sobre alergênicos deverão ser exibidos logo postos. A região Nordeste concentra 40% das vagas
abaixo da lista de ingredientes. Além disso, as palavras têm de disponíveis. Elas ficam em 223 municípios da região, dentre as
estar em caixa alta, em negrito e com cor diferente do rótulo. A quais 57 são capitais.
letra não pode ser menor do que a da lista de ingredientes. Em setembro, o governo já tinha anunciado a pretensão de
Os fabricantes tiveram um ano para adequar as reduzir em 35% a participação de médicos cubanos no
embalagens às novas regras. Os produtos fabricados até o fim programa em três anos. A meta do governo federal é que a
do prazo de adequação, este sábado (02/07/2016), poderão quantidade de médicos da ilha caribenha atuando no
ser comercializados até o fim do prazo de validade. programa passe de 11,4 mil para 7,4 mil nesse período. Apesar
Segundo o diretor-relator da matéria, Renato Porto, a disso, caso as vagas não sejam preenchidas por brasileiros,
demanda nasceu “fortemente da sociedade”, o que fez com que existe a possibilidade de convocar novos grupos de
toda a diretoria votasse unilateralmente pela regulamentação. profissionais cubanos.
“A sociedade pode agora ter certeza que terá rótulos de Segundo o ministro da Saúde, Ricardo Barros, o Brasil vai
produtos muito mais adequados, que vão dar a possibilidade manter o contrato com a Organização Pan-americana da Saúde
do consumidor de escolher adequadamente seus produtos, (Opas), responsável pelo convênio com Cuba, enquanto os
dado que a melhor maneira de se prevenir [de uma crise brasileiros não ocuparem todas as vagas. De acordo com o
alérgica] é evitando o consumo”, explicou. ministro, a vinda dos profissionais cubanos correspondia,
Segundo a Anvisa, no Brasil, de 6% a 8% das crianças de 6 desde o princípio, a uma política temporária de saúde, e que a
a 8 anos sofrem de algum tipo de alergia. prioridade é contratar profissionais brasileiros.
Ao todo, existem 11.429 médicos cubanos atuando no
Busca por medicamentos é principal motivo para Brasil. O número corresponde a 62,6% dos 18.240 médicos
ações judiciais na saúde58 participantes no programa Mais Médicos. O índice de
Uma pesquisa mostra que quase a metade das ações profissionais com registro médico brasileiro é de apenas 29%.
judiciais na área da saúde é proposta por doentes tentando Os intercambistas correspondem a 8,4%.
exigir, do governo ou dos planos, o pagamento por
tratamentos. A maioria dos pedidos é para conseguir Ministério adota mais critérios para diagnosticar
medicamentos, que às vezes nem foram incorporados pelo danos por zika em bebês60
SUS. São Paulo é o campeão desta judicialização da saúde. O Ministério da Saúde informou nesta sexta-feira
Segundo a pesquisa feita pela Interfarma, das quase 15 mil (18/11/2016) que vai passar a levar em conta critérios como
ações na área da saúde em São Paulo, Rio Grande do Sul, Rio de perda de visão, audição, comprometimento e deficiência de
Janeiro e Minas Gerais, quase a metade é de judicialização da membros para avaliar se uma criança tem danos de
saúde, quando a ação é para obrigar o poder público ou o plano desenvolvimento provocados pelo vírus da zika. Até agora, o
de saúde a pagar por tratamento, hospitalização e principal critério levado em consideração era o perímetro da
medicamentos. cabeça dos bebês.
Os remédios lideram estes pedidos: 64% das ações são A nova medida foi tomada com base em evidências
para consegui-los. A Justiça concede liminar em pelo menos científicas de que mesmo crianças com tamanho de cabeça
87% dos casos. O estudo cita como causas para a judicialização dentro dos parâmetros considerados "normais" podem
da saúde os cortes no orçamento e a demora para incorporar apresentar consequências do vírus.
novas drogas no SUS. Desde a descoberta da microcefalia, pesquisadores
apontaram outras doenças relacionadas à infecção e isso levou
Para substituir cubanos, Mais Médicos abre mil vagas à Organização Munidal da Saúde (OMS) a determinar a
para brasileiros59 síndrome congênita do zika.
O Ministério da Saúde lançará nesta sexta-feira Segundo a pasta, o objetivo é garantir cuidado a todas as
(11/11/2016) um edital para chamar 1.004 médicos crianças, inclusive as que apresentaram alterações tardias - o
brasileiros em 462 municípios para o programa Mais Médico. ministério informou que vai passar a acompanhar por três
A intenção do governo é substituir 838 vagas ocupadas hoje anos crianças de mães que tiveram zika durante a gravidez. Até
por profissionais cubanos e repor 166 cargos de médicos que então, só eram monitorados bebês com microcefalia. Os novos
desistiram do programa. critérios afetam as regras para notificação, investigação e
classificação final de casos suspeitos da doença.

58 28/09/2016 – Fonte: http://g1.globo.com/bom-dia- 60 18/11/2016 Fonte:


brasil/noticia/2016/09/busca-por-medicamentos-e-principal-motivo-para- http://g1.globo.com/bemestar/noticia/2016/11/ministerio-adota-novos-
acoes-judiciais-na-saude.html criterios-para-diagnosticar-microcefalia-em-bebes.html
59 08/11/2016 fonte:
http://g1.globo.com/bemestar/noticia/2016/11/para-substituir-cubanos-
mais-medicos-abre-mil-vagas-para-brasileiros.html

Atualidades e Deveres dos Servidores Públicos 28


Apostila Digital Licenciada para Alice Caroline Guarino dos Santos - alice.guarino@hotmail.com (Proibida a Revenda)
APOSTILAS OPÇÃO

O governo também passa a recomendar uma segundo Brasil tem mais casos de chikungunya do que de zika
ultrassom no sétimo mês de gravidez. Hoje, o exame já é em 201662
realizado no primeiro mês de gestação. O objetivo é identificar O Ministério da Saúde anunciou nesta quinta-feira
possíveis malformações na etapa final da gestação, atendendo (24/11/2016) que os casos de chikungunya tiveram um
a orientações da OMS. crescimento de 850% em 2016, e que o ano de 2017 terá ainda
Para garantir o exame, o ministro anunciou que o governo mais casos. Os dados são do Levantamento Rápido do Índice
vai liberar repasses mensais aos estados para que a rede de Infestação pelo Aedes aegypti.
pública realize essa segunda ultrassonografia. A estimativa é De acordo com o ministro Ricardo Barros, foram 251.051
de gastar até R$ 52,6 milhões por ano com este exame. casos da doença neste ano, contra 26.435 em 2015 - os
registros superam os de zika no mesmo período, com 208.867
Microcefalia notificações e apenas 3 mortes.
O ministério também mudou os critérios de medição da Em 2016, morreram 138 pessoas por chikungunya
cabeça do bebê. São suspeitos de microcefalia meninos com enquanto que, no ano passado, foram somente seis óbitos. O
cabeça menor que 30,5 cm e meninas com 30,2 cm (até então pico da doença ocorreu em março.
as medidas eram 31,9 cm e 31,5 cm, respectivamente). Esta é O ministério não falhou nas ações e tem feito o
a segunda vez em que o ministério faz uma alteração do tipo. planejamento para quando houver surtos de doenças. "Não
Segundo o coordenador de Vigilância Epidemiológica, [falhou], é uma incidência, gente. Não há como se prever a
Wanderson Oliveira, a mudança segue uma regra da OMS de incidência das doenças. O que temos feito é uma estatística e
30 de agosto. “Acompanhamos [as novas medições] por dois um planejamento para que estejamos preparados, para que
meses para ver o perfil epidemiológico no Brasil, e por isso quando acontece, a saúde pública esteja em condições de
estamos adotando as medidas neste momento. É uma atender".
qualificação das medidas que já foram adotadas.”
Todas essas novas regras serão divulgadas para As doenças do Aedes
profissionais de saúde, cuidadores e familiares de pacientes. Além do alerta sobre a chikungunya, Barros fez um balanço
Além das mudanças das normas sobre o vírus da zika, as novas sobre as doenças causadas pelo mosquito Aedes. Ele afirmou
diretrizes vêm com passos para que qualquer profissional da que os casos de zika e dengue deverão se manter estáveis em
Atenção Básica também possa tratar crianças com alterações 2017 e ter o mesmo número de registros. "Nós não esperamos
no desenvolvimento. Também há um guia sobre como cuidar que aumentem os casos de dengue e zika", disse. "Este ano a
de bebês recém-nascidos e estimulá-los (com massagens e nossa maior preocupação é a chikungunya”, completou.
estratégias para estimular a amamentação, por exemplo). No caso da dengue, foram 1.458.355 casos prováveis em
Desde novembro do ano passado, o ministério contabiliza 2016, contra 1.543.000 em 2015. Houve uma redução de 50%
2.143 casos confirmados de Zika, 3.086 em investigação e nos casos graves da doença e de 36% em relação aos óbitos do
4.890 descartados. Ao todo, 78% dos casos confirmados se ano anterior.
concentram na região Nordeste do país. Os casos confirmados de microcefalia chegam a 2.159,
sendo que 3.115 continuam em observação e 4.925 foram
Sobre a síndrome descartados. O Nordeste concentra 66% do total de casos
A síndrome congênita do zika, reconhecida pela OMS, é um confirmados da malformação. A zika atingiu 2.281 municípios
conjunto de malformações e problemas apresentados por e teve seu pico em fevereiro, com 17.541 ocorrência por
bebês que tiveram mães infectadas pelo vírus da zika durante semana, em média. A pasta estima que, dos mais de 200 mil
a gestação. A microcefalia é só uma das consequências. As casos notificados, 16.696 casos tenham sido em gestantes.
crianças também podem ter o sistema nervoso central afetado,
apresentando epilepsia, deficiências auditivas e visuais, Investimento
prejuízo no desenvolvimento psicomotor, bem como efeitos Segundo o ministro, no ano passado foram gastos cerca de
negativos sobre ossos e articulações. R$ 500 milhões no combate às doenças propagadas pelo Aedes
aegypti. O orçamento para a área em 2017 ainda não foi
'Bactéria do bem' é a mais promissora pesquisa definido e está em votação no Congresso.
contra zika, diz ministro61 Ao todo, segundo levantamento da pasta, 855 cidades, que
O ministro da Saúde, Ricardo Barros, disse nesta quarta- representam 34% dos municípios pesquisados, se encontram
feira (16/11/2016) que a pesquisa que utiliza a bactéria em situação de alerta e risco de surto de dengue, zika e
Wolbachia, também chamada de “bactéria do bem”, é a “mais chikungunya. Atualmente, a adesão à pesquisa do ministério é
promissora” no combate ao vírus da zika. voluntária, mas Barros anunciou que vai propor que, a partir
Os resultados da pesquisa que envolve a bactéria de 2017, ela seja obrigatória a todos os municípios com mais
Wolbachia têm animado os pesquisadores da Fundação de 2 mil imóveis.
Oswaldo Cruz (Fiocruz), que começou os estudos no Rio de Das 22 capitais levantadas pelo ministério, Cuiabá é a única
Janeiro e em Niterói. Se injetada em ovos do mosquito, a em situação de alto risco. Já nove estão em alerta e 12 em
“bactéria do bem” pode neutralizar a transmissão das doenças condições satisfatórias.
causadas pelo Aedes. O ministro também lembrou do início da
campanha do governo contra o Aedes aegypti, no domingo O que é a chikungunya?
(20/11/2016), e o Dia Nacional de Mobilização contra o A doença transmitida pelo mesmo mosquito da dengue e
mosquito será no dia 25. De acordo com o ministério, a do zika é uma epidemia de dores nas articulações. Estudos
campanha deste ano deverá conscientizar sobre as mostram que a chikungunya deixa rastros, metade deles
consequências das diversas doenças causadas pelo mosquito. crônicos. O vírus tem uma predileção pelas articulações e
No último ano, com o aumento dos casos de dengue, a doença causa uma cascata inflamatória no local. Os pacientes,
ficou em destaque. Marcela Temer, mulher do presidente principalmente as mulheres, desenvolvem dores
Michel Temer, poderá ser madrinha da campanha deste ano. incapacitantes crônicas e não conseguem abotoar uma camisa,
pentear o cabelo, sem dores.

61 16/11/2016 Fonte: 62 24/11/2016 Fonte: http://g1.globo.com/bemestar/noticia/brasil-tem-

http://g1.globo.com/bemestar/noticia/2016/11/bacteria-do-bem-e-mais- alta-850-nos-casos-de-chikungunya-diz-ministerio.ghtml
promissora-pesquisa-contra-zika-diz-ministro.html

Atualidades e Deveres dos Servidores Públicos 29


Apostila Digital Licenciada para Alice Caroline Guarino dos Santos - alice.guarino@hotmail.com (Proibida a Revenda)
APOSTILAS OPÇÃO

O tratamento é feito com analgésicos e anti-inflamatórios. O número de pessoas empregadas aumentou em 980 mil
Entretanto, é preciso procurar um médico, porque a doença (ou 0,3%) em relação a julho do ano passado, para 64,79
pode trazer risco de sangramento. Tratamentos não milhões. Para cada 100 solicitantes de emprego, foram
medicamentosos como acupuntura, fisioterapia e compressas oferecidos no Japão em julho 137 postos de trabalho, mesmo
podem ajudar a aliviar a dor. número que em junho. O dado volta a ressaltar a extrema
rigidez vivida pelo mercado de trabalho japonês.
RELAÇÕES INTERNACIONAIS No entanto, o dado recorde foi afetado pela divulgação
simultânea da despesa das famílias japonesas também em
Chanceleres do Mercosul decidem que Venezuela não julho, mês no qual caiu, em termos reais, 0,5% em taxa
assumirá presidência rotativa do bloco63 anualizada. Embora este retrocesso do indicador - o quinto
Os chanceleres do Brasil, da Argentina, do Paraguai e do consecutivo - seja menor do que o previsto pelos analistas, o
Uruguai decidiram na terça-feira (13) que a Venezuela não número destaca o enfraquecimento da demanda interna,
assumirá a presidência rotativa do Mercosul, como estava principal motor da terceira maior economia mundial, e a
previsto no cronograma do bloco. Segundo o Itamaraty, os aparente incapacidade do governo japonês para virar esta
ministros decidiram mudar a regra do bloco porque a tendência. A maioria dos economistas consultados nesta terça-
Venezuela descumpriu compromissos assumidos no Protocolo feira (30/08/16) pelo jornal econômico "Nikkei" situam as
de Adesão ao Mercosul, assinado em Caracas em 2006. razões por trás desta situação na ausência de aumentos
A ministra de Relações Exteriores da Venezuela, Delcy salariais significativos, apesar da melhora do mercado de
Rodríguez, disse que o governo venezuelano manterá sua luta trabalho motivada pelas perspectivas pessimistas, que
em defesa do tratado constitutivo do Mercosul, que prevê a proporcionam um menor crescimento econômico no país este
troca da presidência pro temporea cada seis meses e por ano. A despesa média mensal das famílias com dois ou mais
ordem alfabética. Ela considera que o país está no exercício residentes foi de 278.067 ienes (US$ 2.731). A renda média por
pleno do comando do Mercosul desde o fim de julho e rechaça família assalariada caiu 1,8% (taxa anualizada) em julho, para
a declaração conjunta divulgada ontem pelos parceiros do 574.227 ienes (US$ 5.639).
bloco, que, segundo ela, “enfraquece a legalidade” da
organização. Vai a 247 o nº de mortos após forte terremoto na
Itália65
União Europeia prepara força-tarefa para gerenciar O número de mortos após o forte terremoto que atingiu a
transição de saída do Reino Unido região central da Itália na quarta-feira (24/08/16) subiu para
A Comissão Europeia criou uma força-tarefa na quarta- 247, informou a Defesa Civil nesta quinta (25/08/16), 27
feira (14/09/16) para a preparação e o desenvolvimento das horas após o tremor. O último balanço aponta ainda que 350
negociações com o Reino Unido para a saída do país do bloco. ficaram feridos e centenas seguem desaparecidos.
Ao lado do chefe das negociações da comissão, o francês O tremor matou, por enquanto, 190 na província de Rieti e
Michel Barnier, a alemã Sabine Weyand será a responsável por 57 na província de Ascoli. As buscas por sobreviventes não tem
ajudar na transição. A primeira reunião do grupo foi agendada previsão de interrupção durante a madrugada, segundo as
para o dia 1º de outubro. autoridades. Escavadeiras estão sendo usadas nos maiores
desmoronamentos, mas em diversos pontos bombeiros e
Hillary Clinton enfrenta crescimento de Donald socorristas usam as próprias mãos para retirar escombros e
Trump nas pesquisas em estados-chave tentar alcançar vítimas.
Segundo pesquisa nacional da rede de televisão CBS em Na cidade de Accumoli, cerca de 150 sobreviventes
parceria com o jornal The New York Times, a vitória de Hillary passaram a primeira noite após o terremoto em um abrigo
Clinton é de 46% contra 44% de Donald Trump, uma diferença improvisado em um parque público, onde voluntários também
de apenas dois pontos percentuais, muito pouco para dar estavam recebendo doações. O município fica no alto de uma
tranquilidade à candidata democrata. colina e sofreu graves danos, com o desabamento de várias
Em Ohio, um estado que é considerado um termômetro da casas e rachaduras nas ruas. Sem luz, os moradores também
tendência do eleitor norte-americano, os números são ainda foram obrigados a enfrentar baixas temperaturas durante a
mais preocupantes para o Partido Democrata: Trump está madrugada.
ganhando com 45% contra 37% de Hillary Clinton na
preferência do eleitor, de acordo com pesquisa da Mais de 4 mil migrantes morreram até julho no
Universidade de Monmouth. Mediterrâneo66
Mais de 4 mil migrantes e refugiados perderam a vida
Desemprego no Japão cai para 3% em julho, melhor durante o ano, o que representa um aumento de 26% em
resultado em 20 anos64 relação ao mesmo período de 2015, anunciou nesta terça-feira
A taxa de desemprego no Japão caiu em julho um décimo (02/08/16) a Organização Internacional para as Migrações
de ponto percentual, para 3%, o que representa o melhor (OIM). Um total de 4.027 migrantes morreram quando
resultado do país nas duas últimas décadas, segundo o tentavam cruzar o Mediterrâneo, mas também nas estradas do
governo. Esta é a taxa de desemprego mais baixa no Japão norte da África e na fronteira entre Turquia e Síria, informa um
desde maio de 1995, afirmou um porta-voz do Ministério do comunicado da agência com sede em Genebra. Deles, 3.120
Interior e Comunicações. pessoas morreram no Mediterrâneo entre 1º de janeiro e 31
O dado correspondente ao sétimo mês do ano está um de julho deste ano, segundo a OIM.
décimo acima da maioria das previsões feitas pelos A rota marítima mais perigosa continua sendo a travessia
economistas. O número de pessoas sem emprego em julho foi à Itália, que deixou 2.692 mortos, muito à frente dos itinerários
de 2,03 milhões, o que representa 190 mil pessoas ou 8,6% a à Grécia (383 mortos) e à Espanha (45 mortos). Quanto aos
menos que no mesmo mês de 2015. desaparecidos no Mediterrâneo, a OIM aumentou nesta terça-

63 16/09/2016 – Fonte: 65 25/08/2016 – Fonte:


http://guiadoestudante.abril.com.br/blogs/atualidades-vestibular/ http://g1.globo.com/mundo/noticia/2016/08/vai-247-o-n-de-mortos-apos-
64 30/08/2016 – Fonte: forte-terremoto-na-italia.html
http://g1.globo.com/economia/noticia/2016/08/desemprego-no-japao-cai- 66 02/08/2016 – Fonte:
para-3-em-julho-melhor-resultado-em-20-anos.html http://g1.globo.com/mundo/noticia/2016/08/mais-de-4-mil-migrantes-ate-
julho-de-2016.html

Atualidades e Deveres dos Servidores Públicos 30


Apostila Digital Licenciada para Alice Caroline Guarino dos Santos - alice.guarino@hotmail.com (Proibida a Revenda)
APOSTILAS OPÇÃO

feira o balanço após a descoberta recente de 120 corpos nas A Argentina propõe que seja feita uma reunião de
praias da cidade líbia de Sabrata. A organização e a guarda- coordenadores, com representantes de Brasil, Paraguai e da
costeira líbia não souberam informar se estes própria Argentina, para destravar as divergências sobre a
desaparecimentos se devem a um ou a vários naufrágios de Venezuela assumir a presidência do bloco.
embarcações de migrantes. Além do Mediterrâneo, o norte da
África foi neste ano a região mais perigosa, com 342 migrantes Entenda as diferenças entre União Europeia, zona do
mortos. euro e espaço Schengen69
Em um referendo histórico realizado no dia 23 de junho de
França faz funeral de padre morto por extremistas 2016, os britânicos votaram pela saída do Reino Unido da
sob forte segurança67 União Europeia, bloco econômico do qual o Estado começou a
O funeral do padre assassinado há uma semana por dois fazer parte em 1973. Essa é a primeira vez em que um membro
extremistas islâmicos em uma igreja na Normandia foi da UE opta por sair do bloco europeu - decisão que tem muito
realizado nesta terça-feira (02/08/16) em meio a um forte potencial para influenciar a geopolítica mundial nos próximos
esquema de segurança na Catedral de Rouen. Cerca de 2 mil anos.
pessoas dentro e fora da catedral gótica enfrentaram chuva O Reino Unido já não fazia parte de outros dois acordos
para participar do tributo ao padre Jacques Hamel, de 85 anos, característicos da União Europeia: a zona do euro e o espaço
morto enquanto celebrava uma missa para cinco fiéis em sua Schengen. Entenda, a seguir, quais são as diferenças entre cada
igreja. O caixão, carregado por quatro pessoas, entrou no um deles.
prédio precedido e seguido por uma procissão de muitos União Europeia: é uma união econômica e política
padres de estola roxa, a cor do luto. formada atualmente por 27 países europeus. Ela tem suas
Alguns muçulmanos também estiveram presentes. O origens na Comunidade Europeia do Carvão e do Aço (CECA) e
assassinato, cometido por dois extremistas de 19 anos que na Comunidade Econômica Europeia (CEE), organizações
juraram fidelidade ao grupo Estado Islâmico (EI), causou forte internacionais formadas pela França, Itália, Alemanha
comoção na França. O padre Jacques Hamel era conhecido Ocidental, Bélgica, Holanda e Luxemburgo nos anos 50.
localmente por seu envolvimento no diálogo interreligioso Tratado de Maastricht: a União Europeia do modo como
com os muçulmanos de Saint-Etienne-du-Rouvray. conhecemos hoje foi instituída em 1993 com o Tratado de
Maastricht. A partir dele, foram estabelecidas metas de livre
Segurança reforçada: O esquema de segurança em torno circulação de mercadorias, de pessoas, de serviços e de
da catedral foi reforçado para a cerimônia, que contou com a capitais, tendo como objetivo a estabilidade política do
presença dos ministro francês do Interior e dos Assuntos continente europeu.
Religiosos, Bernard Cazeneuve. Vinte veículos da polícia Mercado e política: o mercado comum da UE possui um
estavam nos arredores e todas as pessoas que entravam no sistema de políticas de regulamentação que são aplicadas a
prédio da igreja eram revistadas. No exterior, um telão todos os seus Estados-membros. O bloco econômico também
retransmitia a cerimônia. O padre Hamel, originário do norte possui diversos tratados e leis em comum. O conselho da União
da França, será enterrado em um lugar que a família quer Europeia é a instituição da UE onde os ministros de cada
manter em segredo. Estado-membro aprovam leis e coordenam políticas internas
Sua igreja de Saint-Etienne-du-Rouvray, onde oficiava há e externas. Sua sede fica em Bruxelas, cidade que é
quase 50 anos, permanece fechada. Segundo a diocese, ela só considerada a capital da União Europeia.
será reaberta após um "rito penitencial de reparação", que Zona do euro: já a zona do euro, em que é adotada como
permitirá a retomada do culto após a profanação do templo. moeda comum o euro, foi criada somente em 1998. Ela não é
O funeral ocorre dois dias depois que centenas de composta por todos os países que fazem parte da União
muçulmanos foram orar nas igrejas da França ao lado de Europeia – atualmente, apenas 19 dos 27 Estados-membros da
católicos na missa de domingo, como um sinal de UE utilizam o euro como moeda. É o caso, por exemplo, da
"solidariedade" e "esperança". República Tcheca, que faz parte da UE mas utiliza como moeda
a coroa tcheca(CZK). Existem ainda alguns pequenos países
Em carta ao bloco, Serra diz que presidência do que não fazem parte da UE oficialmente mas que utilizam o
Mercosul está vaga68 euro, como é o caso de Andorra, Mônaco e do Vaticano.
O ministro das Relações Exteriores, José Serra, enviou Critérios: a criação do euro teve como intenção fortalecer
carta aos chanceleres do Ururguai, Argentina e Paraguai na a economia dos países que o adotassem como moeda e
qual declara que o Brasil considera vaga a presidência do também facilitar e ampliar as relações comerciais entre países
Mercosul. Na última sexta-feira (29/07/16), o Uruguai, então da UE. No entanto, foram estabelecidos alguns critérios para
ocupante do cargo, afirmou que seu período na presidência do adesão à moeda – o país que desejar aderir ao euro deve ter
bloco havia terminado, sem transferir o mandato para um país alta estabilidade nos preços e não apresentar déficit excessivo
sucessor. A sucessora seria a Venezuela. A presidência pro no orçamento nacional, por exemplo.
tempore do Mercosul muda de seis em seis meses, na ordem Espaço Schengen: é a área resultante de uma políticade
alfabética dos países. No entanto, Brasil, Argentina e Paraguai abertura de fronteiras, ou seja, onde a circulação de pessoas é
são contra a presidência dos venezuelanos. Na carta, Serra livre. Ele coincide com grande parte do território da União
disse que a Venezuela não preenche requisitos para presidir o Europeia – atualmente, apenas a Irlanda e Reino (que
Mercosul. atualmente estuda um processo de saída da EU) integram o
Proposta da Argentina: Nesta segunda-feira bloco e não assinaram o acordo de Schegen. Além disso,
(1/08/2016), a Venezuela rejeitou a reunião de Islândia, Noruega e Suíça, que não são Estados-membros da
coordenadores proposta pela Argentina visando resolver as UE, fazem parte do acordo.
divergências em torno da presidência do Mercosul. Fronteiras: dentro do espaço Schengen, as viagens entre
um país e outro são consideradas domésticas. Por isso, não há

67 02/08/2016 – Fonte: 69 22/07/2016 – Fonte:


http://g1.globo.com/mundo/noticia/2016/08/franca-realiza-funeral-de- http://guiadoestudante.abril.com.br/estudar/atualidades/entenda-diferencas-
padre-frances-morto-por-extremistas.html uniao-europeia-zona-euro-espaco-schengen-961601.shtml
68 02/08/2016 – Fonte:
http://g1.globo.com/politica/noticia/2016/08/em-carta-ao-bloco-serra-diz-
que-presidencia-do-mercosul-esta-vaga.html

Atualidades e Deveres dos Servidores Públicos 31


Apostila Digital Licenciada para Alice Caroline Guarino dos Santos - alice.guarino@hotmail.com (Proibida a Revenda)
APOSTILAS OPÇÃO

obrigatoriedade de controle de passaporte nas fronteiras, a particular. A acusação tem como base desdobramentos da
não ser quando se chega de um país de fora ou quando se deixa Operação Acrônimo, que investiga um suposto esquema de
a zona Schegen. financiamento ilegal de campanhas políticas.
Documentos: cidadãos residentes dos países que fazem (Folha de S.Paulo, 06.05.2016. Adaptado)
parte do espaço Schengen podem viajar dentro da zona sem O governador denunciado foi
passaporte, portanto apenas um documento de identidade. Os (A) Geraldo Alckmin, de São Paulo.
turistas, no entanto, devem portar obrigatoriamente o (B) Fernando Pimentel, de Minas Gerais.
passaporte. (C) Luiz Fernando Pezão, do Rio de Janeiro.
Estado de emergência: em casos de risco para a (D) Beto Richa, do Paraná.
segurança nacional de algum país, é possível que ele reative o (E) Flávio Dino, do Maranhão.
controle das suas fronteiras mesmo dentro do espaço
Schengen. É o caso da França, por exemplo, que decretou um 03. (MPE-SP - Analista Técnico Científico – Biólogo –
estado de emergência para o país após os atentados terroristas VUNESP/2016) Depois de diminuir de cinco para quatro os
de novembro de 2015. Com essa medida, a polícia francesa dias úteis do serviço público, o presidente anunciou nesta
voltou a fazer controle de passaporte tanto na entrada quanto quinta-feira (14 de abril) que mudará o fuso horário do país
na saída do país, independentemente da origem ou destino do para economizar energia. O horário de verão é mais uma
viajante. medida do governo para tentar resolver a crise energética. Nos
últimos meses, a seca provocada pelo El Niño diminuiu ainda
Estados Unidos e França podem atacar Mossul, no mais a geração de energia, já afetada pela falta de
Iraque, diz porta-voz francês70 infraestrutura.
O porta-voz do governo francês, Stephane Le Foll, afirmou (Folha de S.Paulo, 14.04.2016. Adaptado)
que os Estados Unidos e a França estão preparando um ataque A notícia trata da situação energética
conjunto contra a cidade de Mossul, no Iraque, que permanece (A) na Venezuela.
sob o controle do grupo terrorista Estado Islâmico, informou a (B) na Bolivia.
agência Reuters nesta quarta-feira (20/07/2016). Mossul é a (C) no Paraguai.
segunda maior cidade do Iraque e a sua população é superior (D) no Peru.
a 500 mil pessoas. Desde 2014, os militantes do Estado (E) no Equador
Islâmico consideram a cidade como sua capital no Iraque.
Combatentes curdos Peshmerga (forças armadas do Curdistão 04. (ELETROBRAS-ELETROSUL - Técnico de Segurança
iraquiano), o exército do Iraque e vários grupos xiitas têm do Trabalho – FCC/2016) As agências Standard & Poor's,
tentado retomar a cidade. A coalizão liderada pelos Estados Moody's e Fitch tornaram-se mais conhecidas dos brasileiros
Unidos é composta por mais de 60 países e vem realizando a partir do ano de 2015 e ainda são notícia neste ano de 2016
ataques aéreos na Síria e no Iraque desde 2014. Na Síria, a porque
campanha é feita sem o consentimento do governo local. (A) indicam a venda do pré-sal como medida de
saneamento econômico da Petrobras e mantêm a
Questões recomendação para que os juros continuem elevados.
(B) defendem amplas reformas políticas, sobretudo no
01. (MPE-SP - Analista Técnico Científico – Biólogo – poder executivo e recomendam novas medidas econômicas,
VUNESP/2016) dentre as quais a reinstalação da CPMF.
Justiça homologa acordo de leniência com Andrade (C) sustentam a perspectiva de deterioração da economia
Gutierrez brasileira e continuam mantendo o rebaixamento do grau de
O juiz federal Sérgio Moro homologou o acordo de investimento do país.
leniência entre a empreiteira Andrade Gutierrez e o Ministério (D) são porta-vozes do governo estadunidense que
Público Federal. pretende a adesão do Brasil ao bloco Transpacífico e
(G1, 08.05.2016.) aconselham o governo a ampliar as reservas cambiais.
Nos termos do acordo, em troca de poder continuar (E) contestam legalmente o afastamento da Presidente e
mantendo contratos com o poder público, a empresa defendem a redução da carga tributária que onera os produtos
(A) decidiu não mais financiar campanhas de candidatos destinados à exportação.
e partidos políticos, assim como se comprometeu a desmontar
o seu escritório de lobby em Brasília. 05. (Prefeitura de Guarulhos – SP - Agente Escolar –
(B) firmou que os seus executivos devem se manifestar VUNESP/2016) O governo e a guerrilha das Farc firmaram
apenas no que for estabelecido expressamente pelos contratos histórico acordo que pretendia por fim a mais de meio século
firmados, para evitar práticas de suborno e corrupção. de conflito armado. O governo e as Farc chegaram ao acordo
(C) resolveu submeter todos os seus contratos a de paz depois de quase quatro anos de negociações. O pacto só
auditorias externas e, a título de transparência, repatriou os se tornaria efetivo se fosse aprovado pela população em um
seus recursos depositados em paraísos fiscais. referendo ocorrido em 2 de outubro.
(D) aceitou pagar R$ 1 bilhão em multas, além de garantir O acordo, rejeitado pela população no referendo, referia-se
a colaboração em todas as investigações de corrupção em que (A) à Bolívia.
possa estar envolvida. (B) à Colômbia.
(E) estabeleceu limites para o valor das obras públicas (C) ao Panamá.
das quais participará de agora em diante, evitando os projetos (D) ao Chile.
mais custosos e com maior risco de corrupção. (E) ao Equador.

02. (MPE-SP - Analista Técnico Científico – Biólogo – 06. (Prefeitura de Guarulhos – SP - Agente Escolar –
VUNESP/2016) A Procuradoria-Geral da República ofereceu VUNESP/2016) O candidato republicano à Presidência dos
nesta sexta- -feira (6 de maio) denúncia ao STJ (Superior Estados Unidos, Donald Trump, foi ao México para se reunir
Tribunal de Justiça) contra o governador pelos crimes de com o presidente Enrique Peña Nieto, um encontro duramente
corrupção, lavagem de dinheiro e falsidade de documento

70 20/07/2016 – Fonte: G1.com.br

Atualidades e Deveres dos Servidores Públicos 32


Apostila Digital Licenciada para Alice Caroline Guarino dos Santos - alice.guarino@hotmail.com (Proibida a Revenda)
APOSTILAS OPÇÃO

criticado pelos mexicanos que desaprovam o discurso do inicial, o risco está associado aos primeiros três meses de
magnata nova-iorquino. gravidez.
(G1, 31.08.2016. Disponível em: <http://goo.gl/XslVgV>. Preenche adequadamente a lacuna no fragmento acima:
Adaptado) (A) O surto de microcefalia na região Nordeste.
Uma das críticas dos mexicanos a Trump deve-se (B) O surto de encefalopatia na região Sul.
(A) à sua proposta de construir um muro na fronteira entre (C) A febre chikungunya na região Sudeste.
os dois países. (D) A dengue hemorrágica na região Norte.
(B) ao seu projeto de criação de uma área de livre comércio (E) A macrocefalia na região Sul.
na América do Norte.
(C) à sua intenção de expulsar todos os mexicanos Respostas
residentes nos EUA, legais ou ilegais.
(D) à sua tentativa de intervir na política interna do país, 01. D / 02. B / 03. A / 04. C / 05. B / 06. A / 07. C
apoiando alguns políticos e não outros.
(E) à sua defesa da entrada da polícia dos EUA no país 08. B / 09. D / 10. A
vizinho com o objetivo de lutar contra o tráfico.

07. (Prefeitura de Piedade – SP - Técnico Legislativo –


PUBLICONSULT/2016) “__________________ foi o responsável
2. Estatuto dos Funcionários
por acender a pira olímpica na cerimônia de abertura dos Públicos Civis do Estado de
Jogos do Rio-2016, nesta sexta-feira (5/08/2016), no São Paulo (Lei n.º 10.261/68)
Maracanã. Ocupou o lugar de Pelé, que era a primeira opção da - artigos 239 a 250; com as
organização. Porém, o ex-jogador de futebol recusou o convite
alterações vigentes até a
alegando problemas físicos que o impossibilitariam de
participar do evento”. publicação deste Edital.
(Fonte: olimpiadas.uol.com.br, 06/08/2016)
Assinale a alternativa que completa corretamente a lacuna:
(A) A ex-jogadora de basquetebol Hortência Marcari
LEI Nº 10.261, DE 28 DE OUTUBRO DE 1968
(B) O tenista tricampeão de Roland Garros, Gustavo
Kuerten
Dispõe sobre o Estatuto dos Funcionários Públicos Civis
(C) O ex-maratonista Vanderlei Cordeiro de Lima
do Estado
(D) O ex-jogador de basquetebol, Oscar Schmidt
[...]
08. (Prefeitura de Cipotânea – MG – Enfermeiro - REIS
CAPÍTULO VII
& REIS/2016) “Apontada como um mecanismo importante de
Do Direito de Petição
financiamento cultural no Brasil, a ________________ é
constantemente alvo de críticas e voltou ao debate nacional
Artigo 239 - É assegurado a qualquer pessoa, física ou
por causa da extinção – agora revertida – do Ministério da
jurídica, independentemente de pagamento, o direito de
Cultura na gestão interina de Michel Temer. Esta Lei foi criada
petição contra ilegalidade ou abuso de poder e para defesa de
em 1991, durante o governo Collor, e permite que produtores
direitos.
e instituições captem, junto a pessoas físicas e jurídicas,
§ 1º - Qualquer pessoa poderá reclamar sobre abuso, erro,
recursos para financiar projetos culturais. O valor destinado a
omissão ou conduta incompatível no serviço público.
esses projetos pode ser deduzido integralmente do Imposto de
§ 2º - Em nenhuma hipótese, a Administração poderá
Renda a pagar.”
recusar-se a protocolar, encaminhar ou apreciar a petição, sob
Marque a alternativa que completa corretamente o
pena de responsabilidade do agente.
enunciado acima:
(A) Lei Collor.
Artigo 240 - Ao servidor é assegurado o direito de requerer
(B) Lei Rouanet.
ou representar, bem como, nos termos desta lei
(C) Lei de Diretrizes e Bases da Educação.
complementar, pedir reconsideração e recorrer de decisões,
(D) Lei Echer.
no prazo de 30 (trinta) dias, salvo previsão legal específica.
09. (Prefeitura de Portão – RS - Agente Admnistrativo -
TÍTULO VI
LEGALLE Concursos/2016) Como se chama a maior
DOS DEVERES, DAS PROIBIÇÕES E DAS
investigação sobre corrupção conduzida até hoje no país, que
RESPONSABILIDADES
começou investigando uma rede de doleiros que atuavam em
CAPÍTULO I
vários Estados e descobriu a existência de um vasto esquema
Dos Deveres e das Proibições
de corrupção na Petrobrás?
SEÇÃO I
(A) Operação Mala Preta.
Dos Deveres
(B) Operação Aguas Profundas.
(C) Operação 13 de maio.
Artigo 241 - São deveres do funcionário:
(D) Operação Lava Jato.
I - ser assíduo e pontual;
(E) Operação Jaguatirica.
II - cumprir as ordens superiores, representando quando
forem manifestamente ilegais;
10. (Prefeitura de Portão – RS - Agente Admnistrativo -
III - desempenhar com zelo e presteza os trabalhos de que
LEGALLE Concursos/2016) O Ministério da Saúde (BRASIL,
for incumbido;
2016) confirmou recentemente a relação entre o vírus Zika e
IV - guardar sigilo sobre os assuntos da repartição e,
_______________. As investigações sobre o tema devem continuar
especialmente, sobre despachos, decisões ou providências;
para esclarecer questões como a transmissão desse agente, a
V - representar aos superiores sobre todas as
sua atuação no organismo humano, a infecção do feto e
irregularidades de que tiver conhecimento no exercício de
período de maior vulnerabilidade para a gestante. Em análise
suas funções;

Atualidades e Deveres dos Servidores Públicos 33


Apostila Digital Licenciada para Alice Caroline Guarino dos Santos - alice.guarino@hotmail.com (Proibida a Revenda)
APOSTILAS OPÇÃO

VI - tratar com urbanidade os companheiros de serviço e IX - constituir-se procurador de partes ou servir de


as partes; intermediário perante qualquer repartição pública, exceto
VI - tratar com urbanidade as pessoas; quando se tratar de interesse de cônjuge ou parente até
VII - residir no local onde exerce o cargo ou, onde segundo grau;
autorizado; X - receber estipêndios de firmas fornecedoras ou de
VIII - providenciar para que esteja sempre em ordem, no entidades fiscalizadas, no País, ou no estrangeiro, mesmo
assentamento individual, a sua declaração de família; quando estiver em missão referente à compra de material ou
IX - zelar pela economia do material do Estado e pela fiscalização de qualquer natureza;
conservação do que for confiado à sua guarda ou utilização; XI - valer-se de sua qualidade de funcionário para
X - apresentar -se convenientemente trajado em serviço ou desempenhar atividade estranha às funções ou para lograr,
com uniforme determinado, quando for o caso; direta ou indiretamente, qualquer proveito; e
XI - atender prontamente, com preferência sobre qualquer XII - fundar sindicato de funcionários ou deles fazer parte.
outro serviço, às requisições de papéis, documentos, Parágrafo único — Não está compreendida na proibição
informações ou providências que lhe forem feitas pelas dos itens II e VI deste artigo, a participação do funcionário em
autoridades judiciárias ou administrativas, para defesa do sociedades em que o Estado seja acionista, bem assim na
Estado, em Juízo; direção ou gerência de cooperativas e associações de classe, ou
XII - cooperar e manter espírito de solidariedade com os como seu sócio.
companheiros de trabalho,
XIII - estar em dia com as leis, regulamentos, regimentos, Artigo 244 - É vedado ao funcionário trabalhar sob as
instruções e ordens de serviço que digam respeito às suas ordens imediatas de parentes, até segundo grau, salvo quando
funções; e se tratar de função de confiança e livre escolha, não podendo
XIV - proceder na vida pública e privada na forma que exceder a 2 (dois) o número de auxiliares nessas condições.
dignifique a função pública.
CAPÍTULO II
SEÇÃO II Das Responsabilidades
Das Proibições
Artigo 245 - O funcionário é responsável por todos os
Artigo 242 - Ao funcionário é proibido: prejuízos que, nessa qualidade, causar à Fazenda Estadual, por
I - (Revogado). dolo ou culpa, devidamente apurados.
II - retirar, sem prévia permissão da autoridade Parágrafo único - Caracteriza-se especialmente a
competente, qualquer documento ou objeto existente na responsabilidade:
repartição; I - pela sonegação de valores e objetos confiados à sua
III - entreter-se, durante as horas de trabalho, em guarda ou responsabilidade, ou por não prestar contas, ou por
palestras, leituras ou outras atividades estranhas ao serviço; não as tomar, na forma e no prazo estabelecidos nas leis,
IV - deixar de comparecer ao serviço sem causa justificada; regulamentos, regimentos, instruções e ordens de serviço;
V - tratar de interesses particulares na repartição; II - pelas faltas, danos, avarias e quaisquer outros prejuízos
VI - promover manifestações de apreço ou desapreço que sofrerem os bens e os materiais sob sua guarda, ou sujeitos
dentro da repartição, ou tornar-se solidário com elas; a seu exame ou fiscalização;
VII - exercer comércio entre os companheiros de serviço, III - pela falta ou inexatidão das necessárias averbações nas
promover ou subscrever listas de donativos dentro da notas de despacho, guias e outros documentos da receita, ou
repartição; e que tenham com eles relação; e
VIII - empregar material do serviço público em serviço IV - por qualquer erro de cálculo ou redução contra a
particular. Fazenda Estadual.

Artigo 243 - É proibido ainda, ao funcionário: Artigo 246 - O funcionário que adquirir materiais em
I - fazer contratos de natureza comercial e industrial com desacordo com disposições legais e regulamentares, será
o Governo, por si, ou como representante de outrem; responsabilizado pelo respectivo custo, sem prejuízo das
II - participar da gerência ou administração de empresas penalidades disciplinares cabíveis, podendo-se proceder ao
bancárias ou industriais, ou de sociedades comerciais, que desconto no seu vencimento ou remuneração.
mantenham relações comerciais ou administrativas com o
Governo do Estado, sejam por este subvencionadas ou estejam Artigo 247 - Nos casos de indenização à Fazenda Estadual,
diretamente relacionadas com a finalidade da repartição ou o funcionário será obrigado a repor, de uma só vez, a
serviço em que esteja lotado; importância do prejuízo causado em virtude de alcance,
III - requerer ou promover a concessão de privilégios, desfalque, remissão ou omissão em efetuar recolhimento ou
garantias de juros ou outros favores semelhantes, federais, entrada nos prazos legais.
estaduais ou municipais, exceto privilégio de invenção Artigo 248 - Fora dos casos incluídos no artigo anterior, a
própria; importância da indenização poderá ser descontada do
IV - exercer, mesmo fora das horas de trabalho, emprego vencimento ou remuneração não excedendo o desconto à 10ª
ou função em empresas, estabelecimentos ou instituições que (décima) parte do valor destes.
tenham relações com o Governo, em matéria que se relacione Parágrafo único - No caso do item IV do parágrafo único do
com a finalidade da repartição ou serviço em que esteja lotado; art. 245, não tendo havido má-fé, será aplicada a pena de
V - aceitar representação de Estado estrangeiro, sem repreensão e, na reincidência, a de suspensão.
autorização do Presidente da República;
VI - comerciar ou ter parte em sociedades comerciais nas Artigo 249 - Será igualmente responsabilizado o
condições mencionadas no item II deste artigo, podendo, em funcionário que, fora dos casos expressamente previstos nas
qualquer caso, ser acionista, quotista ou comanditário; leis, regulamentos ou regimentos, cometer a pessoas
VII - incitar greves ou a elas aderir, ou praticar atos de estranhas às repartições, o desempenho de encargos que lhe
sabotagem contra o serviço público; competirem ou aos seus subordinados.
VIII - praticar a usura;

Atualidades e Deveres dos Servidores Públicos 34


Apostila Digital Licenciada para Alice Caroline Guarino dos Santos - alice.guarino@hotmail.com (Proibida a Revenda)
APOSTILAS OPÇÃO

Artigo 250 - A responsabilidade administrativa não exime 05. Acerca do Estatuto dos Funcionários Públicos Civis do
o funcionário da responsabilidade civil ou criminal que no caso Estado de São Paulo, julgue o item abaixo:
couber, nem o pagamento da indenização a que ficar obrigado, A responsabilidade administrativa é independente da civil
na forma dos arts. 247 e 248, o exame da pena disciplinar em e da criminal.
que incorrer. (...) Certo
§ 1º - A responsabilidade administrativa é independente (...) Errado
da civil e da criminal.
§ 2º - Será reintegrado ao serviço público, no cargo que Respostas
ocupava e com todos os direitos e vantagens devidas, o
servidor absolvido pela Justiça, mediante simples 01. E / 02. E / 03. B / 04. Certo / 05. Certo
comprovação do trânsito em julgado de decisão que negue a
existência de sua autoria ou do fato que deu origem à sua
demissão. 3. Lei Federal nº 8.429/92
§ 3º - O processo administrativo só poderá ser sobrestado
para aguardar decisão judicial por despacho motivado da (Lei de Improbidade
autoridade competente para aplicar a pena. Administrativa) artigos 1º
ao 11º – com as alterações
Questões vigentes até a publicação do
01. A Lei n° 10.261/68 dispõe que ao funcionário público
Edital.
é proibido:
(A) fazer parte dos quadros sociais de qualquer tipo de
sociedade comercial. IMPROBIDADE ADMINISTRATIVA
(B) deixar de comparecer ao serviço, mesmo que por causa
justificada. Noções Gerais.
(C) participar da gerência de sociedades comerciais,
mesmo daquelas que não mantenham relações comerciais ou Improbidade administrativa é um termo técnico,
administrativas com o Governo do Estado. designativo para falar de corrupção administrativa.
(D) exercer, mesmo fora das horas de trabalho, emprego Ela se promove sobre diversas formas, entre elas: pelo
ou função em qualquer tipo de empresa. desvirtuamento da função pública (a Administração Pública) e
(E) empregar material do serviço público em serviço da ordem jurídica.
particular. A improbidade se revela com a aquisição de vantagens
patrimoniais indevidas (a expensas do erário).
02. Sobre a responsabilidade dos funcionários públicos, é
correto afirmar, nos moldes da Lei n° 10.261/68, que: O ilícito de improbidade possui natureza jurídica de ilícito
(A) o funcionário é responsável por todos os prejuízos que, civil, entretanto, isso não afasta possível incidência das esferas
nessa qualidade, causar à Fazenda Estadual, penal e administrativas (lembre-se que as esferas cível, penal
independentemente de dolo ou culpa, devidamente apurados. e administrativas são independentes entre si).
(B) a responsabilidade administrativa exime o funcionário
da responsabilidade civil. O art. 2º da Lei 8.429/92 dispõe que agente público que
(C) a responsabilidade administrativa do funcionário exerce, ainda que transitoriamente ou sem remuneração, por
depende da criminal e da civil. eleição, nomeação, designação, contratação ou qualquer outra
(D) o funcionário que for absolvido pela justiça em forma de investidura ou vínculo, mandato, cargo, emprego ou
processo criminal, por qualquer motivo, não responderá civil função em qualquer entidade da administração direta,
e administrativamente pelo mesmo fato. indireta, fundacional ou autárquica de qualquer dos Poderes
(E) o processo administrativo só poderá ser sobrestado da União, Estados ou Municípios e de empresas incorporadas
para aguardar decisão judicial por despacho motivado da ao patrimônio público poderão responder por atos ímprobos.
autoridade competente para aplicar a pena.
Atenção!
03. Nos termos do que expressamente estabelece a Lei n° Há grande divergência quanto à aplicação da lei de
10.261/68, é dever do funcionário público: improbidade aos agentes políticos. Veja como tem se
(A) cumprir as ordens superiores, mesmo quando forem posicionado o STF e o STJ sobre o assunto:
manifestamente ilegais. STF: tem entendido que os agentes políticos não estão
(B) residir no local onde exerce o cargo ou onde sujeitos à Lei de improbidade, isso porque o crime de
autorizado. responsabilidade estipula sanções de natureza civil e seria bis
(C) guardar sigilo sobre os assuntos da repartição, exceto in idem admitir as duas punições.
sobre despachos, decisões ou providências. STJ: excetuada a hipótese de atos de improbidade
(D) manter sigilo sobre as irregularidades de que tiver praticados pelo Presidente da República e Ministros de Estado
conhecimento no exercício de suas funções, deixando eventual em crimes conexos com este, não há norma alguma que proíba
investigação para as autoridades competentes. que os agentes políticos respondam por crimes de
(E) providenciar para que estejam sempre em ordem todas responsabilidade e por atos de improbidade. (Reclamações
as mesas de trabalho da repartição onde exerce suas funções. 2790 e 2115).

04. Acerca do Estatuto dos Funcionários Públicos Civis do Os atos de improbidade estão disciplinados nos arts. 9º a
Estado de São Paulo, julgue o item abaixo: 11 da Lei 8.429/92 e dispõe sobre hipóteses de enriquecimento
É assegurado a qualquer pessoa, física ou jurídica, ilícito, atos que causem prejuízo ao erário e atos que atentam
independentemente de pagamento, o direito de petição contra contra os princípios da Administração Pública. Esses eram os
ilegalidade ou abuso de poder e para defesa de direitos. atos previstos até o final do ano 2016, com a LC 157/16
(...) Certo introduziu-se nova Seção ao Capítulo. Agora, além daqueles
(...) Errado atos acima descritos poderá configura ato de improbidade a

Atualidades e Deveres dos Servidores Públicos 35


Apostila Digital Licenciada para Alice Caroline Guarino dos Santos - alice.guarino@hotmail.com (Proibida a Revenda)
APOSTILAS OPÇÃO

ação ou omissão decorrentes de concessão ou aplicação Parágrafo único. A indisponibilidade a que se refere o
indevida de benefício financeiro ou tributário (art. 10-A). caput deste artigo recairá sobre bens que assegurem o integral
ressarcimento do dano, ou sobre o acréscimo patrimonial
Quanto ao elemento subjetivo desses atos, regra geral, ato resultante do enriquecimento ilícito.
de improbidade exige DOLO.
Art. 8° O sucessor daquele que causar lesão ao patrimônio
Vamos à leitura na íntegra da lei: público ou se enriquecer ilicitamente está sujeito às
cominações desta lei até o limite do valor da herança.
LEI Nº 8.429, DE 2 DE JUNHO DE 1992.
CAPÍTULO II
Dispõe sobre as sanções aplicáveis aos agentes públicos Dos Atos de Improbidade Administrativa
nos casos de enriquecimento ilícito no exercício de mandato, Seção I
cargo, emprego ou função na administração pública direta, Dos Atos de Improbidade Administrativa que
indireta ou fundacional e dá outras providências. Importam Enriquecimento Ilícito

O PRESIDENTE DA REPÚBLICA, Faço saber que o Art. 9° Constitui ato de improbidade administrativa
Congresso Nacional decreta e eu sanciono a seguinte lei: importando enriquecimento ilícito auferir qualquer tipo de
vantagem patrimonial indevida em razão do exercício de
CAPÍTULO I cargo, mandato, função, emprego ou atividade nas entidades
Das Disposições Gerais mencionadas no art. 1° desta lei, e notadamente:
I - receber, para si ou para outrem, dinheiro, bem móvel ou
Art. 1° Os atos de improbidade praticados por qualquer imóvel, ou qualquer outra vantagem econômica, direta ou
agente público, servidor ou não, contra a administração direta, indireta, a título de comissão, percentagem, gratificação ou
indireta ou fundacional de qualquer dos Poderes da União, dos presente de quem tenha interesse, direto ou indireto, que
Estados, do Distrito Federal, dos Municípios, de Território, de possa ser atingido ou amparado por ação ou omissão
empresa incorporada ao patrimônio público ou de entidade decorrente das atribuições do agente público;
para cuja criação ou custeio o erário haja concorrido ou II - perceber vantagem econômica, direta ou indireta, para
concorra com mais de cinquenta por cento do patrimônio ou facilitar a aquisição, permuta ou locação de bem móvel ou
da receita anual, serão punidos na forma desta lei. imóvel, ou a contratação de serviços pelas entidades referidas
Parágrafo único. Estão também sujeitos às penalidades no art. 1° por preço superior ao valor de mercado;
desta lei os atos de improbidade praticados contra o III - perceber vantagem econômica, direta ou indireta, para
patrimônio de entidade que receba subvenção, benefício ou facilitar a alienação, permuta ou locação de bem público ou o
incentivo, fiscal ou creditício, de órgão público bem como fornecimento de serviço por ente estatal por preço inferior ao
daquelas para cuja criação ou custeio o erário haja concorrido valor de mercado;
ou concorra com menos de cinquenta por cento do patrimônio IV - utilizar, em obra ou serviço particular, veículos,
ou da receita anual, limitando-se, nestes casos, a sanção máquinas, equipamentos ou material de qualquer natureza, de
patrimonial à repercussão do ilícito sobre a contribuição dos propriedade ou à disposição de qualquer das entidades
cofres públicos. mencionadas no art. 1° desta lei, bem como o trabalho de
servidores públicos, empregados ou terceiros contratados por
Art. 2° Reputa-se agente público, para os efeitos desta lei, essas entidades;
todo aquele que exerce, ainda que transitoriamente ou sem V - receber vantagem econômica de qualquer natureza,
remuneração, por eleição, nomeação, designação, contratação direta ou indireta, para tolerar a exploração ou a prática de
ou qualquer outra forma de investidura ou vínculo, mandato, jogos de azar, de lenocínio, de narcotráfico, de contrabando, de
cargo, emprego ou função nas entidades mencionadas no usura ou de qualquer outra atividade ilícita, ou aceitar
artigo anterior. promessa de tal vantagem;
VI - receber vantagem econômica de qualquer natureza,
Art. 3° As disposições desta lei são aplicáveis, no que direta ou indireta, para fazer declaração falsa sobre medição
couber, àquele que, mesmo não sendo agente público, induza ou avaliação em obras públicas ou qualquer outro serviço, ou
ou concorra para a prática do ato de improbidade ou dele se sobre quantidade, peso, medida, qualidade ou característica de
beneficie sob qualquer forma direta ou indireta. mercadorias ou bens fornecidos a qualquer das entidades
mencionadas no art. 1º desta lei;
Art. 4° Os agentes públicos de qualquer nível ou hierarquia VII - adquirir, para si ou para outrem, no exercício de
são obrigados a velar pela estrita observância dos princípios mandato, cargo, emprego ou função pública, bens de qualquer
de legalidade, impessoalidade, moralidade e publicidade no natureza cujo valor seja desproporcional à evolução do
trato dos assuntos que lhe são afetos. patrimônio ou à renda do agente público;
VIII - aceitar emprego, comissão ou exercer atividade de
Art. 5° Ocorrendo lesão ao patrimônio público por ação ou consultoria ou assessoramento para pessoa física ou jurídica
omissão, dolosa ou culposa, do agente ou de terceiro, dar-se-á que tenha interesse suscetível de ser atingido ou amparado
o integral ressarcimento do dano. por ação ou omissão decorrente das atribuições do agente
público, durante a atividade;
Art. 6° No caso de enriquecimento ilícito, perderá o agente IX - perceber vantagem econômica para intermediar a
público ou terceiro beneficiário os bens ou valores acrescidos liberação ou aplicação de verba pública de qualquer natureza;
ao seu patrimônio. X - receber vantagem econômica de qualquer natureza,
direta ou indiretamente, para omitir ato de ofício, providência
Art. 7° Quando o ato de improbidade causar lesão ao ou declaração a que esteja obrigado;
patrimônio público ou ensejar enriquecimento ilícito, caberá a XI - incorporar, por qualquer forma, ao seu patrimônio
autoridade administrativa responsável pelo inquérito bens, rendas, verbas ou valores integrantes do acervo
representar ao Ministério Público, para a indisponibilidade patrimonial das entidades mencionadas no art. 1° desta lei;
dos bens do indiciado.

Atualidades e Deveres dos Servidores Públicos 36


Apostila Digital Licenciada para Alice Caroline Guarino dos Santos - alice.guarino@hotmail.com (Proibida a Revenda)
APOSTILAS OPÇÃO

XII - usar, em proveito próprio, bens, rendas, verbas ou Seção II-A


valores integrantes do acervo patrimonial das entidades Dos Atos de Improbidade Administrativa Decorrentes
mencionadas no art. 1° desta lei. de Concessão ou Aplicação Indevida de Benefício
Financeiro ou Tributário
Seção II (Incluído pela Lei Complementar 157/16)
Dos Atos de Improbidade Administrativa que Causam
Prejuízo ao Erário Art. 10-A. Constitui ato de improbidade administrativa
qualquer ação ou omissão para conceder, aplicar ou manter
Art. 10. Constitui ato de improbidade administrativa que benefício financeiro ou tributário contrário ao que dispõem o
causa lesão ao erário qualquer ação ou omissão, dolosa ou caput e o §1º do art. 8º-A da Lei Complementar nº 116, de 31
culposa, que enseje perda patrimonial, desvio, apropriação, de julho de 2003.
malbaratamento ou dilapidação dos bens ou haveres das
entidades referidas no art. 1º desta lei, e notadamente: Seção III
I - facilitar ou concorrer por qualquer forma para a Dos Atos de Improbidade Administrativa que
incorporação ao patrimônio particular, de pessoa física ou Atentam Contra os Princípios da Administração Pública
jurídica, de bens, rendas, verbas ou valores integrantes do
acervo patrimonial das entidades mencionadas no art. 1º desta Art. 11. Constitui ato de improbidade administrativa que
lei; atenta contra os princípios da administração pública qualquer
II - permitir ou concorrer para que pessoa física ou jurídica ação ou omissão que viole os deveres de honestidade,
privada utilize bens, rendas, verbas ou valores integrantes do imparcialidade, legalidade, e lealdade às instituições, e
acervo patrimonial das entidades mencionadas no art. 1º desta notadamente:
lei, sem a observância das formalidades legais ou I - praticar ato visando fim proibido em lei ou regulamento
regulamentares aplicáveis à espécie; ou diverso daquele previsto, na regra de competência;
III - doar à pessoa física ou jurídica bem como ao ente II - retardar ou deixar de praticar, indevidamente, ato de
despersonalizado, ainda que de fins educativos ou ofício;
assistências, bens, rendas, verbas ou valores do patrimônio de III - revelar fato ou circunstância de que tem ciência em
qualquer das entidades mencionadas no art. 1º desta lei, sem razão das atribuições e que deva permanecer em segredo;
observância das formalidades legais e regulamentares IV - negar publicidade aos atos oficiais;
aplicáveis à espécie; V - frustrar a licitude de concurso público;
IV - permitir ou facilitar a alienação, permuta ou locação de VI - deixar de prestar contas quando esteja obrigado a fazê-
bem integrante do patrimônio de qualquer das entidades lo;
referidas no art. 1º desta lei, ou ainda a prestação de serviço VII - revelar ou permitir que chegue ao conhecimento de
por parte delas, por preço inferior ao de mercado; terceiro, antes da respectiva divulgação oficial, teor de medida
V - permitir ou facilitar a aquisição, permuta ou locação de política ou econômica capaz de afetar o preço de mercadoria,
bem ou serviço por preço superior ao de mercado; bem ou serviço.
VI - realizar operação financeira sem observância das VIII - XVI a XXI - (Vide Lei nº 13.019, de 2014)
normas legais e regulamentares ou aceitar garantia IX - (Vide Lei nº 13.146, de 2015)
insuficiente ou inidônea;
VII - conceder benefício administrativo ou fiscal sem a Questões
observância das formalidades legais ou regulamentares
aplicáveis à espécie; 01. (TJ-RS - Assessor Judiciário – FAURGS/2016)
VIII - frustrar a licitude de processo licitatório ou dispensá- Quanto à Lei de Improbidade Administrativa, assinale a
lo indevidamente; (Vide Lei nº 13.019, de 2014) alternativa correta.
IX - ordenar ou permitir a realização de despesas não (A) As normas da Lei de Improbidade Administrativa não
autorizadas em lei ou regulamento; se aplicam a quem não seja agente público.
X - agir negligentemente na arrecadação de tributo ou (B) Os agentes de sociedades de economia mista, por
renda, bem como no que diz respeito à conservação do estarem submetidos a uma relação de emprego, não estão
patrimônio público; submetidos aos comandos da Lei de Improbidade
XI - liberar verba pública sem a estrita observância das Administrativa.
normas pertinentes ou influir de qualquer forma para a sua (C) A ação civil por ato de improbidade administrativa, por
aplicação irregular; depender de atuação institucional e apuração de
XII - permitir, facilitar ou concorrer para que terceiro se responsabilidades, só pode ser movida pelo Ministério
enriqueça ilicitamente; Público.
XIII - permitir que se utilize, em obra ou serviço particular, (D) Quando o Ministério Público ingressa com a ação civil
veículos, máquinas, equipamentos ou material de qualquer de improbidade administrativa, a pessoa jurídica de direito
natureza, de propriedade ou à disposição de qualquer das público ou de direito privado, cujo ato seja objeto de
entidades mencionadas no art. 1° desta lei, bem como o impugnação, poderá abster-se de contestar o pedido, ou
trabalho de servidor público, empregados ou terceiros poderá atuar ao lado do autor, desde que isso se afigure útil ao
contratados por essas entidades. interesse público, a juízo do respectivo representante legal ou
XIV – celebrar contrato ou outro instrumento que tenha dirigente.
por objeto a prestação de serviços públicos por meio da gestão (E) O sucessor do causador do dano em ato de
associada sem observar as formalidades previstas na lei; improbidade administrativa não sofre qualquer
XV – celebrar contrato de rateio de consórcio público sem responsabilização patrimonial, ainda que tenha recebido
suficiente e prévia dotação orçamentária, ou sem observar as herança ou legado do infrator.
formalidades previstas na lei.
XVI a XXI - (Vide Lei nº 13.019, de 2014) 02. (PC-PE - Escrivão de Polícia – CESPE/2016) Assinale
a opção correta com referência a improbidade administrativa
e à Lei de Improbidade Administrativa (Lei n.° 8.429/1992).
(A) A aplicação administrativa da pena de demissão
prevista em lei reguladora de carreira pública exige que se

Atualidades e Deveres dos Servidores Públicos 37


Apostila Digital Licenciada para Alice Caroline Guarino dos Santos - alice.guarino@hotmail.com (Proibida a Revenda)
APOSTILAS OPÇÃO

aguarde o trânsito em julgado da ação de improbidade


administrativa.
(B) Os atos de improbidade descritos no art. 11 da Lei n.°
8.429'1992 não exigem a presença do dolo para sua Anotações
configuração.
(C) Os atos de improbidade descritos no art. 11 da Lei n.°
8.429'1992, para sua configuração, exigem a demonstração da
ocorrência de dano para a administração pública ou
enriquecimento ilícito do agente.
(D) A punição administrativa do servidor faltoso impede a
aplicação das penas previstas na Lei de Improbidade
Administrativa (Lei n.° 8.429/1992).
(E) O atentado à vida e à liberdade individual de
particulares, se praticado por agentes públicos armados, pode
configurar improbidade administrativa.

03. (FUB - Conhecimentos Básicos - Cargos de 1 a 7 –


CESPE/2016) Julgue o item que se segue, de acordo com o
disposto na Lei de Improbidade Administrativa.
Não dar publicidade a ato oficial configura ato de
improbidade administrativa.
(...) Certo
(...) Errado

04. (DER-CE - Procurador Autárquico - UECE-


CEV/2016) É exemplo de ato de improbidade administrativa
que importa enriquecimento ilícito
(A) permitir ou facilitar a aquisição, permuta ou locação de
bem ou serviço por preço superior ao de mercado.
(B) realizar operação financeira sem observância das
normas legais e regulamentares ou aceitar garantia
insuficiente ou inidônea.
(C) retardar ou deixar de praticar, indevidamente, ato de
ofício.
(D) aceitar emprego, comissão ou exercer atividade de
consultoria ou assessoramento para pessoa física ou jurídica
que tenha interesse suscetível de ser atingido ou amparado
por ação ou omissão decorrente das atribuições do agente
público, durante a atividade.

05. (ANS - Técnico Administrativo – FUNCAB/2016)


Assinale a alternativa correta de acordo com a Lei n°
8.429/1992 e os atos de improbidade administrativa.
(A) Apenas os agentes públicos de hierarquia superior são
obrigados a velar pela estrita observância dos princípios de
legalidade, impessoalidade, moralidade e publicidade no trato
dos assuntos que lhe são afetos.
(B) Somente os agentes públicos poderão representar à
autoridade administrativa competente para que seja
instaurada investigação destinada a apurar a prática de ato de
improbidade.
(C) O sucessor daquele que causar lesão ao patrimônio
público ou se enriquecer ilicitamente não está sujeito às
cominações desta lei.
(D) Não constitui ato de improbidade administrativa
importando enriquecimento ilícito perceber vantagem
econômica para intermediar a liberação ou aplicação de verba
pública de qualquer natureza.
(E) As disposições dessa lei são aplicáveis, no que couber,
àquele que, mesmo não sendo agente público, induza ou
concorra para a prática do ato de improbidade ou dele se
beneficie sob qualquer forma direta ou indireta.

Respostas

01. D. / 02. E. / 03. Certo. / 04. D. / 05. E.

Atualidades e Deveres dos Servidores Públicos 38


Apostila Digital Licenciada para Alice Caroline Guarino dos Santos - alice.guarino@hotmail.com (Proibida a Revenda)
INFORMÁTICA

Apostila Digital Licenciada para Alice Caroline Guarino dos Santos - alice.guarino@hotmail.com (Proibida a Revenda)
Apostila Digital Licenciada para Alice Caroline Guarino dos Santos - alice.guarino@hotmail.com (Proibida a Revenda)
APOSTILAS OPÇÃO
Domine a Web com o Microsoft Edge
O Microsoft Edge é o primeiro navegador que permite fazer
anotações, escrever, rabiscar e realçar diretamente em páginas
da Web. Use a lista de leitura Ícone Lista de leitura para salvar
seus artigos favoritos para mais tarde e lê-los no modo de leitura
Ícone Modo de leitura. Focalize guias abertas para visualizá-las e
leve seus favoritos e sua lista de leitura com você quando usar o
Microsoft Edge em outro dispositivo.
MS-Windows 10: conceito de
pastas, diretórios, arquivos e
atalhos, área de trabalho, área
de transferência, manipulação
de arquivos e pastas, uso dos
menus, programas e aplicativos,
interação com o conjunto de
aplicativos MS-Office 2016,

Windows 101

O Windows 10 está repleto de novos recursos e melhorias.


Confira os destaques! Onde você pode digitar, também pode escrever
O menu Iniciar está de volta. E ele está mais pessoal, mais O Microsoft Edge não é o único aplicativo em que você pode
organizado e mais divertido do que nunca. escrever. Use sua caneta eletrônica, o dedo ou o mouse para es-
crever em todos os lugares onde antes você digitava. Ou simples-
mente rabisque no OneNote. Não contaremos a ninguém.

Selecione o botão Iniciar na barra de tarefas. Você encontra-


rá seus aplicativos mais usados no lado esquerdo, a lista Todos
os aplicativos e atalhos para outros locais no computador, como Todas as suas fotos em um só lugar
Explorador de Arquivos e Configurações. Chega de pesquisas infinitas. O aplicativo Fotos reúne todas
Baixe aplicativos, músicas e muito mais... as suas fotos e vídeos em um único local. De seu telefone, com-
putador e OneDrive. Em seguida, ele organiza suas memórias em
álbuns para você aproveitar e compartilhar.

Aplicativo Fotos aprimorado e muito mais

A Loja é uma loja centralizada para músicas, vídeos, jogos e


aplicativos. Crie seus próprios álbuns de fotos ou curta belos álbuns que
Experimente um aplicativo antes de comprá-lo ou escolha o aplicativo Fotos avançado cria para você com suas melhores
um gratuito. Seus aplicativos Windows 10 funcionarão em todos fotos. Também é mais fácil encontrar essas fotos, com formas
os seus dispositivos Windows 10. melhores de navegar pelas pastas de fotos e suas subpastas —
do seu disco rígido, de uma unidade externa ou do OneDrive. E,
se você tiver imagens em ação em seu telefone Windows, com-
1 Fonte: http://windows.microsoft.com/pt-br/windows-10/getstar- partilhe-as por email e nas mídias sociais.
ted-search-for-help

Informática 1
Apostila Digital Licenciada para Alice Caroline Guarino dos Santos - alice.guarino@hotmail.com (Proibida a Revenda)
APOSTILAS OPÇÃO
Melhor multitarefa para fazer o trabalho Procure ajuda
Precisa de ajuda com o Windows 10? Marque a caixa de sele-
ção e digite uma palavra-chave ou uma pergunta, e você encon-
trará ajuda da Microsoft.

Encontre rápido
Se você não sabe onde encontrar uma configuração ou um
recurso, há uma grande chance de que uma única palavra mos-
trará o caminho para você. Por exemplo, digite suspensão e você
será direcionado para a página de configurações, onde poderá
alterar as configurações de suspensão do computador. Ou digite
desinstalar para encontrar a página de configurações, onde você
pode exibir ou desinstalar aplicativos.

Deslize a borda compartilhada de aplicativos da área de tra-


balho ajustados para onde quiser, redimensionando com facili-
dade ambos os aplicativos com um movimento, assim como no
modo tablet.

Procure por qualquer coisa, em qualquer lugar


Use a barra de tarefas para pesquisar em seu computador e
na Web para encontrar ajuda, aplicativos, arquivos, configura-
ções, o que você quiser.

Use a caixa de pesquisa


Digite o que você está procurando na caixa de pesquisa da
barra de tarefas. Você receberá sugestões e respostas para suas
dúvidas e resultados de pesquisa de seu computador e da Inter-
net.

Pesquisar meu conteúdo


Depois de digitar um termo de pesquisa, selecione Meu
conteúdo para encontrar resultados para arquivos, aplicativos,
configurações, fotos, vídeos e músicas em seu computador e no
OneDrive.

Respostas rápidas
Para algumas das perguntas mais frequentes sobre o Win-
dows, há uma resposta pronta. Basta digitar uma pergunta, por
exemplo: Como faço para excluir meu histórico de navegação ou
Como usar várias áreas de trabalho no Windows 10.

Iniciar
O menu Iniciar voltou e está melhor do que nunca! Basta
selecionar o botão Iniciar Ícone Iniciar na barra de tarefas. Em
seguida, personalize-o fixando aplicativos e programas ou mo-
vendo e reagrupando blocos. Se precisar de mais espaço, redi-
mensione o menu Iniciar para aumentá-lo.
Seus aplicativos e programas — ao alcance de seus dedos
Acesse Explorador de Arquivos, Configurações e outros apli-
cativos usados com frequência do lado esquerdo do menu Ini-
ciar. Para ver todos os seus aplicativos e programas, selecione
Todos os aplicativos.

Informática 2
Apostila Digital Licenciada para Alice Caroline Guarino dos Santos - alice.guarino@hotmail.com (Proibida a Revenda)
APOSTILAS OPÇÃO

Dica
Está vendo uma seta à direita de um aplicativo? Selecione-a Veja o menu Iniciar em tela inteira
para ver as tarefas ou itens específicos do aplicativo. Para exibir o menu Iniciar em tela inteira e ver tudo em uma
única exibição, selecione o botão Iniciar ícone Iniciar, Configu-
rações > Personalização > Iniciar e ative Usar Iniciar em tela in-
teira.
Selecione o Menu no canto superior esquerdo da tela para
obter a imagem de sua conta, as listas Todos os aplicativos e
Mais usados e o botão de energia.

Você tem a força


Bloqueie o computador ou saia dele, mude para outra conta
ou altere a imagem da conta selecionando seu nome na parte
superior do menu Iniciar.

Se você deseja apenas redimensionar um pouco o menu Ini-


ciar para torná-lo mais alto ou mais largo, selecione a borda su-
perior ou lateral e arraste-a.

Encontre todos os seus aplicativos e programas


Procurando por aquele aplicativo ou programa perdido? Não
se preocupe! O menu Iniciar é o lugar certo para encontrar uma
lista completa de todos eles. Selecione o botão Iniciar ícone Ini-
ciar e, em seguida, selecione Todos os aplicativos no canto infe-
rior esquerdo.
Para manter a rolagem no mínimo, vá para uma parte especí-
fica da lista. Selecione um dos divisores de seção e escolha a letra
Se você quiser sair de perto do computador por um instante, com a qual o nome do aplicativo começa.
o botão de energia fica na parte inferior do menu Iniciar para
que você possa colocar o computador no modo de suspensão,
reiniciá-lo ou desligá-lo totalmente.
Se você quiser fazer outras alterações na aparência do menu
Iniciar, acesse Configurações. Selecione o botão Iniciar ícone Ini-
ciar e selecione Configurações > Personalização > Iniciar para
alterar quais aplicativos e pastas aparecem no menu Iniciar.

Informática 3
Apostila Digital Licenciada para Alice Caroline Guarino dos Santos - alice.guarino@hotmail.com (Proibida a Revenda)
APOSTILAS OPÇÃO
do, incluindo opções avançadas no Painel de Controle.
A maioria dos aplicativos tem suas próprias configurações -
procure por este ícone no aplicativo.

Personalize sua tela de bloqueio


Para ajustar sua tela de bloqueio de acordo com sua prefe-
rência, selecione o botão Iniciar ícone Iniciar e Configurações
> Personalização > Tela de bloqueio. Experimente mudar a tela
de fundo para uma foto favorita ou apresentação de slides, ou
escolha qualquer combinação de notificações de status detalha-
das e rápidas para mostrar a você eventos futuros do calendário,
atualizações de redes sociais e outras notificações de aplicativo
e do sistema.

E lembre-se, se você ainda não conseguir encontrar o que


está procurando, use a pesquisa! Use a caixa de pesquisa na bar-
ra de tarefas ou pressione a tecla do logotipo do Windows em
seu teclado e comece a digitar.

Reproduza músicas com o Groove


Curta todas as músicas que você adora em um aplicativo
Windows 10 simples, rápido e elegante.
Por si só, o Groove torna mais fácil reproduzir e gerenciar
suas músicas e listas de reprodução. Mas quando você adicionar
suas músicas ao OneDrive, o Groove permite que você repro-
duza-as — de graça — em todos os seus dispositivos favoritos:
computador, Xbox, Android, iPhone, telefone Windows, Sonos e
na Web.
No computador, basta adicionar seus MP3s, até mesmo mú- Aprenda a usar temas
sicas do iTunes, à pasta de música do OneDrive. Depois, você Selecione o botão Iniciar e, em seguida, selecione Configu-
terá todo esse conteúdo ao seu alcance no Groove em qualquer rações > Personalização > Temas > Configurações de tema. Em
um de seus dispositivos. seguida, escolha um tema padrão ou selecione Obter mais temas
Quer mais maneiras de ouvir música? Compre um Groove online para baixar temas novos que apresentam criaturas boni-
Music Pass e tenha acesso a rádios baseadas em artistas, playlists tas, recordações de férias e outras opções alegres.
selecionadas e um dos maiores catálogos de música do planeta.
O Groove Music Pass permite transmitir e baixar sucessos novos
e favoritos conhecidos de uma seleção de mais de 40 milhões de
músicas. Ouça — sem anúncios — em qualquer lugar.

Aplicativos do Office no Windows 10


Dependendo de suas necessidades e do dispositivo usado,
você poderá se perguntar que versão do Office poderia ser usa-
da no Windows 10.

Aplicativos de área de trabalho do Office


Esses são os programas tradicionais e incluem recursos mais
avançados que as versões móveis. Se você estiver trabalhando
em projetos mais complexos, para o trabalho (faturamento de
uma planilha), para estudos (monografia) ou diversão (folhetos
de um bazar na garagem), o Office 2016 pode ser a melhor opção Mude as cores e a tela de fundo da área de trabalho
para você. Selecione o botão Iniciar e, em seguida, selecione Configu-
rações > Personalização para escolher uma imagem digna de
Uma nova aparência para as configurações enfeitar a tela de fundo da sua área de trabalho e para alterar a
As Configurações sofreram uma transformação — e tiraram cor de destaque de Iniciar, da barra de tarefas e de outros itens.
o “PC” do nome. A janela de visualização oferece uma prévia das suas mudanças
conforme elas acontecem.
Em Tela de fundo, selecione uma imagem, uma cor sólida ou
crie uma apresentação de slides de imagens.

Acesse Configurações selecionando o botão Iniciar e depois


selecionando Configurações. A partir daí, navegue pelas catego-
rias ou use a pesquisa para encontrar o que você está procuran-

Informática 4
Apostila Digital Licenciada para Alice Caroline Guarino dos Santos - alice.guarino@hotmail.com (Proibida a Revenda)
APOSTILAS OPÇÃO
arquivos que deseja compartilhar, acesse a guia Compartilhar,
selecione o botão Compartilhar e, em seguida, escolha um apli-
cativo. Para saber mais sobre as opções de compartilhamento,
confira Compartilhar arquivos no Explorador de Arquivos.
- Se você está migrando do Windows 7, veja algumas dife-
renças mais:
- Meu computador agora é chamado This PC e ele não apa-
recerá na área de trabalho por padrão. Para descobrir como adi-
cionar This PC à sua área de trabalho ou ao menu Iniciar, confira
Meu Computador agora é This PC.
- Da mesma forma, bibliotecas não aparecerão no Explora-
dor de Arquivos, a menos que você quiser. Para adicioná-las ao
painel esquerdo, selecione a guia Exibição > Painel de navegação
> Mostrar bibliotecas.

Torne seu computador mais fácil de usar


Configurar o computador para torná-lo mais acessível re-
quer apenas algumas etapas. A maioria das opções de Facilidade
de Acesso estão disponíveis em Configurações. Selecione o bo-
tão Iniciar ícone Iniciar e, em seguida, selecione Configurações
Em Cores, deixe o Windows puxar uma cor de destaque da > Facilidade de Acesso.
sua tela de fundo, ou aventure-se nas cores por conta própria. As opções comumente usadas também estão disponíveis na
tela de credenciais. Selecione o botão Facilidade de Acesso no
canto inferior direito para vê-las.

O que mudou no Explorador de Arquivos


Como muitas das coisas mais refinadas da vida, o Explorador
de Arquivos está ficando melhor com idade. Para conferir seus
novos benefícios, abra-o a partir da barra de tarefas ou do menu
Iniciar, ou pressionando a tecla do logotipo do Windows + E no
seu teclado.

Veja algumas mudanças importantes:


- O OneDrive agora faz parte do Explorador de Arquivos.
Para ver uma rápida cartilha sobre como ele funciona no Windo-
ws 10, confira OneDrive em seu computador.
- Quando o Explorador de Arquivos for aberto, você entrará
no Acesso rápido. As pastas usadas com frequência e os arquivos
usados recentemente ficam listados ali, assim você não precisa As opções da Facilidade de Acesso podem tornar mais fácil:
procurar por eles uma série de pastas para encontrá-los. Você - Usar o computador sem um vídeo. O Narrador permite que
também pode fixar suas pastas favoritas ao Acesso rápido para você ouça as descrições de áudio de elementos na tela, como tex-
mantê-las à mão. Para saber mais, confira Fixar, remover e per- to e botões.
sonalizar no Acesso rápido. - Ver o que está na tela. Aumente o conteúdo na tela usando a
Lupa ou use um modo de exibição com alto contraste.
- Usar o teclado. Ative teclas de aderência, teclas de alternân-
cia ou o teclado virtual.
- Usar o mouse. Mude o tamanho do ponteiro ou ative as te-
clas para mouse e use o teclado para mover o mouse.

Agora, você pode usar aplicativos para compartilhar arqui-


vos e fotos diretamente de Explorador de Arquivos. Selecione os

Informática 5
Apostila Digital Licenciada para Alice Caroline Guarino dos Santos - alice.guarino@hotmail.com (Proibida a Revenda)
APOSTILAS OPÇÃO

Questões

01. (Prefeitura de Goiânia - GO - Assistente Administra-


tivo Educacional - CS-UFG/2016). Que forma de exibição de
ícones NÃO está presente no explorador de arquivos do Sistema - Abrir: Abre o ícone imediatamente.
Operacional Windows 10? - Compartilhar com: Compartilhar o ícone na Rede. *com
(A) Lista. exceção de ícones do tipo atalho.
(B) Personalizados. - Enviar para: Envia o arquivo diretamente para o destino
(C) Detalhes. escolhido. Ex: Pendrive.
(D) Blocos. - Recortar: Outra maneira de mover um ícone. Usado junto
com a função “Colar”.
02. (Sercomtel S.A Telecomunicações - Agente - - Copiar: Copia um ícone. Usado junto com a função “Colar”.
CONCESP/2015). Qual é o nome do sistema operacional da - Criar Atalho: Cria um caminho para o ícone, o seu tamanho
empresa Microsoft que foi lançado oficialmente em 29 de julho é padrão de 1kb.
de 2015? - Excluir
(A) Windows 9 - Renomear
(B) Windows 10 - Propriedades: Acessa informações como, tamanho do
(C) Windows 8.1 arquivo, formato, data de modificação, data de criação e etc.
(D) Windows 9.1 Estas opções são padrão para qualquer ícone. As outras
(E) Windows Ultimate opções que aparecem na figura acima são especificas de um
determinado arquivo, no exemplo foi utilizado um documento
Respostas do Word.
01. B\02. B
Janelas
Gerênciamento de arquivos e pastas2 O sistema operacional Windows ganhou este nome por
utilizar o conceito de “janelas”. Uma janela acessa as informações
Ícones do arquivo, pasta ou programa e as exibe em um modelo padrão.
Os ícones são representações de arquivos, atalhos ou pastas. As opções de cada janela mudam de acordo com o programa
Podem ser visualizados na Área de Trabalho ou dentro de pastas. que a utiliza.

As opções gerais, que estão presentes em todas as janelas


são:
Menu de opções de um ícone: 1. Opções de Janela:

2 Fonte: http://www.inf.pucpcaldas.br/extensao/cereadd/
apostilas/windows7v1.pdf

Informática 6
Apostila Digital Licenciada para Alice Caroline Guarino dos Santos - alice.guarino@hotmail.com (Proibida a Revenda)
APOSTILAS OPÇÃO
Minimizar : Envia a janela para a barra de tarefas.

Maximizar : Aumenta o tamanho da janela para preencher


a tela inteira. Se já estiver maximizada, o botão mudará para
restaurar o tamanho antigo.

Para saber o destino do atalho, acesse suas propriedades no


menu.

Arquivos e Pastas
Fechar : Finaliza a janela. Disco Rígido
O disco rígido (também conhecido como HD) é a parte do
2. Barra de Rolagem: Quando o conteúdo não cabe totalmente computador que guarda todos os arquivos do sistema. Isso inclui
dentro da janela, a barra de rolagem é acionada e é possível arquivos gerais do Windows (e outros programas) e arquivos
“rolar” ela para baixo ou para cima para visualizar o conteúdo. pessoais, como fotos, filmes, documentos e etc.
Mover a janela: Quando não esta maximizada, é possível O HD é um recipiente. Ele guarda coisas. E tudo que serve para
move-la para qualquer lugar dentro da área de trabalho. Para guardar algo tem um tamanho limite. A informação principal de
fazer isso posicione o mouse sobre a barra azul na parte superior um disco rígido é a sua capacidade de armazenamento, ou seja,
marcada (qualquer lugar que não tenha um botão), clique e o quanto de coisas ele pode guardar.
arraste. Exemplo: O HD do meu computador é de 300gb. (leia-se 300
gigabytes)

Redimensionamento: Posicionando o mouse sobre as bordas


da janela, o ponteiro do mouse muda de desenho, quando isso
acontecer significa que você poderá mudar o formato da janela.

Unidades de Medida:
Byte = o tamanho de uma letra.
Kilobyte ou Kb = 1.000 bytes.
Megabyte ou Mb = 1.000.000 bytes.
Gygabyte ou Gb = 1.000.000.000 bytes.
Quando o disco rígido esta muito cheio, recomenda-se
esvaziá-lo para evitar lentidão.

Unidades do Disco Rígido:


Atalho Para conhecer as unidades de disco rígido do seu computador,
Atalhos são muito utilizados no Windows 7. Eles fazem a acesse o Menu Iniciar -> Computador.
nossa interação com a interface algo mais pratico e rápido. Nós
podemos criar nossos próprios atalhos para suprirem nossas
necessidades.

Atenção: Todo dispositivo de armazenamento recebe uma


letra que o identifica. Esta letra representa a raiz daquele volume
em especifico. Ela representa um endereço.
Para identificar a raiz, basta procurar uma letra seguida de
dois pontos ( : ). O endereço é digitado na “barra de endereços”
da janela. Por exemplo: C:\ Fotos\Vizinha, isso significa: Dentro
do meu disco rígido, o primeiro local a existir é a raiz, portanto,
TUDO ESTA DENTRO DE C:
Atalhos tem um tamanho muito pequeno, pois carregam Na raiz da unidade existe uma pasta chamada Fotos. E dentro
apenas a informação do seu destino. Excluir atalhos não da pasta fotos, existe uma chamada Vizinha. A barra serve para
influencia em nada o funcionamento do arquivo (ícone) a qual delimitar espaços:
ele esta vinculado.
Atalhos são diferenciados através de uma seta presente Quando algo esta entre barras: \Fotos\ . Representa uma
próxima ao desenho do ícone: pasta.
Quando não existem barras antes: C:\. Representa uma

Informática 7
Apostila Digital Licenciada para Alice Caroline Guarino dos Santos - alice.guarino@hotmail.com (Proibida a Revenda)
APOSTILAS OPÇÃO
unidade de disco rígido. será perdido ao fechar. Da mesma forma, se você modificar
Quando não existem barras depois: \Curriculum.txt o conteúdo, deverá salvar, pois caso contrario, ao fechar o
Representa um arquivo. programa, sua modificação não terá efeito.
Exemplo: Para localizar uma pessoa no nosso mundo, como Exemplo: Abrir o programa Bloco de Notas, ir no menu
o Windows faz, seria assim: Arquivo->Salvar. O programa Bloco de Notas, vai gerar um
PlanetaTerra:\Brasil\Minas Gerais\Poços de Caldas\Meu arquivo de extensão .txt contendo como conteúdo, o texto que
Bairro\Minha Rua\João da Silva.humano foi digitado no programa. Extensões de Arquivos = Programa
Tudo que nós conhecemos esta no planeta terra. Ou seja, se Associado:
eu sou um arquivo em uma unidade de disco, tudo ao meu redor, .doc = Microsoft Word
inclusive eu, esta inserido nesta unidade. .exe = Executavel
\Brasil\...\MinhaRua\ Diz respeito as direções que devo .rar = WinRAR
seguir. As pastas são como as divisas entre estados, países, .txt = Bloco de Notas
cidades.
João da Silva.humano , representa o arquivo, no caso, a
pessoa. Primeiro, o nome que o identifica, depois, seguido de
ponto, o tipo dele. No caso, João é do tipo Humano.
Outro exemplo é o endereço da área de trabalho:

A extensão dos arquivos é oculta, mas é possível visualizá-las


alterando uma determinada opção ou abrindo as propriedades
do arquivo.
Quando alteramos a extensão do arquivo, estamos alterando
o programa com o qual ele vai ser associado, mas não o seu
Onde: conteúdo! Porem, se após a modificação, tentarmos abrir o
C:\ = Raiz da unidade. arquivo com o programa errado, então o programa poderá
\Users\ = Pasta que contem todos os usuários do Windows. modificar os dados do
\Meu Nome\ = Pasta que identifica o usuário. arquivo, tornando-o inutilizável.
\Desktop\ = Pasta que identifica a área de trabalho. É possível existir 2 arquivos de tipos diferentes com o mesmo
Portanto, quando você envia um ícone para a Área de nome. Mas não é possível existir 2 arquivos com o mesmo tipo
Trabalho, na verdade, o computador esta mudando o endereço e nome.
do ícone.
Excluindo um Arquivo
Partição Como visto anteriormente, arquivos ocupam espaço do disco
Uma partição é uma divisão do disco rígido. rígido. Quando não são mais utilizados, podem ser excluídos.
Para que um arquivo possa ser excluído, ele precisa:
- Não estar sendo usado (aberto) pelo usuário no momento
da exclusão.
- Não estar sendo usado por um programa no momento da
exclusão.
- Não estar protegido pelo sistema.
Um arquivo pode ser excluído, ou deletado utilizando o menu
do arquivo ou selecionando o ícone e pressionando a tecla Del.

Pastas
Pastas são delimitações do espaço virtual para organizar
arquivos e por isso, uma pasta vazia não tem tamanho, ou seja,
não ocupa espaço.

Criando uma Pasta


Uma pasta só pode ser criada na raiz da unidade de disco
ou dentro de outra pasta (neste caso a área de trabalho é
Na figura, tem-se um disco rígido particionado em 3. Cada considerada uma pasta) através do menu:
partição é uma unidade de disco e por isso recebe uma letra. Novo->Pasta.
P:Qual o beneficio de usar uma partição? Crie uma pasta com o seu nome.
R:Geralmente uma utilidade é fazer Backup, ou seja, usar
uma partição vazia para colocar dados que sejam importantes, Compartilhando uma Pasta
caso aconteça algum dano com a partição principal. Outra, é Quando o nosso computador esta em rede, pode ser
instalar dois sistemas operacionais. Por exemplo: Uma maquina interessante compartilhar uma determinada pasta, desta forma,
que tenha todos os computadores da rede podem acessá-la.
Windows e Linux. Para compartilhar basta usar o menu->Compartilhar e
escolher uma das opções:
Arquivo Compartilhamento para Leitura: O conteúdo não pode ser
Um arquivo é o resultado de um programa, contem em seu modificado por usuários da rede.
interior dados armazenados, que são tratados pelo programa Compartilhamento para Leitura/Gravação: O conteúdo pode
que o criou/utiliza. ser modificado por usuários da rede.
Um arquivo tem: Nome, Extensão (tipo), Tamanho, Ícone, Para abrir uma pasta compartilhada em rede:
Endereço. Gerando e Salvando um Arquivo Painel de Controle-> Redes e Internet->Exibir Computadores
Para criar um arquivo devemos fazer uso de um determinado e Dispositivos de Rede
programa. Será apresentado uma lista de computadores conectados,
Quando você inicia as atividades em um programa, você escolha o computador que esta compartilhando o conteúdo
precisa salvar o seu progresso, pois caso contrario, tudo desejado.

Informática 8
Apostila Digital Licenciada para Alice Caroline Guarino dos Santos - alice.guarino@hotmail.com (Proibida a Revenda)
APOSTILAS OPÇÃO
Excluindo uma Pasta arquivos. Assinale a alternativa que apresenta um nome válido
Mesmo processo para excluir um arquivo. Lembrando que, para arquivos e pastas no Windows:
deletar uma pasta, ira deletar todo o seu conteúdo. (A) Arquivos+selecionados(razão?).
Pastas não podem ser excluídas se: (B) Registro_de_Taregas_>mêsAtual.
- Estiver aberta. (C) Endereços/Emails/Telefones importantes.
- Estiver sendo usada por um programa. (D) Despesas_de_*2015*.
- Estiver sendo usada pelo sistema. (E) ControleMensagens(+)Recebidos&Mensagens(-)
Enviados.
Windows Explorer
O Windows Explorer é um gerenciador de pastas e arquivos 02. (Prefeitura de Cuiabá – MT - Técnico em
que os organiza e reúne em um só lugar. No Windows 7 ele esta Administração Escolar – FGV/2015). Analise a tela produzida
presente na barra de ferramentas, e tem a função de organizar no Windows 7 mostrada a seguir.
as pastas. Quando aberto, possui muitas utilidades, como por
exemplo:

Está correto concluir que essa tela exibe


(A) o conteúdo dos programas excluídos por meio do Painel
de Controle.
(B) o resultado de uma busca por arquivos a partir do menu
Iniciar.
(C) a lista de arquivos recentemente recuperados de
desligamentos forçados.
(D) o conteúdo da lixeira do Windows.
(E) o conteúdo do desktop do usuário.

03. (Prefeitura de Cuiabá – MT - Técnico em


Administração Escolar – FGV/2015). Analise, na imagem
a seguir, o menu apresentado no Windows 7 após o clique do
botão direito do mouse sobre o nome de um arquivo

Favoritos: Local para inserir atalhos de arquivos e pastas


para acesso rápido.
- Bibliotecas: Pastas separadas por categoria.
- Grupo Doméstico: Grupo de rede do qual este computador
faz parte.
- Computador: Acesso imediato a raiz das unidades de disco.
- Rede: Acesso rápido aos computadores da rede.
Pesquisar, exibição e organização de itens:

Menu com algumas opções de um ícone selecionado:

A opção “Abrir com”, nesse contexto, permite


(A) usar o comando como um atalho para a compactação de
Questões um arquivo.
(B) abrir o arquivo com um aplicativo diferente daquele que
01. (ISSBLU – SC - Agente Administrativo – FURB/2015). é o padrão no Windows.
Atribuir nomes significativos a pastas e arquivos, isto é, que (C) enviar um arquivo para o pen-drive, criando ali uma nova
indiquem a natureza do seu conteúdo, é um importante passo cópia.
para a gestão e localização dessas pastas e arquivos. Em se (D) utilizar uma versão mais antiga do Windows para abrir
tratando de um sistema operacional Windows, os nomes dos o arquivo.
arquivos e pastas podem conter até 256 caracteres entre letras, (E) especificar modos de segurança especiais na manipulação
números e caracteres especiais, à exceção daqueles chamados do arquivo depois de aberto.
reservados, que não podem ser usados em nomes de pastas e

Informática 9
Apostila Digital Licenciada para Alice Caroline Guarino dos Santos - alice.guarino@hotmail.com (Proibida a Revenda)
APOSTILAS OPÇÃO
04. (TRE-PB - Técnico Judiciário - Área Administrativa –
FCC/2015). Em um computador com o Windows 7 Professional,
em português, um técnico clicou no botão Iniciar e na opção
Computador para visualizar as unidades de disco disponíveis.
Após conectar um pen drive em uma das portas USB, percebeu
que a unidade deste dispositivo foi identificada pela letra E.
Ao clicar sobre esta unidade, foram exibidos arquivos na raiz
e pastas contidas neste pen drive. Ao arrastar um arquivo,
utilizando o mouse, da raiz do pen drive para uma das pastas, o
técnico percebeu que:
(A) o arquivo foi copiado para a pasta.
(B) ocorreu um erro, pois este procedimento não é permitido.
(C) o arquivo foi movido para a pasta.
(D) o arquivo foi aberto pelo software no qual foi criado.
(E) o arquivo foi apagado.

05. (CRM-SC - Assistente Administrativo – IASES/2015).


Considera as seguintes afirmativas sobre os possíveis motivos
de não se poder apagar um arquivo ou pasta:
I. O arquivo ou pasta está sendo utilizado.
II. Você não possui direitos para apagar este arquivo ou
pasta.
III. O arquivo está na sua área de trabalho.
Analise as afirmativas e assinale a alternativa correta:
(A) As afirmativas I, II e III estão corretas.
(B) Somente as afirmativas II e III estão corretas.
(C) Somente as afirmativas I e III estão corretas.
(D) Somente as afirmativas I e II estão corretas.

Respostas
01. E\02. D\03. B\04. C\05. D

MS-Word 2016: estrutura básica


dos documentos, edição
e formatação de textos,
cabeçalhos, parágrafos,
fontes, colunas, marcadores
simbólicos e numéricos, tabelas,
impressão, controle de quebras
e numeração de páginas,
legendas, índices, inserção de
objetos, campos predefinidos,
caixas de texto.

Word 20163

3 Fonte: https://support.office.com/pt-BR/article/Guias-de-
In%C3%ADcio-R%C3%A1pido-do-Office-2016-25f909da-3e76-443d-
94f4-6cdf7dedc51e

Informática 10
Apostila Digital Licenciada para Alice Caroline Guarino dos Santos - alice.guarino@hotmail.com (Proibida a Revenda)
APOSTILAS OPÇÃO

Mantenha-se conectado
Você precisa trabalhar fora do escritório e em dispositivos
diferentes? Clique em Arquivo > Conta para entrar e acessar os
arquivos usados recentemente, em praticamente qualquer lugar
e em qualquer dispositivo, por meio da integração perfeita entre
o Office, o OneDrive, o OneDrive for Business e o SharePoint.

Localizar arquivos recentes


Se você trabalha apenas com arquivos armazenados no disco
rígido local do seu computador ou usa vários serviços de nuvem,
clique em Arquivo > Abrir para acessar os documentos usados
recentemente e os arquivos que fixou à sua lista.

Descubra as ferramentas contextuais


Para disponibilizar comandos contextuais da faixa de opções,
Criar alguma coisa escolha objetos relevantes em seu documento. Por exemplo,
Inicie com um Documento em branco para começar a clicar dentro de uma tabela exibe a guia Ferramentas de tabela,
trabalhar. Se preferir, para economizar bastante tempo, selecione que oferece opções adicionais para o Design e o Layout das suas
e personalize um modelo que atenda à sua necessidade. Clique tabelas.
em Arquivo > Novo e, em seguida, escolha ou pesquise o modelo
desejado.

Compartilhe seu trabalho com outras pessoas


Para convidar outras pessoas para exibir ou editar seus

Informática 11
Apostila Digital Licenciada para Alice Caroline Guarino dos Santos - alice.guarino@hotmail.com (Proibida a Revenda)
APOSTILAS OPÇÃO
documentos na nuvem, clique no botão Compartilhar, no canto
superior direito da janela do aplicativo. No painel Compartilhar
exibido, você pode obter um link de compartilhamento ou enviar
convites para as pessoas escolhidas.

Encontre tudo o que precisar


Digite uma palavra-chave ou frase na caixa de pesquisa Diga-
me o que você deseja fazer, na faixa de opções, para localizar
rapidamente os comandos e recursos do Word que você está
procurando, para saber mais sobre o conteúdo de Ajuda online
ou obtenha mais informações online.
Revisar e controlar alterações
Caso pretenda apenas verificar a ortografia, manter o
controle da contagem de palavras ou colaborar com outras
pessoas, a guia Revisão revela comandos essenciais para
controle, discussão e gerenciamento de todas as alterações
feitas nos documentos.

Confira quem mais está digitando


A coautoria de documentos do Word que são compartilhados
no OneDrive ou no site do SharePoint acontecem em tempo real,
o que significa que você pode ver facilmente onde os outros
autores estão fazendo modificações no mesmo documento em
que você está trabalhando atualmente.
Pesquisar informações relevantes
Com a Pesquisa Inteligente, o Word faz uma busca na internet
para obter informações relevantes para definir palavras, frases e
conceitos. Pesquisar os resultados exibidos no painel de tarefas
pode fornecer conteúdo útil para as ideias que você estruturou
nos seus documentos.

Formate documentos com estilo


O painel Estilos permite que você reveja, aplique e crie
visualmente os estilos de formatação no seu documento atual.
Para abri-lo, clique na guia Página Inicial e, em seguida, clique
na seta pequena seta no canto superior direito da galeria Estilos.

Abrir um documento
Sempre que iniciar o Word, verá uma lista dos documentos
utilizados mais recentemente na coluna da esquerda. Se o
documento de que está à procura não aparecer na lista, clique
em Abrir Outros Documentos.

Informática 12
Apostila Digital Licenciada para Alice Caroline Guarino dos Santos - alice.guarino@hotmail.com (Proibida a Revenda)
APOSTILAS OPÇÃO
- Clique em Revisão e, em seguida, no botão Controlar
Alterações, selecione a opção Controlar Alterações.

Abrir outros documentos

Se já estiver no Word, clique em Arquivo > Abrir e a seguir


procure a localização do arquivo.
Quando abrir um documento criado em versões anteriores Registar Alterações
do Word, é apresentada a indicação Modo de Compatibilidade
na barra de título da janela do documento. Você pode trabalhar Imprimir o documento
no modo de compatibilidade ou pode atualizar o documento Para imprimir um documento de texto defina as opções de
para utilizar as funcionalidades novas ou melhoradas do Word impressão e imprima o arquivo.
2013. - Clique na guia Arquivo e em seguida clique em Imprimir.
- Faça o seguinte:
Salvar um documento - Em Imprimir, na caixa Cópias escreva o número de cópias
Para salvar um documento pela primeira vez, faça o seguinte: que deseja.
- Clique na guia Arquivo. - Em Impressora, certifique-se de que a impressora utilizada
- Clique em Salvar Como. está selecionada.
- Procure a localização em que o documento será salvo. - Em Configurações, as predefinições de impressão para a
- Clique em Salvar. sua impressora são selecionadas automaticamente. Se quiser
alterar uma configuração, faça as alterações necessárias.
Obs: Para salvar o documento no seu computador, escolha - Quando estiver satisfeito com as configurações da página,
uma pasta do Computador ou clique em Procurar. Para salvar o clique em Imprimir.
documento online, escolha a localização em Locais ou Adicionar
uma Localização. Quando os arquivos estiverem online, poderá PROTEGER COM SENHA
compartilhar, comentar e colaborar através dos arquivos em Ao selecionar a opção informações no menu “Arquivo” você
tempo real. pode proteger o documento. As opções de proteção são:
O Word salva os documentos automaticamente no formato
.docx. Para salvar o documento em um formato diferente, clique
na lista tipo e selecione o formato de arquivo que deseja.
Para salvar o documento à medida em trabalha, clique em
Salvar na Barra de Ferramentas de Acesso Rápido.

Salvar na barra de ferramenteas de acesso rápido

Ler documentos
Abra o documento em Modo de Leitura para ocultar a
maioria dos botões e ferramentas e assim concentrar-se na sua
Opções de proteção de um documento
leitura sem distrações.
Obs: Alguns arquivos de texto, tais como documentos ou
- Marcar como final: Torna o documento somente leitura.
anexos protegidos, são automaticamente abertos em Modo de
Quando um documento é marcado como final, a digitação,
Leitura.
a edição de comandos e as marcas de revisão de texto são
- Clique em Exibiçãor > Modo de Leitura.
desabilitadas ou desativadas, e o documento se torna somente
- Para se mover entre páginas num documento, siga um dos
leitura. O comando Marcar como Final o ajuda a comunicar
seguintes passos:
que você está compartilhando uma versão concluída de um
- Clique nas setas dos lados esquerdo e direito das páginas.
documento. Ele também ajuda a impedir que revisores ou
- Pressione as teclas página abaixo, página acima ou a barra
leitores façam alterações inadvertidas no documento.
da espaços e retrocesso do teclado. Também pode utilizar as
- Criptografar com senha: Define uma senha para o
teclas de setas ou a rolagem do mouse.
documento. Quando você seleciona Criptografar com Senha,
- Se estiver a utilizando um dispositivo de toque, percorra
a caixa de diálogo Criptografar Documento é exibida. Na caixa
com o dedo para a esquerda ou direita.
Senha, digite uma senha.
Obs: Clique em Modo de Exibição > Editar Documento para
- Restringir edição: Controla os tipos de alterações que
voltar a editar o documento.
podem ser feitas no documento.
- Restringir acesso: Você pode aplicar permissões ou
Registar Alterações
restrições para um grupo de pessoas. Quando você seleciona
Quando estiver a trabalhando e um documento com outras
Restringir Edição, três opções são exibidas: Restrições de
pessoas ou editá-lo sozinho, ative a opção Controlar Alterações
Formatação: reduz as opções de formatação, preservando
para ver todas as alterações. O Word assinala todas as inserções,
a aparência. Clique em Configurações para selecionar quais
remoções, mudanças e alterações de formatação realizadas no
estilos são permitidos. Restrições de edição: você controla
documento.
como o arquivo pode ser editado ou pode desativar a edição.
- Abra o documento a ser revisto.

Informática 13
Apostila Digital Licenciada para Alice Caroline Guarino dos Santos - alice.guarino@hotmail.com (Proibida a Revenda)
APOSTILAS OPÇÃO
Clique em Exceções ou Mais usuários para controlar quem pode a fonte e o parágrafo, configure estilos de formatação e permite
editar. Aplicar proteção. Clique em Sim, Aplicar Proteção para localizar substituir ou selecionar determinadas partes do texto.
selecionar a proteção de senha ou a autenticação do usuário.
- Adicionar uma assinatura digital: As assinaturas digitais ÁREA DE TRANSFERÊNCIA
autenticam informações digitais, como documentos, mensagens Auxilia nos procedimentos de Copiar, Recortar, Colar e na
de e-mail e macros, usando a criptografia do computador. utilização do pincel de formatação.

PROPRIEDADES
Ainda na opção informações é possível visualizar as
propriedades do documento.
As propriedades de um documento são detalhes de um
arquivo que o descrevem ou identificam. As propriedades
incluem detalhes como título, nome do autor, assunto e palavras- Opções da Área de Transferência
chave que identificam o tópico ou o conteúdo do documento.
Colar: Permite adicionar ao documento uma imagem ou texto
Estrutura básica dos documentos copiado do navegador de internet, de uma planilha do Excel, de
Os Editores de texto, assim como é o Microsoft Word 2013, uma apresentação do Power Point ou mesmo do próprio Word.
são programas de computadores elaborados para edição e A tecla de atalho utilizada é a combinação (CTRL + V)
formatação de textos, essas formatações podem ser aplicadas Recortar: Remove a seleção, adicionando-a na área de
em cabeçalhos e rodapés, fontes, parágrafos, tabelas, trabalhos transferência, para que o conteúdo seja colado em outro local,
com textos em colunas, numerações de páginas, referências seja ele no mesmo documento ou em outro. A tecla de atalho
como índices, notas de rodapé e inserção de objetos. utilizada é a combinação (CTRL + X)
Seu formato de gravação é DOCX e os documentos além Copiar: Copia a seleção, adicionando-a na área de
das características básicas citadas acima possuem a seguinte transferência, para que o conteúdo seja colado em outro local,
estrutura: seja ele no mesmo documento ou em outro. A tecla de atalho
• Cabeçalho; utilizada é a combinação (CTRL + C)
• Rodapé; Pincel de Formatação: Permite que a formatação de um texto
• Seção; por exemplo, seja copiada, ao visualizar determinada formatação
• Parágrafos; você pode selecioná-la, clicar no pincel de formatação, neste
• Linhas; momento o cursor do mouse vai ficar no formato de um pincel,
• Paginas; agora todo o texto que você selecionar receberá a mesma
• Números de Páginas; formatação da seleção que foi feita anteriormente. A tecla de
• Margens; atalho utilizada é a combinação (CTRL + Shift + C) para copiar e
(CTRL + Shift + V) para colar.
Abaixo, seguem alguns exemplos:
FONTE
As fontes são definidas a partir de seu estilo, tipo e tamanho,
o Word, trabalha com as chamadas fontes True Type gravadas
sob o formato .ttf, o local de armazenamento das fontes é no
diretório Fonts dentro da pasta Windows, essas não ficam só
disponíveis para o Word, mas sim para todos os programas do
computador.
Na barra de ferramentas padrão da aba página inicial do
Word, estão disponíveis em forma de ícones todas as opções
para formatações de texto, como veremos a seguir:
Nome da Fonte: Os nomes das fontes estão relacionados
diretamente com seus estilos, por padrão o Word 2013 o Word
sugere a utilização das fontes Calibri e Cambria, também existe
uma área onde ficam armazenas as fontes que foram usadas
recentemente.
Tamanho da Fonte: ao lado da caixa onde fica definido o
nome da fonte utilizada temos a caixa de seleção dos tamanhos
das fontes, exemplo: 8, 9, 10, 11 e assim por diante, se necessário,
o usuário também pode digitar um valor numérico nesta caixa e
pressionar a tecla Enter para fixar o tamanho desejado, ainda
podemos utilizar os ícones aumentar ou diminuir o tamanho do
texto. Há a possibilidade de utilizar também as teclas de atalho
(Ctrl + Shift + >) para aumentar o tamanho da fonte ou (Ctrl +
Shift + <) para diminuir o tamanho da fonte.

Atalhos para fonte

Maiúsculas e Minúsculas: Altera todo o texto selecionado de


acordo com as opções a seguir:

GUIA PÁGINA INICIAL


A aba página inicial permite que você adicione texto, formate

Informática 14
Apostila Digital Licenciada para Alice Caroline Guarino dos Santos - alice.guarino@hotmail.com (Proibida a Revenda)
APOSTILAS OPÇÃO
Lista de vários Níveis (são exibidos níveis para o marcador
exemplo, 1.1 ou 2.1.3)
Níveis de Recuo – São utilizados para aumentar ou diminuir
nível de recuo do parágrafo desejado.

Exemplo do nível de recuo para a primeira linha


Opções do menu Maiúsculas e Minúsculas

Limpar Formatação: Limpa toda a formatação do texto.


Deixando-o com a formatação do estilo Normal.
Negrito: Torna o traço da escrita mais grosso que o comum.
Pode ser aplicado ao selecionar um texto ou palavra e clicar no
atalho do grupo de opções fonte ou usando a combinação (Ctrl
+ N).
Itálico: Deixa a fonte levemente inclinada à direita. Pode ser
aplicado ao selecionar um texto ou palavra e clicar no atalho do Exemplo de uso da régua em um documento.
grupo de opções fonte ou usando a combinação (Ctrl + I). Ex:
Sublinhado: Sublinha o texto, frase ou palavra selecionada, 1- Margem esquerda: É o limite da impressão no papel do
inserindo uma linha abaixo da mesma. Pode ser aplicado ao lado esquerdo, tudo o que ficar além do limite não irá aparecer
selecionar um texto ou palavra e clicar no atalho do grupo de na impressão.
opções fonte ou usando a combinação (Ctrl + S). 2- Recuo deslocado: Ao clicar e arrastar usando este ícone, o
Tachado: Desenha uma linha no meio do texto selecionado. texto que faz parte da segunda linha em diante terá seus valores
Ex: de recuo alterados de acordo com o tanto que o objeto for
Exemplo de texto tachado. deslocado, para direita ou esquerda.
3- Recuo à esquerda: Ao clicar e arrastar usando este ícone,
o texto que faz parte da segunda linha em diante terá seus
valores de recuo alterados de acordo com o tanto que o objeto
for deslocado, para direita ou esquerda. Com uma diferença
da opção anterior, pois ao deslocarmos este objeto, o recuo da
primeira linha será deslocado junto, mantendo assim a mesma
proporção de distância entre o texto da primeira e das demais
linhas do parágrafo.
4- Recuo da primeira linha: Ao clicar e arrastar usando
Ao selecionar o texto do exemplo acima o atalho usado para este ícone, o texto da primeira linha terá seus valores de recuo
o comando tachado fica destacado. alterados de acordo com o tanto que o objeto for deslocado, para
direita ou esquerda.
Subscrito: Cria letras ou números pequenos abaixo do texto. 5- Recuo à direita: Define o lime para o texto à direita da
Tem como atalho a combinação de teclas (Ctrl + =). Ex: página.
H2O 6- Margem direita: Define o limite da área impressa do lado
Sobrescrito: Cria letras ou números pequenos acima do direito da página.
texto. Tem como atalho a combinação de teclas (Ctrl + Shift + +).
158 Espaçamento de Linhas e Parágrafos – Altera os espaços
Efeitos de texto: Permite adicionar efeitos ao texto como entre as linhas do texto, estes espaçamentos são definidos por
sombra, reflexo ou brilho. Ao clicar na seta ao lado do atalho de pontos, e podem ser atribuídos antes ou após os parágrafos.
efeitos temos algumas opções disponíveis para aplicar no texto Sombreamento nos parágrafos – Realça todo o parágrafo,
selecionado. diferenciando do item
Cor do Realce do texto: Faz com que o texto selecionado fique Sombreamento
como se tivesse sido selecionado por um marcador de texto. Diferente do realce que sombreia apenas o texto selecionado,
Cor da fonte: Muda a cor do texto selecionado. Pod­emos o sombreamento muda a cor da linha toda.
escolher uma cor sugerida ou clicar em mais cores para visualizar
mais opções de cores, ou ainda utilizar a opção gradiente que Bordas – as bordas inferiores são utilizadas para criar linhas
permite escolher uma combinação de cor para a fonte. em volta do texto selecionado, basta selecionar o texto desejado
Formatação de Parágrafos, são utilizadas para alinhar o e escolher as bordas desejadas.
texto, criar recuos e espaçamentos entre parágrafos, conforme Estilos
a necessidade do usuário, veja nos exemplos a seguir cada uma Estilos são formatações e alinhamentos pré-definidos que
dessas formatações: são utilizados para poupar tempo e proporcionar ao documento
1- Texto alinhado à Esquerda – Alinha todo o texto um bom aspecto, basta selecionar ou criar o seu próprio estilo e
selecionado a esquerda. aplicar ao texto selecionado.
2- Texto Centralizado – Centraliza o texto no meio da
página. GUIA INSERIR
3- Texto alinhado a Direita – Faz com que o texto As ferramentas dessa área são utilizadas para inserção de
selecionado fique alinhado a direita. objetos nas páginas do documentos, estas, são divididas pelas
4- Texto alinhado Justificado – Alinha todo o texto de seguintes categorias:
forma justificada, ou seja, o texto selecionado fica alinhado
perfeitamente tanto esquerda.
Marcadores e Numeração - é uma ferramenta fundamental
para elaboração de textos seja um texto profissional, doméstico
ou acadêmico. O Word disponibiliza três tipos de marcadores
que são:
Marcadores (são exibidos em forma de símbolos)
Numeração (são exibidos em forma de números e até
mesmo letas)

Informática 15
Apostila Digital Licenciada para Alice Caroline Guarino dos Santos - alice.guarino@hotmail.com (Proibida a Revenda)
APOSTILAS OPÇÃO
Página – Insere ao documento objetos como folha de rosto, assinatura digital, então poderá utilizá-la, o campo Data e Hora
página em branco ou quebra de página (envia o texto ou cursor insere em diversos formatos a data e/ou hora do computador.
para a próxima página). Símbolos – utilizado para inserção de fórmulas matemáticas
Tabelas – Cria no documento tabelas com o número de (já existentes no computador ou criadas pelo usuário) ou
colunas e linhas especificado pelo usuário, nesse MENU, também símbolos não disponíveis no teclado.
são disponibilizadas ferramentas como “desenhar tabela”
(permite que o usuário fique livre para desenhar sua tabela), GUIA DESIGN
“Planilha do Excel” (importa uma planilha do Excel para dentro Formatação do documento: Permite aplicar diferentes
do Documento do Word) e “Tabelas Rápidas” (Cria modelos de formatações para o texto, definir temas para o documento, e
tabelas pré-definidos como calendários, matrizes, etc.). selecionar cores pré-definidas ou personalizadas.
No Word 2013, sempre que inserimos algum objeto Plano de fundo da página: Permite adicionar um plano de
que possua configurações adicionais, ou seja que não estão fundo, bordas ou marca d`água em um documento.
disponíveis nos sete menus iniciais, submenus são adicionados Layout da Página
para auxiliar na formatação do objeto, quando inserimos uma Nessa área ficam dispostas as opções de formatações gerais
tabela por exemplo, as abas Design e Layout ficam disponíveis, de Layout da página ou do documento a ser trabalhado, como
pois são abas que só aparecem quando estamos formatando configurações de margens, orientações da página, colunas e
uma tabela. tamanhos:

Ferramentas de Tabela aba Design


Margens – permite que o usuário atribua configure as
1- Opção ferramentas de tabela, traz as abas Design e Layout margens superior, inferior, direita e esquerda da página, o
que são usadas para a formatação de tabelas. Word 2010 já traz em sua configuração padrão margens pré-
2- Aba Design: Permite configurar cores, estilos de borda e configuradas, porém, mas é possível incluir suas próprias
sombreamento de uma tabela. configurações, clicando em “Margens Personalizadas”.
3- Aba Layout: Permite configurar a disposição do texto ou
imagem dentro da tabela, configurar o tamanho das colunas e Orientação – Altera o layout da página para retrato ou
linhas e trabalhar com os dados da tabela. paisagem.

Obs: Quando estamos trabalhando com tabelas e desejamos


apagar os dados que estão dentro dela usamos a tecla “Delete”,
a tecla Backspace é usada quando desejamos excluir linhas,
colunas ou a tabela.
Ilustrações – Permite a inserção de Imagens (arquivos de
imagens do computador), ClipArt (arquivos de mídia, como
ilustrações, fotografias, sons, animações ou filmes, que são
fornecidos no Microsoft Office), Formas (Formas geométricas),
SmartArts (Diagramas), Gráficos(Importa do Excel gráficos para
ilustração de dados), Instantaneo(insere uma imagem de um
programa que esteja minimizado na barra de tarefas).
Ao inserir uma imagem temos acesso as opções de
formatação de imagem, que vem através de uma nova aba.
Através dela é possivel fazer ajustes na imagem, definir estilos,
organizar ela no texto e definir seu tamanho.
Tamanho – Permite que o usuário escolher um tamanho
Ao inserir formas também temos acesso a uma nova aba de papel para o documento, assim como em todas as outras
Formatar que faz parte da opção ferramentas de Desenho. Onde configurações existem tamanhos padrões, mas é possível
é possivel escolher outras formas, colorir, definir textos para as personaliza-los.
formas, organiza-la no documento e configurar seu tamanho. Colunas – divide o texto da página em uma ou mais colunas.
Link – Utilizado para criar ligações com alguma página WEB Essa opção é muito utilizada para diagramações de livros,
ou para ativar algum cliente de e-mail ativo no computador e apostilas, revistas, etc.
também criar referência cruzada, ou seja, referência algum item Quebra de Página – Adiciona Página, seção ou quebras de
do documento. coluna ao documento como mostra no exemplo a seguir:
Cabeçalho e Rodapé – Edita o cabeção e rodapé do Números de linha: Fazer referência a linhas específicas no
documento, aplicando sua configuração a todas as páginas. documento com rapidez e facilidade, usando números de linha
Sendo que o cabeçalho está localizado na parte de cima do na margem.
documento e o rodapé na parte de baixo, conforme demonstrado Hifenização: Quando uma palavra fica sem espaço, o Word
na imagem localizada no item estrutura básica dos documentos. normalmente a move para a próxima linha. Quando você ativa a
Número de Página – Insere uma sequência numérica às hifenização, ajuda a criar um espaço mais uniforme e economiza
páginas, sendo no cabeçalho ou no rodapé e na esquerda ou espaço no documento.
direita.
Textos – Caixa de Texto (insere uma caixa de texto pré- GUIA REFERÊNCIAS
formatada), Partes Rápidas (insere trechos de textos reutilizáveis A aba de Referencias possui um amplo conjunto de
configurados pelo usuário), WordArt (inclui um texto decorativo ferramentas a serem utilizadas no documento, como por
ao documento) e Letras Capitular (cria uma letra maiúscula exemplo, índices, notas de rodapé, legendas, etc.
grande no início do parágrafo). Sumário – Ferramenta para elaboração do Índice principal
Campos pré-definidos (Linha de Assinatura e Data e do documento, este pode ser criado a partir de Estilos pré-
Hora) – A Linha de Assinatura insere um campo automático estabelecidos ou por meio de inserção de itens manualmente.
que necessita de prévia configuração com a especificação para Nota de Rodapé – Utilizada para referenciar algo do texto
uma pessoa assinar o documento, caso o usuário possua uma no rodapé da página, essas são numeradas automaticamente.

Informática 16
Apostila Digital Licenciada para Alice Caroline Guarino dos Santos - alice.guarino@hotmail.com (Proibida a Revenda)
APOSTILAS OPÇÃO
Citação Bibliográfica – Permite que sejam inseridas GUIA EXIBIÇÃO
informações como autor, título, ano, cidade e editora na citação. Modos de exibição: Opções de configuração de exibição do
Legendas – Utilizada para criar legendas de tabelas e documento.
figuras, pode ser utilizado como índice de ilustrações e tabelas. Mostrar: Opções de exibição de ferramentas do Word.
Índice - É uma lista de palavras encontradas no documento, Zoom: Opções de zoom, para aumentar ou diminuir a tela.
juntamente com o número das página em que as palavras Esta opção não aumenta o tamanho da letra ou dos objetos que
aparecem. fazem parte do documento.
Índice de autoridades – Adiciona um índice de casos, Janela: Opção para organizar como se comportam as janelas,
estatutos e outras autoridades citadas no documento. quando mais de um documento sendo visualizado ao mesmo
tempo.
GUIA CORRESPONDÊNCIAS Macros: Exibe uma listagem de macros que podem ser
Criar: Permite criar um documento baseado em uma modelo utilizas no documento.
de etiqueta ou envelope.
Iniciar mala direta: Opção para criar mala direta ou Questões
selecionar destinatários que já existem em algum tipo de banco
de dados. 01. (COSEAC - Prefeitura de Niterói - RJ - Técnico em
Gravar e inserir campos: Opções para inserção dos Informática/2016). No processador de textos Word 2016, são
campos correspondentes aos destinatários. Através desta opção modos de exibição de documentos:
podemos adicionar um texto direcionado aos destinatários. (A) rascunho e estrutura de tópicos.
Visualizar resultados: Opção usada para verificar como (B) layout da web e revisão.
ficou o documento com a mala direta, podendo alternar entre a (C) retrato e paisagem.
visualização dos campos ou destinatários. (D) negrito e sublinhado.
Concluir: Permite imprimir o documento para todos os (E) justificar e centralizar.
destinatários.
02. (IADES - CRESS-MG - Auxiliar Administrativo/2016).
GUIA REVISÃO Considerando o Microsoft Word 2016, assinale a alternativa que
A guia revisão nos traz ferramentas de ortografia e indica o procedimento efetuado para alterar o espaçamento
gramática, Contador de palavras, Comentários e etc. Todas entre linhas de um documento inteiro, sem selecionar o texto
as funcionalidades desta guia servem para a realização uma do documento.
revisão geral no documento com a finalidade de realizar buscas (A) Clique em Design > Espaçamento entre Parágrafos, e
de erros no texto. escolha uma das opções de espaçamento interno.
A opção de Ortografia e gramatica serve para auxiliar a (B) Clique em Inserir > Espaçamento de Linha e Parágrafo, e
correção do documento, onde é possível corrigir palavras escolha uma das opções de espaçamento interno.
escritas de forma errada ou corrigir a forma como determinados (C) Selecione os parágrafos que você deseja alterar, clique
símbolos foram inseridos. em Página Inicial > Espaçamento de Linha e Parágrafo, e escolha
O Word identifica erros de ortografia e gramatica através uma das opções.
de sublinhados, o sublinhado vermelho abaixo de uma palavra (D) Clique em Página Inicial > Parágrafo > Marcadores ou
no Word indica possíveis erros de ortografia, é uma palavra Numeração, e escolha uma das opções.
não reconhecida, onde o usuário pode optar por corrigi-la ou (E) Não é possível alterar o espaçamento entre linhas de um
adicionar esta palavra ao dicionário. Basta clicar com o botão documento inteiro sem selecionar o respectivo texto.
direito do mouse sobre a palavra para ver as sugestões. Faz
parte das opções de ortografia e gramática a sugestão de escrita 03. No Microsoft Word, em sua configuração padrão, as
da pala, que na imagem abaixo sugere que a palavra seja escrita configurações de parágrafo e estilo são encontradas na guia:
com letra maiúscula, podemos ignora o aviso do Word, assim o
sublinhado desaparece desta palavra, podemos ignorar tudo, (A) Inserir.
para que não apareça o sublinhado todo o documento onde (B) Layout da Página.
a palavra está escrita ou adicionar ao dicionário para que a (C) Página Inicial.
palavra não seja reconhecida como errada novamente em (D) Exibição.
nenhum documento do Word escrito neste computador, porém (E) Revisão.
o usuário deve tomar cuidado pois ao adicionar uma palavra
escrita de forma errado no dicionário a correção ortográfica não 04. (ANAC - Técnico Administrativo - ESAF/2016). No MS
irá sugerir correção para a mesma em nenhum momento. Word ,
(A) é possível aplicar os recursos de formatação (aplicar
negrito, centralizar, etc.) em um símbolo, bastando aplicar o
comando antes de selecioná-lo como um caractere normal.
(B) é possível aplicar os recursos de formatação (aplicar
negrito, centralizar, etc.) em um símbolo, bastando selecioná-lo
como um caractere normal antes da aplicação do comando.
(C) é possível criar uma tecla de atalho para inserção de
símbolo, desde que seja aberta a caixa de diálogo no momento
de cada inserção
(D) é possível utilizar o Clip-art.com para inserir imagens,
sem haver conexão com a internet.
(E) não há possibilidade de ajustar uma tabela gerada por
auto-formatação.

05. Assinale a alternativa correta, sobre o documento a


Opções de correção ortográfica. seguir, criado no Microsoft Word, em sua configuração original,
com o cursor posicionado na segunda página.
Obs: Nenhum dos sublinhado que aparecer devido a um
erro ortográfico será impresso, essas marcas só são visíveis no
computador.
COMENTÁRIOS: Permite que um comentário seja adicionado
em uma seleção.

Informática 17
Apostila Digital Licenciada para Alice Caroline Guarino dos Santos - alice.guarino@hotmail.com (Proibida a Revenda)
APOSTILAS OPÇÃO

(A) O documento contém 1 página e está formatado com 2


colunas.
(B) A primeira página está sendo exibida em modo de
impressão e a segunda página, em modo de layout web.
(C) O documento contém 2 páginas, sendo a primeira em
orientação paisagem e a segunda, em orientação retrato.
(D) O documento contém 1 página, sendo que o primeiro
quadro é dedicado a anotações do autor do texto.
(E) O documento está 40% preenchido.

Respostas
01. A/02. A/03. C/04. B/05. C

MSExcel 2016: estrutura básica


das planilhas, conceitos de
células, linhas, colunas, pastas
e gráficos, elaboração de
tabelas e gráficos, uso de
fórmulas, funções e macros,
impressão, inserção de objetos,
campos predefinidos, controle
de quebras e numeração de
páginas, obtenção de dados
externos, classificação de dados.

Excel 2016

Informática 18
Apostila Digital Licenciada para Alice Caroline Guarino dos Santos - alice.guarino@hotmail.com (Proibida a Revenda)
APOSTILAS OPÇÃO
o Office, o OneDrive, o OneDrive for Business e o SharePoint.

Localizar arquivos recentes


Se você trabalha apenas com arquivos armazenados no
disco rígido local do seu computador ou se usa vários serviços
de nuvem, clique em Arquivo > Abrir para acessar as pasta de
trabalho usadas recentemente e os arquivos que fixou à sua lista.

Descubra as ferramentas contextuais


Escolha objetos relevantes na sua pasta de trabalho para
disponibilizar os comandos textuais. Por exemplo, clicar em um
elemento do gráfico exibe a guia Ferramentas de gráfico com as
opções Design e Formato de um gráfico selecionado.

Criar alguma coisa


Escolha uma Pasta de trabalho em branco para começar a
trabalhar. Se preferir, para economizar bastante tempo, selecione
e personalize um modelo que atenda à sua necessidade. Clique
em Arquivo > Novo e, em seguida, escolha ou pesquise o modelo Compartilhe seu trabalho com outras pessoas
desejado. Para convidar outras pessoas para exibir ou editar suas pastas
de trabalho na nuvem, clique no botão Compartilhar, no canto
superior direito da janela do aplicativo. No painel Compartilhar
exibido, você pode obter um link de compartilhamento ou enviar
convites para as pessoas escolhidas.

Mantenha-se conectado
Precisa trabalhar fora do escritório e em dispositivos
diferentes? Clique em Arquivo > Conta para entrar e acessar os
arquivos usados recentemente, em praticamente qualquer lugar
e em qualquer dispositivo, por meio da integração perfeita entre

Informática 19
Apostila Digital Licenciada para Alice Caroline Guarino dos Santos - alice.guarino@hotmail.com (Proibida a Revenda)
APOSTILAS OPÇÃO

Gerencie os dados com as tabelas do Excel Pesquisar informações relevantes


Você pode optar por formatar um intervalo de células na sua Com a Pesquisa Inteligente, o Excel pesquisa na Internet
pasta de trabalho atual como uma tabela do Excel. As tabelas para obter informações relevantes para definir palavras, frases e
do Excel permitem que você analise e gerencie facilmente um conceitos. Pesquisar os resultados exibidos no painel de tarefas
grupo de dados relacionados independentemente das linhas e pode fornecer conteúdo útil para os dados e as informações na
das colunas na sua pasta de trabalho. sua pasta de trabalho.

Insira funções e crie fórmulas


Na guia Fórmulas, clique em Inserir Função para exibir a
caixa de diálogo Inserir Função. Aqui, você pode pesquisar e
inserir funções, procurar a sintaxe correta e até mesmo obter
ajuda detalhada sobre as funções selecionadas.

Estrutura básica das pastas e planilhas


É a estrutura que compõe a tela do programa.
1) Barra de Ferramentas de Acesso Rápido:
São ícones de atalho configurados no canto superior
esquerdo da tela, para agilizar o acesso a comandos comumente
utilizados.
Os ícones padrão são:

A – Botão do Excel: possibilita a realização de funções com


a janela como Restaurar, Mover, alterar o Tamanho, Minimizar,
Maximizar e Fechar (Alt+F4).
B – Salvar: quando criamos um documento e desejamos gravá-
lo no computador ou em outro dispositivo de armazenamento.
C – Desfazer (Ctrl+Z): Desfaz as últimas ações realizadas.
D – Refazer (Ctrl+Y): Refaz a última ação.
E – Persolnalizar a Barra de Ferramentas de Acesso Rápido:
Com ela determinamos quais os ícones que farão parte desta
barra, habilitando ou desabilitando para mostrar ou ocultar,
respectivamente, determinado ícone.
2) Barra de Título: mostra o nome da pasta e o nome do
programa. O nome padrão dos arquivos no Excel é “Pasta”. Esse
nome pode ser alterado pelo usuário quando o arquivo é salvo.
3) Ajuda do Microsoft Excel (F1): abre o assistente de ajuda
Encontre tudo o que precisar do Excel, no modo offline (apenas com a ajuda instalada no
Digite uma palavra-chave ou frase na caixa de pesquisa Diga- computador) ou online (permitindo a pesquisa na Internet).
me o que você deseja fazer, na faixa de opções, para localizar 4) Opções de Exibição da Faixa de Opções: permite a
rapidamente os comandos e recursos do Excel que você está configuração de opções de mostrar ou ocultar na Faixa de
procurando, para saber mais sobre o conteúdo de Ajuda online Opções.
ou obtenha mais informações online. - Ocultar a Faixa de Opções Automaticamente: Oculta a Faixa
de Opções e com um clique na parte superior do aplicativo volta
a exibí-la.

Informática 20
Apostila Digital Licenciada para Alice Caroline Guarino dos Santos - alice.guarino@hotmail.com (Proibida a Revenda)
APOSTILAS OPÇÃO
- Mostrar Guias: Mostra somente as guias da Faixa de Opções.
Clicando em uma das guias, mostra os comandos.
- Mostrar Guias e Comandos: Mostra as guias e da Faixa de
Opções e comandos o tempo todo.
5) Minimizar: reduz a janela a um botão na barra de tarefas
do sistema operacional.
6) Maximizar ou Restaurar: sua função depende do estado
atual da janela. Se ela estiver maximizada, ou seja, ocupando
toda a área da tela do computador, este botão transforma-se
no Restaurar, que volta a janela ao tamanho anteriormente
determinado pelo usuário. Se ela estiver com tamanho alterado, Nome dado a um grupo de células
o botão transforma-se no Maximizar, que faz com que a janela
ocupe a maior área possível da tela. Clicando no drop down da caixa de nomes, serão listados
7) Fechar: finaliza o documento. todos os nomes atribuídos pelo uusário nas células da planilha,
8) Faixa de opções: é composta pela Barra de Ferramentas permitindo o deslocamento para elas apenas com um clique:
de Acesso Rápido, Barra de Título, Botões de Ajuda, Opções de
Exibição da Faixa de Opções, Minimizar, Restaurar/Maximizar e
Fechar, Guias e Comandos.

Nomes dados às células da planilha

B) Botões Cancelar,Inserir e Inserir função:


Grupos - Cancelar:
As guias são as “abas” que englobam grupos e comandos. Quando estamos com uma célula ativa e desejamos anular
Os comandos são os botões que realizam funções específicas o conteúdo digitado dentro dela, podemos usar o cancelar. O
presentes em cada grupo. que foi digitado será excluído e a célula voltará ao seu estado
Os grupos organizam comandos característicos. original.
Por exemplo, temos na Guia “Página Inicial” os grupos Área - Inserir:
de Transferência, Fonte, Alinhamento, Número, Estilo, Células e O botão inserir confirma a inclusão de um conteúdo em uma
Edição. célula. Após escrevermos um texto e clicarmos nele, o texto será
Cada guia possui vários grupos e vários botões de comandos. confimado dentro da célula.
No grupo Área de Transferência, temos os botões de Não é necessário o uso contínuo dos botões Cancelar e
comandos Colar, Recortar, Copiar, Pindel de Formatação e o Inserir, visto que uma vez que o conteúdo da célula não esteja
botão mostrar, que exibe todas as opções que compõem esse correto ou não seja desejado, pode ser excluído facilmente com
grupo. o auxílio das teclas “Del” ou “Backspace”. Quando terminamos
Essa termologia é importante para entendermos os de digitar um conteúdo em uma célula e pressionamos “ENTER”
enunciados das questões de concursos públicos e também nos ou clicamos em outra célula, este conteúdo já é confirmado na
localizarmos durante a prática do programa. célula.
9) Caixa de nomes, cancelar, inserir, inserir função, barra de - Inserir função:
fórmulas: Abre a janela “Inserir função” que atua como assistente na
Logo abaixo da Faixa de Comandos temos: inserção e uso de funções.

A) Caixa de nomes: nesta caixa fica a localização da célula


ativa, ou seja, aquela que está marcada ou em uso no momento.
Além de exibir o nome da célula, caso já saibamos para
que célula desejamos ir, basta digitar o nome na referida caixa
e pressionar a tecla ENTER. Com este procedimento, seremos
levados diretamente para a célula digitada.
Outra função da caixa de nomes é atribuir nomes diferentes
às células. Esse recurso é muito útil quando trabalhamos com
uma planilha grande, por exemplo. Para não precisar lembrar
qual a coluna e linha de uma célula para encontrá-la no meio
de tantas outras sem percorrer toda a planilha a sua procura,
podemos dar um nome específico a ela, como, por exemplo
“total”.
Para trocar o nome de uma célula, basta: Inserir função
- Com a célula selecionada vá até a caixa de nomes e digite
o nome desejando. Pressione ENTER. Pronto! A célula já estará A célula que estava ativa e a barra de fórmulas recebem
com seu nome alterado. O mesmo procedimento pode ser automaticamente o sinal fundamental e indispensável para que
realizado para atribuir um nome a um grupo de células. o Excel considere como função o que será inserido na fórmula
depois dele: o sinal de “=” (igual).
É possível procurar uma função digitando seu nome e
clicando em “Ir”; selecionar uma categoria para localizar mais
facilmente a função desejada; ou clicar diretamente no seu nome.
Além de facilitar a localização da função, o “Inserir função” exibe
a sintaxe e uma breve explicação do que a função selecionada
faz.

Informática 21
Apostila Digital Licenciada para Alice Caroline Guarino dos Santos - alice.guarino@hotmail.com (Proibida a Revenda)
APOSTILAS OPÇÃO
Quando clicamos em “OK”, a função escolhida é inserida na - Excluir: exclui a planilha selecionada.
célula e o programa direciona o usuário para a inserção dos - Renomear: permite alterar o nome da planilha ativa.
argumentos da função, implementando explicações e oferecendo - Mover ou copiar: permite a locomoção da planilha dentro
os caminhos necessários para a completa conclusão: da pasta de trabalho, deixando-a antes ou depois de determinada
planilha e alterando a ordem em que se encontrava entre as
guias das planilhas. Permite também criar uma cópia da planilha
selecionada e movê-la para a ordem determinada pelo usuário.
- Exibir código: abre o Visual Basic for Applications para que
códigos de programação sejam vinculados à planilha.
- Proteger Planilha: permite atribuir senhas que impeçam
a alteração do conteúdo da planilha por terceiros. Protege e
planilha e o conteúdo de células bloqueadas.
- Cor da Guia: permite alterar a cor da guia para destacar e
organizar as planilhas.
- Ocultar: esconte a planilha da lista de visualização. Após
usar este recurso, basta usar o Re-exibir para voltar a mostrar
a planilha.
- Selecionar todas: agrupa todas as planilhas permitindo
formatações e ações em conjunto como, por exemplo, atribuir
Argumentos da função uma cor a todas as guias.
Quando existem muitas planilhas para serem visualizadas,
C) Barra de fórmulas: aparecem no início e no final da lista de guias sinais de “...”
Tendo uma célula ativa, podemos inserir dados nela também reticências, indicando sua continuidade. Para exibir guias que
pela barra de fórmulas. Ou seja, clicamos da célula para ativá- não estão sendo visualizadas, basta clicar nesses sinais.
la, mas para uma melhor visualização, digitamos o conteúdo na Indicado pela seta da figura acima, temos o botão “Nova
barra de fórmulas. Mas a principal função dessa barra é mostrar Planilha”, que insere mais uma planilha automaticamente na
o conteúdo real da célula, pois quando o conteúdo da célula se lista das guias.
trata de uma função, a célula mostrará apenas o resultado dela. Barra de Status: mostra informações sobre a planilha a
Já a barra de fórmulas mostra o real conteúdo da célula, seja forma de exibição do documento (normal, layout da página,
texto, seja fórmula. visualização de quebra de página) e o controle do zoom.

Novas pastas
Para criar uma nova pasta, quando iniciamos o programa é
exibida a seguinte tela:

Barra de fórmulas

Continuando a estrutura de nossa pasta e planilhas, teremos,


na ordem, o cabeçalho das colunas, das linhas que finalmente
darão origem às céluas. Teremos então, a planilha em si:

Tela Inicial Excel 2013

Com a tela inicial do Excel 2013 podemos escolher inicar


uma nova pasta clicando na opção:
Planilha
- Pasta de trabalho em branco - abrirá um arquivo vazio para
darmos início ao nosso trabalho;
Na figura a cima, o canto marcado com uma seta mostra o
- Em um dos modelos como “Amortização de empréstimos”,
local em que podemos, com apenas um clique, selecionar a
“Cartão de ponto” ou outros, o arquivo abrirá com formatações
planilha inteira.
prontas, bastando alterar o texto nos lugares indicados para a
Barras de rolagem: localizadas nas extremidades direita
criação de uma pasta com aparência profissional.
e inferior da página, permitem rolar a planilha na tela dando
Após a abertura do programa, para dar incício a uma nova
visibilidade a todo o seu conteúdo.
planilha, aciona-se, na guia Arquivo, que é a primeira no canto
Permitem navegar entre as planilhas existentes, indo para a
superior esquerdo da Faixa de opções, a opção Novo, conforme
planilha anterior, para a próxima ou:
indicado na figura a seguir:
- Ctrl+ botão esquerdo do mouse: rola a visualização para a
primeira ou para a última planilha, dependendo do navegador
selecionado.
- Clique com o botão direito do mouse: abre a lista com os
nomes de todas as planilhas, facilitando seu acesso com apenas
um clique:

Guias das planilhas:


Cada planilha tem sua própria guia, ou seja, sua própria aba,
que recebe seu nome e pode sofrer algumas formatações como
ter a cor da guia alterada.
Clicando com o botão direito do mouse em uma guia é
possível:
- Inserir: abre a janela Inserir, que permite a inserção de uma
nova planilha em branco ou com outros objetos como gráficos e
macros.

Informática 22
Apostila Digital Licenciada para Alice Caroline Guarino dos Santos - alice.guarino@hotmail.com (Proibida a Revenda)
APOSTILAS OPÇÃO
realizadas, é possível acionar as telas que vimos através da guia
“Arquivo”, opção “Salvar como”. Com ela sempre será possível
escolher outro local, outro nome e outro tipo para o arquivo.

Abrir arquivos
Para abrir arquivos existentes, após a abertura do
programa, usamos a guia “Arquivo” e a opção “Abrir”. Com estes
procedimentos, será aberta uma tela com a lista das pastas
recentemente utilizadas, para facilitar a sua abertura com
apenas um clique em seu nome.
Caso a pasta desejada não esteja na lista das pastas de
trabalho recentes, podemos clicar no ícone “Computador” e
procurá-la em uma das pastas da lista, ou pelo botão “Procurar”.
Quando clicamos em “Procurar”, é aberta uma tela que dá
acesso a todo o conteúdo do computador. Nela, clicando nas
pastas e subpastas ou em dispositivos de armazenamento e de
Nova pasta rede conectados a máquina, seguimos o caminho que nos leva a
pasta de trabalho desejada. Ao encontrá-la, clicamos duas vezes
Após essas ações, continuamos a criação da nova pasta, sobre seu ícone e será aberta.
como visto anteriormente, ou seja, clicando sobre o modelo que
desejamos usar. Elaboração de tabelas e gráficos
Os recursos de edição de uma planilha compreendem a
Salvar arquivo digitação, os comandos Desfazer/Refazer, Repetir, os comandos
A criação do arquivo é efetivada após seu salvamento, ou dos grupos Área de Transferência, Fonte, Alinhamento, Número,
seja, sua gravação em alguma mídia de armazenamento, como o Estilo Células e Edição, que encontramos na guia Página Inicial.
HD do computador ou um pendrive. Iniciaremos com os grupos Área de trasferência e Edição
Com o arquivo aberto, clique no ícone Salvar, ou use as teclas para depois tratarmos separadamente dos grupos Fonte,
de atalho Ctrl+B, conforme indicado na figura: Alinhamento, Número, Estilo Células, que compreendem a
Outra forma de acionar o mesmo comando, é clicando na formatação da planilha, propriamete dita.
guia Arquivo e depois na opção Salvar.
Com a tela acima vemos o conteúdo da pasta atualmente Grupo Área de Transferência:
selecinada para gravação. Clicando em “Procurar”, será aberta Possui os botões de comando para colar, recortar, copiar e
uma tela onde é possível definir o local, o nome e o tipo de pincel de formatação. Quando usamos os recursos de recortar
arquivo que guardará nosso trabalho. e copiar, o conteúdo recortado ou copiado fica armazenado na
memória RAM do computador, em uma área denominada área
de transferência.
Colar – aplica no documento um texto ou imagem que foi
copiada ou recortada. Teclas de atalho CTRL + V.
Recortar – retira do documento e coloca na área de
transferência um texto ou imagem selecionada. Teclas de atalho
CTRL + X.
Copiar – cria uma cópia do texto ou imagem selecionado e
deixa na área de transferência. Teclas de atalho CTRL + C.
Pincel de formatação – Guarda a formatação do texto
selecionado para aplicar em outro ponto do texto. Teclas de
atalho CTRL + SHIFT + C.
Grupo Edição:
Soma: exibe a soma das células selecionadas diretamente
após essas células.
Classificar e filtrar: organiza os dados para que sejam mais
facilmente analisados.
Localizar e Selecionar: localiza e seleciona um texto
Salvar como específico, uma formatação ou um tipo de informação na pasta
de trabalho.
1 e 2– Escolhemos o local onde o arquivo será gravado, Preencher: continua um padrão em uma ou mais células
clicando (1) na pasta e (2) na subpasta de destino. adjacentes.
3 – Digitamos seu nome ou mantemos o nome padrão. Limpar: exclui todos os elementos da célula ou remove
4 – Determinamos o tipo do arquivo. seletivamente a formatação, o conteúdo ou os comentários.
São vários os tipos de arquivo que podemos escolher. O
tipo do arquivo determina qual será a sua extensão. A extensão Grupo Fonte:
é o conjunto de três ou quatro letras colocadas pelo programa Permite a formatação da fonte das palavras ou caracteres
após seu nome e um . (ponto). Por exemplo: material escrito. selecionados, mudando sua forma, tamanho e usando efeitos
xlsx. No exemplo, “material escrito” é o nome do arquivo e para realçar ou fazer indicações especiais no texto, como a
“.xlsx” é a extensão. Ela serve de referência para sabermos em aplicação de itálico, para indicar uma palavra em outro idioma,
qual programa um determinado arquivo foi criado e em quais ou sublinhado para indicar um link.
programas poderá ser aberto.
A opção “Salvar”, em todas as suas formas de acesso (Ctrl+B,
ícone Salvar ou Arquvio, Salvar), abre a janela Salvar como
apenas na primeira vez que é acionado para cada arquivo. Depois
que ele já está gravado, se for acionada novamente, irá gravar as
alterações efetuadas automaticamente no mesmo arquivo. Essa
ação faz com que tenhamos apenas o arquivo em sua forma mais
atual.
Se a intensão for manter o arquivo original e salvar um
outro arquivo que tenha o conteúdo inicial mais as alterações

Informática 23
Apostila Digital Licenciada para Alice Caroline Guarino dos Santos - alice.guarino@hotmail.com (Proibida a Revenda)
APOSTILAS OPÇÃO
Para criarmos um gráfico:

1º) Selecionamos um grupo de células que, obrigatoriamente,


têm que envolver dados numéricos. Somente com dados
numéricos contidos nesta seleção será possível criar um gráfico,
pois os gráficos representam (expressam) dados numéricos.

Grupo fonte

1 – Fonte (Ctrl+Shift+F): permite a seleção de uma nova


fonte para o texto. Seleção das células para criação do gráfico
2 – Tamanho da fonte (Ctrl+Shift+P): muda o tamanho dos
caracteres do texto selecionado. 2º) Escolher um tipo de gráfico que represente
3 – Aumentar tamanho fonte (Ctrl+>): aumenta o tamanho adequadamente o que desejamos. Temos que tomar um cuidado
da fonte da palavra, letra ou caracteres selecionados. especial na hora de escolher o tipo de gráfico, pois nem sempre
4 – Reduzir tamanho da fonte (Ctrl+<): reduz o tamanho da ele consegue representar o que desejamos. Por isso, devemos
fonte da palavra, letra ou caracteres selecionados. ler atentamente a breve explicação que aparece sob os tipos de
5 – Negrito (Ctrl+N): aplica negrito ao texto selecionado. gráficos para escolhermos o mais adequado:
6 – Itálico (Ctrl+I): aplica itálico ao texto selecionado.
7 – Sublinhado (Ctrl+S): permite sublinhar, ou seja, desenhar
uma linha na base da palavra selecionada.
8 – Bordas: aplica e configura bordas para as células
selecionadas.
9 – Cor de Preenchimento: aplica cor ao plano de fundo das
células para destaca-las.
10 – Cor da fonte: muda a cor do texto.

Grupo Número:
Escolhe como os valores de uma célula serão exibidos: como
percentual, moeda, data ou hora. Os botões exibidos na imagem
acima, respectivamente, transformam os números em: estilo Aplicação do gráfico
da moeda, percentual, milhar, diminuem as casas decimais e
aumentam as casas decimais. Os gráficos podem ser:
- Colunas: usados para comparar valores em diversas
Grupo Estilo: categorias.
Formatação condicional: realça as células desejadas, - Linhas: são usados para exibir tendências ao longo do
enfatizando valores que temos a intenção de ressaltar para o tempo.
usuário, seja por representarem o resultado final de uma função - Pizza: exibem a comparação de valores em relação a um
ou uma condição. Podemos usar, para essa formatação, estilo de total.
fonte, de preenchimento, entre outros recursos. Por exemplo, - Barras: comparam múltiplos valores.
se desejarmos que uma célula fique com a cor da fonte em - Área: mostram as diferenças entre vários conjuntos de
vermelho, sempre que seu valor for negativo, podemos usar a dados ao longo de um período de tempo.
formatação condicional. - Dispersão: compara pares de valores.
Formatar como tabela: formata rapidamente um intervalo - Outros gráficos: possibilita a criação de gráficos como
de células e o converte em tabela, escolhendo um estilo de tabela Ações, Superfície, Rosca, Bolhas e outros.
predefinido.
Estilo de célula: formata rapidamente uma célula escolhendo Uso de fórmulas e funções
um dos estilos predefinidos. A principal função do Excel é facilitar os cálculos com o uso
de suas fórmulas. A partir de agora, estudaremos várias de suas
Grupo Células: fórmulas. Para iniciar, tenhamos em mente que, para qualquer
Inserir: insere linhas, células, colunas e tabelas. fórmula que será inserida em uma célula, devemos utilizar o
Excluir: exclui linhas, células, colunas e tabelas. sinal de “=” no seu início. Esse sinal oferece uma entrada no
Formatar: altera a altura da linha ou a largura da coluna, Excel que o faz diferenciar textos ou números comuns de uma
organiza planilhas ou protege/oculta células. fórmula.
Guia Inserir: SOMAR
Grupo Gráficos: Se tivermos uma sequência de dados numéricos e quisermos
realizar a sua soma, temos as seguintes formas de fazê-la:
Usamos, nesse exemplo, a fórmula =B2+B3+B4.
Após o sinal de “=” (igual), clicar em uma das células, digitar
o sinal de “+” (mais) e continuar essa sequência até o último
valor.
Após a sequência de células a serem somadas, clicar no ícone
soma, ou usar as teclas de atalho Alt+=.
Após inserir o sibal, o Excel mostrará um pequeno lembrete
Grupo gráficos sobre a função que usaremos, onde é possível clicar e obter
ajuda, também. Utilizaremos, no exemplo a seguir, a função =
Após selecionar células, podemos escolher um dos tipos de soma(B2:B4).
gráficos para serem criados na planilha referente aos dados ou
em uma nova planilha separadamente.

Informática 24
Apostila Digital Licenciada para Alice Caroline Guarino dos Santos - alice.guarino@hotmail.com (Proibida a Revenda)
APOSTILAS OPÇÃO
B2 – refere-se ao endereço do valor da compra
* - sinal de multiplicação
5/100 – é o valor do desconto dividido por 100 (5%). Ou
seja, você está multiplicando o endereço do valor da compra por
5 e dividindo por 100, gerando assim o valor do desconto.
Lembrete mostrado pelo Excel. Se preferir pode fazer o seguinte exemplo:
=B2*5% Onde:
No “lembrete” exibido na figura acima, vemos que após a B2 – endereço do valor da compra
estrutura digitada, temos que clicar em um número, ou seja, em * - sinal de multiplicação
uma célula que contém um número, depois digitar “;” (ponto 5% - o valor da porcentagem.
e vírgula) e seguir clicando nos outros números ou células Depois para saber o Valor a Pagar, basta subtrair o Valor da
desejadas. Compra – o Valor do Desconto, como mostra no exemplo.
Aqui vale uma explicação: o “;” (ponto e vírgula) entre as
sentenças usadas em uma função indicam que usaremos uma MÁXIMO
célula e outra. Podem ser selecionadas mantendo a tecla CTRL Mostra o maior valor em um intervalo de células selecionadas.
pressionada, por exemplo. Na figura a seguir, calcularemos a maior idade digitada
Existem casos em que usaremos no lugar do “;” (ponto e no intervalo de células de A2 até A5. A função digitada será =
vírgula) os “:”, dois pontos, que significam que foi selecionada máximo (A2:A5).
uma sequência de valores, ou seja, de um valor até outro, ou de
uma célula até outra.
Dando continuidade, se clicarmos sobre a palavra “soma”,
do nosso “lembrete”, será aberta uma janela de ajuda no Excel,
onde podemos obter todas as informações sobre essa função.
Apresenta informações sobre a sintaxe, comentários e exemplos
de uso da função. Esses procedimentos são válidos para todas as
funções, não sendo exclusivos da função “Soma”. Exemplo da função máximo

SUBTRAÇÃO Onde: “= máximo” – é o início da função; (A2:A5) – refere-


A subtração será feita sempre entre dois valores, por isso se ao endereço dos valores onde você deseja ver qual é o maior
não precisamos de uma função específica. valor. No caso a resposta seria 10.
Tendo dois valores em células diferentes, podemos apenas
clicar na primeira, digitar o sinal de “-” (menos) e depois clicar MÍNIMO
na segunda célula. Usamos na figura a seguir a fórmula = B2-B3. Mostra o menor valor existente em um intervalo de células
selecionadas.
MÉDIA
A função da média soma os valores de uma sequência
selecionada e divide pela quantidade de valores dessa sequência.
Todas as funções, quando um de seus itens for alterado,
recalculam o valor final.
Exemplo de subtração
DATA
MULTIPLICAÇÃO Esta fórmula insere a data automática em uma planilha.
Para realizarmos a multiplicação, procedemos de forma
semelhante à subtração. Clicamos no primeiro número, INTEIRO
digitamos o sinal de multiplicação que, para o Excel é o “*” Com essa função podemos obter o valor inteiro de uma
asterisco, e depois clicamos no último valor. fração. A função a ser digitada é =int(A2). Lembramos que A2 é a
Outra forma de realizar a multiplicação é através da seguinte célula escolhida e varia de acordo com a célula a ser selecionada
função: na planilha trabalhada.
=MULT(B2;C2) multiplica o valor da célula B2 pelo valor da
célula C2. ARREDONDAR PARA CIMA
Com essa função é possível arredondar um número com
A B C E casas decimais para o número mais distante de zero.
Sua sintaxe é:
1 PRODUTO VALOR QUANT. TOTAL = ARREDONDAR.PARA.CIMA(núm;núm_dígitos)
2 Feijão 1,50 50 =MULT(B2;C2) Onde:
Núm: é qualquer número real que se deseja arredondar.
Núm_dígitos: é o número de dígitos para o qual se deseja
DIVISÃO
arredondar núm.
Para realizarmos a divisão, procedemos de forma semelhante
à subtração e multiplicação. Clicamos no primeiro número,
ARREDONDAR PARA BAIXO
digitamos o sinal de divisão que, para o Excel é a “/” barra, e
Arredonda um número para baixo até zero.
depois clicamos no último valor.
ARREDONDAR.PARA.BAIXO(núm;núm_dígitos)
Núm: é qualquer número real que se deseja arredondar.
PORCENTAGEM
Núm_dígitos: é o número de dígitos para o qual se deseja
Para aprender sobre porcentagem, vamos seguir um
arredondar núm.
exemplo: um cliente de sua loja fez uma compra no valor de
R$1.500,00 e você deseja dar a ele um desconto de 5% em cima
RESTO
do valor da compra. Veja como ficaria a fórmula na célula C2.
Com essa função podemos obter o resto de uma divisão. Sua
sintaxe é a seguinte:
= mod (núm;divisor)
Onde:
Núm: é o número para o qual desejamos encontrar o resto.
Exemplo de porcentagem Divisor: é o número pelo qual desejamos dividir o número.
Onde:

Informática 25
Apostila Digital Licenciada para Alice Caroline Guarino dos Santos - alice.guarino@hotmail.com (Proibida a Revenda)
APOSTILAS OPÇÃO
VALOR ABSOLUTO
Com essa função podemos obter o valor absoluto de um Gráficos de linhas
número. O valor absoluto é o número sem o sinal. A sintaxe da exibem dados contínuos
função é a seguinte: ao longo do tempo,
=abs(núm) ideais para mostrar
Onde: tendências em dados a
ABS(núm)
intervalos iguais.
Núm: é o número real cujo valor absoluto você deseja obter.

DIAS 360 Gráficos de pizza


Retorna o número de dias entre duas datas com base em um mostram o tamanho
ano de 360 dias (doze meses de 30 dias). Sua sintaxe é: de itens em uma série
= DIAS360(data_inicial;data_final) de dados, de modo
Onde: proporcional à soma
Data_inicial = a data de início de contagem. dos itens. Os pontos de
Data_final = a data à qual quer se chegar. dados em um gráfico de
pizza são exibidos como
FUNÇÃO SE um percentual de toda
A função se é uma função lógica e condicional, ou seja, ela a pizza.
trabalha com condições para chegar ao seu resultado.
Sua sintaxe é:
= se (teste_lógico; “valor_se_verdadeiro”; “valor_se_falso”) Gráficos de barras
Onde: apresentam a relação de
= se( = início da função. cada item com o todo,
Teste_lógico = é a comparação que se deseja fazer. exibindo os dados em
Vale lembrar que podemos fazer vários tipos de comparações. três eixos.
Para fins didáticos, usaremos células A1 e A2, supondo que
estamos comparando valores digitados nessas duas células. Os Gráficos de área
tipos de comparação possíveis e seus respectivos sinais são: enfatizam a magnitude
A1=A2 → verifica se o valor de A1 é igual ao valor de A2 da mudança no decorrer
A1<>A2 → verifica se o valor de A1 é diferente do valor de A2 do tempo e podem ser
A1>=A2 → verifica se o valor de A1 é maior ou igual ao valor usados para chamar
de A2 atenção para o valor
A1<=A2 → verifica se o valor de A1 é menor ou igual ao valor
total ao longo de uma
de A2
A1>A2 → verifica se o valor de A1 é maior do que o valor de tendência.
A2 Um gráfico de dispersão
A1<A2 → verifica se o valor de A1 é menor do que o valor possui dois eixos de
de A2 valores, mostrando
um conjunto de dados
No lugar das células podem ser colocados valores e até numéricos ao longo
textos. do eixo horizontal e
Valor_se_verdadeiro = é o que queremos que apareça outro ao longo do eixo
na célula, caso a condição seja verdadeira. Se desejarmos que
vertical, indicado para
apareça uma palavra ou frase, dentro da função, essa deve estar
entre “” (aspas). exibição e comparação
Valor_se_falso= é o que desejamos que apareça na célula, de valores numéricos,
caso a condição proposta não seja verdadeira. como dados científicos,
estatísticos e de
Gráficos engenharia.
Um gráfico é uma representação visual de seus dados. Tipos de Gráficos5
Usando elementos como colunas (em um gráfico de colunas) ou
linhas (em um gráfico de linhas), um gráfico exibe uma série de Impressão e controle de quebras
dados numéricos em um formato gráfico4. Para imprimir um arquivo podemos antes definir a Área
O Excel, disponibiliza os gráficos em diversos formatos, de Impressão, ou seja, qual parte da planilha será impressa.
facilitando a interpretação dos dados relacionados. Os tipos de Para isso, devemos selecionar as células que desejamos, clicar
gráficos disponíveis estão contido na aba Inserir da Barra de no Botão de Comando Área de Impressão e em Definir Área de
Ferramentas: Impressão.
Caso deseje alterar a parte da planilha a ser impressa, clique
Tipos de Gráficos novamente na Guia Layout da Página e, no Grupo Configurar
Página, acione o Botão de Comando Área de Impressão e Limpar
Característica Exemplo
Área de Impressão.
Após definir a Área de Impressão é preciso acionar o
Gráficos de colunas comando Imprimir, que fica na Guia Arquivo, mas não é
obrigatória a definição da área de impressão. Se ela não for
apresentam valores
definida, toda a planilha será impressa.
comparados através de Para efetivar a impressão de um arquivo, clique na Guia
retângulos na vertical. Arquivo → Imprimir.
Ao acionar a Guia Arquivo → Imprimir, é possível:
- clicar diretamente no botão Imprimir, para enviar o arquivo
para a impressão;
- selecionar o número de cópias;
4 Criar gráficos com seus dados em uma planilha - https://
support.office.com/pt-br/article/In%C3%ADcio-r%C3%A1pido- 5 Tipos de Gráficos Disponíveis - https://support.office.
crie-gr%C3%A1ficos-com-seus-dados-45af 7d1b-4a45-4355-9698- com/pt-br/article/Tipos-de-gr%C3%A1ficos-dispon%C3%ADveis-
01126488e689 a6187218-807e-4103-9e0a-27cdb19afb90#bmcolumncharts

Informática 26
Apostila Digital Licenciada para Alice Caroline Guarino dos Santos - alice.guarino@hotmail.com (Proibida a Revenda)
APOSTILAS OPÇÃO
- terminar em qual impressora a impressão será realizada e da Página, no Grupo Configurar Página. As quebras de página
definir propriedades da impressora. de uma planilha podem ser removidas e redefinidas através do
É possível realizar diversas configurações de impressão: mesmo botão de comando.
- Definir o que será impresso da pasta:
- Imprimir somente as planilhas ativas, ou seja, as que Classificação
estão selecionadas; Clique na guia Dados, no grupo Classificar e Filtrar > clique
- Imprimir toda a pasta de trabalho, ou seja, imprimir em Classificar. Procurando este ícone:
todas as planilhas existentes no arquivo aberto.
- Imprimir seleção, ou seja, levar à impressora apenas
a parte selecionada na planilha.
- Selecionar as páginas a serem impressas, marcando a
página inicial em “Páginas” e a final em “para”. Este procedimento abrirá a tela Classificar, que permite
- Definir se a impressão sairá “Agrupada”, isto é, supondo diversas configurações para que a classificação dos dados
que sejam várias cópias, sairão todas em ordem crescente de selecionados atenda a necessidade do usuário.
numeração de página. Desta forma será impressa primeiro a Podemos usar os botões de comando “Classificar de A a
página 01, depois a 02 e assim por diante, até finalizar a primeira Z” e “Classificar de Z a A”, do mesmo grupo. Esses comandos,
cópia. A segunda cópia será iniciada após o término da primeira. respectivamente, classificam do menor para o maior valor e do
- Definir se a impressão sairá “Desagrupada”, o que faria que maior para o menor.
fossem impressas todas as páginas 01 de cada cópia, depois as
páginas 02 de todas as cópias e assim continuamente até que Macros
todas as páginas fossem impressas de todas as cópias. Por Segundo informações do próprio programa Microsoft Excel:
exemplo, as páginas de número 02 só serão impressas após “Para automatizar tarefas repetitivas, você pode gravar uma
todas as primeiras páginas, de todas as cópias, serem impressas. macro (macro: uma ação ou um conjunto de ações que você
- Escolher a Orientação do Papel. Esta configuração define se pode usar para automatizar tarefas. As macros são gravadas
a impressão sairá no papel em sua posição Retrato ou Paisagem. na linguagem de programação Visual Basic for Applications.)
O recurso de Orientação do Papel também é encontrado na Guia rapidamente no Microsoft Office Excel. Você também pode criar
Layout de Página, Grupo Configurar Página, na forma do Botão uma macro usando o Editor do Visual Basic (Editor do Visual
de Comando Orientação. Basic: um ambiente no qual você escreve um novo código e
- Configurar o Tamanho do Papel, escolhendo entre A4, novos procedimentos do Visual Basic for Applications e edita os
Tablóide, Ofício, Executivo e outros. O recurso Tamanho do já existentes. O Editor do Visual Basic contém um conjunto de
Papel também é encontrado na Guia Layout de Página, Grupo ferramentas de depuração completo para localizar problemas
Configurar Página, na forma do Botão de Comando Tamanho. lógicos, de sintaxe e tempo de execução em seu código.), no
- Determinar as margens do arquivo, alterando as margens Microsoft Visual Basic, para gravar o seu próprio script de macro
esquerda, direita, inferior e superior para tamanhos pré- ou para copiar toda a macro, ou parte dela, para uma nova. Após
definidos ou personalizá-las, digitando tamanhos alternativos. criar uma macro, você poderá atribuí-la a um objeto (como
As margens também podem ser configuradas pelo Botão de um botão da barra de ferramentas, um elemento gráfico ou
Comando Margens, encontrado na Guia Layout de Página, Grupo um controle) para poder executá-la clicando no objeto. Se não
Configurar Página, Grupo Configurar Página. precisar mais usar a macro, você poderá excluí-la.”
- Ajustar o tamanho da planilha na impressão. Este recurso
traz várias opções de dimensionamento: Gravar uma macro
- Sem dimensionamento: imprime a planilha em Para gravar uma macro, observe se a guia Desenvolvedor
tamanho real; está disponível. Caso não esteja, siga os seguintes passos:
- Ajustar planilha em uma página: reduz a cópia 1. “Clique no Botão do Microsoft Office e, em seguida,
impressa para ajustá-la em uma página; clique em Opções do Excel.
- Ajustar todas as colunas em uma página: reduz a 2. Na categoria Popular, em Opções principais para o
cópia impressa para que caiba na largura de uma página; trabalho com o Excel, marque a caixa de seleção Mostrar guia
- Ajustar todas as linhas em uma página: reduz a cópia Desenvolvedor na Faixa de Opções e clique em OK.
impressa para que caiba na altura de uma página; Para definir o nível de segurança temporariamente e
- Opções de dimensionamento personalizado: permite habilitar todas as macros, faça o seguinte:
que o usuário escolha a porcentagem que será impressa do Na guia Desenvolvedor, no grupo Código, clique em
tamanho real. Segurança de Macro.
No menu Imprimir encontramos a opção “Configurar Página”.
Clicando nesta opção, será aberta a janela “Configurar Página”.
Nesta janela, realizamos configurações de página, margens,
cabeçalho e rodapé e forma de impressão da planilha.
Se for necessário escolher linhas e colunas a serem repetidas
em cada página impressa; por exemplo, linhas e colunas como
rótulos ou cabeçalhos, use o Botão de Comando Imprimir
Títulos, da Guia Layout de Página.
Na mesma Guia, com o Botão Plano de Fundo, determinamos Em Configurações de Macro, clique em Habilitar todas
uma imagem que ficará como plano de fundo de toda a planilha. as macros (não recomendável; códigos possivelmente
O controle de quebras adiciona quebras de página no local perigosos podem ser executados) e em OK.
que você quer que a próxima página comece na cópia impressa. Para incluir uma descrição da macro, na caixa Descrição,
A quebra de página será inserida à cima e à esquerda da sua digite o texto desejado.
sessão. Clique em OK para iniciar a gravação.
Execute as ações que deseja gravar.
Na guia Desenvolvedor, no grupo Código, clique em Parar
Gravação.

Questões

01. (IADES - CRESS-MG - Auxiliar Administrativo/2016).


Quebra de Página Como mesclar células em uma tabela no Microsoft Excel 2016?
(A) Selecionar a célula da esquerda, depois clicar em Início >
O comando Quebra de Página é encontrado na Guia Layout

Informática 27
Apostila Digital Licenciada para Alice Caroline Guarino dos Santos - alice.guarino@hotmail.com (Proibida a Revenda)
APOSTILAS OPÇÃO
Mesclar e Centralizar. (A) 9 (C) 5 (E) 3
(B) Na guia Página Inicial, no grupo Células, clicar em (B) 7 (D) 4
Formatar. Respostas
(C) Selecionar duas ou mais células adjacentes que se deseja 01. C/02. E/03. B /04. E /05. B
mesclar, depois clicar em Início > Mesclar e Centralizar.
(D) Na guia Página Inicial, no grupo Células, clicar em Inserir.
(E) Selecionar duas ou mais células adjacentes que se Correio Eletrônico: uso de correio
deseja mesclar, depois clicar em Início > Quebrar Texto eletrônico, preparo e envio
Automaticamente. de mensagens, anexação de
02. (FAURGS - TJ-RS - Assessor Judiciário/2016). arquivos.
Deseja-se calcular, por meio do MS-Excel, um valor usado em
DOC bancário, chamado “Fator de Vencimento”. Tal valor é o
número de dias decorridos desde a implantação do sistema OUTLOOK 20136
(07/10/1997) até a data de vencimento desejada. O Cálculo será
feito na célula C1, que já foi formatada para número com zero
casas decimais.

Considerando-se que se tem na célula A1 a data de


07/10/1997 e na célula B1 a data desejada, 22/05/2016, qual
das fórmulas abaixo NÃO faria o cálculo correto?
(A) = B1-A1
(B) = DATA(2016;5;22)-A1
(C) = B1-DATA(1997;10;7) Torne-o seu: Personalize o Outlook. Escolha entre cores
(D) = DATA.VALOR(“22/05/2016”)-DATA.VALOR de esquema e fundos diferentes e sincronize-os com outros
(“07/10/1997”) computadores que possui.
(E) = DATA(22;5;2016)-DATA.VALOR(“07/10/1997”) 1- Procurar: Você pode procurar uma pasta, subpastas e até
outras caixas de correio.
03. (Petrobras - Técnico de Administração e Controle 2- Faça gestão de tarefas de mensagem na Lista de
Júnior - CESGRANRIO – 2014 - Adaptada) Com referência ao Mensagens: Categorize, Sinalize ou Elimine mensagens onde
MS Excel, a fórmula que está incorretamente escrita é: residem – na Lista de Mensagens.
3- Faça mais na Barra de Ações a Fazer: A Barra de Ações
(A) =SOMA((B2+C3)/A1) a Fazer apresenta o navegador de data, os seus compromissos,
(B) =((D3+C3)/A1)-D2) pessoas e a sua lista de tarefas.
(C) =SOMA(B2) 4- Painel Pessoas: Consulte detalhes sobre todos nas linhas
(D) =(B2+C4-X2)/B2/B2 Para, De ou Cc de cada mensagem.
(E) =$Y2+Y$2/MÉDIA(B2:B10) 5- Vista Backstage: Clique no separador Ficheiro para abrir a
vista Backstage, onde pode adicionar contas e alterar definições.
04. (Prefeitura de Florianópolis - SC - Fiscal de Serviços 6- Responda a partir do painel de leitura: Pode Responder
Públicos – FGV) Em determinada planilha Excel, as células A1,
e Reencaminhar diretamente a partir do painel de leitura.
A2, A3, A4 e A5 contêm, respectivamente, os valores numéricos
Também pode escrever uma resposta no painel de leitura.
5, 9, 15, 21 e 35.
7- Pré-visualize: Veja uma vista rápida do seu Calendário,
Os conteúdos das células B1, B2 e B3 são respectivamente: Pessoas e Tarefas. Veja também compromissos futuros.

=A1+A3 ← conteúdo da célula B1


=A2+A4+A5 ← conteúdo da célula B2
=(B1*2)+(B2*2) ← conteúdo da célula B3

Sendo assim, qual é o resultado numérico da fórmula da


célula B3?

(A) 20 (C) 65 (E) 170


(B) 28 (D) 85
05. (SP-URBANISMO - Assistente Administrativo-
VUNESP - Adaptada) Observe a planilha a seguir, que está sendo
editada por meio do MS-Excel, em sua configuração padrão.

Adicionar a sua conta.


Antes de poder enviar ou receber mensagens, terá de ligar
a sua conta de e-mail. Se a sua empresa utiliza o Microsoft
Exchange, o Outlook 2013 tentará configurar a sua conta de
e-mail. Se utiliza serviços de e-mail baseados na Internet (tais
como o Hotmail, Gmail ou Yahoo!), introduza o seu nome,
Assinale a alternativa que contém o resultado exibido na endereço de e-mail e palavra-passe para configurar a sua conta.
célula D1, após ser preenchida com a fórmula =MAIOR(A1:C3;3).
6 Fonte: Guia de introdução ao Outlook 2013 - Microsoft

Informática 28
Apostila Digital Licenciada para Alice Caroline Guarino dos Santos - alice.guarino@hotmail.com (Proibida a Revenda)
APOSTILAS OPÇÃO
Inserir uma Inserir A partir de uma nova
imagem ou clip mensagem de correio,
art clique em Ilustrações
e, em seguida, clique
em um dos seguintes:
Imagens, Imagens
Online, Formas, Smart
Art, Gráfico ou Captura
de Ecrã.
Gerir Base Na vista de Correio,
mensagens clique em Mover >
de e-mail Regras.
atribuindo
Alterar o Tema do Office regras
Reestruturámos o Office 2013 para disponibilizar uma Inserir um Inserir A partir de uma nova
experiência limpa e uniforme - como uma folha de papel em símbolo mensagem de correio,
branco. Se pretender ver as diferentes áreas do Outlook de ou caráter clique em Símbolos >
forma mais distinta, pode alterar o Tema do Office. especial Símbolo.
Utilize as definições em Ficheiro > Conta do Office para alterar Adicionar Ficheiro Clique em Opções >
o esquema de cores do Office 2013 em todos os computadores, feriados ao Calendário. Em Opções
ou utilize as Opções do Outlook para alterar o esquema de cores calendário do calendário clique em
apenas no computador que está a utilizar. Pode escolher entre Adicionar Feriados.
Branco, Cinzento Claro ou Cinzento Escuro. Partilhar um Base A partir da vista de
calendário Calendário, no grupo
Partilhar, clique em
Enviar Calendário
por E-mail > Partilhar
Calendário (esta
funcionalidade necessita
de uma conta do
Microsoft Exchange
Server) ou em Publicar
Online.

O correio não é a única função

A comunicação é importante, mas o seu tempo também. O


Outlook 2013 atualizou a forma de gerir o seu tempo, tarefas e
pessoas.
Algumas coisas que poderá procurar
Utilize a seguinte lista para localizar algumas das ferramentas 1- Agende uma Reunião do Lync: Encontre-se com alguém
e comandos mais comuns no Outlook 2013. pessoalmente ou poupe a viagem agendando uma reunião
online com o Lync 2013.
Para... Clique em... E, em seguida, veja no... 2- Faça planos conforme o tempo
Aplicar papel Ficheiro Clique em Ficheiro > Veja a previsão meteorológica para a sua e outras cidades
de carta ou Opções > Correio > (máximo de cinco).
fundos a uma Papel de Carta e Tipos Quando planear uma reunião fora da cidade, saberá o que
mensagem de Letra. No separador arrumar na mala.
Papel de Carta Pessoal, 3- Pré-visualize: Paire o cursor sobre uma reunião ou
clique em Tema. compromisso para ver os seus detalhes.
Enviar Ficheiro Em Informações de 4- Veja calendários de outras pessoas Consulte rapidamente
respostas Conta, clique em as melhores alturas para agendar uma reunião com outras
automáticas Respostas Automáticas pessoas.
quando > Enviar respostas
estiver fora do automáticas e, em
escritório seguida, selecione
as suas opções (esta
funcionalidade necessita
de uma conta do
Microsoft Exchange
Server).

Informática 29
Apostila Digital Licenciada para Alice Caroline Guarino dos Santos - alice.guarino@hotmail.com (Proibida a Revenda)
APOSTILAS OPÇÃO
Criar uma assinatura de e-mail Questões
Para criar uma nova assinatura de e-mail, faça o seguinte:
1. Clique em Ficheiro > Opções > Correio. Em Compor 01. (EPE - Assistente Administrativo –
mensagens, clique em CESGRANRIO/2010) Qual o procedimento para escrever um
Assinaturas. e-mail usando o OUTLOOK?
2. No separador Assinatura de E-mail, clique em Novo e, em (A) Selecione o grupo e-mail na lateral esquerda, e depois
seguida, escreva um nome para a assinatura. selecione Novo na parte superior.
3. Na caixa Editar assinatura, escreva o texto que pretende (B) Selecione o grupo Contato na lateral esquerda, e depois
incluir na assinatura e, em seguida, utilize as ferramentas selecione Novo na parte superior.
incorporadas para a formatar. (C) Clique em Ferramentas e depois em Configurações de
conta e clique em Novo.
(D) Clique em Ferramentas e depois em Enviar/Receber.
(E) Clique em Ir e depois em e-mail.

02. (Prefeitura de Paulista-PE – Digitador –


UPENET/2014) Na configuração padrão, os arquivos de dados
do Outlook são armazenados com a extensão
(A) .dat.
(B) .oab.
(C) .ost.
(D) .pab.
(E) .pst.
Caso já tenha criado uma assinatura, pode copiá-la de uma Respotas
das mensagens enviadas e colá-la aqui. 01. A\02. E

Internet: navegação internet,


conceitos de URL, links, sites,
busca e impressão de páginas.

INTERNET7

A internet é um meio de comunicação muito importante,


onde o conjunto de várias redes interligadas proporcionam que
computadores possam se comunicar através dos protocolos
TCP/IP.
Com a internet podemos utilizar serviços como Web (a parte
multimídia da rede), correios eletrônicos, redes sociais, fazer
transferência de arquivos, etc.
Adicionar automaticamente uma assinatura a Word Wide Web
A Word Wide Web(rede de alcance mundial) é também
mensagens.
conhecida como Web ou WWW. O serviço WWW surgiu em 1989
como um integrador de informações, onde a grande maioria das
Para adicionar automaticamente uma assinatura a novas informações disponíveis na Internet podem ser acessadas de
mensagens de e-mail, efetue o seguinte procedimento: forma simples e consistente. A forma padrão das informações
1. A partir de qualquer vista, clique em Ficheiro > Opções > do WWW é o hipertexto, o que permite a interligação entre
Correio. Em Compor mensagens, clique em Assinaturas. diferentes documentos que possivelmente estão localizados em
2. Em Escolher assinatura predefinida escolha a assinatura diferentes servidores. O hipertexto é codificado com a linguagem
a adicionar a Novas Mensagens. Se quiser, pode escolher uma HTML (Hypertext Markup Language), que é a linguagem
assinatura diferente para Respostas/ reencaminhamentos. interpretada pelo o que chamamos de browsers exemplo de um
3. Para adicionar manualmente uma assinatura a uma nova browser é o Internet Explorer.
mensagem, no separador Mensagem, no grupo Incluir, clique em
Assinatura e, em seguida, clique na assinatura que pretende. INTRANET8

As intranets são redes internas às organizações que usam as


tecnologias utilizadas na rede mundial Internet. Através de tais
tecnologias é possível implementar uma poderosa ferramenta
de comunicação e de trabalho em grupo. A implantação de
uma intranet tem o potencial de aumentar a produtividade
pois facilita o acesso às informações dispersas nos diversos
computadores da organização.
Os serviços tipicamente disponibilizados através de uma
intranet são: correio-eletrônico transferência de arquivos,
grupos de usuários e Web. Embora a aplicação mais comum
na maioria das intranets seja o correio-eletrônico, é o uso da
tecnologia Web que caracteriza uma intranet. A tecnologia Web
facilita o acesso às informações, tem uma arquitetura aberta
e funciona independente da plataforma de hardware ou do
7 Fonte: http://www.inf.pucpcaldas.br/extensao/cereadd/
apostilas/internet_final.pdf
8 Fonte: file:///C:/Users/Ewertonjs/Downloads/intranet.un-
locked.pdf

Informática 30
Apostila Digital Licenciada para Alice Caroline Guarino dos Santos - alice.guarino@hotmail.com (Proibida a Revenda)
APOSTILAS OPÇÃO
sistema operacional. Podemos começar nossa navegação diretamente digitando
Exemplos de aplicações que podem ser disponibilizadas o endereço a ser acessado no browser e apertando ENTER no
através da tecnologia Web em uma intranet são: tutorias sobre teclado ou clicando no botão IR.
procedimentos adotados dentro da organização, informações
sobre aplicativos usados na organização, manuais usados na Páginas Favoritas e Histórico
organização, resumo de notícias que sejam de importância Se você utiliza a Internet constantemente, possivelmente
para a organização, informações sobre produtos e serviços, gostaria de ter um mecanismo fácil e simples para guardar
calendário de eventos e cursos oferecidos. as páginas que mais acessa. O menu Favoritos proporciona
esta funcionalidade. Esta opção funciona como um caminho
Conectando-se com a Internet permanente de acesso à lista de todos os sites favoritos, além de
Para se conectar a internet é necessário um aparelho mantê-los organizados.
qualquer (computador, celular, vídeo games) que possua um A história de todos os sites visitados também é mantida no
dispositivo que permita a comunicação, seja ela sem fio ou não. seu navegador (browser).
Vale lembrar que a internet deu seus primeiros passos a partir Você pode abrir a pasta histórico e visualizar a lista de sites
de cabos e fios e com o passar do tempo surgiram as conexões visitados no dia ou até mesmo na semana ou no mês.
sem fio. Essas duas funções do navegador podem ser manipuladas
Com um aparelho que possua o dispositivo de comunicação pelo usuário podemos adicionar ou excluir uma pagina a
em mãos temos que escolher o tipo de conexão mais apropriada, favoritos e também podemos excluir nosso histórico caso seja
abaixo segue algumas das conexões mais utilizadas: necessário.
Conexões que necessitam de fios (cabos):
A internet deu seus primeiros passos a partir de cabos e Endereços na Internet
fios. Apesar de soar como algo bastante antiquado, esses tipos Todos os endereços da Internet seguem uma norma
de conexões ainda são amplamente utilizados, principalmente estabelecida pelo InterNic, órgão americano pertencente a ISOC
devido à alta velocidade obtida por alguns. (Internet Society). No Brasil a responsabilidade pelo registro
Dial Modem de domínios (endereços) na rede Internet é do Comitê Gestor
A famosa internet discada foi praticamente o pontapé inicial Internet Brasil (GC).
da rede no Brasil. Apesar de ainda ser utilizada, não é mais tão Exemplo de endereço: http://www.google.com.br
popular quanto foi no início dos anos 2000. Onde:
Cabo 1. http:// - O Hyper Text Transfer Protocol, o protocolo
Já ouvimos falar de TV a cabo, certo? Algumas empresas padrão que permite que os computadores se comuniquem.
decidiram aliar a ela o acesso à internet. Com isso, uma linha O http:// é inserido pelo browser, portanto não é necessário
telefônica não era mais pré-requisito para se conectar, o que deu digitá-lo.
mais liberdade ao usuário. 2. www – padrão para a Internet gráfica.
Conexões sem fio (wireless) 3. google– geralmente é o nome da empresa cadastrada
Com a correria do dia a dia, ficar preso a um desktop junto ao Comitê Gestor.
para acessar a internet é algo fora de questão. Os notebooks 4. com – indica que a empresa é comercial. Algumas
trouxeram mais mobilidade e abriram as portas para as conexões categorias de domínios existentes são:
que dispensam a utilização de fios e cabos. A internet wireless Gov.br - Entidades governamentais
mostrou que a internet está em qualquer lugar. Org.br - Entidades não-governamentais
Wi-Fi Com.br - Entidades comerciais
Esse tipo de conexão, antes exclusiva dos laptops, tornou- Mil.br - Entidades militares
se tão popular que vários outros equipamentos passaram a Net.br - Empresas de telecomunicações
adotá-la. É o caso de celulares, smartphones e até mesmo alguns Edu.br - Entidades de ensino superior
computadores domésticos, que adicionaram um adaptador .br - Sites no Brasil
wireless para captar o sinal. .jp - Sites no Japão
Rádio
A conexão via rádio é bastante útil devido ao seu longo Protocolos para Internet9
alcance, o que favorece quem mora em cidades onde o sinal HTTP (Hyper Text Transfer Protocol) – É o protocolo utilizado
telefônico ou via cabo não alcança. O único problema é que, para para controlar a comunicação entre o servidor de Internet e o
obter o máximo da conexão, o sinal deve chegar à torre sem browser. Quando se abre uma página da Internet, vemos texto,
encontrar nenhum tipo de barreira, e até mesmo chuvas podem imagens, links ou outros serviços associados à Internet ou a uma
desestabilizá-la. Intranet. O HTTP é o responsável por redireccionar os serviços
quando seleccionamos alguma das opções da página web.
A moda da internet de bolso SMTP (Simple Mail Transfer Protocol) – Como o nome indica,
Os usuários de telefones celulares sempre desejaram este protocolo serve para efectuar a transferência de emails en-
conectar-se à internet através de seus aparelhos móveis. Hoje tre os servidores. O servidor de email utiliza o POP ou IMAP para
em dia podemos conferir emails ou saber das novidades online enviar as mensagens de email aos utilizadores.
em qualquer lugar através de algumas das conexões existentes FTP (File Transfer Protocol) – Este protocolo permite trans-
citadas a baixo. ferência de dados ou ficheiros entre computadores, mesmo com
WAP sistemas operativos diferentes como o Linux e o Windows. O
A primeira grande tentativa de integrar os aparelhos celulares FTP é também um comando que permite ligação de um cliente
à internet. A conexão WAP era uma espécie de adaptação da a um servidor FTP de forma a transferir dados via Internet ou
web, já que só podia acessar páginas feitas especialmente para Intranet.
este tipo de conexão. SNMP (Simple Network Management Protocol) – É um proto-
3G colo de comunicação que permite recolher informação sobre to-
Funciona de maneira semelhante à conexão a rádio e os dos os componentes que estão na rede como switches, routers,
sinais são enviados praticamente pelas mesmas torres que bridges e os computadores ligados em rede.
enviam o sinal de telefonia para o aparelho, o que significa um TCP (Transfer Control Protocol) – O TCP permite dar segu-
amplo raio de alcance. rança à transferência de informações e verificar se a mesma foi
bem sucedida pelo computador receptor. Caso contrário volta a
Navegando na Internet com um Browser (navegador) enviar essa informação. A mesma circula pela rede em forma de
Para podermos navegar na Internet é necessário um software fragmentosdesignados por datagrams e que contém um cabeça-
navegador (browser) como o Internet Explorer, Mozilla Firefox lho. Esse cabeçalho contém informação como a porta de origem
ou Google Chrome. (Estes são uns dos mais conhecidos, embora 9 Fonte: http://faqinformatica.com/quais-sao-os-protocolos-do-
existam diversos navegadores). -tcpip/

Informática 31
Apostila Digital Licenciada para Alice Caroline Guarino dos Santos - alice.guarino@hotmail.com (Proibida a Revenda)
APOSTILAS OPÇÃO
e a porta de destino da informação, o ACK, entre outra informa- com.br onde:
ção, de modo a manter a circulação de dados estável e credível. seunome = identificação do email, geralmente usamos algo
UDP (User Datagram Protocol) – O UDP é um protocolo de relacionado a nosso nome ou empresa.
transporte de informação, mas não é tão fiável com o TCP. O UDP nomedoseuprovedor = é o serviço do correio eletrônico
não estabelece uma sessão de ligação em que os pacotes contêm escolhido (Gmail, Hotmail, Yahoo, bol, etc).
um cabecalho. Simplesmente faz a ligação e envia os dados, o Caixa de entrada
que o torna mais rápido mas menos eficiente. A caixa de entrada é o diretório onde encontramos todos
ARP (Address Resolution Protocol) é o ARP estabelece uma li- os e-mails recebidos, através da caixa de entrada podemos
gação entre o endereço físico da placa de rede e o endereço de IP. visualizar quem enviou o e-mail e qual o seu conteúdo seja ele
A placa de rede de um PC contém uma tabela onde faz a ligação um texto ou um arquivo qualquer.
entre os endereços físicos e lógicos dos computadores presentes Escrever e-mail
na rede. Quando um PC quer comunicar com outro, vai verificar Clicando no botão “escrever e-mail” podemos enviar um
nessa tabela se o computador está presente na rede. Se estiver, e-mail (mensagem) a qualquer pessoa que também possua uma
envia os dados e o tráfego na rede é dminuído, caso contrário conta de e-mail seja essa conta do Gmail, Hotmail, Yahoo, Bol
envia um sinal designado por pedido ARP para determinar o seu entre outras.
endereço. Anexando um arquivo
IP (Internet Protocol) – É responsável por estabelecer o con- Podemos anexar qualquer tipo de arquivo a mensagem
tacto entre os computadores emissores e receptores de maneira enviada, desde que não ultrapasse o tamanho permitido.
a qua a informação não se perca na rede. Juntamente com o TCP Contatos
é o protocolo mais importante de todos este conjunto. É onde você pode visualizar e também adicionar novos
ICMP (Internet Control Message Protocol) – O ICMP trabalha contatos de e-mail seja um conhecido um amigo ou até mesmo
em conjunto com o IP e serve para enviar mensagens para res- um contato de negócio.
ponder a pacotes de informação que não foram entregues cor-
rectamente. Desta forma é enviada uma mensagem ICMP e volta Serviço de correio eletrônico
a ser enviado o pacote de informação não recebido. Correio eletrônico, ou simplesmente email, é um dos
IGMP (Internet Group Management Protocol) – Este proto- serviços da Internet mais conhecidos e amplamente utilizados.
colo é responsável pela gestão de informação que circula pela Hoje em dia é muito comum que uma pessoa possua um email
Internet e Intranet através do protocolo TCP/IP. para contatos pessoais e profissionais. Uma das principais
vantagens do surgimento do serviço de mensagem eletrônica
Portais/Sites é a possibilidade de enviar mensagens a quem você desejar,
Uma das melhores maneiras de se “ambientar” na Internet sem pagar nada pelo serviço. Por exemplo, é possível trocar
é através de sites chamados de Portais. A definição de Portal mensagens com professores de outras instituições de ensino,
surgiu pelo fato de estes sites possuírem informações variadas a quilômetros de distância, enviar mensagens aos amigos
que permitem ao internauta procurar e estar por dentro de distantes e resolver pendências profissionais, tudo via correio
novidades já que os portais oferecem uma grande quantidade eletrônico.
de noticias e são atualizados com freqüência.
Exemplo de alguns dos portais mais conhecidos no Brasil: Redes Sociais
www.uol.com.br As redes sociais são relações entre os indivíduos na
www.globo.com comunicação por meio de computadores e da internet. O que
www.terra.com.br também pode ser chamado de interação social, cujo objetivo é
www.ig.com.br buscar conectar pessoas e proporcionar a comunicação entre
elas criando grupos com o objetivo de se relacionar virtualmente
Mecanismos de busca na internet através das redes sociais.
Há mais informações na Web do que se possa imaginar. O Existem varias redes sociais, algumas das mais populares
segredo é encontrar exatamente o que se quer. hoje em dia são:
Orkut http://www.orkut.com
O que são mecanismos de busca? Facebook http://www.facebook.com
Mecanismos de Busca são sites de informações sobre as Twitter http://twitter.com
páginas da internet e podemos utilizar esses mecanismos MySpace http://br.myspace.com
para encontrar palavras, textos, sites, diretórios, servidores de Para fazer parte de umas dessa redes sociais basta se
arquivos, etc. Com essas ferramentas, encontrar informações na cadastrar criando uma conta no site da rede e se interagir.
Internet torna-se uma tarefa bem simples.
Mas como posso encontrar o que eu quero? Grupos, fórum, chat e wiki10
Utilizando algumas ferramentas de pesquisa disponíveis na Agora iremos conhecer a criação de grupos e os três recursos
internet podemos associar o que procuramos com informações do moodle que potencializam a discussão e a construção do
disponíveis na rede mundial (internet) fazendo uma espécie de conhecimento, e que podem ser explorados no ambiente de
filtro de informações. experimentação. São eles:
Alguns dos mecanismos de busca atuais mais populares: Grupos
Yahoo http://cade.search.yahoo.com/ É possível desenvolver trabalhos separando os alunos por
Uol http://busca.uol.com.br/ diferentes grupos e, se necessário, permitir o acompanhamento e
IG http://busca.igbusca.com.br/ as orientações do tutor aos seus respectivos grupos, viabilizando
Google http://www.google.com.br/ uma maior interatividade e proximidade entre as pessoas. A
opção de criação de grupos no Moodle possibilita a organização
Correio Eletrônico dos cursistas em pequenos grupos para o desenvolvimento de
O correio eletrônico é uma das maravilhas da internet, com atividades no curso, ou mesmo para a divisão dos grupos por
ele podemos enviar e receber documentos. Seu crescimento tutor.
repentino na internet se deve a velocidade de se enviar e Para criar grupos dentro de um curso, basta clicar em
receber textos, imagens e qualquer tipo de documento de um “Grupos”, no bloco “Administração” e você será redirecionado
computador para outro independentemente do local onde se para uma tela que contém dois quadros, e os botões necessários
encontram os computadores. para você editar esses grupos.
E-mail Para adicionar um grupo, digite o nome do grupo e clique em
Para que possamos fazer o uso do correio eletrônico é “Criar grupo”. Após isso, o novo grupo já aparecerá na relação do
necessário um endereço na rede mundial denominado endereço quadro Grupos.
de e-mail. 10 Fonte: http://www.moodle.ufba.br/mod/resource/view.
A estrutura de um e-mail é seunome@nomedoseuprovedor. php?id=33426

Informática 32
Apostila Digital Licenciada para Alice Caroline Guarino dos Santos - alice.guarino@hotmail.com (Proibida a Revenda)
APOSTILAS OPÇÃO
O primeiro quadro mostra todos os grupos existentes no Armazenamento das discussões para posterior leitura dos
curso. alunos que não participaram da seção;
Selecionando em adicionar ou remover pessoas, abrirá Dinâmica colaborativa onde todos podem contribuir com a
uma tela com duas colunas, uma coluna informa os nomes discussão em tempo real.
das pessoas que já fazem parte do grupo e outra os potenciais Para criar um chat basta clicar na opção ‘acrescentar
membros. Entre as duas colunas há setas - uma para direita e atividade’, no tópico onde desejamos acrescentar o recurso,
outra para a esquerda - que possibilitarão adicionar ou remover selecionar ‘chat’ e configurar conforme as necessidades do curso,
membros. Selecionando um aluno de uma das colunas e clicando colocando nome, data e uma descrição objetiva. Lembramos que
na seta desejada, o aluno será adicionado ou removido do grupo. o botão ‘Ativar Edição’ deve estar acionado.
O Fórum - O fórum é uma interface assíncrona, que possibilita
a interação e discussão entre os participantes do curso sobre WIKI
determinado assunto. As mensagens são estruturadas de forma Um recurso assíncrono colaborativo que possibilita a
hierárquica, apresentando os assuntos em destaque. Apesar construção coletiva de diferentes tipos de textos, por vários
dessa hierarquia, o fórum traz o potencial do meio digital, por autores. A Wiki do Moodle permite que os participantes de
permitir dinâmicas hipertextuais e agregação de várias mídias um curso trabalhem juntos, acrescentando ou alterando seu
conteúdo. As versões anteriores não são excluídas, podendo ser
Tipos de Fórum recuperadas. Numa Wiki pode-se inserir novas páginas ou novos
Esse recurso oferece a opção de configurá-lo de acordo com hiperlinks. Trazemos um exemplo desse recurso na página
as necessidades de cada professor na elaboração de seu curso; principal.
dessa forma é possível escolher entre os seguintes tipos de Possibilidades:
fórum no Moodle: Desenvolvimento de projetos, concepção de livros,
Cada usuário inicia apenas UM NOVO tópico - Essa opção Trabalhos em grupos, produção de qualquer tipo de texto
permite que cada participante possa abrir apenas um novo colaborativo.
tópico, no entanto, todos podem responder livremente, sem Para criar um wiki basta clicar na opção ‘acrescentar
limites de quantidade; atividade’, no tópico onde desejamos acrescentar o recurso e
Fórum Geral - Permite que os participantes do curso possam selecionar ‘wiki’. Lembramos que o botão ‘Ativar Edição’ deve
inserir tantos tópicos quantos desejarem; estar acionado.
Fórum P e R (Pergunta e Resposta) - Permite ao professor
elaborar questionamentos no fórum para discussão. Porém, TRANSFERÊNCIA DE ARQUIVOS11
o aluno somente consegue visualizar as respostas dos outros Para transferir dados deve existir uma conexão de dados
participantes a partir do momento que este posta a sua própria entre portas apropriadas e deve ser feita uma escolha de
resposta; parâmetros de transferência. Os processos Cliente-DTP e
Fórum de uma única discussão - Com esse fórum, o tópico Servidor-DTP possuem portas com valores default que devem
aparece em uma única página, este tipo de fórum é recomendado ser suportadas por todas as versões de FTP. Entretanto, o cliente
para organizar discussões com foco em um tema único e preciso. pode alterar o valor de tais portas.
Como inserir um fórum no curso Logo que inicia a transferência de dados, o gerenciamento da
Para a criação de um novo fórum no ambiente Moodle, basta conexão de transferência de dados passa a ser responsabilidade
clicar em ‘Adicionar atividade’, selecionar ‘fórum’ e configurar do servidor; salvo uma transferência sem erros e em que os
conforme as necessidades do curso. dados estão indo do cliente para o servidor. Nesse caso, em vez
O fórum do Moodle permite também a configuração de enviar um End of File, torna-se responsabilidade do cliente
de ações a serem executadas pelos participantes do curso. fechar a conexão para indicar o fim de arquivo.
Outra particularidade do Moodle é o recebimento ou não das Acrescentando às definiçoes existentes do FTP, pode-se
mensagens postadas no fórum via e-mail pessoal. Cabe a cada definir - também, o modo de transferência dos arquivos, de
equipe de trabalho definir se os alunos serão ou não assinantes. forma a otimizar e melhorar a transferência dos dados. O modo
A definição de tipos de grupo é uma outra ação importante de transmissão pode ser por fluxo contínuo, modo blocado e
possibilitada pelo Moodle, pois permite escolher de que maneira modo comprimido.
os usuários utilizam o fórum. São basicamente três formatos: O FTP não se preocupa com a perda ou a adulteração de bits
Nenhum grupo: não há separação em grupos; durante a transferência, pois é atribuição do TCP - protocolo do
Grupos separados: membros de grupos iguais interagem nível de transporte, mas provê mecanismos para um eventual
entre si mas não com membros de outros grupos; reinício da transferência quando ela for interrompida por
Grupos visíveis: eles não interagem mas podem ver as problemas externos ao sistema (como uma falha na alimentação
mensagens de outros grupos. elétrica).
Como inserir um novo tópico de Discussão Este procedimento de reinício só está disponível nos modos
No fórum, é possível que tanto os professores quanto os de transferência que permitem inserir controles no meio do
alunos possam adicionar tópicos para as discussões. Para criar fluxo de dados (modo de transferência blocado e comprimido).
um novo tópico no Moodle, basta clicar em “acrescentar um
novo tópico”. Em seguida você poderá adicionar uma mensagem Questões
relativa ao tema do fórum e para finalizar, clique em “enviar
mensagem para o fórum”. De imediato surge o registro da 01. (CEP 28 - Assistente Administrativo – IBFC/2015).
intervenção efetuada. A Intranet possui características técnicas próprias que a
diferenciam quanto a Internet. Uma dessas características
Chat técnicas que a distingue é o fato da Intranet ser:
Também conhecido como bate–papo, traz como principal (A) desenvolvida com base no protocolo TCP/IP.
característica a comunicação síncrona, ou seja, a possibilidade de (B) a única que possui Grupos de Discussão.
podermos interagir no mesmo momento, enviando e recebendo (C) a única que possui banda larga.
mensagens de forma imediata. Uma opção interessante do chat (D) privada e não pública
do Moodle é a de separarmos, ou não, por grupo os participantes,
e de podermos ‘salvar as sessões encerradas’. Quando ativamos 02. (UEG - Assistente de Gestão Administrativa – Geral
essa função, o moodle automaticamente registra a conversa e é – FUNIVERSA/2015). Assinale a alternativa em que são
possível disponibilizá-la para todos os participantes do curso. apresentados apenas nomes de sítios de busca e pesquisa na
Trazemos um exemplo de chat na página principal. Internet.
Possibilidades: (A) Linux e Facebook
Interação, por proporcionar o esclarecimento de dúvidas, (B) Google e Gmail
discussões e criação de vínculos; (C) Internet Explorer e Mozilla Firefox
Definição de tópicos para a discussão; 11 Fonte: http://penta.ufrgs.br/rc952/trab1/ftp3.html

Informática 33
Apostila Digital Licenciada para Alice Caroline Guarino dos Santos - alice.guarino@hotmail.com (Proibida a Revenda)
APOSTILAS OPÇÃO
(D) BrOffice e Bing minha cabeça dói, diga dor de cabeça, já que é o termo que um
(E) Google e Yahoo! site de medicina usaria.

03. (SSP-AM - Assistente Operacional – FGV/2015). A Quanto menos, melhor


Wikipedia, um famoso site da Internet, fornece o endereço: Um ou dois termos simples de pesquisa retornarão
https://secure.wikimedia.org/wikipedia/pt/wi ki/Página_ resultados mais abrangentes. Comece com termos de pesquisa
principal curtos e, em seguida, refine seus resultados incluindo mais
para acessar e editar o conteúdo dos sites. O uso do prefixo palavras.
“https:” significa que a comunicação com o site é feita de forma:
(A) anônima; Pesquise uma frase exata
(B) segura; Coloque palavras entre aspas “[qualquer palavra]” para
(C) compactada; pesquisar por uma frase exata e na ordem exata. Lembre-se de
(D) prioritária; que as pesquisas com aspas podem excluir resultados relevantes.
(E) somente leitura. Por exemplo, uma pesquisa por “Alexander Bell” excluirá páginas
que se referem a Alexander G. Bell.
04. (MPCM - Técnico em Informática – Desenvolvimento
– CETAP/2015). A velocidade de transmissão dos atuais links Use palavras descritivas
de acesso a Internet em banda larga e usualmente medida em: Quanto mais específica for a palavra, maior será a chance
(A) Mbits/s; que significa “Megabits porsegundo” . de obter resultados relevantes. Dessa forma [toques de
(B) MBytes/s; que significa “Megabytes porsegundo” . celebridades] é melhor do que [sons de celebridades]. Porém,
(C) Mbits/s; que significa “Megabytes por segundo” lembre-se de que se a palavra tiver o significado correto, mas
(D) MBytes/s; que significa “Megabits porsegundo”. não for a mais usada pelas pessoas, ela poderá não corresponder
(E) GBytes; que significa “Gigabytes por segundo” . às páginas que você busca.

05. (DPE-MT - Assistente Administrativo – FGV/2015). Não se preocupe com maiúsculas e minúsculas
A ferramenta da Internet que consiste em criar uma abstração A pesquisa não diferencia maiúsculas e minúsculas. Uma
do terminal, permitindo ao usuário a criação de uma conexão pesquisa por new york times apresenta os mesmos resultados
com o computador remoto sem conhecer as suas características, que New York Times.
possibilitando o envio de comandos e instruções de maneira
interativa, é denominada Pesquise em um site específico
(A) Telecommunications Networks. Antes de sua consulta, digite site: se desejar que sua resposta
(B) File Transfer Protocol. venha de um site ou tipo de site específico (.org, .edu). Por
(C) Trivial File Transfer. exemplo: site:edu ou site:nytimes.com.
(D) Domain Name System.
(E) Dynamic Host Configuration. Não se preocupe com pontuação
A pesquisa ignora pontuação. Isso inclui @#%^*()=[]\ e
Respostas outros caracteres especiais.
01. D\02. E\03. B\04. A\05. A
Pesquise por tipo de arquivo
Busca e pesquisa no Google12 Pesquise por tipos específicos de arquivos, como PDF, PPT,
O Google é, atualmente, o melhor site de pesquisa. Pesquisa ou XLS, adicionando filetype: e a abreviação de três letras.
por palavra ou categoria. Tem versões em várias línguas. Você
pode ter a Barra de Ferramentas do Google na seu próprio Inclua ou ignore palavras e caracteres em sua pesquisa
navegador e aumentar sua habilidade de encontrar informações Destaque palavras e caracteres comuns como o e e se eles
de qualquer lugar na rede e a instalação leva apenas alguns forem essenciais para a sua pesquisa (como no título de um
segundos. A barra bloqueia os “pop-ups” (páginas que filme ou livro) colocando-os entre aspas: “o”. Você também pode
vão aparecendo com propagandas). Quando instalada, ela usar o sinal de menos “-” para especificar itens específicos que
automaticamente aparece junto à barra de ferramentas do você não deseja ver nos resultados, como ingredientes em uma
Internet Explorer e do Firefox. Isto significa que você pode, receita.
rápida e facilmente, usá-la para efetuar buscas de qualquer
website, sem retornar à página do Google.13 Encontre páginas relacionadas
Use o operador related: para encontrar páginas com
Comece de forma simples conteúdo similar. Digite “related:” seguido pelo endereço do site.
Não importa o que você procura, nosso lema é “mantenha Por exemplo, se você encontrar um site de que gostou, tente usar
a simplicidade”. Comece inserindo um nome ou uma palavra. related:[Insira o URL] para localizar sites similares.
Se estiver procurando por um local ou produto em um lugar
específico, insira o nome junto com a cidade ou o CEP. Pesquise por números em uma faixa
Fique dentro do seu orçamento pesquisando apenas itens
em uma faixa de números colocando “..” entre os valores.

Faça conversões numéricas


Converta qualquer medida (por exemplo, milhas para
Ignore a ortografia quilômetros ou onças para litros) digitando o número e a
O corretor ortográfico do Google padroniza automaticamente unidade de medida.
para a escrita mais comum de uma palavra, mesmo que você não
tenha digitado corretamente. Veja que horas são
Para ver a hora certa em qualquer lugar do mundo, pesquise
Use palavras comuns na web por horário e a cidade ou país.
Um mecanismo de pesquisa funciona relacionando as
palavras inseridas com páginas da web. Ou seja, usar as palavras Faça conversões monetárias
que tem mais probabilidade de aparecer naquelas páginas Veja as taxas de conversão monetária pesquisando por
oferecerá os melhores resultados. Por exemplo, em vez de dizer [moeda 1] em [moeda 2].
12 Fonte: http://www.google.com/intl/pt-BR_ALL/
insidesearch/tipstricks/all.html Pesquise na web em todos os idiomas
13 Fonte: http://www.gregoriano.org.br/portinha/017.htm Quando estiver pesquisando, clique em “Mais ferramentas

Informática 34
Apostila Digital Licenciada para Alice Caroline Guarino dos Santos - alice.guarino@hotmail.com (Proibida a Revenda)
APOSTILAS OPÇÃO
de pesquisa” no painel à esquerda da página de resultados e, a Termos semelhantes
seguir, selecione “Páginas estrangeiras traduzidas”. Esse recurso Obtenha resultados que incluam sinônimos colocando o
escolherá o melhor idioma para pesquisar e fornecer resultados sinal “~” em frente ao termo de pesquisa. Uma pesquisa sobre
traduzidos. [receitas ~sobremesas Natal], por exemplo, retornará resultados
para sobremesas, além de doces, biscoitos e outras guloseimas.
Verifique o clima
Pesquise por meteorologia seguido por um CEP dos EUA ou Pesquise por voz
o nome de qualquer cidade no mundo para obter a meteorologia Fale para pesquisar. Toque no botão de microfone na caixa
atual e a previsão. Insira apenas meteorologia para obter a de pesquisa do Google, ou pressione por alguns segundos o
meteorologia do seu local atual. botão de pesquisa do telefone para ativar a tela “Fale agora”. A
Pesquisa por voz para Android suporta Voice Actions no Android
Verifique cotações da bolsa em tempo real 2.2 (Froyo) e superior.
Ao digitar o símbolo de uma ação na caixa de pesquisa,
você obterá cotações da ação atualizadas em tempo real nos Examine dados públicos
resultados de pesquisa. Clique no link da página de resultados Pesquise termos demográficos como população ou taxa de
de pesquisa para obter uma análise detalhada do mercado do desemprego, seguido por uma cidade, estado ou país e você
Google Finance. receberá dados instantâneos sobre o local escolhido diretamente
do U.S. Census Bureau e do Bureau of Labor Statistics. A partir
Verifique resultados e programações esportivas daí, você poderá clicar para comparar taxas em locais diferentes.
Saiba os resultados e o horário dos jogos do seu time favorito,
pesquisando pelo time ou liga na caixa de pesquisa. Obtenha informações financeiras interativas
Para obter resultados financeiros interativos, vá para google.
Calcule qualquer coisa com no iPhone ou celular com Android (2.1 ou superior) e
Insira qualquer equação matemática na caixa de pesquisa e pesquise pelo símbolo de uma ação. Você verá um gráfico
sua resposta será calculada. interativo em um cartão e poderá alternar as visualizações
conforme períodos diferentes tocando nos botões abaixo do
Localize a atividade de terremotos gráfico.
Digite terremoto na caixa de pesquisa e os resultados
apresentarão informações do US Geological Survey mostrando o Pesquise por local
horário, o local e a magnitude de terremotos recentes. Se você deseja encontrar sushi perto de você, vá para
Google.com no seu smartphone e digite “sushi”. Se tiver optado
Pesquise por local por compartilhar seu local com o Google, você verá resultados
Adicione um CEP ao final da pesquisa de local, como costelas de restaurantes próximos ao seu local atual. Se você desejar
de porco e obtenha resultados que mostram as churrascarias pesquisar em outro local, especifique uma localização na
mais próximas, junto com números de telefone, um mapa e até consulta, como “pizza Kansas City”.
comentários. Se você não incluir seu CEP, poderemos sugerir
locais próximos a você. Interaja com a meteorologia
Para ver um snippet novo de resultados de pesquisa
Pesquise locais por CEP e códigos de área sobre meteorologia que permite que você se envolva com os
Deseja saber onde se localiza um CEP ou um código de área? resultados, acesse google.com no iPhone ou em um aparelho
Digite-o na caixa de pesquisa. Android e pesquise por “meteorologia”.

Compre e compare Como pesquisar no Google14


Use o link “Compras” no painel à esquerda na página de Dica 1: comece com o básico
resultados para procurar produtos e comparar preços na web. Não importa o que você esteja procurando, faça uma
pesquisa simples, por exemplo, onde fica o aeroporto mais
Encontre empresas locais próximo? Inclua palavras descritivas, se necessário.
Para encontrar uma loja, um restaurante ou outra empresa Se você estiver procurando um lugar ou produto em um local
em uma área específica, insira o tipo de negócio e o local e específico, inclua o local, por exemplo, padaria Campinas.
apresentaremos uma lista de locais próximos, junto com um Dica 2: pesquise usando sua voz
mapa, análises e informações de contato. Se você não incluir o Se não quiser digitar, diga “Ok Google” ou escolha o ícone de
local em sua pesquisa, encontraremos locais próximos a você. microfone para pesquisar usando sua voz. Saiba mais sobre “Ok
Google” e a pesquisa por voz.
Leia livros de domínio público Dica 3: escolha as palavras com cuidado
Leia gratuitamente os textos completos de obras de domínio Quando pensar nas palavras para sua pesquisa, escolha as
público como Moby Dick. Basta selecionar “Livros” no painel que tenham mais chances de aparecer no site que você está
esquerdo dos resultados de pesquisa. procurando. Por exemplo, não diga minha cabeça dói, e sim
dor de cabeça, pois é a palavra usada em sites com informações
Delimite uma faixa médicas.
Para especificar uma determinada faixa de números, digite Dica 4: não se preocupe com pequenos detalhes
“..”, um espaço e os números dentro daquela faixa. Por exemplo, Ortografia: o corretor ortográfico do Google usa
se estiver procurando por carros com mais de 300 cavalos de automaticamente a grafia mais comum de uma palavra, mesmo
potência, pesquise por carros “300.. cavalos de potência”. Veja que não tenha digitado corretamente.
outros exemplos: “220.. V” ou “1.. RPM” ou bateria “8000.. mAh” Letras maiúsculas: pesquisar por Folha de São Paulo é o
mesmo que digitar folha de são paulo.
Veja notícias do mundo todo Dica 5: encontre respostas rápidas
Pesquise por tópicos e clique na guia “Notícias” no painel Em muitas pesquisas, o Google mostra uma resposta à sua
esquerdo para obter resultados de notícias de fontes de todo o pergunta nos resultados da pesquisa. Alguns recursos, como
mundo. informações sobre equipes esportivas, não estão disponíveis em
todas as regiões.
Pesquise por um endereço - Tempo: pesquise clima para ver o clima de onde você está
Para mapear qualquer localidade, digite o nome ou CEP da ou inclua o nome de uma cidade, como clima Campinas, para
cidade seguido pela palavra “mapa” e você verá o mapa do lugar. saber o clima de um lugar específico.
Clique nele para vê-lo no Google Maps. 14 Fonte: https://support.google.com/websearch/
answer/134479?hl=pt-BR&ref_topic=3081620

Informática 35
Apostila Digital Licenciada para Alice Caroline Guarino dos Santos - alice.guarino@hotmail.com (Proibida a Revenda)
APOSTILAS OPÇÃO
- Dicionário: coloque definição de na frente de qualquer Oprdr Como usar
palavra para ver seu significado.
- Cálculos: digite uma equação matemática como 3*9123 ou site: Consiga resultados a partir de
resolva equações gráficas complexas. determinados sites ou domínios.
- Conversões de unidades: digite uma conversão, como 3 Exemplos: olimpíadas site:nbc.com e
dólares em euros. related: Encontre sites semelhantes a um
- Esportes: procure o nome do seu time para ver a endereço da Web que você já conhece.
programação, os resultados de jogos e muito mais. Exemplo: related:time.com
- Fatos rápidos: pesquise o nome de celebridades, locais, OU Encontre páginas que podem
filmes ou músicas para encontrar informações relacionadas. usar uma das várias palavras.
Exemplo: maratona OU corrida
Operadores de pesquisa15
info: Receba informações sobre um endereço da Web,
É possível usar operadores de pesquisa e outra pontuação
incluindo a versão em cache da página, páginas
para ver resultados mais específicos. Com exceção dos exemplos
semelhantes e páginas vinculadas ao site.
abaixo, a Pesquisa Google geralmente ignora pontuação.
Exemplo: info:google.com.br
Pontuação e símbolos cache: Veja como estava a página na última
É possível usar os sinais de pontuação abaixo ao pesquisar. vez que o Google visitou o site.
No entanto, incluí-los nem sempre melhora os resultados. Se não Exemplo: cache:washington.edu
acharmos que a pontuação dará resultados melhores, poderão
ser exibidos resultados sugeridos para aquela pesquisa sem a Observação: ao fazer uma pesquisa usando operadores ou
pontuação. sinais de pontuação, não adicione espaços entre o operador e
os termos de pesquisa. Uma pesquisa por site:nytimes.com
funcionará, mas por site: nytimes.com não.
Símb. Como usar
+ Pesquise por páginas do Encontrar imagens sem restrições de uso16
Google+ ou tipos sanguíneos Quando você faz uma Pesquisa Google, pode filtrar os
Exemplos: +Chrome ou AB+ resultados para encontrar imagens, vídeos ou texto que você
@ Encontre tags sociais tenha permissão de usar. Para fazer isso, é necessário usar um
Exemplo: @agoogler filtro da Pesquisa avançada chamado “direitos de uso”, que
permite saber quando usar, compartilhar ou modificar algo que
$ Encontre preços você encontra on-line.
Exemplo: nikon R$400
# Encontre os tópicos mais Encontrar imagens, textos e vídeos que possam ser reutilizados
comuns marcados por hashtags - Acesse a Pesquisa de imagens avançada para imagens ou
Exemplo: #desafiodogelo Pesquisa avançada para todos os outros conteúdos.
- Quando você usa um traço antes de uma - Na caixa “todas estas palavras”, digite o que você deseja
palavra ou site, ele exclui os resultados que pesquisar.
incluem essa palavra ou site. Isso é útil para - Na seção “Direitos de uso”, use a lista suspensa para
palavras com vários significados, como escolher o tipo de licença que você deseja que o conteúdo tenha.
Jaguar, a marca do carro, e jaguar, o animal. - Selecione Pesquisa avançada.
Exemplos: velocidade do jaguar
-carro ou pandas -site:wikipedia.org Observação: antes de reutilizar o conteúdo, verifique se sua
licença é legítima e confira os termos exatos de reutilização. Por
“ Quando você coloca uma palavra ou frase
exemplo, a licença pode exigir que você dê crédito ao criador da
entre aspas, os resultados incluem apenas
imagem ao usar a imagem. O Google não pode dizer se a licença
páginas com as mesmas palavras e na mesma
é legítima. Portanto, não sabemos se o conteúdo foi legalmente
ordem do que está dentro das aspas. Use isso
licenciado.
apenas se você estiver procurando por uma
palavra ou frase exata. Caso contrário, você
Tipos de direito de uso
excluirá muitos resultados úteis por engano.
- Sem restrições de uso ou compartilhamento: permite que
Exemplo: “imagine all the people”
você copie ou redistribua o conteúdo, desde que esse conteúdo
* Adicione um asterisco como um permaneça inalterado.
marcador para termos desconhecidos ou - Sem restrições de uso, compartilhamento ou modificação:
permite que você copie, modifique ou redistribua o conteúdo
caracteres curinga. conforme especificado pela licença.
Exemplo: “melhor um * na mão do - Comercialmente: se você procura conteúdo para uso
comercial, não se esqueça de selecionar uma opção que inclui a
que dois *” palavra “comercialmente”.
.. Separe os números por dois pontos sem
Como funcionam os direitos de uso
espaços para ver resultados dentro de Os direitos de uso ajudam a encontrar conteúdo que possa
um intervalo. ser usado além do uso justo. Proprietários de sites podem usar
as licenças para informar se e como o conteúdo dos sites deles
Exemplo: câmera R$50..R$100 pode ser reutilizado.
O filtro de direitos de uso da Pesquisa avançada mostra
Operadores de pesquisa conteúdo identificado com uma licença da Creative Commons ou
Operadores de pesquisa são palavras que podem ser similar, ou como sendo de domínio público. No caso de imagens,
adicionadas às pesquisas para ajudar a restringir os resultados. o filtro de direitos de uso também mostra imagens identificadas
Não se preocupe em memorizar cada operador, pois você com a licença de Documentação livre do GNU.
também pode usar a página Pesquisa avançada para criar essas
pesquisas. Informar direitos de uso incorretos
Caso encontre conteúdo com direitos de uso incorretos nos
15 Fonte: https://support.google.com/websearch/ 16 Fonte: https://support.google.com/websearch/
answer/2466433?hl=pt-BR&ref_topic=3081620 answer/29508?hl=pt-BR&ref_topic=3081620

Informática 36
Apostila Digital Licenciada para Alice Caroline Guarino dos Santos - alice.guarino@hotmail.com (Proibida a Revenda)
APOSTILAS OPÇÃO
resultados de pesquisa, informe-nos por meio do Fórum da - Particular: se você está conectado à sua Conta do Google,
Pesquisa Google. pode ver o conteúdo que foi compartilhado com você no Google+
ou no Gmail.
Filtrar seus resultados de pesquisa17 - Local próximo: veja resultados para seu local atual.
É possível filtrar e personalizar os resultados de pesquisa - Vídeos: filtre vídeos por duração, qualidade e fonte, por
para encontrar exatamente o que você deseja. Por exemplo, é exemplo youtube.com.br.
possível encontrar sites atualizados nas últimas 24 horas ou - Receitas: filtre por ingredientes, tempo de preparo e
fotos de uma determinada cor. quantidade de calorias.
- Aplicativos: escolha o preço e o sistema operacional para os
Observação: os filtros de resultados de pesquisa não aplicativos disponíveis.
alteram nenhuma das configurações de pesquisa. Para alterar - Patentes: selecione a data do depósito do pedido ou
configurações, como SafeSearch, resultados por página e publicação de uma patente, o escritório de patentes onde
idiomas, ou excluir pesquisas anteriores, visite a página foi depositado o pedido, o status do pedido e o tipo. Também
Configurações de pesquisa. é possível fazer uma pesquisa de patentes diretamente em
patents.google.com.br.
Adicionar ou remover filtros
Computador Ferramentas de busca que podem ser usadas para resultados
- Faça uma pesquisa no Google. com imagens
- Escolha o tipo de resultado que você deseja ver, por exemplo - Tamanho: escolha entre grande, médio ou ícone ou
Imagens ou Notícias, abaixo da caixa de pesquisa. Clique em configure as dimensões exatas.
Mais para ver outras opções. - Cor: encontre imagens para uma cor específica certo, preto
- Abaixo da caixa de pesquisa, clique em Ferramentas de e branco ou transparente.
pesquisa para ver mais filtros que podem ser aplicados à sua - Tipo: veja somente imagens de rostos, fotos, clip art, linhas
pesquisa. As ferramentas de pesquisa que você vê variam de de desenho ou GIFs animados.
acordo com sua pesquisa e com os filtros que você já usou. - Tempo: encontre uma foto que tenha sido publicada
Portanto, você não verá sempre as mesmas opções. recentemente ou em uma data específica.
- Para remover os filtros que você já adicionou nas - Direitos de uso: veja fotos que podem ser reutilizadas ou
ferramentas de pesquisa, clique em Limpar. modificadas. Saiba como encontrar conteúdo para reutilização.

Navegador para dispositivos móveis Operadores de pesquisa


Use estas instruções se estiver pesquisando em um Outra maneira de filtrar resultados é usar operadores de
navegador para dispositivos móveis, como o Chrome ou o Safari, pesquisa, que são palavras ou símbolos que ajudam a restringir
no seu smartphone ou tablet. seus resultados.
- Faça uma pesquisa no Google.
- Escolha o tipo de resultado que você deseja ver, por exemplo Pesquisa avançada18
Imagens ou Notícias, abaixo da caixa de pesquisa. Toque em Mais Restrinja os resultados de pesquisas complexas, usando a
para ver mais opções. página “Pesquisa avançada”. Por exemplo, é possível encontrar
- Toque em Mais > Ferramentas de pesquisa para ver mais sites atualizados nas últimas 24 horas ou imagens em preto e
filtros que você pode aplicar na pesquisa. As ferramentas de branco.
pesquisa que você vê variam de acordo com sua pesquisa e com
os filtros que você já usou. Portanto, você não verá sempre as Fazer uma pesquisa avançada
mesmas opções. 1- Acesse a página “Pesquisa avançada”.
- Para remover todos os filtros adicionados por meio da - Pesquisa avançada para websites
Ferramenta de pesquisa, toque em Limpar. - Pesquisa de imagens avançada
2- Digite os termos da pesquisa na seção “Localizar páginas
Maneiras de filtrar resultados de pesquisa com”.
- Tipo de resultado: na parte superior ou inferior de uma 3- Na seção “Em seguida, limite seus resultados por”, escolha
página de resultados de pesquisa, você vê uma série de maneiras os filtros que você deseja usar. É possível usar um ou mais filtros.
de filtrar os resultados para ver um tipo de conteúdo. Por 4- Clique em Pesquisa avançada.
exemplo, clique em “Imagens” para que todos os resultados da
pesquisa sejam de imagens ou “Compras” para ver resultados Dica: também é possível usar muitos desses filtros na caixa
da pesquisa que ajudem você a encontrar meios de comprar as de pesquisa com operadores de pesquisa.
coisas que pesquisou.
- Ferramentas de pesquisa: depois de decidir que tipo de Filtros da “Pesquisa avançada” que podem ser usados
resultado que você deseja, refine ainda mais os resultados Sites
usando “Ferramentas de pesquisa”. As ferramentas de pesquisa - Idioma
podem incluir itens como localização, cor, tamanho, bem como a - Região
data em que a página foi publicada. - Data da última atualização
- Site ou domínio
Tipos de ferramentas de pesquisa - Onde os termos de pesquisa aparecem na página
Algumas ferramentas de pesquisa não estão disponíveis em - SafeSearch
todos os idiomas ou aparecem apenas se você estiver conectado - Nível de leitura
à sua Conta do Google. - Tipo de arquivo
- Direitos de uso (encontre páginas que você tenha permissão
Ferramentas de pesquisa que podem ser usadas para para usar)
resultados da Web
As ferramentas de pesquisa variam de acordo com o tipo de Imagens
resultado que você está procurando, mas podem incluir: - Tamanho
- Data de publicação: limite os resultados de acordo com a - Proporção
data em que eles foram publicados na Web. - Cor
- Palavra por palavra: pesquise por palavras ou frases exatas. - Tipo (página de início, animada etc.)
- Dicionário: encontre definições, sinônimos, imagens e - Site ou domínio
muito mais para seu termo de pesquisa. - Tipo de arquivo
17 Fonte: https://support.google.com/websearch/ 18 Fonte: https://support.google.com/websearch/
answer/142143?hl=pt-BR&ref_topic=3081620 answer/35890?hl=pt-BR&ref_topic=3081620

Informática 37
Apostila Digital Licenciada para Alice Caroline Guarino dos Santos - alice.guarino@hotmail.com (Proibida a Revenda)
APOSTILAS OPÇÃO
- SafeSearch palavras que você pesquisou aparecem em negrito para ajudar a
-Direitos de uso (encontre imagens que você tenha permissão identificar se a página tem o que você está procurando.
para usar)
Anúncios (Se acharmos que um anúncio pode ajudar você
Página de resultados da Pesquisa Google19 a encontrar o que está procurando, ele será exibido na parte
superior ou no lado direito da página de resultados. Você saberá
Use esta página para saber mais sobre as diferentes partes que é um anúncio e não um resultado de pesquisa, por causa do
de uma página de resultados da Pesquisa Google, incluindo o ícone amarelo de anúncio ao lado do URL.)
significado de diferentes ícones e botões. Clique em um dos links
abaixo para saber mais sobre a parte correspondente da página [4] Parte inferior da página
de resultados. Local (O local atual que o Google tem para você é exibido
juntamente com uma opção para atualizar seu local ou usar seu
Partes da página de resultados de pesquisa local exato.)
- Parte superior da página Enviar feedback (Se houver um erro na página de resultados
- Filtros e configurações de pesquisa ou se você desejar sugerir formas de melhorá-la, clique em
- Resultados e anúncios de pesquisa Enviar feedback.)
- Parte inferior da página
Dica: se sua página de resultados de pesquisa parece muito
pequena, é possível aumentar o texto e o tamanho da página.
- Em um Mac, mantenha a tecla “Command” e a tecla +
pressionadas ao mesmo tempo.
- Em um PC, mantenha as teclas Ctrl e + pressionadas ao
mesmo tempo.
Questões

01. (Prefeitura de Niterói – RJ - Agente Fazendário –


FGV/2015). O Google permite o emprego de expressões que
refinam o processo de busca. Analise, na figura abaixo, a primeira
parte do resultado de uma busca realizada pelo Google.

[1] Parte superior da página


Ícone de microfone (Toque no ícone de microfone para fazer
uma pesquisa usando a voz em vez de digitação.)
Nome do Google+ (Clique no seu nome para acessar sua
página do Google+.)
Aplicativos (Clique em “Aplicativos” para ter acesso rápido Nesse caso, o texto digitado na caixa de busca foi:
a outros produtos do Google, como Gmail, YouTube e Google (A) define prova.
Agenda.) (B) dicionário prova.
Sua foto (Clique na sua foto para sair da Conta do Google ou (C) ++prova.
para adicionar uma nova conta.) (D) $prova.
(E) ?prova.
[2] Filtros e configurações de pesquisa
Filtros e ferramentas de pesquisa (Filtros: clique em qualquer 02. (TRE-MT - Conhecimentos Gerais para o Cargo 6
um dos links abaixo da caixa de pesquisa para selecionar o – CESP/2015). Assinale a opção que apresenta uma forma
tipo de resultado que você deseja ver. Por exemplo, para ver adequada e específica de buscar no Google por arquivos pdf
só imagens, clique em “Imagens”. Ferramentas de pesquisa: relacionados ao BrOffice.
clique em “Ferramentas de pesquisa” para ver as formas mais (A) filetype:pdf broffice
avançadas de filtrar os resultados, por exemplo por cor, hora ou (B) related:pdf broffice
local.) (C) link:pdf broffice
Resultados particulares (Clique em para ver resultados (D) link broffice
públicos. Clique em para ver resultados particulares, como (E) type:file broffice
informações do seu Gmail ou do Google Agenda, na página de
resultados.) 03. (Prefeitura de Cuiabá – MT - Técnico em
Configurações (Clique em “Opções” para alterar qualquer Administração Escolar – FGV/2015). Nos buscadores da
uma das suas configurações, como o idioma dos seus resultados Internet, especialmente o Google, um trecho entre aspas duplas
de pesquisa, o número de resultados por página, o SafeSearch e na caixa de busca significa que devem ser incluídas no resultado
se suas pesquisas anteriores são salvas na sua Conta do Google da busca páginas que contêm
ou não.) (A) qualquer das palavras importantes contidas no trecho.
(B) todas as palavras importantes contidas no trecho, em
[3] Resultados e anúncios de pesquisa qualquer ordem.
Resultados de pesquisa (Cada resultado de pesquisa tem (C) todas as palavras importantes do trecho ou seus
três partes: sinônimos, em qualquer ordem.
- Título: a primeira linha azul de qualquer resultado de (D) todas as palavras importantes contidas no trecho, na
pesquisa é o título da página. Clique no título para acessar o site. ordem em que se encontram.
- URL: o endereço da Web do site é exibido em verde. (E) todas as palavras importantes do trecho ou seus
- Snippet: abaixo do URL fica uma descrição da página da sinônimos, na ordem em que se encontram.
Web, que pode incluir palavras que fazem parte da página. As
04. (TRE-AP - Técnico Judiciário – Administrativa –
19 Fonte: https://support.google.com/websearch/
FCC/2015). Um usuário da internet está realizando uma busca
answer/35891?hl=pt-BR&ref_topic=3081620

Informática 38
Apostila Digital Licenciada para Alice Caroline Guarino dos Santos - alice.guarino@hotmail.com (Proibida a Revenda)
APOSTILAS OPÇÃO
sobre terremotos que aconteceram entre os anos de 1970 e
1980 utilizando o site de busca Google. Para restringir e otimizar
a busca para apresentar apenas os resultados que atendem ao
desejado, ele deve inserir o seguinte texto de busca
(A) Terremoto+1970+1980.
(B) Terremoto 1970-1980.
(C) “Terremoto 1970 1980”.
(D) Terremoto-1970-1980.
(E) Terremoto 1970..1980.

05. (PC-SE - Escrivão Substituto – IBFC/2014). Na


pesquisa avançada do Google, temos como padrão, condições de
limitar os resultados de pesquisa por:
I. tipo de arquivo
II. tamanho da página
III. idioma

Estão corretos os itens:


(A) I e II
(B) II e III.
(C) I e III
(D) todos os itens

Respostas
01. A\02. A\03. D\04. E\05. C

Anotações

Informática 39
Apostila Digital Licenciada para Alice Caroline Guarino dos Santos - alice.guarino@hotmail.com (Proibida a Revenda)
APOSTILAS OPÇÃO

Informática 40
Apostila Digital Licenciada para Alice Caroline Guarino dos Santos - alice.guarino@hotmail.com (Proibida a Revenda)
CONHECIMENTOS ESPECÍFICOS

Apostila Digital Licenciada para Alice Caroline Guarino dos Santos - alice.guarino@hotmail.com (Proibida a Revenda)
Apostila Digital Licenciada para Alice Caroline Guarino dos Santos - alice.guarino@hotmail.com (Proibida a Revenda)
APOSTILAS OPÇÃO

em nossa sociedade, não se configuram como homogêneas,


mas são permeadas por diversas clivagens, tensões e
confrontos internos. Isso porque, a compreensão
teórico/metodológica da realidade, fundada no acervo
intelectual que se constituiu a partir das principais matrizes do
pensamento social e de suas expressões nos diferentes campos
do conhecimento humano, é processo que se constrói na
interlocução com o próprio movimento da sociedade.
O ponto de partida consiste, pois, da análise ainda que
1. fundamentos teórico- sumária, do processo de incorporação pela profissão:
metodológicos, ético-políticos, - de ideias e conteúdos doutrinários do pensamento social da
Igreja Católica, em seu processo de institucionalização no Brasil;
técnico-operativos do Serviço - das principais matrizes teórico metodológicas acerca do
Social; conhecimento do social na sociedade burguesa;
- Tecer algumas considerações sobre este processo é buscar
compreender diferentes posicionamentos, lógicas e estratégias
Este texto coloca em questão os fundamentos históricos e que permearam o pensamento e a ação profissional do serviço
teórico/metodológicos do Serviço Social brasileiro na social em sua trajetória e que persistem até os dias atuais com
contemporaneidade, particularizando as décadas de 80, 90 e novas articulações, expressões e redefinições.
os primeiros anos do século XXI. Algumas referências acerca
do Serviço Social latino‐americano também serão Quanto ao primeiro aspecto, é por demais conhecida a
apresentadas. Parte do pressuposto de que a profissão e o relação entre a profissão e o ideário católico na gênese do
conhecimento que a ilumina, se explicam no movimento Serviço Social brasileiro, no contexto de expansão e
histórico da sociedade. Sociedade que é produto de relações secularização do mundo capitalista. Relação que vai imprimir
sociais, de ações recíprocas dos homens entre si, no complexo à profissão caráter de apostolado fundado em uma abordagem
processo de reprodução social da vida. O mundo social é um da "questão social" como problema moral e religioso e numa
mundo de relações. intervenção que prioriza a formação da família e do indivíduo
São múltiplas as mediações que constituem o tecido de para solução dos problemas e atendimento de suas
relações sociais que envolvem esse processo de produção e necessidades materiais, morais e sociais. O contributo do
reprodução social da vida em suas expressões materiais e Serviço Social, nesse momento, incidirá sobre valores e
espirituais. Essas relações que constituem a sociabilidade comportamentos de seus "clientes" na perspectiva de sua
humana, implicam âmbitos diferenciados e uma trama que integração à sociedade, ou melhor, nas relações sociais
envolve o social, o político, o econômico, o cultural, o religioso, vigentes.
as questões de gênero, a idade, a etnia etc. Dimensões com as Os referenciais orientadores do pensamento e da ação do
quais se defronta cotidianamente o Serviço Social e em relação emergente Serviço Social tem sua fonte na Doutrina Social da
às quais se posiciona quer do ponto de vista explicativo quer Igreja, no ideário franco‐belga de ação social e no pensamento
do interventivo, considerados nesta abordagem como de São Tomás de Aquino (séc. XII): o tomismo e o neotomismo
dimensões de uma mesma totalidade. (retomada em fins do século XIX do pensamento tomista por
A análise dos principais fundamentos que configuram o Jacques Maritain na França e pelo Cardeal Mercier na Bélgica
processo através do qual a profissão busca explicar e intervir tendo em vista "aplicá‐lo" às necessidades de nosso tempo).
sobre a realidade, definindo sua direção social, constitui o É, pois, na relação com a Igreja Católica que o Serviço Social
principal objetivo deste texto. É necessário assinalar que essa brasileiro vai fundamentar a formulação de seus primeiros
análise das principais tendências históricas e teórico objetivos político/sociais orientando‐se por posicionamentos
metodológicas da profissão, sobretudo nas três últimas de cunho humanista conservador contrários aos ideários
décadas não é tarefa fácil ou simples, pois exige o liberal e marxista na busca de recuperação da hegemonia do
conhecimento do processo histórico de constituição das pensamento social da Igreja face à "questão social". Entre os
principais matrizes de conhecimento do social, do complexo postulados filosóficos tomistas que marcaram o emergente
movimento histórico da sociedade capitalista brasileira e do Serviço Social temos a noção de dignidade da pessoa humana;
processo pelo qual o Serviço Social incorpora e elabora sua perfectibilidade, sua capacidade de desenvolver
análises sobre a realidade em que se insere e explica sua potencialidades; a natural sociabilidade do homem, ser social
própria intervenção. e político; a compreensão da sociedade como união dos
Assim sendo, este texto apresenta‐se organizado em três homens para realizar o bem comum (como bem de todos) e a
partes: em uma primeira introdutória, onde são apresentados necessidade da autoridade para cuidar da justiça geral.
alguns fundamentos relativos ao processo histórico de No que se refere à Doutrina Social da Igreja merecem
constituição das principais matrizes do conhecimento e da destaque nesse contexto as encíclicas “Rerum Novarum” do
ação do Serviço Social brasileiro e em três outras, nas quais se Papa Leão XIII de 1891, que vai iniciar o magistério social da
busca uma aproximação às principais tendências históricas e Igreja no contexto de busca de restauração de seu papel social
teórico metodológicas do debate profissional nos anos 80, 90 sociedade moderna e a “Quadragésimo Anno” de Pio XI de
e 2000. Encerram o texto algumas reflexões acerca das 1931 que, comemorando 40 anos da “Rerum Novarum” vai
polêmicas atuais da profissão. tratar da questão social, apelando para a renovação moral da
sociedade e a adesão à Ação Social da Igreja.
O processo de constituição das principais matrizes do
conhecimento e da ação do Serviço Social brasileiro É necessário assinalar que esta matriz encontra‐se na
gênese da profissão em toda a América Latina, embora com
A questão inicial que se coloca é explicitar como se particularidades diversas como, por exemplo, na Argentina e
constituem e se desenvolvem no Serviço Social brasileiro as no Chile onde vai somar‐se ao racionalismo higienista. (Ideário
tendências de análise e as interpretações acerca de sua própria do movimento de médicos higienistas que exigiam a
intervenção e sobre a realidade social na qual se move. É claro intervenção ativa do Estado sobre a questão social pela criação
que estas tendências, derivadas das transformações sociais da assistência pública que deveria assumir um amplo
que vem particularizando o desenvolvimento do capitalismo programa preventivo na área sanitária, social e moral).

Conhecimentos Específicos 1
Apostila Digital Licenciada para Alice Caroline Guarino dos Santos - alice.guarino@hotmail.com (Proibida a Revenda)
APOSTILAS OPÇÃO

O conservadorismo católico que caracterizou os anos O questionamento a este referencial tem início no contexto
iniciais do Serviço Social brasileiro começa, especialmente a de mudanças econômicas, políticas, sociais e culturais que
partir dos anos 40, a ser tecnificado ao entrar em contato com expressam, nos anos 60, as novas configurações que
o Serviço Social norte-americano e suas propostas de trabalho caracterizam a expansão do capitalismo mundial, que impõem
permeados pelo caráter conservador da teoria social à América Latina um estilo de desenvolvimento excludente e
positivista. subordinado. A profissão assume as inquietações e
Efetivamente, a reorientação da profissão, para atender às insatisfações deste momento histórico e direciona seus
novas configurações do desenvolvimento capitalista, exige a questionamentos ao Serviço Social tradicional através de um
qualificação e sistematização de seu espaço sócio-ocupacional amplo movimento, de um processo de revisão global, em
tendo em vista atender às requisições de um Estado que diferentes níveis: teórico, metodológico, operativo e político.
começa a implementar políticas no campo social. Este movimento de renovação que surge no Serviço Social na
Nesse contexto, a legitimação do profissional, expressa em sociedade latino‐americana impõe aos assistentes sociais a
seu assalariamento e ocupação de um espaço na divisão sócio necessidade de construção de um novo projeto comprometido
técnica do trabalho, vai colocar o emergente Serviço Social com as demandas das classes subalternas, particularmente
brasileiro frente à matriz positivista, na perspectiva de expressas em suas mobilizações. É no bojo deste movimento,
ampliar seus referenciais técnicos para a profissão. Este de questionamentos à profissão, não homogêneos e em
processo, que vai constituir o que Iamamoto denomina de conformidade com as realidades de cada país, que a
"arranjo teórico doutrinário", caracterizado pela junção do interlocução com o marxismo vai configurar para o Serviço
discurso humanista cristão com o suporte técnico‐científico de Social latinoamericano a apropriação de outra matriz teórica:
inspiração na teoria social positivista, reitera para a profissão a teoria social de Marx. Embora esta apropriação se efetive em
o caminho do pensamento conservador (agora, pela mediação tortuoso processo.
das Ciências Sociais).
Cabe aqui uma explicação: nem o doutrinarismo, nem o É importante assinalar que é no âmbito do movimento de
conservadorismo constituem teorias sociais. A doutrina Reconceituação e em seus desdobramentos, que se definem de
caracteriza‐se por ser uma visão de mundo abrangente forma mais clara e se confrontam, diversas tendências
fundada na fé em dogmas. Constitui‐se de um conjunto de voltadas a fundamentação do exercício e dos posicionamentos
princípios e crenças que servem como suporte a um sistema teóricos do Serviço Social. Tendências que resultam de
religioso, filosófico, político, entre outros. O conservadorismo conjunturas sociais particulares dos países do Continente e
como forma de pensamento e experiência prática é resultado que levam, por exemplo, no Brasil, o movimento em seus
de um contramovimento aos avanços da modernidade, e nesse primeiros momentos, (em tempos de ditadura militar e de
sentido, suas reações são restauradoras e preservadoras, impossibilidade de contestação política) a priorizar um
particularmente da ordem capitalista. A teoria social por sua projeto tecnocrático/modernizador, do qual Araxá e
vez constitui conjunto explicativo totalizante, ontológico, e, Teresópolis são as melhores expressões.
portanto organicamente vinculado ao pensamento filosófico, Já o tronco latino americano do movimento, sobretudo no
acerca do ser social na sociedade burguesa, e a seu processo de Cone Sul, assume claramente uma perspectiva crítica de
constituição e de reprodução. A teoria reproduz contestação política e a proposta de transformação social.
conceitualmente o real, é, portanto, construção intelectual que Posição que, dificilmente poderá levar à prática frente à
proporciona explicações aproximadas da realidade e, assim explosão de governos militares ditatoriais e pela ausência de
sendo, supõe uma forma de autoconstituição, um padrão de suportes teóricos claros.
elaboração: o método. Neste sentido, cada teoria social é um Sem dúvida, as ditaduras que tiveram vigência no
método de abordar o real. O método é, pois a trajetória teórica, Continente deixaram suas marcas nas ciências sociais e na
o movimento teórico que se observa na explicação sobre o ser profissão, que depois de avançar em uma produção crítica nos
social. É o posicionamento do sujeito que investiga face ao anos 60/70 (nos países onde isso foi permitido) é obrigada a
investigado e desta forma é "questão da teoria social e não longo silêncio. Até o final da década de 70, o pensamento de
problema particular desta ou daquela 'disciplina'. autores latino‐americanos ainda orienta ao lado da iniciante
No caso do Serviço Social, um primeiro suporte produção brasileira (particularmente divulgada pelo CBCISS),
teóricometodológico necessário à qualificação técnica de sua a formação e o exercício profissional no país. Situação que, aos
prática e à sua modernização vai ser buscado na matriz poucos se vai modificando com o desenvolvimento do debate
positivista e em sua apreensão manipuladora, instrumental e e da produção intelectual do Serviço Social brasileiro e que
imediata do ser social. Este horizonte analítico aborda as resulta de desdobramentos e da explicitação das seguintes
relações sociais dos indivíduos no plano de suas vivências vertentes de análise que emergiram no bojo do Movimento de
imediatas, como fatos, como dados, que se apresentam em sua Reconceituação:
objetividade e imediaticidade. O método positivista trabalha
com as relações aparentes dos fatos, evolui dentro do já - a vertente modernizadora caracterizada pela
contido e busca a regularidade, as abstrações e as relações incorporação de abordagens funcionalistas, estruturalistas e
invariáveis. mais tarde sistêmicas (matriz positivista), voltadas a uma
É a perspectiva positivista que restringe a visão de teoria modernização conservadora e à melhoria do sistema pela
ao âmbito do verificável, da experimentação e da mediação do desenvolvimento social e do enfrentamento da
fragmentação. Não aponta para mudanças, senão dentro da marginalidade e da pobreza na perspectiva de integração da
ordem estabelecida, voltando‐se antes para ajustes e sociedade. Os recursos para alcançar estes objetivos são
conservação. Particularmente em sua orientação funcionalista, buscados na modernização tecnológica e em processos e
esta perspectiva é absorvida pelo Serviço Social, configurando relacionamentos interpessoais. Estas opções configuram um
para a profissão propostas de trabalho ajustadoras e um perfil projeto renovador tecnocrático fundado na busca da eficiência
manipulatório, voltado para o aperfeiçoamento dos e da eficácia que devem nortear a produção do conhecimento
instrumentos e técnicas para a intervenção, com as e a intervenção profissional;
metodologias de ação, com a "busca de padrões de eficiência,
sofisticação de modelos de análise, diagnóstico e - a vertente inspirada na fenomenologia, que emerge como
planejamento; enfim, uma tecnificação da ação profissional metodologia dialógica, apropriando‐se também da visão de
que é acompanhada de uma crescente burocratização das pessoa e comunidade de E. Mounier dirige‐se ao vivido humano,
atividades institucionais". aos sujeitos em suas vivências, colocando para o Serviço Social a

Conhecimentos Específicos 2
Apostila Digital Licenciada para Alice Caroline Guarino dos Santos - alice.guarino@hotmail.com (Proibida a Revenda)
APOSTILAS OPÇÃO

tarefa de "auxiliar na abertura desse sujeito existente, singular, e as atuais diretrizes curriculares); os eventos acadêmicos e
em relação aos outros, ao mundo de pessoas". Esta tendência aqueles resultantes da experiência associativa dos
que no Serviço Social brasileiro vai priorizar as concepções de profissionais, como suas Convenções, Congressos, Encontros e
pessoa, diálogo e transformação social (dos sujeitos) é analisada Seminários; está presente na regulamentação legal do
por Netto como uma forma de reatualização do exercício profissional e em seu Código de Ética. Sob sua
conservadorismo presente no pensamento inicial da profissão; influência ganha visibilidade um novo momento e uma nova
- a vertente marxista que remete a profissão à consciência qualidade no processo de recriação da profissão na busca de
de sua inserção na sociedade de classes e que no Brasil vai sua ruptura com seu histórico conservadorismo e no avanço
configurar‐se, em um primeiro momento, como uma da produção de conhecimentos, nos quais a tradição marxista
aproximação ao marxismo sem o recurso ao pensamento de aparece hegemonicamente como uma das referências básicas.
Marx. Nesta tradição o Serviço Social vai apropriar‐se a partir dos
anos 80 do pensamento de Antonio Gramsci e particularmente
Efetivamente, a apropriação da vertente marxista no de suas abordagens acerca do Estado, da sociedade civil, do
Serviço Social (brasileiro e latino‐americano) não se dá sem mundo dos valores, da ideologia, da hegemonia, da
incontáveis problemas, que aqui não abordaremos, e que se subjetividade e da cultura das classes subalternas. Vai chegar
caracterizam, quer pelas abordagens reducionistas dos a Agnes Heller e à sua problematização do cotidiano, à Georg
marxismos de manual, quer pela influência do cientificismo e Lukács e à sua ontologia do ser social fundada no trabalho, à
do formalismo metodológico (estruturalista) presente no E.P. Thompson e à sua concepção acerca das "experiências
"marxismo" althusseriano (referência a Louis Althusser, humanas", à Eric Hobsbawm um dos mais importantes
filosofo francês cuja leitura da obra de Marx vai influenciar a historiadores marxistas da contemporaneidade e a tantos
proposta marxista do Serviço Social nos anos 60/70 e outros cujos pensamentos começam a permear nossas
particularmente o Método de B.H. Um marxismo equivocado produções teóricas, nossas reflexões e posicionamentos
que recusou a via institucional e as determinações sócio ideopolíticos.
históricas da profissão. Obviamente, este processo de construção da hegemonia de
No entanto, é com este referencial, precário em um novos referenciais teóricometodológicos e interventivos, a
primeiro momento, do ponto de vista teórico, mas posicionado partir da tradição marxista, para a profissão ocorre em um
do ponto de vista sócio‐político, que a profissão questiona sua amplo debate em diferentes fóruns de natureza acadêmica
prática institucional e seus objetivos de adaptação social ao e/ou organizativa, além de permear a produção intelectual da
mesmo tempo em que se aproxima dos movimentos sociais. área. Trata‐se de um debate plural, que implica na convivência
Inicia‐se aqui a vertente comprometida com a ruptura com o e no diálogo de diferentes tendências, mas que supõe uma
Serviço Social tradicional. direção hegemônica. A questão do pluralismo, sem dúvida uma
Estas tendências, que configuram para a profissão linhas das questões do tempo presente, desde os anos 80 vem‐se
diferenciadas de fundamentação teóricometodológica constituindo objeto de polêmicas e reflexões do Serviço Social.
tenderão a acompanhar a trajetória do pensamento e da ação Temática complexa que constitui como afirma Coutinho um
profissional nos anos subsequentes ao movimento de fenômeno do mundo moderno e da visão individualista do
Reconceituação e se conservarão presentes até os anos homem. É o autor em questão que problematiza a proposta de
recentes, apesar de seus movimentos, redefinições e da hegemonia com pluralismo, no necessário diálogo e no debate
emergência de novos referenciais nesta transição de milênio. de ideias, apontando os riscos de posicionamentos ecléticos
(que conciliam o inconciliável ao apoiarem‐se em
O Serviço Social nos anos 80: as tendências históricas e pensamentos divergentes).
teórico metodológicas do debate profissional
Assim, em diferentes espaços, o conjunto de tendências
É, sobretudo com Iamamoto no início dos anos 80 que a teóricometodológicas e posições ideopolíticas se confrontam,
teoria social de Marx inicia sua efetiva interlocução com a sendo inegável a centralidade assumida pela tradição marxista
profissão. Como matriz teóricometodológica esta teoria nesse processo.
apreende o ser social a partir de mediações. Ou seja, parte da Este debate se expressa na significativa produção teórica
posição de que a natureza relacional do ser social não é do Serviço Social brasileiro, que vem gerando uma bibliografia
percebida em sua imediaticidade. "Isso porque, a estrutura de própria, e que tem na criação e expansão da pós‐graduação,
nossa sociedade, ao mesmo tempo em que põe o ser social com seus cursos de mestrado e doutorado, iniciada na década
como ser de relações, no mesmo instante e pelo mesmo de 70, um elemento impulsionador.
processo, oculta a natureza dessas relações ao observador". Ou
seja, as relações sociais são sempre mediatizadas por É importante lembrar que a pós‐graduação configura‐se,
situações, instituições etc, que ao mesmo tempo por definição, como espaço privilegiado de interlocução e
revelam/ocultam as relações sociais imediatas. Por isso nesta diálogo entre as áreas do saber e entre diversos paradigmas
matriz o ponto de partida é aceitar fatos, dados como teóricometodológicos. Neste espaço o Serviço Social brasileiro
indicadores, como sinais, mas não como fundamentos últimos vem dialogando e se apropriando do debate intelectual
do horizonte analítico. Trata‐se, portanto de um conhecimento contemporâneo no âmbito das ciências sociais do país e do
que não é manipulador e que apreende dialeticamente a exterior. Também neste espaço, o Serviço Social brasileiro
realidade em seu movimento contraditório. Movimento no desenvolveu‐se na pesquisa acerca da natureza de sua
qual e através do qual se engendram, como totalidade, as intervenção, de seus procedimentos, de sua formação, de sua
relações sociais que configuram a sociedade capitalista. história e, sobretudo acerca da realidade social, política,
É no âmbito da adoção do marxismo como referência econômica e cultural onde se insere como profissão na divisão
analítica, que se torna hegemônica no Serviço Social no país, a social e técnica do trabalho. Avançou na compreensão do
abordagem da profissão como componente da organização da Estado capitalista, das políticas sociais, dos movimentos
sociedade inserida na dinâmica das relações sociais sociais, do poder local, dos direitos sociais, da cidadania, da
participando do processo de reprodução dessas relações. democracia, do processo de trabalho, da realidade
Este referencial, a partir dos anos 80 e avançando nos anos institucional e de outros tantos temas. Enfrentou o desafio de
90, vai imprimir direção ao pensamento e à ação do Serviço repensar a assistência social colocando‐a como objeto de suas
Social no país. Vai permear as ações voltadas à formação de investigações. Obteve o respeito de seus pares no âmbito
assistentes sociais na sociedade brasileira (o currículo de 1982 interdisciplinar e alcançou visibilidade na interlocução com as

Conhecimentos Específicos 3
Apostila Digital Licenciada para Alice Caroline Guarino dos Santos - alice.guarino@hotmail.com (Proibida a Revenda)
APOSTILAS OPÇÃO

ciências sociais, apesar das dificuldades decorrentes da falta O Serviço Social nos anos 90: as tendências históricas e
de experiência em pesquisa, do fato de defrontar com teórico-metodológicas do debate profissional
restrições por se constituir em disciplina interventiva (de
"aplicação") e das dificuldades na apropriação das teorias Inicialmente, não podemos esquecer que, nos marcos da
sociais. Nesta década o serviço Social ganha espaço no CNPq reestruturação dos mecanismos de acumulação do capitalismo
como área de pesquisa. globalizado, os anos 80 e 90 foram anos adversos para as
políticas sociais e se constituíram em terreno particularmente
Cabe também assinalar que nos anos 80 começam a se fértil para o avanço da regressão neoliberal que erodiu as
colocar para o Serviço Social brasileiro demandas, em nível de bases dos sistemas de proteção social e redirecionou as
pós‐graduação, de instituições portuguesas, e latino intervenções do Estado em relação à questão social. Nestes
americanas (Argentina, Uruguai, Chile), o que vem permitindo anos, em que as políticas sociais vêm sendo objeto de um
ampliar a influência do pensamento profissional brasileiro processo de reordenamento, subordinado às políticas de
nestes países. estabilização da economia, em que a opção neoliberal na área
Também no âmbito da organização e representação social passa pelo apelo à filantropia e à solidariedade da
profissional o quadro que se observa no Serviço Social sociedade civil e por programas seletivos e focalizados de
brasileiro é de maturação. Maturação que expressa na combate à pobreza no âmbito do Estado (apesar da
passagem dos anos 80 para os anos 90 rupturas com o seu Constituição de 1988), novas questões se colocam ao Serviço
tradicional conservadorismo, embora como bem lembre o Social, quer do ponto de vista de sua intervenção, quer do
autor “essa ruptura não signifique que o conservadorismo (e ponto de vista da construção de seu corpo de conhecimentos.
com ele, o reacionarismo) foi superado no interior da categoria Assim, a profissão enfrenta o desafio de decifrar algumas
profissional". Pois, a herança conservadora e antimoderna, lógicas do capitalismo contemporâneo particularmente em
constitutiva da gênese da profissão atualiza‐se e permanece relação às mudanças no mundo do trabalho e sobre os
presente nos tempos de hoje. Essa maturidade profissional processos desestruturadores dos sistemas de proteção social
que avança no início do novo milênio, se expressa pela e da política social em geral. Lógicas que reiteram a
democratização da convivência de diferentes posicionamentos desigualdade e constroem formas despolitizadas de
teóricometodológicos e ideopolíticos desde o final da década abordagem da questão social, fora do mundo público e dos
de 1980. Maturação que ganhou visibilidade na sociedade fóruns democráticos de representação e negociação dos
brasileira, entre outros aspectos, pela intervenção dos interesses em jogo nas relações Estado / Sociedade.
assistentes sociais, através de seus organismos Efetivamente, a opção neoliberal por programas seletivos
representativos, nos processos de elaboração e e focalizados de combate à pobreza e o avanço do ideário da
implementação da Lei Orgânica da Assistência Social ‐ LOAS "sociedade solidária" que implica no deslocamento para
(dezembro de 1993). É também no âmbito da implementação sociedade das tarefas de enfrentar a pobreza e a exclusão
da LOAS, e de outras políticas sociais públicas, com os social, começa a parametrar diferentes modalidades de
processos descentralizadores que se instituem no país, no intervenção no campo social na sociedade capitalista
âmbito dessas políticas, que observa-se a diversificação das contemporânea Exemplos desta opção podem ser observados
demandas ao profissional de serviço social. em diversos países do Continente latino‐americano como no
É nesse contexto histórico, pós Constituição de 1988 que Chile e na Argentina.
os profissionais de serviço social, iniciam o processo de Inserido neste processo contraditório o Serviço Social da
ultrapassagem da condição de executores de políticas sociais, década de 90, se vê confrontado com este conjunto de
para assumir posições de planejamento e gestão dessas transformações societárias no qual é desafiado a compreender
políticas. e intervir nas novas configurações e manifestações da
"questão social", que expressam a precarização do trabalho e
A conjuntura econômica é dramática, dominada pela a penalização dos trabalhadores na sociedade capitalista
distância entre minorias abastadas e massas miseráveis. Não contemporânea.
devemos esquecer que nos anos 80 (a “década perdida” do Trata‐se de um contexto em que são apontadas
ponto de vista econômico para a CEPAL) a pobreza vai se alternativas privatistas e refilantropizadas para questões
converter em tema central na agenda social, quer por sua relacionadas à pobreza e à exclusão social. Cresce o
crescente visibilidade, pois a década deixou um aumento denominado terceiro setor, amplo conjunto de organizações e
considerável do número absoluto de pobres, quer pelas iniciativas privadas, não lucrativas, sem clara definição,
pressões de democratização que caracterizaram a transição. A criadas e mantidas com o apoio do voluntariado e que
situação de endividamento (que cresce 61% nos anos 80), a desenvolvem suas ações no campo social, no âmbito de um
presença dos organismos de Washington (FMI, BANCO vastíssimo conjunto de questões, em espaços de
MUNDIAL), o consenso de Washington, as reformas desestruturação (não de eliminação) das políticas sociais, e de
neoliberais e a redução da autonomia nacional, a adoção de implementação de novas estratégias programáticas como, por
medidas econômicas e o ajuste fiscal vão se expressar no exemplo, os programas de Transferência de Renda, em suas
crescimento dos índices de pobreza e indigência. É sempre diferentes modalidades.
oportuno lembrar que, nos anos 80 e 90 a somatória de Nessa conjuntura, emergem processos e dinâmicas que
extorsões que configurou um novo perfil para a questão social trazem para a profissão, novas temáticas, novos, e os de
brasileira, particularmente pela via da vulnerabilização do sempre, sujeitos sociais e questões como: o desemprego, o
trabalho, conviveu com a erosão do sistema público de trabalho precário, os sem-terra, o trabalho infantil, a moradia
proteção social, caracterizada por uma perspectiva de retração nas ruas ou em condições de insalubridade, a violência
dos investimentos públicos no campo social, seu doméstica, as discriminações por questões de gênero e etnia,
reordenamento e pela crescente subordinação das políticas as drogas, a expansão da AIDS, as crianças e adolescentes de
sociais às políticas de ajuste da economia, com suas restrições rua, os doentes mentais, os indivíduos com deficiências, o
aos gastos públicos e sua perspectiva privatizadora. É nesse envelhecimento sem recursos, e outras tantas questões e
contexto, e na “contra mão” das transformações que ocorrem temáticas relacionadas à pobreza, à subalternidade e à
na ordem econômica internacional mundializada que o Brasil exclusão com suas múltiplas faces.
vai instituir constitucionalmente em 1988, seu sistema de
Seguridade Social.

Conhecimentos Específicos 4
Apostila Digital Licenciada para Alice Caroline Guarino dos Santos - alice.guarino@hotmail.com (Proibida a Revenda)
APOSTILAS OPÇÃO

Ao longo da década a profissão se coloca diante destas e de do trabalho e do restabelecimento exacerbado dos valores da
outras questões. Destacam‐se como alguns dos eixos competitividade e do individualismo. Não podemos esquecer
articuladores do debate profissional e que tem rebatimentos que a reestruturação dos mercados de trabalho no capitalismo
em sua ação e produção: contemporâneo vem se fazendo via rupturas, apartheid e
degradação humana.
- a Seguridade Social, em construção no país, após a Carta Do ponto de vista das referências teóricometodológicas a
Constitucional de 1988, que afirma o direito dos cidadãos questão primeira que se coloca para a profissão já no início da
brasileiros a um conjunto de direitos no âmbito das políticas década é o confronto com a denominada "crise" dos modelos
sociais (Saúde, Previdência e Assistência Social). A noção de analíticos, explicativos nas ciências sociais, que buscam captar
Seguridade supõe que os cidadãos tenham acesso a um o que está acontecendo no fim de século e as grandes
conjunto de certezas e seguranças que cubram, reduzam ou transformações que alcançam múltiplos aspectos da vida
previnam situações de risco e de vulnerabilidades sociais. Essa social. No mundo do conhecimento começam as interferências,
cobertura é social e não depende do custeio individual direto. não sem conflitos, do denominado pensamento pós moderno,
A inserção do Serviço Social brasileiro nos debates sobre essa "notadamente em sua versão neoconservadora" que questiona
cobertura social marcou a década; e nivela os paradigmas marxista e positivista. Estes
- a Assistência Social, qualificada como política pública, de questionamentos se voltam contra os diferentes "modelos"
Proteção Social, constitutiva da Seguridade Social, constituiu‐ explicativos por suas macroabordagens apontando que nestas
se em tema de estudos, pesquisas e campo de interlocução do macronarrativas são deixados de lado valores e sentimentos
Serviço Social com amplos movimentos da sociedade civil que fundamentais dos homens, seu imaginário, suas crenças,
envolveram fóruns políticos, entidades assistenciais e afeições, a beleza, os saberes do cotidiano, os elementos
representativas dos usuários de serviços assistenciais; étnicos, religiosos, culturais, os fragmentos da vida enfim. A
- a questão da municipalização e da descentralização das abordagem pós‐moderna dirige sua crítica à razão afirmando‐
políticas sociais públicas e outros aspectos daí decorrentes, a como instrumento de repressão e padronização, propõe a
seja na ótica da racionalização de recursos, humanos e sociais superação das utopias, denuncia a administração e o
com vistas a seus efetivos resultados, tanto na perspectiva de disciplinamento da vida, recusa a abrangência das teorias
aproximar a gestão destas políticas dos cidadãos. Notável é sociais com suas análises totalizadoras e ontológicas
desde os anos 1990, em todo o território nacional a presença e sustentadas pela razão e reitera a importância do fragmento,
o protagonismo do assistente social em fóruns e conselhos do intuitivo, do efêmero e do microssocial (em si mesmos)
vinculados às políticas de saúde, de assistência social, da restaurando o pensamento conservador e antimoderno. Seus
criança e do adolescente, entre outras, participando questionamentos são também dirigidos à ciência que esteve
ativamente na defesa de direitos e no controle social das mais a serviço da dominação do que da felicidade dos homens.
políticas públicas. Assim ao afirmar a rejeição à ciência o pensamento pós‐
moderno rejeita as categorias da razão (da Modernidade) que
É importante observar que esta presença tem início em transformaram os modos de pensar da sociedade, mas não
uma conjuntura contraditória e adversa, na qual os impactos emanciparam o homem, não o fizeram mais feliz e não
devastadores sobre o processo de reprodução social da vida se resolveram problemas de sociedades que se complexificam e
fazem notar de múltiplas formas, mas, sobretudo pela se desagregam. O posicionamento pós‐moderno busca
precarização do trabalho e pela desmontagem de direitos. resgatar valores negados pela modernidade e cria um universo
descentrado, fragmentado relativo e fugaz. Para Harvey
É fundamental assinalar que as transformações societárias (1992) as características da pós-modernidade são produzidas
que caracterizam esta década, vão encontrar um Serviço Social historicamente e se relacionam com a emergência de modos
consolidado e maduro na sociedade brasileira, uma profissão mais flexíveis de acumulação do capital.
com avanços e acúmulos, que, ao longo desta década construiu,
com ativa participação da categoria profissional, através de Observe‐se que a complexidade da questão não está na
suas entidades representativas um projeto ético político abordagem de questões micro sociais, locais ou que envolvam
profissional para o Serviço Social brasileiro, que integra dimensões dos valores, afetos e da subjetividade humana
valores, escolhas teóricas e interventivas, ideológicas, (questões de necessário enfrentamento), mas está na recusa
políticas, éticas, normatizações acerca de direitos e deveres, da Razão e na descontextualização, na ausência de referentes
recursos político‐organizativos, processos de debate, históricos, estruturais no não reconhecimento de que os
investigações e, sobretudo interlocução crítica com o sujeitos históricos encarnam processos sociais, expressam
movimento da sociedade na qual a profissão é parte e visões de mundo e tem suas identidades sociais construídas na
expressão. tessitura das relações sociais mais amplas. Relações que se
A direção social que orienta este projeto de profissão tem explicam em teorias sociais abrangentes, que configuram
como referência a relação orgânica com o projeto das classes visões de mundo onde o particular ganha sentido referido ao
subalternas, reafirmado pelo Código de Ética de 1993, pelas genérico.
Diretrizes Curriculares de 1996 e pela Legislação que Cabe assinalar ainda que, todo este debate que é
regulamenta o exercício profissional (Lei n. 8662 de apresentado no âmbito das ciências sociais contemporâneas
07/06/93). como crise de paradigmas, em termos da capacidade
Cabe ainda assinalar outra questão que vem permeou o explicativa das teorias recoloca a polêmica Razão/Intuição que
debate dos assistentes sociais nesta conjuntura: trata‐se do tem repercussões significativas na pesquisa, na construção de
movimento de precarização e de mudanças no mercado de explicações sobre a realidade e na definição de caminhos para
trabalho dos profissionais brasileiros, localizado no quadro a ação.
mais amplo de desregulamentação dos mercados de trabalho Especificamente no Serviço Social estas questões também
de modo geral, quadro em que se alteram as profissões, se colocam, apesar da vitalidade do marxismo como
redefinem‐se suas demandas, monopólios de competência e as paradigma de análise e compreensão da realidade e apesar da
próprias relações de trabalho. Aqui situamos processos como manutenção da hegemonia do projeto profissional
a terceirização, os contratos parciais, temporários, a redução caracterizado pela ruptura com o conservadorismo que
de postos de trabalho, a emergência de novos espaços de caracterizou a trajetória do Serviço Social no país. Colocam‐se
trabalho como o Terceiro Setor, a exigência de novos nos desdobramentos e nas polêmicas em torno dos
conhecimentos técnico‐operativos, ao lado do declínio da ética paradigmas clássicos e na busca de construção de novos

Conhecimentos Específicos 5
Apostila Digital Licenciada para Alice Caroline Guarino dos Santos - alice.guarino@hotmail.com (Proibida a Revenda)
APOSTILAS OPÇÃO

paradigmas; se colocam pela apropriação do pensamento de governo Lula, como é o caso dos programas de Transferência
autores contemporâneos de diversas tendências de Renda.
teóricometodológicas como Anthony Giddens, Hannah Arendt,
Pierre Bourdieu, Michel Foucault, Juergen Habermas, Edgard Efetivamente, no país, apesar dos consideráveis avanços
Morin, Boaventura Souza Santos, Eric Hobsbawm, E.P. na Proteção Social, garantidos na Constituição Federal de 1988
Thompson e tantos outros. Se colocam também nas formas de e expressos, por exemplo, no ECA, na LOAS e no SUS, esses
abordagem das temáticas relevantes para a profissão nesta últimos anos não romperam com as características neoliberais
transição de milênio, na busca de interligação entre sujeito e que se expandiram desde os anos 90, face às necessidades
estrutura e entre concepções macro e micro da vida social, na sociais da população.
retomada e valorização das questões concernentes à cultura No caso da Assistência Social merece destaque a Política
das classes subalternas e em outras clivagens e questões Nacional de Assistência Social – PNAS (2004) que propõe uma
relativas aos dominados tanto no plano das relações culturais nova arquitetura institucional e política para essa política com
como nas lutas pelo empowerment e contra a discriminação a criação de um Sistema Único de Assistência Social ‐ SUAS. O
pelo gênero, pela etnia, pela idade. SUAS é constituído pelo conjunto de serviços, programas,
projetos e benefícios no âmbito da assistência social. É um
No âmbito da produção inspirada na tradição marxista, modo de gestão compartilhada que divide responsabilidades
estas questões aparecem com o recurso à pensadores que para instalar, regular, manter e expandir ações de assistência
abordam temáticas da cultura das classes subalternas, do social.
sujeito e da experiência cotidiana da classe como Gramsci, Desde então, são os assistentes sociais que estão
Heller e Thompson. implementando o SUAS, enfrentando inúmeros desafios entre
os quais destacamos a reafirmação da Assistência Social como
Efetivamente, os desdobramentos desta "crise" de política de Seguridade Social, a consolidação e a
referenciais analíticos, permeiam polêmica profissional dos democratização dos Conselhos e dos mecanismos de
dias atuais e se expressam pelos confrontos com o participação e controle social; a organização e apoio à
conservadorismo que atualiza‐se em tempos pós modernos. representação dos usuários; a participação nos debates sobre
o SUAS, a NOB, os CRAS e os CREAS; a elaboração de
Assim, coloca‐se como desafio à profissão ao longo de toda diagnósticos de vulnerabilidade dos municípios; o
a década de 90, e neste início de milênio a consolidação do monitoramento e a avaliação da política; o estabelecimento de
projeto ético político, teórico metodológico e operativo que indicadores e padrões de qualidade e de custeio dos serviços;
vem construindo particularmente sob a influência da tradição contribuindo para a construção de uma cultura democrática,
marxista, "mas incorporando valores auridos noutras fontes e do direito e da cidadania.
vertentes e, pois sem vincos estreitos ou sectários, aquelas Outro desafio colocado aos assistentes sociais brasileiros
matrizes estão diretamente conectadas ao ideal de socialidade neste início dos anos 2000, refere‐se aos Programas de
posto pelo programa da modernidade ‐ neste sentido, tais Transferência de Renda, sem dúvida uma das faces mais
matrizes não são 'marxistas' nem dizem respeito apenas aos importantes da Política Social brasileira, conforme dados
marxistas, mas remetem a um largo rol de conquistas oficiais (PNAD) chegam a quem precisam chegar (11milhões
civilizatórias e, do ponto de vista profissional, concretizam um de famílias) Destas, 91% tem como renda mensal per capita
avanço que é pertinente a todos os profissionais que, na luta até 1 salário mínimo e 75% delas tem menos de meio salário
contra o conservadorismo, não abrem mão daquilo que o velho mensal de renda per capita. Essa PNAD também revela uma
Lukács chamava de 'herança cultural'." questão essencial: os PTR não retiram os beneficiários do
trabalho (79.1% dos beneficiários trabalham). Ou seja o Bolsa
As polêmicas dos dias atuais Família não pretende substituir a renda do trabalho e apesar
das polêmicas que cercam o Programa, seu impacto sobre as
No início do milênio o Serviço Social brasileiro enfrenta a condições de vida das famílias mais pobres, sobretudo no
difícil herança do final do século anterior, com seus processos Nordeste é incontestável. Ele significa basicamente mais
de globalização em andamento, com sua valorização do capital comida na mesa dos miseráveis.
financeiro, suas grandes corporações transnacionais, seus
mercados, suas mídias, suas estruturas mundiais de poder e as É bom lembrar que se escapa às políticas sociais, às suas
graves consequências desta conjuntura para o tecido social em capacidades, desenhos e objetivos reverter níveis tão elevados
geral, configurando um novo perfil para a questão social; no de desigualdade, como os encontrados no Brasil, essas
qual destacamos a precarização, a insegurança e a políticas também respondem a necessidades e direitos
vulnerabilidade do trabalho e das condições de vida dos concretos de seus usuários.
trabalhadores que perdem suas proteções e enfrentam E os assistentes sociais vêm, em muito, contribuindo, nas
problemas como o desemprego, o crescimento do trabalho últimas décadas, para a construção de uma cultura do direito
informal (hoje mais da metade da força de trabalho do país) e e da cidadania, resistindo ao conservadorismo e considerando
das formas de trabalho precarizado e sem proteção social. as políticas sociais como possibilidades concretas de
Trata‐se de um contexto que interpela a profissão sob construção de direitos e iniciativas de “contradesmanche”
vários aspectos: das novas manifestações e expressões da nessa ordem social injusta e desigual.
questão social, aos processos de redefinição dos sistemas de No âmbito da pesquisa e da produção de conhecimentos o
proteção social e da política social em geral, que emergem Serviço Social brasileiro chega a 2007 com uma maturidade
nesse contexto. expressa em seus 25 Programas de Pós-Graduação
Nesses anos, assim como na última década do século XX, direcionados à formação de recursos humanos com
tornaram‐se evidentes as inspirações neoliberais da política capacidade para atuar criticamente na realidade social.
social brasileira, face às necessidades sociais da população. Do ponto de vista dos referentes teórico metodológicos,
Uma retomada analítica dessas políticas sociais revela sua permanecem as tensões e ambiguidades que caracterizaram o
direção compensatória e seletiva, centrada em situações Serviço social brasileiro na década de 1990: apesar da ruptura
limites em termos de sobrevivência e seu direcionamento aos com o histórico conservadorismo e da legitimidade alcançada
mais pobres dos pobres, incapazes de competir no mercado. pelo pensamento marxista ampliam‐se as interferências de
Estas políticas focalizadas permaneceram e se expandiram no outras correntes teórico metodológicas, particularmente no
âmbito da influência do pensamento pós‐moderno e

Conhecimentos Específicos 6
Apostila Digital Licenciada para Alice Caroline Guarino dos Santos - alice.guarino@hotmail.com (Proibida a Revenda)
APOSTILAS OPÇÃO

neoconservador e das teorias herdeiras da “perspectiva afetivos - relacionais e de pertencimento social


modernizadora”. (discriminações etárias, étnicas, de gênero ou por deficiências,
dentre outras).
Há pouco mais de uma década, Netto já apontava como Prevê o desenvolvimento de serviços, programas e
hipóteses para o encaminhamento dessa tensão uma dupla projetos locais de acolhimento, convivência e socialização de
perspectiva: de um lado a consolidação e o aprofundamento da famílias e de indivíduos, conforme identificação da situação de
hegemonia da atual direção social e de outro a possibilidade vulnerabilidade apresentada. Deverão incluir as pessoas com
de sua reversão ou mudança. Afirmava o autor "[...] num deficiência e ser organizados em rede, de modo a inseri-las nas
ordenamento social com regras democráticas, uma profissão é diversas ações ofertadas. Os benefícios, tanto de prestação
sempre um campo de lutas, em que os diferentes segmentos da continuada como os eventuais compõem a proteção social
categoria, expressando a diferenciação ideopolítica existente básica, dada a natureza de sua realização.
na sociedade, procuram elaborar uma direção social Os serviços, programas, projetos e benefícios de proteção
estratégica para a sua profissão". social básica deverão ainda se articular com as demais
Para finalizar é necessário assinalar que a reafirmação das políticas públicas locais, de forma a garantir a sustentabilidade
bases teóricas do projeto ético político, teórico metodológico e das ações desenvolvidas e o protagonismo das famílias e
operativo, centrada na tradição marxista, não pode implicar na indivíduos atendidos, de forma a superar as condições de
ausência de diálogo com outras matrizes de pensamento vulnerabilidade e a prevenir as situações que indicam risco
social, nem significa que as respostas profissionais aos potencial. Deverão, ainda, se articular aos serviços de proteção
desafios desse novo cenário de transformações possam ou especial, garantindo a efetivação dos encaminhamentos
devam ser homogêneas. Embora possam e devam ser criativas necessários.
e competentes. Os serviços de proteção social básica serão executados de
forma direta nos Centros de Referência da Assistência Social
(CRAS) e em outras unidades básicas e públicas de Assistência
2. políticas Públicas e Social, bem como de forma indireta nas entidades e
Serviço Social; organizações de Assistência Social da área de abrangência dos
CRAS.

A Política Pública de Assistência Social realiza-se de forma Centro de Referência da Assistência Social e os serviços
integrada às políticas setoriais, considerando as desigualdades de proteção básica
socioterritoriais, visando seu enfrentamento, à garantia dos
mínimos sociais, ao provimento de condições para atender O Centro de Referência da Assistência Social – CRAS é uma
contingências sociais e à universalização dos direitos sociais. unidade pública estatal de base territorial, localizado em áreas
Sob esta perspectiva, objetiva: de vulnerabilidade social, que abrange a um total de até 1.000
- prover serviços, programas, projetos e benefícios de famílias/ano. Executa serviços de proteção social básica,
proteção social básica e, ou, especial para famílias, indivíduos organiza e coordena a rede de serviços socioassistenciais
e grupos que deles necessitar; locais da política de assistência social.
- contribuir com a inclusão e a equidade dos usuários e O CRAS atua com famílias e indivíduos em seu contexto
grupos específicos, ampliando o acesso aos bens e serviços comunitário, visando a orientação e o convívio sócio familiar e
socioassistenciais básicos e especiais, em áreas urbana e rural; comunitário. Neste sentido, é responsável pela oferta do
- assegurar que as ações no âmbito da assistência social Programa de Atenção Integral às Famílias. Na proteção básica,
tenham centralidade na família, e que garantam a convivência o trabalho com famílias deve considerar novas referências
familiar e comunitária; para a compreensão dos diferentes arranjos familiares,
superando o reconhecimento de um modelo único baseado na
Usuários família nuclear, e partindo do suposto de que são funções
básicas das famílias: prover a proteção e a socialização dos
Constitui o público usuário da política de Assistência seus membros; constituir-se como referências morais, de
Social, cidadãos e grupos que se encontram em situações de vínculos afetivos e sociais; de identidade grupal, além de ser
vulnerabilidade e riscos, tais como: famílias e indivíduos com mediadora das relações dos seus membros com outras
perda ou fragilidade de vínculos de afetividade, pertencimento instituições sociais e com o Estado.
e sociabilidade; ciclos de vida; identidades estigmatizadas em O grupo familiar pode ou não se mostrar capaz de
termos étnico, cultural e sexual; desvantagem pessoal desempenhar suas funções básicas. O importante é notar que
resultante de deficiências; exclusão pela pobreza e, ou, no esta capacidade resulta não de uma forma ideal e sim de sua
acesso às demais políticas públicas; uso de substâncias relação com a sociedade, sua organização interna, seu
psicoativas; diferentes formas de violência advinda do núcleo universo de valores, entre outros fatores, enfim, do estatuto
familiar, grupos e indivíduos; inserção precária ou não mesmo da família como grupo cidadão. Em consequência,
inserção no mercado de trabalho formal e informal; qualquer forma de atenção e, ou, de intervenção no grupo
estratégias e alternativas diferenciadas de sobrevivência que familiar precisa levar em conta sua singularidade, sua
podem representar risco pessoal e social. vulnerabilidade no contexto social, além de seus recursos
simbólicos e afetivos, bem como sua disponibilidade para se
Assistência Social e as proteções afiançadas transformar e dar conta de suas atribuições.
Além de ser responsável pelo desenvolvimento do
Proteção Social Básica Programa de Atenção Integral às Famílias - com referência
territorializada, que valorize as heterogeneidades, as
A proteção social básica tem como objetivos prevenir particularidades de cada grupo familiar, a diversidade de
situações de risco através do desenvolvimento de culturas e que promova o fortalecimento dos vínculos
potencialidades e aquisições, e o fortalecimento de vínculos familiares e comunitários – a equipe do CRAS deve prestar
familiares e comunitários. Destina-se à população que vive em informação e orientação para a população de sua área de
situação de vulnerabilidade social decorrente da pobreza, abrangência, bem como articular-se com a rede de proteção
privação (ausência de renda, precário ou nulo acesso aos social local no que se refere aos direitos de cidadania,
serviços públicos, dentre outros) e, ou, fragilização de vínculos mantendo ativo um serviço de vigilância da exclusão social na

Conhecimentos Específicos 7
Apostila Digital Licenciada para Alice Caroline Guarino dos Santos - alice.guarino@hotmail.com (Proibida a Revenda)
APOSTILAS OPÇÃO

produção, sistematização e divulgação de indicadores da área e de suas dificuldades de auto-organização e de participação


de abrangência do CRAS em conexão com outros territórios. social.
Realiza ainda, sob orientação do gestor municipal de Assim, as linhas de atuação com as famílias em situação de
Assistência Social, o mapeamento e a organização da rede risco devem abranger desde o provimento de seu acesso a
socioassistencial de proteção básica e promove a inserção das serviços de apoio e sobrevivência até sua inclusão em redes
famílias nos serviços de assistência social local. Promove sociais de atendimento e de solidariedade.
também o encaminhamento da população local para as demais As situações de risco demandarão intervenções em
políticas públicas e sociais possibilitando o desenvolvimento problemas específicos e, ou, abrangentes. Nesse sentido, é
de ações intersetoriais que visem a sustentabilidade, de forma preciso desencadear estratégias de atenção sócio familiar que
a romper com o ciclo de reprodução intergeracional do visem a reestruturação do grupo familiar e a elaboração de
processo de exclusão social, e evitar que estas famílias e novas referências morais e afetivas, no sentido de fortalecê-lo
indivíduos tenham seus direitos violados, recaindo em para o exercício de suas funções de proteção básica ao lado de
situações de vulnerabilidades e riscos. sua auto-organização e conquista de autonomia. Longe de
São considerados serviços de proteção básica de significar um retorno à visão tradicional, e considerando a
assistência social aqueles que potencializam a família como família como uma instituição em transformação, a ética da
unidade de referência, fortalecendo seus vínculos internos e atenção da proteção especial pressupõe o respeito à cidadania,
externos de solidariedade, através do protagonismo de seus o reconhecimento do grupo familiar como referência afetiva e
membros e da oferta de um conjunto de serviços locais que moral e a reestruturação das redes de reciprocidade social.
visam à convivência, socialização e ao acolhimento, em A ênfase da proteção social especial deve priorizar a
famílias cujos vínculos familiar e comunitário não foram reestruturação dos serviços de abrigamento dos indivíduos
rompidos, tais como: que, por uma série de fatores, não contam mais com a proteção
- Programa de Atenção Integral às Famílias; e o cuidado de suas famílias, para as novas modalidades de
- Projetos de Geração de Trabalho e Renda; atendimento. A história dos abrigos e asilos é antiga no Brasil.
- Centros de Convivência para Idosos; A colocação de crianças, adolescentes, pessoas com
- Serviços para crianças de 0 a 6 anos, que visem o deficiências e idosos em instituições para protegê-los ou
fortalecimento dos vínculos familiares, o direito de brincar, afastá-los do convívio social e familiar foi, durante muito
ações de socialização e de sensibilização para a defesa dos tempo, materializada em grandes instituições de longa
direitos das crianças; permanência, ou seja, espaços que atendiam a um grande
- Serviços socioeducativos para crianças e adolescentes na número de pessoas, que lá permaneciam por longo período –
faixa etária de 6 a 14 anos, visando sua proteção, socialização às vezes a vida toda. São os chamados, popularmente, como
e o fortalecimento dos vínculos familiares e comunitários; orfanatos, internatos, educandários, asilos, entre outros.
- Programas de incentivo ao protagonismo juvenil, e de São destinados, por exemplo, às crianças, aos adolescentes,
fortalecimento dos vínculos familiares e comunitários; aos jovens, aos idosos, às pessoas com deficiência e às pessoas
- Centros de Informação e de educação para o trabalho, em situação de rua que tiverem seus direitos violados e, ou,
voltado para jovens e adultos. ameaçados e cuja convivência com a família de origem seja
considerada prejudicial à sua proteção e ao seu
Proteção Social Especial desenvolvimento. No caso da proteção social especial à
população em situação de rua serão priorizados os serviços
Além de privações e diferenciais de acesso a bens e que possibilitem a organização de um novo projeto de vida,
serviços, a pobreza associada à desigualdade social e a visando criar condições para adquirirem referências na
perversa concentração de renda, revela-se numa dimensão sociedade brasileira, enquanto sujeitos de direito.
mais complexa: a exclusão social. O termo exclusão social A proteção social especial é modalidade de atendimento
confunde-se, comumente, com desigualdade, miséria, assistencial destinada a famílias e indivíduos que se
indigência, pobreza (relativa ou absoluta), apartação social, encontram em situação de risco pessoal e social, por
dentre outras. Naturalmente existem diferenças e ocorrência de abandono, maus tratos físicos e, ou, psíquicos,
semelhanças entre alguns desses conceitos, embora não exista abuso sexual, uso de substâncias psicoativas, cumprimento de
consenso entre os diversos autores que se dedicam ao tema. medidas socioeducativas, situação de rua, situação de trabalho
Entretanto, diferentemente de pobreza, miséria, desigualdade infantil, dentre outras.
e indigência que são situações, a exclusão social é um processo São serviços que requerem acompanhamento individual e
que pode levar ao acirramento da desigualdade e da pobreza maior flexibilidade nas soluções protetivas. Da mesma forma,
e, enquanto tal, apresenta-se heterogênea no tempo e no comportam encaminhamentos monitorados, apoios e
espaço. processos que assegurem qualidade na atenção protetiva e
A realidade brasileira nos mostra que existem famílias com efetividade na reinserção almejada.
as mais diversas situações socioeconômicas que induzem à Os serviços de proteção especial têm estreita interface com
violação dos direitos de seus membros, em especial, de suas o sistema de garantia de direito exigindo, muitas vezes, uma
crianças, adolescentes, jovens, idosos e deficientes, além da gestão mais complexa e compartilhada com o Poder Judiciário,
geração de outros fenômenos como, por exemplo, pessoas em Ministério Público e outros órgãos e ações do Executivo.
situação de rua, migrantes, idosos abandonados que estão
nesta condição não pela ausência de renda, mas por outras Proteção Social Especial de média complexidade
variáveis da exclusão social. Percebe-se que estas situações se
agravam justamente nas parcelas da população onde há São considerados serviços de média complexidade aqueles
maiores índices de desemprego e de baixa renda dos adultos. que oferecem atendimentos às famílias e indivíduos com seus
As dificuldades em cumprir com funções de proteção direitos violados, mas cujos vínculos familiar e comunitário
básica, socialização e mediação, fragilizam, também, a não foram rompidos. Neste sentido, requerem maior
identidade do grupo familiar, tornando mais vulneráveis seus estruturação técnico operacional e atenção especializada e
vínculos simbólicos e afetivos. A vida destas famílias não é mais individualizada, e, ou, de acompanhamento sistemático e
regida apenas pela pressão dos fatores socioeconômicos e monitorado. Tais como:
necessidade de sobrevivência. Elas precisam ser - Serviço de orientação e apoio sócio familiar;
compreendidas em seu contexto cultural, inclusive ao se tratar - Plantão Social;
da análise das origens e dos resultados de sua situação de risco - Abordagem de Rua;

Conhecimentos Específicos 8
Apostila Digital Licenciada para Alice Caroline Guarino dos Santos - alice.guarino@hotmail.com (Proibida a Revenda)
APOSTILAS OPÇÃO

- Cuidado no Domicílio; que cabe ao profissional apenas ter habilidade técnica de


- Serviço de Habilitação e Reabilitação na comunidade das manusear um instrumento de trabalho, o Assistente Social
pessoas com deficiência; perderá a dimensão do por que ele está utilizando
- Medidas socioeducativas em meio-aberto (PSC – determinado instrumento. Sua prática se torna mecânica,
Prestação de Serviços à Comunidade e LA – Liberdade repetitiva, burocrática. Mais do que meramente aplicar
Assistida). técnicas “prontas” – como se fossem “receitas de bolo”, o
A proteção especial de média complexidade envolve diferencial de um profissional é saber adaptar um
também o Centro de Referência Especializado da Assistência determinado instrumento às necessidades que precisa
Social, visando a orientação e o convívio sócio familiar e responder no seu cotidiano. E como a realidade é dinâmica,
comunitário. Difere-se da proteção básica por se tratar de um faz-se necessário compreender quais mudanças são essas
atendimento dirigido às situações de violação de direitos. para que o instrumental utilizado seja o mais eficaz
possível, e, de fato, possa produzir as mudanças desejadas
Proteção Social Especial de alta complexidade pelo Assistente Social – ou chegar o mais próximo possível.

Os serviços de proteção social especial de alta Ora, isso pressupõe que, mais do que copiar e seguir
complexidade são aqueles que garantem proteção integral – manuais de instruções, o que se coloca para o Assistente Social
moradia, alimentação, higienização e trabalho protegido para hoje é sua capacidade criativa, o que inclui o potencial de
famílias e indivíduos que se encontram sem referência e, ou, utilizar instrumentos consagrados da profissão, mas também
em situação de ameaça, necessitando ser retirados de seu de criar outros tantos que possam produzir mudanças na
núcleo familiar e, ou, comunitário. Tais como: realidade social, tanto em curto quanto em médio e longo
- Atendimento Integral Institucional; prazos.
- Casa Lar; Isso é primordial para que possamos desempenhar com
- República; competência as atribuições que foram definidas para o
- Casa de Passagem; Assistente Social na Lei de Regulamentação Profissional – e
- Albergue; que foi citado na Introdução desse texto. Vejamos: se o Serviço
- Família Substituta; Social, em sua trajetória histórica, não tivesse criado novos
- Família Acolhedora; instrumentos e novas técnicas de intervenção, teria
- Medidas socioeducativas restritivas e privativas de conseguido sair da condição de mero executor das políticas
liberdade (Semiliberdade, Internação provisória e sociais e hoje desempenhar funções de elaboração,
sentenciada); planejamento e gerência das mesmas? Certamente não.
- Trabalho protegido.
Assim, pensar a instrumentalidade do Serviço Social é
A INSTRUMENTALIDADE DO SERVIÇO SOCIAL1 pensar para além da “especificidade” da profissão: é pensar
que são infinitas as possibilidades de intervenção profissional,
Expressar os objetivos que se quer alcançar não significa e que isso requer, nas palavras de Iamamoto, “tomar um banho
que eles necessariamente serão alcançados. Nunca podemos de realidade”. Guerra resume, em poucas palavras, o sentido
perder de vista que qualquer ação humana está condicionada dessa reflexão.
ao momento histórico em que ela é desenvolvida. A realidade A clara definição do ‘Para quê’ da profissão, possível desde
social é complexa, heterogênea e os impactos de qualquer que iluminada por uma racionalidade (como forma de ser e
intervenção dependem de fatores que são externos a quem pensar) que seja dialética e crítica, conectada à capacidade de
quer que seja – inclusive ao Serviço Social. Como analisa responder eficazmente às demandas sociais, se constituirão na
Iamamoto, reconhecer as possibilidades e limitações condição necessária, talvez não suficiente, à manutenção da
históricas, dadas pela própria realidade social, é funda- mental profissão. Aqui se coloca a necessidade de dominar um
para que o Serviço Social não adote, por um lado, uma postura repertório de técnicas, legada do desenvolvi- mento das
fatalista (ou seja, acreditar que a realidade já está dada e não ciências sociais, fruto das pesquisas e do avanço tecnológico e
pode ser mudada), ou por outro lado, uma postura messiânica patrimônio das profissões sociais (e não exclusividade de uma
(achar que o Serviço Social é o “messias”, que é a profissão que categoria profissional), mas também um conjunto de
vai transformar todas as relações sociais). É importante ter estratégias e táticas desenvolvidas, criadas e recriadas no
essa compreensão para localizarmos o lugar ocupado pelos processo histórico, no movimento da realidade.
instrumentos de trabalho utilizados pelo Assistente Social em
sua prática. Instrumentalidade e linguagem
Se são os objetivos profissionais (construí- dos a partir de
uma reflexão teórica, ética e política e um método de É sabido que o estudo sobre linguagem é bastante amplo
investigação) que definem os instrumentos e técnicas de no âmbito das Ciências Sociais. Contudo, esse ainda é um tema
intervenção (as metodologias de ação), conclui-se que essas pouco explorado na literatura do Serviço Social – clássica ou
metodologias não estão prontas e acabadas. Elas são contemporânea. A título de referência, usaremos aqui os
necessárias em qualquer processo racional de intervenção, conceitos trabalhados por Magalhães. Para ela, segundo os
mas elas são construídas a partir das finalidades estabelecidas linguistas, o homem se comunica através de signos, e estes são
no planejamento da ação realizado pelo Assistente Social. organizados através de códigos e linguagens. Pelo processo
Primeiro, ele define “para quê fazer”, para depois se definir socializador, ele desenvolve e amplia suas aptidões de
“como fazer”. Mais uma vez, podemos aqui identificar a comunicação, utilizando os modos e usos de fala que estão
estreita relação entre as competências teóricometodológicas, configurados no contexto sociocultural dos diferentes grupos
ético-política e técnico-operativa. sociais dos quais faz parte.
Assim, os seres humanos dão significados às categorias
Em outras palavras, os instrumentos e técnicas de que existem na realidade (ontológicas) através de códigos-
intervenção não podem ser mais importantes que os palavras. Portanto, uma palavra só tem significado se
objetivos da ação profissional. Se partirmos do pressuposto

1Texto adaptado de SOUSA, C. T. de. Assistente Social do Ministério


Público.

Conhecimentos Específicos 9
Apostila Digital Licenciada para Alice Caroline Guarino dos Santos - alice.guarino@hotmail.com (Proibida a Revenda)
APOSTILAS OPÇÃO

compreendida no contexto social e político no qual ela é Os instrumentos de trabalho diretos ou “face a face”
utilizada.
Indo mais além, a autora afirma que as linguagens Sobre a interação face a face, esta permite que a
construídas são produtos do processo de socialização dos enunciação de um discurso se expresse não só pela palavra,
seres humanos, o que remete a uma concepção social das mas também pelo olhar, pela linguagem gestual, pela
diferentes linguagens existentes em uma mesma sociedade: entonação, que vão contextualizar e, possivelmente, identificar
elas (as linguagens) indicam modos de ser e de viver de classes subjetividades de uma forma mais evidenciada. Sob esse
e grupos sociais diferentes entre si. Em outras palavras, a enfoque, pode-se dizer que o discurso direto expressa uma
linguagem possibilita a construção da identidade de um interação dinâmica.
determinado grupo social. Assim, podemos identificar alguns instrumentos de
Considera-se que a linguagem é o instrumento número um trabalho “face a face” consagrados na história da profissão, e
de todos os profissionais, pois ela possibilita a comunicação que abaixo apresentamos de forma bastante sucinta:
entre estes e aqueles com quem interagem. Ou, como afirma
Iamamoto, no nosso caso: “o Serviço Social, como uma das Observação participante
formas institucionalizadas de atuação nas relações entre os
homens no cotidiano da vida social, tem como recurso básico Observar é muito mais do que ver ou olhar. Observar é
de trabalho a linguagem”. E é a partir das formas de estar atento, é direcionar o olhar, é saber para onde se olha.
comunicação que se estabelecem no espaço das instituições Na definição clássica, a observação é o uso dos sentidos
onde trabalha o Assistente Social que este profissional poderá humanos (visão, audição, tato, olfato e paladar) para o
construir e utilizar instrumentos e técnicas de intervenção conhecimento da realidade. Mas não um uso ingênuo dos
social. sentidos, e sim, um uso que tem como objetivo produzir um
conheci- mento sobre a realidade – tem-se um objetivo a
Segundo Martinelli & Koumrouyan, define-se por alcançar.
instrumental o conjunto articulado de instrumentos e técnicas Porém, o Assistente Social, ao estabelecer uma interação
que permitem a operacionalização da ação profissional. Nessa face a face, estabelece uma relação social com outro(s) ser (es)
ideia, o instrumento é estratégia ou tática por meio da qual se humano(s), que possui (em) expectativas quanto às
realiza a ação; a técnica é a habilidade no uso do instrumento. intervenções que serão realizadas pelo profissional. Assim,
O uso do instrumental pressupõe interações de além de observador, o profissional também é observado.
comunicação, isto é, do uso de linguagens por parte do E ainda: na medida em que o Assistente Social realiza
Assistente Social. Se a linguagem é um meio através do qual um intervenções, ele participa diretamente do processo de
determinado grupo social cria uma identidade social, não será conhecimento acerca da realidade que está sendo investigada.
diferente para uma profissão que tem a linguagem como o Por isso, não se trata de uma observação fria, ou como querem
principal recurso de trabalho. O que queremos dizer é que o alguns, “neutra”, em que o profissional pensa estar em uma
Assistente Social diz quem ele é, seja para a população posição de não-envolvimento com a situação. Por isso, trata-se
atendida ou para quem com que estabelecer alguma relação, a de uma observação participante – o profissional, além de
partir das formas de comunicação e de interação que ele observar, interage com o outro, e participa ativamente do
estabelece com esses sujeitos. Assim, a definição do processo de observação.
instrumental a ser utilizado na intervenção profissional deve
sempre levar em consideração o agente receptor da Entrevista individual e grupal
mensagem, ou seja, o destinatário da mesma.
A entrevista nada mais é do que um diálogo, um processo
Assim, para além da linguagem que é própria da matéria de comunicação direta entre o Assis- tente Social e um usuário
de Serviço Social, isto é, aquela que é utilizada quando a (entrevista individual), ou mais de um (entrevista grupal).
“questão social” está sendo refletida e trabalhada, nunca nos Contudo, o que diferencia a entrevista de um diálogo comum é
esqueçamos que o Assistente Social é um profissional de nível o fato de existir um entrevistador e um entrevistado, isto é, o
superior. Com tal grau de escolaridade, “é de esperar que Assistente Social ocupa um papel diferente – e, sob
sigam a norma culta da língua [portuguesa] e não adentrem determinado ponto de vista, desigual – do papel do usuário.
seus escritos [e falas] para uma linguagem coloquial ou do O papel do profissional entrevistador é dado pela
senso comum”. Nesse sentido, é primordial que o Assistente instituição que o contrata – no momento da interação com o
social saiba falar e escrever cor- retamente, bem como usuário, o Assistente Social fala em nome da instituição.
comunicar-se articulada- mente. Isto é criar uma identidade Ambos os sujeitos (Assistente Social e usuário) possuem
social de um profissional competente, que articula teoria e objetivos com a realização da entrevista – objetivos esses
prática, e que detém uma forma coerente de pensar e de necessariamente diferentes. Mas o papel de entrevistador que
expressar o pensamento. cabe ao Assistente Social coloca-lhe a tarefa de conduzir o
Definido isto, podemos identificar duas categorias de diálogo, de direcionar para os objetivos que se pretendem
linguagens comumente utilizadas pelo Serviço Social: a alcançar.
linguagem oral ou direta e a linguagem escrita ou indireta, e Nem sempre é possível conciliar os objetivos do usuário e
com elas, estabelecer as interações. Desse modo, podemos os do Assistente Social (e alcançar essa conciliação não é uma
classificar os instrumentos de trabalho como instrumentos regra). Entretanto, estabelece-se uma relação de poder entre
diretos (ou “face a face”) e instrumentos indiretos (ou “por esses dois sujeitos – relação essa em que o Assistente Social
escrito”). aparece em uma posição hierarquicamente superior. Mas se
Não é possível aqui esgotar todas as possibilidades de defendemos a democracia e o respeito à diversidade como
utilização dos instrumentos de trabalho, pois cada um deles valores éticos fundamentais da nossa profissão, o momento da
possui características muito peculiares. O que será aqui entrevista é um espaço que o usuário pode exprimir suas
desenvolvido é uma breve apresentação dos principais ideias, vontades, necessidades, ou seja, que ele possa ser
instrumentos utilizados pelo Serviço Social no cotidiano de sua ouvido (em tempo: ser ouvido não é concordar com tudo o que
prática – e nunca perdendo de vista que se trata de alguns usuário diz). Estabelecer essa relação é fundamental, pois se o
instrumentos, uma vez que a definição dos mesmos depende usuário não é respeitado nesse direito básico, não apenas
do objetivo estabelecido pelo profissional. estaremos desrespeitando-o, como prejudicando o próprio
processo de construção de um conheci- mento sólido sobre a

Conhecimentos Específicos 10
Apostila Digital Licenciada para Alice Caroline Guarino dos Santos - alice.guarino@hotmail.com (Proibida a Revenda)
APOSTILAS OPÇÃO

realidade social que ele está trazendo, comprometendo toda a reunião possa alcançar o objetivo de tomar uma decisão que
intervenção. envolva todos os seus participantes.
Importante ressaltar que, por ser um observador
participante, o Assistente Social também emite suas opiniões, Mobilização de comunidades
valores, a partir dos conhecimentos que já possui. Desse modo,
entrevistar é mais do que apenas “conversar”: requer um Muitos Assistentes Sociais desenvolvem trabalhos em
rigoroso conhecimento teóricometodológico, a fim de comunidades de um modo geral. Contudo, faz-se necessário
possibilitar um planejamento sério da entrevista, bem como a clarificar o que se entende por comunidade. Segundo a
busca por alcançar os objetivos estabelecidos para sua definição de Souza, comunidade é um conjunto de grupos e
realização. subgrupos de uma mesma classe social, que têm interesses e
preocupações comuns sobre condições de vivência no espaço
Dinâmica de Grupo de moradia e que, dadas as suas condições fundamentais de
existência, tendem a ampliar continuamente o âmbito de
Descendente da Psicologia Social, a dinâmica de grupo repercussão dos seus interesses, preocupações e
surgiu como um instrumento de pesquisa do comportamento enfrentamentos comuns.
humano em pequenos grupos. Em seguida, tornou-se um Assim, temos algumas características que definem o que
instrumento bastante utilizado na área social – em especial na entendemos por comunidade: falamos de um território
saúde mental – e hoje é muito utilizada em empresas. A geograficamente definido, mas ao mesmo tempo, entendendo
dinâmica de grupo foi amplamente usada como uma forma de que a divisão geográfica do espaço territorial reflete as
garantir controles coletivos, manipular comportamentos, diferentes divisões da sociedade em classes sociais e
valendo-se das relações grupais. segmentos de classes sociais. Assim, trabalhar em uma
comunidade significa compreendê-la dentro de um contexto
Contudo, a dinâmica de grupo é um recurso que pode ser econômico, social, político e cultural de uma sociedade
utilizado pelo Assistente Social em diferentes momentos de sua dividida em classes sociais – e que ela não está descolada da
intervenção. Para levantar um debate sobre determinado tema totalidade da realidade social.
com um número maior de usuários, bem como atender um maior Trabalhar em projetos comunitários na perspectiva ético-
número de pessoas que estejam vivenciando situações parecidas. política defendida pelo Serviço Social, hoje, significa criar
E nunca é demais lembrar que é o instrumento que se adapta aos estratégias para mobilizar e envolver os membros de uma
objetivos profissionais – no caso, a dinâmica de grupo deve estar população situada historicamente no tempo e no espaço nas
em consonância com as finalidades estabelecidas pelo decisões das ações que serão desenvolvidas, uma vez que são
profissional. eles o público-alvo do trabalho do Assistente Social. Assim,
trata-se de um processo de mobilização comunitária.
Sucintamente, a dinâmica de grupo é uma técnica que Para tal, é necessário que o Assistente Social conheça a
utiliza jogos, brincadeiras, simulações de determinadas comunidade, os atores sociais que lá atuam: os agentes
situações, com vistas a permitir que os membros do grupo políticos, as instituições existentes, as organizações
produzam uma reflexão acerca de uma temática definida. No (religiosas, comerciais, políticas) e como se constroem as
caso do Ser- viço Social, uma temática que tenha relação com o relações de poder dentro da comunidade trabalhando em rede.
objeto de sua intervenção – as diferentes expressões da Mas também é necessário conhecer quais são as principais
“questão social”. Para tanto, o Assistente Social age como um demandas e necessidades da comunidade, de modo a propor
facilitador, um agente que provoca situações que levem à principal objetivo conhecer as condições e modos de vida da
reflexão do grupo. Isso requer tanto habilidades teóricas (a população usuária em sua realidade cotidiana, ou seja, no local
escolha do tema e como ele será trabalhado), como uma onde ela estabelece suas relações do dia a dia: em seu
postura política democrática (que deixa o grupo produzir), domicílio.
mas também uma necessidade de controle do processo de
dinâmica – caso contrário, a dinâmica vira uma “brincadeira” A visita domiciliar
e não alcança os objetivos principais: provocar a reflexão do
grupo. A visita domiciliar é um instrumento que, ao final,
aproxima a instituição que está atendendo ao usuário de sua
Reunião realidade, via Assistente Social. Assim as instituições devem
garantir as condições para que a visita domiciliar seja
Assim como a dinâmica de grupo, as reuniões são espaços realizada (transporte, por exemplo).
coletivos. São encontros grupais, que têm como objetivo Como os demais instrumentos, a visita domiciliar não é
estabelecer alguma espécie de reflexão sobre determinado exclusividade do Assistente Social: ela só é realizada quando o
tema. Mas, sobre- tudo, uma reunião tem como objetivo a objetivo da mesma é analisar as condições sociais de vida e de
tomada de uma decisão sobre algum assunto. existência de uma família ou de um usuário – pois é esse
As reuniões podem ocorrer com diferentes sujeitos – “olhar” que determina a inserção do Serviço Social na divisão
podem ser realizadas junto à população usuária, junto à equipe social do trabalho.
de profissionais que trabalham na instituição. Enfim ela se Contudo, a visita domiciliar sempre foi um dos principais
realiza em todo espaço em que se pretende que uma instrumentos de controle das classes populares que as
determinada decisão não seja tomada individualmente, mas instituições utilizavam. Uma vez que o usuário está sendo
coletivamente. Essa postura já indica que, ao coletivizar a atendido na instituição, ele está acionando um espaço público:
decisão, o coordenador de uma reunião se coloca em uma quando a instituição se propõe a ir até a casa do usuário, ela
posição democrática. está adentrando no terreno do privado. A residência é o espaço
Entretanto, colocar-se como um líder democrático não privado da família que lá vive. Ter essa dimensão é
significa não ter firmeza quanto ao cumprimento dos objetivos fundamental para que o Assistente Social rompa com uma
da reunião. O espaço de tomada de decisões é um espaço postura autoritária, controladora e fiscalizadora.
essencial- mente político, pois diferentes interesses estão em Porém, é de suma importância que o profissional que
confronto. Saber reconhecê-los e como se relacionar com eles realiza a visita tenha competência teórica para saber
requer uma competência teórica e política, de modo que a identificar que as condições de moradia não estão descoladas
das condições de vida de uma comunidade onde a casa se

Conhecimentos Específicos 11
Apostila Digital Licenciada para Alice Caroline Guarino dos Santos - alice.guarino@hotmail.com (Proibida a Revenda)
APOSTILAS OPÇÃO

localiza, e que, por sua vez, não estão separadas do contexto Assistente Social dentro da instituição – é essencial para
social e histórico. Assim, o profissional consegue romper uma qualquer proposta de construção de um conheci- mento sobre
mera “constatação” da singularidade, mas situá-la no campo da a realidade social.
universalidade, ou seja, no contexto sócio econômico vigente. Assim, podemos identificar alguns instrumentos de
trabalho “por escrito” consagrados na história da profissão, e
Visita institucional que abaixo apresentamos de forma bastante sucinta.

Assim como a visita domiciliar, aqui se fala de quando o Atas de reunião


Assistente Social realiza visita a instituições de diversas É o registro de todo o processo de uma reunião, das
naturezas – entidades públicas, empresas, ONGs etc. discussões realizadas, das opiniões emitidas, e, sobretudo, da
Muitas podem ser as motivações para que o Assistente decisão tomada – e da forma como o grupo chegou a ela (por
Social realize uma visita institucional. Enumeramos três delas: votação, por consenso, ou outra forma).
Geralmente o relator de uma ata de reunião é designado
1. Quando o Assistente Social está trabalhando em um para tal. Pode ser um membro do grupo ou um funcionário da
determinada situação singular, e resolve visitar uma instituição instituição. Comumente, as atas de reuniões são lidas ao final
com a qual o usuário mantém alguma espécie de vínculo; da mesma, e, após sua aprovação, todos os participantes
2. Quando o Assistente Social quer conhecer um assinam – com garantia de que a discussão realizada assim
determinado trabalho desenvolvido por uma instituição; como a decisão tomada é de ciência de todos.
3. Quando o Assistente Social precisa realizar uma avaliação
da cobertura e da qualidade dos serviços prestados por uma Livros de Registro
instituição.
O Livro de Registro é um instrumento bastante utilizado,
Em todos os casos, sobretudo nos 02 últimos, o que se quer sobretudo em locais onde circula um grande número de
fazer é conhecer e avaliar a qualidade da política social – o que profissionais. Trata-se de um livro onde são anotadas as
requer do profissional um intenso conhecimento teórico e atividades realiza- das, telefonemas recebidos, questões
técnico sobre políticas sociais. pendentes, atendimentos realizados, dentre outras questões,
Pode-se perceber, a partir do elencado acima, que os de modo que toda a equipe tenha acesso ao que está sendo
instrumentos de trabalho não são atomizados ou estáticos: desenvolvido ações que visem ao atendimento das mesmas.
eles podem coexistir em um mesmo momento. A observação
participante está presente em todos os demais; em uma visita Uma interessante reflexão sobre o papel histórico que a
domiciliar a entrevista pode ser utilizada; no trabalho de visita
mobilização comunitária, reuniões podem ocorrer, além de
visitas institucionais, dentre outras situações. Várias Diário de Campo
combinações entre eles podem ser descritas, porque a
realidade da prática profissional é muito mais dinâmica e rica Como afirmamos anteriormente, o profissional está em
do que qualquer tentativa de classificação dos instrumentos de constante transformação, em constante aprendizagem e
trabalho. aperfeiçoamento. Contudo, ele precisa se reconhecer no
trabalho
Os instrumentos de trabalho indiretos ou “por escrito” – identificar onde residem suas dificuldades, e localizar os
limites e as possibilidades de trabalho.
Sobre os instrumentos de trabalho indiretos, eles O diário de campo é um instrumento que auxilia bastante
necessariamente são utilizados após a utilização do o profissional nesse processo. Trata- se de anotações livres do
instrumental face a face, que é caracterizado por uma forma de profissional, individuais, em que o mesmo sistematiza suas
comunicação mais ativa. É o registro do trabalho direto atividades e suas reflexões sobre o cotidiano do seu trabalho.
realizado. Assim, no caso da interação por escrito, esta tende a O diário de campo é importante porque o Assistente Social, na
ser mais passiva. A comunicação que se estabelece entre medida em que vai refletindo sobre o processo, pode perceber
locutor e interlocutor, embora possibilite reações e onde houve avanços, recuos, melhorias na qualidade dos
interpretações, não conta com a presença física do seu autor serviços, aperfeiçoamento nas intervenções realizadas – além
que, nessa forma de diálogo, fica à mercê da unilateralidade de de ser um instrumento bastante interessante para a realização
interpretação”. de futuras pesquisas. Ele é de extrema utilidade nos processos
Enquanto a comunicação direta, como o próprio nome diz, de análise institucional, o que é fundamental para localizar
permite uma intervenção direta junto ao interlocutor, a qualquer pro- posta de inserção interventiva do Serviço Social.
comunicação escrita possibilita que outros agentes tenham
acesso ao trabalho que foi desenvolvido pelo Assistente Social. Relatório Social
Sendo assim, os instrumentos de trabalho por escrito, não
raramente, implicam que outros profissionais e/ou outras Esse instrumento é uma exposição do trabalho realizado e
instituições desenvolverão ações interventivas a partir da das informações adquiridas durante a execução de
intervenção do Assistente Social. Por isso a necessidade do determinada atividade. Semanticamente falando, é o relato
texto estar bem escrito, claro e coerente, para que não haja dos dados coletados e das intervenções realizadas pelo
dúvidas quanto à mensagem que o Assistente Social quer Assistente Social.
emitir. O relatório social pode ser referente a qual- quer um dos
Contudo, a utilização dos instrumentos de trabalho por instrumentos face a face, bem como pode descrever todas as
escrito também possui uma funda- mental importância: é aqui atividades desenvolvidas pelo profissional (relatório de
que se torna possível ao Assistente Social sistematizar a atividades). Desse modo, os diferentes relatórios sociais são os
prática. Todo processo de registro e avaliação de qualquer instrumentos privilegiados para a sistematização da prática do
ação é um conhecimento prático que se produz, e que não se Assistente Social.
perde, garantindo visibilidade e importância à atividade Os tipos de relatórios produzidos pelo Assistente Social
desenvolvida. E mais: sistematizar a prática e arquivá-la, é dar são tão iguais à quantidade de possibilidades de realizar
uma história ao Serviço Social, uma história ao(s) usuário(s) diferentes atividades no campo de trabalho. Assim, qualquer
atendido(s), uma história da inserção profissional do

Conhecimentos Específicos 12
Apostila Digital Licenciada para Alice Caroline Guarino dos Santos - alice.guarino@hotmail.com (Proibida a Revenda)
APOSTILAS OPÇÃO

tentativa de classificação dos relatórios é tão-somente uma vulnerabilidades sociais. A partir dessa nova concepção foi
breve aproximação com essa gama de probabilidades. instituído o reconhecimento do direito universal,
Não é nosso objetivo aqui descrever detalhadamente como independente se o cidadão contribuísse com o sistema
se produz um relatório. Isso de- pende do objetivo do trabalho, previdenciário ou não. (YASBECK, 1997).
do tipo de atividade desenvolvida etc. Entretanto, retomando
a discussão de Magalhães, um dado é fundamental para Todo esse processo de ampliação do conceito de direitos
qualquer elaboração textual: o destinatário do texto – o agente sociais e de políticas públicas culminou na organização das
interlocutor. É importante saber para quem se escreve (e, definições das frentes de ação que caracterizariam o Sistema
portanto, escrever bem). É outro Assistente Social, um gestor, de Proteção Social brasileiro: Saúde, Previdência Social e
um profissional da área jurídica, um profissional da área Assistência Social, o qual é chamado hoje de tripé da
médica, um Psicólogo, um Administrador. Ou também o Seguridade Social, cada uma com suas respectivas atribuições
relatório pode ser produzido para o próprio Assistente Social no que toca o enfrentamento das expressões da questão social
– ou para a própria equipe de Serviço Social de onde o e na viabilização do acesso aos direitos, tornando-se uma
Assistente Social está desenvolvendo trabalho. Nesse sentido, política pública que significa direito do cidadão e dever do
cabe uma breve classificação entre relatórios internos (que Estado. Desta forma, a assistência social, pela primeira vez em
serão de uso e manuseio do Assistente Social ou da equipe que sua história, foi arquitetada como uma das três instituições
ele compõe) e relatórios externos (que serão de uso e políticas basilares da Seguridade Social. Avanço que expressa
manuseio de agentes exteriores à equipe). à superação do conceito de assistencialismo, da filantropia e
Outro dado também é fundamental nessa discussão sobre da benemerência social, e passa a constituir-se como a
o relato do trabalho. Não se trata de qualquer relatório, e sim, profissionalização da atividade pública, a qual não somente
de um relatório social. Isso repõe o debate sobre a inserção do está atrelada ao atendimento às necessidades básicas da
Serviço Social na divisão do trabalho – um profissional que população, como e, sobretudo junto à população em situação
trabalha com as diferentes manifestações, na vida social, da de risco e vulnerabilidade social.
“questão social”. Desse modo, os dados relatados são de Na Constituição Federal, os artigos 203 e 204 idealizam a
natureza social, isto é, as informações que dizem respeito a Assistência Social enquanto política, sendo a mesma de
essas características. responsabilidade do Estado e direito de todo cidadão. Como
ratifica o artigo primeiro da Lei Orgânica da Assistência Social
CONCEITO DE ASSISTÊNCIA SOCIAL ENQUANTO (LOAS), n° 8.742 de 7 de dezembro de 1993, que dispõe sobre
POLÍTICA PÚBLICA: uma construção lenta e desafiante 2 a organização da assistência social no Brasil;
A Assistência Social, direito do cidadão e dever do Estado,
Para problematizar a política de Assistência Social na é política de Seguridade Social não contributiva, que provê os
atualidade, faz-se necessário situar em breves linhas a sua mínimos sociais, realizada através de um conjunto integrado
retrospecção histórica, no sentido de compreender os desafios de ações de iniciativa pública e da sociedade, para garantir o
e dilemas presentes na sua operacionalização na atendimento as necessidades básicas. (BRASIL, Lei n° 8.742,
contemporaneidade. 1993).
Ao fazermos uma retrospectiva nos fatos concernentes ao A Lei Orgânica da Assistência Social atribui um caráter de
social, podemos perceber que a origem da assistência social no maturidade legal aos serviços socioassistenciais, tendo como
Brasil e no mundo, tem suas raízes na caridade, na filantropia instância de coordenação o Ministério do Desenvolvimento
e na solidariedade religiosa. Sendo que tais práticas Social e Combate à Fome (MDS). Tal lei instituiu o Conselho
compreendiam ações paternalistas e/ou clientelistas do poder Nacional de Assistência Social (CNAS) enquanto instância
público, favores concedidos aos indivíduos, pressupondo que máxima de deliberação. Suas competências principais consistem
tais pessoas atendidas eram favorecidas e não cidadãs ou em aprovar a Política Nacional de Assistência Social (PNAS),
usuários de um serviço ao qual tinham direito. Portanto, a normatizar e regular a prestação de serviços sejam eles de
assistência confundia-se com a benesse, ou seja, ajuda aos caráter público ou privado no campo da política em questão.
pobres e necessitados, configurando-se mais como uma Acompanhar e fiscalizar as entidades e organizações de
prática do que como uma política. assistência social, zelar pela efetivação do sistema participativo
O período histórico, a partir da década de 1980, e descentralizado; acompanhar e avaliar a gestão dos recursos,
configurou-se como um divisor de águas no que diz respeito ao divulgar no Diário Oficial da União todas as decisões, bem como
campo dos direitos sociais. Ocorreu nesse período um forte do Fundo Nacional de Assistência Social (FNAS), dentre outras.
engajamento e pressão da sociedade civil no que concerne à (BRASIL, CNAS, 2010).
discussão das políticas sociais, na qual denotou-se uma ampla
articulação dos movimentos sociais, principalmente no campo A LOAS como:
da Assistência Social. Segundo Rizotti (1998) referenciando- Lei inova ao afirmar para a Assistência Social seu caráter de
se em Cardoso (1994), os movimentos sociais contribuíram direito não contributivo (independentemente de contribuição à
para a conformação dos novos arranjos das políticas sociais Seguridade e para além dos interesses do mercado), ao apontar
brasileiras, fazendo sua inserção tanto na gestão quanto no a necessária integração entre o econômico e o social e ao
controle social. apresentar novo desenho institucional para a Assistência Social.
Foi a partir da Constituição Federal de 1988, que houve um (YASBECK, 2006).
reconhecimento dos direitos humanos sociais como um
avanço significativo. Pela primeira vez o homem brasileiro era Portanto, fica instituído que cabe ao Estado e suas
tratado como cidadão, como sujeito e possuidor de direitos, instituições consolidarem em rede uma política pública de
dentre os quais estava o direito à Seguridade Social. direito, rompendo com práticas remotas de benemerência e
Nesse sentido a Seguridade Social implica que todo filantropia.
cidadão tenha acesso a um conjunto de certezas e seguranças A PNAS, que é um documento normatizador das ações de
que venham cobrir, diminuir ou precaver os riscos e as assistência social, Resolução n° 145 de 15/10/2004, promove

2 SANTANA, E. P. de; SILVA, J. A. dos S. da; SILVA, V. S. da. http://www.joinpp.ufma.br/jornadas/joinpp2013/JornadaEixo2


HISTÓRICO DA POLÍTICA DE ASSISTÊNCIA SOCIAL: uma construção 013/anais-eixo8
lenta e desafiante, do âmbito das benesses ao campo dos direitos direitosepoliticaspublicas/pdf/historicodapoliticadeassistencias
sociais. Disponível em: ocial.pdf

Conhecimentos Específicos 13
Apostila Digital Licenciada para Alice Caroline Guarino dos Santos - alice.guarino@hotmail.com (Proibida a Revenda)
APOSTILAS OPÇÃO

principalmente a defesa e a atenção aos interesses e proverem as necessidades voltadas à “defesa dos direitos”,
necessidades às pessoas em situação de risco e/ou principalmente os direitos socioassistenciais.
vulnerabilidade social, cabendo a mesma, ações que Entretanto, no campo dos direitos sociais, houve um
promovam a prevenção, a proteção, a promoção e a inserção retrocesso decorrente do modo de produção capitalista com
social, como também um conjunto de garantias e seguranças. vistas ao modelo neoliberal, intensificado na década de 1990,
A mesma é construída a fim de integrar as demais políticas período marcado por fragilizações no âmbito das políticas
sociais, considerando as peculiaridades sociais e territoriais, sociais. São incorporadas novas percepções referentes aos
efetivando assim, a garantia dos mínimos sociais, bem como a direitos e a justiça social, novas formas de alocação de recursos
universalização dos direitos sociais (MDS, PNAS, 2004), e tem públicos e novos parâmetros de regulação ocasionando uma
como órgão gestor, em âmbito nacional o Ministério de série de reformas as quais acarretaram mudanças cruciais
Desenvolvimento Social e Combate à Fome (MDS). Fica nesse processo. Assim é importante frisar que no contexto de
instituído, também na LOAS em seus artigos 4° e 5º, enquanto emergência de práticas neoliberais, surgem novas
modelo de gestão, o Sistema Único da Assistência Social configurações da questão social tais como a fragilização das
(SUAS). relações de trabalho e o sucateamento do aparelho estatal no
O SUAS é fruto de um acordo federativo entre as três que tange a garantia da proteção social. Os quais resultam,
instâncias de governo (federal, estaduais e municipais) a fim dentre outros fatores, em ações e/ou posturas que tendem a
de promover uma gestão descentralizada no que toca o retroceder às práticas filantrópicas em uma nova roupagem
financiamento e monitoramento dos serviços que torna cada vez mais tênue o princípio da universalidade
socioassistenciais. Tendo como ponto de partida para sua dos diretos sociais. O contexto vigente,
implementação a Norma Operacional Básica (NOB/SUAS), (...) construiu para a Assistência Social um perfil ainda longe
aprovada pelo CNAS - Resolução n° 130 de 15/10/05. Tem proposto pela LOAS, perfil desarticulado que colocou em
como características principais a proteção social, a defesa de evidência um caráter seletivo, focalista e fragmentador para as
direitos socioassistenciais e a vigilância social, atribuindo a suas intervenções com medidas assistenciais meramente
política em questão uma nova lógica de organização das ações, compensatórias face aos efeitos dos ajustes estruturais da
classificadas em níveis de complexidade, considerando as economia (Yazbek, 2006)
peculiaridades territoriais e regionais. Tal ação visa viabilizar
um sistema participativo e descentralizado como forma de Nessa conjuntura há um difícil processo de ruptura com os
materializar a LOAS, tendo como finalidade garantir os traços assistencialistas que ainda pairam o campo da
princípios previstos pela Política de Assistência Social. assistência social. Há um retrocesso na década de 1990, ainda
Tomando como base a análise, supracitada, da que de forma velada, às práticas pautadas na filantropia e no
historicidade que perpassou a construção da proteção social voluntariado, as quais ainda persistem existir,
brasileira enquanto política pública de direito, é perceptível descaracterizando e depreciando todo um contexto de lutas
que foi um processo lento e gradual, mas, que teoricamente pela conquista dos marcos legais que norteiam a política
pode ser considerando como um modelo que evidencia um supracitada. Insistem ainda os resquícios da cultura moralista,
novo paradigma no que tange os direitos sociais. Entretanto, a qual culpabiliza o indivíduo por sua precária condição
faz-se necessário salientar, que mesmo com todos esses econômica e social. Desconsiderando assim, que vários são os
amparos legais ainda existe um déficit na efetivação de tal determinantes estruturais que compõem a totalidade da vida
política, principalmente no que se refere à negligência de social, os quais se pulverizam a partir de mediações e incidem
princípios básicos preconizados pela Lei: Universalidade; negativamente sobre a massa já destituída da riqueza
Supremacia do atendimento às necessidades sociais; Respeito socialmente produzida.
à dignidade do cidadão; Igualdade de direito no acesso ao Portanto, foram imprescindíveis os avanços conquistados
atendimento; Divulgação ampla dos benefícios, serviços, no que toca a política social referida, no entanto, a mesma
programas e projetos; entre outros; denotando assim, que ainda necessita romper com diversos paradoxos e retrocessos
ainda existe uma desafiante trajetória para o cumprimento do a fim de atingir uma posição de consolidação com vistas ao
que de fato está na teoria. fomento das potencialidades humanas, consubstanciadas na
Já de posse de uma sucinta explanação acerca dos marcos materialização e/ou viabilização dos direitos sociais.
legais que norteiam a Assistência Social, enquanto eixo
estruturante da Seguridade Social cabe trazermos uma Conclusão
apreciação acerca da referida política social em suas tímidas
inovações e intensos percalços na contemporaneidade. As análises trazidas a partir desse artigo reafirmam a
constatação da louvável evolução da Política de Assistência
A DICOTOMIA DA ASSISTÊNCIA SOCIAL: emancipação Social brasileira, sobre tudo ao que se refere ao seu cariz
x subordinação conceitual e operacional, os quais evidenciam uma construção
teórica de um modelo pautado na lógica da garantia de
Com o intuito de refletirmos sobre a atual condição da direitos. Construção essa, que vem efervescer perante os
Política de Assistência Social, faz-se necessário evidenciar cidadãos a certeza que o Estado tem como obrigação garantir
alguns de seus avanços e entraves na contemporaneidade. e subsidiar os mínimos sociais que deem condições dignas de
Como inovações na política em questão, é crucial citar a sobrevivência. Essa certeza é fruto dos avanços da política
reordenação de sua gestão, como um sistema político social brasileira consolidada, sobretudo, a partir da
administrativo descentralizado e participativo, abarcando os Constituição de 1988. Destarte, um longo caminho foi
três níveis do governo. Foram ampliados os espaços de percorrido com progressivos ganhos que vem beneficiando
participação política e social nos tramites de gestão e toda a sociedade e em especial as famílias em situação de risco
deliberação da política de assistência social, além de colaborar e/ou vulnerabilidade social, através de programas que
para o reconhecimento das peculiaridades (culturais, naturais, priorizam o combate à pobreza, e que, de fato, tem
históricas, dentre outras) de cada município. Abrindo cada vez conseguindo modificar, pelo menos em parte, a realidade da
mais o rol de ações articuladas às demais políticas sociais. Com ausência do mínimo necessário para a vida.
esses novos rearranjos são incorporadas também, a atuação de Todavia, embora haja no Brasil uma estrutura formal de
instituições privadas e mistas, como as organizações não proteção social, estabelecida pela Constituição como
governamentais, as quais são integradas ao processo por Seguridade Social – Assistência Social – o seu alcance ainda é
restrito e insuficiente; é valido salientar que existem ainda

Conhecimentos Específicos 14
Apostila Digital Licenciada para Alice Caroline Guarino dos Santos - alice.guarino@hotmail.com (Proibida a Revenda)
APOSTILAS OPÇÃO

muitos desafios a enfrentar, sejam pela ampliação das como, obediência ao princípio constitucional da Dignidade da
políticas, a universalização, a melhoria na qualidade dos pessoa humana.
serviços, há também a necessidade de capacitar e valorizar os Desse modo, resta claro que, aqueles que possuírem
trabalhadores, visto que são os agentes executores da política recursos financeiros a sua manutenção e subsistência, não
em questão, padronização de um mínimo de qualidade em fazem jus a percepção de qualquer benefício consistente em
todos os municípios para que possam desenvolver políticas pecúnia.
públicas comprometidas com a melhoria de vida de todos os
cidadãos e que possam , verdadeiramente, combater a Na Constituição Federal a assistência social é tratada da
pobreza e a desigualdade que ainda persistem em existir. seguinte forma:
Por fim, é válido ressaltar que esses avanços e retrocessos,
de maneira especial os retrocessos, são resultantes da atual “Art. 203. A assistência social será prestada a quem dela
conjuntura, a qual é marcada pela égide neoliberal e seus necessitar, independentemente de contribuição à seguridade
princípios, que priorizam os interesses do capital. Somando-se social, e tem por objetivos:
a esse fato, ainda persiste a herança da postura clientelista com I - a proteção à família, à maternidade, à infância, à
práticas paternalistas de muitos governos, resultando assim, adolescência e à velhice;
em ações da “Assistência Social” de cunho assistencialistas, II - o amparo às crianças e adolescentes carentes;
transformando o direito em benemerência e favor, ou até III - a promoção da integração ao mercado de trabalho;
mesmo com práticas pontuais e focalizadas. Para que de fato a IV - a habilitação e reabilitação das pessoas portadoras de
teoria seja o par dialético da prática, faz-se necessário que as deficiência e a promoção de sua integração à vida comunitária;
ações voltadas à Política de Assistência Social, sejam pautadas V - a garantia de um salário mínimo de benefício mensal à
nos ditames legais, esboçados anteriormente, assegurando a pessoa portadora de deficiência e ao idoso que comprovem
efetivação da política como direito. Convém ainda frisar a não possuir meios de prover à própria manutenção ou de tê-la
relevância da prática profissional do serviço social, no âmbito provida por sua família, conforme dispuser a lei.”
da Assistência Social.
O Assistente Social enquanto um profissional inserido na Seja da análise do art. 203 da CF/88 ou do art. 2º da Lei nº
divisão sociotécnica do trabalho é também um agente 8.742/93, o que se depreende é a preocupação estatal com
mobilizador e articulador de práticas essencialmente aqueles que, de alguma forma são ou estão desprovidos de
pedagógicas as quais podem contribuir significativamente qualquer condição econômica suficiente de se amparar
para fomento de um processo reflexivo/ pedagógico com autonomamente, ou seja, visa combater a pobreza, atendendo
vistas ao empoderamento de seus respectivos usuários, bem as necessidades e garantir os direitos sociais.
como dos trabalhadores da política de Assistência Social. A fim
de materializar os princípios básicos dos marcos legais que Da base de financiamento
norteiam a política supracitada e, consequentemente, instigar
o empoderamento dos usuários e/ou trabalhadores com vista Organizada administrativamente de forma
à emancipação humana e social. descentralizada, e coordenado pela União, a assistência social
é subvencionada por recursos oriundos da seguridade social,
ASSISTÊNCIA SOCIAL COMO DIREITO cabendo aos entes estatais, distritais e municipais a execução
dos programas sociais.
Assistência Social: uma garantia de atenção as
necessidades básicas do cidadão carente3 Do benefício

Instituída pela Constituição Federal de 1988, em seu art. - BPC


203, a Assistência Social é disciplinada pela Lei nº 8.742/93, e
conceituada como: Disciplinado pela Lei nº 8.742, Lei Orgânica da Assistência
Social, o respectivo benefício denominado de Benefício de
“Direito do cidadão e dever do Estado, é Política de prestação continuada/Amparo assistencial, embora não
Seguridade Social não contributiva, que provê os mínimos possua natureza previdenciária por não exigir a contribuição
sociais, realizada através de um conjunto integrado de ações de do beneficiário, é operacionalizado pelo INSS em razão de sua
iniciativa pública e da sociedade, para garantir o atendimento estrutura própria que abrange todo território nacional.
às necessidades básicas.” O benefício decorrente da assistência social consiste,
segundo o art. 203, V, da Constituição Federal, no pagamento
O campo de abrangência da assistência é diverso do da mensal de um salário mínimo, tendo como beneficiário direto
previdência, pois, aquela, segundo o art. 203 da Constituição o brasileiro e também o indígena, desde que idoso, idade igual
Federal, será prestada a quem dela necessitar, sendo ou superior a 65 (sessenta e cinco) anos, apesar da Lei n
desnecessária a sua vinculação a contribuição obrigatória. O 10.741/03 qualificar como idoso aquele que tiver 60
que garante a o auxílio assistencial é a necessidade do homem, (sessenta) anos, e a pessoa portadora de deficiência
e não a sua capacidade contributiva. impossibilitadas de proverem sua própria manutenção, nem
A necessidade, conforme entendimento do Ministro do de tê-la provida por sua família.
SFT, Gilmar Mendes, não deve ser aferida com base na renda Para a fixação da renda per capta, entende a LOAS que
de ¼ do salário mínimo, devendo o órgão responsável pela compreende a família apenas o requerente,
concessão do respectivo benefício, usar de outros fatores cônjuge/companheiro, pais, irmãos e filhos/enteados e
indicativos do estado de penúria do cidadão. tutelados, todos, desde que habitem sob o mesmo teto.
Apesar de o julgador poder ir atrás da verdade real dos Frise-se que, o respectivo benefício poderá ser concedido
fatos, deve-se pautar em parâmetros objetivos para a para mais de um membro de uma única família, devendo ser
concessão de direitos, analisando ponderadamente a devidamente comprovadas todas as exigências legais.
realidade social vivenciada pelo beneficiário, para não ser este Entretanto, para haver uma segunda concessão, há
e/ou seus dependentes restritos de uma garantia legal, tal divergência em um dos requisitos, pois, para uma nova

3 SEGUNDO, C. B. de A. A seguridade social e assistência social:

direito do cidadão e dever do Estado.

Conhecimentos Específicos 15
Apostila Digital Licenciada para Alice Caroline Guarino dos Santos - alice.guarino@hotmail.com (Proibida a Revenda)
APOSTILAS OPÇÃO

concessão a membros de uma família de um inválido Disciplinados pelo Decreto nº 6.917/09 e pela portaria
beneficiário, o valor percebido por este passa a integrar MDS 341/2008, os valores do referido programa deverão ser
formalmente a renda família para fins de cálculo da renda per pagos preferencialmente a mulher, sem acarretar qualquer
capta. violação ao princípio constitucional da isonomia.
Por inteligência do Estatuto do Idoso, a regra aplicável ao
deficiente não se aplica ao idoso, ou seja, para a concessão do A Política Nacional de Assistência Social – PNAS, o Sistema
benefício assistencial a um segundo idoso integrante do Único de Assistência Social – SUAS4
mesmo núcleo familiar, a renda percebida por outro, não serve
como base para cálculo da renda familiar. Sem dúvida, Assistência Social, como política de Proteção
O mesmo entendimento deveria alcançar a situação em Social, não contributiva, inserida constitucionalmente na
que sendo um casal de idosos, um receberia um benefício Seguridade Social brasileira, avançou muitíssimo no país ao
assistencial e o outro, um benefício previdenciário, desde que longo dos últimos anos, nos quais foram e vem sendo
o valor pago a título de benefício previdenciário seja implementados mecanismos viabilizadores da construção de
equivalente ao benefício assistencial. direitos sociais da população usuária dessa Política, conjunto
O BPC possui caráter personalíssimo, não podendo ser em que se destacam a Política Nacional de Assistência Social -
transferido, extinguindo-se, portanto, com a morte do PNAS e do Sistema Único de Assistência Social – SUAS.
beneficiário. Este conjunto, sem dúvida, vem criando uma nova
O auxílio também poderá ser extinto desde que, no prazo arquitetura institucional, ética, política e informacional para a
de dois anos, o beneficiário não se submeta ao procedimento Assistência Social brasileira e a partir dessa arquitetura e das
de avaliação da continuidade das condições que lhe deram mediações que a tecem podemos efetivamente, realizar, na
origem. esfera pública, direitos concernentes à Assistência Social.
Ademais, integram ainda as hipóteses de extinção do Nesta direção, o SUAS vem buscando incorporar as
benefício à superação das condições que lhe deram origem; demandas presentes na sociedade brasileira no que diz
morte presumida do beneficiário, declarada em juízo; ausência respeito à efetivação da assistência social como direito de
declarada do beneficiário; falta de apresentação de declaração cidadania e responsabilidade do Estado. Tem como principal
de composição do grupo e renda familiar no momento da objetivo a gestão integrada de ações descentralizadas e
revisão do benefício. participativas de assistência social no Brasil. Essa gestão
Questão relevante a ser discutida é a concessão de BPC ao supõe a articulação de serviços, programas e benefícios bem
estrangeiro residente em território nacional, uma vez que o como da ampliação de seu financiamento e o estabelecimento
estrangeiro naturalizado brasileiro e domiciliado no Brasil não de padrões de qualidade e de custeio desses serviços; supõe
coberto por sistema previdenciário de seu país de origem é também a qualificação dos recursos humanos nele envolvidos;
acobertado pela assistência social. a clara definição das relações público/privado na construção
Sabe-se que a saúde possui cobertura universal, o que da Rede socioassistencial; a expansão e multiplicação dos
também poderia se estender a assistência social, uma vez que, mecanismos participativos, a democratização dos Conselhos e
autorizada a permanência do estrangeiro no Brasil, este a construção de estratégias de resistência à cultura política
também poderia contribuir para o custeio do sistema. conservadora; e finalmente, exige que as provisões
O presente tema, também já foi alvo de discussão na África assistenciais sejam prioritariamente pensadas no âmbito das
do Sul, ficando decidido que, o país deve garantir aos seus garantias de cidadania sob vigilância do Estado, cabendo a
habitantes o atendimento mínimo existencial, uma vez que o este a universalização da cobertura e garantia de direitos e de
Estado não foi hábil em provar através de dados concretos, acesso para os serviços, programas e projetos sob sua
que, a inclusão de estrangeiros na esteira de beneficiários da responsabilidade.
assistência social, provocaria a falência no sistema No processo de construção do SUAS estamos nos
previdenciário. defrontando com uma pesada herança que marcou a história
A LOAS prevê ainda a concessão de eventuais pagamentos dessa área de Política Social e lhe atribuiu historicamente
a título de auxílio natalidade, morte e calamidade pública às características particulares dentro do contexto histórico da
famílias de baixa renda, visando reduzir as necessidades Proteção Social dos cidadãos brasileiros.
oriundas de situações excepcionais e temporárias. Atualmente, estas características estão permeadas por
valores e tendências, tanto conservadoras como
Dos programas sociais “emancipatórias”. É fundamental, todo o tempo, fortalecer
aquelas que operam a formulação da Assistência Social como
A assistência social vai além do Benefício de Prestação política pública regida pelos princípios universais dos direitos
continuada. Visando combater a pobreza extrema, o Governo e da cidadania.
Federal cria um programa chamado de Bolsa Família, Obviamente há muito para ser realizado nesse campo e a
consistente no pagamento de valores os beneficiários, com o luta pela construção democrática do Sistema, traz muitas
objetivo maior de garantir além de uma alimentação ao núcleo exigências, mas, sobretudo exige a gestão competente da
familiar, um acompanhamento de saúde e educacional dos política, bem como dos interesses, demandas e necessidades
menores. da população usuária da Assistência Social e dos recursos
O Bolsa família é uma uniformização de diversos outros humanos que nela trabalham.
programas, tais como o Bolsa Escola, Bolsa Alimentação, Bolsa
Escola, Auxílio Gás. Para alcançar seus objetivos, a Política de Assistência
Social deve ser realizada de forma integrada e articulada às
Apesar da atual legislação (Lei nº 10.836/04) prever que demais políticas sociais setoriais, para atender às demandas
para o cálculo da renda mensal familiar, os rendimentos de seus usuários. Essa definição está na LOAS, que pressupõe
oriundos de programas oficiais de transferência de renda para a assistência social o provimento dos “mínimos sociais”,
estão excluídos, não faz qualquer menção a respeito da cuja garantia exige a realização de um conjunto articulado de
acumulação de benefícios assistenciais. ações, envolvendo a participação de diferentes políticas
públicas e da sociedade civil. A LOAS afirma também, que o

4 Texto adaptado e extraído de YAZBEK, M. C. Sistema de Proteção

Social Brasileiro: Modelo, Dilemas e Desafios.

Conhecimentos Específicos 16
Apostila Digital Licenciada para Alice Caroline Guarino dos Santos - alice.guarino@hotmail.com (Proibida a Revenda)
APOSTILAS OPÇÃO

destinatário da “ação social” deve ser alcançável pelas demais argumento do crescimento da pobreza e à impossibilidade de
políticas públicas. Não podemos esquecer que o sujeito alvo sustentabilidade financeira de ambas as políticas que
dessas políticas não se fragmenta por suas demandas e requererem sistemas privados complementares e redução de
necessidades que são muitas e heterogêneas. Estamos serviços e benefícios.
tratando das condições de pobreza e vulnerabilidade que
afetam múltiplas dimensões de vida e de sobrevivência dos Vale salientar que essa hipótese somente tem validade se
cidadãos e de suas famílias. O Estado é o garantidor do considerarmos a seguridade social uma das mediações do
cumprimento dos direitos, responsável pela formulação das processo de reprodução social, um mecanismo que tem uma
políticas públicas, expressando as relações de forças presentes base material, fundado em necessidades objetivas, mas que, ao
no seu interior e fora dele. Evaldo Vieira em sua análise da transitar na esfera das superestruturas, como instituição
política social e dos direitos sociais afirma: “sem justiça e sem social e mecanismo de enfrentamento da desigualdade,
direitos, a política social não passa de ação técnica, de medida adquire um caráter ideológico e político. Segundo Neves, que
burocrática, de mobilização controlada ou de controle da trata sobre o assunto para discutir sobre a pedagogia da
política quando consegue traduzir-se nisto”. hegemonia na esteira do pensamento gramsciano, existe uma
indissociabilidade entre estrutura e superestrutura na
Contudo, sabemos que o desafio de construir Políticas história, embora tal não se dê de forma mecânica ao ponto de
Públicas e especialmente Políticas no campo da Proteção impedir qualquer possibilidade de relativa autonomia das
Social nesses tempos de crise e de transformações no relações superestruturais em condições históricas muito
capitalismo e na vida social não é fácil. precisas. Todavia, assinala a autora que, tal fato [...] não
Sabemos também que a extensão das proteções é um descarta, mas reforça a mencionada articulação entre os
processo histórico de longa duração, que funciona em grande planos, porque essa relação de reciprocidade entre forças
parte de mãos dadas com o desenvolvimento do Estado e as materiais e ideologias aponta para uma possibilidade concreta
exigências da democracia, e sem dúvida jamais esteve tão de o desenvolvimento das formações sociais capitalista ser
presente como hoje quando o direito a ser protegido [...] uma resultante da simultaneidade entre instrumentos de
exprime uma necessidade inscrita no cerne da condição do ser coerção, persuasão das forças político-sociais em disputa pela
humano...”. hegemonia político-social e alterações concretas nas forças
Como afirma Vieira: “Sem justiça e sem direitos, a política materiais de produção social.
social não passa de ação técnica, de medida burocrática, de
mobilização controlada ou de controle da política quando Por isso mesmo, ao tratarmos a seguridade social como
consegue traduzir-se nisto”. Não existe direito sem sua mediação da reprodução social, não estamos nos referindo
realização e sem suas mediações e a Política Social é sem especificamente à reprodução material da força de trabalho,
dúvida mediação fundamental, nesse sentido. Se entendermos mas à totalidade da esfera da reprodução das relações sociais,
que, no contexto de crise e na nova ordem das coisas, está em nela incluídas a reprodução das contradições sociais e as lutas
disputa, uma direção para a sociedade brasileira, cabe de classe. Longe de qualquer ilação, o processo histórico
interferir na construção dessa direção onde a medida sejam os ampara a nossa assertiva, indicando como a burguesia
interesses das classes subalternas na sociedade. Cabe brasileira, subordinando o Estado aos seus interesses, utilizou
construir sua hegemonia, criar uma cultura que torne medidas relacionadas à proteção social para legitimar-se.
indeclináveis seus interesses. Para isso é preciso enfrentar
desafios e nos desvencilhar de certas determinações e de É oportuno destacar que autoras como Ivanete
certos condicionamentos impostos pela realidade mesma em Boschetti e Beatriz Paiva fazem incursões sobre o tema da
que estamos inseridos e de algum modo limitados por ela. seguridade social, apresentando elaborações que nos
Estamos no olho do furacão. E, embora saibamos que escapa auxiliam ao tratar a tese central deste texto. Embora
às políticas sociais, às suas capacidades, desenhos e objetivos Boschetti não esteja discutindo o mérito da expansão da
reverter níveis tão elevados de desigualdade, como os assistência social em face da privatização da previdência e
encontrados no Brasil, não podemos duvidar das virtualidades da saúde, apresenta uma interessante discussão sobre o
possíveis dessas políticas. Elas podem ser possibilidade de modelo de seguridade social adotado no Brasil, criticando
construção de direitos e iniciativas de "contradesmanche" de a ausência de uma concepção universal de proteção social.
uma ordem injusta e desigual.
A autora identifica uma relação contraditória entre
previdência e assistência em função da vinculação das
3. seguridade Social e coberturas da previdência com a existência de vínculos com o
Assistência Social; trabalho assalariado e da assistência com os não-inseridos no
mundo do trabalho, os pobres. Afirma que a combinação entre
previdência – decorrente do exercício do trabalho – e
Seguridade Social Brasileira5. assistência aos pobres inaptos para o trabalho pode parecer
coerente e garantir proteção social universal nos países em
A tese central a ser trabalhada neste texto é a de que as que predomina ou predominou o Welfare, mas numa
políticas que integram a seguridade social brasileira, longe de sociedade do trabalho precarizado ela deixa completamente a
formarem um amplo e articulado mecanismo de proteção, descoberto os pobres economicamente ativos, ou seja, os que
adquiriram a perversa posição de conformarem uma unidade têm trabalho, mas com renda insuficiente para garantir suas
contraditória: enquanto a mercantilização da saúde e da condições de vida.
previdência precariza o acesso aos benefícios e serviços, a
assistência social se amplia, transformando-se num novo Paiva, por sua vez, reconhece que a expansão do
fetiche de enfrentamento à desigualdade social. desemprego impactou na perda de vínculos com a seguridade
Esta afirmação encontra amparo na expansão da contributiva (previdência), pressionando os sistemas de
assistência social e nas condicionalidades restritivas da proteção a acionarem “outras redes de segurança, mais
previdência e da saúde, movimentos que vêm sendo precisamente a assistência social”, e admite que alguns
sustentados por uma razão instrumental, circunscrita ao direitos preexistentes foram progressivamente abandonados

5 Texto adaptado de MOTA, A. E.

Conhecimentos Específicos 17
Apostila Digital Licenciada para Alice Caroline Guarino dos Santos - alice.guarino@hotmail.com (Proibida a Revenda)
APOSTILAS OPÇÃO

e “introduzidos mecanismos de mercadorização sutilmente sociedade brasileira, já que o cenário internacional era
incorporados”. Com esta colocação, a autora demonstra tanto adverso a qualquer iniciativa de intervenção social do Estado
o aumento da demanda por assistência social como sinaliza a em face do avanço das críticas neoliberais ao Welfare State nos
possibilidade de uma nova fratura na universalidade da países desenvolvidos.
seguridade social, que deveria ser assegurada pelo conjunto Essa disjunção entre as tendências internacionais e a
articulado das políticas de saúde, previdência e assistência que particularidade local apontava, ainda que “tardiamente”, para
a integram. a expansão e a consolidação de um sistema de seguridade
Ora, se historicamente a política de assistência social, no social, nele incluindo a Assistência Social como uma das
caso da seguridade social brasileira, ocupava uma posição políticas que, ao lado da Previdência e da Saúde, integravam o
residual, sua ampliação e reorganização, expressas na Política sistema. Essa expectativa, entretanto, a partir dos anos 1990
Nacional de Assistência Social e no SUAS, recomporiam a começa a ser ameaçada por uma agenda de prescrições
lógica da universalidade da proteção social brasileira. Neste neoliberais e de reformas voltadas à descaracterização dos
caso, tanto as considerações de Boschetti como as de Paiva recém-instituídos direitos sociais.
seriam, em tese, contempladas e com elas concordaríamos.
Mas não é isso que a realidade está apresentando: de um Numa sociedade cuja seguridade social ainda engatinhava
lado, o mercado passa a ser uma mediação explícita; de outro, rumo à universalização, esta ofensiva deve ser creditada:
a expansão da assistência recoloca duas novas questões: o
retrocesso no campo dos direitos já consolidados na esfera da
saúde e da previdência e a relação entre trabalho e assistência – Às condições particulares sob as quais a burguesia
social em tempos de desemprego e precarização do trabalho. brasileira, associada ao grande capital transacional,
O desdobramento no Brasil real parece indicar que, mais incorporou à sua ordem as necessidades sociais dos
uma vez, o grande capital utiliza o social como pretexto para trabalhadores, particularmente a partir dos anos 90,
ampliar seu espaço de acumulação, como discorre de um substituindo os mecanismos coercitivos por persuasivos,
jovem e promissor intelectual, o aumento dos investimentos após os grandes embates políticos dos anos 80, e
em uma política social para os pobres esconde a abertura de inaugurando novas táticas e estratégias de dominação;
novos e lucrativos mercados de investimentos para o capital – À conjuntura econômica e política do período, cujos
privado, em detrimento do serviço público. Assim é que, ajustes macroestruturais, determinados pela inserção
atualmente, a “inclusão dos excluídos” serve de discurso de subalterna do Brasil na economia internacional,
legitimação para o avanço do capital sobre os ativos públicos e ocasionaram os arranjos da restauração capitalista,
para o andamento das reformas neoliberais. implicando a reestruturação produtiva, os rumos da
Arma-se a burguesia de instrumentos para esgarçar a política econômica, a reforma do Estado e,
histórica relação entre trabalho e proteção social, visto que a enfaticamente, as injunções dos organismos financeiros
partir de então a tendência é ampliar as ações compensatórias internacionais, que impuseram as contrarreformas
ou de inserção, antes restritas àqueles impossibilitados de sociais de cunho neoliberal.
prover o seu sustento e, ao mesmo tempo, impor novas
condicionalidades de acesso aos benefícios sociais e materiais
nos casos de afastamento do trabalho por doenças, acidentes, Ora, a expansão da seguridade no pós-1964 já se fez
invalidez e desemprego temporário, para não falar da perda do mediante a fragmentação dos meios de consumo coletivo,
poder aquisitivo das aposentadorias e pensões por velhice, franqueando ao capital privado a prestação de serviços
morte, invalidez e tempo de contribuição/idade. No caso da considerados rentáveis, como foi o caso da saúde e da
saúde, a despeito do estatuto universal, a realidade aponta previdência via mercado de seguros, medicina de grupo e
para dois mecanismos: o do acesso a serviços privados como fundos de pensão. A estes, juntou-se o instituto da renúncia
parte dos benefícios ocupacionais oferecidos pelos fiscal, que, sob o argumento da colaboração empresarial,
empregadores e o da expansão de planos de saúde populares, estimulou as empresas a ofertar serviços sociais e benefícios
com oferta precária e restrita a serviços ambulatoriais de aos seus empregados.
baixo custo operacional. Este modelo criado pela ditadura permaneceu vigente com
a criação dos novos e mais abrangentes direitos
É importante lembrar que, embora a seguridade brasileira regulamentados pela Constituição de 1988. Tanto é assim que
pós-1988 incorpore a orientação das políticas de proteção o texto constitucional faz menção à existência de sistemas
social nos países desenvolvidos e que seguiram a tradição complementares tanto no caso da saúde como da previdência.
beveridgiana, no Brasil, a adoção da concepção de seguridade Dessa forma, ao criar as condições para institucionalizar a
social, como um sistema de proteção integrado e universal, não inclusão de alguns trabalhadores não-acobertados pelo
aconteceu nem do ponto de vista da sua organização sistema de proteção social, o Estado também facilitava a
administrativa e financeira, nem do posto de vista do acesso. abertura do mercado privado de serviços sociais. Enquanto
Sob a ótica do funcionamento, não se vislumbra nenhuma ampliava alguns benefícios e serviços, incluindo no sistema
iniciativa de mudança, permanecendo a estrutura atual com segmentos não-assalariados ou em situações sociais de
administrações e orçamentos distintos; do ponto de vista da precariedade, ao mesmo tempo, criava condições para o
cobertura universal, a despeito da nova política de assistência afastamento dos setores médios assalariados e autônomos
e dos inúmeros avanços que ela contém, tememos pela (principalmente profissionais liberais) do sistema público.
inversão da equação apontada originalmente por Boschetti: se
antes a centralidade da seguridade girava em torno da Se nos primeiros anos da década de 1990 esta era uma
previdência, ela agora gira em torno da assistência, que suposição, nos anos finais desse período as duas reformas
assume a performance de uma política estruturadora, e não previdenciárias e a realidade da saúde já tinham como dadas
como mediadora de acesso a outras políticas e a outros as possibilidades de um sistema básico e um complementar,
direitos, como é o caso do trabalho. como de fato veio a acontecer. O espólio da expansão
fragmentada e mercantil da seguridade criada pela ditadura
Apesar de tudo, é importante destacar que o momento militar vem sendo refuncionalizado, inclusive sob os auspícios
político que marcou a definição da Seguridade Social na do capital financeiro.
Constituição de 1988 deve ser reconhecido como expressão de A outra ordem de fatores responde pelas necessidades
resistência e autonomia dos setores progressistas da advindas do processo de reestruturação da economia

Conhecimentos Específicos 18
Apostila Digital Licenciada para Alice Caroline Guarino dos Santos - alice.guarino@hotmail.com (Proibida a Revenda)
APOSTILAS OPÇÃO

capitalista, tais como a globalização, a reestruturação sociedade a legitimidade do projeto societário formulado e
produtiva, uma nova divisão do trabalho e a reforma do colocado em execução [...]”. E continua a autora alertando para
Estado. Para intensificar a produtividade do trabalho, algumas o fato de que esse projeto supõe que os “incluídos” passem não
estratégias vêm sendo implementadas de modo a consolidar apenas a usufruir dos serviços sociais oferecidos, mas se
novos modos de o capital consumir e gerir a força de trabalho. transformem em colaboradores dos mecanismos de consenso,
Nesse ambiente, uma parcela de ex-trabalhadores que em situação contrária, poderiam representar uma ameaça
assalariados, agora considerados “pequenos ao status quo.
empreendedores” ou trabalhadores por conta própria, se
transformou em consumidores dos serviços disponíveis no Sob esta perspectiva, aloja-se a despolitização das lutas e
mercado, como é o caso dos seguros-saúde e planos de do caráter classista das desigualdades sociais, que passam a
previdência privada para os que conseguem pagar. Os demais ser entendidas ora como exclusão, ora como evidência da
desempregados e desorganizados politicamente engrossam as desfiliação em relação à proteção estatal, ambas confluindo na
fileiras do “mundo da pobreza”. Mais do que viabilizar medidas defesa de estratégias de inclusão e inserção, permitindo que o
que alteram o escopo da seguridade social brasileira inscrita existente se transforme em “ideal” e sitiando, assim, a
na Constituição de 1988, o que está em discussão é o próprio construção de projetos societais.
desenho da proteção social no Brasil, em face da construção de Mas até quando as classes dominantes e o seu Estado
um novo modo de tratar a “questão social” brasileira, focando- poderão tratar o processo de pauperização da população
a enquanto objeto de ações e programas de combate à pobreza brasileira como uma questão de assistência social?
à moda dos organismos financeiros internacionais, donde a
centralidade dos programas de transferência de renda. Seguridade Social, estratégias de classe e desafios ao Serviço
Social
A conjunção entre expansão pública e mercantilização faz
parte das proposições neoliberais, postuladas como Situamos a seguridade social na dinâmica da reprodução
assecuratórias da equidade, cujo princípio é dar mais a quem social e aventamos a hipótese de que está em processo de
tem menos. O que chama atenção é a capacidade que tiveram consolidação uma nova estratégia de dominação política: uma
as classes dominantes de capitalizar politicamente a nova reforma social e moral da burguesia, reveladora da sua
assistência social, transformando- a no principal instrumento pedagogia da hegemonia e que se realiza através do
de enfrentamento da crescente pauperização relativa, atendimento de algumas necessidades objetivas das classes
ampliando o exército industrial de reserva no seio das classes trabalhadoras, integrando-as à sua lógica reprodutiva. Essa
trabalhadoras. Em tal contexto, um dos instrumentos de reforma implica uma passivização da “questão social”, que se
repolitização da política, como parte da pedagogia da desloca do campo do trabalho para se apresentar como
hegemonia, consistiu em definir este segmento de classe como sinônimo das expressões da pobreza e, por isso mesmo, objeto
“excluídos” e os programas de assistência social como do direito à assistência, e não ao trabalho.
estratégia de inclusão. Se esta linha de reflexão e argumentação estiver correta,
Não é por acaso que a Assistência Social adquire dada a imanência da questão social ao processo capitalista de
centralidade como mecanismo de enfrentamento das reprodução social, as novas formas de acumulação e as
desigualdades sociais. Instala-se uma fase na qual a assistência inflexões por elas produzidas nas transformações em curso
social, mais do que uma política de proteção social, se constitui parecem afetar diretamente os meios de enfrentamento da
num mito social. Menos pela sua capacidade de intervenção questão social do que propriamente suas novas expressões. O
direta e imediata, particularmente através dos programas de que está posto é um novo tratamento e uma proposta de
transferência de renda, que têm impactos objetivos no enfrentar a questão social.
atendimento dos mínimos sociais de subsistência para a Não temos dúvidas de que a seguridade social é produto
população pauperizada; e mais pela sua condição de ideologia histórico das lutas do trabalho, na medida em que respondem
e prática política, robustecidas no plano superestrutural pelo pelo atendimento de necessidades inspiradas em princípios e
apagamento do lugar que a precarização do trabalho e o valores socializados pelos trabalhadores e reconhecidos pelo
aumento da superpopulação relativa têm no processo de Estado. Quaisquer que sejam seus objetos específicos de
reprodução social. intervenção, sua institucionalização depende tanto do nível de
socialização da política conquistado pelas classes
Na impossibilidade de garantir o direito ao trabalho trabalhadoras como das estratégias do capital na incorporação
(postulado inexistente na sociedade regida pelo capital), seja das necessidades do trabalho, configurando-se historicamente
pelas condições que ele assume contemporaneamente, seja como um campo de disputas e negociações na ordem
pelo nível de desemprego, ou pelas orientações burguesa.
macroeconômicas vigentes, o Estado capitalista amplia o
campo de ação da Assistência Social ao mesmo tempo em que Razão pela qual ela é sempre e continuamente objeto de
limita o acesso à saúde e à previdência social públicas. Não se investidas do capital, no sentido de “adequá-la” aos seus
trata de uma visão estreita ou residual da política de interesses; as investidas do mercado financeiro em
Assistência Social – seja ela concebida como política setorial transformar os serviços sociais em campos de investimento e
ou intersetorial – o que está em discussão é o estatuto que ela negócios lucrativos é uma prova cabal. Por isso mesmo, aquilo
assume nessa conjuntura. que se afigura como déficit para a população transforma-se em
A rigor, não podemos mistificar a assistência social pelo demandas de mercado, objeto do processo de
fato de ela ser uma política não contributiva voltada aos que supercapitalização, como é o caso dos serviços médico-
dela necessitam. O nosso tratamento da assistência não se dá hospitalares, das escassas vagas para a educação superior
por essa diferenciação, mas pela centralidade que ela ocupa, pública, dos baixos rendimentos de aposentadoria etc.
neste momento, no conjunto da Seguridade Social. Assim, frações dirigentes da classe dominante –
As classes dominantes invocam a política de Assistência paralelamente à crise político organizativa dos que vivem do
Social como solução para combater a pobreza relativa e nela seu trabalho – constroem, na atualidade, outra reforma social;
imprimem o selo do enfrentamento da desigualdade ao tempo não mais sustentada pelo pacto fordista-keynesiano, nem pela
que exercitam a sua condição de classe dirigente, visto que “os exclusividade da mercantilização, mas hipotecando à
movimentos definidos como de repolitização da política sociedade e às políticas de combate à pobreza a solução para o
objetivam mostrar a camadas cada vez mais consideráveis da enfrentamento do pauperismo. Por isso mesmo, também as

Conhecimentos Específicos 19
Apostila Digital Licenciada para Alice Caroline Guarino dos Santos - alice.guarino@hotmail.com (Proibida a Revenda)
APOSTILAS OPÇÃO

novas conceituações de questão social e das políticas de social”, aliás, um novo e instigante tema para a agenda política
seguridade social passam a ser chaves para esgarçar qualquer do Serviço Social brasileiro.
tentativa de vinculação entre pobreza e acumulação da
riqueza. Como afirmado em outra ocasião, reside neste processo o
Ocorre um verdadeiro transformismo nos conteúdos que desafio de superar a aparência dos fenômenos, identificando
informam a questão social, passando a mesma a adquirir as múltiplas determinações do real. Sem desconsiderar as
novos significados, tais como: questão social como exclusão; condições objetivas que estão envolvidas na prática
questão social como objeto da política social; questão social profissional e que requerem conhecimentos específicos que
como ausência de cidadania e direitos sociais; questão social deem conta das particularidades e singularidades da
como desemprego. Essas tendências estão subjacentes às Assistência Social, reafirmamos uma célebre citação marxiana
respostas presentes no conjunto dos programas e iniciativas para justificar a importância da formação profissional no
vigentes no atual perfil da seguridade social. desvelamento da realidade sobre a qual incidirá a política de
Assistência Social: se aparência e essência coincidissem, todo
Em nossa análise e avaliação, esse conjunto de o esforço do conhecimento seria inútil.
abordagens pragmáticas da questão social é responsável Se é inconteste a impossibilidade de o capitalismo superar
por um dado modo do seu enfrentamento – modo as sequelas sociais e materiais do processo de acumulação,
constituidor da hegemonia do capital, formador de cultura, também o é a possibilidade de utilizar meios para enfrentá-las
e que se espraia como possibilidade de tratamento teórico no limite da ordem burguesa, permitindo que proprietários e
e, especialmente, de proposições de seu enfrentamento trabalhadores sejam tornados cidadãos sujeitos de direitos. A
dentro da ordem, dando corpo e substância à hegemonia luta por direitos e a institucionalização da seguridade social se
burguesa. dão no limite da emancipação política na sociedade brasileira,
sem que isso elida os confrontos de classe e suas dimensões
Amparada pela naturalização da ordem, a burguesia busca, políticas.
dentre outros objetivos, transformar o cidadão sujeito de
direitos num consumidor; o trabalhador, num contribuinte Essa afirmativa não implica a subtração da dimensão
autônomo; o desempregado, num beneficiário da assistência civilizatória da seguridade social, tampouco o fato de ela ser
social; e a família e as comunidades, em células de uma resultante das lutas sociais dos trabalhadores. Aqui, salvo
“sociedade solidária, socialmente responsável e cooperativa”. melhor juízo, é necessário fazer uma distinção entre as
Ao absorver as demandas derivadas do agravamento das especificidades de uma prática político-organizativa, que
condições de vida e trabalho da população brasileira através mesmo respondendo às legitimas necessidades postas pela
de políticas compensatórias, como é o caso da Assistência realidade, pode não condensar uma consciência teórica,
Social, o Estado brasileiro define sua principal estratégia de subsumindo o ideal ao real. Ou, em outros termos, abdicando
enfrentamento da “questão social”. Nesses termos, a da emancipação humana em prol dos limites históricos da
Assistência Social passa a assumir, para uma parcela emancipação política. No dizer de Lessa, “[...] a emancipação
significativa da população, a tarefa de ser a política de proteção política é o ‘enorme progresso’ de constituição histórica da
social, e não parte da política de proteção social. sociabilidade regida pela propriedade privada burguesa. A
Na conjuntura atual, esta dimensão compensatória é emancipação humana, por sua vez, é a superação da
redimensionada em função do crescimento do desemprego e propriedade privada e a constituição de uma sociabilidade
das massas de trabalhadores supérfluos para o capital. Como comunista”.
tal, a assistência está assumindo um papel na esfera da
proteção social, que termina por suprir necessidades que Política de Previdência Social Brasileira.
seriam do âmbito de outras políticas e constitutiva de uma luta
que mobiliza os trabalhadores desde os idos do séc. XIX: o A Previdência Social é o seguro social para a pessoa que
direito ao trabalho. contribui. É uma instituição pública que tem como objetivo
reconhecer e conceder direitos aos seus segurados. A renda
Estas constatações parecem apontar para algo que não transferida pela Previdência Social é utilizada para substituir
vem sendo posto no debate: a Assistência Social no séc. XXI a renda do trabalhador contribuinte, quando ele perde a
está adquirindo a condição de mecanismo integrador, em lugar capacidade de trabalho, seja pela doença, invalidez, idade
do papel desempenhado pelo trabalho. Aqui o maior destaque avançada, morte e desemprego involuntário, ou mesmo a
fica por conta dos programas de transferência de renda e pela maternidade e a reclusão.
condição de política estruturadora que a PNAS tenderá a Previdência Social é o resultado de contribuições feitas por
assumir nos municípios brasileiros. No nosso entender, esta é trabalhadores para prover alguma subsistência na
a maior tensão presente na Política de Assistência Social, haja incapacidade de trabalhar.
vista a impossibilidade estrutural de ela assumir este papel. Dentre as políticas públicas desenvolvidas pelos governos
Não seria leviano admitir que o desenvolvimento desse nacionais, uma parte delas visa atender ao que se denomina
processo responde por uma aliança invisível entre a como elementos constituintes dos direitos sociais. Estes
manutenção do antirreformismo, a preservação da dinâmica englobam lazer, educação, cultura e seguridade. Por sua vez, a
capitalista e o atendimento de necessidades imediatas da seguridade social consiste em um conjunto de ações com o
população, sem que se projete qualquer iniciativa que reponha intuito de amparar o cidadão e sua família em casos de velhice,
a centralidade do trabalho, seu valor de uso social no horizonte doença e desemprego, prestando serviços de saúde pública e
do enfrentamento das desigualdades sociais, geradoras das de assistência social. É dentro dessa política de seguridade
necessidades da assistência social. Para isso, utilizaram uma social que se inclui a Previdência Social.
tática transformista que desmonta e desqualifica os projetos,
as ideologias e as práticas vinculadas ao campo do trabalho. A Previdência Social é organizada sob contribuições
Nesse sentido, coloca-se o maior desafio para os que previdenciárias obrigatórias que seguem princípios como a
professam “o pessimismo da razão e o otimismo da vontade”: universalidade da participação e cálculo dos benefícios e
distinguir e compreender a necessidade objetiva da ampliação preservação do seu valor real. O recolhimento das taxas
da assistência diante do agravamento da pobreza, sem previdenciárias é socialmente importante porque assegura
hiperdimensionar seu papel no enfrentamento da “questão alguma proteção e dignidade, contribuindo para a redução da
pobreza, é economicamente importante porque arrecada muito

Conhecimentos Específicos 20
Apostila Digital Licenciada para Alice Caroline Guarino dos Santos - alice.guarino@hotmail.com (Proibida a Revenda)
APOSTILAS OPÇÃO

por ano e é politicamente importante porque colabora com uma único limite real são os limites de soberania de outras
paz social. instituições sobre estas práticas sociais. Toda instituição tende
a ampliar seu âmbito de ação, e se estenderia indefinidamente
No Brasil, a Previdência Social é administrada pelo até esbarrar com o âmbito de outras instituições, de modo que
Ministério da Previdência Social, criado no dia primeiro de o âmbito de uma instituição é definido pela história das lutas
maio de 1974. Suas ações são executadas pelo Instituto de sua institucionalização, no conflito com outras instituições.
Nacional do Seguro Social (INSS). Os trabalhadores O âmbito do Serviço Social portanto não é fixo, nem
contribuem com seus tributos diretamente ou por meio de definido a priori pelos profissionais e professores do Serviço
seus empregadores, que repassam o valor dos impostos já no Social, e sim pelo contexto, que é conflitante. Os atores
pagamento dos salários. Em geral, todos os trabalhadores institucionais se dividem em internos e externos.
estão submetidos a um mesmo regime de contribuição Atores institucionais internos “são aqueles que
previdenciária, à exceção de funcionários públicos, diretamente “fazem” a instituição, desenvolvendo
Organizações Não-Governamentais e organizações religiosas. positivamente sua ação no quadro de um aparelho
Embora o Estado se encarregue do recolhimento dos tributos, determinado”. São os agentes em geral (em geral
os trabalhadores ainda podem optar por sistemas de profissionais) e a clientela. São subcategorizados de acordo
previdência privada, que possuem suas próprias regras e com o grau de “saber” em relação ao objeto institucional, o que
valores de contribuição e repasse dos benefícios. O recurso é lhes confere poder e posição estratégica em relação à ação
utilizado por aqueles que se interessam em garantir uma institucional.
renda extra em casos de necessidade, já que a previdência Agentes privilegiados são aqueles cuja prática concretiza
privada não substitui a pública e, por isso, é chamada também plenamente a ação institucional. Têm um saber pleno quanto
de complementar. ao objeto institucional e a partir desse saber uma posição
Mediante contribuição para a Previdência Social, o estratégica em relação à ação institucional.
trabalhador recebe amparo em casos de doença, invalidez, Os assistentes sociais são agentes privilegiados em
velhice, morte e maternidade, concedendo auxílios, instituições de Serviço Social, como a antiga LBA, o SESC, o
aposentadorias e pensões. SESI. Porém nada garante que os profissionais que dão nome a
uma instituição sejam seus agentes privilegiados. É necessária
uma análise concreta para elucidação da prática institucional.
4. prática profissional x Por exemplo, em muitas instituições religiosas assistenciais o
agente privilegiado é o padre.
prática social x prática Agentes subordinados (ou agentes institucionalmente
institucional x avaliações; complementares) são atores cujas práticas também estão
ligadas a ação ou objeto institucional, mas estão subordinados
aos agentes privilegiados. Têm saber relativo quanto ao objeto,
mas mantêm uma posição estratégica em relação à ação
A Prática Profissional do Assistente Social na
institucional do ponto de vista de seu âmbito.
Instituição e Análise Institucional6.
O Assistente Social é agente subordinado em organizações
institucionais como hospitais e hospícios. No hospício o agente
Prática Institucional
privilegiado é o psiquiatra, mas como a saúde mental (o objeto
institucional) depende da ação do assistente social junto à
Estritamente falando, a ação institucional nada mais é do
família, à comunidade, às relações sociais do doente mental, o
que a prática de um de seus agentes, mandantes, clientes etc.
assistente é chamado a agir em nome do objeto institucional,
Ou, mais precisamente, o resultado de relações sociais entre
porém subordinado ao poder-saber do psiquiatra.
agentes e mandantes ou agentes e clientes, e assim por diante.
Ora, a prática de cada um desses atores é, com frequência,
Agentes organizacionais (ou “pessoal institucional”) são
divergente, e muitas vezes complementar e contraditória com
aqueles que prestam serviços indispensáveis à manutenção da
respeito à de outros. A prática institucional é, portanto, a
organização. Eles não são, portanto, ligados à ação
resultante das práticas conflitantes dos diversos atores.
institucional e sim à ação organizacional. Como agentes, não
A prática institucional é resultado da prática de seus
têm um saber sobre o objeto institucional.
agentes concretos e só existe quando encarnada em atores
Em algumas empresas industriais, o Serviço Social
concretos que constituem. Para definir os elementos que
trabalha apenas na administração de benefícios aos
estruturam a prática institucional, vamos categorizar o objeto
trabalhadores. Nelas, o assistente social trabalha tão afastado
institucional, o âmbito institucional e os atores institucionais.
do objeto institucional (a produção) que pode se tornar um
Objeto Institucional é aquilo sobre cuja propriedade a
agente organizacional.
instituição reivindica o monopólio da legitimidade. O objeto
Clientela é o conjunto de indivíduos atingidos efetivamente
institucional não é um objeto material, mas imaterial,
pela ação institucional. São os atores cuja relação com a
impalpável, pois é esse caráter que facilita a sua apropriação
instituição é objeto da ação institucional. É a população
pela instituição e sua tendência a alargar indefinidamente os
atingida ou o “público-alvo”. Nega-se que os clientes tenham
limites de seu âmbito.
“saber” sobre o objeto institucional. Na medida do possível, o
O objeto institucional da instituição Serviço Social é muito
cliente está reduzido à passividade (nega-se, no mais das
discutido ao longo da sua história. Weisshaupt formula a
vezes, sua condição de sujeito), no limite, o cliente é tornado
hipótese de que o objeto institucional do Serviço Social será
“paciente”.
algo como a cidadania efetiva da população, entendida como o
Quando o assistente social, agente subordinado, trabalha
uso adequado dos aparatos de Estado.
numa empresa, sua clientela são os empregados da própria
empresa, portanto seus colegas; quando trabalha num
O assistente social tem a função política de mediar as
hospital, sua clientela não tem vínculo empregatício com esta
relações entre a população e o Estado no usufruto de seus
organização institucional, são os pacientes do hospital; quando
aparatos.
trabalha em instituições do Estado, em programas de
Âmbito Institucional compreende as relações e práticas
comunidade, sua clientela é o público da instituição. Logo,
sociais concretas que sustentam o objeto institucional. Seu

6Texto adaptado de SANTOS, M. F. Professora de Serviço de Social.

Conhecimentos Específicos 21
Apostila Digital Licenciada para Alice Caroline Guarino dos Santos - alice.guarino@hotmail.com (Proibida a Revenda)
APOSTILAS OPÇÃO

pelos exemplos, o assistente social pode ter clientela distinta (institucional), que têm uma concessão temporária de mando
da clientela da organização institucional em que trabalha. e que, portanto, pode ser retirada a qualquer momento, de
Quando o assistente social agente privilegiado trabalha em acordo com o estatuto da organização: são cargos de confiança
uma instituição de Serviço Social, sua clientela é a clientela da do mandante. Nas pequenas empresas privadas, o dono pode
sua própria instituição. Esses exemplos são para mostrar como ser presidente ou diretor, portanto mandante funcional,
o assistente social pode ter diferentes implicações com sua patrimonial e dirigente ao mesmo tempo.
clientela.
Atores institucionais externos (ou contextuais) são Os mandantes assentados na relação institucional
agentes que não têm ação direta nas instituições, estão fora (mandantes institucionais propriamente dito – mandantes, em
delas, mas podem interferir “negativamente” nelas. “Os nome do “saber”, definidos na instância “ideológica”) são os
mandantes podem paralisar a ação institucional, impedindo os guardiões da legitimidade de que a instituição se reveste. São
agentes institucionais de agir, mas não podem agir em seu os que se apropriam do saber relevante para a instituição e
lugar, senão tornando-se, por sua vez, agente institucional”. decidem quais são as práticas permitidas ou proibidas,
São os mandantes e o público. Os mandantes assentam seu segundo um critério de “verdade”.
poder na apropriação dos meios de produção e reprodução Mas instituições de Serviço Social os guardiões da “verdade
social, de acordo com a metodologia que categoriza as relações profissional” são os Conselhos Regionais de Serviço Social, os
sociais em três instâncias: econômica, política e ideológica. O CRESS’s, e o Conselho Federal de Serviço Social, os CFESS’s (e
poder do mandante deriva da: as escolas também, até certo ponto – a “academia”). O
assistente social está submetido a estes órgãos mesmo que
1. Apropriação dos meios de produção e reprodução trabalhe subordinado em uma instituição de outra área. O
material da sociedade na relação de propriedade, de acordo conflito entre os vários mandantes propicia um grau de
com o direito de propriedade capitalista que rege a relação autonomia aos agentes internos.
material entre os agentes sociais; Público é o conjunto dos atores coletivos ou individuais
2. Apropriação dos meios de produção e reprodução para quem a ação institucional é visível, podendo
social da sociedade na relação de hierarquia, de acordo com eventualmente integrar a clientela. Por exemplo, um posto de
a constituição político-jurídica que rege a relação de mando assistência médica tem todo o bairro como público, mas só os
social; pacientes são clientela. Outro exemplo, num hospital, os
3. Apropriação dos meios de produção e reprodução pacientes são clientela e seus parentes podem ser
imaginária de sociedade na relação de autoridade, via de simplesmente público.
regra geral em nossa sociedade, de acordo com a Observa-se que a opinião do público pode ser mais decisiva
racionalidade científica que rege a relação cultural central do que a opinião da clientela para a prática institucional. Poe
de valorização dos saberes dos agentes sociais. exemplo, no programa de privatização do atual governo, a
opinião pública tem sido testada na condução do processo de
Os mandantes assentados na relação de propriedade venda das estatais. Num posto de assistência médica, a
(mandantes patrimoniais – mandantes, em nome do “ter”, associação de moradores pode ter influência na política de
definido a partir da instância econômica) são os que sustentam atendimento. O público, nas democracias, deveria ser o
economicamente a instituição (garantem a sua reprodução “mandante privilegiado” das instituições. O público
material) e, baseados nessa propriedade, exigem uma identificar-se-ia com a totalidade das relações sociais.
prestação de contas em relação à ação institucional e, em
particular, dos agentes privilegiados, que se apropriam apenas Análise da prática institucional e Serviço Social
institucionalmente (não materialmente) do objeto. Esses
mandantes, portanto, se apropriam não do objeto As relações entre o assistente social, o usuário e seu objeto
institucional, mas do que a instituição produz da prática estão submetidos a limites organizacionais e
economicamente. institucionais, e situados em um dado contexto histórico. Sua
Nas instituições públicas, o mandante patrimonial é o autonomia fica sujeita às relações de propriedade, saber e
Estado, seja no âmbito federal, estadual ou municipal. Nas poder entre os atores; fica limitada pelo ponto de vista da
instituições privadas é o Capital (os acionistas ou os donos). O racionalidade da organização, da legitimidade da instituição e
dono pode ser uma pessoa jurídica, isto é, uma empresa é dona da articulação “contextual” que determina o lugar do Serviço
de outras empresas, por exemplo. Uma situação em que alguns Social na divisão sociotécnica do trabalho.
atores concretos podem desempenhar vários papéis Desta forma, o assistente social se queixa que suas
institucionais ocorre, por exemplo, nos clubes, associações e dificuldades estão ligadas a uma deficiência teórica no período
cooperativas, em que os agentes sócios, associados, de formação, que não permite dar conta da complexidade da
cooperandos podem ser ao mesmo tempo, clientes, agentes e prática, e desloca sua queixa para questões técnicas. É
mandantes patrimoniais, porque são, coletivamente, os frequente o aluno de Serviço Social se queixar da falta de
proprietários. ferramental para a realidade de seu estágio, porém a questão
operacional no Serviço social é até privilegiada em termos de
Os mandantes assentados na relação de hierarquia técnicas, pois as pega de vários campos de conhecimentos
(mandantes funcionais – mandantes, em nome do “poder”, diferentes.
definidos a partir da instância política) são aqueles que
nomeiam o corpo de agentes institucionais e, sobretudo, dos No cotidiano da prática, junto às questões emergentes,
agentes que exercerão a direção da organização institucional. pode parecer ao assistente social que lhe falte técnica para
Nas instituições públicas esses mandantes são os exercer sua profissão. A questão é que, para aplicar qualquer
representantes do Estado (o governo, mandatário, tem um técnica, o profissional está submetido às normas
mandato legitimado pela Constituição que lhe confere um organizacionais, a saberes institucionais e ao contexto
poder por um tempo estipulado). Nas empresas privadas histórico. O assistente social critica o contexto histórico (a luta
capitalistas, os donos ou o corpo de acionistas, apoiados na de classes), mas não estuda concretamente as relações entre
legislação comercial e fiscal, exercem diretamente, ou delegam saber e poder e a micropolítica que se dá dentro de uma
o direito de administrar a produção a um agente que deverá organização institucional. Toda prática profissional tem uma
prestar contas de sua gestão. Note-se que os dirigentes não são dimensão técnica, uma dimensão institucional e uma
mandantes, são agentes, internos à organização dimensão contextual. Falta, portanto, o estudo da dimensão

Conhecimentos Específicos 22
Apostila Digital Licenciada para Alice Caroline Guarino dos Santos - alice.guarino@hotmail.com (Proibida a Revenda)
APOSTILAS OPÇÃO

institucional da prática profissional: “Concretamente, continuado de avaliação e monitoramento “são providências


evidencia-se um problema de formação, já que a urgentes e ferramentas essenciais a serem desencadeadas
representação implícita do campo profissional transmitida na para a consolidação da PNAS e para a implementação do
escola, em geral, não corresponde às condições institucionais Sistema Único de Assistência Social - SUAS. Trata-se, pois, de
de fato impostas à prática”. construção prioritária e fundamental que deve ser coletiva e
envolver esforços dos três entes da federação”. Assim as
Avaliação de Programas Sociais7. igrejas, entidades, ONG´s têm papel investigativo e de alta
relevância à sociedade.
A partir da compreensão da complexidade das políticas Segundo Evelyne Leandro, em seu artigo “Avaliação e
públicas e de sua importância na construção da hegemonia monitoramento de projetos” as ONG´s em sua maioria
política, a avaliação e monitoramento dos projetos sociais nos procuram atentar mais focadamente ao funcionamento do
aspectos macro e micros sociais, torna processo de avaliação e serviço público por meio do fortalecimento dos conselhos
monitoramento etapa crucial, principalmente os de relevância gestores e movimentos sociais. Mas com a inserção do capital
social. privado, como também na participação de entidades não
Avaliação e monitoramento são concebidos segundo governamentais, entidades associativas, igrejas na prestação
Samira Kauchakje, em seu livro Gestão Públicos de Serviços de serviços públicos, sobretudo no controle social, recai sobre
Sociais, como: as tais, a responsabilidade implementar processos
continuados de avaliação e monitoramentos dos projetos e
“(...) processo contínuo, estando presente em todos os programas sociais, que garantam a transparência na aplicação
momentos do projeto. Permite identificar problemas no dos financiamentos. Logo as iniciativas privadas (igrejas,
desenvolvimento do projeto e suas possíveis causas, de modo a associações etc.) também se tornaram alvo de monitoração e
antecipar medidas para a recondução das atividades conforme avaliação por parte do governo e da sociedade em geral.
o planejado ou de acordo com necessidade de novos Para Eveline a monitoração e a avaliação precisam
direcionamentos”. acontecer respectivamente, pois:

Logo, avaliação enquanto processo continuado e “O monitoramento consiste, basicamente, em acompanhar o


sistematizado traz aos projetos a possibilidade de andamento do projeto no dia-a-dia (...) possibilita a
readequação as diversas variações das premissas que podem identificação de problemas e possibilita solução. A avaliação,
levar ao sucesso ou a ineficiência de sua execução. Conforme por sua vez, destina-se a verificação dos indicadores
Samira, a avaliação não deve ser executada como etapa final de quantitativos e qualitativos (...) onde são postos à prova os
implementação de projetos, mas devem ser conjugados a cada mecanismos de gestão adotados pela entidade. A avaliação
etapa de execução. permite identificar pontos críticos e proporciona a resolução
Neste sentido, destaca Kauchakje a correlação quanto ao desses antes que comprometam o resultado final do projeto.”
objetivo da avaliação e aquilo que se deseja dar visibilidade.
Destacam-se as seguintes metodologias e seus objetivos: Verifica-se então que o sucesso do projeto está
intimamente ligado a um bom processo de monitoramento e
a) Avaliação de processo _ relação custo e benefício _ eficácia avaliação do mesmo atrelado à capacidade de interpretação e
e eficiência; problematização das premissas que lhe são inerentes.
b) Avaliação de impacto _ impacto sobre o diagnóstico _ Destaca-se ainda que o acesso a financiamentos sociais, ou
efetividade; a continuidade de seu fornecimento tem como pré-requisito
c) Avaliação política _ processo de aprendizado _ impactos básico a transparência do emprego destes recursos, que só
sociais; podem ser obtidas através da contínua avaliação e
monitoração dos serviços executados pelos mesmos.
Evidencia-se, portanto, a inevitabilidade de se diagnosticar Contudo, a avaliação social teve no Brasil seu tardio
a situação na qual imerge o objeto de intervenção missionária reconhecimento enquanto ferramenta de validação de
e assim se possa inferir medidas, ações, alterações no projeto projetos sociais. A importância, embora indiscutível na
confessional que está sendo executado a fim de que este atualidade, é contraposta às poucas inovações postas nos
alcance os resultados planejados. projetos sociais.
Da mesma forma, pode-se atribuir a capacidade de A crescente demanda por serviços públicos, advindos do
explicitar possíveis premissas que oferecem risco ou podem acirramento das desigualdades sociais, da falta de acesso às
potencializar a missão, à atividade avaliativa (inserida no riquezas produzidas pela sociedade, traz ao
processo). São a partir destas práticas que se desenham ações programa/projeto, profissional na assistência social, a
interventivas que culminarão no êxito do programa ou de um dificuldade de se reservar tempo para validar dados, elaborar
projeto. documentos, sistematizar atendimentos, reduzindo a atuação
profissional ao atendimento das demandas sociais.
De avaliadores e monitoradores a avaliados e monitorados
A incorporação de meios de avaliação e monitoração de
A sociedade civil, segundo Gramsci, tem sua definição projetos ou de programas assistenciais representa um grande
delineada enquanto “Estado Ético” como classe (aparelho desafio aos assistentes sociais. A capacidade de focar
privado, no qual a igreja constitui-se como meio) que busca a atividades e delas extrair indicadores sociais requer do
hegemonia e tem por meio do consenso e da coerção, programa/projeto o domínio metodológico e um acúmulo
estratégias de galgar o poder. Por conseguinte configura-se teórico-prático das diversas dimensões da assistência social.
imprescindível sua fiscalização do funcionamento do O fortalecimento da igreja, da sociedade civil e dos
equipamento público seja na monitoração ou na avaliação movimentos sociais em espaços que possibilitem o controle e
através dos conselhos gestores. a participação social é essencial à consolidação das políticas
Segundo a PNAS no item 3.1.7 sobre a Informação, públicas e legitima meios democráticos do país.
Monitoramento e Avaliação, contata-se e o processo

7 Texto adaptado de RIBAS, S. Gestão de Políticas, Programas e

Projetos Sociais.

Conhecimentos Específicos 23
Apostila Digital Licenciada para Alice Caroline Guarino dos Santos - alice.guarino@hotmail.com (Proibida a Revenda)
APOSTILAS OPÇÃO

É preciso, portanto buscar a superação de ações cujo profissional de Serviço Social, e é orientado pela perspectiva
planejamento equivocado às necessidades sociais que metodológica histórica-estrutural.
acarretarão no desperdício do financiamento público, da Nas formulações a nível de Comunidade, o campo de
igreja, dos mantenedores, além de reproduzirem diferenças atuação do Serviço Social, são observadas ambas as tendências
sociais que assolam a população brasileira. referidas acima, com características bipolares acentuadas. A
primeira decorrente da influência norte-americana e a
DIAGNÓSTICO segunda, produto do movimento de reconceituação latino-
americano. Essas duas tendências do Serviço Social serão
No Serviço Social, o termo diagnóstico8 vem sendo denominadas nesse trabalho de “ajustamento social” e
conceituado por diversos autores com interpretações “transformação social” respectivamente.
variadas. Alguns baseiam-se em sua etimologia, outros
afastam-se dela, definindo-o como uma interpretação, uma Na América Latina, apesar da influência marcante dessas
descrição ou explicação de uma situação ou de um sujeito ou, propostas, verifica-se, a partir da década de 60, a presença de
ainda, como tentativa de compreensão da realidade. uma nova alternativa de prática, deslocando o polo de
Há, no entanto, insuficiência de estudos que esclareçam, intervenção da disfunção social para a proposta de
com maior precisão e profundidade, os aspectos teóricos e transformação das relações sociais. O conceito de comunidade
metodológicos do diagnóstico em Serviço Social. nesta perspectiva se modifica, nega a identidade de interesses,
Este texto constitui uma tentativa de abordagem do tema salienta o interesse de classe, os conflitos e as contradições,
numa perspectiva crítica, tendo como escopo fornecer considerando-os como força motriz para a transformação
elementos para novas discussões e estudos em torno dos social
discursos correntes na literatura do Serviço Social. O diagnóstico em Serviço Social, em qualquer dessas
Pode-se observar mediante o estudo das tendências de vertentes, dá-se através da relação do Assistente Social e a
conhecimento e ação do Serviço Social, frente à realidade realidade da comunidade-base.
social, fundamentações distintas originadas nas grandes Nessa relação profissional x comunidade o momento em
correntes metodológicas, segundo as orientações que predomina o conhecimento denomina-se classicamente
funcionalistas empiristas, de um lado, e histórico-estrutural, de diagnóstico. O diagnóstico é uma construção, ou seja, uma
de outro. Os conhecimentos e as ações se diversificam em produção de conhecimentos e significa um processo de
decorrência destas distintas orientações teórico-valorativo, reflexão na realidade social, área de atuação, integrando o
procedimentos metodológicos e objeto de conhecimento e fazer profissional. Essa prática de produção de conhecimentos
ação. tem uma estrutura que lhe dá o caráter de totalidade e consiste
De acordo com Carlos Boggio, o conceito de diagnóstico no num conjunto de elementos interdependentes e que
Serviço Social está relacionado intimamente com o próprio constituem uma unidade: referencial teórico-valorativo,
Serviço Social, que se vem reformulando historicamente. procedimento metodológico e objeto de conhecimento e
Segundo a concepção clássica do /serviço Social como ação.
tecnologia, o diagnóstico é considerado uma etapa do processo
de planejamento linear da ação, antepondo-se à programação. O referencial Teórico-Valorativo compreende os
Apresenta uma listagem bastante exaustiva de dados dos mais conceitos que auxiliam na interpretação da realidade social;
variados aspectos com vista a identificar uma determinada estes estão impregnados de conceitos valorativos, de natureza
realidade. Neste caso, a postura que tem orientado o ético-normativa e ideológica. Tais pressupostos fundamentam
diagnóstico caracteriza-se pela influência das disciplinas os Assistentes Sociais na sua atuação e, especificamente, no
terapêutica e pelas perspectivas metodológicas empirista e diagnóstico, seja na definição dos aspectos a serem estudados,
funcionalista. Cabe ao assistente social, dentro dessa no procedimento adotado para a sua análise e na própria
orientação, julgar aquilo que constitui problemas ou interpretação da realidade.
necessidades da clientela. Essa postura se vincula a uma Os Procedimentos Metodológicos consistem num
perspectiva de adaptação e ajustamento social. conjunto de passos ou regras que orientam a formulação do
O diagnóstico em Serviço Social de comunidade, por diagnóstico em Serviço Social a nível de comunidade ou nos
exemplo, conforme se apresenta em alguns posicionamentos momentos de aproximativos da realidade social que
mais recentes, constitui-se num componente inerente a configuram o processo de conhecimento e ação.
própria ação e tem como características: Por Objeto de Conhecimento e Ação compreende-se um
conjunto de fenômenos sociais determinados ou que emergem
- Fazer parte de um processo desenvolvido pela na relação entre o Assistente Social e população de uma área
população residente na comunidade-área de atuação do específica de atuação. Poderá se constituir em categorias
Serviço Social, tendo o Assistente Social como preestabelecidas a serem identificadas na realidade social ou
coparticipante; em categorias a serem configuradas no decorrer da ação.
- É um processo desenvolvido conjugadamente pela
população e profissional, não sendo, predominante, uma O Documento de Araxá, de uso corrente no Brasil, explicita
fase do método utilizado pelo Assistente Social; esta postura quando enuncia os postulados e princípios que
- Os aspectos da realidade a serem estudados são fundamentam a ação do Serviço Social.
determinados pela população sujeita do processo; A
interpretação da realidade da comunidade local vincula-se Postulados:
com a sociedade global, situada e datada historicamente;
- O diagnóstico, como toda a prática nessa perspectiva a) postulado da dignidade da pessoa humana: que se
se dirige para fins de transformação das relações sociais. entende como uma concepção do ser humano numa posição de
eminência ontológica na ordem universal e ao qual todas as
Nesta concepção de transformação social o diagnóstico coisas devem estar referidas;
constitui-se em etapas sucessivas e aproximativas da b) postulado da sociabilidade essencial da pessoa humana:
realidade, vivenciadas de forma conjunta pela população e o que é o reconhecimento da dimensão social intrínseca à
natureza humana e, em decorrência do que se afirmar, o

8 Texto adaptado de CARDOSO, E.

Conhecimentos Específicos 24
Apostila Digital Licenciada para Alice Caroline Guarino dos Santos - alice.guarino@hotmail.com (Proibida a Revenda)
APOSTILAS OPÇÃO

direito de a pessoa humana encontrar na sociedade, as movimento de reflexão-decisão-ação que o caracteriza vai se
condições para sua a auto realização. realizando de acordo com as seguintes aproximações. São elas:
c) Postulado da perfectibilidade humana: compreende-se
como o reconhecimento de que o homem é, na ordem - 1ª a 5ª aproximação: Reflexão
ontológica, um ser que se auto realiza no plano da Delimitação do objeto/reconstrução do objeto; Estudo de
historicidade humana, em decorrência do que se admite a situação; Construção de referenciais teórico-práticos;
capacidade e potencialidades naturais dos indivíduos, grupos, Levantamento de hipóteses preliminares e Coleta de dados.
comunidades e populações para progredirem e se auto - 6ª a 8ª aproximação: Decisão
promoverem. Organização e análise; Identificação de prioridades de
intervenção e Definição de objetivos e estabelecimento de metas.
Princípios Operacionais: - 9ª a 13ª aproximação: Ação
Planificação; Implementação; Implantação e execução e
a) estímulo ao exercício da livre escolha e de Definição de parâmetros de avaliação e Controle.
responsabilidade de decisões;
b) respeito aos valores, padrões e pautas culturais; Estas aproximações são apresentadas nessa sequência,
c) anseio à mudança nos sentido da autopromoção e do mas na prática esses processos muitas vezes alteram essa
enriquecimento do indivíduo, grupo, da comunidade, das ordem. Compreende-se que as aproximações da 1ª a 5ª
populações; relacionam-se como fase de reflexão, as aproximações de 6ª a
d) atuação dentro de uma perspectiva de globalidade na 8ª são reconhecidas como fase de decisão e as aproximações
realidade social. de 9ª a 13ª são reconhecidas como fase de ação. Podemos
assim entender que o processo de planejamento faz parte de
PLANEJAMENTO DE TRABALHOS E PROJETOS uma contínua análise, ou seja, se inicia com a reflexão de uma
SOCIAIS9 situação e simultaneamente o processo, devendo ser este
contínuo, cíclico e reflexivo. Marx denomina este processo de
Nossa vida cotidiana é cercada de planejamento, o homem união do pensamento e da ação como práxis social. É neste
em sua essência planeja desde que toma consciência de sua cotidiano que estabelecemos a compreensão para que as
importância e contribuição para o meio social. Assim, serão decisões elaboradas no planejamento sejam concluídas.
apresentadas neste trabalho as aproximações que compõe o A realidade social, ou seja, a práxis, é determinante das
processo de planejamento e que exige do planejador o relações sociais, fato que engloba aspectos políticos e
movimento de reflexão-decisão-ação. O planejamento é um econômicos. Dessa maneira, entendemos que é dentro da
processo racional que precisa de uma sequência antecipada de realidade que o planejamento torna-se etapa indispensável
tempo não se manifestando em um dado momento, mas sim para que se chegue a um resultado final dentro do processo.
realizando-se de foram contínua ao longo da história passando Para tanto, detalharemos todas as três fases do Planejamento
por vários estágios. outrora mencionado. São elas:
O planejamento social é um instrumento de fundamental
importância para o desenvolvimento de trabalho do - (Re)Construção do objeto
profissional de Serviço Social, pois este tem necessidades de
conhecer e compreender a realidade do planejamento para Essa aproximação é o primeiro passo para o processo de
que o profissional consiga realizar intervenções com planejamento social. É necessário saber o que planejar e qual
qualidade. Serão apresentadas as etapas do planejamento, a o segmento da realidade que será colocado em desafio, inicia-
divisão de conceitos e a importância do mesmo para a se assim essa etapa compreendida como processo de reflexão.
profissão e para o profissional. Nesta etapa consideramos a realidade onde será formulado o
conjunto de proposições para uma intervenção mais
A importância do planejamento social qualificada, e ao delimitarmos o objeto de intervenção,
estaremos olhando de fora para dentro, confrontando as ações
O presente texto se apropria das ideias de Marx para planejadas com as mudanças que ocorrem na realidade.
compreender seu método de interpretação da realidade social É importante ter em mente qual planejamento que será
para, então, aplicá-lo no âmbito da profissão de serviço social. realizado em uma realidade, em qual conjuntura, em qual
O modo como Marx compreende a práxis do planejamento, organização especifica e quais práticas que serão utilizadas, e
possibilita entender de maneira mais completa a totalidade assim têm-se a capacidade de formular mudanças.
social e todas as suas contradições. Marx compreende que o A reconstrução do objeto é o movimento que traduz a
ato de planejar é de natureza do ser humano, o ser humano relação, a ação e o conhecimento. Segundo Baptista, "o
projeta em sua mente o ato para depois executar e antes de profissional precisa se preparar, [...] conhecer suas
executar qualquer ato ele planeja. Isso é consciência representações, seus sistemas e valores, suas noções e práticas
teleológica. [...]". A cada mudança que o planejador faz ele está construindo
O planejamento social busca utilizar de forma harmônica o um novo conhecimento sobre novas situações e esse processo
planejamento estratégico, ampliando a participação dos vários é cíclico e constante em todas as relações sociais.
níveis profissionais existentes dentro da sociedade. Nesse - Estudo de situação
sentido, a tomada de decisão se torna elemento fundamental, Segundo Baptista, o estudo de situação compõe-se da
pois corresponde com as diferentes escolhas dentro do descrição interpretativa, da caracterização, da compreensão e
processo. Um elemento importante no planejamento social é a da explicação de uma situação para o planejamento,
operacionalização, onde relaciona as atividades necessárias determinando suas limitações. O estudo da situação é o
para efetuar as decisões tomadas. Nessa fase o planejador conjunto de informações que provém em contribuição para
social (o assistente social) deve acompanhar a implantação, o tomar decisões, ampliando o conhecimento das realidades
controle e a avaliação do planejamento do projeto social que o concretas. O objeto do planejamento não pode ser tratado
mesmo for implantar em determinada instituição pública ou separadamente, devem se levar em consideração as propostas
privada. O planejamento é um processo contínuo e dinâmico, que estejam abertas às modificações perante a sociedade.
tendo o planejamento como uma decisão de planejar o

9 Texto adaptado de VIEIRA, R. C.

Conhecimentos Específicos 25
Apostila Digital Licenciada para Alice Caroline Guarino dos Santos - alice.guarino@hotmail.com (Proibida a Revenda)
APOSTILAS OPÇÃO

Alguns objetivos, segundo Mattelart podem ser considerados empiricamente e sem qualquer especulação ou mistificação - a
como estudo de situação: conexão entre a estrutura social e política e a produção."

"Configuração do marco de situações ou de antecedentes, - Coleta de dados


acompanhada de análise compreensiva e explicativa de suas
determinações; a Identificação sistemática e contínua das áreas A partir da busca e do levantamento de informações, dá-se
críticas e de necessidades, a que se pode acrescentar, ainda, de início a coleta de dados, onde o planejador consegue relacionar
oportunidade e de ameaças; a Determinação de elementos que as informações que já foram anteriormente organizadas,
permitam justificar a ação sobre o objeto; o Estabelecimento de consegue programar as investigações e pesquisa as situações
prioridades; a Análise dos instrumentos e técnicas que podem que forem necessárias para que haja um aprofundamento para
ser operados na ação; a Identificação de alternativas de iniciar a tomada de decisão.
intervenção.". A coleta de dados deve se ater a alguns aspectos como:
coleta de dados de situação; dados da instituição; dados das
O planejamento é um processo feito a partir de uma políticas públicas, da legislação, do planejamento jurídico, da
realidade, de um estudo de uma situação, que deve ser rede de apoio existente e os dados da prática. Os dados de
considerado sob o panorama de um conjunto dinâmico de situação constituem-se em objetivos do estudo. Estes
informações durante esse processo. Esse conjunto de estabelecem a compreensão do objeto que está em ação,
informações deverão se constituir em recursos básicos e estabelecendo assim a natureza e a problemática desse objeto.
permanentes para o planejamento da ação, localizando, Os dados levantados estão ligados à realidade e com os fatores
compreendendo, controlando e prevendo as situações de um de ordem social, econômica e cultural que compõem os
modo geral, fornecendo elementos que permita mostrar problemas e as possibilidades. Sendo assim os dados de
alternativas de intervenção. O estudo da situação nada mais é situação procura entender a dimensão do objeto da ação que
que a reflexão, a compreensão, a explicação e a expressão de está em questão, como está sendo percebido pelas pessoas,
causas antes dos dados da realidade em relação ao seu pela sociedade e pelos profissionais.
conjunto de aspectos especiais.
- Organização e análise
- Construção de referenciais teórico-práticos
A fase de organização e análise inicia um novo processo
Para a construção de referenciais teóricos temos que que não se desmembra dos demais, todavia inicia uma nova
entendê-los como conhecimentos que alimentam o estudo das fase que é a da decisão. É nessa fase que o planejador social
situações para que haja o planejamento e podem ser de deve se aprofundar na observação, pois nessa fase que se
natureza: cientifica, documental, técnicos e periódicos. Seu descrevem os dados e os interpreta de maneira organizada,
principal objetivo é a análise e a exposição da realidade que sistemática e analítica. Observar se os dados obtidos durante a
será planejada. Nesse processo precisamos conhecer a investigação são suficientes para proporcionar respostas ao
realidade de um modo geral, torna-se necessário a união do objetivo proposto. Depois destas observações damos início ao
pensamento e da ação, a realidade social apresenta várias processo de olhar atentamente para os dados coletados
dimensões, sendo elas: sociais, culturais, psicológicas, políticas
e econômicas. - Definição de objetivos e estabelecimento de metas
Os estudos devem ser organizados de uma forma simples
e clara para que possa ser confrontado com os dados A definição dos objetivos e o estabelecimento das metas
concretos, sendo assim deve se fazer uma operacionalização dão o real sentido e fundamento ao processo de planejamento.
dos conceitos que serão ou foram trabalhados. A A função específica do objetivo é posicionar a organização,
operacionalização dos conceitos é o estabelecimento da orientar a ação, definir o ritmo do planejamento, motivar os
relação entre os elementos da situação juntamente com os atores envolvidos no processo, facilitar a avaliação de
elementos que não são observáveis. Para operacionalizar os desempenho e incorporar a racionalidade, entre outros. Este
conceitos é necessário observar o objeto de estudo e os fatos a pode também ser classificado em três aspectos: Se é um
ele relacionados, com a observação de estudo podem-se objetivo geral ou específico; a média de tempo previsto a ser
encontrar diferentes elementos para serem trabalhados. Desta utilizado; se é a longo, médio ou curto prazo e a delimitação de
maneira, segundo Baptista o profissional deve sempre forma, se são quantitativos e/ou qualitativos. Dessa maneira,
compreender essa realidade posta de forma que "essa teoria os objetivos e metas "deverão ser conquistados para
possibilite formular seu esquema de análise trazendo-lhe transformar a visão em realidade, [...] (pois) buscam alcançar
referência, supostos, concepções amplas [...] que lhe vai resultados especificados em um tempo pré-estipulado".
permitir apreender a realidade". Após todas as etapas anteriores que foram a Reflexão e a
Decisão, temos uma nova etapa que não se desmembra das
- Levantamento de pressupostos demais, mas as completa de forma sucinta e eficiente que é a
fase da Ação.
O levantamento de pressupostos ou hipóteses é
estabelecido na hora da elaboração teórica podendo ser - Planificação
desenvolvido de forma implícita ou explícita, como o
planejamento social trabalha com o ser humano, e por esses No processo de planejamento, e a planificação é realizada
estarem sob um processo de evolução constante, os resultados depois de passar por um conjunto de decisões. Decisões essas
ficam aproximados e não exatos, uma vez que há mudanças que são sistematizadas, interpretadas e detalhadas em
constantes nas estruturas sociais e políticas a que os documentos que apresentem níveis de decisão composto por
indivíduos estão inseridos. Assim, segundo Marx, diversificados planos, programas e projetos.
"O fato, portanto, é o seguinte: indivíduos determinados, O plano demarca as decisões gerais do sistema, suas
que como produtores atuam de um modo também estratégias e suas diretrizes e deve ser formulado de forma
determinado, estabelecem entre si relações políticas e sociais clara e simples. São organizados os objetivos e metas.
determinadas. É preciso que, em cada caso particular, a O programa detalha os documentos por setores, ou seja, faz
observação empírica coloque necessariamente em relevo projeções detalhadas das informações que são consideradas

Conhecimentos Específicos 26
Apostila Digital Licenciada para Alice Caroline Guarino dos Santos - alice.guarino@hotmail.com (Proibida a Revenda)
APOSTILAS OPÇÃO

específicas em relação aos níveis, modalidade e especificação profissional, ou seja, o profissional deve investigar e analisar a
do setor. realidade para assim propor uma intervenção eficaz.
O projeto estabelece-se como documento que estabelece Para o profissional de serviço social o planejamento deixa
um plano prévio da operação de um conjunto de ações, é de ser um método de estudo e passa a ser um procedimento
também a racionalização das decisões. Na planificação o importante para a profissão, torna-se instrumento essencial
projeto vem antes da indicação dos resultados, é instrumento para compreender a profissão que trabalha com e na
que está mais próximo da execução, sendo assim faz-se o realidade, profissão esta que precisa repensar suas práticas
detalhamento das atividades que serão desenvolvidas para atender as mais diversas realidades e expressões da
estabelecendo-se os prazos e especificando os recursos que questão social que surgem no cotidiano profissional.
serão utilizados. Para Zanoni e Bogado "como planificação da
ação, o projeto pressupõe a indicação aos resultados Referência:
perseguidos. É o instrumental mais próximo da execução, BAPTISTA, Miryan Veras. Planejamento social:
devendo detalhar as atividades a serem desenvolvidas, intencionalidade e instrumentalização. São Paulo: Veras,
estabelecer prazos e especificar recursos". 2000

- Implementação
5. laudos, relatórios e
Para a implementação, destacamos a estratégia como um pareceres;
conjunto de ações e está ligada à realidade do homem desde os
primórdios, sendo que esta se constitui em um conjunto amplo
de ações e providências "destinado a viabilizar o seu avanço Elaboração de estudo social, relatório, laudo e
com a maior segurança possível [...]. Mobilizando, motivando e parecer10.
condicionando colaboradores para atingir um elenco de
objetivos previamente estabelecidos". Através do estudo social, o assistente social busca
Dessa maneira, a implementação abrange a coordenação e identificar as condições de vida dos sujeitos trazendo uma
a integração de todas as áreas do planejamento de um amostra documentada da realidade de indivíduos envolvidos
processo, precisa da colaboração, da competência técnica e em situações que podem requerer intervenção profissional. O
gerencial de todos os envolvidos para a resolução das objeto de investigação desse profissional é a realidade social
problemáticas, assim faz-se necessário a articulação e vivida pelo sujeito, ou seja, nos seus aspectos econômicos,
interdisciplinaridade para que haja a implementação das familiares, culturais e comportamentais, priorizando o
soluções. contexto e não os fatos. Cabe-lhe conhecer com profundidade
e de forma crítica uma determinada situação, buscando
- Implantação e execução estabelecer as interrelações dos diversos fatores que a
constituem e apresentar com mais verossimilhança a
A implantação é instituir na prática todas as ações que já realidade investigada.
foram planejadas, arquitetadas e avaliadas ao longo do tempo,
passando a cumprir os passos dos objetivos estabelecidos de A profissão tem como compromisso ético a contribuição
forma criteriosa para que essa execução se dê de forma para a garantia dos direitos humanos e sociais e a efetiva
contundente. Todavia, é naturalmente compreensível que proteção da pessoa em situação de vulnerabilidade. O
onde o plano estabelecido for implantado tornar-se alvo de assistente social deve buscar o estado atual dos fatos, por
insatisfação e/ou incompreensão por razões de meios científicos de captação, armazenagem, averiguação e
peculiaridades, uma vez que ao se trabalhar com pessoas, exposição dos mesmos apresentando o resultado do estudo
atores de sua própria história haja opiniões distintas. com as interpretações, diagnóstico e parecer, frutos de um
processo metodológico específico do serviço social, na forma
- Parâmetros de Avaliação e Controle de relatório ou laudo social.
Na construção de um estudo social deve-se ter em mente
A avaliação é o caminho onde o planejador poderá aferir a que, mesmo quando se trabalha com apenas um usuário, ele é
efetividade e o impacto que sua ação e decisão tiveram sobre um indivíduo social, e a realidade social que condicionou a sua
as outras etapas do processo. Por ser este processo dinâmico história, bem como o fato que motivou a realização do estudo,
e contínuo deve ser pautado sempre na reflexão. Já o controle devem ser trazidos à tona por competência do assistente
é o instrumento que verifica o que já foi previsto e o que está social.
acontecendo. O controle define os parâmetros de avaliação, o
estabelecimento e a verificação dos pontos de controle, a Técnicas utilizadas para realização do estudo social
correção dos possíveis desafios e a reflexão contínua do
processo em análise. - Entrevistas.
Mesmo que por algum motivo o planejador social venha a - Visita domiciliar.
se desviar de seus propósitos iniciais há que se voltar atrás, - Observação.
avaliar o refletir cautelosamente a respeito das decisões - Análise de documentação.
tomadas a respeito de seu trabalho, para que futuras falhas e - Informações e entendimentos com colaterais ou
erros não impeçam o contínuo andamento desse processo que entidades de bem estar social da comunidade.
deve ser dinâmico e responsável em todas as esferas da vida
social dos cidadãos que dele necessitarem. Relatório Social
Contudo, o planejamento é um importante aliado ao É uma apresentação descritiva e interpretativa de uma
exercício do trabalho profissional, pois permite antecipar situação ou expressão da questão social, enquanto objeto da
possíveis e certas mudanças do ambiente externo em que a intervenção desse profissional, no seu cotidiano laborativo.
sociedade está inserida continuamente. O planejamento deve Seu uso se dá com a finalidade de informar, esclarecer,
ser tratado como um processo primordial ao trabalho subsidiar, documentar um auto processual relacionado a
profissional, pois é um método aplicado para a intervenção alguma medida protetiva ou sócio educativa, prevista no ECA

10 Texto adaptado de FÁVERO, E. T.

Conhecimentos Específicos 27
Apostila Digital Licenciada para Alice Caroline Guarino dos Santos - alice.guarino@hotmail.com (Proibida a Revenda)
APOSTILAS OPÇÃO

ou enquanto parte de registros a serem utilizados para a quanto ao processo realizado e as exigências institucionais e
elaboração de um laudo ou parecer. Como qualquer outra legais, estabelecendo os procedimentos adotados para a
documentação do âmbito da profissão, os princípios éticos realização do estudo e, inclusive, socializando com o usuário a
devem guiar a escolha do que é pertinente ou não de se conclusão do estudo realizado. (Desmistificando assim o
registrar em documento que permaneçam em prontuário aparato e o poder institucional). Lúcia Lopes ressalta que a
próprio do Serviço Social e naqueles que serão expostos à pertinência da emissão do parecer ou a emissão por iniciativa
análise de outro agente, ou que poderão vir a público. própria deverá ser “o conhecimento acerca da burocracia
institucional, dos direitos dos usuários e, sem dúvida, da
Laudo Social. possibilidade de viabilização de direitos através desse
É mais um elemento de “prova”, com a finalidade de dar instrumento”.
suporte à decisão judicial, a partir de uma determinada área
de conhecimento. Ele, na maioria das vezes contribui para a Perícia Social
formação de um juízo por parte do magistrado no sentido dele A perícia, quando solicitada a um profissional de Serviço
ter elementos que possibilitem o exercício da faculdade de social, é chamada de perícia social, recebendo esta
julgar, a qual se traduz em “avaliar, escolher, decidir”. Esse denominação por se tratar de estudo e parecer cuja finalidade
laudo oferece elementos de base social para a formação de um é subsidiar uma decisão, via de regra, judicial. Ela é realizada
juízo e tomada de decisão que envolve direitos fundamentais por meio do estudo social e implica na elaboração de um laudo
e sociais. e emissão de um parecer. Para sua construção o profissional
Ele possui a seguinte estrutura: faz uso dos instrumentos e técnicas pertinentes ao exercício da
profissão, sendo facultado a ele a realização de tantas
- Introdução: que indica a demanda judicial e objetivos. entrevistas, contatos, visitas, pesquisa documental e
- Identificação breve dos sujeitos envolvidos. bibliografia que considerar necessárias para análise e a
- A metodologia interpretação da situação em questão e a elaboração de
- Um relato analítico parecer. Assim, a perícia é o estudo social, realizado com base
nos fundamentos teórico-metodológicos, ético-político e
Parecer Social. técnico operativo, próprios do serviço social, e com finalidade
Diz respeito a esclarecimentos e análises, com base em relacionada a avaliações e julgamento. (CFESS,2007, p.43-44).
conhecimento específico, a uma questão ou questões
relacionadas a decisões a serem tomadas. Trata-se de
exposição e manifestação sucinta, enfocando-se objetivamente 6. análise e fundamentação das
a questão ou situação social analisada, e os objetivos do
trabalho solicitado e apresentado; a análise da situação,
relações sociais no âmbito das
referenciada em fundamentos teóricos-éticos e técnicos e uma instituições e dos movimentos
finalização, de caráter conclusivo ou indicativo. Ele pode ser populares;
emitido enquanto parte final ou conclusão de um laudo, bem
como enquanto resposta à consulta ou a determinação da
autoridade judiciária a respeito de alguma questão constante
Movimentos Sociais, Sociedade Civil e Transformação
em processo já acompanhado pelo profissional.
Social no Brasil11
Elementos constitutivos do parecer social:
As representações, os valores e os significados presentes
Os movimentos sociais vêm acompanhando os passos
no contexto sociocultural onde o usuário desenvolve relações
democráticos de diversas nações, inclusive do Brasil, nas
sociais e de convivência deverão ser considerados. Os
últimas décadas, presentes constantemente em
seguintes aspectos deverão nortear os elementos
acontecimentos históricos relevantes, principalmente no
constitutivos do parecer:
âmbito das conquistas sociais. Na verdade, consistem num
mecanismo que os cidadãos utilizam para reivindicar e ver
- qual o núcleo familiar;
reconhecidos seus interesses e anseios coletivos.
- satisfação das necessidades básicas;
Avritzer afirma que “os movimentos sociais constituem
- posição do usuário no grupo familiar;
aquela parte da realidade social na qual as relações sociais
- inserção no mercado de trabalho.
ainda não estão cristalizadas em estruturas sociais, onde a
ação é a portadora imediata da tessitura relacional da
Elaboração do parecer social
sociedade e do seu sentido”. Eles não constituem um simples
objeto social e sim uma lente por intermédio da qual
Tem como base a observação e a realização do estudo
problemas mais gerais podem ser abordados.
sócio- econômico de uma dada situação. O Assistente Social
A influência dos movimentos sociais vai muito além dos
deve estar atento para não definir a concessão do benefício
efeitos políticos produzidos por eles, pois suas ações
ou da avaliação médico pericial, situando o parecer no
determinam a modificação de comportamentos e de regras por
âmbito do Serviço Social. O estudo socioeconômico não
parte do sistema político. E, além do mais, há uma dimensão
deverá ser encaminhado aos setores solicitantes, mas
simbólica muito mais complexa sobre a qual os movimentos
permanecer no prontuário do Assistente Social, em arquivo
sociais exercem grande impacto que é a transformação social.
próprio. Somente o parecer social é que deverá ser
Hoje, a partir dessas novas mobilizações, os cidadãos e as
encaminhado (O sigilo profissional deve ser prevalecido).
sociedades conjugam a gramática da igualdade de gênero,
preocupações ecológicas, conservação do meio ambiente,
É necessário que o conteúdo não seja excessivo e deve ser
direitos dos nascituros, impensáveis antes da emergência de
expresso de maneira clara e objetiva. A autora sublinha que
movimentos sociais com essas novas agendas.
deve-se estar atento na coleta de dados, ao realizar a
Para Correia, a sociedade civil serve-se dos movimentos
entrevista, explicitando para o usuário o nosso objetivo,
sociais para conquistar direitos negados ou não
buscando uma postura horizontal com o mesmo, situando-o
disponibilizados pelo Estado. É nesse contexto de carências, de

11 Texto adaptado de AZEVEDO, D. A. de. Movimentos Sociais,

Sociedade Civil e Transformação Social no Brasil.

Conhecimentos Específicos 28
Apostila Digital Licenciada para Alice Caroline Guarino dos Santos - alice.guarino@hotmail.com (Proibida a Revenda)
APOSTILAS OPÇÃO

exclusão e necessidades sociais, que se situam as práticas Nesse âmbito também começa a se falar em crise dos
cotidianas de movimentos sociais, que ainda com certas “movimentos sociais urbanos”, esta não representava o seu
limitações, são meios potencializadores de novas formas de se desaparecimento nem o seu enfraquecimento enquanto atores
fazer política, de participação social, de construção do sociopolíticos, mas sim uma rearticulação interna e externa de
processo democrático e de transformação social. Presume-se seu papel na sociedade. As transformações no contexto
que os movimentos sociais são tentativas coletivas e político levam também a emergência, ou ao fortalecimento, de
organizadas que têm a finalidade de buscar determinadas outros atores sociais, como as ONGs e outras entidades do
mudanças ou até mesmo estipular a possibilidade de terceiro setor. Assim, os movimentos populares passam a ser
construção de uma nova ordem social. aliados ou até mesmo disputar com
Na realidade histórica, os movimentos sociais sempre
existiram e cremos que sempre existirão. Isto porque eles Apesar do enfraquecimento dos movimentos sociais e da
representam forças sociais organizadas que aglutinam as rearticulação do papel destes na sociedade, é imprescindível
pessoas não como força tarefa, de ordem numérica, mas como considerarmos a sua grande relevância no processo
campo de atividades e de experimentação social, e essas democrático brasileiro, mediante sua atuação voltada a
atividades são fontes geradoras de criatividade e inovações reivindicação dos direitos, até então, não disponibilizados aos
socioculturais. cidadãos. Desta forma, as lutas desencadeadas na sociedade
civil, são absolutamente essenciais num processo de efetiva
Breve Contextualização dos Movimentos Sociais Brasileiros transformação social, a caminho da emancipação humana.

Por volta dos anos 60 e 70, se disseminam os movimentos A Emergência da Sociedade Civil no Brasil
populares e sociais no Brasil. Para Pinsky, “os movimentos
populares se caracterizaram por um alcance limitado a Em relação a trajetória histórica da sociedade civil no
questões localizadas na vida prática da comunidade”. Brasil, situa-se, aqui, apenas as três últimas décadas do século
A força do movimento operário e dos movimentos XX, onde se acentua o fortalecimento desta categoria no
populares (como das classes dominantes) determina a movimento de redemocratização. Mas não se pode negar a
estratégia dos grupos populares. “Se a conjuntura é favorável organização anterior desta no decorrer da história brasileira.
à mobilização popular e expansão das lutas, a estratégia pode Ao contextualizar-se a emergência da sociedade civil,
ser mais ofensiva, se o momento se apresenta desfavorável é Avritzer introduz que o surgimento de novos atores sociais no
marcado por uma retração da forças populares a estratégia é processo de construção da sociedade civil no Brasil se associa
defensiva” (FALEIROS, 1985). ao rápido processo de modernização ao qual está ligado o
A partir do final dos anos 1970, o movimento sindical e as autoritarismo brasileiro. Contudo, é possível localizar a
organizações estudantis ganharam força. As greves dos emergência da sociedade civil brasileira no período final do
metalúrgicos paralisaram as indústrias de São Paulo, logo regime autoritário e início da redemocratização do país.
acompanhadas por greves dos bancários. A crise do “milagre Entende-se, que desde o seu surgimento a sociedade civil
econômico” foi o estopim desses movimentos, a luta contra a contribui para a construção do processo democrático
carestia balançou os alicerces da ditadura. Paralelamente, brasileiro, por meio de organizações sociais, lutas, associações,
surgiram grupos ligados a questões específicas: mulheres, movimentos, etc, constantemente, buscando o
povos indígenas, negros e homossexuais. reconhecimento de seu projeto societário numa realidade
É na década de 1980 que os movimentos sociais vão incluir contraditória pelo desenvolvimento do capital e, ao mesmo
parcelas mais amplas da sociedade. Surgem os movimentos tempo, pelo desenvolvimento da cidadania.
ecológicos, que transcendiam a divisão política entre direita e Os novos atores que emergiram na cena política
esquerda e, também, o movimento em defesa dos direitos do necessitam de espaços na sociedade civil – instituições
consumidor. Já na década de 90, surge no Brasil, um tipo de próprias, para participarem de novos pactos políticos que
organização inexistente até então, as organizações não- redirecionam o modelo político vigente. O saudoso Betinho já
governamentais (ONGs) para designar as entidades da nos dizia: “a sociedade civil tem um papel central, o poder está
sociedade civil, em referência a todo movimento de cunho na sociedade civil, não no Estado. O Estado é instrumento”
social. (SOUZA 2005).
A questão do protagonismo dos movimentos sociais no Em face disso, explicita-se a capacidade de mobilização,
Brasil, a partir dos anos 1990, começa a perder visibilidade participação e conscientização política da sociedade civil,
política no cenário urbano. A partir disso, referem-se três configurada numa importante referência ao aprimoramento e
momentos: 1990-1995; 1995 a 2000; e do início deste novo reafirmação do Estado Democrático de Direito.
século até os dias atuais, que diagnosticam uma crise dos Tal como evidenciado no debate internacional, Duriguetto
movimentos sociais populares urbanos, nos primeiros cinco sustenta que a categoria sociedade civil foi comumente
anos dos anos 1990, no sentido de que reduziram parte de seu empregada no contexto brasileiro a partir do final da década
poder de pressão direta que haviam conquistado nos anos de 1970, para expressar a reativação do movimento sindical e
1980. Nesse momento, o país saía de uma etapa de conquista a ação dos chamados “novos movimentos sociais”, que
dos direitos constitucionais, os quais necessitavam ser passaram a dinamizar processos de mobilização de defesa,
regulamentados. Ao mesmo tempo, o governo federal, passou conquista e ampliação de direitos civis, políticos, sociais e
a implementar ou a aprofundar, em todos os níveis, as políticas trabalhistas.
neoliberais, as quais geraram desemprego, aumento da Ainda, a autora refere à emergência de várias iniciativas de
pobreza e da violência urbana e rural (GOHN, 2005). parceria entre a sociedade civil organizada e o poder público,
impulsionadas por políticas estatais, como a experiência do
O Estado, diante de tal realidade, fecha as portas da Orçamento Participativo, no Rio Grande do Sul, em Porto
negociação porque as concessões solicitadas não são Alegre. Com o Orçamento Participativo, o próprio Estado
aceitáveis ao estado de acumulação de capital que ele visa. evolui gradativamente, começando a perceber a importância
Essa estratégia pode retirar a legitimidade da classe no poder da participação popular nas decisões políticas e sociais.
se ela defende (discurso) a democracia e a participação. O Em nível de Brasil, as práticas de deliberação participativa
Estado se vê então colocado numa situação de defesa clara da estiveram, desde o seu início, ligadas à visibilidade política dos
acumulação do capital ou de sua legitimidade (FALEIROS, novos movimentos sociais e à redefinição de práticas do
1985). movimento operário nas décadas de 1970 e 1980. Elas foram

Conhecimentos Específicos 29
Apostila Digital Licenciada para Alice Caroline Guarino dos Santos - alice.guarino@hotmail.com (Proibida a Revenda)
APOSTILAS OPÇÃO

entendidas através de uma renovada teoria do conflito social tempo não é razoável imaginar que aqueles que pretendam
que apontava para formas de participação popular e lutas dirigir o futuro consigam avançar se se puserem fora do Estado
plurais demandantes de representação autônoma no processo ou sem um Estado.
de bens públicos e formulação de políticas públicas. Diante disso, torna-se relevante mencionar que as últimas
As novas democracias devem se transformar em novos duas décadas do século XX, marcam o avanço da
movimentos sociais, no sentido que o estado deve se democratização e da progressiva valorização da democracia
transformar em um local de experimentação distributiva e participativa na sociedade brasileira. Para o ideal
cultural. É na originalidade das novas formas de desenvolvimentista, a resolução dos problemas sociais ou o
experimentação institucional que podem estar os potenciais combate à inflação requereriam bem mais autoridade e
emancipatórios ainda presentes nas sociedades centralização decisória do que democracia. Antes, a convicção
contemporâneas (SANTOS). era a de que processos participativos ou, mais genericamente,
mecanismos de consulta popular, negociação e formação
É possível assinalar que os novos movimentos sociais ampliada de consensos, agiriam “contra” o crescimento
contemplam uma identidade em função da defesa de seu econômico, na medida em que dificultariam a tomada rápida
projeto societário. Identidade esta, de caráter coletivo, de decisões e, com isso, prolongariam indevidamente o tempo
construída dentro do grupo num processo de inter-relações de formulação e de implementação de políticas. Pouco a pouco,
que vem impondo desafios para pensar a relação do sujeito a opinião prevalecente foi-se deslocando para o lado oposto,
com as instituições existentes. com o correspondente reconhecimento de que a participação
O século XX foi efetivamente um século de intensa disputa não somente conteria um valor em si, como também seria
em torno da questão democrática. Para Santos (2002), haveria, particularmente relevante no fornecimento de
portanto, uma tensão entre capitalismo e democracia, tensão sustentabilidade às políticas públicas.
essa que, uma vez resolvida a favor da democracia, colocaria É importante mencionar, que no caso brasileiro, os
limites à propriedade e implicaria em ganhos distributivos autores citados mostram que a motivação pela participação é
para os setores sociais desfavorecidos. Aos marxistas, por seu parte de uma herança comum do processo de democratização
lado, entendiam que essa solução exigia a descaracterização que levou atores sociais democráticos, oriundos,
total da democracia, uma vez que nas sociedades capitalistas especialmente do movimento comunitário, a disputarem o
não era possível democratizar a relação fundamental em que significado do termo participação, a partir da abertura de
se assentava a produção material, a relação entre o capital e o espaços reais de participação pela sociedade política, dando
trabalho. Daí que no âmbito desse debate, se discutissem ênfase a deliberação em nível local.
modelos de democracia alternativos ao modelo liberal, entre A democracia participativa apenas pode produzir seus
eles: a democracia participativa. desdobramentos ótimos se dispuser de cidadãos e de
É possível mostrar que os atores que implantaram as associações com disposição política para experenciar
experiências de democracia participativa colocaram em dinâmicas coletivas e cooperativas. Pensada como recurso de
questão uma identidade que lhes fora atribuída externamente transformação social, somente pode avançar se seus cidadãos
por um Estado colonial (Estado autoritário e discriminador). forem alcançados por processos fortes de educação política, de
Caracterizando-se pela reivindicação de direitos de moradia, conscientização e de politização.
direitos à bens públicos distribuídos localmente, direitos de
participação, de reconhecimento das diferenças, implicam, de
certo modo, questionar uma gramática social e estatal de 7. Serviço Social e
exclusão e propor, como alternativa, uma outra mais inclusiva. interdisciplinaridade;
É no âmbito do projeto democrático que se põe
efetivamente a questão da sociedade civil. Para Nogueira, isso
quer dizer que precisamos de uma perspectiva que não só A interdisciplinaridade12 parece indicar um
valorize a sociedade civil e celebre seu crescente amadurecimento entre as áreas acerca da incompletude que
protagonismo, mas também colabore para politizá-la, cada uma delas carrega invariavelmente. Como lembra
libertando-a das amarras reducionistas dos interesses Fazenda (1995) a interdisciplinaridade se define como um
particulares, aproximando-a do universo mais rico e generoso regime de co-propriedade, de interação, que viabilizará
dos interesses gerais, da hegemonia, em uma palavra, do diálogo entre os atores envolvidos.
Estado. A interdisciplinaridade, favorecendo o alargamento e a
Ao longo do processo de reprodução do capitalismo, a flexibilização no âmbito do conhecimento, pode significar uma
sociedade civil ganhou corpo graças a uma modernização que instigante disposição para os horizontes do saber. (...) Penso a
se afirmou, de modo muito agressivo, uma multidão de interdisciplinaridade, inicialmente, como postura profissional
interesses particulares. A sociedade civil cresceu à base desse que permite se pôr a transitar o “espaço da diferença” com
processo e viu-se confrontada com os mais diversos estímulos sentido de busca, de desenvolvimento da pluralidade de
tendentes a separá-la da política, a entregá-la a valores mais ângulos que um determinado objeto investigado é capaz de
individualistas que solidários, mais competitivos que proporcionar, que uma determinada realidade é capaz de
cooperativos. Percebe-se que o conjunto das mudanças afetou gerar, que diferentes formas de abordar o real podem trazer.
comportamentos e expectativas políticas, forçando a abertura
de espaços para a vocalização de novos interesses, Para efetivar a chamada interdisciplinaridade é
transformou os padrões de participação e de competição fundamental para qualquer profissão ater-se para o fato de
eleitoral. que não se encontra isolada de outras profissões, além disso,
Compreende-se que a sociabilidade está mais complexa e o ter a plena consciência de que para bem interagir com as
Estado passou a falhar em seu desempenho, é inevitável que se mesmas, é sempre necessário saber definir e ser competente
projete uma situação na qual os espaços sociais sejam em seu campo. Este é um elemento fundamental para que haja,
radicalmente valorizados. Para o autor citado acima, tudo leva de fato, troca entre os saberes – o que define a equipe
a crer que o Estado não terá como voltar a desempenhar os interdisciplinar.
mesmos papéis que desempenhou antes, mas ao mesmo

12 CARVALHO, F. A. de O serviço social e a interdisciplinaridade.

http://www.proceedings.scielo.br/pdf/cips/n4v1/27.pdf

Conhecimentos Específicos 30
Apostila Digital Licenciada para Alice Caroline Guarino dos Santos - alice.guarino@hotmail.com (Proibida a Revenda)
APOSTILAS OPÇÃO

Parece brindar-nos de obviedade que só oferecemos aquilo profissionais que atuam dia após dia, sem compreensão do que
que de fato, temos propriedade. Ou seja, o conhecimento e se trata uma e outra, e sentindo-se responsável pelas duas. É
domínio das possibilidades e limites da própria profissão possível registrar que a política social como direito de
tornam-se indispensáveis para que haja interação com as cidadania ou como elemento redistributivo são insuficientes
demais profissões. para a demanda que a população apresenta.
Em sendo assim, procuramos nesse momento, É nessa contradição - trabalhar para e com a proteção
compartilhar algumas informações que se constituem em social, muitas vezes sem os recursos necessários – sobretudo
elementos que fundamentam teórica e metodologicamente o de natureza estrutural - que atua o serviço social. Podemos
serviço social. afirmar que os limites da práxis profissional podem ser
O assistente social é um dos profissionais que trabalha superados pela práxis social que extrapola os limites da
diretamente sobre as diferenciadas expressões da questão categoria e trabalha para a construção de outra sociedade.
social e possui nas mesmas um campo qualificado de atuação O Serviço Social possui diferentes espaços sócio
direta. A questão social é compreendida como, um complexo ocupacionais e atua em consonância com os dispositivos
social que faz parte da natureza da propriedade privada no legais, tais como: a Constituição Federal de 1988, o SUAS –
capitalismo, ou seja, é manifestação direta da apropriação Sistema Único de Assistência Social, SUS – Sistema Único da
privada da produção social e da lei geral da acumulação Saúde, ECA – Estatuto da criança e do Adolescente, além de
capitalista. inúmeras outras diretrizes legais que diferenciam-se de
acordo com área de atendimento e população atendida. Porém,
Ainda, lembra-nos Iamamoto (2001) que a questão social há o destaque para alguns princípios específicos da profissão:
expressa as diferentes disparidades – econômicas, políticas e seu Projeto Ético Político e Código de Ética.
culturais das classes - mediatizadas por relações de gênero, O Projeto Éticopolítico profissional atem-se aos interesses
características étnico-raciais e formações regionais. da classe trabalhadora, como conferimos a seguir:
Desse modo, é importante zelar para que o trato da
questão social não passe a ser orientado pela teoria de A direção sociopolítica estratégica do projeto profissional
integração social. Impedindo assim, que se naturalizem as pressupõe um processo de lutas sociais e mobilização de massas,
desigualdades sociais e a perda do caráter de conquista das que possam reverter o quadro de barbárie social em que
políticas sociais. Nesse caso, os usuários que eram antes estamos mergulhados e que foram impostos pela ditadura do
entendidos como sujeito de direitos são diminuídos à condição capital. Lutar por consolidação das massas trabalhadoras, da
de beneficiários e assistidos. cidade e do campo, em uma perspectiva de classe,
Esse tipo de direcionamento tende a concretizar soluções anticapitalista, anti-imperialista e socialista no interior do
que misturam a integração social - constituída de políticas processo de ruptura profissional com o conservadorismo.
compensatórias - com as medidas repressivas, de acordo com
conjuntura histórica. Nesse sentindo, as políticas universais Em sendo assim, associa-se a um projeto societário, e
tornam-se as únicas respostas aceitáveis se a compreensão da propõe outra construção de homens e mulheres. Sem a
questão social ocorre no interior da luta de classes. presença de qualquer exploração ou dominação. Seja esta de
O serviço social, desse modo, defende a política de direitos classes, etnia e/ou gênero. Interessante destacar que esse
universais! Uma curiosidade para os que não são formados no mesmo projeto refuta qualquer tipo de preconceito e afirma o
serviço social, trata-se da trajetória histórica do mesmo. reconhecimento do pluralismo e da defesa dos direitos
Originado das ações das damas de caridade da igreja humanos.
católica, veio se transformando historicamente. Embora o Já acerca do Código de Ética do Assistente Social do ano de
serviço social tenha concretizado o Movimento de 1986, pode-se destacar que o mesmo avaliou a necessidade de
Reconceituação e assim, sua tentativa de ruptura com o superar o ponto de vista que mantinha os valores éticos como
serviço social tradicional, pode-se afirmar que a apropriação universais acima dos interesses de classe em que se organiza
teórica marxiana deixou a desejar naquele momento, talvez sociedade. Realizando então um rompimento com a tradição
até pelo episódio histórico do país, extremamente repressivo neotomista. Porém, é em 1993 que o novo Código de Ética, de
– em tempos de ditadura militar. Fato que contribuiu para que base marxista, aponta a emancipação do ser social e
de modo equivocado, houvesse a atribuição da revolução fundamenta a liberdade como seu valor ético central. Pois,
legada ao serviço social. (Silva, 2007) Se o código de 1986 rompia com o tradicionalismo, com o
Compreendemos assim que a perspectiva da revolução não personalismo cristão e com os princípios abstratos e neutros -
foi apanhada na sua complexidade. E a revolução, então, característicos de neotomismo; o código de 1993 preconiza
aparece como uma tarefa do Serviço Social e de um conjunto uma apropriação teórica da produção marxiana, pautando–se
de profissionais messianicamente comprometidos com a na ontologia social de Marx e no seu projeto societário, o que
“capacitação”, com a “organização” das massas e com a se constitui como embasamento filosófico a leitura da
“transformação da sociedade”. Assim, também foi importante realidade e atuação profissional.
para a categoria perceber que a cidadania burguesa e os É sempre uma profunda responsabilidade realizar
direitos básicos, não se tratam de emancipação humana. Ainda intervenção na vida de seres humanos, homens, mulheres e
que conquistemos a cidadania mínima (o que seria com crianças que possuem história, desejos, sonhos, dificuldades e
certeza uma conquista), não teríamos simplesmente com ela fragilidades – como qualquer outro, com a diferença de que
qualquer transformação estrutural. Ou seja, tiveram oportunidades concretas bastante diferenciadas.
A afirmação dos direitos, não realiza, mesmo que Assim, respaldar-se para realizar as chamadas “escolhas de
radicalizada, a emancipação humana. O seu empobrecimento e intervenção” demanda superar limites já estabelecidos,
banalização, ou em outras palavras, a sua captura e utilização, sobretudo, aqueles que se referem à avaliação da própria
a partir dos interesses de mercado que restringem e atuação. Espaço privilegiado quando aproveitamos os
empobrecem as noções de democracia e cidadania ‘para os diferentes conhecimentos e áreas com os quais estamos
mais fortes’, também são sérios e concretos obstáculos à envolvidos.
emancipação humana.
Qual é o cenário da prática profissional do serviço social?
Reconhecer os limites do serviço social é básico para a Iamamoto (2001) indica como novas expressões da questão
atuação profissional, a confusão entre emancipação política e social diversos elementos, tais como: a lógica financeira do
emancipação humana é elemento de desgaste para inúmeros regime de acumulação que tende a provocar crises mundiais

Conhecimentos Específicos 31
Apostila Digital Licenciada para Alice Caroline Guarino dos Santos - alice.guarino@hotmail.com (Proibida a Revenda)
APOSTILAS OPÇÃO

gerando recessão; o espaço antes fordista-taylorista com Se a existência humana não é garantida pela natureza, não
tendência à liderança a especialização flexível; as intensas é uma dádiva natural, mas tem de ser produzida pelos próprios
mudanças na relação Estado-sociedade civil orientada pelo homens, sendo, pois, um produto do trabalho, isso significa
neoliberalismo; a afetação da esfera da sociabilidade que o homem não nasce homem. Ele forma-se homem. Ele não
invadindo as diferentes relações sociais com uma lógica nasce sabendo produzir-se como homem. Ele necessita
pragmática e produtivista, com mentalidade utilitária e aprender a ser homem, precisa aprender a produzir sua
individualista. própria existência. Portanto, a produção do homem é, ao
É exatamente a partir desse quadro que se evidenciam a mesmo tempo, a formação do homem, isto é, um processo
globalização econômica e a ideologia neoliberal. Repercutindo educativo. A origem da educação coincide, então, com a origem
na flexibilização no mundo do trabalho que agrega diversos do homem mesmo.
processos, como exemplo citamos a criação de novas formas
de trabalho, parcial, temporário, subcontratado, vinculado à É com este homem - sujeito e construtor - que trabalha
chamada economia informal. Onde se apresenta a baixa tanto a psicologia como o serviço social e a pedagogia.
remuneração e a escassa inserção nas políticas assistenciais. Profissões que possuem diferentes correntes
Pastorini (2007). teóricometodológica tanto internamente, como também entre
si, e o exercício da interdisciplinaridade é exatamente esse:
O Estado Intervencionista transita para o papel de Estado lidar com a diferença e convertê-la em saldo e não em débito
Mínimo com a vigência da ideologia neoliberal. Fazendo para o profissional, para sua categoria, para sua equipe, para
ocorrer uma minimização do Estado Interventor diante das seu usuário e para a política universal de direitos – eixo
questões sociais, onde ao mesmo é atribuída a função de comum entre todas. Seja em qualquer subárea de atuação –
garantir as propriedades e liberdades individuais. saúde, educação, assistência social, lidar com o conteúdo do
Estarrecedor perceber que o Estado agirá apenas em esferas outro e respeita-lo como digno de troca faz parte do princípio
que o mercado não possa ou não se interesse em responder. da interdisciplinaridade.
Não é novidade compartilhar que é nesse contexto que
ocorre a redução de direitos sociais, das políticas sociais, e
sendo necessário, dos direitos políticos, com a justificativa dos
direitos civis. 8. Direitos Humanos;
É exatamente no bojo das contradições que se materializa
para o serviço social a tarefa de perquirir e perseguir o real
como “concreto pensado”. Programa Nacional de Direitos Humanos- 1 versão13
Para que desse modo, o assistente social não seja enredado
em culpabilizar o usuário e sua família, ou até mesmo em Os Direitos Humanos são os direitos de todos e devem ser
legitimar um determinado estigma, ora, atribuído à profissão, protegidos em todos Estados e nações.
como meramente interventiva, de profissional meramente Os assassinatos, as chacinas, o extermínio, os sequestros, o
executor. crime organizado, o tráfico de drogas e as mortes no trânsito
Contudo o que todas essas informações podem contribuir não podem ser consideradas normais, especialmente em um
para uma conversa sobre interdisciplinaridade entre serviço Estado e em uma sociedade que se desejam modernos e
social, pedagogia e psicologia? Ora, esse foi um primeiro democráticos.
momento de rápida e sucinta apresentação da área do serviço É preciso dizer não à banalização da violência e proteger a
social. Que se trata de uma profissão extremamente existência humana. É neste contexto que o Governo brasileiro,
comprometida com o objetivo de defender e consolidar sob a presidência de Fernando Henrique Cardoso, decidiu
direitos! Bem sabemos que o serviço social, felizmente, não elaborar o Programa Nacional de Direitos Humanos.
está sozinho nessa empreitada. Direitos humanos são os direitos fundamentais de todas as
Nesse sentido, é importante refletirmos que há, ainda, pessoas, sejam elas mulheres, negros, homossexuais, índios,
outro elemento fundamental para a interdisciplinaridade: a idosos, pessoas portadoras de deficiências, populações de
capacidade do profissional em reconhecer mérito e fronteiras, estrangeiros e emigrantes, refugiados, portadores
legitimidade nos conhecimentos das demais profissões entre de HIV positivo, crianças e adolescentes, policiais, presos,
si. despossuídos e os que têm acesso a riqueza. Todos, enquanto
Devo confessar que a temática desafiou-me desde o início, pessoas, devem ser respeitados e sua integridade física
pois além de profissional da área do serviço social, também protegida e assegurada.
sou profissional da educação. Áreas distintas, com Direitos humanos referem-se a um sem número de campos
competências diferenciadas e por diversas vezes, públicos da atividade humana: o direito de ir e vir sem ser molestado; o
comuns. E a psicologia? Além de também possuir seu universo direito de ser tratado pelos agentes do Estado com respeito e
particular, também transita dentre as áreas já mencionadas. dignidade, mesmo tendo cometido uma infração; o direito de
Ora, quais são as interfaces das áreas distintas? O que as ser acusado dentro de um processo legal e legítimo, onde as
diferencia? Quais são seus objetivos, potencialidades e limites? provas sejam conseguidas dentro da boa técnica e do bom
Como podem coexistir garantindo sua identidade e direito, sem estar sujeito a torturas ou maus tratos; o direito
simultaneamente a troca de saberes que a de exigir o cumprimento da Lei e, ainda, de ter acesso a um
interdisciplinaridade exige? Judiciário e a um Ministério Publico que, ciosos de sua
Parece-me que estas questões são um o desafio desse importância para o Estado democrático, não descansem
debate, cujo convite está alicerçado nas diferenças de cada enquanto graves violações de direitos humanos estejam
área, na beleza de suas potencialidades e na honestidade de impunes e seus responsáveis soltos e sem punição, como se
seus limites. estivessem acima das normas legais; o direito de dirigir seu
Podemos destacar que as três áreas possuem como sujeito carro dentro da velocidade permitida e com respeito aos sinais
– objeto - sujeito de ação, o próprio ser humano. Talvez seja de trânsito e às faixas de pedestres, para não matar um ser
esse não somente o primeiro, mas o principal ponto a ser humano ou lhe causar acidente; o direito de ser, pensar, crer,
destacado. de manifestar- se ou de amar sem tornar-se alvo de

13 Texto adaptado disponível em http://dhnet.org.br/

Conhecimentos Específicos 32
Apostila Digital Licenciada para Alice Caroline Guarino dos Santos - alice.guarino@hotmail.com (Proibida a Revenda)
APOSTILAS OPÇÃO

humilhação, discriminação ou perseguição. São aqueles esta conferência, setores do Estado e diversas entidades de
direitos que garantem existência digna a qualquer pessoa. direitos humanos foram convocados pelo então Ministro da
O entendimento deste princípio é indispensável para que Justiça, Maurício Corrêa, com a finalidade de elaborar uma
haja uma mutação cultural e, em consequência, uma mudança Agenda Nacional de Direitos Humanos.
nas práticas dos Governos, dos Poderes da República nas suas Em 7 de setembro último, o Presidente Fernando Henrique
várias esferas e, principalmente, da própria sociedade. É Cardoso reiterou que os direitos humanos são parte essencial
justamente quando a sociedade se conscientiza dos seus de seu programa de Governo. Para o Presidente, no limiar do
direitos e exige que estes sejam respeitados que se fortalecem século XXI, a "luta pela liberdade e pela democracia tem um
a Democracia e o Estado de Direito. nome específico: chama-se direitos humanos". Determinou,
O esforço dos Governos federais, estaduais, municipais, das então, ao Ministério da Justiça a elaboração de um Programa
autoridades judiciárias, legislativas e da própria sociedade Nacional de Direitos Humanos, conforme previsto na
como um todo ainda não foram capazes de diminuir o Declaração e Programa de Ação de Viena, adotada
desrespeito diário aos direitos humanos no Brasil consensualmente na Conferência Mundial dos Direitos
A falta de segurança das pessoas, o aumento da escalada da Humanos, em 25 de junho de 1993, na qual o Brasil teve uma
violência, que a cada dia se revela mais múltipla e perversa, destacada participação.
exigem dos diversos atores sociais e governamentais uma O Governo brasileiro, embora considere que a
atitude firme, segura e perseverante no caminho do respeito normatização constitucional e a adesão a tratados
aos direitos humanos. internacionais de direitos humanos sejam passos essenciais e
O Programa Nacional de Direitos Humanos aponta nessa decisivos na promoção destes direitos, está consciente de que
direção, e está dirigido para o conjunto dos cidadãos a sua efetivação, no dia a dia de cada um, depende da atuação
brasileiros. O Programa é uma clara afirmação do Governo constante do Estado e da Sociedade. Com este objetivo se
Federal com os compromissos assumidos, pelo Brasil, elaborou o Programa Nacional de Direitos Humanos que ora se
externamente e com a população na luta contra a violência em submete a toda a Nação.
geral.
O Governo Federal, com a iniciativa do Programa Nacional Atualidade dos Direitos Humanos -
de Direitos Humanos, quer ir além de um quadro
profundamente preocupante, marcado no passado por um A adoção pela Assembleia Geral das Nações Unidas da
Poder Público deficiente e indiferente ao desrespeito à Declaração Universal de Direitos Humanos, em 1948, constitui
tranquilidade e segurança do cidadão comum. A o principal marco no desenvolvimento da ideia
inconformidade da sociedade brasileira com esta situação é contemporânea de direitos humanos. Os direitos inscritos
essencial para que este estado inaceitável de coisas seja afinal nesta Declaração constituem um conjunto indissociável e
superado. interdependente de direitos individuais e coletivos, civis,
O objetivo do Programa Nacional de Direitos Humanos políticos, econômicos, sociais e culturais, sem os quais a
(PNDH), elaborado pelo Ministério da Justiça em conjunto com dignidade da pessoa humana não se realiza por completo. A
diversas organizações da sociedade civil, é, identificando os Declaração transformou-se, nesta última metade de século,
principais obstáculos à promoção e proteção dos direitos numa fonte de inspiração para a elaboração de diversas cartas
humanos no Brasil, eleger prioridades e apresentar propostas constitucionais e tratados internacionais voltados à proteção
concretas de caráter administrativo, legislativo e político- dos direitos humanos. Este documento, chave do nosso tempo,
cultural que busquem equacionar os mais graves problemas tornou-se um autêntico paradigma ético a partir do qual se
que hoje impossibilitam ou dificultam a sua plena realização. pode medir e contestar a legitimidade de regimes e Governos.
O PNDH é resultante de um longo e muitas vezes penoso Os direitos ali inscritos constituem hoje um dos mais
processo de democratização da Sociedade e do Estado importantes instrumentos de nossa civilização visando a
brasileiro. assegurar um convívio social digno, justo e pacífico.
A Constituição de 1988 estabelece a mais precisa e Os direitos humanos não são, porém, apenas um conjunto
pormenorizada carta de direitos de nossa história, que inclui de princípios morais que devem informar a organização da
uma vasta identificação de direitos civis, políticos, econômicos, sociedade e a criação do direito. Enumerados em diversos
sociais, culturais, além de um conjunto preciso de garantias tratados internacionais e constituições, asseguram direitos
constitucionais. A Constituição também impõe ao Estado aos indivíduos e coletividades e estabelecem obrigações
brasileiro reger-se, em suas relações internacionais, pelo jurídicas concretas aos Estados. Compõem-se de uma série de
princípio da "prevalência dos Direitos Humanos" (artigo normas jurídicas claras e precisas, voltadas a proteger os
4&deg, II). Resultado desta nova diretiva constitucional foi a interesses mais fundamentais da pessoa humana. São normas
adesão do Brasil, no início dos anos noventa, aos Pactos cogentes ou programáticas que obrigam os Estados no plano
Internacionais de Direitos Civis e Políticos, e de Direitos interno e externo.
Econômicos, Sociais e Culturais, às Convenções Americana de Com o estabelecimento das Nações Unidas, em 1945, e a
Direitos Humanos e contra a Tortura e Outros Tratamentos ou adoção de diversos tratados internacionais voltados à
Penas Cruéis, Desumanos ou Degradantes, que se encontram proteção da pessoa humana, os direitos humanos deixaram de
entre os mais importantes instrumentos internacionais de ser uma questão afeta exclusivamente aos Estados nacionais,
proteção aos direitos humanos. Paralelamente a esta mudança passando a ser matéria de interesse de toda a comunidade
no quadro normativo, o Governo Federal vem tomando várias internacional. A criação de mecanismos judiciais
iniciativas nas esferas internacional e interna que visam a internacionais de proteção dos direitos humanos, como a
promover e proteger os direitos humanos. Corte Interamericana e a Corte Europeia de Direitos Humanos,
Por iniciativa do então Chanceler Fernando Henrique ou quase judiciais como a Comissão Interamericana de
Cardoso reuniram-se, no Ministério das Relações Exteriores, Direitos Humanos ou Comitê de Direitos Humanos das Nações
em maio de 1993, representantes do Ministério da Justiça, da Unidas, deixam claro esta mudança na antiga formulação do
Procuradoria Geral da República, além de parlamentares, e as conceito de soberania. É certo, porém, que a obrigação
mais importantes organizações não-governamentais de primária de assegurar os direitos humanos continua a ser
direitos humanos, com a finalidade de elaborar um relatório responsabilidade interna dos Estados.
com diagnóstico das principais dificuldades do país, de modo
a definir a agenda do Brasil para a Conferência Mundial de
Direitos Humanos realizada em Viena, em junho de 1993. Após

Conhecimentos Específicos 33
Apostila Digital Licenciada para Alice Caroline Guarino dos Santos - alice.guarino@hotmail.com (Proibida a Revenda)
APOSTILAS OPÇÃO

A natureza do Programa Nacional de Direitos Humanos brasileira deverá ter um papel ativo para que este Programa
- se efetive como realidade.
O Programa Nacional de Direitos Humanos abre uma nova
O Programa Nacional de Direitos Humanos, como qualquer dinâmica. Governo e sociedade civil respeitam a mesma
plano de ação que se pretenda exequível, deve explicitar gramática e articulam esforços comuns. O Programa passa,
objetivos definidos e precisos. Assim, sem abdicar de uma desta forma, a ser um marco referencial para as ações
compreensão integral e indissociável dos direitos humanos, o governamentais e para a construção, por toda a sociedade, da
Programa atribui maior ênfase aos direitos civis, ou seja, os convivência sem violência que a democracia exige.
que ferem mais diretamente a integridade física e o espaço de
cidadania de cada um. PROPOSTAS DE AÇÕES GOVERNAMENTAIS

O fato de os direitos humanos em todas as suas três Políticas públicas para proteção e promoção dos direitos
gerações - a dos direitos civis e políticos, a dos direitos sociais, humanos no Brasil:
econômicos e culturais, e a dos direitos coletivos - serem Apoiar a formulação e implementação de políticas públicas
indivisíveis não implica que, na definição de políticas e privadas e de ações sociais para redução das grandes
específicas - dos direitos civis - o Governo deixe de contemplar desigualdades econômicas, sociais e culturais ainda existentes
de forma específica cada uma dessas outras dimensões. O no país, visando a plena realização do direito ao
Programa, apesar de inserir-se dentro dos princípios definidos desenvolvimento.
pelo Pacto Internacional de Direitos Civis e Políticos, Criar um Cadastro Federal de Inadimplentes Sociais, que
contempla um largo elenco de medidas na área de direitos relacione os estados e municípios que não cumpram
civis que terão consequências decisivas para a efetiva proteção obrigações mínimas de proteção e promoção dos direitos
dos direitos sociais, econômicos e culturais, como, por humanos, com vistas a evitar o repasse de recursos, subsídios
exemplo, a implementação das convenções internacionais dos ou favorecimento a esses inadimplentes.
direitos das crianças, das mulheres e dos trabalhadores. Proteção do direito à vida
Numa sociedade ainda injusta como é a do Brasil, com
graves desigualdades de renda, promover os direitos humanos Segurança das pessoas
tornar-se-á mais factível se o equacionamento dos problemas
estruturais - como aqueles provocados pelo desemprego, Curto prazo
fome, dificuldades do acesso à terra, à saúde, à educação,
concentração de renda - for objeto de políticas - Promover a elaboração do mapa da violência urbana, com
governamentais. Mas, para que a população possa assumir que base em dados e indicadores de desenvolvimento urbano e
os direitos humanos são direitos de todos, e as entidades da qualidade de vida, a partir de quatro grandes cidades;
sociedade civil possam lutar por esses direitos e organizar-se - Elaborar um mapa da violência rural a partir de uma
para atuar em parceria com o Estado, é fundamental que seus região do país, visando a identificar áreas de conflitos e
direitos civis elementares sejam garantidos e, especialmente, possibilitar análise mais aprofundada da atuação do Estado.
que a Justiça seja uma instituição garantidora e acessível para - Apoiar programas para prevenir a violência contra
qualquer um. grupos em situação mais vulnerável, caso de crianças e
Serão abordados, no Programa, os entraves à cidadania adolescentes, idosos, mulheres, negros, indígenas, migrantes,
plena, que levam à violação sistemática dos direitos, visando a trabalhadores sem-terra e homossexuais.
proteger o direito à vida e à integridade física; o direito à - Aperfeiçoar a legislação sobre venda, posse, uso e porte
liberdade; o direito à igualdade perante à lei. de armas e munições pelos cidadãos, condicionando-os a
O Programa contempla, igualmente, iniciativas que rigorosa comprovação de necessidade, aptidão e capacidade
fortalecem a atuação das organizações da sociedade civil, para de manuseio.
a criação e consolidação de uma cultura de direitos humanos. - Propor projeto de lei regulando o uso de armas e
Nada melhor para atingir esse objetivo do que atribuir a essas munições por policiais nos horários de folga e aumentando o
organizações uma responsabilidade clara na promoção dos controle nos horários de serviço.
direitos humanos, especialmente nas iniciativas voltadas para - Apoiar a criação de sistemas integrados de controle de
a educação e a formação da cidadania. armamentos e munições pelos Governos estaduais, em
Na elaboração do Programa foram realizados entre parceria com o Governo federal.
novembro de 1995 e março de 1996 seis seminários regionais - Implementar programas de desarmamento, com ações
- São Paulo, Rio de Janeiro, Recife, Belém, Porto Alegre e Natal, coordenadas para apreender armas e munições de uso
com 334 participantes, pertencentes a 210 entidades. Foram proibido ou possuídas ilegalmente.
realizadas consultas, por telefone e fax, a um largo espectro de - Estimular o aperfeiçoamento dos critérios para seleção,
centros de direitos humanos e personalidades. Foi realizada admissão, capacitação, treinamento e reciclagem de policiais.
uma exposição no Encontro do Movimento Nacional dos - Incluir nos cursos das academias de polícia matéria
Direitos Humanos, em Brasília, no mês de fevereiro de 1996. específica sobre direitos humanos.
Finalmente, o projeto do Programa foi apresentado e debatido - Implementar a formação de grupo de consultoria para
na I Conferência Nacional de Direitos Humanos, promovida educação em direitos humanos, conforme o Protocolo de
pela Comissão de Direitos Humanos da Câmara de Deputados, Intenções firmado entre o Ministério da Justiça e a Anistia
com o apoio do Fórum das Comissões Legislativas de Direitos Internacional para ministrar cursos de direitos humanos para
Humanos, Comissão de Direitos Humanos da OAB Federal, as polícias estaduais.
Movimento Nacional de Direitos Humanos, CNBB, FENAJ, - Estruturar a Divisão de Direitos Humanos, criada
INESC, SERPAJ e CIMI, em abril de 1996. O Programa foi recentemente no organograma da Polícia Federal.
encaminhado, ainda, a várias entidades internacionais. Neste - Estimular a criação e o fortalecimento das corregedorias
processo de elaboração, foi colocada em prática a parceria de polícia, com vistas a limitar abusos e erros em operações
entre o Estado e as organizações da sociedade civil. Na policiais e emitir diretrizes claras a todos os integrantes das
execução concreta do Programa, a mesma parceria será forças policiais com relação à proteção dos direitos humanos.
intensificada. Além das organizações de direitos humanos, - Propor o afastamento nas atividades de policiamento de
universidades, centros de pesquisa, empresas, sindicatos, policiais acusados de violência contra os cidadãos, com
associações empresariais, fundações, enfim, toda a sociedade

Conhecimentos Específicos 34
Apostila Digital Licenciada para Alice Caroline Guarino dos Santos - alice.guarino@hotmail.com (Proibida a Revenda)
APOSTILAS OPÇÃO

imediata instauração de sindicância, sem prejuízo do devido procedimentos e aumentar as garantias do tratamento
processo criminal. igualitário de todos perante a lei.
- Incentivar a criação de Ouvidorias de Polícia, com - Apoiar a expansão dos serviços de prestação da justiça,
representantes da sociedade civil e autonomia de investigação para que estes se façam presentes em todas as regiões do País.
e fiscalização. - Apoiar a multiplicação e manutenção, pelos Estados, de
- Estimular a implementação de programas de seguro de juizados especiais civis e criminais.
vida e de saúde para policiais - Incentivar a prática de plantões permanentes no
- Apoiar a criação de um sistema de proteção especial à Judiciário, Ministério Público, Defensoria Pública e Delegacias
família dos policiais ameaçados em razão de suas atividades. de Polícia.
- Estimular programas de cooperação e entrosamento - Estudar a viabilidade de um sistema de juízes,
entre policiais civis e militares e entre estes e o Ministério promotores e defensores públicos itinerantes, especialmente
Público. nas regiões distantes dos centros urbanos, para ampliar o
- Apoiar, com envio de pedido de urgência o projeto de lei acesso à justiça.
n&ordm; 73 que estabelece o novo Código de Trânsito. - Apoiar medidas de fortalecer as corregedorias internas
- Promover programas de caráter preventivo que do Ministério Público e do Poder Judiciário, para aumentar a
contribuam para diminuir a incidência de acidentes de fiscalização e monitoramento das atividades dos promotores e
trânsito. juízes.
- Regulamentar o artigo 129, VII, da Constituição Federal,
Médio prazo que trata do controle externo da atividade policial pelo
Ministério Público.
- Incentivar programas de capacitação material das - Apoiar a criação nos Estados de programas de proteção
polícias, com a necessária e urgente renovação e de vítimas e testemunhas de crimes, expostas a grave e atual
modernização dos equipamentos de prestação da segurança perigo em virtude de colaboração ou declarações prestadas em
pública. investigação ou processo penal.
- Apoiar as experiências de polícias comunitárias ou - Propugnar pela aprovação do projeto de lei N&ordm;
interativas, entrosadas com conselhos comunitários, que 4.716-A/94 que tipifica o crime de tortura.
encarem o policial como agente de proteção dos direitos - Reformular o Conselho de Defesa dos Direitos da Pessoa
humanos. Humana (CDDPH), ampliando a participação de
- Apoiar programas de bolsas de estudo para representantes da sociedade civil e a sua competência.
aperfeiçoamento técnico dos policiais. - Incentivar a criação e fortalecimento de conselhos de
- Rever a legislação regulamentadora dos serviços defesa dos direitos humanos nos Estados e Municípios.
privados de segurança, com o objetivo de limitar seu campo de - Apoiar a atuação da Comissão de Direitos Humanos da
atuação, proporcionar seleção rigorosa de seus integrantes e Câmara dos Deputados e de comissões de direitos humanos
aumentar a supervisão do poder público. nos Estados e Municípios, e às comissões parlamentares de
- Estimular a regionalização do intercâmbio de inquérito - tanto no Congresso Nacional como nas Assembleias
informações e cooperação de atividades de segurança pública, Legislativas - para a investigação de crimes contra os direitos
com apoio aos atuais Conselhos de Segurança Pública do humanos.
Nordeste, do Sudeste e do Entorno, e a outros que venham a - Apoiar a criação de comissões de direitos humanos no
ser criados. Senado Federal e nas assembleias legislativas e câmaras
- Apoiar a expansão dos serviços de segurança pública, municipais onde estas comissões não tenham ainda sido
para que estes se façam presentes em todas as regiões do País. criadas.
- Fortalecer e ampliar a esfera de atuação da Ouvidoria
Luta contra a impunidade Geral da República, a fim de ampliar a participação da
população no monitoramento e fiscalização das atividades dos
Curto prazo órgãos e agentes do poder público.
- Estimular a criação do serviço "Disque Denúncia" em todo
- Atribuir à Justiça Federal a competência para julgar (a) os País e Instituir esse serviço nas repartições públicas federais
crimes praticados em detrimento de bens ou interesses sob a que integram o sistema federal de segurança pública.
tutela de órgão federal de proteção a direitos humanos (b) as
causas civis ou criminais nas quais o referido órgão ou o Médio prazo
Procurador-Geral da República manifeste interesse.
- Atribuir à Justiça Comum a competência para processar e - Propor a revisão da legislação sobre abuso e desacato à
julgar os crimes cometidos por policiais militares no autoridade.
policiamento civil ou com arma da corporação, apoiando - Fortalecer os Institutos Médico-Legais ou de
projeto específico já aprovado na Câmara dos Deputados. Criminalística, adotando medidas que assegurem a sua
- Propor projeto de lei para tornar obrigatória a presença excelência técnica e progressiva autonomia, articulando-os
no local, do juiz ou do representante do Ministério Público, à com universidades, com vista a aumentar a absorção de
ocasião do cumprimento de mandado de manutenção ou tecnologias.
reintegração de posse de terras, quando houver pluralidade de - Implantar o Programa de Integração das Informações
réus, para prevenir conflitos violentos no campo, ouvido Criminais, visando à criação de uma cadastro nacional de
também o órgão administrativo da reforma agrária. identificação criminal.
- Apoiar proposições legislativas que objetivem dinamizar - Dar continuidade à estruturação da Defensoria Pública da
os processos de expropriação para fins de reforma agrária, União, bem como incentivar a criação de Defensorias Públicas
assegurando-se, para prevenir violências, mais cautela na junto a todas as comarcas do país.
concessão de liminares.
- Apoiar, no contexto da reforma do Estado, coordenada Longo prazo
pelo Ministério da Administração e Reforma do Estado,
propostas para modernizar o Judiciário e para fortalecer o - Apoiar a criação do Conselho Nacional de Justiça, com a
sistema de proteção e promoção dos direitos humanos, de função de fiscalizar as atividades do Poder Judiciário.
forma a agilizar os processos, simplificar as regras e

Conhecimentos Específicos 35
Apostila Digital Licenciada para Alice Caroline Guarino dos Santos - alice.guarino@hotmail.com (Proibida a Revenda)
APOSTILAS OPÇÃO

Proteção do direito à liberdade - Incentivar a implementação de Conselhos Comunitários,


conforme determina a Lei de Execuções Penais, em todas as
Liberdade de Expressão e Classificação Indicativa regiões, para auxiliar, monitorar e fiscalizar os procedimentos
ditados pela Justiça criminal.
Curto Prazo - Levar à discussão, em âmbito nacional, sobre a
necessidade de se repensar as formas de punição ao cidadão
- Promover o debate, mediante encontros, seminários, com infrator, incentivando o Poder Judiciário a utilizar as penas
todos os setores vinculados ao tema da liberdade de expressão alternativas contidas nas leis vigentes com vistas a minimizar
e da classificação indicativa de espetáculos e diversões a crise do sistema penitenciário.
públicas, buscando, via de regra, uma ação integrada e voltada - Propor legislação para introduzir penas alternativas à
para o interesse público nesse assunto. prisão para os crimes não violentos.
- Propor alteração na legislação existente sobre faixa etária - Estimular a criação de cursos de formação de agentes
com vistas a adequá-las aos dias e necessidades atuais. penitenciários.
- Estabelecer com os produtores e distribuidores de - Propor normatização dos procedimentos de revista aos
programação um diálogo, franco, cordial e aberto visando a visitantes de estabelecimentos prisionais, com o objetivo de
cooperação e sensibilização desses setores para o coibir quaisquer ações que atentem contra dignidade e os
cumprimento da legislação em vigor, convidando-os a uma direitos humanos dessas pessoas.
participação efetiva neste processo.
- Estruturar o Departamento de Classificação Indicativa do Médio prazo
Ministério da Justiça, de modo dotá-lo de capacidade operativa
compatível com sua missão institucional. - Incentivar a agilização dos procedimentos judiciais, a fim
de reduzir o número de detidos à espera de julgamento.
Médio Prazo - Promover programas de educação, treinamento
profissional e trabalho para facilitar a reeducação e
- Criar um sistema de avaliação permanente sobre os recuperação do preso.
critérios de classificação indicativa e faixa etária. - Desenvolver programas de assistência integral à saúde do
- Promover o mapeamento dos programas radiofônicos e preso e de sua família.
televisivos que estimulem a apologia do crime, da violência, da - Proporcionar incentivos fiscais, creditícios e outros às
tortura, das discriminações, do racismo, da ação de grupos de empresas que empreguem egressos do sistema penitenciário.
extermínio, de grupos paramilitares e da pena de morte, com - Realizar levantamento epidemiológico da população
vistas a identificar responsáveis e adotar as medidas legais carcerária brasileira.
pertinentes. - Incrementar a descentralização dos estabelecimentos
penais, com a construção de presídios de pequeno porte que
Trabalho forçado facilitem a execução da pena próximo aos familiares dos
presos.
Curto prazo
Longo Prazo
- Rever a legislação para coibir o trabalho forçado.
- Fortalecer os mecanismos para fiscalizar e coibir o - Incrementar a desativação da Casa de Detenção de São
trabalho forçado, com vista a eficácia do Programa de Paulo (Carandiru), e de outros estabelecimentos
Erradicação do Trabalho Forçado e do aliciamento de penitenciários que contrariem as normas mínimas
trabalhadores - PERFOR, criado pelo Decreto de 03 de penitenciárias internacionais.
setembro de 1992.
- Apoiar o Grupo Executivo de Repressão ao Trabalho Proteção do direito a tratamento igualitário perante a
Forçado - GERTRAF, vinculado ao Ministério do Trabalho. lei
- Incentivar a ampliação dos Serviços de Fiscalização Móvel
do Ministério do Trabalho com vistas à coibição do trabalho Direitos Humanos, Direitos de Todos:
forçado.
Curto Prazo
Médio prazo
- Propor legislação proibindo todo tipo de discriminação,
- Criar, nas organizações policiais, divisões especializadas com base em origem, raça, etnia, sexo, idade, credo religioso,
de coibição ao trabalho forçado, com atenção especial para as convicção política ou orientação sexual, e revogando normas
crianças, adolescentes, estrangeiros e migrantes brasileiros. discriminatórias na legislação infraconstitucional, de forma a
reforçar e consolidar a proibição de práticas discriminatórias
Penas privativas de liberdade existente na legislação constitucional.
- Estimular a criação de canais de acesso direto e regular
Curto prazo da população a informações e documentos governamentais
para tornar o funcionamento do Executivo, Legislativo e
- Reativar e difundir nos Estados o sistema de informática Judiciário mais transparente, como, por exemplo, a criação de
penitenciária - INFORPEN, de forma a agilizar processos e um banco de dados que possibilite, inclusive, o
julgamentos e evitar excessos no cumprimento de pena. acompanhamento da tramitação de investigações e processos
- Apoiar programas de emergência para corrigir as legais relativos a casos de violação de direitos humanos.
condições inadequadas das prisões, criar novos - Lançar uma campanha nacional, envolvendo Estados e
estabelecimentos e aumentar o número de vagas no país, em Municípios, com o objetivo de dotar todos os cidadãos, num
parceria com os Estados, utilizando-se recursos do Fundo prazo de um ano, dos documentos fundamentais de cidadania,
Penitenciário Nacional - FUNPEN. tais como certidão de nascimento, carteira de identidade,
- Estimular a aplicação dos dispositivos da Lei de carteira de trabalho, título de eleitor e certificado de
Execuções Penais referentes a regimes de prisão semiaberto e alistamento militar (ou certificado de reservista ou certificado
aberto. de dispensa de incorporação).

Conhecimentos Específicos 36
Apostila Digital Licenciada para Alice Caroline Guarino dos Santos - alice.guarino@hotmail.com (Proibida a Revenda)
APOSTILAS OPÇÃO

- Instituir a concessão gratuita das certidões de Proteção de Crianças e Adolescentes em Matéria de Adoção
nascimento e de óbito para todos os cidadãos. Internacional, realizada em Haia (1993), com a designação de
- Melhorar a qualidade do tratamento das pessoas uma autoridade central em matéria de adoções internacionais
dependentes do consumo de drogas ilícitas, o que deve incluir no Brasil.
a ampliação da acessibilidade e a diminuição do seu custo. - Apoiar a criação, pelos tribunais de justiça dos Estados,
- Incentivar estudos, pesquisas e programas para limitar a de comissões de adoção.
incidência e o impacto do consumo de drogas ilícitas. - Incentivar a criação de estruturas para o
- Apoiar ações para implementação do PANAD - Programa desenvolvimento de programas sócio-educativos para o
de Ação Nacional Antidrogas. atendimento de adolescentes infratores.
- Apoiar a participação das pessoas portadoras de - Promover, em parceria com Governos estaduais e
HIV/AIDS e suas organizações na formulação e implementação municipais e com a sociedade civil, campanhas educativas
de políticas e programas de combate e prevenção do relacionadas às situações de risco vivenciadas pela criança e
HIV/AIDS. pelo adolescente, como violência doméstica e sexual,
- Incentivar campanhas de informação sobre HIV/AIDS, prostituição, exploração no trabalho e uso de drogas, visando
visando esclarecer a população sobre os comportamentos que a criar e manter um padrão cultural favorável aos direitos da
facilitem ou dificultem a sua transmissão. criança e do adolescente.
- Apoiar a melhoria da qualidade do tratamento das - Estender o Programa de Merenda Escolar às creches.
pessoas com HIV/AIDS, o que deve incluir a ampliação da - Apoiar o Programa Brasil Criança Cidadã, desenvolvido
acessibilidade e a diminuição do seu custo. pela Secretaria de Assistência Social do MPAS.
- Incentivar estudos, pesquisas e programas para limitar a - Apoiar o Fórum Nacional de Prevenção e Erradicação do
incidência e o impacto do HIV/AIDS. Trabalho Infantil, coordenado pelo Ministério do Trabalho.
- Estimular a criação de PROCONs municipais. - Promover a discussão do papel dos meios de
- Estimular, a exemplo da Ordem dos Advogados do Brasil comunicação no combate à exploração sexual infanto-juvenil.
e da Federação Nacional de Jornalistas, a criação de
secretarias, departamentos ou comissões de direitos humanos Médio Prazo
e cidadania nos sindicatos, centrais de trabalhadores,
federações e entidades populares e estudantis. - Investir na formação e capacitação de profissionais e
encarregados da implementação da política de direitos da
Médio Prazo criança e do adolescente nos Governos estaduais e municipais
e nas organizações não governamentais,
- Instituir a carteira nacional de identidade. - Implantar sistema nacional e sistemas estaduais de
informação e monitoramento da situação da criança e do
Crianças e Adolescentes. adolescente, focalizando principalmente: (a) criação e
funcionamento de Conselhos de Direitos da Criança e do
Curto Prazo Adolescente e Conselhos Tutelares; (b) localização e
identificação de crianças e adolescentes desaparecidos; (c)
- Apoiar o funcionamento do Conselho Nacional dos violação de direitos de crianças e adolescentes, que contemple
Direitos da Criança e do Adolescente - CONANDA. o número de denúncias, número de processos, local da
- Incentivar programas de orientação familiar com o ocorrência, faixa etária e cor das crianças e adolescentes
objetivo de capacitar as famílias a resolver conflitos familiares envolvidos, número de casos; (d) prostituição Infanto-juvenil;
de forma não violenta, e a cumprir suas responsabilidades de (e) mortes violentas de crianças e adolescentes.
cuidar e proteger as crianças.
- Propor alterações na legislação penal com o objetivo de Longo Prazo
limitar a incidência da violência doméstica contra as crianças
e adolescentes. - Incentivar o reordenamento das instituições privativas
- Propor alterações na legislação penal e incentivar ações de liberdade para menores infratores, reduzindo o número de
com o objetivo de eliminar o trabalho infantil, punindo a adolescentes autores de ato infracional por unidade de
prática de sua exploração. atendimento, com prioridade na implementação das demais
- Dar continuidade à Campanha Nacional de Combate à medidas socioeducativas previstas no Estatuto da Criança e do
Exploração Sexual Infanto-juvenil. Adolescente.
- Incentivar a criação de estruturas para o - Apoiar a criação, pelo Poder Judiciário, Ministério Público
desenvolvimento de programas socioeducativos para o e pelos Governos estaduais, de varas, promotorias e delegacias
atendimento de adolescentes infratores. especializadas em infrações penais envolvendo menores,
- Propor a alteração da legislação no tocante à tipificação como previsto no Estatuto da Criança e do Adolescente.
de crime de exploração sexual infanto-juvenil, com
penalização para o explorador e usuário. Mulheres.
- Incentivar a criação, nos Estados e Municípios do País, dos
Conselhos dos Direitos da Criança e do Adolescente, Conselhos Curto prazo
Tutelares e Fundos dos Direitos da Criança e do Adolescente
- Incentivar os programas de capacitação de conselheiros - Apoiar o Conselho Nacional dos Direitos da Mulher na
à distância. formulação e implementação de políticas públicas para a
- Apoiar a produção e publicação de documentos que defesa dos direitos da mulher.
contribuam para a divulgação e aplicação do Estatuto da - Apoiar o Programa Nacional de Combate à Violência
Criança e do Adolescente. Contra a Mulher, do Governo federal.
- Instituir uma política nacional de estímulo à adoção, - Incentivar a criação de centros integrados de assistência
principalmente por famílias brasileiras, de crianças e a mulheres sob risco de violência doméstica e sexual.
adolescentes efetivamente abandonadas, a fim de lhes - Apoiar as políticas dos Governos estaduais e municipais
possibilitar a convivência familiar. para prevenção da violência doméstica e sexual contra as
- Apoiar a regulamentação do decreto legislativo que mulheres.
promulgou a Convenção sobre Cooperação Internacional e

Conhecimentos Específicos 37
Apostila Digital Licenciada para Alice Caroline Guarino dos Santos - alice.guarino@hotmail.com (Proibida a Revenda)
APOSTILAS OPÇÃO

- Incentivar a pesquisa e divulgação de informações sobre pelos órgãos da administração direta e indireta e por empresas
a violência e discriminação contra a mulher e sobre formas de estatais do Governo Federal.
proteção e promoção dos direitos da mulher. - Apoiar a definição de ações de valorização para a
- Assegurar o cumprimento dos dispositivos existentes na população negra e com políticas públicas.
Lei n&ordm; 9.029/95, que dá proteção às mulheres contra - Apoiar as ações da iniciativa privada que realizem
discriminação em razão de gravidez. discriminação positiva.
- Apoiar o Grupo GTEDEO, criado pelo decreto 20.3.96 para - Estimular as Secretarias de Segurança Pública dos
definir Ações de Combate à Discriminação da Mulher. Estados a realizarem cursos de reciclagem e seminários sobre
- Apoiar o projeto de lei que altera o Código Penal nos discriminação racial.
crimes de estupro e atentado violento à mulher.
Médio prazo
Médio prazo
- Revogar normas discriminatórias ainda existentes na
- Regulamentar o artigo 7&deg;, inciso XX, da Constituição legislação infraconstitucional.
Federal, que prevê a proteção do mercado de trabalho da - Aperfeiçoar as normas de combate à discriminação
mulher através de incentivos específicos. contra a população negra.
- Revogar as normas discriminatórias ainda existentes na - Criar banco de dados sobre a situação dos direitos civis,
legislação infraconstitucional, incluindo particularmente as políticos, sociais, econômicos e culturais da população negra
normas do Código Civil Brasileiro que tratam do pátrio poder, na sociedade brasileira que oriente políticas afirmativas
chefia da sociedade conjugal, direito da anulação do visando a promoção dessa comunidade.
casamento pelo homem quando a mulher não é virgem, - Promover o mapeamento e tombamento dos sítios e
privilégio do homem na fixação do domicílio familiar. documentos detentores de reminiscências históricas, bem
- Reformular as normas de combate à violência e como a proteção das manifestações culturais afro-brasileiras.
discriminação contra as mulheres, em particular, apoio ao - Propor projeto de lei, visando a regulamentação dos art.
projeto do Governo que trata o estupro como crime contra a 215, 216 e 242 da Constituição Federal.
pessoa e não mais como crime contra os costumes. - Desenvolver ações afirmativas para o acesso dos negros
- Incentivar a inclusão da perspectiva de gênero na aos cursos profissionalizantes, à universidade e ás áreas de
educação e treinamento de funcionários públicos, civis e tecnologia de ponta.
militares e nas diretrizes curriculares para o ensino - Determinar ao IBGE a adoção do critério de se considerar
fundamental e médio, com o objetivo de promover mudanças os mulatos, os pardos e os pretos como integrantes do
na mentalidade e atitude e o reconhecimento da igualdade de contingente da população negra.
direitos das mulheres, não apenas na esfera dos direitos civis - Adotar o princípio da criminalização da prática do
e políticos, mas também na esfera dos direitos econômicos, racismo, nos Códigos Penal e de Processo Penal.
sociais e culturais. - Estimular que os livros didáticos enfatizem a história e as
- Incentivar a geração de estatísticas que evidenciem lutas do povo negro na construção do nosso país, eliminando
salários, jornadas de trabalho, ambientes de trabalho, doenças estereótipos e discriminações.
profissionais e direitos trabalhistas da mulher. - Divulgar as Convenções Internacionais, os dispositivos da
Constituição Federal e a legislação infraconstitucional que
Longo prazo tratam do racismo.
- Apoiar a produção e publicação de documentos que
- Definir políticas e programas governamentais, nas contribuam para a divulgação da legislação anti-
esferas federal, estadual e municipal, para implementação das discriminatória.
leis que asseguram a igualdade de direitos das mulheres e dos - Facilitar a discussão e a articulação entre as entidades da
homens em todos os níveis, incluindo saúde, educação e comunidade negra e os diferentes setores do Governo, para
treinamento profissional, trabalho, segurança social, desenvolver planos de ação e estratégias na valorização da
propriedade e crédito rural, cultura, política e justiça. comunidade negra.

População Negra. Longo prazo

Curto prazo - Incentivar ações que contribuam para a preservação da


memória e fomento à produção cultural da comunidade negra
- Apoiar o grupo de trabalho interministerial criado por no Brasil.
Decreto Presidencial de 20 de novembro de 1995 com o - Formular políticas compensatórias que promovam social
objetivo de sugerir ações e políticas de valorização da e economicamente a comunidade negra.
população negra.
- Inclusão do quesito "cor" em todos e quaisquer sistemas Sociedades Indígenas
de informação e registro sobre a população e bancos de dados
públicos. Curto prazo
- Apoiar o Grupo de Trabalho para a Eliminação da - Formular e implementar políticas de proteção e
Discriminação no Emprego e na Ocupação - GTEDEO, promoção dos direitos das sociedades indígenas, em
instituído no âmbito do Ministério do Trabalho, pelo decreto substituição a políticas assimilacionistas e assistencialistas.
de 20 de março de 1996. O GTEDEO de constituição tripartite, - Apoiar a revisão do Estatuto do Índio (Lei 6.001/73), no
deverá definir um programa de ações e propor estratégias de sentido apontado pelo projeto de lei do Estatuto das
combate à discriminação no emprego e na ocupação, conforme Sociedades Indígenas, já aprovado na Câmara dos Deputados.
os princípios da Convenção 111, da Organização Internacional - Assegurar a participação das sociedades indígenas e de
do Trabalho - OIT. suas organizações na formulação e implementação de políticas
- Incentivar e apoiar a criação e instalação, a níveis de proteção e promoção de seus direitos.
estadual e municipal, de Conselhos da Comunidade Negra. - Assegurar o direito das sociedades indígenas às terras
- Estimular a presença dos grupos étnicos que compõem a que eles tradicionalmente ocupam.
nossa população em propagandas institucionais contratadas

Conhecimentos Específicos 38
Apostila Digital Licenciada para Alice Caroline Guarino dos Santos - alice.guarino@hotmail.com (Proibida a Revenda)
APOSTILAS OPÇÃO

- Demarcar e regularizar as terras tradicionalmente Terceira Idade


ocupadas por sociedades indígenas que ainda não foram
demarcadas e regularizadas. Curto prazo
- No contexto do processo de demarcação das terras
indígenas, apoiar ações que contribuam para o aumento do - Estabelecer prioridade obrigatória de atendimento às
grau de confiança e de estabilidade das relações entre as pessoas idosas em todas as repartições públicas e
organizações governamentais e não governamentais, através estabelecimentos bancários do país.
de seminários, oficinas e projetos que contribuam para - Facilitar o acesso das pessoas idosas a cinemas, teatros,
diminuir a desinformação, o medo e outros fatores que shows de música e outras formas de lazer público.
contribuam para o acirramento dos conflitos e para violência - Apoiar as formas regionais denominadas ações
contra os índios. governamentais integradas, para o desenvolvimento da
- Dotar a FUNAI de recursos suficientes para a realização Política Nacional do Idoso.
de sua missão de defesa dos direitos das sociedades indígenas,
particularmente no processo de demarcação das terras Médio Prazo
indígenas.
- Garantir às sociedades indígenas assistência na área da - Criar e fortalecer conselhos e organizações de
saúde, com a implementação de programas de saúde representação dos idosos, incentivando sua participação nos
diferenciados, considerando as especificidades dessas programas e projetos governamentais de seu interesse.
populações. - Incentivar o equipamento de estabelecimentos públicos e
- Assegurar à sociedades indígenas uma educação escolar meios de transporte de forma a facilitar a locomoção dos
diferenciada, respeitando o seu universo sociocultural. idosos.
- Promover a divulgação de informação sobre os indígenas
e os seus direitos, principalmente nos meios de comunicação e Longo prazo
nas escolas, como forma de eliminar a desinformação (uma das
causas da discriminação e da violência contra os indígenas e - Generalizar a concessão de passe livre e precedência de
suas culturas). acesso aos idosos em todos os sistemas de transporte público
urbano.
Médio prazo - Criar, fortalecer e descentralizar programas de
assistência aos idosos, de forma a contribuir para sua
- Implantar sistema de vigilância permanente em terras integração à família e à sociedade e incentivar o seu
indígenas, com unidades móveis de fiscalização, com atendimento no seu próprio ambiente.
capacitação de servidores e membros da própria comunidade
indígena. Pessoas portadoras de deficiência
- Levantar informações sobre conflitos fundiários e
violência em terras indígenas, a ser integrado ao mapa dos Curto prazo
conflitos fundiários e violência rural no Brasil.
- Formular políticas de atenção às pessoas portadoras de
Longo prazo deficiência, para a implementação de uma estratégia nacional
de integração das ações governamentais e não-
- Reorganizar a FUNAI para compatibilizar a sua governamentais, com vistas ao efetivo cumprimento do
organização com a função de defender os direitos das Decreto n&ordm; 914, de 06 de setembro de 1993
sociedades indígenas. - Propor normas relativas ao acesso do portador de
- Apoiar junto às comunidades indígenas o deficiência ao mercado de trabalho e no serviço público, nos
desenvolvimento de projetos auto-sustentáveis do ponto de termos do art. 37, VIII da Constituição Federal.
vista econômico, ambiental e cultural. - Adotar medidas que possibilitem o acesso das pessoas
portadores de deficiências às informações veiculadas pelos
Estrangeiros, Refugiados e Migrantes Brasileiros meios de comunicação.

Curto prazo Médio prazo

- Desenvolver programa e campanha visando à - Formular programa de educação para pessoas


regularização da situação dos estrangeiros atualmente no país. portadoras de deficiência.
- Adotar medidas para impedir e punir a violência e - Implementar o programa de remoção de barreiras físicas
discriminação contra estrangeiros no Brasil e migrantes que impedem ou dificultam a locomoção das pessoas
brasileiros no exterior. portadoras de deficiência, ampliando o acesso às cidades
- Propor projeto de lei estabelecendo o estatuto dos históricas, turísticas, estâncias hidrominerais e grandes
refugiados. centros urbanos, como vistos no projeto "Cidade para todos".

Médio prazo Longo prazo

- Estabelecer política de proteção aos direitos humanos - Conceber sistemas de informações com a definição de
das comunidades estrangeiras no Brasil bases de dados relativamente a pessoas portadores de
- Estabelecer política de proteção aos direitos humanos deficiência, à legislação, ajudas técnicas, bibliografia e
das comunidades brasileiras no exterior capacitação na área de reabilitação e atendimento.

Longo prazo Educação e Cidadania. Bases para uma cultura de


Direitos Humanos.
- Reformular a Lei dos Estrangeiros, através da apreciação
pelo Congresso do projeto de lei 1813/91, que regula a Produção e Distribuição de Informações e Conhecimento
situação jurídica do estrangeiro no Brasil.

Conhecimentos Específicos 39
Apostila Digital Licenciada para Alice Caroline Guarino dos Santos - alice.guarino@hotmail.com (Proibida a Revenda)
APOSTILAS OPÇÃO

Curto prazo Longo prazo

- Criar e fortalecer programas de educação para o respeito - Incentivar campanhas de esclarecimento da opinião
aos direitos humanos nas escolas de primeiro, segundo e pública sobre os candidatos a cargos públicos e lideranças da
terceiro grau, através do sistema de "temas transversais" nas sociedade civil comprometidos com a proteção e promoção
disciplinas curriculares, atualmente adotado pelo Ministério dos direitos humanos.
da Educação e do Desporto, e através da criação de uma
disciplina sobre direitos humanos. Ações internacionais para proteção e promoção dos
- Apoiar a criação e desenvolvimento de programas de Direitos Humanos:
ensino e de pesquisa que tenham como tema central a
educação em direitos humanos. Ratificação de atos internacionais
- Incentivar campanha nacional permanente que amplie a
Curto Prazo
compreensão da sociedade brasileira sobre o valor da vida
humana e a importância do respeito aos direitos humanos. - Ratificar a Convenção 138 e implementar a
- Incentivar, em parceria com a sociedade civil, a criação de Recomendação 146 da OIT, que tratam da idade mínima para
prêmios, bolsas e outras distinções regionais para entidades e admissão no emprego.
personalidades que tenham se destacado periodicamente na - Ratificar a Convenção 169 sobre Povos Indígenas e
luta pelos direitos humanos. Tribais em Países Independentes, aprovada pela OIT em 1989.
- Estimular os partidos políticos e os tribunais eleitorais a - Ratificar a Convenção Internacional para Proteção dos
Direitos dos Migrantes e de suas Famílias, aprovada pela ONU
reservarem parte do seu espaço específico à promoção dos
em 1990.
direitos humanos. - Ratificar a Convenção Interamericana de
- Atribuir, anualmente, o Prêmio Nacional de Direitos Desaparecimento Forçado de Pessoas, assinada em Belém/PA
Humanos. em 9/06/94;
- Adotar legislação interna que permita o cumprimento
Médio prazo pelo Brasil dos compromissos assumidos internacionalmente,
como Estado parte, em convenções e tratados de direitos
humanos.
- Incentivar a criação de canais de acesso direto da
- Dar continuidade à política de adesão a tratados
população a informações e meios de proteção aos direitos internacionais para proteção e promoção dos direitos
humanos, como linhas telefônicas especiais. humanos, através da sua ratificação e implementação.

Conscientização e Mobilização pelos Direitos Humanos Implementação e divulgação de atos internacionais

Curto prazo
Curto prazo
- Desenvolver campanhas de divulgação, através de
- Apoiar programas de informação, educação e veículos de comunicação, das principais declarações e
treinamento de direitos humanos para profissionais de direito, convenções internacionais para proteção e promoção dos
policiais, agentes penitenciários e lideranças sindicais, direitos humanos assinadas pelo Brasil, a fim de deixar claro
associativas e comunitárias, para aumentar a capacidade de quais são os compromissos assumidos pelo Brasil na área da
proteção e promoção dos direitos humanos na sociedade proteção e promoção dos direitos humanos.
- Implementar a Conferência Mundial dos Direitos
brasileira.
Humanos - 1993, em Viena, que define a violência contra as
- Orientar tais programas na valorização da moderna mulheres como violência contra os direitos humanos;
concepção dos direitos humanos segundo a qual o respeito à - Implementar a Convenção Interamericana para prevenir,
igualdade supõe também a tolerância com as diferenças e punir e erradicar a violência contra a mulher (junho de 1994).
peculiaridades de cada indivíduo. - Implementar a IV Conferência Mundial da Mulher
- Apoiar a realização de fóruns, seminários e "workshops" (Beijing, setembro de 1995);
na área de direitos humanos. - Apoiar, junto com o Ministério das Relações Exteriores, a
elaboração do Plano Hemisférico de Direitos Humanos, no
contexto da implementação da Declaração de Princípios e do
Médio prazo Plano de Ação, aprovados pela Cúpula de Américas realizada
em Miami em 1994.
- Incentivar a criação de bancos de dados sobre entidades, No contexto da implementação da Declaração de
representantes políticos, empresas, sindicatos, igrejas, escolas Princípios e do Plano de Ação da Cúpula das Américas, criar
e associações comprometidos com a proteção e promoção dos um sistema hemisférico de divulgação dos princípios e ações
de proteção à cidadania e aos direitos humanos, apoiar
direitos humanos.
programas internacionais para limitar a incidência e impacto
- Apoiar a representação proporcional de grupos e do terrorismo, do tráfico de drogas e do HIV/AIDS.
comunidades minoritárias do ponto de vista étnico, racial e de - Implementar as convenções internacionais das quais o
gênero nas campanhas de publicidade e de comunicação de Brasil é signatário, como as que tratam dos direitos da criança
agências governamentais. e do adolescente, em particular cumprindo prazos na entrega
de planos de ação e relatórios.
- Implementar propostas de proteção dos direitos da
mulher contidas nos seguintes documentos: A) Declaração e
Proposta de Ação da Conferência Mundial de Direitos
Humanos, realizada em Viena em 1993; B) Declaração sobre

Conhecimentos Específicos 40
Apostila Digital Licenciada para Alice Caroline Guarino dos Santos - alice.guarino@hotmail.com (Proibida a Revenda)
APOSTILAS OPÇÃO

Eliminação da Violência Contra a Mulher, aprovada pela ONU humanos tanto no Programa Nacional quanto no plano
em 1993; C) Convenção Interamericana para Prevenir, Punir e internacional.
Erradicar a Violência contra a Mulher ("Convenção de Belém - Desenvolver campanha publicitária no âmbito nacional,
do Pará"), aprovada pela OEA em 1994. através dos vários meios de comunicação social, com o
- Implementar a Convenção Internacional sobre a objetivo de esclarecer e sensibilizar o País para a importância
Eliminação de Todas as Formas de Discriminação Racial. dos direitos humanos e do Programa Nacional de Direitos
- Implementar as Convenções 29, 105 e 111 da OIT, que Humanos.
tratam do trabalho forçado e da discriminação nos locais de - Atribuir ao Ministério da Justiça, através de órgão a ser
trabalho. designado, a responsabilidade pela coordenação da
- Dar cumprimento à obrigação de submeter relatórios implementação e atualização do Programa Nacional de
periódicos sobre a implementação de convenções e tratados Direitos Humanos, inclusive sugestões e queixas sobre o seu
de direitos humanos, dos quais o Brasil seja parte. cumprimento. Atribuir a entidades equivalentes a
responsabilidade pela coordenação da implementação do
Médio prazo Programa nos estados e municípios.
- Promover estudos visando à criação de um sistema de
- Dar publicidade e divulgação aos textos dos tratados e concessão de incentivos por parte do Governo federal aos
convenções internacionais de direitos humanos de que o Brasil Governos estaduais que implementarem medidas favoráveis
seja parte. aos direitos humanos previstas no Programa Nacional de
Direitos Humanos.
Apoio a organizações e operações de defesa dos
direitos humanos Monitoramento

Curto prazo - Atribuir ao Ministério da Justiça a responsabilidade de


apresentar ao Presidente da República relatórios
- Promover o intercâmbio internacional de experiências quadrimestrais sobre a implementação do Programa Nacional
em matéria de proteção e promoção dos direitos humanos. de Direitos Humanos, face à situação dos direitos humanos no
- Promover o intercâmbio internacional de experiências na Brasil.
área da educação e treinamento de forças policiais visando - Destinar aos Governos estaduais a responsabilidade de
melhor prepará-las para limitar a incidência e o impacto de elaborar e apresentar ao Ministério da Justiça relatórios
violações dos direitos humanos no combate à criminalidade e quadrimestrais e anuais sobre a implementação do Programa
à violência. Nacional de Direitos Humanos e a situação dos direitos
- Criar e fortalecer programas internacionais de apoio a humanos no respectivo Estado.
projetos nacionais que visem a proteção e promoção dos
direitos humanos, em particular da reforma e melhoria dos Programa Nacional de Direitos Humanos- 2 versão
sistemas judiciários e policiais.
- Apoiar a elaboração do protocolo facultativo adicional à Introdução
Convenção contra tortura e outros tratamentos, ou penas
cruéis, desumanas ou degradantes. Decorridos mais de cinco anos do lançamento do Programa
Nacional de Direitos Humanos - PNDH, pode-se afirmar com
Médio prazo segurança que o Brasil avançou significativamente na questão
da promoção e proteção dos direitos humanos. Graças ao
- Fortalecer a cooperação com organismos internacionais PNDH, foi possível sistematizar demandas de toda a sociedade
de proteção aos direitos humanos, em particular a Comissão brasileira com relação aos direitos humanos e identificar
de Direitos Humanos da ONU, a Comissão Interamericana de alternativas para a solução de problemas estruturais,
Direitos Humanos, a Corte Interamericana de Direitos subsidiando a formulação e implementação de políticas
Humanos e o Instituto Interamericano de Direitos Humanos. públicas e fomentando a criação de programas e órgãos
- Apoiar a elaboração da Declaração sobre os Direitos das estaduais concebidos sob a ótica da promoção e garantia dos
sociedades Indígenas, da ONU. direitos humanos.
- Incentivar a ratificação dos instrumentos internacionais A criação da Secretaria de Estado dos Direitos Humanos,
de proteção e promoção dos direitos humanos pelos países no âmbito do Ministério da Justiça, possibilitou o engajamento
com os quais o Brasil possui relações diplomáticas. efetivo do Governo Federal em ações voltadas para a proteção
- Desenvolver no País o Plano de Ação da Década para a e promoção de direitos humanos. As metas do PNDH foram, em
Educação em Direitos Humanos, aprovado pela Organização sua maioria, sendo incorporadas aos instrumentos de
das Nações Unidas em 1994 para o período 1995-2004. planejamento e orçamento do Governo Federal, convertendo-
se em programas e ações específicas com recursos financeiros
Implementação e Monitoramento do Programa assegurados nas Leis Orçamentárias Anuais, conforme
Nacional de Direitos Humanos determina o Plano Plurianual (PPA).

Implementação Entre as principais medidas legislativas que resultaram de


proposições do PNDH figuram o reconhecimento das mortes
- Criar um serviço civil constituído por jovens formados de pessoas desaparecidas em razão de participação política
como agentes da cidadania, que possam atuar na proteção dos (Lei nº 9.140/95), pela qual o Estado brasileiro reconheceu a
direitos humanos em todos os estados do país. responsabilidade por essas mortes e concedeu indenização
- Elaborar um Manual dos Direitos Humanos, a ser aos familiares das vítimas; a transferência da justiça militar
distribuído nos Estados e Municípios, para informar, educar e para a justiça comum dos crimes dolosos contra a vida
treinar os integrantes de organizações governamentais e não praticados por policiais militares (Lei 9.299/96), que permitiu
governamentais responsáveis pela implementação do o indiciamento e julgamento de policiais militares em casos de
Programa Nacional de Direitos Humanos, e para deixar claro múltiplas e graves violações como os do Carandiru,
os compromissos assumidos pelo Brasil na área de direitos Corumbiara e Eldorado dos Carajás; a tipificação do crime de
tortura (Lei 9.455/97), que constituiu marco referencial para

Conhecimentos Específicos 41
Apostila Digital Licenciada para Alice Caroline Guarino dos Santos - alice.guarino@hotmail.com (Proibida a Revenda)
APOSTILAS OPÇÃO

o combate a essa prática criminosa no Brasil; e a construção da reivindicação formulada pela sociedade civil por ocasião da IV
proposta de reforma do Poder Judiciário, na qual se inclui, Conferência Nacional de Direitos Humanos, realizada em 13 e
entre outras medidas destinadas a agilizar o processamento 14 de maio de 1999 na Câmara dos Deputados, em Brasília.
dos responsáveis por violações, a chamada 'federalização' dos
crimes de direitos humanos. A atualização do Programa Nacional oferece ao governo e
O PNDH contribuiu ainda para ampliar a participação do à sociedade brasileira a oportunidade de fazer um balanço dos
Brasil nos sistemas global (da Organização das Nações Unidas progressos alcançados desde 1996, das propostas de ação que
- ONU) e regional (da Organização dos Estados Americanos - se tornaram programas governamentais e dos problemas
OEA) de promoção e proteção dos direitos humanos, por meio identificados na implementação do PNDH. A inclusão dos
da continuidade da política de adesão a pactos e convenções direitos econômicos, sociais e culturais, de forma consentânea
internacionais de direitos humanos e de plena inserção do País com a noção de indivisibilidade e interdependência de todos
no sistema interamericano. O aumento da cooperação com os direitos humanos expressa na Declaração e Programa de
órgãos internacionais de salvaguarda se evidenciou no Ação de Viena (1993), orientou-se pelos parâmetros definidos
número de relatores especiais das Nações Unidas que na Constituição Federal de 1988, inspirando-se também no
realizaram visitas ao Brasil nos últimos anos. Essas visitas Pacto Internacional sobre Direitos Econômicos, Sociais e
resultaram na elaboração de relatórios contendo conclusões e Culturais de 1966 e no Protocolo de São Salvador em Matéria
recomendações de grande utilidade para o aprimoramento de de Direitos Econômicos, Sociais e Culturais, ratificados pelo
diagnósticos e a identificação de medidas concretas para a Brasil em 1992 e 1996, respectivamente.
superação de problemas relacionados aos direitos humanos O PNDH II incorpora ações específicas no campo da
no Brasil. garantia do direito à educação, à saúde, à previdência e
Já visitaram o País os relatores da ONU sobre os temas da assistência social, ao trabalho, à moradia, a um meio ambiente
venda de crianças, prostituição e pornografia infantis; da saudável, à alimentação, à cultura e ao lazer, assim como
violência contra a mulher; do racismo, discriminação racial, propostas voltadas para a educação e sensibilização de toda a
xenofobia e intolerância correlata; dos direitos humanos e sociedade brasileira com vistas à construção e consolidação de
resíduos tóxicos; e, mais recentemente, sobre a tortura. Dando uma cultura de respeito aos direitos humanos. Atendendo a
seguimento à cooperação com os mecanismos temáticos das anseios da sociedade civil, foram estabelecidas novas formas
Nações Unidas, o Governo brasileiro estendeu convite aos de acompanhamento e monitoramento das ações
relatores especiais sobre execuções extrajudiciais, sumárias e contempladas no Programa Nacional, baseadas na relação
arbitrárias e sobre o direito à alimentação para que visitem o estratégica entre a implementação do programa e a elaboração
País no transcurso de 2002. dos orçamentos em nível federal, estadual e municipal. O
Da mesma forma, a cooperação com os órgãos de PNDH II deixa de circunscrever as ações propostas a objetivos
supervisão da OEA tem ensejado a busca de soluções de curto, médio e longo prazo, e passa a ser implementado por
amistosas para casos de violação em exame pela Comissão meio de planos de ação anuais, os quais definirão as medidas a
Interamericana de Direitos Humanos, possibilitando a serem adotadas, os recursos orçamentários destinados a
concessão de reparações e indenizações às vítimas dessas financiá-las e os órgãos responsáveis por sua execução.
violações ou a seus familiares, bem como a adoção de medidas O PNDH II será implementado, a partir de 2002, com os
administrativas e legislativas para prevenir a ocorrência de recursos orçamentários previstos no atual Plano Plurianual
novas violações. A aceitação da jurisdição compulsória da (PPA 2000-2003) e na lei orçamentária anual. Embora a
Corte Interamericana de Direitos Humanos representa, revisão do Programa Nacional esteja sendo apresentada à
ademais, garantia adicional a todos os brasileiros de proteção sociedade brasileira a pouco mais de um ano da posse do novo
dos direitos consagrados na Convenção Americana sobre governo, os compromissos expressos no texto quanto à
Direitos Humanos, quando as instâncias nacionais se promoção e proteção dos direitos humanos transcendem a
mostrarem incapazes de assegurar a realização da justiça. atual administração e se projetam no tempo,
No plano interno, os resultados da elaboração e independentemente da orientação política das futuras gestões.
implementação do PNDH podem ser medidos pela ampliação Nesse sentido, o PNDH II deverá influenciar a discussão, no
do espaço público de debate sobre questões afetas à proteção transcurso de 2003, do Plano Plurianual 2004-2007. O
e promoção dos direitos humanos, tais como o combate à Programa Nacional servirá também de parâmetro e orientação
exploração sexual de crianças e adolescentes, a reforma dos para a definição dos programas sociais a serem desenvolvidos
mecanismos de reinserção social do adolescente em conflito no País até 2007, ano em que se procederia a nova revisão do
com a lei, a manutenção da idade de imputabilidade penal, o PNDH.
combate a todas formas de discriminação, a adoção de As propostas de atualização foram discutidas em
políticas de ação afirmativa e de promoção da igualdade e o seminários regionais, com ampla participação de órgãos
combate à prática da tortura. Os esforços empreendidos no governamentais e de entidades da sociedade civil e,
campo da promoção e proteção dos direitos humanos se posteriormente, registradas e consolidadas pelo Núcleo de
pautaram na importância estratégica da coordenação entre os Estudos da Violência da Universidade de São Paulo - NEV/USP.
três níveis de governo e os poderes Executivo, Legislativo e Após esforço de sistematização e aglutinação, o PNDH
Judiciário, assim como da parceria entre órgãos atualizado engloba um total de 500 propostas, consideradas
governamentais e entidades da sociedade civil. todas as categorias de direitos, enquanto o programa original
Ao adotar, em 13 de maio de 1996, o Programa Nacional de previa 227 ações.
Direitos Humanos, o Brasil se tornou um dos primeiros países A etapa seguinte, de editoração e complementação de
do mundo a cumprir recomendação específica da Conferência propostas, foi desenvolvida pela Secretaria de Estado dos
Mundial de Direitos Humanos (Viena, 1993), atribuindo Direitos Humanos, do Ministério da Justiça, com a inestimável
ineditamente aos direitos humanos o status de política pública colaboração de todos os Ministérios e órgãos da área social e
governamental. Sem abdicar de uma compreensão integral e sob a coordenação da Casa Civil da Presidência da República.
indissociável dos direitos humanos, o programa original Finalizado o projeto de atualização do PNDH, a Secretaria de
conferiu maior ênfase à garantia de proteção dos direitos civis. Estado dos Direitos Humanos realizou, no período de 19 de
O processo de revisão do PNDH constitui um novo marco na dezembro de 2001 a 10 de janeiro de 2002, consulta pública
promoção e proteção dos direitos humanos no País, ao elevar através da internet, dela resultando, após correções e ajustes
os direitos econômicos, sociais e culturais ao mesmo patamar finais, o texto do PNDH II que ora se encaminha à publicação
de importância dos direitos civis e políticos, atendendo a no Diário Oficial da União.

Conhecimentos Específicos 42
Apostila Digital Licenciada para Alice Caroline Guarino dos Santos - alice.guarino@hotmail.com (Proibida a Revenda)
APOSTILAS OPÇÃO

Propostas Gerais eventos, manifestações públicas e conflitos, assim como a


proibição da exportação de armas de fogo para países
1. Apoiar a formulação, a implementação e a avaliação de limítrofes.
políticas e ações sociais para a redução das desigualdades 17. Promover, em parceria com entidades não-
econômicas, sociais e culturais existentes no país, visando à governamentais, a elaboração de mapas de violência urbana e
plena realização do direito ao desenvolvimento e conferindo rural, identificando as regiões que apresentem maior
prioridade às necessidades dos grupos socialmente incidência de violência e criminalidade e incorporando dados
vulneráveis. e indicadores de desenvolvimento, qualidade de vida e risco de
2. Apoiar, na esfera estadual e municipal, a criação de violência contra grupos vulneráveis.
conselhos de direitos dotados de autonomia e com composição 18. Estimular o aperfeiçoamento dos critérios para seleção
paritária de representantes do governo e da sociedade civil. e capacitação de policiais e implantar, nas Academias de
3. Apoiar a formulação de programas estaduais e polícia, programas de educação e formação em direitos
municipais de direitos humanos e a realização de conferências humanos, em parceria com entidades não-governamentais.
e seminários voltados para a proteção e promoção de direitos 19. Incluir no currículo dos cursos de formação de policiais
humanos. módulos específicos sobre gênero e raça, gerenciamento de
4. Apoiar a atuação da Comissão de Direitos Humanos da crises, técnicas de investigação, técnicas não-letais de
Câmara dos Deputados, a criação de comissões de direitos intervenção policial e mediação de conflitos.
humanos nas assembléias legislativas estaduais e câmaras 20. Propor a criação de programas de atendimento
municipais e o trabalho das comissões parlamentares de psicossocial para o policial e sua família, a obrigatoriedade de
inquérito constituídas para a investigação de crimes contra os avaliações periódicas da saúde física e mental dos
direitos humanos. profissionais de polícia e a implementação de programas de
5. Estimular a criação de bancos de dados com indicadores seguro de vida e de saúde, de aquisição da casa própria e de
?????z??¾sociais e econômicos sobre a situação dos direitos estímulo à educação formal e à profissionalização.
humanos nos estados brasileiros, a fim de orientar a definição 21. Apoiar estudos e programas para a redução da
de políticas públicas destinadas à redução da violência e à letalidade em ações envolvendo policiais.
inclusão social. 22. Apoiar o funcionamento e a modernização de
6. Apoiar, em todas as unidades federativas, a adoção de corregedorias estaduais independentes e desvinculadas dos
mecanismos que estimulem a participação dos cidadãos na comandos policiais, com vistas a limitar abusos e erros em
elaboração dos orçamentos públicos. operações policiais e a emitir diretrizes claras aos integrantes
7. Estimular a criação de mecanismos que confiram maior das forças policiais com relação à proteção dos direitos
transparência à destinação e ao uso dos recursos públicos, humanos.
aprimorando os mecanismos de controle social das ações 23. Fortalecer o Fórum Nacional de Ouvidores de Polícia -
governamentais e de combate à corrupção. FNOP, órgão de caráter consultivo vinculado à Secretaria de
Estado dos Direitos Humanos, e incentivar a criação e o
8. Ampliar, em todas as unidades federativas, as iniciativas fortalecimento de ouvidorias de polícia dotadas de autonomia
voltadas para programas de transferência direta de renda, a e poderes para receber, acompanhar e investigar denúncias.
exemplo dos programas de renda mínima, e fomentar o 24. Apoiar medidas destinadas a garantir o afastamento
envolvimento de organizações locais em seu processo de das atividades de policiamento de policiais envolvidos em
implementação. ocorrências letais e na prática de tortura, submetendo-os à
9. Realizar estudos para que o instrumento de ação direta avaliação e tratamento psicológico e a imediata instauração de
de inconstitucionalidade possa ser invocado no caso de processo administrativo, sem prejuízo do devido processo
adoção, por autoridades municipais, estaduais e federais, de criminal.
políticas públicas contrárias aos direitos humanos. 25. Fortalecer a Divisão de Direitos Humanos do
10. Garantir o acesso gratuito e universal ao registro civil Departamento de Polícia Federal.
de nascimento e ao assento de óbito. 26. Criar a Ouvidoria da Polícia Federal - OPF.
11. Apoiar a aprovação do Projeto de Lei nº 4715/1994, 27. Apoiar programas estaduais voltados para a integração
que transforma o Conselho de Defesa dos Direitos da Pessoa entre as polícias civil e militar, em especial aqueles com ênfase
Humana - CDDPH em Conselho Nacional dos Direitos na unificação dos comandos policiais.
Humanos - CNDH, ampliando sua competência e a participação 28. Reforçar a fiscalização e a regulamentação das
de representantes da sociedade civil. atividades das empresas de segurança privada, com
participação da Polícia Civil no controle funcional e da Polícia
Garantia do Direito à Vida Militar no controle operacional das ações previstas, bem como
determinar o imediato recadastramento de todas as empresas
12. Apoiar a execução do Plano Nacional de Segurança de segurança em funcionamento no País, proibindo o
Pública - PNSP. funcionamento daquelas em situação irregular.
13. Apoiar programas e ações que tenham como objetivo 29. Apoiar ações destinadas a reduzir a contratação ilegal
prevenir a violência contra grupos vulneráveis e em situação de profissionais de polícia e guardas municipais por empresas
de risco. de segurança privada.
14. Apoiar a implementação de ações voltadas para o 30. Incentivar ações educativas e preventivas destinadas a
controle de armas e o desarmamento, tais como a coordenação reduzir o número de acidentes e mortes no trânsito.
centralizada do controle de armas, o Cadastro Nacional de 31. Incentivar a implantação da polícia ou segurança
Armas Apreendidas - CNAA e o Sistema Nacional de Armas - comunitária e de ações de articulação e cooperação entre a
SINARM, assim como campanhas de desarmamento e ações de comunidade e autoridades públicas com vistas ao
recolhimento/apreensão de armas ilegais. desenvolvimento de estratégias locais de segurança pública,
15. Propor a edição de norma federal regulamentando a visando a garantir a proteção da integridade física das pessoas
aquisição de armas de fogo e munição por policiais, guardas e dos bens da comunidade e o combate à impunidade.
municipais e agentes de segurança privada. 32. Apoiar a criação e o funcionamento de centros de apoio
16. Apoiar a edição de norma federal que regule o uso de a vítimas de crime nas áreas com maiores índices de violência,
armas de fogo e munição por policiais, guardas municipais e com vistas a disponibilizar assistência social, jurídica e
agentes de segurança privada, especialmente em grandes

Conhecimentos Específicos 43
Apostila Digital Licenciada para Alice Caroline Guarino dos Santos - alice.guarino@hotmail.com (Proibida a Revenda)
APOSTILAS OPÇÃO

psicológica às vítimas de violência e a seus familiares e 47. Criar e fortalecer ouvidorias nos órgãos públicos da
dependentes. União e dos estados para o atendimento de denúncias de
33. Apoiar a realização de estudos e pesquisas de violação de direitos fundamentais, com ampla divulgação de
vitimização, com referência específica a indicadores de gênero sua finalidade nos meios de comunicação.
e raça, visando a subsidiar a formulação, implementação e 48. Criar e fortalecer a atuação de ouvidorias gerais nos
avaliação de programas de proteção dos direitos humanos. Estados.
34. Estimular a avaliação de programas e ações na área de 49. Apoiar a expansão dos serviços de prestação da justiça,
segurança pública e a identificação de experiências inovadoras para que estes se façam presentes em todas as regiões do país.
e bem sucedidas que possam ser reproduzidas nos estados e 50. Apoiar medidas legislativas destinadas a transferir, da
municípios. Justiça Militar para a Comum, a competência para processar e
35. Implantar e fortalecer sistemas de informação nas julgar todos os crimes cometidos por policiais militares no
áreas de segurança e justiça, como o INFOSEG, de forma a exercício de suas funções.
permitir o acesso à informação e a integração de dados sobre 51. Incentivar a prática de plantões permanentes no
identidade criminal, mandados de prisão e situação da Judiciário, Ministério Público, Defensoria Pública e Delegacias
população carcerária em todas as unidades da Federação. de Polícia.
36. Criar bancos de dados sobre a organização e o 52. Fortalecer os Institutos Médico-Legais ou de
funcionamento das polícias e sobre o fluxo das ocorrências no Criminalística, adotando medidas que assegurem a sua
sistema de justiça criminal. excelência técnica e progressiva autonomia.
37. Apoiar a implementação de programas de prevenção 53. Apoiar o fortalecimento da Defensoria Pública da União
da violência doméstica. e das Defensorias Públicas Estaduais, assim como a criação de
Defensorias Públicas junto a todas as comarcas do país.
Garantia do Direito à Justiça 54. Estimular a criação de serviços de orientação jurídica
gratuita, a exemplo dos balcões de direito e dos serviços de
38. Adotar, no âmbito da União e dos estados, medidas disque-denúncia, assim como o desenvolvimento de
legislativas e administrativas para a resolução de casos de programas de formação de agentes comunitários de justiça.
violação de direitos humanos, particularmente aqueles em 55. Estimular a criação e o fortalecimento de órgãos de
exame pelos órgãos internacionais de supervisão, garantindo defesa do consumidor, em nível estadual e municipal, assim
a apuração dos fatos, o julgamento dos responsáveis e a como apoiar as atividades das organizações da sociedade civil
reparação dos danos causados às vítimas. atuantes na defesa do consumidor.
39. Apoiar a Proposta de Emenda à Constituição nº 56. Apoiar a instalação e manutenção, pelos estados, de
29/2000, sobre a reforma do Poder Judiciário, com vistas a: a) juizados especiais civis e criminais.
assegurar a todos, no âmbito judicial e administrativo, a 57. Incentivar projetos voltados para a criação de serviços
razoável duração dos processos e os meios que garantam a de juizados itinerantes, com a participação de juízes,
celeridade de sua tramitação; b) conferir o status de emenda promotores e defensores públicos, especialmente nas regiões
constitucional aos tratados e convenções internacionais sobre mais distantes dos centros urbanos, para ampliar o acesso à
direitos humanos aprovados pelo Congresso Nacional; c) justiça.
garantir o incidente de deslocamento, da Justiça Estadual para 58. Estimular a criação de centros integrados de cidadania
a Justiça Federal, da competência processual nas hipóteses de próximos às comunidades carentes e periferias, que
graves crimes contra os direitos humanos, suscitadas pelo contenham os órgãos administrativos para atendimento ao
Procurador Geral da República perante o Superior Tribunal de cidadão, delegacias de polícias e varas de juizado especial com
Justiça; d) adotar a súmula vinculante, dispondo sobre a representantes do Ministério Público e da Defensoria Pública.
validade, a interpretação e a eficácia das normas legais e seu 59. Implementar a Campanha Nacional de Combate à
efeito vinculante em relação aos demais órgãos do Poder Tortura por meio da veiculação de filmes publicitários, da
Judiciário; e) estabelecer o controle externo do Poder sensibilização da opinião pública e da capacitação dos
Judiciário, com a criação do Conselho Nacional de Justiça, operadores do direito.
encarregado do controle da atuação administrativa e 60. Fortalecer a Comissão Especial de Combate à Tortura,
financeira do Poder Judiciário e do cumprimento dos deveres criada por meio da Resolução nº 2, de 5 de junho de 2001, no
funcionais dos juízes; f) criar o Conselho Nacional do âmbito do Conselho de Defesa dos Direitos da Pessoa Humana
Ministério Público e do Conselho Superior da Justiça do - CDDPH.
Trabalho. 61. Elaborar e implementar o Plano Nacional de Combate
40. Apoiar a criação de promotorias de direitos humanos à Tortura, levando em conta as diretrizes fixadas na Portaria
no âmbito do Ministério Público. nº 1.000 do Ministério da Justiça, de 30 de outubro de 2001, e
41. Propor legislação visando a fortalecer a atuação do as recomendações do Relator Especial das Nações Unidas para
Ministério Público no combate ao crime organizado. a Tortura, elaboradas com base em visita realizada ao Brasil
42. Fortalecer as corregedorias do Ministério Público e do em agosto/setembro de 2000.
Poder Judiciário, como forma de aumentar a fiscalização e o 62. Fomentar um pacto nacional com as entidades
monitoramento das atividades dos promotores e juízes. responsáveis pela aplicação da Lei nº 9.455, de 7 de abril de
43. Regulamentar o artigo 129, inciso VII, da Constituição 1997, que tipifica o crime de tortura, e manter sistema de
Federal, que trata do controle externo da atividade policial recepção, tratamento e encaminhamento de denúncias para
pelo Ministério Público. prevenção e apuração de casos - SOS Tortura.
44. Apoiar a atuação da Procuradoria Federal dos Direitos 63. Apoiar a criação e o funcionamento, nos estados, de
do Cidadão no âmbito da União e dos estados. programas de proteção de vítimas e testemunhas de crimes,
45. Propor medidas destinadas a incentivar a agilização expostas a grave e real ameaça em virtude de colaboração ou
dos procedimentos judiciais, a fim de reduzir o número de declarações prestadas em investigação ou processo penal,
detidos à espera de julgamento. assim como estruturar o serviço de proteção ao depoente
46. Fortalecer a Ouvidoria Geral da República, a fim de especial, instituído pela Lei nº 9.807/99 e regulamentado pelo
ampliar a participação da população no monitoramento e Decreto 3.518/00.
fiscalização das atividades dos órgãos e agentes do poder 64. Estudar a possibilidade de revisão da legislação sobre
público. abuso e desacato à autoridade.

Conhecimentos Específicos 44
Apostila Digital Licenciada para Alice Caroline Guarino dos Santos - alice.guarino@hotmail.com (Proibida a Revenda)
APOSTILAS OPÇÃO

65. Apoiar a aplicação da Lei Complementar nº 88/96, carcerárias e o cumprimento de penas privativas de liberdade
relativa ao rito sumário, assim como outras proposições e penas alternativas, bem como promover a participação de
legislativas que objetivem dinamizar os processos de organizações da sociedade civil em programas de assistência
expropriação para fins de reforma agrária, assegurando-se, aos presos e na fiscalização das condições e do tratamento a
para prevenir atos de violência, maior cautela na concessão de que são submetidos nos estabelecimentos prisionais.
liminares. 81. Desenvolver programas de atenção integral à saúde da
66. Assegurar o cumprimento da Lei nº 9.416, que torna população carcerária.
obrigatória a presença do juiz ou de representante do 82. Estimular a aplicação dos dispositivos da Lei de
Ministério Público no local, por ocasião do cumprimento de Execuções Penais referentes a regimes semiabertos de prisão.
mandado de manutenção ou reintegração de posse de terras, 83. Fortalecer o programa nacional de capacitação do
quando houver pluralidade de réus, para prevenir conflitos servidor prisional, com vistas a assegurar a formação
violentos no campo, ouvido também o órgão administrativo da profissional do corpo técnico, da direção e dos agentes
reforma agrária. penitenciários.
67. Promover a discussão, em âmbito nacional, sobre a 84. Propor a normatização dos procedimentos de revista
necessidade de se repensar as formas de punição ao cidadão aos visitantes de estabelecimentos prisionais, com o objetivo
infrator, incentivando o Poder Judiciário a utilizar as penas de evitar constrangimentos desnecessários aos familiares dos
alternativas previstas nas leis vigentes com a finalidade de presos.
minimizar a crise do sistema penitenciário. 85. Promover programas de educação, treinamento
68. Estimular a aplicação de penas alternativas à prisão profissional e trabalho para facilitar a recuperação do preso,
para os crimes não violentos. visando a sua futura reinserção na sociedade.
69. Apoiar o funcionamento da Central Nacional - CENAPA 86. Realizar levantamento epidemiológico da população
e das centrais estaduais de penas alternativas, estimulando a carcerária brasileira.
disseminação de informações e a reprodução dessas 87. Apoiar a realização de Mutirões da Execução Penal com
iniciativas, assim como a criação do Conselho Nacional de vistas à concessão de progressão de regime e soltura dos
Penas e Medidas Alternativas. presos que já cumpriram integralmente suas penas.
88. Apoiar programas que tenham como objetivo a
70. Adotar medidas para assegurar a obrigatoriedade de reintegração social do egresso do sistema penitenciário e a
apresentação da pessoa presa ao juiz no momento da redução das taxas de reincidência penitenciária.
homologação da prisão em flagrante e do pedido de prisão 89. Proporcionar incentivos fiscais, creditícios e outros às
preventiva, como forma de garantir a sua integridade física. empresas que empreguem egressos do sistema penitenciário.
71. Ampliar a representação da sociedade civil no Conselho 90. Apoiar a desativação de estabelecimentos
Nacional de Política Criminal e Penitenciária - CNPCP. penitenciários que contrariem as normas mínimas
72. Realizar periodicamente o Censo Penitenciário, de penitenciárias internacionais, a exemplo da Casa de Detenção
modo a possibilitar um planejamento adequado da oferta de de São Paulo - Carandiru.
vagas, das ações gerenciais e de outras medidas destinadas a
assegurar a melhoria do sistema. Garantia do Direito à Liberdade
73. Dar continuidade ao processo de articulação do
INFOSEG com o Sistema de Informática Penitenciária - Opinião e Expressão
INFOPEN, de forma a acompanhar a passagem do detento por
todas as etapas do sistema de justiça penal, desde a detenção 91. Promover debate com todos os setores vinculados ao
provisória até o relaxamento da prisão, seja pelo cumprimento tema da liberdade de expressão e da classificação indicativa de
da pena, seja pela progressão de regime. espetáculos e diversões públicas, buscando uma ação
74. Apoiar programas que tenham como objetivo a integrada e voltada para o interesse público nesse assunto.
transferência de pessoas submetidas à detenção provisória de 92. Estabelecer diálogo com os produtores e distribuidores
carceragens de delegacias de Polícia para centros de detenção de programação visando à cooperação e sensibilização desses
provisória, núcleos de custódia e/ou cadeias públicas, ou, no segmentos para o cumprimento da legislação em vigor e
caso de proferida sentença condenatória, diretamente para construção de uma cultura de direitos humanos.
estabelecimentos prisionais. 93. Apoiar o funcionamento da Coordenação Geral de
75. Implementar políticas visando a garantir os direitos Justiça, Classificação, Títulos e Qualificação, da Secretaria
econômicos, sociais e culturais das pessoas submetidas à Nacional de Justiça do Ministério da Justiça, de modo a dotá-la
detenção. de capacidade operativa compatível com sua missão
76. Apoiar programas de emergência para corrigir as institucional.
condições inadequadas dos estabelecimentos prisionais 94. Criar um sistema de avaliação permanente sobre os
existentes, assim como para a construção de novos critérios de classificação indicativa e faixa etária.
estabelecimentos, federais e estaduais, com a utilização de 95. Promover o mapeamento dos programas radiofônicos
recursos do Fundo Penitenciário Nacional - FUNPEN. e televisivos que estimulem a apologia do crime, a violência, a
77. Estabelecer níveis hierárquicos de segurança para tortura, o racismo e outras formas de discriminação, a ação de
estabelecimentos prisionais de modo a abrigar criminosos grupos de extermínio e a pena de morte, com vistas a
reincidentes, perigosos e organizados em estabelecimentos identificar responsáveis e a adotar as medidas legais
mais seguros. pertinentes.
78. Incrementar a descentralização dos estabelecimentos 96. Apoiar a instalação, no âmbito do Poder Legislativo, do
penais, promovendo a sua interiorização, com a construção de Conselho de Comunicação Social, com o objetivo de garantir o
presídios de pequeno porte que facilitem a execução da pena controle democrático das concessões de rádio e televisão,
nas proximidades do domicílio dos familiares dos presos. regulamentar e monitorar o uso dos meios de comunicação
79. Integrar Juizado, Ministério Público, Defensoria Pública social e coibir incentivos a práticas de violação dos direitos
e Assistência Social na região de inserção dos humanos.
estabelecimentos prisionais. 97. Garantir a possibilidade de fiscalização da
80. Incentivar a implantação e o funcionamento, em todas programação das emissoras de rádio e televisão, com vistas a
as regiões, dos conselhos comunitários previstos na Lei de assegurar o controle social sobre os meios de comunicação e a
Execuções Penais, para monitorar e fiscalizar as condições penalizar, na forma da lei, as empresas de telecomunicação

Conhecimentos Específicos 45
Apostila Digital Licenciada para Alice Caroline Guarino dos Santos - alice.guarino@hotmail.com (Proibida a Revenda)
APOSTILAS OPÇÃO

que veicularem programação ou publicidade atentatória aos governamentais, especialmente a dados sobre a tramitação de
direitos humanos. investigações e processos legais relativos a casos de violação
98. Coibir a propaganda de ideias neonazistas e outras de direitos humanos.
ideologias que pregam a violência, particularmente contra 117. Apoiar a adoção, pelo poder público e pela iniciativa
grupos minoritários. privada, de políticas de ação afirmativa como forma de
99. Propor legislação visando a coibir o uso da Internet combater a desigualdade.
para incentivar práticas de violação dos direitos humanos. 118. Promover estudos para alteração da Lei de Licitações
100. Garantir a imparcialidade, o contraditório e o direito Públicas de modo a possibilitar que, uma vez esgotados todos
de resposta na veiculação de informações, de modo a os procedimentos licitatórios, configurando-se empate, o
assegurar a todos os cidadãos o direito de informar e ser critério de desempate - hoje definido por sorteio - seja
informado. substituído pelo critério de adoção, por parte d?os licitantes,
101. Apoiar formas de democratização da informação, a de políticas de ação afirmativa em favor de grupos
exemplo das rádios e televisões comunitárias. discriminados.
102. Coibir a utilização de recursos públicos, inclusive de 119. Apoiar a inclusão nos currículos escolares de
bancos oficiais, fundações, empresas públicas e de economia informações sobre o problema da discriminação na sociedade
mista, para patrocinar eventos e programas que estimulem a brasileira e sobre o direito de todos os grupos e indivíduos a
prática de violência. um tratamento igualitário perante a lei.
103. Contemplar, nos meios de comunicação, elementos
destinados a elevar a autoestima dos afrodescendentes, povos Crianças e Adolescentes
indígenas e outros grupos historicamente vitimizados pelo
racismo e outras formas de discriminação. 120. Fortalecer o papel do Conselho Nacional dos Direitos
da Criança e do Adolescente - CONANDA na formulação e no
Crença e Culto acompanhamento de políticas públicas para a infância e
adolescência.
104. Garantir o direito à liberdade de crença e culto a todos 121. Incentivar a criação e o funcionamento, nos estados e
os cidadãos brasileiros, independentemente de sua orientação municípios, dos Conselhos dos Direitos da Criança e do
religiosa. Adolescente, Conselhos Tutelares e Fundos dos Direitos da
105. Prevenir e combater a intolerância religiosa, inclusive Criança e do Adolescente.
no que diz respeito a religiões minoritárias e a cultos afro- 122. Promover campanhas de esclarecimento sobre os
brasileiros. Fundos dos Direitos da Criança e do Adolescente, informando
106. Implementar os dispositivos da Declaração Sobre a sobre as vantagens de aplicação para pessoas físicas e
Eliminação de Todas as Formas de Intolerância e jurídicas, assim como criar mecanismos de incentivo à
Discriminação Fundadas em Religião ou Crença, adotada pela captação de recursos, garantindo formas de controle social de
Assembléia Geral das Nações Unidas em 25 de novembro de sua aplicação.
1981. 123. Apoiar a produção e publicação de estudos e
107. Proibir a veiculação de propaganda e mensagens pesquisas que contribuam para a divulgação e aplicação do
racistas e/ou xenofóbicas que difamem as religiões e incitem Estatuto da Criança e do Adolescente - ECA.
ao ódio contra valores espirituais e/ou culturais. 124. Assegurar a implantação e o funcionamento adequado
108. Incentivar o diálogo entre movimentos religiosos sob dos órgãos que compõem o Sistema de Garantia de Direitos de
o prisma da construção de uma sociedade pluralista, com base Crianças e Adolescentes, estimulando a criação de Núcleos de
no reconhecimento e no respeito às diferenças de crença e Defensorias Públicas Especializadas no Atendimento a
culto. Crianças e Adolescentes (com os direitos violados), de
Delegacias de Investigação de Crimes Praticados Contra
Orientação Sexual Crianças e Adolescentes e de Varas Privativas de Crimes
Contra Crianças e Adolescentes.
109. Propor emenda à Constituição Federal para incluir a 125. Promover a discussão do papel do Poder Judiciário, do
garantia do direito à livre orientação sexual e a proibição da Ministério Público, da Defensoria Pública e do Poder
discriminação por orientação sexual. Legislativo, ao lado do Poder Executivo, bem como da
110. Apoiar a regulamentação da parceria civil registrada integração de suas ações, na implementação do ECA.
entre pessoas do mesmo sexo e a regulamentação da lei de 126. Investir na formação e capacitação de profissionais
redesignação de sexo e mudança de registro civil para encarregados da promoção e proteção dos direitos de crianças
transexuais. e adolescentes no âmbito de instituições públicas e de
111. Propor o aperfeiçoamento da legislação penal no que organizações não-governamentais.
se refere à discriminação e à violência motivadas por 127. Capacitar os professores do ensino fundamental e
orientação sexual. médio para promover a discussão de temas específicos, tais
112. Excluir o termo 'pederastia' do Código Penal Militar. como: a educação sexual, o consumo de drogas, a ética e a
cidadania.
113. Incluir nos censos demográficos e pesquisas oficiais 128. Apoiar campanhas voltadas para a paternidade
dados relativos à orientação sexual. responsável.
129. Promover, em parceria com governos estaduais e
Garantia do Direito à Igualdade municipais e com entidades da sociedade civil, campanhas
educativas relacionadas às situações de violação de direitos
114. Apoiar o funcionamento e a implementação das vivenciadas pela criança e o adolescente, tais como: a violência
resoluções do Conselho Nacional de Combate à Discriminação doméstica, a exploração sexual, a exploração no trabalho e o
- CNCD, no âmbito do Ministério da Justiça. uso de drogas, visando à criação de padrões culturais
115. Estimular a divulgação e a aplicação da legislação favoráveis aos direitos da criança e do adolescente.
antidiscriminatória, assim como a revogação de normas 130. Viabilizar programas e serviços de atendimento e de
discriminatórias na legislação infraconstitucional. proteção para crianças e adolescentes vítimas de violência,
116. Estimular a criação de canais de acesso direto e assim como de assistência e orientação para seus familiares.
regular da população a informações e documentos

Conhecimentos Específicos 46
Apostila Digital Licenciada para Alice Caroline Guarino dos Santos - alice.guarino@hotmail.com (Proibida a Revenda)
APOSTILAS OPÇÃO

131. Propor alterações na legislação penal com o objetivo 149. Priorizar as medidas socioeducativas em meio aberto
de limitar a incidência da violência doméstica contra crianças para o atendimento dos adolescentes em conflito com a lei.
e adolescentes. 150. Incentivar o reordenamento das instituições
132. Incentivar programas de orientação familiar com privativas de liberdade para adolescentes em conflito com a
vistas a capacitar as famílias para a resolução de conflitos de lei, reduzindo o número de internos por unidade de
forma não violenta, bem como para o cumprimento de suas atendimento e conferindo prioridade à implementação das
responsabilidades para com as crianças e adolescentes. demais medidas socioeducativas previstas no ECA, em
133. Garantir a expansão de programas de prevenção da consonância com as resoluções do CONANDA.
violência voltados para as necessidades específicas de crianças 151. Incentivar o desenvolvimento, monitoramento e
e adolescentes. avaliação de programas socioeducativos para o atendimento
134. Fortalecer os programas que ofereçam benefícios a de adolescentes autores de ato infracional, com a participação
adolescentes em situação de vulnerabilidade, e que de seus familiares.
possibilitem o seu envolvimento em atividades comunitárias 152. Fortalecer a atuação do Poder Judiciário e do
voltadas para a promoção da cidadania, saúde e meio Ministério Público na fiscalização e aplicação das medidas
ambiente. socioeducativas a adolescentes em conflito com a lei.
135. Apoiar a implantação e implementação do Plano 153. Promover a integração operacional de órgãos do
Nacional de Enfrentamento da Violência Sexual Infanto- Poder Judiciário, Ministério Público, Defensorias Públicas e
Juvenil nos estados e municípios. Secretarias de Segurança Pública com as delegacias
136. Dar continuidade à Campanha Nacional de Combate à especializadas em investigação de atos infracionais praticados
Exploração Sexual Infanto-Juvenil, estimulando o lançamento por adolescentes e às entidades de atendimento, bem como
de campanhas estaduais e municipais que visem a modificar ações de sensibilização dos profissionais indicados para esses
concepções, práticas e atitudes que estigmatizam a criança e o órgãos quanto à aplicação do ECA.
adolescente em situação de violência sexual, utilizando como 154. Assegurar atendimento sistemático e proteção
marco conceitual o ECA e as normas internacionais integral à criança e ao adolescente testemunha, sobretudo
pertinentes. quando se tratar de denúncia envolvendo o narcotráfico e
137. Propor a alteração da legislação no tocante à grupos de extermínio.
tipificação de crime de exploração sexual infanto-juvenil, com
penalização para o explorador e o usuário. 155. Estender a assistência jurídica às crianças que se
138. Criar informativo, destinado a turistas estrangeiros, encontram em abrigos públicos ou privados, com vistas ao
cobrindo aspectos relacionados aos crimes sexuais e suas restabelecimento de seus vínculos familiares, quando possível,
implicações pessoais, sociais e judiciais. ou a sua colocação em família substituta, como medida
139. Promover a discussão do papel dos meios de subsidiária.
comunicação em situações de violação de direitos de crianças 156. Instituir uma política nacional de estímulo à adoção
e adolescentes. de crianças e adolescentes privados da convivência familiar.
140. Ampliar o Programa de Erradicação do Trabalho 157. Promover a implementação da Convenção da Haia
Infantil - PETI de modo a focalizar as crianças de áreas urbanas sobre a Proteção das Crianças e a Cooperação em Matéria de
em situação de risco, especialmente aquelas utilizadas em Adoção Internacional, por meio do fortalecimento da
atividades ilegais como a exploração sexual infanto-juvenil e o Autoridade Central Brasileira, instituída pelo Decreto n.º
tráfico de drogas. 3.174/99 e dos órgãos que a integram.
141. Apoiar iniciativas de geração de renda para as famílias 158. Apoiar proposta legislativa destinada a regulamentar
de crianças atendidas pelo PETI. o funcionamento da Autoridade Central Brasileira e do
142. Promover e divulgar experiências de ações Conselho das Autoridades Centrais, órgãos responsáveis pela
socioeducativas junto às famílias de crianças atendidas pelo cooperação em matéria de adoção internacional.
PETI. 159. Promover ações e iniciativas com vistas a reforçar o
143. Apoiar e fortalecer o Fórum Nacional de Prevenção e caráter excepcional das adoções internacionais.
Erradicação do Trabalho Infantil. 160. Promover a implementação da Convenção da Haia
144. Implantar e implementar as diretrizes da Política sobre os Aspectos Civis do Sequestro Internacional de
Nacional de Combate ao Trabalho Infantil e de Proteção do Crianças, no que se refere à estruturação da Autoridade
Adolescente Trabalhador. Central designada pelo Decreto nº 3951/01.
145. Ampliar programas de aprendizagem profissional 161. Apoiar medidas destinadas a assegurar a localização
para adolescentes em organizações públicas e privadas, de crianças e adolescentes deslocados e retidos ilicitamente,
respeitando as regras estabelecidas pelo ECA. garantindo o regresso a seu local de origem.
146. Dar continuidade à implantação e implementação, no
âmbito federal e de forma articulada com estados e Mulheres
municípios, do Sistema de Informação para a Infância e a
Adolescência - SIPIA, no que se refere aos Módulos: I - 162. Apoiar as atividades do Conselho Nacional dos
monitoramento da situação de proteção da criança e do Direitos da Mulher - CNDM, assim como dos conselhos
adolescente, sob a ótica da violação e ressarcimento de estaduais e municipais dos direitos da mulher.
direitos; II - monitoramento do fluxo de atendimento ao 163. Estimular a formulação, no âmbito federal, estadual e
adolescente em conflito com a lei; III - monitoramento da municipal, de programas governamentais destinados a
colocação familiar e das adoções nacionais e internacionais; e assegurar a igualdade de direitos em todos os níveis, incluindo
IV - acompanhamento da implantação dos Conselhos de saúde, educação e treinamento profissional, trabalho,
Direitos, Conselhos Tutelares e Fundos para a Infância e a segurança social, propriedade e crédito rural, cultura, política
Adolescência. e justiça.
147. Apoiar a criação de serviços de identificação, 164. Incentivar a capacitação dos professores do ensino
localização, resgate e proteção de crianças e adolescentes fundamental e médio para a aplicação dos Parâmetros
desaparecidos. Curriculares Nacionais - PCNs no que se refere às questões de
148. Promover iniciativas e campanhas de esclarecimento promoção da igualdade de gênero e de combate à
que tenham como objetivo assegurar a inimputabilidade penal discriminação contra a mulher.
até os 18 anos de idade.

Conhecimentos Específicos 47
Apostila Digital Licenciada para Alice Caroline Guarino dos Santos - alice.guarino@hotmail.com (Proibida a Revenda)
APOSTILAS OPÇÃO

165. Incentivar a criação de cursos voltados para a 183. Adotar, no âmbito da União, e estimular a adoção,
capacitação política de lideranças locais de mulheres, com pelos estados e municípios, de medidas de caráter
vistas ao preenchimento da quota estabelecida para a compensatório que visem à eliminação da discriminação racial
candidatura de mulheres a cargos eletivos. e à promoção da igualdade de oportunidades, tais como:
166. Apoiar a regulamentação do Artigo 7º, inciso XX da ampliação do acesso dos afrodescendentes às universidades
Constituição Federal, que prevê a proteção do mercado de públicas, aos cursos profissionalizantes, às áreas de tecnologia
trabalho da mulher. de ponta, aos cargos e empregos públicos, inclusive cargos em
167. Incentivar a geração de estatísticas sobre salários, comissão, de forma proporcional a sua representação no
jornadas de trabalho, ambientes de trabalho, doenças conjunto da sociedade brasileira.
profissionais e direitos trabalhistas da mulher. 184. Criar bancos de dados sobre a situação dos direitos
168. Assegurar o cumprimento dos dispositivos existentes civis, políticos, sociais, econômicos e culturais dos
na Lei nº 9.029/95, que garante proteção às mulheres contra a afrodescendentes na sociedade brasileira, com a finalidade de
discriminação em razão de gravidez. orientar a adoção de políticas públicas afirmativas.
169. Apoiar a implementação e o fortalecimento do 185. Estudar a viabilidade da criação de fundos de
Programa de Assistência Integral à Saúde da Mulher - PAISM. reparação social destinados a financiar políticas de ação
170. Apoiar programas voltados para a sensibilização em afirmativa e de promoção da igualdade de oportunidades.
questões de gênero e violência doméstica e sexual praticada 186. Apoiar as ações da iniciativa privada no campo da
contra mulheres na formação dos futuros profissionais da área discriminação positiva e da promoção da diversidade no
de saúde, dos operadores do direito e dos policiais civis e ambiente de trabalho.
militares, com ênfase na proteção dos direitos de mulheres 187. Implementar a Convenção Internacional Sobre a
afrodescendentes. Eliminação de Todas as Formas de Discriminação Racial, a
171. Apoiar a alteração dos dispositivos do Código Penal Convenção nº 111 da Organização Internacional do Trabalho -
referentes ao estupro, atentado violento ao pudor, posse OIT, relativa à discriminação em matéria de emprego e
sexual mediante fraude, atentado ao pudor mediante fraude e ocupação, e a Convenção Contra a Discriminação no Ensino.
o alargamento dos permissivos para a prática do aborto legal, 188. Estimular a criação e o funcionamento de programas
em conformidade com os compromissos assumidos pelo de assistência e orientação jurídica para ampliar o acesso dos
Estado brasileiro no marco da Plataforma de Ação de Pequim. afrodescendentes à justiça.
172. Adotar medidas com vistas a impedir a utilização da 189. Apoiar a regulamentação do artigo 68 do Ato das
tese da "legítima defesa da honra" como fator atenuante em Disposições Constitucionais Transitórias - ADCT, que dispõe
casos de homicídio de mulheres, conforme entendimento já sobre o reconhecimento da propriedade definitiva das terras
firmado pelo Supremo Tribunal Federal. ocupadas pelos remanescentes das comunidades dos
173. Fortalecer o Programa Nacional de Combate à quilombos.
Violência Contra a Mulher. 190. Promover o cadastramento e a identificação das
174. Apoiar a criação e o funcionamento de delegacias comunidades remanescentes de quilombos, em todo o
especializadas no atendimento à mulher - DEAMs. território nacional, com vistas a possibilitar a emissão dos
175. Incentivar a pesquisa e divulgação de informações títulos de propriedade definitiva de suas terras.
sobre a violência e discriminação contra a mulher e sobre 191. Apoiar medidas destinadas à remoção de grileiros e
formas de proteção e promoção dos direitos da mulher. intrusos das terras já tituladas das comunidades de quilombos.
192. Apoiar projetos de infraestrutura para as
176. Apoiar a implantação, nos estados e municípios, de comunidades remanescentes de quilombos, como forma de
serviços de disque-denúncia para casos de violência contra a evitar o êxodo rural e promover o desenvolvimento social e
mulher. econômico dessas comunidades.
177. Apoiar programas voltados para a defesa dos direitos 193. Criar unidade administrativa específica para tratar da
de profissionais do sexo. titulação de terras de quilombos e prestar apoio a associações
178. Apoiar programas de proteção e assistência a vítimas de pequenos(as) agricultores(as) afrodescendentes em
e testemunhas da violência de gênero, contemplando serviços projetos de desenvolvimento das comunidades quilombolas,
de atendimento jurídico, social, psicológico, médico e de no Instituto Nacional de Colonização e Reforma Agrária -
capacitação profissional, assim como a ampliação e o INCRA.
fortalecimento da rede de casas-abrigo em todo o país. 194. Incentivar ações que contribuam para a preservação
179. Estimular a articulação entre os diferentes serviços de da memória e fomento à produção cultural da comunidade
apoio a mulheres vítimas de violência doméstica e sexual no afrodescendente no Brasil.
âmbito federal, estadual e municipal, enfatizando a ampliação 195. Promover o mapeamento e tombamento dos sítios e
dos equipamentos sociais de atendimento à mulher vitimizada documentos detentores de reminiscências históricas, bem
pela violência. como a proteção das manifestações culturais afro-brasileiras.
180. Apoiar as políticas dos governos estaduais e 196. Estimular a presença proporcional dos grupos raciais
municipais para a prevenção da violência doméstica e sexual que compõem a população brasileira em propagandas
contra as mulheres. institucionais contratadas pelos órgãos da administração
direta e indireta e por empresas estatais.
Afrodescendentes
197. Incentivar o diálogo com entidades de classe e agentes
181. Apoiar o reconhecimento, por parte do Estado de publicidade visando ao convencimento desses setores
brasileiro, de que a escravidão e o tráfico transatlântico de quanto à necessidade de que as peças publicitárias reflitam
escravos constituíram violações graves e sistemáticas dos adequadamente a composição racial da sociedade brasileira e
direitos humanos que hoje seriam consideradas crimes contra evitem o uso de estereótipos depreciativos.
a humanidade. 198. Examinar a viabilidade de alterar o artigo 61 do
182. Apoiar o reconhecimento, por parte do Estado Código Penal brasileiro, de modo a incluir entre as
brasileiro, da marginalização econômica, social e política a que circunstâncias agravantes na aplicação das penas o racismo, a
foram submetidos os afrodescendentes em decorrência da discriminação racial, a xenofobia e formas correlatas de
escravidão. intolerância.

Conhecimentos Específicos 48
Apostila Digital Licenciada para Alice Caroline Guarino dos Santos - alice.guarino@hotmail.com (Proibida a Revenda)
APOSTILAS OPÇÃO

199. Propor medidas destinadas a fortalecer o papel do 218. Garantir aos povos indígenas assistência na área da
Ministério Público na promoção e proteção dos direitos e saúde, com a implementação de programas de saúde
interesses das vítimas de racismo, discriminação racial e diferenciados, considerando as especificidades dessa
formas correlatas de intolerância. população e priorizando ações na área de medicina preventiva
200. Apoiar a inclusão do quesito 'raça/cor' nos sistemas e segurança alimentar.
de informação e registro sobre população e em bancos de
dados públicos. 219. Assegurar aos povos indígenas uma educação escolar
201. Apoiar as atividades do Grupo de Trabalho para a diferenciada, respeitando o seu universo sociocultural, e
Eliminação da Discriminação no Emprego e na Ocupação - viabilizar apoio aos estudantes indígenas do ensino
GTEDEO, instituído no âmbito do Ministério do Trabalho e fundamental, de segundo grau e de nível universitário.
Emprego - MTE. 220. Promover a criação de linhas de crédito e a concessão
202. Incentivar a participação de representantes de bolsas de estudo específicas para estudantes indígenas
afrodescendentes nos conselhos federais, estaduais e universitários.
municipais de defesa de direitos e apoiar a criação de 221. Implementar políticas de comunicação e divulgação
conselhos estaduais e municipais de defesa dos direitos dos de informações sobre os povos indígenas, especialmente nas
afrodescendentes. escolas públicas e privadas do ensino médio e fundamental,
203. Estimular as secretarias de segurança pública dos com vistas à promoção da igualdade e ao combate à
estados a realizarem cursos de capacitação e seminários sobre discriminação.
racismo e discriminação racial. 222. Implementar políticas de proteção e gestão das terras
204. Propor projeto de lei regulamentando os artigos 215, indígenas, com a implantação de sistemas de vigilância
216 e 242 da Constituição Federal, que dizem respeito ao permanente dessas terras e de seu entorno, a promoção de
exercício dos direitos culturais e à constituição do patrimônio parcerias com a Polícia Federal, o IBAMA e as Secretarias
cultural brasileiro. Estaduais de Meio Ambiente, e a capacitação de servidores e
205. Propor ao Instituto Brasileiro de Geografia e membros das comunidades indígenas.
Estatística - IBGE a adoção de critério estatístico abrangente a 223. Viabilizar programas e ações na área de etno-
fim de considerar pretos e pardos como integrantes do desenvolvimento voltados para a ocupação sustentável de
contingente da população afrodescendente. espaços estratégicos no interior das terras indígenas, tais
206. Apoiar a criminalização da prática do racismo nos como áreas desocupadas por invasores e/ou áreas de ingresso
Códigos Penal e de Processo Penal. de madeireiros e garimpeiros.
207. Apoiar o processo de revisão dos livros didáticos de 224. Implantar banco de dados que permita colher e
modo a resgatar a história e a contribuição dos sistematizar informações sobre conflitos fundiários e violência
afrodescendentes para a construção da identidade nacional. em terras indígenas, a ser integrado aos mapas de conflitos
208. Promover um ensino fundado na tolerância, na paz e fundiários e de violência.
no respeito à diferença, que contemple a diversidade cultural 225. Apoiar a edição de publicações com dados relativos à
do país, incluindo o ensino sobre cultura e história dos discriminação e à violência contra os povos indígenas.
afrodescendentes. 226. Apoiar e assessorar as comunidades indígenas na
209. Apoiar o fortalecimento da Fundação Cultural elaboração de projetos e na execução de ações de etno-
Palmares - FCP, assegurando os meios para o desempenho de desenvolvimento de caráter sustentável.
suas atividades. 227. Apoiar a criação e o desenvolvimento dos
mecanismos de gestão dos programas multissetoriais
Povos Indígenas gerenciados pela FUNAI, no âmbito dos Planos Plurianuais e
dos orçamentos federais.
210. Formular e implementar políticas de proteção e 228. Apoiar a criação de serviços específicos de assistência
promoção dos direitos dos povos indígenas, em substituição a jurídica para indivíduos e comunidades indígenas.
políticas integracionistas e assistencialistas. 229. Garantir o direito constitucional dos povos indígenas
211. Apoiar o processo de reestruturação da Fundação ao uso exclusivo da biodiversidade existente em suas terras,
Nacional do Índio - FUNAI, de forma que a instituição possa implementando ações que venham a coibir a biopirataria dos
garantir os direitos constitucionais dos povos indígenas. recursos e conhecimentos tradicionais dos indígenas.
212. Dotar a FUNAI de recursos humanos e financeiros 230. Desenvolver políticas de proteção do patrimônio
suficientes para o cumprimento de sua missão institucional de cultural e biológico e dos conhecimentos tradicionais dos
defesa dos direitos dos povos indígenas. povos indígenas, em especial as ações que tenham como
213. Apoiar a revisão do Estatuto do Índio (Lei 6.001/73), objetivo a catalogação, o registro de patentes e a divulgação
com vistas à rápida aprovação do projeto de lei do Estatuto das desse patrimônio.
Sociedades Indígenas, bem como a promover a ratificação da
Convenção nº 169 da OIT, sobre Povos Indígenas e Tribais em Gays, Lésbicas, Travestis, Transexuais e Bissexuais -
Países Independentes. GLTTB
214. Assegurar a efetiva participação dos povos indígenas,
de suas organizações e do órgão indigenista federal no 231. Promover a coleta e a divulgação de informações
processo de formulação e implementação de políticas públicas estatísticas sobre a situação sócio demográfica dos GLTTB,
de proteção e promoção dos direitos indígenas. assim como pesquisas que tenham como objeto as situações de
215. Assegurar o direito dos povos indígenas às terras que violência e discriminação praticadas em razão de orientação
tradicionalmente ocupam, às reservadas e às de domínio. sexual.
216. Demarcar e regularizar as terras indígenas 232. Implementar programas de prevenção e combate à
tradicionalmente ocupadas, as reservadas e as de domínio que violência contra os GLTTB, incluindo campanhas de
ainda não foram demarcadas e regularizadas. esclarecimento e divulgação de informações relativas à
217. Divulgar medidas sobre a regularização de terras legislação que garante seus direitos.
indígenas, especialmente para os municípios brasileiros 233. Apoiar programas de capacitação de profissionais de
localizados nessas regiões, de modo a aumentar o grau de educação, policiais, juízes e operadores do direto em geral
confiança e estabilidade nas relações entre os povos indígenas para promover a compreensão e a consciência ética sobre as
e a sociedade envolvente.

Conhecimentos Específicos 49
Apostila Digital Licenciada para Alice Caroline Guarino dos Santos - alice.guarino@hotmail.com (Proibida a Revenda)
APOSTILAS OPÇÃO

diferenças individuais e a eliminação dos estereótipos 254. Estimular e apoiar as municipalidades nas quais se
depreciativos com relação aos GLTTB. identifica a presença de comunidades ciganas com vistas ao
234. Inserir, nos programas de formação de agentes de estabelecimento de áreas de acampamento dotadas de
segurança pública e operadores do direito, o tema da livre infraestrutura e condições necessárias.
orientação sexual. 255. Sensibilizar as comunidades ciganas para a
235. Apoiar a criação de instâncias especializadas de necessidade de realizar o registro de nascimento dos filhos,
atendimento a casos de discriminação e violência contra assim como apoiar medidas destinadas a garantir o direito ao
GLTTB no Poder Judiciário, no Ministério Público e no sistema registro de nascimento gratuito para as crianças ciganas.
de segurança pública.
236. Estimular a formulação, implementação e avaliação Pessoas Portadoras de Deficiência
de políticas públicas para a promoção social e econômica da
comunidade GLTTB. 256. Apoiar as atividades do Conselho Nacional dos
237. Incentivar ações que contribuam para a preservação Direitos da Pessoa Portadora de Deficiência - CONADE, bem
da memória e fomento à produção cultural da comunidade como dos conselhos estaduais e municipais.
GLTTB no Brasil. 257. Instituir medidas que propiciem a remoção de
238. Incentivar programas de orientação familiar e escolar barreiras arquitetônicas, ambientais, de transporte e de
para a resolução de conflitos relacionados à livre orientação comunicação para garantir o acesso da pessoa portadora de
sexual, com o objetivo de prevenir atitudes hostis e violentas. deficiência aos serviços e áreas públicas e aos edifícios
239. Estimular a inclusão, em programas de direitos comerciais.
humanos estaduais e municipais, da defesa da livre orientação 258. Formular plano nacional de ações integradas na área
sexual e da cidadania dos GLTTB. da deficiência, objetivando a definição de estratégias de
240. Promover campanha junto aos profissionais da saúde integração das ações governamentais e não-governamentais,
e do direito para o esclarecimento de conceitos científicos e com vistas ao cumprimento do Decreto nº 3298/99.
éticos relacionados à comunidade GLTTB. 259. Adotar medidas que possibilitem o acesso das pessoas
portadoras de deficiência às informações veiculadas em todos
241. Promover a sensibilização dos profissionais de os meios de comunicação.
comunicação para a questão dos direitos dos GLTTB. 260. Estender a estados e municípios o Sistema Nacional
de Informações sobre Deficiência - SICORDE.
Estrangeiros, Refugiados e Migrantes 261. Apoiar programas de tratamentos alternativos à
internação de pessoas portadoras de deficiência mental e
242. Apoiar, no âmbito do Ministério da Justiça, o portadores de condutas típicas - autismo.
funcionamento do Comitê Nacional para Refugiados - 262. Apoiar programas de educação profissional para
CONARE. pessoas portadoras de deficiência.
243. Implementar a Convenção da ONU relativa ao 263. Apoiar o treinamento de policiais para lidar com
Estatuto dos Refugiados, de 1951, e o Protocolo Adicional de portadores de deficiência mental, auditiva e condutas típicas -
1966. autismo.
244. Promover a capacitação das autoridades nacionais
diretamente envolvidas na execução da política nacional para 264. Adotar medidas legais e práticas para garantir o
refugiados. direito dos portadores de deficiência ao reingresso no
245. Desenvolver programa e campanha visando à mercado de trabalho, mediante adequada reabilitação
regularização da situação dos estrangeiros atualmente no país, profissional.
atendendo a critérios de reciprocidade de tratamento. 265. Ampliar a participação de representantes dos
246. Adotar medidas para impedir e punir a violência e portadores de deficiência na discussão de planos diretores das
discriminação contra estrangeiros no Brasil e brasileiros no cidades.
exterior. 266. Desenvolver ações que assegurem a inclusão do
247. Promover estudos e pesquisas relativos à proteção, quesito acessibilidade, de acordo com as especificações da
promoção e difusão dos direitos dos refugiados, incluindo as Associação Brasileira de Normas Técnicas - ABNT, nos
soluções duráveis (reassentamento, integração local e projetos de moradia financiados por programas habitacionais.
repatriação), com especial atenção para a situação das 267. Adotar políticas e programas para garantir o acesso e
mulheres e crianças refugiadas. a locomoção das pessoas portadoras de deficiência, segundo as
248. Estabelecer políticas de promoção e proteção dos normas da ABNT.
direitos das comunidades brasileiras no exterior e das 268. Garantir a qualidade dos produtos para portadores de
comunidades estrangeiras no Brasil. deficiência adquiridos e distribuídos pelo Poder Público -
249. Propor a elaboração de uma nova lei de imigração e órteses e próteses.
naturalização, regulando a situação jurídica dos estrangeiros 269. Apoiar a inclusão de referências à acessibilidade para
no Brasil. pessoas portadoras de deficiência nas campanhas promovidas
pelo Governo Federal e pelos governos estaduais e municipais.
Ciganos 270. Promover a capacitação de agentes públicos,
profissionais de saúde, lideranças comunitárias e membros de
250. Promover e proteger os direitos humanos e conselhos sobre questões relativas às pessoas portadoras de
liberdades fundamentais dos ciganos. deficiência.
251. Apoiar a realização de estudos e pesquisas sobre a
história, cultura e tradições da comunidade cigana. Idosos
252. Apoiar projetos educativos que levem em
consideração as necessidades especiais das crianças e 271. Criar, fortalecer e descentralizar programas de
adolescentes ciganos, bem como estimular a revisão de assistência ao?s idosos, de forma a contribuir para sua
documentos, dicionários e livros escolares que contenham integração à família e à sociedade e a incentivar o atendimento
estereótipos depreciativos com respeito aos ciganos. no seu próprio ambiente.
253. Apoiar a realização de estudos para a criação de
cooperativas de trabalho para ciganos.

Conhecimentos Específicos 50
Apostila Digital Licenciada para Alice Caroline Guarino dos Santos - alice.guarino@hotmail.com (Proibida a Revenda)
APOSTILAS OPÇÃO

272. Adotar medidas para estimular o atendimento e em espaços públicos, especialmente nas escolas, bibliotecas
prioritário às pessoas idosas nas instituições públicas e e espaços comunitários.
privadas. 292. Garantir a universalização, a obrigatoriedade e a
273. Apoiar a formação de conselhos de defesa dos direitos qualidade do ensino fundamental, estimulando a adoção da
dos idosos e a implementação de programas de proteção, com jornada escolar ampliada, a valorização do magistério e a
a participação de organizações não-governamentais. participação da comunidade na gestão das escolas, e
274. Desenvolver programas de habitação adequados às garantindo apoio ao transporte escolar.
necessidades das pessoas idosas, principalmente em áreas 293. Promover a equidade nas condições de acesso,
carentes. permanência e êxito escolar do aluno no ensino fundamental,
275. Adotar medidas para assegurar a responsabilização por meio da ampliação de programas de transferência direta
de familiares pelo abandono de pessoas idosas. de renda vinculada à educação (bolsa-escola) e de aceleração
276. Promover a remoção de barreiras arquitetônicas, da aprendizagem.
ambientais, de transporte e de comunicação para facilitar o 294. Garantir o suprimento de livros gratuitos e de
acesso e a locomoção da pessoa idosa aos serviços e áreas qualidade às escolas públicas do ensino fundamental.
públicas e aos edifícios comerciais. 295. Suprir parcialmente as necessidades nutricionais dos
277. Estimular a adoção de medidas para que o documento alunos das escolas públicas e das escolas mantidas por
de identidade seja aceito como comprovante de idade para a entidades filantrópicas por meio do oferecimento de, no
concessão do passe livre nos sistemas de transporte público. mínimo, uma refeição diária adequada, estimulando bons
278. Apoiar a criação e o fortalecimento de programas hábitos alimentares e procurando diminuir a evasão e a
descentralizados de assistência aos idosos, com ênfase na repetência.
integração social da pessoa idosa e permanência junto à 296. Promover a expansão do acesso ao ensino médio com
família. equidade e adequar a oferta atual, de forma ordenada e
279. Estimular o combate à violência e à discriminação atendendo a padrões básicos mínimos.
contra a pessoa idosa, inclusive por meio de ações de 297. Adotar uma concepção para o ensino médio que
sensibilização e capacitação que contribuam para prevenir a corresponda às determinações da Lei de Diretrizes e Bases da
violação de seus direitos. Educação - LDB, assim como à demanda e às necessidades do
280. Incentivar a criação, nos estados e municípios, de país.
serviços telefônicos de informação, orientação e recepção de 298. Implementar a reforma curricular e assegurar a
denúncias (disque-idoso). formação continuada de docentes e gestores de escolas de
ensino médio.
Garantia do Direito à Educação 299. Equipar progressivamente as escolas de ensino médio
com bibliotecas, laboratórios de informática e ciências e kit
281. Contribuir para a formulação de diretrizes e normas tecnológico para recepção da TV Escola.
para a educação infantil de modo a garantir padrões básicos de 300. Estimular a melhoria dos processos de gestão dos
atendimento em creches e pré-escolas. sistemas educacionais nos estados e municípios.
282. Contribuir para o planejamento, desenvolvimento e 301. Promover a articulação e a complementaridade entre
avaliação de práticas educativas, além da construção de a educação profissional e o ensino médio.
propostas educativas que respondam às necessidades das 302. Criar cursos que garantam perspectiva de trabalho
crianças e de seus familiares nas diferentes regiões do país. para os jovens, que facilitem seu acesso ao mercado e que
283. Promover um ensino fundado na tolerância, na paz e atendam também aos profissionais já inseridos no mercado de
no respeito às diferenças, que contemple a diversidade trabalho.
cultural do país. 303. Estimular a educação continuada e permanente como
284. Ampliar programas voltados para a redução da forma de atualizar os conhecimentos de jovens e adultos, com
violência nas escolas, a exemplo do programa 'Paz nas Escolas', base em competências requeridas para o exercício
especialmente em áreas urbanas que apresentem aguda profissional.
situação de carência e exclusão. 304. Apoiar a criação de mecanismos permanentes para
285. Incentivar a associação estudantil em todos os níveis fomentar a articulação entre escolas, trabalhadores e
e a criação de conselhos escolares compostos por familiares, empresários, com vistas à definição e revisão das
entidades, organizações não-governamentais e associações, competências necessárias às diferentes áreas profissionais.
para a fiscalização, avaliação e elaboração de programas e 305. Identificar oportunidades, estimular iniciativas, gerar
currículos escolares. alternativas e apoiar negociações que encaminhem o melhor
286. Propor medidas destinadas a democratizar o atendimento educacional às pessoas com necessidades
processo de escolha dos dirigentes de escolas públicas, educativas especiais, de forma a garantir a sua integração
estaduais e municipais, com a participação das comunidades escolar e social.
escolares e locais. 306. Garantir a ampliação da oferta do ensino superior de
modo a atender a demanda gerada pela expansão do ensino
287. Incrementar a qualidade do ensino, com intervenções médio no país.
em segmentos determinantes do sucesso escolar. 307. Estabelecer políticas e mecanismos que possibilitem
288. Consolidar um sistema de avaliação dos resultados do a oferta de cursos de graduação por meio de metodologias
ensino público e privado em todo o país. alternativas tais como a educação à distância e a capacitação
289. Assegurar o financiamento e a otimização do uso dos em serviço.
recursos públicos destinados à educação. 308. Apoiar a criação, nas universidades, de cursos de
290. Realizar periodicamente censos educacionais em extensão voltados para a proteção e promoção de direitos
parceria com as secretarias de educação dos estados e do humanos.
Distrito Federal, com o objetivo de produzir dados estatístico- 309. Propor a criação de ouvidorias nas universidades.
educacionais para subsidiar o planejamento e a gestão da 310. Propor medidas destinadas à garantia e promoção da
educação nas esferas governamentais. autonomia universitária.
291. Apoiar a popularização do uso do microcomputador e 311. Reduzir o índice de analfabetismo da população
da internet, através da massificação dessa tecnologia e da brasileira, elevando a média do tempo de estudos e ampliando
realização de cursos de treinamento em comunidades carentes programas de alfabetização para jovens e adultos.

Conhecimentos Específicos 51
Apostila Digital Licenciada para Alice Caroline Guarino dos Santos - alice.guarino@hotmail.com (Proibida a Revenda)
APOSTILAS OPÇÃO

312. Estabelecer mecanismos de promoção da equidade de 334. Garantir a vigilância sanitária de medicamentos,
acesso ao ensino superior, levando em consideração a alimentos e outros produtos.
necessidade de que o contingente de alunos universitários 335. Promover a produção de medicamentos genéricos e
reflita a diversidade racial e cultural da sociedade brasileira. divulgar, junto à sociedade brasileira, o seu significado e custo.
313. Assegurar aos quilombolas e povos indígenas uma 336. Ampliar e fortalecer os programas de assistência aos
educação escolar diferenciada, respeitando o seu universo portadores de anemia falciforme.
sociocultural e linguístico. 337. Assegurar o cumprimento da obrigatoriedade, no
314. Implantar a educação nos presídios seguindo as serviço público de saúde, da realização do teste de traços
diretrizes da LDB. falcêmicos e da anemia falciforme em recém-nascidos.
338. Garantir o acesso aos exames diagnósticos e à
Garantia do Direito à Saúde, à Previdência e à terapêutica de anormalidades no metabolismo.
Assistência Social 339. Intensificar as ações destinadas a eliminar a
hanseníase como problema de saúde pública no país, visando
315. Assegurar o princípio da universalização do acesso à a garantir o diagnóstico precoce e o tratamento dos
saúde, fortalecendo o Sistema Único de Saúde - SUS, portadores, bem como a promover medidas destinadas a
assegurando sua autonomia e democratização, bem como a combater o preconceito contra a doença.
sua consolidação em todos os estados e municípios brasileiros. 340. Intensificar as ações destinadas a controlar a
316. Promover a humanização e a qualidade do tuberculose no país, visando a garantir o diagnóstico precoce
atendimento do SUS, bem como a integralidade e a equidade e o tratamento dos portadores, bem como a promover medidas
de atenção à saúde da população. destinadas a combater o preconceito contra a doença.
317. Ampliar o acesso da população aos serviços básicos 341. Garantir a atenção integral à saúde dos idosos,
de saúde, tendo como eixo central de atuação as equipes de promovendo o acesso aos medicamentos específicos no
saúde da família. âmbito do SUS.
318. Garantir o processo de transformação do modelo de 342. Garantir a atenção integral à saúde dos adolescentes,
atenção à saúde a partir do fortalecimento da atenção básica, levando em conta as necessidades específicas desse segmento
valendo-se, para tanto, da expansão e consolidação do populacional.
Programa de Saúde da Família - PSF. 343. Garantir a atenção integral à saúde dos povos
319. Apoiar o fortalecimento de programas voltados para indígenas, levando em consideração as suas necessidades
a assistência integral à saúde da mulher. específicas.
320. Divulgar o conceito de direitos reprodutivos, com
base nas plataformas do Cairo e de Pequim, desenvolvendo 344. Promover o controle dos fundos de pensão e dos
campanhas de pré-natal e parto humanizado, bem como planos privados de saúde, divulgando amplamente os direitos
implementando comitês de prevenção da mortalidade dos pacientes e seus mecanismos de efetivação.
materna e da gravidez na adolescência. 345. Criar o sistema de vigilância epidemiológica da saúde
321. Implementar, em todos os municípios brasileiros, o do trabalhador.
Programa de Humanização do Parto e Nascimento, que visa a 346. Delinear uma política de atendimento à saúde nos
assegurar a realização de, pelo menos, seis consultas de pré- estabelecimentos penitenciários do país no âmbito do SUS.
natal e de todos os exames, bem como a definição do serviço 347. Fortalecer a integração de ações entre o Ministério
de saúde onde será realizado o parto. Público, o Ministério da Saúde, a Comissão de Direitos
322. Considerar o aborto como tema de saúde pública, com Humanos da Câmara dos Deputados, bem como organizações
a garantia do acesso aos serviços de saúde para os casos da sociedade civil.
previstos em lei. 348. Garantir o diagnóstico, tratamento e
323. Desenvolver programas educativos sobre acompanhamento, no âmbito da atenção básica de saúde, dos
planejamento familiar, promovendo o acesso aos métodos portadores de hipertensão arterial e de diabetes.
anticoncepcionais no âmbito do SUS. 349. Acelerar a implementação de medidas destinadas a
324. Ampliar e fortalecer programas voltados para a desburocratizar os serviços do Instituto Nacional do Seguro
assistência domiciliar terapêutica. Social - INSS para a concessão de aposentadorias e benefícios.
325. Reduzir a morbimortalidade de crianças de zero a 350. Implementar programa de remuneração para mães
cinco anos de idade, por meio de programas de atenção não amparadas pela seguridade.
integral à saúde da criança e de incentivo ao aleitamento 351. Estudar a possibilidade de introdução de recorte
materno. racial na concessão dos benefícios continuados de assistência
326. Ampliar e fortalecer programas de combate à social.
mortalidade materna e infantil. 352. Estimular a adesão do trabalhador urbano e rural ao
327. Criar o sistema de vigilância epidemiológica de regime geral de previdência social.
acidentes e violência e implementar programas de prevenção 353. Implementar mecanismos de controle social da
à violência pública e doméstica, esclarecendo seus riscos para previdência social.
a saúde e as implicações judiciais da mesma.
328. Assegurar a assistência adequada e oportuna às Saúde Mental
vítimas de acidentes e violência.
329. Estimular e fortalecer a participação social no SUS, 354. Apoiar a divulgação e a aplicação da Lei nº 10.216, de
inclusive na identificação de prioridades na área da saúde. 6 de abril de 2001, com vistas à desconstrução do aparato
330. Promover o treinamento e a capacitação sistemática manicomial sob a perspectiva da reorientação do modelo de
de agentes comunitários de saúde. atenção em saúde mental.
331. Apoiar programas que tenham como objetivo 355. Estabelecer mecanismos de normatização e
prevenir e reduzir os riscos, acidentes e doenças relacionadas acompanhamento das ações das secretarias de justiça e
ao ambiente e ao processo de trabalho. cidadania nos estados, no que diz respeito ao funcionamento
332. Apoiar programas voltados para a proteção da saúde dos hospitais de custódia e tratamento psiquiátrico.
de profissionais do sexo. 356. Promover esforço intersetorial em favor da
333. Garantir a assistência farmacêutica básica no âmbito substituição do modelo de atenção dos hospitais de custódia e
do SUS. tratamento por tratamento referenciado na rede SUS.

Conhecimentos Específicos 52
Apostila Digital Licenciada para Alice Caroline Guarino dos Santos - alice.guarino@hotmail.com (Proibida a Revenda)
APOSTILAS OPÇÃO

357. Promover debates sobre a inimputabilidade penal das 373. Diagnosticar e monitorar o processo de
pessoas acometidas por transtornos psíquicos. implementação das cooperativas de trabalho, com ênfase na
358. Criar programas de atendimento às pessoas observância dos direitos trabalhistas.
portadoras de doenças mentais, apoiando tratamentos 374. Organizar banco de dados com indicadores sociais,
alternativos à internação, de forma a conferir prioridade a que traduzam as condições de emprego, subemprego e
modelos de atendimento psicossocial, com a eliminação desemprego, sob a perspectiva de gênero e raça.
progressiva dos manicômios. 375. Fortalecer a política de concessão do seguro-
359. Criar uma política de atenção integral às vítimas de desemprego e assegurar o desenvolvimento de programas de
sofrimento psíquico na área da saúde mental, assegurando o qualificação e requalificação profissional.
cumprimento da carta de direitos dos usuários de saúde 376. Estimular a adoção de políticas de ação afirmativa no
mental e o monitoramento dos hospitais psiquiátricos. serviço público e no setor privado, com vistas a estimular
maior participação dos grupos vulneráveis no mercado de
Dependência Química trabalho.
377. Estimular programas de voluntariado em instituições
360. Promover campanhas nacionais de prevenção do públicas e privadas como forma de promoção dos direitos
alcoolismo e do uso de drogas que geram dependência humanos.
química, incentivando estudos, pesquisas e programas para 378. Criar um programa de atenção especial aos direitos do
limitar a incidência e o impacto do consumo de drogas ilícitas. trabalhador rural.
361. Propor o tratamento dos dependentes de drogas sob 379. Apurar denúncias de desrespeito aos direitos dos
o enfoque de saúde pública. trabalhadores, em especial aos assalariados rurais.
362. Apoiar ações para implementação do Programa de 380. Ampliar programas de erradicação do trabalho
Ação Nacional Antidrogas - PANAD. infantil, com vistas a uma ação particularmente voltada para
363. Apoiar programas de assistência e orientação para crianças de área urbana em situação de risco, priorizando a
usuários de drogas, em substituição ao indiciamento em repressão a atividades ilegais que utilizam crianças e
inquérito policial e processo judicial. adolescentes, tais como a exploração sexual e prostituição
infantis e o tráfico de drogas.
HIV/AIDS 381. Fortalecer as ações do Fórum Nacional de Prevenção
e Erradicação do Trabalho Infantil.
364. Apoiar a participação dos portadores de doenças 382. Apoiar a aprovação da proposta de emenda
sexualmente transmissíveis - DST e de pessoas com HIV/AIDS constitucional que altera o Artigo nº 243 da Constituição
e suas organizações na formulação e implementação de Federal, incluindo entre as hipóteses de expropriação de
políticas e programas de combate e prevenção das DST e do terras, além do cultivo de plantas psicotrópicas, a ocorrência
HIV/AIDS. de trabalho forçado.
365. Incentivar campanhas de informação sobre DST e 383. Garantir o cumprimento das Convenções nº 29, 105 e
HIV/AIDS, visando a esclarecer a população sobre os 111 da OIT, que tratam do trabalho forçado e da discriminação
comportamentos que facilitem ou dificultem a sua nos locais de trabalho.
transmissão. 384. Apoiar a reestruturação do Grupo Executivo de
366. Apoiar a melhoria da qualidade do tratamento e Repressão ao Trabalho Forçado - GERTRAF, vinculado ao
assistência das pessoas com HIV/AIDS, incluindo a ampliação Ministério do Trabalho e Emprego - MTE, assegurando a maior
da acessibilidade e a redução de custos. participação de entidades da sociedade civil em sua
367. Assegurar atenção às especificidades e diversidade composição.
cultural das populações, as questões de gênero, raça e 385. Fortalecer a atuação do Grupo Especial de
orientação sexual nas políticas e programas de combate e Fiscalização Móvel do Ministério do Trabalho e Emprego com
prevenção das DST e HIV/AIDS, nas campanhas de informação vistas à erradicação do trabalho forçado.
e nas ações de tratamento e assistência. 386. Criar, nas organizações policiais, divisões
especializadas na repressão ao trabalho forçado, com atenção
368. Incentivar a realização de estudos e pesquisas sobre especial para as crianças, adolescentes, estrangeiros e
DST e HIV/AIDS nas diversas áreas do conhecimento, migrantes brasileiros.
atentando para princípios éticos de pesquisa. 387. Criar e capacitar, no âmbito do Departamento da
Polícia Federal, grupo especializado na repressão do trabalho
Garantia do Direito ao Trabalho forçado para apoio consistente às ações da fiscalização móvel
do MTE.
369. Assegurar e preservar os direitos do trabalhador 388. Promover campanhas de sensibilização sobre o
previstos na legislação nacional e internacional. trabalho forçado e degradante e as formas contemporâneas de
370. Zelar pela implementação da legislação que promove escravidão nos estados onde ocorre trabalho forçado e nos
a igualdade em relação ao mercado de trabalho, sem polos de aliciamento de trabalhadores.
discriminação de idade, raça, sexo, orientação sexual, credo, 389. Sensibilizar juízes federais para a necessidade de
convicções filosóficas, condição social e estado sorológico, manter, no âmbito federal, a competência para julgar crimes
levando em consideração as pessoas com necessidades de trabalho forçado.
especiais, tipificando tal discriminação e definindo as penas 390. Estudar a possibilidade de aumentar os valores das
aplicáveis. multas impostas aos responsáveis pela exploração de trabalho
371. Apoiar, promover e fortalecer programas de forçado.
economia solidária, a exemplo das políticas de microcrédito, 391. Propor nova redação para o artigo 149 do Código
ampliando o acesso ao crédito para pequenos Penal, de modo a tipificar de forma mais precisa o crime de
empreendedores e para a população de baixa renda. submeter alguém à condição análoga a de escravo.
372. Promover políticas destinadas ao primeiro emprego,
incorporando questões de gênero e raça, e criar um banco de 392. Apoiar programas voltados para o reaparelhamento
dados, com ampla divulgação, voltado para o público juvenil dos estabelecimentos penais, com vistas a proporcionar
que busca o primeiro emprego. oportunidades de trabalho aos presos.

Conhecimentos Específicos 53
Apostila Digital Licenciada para Alice Caroline Guarino dos Santos - alice.guarino@hotmail.com (Proibida a Revenda)
APOSTILAS OPÇÃO

Acesso a Terra Portanto, a realização da 11ª CNDH e a elaboração do


PNDH-3 são ações compartilhadas entre governo e sociedade
393. Promover a segurança da posse, compreendendo a civil, e por isso capazes de gerar as bases para a formulação e
urbanização de áreas informalmente ocupadas e a fortalecimento de ações que convergem para uma Política
regularização de loteamentos populares, assim como a revisão Nacional de Direitos Humanos como política de Estado.
dos instrumentos legais que disciplinam a posse da terra, O PNDH-3 está estruturado em seis eixos orientadores,
como a lei que regula os registros públicos (Lei 6.015/73) e a subdivididos em 25 diretrizes, 82 objetivos estratégicos que
lei federal de parcelamento do solo urbano (Lei 6.766/79). incorporam ou refletem os 7 eixos, as 36 diretrizes e as 700
394. Promover a igualdade de acesso à terra, por meio do resoluções da 11ª CNDH. O Programa tem ainda, como alicerce
desenvolvimento de uma política fundiária urbana que de sua construção, as resoluções das Conferências Nacionais
considere a função social da terra como base de apoio para a temáticas, os Planos e Programas do governo federal, os
implementação de políticas habitacionais. Tratados internacionais ratificado pelo Estado brasileiro e as
395. Implementar a regularização fundiária, o Recomendações dos Comitês de Monitoramento de Tratados
reassentamento e a reforma agrária, respeitando os direitos à da ONU e dos Relatores especiais.
moradia adequada e acessível, à demarcação de áreas
indígenas e à titulação das terras de remanescentes de Caro candidato, para o acesso na íntegra da terceira
quilombos. versão do PNDH segue o link
396. Criar e apoiar políticas e programas de ação http://www.sdh.gov.br/assuntos/direito-para-
integrados para o assentamento de trabalhadores sem-terra, todos/programas/pdfs/programa-nacional-de-direitos-
com infraestrutura adequada para a produção agrícola, humanos-pndh-3
agroindústria e incentivo a outras atividades econômicas
compatíveis com a defesa do meio ambiente. DECRETO Nº 7.037, DE 21 DE DEZEMBRO DE 2009.
397. Promover a agricultura familiar e modelos de
agricultura sustentável, na perspectiva da distribuição da Aprova o Programa Nacional de Direitos Humanos -
riqueza e do combate à fome. PNDH-3 e dá outras providências.
398. Fortalecer políticas de incentivo à agricultura
familiar, em particular nos assentamentos de reforma agrária, O PRESIDENTE DA REPÚBLICA, no uso da atribuição que
transformando-os em base provedora de segurança alimentar lhe confere o art. 84, inciso VI, alínea “a”, da Constituição,
local e sustentável.
399. Adotar medidas destinadas a coibir práticas de DECRETA:
violência contra movimentos sociais que lutam pelo acesso à
terra. Art. 1o Fica aprovado o Programa Nacional de Direitos
400. Apoiar a aprovação de projeto de lei que propõe que Humanos - PNDH-3, em consonância com as diretrizes,
a concessão de medida liminar de reintegração de posse seja objetivos estratégicos e ações programáticas estabelecidos, na
condicionada à comprovação da função social da propriedade, forma do Anexo deste Decreto.
tornando obrigatória a intervenção do Ministério Público em
todas as fases processuais de litígios envolvendo a posse da Art. 2o O PNDH-3 será implementado de acordo com os
terra urbana e rural. seguintes eixos orientadores e suas respectivas diretrizes:
401. Promover ações integradas entre o INCRA, as I - Eixo Orientador I: Interação democrática entre Estado e
secretarias de justiça, as secretarias de segurança pública, os sociedade civil:
Ministérios Públicos e o Poder Judiciário, para evitar a a) Diretriz 1: Interação democrática entre Estado e
realização de despejos forçados de trabalhadores rurais, sociedade civil como instrumento de fortalecimento da
conforme a Resolução n.º 1993/77 da Comissão de Direitos democracia participativa;
Humanos das Nações Unidas, garantindo o prévio b) Diretriz 2: Fortalecimento dos Direitos Humanos como
reassentamento das famílias desalojadas. instrumento transversal das políticas públicas e de interação
402. Priorizar a regularização fundiária de áreas ocupadas, democrática; e
implantando um padrão mínimo de urbanização, de c) Diretriz 3: Integração e ampliação dos sistemas de
equipamentos e serviços públicos nos empreendimentos informações em Direitos Humanos e construção de
habitacionais e na regularização de áreas ocupadas. mecanismos de avaliação e monitoramento de sua efetivação;
II - Eixo Orientador II: Desenvolvimento e Direitos
Programa Nacional de Direitos Humanos – PNDH-3 Humanos:
a) Diretriz 4: Efetivação de modelo de desenvolvimento
A terceira versão do Programa Nacional de Direitos sustentável, com inclusão social e econômica, ambientalmente
Humanos – PNDH-3 dá continuidade ao processo histórico de equilibrado e tecnologicamente responsável, cultural e
consolidação das orientações para concretizar a promoção e regionalmente diverso, participativo e não discriminatório;
defesa dos Direitos Humanos no Brasil. Avança incorporando b) Diretriz 5: Valorização da pessoa humana como sujeito
a transversalidade nas diretrizes e nos objetivos estratégicos central do processo de desenvolvimento; e
propostos, na perspectiva da universalidade, indivisibilidade e c) Diretriz 6: Promover e proteger os direitos ambientais
interdependência dos Direitos Humanos. como Direitos Humanos, incluindo as gerações futuras como
Em 2008 deu-se início a uma cuidadosa atualização e sujeitos de direitos;
revisão do Programa Nacional de Direitos Humanos I e II, III - Eixo Orientador III: Universalizar direitos em um
tendo como instrumento fundamental a realização da 11ª contexto de desigualdades:
Conferência Nacional dos Direitos Humanos – 11ª CNDH. a) Diretriz 7: Garantia dos Direitos Humanos de forma
universal, indivisível e interdependente, assegurando a
Foram realizados, em todos os Estados e no Distrito cidadania plena;
Federal, 137 encontros prévios às etapas estadual e distrital. b) Diretriz 8: Promoção dos direitos de crianças e
Esses encontros envolveram aproximadamente 14 mil adolescentes para o seu desenvolvimento integral, de forma
participantes, representando a sociedade civil organizada e o não discriminatória, assegurando seu direito de opinião e
poder público, garantindo força institucional ao Programa. participação;
c) Diretriz 9: Combate às desigualdades estruturais; e

Conhecimentos Específicos 54
Apostila Digital Licenciada para Alice Caroline Guarino dos Santos - alice.guarino@hotmail.com (Proibida a Revenda)
APOSTILAS OPÇÃO

d) Diretriz 10: Garantia da igualdade na diversidade; II - Secretaria Especial de Políticas para as Mulheres da
IV - Eixo Orientador IV: Segurança Pública, Acesso à Justiça Presidência da República;
e Combate à Violência: III - Secretaria Especial de Políticas de Promoção da
a) Diretriz 11: Democratização e modernização do sistema Igualdade Racial da Presidência da República;
de segurança pública; IV - Secretaria-Geral da Presidência da República;
b) Diretriz 12: Transparência e participação popular no V - Ministério da Cultura;
sistema de segurança pública e justiça criminal; VI - Ministério da Educação;
c) Diretriz 13: Prevenção da violência e da criminalidade e VII - Ministério da Justiça;
profissionalização da investigação de atos criminosos; VIII - Ministério da Pesca e Aqüicultura;
d) Diretriz 14: Combate à violência institucional, com IX - Ministério da Previdência Social;
ênfase na erradicação da tortura e na redução da letalidade X - Ministério da Saúde;
policial e carcerária; XI - Ministério das Cidades;
e) Diretriz 15: Garantia dos direitos das vítimas de crimes XII - Ministério das Comunicações;
e de proteção das pessoas ameaçadas; XIII - Ministério das Relações Exteriores;
f) Diretriz 16: Modernização da política de execução penal, XIV - Ministério do Desenvolvimento Agrário;
priorizando a aplicação de penas e medidas alternativas à XV - Ministério do Desenvolvimento Social e Combate à
privação de liberdade e melhoria do sistema penitenciário; e Fome;
g) Diretriz 17: Promoção de sistema de justiça mais XVI - Ministério do Esporte;
acessível, ágil e efetivo, para o conhecimento, a garantia e a XVII - Ministério do Meio Ambiente;
defesa de direitos; XVIII - Ministério do Trabalho e Emprego;
V - Eixo Orientador V: Educação e Cultura em Direitos XIX - Ministério do Turismo;
Humanos: XX - Ministério da Ciência e Tecnologia; e
a) Diretriz 18: Efetivação das diretrizes e dos princípios da XXI - Ministério de Minas e Energia.
política nacional de educação em Direitos Humanos para § 2o O Secretário Especial dos Direitos Humanos da
fortalecer uma cultura de direitos; Presidência da República designará os representantes do
b) Diretriz 19: Fortalecimento dos princípios da Comitê de Acompanhamento e Monitoramento do PNDH-3.
democracia e dos Direitos Humanos nos sistemas de educação § 3o O Comitê de Acompanhamento e Monitoramento do
básica, nas instituições de ensino superior e nas instituições PNDH-3 poderá constituir subcomitês temáticos para a
formadoras; execução de suas atividades, que poderão contar com a
c) Diretriz 20: Reconhecimento da educação não formal participação de representantes de outros órgãos do Governo
como espaço de defesa e promoção dos Direitos Humanos; Federal.
d) Diretriz 21: Promoção da Educação em Direitos § 4o O Comitê convidará representantes dos demais
Humanos no serviço público; e Poderes, da sociedade civil e dos entes federados para
e) Diretriz 22: Garantia do direito à comunicação participarem de suas reuniões e atividades.
democrática e ao acesso à informação para consolidação de
uma cultura em Direitos Humanos; e Art. 5o Os Estados, o Distrito Federal, os Municípios e os
VI - Eixo Orientador VI: Direito à Memória e à Verdade: órgãos do Poder Legislativo, do Poder Judiciário e do
a) Diretriz 23: Reconhecimento da memória e da verdade Ministério Público, serão convidados a aderir ao PNDH-3.
como Direito Humano da cidadania e dever do Estado;
b) Diretriz 24: Preservação da memória histórica e Art. 6o Este Decreto entra em vigor na data de sua
construção pública da verdade; e publicação.
c) Diretriz 25: Modernização da legislação relacionada com
promoção do direito à memória e à verdade, fortalecendo a Art. 7o Fica revogado o Decreto no 4.229, de 13 de maio de
democracia. 2002.
Parágrafo único. A implementação do PNDH-3, além dos
responsáveis nele indicados, envolve parcerias com outros Brasília, 21 de dezembro de 2009; 188o da Independência
órgãos federais relacionados com os temas tratados nos eixos e 121o da República.
orientadores e suas diretrizes.
ANEXO
Art. 3o As metas, prazos e recursos necessários para a
implementação do PNDH-3 serão definidos e aprovados em Eixo Orientador I:
Planos de Ação de Direitos Humanos bianuais.
Interação democrática entre Estado e sociedade civil
Art. 4o Fica instituído o Comitê de Acompanhamento e
Monitoramento do PNDH-3, com a finalidade de: A partir da metade dos anos 1970, começam a ressurgir no
I - promover a articulação entre os órgãos e entidades Brasil iniciativas de rearticulação dos movimentos sociais, a
envolvidos na implementação das suas ações programáticas; despeito da repressão política e da ausência de canais
II - elaborar os Planos de Ação dos Direitos Humanos; democráticos de participação. Fortes protestos e a luta pela
III - estabelecer indicadores para o acompanhamento, democracia marcaram esse período. Paralelamente, surgiram
monitoramento e avaliação dos Planos de Ação dos Direitos iniciativas populares nos bairros reivindicando direitos
Humanos; básicos como saúde, transporte, moradia e controle do custo
IV - acompanhar a implementação das ações e de vida. Em um primeiro momento, eram iniciativas
recomendações; e atomizadas, buscando conquistas parciais, mas que ao longo
V - elaborar e aprovar seu regimento interno. dos anos foram se caracterizando como movimentos sociais
§ 1o O Comitê de Acompanhamento e Monitoramento do organizados.
PNDH-3 será integrado por um representante e respectivo Com o avanço da democratização do País, os movimentos
suplente de cada órgão a seguir descrito, indicados pelos sociais multiplicaram-se. Alguns deles institucionalizaram-se
respectivos titulares: e passaram a ter expressão política. Os movimentos populares
I - Secretaria Especial dos Direitos Humanos da Presidência da e sindicatos foram, no caso brasileiro, os principais
República, que o coordenará; promotores da mudança e da ruptura política em diversas

Conhecimentos Específicos 55
Apostila Digital Licenciada para Alice Caroline Guarino dos Santos - alice.guarino@hotmail.com (Proibida a Revenda)
APOSTILAS OPÇÃO

épocas e contextos históricos. Com efeito, durante a etapa de Aperfeiçoar a interlocução entre Estado e sociedade civil
elaboração da Constituição Cidadã de 1988, esses segmentos depende da implementação de medidas que garantam à
atuaram de forma especialmente articulada, afirmando-se sociedade maior participação no acompanhamento e
como um dos pilares da democracia e influenciando monitoramento das políticas públicas em Direitos Humanos,
diretamente os rumos do País. num diálogo plural e transversal entre os vários atores sociais
Nos anos que se seguiram, os movimentos passaram a se e deles com o Estado. Ampliar o controle externo dos órgãos
consolidar por meio de redes com abrangência regional ou públicos por meio de ouvidorias, monitorar os compromissos
nacional, firmando-se como sujeitos na formulação e internacionais assumidos pelo Estado brasileiro, realizar
monitoramento das políticas públicas. Nos anos 1990, conferências periódicas sobre a temática, fortalecer e apoiar a
desempenharam papel fundamental na resistência a todas as criação de conselhos nacional, distrital, estaduais e municipais
orientações do neoliberalismo de flexibilização dos direitos de Direitos Humanos, garantindo-lhes eficiência, autonomia e
sociais, privatizações, dogmatismo do mercado e independência são algumas das formas de assegurar o
enfraquecimento do Estado. Nesse mesmo período, aperfeiçoamento das políticas públicas por meio de diálogo, de
multiplicaram-se pelo País experiências de gestão estadual e mecanismos de controle e das ações contínuas da sociedade
municipal em que lideranças desses movimentos, em larga civil. Fortalecer as informações em Direitos Humanos com
escala, passaram a desempenhar funções de gestores públicos. produção e seleção de indicadores para mensurar demandas,
Com as eleições de 2002, alguns dos setores mais monitorar, avaliar, reformular e propor ações efetivas, garante
organizados da sociedade trouxeram reivindicações históricas e consolida o controle social e a transparência das ações
acumuladas, passando a influenciar diretamente a atuação do governamentais.
governo e vivendo de perto suas contradições internas. A adoção de tais medidas fortalecerá a democracia
Nesse novo cenário, o diálogo entre Estado e sociedade participativa, na qual o Estado atua como instância
civil assumiu especial relevo, com a compreensão e a republicana da promoção e defesa dos Direitos Humanos e a
preservação do distinto papel de cada um dos segmentos no sociedade civil como agente ativo – propositivo e reativo – de
processo de gestão. A interação é desenhada por acordos e sua implementação.
dissensos, debates de idéias e pela deliberação em torno de Diretriz 1: Interação democrática entre Estado e sociedade
propostas. Esses requisitos são imprescindíveis ao pleno civil como instrumento de fortalecimento da democracia
exercício da democracia, cabendo à sociedade civil exigir, participativa.
pressionar, cobrar, criticar, propor e fiscalizar as ações do
Estado. Objetivo estratégico I:

Essa concepção de interação democrática construída entre Garantia da participação e do controle social das
os diversos órgãos do Estado e a sociedade civil trouxe consigo políticas públicas em Direitos Humanos, em diálogo plural
resultados práticos em termos de políticas públicas e avanços e transversal entre os vários atores sociais.
na interlocução de setores do poder público com toda a
diversidade social, cultural, étnica e regional que caracteriza Ações programáticas:
os movimentos sociais em nosso País. Avançou-se
fundamentalmente na compreensão de que os Direitos a)Apoiar, junto ao Poder Legislativo, a instituição do
Humanos constituem condição para a prevalência da Conselho Nacional dos Direitos Humanos, dotado de recursos
dignidade humana, e que devem ser promovidos e protegidos humanos, materiais e orçamentários para o seu pleno
por meio do esforço conjunto do Estado e da sociedade civil. funcionamento, e efetuar seu credenciamento junto ao
Uma das finalidades do PNDH-3 é dar continuidade à Escritório do Alto Comissariado das Nações Unidas para os
integração e ao aprimoramento dos mecanismos de Direitos Humanos como "Instituição Nacional Brasileira",
participação existentes, bem como criar novos meios de como primeiro passo rumo à adoção plena dos "Princípios de
construção e monitoramento das políticas públicas sobre Paris".
Direitos Humanos no Brasil. Responsáveis: Secretaria Especial dos Direitos Humanos
No âmbito institucional o PNDH-3, amplia as conquistas na da Presidência da República; Ministério das Relações
área dos direitos e garantias fundamentais, pois internaliza a Exteriores
diretriz segundo a qual a primazia dos Direitos Humanos b)Fomentar a criação e o fortalecimento dos conselhos de
constitui princípio transversal a ser considerado em todas as Direitos Humanos em todos os Estados e Municípios e no
políticas públicas. Distrito Federal, bem como a criação de programas estaduais
As diretrizes deste capítulo discorrem sobre a importância de Direitos Humanos.
de fortalecer a garantia e os instrumentos de participação Responsável: Secretaria Especial dos Direitos Humanos
social, o caráter transversal dos Direitos Humanos e a da Presidência da República
construção de mecanismos de avaliação e monitoramento de c)Criar mecanismos que permitam ação coordenada entre
sua efetivação. Isso inclui a construção de sistema de os diversos conselhos de direitos, nas três esferas da
indicadores de Direitos Humanos e a articulação das políticas Federação, visando a criação de agenda comum para a
e instrumentos de monitoramento existentes. implementação de políticas públicas de Direitos Humanos.
O Poder Executivo tem papel protagonista na coordenação Responsáveis: Secretaria Especial dos Direitos Humanos
e implementação do PNDH-3, mas faz-se necessária a definição da Presidência da República, Secretaria-Geral da Presidência
de responsabilidades compartilhadas entre a União, Estados, da República
Municípios e do Distrito Federal na execução de políticas d)Criar base de dados dos conselhos nacionais, estaduais,
públicas, tanto quanto a criação de espaços de participação e distrital e municipais, garantindo seu acesso ao público em
controle social nos Poderes Judiciário e Legislativo, no geral.
Ministério Público e nas Defensorias, em ambiente de respeito, Responsáveis: Secretaria-Geral da Presidência da
proteção e efetivação dos Direitos Humanos. O conjunto dos República; Secretaria Especial dos Direitos Humanos da
órgãos do Estado – não apenas no âmbito do Executivo Federal Presidência da República
– deve estar comprometido com a implementação e e)Apoiar fóruns, redes e ações da sociedade civil que fazem
monitoramento do PNDH-3. acompanhamento, controle social e monitoramento das
políticas públicas de Direitos Humanos.

Conhecimentos Específicos 56
Apostila Digital Licenciada para Alice Caroline Guarino dos Santos - alice.guarino@hotmail.com (Proibida a Revenda)
APOSTILAS OPÇÃO

Responsáveis: Secretaria Especial dos Direitos Humanos lusófonos do continente africano, o Timor-Leste, Caribe e a
da Presidência da República; Secretaria-Geral da Presidência América Latina.
da República Responsáveis: Secretaria Especial dos Direitos Humanos
f)Estimular o debate sobre a regulamentação e efetividade da Presidência da República; Ministério das Relações
dos instrumentos de participação social e consulta popular, Exteriores
tais como lei de iniciativa popular, referendo, veto popular e
plebiscito. Objetivo estratégico II:
Responsáveis: Secretaria Especial dos Direitos Humanos Fortalecimento dos instrumentos de interação
da Presidência da República; Secretaria-Geral da Presidência democrática para a promoção dos Direitos Humanos.
da República
g)Assegurar a realização periódica de conferências de Ações programáticas:
Direitos Humanos, fortalecendo a interação entre a sociedade a) Criar o Observatório Nacional dos Direitos Humanos
civil e o poder público. para subsidiar, com dados e informações, o trabalho de
Responsável: Secretaria Especial dos Direitos Humanos monitoramento das políticas públicas e de gestão
da Presidência da República governamental e sistematizar a documentação e legislação,
nacionais e internacionais, sobre Direitos Humanos.
Objetivo estratégico II: Responsável: Secretaria Especial dos Direitos Humanos
Ampliação do controle externo dos órgãos públicos. da Presidência da República
b) Estimular e reconhecer pessoas e entidades com
Ações programáticas: destaque na luta pelos Direitos Humanos na sociedade
a)Ampliar a divulgação dos serviços públicos voltados brasileira e internacional, com a concessão de premiação,
para a efetivação dos Direitos Humanos, em especial nos bolsas e outros incentivos, na forma da legislação aplicável.
canais de transparência. Responsáveis: Secretaria Especial dos Direitos Humanos
Responsável: Secretaria Especial dos Direitos Humanos da Presidência da República; Ministério das Relações
da Presidência da República Exteriores
b)Propor a instituição da Ouvidoria Nacional dos Direitos c) Criar selo nacional "Direitos Humanos", a ser concedido
Humanos, em substituição à Ouvidoria-Geral da Cidadania, às entidades públicas e privadas que comprovem atuação
com independência e autonomia política, com mandato e destacada na defesa e promoção dos direitos fundamentais.
indicação pelo Conselho Nacional dos Direitos Humanos, Responsáveis: Secretaria Especial dos Direitos Humanos
assegurando recursos humanos, materiais e financeiros para da Presidência da República; Ministério da Justiça
seu pleno funcionamento. Diretriz 3: Integração e ampliação dos sistemas de
Responsável: Secretaria Especial dos Direitos Humanos informação em Direitos Humanos e construção de
da Presidência da República mecanismos de avaliação e monitoramento de sua efetivação.
c)Fortalecer a estrutura da Ouvidoria Agrária Nacional.
Responsável: Ministério do Desenvolvimento Agrário Objetivo estratégico I:
Desenvolvimento de mecanismos de controle social
Diretriz 2: Fortalecimento dos Direitos Humanos como das políticas públicas de Direitos Humanos, garantindo o
instrumento transversal das políticas públicas e de monitoramento e a transparência das ações
interação democrática. governamentais.

Objetivo estratégico I: Ações programáticas:


Promoção dos Direitos Humanos como princípios a)Instituir e manter sistema nacional de indicadores em
orientadores das políticas públicas e das relações Direitos Humanos, de forma articulada com os órgãos públicos
internacionais. e a sociedade civil.
Responsável: Secretaria Especial dos Direitos Humanos
Ações programáticas: da Presidência da República
a)Considerar as diretrizes e objetivos estratégicos do b)Integrar os sistemas nacionais de informações para
PNDH-3 nos instrumentos de planejamento do Estado, em elaboração de quadro geral sobre a implementação de
especial no Plano Plurianual, na Lei de Diretrizes políticas públicas e violações aos Direitos Humanos.
Orçamentárias e na Lei Orçamentária Anual. Responsável: Secretaria Especial dos Direitos Humanos
Responsáveis: Secretaria Especial dos Direitos Humanos da Presidência da República
da Presidência da República; Secretaria-Geral da Presidência c)Articular a criação de base de dados com temas
da República; Ministério do Planejamento, Orçamento e relacionados aos Direitos Humanos.
Gestão Responsável: Secretaria Especial dos Direitos Humanos
b)Propor e articular o reconhecimento da Presidência da República
do status constitucional de instrumentos internacionais de d)Utilizar indicadores em Direitos Humanos para
Direitos Humanos novos ou já existentes ainda não ratificados. mensurar demandas, monitorar, avaliar, reformular e propor
Responsáveis: Secretaria Especial dos Direitos Humanos ações efetivas.
da Presidência da República; Ministério da Justiça; Secretaria Responsáveis: Secretaria Especial dos Direitos Humanos
de Relações Institucionais da Presidência da República da Presidência da República; Secretaria Especial de Políticas
c)Construir e aprofundar agenda de cooperação para as Mulheres da Presidência da República; Secretaria
multilateral em Direitos Humanos que contemple Especial de Políticas de Promoção da Igualdade Racial da
prioritariamente o Haiti, os países lusófonos do continente Presidência da República; Ministério da Saúde; Ministério do
africano e o Timor-Leste. Desenvolvimento Social e Combate à Fome; Ministério da
Responsáveis: Secretaria Especial dos Direitos Humanos Justiça; Ministério das Cidades; Ministério do Meio Ambiente;
da Presidência da República; Ministério das Relações Ministério da Cultura; Ministério do Turismo; Ministério do
Exteriores Esporte; Ministério do Desenvolvimento Agrário
d)Aprofundar a agenda Sul-Sul de cooperação bilateral em e)Propor estudos visando a criação de linha de
Direitos Humanos que contemple prioritariamente os países financiamento para a implementação de institutos de pesquisa
e produção de estatísticas em Direitos Humanos nos Estados.

Conhecimentos Específicos 57
Apostila Digital Licenciada para Alice Caroline Guarino dos Santos - alice.guarino@hotmail.com (Proibida a Revenda)
APOSTILAS OPÇÃO

Responsável: Secretaria Especial dos Direitos Humanos serviços, sua distribuição ocorreria de forma a satisfazer as
da Presidência da República necessidades de todas as pessoas. Constatou-se, porém, que,
embora importante, o crescimento do PIB não é suficiente para
Objetivo estratégico II: causar, automaticamente, melhoria do bem estar para todas as
Monitoramento dos compromissos internacionais camadas sociais. Por isso, o conceito de desenvolvimento foi
assumidos pelo Estado brasileiro em matéria de Direitos adotado por ser mais abrangente e refletir, de fato, melhorias
Humanos. nas condições de vida dos indivíduos.
A teoria predominante de desenvolvimento econômico o
Ações programáticas: define como um processo que faz aumentar as possibilidades
a)Elaborar relatório anual sobre a situação dos Direitos de acesso das pessoas a bens e serviços, propiciadas pela
Humanos no Brasil, em diálogo participativo com a sociedade expansão da capacidade e do âmbito das atividades
civil. econômicas. O desenvolvimento seria a medida qualitativa do
Responsáveis: Secretaria Especial dos Direitos Humanos progresso da economia de um país, refletindo transições de
da Presidência da República; Ministério das Relações estágios mais baixos para estágios mais altos, por meio da
Exteriores adoção de novas tecnologias que permitem e favorecem essa
b)Elaborar relatórios periódicos para os órgãos de transição. Cresce nos últimos anos a assimilação das idéias
tratados da ONU, no prazo por eles estabelecidos, com base em desenvolvidas por Amartya Sem, que abordam o
fluxo de informações com órgãos do governo federal e com desenvolvimento como liberdade e seus resultados centrados
unidades da Federação. no bem estar social e, por conseguinte, nos direitos do ser
Responsáveis: Secretaria Especial dos Direitos Humanos humano.
da Presidência da República; Ministério das Relações São essenciais para o desenvolvimento as liberdades e os
Exteriores direitos básicos como alimentação, saúde e educação. As
c)Elaborar relatório de acompanhamento das relações privações das liberdades não são apenas resultantes da
entre o Brasil e o sistema ONU que contenha, entre outras, as escassez de recursos, mas sim das desigualdades inerentes aos
seguintes informações: mecanismos de distribuição, da ausência de serviços públicos
- Recomendações advindas de relatores especiais do e de assistência do Estado para a expansão das escolhas
Conselho de Direitos Humanos da ONU; individuais. Este conceito de desenvolvimento reconhece seu
- Recomendações advindas dos comitês de tratados do caráter pluralista e a tese de que a expansão das liberdades
Mecanismo de Revisão Periódica; não representa somente um fim, mas também o meio para seu
Responsáveis: Secretaria Especial dos Direitos Humanos alcance. Em consequência, a sociedade deve pactuar as
da Presidência da República; Ministério das Relações políticas sociais e os direitos coletivos de acesso e uso dos
Exteriores recursos. A partir daí, a medição de um índice de
d)Definir e institucionalizar fluxo de informações, com desenvolvimento humano veio substituir a medição de
responsáveis em cada órgão do governo federal e unidades da aumento do PIB, uma vez que o Índice de Desenvolvimento
Federação, referentes aos relatórios internacionais de Direitos Humano (IDH) combina a riqueza per capita indicada pelo PIB
Humanos e às recomendações dos relatores especiais do aos aspectos de educação e expectativa de vida, permitindo,
Conselho de Direitos Humanos da ONU e dos comitês de pela primeira vez, uma avaliação de aspectos sociais não
tratados. mensurados pelos padrões econométricos.
Responsáveis: Secretaria Especial dos Direitos Humanos No caso do Brasil, por muitos anos o crescimento
da Presidência da República; Ministério das Relações econômico não levou à distribuição justa de renda e riqueza,
Exteriores mantendo-se elevados índices de desigualdade. As ações de
e)Definir e institucionalizar fluxo de informações, com Estado voltadas para a conquista da igualdade socioeconômica
responsáveis em cada órgão do governo federal, referentes aos requerem ainda políticas permanentes, de longa duração, para
relatórios da Comissão Interamericana de Direitos Humanos e que se verifique a plena proteção e promoção dos Direitos
às decisões da Corte Interamericana de Direitos Humanos. Humanos. É necessário que o modelo de desenvolvimento
Responsáveis: Secretaria Especial dos Direitos Humanos econômico tenha a preocupação de aperfeiçoar os
da Presidência da República; Ministério das Relações mecanismos de distribuição de renda e de oportunidades para
Exteriores todos os brasileiros, bem como incorpore os valores de
f)Criar banco de dados público sobre todas as preservação ambiental. Os debates sobre as mudanças
recomendações dos sistemas ONU e OEA feitas ao Brasil, climáticas e o aquecimento global, gerados pela preocupação
contendo as medidas adotadas pelos diversos órgãos públicos com a maneira com que os países vêm explorando os recursos
para seu cumprimento. naturais e direcionando o progresso civilizatório, está na
Responsáveis: Secretaria Especial dos Direitos Humanos agenda do dia. Esta discussão coloca em questão os
da Presidência da República; Ministério das Relações investimentos em infraestrutura e modelos de
Exteriores desenvolvimento econômico na área rural, baseados, em
grande parte, no agronegócio, sem a preocupação com a
Eixo Orientador II: potencial violação dos direitos de pequenos e médios
Desenvolvimento e Direitos Humanos agricultores e das populações tradicionais.
O desenvolvimento pode ser garantido se as pessoas forem
O tema "desenvolvimento" tem sido amplamente debatido protagonistas do processo, pressupondo a garantia de acesso
por ser um conceito complexo e multidisciplinar. Não existe de todos os indivíduos aos direitos econômicos, sociais,
modelo único e preestabelecido de desenvolvimento, porém, culturais e ambientais, e incorporando a preocupação com a
pressupõe-se que ele deva garantir a livre determinação dos preservação e a sustentabilidade como eixos estruturantes de
povos, o reconhecimento de soberania sobre seus recursos e proposta renovada de progresso. Esses direitos têm como foco
riquezas naturais, respeito pleno à sua identidade cultural e a a distribuição da riqueza, dos bens e serviços.
busca de equidade na distribuição das riquezas. Todo esse debate traz desafios para a conceituação sobre
Durante muitos anos, o crescimento econômico, medido os Direitos Humanos no sentido de incorporar o
pela variação anual do Produto Interno Bruto (PIB), foi usado desenvolvimento como exigência fundamental. A perspectiva
como indicador relevante para medir o avanço de um país. dos Direitos Humanos contribui para redimensionar o
Acreditava-se que, uma vez garantido o aumento de bens e desenvolvimento. Motiva a passar da consideração de

Conhecimentos Específicos 58
Apostila Digital Licenciada para Alice Caroline Guarino dos Santos - alice.guarino@hotmail.com (Proibida a Revenda)
APOSTILAS OPÇÃO

problemas individuais a questões de interesse comum, de b) Expandir políticas públicas de geração e transferência
bem-estar coletivo, o que alude novamente o Estado e o chama de renda para erradicação da extrema pobreza e redução da
à corresponsabilidade social e à solidariedade. pobreza.
Ressaltamos que a noção de desenvolvimento está sendo Responsável: Ministério do Desenvolvimento Social e
amadurecida como parte de um debate em curso na sociedade Combate à Fome
e no governo, incorporando a relação entre os direitos c) Apoiar projetos de desenvolvimento sustentável local
econômicos, sociais, culturais e ambientais, buscando a para redução das desigualdades inter e intrarregionais e o
garantia do acesso ao trabalho, à saúde, à educação, à aumento da autonomia e sustentabilidade de espaços sub-
alimentação, à vida cultural, à moradia adequada, à regionais.
previdência, à assistência social e a um meio ambiente Responsáveis: Secretaria Especial dos Direitos Humanos
sustentável. A inclusão do tema Desenvolvimento e Direitos da Presidência da República; Ministério do Desenvolvimento
Humanos na 11a Conferência Nacional reforçou as estratégias Agrário
governamentais em sua proposta de desenvolvimento. d) Avançar na implantação da reforma agrária, como forma
de inclusão social e acesso aos direitos básicos, de forma
Assim, este capítulo do PNDH-3 propõe instrumentos de articulada com as políticas de saúde, educação, meio ambiente
avanço e reforça propostas para políticas públicas de redução e fomento à produção alimentar.
das desigualdades sociais concretizadas por meio de ações de Responsável: Ministério do Desenvolvimento Agrário
transferência de renda, incentivo à economia solidária e ao e) Incentivar as políticas públicas de economia solidária,
cooperativismo, à expansão da reforma agrária, ao fomento da de cooperativismo e associativismo e de fomento a pequenas e
aquicultura, da pesca e do extrativismo e da promoção do micro empresas.
turismo sustentável. Responsáveis: Ministério do Trabalho e Emprego;
O PNDH-3 inova ao incorporar o meio ambiente saudável Ministério do Desenvolvimento Agrário; Ministério das
e as cidades sustentáveis como Direitos Humanos, propõe a Cidades; Ministério do Desenvolvimento Social e Combate à
inclusão do item "direitos ambientais" nos relatórios de Fome
monitoramento sobre Direitos Humanos e do item "Direitos f) Fortalecer políticas públicas de apoio ao extrativismo e
Humanos" nos relatórios ambientais, assim como fomenta ao manejo florestal comunitário ambientalmente sustentáveis.
pesquisas de tecnologias socialmente inclusivas. Responsáveis: Ministério do Meio Ambiente; Ministério
Nos projetos e empreendimentos com grande impacto do Desenvolvimento Agrário; Ministério do Desenvolvimento,
socioambiental, o PNDH-3 garante a participação efetiva das Indústria e Comércio Exterior
populações atingidas, assim como prevê ações mitigatórias e g) Fomentar o debate sobre a expansão de plantios de
compensatórias. Considera fundamental fiscalizar o respeito monoculturas que geram impacto no meio ambiente e na
aos Direitos Humanos nos projetos implementados pelas cultura dos povos e comunidades tradicionais, tais como
empresas transnacionais, bem como seus impactos na eucalipto, cana-de-açúcar, soja, e sobre o manejo florestal, a
manipulação das políticas de desenvolvimento. Nesse sentido, grande pecuária, mineração, turismo e pesca.
avalia como importante mensurar o impacto da biotecnologia Responsável: Secretaria Especial dos Direitos Humanos
aplicada aos alimentos, da nanotecnologia, dos poluentes da Presidência da República
orgânicos persistentes, metais pesados e outros poluentes h)Erradicar o trabalho infantil, bem como todas as formas
inorgânicos em relação aos Direitos Humanos. de violência e exploração sexual de crianças e adolescentes nas
Alcançar o desenvolvimento com Direitos Humanos é cadeias produtivas, com base em códigos de conduta e no
capacitar as pessoas e as comunidades a exercerem a Estatuto da Criança e do Adolescente.
cidadania, com direitos e responsabilidades. É incorporar, nos Responsáveis: Secretaria Especial dos Direitos Humanos
projetos, a própria população brasileira, por meio de da Presidência da República; Ministério do Turismo
participação ativa nas decisões que afetam diretamente suas i)Garantir que os grandes empreendimentos e projetos de
vidas. É assegurar a transparência dos grandes projetos de infraestrutura resguardem os direitos dos povos indígenas e
desenvolvimento econômico e mecanismos de compensação de comunidades quilombolas e tradicionais, conforme
para a garantia dos Direitos Humanos das populações previsto na Constituição e nos tratados e convenções
diretamente atingidas. internacionais.
Por fim, este PNDH-3 reforça o papel da equidade no Plano Responsáveis: Ministério da Justiça; Ministério dos
Plurianual, como instrumento de garantia de priorização Transportes; Ministério da Integração Nacional; Ministério de
orçamentária de programas sociais. Minas e Energia; Secretaria Especial de Políticas de Promoção
da Igualdade Racial da Presidência da República; Ministério do
Diretriz 4: Efetivação de modelo de desenvolvimento Meio Ambiente; Ministério do Desenvolvimento Social e
sustentável, com inclusão social e econômica, Combate à Fome; Ministério da Pesca e Aquicultura; Secretaria
ambientalmente equilibrado e tecnologicamente Especial de Portos da Presidência da República
responsável, cultural e regionalmente diverso, j)Integrar políticas de geração de emprego e renda e
participativo e não discriminatório. políticas sociais para o combate à pobreza rural dos
agricultores familiares, assentados da reforma agrária,
Objetivo estratégico I: quilombolas, indígenas, famílias de pescadores e comunidades
Implementação de políticas públicas de tradicionais.
desenvolvimento com inclusão social. Responsáveis: Ministério do Desenvolvimento Social e
Combate à Fome; Ministério da Integração Nacional;
Ações programáticas: Ministério do Desenvolvimento Agrário; Ministério do
a) Ampliar e fortalecer as políticas de desenvolvimento Trabalho e Emprego; Ministério da Justiça; Secretaria Especial
social e de combate à fome, visando a inclusão e a promoção de Políticas de Promoção da Igualdade Racial da Presidência
da cidadania, garantindo a segurança alimentar e nutricional, da República; Ministério da Cultura; Ministério da Pesca e
renda mínima e assistência integral às famílias. Aquicultura
Responsável: Ministério do Desenvolvimento Social e k)Integrar políticas sociais e de geração de emprego e
Combate à Fome renda para o combate à pobreza urbana, em especial de
catadores de materiais recicláveis e população em situação de
rua.

Conhecimentos Específicos 59
Apostila Digital Licenciada para Alice Caroline Guarino dos Santos - alice.guarino@hotmail.com (Proibida a Revenda)
APOSTILAS OPÇÃO

Responsáveis: Ministério do Trabalho e Emprego; Responsáveis: Ministério do Trabalho e Emprego;


Ministério do Meio Ambiente; Ministério do Desenvolvimento Ministério do Desenvolvimento Social e Combate à Fome;
Social e Combate à Fome; Ministério das Cidades; Secretaria Ministério do Meio Ambiente; Ministério do Desenvolvimento
dos Direitos Humanos da Presidência da República Agrário; Ministério da Saúde
l)Fortalecer políticas públicas de fomento à aquicultura e à b)Garantir a aplicação do princípio da precaução na
pesca sustentáveis, com foco nos povos e comunidades proteção da agrobiodiversidade e da saúde, realizando
tradicionais de baixa renda, contribuindo para a segurança pesquisas que avaliem os impactos dos transgênicos no meio
alimentar e a inclusão social, mediante a criação e geração de ambiente e na saúde.
trabalho e renda alternativos e inserção no mercado de Responsáveis: Ministério da Saúde; Ministério do Meio
trabalho. Ambiente; Ministério de Ciência e Tecnologia
Responsáveis: Ministério da Pesca e Aquicultura; c)Fomentar tecnologias alternativas para substituir o uso
Ministério do Trabalho e Emprego; Ministério do de substâncias danosas à saúde e ao meio ambiente, como
Desenvolvimento Social e Combate à Fome poluentes orgânicos persistentes, metais pesados e outros
m)Promover o turismo sustentável com geração de poluentes inorgânicos.
trabalho e renda, respeito à cultura local, participação e Responsáveis: Ministério de Ciência e Tecnologia;
inclusão dos povos e das comunidades nos benefícios advindos Ministério do Meio Ambiente; Ministério da Saúde; Ministério
da atividade turística. da Agricultura, Pecuária e Abastecimento; Ministério do
Responsáveis: Ministério do Turismo; Ministério do Desenvolvimento, Indústria e Comércio Exterior
Desenvolvimento, Indústria e Comércio Exterior d)Fomentar tecnologias de gerenciamento de resíduos
sólidos e emissões atmosféricas para minimizar impactos à
Objetivo estratégico II: saúde e ao meio ambiente.
Fortalecimento de modelos de agricultura familiar e Responsáveis: Ministério de Ciência e Tecnologia;
agroecológica. Ministério do Meio Ambiente; Ministério da Saúde; Ministério
das Cidades
Ações programáticas: e)Desenvolver e divulgar pesquisas públicas para
a)Garantir que nos projetos de reforma agrária e diagnosticar os impactos da biotecnologia e da nanotecnologia
agricultura familiar sejam incentivados os modelos de em temas de Direitos Humanos.
produção agroecológica e a inserção produtiva nos mercados Responsáveis: Secretaria Especial dos Direitos Humanos
formais. da Presidência da República; Ministério da Saúde; Ministério
Responsáveis: Ministério do Desenvolvimento Agrário; do Meio Ambiente; Ministério da Agricultura, Pecuária e
Ministério do Desenvolvimento, Indústria e Comércio Exterior Abastecimento; Ministério de Ciência e Tecnologia
b)Fortalecer a agricultura familiar camponesa e a pesca f)Produzir, sistematizar e divulgar pesquisas econômicas e
artesanal, com ampliação do crédito, do seguro, da assistência metodologias de cálculo de custos socioambientais de projetos
técnica, extensão rural e da infraestrutura para de infraestrutura, de energia e de mineração que sirvam como
comercialização. parâmetro para o controle dos impactos de grandes projetos.
Responsáveis: Ministério do Desenvolvimento Agrário; Responsáveis: Ministério da Ciência e Tecnologia;
Ministério da Pesca e Aquicultura Ministério de Minas e Energia; Ministério do Meio Ambiente;
c)Garantir pesquisa e programas voltados à agricultura Secretaria de Assuntos Estratégicos da Presidência da
familiar e pesca artesanal, com base nos princípios da República; Ministério da Integração Nacional
agroecologia.
Responsáveis: Ministério do Desenvolvimento Agrário; Objetivo estratégico IV:
Ministério do Meio Ambiente; Ministério da Agricultura, Garantia do direito a cidades inclusivas e
Pecuária e Abastecimento; Ministério da Pesca e Aquicultura; sustentáveis.
Ministério do Desenvolvimento, Indústria e Comércio Exterior
d)Fortalecer a legislação e a fiscalização para evitar a Ações programáticas:
contaminação dos alimentos e danos à saúde e ao meio a)Apoiar ações que tenham como princípio o direito a
ambiente causados pelos agrotóxicos. cidades inclusivas e acessíveis como elemento fundamental da
Responsáveis: Ministério da Agricultura, Pecuária e implementação de políticas urbanas.
Abastecimento; Ministério do Meio Ambiente; Ministério da Responsáveis: Ministério das Cidades; Secretaria Especial
Saúde; Ministério do Desenvolvimento Agrário dos Direitos Humanos da Presidência da República; Ministério
e)Promover o debate com as instituições de ensino do Desenvolvimento, Indústria e Comércio Exterior
superior e a sociedade civil para a implementação de cursos e b)Fortalecer espaços institucionais democráticos,
realização de pesquisas tecnológicas voltados à temática participativos e de apoio aos Municípios para a implementação
socioambiental, agroecologia e produção orgânica, de planos diretores que atendam aos preceitos da política
respeitando as especificidades de cada região. urbana estabelecidos no Estatuto da Cidade.
Responsáveis: Ministério da Educação; Ministério do Responsável: Ministério das Cidades
Desenvolvimento Agrário c)Fomentar políticas públicas de apoio aos Estados,
Distrito Federal e Municípios em ações sustentáveis de
Objetivo estratégico III: urbanização e regularização fundiária dos assentamentos de
Fomento à pesquisa e à implementação de políticas população de baixa renda, comunidades pesqueiras e de
para o desenvolvimento de tecnologias socialmente provisão habitacional de interesse social, materializando a
inclusivas, emancipatórias e ambientalmente função social da propriedade.
sustentáveis. Responsáveis: Ministério das Cidades; Ministério do Meio
Ambiente; Ministério da Pesca e Aquicultura
Ações programáticas: d)Fortalecer a articulação entre os órgãos de governo e os
a)Adotar tecnologias sociais de baixo custo e fácil consórcios municipais para atuar na política de saneamento
aplicabilidade nas políticas e ações públicas para a geração de ambiental, com participação da sociedade civil.
renda e para a solução de problemas socioambientais e de Responsáveis: Ministério das Cidades; Ministério do Meio
saúde pública. Ambiente; Secretaria de Relações Institucionais da
Presidência da República

Conhecimentos Específicos 60
Apostila Digital Licenciada para Alice Caroline Guarino dos Santos - alice.guarino@hotmail.com (Proibida a Revenda)
APOSTILAS OPÇÃO

e)Fortalecer a política de coleta, reaproveitamento, de obras e empreendimentos com impactos sociais e


triagem, reciclagem e a destinação seletiva de resíduos sólidos ambientais.
e líquidos, com a organização de cooperativas de reciclagem, Responsáveis: Ministério da Integração Nacional;
que beneficiem as famílias dos catadores. Ministério de Minas e Energia; Secretaria Especial dos Direitos
Responsáveis: Ministério das Cidades; Ministério do Humanos da Presidência da República
Trabalho e Emprego; Ministério do Desenvolvimento Social e f)Definir mecanismos para a garantia dos Direitos
Combate à Fome; Ministério do Meio Ambiente Humanos das populações diretamente atingidas e vizinhas aos
f)Fomentar políticas e ações públicas voltadas à empreendimentos de impactos sociais e ambientais.
mobilidade urbana sustentável. Responsável: Secretaria Especial dos Direitos Humanos
Responsável: Ministério das Cidades da Presidência da República
g)Considerar na elaboração de políticas públicas de g)Apoiar a incorporação dos sindicatos de trabalhadores e
desenvolvimento urbano os impactos na saúde pública. centrais sindicais nos processos de licenciamento ambiental
Responsáveis: Ministério da Saúde; Ministério das de empresas, de forma a garantir o direito à saúde do
Cidades trabalhador.
h)Fomentar políticas públicas de apoio às organizações de Responsáveis: Ministério do Meio Ambiente; Ministério
catadores de materiais recicláveis, visando à disponibilização do Trabalho e Emprego; Ministério da Saúde
de áreas e prédios desocupados pertencentes à União, a fim de h)Promover e fortalecer ações de proteção às populações
serem transformados em infraestrutura produtiva para essas mais pobres da convivência com áreas contaminadas,
organizações. resguardando-as contra essa ameaça e assegurando-lhes seus
Responsáveis: Ministério do Planejamento, Orçamento e direitos fundamentais.
Gestão; Ministério das Cidades; Ministério do Trabalho e Responsáveis: Ministério do Meio Ambiente; Ministério
Emprego; Ministério do Desenvolvimento Social e Combate à das Cidades; Ministério do Desenvolvimento Social e Combate
Fome à Fome; Ministério da Saúde
i)Estimular a produção de alimentos de forma
comunitária, com uso de tecnologias de bases agroecológicas, Objetivo estratégico II:
em espaços urbanos e periurbanos ociosos e fomentar a Afirmação dos princípios da dignidade humana e da
mobilização comunitária para a implementação de hortas, equidade como fundamentos do processo de
viveiros, pomares, canteiros de ervas medicinais, criação de desenvolvimento nacional.
pequenos animais, unidades de processamento e
beneficiamento agroalimentar, feiras e mercados públicos Ações programáticas:
populares. a)Reforçar o papel do Plano Plurianual como instrumento
Responsáveis: Ministério do Desenvolvimento Social e de consolidação dos Direitos Humanos e de enfrentamento da
Combate à Fome; Ministério da Agricultura, Pecuária e concentração de renda e riqueza e de promoção da inclusão da
Abastecimento população de baixa renda.
Responsável: Ministério do Planejamento, Orçamento e
Diretriz 5: Valorização da pessoa humana como sujeito Gestão
central do processo de desenvolvimento. b)Reforçar os critérios da equidade e da prevalência dos
Objetivo estratégico I: Direitos Humanos como prioritários na avaliação da
Garantia da participação e do controle social nas programação orçamentária de ação ou autorização de gastos.
políticas públicas de desenvolvimento com grande Responsável: Ministério do Planejamento, Orçamento e
impacto socioambiental. Gestão
c)Instituir código de conduta em Direitos Humanos para
Ações programáticas: ser considerado no âmbito do poder público como critério
a)Fortalecer ações que valorizem a pessoa humana como para a contratação e financiamento de empresas.
sujeito central do desenvolvimento, enfrentando o quadro Responsável: Secretaria Especial dos Direitos Humanos
atual de injustiça ambiental que atinge principalmente as da Presidência da República
populações mais pobres. d)Regulamentar a taxação do imposto sobre grandes
Responsáveis: Secretaria Especial dos Direitos Humanos fortunas previsto na Constituição.
da Presidência da República; Ministério do Meio Ambiente Responsáveis: Ministério da Fazenda; Secretaria Especial
b)Assegurar participação efetiva da população na dos Direitos Humanos da Presidência da República
elaboração dos instrumentos de gestão territorial e na análise e)Ampliar a adesão de empresas ao compromisso de
e controle dos processos de licenciamento urbanístico e responsabilidade social e Direitos Humanos.
ambiental de empreendimentos de impacto, especialmente na Responsáveis: Secretaria Especial dos Direitos Humanos
definição das ações mitigadoras e compensatórias por da Presidência da República; Ministério do Desenvolvimento,
impactos sociais e ambientais. Indústria e Comércio Exterior
Responsáveis: Ministério do Meio Ambiente; Ministério
das Cidades Objetivo estratégico III:
c)Fomentar a elaboração do Zoneamento Ecológico Fortalecimento dos direitos econômicos por meio de
Econômico (ZEE), incorporando o sócio e etnozoneamento. políticas públicas de defesa da concorrência e de proteção
Responsáveis: Ministério das Cidades; Ministério do Meio do consumidor.
Ambiente
d)Assegurar a transparência dos projetos realizados, em Ações programáticas:
todas as suas etapas, e dos recursos utilizados nos grandes a)Garantir o acesso universal a serviços públicos
projetos econômicos, para viabilizar o controle social. essenciais de qualidade.
Responsáveis: Ministério dos Transportes; Ministério da Responsáveis: Ministério da Saúde; Ministério da
Integração Nacional; Ministério de Minas e Energia; Secretaria Educação; Ministério de Minas e Energia; Ministério do
Especial dos Direitos Humanos da Presidência da República Desenvolvimento Social e Combate à Fome; Ministério das
e)Garantir a exigência de capacitação qualificada e Cidades
participativa das comunidades afetadas nos projetos básicos

Conhecimentos Específicos 61
Apostila Digital Licenciada para Alice Caroline Guarino dos Santos - alice.guarino@hotmail.com (Proibida a Revenda)
APOSTILAS OPÇÃO

b)Fortalecer o sistema brasileiro de defesa da No Brasil, ao longo das últimas décadas, os Direitos
concorrência para coibir condutas anticompetitivas e Humanos passaram a ocupar uma posição de destaque no
concentradoras de renda. ordenamento jurídico. O País avançou decisivamente na
Responsáveis: Ministério da Justiça; Ministério da proteção e promoção do direito às diferenças. Porém, o peso
Fazenda negativo do passado continua a projetar no presente uma
c)Garantir o direito à informação do consumidor, situação de profunda iniquidade social.
fortalecendo as ações de acompanhamento de mercado, O acesso aos direitos fundamentais continua enfrentando
inclusive a rotulagem dos transgênicos. barreiras estruturais, resquícios de um processo histórico, até
Responsáveis: Ministério da Justiça; Ministério do secular, marcado pelo genocídio indígena, pela escravidão e
Desenvolvimento, Indústria e Comércio Exterior; Ministério por períodos ditatoriais, práticas que continuam a ecoar em
da Agricultura, Pecuária e Abastecimento comportamentos, leis e na realidade social.
d)Fortalecer o combate à fraude e a avaliação da O PNDH-3 assimila os grandes avanços conquistados ao
conformidade dos produtos e serviços no mercado. longo destes últimos anos, tanto nas políticas de erradicação
Responsáveis: Ministério da Justiça; Ministério do da miséria e da fome, quanto na preocupação com a moradia e
Desenvolvimento, Indústria e Comércio Exterior saúde, e aponta para a continuidade e ampliação do acesso a
tais políticas, fundamentais para garantir o respeito à
Diretriz 6: Promover e proteger os direitos ambientais dignidade humana.
como Direitos Humanos, incluindo as gerações futuras Os objetivos estratégicos direcionados à promoção da
como sujeitos de direitos. cidadania plena preconizam a universalidade, indivisibilidade
Objetivo estratégico I: e interdependência dos Direitos Humanos, condições para sua
Afirmação dos direitos ambientais como Direitos efetivação integral e igualitária. O acesso aos direitos de
Humanos. registro civil, alimentação adequada, terra e moradia, trabalho
decente, educação, participação política, cultura, lazer, esporte
Ações programáticas: e saúde, deve considerar a pessoa humana em suas múltiplas
a)Incluir o item Direito Ambiental nos relatórios de dimensões de ator social e sujeito de cidadania.
monitoramento dos Direitos Humanos. À luz da história dos movimentos sociais e de programas
Responsáveis: Secretaria Especial dos Direitos Humanos de governo, o PNDH-3 orienta-se pela transversalidade, para
da Presidência da República; Ministério do Meio Ambiente que a implementação dos direitos civis e políticos transitem
b)Incluir o tema dos Direitos Humanos nos instrumentos e pelas diversas dimensões dos direitos econômicos, sociais,
relatórios dos órgãos ambientais. culturais e ambientais. Caso contrário, grupos sociais afetados
Responsáveis: Secretaria Especial dos Direitos Humanos pela pobreza, pelo racismo estrutural e pela discriminação
da Presidência da República; Ministério do Meio Ambiente dificilmente terão acesso a tais direitos.
c)Assegurar a proteção dos direitos ambientais e dos As ações programáticas formuladas visam enfrentar o
Direitos Humanos no Código Florestal. desafio de eliminar as desigualdades, levando em conta as
Responsável: Ministério do Meio Ambiente dimensões de gênero e raça nas políticas públicas, desde o
d)Implementar e ampliar políticas públicas voltadas para planejamento até a sua concretização e avaliação. Há, neste
a recuperação de áreas degradadas e áreas de desmatamento sentido, propostas de criação de indicadores que possam
nas zonas urbanas e rurais. mensurar a efetivação progressiva dos direitos.
Responsáveis: Ministério do Meio Ambiente; Ministério Às desigualdades soma-se a persistência da discriminação,
das Cidades que muitas vezes se manifesta sob a forma de violência contra
e)Fortalecer ações que estabilizem a concentração de sujeitos que são histórica e estruturalmente vulnerabilizados.
gases de efeito estufa em nível que permita a adaptação O combate à discriminação mostra-se necessário, mas
natural dos ecossistemas à mudança do clima, controlando a insuficiente enquanto medida isolada. Os pactos e convenções
interferência das atividades humanas (antrópicas) no sistema que integram o sistema regional e internacional de proteção
climático. dos Direitos Humanos apontam para a necessidade de
Responsável: Ministério do Meio Ambiente combinar estas medidas com políticas compensatórias que
f)Garantir o efetivo acesso a informação sobre a acelerem a construção da igualdade, como forma capaz de
degradação e os riscos ambientais, e ampliar e articular as estimular a inclusão de grupos socialmente vulneráveis. Além
bases de informações dos entes federados e produzir disso, as ações afirmativas constituem medidas especiais e
informativos em linguagem acessível. temporárias que buscam remediar um passado
Responsável: Ministério do Meio Ambiente discriminatório. No rol de movimentos e grupos sociais que
g)Integrar os atores envolvidos no combate ao trabalho demandam políticas de inclusão social encontram-se crianças,
escravo nas operações correntes de fiscalização ao adolescentes, mulheres, pessoas idosas, lésbicas, gays,
desmatamento e ao corte ilegal de madeira. bissexuais, travestis, transexuais, pessoas com deficiência,
Responsáveis: Secretaria Especial dos Direitos Humanos pessoas moradoras de rua, povos indígenas, populações
da Presidência da República; Ministério do Trabalho e negras e quilombolas, ciganos, ribeirinhos, varzanteiros e
Emprego; Ministério do Meio Ambiente pescadores, entre outros.
Definem-se, neste capítulo, medidas e políticas que devem
Eixo Orientador III: ser efetivadas para reconhecer e proteger os indivíduos como
Universalizar direitos em um contexto de iguais na diferença, ou seja, para valorizar a diversidade
desigualdades presente na população brasileira para estabelecer acesso
igualitário aos direitos fundamentais. Trata-se de reforçar os
A Declaração Universal dos Direitos Humanos afirma em programas de governo e as resoluções pactuadas nas diversas
seu preâmbulo que o "reconhecimento da dignidade inerente conferências nacionais temáticas, sempre sob o foco dos
a todos os membros da família humana e de seus direitos Direitos Humanos, com a preocupação de assegurar o respeito
iguais e inalienáveis é o fundamento da liberdade, da justiça e às diferenças e o combate às desigualdades, para o efetivo
da paz no mundo". No entanto, nas vicissitudes ocorridas no acesso aos direitos.
cumprimento da Declaração pelos Estados signatários, Por fim, em respeito à primazia constitucional de proteção
identificou-se a necessidade de reconhecer as diversidades e e promoção da infância, do adolescente e da juventude, o
diferenças para concretização do princípio da igualdade. capítulo aponta suas diretrizes para o respeito e a garantia das

Conhecimentos Específicos 62
Apostila Digital Licenciada para Alice Caroline Guarino dos Santos - alice.guarino@hotmail.com (Proibida a Revenda)
APOSTILAS OPÇÃO

gerações futuras. Como sujeitos de direitos, as crianças, os c)Criar bases normativas e gerenciais para garantia da
adolescentes e os jovens são frequentemente subestimadas universalização do acesso ao registro civil de nascimento e à
em sua participação política e em sua capacidade decisória. documentação básica.
Preconiza-se o dever de assegurar-lhes, desde cedo, o direito - Implantar sistema nacional de registro civil para
de opinião e participação. interligação das informações de estimativas de nascimentos,
Marcadas pelas diferenças e por sua fragilidade temporal, de nascidos vivos e do registro civil, a fim de viabilizar a busca
as crianças, os adolescentes e os jovens estão sujeitos a ativa dos nascidos não registrados e aperfeiçoar os
discriminações e violências. As ações programáticas indicadores para subsidiar políticas públicas.
promovem a garantia de espaços e investimentos que - Desenvolver estudo e revisão da legislação para garantir
assegurem proteção contra qualquer forma de violência e o acesso do cidadão ao registro civil de nascimento em todo o
discriminação, bem como a promoção da articulação entre território nacional.
família, sociedade e Estado para fortalecer a rede social de - Realizar estudo de sustentabilidade do serviço notarial e
proteção que garante a efetividade de seus direitos. de registro no País.
- Desenvolver a padronização do registro civil (certidão de
Diretriz 7: Garantia dos Direitos Humanos de forma nascimento, de casamento e de óbito) em território nacional.
universal, indivisível e interdependente, assegurando a - Garantir a emissão gratuita de Registro Geral e Cadastro
cidadania plena. de Pessoa Física aos reconhecidamente pobres.
- Desenvolver estudo sobre a política nacional de
Objetivo estratégico I: documentação civil básica.
Universalização do registro civil de nascimento e Responsáveis: Ministério da Saúde; Ministério do
ampliação do acesso à documentação básica. Desenvolvimento Social e Combate à Fome; Ministério do
Planejamento, Orçamento e Gestão; Ministério da Fazenda;
Ações programáticas: Ministério da Justiça; Ministério do Trabalho e Emprego;
a)Ampliar e reestruturar a rede de atendimento para a Ministério da Previdência Social; Secretaria Especial dos
emissão do registro civil de nascimento visando a sua Direitos Humanos da Presidência da República
universalização. d)Incluir no questionário do censo demográfico perguntas
- Interligar maternidades e unidades de saúde aos para identificar a ausência de documentos civis na população.
cartórios, por meio de sistema manual ou informatizado, para Responsável: Secretaria Especial dos Direitos Humanos
emissão de registro civil de nascimento logo após o parto, da Presidência da República
garantindo ao recém nascido a certidão de nascimento antes
da alta médica. Objetivo estratégico II:
- Fortalecer a Declaração de Nascido Vivo (DNV), emitida Acesso à alimentação adequada por meio de políticas
pelo Sistema Único de Saúde , como mecanismo de acesso ao estruturantes.
registro civil de nascimento, contemplando a diversidade na
emissão pelos estabelecimentos de saúde e pelas parteiras. Ações programáticas:
- Realizar orientação sobre a importância do registro civil a)Ampliar o acesso aos alimentos por meio de programas
de nascimento para a cidadania por meio da rede de e ações de geração e transferência de renda, com ênfase na
atendimento (saúde, educação e assistência social) e pelo participação das mulheres como potenciais beneficiárias.
sistema de Justiça e de segurança pública. Responsáveis: Ministério do Desenvolvimento Social e
- Aperfeiçoar as normas e o serviço público notarial e de Combate à Fome; Secretaria Especial de Políticas para as
registro, em articulação com o Conselho Nacional de Justiça, Mulheres da Presidência da República
para garantia da gratuidade e da cobertura do serviço de b)Vincular programas de transferência de renda à garantia
registro civil em âmbito nacional. da segurança alimentar da criança, por meio do
Responsáveis: Ministério da Saúde; Ministério do acompanhamento da saúde e nutrição e do estímulo de hábitos
Desenvolvimento Social e Combate à Fome; Ministério da alimentares saudáveis, com o objetivo de erradicar a
Previdência Social; Ministério da Justiça; Ministério do desnutrição infantil.
Planejamento, Orçamento e Gestão; Secretaria Especial dos Responsáveis: Ministério do Desenvolvimento Social e
Direitos Humanos da Presidência da República Combate à Fome; Ministério da Educação; Ministério da Saúde
b)Promover a mobilização nacional com intuito de reduzir c)Fortalecer a agricultura familiar e camponesa no
o número de pessoas sem registro civil de nascimento e desenvolvimento de ações específicas que promovam a
documentação básica. geração de renda no campo e o aumento da produção de
- Instituir comitês gestores estaduais, distrital e municipais alimentos agroecológicos para o autoconsumo e para o
com o objetivo de articular as instituições públicas e as mercado local.
entidades da sociedade civil para a implantação de ações que Responsáveis: Ministério do Desenvolvimento Agrário;
visem à ampliação do acesso à documentação básica. Ministério do Desenvolvimento Social e Combate à Fome
- Realizar campanhas para orientação e conscientização da d)Ampliar o abastecimento alimentar, com maior
população e dos agentes responsáveis pela articulação e pela autonomia e fortalecimento da economia local, associado a
garantia do acesso aos serviços de emissão de registro civil de programas de informação, de educação alimentar, de
nascimento e de documentação básica. capacitação, de geração de ocupações produtivas, de
- Realizar mutirões para emissão de registro civil de agricultura familiar camponesa e de agricultura urbana.
nascimento e documentação básica, com foco nas regiões de Responsáveis: Ministério do Desenvolvimento Social e
difícil acesso e no atendimento às populações específicas como Combate à Fome; Ministério da Agricultura, Pecuária e
os povos indígenas, quilombolas, ciganos, pessoas em situação Abastecimento; Ministério do Desenvolvimento Agrário
de rua, institucionalizadas e às trabalhadoras rurais. e)Promover a implantação de equipamentos públicos de
Responsáveis: Ministério da Saúde; Ministério do segurança alimentar e nutricional, com vistas a ampliar o
Desenvolvimento Social e Combate à Fome; Ministério da acesso à alimentação saudável de baixo custo, valorizar as
Defesa; Ministério da Fazenda; Ministério do Trabalho e culturas alimentares regionais, estimular o aproveitamento
Emprego; Ministério da Justiça; Secretaria Especial dos integral dos alimentos, evitar o desperdício e contribuir para a
Direitos Humanos da Presidência da República recuperação social e de saúde da sociedade.

Conhecimentos Específicos 63
Apostila Digital Licenciada para Alice Caroline Guarino dos Santos - alice.guarino@hotmail.com (Proibida a Revenda)
APOSTILAS OPÇÃO

Responsável: Ministério do Desenvolvimento Social e i)Estabelecer que a garantia da qualidade de abrigos e


Combate à Fome albergues, bem como seu caráter inclusivo e de resgate da
f)Garantir que os hábitos e contextos regionais sejam cidadania à população em situação de rua, estejam entre os
incorporados nos modelos de segurança alimentar como critérios de concessão de recursos para novas construções e
fatores da produção sustentável de alimentos. manutenção dos existentes.
Responsável: Ministério do Desenvolvimento Social e Responsáveis: Ministério das Cidades; Ministério do
Combate à Fome Desenvolvimento Social e Combate à Fome
g)Realizar pesquisas científicas que promovam ganhos de j)Apoiar o monitoramento de políticas de habitação de
produtividade na agricultura familiar e assegurar estoques interesse social pelos conselhos municipais de habitação,
reguladores. garantindo às cooperativas e associações habitacionais acesso
Responsáveis: Ministério do Desenvolvimento Social e às informações.
Combate à Fome; Ministério do Desenvolvimento Agrário; Responsável: Ministério das Cidades
Ministério da Agricultura, Pecuária e Abastecimento k)Garantir as condições para a realização de
acampamentos ciganos em todo o território nacional, visando
Objetivo estratégico III: a preservação de suas tradições, práticas e patrimônio
Garantia do acesso à terra e à moradia para a cultural.
população de baixa renda e grupos sociais Responsáveis: Secretaria Especial dos Direitos Humanos
vulnerabilizados. da Presidência da República; Ministério das Cidades

Ações programáticas: Objetivo estratégico IV:


a)Fortalecer a reforma agrária com prioridade à Ampliação do acesso universal a sistema de saúde de
implementação e recuperação de assentamentos, à qualidade.
regularização do crédito fundiário e à assistência técnica aos
assentados, atualização dos índices Grau de Utilização da Ações programáticas:
Terra (GUT) e Grau de Eficiência na Exploração (GEE), a)Expandir e consolidar programas de serviços básicos de
conforme padrões atuais e regulamentação da desapropriação saúde e de atendimento domiciliar para a população de baixa
de áreas pelo descumprimento da função social plena. renda, com enfoque na prevenção e diagnóstico prévio de
Responsável: Ministério do Desenvolvimento Agrário; doenças e deficiências, com apoio diferenciado às pessoas
Ministério da Agricultura, Pecuária e Abastecimento idosas, indígenas, negros e comunidades quilombolas, pessoas
b)Integrar as ações de mapeamento das terras públicas da com deficiência, pessoas em situação de rua, lésbicas, gays,
União. bissexuais, travestis, transexuais, crianças e adolescentes,
Responsável: Ministério do Planejamento, Orçamento e mulheres, pescadores artesanais e população de baixa renda.
Gestão Responsáveis: Ministério da Saúde; Secretaria Especial de
c)Estimular o saneamento dos serviços notariais de Políticas de Promoção da Igualdade Racial da Presidência da
registros imobiliários, possibilitando o bloqueio ou o República; Secretaria Especial de Políticas para as Mulheres da
cancelamento administrativo dos títulos das terras e registros Presidência da República; Ministério da Pesca e Aquicultura
irregulares. b)Criar programas de pesquisa e divulgação sobre
Responsáveis: Ministério da Justiça; Ministério do tratamentos alternativos à medicina tradicional no sistema de
Desenvolvimento Agrário saúde.
d)Garantir demarcação, homologação, regularização e Responsável: Ministério da Saúde
desintrusão das terras indígenas, em harmonia com os c)Reformular o marco regulatório dos planos de saúde, de
projetos de futuro de cada povo indígena, assegurando seu modo a diminuir os custos para a pessoa idosa e fortalecer o
etnodesenvolvimento e sua autonomia produtiva. pacto intergeracional, estimulando a adoção de medidas de
Responsável: Ministério da Justiça capitalização para gastos futuros pelos planos de saúde.
e)Assegurar às comunidades quilombolas a posse dos seus Responsável: Ministério da Saúde
territórios, acelerando a identificação, o reconhecimento, a d)Reconhecer as parteiras como agentes comunitárias de
demarcação e a titulação desses territórios, respeitando e saúde.
preservando os sítios de valor simbólico e histórico. Responsáveis: Ministério da Saúde; Secretaria Especial de
Responsáveis: Secretaria Especial de Políticas de Políticas para as Mulheres da Presidência da República
Promoção da Igualdade Racial da Presidência da República; e)Aperfeiçoar o programa de saúde para adolescentes,
Ministério da Cultura; Ministério do Desenvolvimento Agrário especificamente quanto à saúde de gênero, à educação sexual
f)Garantir o acesso a terra às populações ribeirinhas, e reprodutiva e à saúde mental.
varzanteiras e pescadoras, assegurando acesso aos recursos Responsáveis: Ministério da Saúde; Secretaria Especial de
naturais que tradicionalmente utilizam para sua reprodução Políticas para as Mulheres da Presidência da República;
física, cultural e econômica. Secretaria Especial dos Direitos Humanos da Presidência da
Responsáveis: Ministério do Desenvolvimento Agrário; República
Ministério do Meio Ambiente f)Criar campanhas e material técnico, instrucional e
g)Garantir que nos programas habitacionais do governo educativo sobre planejamento reprodutivo que respeite os
sejam priorizadas as populações de baixa renda, a população direitos sexuais e reprodutivos, contemplando a elaboração de
em situação de rua e grupos sociais em situação de materiais específicos para a população jovem e adolescente e
vulnerabilidade no espaço urbano e rural, considerando os para pessoas com deficiência.
princípios da moradia digna, do desenho universal e os Responsáveis: Ministério da Saúde; Secretaria Especial de
critérios de acessibilidade nos projetos. Políticas para as Mulheres da Presidência da República;
Responsáveis: Ministério das Cidades; Ministério do Secretaria Especial dos Direitos Humanos da Presidência da
Desenvolvimento Social e Combate à Fome República
h)Promover a destinação das glebas e edifícios vazios ou g)Estimular programas de atenção integral à saúde das
subutilizados pertencentes à União, para a população de baixa mulheres, considerando suas especificidades étnico-raciais,
renda, reduzindo o déficit habitacional. geracionais, regionais, de orientação sexual, de pessoa com
Responsáveis: Ministério das Cidades; Ministério do deficiência, priorizando as moradoras do campo, da floresta e
Planejamento, Orçamento e Gestão em situação de rua.

Conhecimentos Específicos 64
Apostila Digital Licenciada para Alice Caroline Guarino dos Santos - alice.guarino@hotmail.com (Proibida a Revenda)
APOSTILAS OPÇÃO

Responsáveis: Ministério da Saúde; Secretaria Especial de t)Implementar medidas destinadas a desburocratizar os


Políticas para as Mulheres da Presidência da República; serviços do Instituto Nacional de Seguro Social para a
Secretaria Especial de Políticas de Promoção da Igualdade concessão de aposentadorias e benefícios.
Racial da Presidência da República; Ministério do Responsável: Ministério da Previdência Social
Desenvolvimento Social e Combate à Fome u)Estimular a incorporação do trabalhador urbano e rural
h)Ampliar e disseminar políticas de saúde pré e neonatal, ao regime geral da previdência social.
com inclusão de campanhas educacionais de esclarecimento, Responsável: Ministério da Previdência Social
visando à prevenção do surgimento ou do agravamento de v)Assegurar a inserção social das pessoas atingidas pela
deficiências. hanseníase isoladas e internadas em hospitais-colônias.
Responsáveis: Ministério da Saúde; Secretaria Especial de Responsáveis: Secretaria Especial dos Direitos Humanos
Políticas para as Mulheres da Presidência da República; da Presidência da República; Ministério da Saúde
Secretaria Especial dos Direitos Humanos da Presidência da w)Reconhecer, pelo Estado brasileiro, as violações de
República direitos às pessoas atingidas pela hanseníase no período da
i)Expandir a assistência pré-natal e pós-natal por meio de internação e do isolamento compulsórios, apoiando iniciativas
programas de visitas domiciliares para acompanhamento das para agilizar as reparações com a concessão de pensão
crianças na primeira infância. especial prevista na Lei no11.520/2007.
Responsável: Ministério da Saúde Responsável: Secretaria Especial dos Direitos Humanos
j)Apoiar e financiar a realização de pesquisas e da Presidência da República
intervenções sobre a mortalidade materna, contemplando o x)Proporcionar as condições necessárias para conclusão
recorte étnico-racial e regional. do trabalho da Comissão Interministerial de Avaliação para
Responsáveis: Ministério da Saúde; Secretaria Especial de análise dos requerimentos de pensão especial das pessoas
Políticas para as Mulheres da Presidência da República atingidas pela hanseníase, que foram internadas e isoladas
k)Assegurar o acesso a laqueaduras e vasectomias ou compulsoriamente em hospitais-colônia até 31 de dezembro
reversão desses procedimentos no sistema público de saúde, de 1986.
com garantia de acesso a informações sobre as escolhas Responsável: Secretaria Especial dos Direitos Humanos
individuais. da Presidência da República
Responsáveis: Ministério da Saúde; Secretaria Especial de
Políticas para as Mulheres da Presidência da República Objetivo estratégico V:
l)Ampliar a oferta de medicamentos de uso contínuo, Acesso à educação de qualidade e garantia de
especiais e excepcionais, para a pessoa idosa. permanência na escola.
Responsável: Ministério da Saúde
m)Realizar campanhas de diagnóstico precoce e Ações programáticas:
tratamento adequado às pessoas que vivem com HIV/AIDS a)Ampliar o acesso a educação básica, a permanência na
para evitar o estágio grave da doença e prevenir sua expansão escola e a universalização do ensino no atendimento à
e disseminação. educação infantil.
Responsável: Ministério da Saúde Responsável: Ministério da Educação
n)Proporcionar às pessoas que vivem com HIV/AIDS b)Assegurar a qualidade do ensino formal público com seu
programas de atenção no âmbito da saúde sexual e monitoramento contínuo e atualização curricular.
reprodutiva. Responsáveis: Ministério da Educação; Secretaria
Responsáveis: Ministério da Saúde; Secretaria Especial de Especial dos Direitos Humanos da Presidência da República
Políticas para as Mulheres da Presidência da República c)Desenvolver programas para a reestruturação das
o)Capacitar os agentes comunitários de saúde que escolas como pólos de integração de políticas educacionais,
realizam a triagem e a captação nas hemorredes para culturais e de esporte e lazer.
praticarem abordagens sem preconceito e sem discriminação. Responsáveis: Ministério da Educação; Ministério da
Responsáveis: Ministério da Saúde; Secretaria Especial Cultura; Ministério do Esporte
dos Direitos Humanos da Presidência da República d)Apoiar projetos e experiências de integração da escola
p)Garantir o acompanhamento multiprofissional a pessoas com a comunidade que utilizem sistema de alternância.
transexuais que fazem parte do processo transexualizador no Responsável: Ministério da Educação
Sistema Único de Saúde e de suas famílias. e)Adequar o currículo escolar, inserindo conteúdos que
Responsáveis: Ministério da Saúde; Secretaria Especial valorizem as diversidades, as práticas artísticas, a necessidade
dos Direitos Humanos da Presidência da República de alimentação adequada e saudável e as atividades físicas e
q)Apoiar o acesso a programas de saúde preventiva e de esportivas.
proteção à saúde para profissionais do sexo. Responsáveis: Ministério da Educação; Ministério da
Responsáveis: Ministério da Saúde; Secretaria Especial de Cultura; Ministério do Esporte; Ministério da Saúde
Políticas para as Mulheres da Presidência da República f)Integrar os programas de alfabetização de jovens e
r)Apoiar a implementação de espaços essenciais para adultos aos programas de qualificação profissional e educação
higiene pessoal e centros de referência para a população em cidadã, apoiando e incentivando a utilização de metodologias
situação de rua. adequadas às realidades dos povos e comunidades
Responsáveis: Secretaria Especial dos Direitos Humanos tradicionais.
da Presidência da República; Ministério do Desenvolvimento Responsáveis: Ministério da Educação; Ministério do
Social e Combate à Fome Desenvolvimento Social e Combate à Fome; Ministério do
s)Investir na política de reforma psiquiátrica fomentando Trabalho e Emprego; Ministério da Pesca e Aquicultura
programas de tratamentos substitutivos à internação, que g)Estimular e financiar programas de extensão
garantam às pessoas com transtorno mental a possibilidade de universitária como forma de integrar o estudante à realidade
escolha autônoma de tratamento, com convivência familiar e social.
acesso aos recursos psiquiátricos e farmacológicos. Responsável: Ministério da Educação
Responsáveis: Ministério da Saúde; Secretaria Especial h)Fomentar as ações afirmativas para o ingresso das
dos Direitos Humanos da Presidência da República; Ministério populações negra, indígena e de baixa renda no ensino
da Cultura superior.

Conhecimentos Específicos 65
Apostila Digital Licenciada para Alice Caroline Guarino dos Santos - alice.guarino@hotmail.com (Proibida a Revenda)
APOSTILAS OPÇÃO

Responsáveis: Ministério da Educação; Secretaria h)Acompanhar a implementação do Programa Nacional de


Especial de Políticas de Promoção da Igualdade Racial da Ações Afirmativas, instituído pelo Decreto no 4.228/2002, no
Presidência da República; Ministério da Justiça âmbito da administração pública federal, direta e indireta, com
i)Ampliar o ensino superior público de qualidade por meio vistas à realização de metas percentuais da ocupação de cargos
da criação permanente de universidades federais, cursos e comissionados pelas mulheres, população negra e pessoas
vagas para docentes e discentes. com deficiência.
Responsável: Ministério da Educação Responsável: Secretaria Especial dos Direitos Humanos
j)Fortalecer as iniciativas de educação popular por meio da da Presidência da República
valorização da arte e da cultura, apoiando a realização de i)Realizar campanhas envolvendo a sociedade civil
festivais nas comunidades tradicionais e valorizando as organizada sobre paternidade responsável, bem como ampliar
diversas expressões artísticas nas escolas e nas comunidades. a licença-paternidade, como forma de contribuir para a
Responsáveis: Ministério da Educação; Ministério da corresponsabilidade e para o combate ao preconceito quanto
Cultura; Secretaria Especial dos Direitos Humanos da à inserção das mulheres no mercado de trabalho.
Presidência da República Responsáveis: Secretaria Especial de Políticas para as
k)Ampliar o acesso a programas de inclusão digital para Mulheres da Presidência da República; Ministério do Trabalho
populações de baixa renda em espaços públicos, e Emprego
especialmente escolas, bibliotecas e centros comunitários. j)Elaborar diagnósticos com base em ações judiciais que
Responsáveis: Ministério da Educação; Ministério da envolvam atos de assédio moral, sexual e psicológico, com
Cultura; Ministério da Ciência e Tecnologia; Ministério da apuração de denúncias de desrespeito aos direitos das
Pesca e Aquicultura trabalhadoras e trabalhadores, visando orientar ações de
l)Fortalecer programas de educação no campo e nas combate à discriminação e abuso nas relações de trabalho.
comunidades pesqueiras que estimulem a permanência dos Responsáveis: Ministério do Trabalho e Emprego;
estudantes na comunidade e que sejam adequados às Secretaria Especial de Políticas de Promoção da Igualdade
respectivas culturas e identidades. Racial da Presidência da República; Secretaria Especial de
Responsáveis: Ministério da Educação; Ministério do Políticas para as Mulheres da Presidência da República;
Desenvolvimento Agrário; Ministério da Pesca e Aquicultura Secretaria Especial dos Direitos Humanos da Presidência da
República
Objetivo estratégico VI: k)Garantir a igualdade de direitos das trabalhadoras e
Garantia do trabalho decente, adequadamente trabalhadores domésticos com os dos demais trabalhadores.
remunerado, exercido em condições de equidade e Responsáveis: Ministério do Trabalho e Emprego;
segurança. Secretaria Especial de Políticas para as Mulheres da
Presidência da República; Ministério da Previdência Social
Ações programáticas: l)Promover incentivos a empresas para que empreguem os
a)Apoiar a agenda nacional de trabalho decente por meio egressos do sistema penitenciário.
do fortalecimento do seu comitê executivo e da efetivação de Responsáveis: Ministério da Fazenda; Ministério do
suas ações. Trabalho e Emprego; Ministério da Justiça
Responsável: Ministério do Trabalho e Emprego m)Criar cadastro nacional e relatório periódico de
b)Fortalecer programas de geração de emprego, empregabilidade de egressos do sistema penitenciário.
ampliando progressivamente o nível de ocupação e Responsável: Ministério da Justiça
priorizando a população de baixa renda e os Estados com n)Garantir os direitos trabalhistas e previdenciários de
elevados índices de emigração. profissionais do sexo por meio da regulamentação de sua
Responsável: Ministério do Trabalho e Emprego profissão.
c)Ampliar programas de economia solidária, mediante Responsáveis: Ministério do Trabalho e Emprego;
políticas integradas, como alternativa de geração de trabalho Secretaria Especial de Políticas para as Mulheres da
e renda, e de inclusão social, priorizando os jovens das famílias Presidência da República.
beneficiárias do Programa Bolsa Família.
Responsáveis: Ministério do Trabalho e Emprego; Objetivo estratégico VII:
Ministério do Desenvolvimento Social e Combate à Fome Combate e prevenção ao trabalho escravo.
d)Criar programas de formação, qualificação e inserção
profissional e de geração de emprego e renda para jovens, Ações programáticas:
população em situação de rua e população de baixa renda. a)Promover a efetivação do Plano Nacional para
Responsáveis: Ministério do Trabalho e Emprego; Erradicação do Trabalho Escravo.
Ministério do Desenvolvimento Social e Combate à Fome; Responsáveis: Ministério do Trabalho e Emprego;
Ministério da Educação Secretaria Especial dos Direitos Humanos da Presidência da
e)Integrar as ações de qualificação profissional às República
atividades produtivas executadas com recursos públicos, b)Apoiar a coordenação e implementação de planos
como forma de garantir a inserção no mercado de trabalho. estaduais, distrital e municipais para erradicação do trabalho
Responsáveis: Ministério do Trabalho e Emprego; escravo.
Ministério do Desenvolvimento Social e Combate à Fome Responsável: Secretaria Especial dos Direitos Humanos
f)Criar programas de formação e qualificação profissional da Presidência da República
para pescadores artesanais, industriais e aquicultores c)Monitorar e articular o trabalho das comissões estaduais,
familiares. distrital e municipais para a erradicação do trabalho escravo.
Responsáveis: Ministério do Trabalho e Emprego; Responsáveis: Ministério do Trabalho e Emprego;
Ministério da Pesca e Aquicultura Secretaria Especial dos Direitos Humanos da Presidência da
g)Combater as desigualdades salariais baseadas em República
diferenças de gênero, raça, etnia e das pessoas com deficiência. d)Apoiar a alteração da Constituição para prever a
Responsáveis: Ministério do Trabalho e Emprego; expropriação dos imóveis rurais e urbanos nos quais forem
Secretaria Especial dos Direitos Humanos da Presidência da encontrados trabalhadores reduzidos à condição análoga a de
República escravos.

Conhecimentos Específicos 66
Apostila Digital Licenciada para Alice Caroline Guarino dos Santos - alice.guarino@hotmail.com (Proibida a Revenda)
APOSTILAS OPÇÃO

Responsáveis: Ministério do Trabalho e Emprego; i)Fortalecer e ampliar programas que contemplem


Secretaria de Relações Institucionais da Presidência da participação dos idosos nas atividades de esporte e lazer.
República; Secretaria Especial dos Direitos Humanos da Responsáveis: Ministério do Esporte; Secretaria Especial
Presidência da República dos Direitos Humanos da Presidência da República
e)Identificar periodicamente as atividades produtivas em j)Potencializar ações de incentivo ao turismo para pessoas
que há ocorrência de trabalho escravo adulto e infantil. idosas.
Responsáveis: Ministério do Trabalho e Emprego; Responsáveis: Ministério do Turismo; Secretaria Especial
Secretaria Especial dos Direitos Humanos da Presidência da dos Direitos Humanos da Presidência da República
República
f)Propor marco legal e ações repressivas para erradicar a Objetivo estratégico IX:
intermediação ilegal de mão de obra. Garantia da participação igualitária e acessível na vida
Responsáveis: Ministério do Trabalho e Emprego; política.
Secretaria Especial dos Direitos Humanos da Presidência da
República; Secretaria Especial de Políticas de Promoção da Ações programáticas:
Igualdade Racial a)Apoiar campanhas para promover a ampla divulgação do
g)Promover a destinação de recursos do Fundo de Amparo direito ao voto e participação política de homens e mulheres,
ao Trabalhador (FAT) para capacitação técnica e por meio de campanhas informativas que garantam a escolha
profissionalizante de trabalhadores rurais e de povos e livre e consciente.
comunidades tradicionais, como medida preventiva ao Responsáveis: Ministério da Justiça; Secretaria Especial
trabalho escravo, assim como para implementação de política de Políticas para as Mulheres da Presidência da República
de reinserção social dos libertados da condição de trabalho b)Apoiar o combate ao crime de captação ilícita de
escravo. sufrágio, inclusive com campanhas de esclarecimento e
Responsáveis: Ministério do Trabalho e Emprego; conscientização dos eleitores.
Secretaria Especial dos Direitos Humanos da Presidência da Responsável: Ministério da Justiça
República c)Apoiar os projetos legislativos para o financiamento
h)Atualizar e divulgar semestralmente o cadastro de público de campanhas eleitorais.
empregadores que utilizaram mão-de-obra escrava. Responsável: Ministério da Justiça
Responsáveis: Ministério do Trabalho e Emprego; d)Garantir acesso irrestrito às zonas eleitorais por meio de
Secretaria Especial dos Direitos Humanos da Presidência da transporte público e acessível e apoiar a criação de zonas
República eleitorais em áreas de difícil acesso.
Responsáveis: Ministério da Justiça; Ministério das
Objetivo estratégico VIII: Cidades
Promoção do direito à cultura, lazer e esporte como e)Promover junto aos povos indígenas ações de educação
elementos formadores de cidadania. e capacitação sobre o sistema político brasileiro.
Responsável: Ministério da Justiça
Ações programáticas: f)Apoiar ações de formação política das mulheres em sua
a)Ampliar programas de cultura que tenham por diversidade étnico-racial, estimulando candidaturas e votos de
finalidade planejar e implementar políticas públicas para a mulheres em todos os níveis.
proteção e promoção da diversidade cultural brasileira, em Responsável: Secretaria Especial de Políticas para as
formatos acessíveis. Mulheres da Presidência da República
Responsáveis: Ministério da Cultura; Ministério do g)Garantir e estimular a plena participação das pessoas
Esporte com deficiência no ato do sufrágio, seja como eleitor ou
b)Elaborar programas e ações de cultura que considerem candidato, assegurando os mecanismos de acessibilidade
os formatos acessíveis, as demandas e as características necessários, inclusive a modalidade do voto assistido.
específicas das diferentes faixas etárias e dos grupos sociais. Responsável: Secretaria Especial dos Direitos Humanos
Responsável: Ministério da Cultura da Presidência da República
c)Fomentar políticas públicas de esporte e lazer,
considerando as diversidades locais, de forma a atender a Diretriz 8: Promoção dos direitos de crianças e
todas as faixas etárias e aos grupos sociais. adolescentes para o seu desenvolvimento integral, de
Responsável: Ministério do Esporte forma não discriminatória, assegurando seu direito de
d)Elaborar inventário das línguas faladas no Brasil. opinião e participação.
Responsável: Ministério da Cultura
e)Ampliar e desconcentrar os pólos culturais e pontos de Objetivo estratégico I:
cultura para garantir o acesso das populações de regiões Proteger e garantir os direitos de crianças e
periféricas e de baixa renda. adolescentes por meio da consolidação das diretrizes
Responsável: Ministério da Cultura nacionais do ECA, da Política Nacional de Promoção,
f)Fomentar políticas públicas de formação em esporte e Proteção e Defesa dos Direitos da Criança e do
lazer, com foco na intersetorialidade, na ação comunitária na Adolescente e da Convenção sobre os Direitos da Criança
intergeracionalidade e na diversidade cultural. da ONU.
Responsável: Ministério do Esporte
g)Ampliar o desenvolvimento de programas de produção Ações programáticas:
audiovisual, musical e artesanal dos povos indígenas. a)Formular plano de médio prazo e decenal para a política
Responsáveis: Ministério da Cultura; Ministério da Justiça nacional de promoção, proteção e defesa dos direitos da
h)Assegurar o direito das pessoas com deficiência e em criança e do adolescente.
sofrimento mental de participarem da vida cultural em Responsável: Secretaria Especial dos Direitos Humanos
igualdade de oportunidade com as demais, e de desenvolver e da Presidência da República
utilizar o seu potencial criativo, artístico e intelectual. b)Desenvolver e implementar metodologias de
Responsáveis: Ministério do Esporte; Ministério da acompanhamento e avaliação das políticas e planos nacionais
Cultura; Secretaria Especial dos Direitos Humanos da referentes aos direitos de crianças e adolescentes.
Presidência da República

Conhecimentos Específicos 67
Apostila Digital Licenciada para Alice Caroline Guarino dos Santos - alice.guarino@hotmail.com (Proibida a Revenda)
APOSTILAS OPÇÃO

Responsável: Secretaria Especial dos Direitos Humanos Objetivo estratégico III:


da Presidência da República Proteger e defender os direitos de crianças e
c)Elaborar e implantar sistema de coordenação da política adolescentes com maior vulnerabilidade.
dos direitos da criança e do adolescente em todos os níveis de
governo, para atender às recomendações do Comitê sobre Ações programáticas:
Direitos da Criança, dos relatores especiais e do Comitê sobre a)Promover ações educativas para erradicação da
Direitos Econômicos, Sociais e Culturais da ONU. violência na família, na escola, nas instituições e na
Responsáveis: Secretaria Especial dos Direitos Humanos comunidade em geral, implementando as recomendações
da Presidência da República; Ministério das Relações expressas no Relatório Mundial de Violência contra a Criança
Exteriores da ONU.
d)Criar sistema nacional de coleta de dados e Responsável: Secretaria Especial dos Direitos Humanos
monitoramento junto aos Municípios, Estados e Distrito da Presidência da República
Federal acerca do cumprimento das obrigações da Convenção b)Desenvolver programas nas redes de assistência social,
dos Direitos da Criança da ONU. de educação e de saúde para o fortalecimento do papel das
Responsável: Secretaria Especial dos Direitos Humanos famílias em relação ao desenvolvimento infantil e à disciplina
da Presidência da República não violenta.
e)Assegurar a opinião das crianças e dos adolescentes que Responsáveis: Secretaria Especial dos Direitos Humanos
estiverem capacitados a formular seus próprios juízos, da Presidência da República; Ministério da Educação;
conforme o disposto no artigo 12 da Convenção sobre os Ministério do Desenvolvimento Social e Combate à Fome;
Direitos da Criança, na formulação das políticas públicas Ministério da Saúde
voltadas para estes segmentos, garantindo sua participação c)Propor marco legal para a abolição das práticas de
nas conferências dos direitos das crianças e dos adolescentes. castigos físicos e corporais contra crianças e adolescentes.
Responsável: Secretaria Especial dos Direitos Humanos Responsáveis: Secretaria Especial dos Direitos Humanos
da Presidência da República da Presidência da República; Ministério da Justiça
d)Implantar sistema nacional de registro de ocorrência de
Objetivo estratégico II: violência escolar, incluindo as práticas de violência gratuita e
Consolidar o Sistema de Garantia de Direitos de reiterada entre estudantes (bullying), adotando formulário
Crianças e Adolescentes, com o fortalecimento do papel unificado de registro a ser utilizado por todas as escolas.
dos Conselhos Tutelares e de Direitos. Responsável: Ministério da Educação
e)Apoiar iniciativas comunitárias de mobilização de
Ações programáticas: crianças e adolescentes em estratégias preventivas, com vistas
a)Apoiar a universalização dos Conselhos Tutelares e de a minimizar sua vulnerabilidade em contextos de violência.
Direitos em todos os Municípios e no Distrito Federal, e Responsáveis: Secretaria Especial dos Direitos Humanos
instituir parâmetros nacionais que orientem o seu da Presidência da República; Ministério da Justiça; Ministério
funcionamento. do Esporte; Ministério do Turismo
Responsável: Secretaria Especial dos Direitos Humanos f)Extinguir os grandes abrigos e eliminar a longa
da Presidência da República permanência de crianças e adolescentes em abrigamento,
b)Implantar escolas de conselhos nos Estados e no Distrito adequando os serviços de acolhimento aos parâmetros
Federal, com vistas a apoiar a estruturação e qualificação da aprovados pelo CONANDA e CNAS.
ação dos Conselhos Tutelares e de Direitos. Responsável: Ministério do Desenvolvimento Social e
Responsável: Secretaria Especial dos Direitos Humanos Combate à Fome
da Presidência da República g)Fortalecer as políticas de apoio às famílias para a
c)Apoiar a capacitação dos operadores do sistema de redução dos índices de abandono e institucionalização, com
garantia dos direitos para a proteção dos direitos e promoção prioridade aos grupos familiares de crianças com deficiências.
do modo de vida das crianças e adolescentes indígenas, Responsáveis: Secretaria Especial dos Direitos Humanos
afrodescendentes e comunidades tradicionais, contemplando da Presidência da República; Ministério do Desenvolvimento
ainda as especificidades da população infanto-juvenil com Social e Combate à Fome
deficiência. h)Ampliar a oferta de programas de famílias acolhedoras
Responsáveis: Secretaria Especial dos Direitos Humanos para crianças e adolescentes em situação de violência, com o
da Presidência da República; Ministério da Justiça objetivo de garantir que esta seja a única opção para crianças
d)Fomentar a criação de instâncias especializadas e retiradas do convívio com sua família de origem na primeira
regionalizadas do sistema de justiça, de segurança e infância.
defensorias públicas, para atendimento de crianças e Responsável: Ministério do Desenvolvimento Social e
adolescentes vítimas e autores de violência. Combate à Fome
Responsáveis: Secretaria Especial dos Direitos Humanos i)Estruturar programas de moradia coletivas para
da Presidência da República; Ministério da Justiça adolescentes e jovens egressos de abrigos institucionais.
e)Desenvolver mecanismos que viabilizem a participação Responsável: Ministério do Desenvolvimento Social e
de crianças e adolescentes no processo das conferências dos Combate à Fome
direitos, nos conselhos de direitos, bem como nas escolas, nos j)Fomentar a adoção legal, por meio de campanhas
tribunais e nos procedimentos judiciais e administrativos que educativas, em consonância com o ECA e com acordos
os afetem. internacionais.
Responsável: Secretaria Especial dos Direitos Humanos Responsáveis: Secretaria Especial dos Direitos Humanos
da Presidência da República da Presidência da República; Ministério das Relações
f)Estimular a informação às crianças e aos adolescentes Exteriores
sobre seus direitos, por meio de esforços conjuntos na escola, k)Criar serviços e aprimorar metodologias para
na mídia impressa, na televisão, no rádio e na Internet. identificação e localização de crianças e adolescentes
Responsáveis: Secretaria Especial dos Direitos Humanos desaparecidos.
da Presidência da República; Ministério da Educação Responsável: Secretaria Especial dos Direitos Humanos
da Presidência da República

Conhecimentos Específicos 68
Apostila Digital Licenciada para Alice Caroline Guarino dos Santos - alice.guarino@hotmail.com (Proibida a Revenda)
APOSTILAS OPÇÃO

l)Exigir em todos os projetos financiados pelo Governo Objetivo estratégico V:


Federal a adoção de estratégias de não discriminação de Garantir o atendimento especializado a crianças e
crianças e adolescentes em razão de classe, raça, etnia, crença, adolescentes em sofrimento psíquico e dependência
gênero, orientação sexual, identidade de gênero, deficiência, química.
prática de ato infracional e origem.
Responsável: Secretaria Especial dos Direitos Humanos Ações programáticas:
da Presidência da República a)Universalizar o acesso a serviços de saúde mental para
m)Reforçar e centralizar os mecanismos de coleta e análise crianças e adolescentes em cidades de grande e médio porte,
sistemática de dados desagregados da infância e adolescência, incluindo a garantia de retaguarda para as unidades de
especialmente sobre os grupos em situação de internação socioeducativa.
vulnerabilidade, historicamente vulnerabilizados, vítimas de Responsável: Ministério da Saúde
discriminação, de abuso e de negligência. b)Fortalecer políticas de saúde que contemplem
Responsável: Secretaria Especial dos Direitos Humanos programas de desintoxicação e redução de danos em casos de
da Presidência da República dependência química.
n)Estruturar rede de canais de denúncias (Disques) de Responsável: Ministério da Saúde
violência contra crianças e adolescentes, integrada aos
Conselhos Tutelares. Objetivo estratégico VI:
Responsável: Secretaria Especial dos Direitos Humanos Erradicação do trabalho infantil em todo o território
da Presidência da República nacional.
o)Estabelecer instrumentos para combater a
discriminação religiosa sofrida por crianças e adolescentes. Ações programáticas:
Responsáveis: Secretaria Especial dos Direitos Humanos a)Erradicar o trabalho infantil, por meio das ações
da Presidência da República intersetoriais no Governo Federal, com ênfase no apoio às
famílias e educação em tempo integral.
Objetivo estratégico IV: Responsáveis: Ministério do Trabalho e Emprego;
Enfrentamento da violência sexual contra crianças e Ministério da Educação; Secretaria Especial dos Direitos
adolescentes. Humanos da Presidência da República
b)Fomentar a implantação da Lei de Aprendizagem (Lei
Ações programáticas: no 10.097/2000), mobilizando empregadores, organizações de
a)Revisar o Plano Nacional de Enfrentamento à Violência trabalhadores, inspetores de trabalho, Judiciário, organismos
Sexual contra Crianças e Adolescentes, em consonância com internacionais e organizações não governamentais.
as recomendações do III Congresso Mundial sobre o tema. Responsável: Ministério do Trabalho e Emprego
Responsável: Secretaria Especial dos Direitos Humanos c)Desenvolver pesquisas, campanhas e relatórios
da Presidência da República periódicos sobre o trabalho infantil, com foco em temas e
b)Ampliar o acesso e qualificar os programas públicos que requerem abordagens específicas, tais como
especializados em saúde, educação e assistência social, no agricultura familiar, trabalho doméstico, trabalho de rua.
atendimento a crianças e adolescentes vítimas de violência Responsáveis: Ministério do Trabalho e Emprego;
sexual e de suas famílias Secretaria Especial de Políticas para as Mulheres da
Responsáveis: Ministério da Saúde; Ministério da Presidência da República; Ministério do Desenvolvimento
Educação; Ministério do Desenvolvimento Social e Combate à Agrário; Secretaria Especial dos Direitos Humanos da
Fome; Secretaria Especial dos Direitos Humanos da Presidência da República; Ministério do Desenvolvimento
Presidência da República Social e Combate à Fome; Ministério da Justiça
c)Desenvolver protocolos unificados de atendimento
psicossocial e jurídico a vítimas de violência sexual. Objetivo estratégico VII:
Responsáveis: Secretaria Especial dos Direitos Humanos Implementação do Sistema Nacional de Atendimento
da Presidência da República; Ministério da Saúde; Ministério Socioeducativo (SINASE).
do Desenvolvimento Social e Combate à Fome; Secretaria
Especial de Políticas para as Mulheres da Presidência da Ações programáticas:
República a)Elaborar e implementar um plano nacional
d)Desenvolver ações específicas para combate à violência socioeducativo e sistema de avaliação da execução das
e à exploração sexual de crianças e adolescentes em situação medidas daquele sistema, com divulgação anual de seus
de rua. resultados e estabelecimento de metas, de acordo com o
Responsáveis: Secretaria Especial dos Direitos Humanos estabelecido no ECA.
da Presidência da República; Ministério do Desenvolvimento Responsável: Secretaria Especial dos Direitos Humanos
Social e Combate à Fome. da Presidência da República
e)Estimular a responsabilidade social das empresas para b)Implantar módulo específico de informações para o
ações de enfrentamento da exploração sexual e de combate ao sistema nacional de atendimento educativo junto ao Sistema
trabalho infantil em suas organizações e cadeias produtivas. de Informação para a Infância e Adolescência, criando base de
Responsáveis: Secretaria Especial dos Direitos Humanos dados unificada que inclua as varas da infância e juventude, as
da Presidência da República; Ministério do Trabalho e unidades de internação e os programas municipais em meio
Emprego; Ministério do Turismo; aberto.
f)Combater a pornografia infanto-juvenil na Internet, por Responsável: Secretaria Especial dos Direitos Humanos
meio do fortalecimento do Hot Line Federal e da difusão de da Presidência da República
procedimentos de navegação segura para crianças, c)Implantar centros de formação continuada para os
adolescentes, famílias e educadores. operadores do sistema socioeducativo em todos os Estados e
Responsáveis: Ministério da Justiça; Secretaria Especial no Distrito Federal.
dos Direitos Humanos da Presidência da República; Ministério Responsáveis: Secretaria Especial dos Direitos Humanos
da Educação da Presidência da República; Ministério da Educação;
Ministério do Desenvolvimento Social e Combate à Fome

Conhecimentos Específicos 69
Apostila Digital Licenciada para Alice Caroline Guarino dos Santos - alice.guarino@hotmail.com (Proibida a Revenda)
APOSTILAS OPÇÃO

d)Desenvolver estratégias conjuntas com o sistema de Responsáveis: Secretaria Especial de Políticas de


justiça, com vistas ao estabelecimento de regras específicas Promoção da Igualdade Racial da Presidência da República;
para a aplicação da medida de privação de liberdade em Ministério da Educação; Ministério do Trabalho e Emprego;
caráter excepcional e de pouca duração. Ministério do Desenvolvimento Social e Combate à Fome;
Responsável: Secretaria Especial dos Direitos Humanos Ministério da Saúde
da Presidência da República c)Elaborar programas de combate ao racismo institucional
e)Apoiar a expansão de programas municipais de e estrutural, implementando normas administrativas e
atendimento socioeducativo em meio aberto. legislação nacional e internacional.
Responsáveis: Ministério do Desenvolvimento Social e Responsável: Secretaria Especial de Políticas de
Combate à Fome; Secretaria Especial dos Direitos Humanos da Promoção da Igualdade Racial da Presidência da República
Presidência da República d)Realizar levantamento de informações para produção de
f)Apoiar os Estados e o Distrito Federal na implementação relatórios periódicos de acompanhamento das políticas contra
de programas de atendimento ao adolescente em privação de a discriminação racial, contendo, entre outras, informações
liberdade, com garantia de escolarização, atendimento em sobre inclusão no sistema de ensino (básico e superior),
saúde, esporte, cultura e educação para o trabalho, inclusão no mercado de trabalho, assistência integrada à
condicionando a transferência voluntária de verbas federais à saúde, número de violações registradas e apuradas,
observância das diretrizes do plano nacional. recorrências de violações, e dados populacionais e de renda.
Responsáveis: Secretaria Especial dos Direitos Humanos Responsáveis: Secretaria Especial de Políticas de
da Presidência da República; Ministério da Educação; Promoção da Igualdade Racial da Presidência da República;
Ministério da Saúde; Ministério do Esporte; Ministério da Secretaria Especial dos Direitos Humanos da Presidência da
Cultura; Ministério do Trabalho e Emprego República
g)Garantir aos adolescentes privados de liberdade e suas e)Analisar periodicamente os indicadores que apontam
famílias informação sobre sua situação legal, bem como acesso desigualdades visando à formulação e implementação de
à defesa técnica durante todo o período de cumprimento da políticas públicas afirmativas que valorizem a promoção da
medida socioeducativa. igualdade racial.
Responsáveis: Secretaria Especial dos Direitos Humanos Responsáveis: Secretaria Especial de Políticas de
da Presidência da República; Ministério da Justiça Promoção da Igualdade Racial da Presidência da República;
h)Promover a transparência das unidades de internação Secretaria Especial dos Direitos Humanos da Presidência da
de adolescentes em conflito com a lei, garantindo o contato República; Ministério da Educação; Ministério do Trabalho e
com a família e a criação de comissões mistas de inspeção e Emprego; Ministério do Desenvolvimento Social e Combate à
supervisão. Fome; Ministério da Saúde
Responsável: Secretaria Especial dos Direitos Humanos f)Fortalecer a integração das políticas públicas em todas as
da Presidência da República comunidades remanescentes de quilombos localizadas no
i)Fomentar a desativação dos grandes complexos de território brasileiro.
unidades de internação, por meio do apoio à reforma e Responsáveis: Secretaria Especial de Políticas de
construção de novas unidades alinhadas aos parâmetros Promoção da Igualdade Racial da Presidência da República;
estabelecidos no SINASE e no ECA, em especial na observância Ministério da Cultura
da separação por sexo, faixa etária e compleição física. g)Fortalecer os mecanismos existentes de reconhecimento
Responsável: Secretaria Especial dos Direitos Humanos das comunidades quilombolas como garantia dos seus direitos
da Presidência da República específicos .
j)Desenvolver campanhas de informação sobre o Responsáveis: Ministério do Desenvolvimento
adolescente em conflito com a lei, defendendo a não redução Agrário; Ministério da Cultura; Secretaria Especial de Política
da maioridade penal. de Promoção da Igualdade Racial da Presidência da República
Responsável: Secretaria Especial dos Direitos Humanos h)Fomentar programas de valorização do patrimônio
da Presidência da República cultural das populações negras.
k)Estabelecer parâmetros nacionais para a apuração Responsável: Ministério da Cultura; Secretaria Especial
administrativa de possíveis violações dos direitos e casos de de Promoção da Igualdade Racial da Presidência da República
tortura em adolescentes privados de liberdade, por meio de i)Assegurar o resgate da memória das populações negras,
sistema independente e de tramitação ágil. mediante a publicação da história de resistência e resgate de
Responsável: Secretaria Especial dos Direitos Humanos tradições das populações das diásporas.
da Presidência da República Responsável: Secretaria Especial de Políticas de
Promoção da Igualdade Racial da Presidência da República
Diretriz 9: Combate às desigualdades estruturais.
Objetivo estratégico II:
Objetivo estratégico I:
Igualdade e proteção dos direitos das populações Garantia aos povos indígenas da manutenção e resgate
negras, historicamente afetadas pela discriminação e das condições de reprodução, assegurando seus modos de
outras formas de intolerância. vida.

Ações programáticas: Ações programáticas:


a)Apoiar, junto ao Poder Legislativo, a aprovação do a)Assegurar a integridade das terras indígenas para
Estatuto da Igualdade Racial. proteger e promover o modo de vida dos povos indígenas.
Responsáveis: Secretaria Especial de Políticas de Responsável: Ministério da Justiça
Promoção da Igualdade Racial da Presidência da República; b)Proteger os povos indígenas isolados e de recente
Secretaria Especial dos Direitos Humanos da Presidência da contato para garantir sua reprodução cultural e
República etnoambiental.
b)Promover ações articuladas entre as políticas de Responsável: Ministério da Justiça
educação, cultura, saúde e de geração de emprego e renda, c)Aplicar os saberes dos povos indígenas e das
visando incidir diretamente na qualidade de vida da população comunidades tradicionais na elaboração de políticas públicas,
negra e no combate à violência racial. respeitando a Convenção no 169 da OIT.

Conhecimentos Específicos 70
Apostila Digital Licenciada para Alice Caroline Guarino dos Santos - alice.guarino@hotmail.com (Proibida a Revenda)
APOSTILAS OPÇÃO

Responsável: Ministério da Justiça de violências contra a mulher, assistência à saúde integral,


d)Apoiar projetos de lei com objetivo de revisar o Estatuto dados reprodutivos, mortalidade materna e escolarização.
do Índio com base no texto constitucional de 1988 e na Responsável: Secretaria Especial de Políticas para as
Convenção no 169 da OIT. Mulheres da Presidência da República
Responsável: Ministério da Justiça d)Divulgar os instrumentos legais de proteção às
e)Elaborar relatório periódico de acompanhamento das mulheres, nacionais e internacionais, incluindo sua publicação
políticas indigenistas que contemple dados sobre os processos em formatos acessíveis, como braile, CD de áudio e demais
de demarcações das terras indígenas, dados sobre intrusões e tecnologias assistivas.
conflitos territoriais, inclusão no sistema de ensino (básico e Responsável: Secretaria Especial de Políticas para as
superior), assistência integrada à saúde, número de violações Mulheres da Presidência da República
registradas e apuradas, recorrências de violações e dados e)Ampliar o financiamento de abrigos para mulheres em
populacionais. situação de vulnerabilidade, garantindo plena acessibilidade.
Responsável: Ministério da Justiça Responsáveis: Secretaria Especial de Políticas para as
f)Proteger e promover os conhecimentos tradicionais e Mulheres da Presidência da República; Ministério do
medicinais dos povos indígenas. Desenvolvimento Social e Combate à Fome
Responsáveis: Ministério da Justiça; Ministério da Saúde f)Propor tratamento preferencial de atendimento às
g)Implementar políticas de proteção do patrimônio dos mulheres em situação de violência doméstica e familiar nos
povos indígenas, por meio dos registros material e imaterial, Conselhos Gestores do Fundo Nacional de Habitação de
mapeando os sítios históricos e arqueológicos, a cultura, as Interesse Social e junto ao Fundo de Desenvolvimento Social.
línguas e a arte. Responsáveis: Secretaria Especial de Políticas para as
Responsáveis: Ministério da Cultura; Ministério da Justiça Mulheres da Presidência da República; Ministério das Cidades;
h)Promover projetos e pesquisas para resgatar a história Ministério do Desenvolvimento Social e Combate à Fome
dos povos indígenas. g)Apoiar a aprovação do projeto de lei que descriminaliza
Responsável: Ministério da Justiça o aborto, considerando a autonomia das mulheres para decidir
i)Promover ações culturais para o fortalecimento da sobre seus corpos.
educação escolar dos povos indígenas, estimulando a g) Considerar o aborto como tema de saúde pública, com a
valorização de suas próprias formas de produção do garantia do acesso aos serviços de saúde. (Redação dada pelo
conhecimento. Decreto nº 7.177, de 2010)
Responsáveis: Ministério da Cultura; Ministério da Justiça Responsáveis: Ministério da Saúde; Secretaria Especial de
j)Garantir o acesso à educação formal pelos povos Políticas para as Mulheres da Presidência da República;
indígenas, bilíngues e com adequação curricular formulada Ministério da Justiça
com a participação de representantes das etnias indigenistas e h)Realizar campanhas e ações educativas para
especialistas em educação. desconstruir os estereótipos relativos às profissionais do sexo.
Responsáveis: Ministério da Justiça; Ministério da Responsável: Secretaria Especial de Políticas para as
Educação Mulheres da Presidência da República
k)Assegurar o acesso e permanência da população
indígena no ensino superior, por meio de ações afirmativas e Diretriz 10: Garantia da igualdade na diversidade.
respeito à diversidade étnica e cultural.
Responsáveis: Ministério da Justiça; Ministério da Objetivo estratégico I:
Educação Afirmação da diversidade para construção de uma
l)Adotar medidas de proteção dos direitos das crianças sociedade igualitária.
indígenas nas redes de ensino, saúde e assistência social, em
consonância com a promoção dos seus modos de vida. Ações programáticas:
Responsáveis: Ministério da Educação; Ministério da a)Realizar campanhas e ações educativas para
Saúde; Ministério do Desenvolvimento Social e Combate à desconstrução de estereótipos relacionados com diferenças
Fome; Secretaria Especial dos Direitos Humanos da étnico-raciais, etárias, de identidade e orientação sexual, de
Presidência da República pessoas com deficiência, ou segmentos profissionais
socialmente discriminados.
Objetivo estratégico III: Responsáveis: Secretaria Especial dos Direitos Humanos
Garantia dos direitos das mulheres para o da Presidência da República; Secretaria Especial de Políticas
estabelecimento das condições necessárias para sua de Promoção da Igualdade Racial da Presidência da República;
plena cidadania. Secretaria Especial de Políticas para as Mulheres da
Presidência da República; Ministério da Cultura
Ações programáticas: b)Incentivar e promover a realização de atividades de
a)Desenvolver ações afirmativas que permitam incluir valorização da cultura das comunidades tradicionais, entre
plenamente as mulheres no processo de desenvolvimento do elas ribeirinhos, extrativistas, quebradeiras de coco,
País, por meio da promoção da sua autonomia econômica e de pescadores artesanais, seringueiros, geraizeiros, varzanteiros,
iniciativas produtivas que garantam sua independência. pantaneiros, comunidades de fundo de pasto, caiçaras e
Responsável: Secretaria Especial de Políticas para as faxinalenses.
Mulheres da Presidência da República Responsáveis: Ministério da Cultura; Ministério do
b)Incentivar políticas públicas e ações afirmativas para a Desenvolvimento Social e Combate à Fome; Ministério do
participação igualitária, plural e multirracial das mulheres nos Esporte
espaços de poder e decisão. c)Fomentar a formação e capacitação em Direitos
Responsável: Secretaria Especial de Políticas para as Humanos, como meio de resgatar a autoestima e a dignidade
Mulheres da Presidência da República das comunidades tradicionais, rurais e urbanas.
c)Elaborar relatório periódico de acompanhamento das Responsáveis: Secretaria Especial de Políticas de
políticas para mulheres com recorte étnico-racial, que Promoção da Igualdade Racial da Presidência da República;
contenha dados sobre renda, jornada e ambiente de trabalho, Ministério da Justiça; Ministério da Cultura
ocorrências de assédio moral, sexual e psicológico, ocorrências d)Apoiar políticas de acesso a direitos para a população
cigana, valorizando seus conhecimentos e cultura.

Conhecimentos Específicos 71
Apostila Digital Licenciada para Alice Caroline Guarino dos Santos - alice.guarino@hotmail.com (Proibida a Revenda)
APOSTILAS OPÇÃO

Responsável: Ministério do Desenvolvimento Social e Responsáveis: Secretaria Especial dos Direitos Humanos
Combate à Fome da Presidência da República; Ministério da Cultura
e)Apoiar e valorizar a associação das mulheres h)Elaborar programas de capacitação para os operadores
quebradeiras de coco, protegendo e promovendo a dos direitos da pessoa idosa.
continuidade de seu trabalho extrativista. Responsável: Secretaria Especial dos Direitos Humanos
Responsável: Ministério do Desenvolvimento Social e da Presidência da República.
Combate à Fome i)Elaborar relatório periódico de acompanhamento das
f)Elaborar relatórios periódicos de acompanhamento das políticas para pessoas idosas que contenha informações sobre
políticas direcionadas às populações e comunidades os Centros Integrados de Atenção a Prevenção à Violência, tais
tradicionais, que contenham, entre outras, informações sobre como: quantidade existente; sua participação no
população estimada, assistência integrada à saúde, número de financiamento público; sua inclusão nos sistemas de
violações registradas e apuradas, recorrência de violações, atendimento; número de profissionais capacitados; pessoas
lideranças ameaçadas, dados sobre acesso à moradia, terra e idosas atendidas; proporção dos casos com resoluções; taxa de
território e conflitos existentes. reincidência; pessoas idosas seguradas e aposentadas;
Responsáveis: Ministério da Justiça; Ministério da Saúde; famílias providas por pessoas idosas; pessoas idosas em
Secretaria Especial de Políticas de Promoção da Igualdade abrigos; pessoas idosas em situação de rua; principal fonte de
Racial da Presidência da República; Secretaria Especial dos renda dos idosos; pessoas idosas atendidas, internadas e
Direitos Humanos da Presidência da República mortas por violência ou maus-tratos.
Responsáveis: Secretaria Especial dos Direitos Humanos
Objetivo estratégico II: da Presidência da República; Ministério da Saúde; Ministério
Proteção e promoção da diversidade das expressões da Previdência Social; Ministério da Justiça; Ministério do
culturais como Direito Humano. Desenvolvimento Social e Combate à Fome

Ações programáticas: Objetivo estratégico IV:


a)Promover ações de afirmação do direito à diversidade Promoção e proteção dos direitos das pessoas com
das expressões culturais, garantindo igual dignidade e deficiência e garantia da acessibilidade igualitária.
respeito para todas as culturas.
Responsável: Ministério da Cultura Ações programáticas:
b)Incluir nos instrumentos e relatórios de políticas a)Garantir às pessoas com deficiência igual e efetiva
culturais a temática dos Direitos Humanos. proteção legal contra a discriminação.
Responsável: Ministério da Cultura Responsáveis: Secretaria Especial dos Direitos Humanos
da Presidência da República; Ministério da Justiça
Objetivo estratégico III: b)Garantir salvaguardas apropriadas e efetivas para
Valorização da pessoa idosa e promoção de sua prevenir abusos a pessoas com deficiência e pessoas idosas.
participação na sociedade. Responsável: Secretaria Especial dos Direitos Humanos
da Presidência da República
Ações programáticas: c)Assegurar o cumprimento do Decreto de Acessibilidade
a)Promover a inserção, a qualidade de vida e a prevenção (Decreto no 5.296/2004), que garante a acessibilidade pela
de agravos aos idosos, por meio de programas que fortaleçam adequação das vias e passeios públicos, semáforos,
o convívio familiar e comunitário, garantindo o acesso a mobiliários, habitações, espaços de lazer, transportes, prédios
serviços, ao lazer, à cultura e à atividade física, de acordo com públicos, inclusive instituições de ensino, e outros itens de uso
sua capacidade funcional. individual e coletivo.
Responsáveis: Secretaria Especial dos Direitos Humanos Responsáveis: Secretaria Especial dos Direitos Humanos
da Presidência da República; Ministério da Cultura; Ministério da Presidência da República; Ministério do Trabalho e
do Esporte Emprego; Ministério das Cidades
b)Apoiar a criação de centros de convivência e desenvolver d)Garantir recursos didáticos e pedagógicos para atender
ações de valorização e socialização da pessoa idosa nas zonas às necessidades educativas especiais.
urbanas e rurais. Responsável: Ministério da Educação
Responsáveis: Secretaria Especial dos Direitos Humanos e)Disseminar a utilização dos sistemas braile, tadoma,
da Presidência da República; Ministério da Cultura escrita de sinais e libras tátil para inclusão das pessoas com
c)Fomentar programas de voluntariado de pessoas idosas, deficiência em todo o sistema de ensino.
visando valorizar e reconhecer sua contribuição para o Responsáveis: Secretaria Especial dos Direitos Humanos
desenvolvimento e bem-estar da comunidade. da Presidência da República; Ministério da Educação
Responsável: Secretaria Especial dos Direitos Humanos f)Instituir e implementar o ensino da Língua Brasileira de
da Presidência da República Sinais como disciplina curricular facultativa.
d)Desenvolver ações que contribuam para o protagonismo Responsáveis: Secretaria Especial dos Direitos Humanos
da pessoa idosa na escola, possibilitando sua participação da Presidência da República; Ministério da Educação
ativa na construção de uma nova percepção intergeracional. g)Propor a regulamentação das profissões relativas à
Responsável: Secretaria Especial dos Direitos Humanos implementação da acessibilidade, tais como: instrutor de
da Presidência da República Libras, guia-intérprete, tradutor-intérprete, transcritor,
e)Potencializar ações com ênfase no diálogo revisor e ledor da escrita braile e treinadores de cães-guia.
intergeracional, valorizando o conhecimento acumulado das Responsável: Ministério do Trabalho e Emprego
pessoas idosas. h)Elaborar relatórios sobre os Municípios que possuam
Responsável: Secretaria Especial dos Direitos Humanos frota adaptada para subsidiar o processo de monitoramento
da Presidência da República do cumprimento e implementação da legislação de
f)Desenvolver ações intersetoriais para capacitação acessibilidade.
continuada de cuidadores de pessoas idosas. Responsáveis: Ministério das Cidades; Secretaria Especial
Responsáveis: Ministério da Saúde; Ministério da Cultura dos Direitos Humanos da Presidência da República
g)Desenvolver política de humanização do atendimento ao
idoso, principalmente em instituições de longa permanência.

Conhecimentos Específicos 72
Apostila Digital Licenciada para Alice Caroline Guarino dos Santos - alice.guarino@hotmail.com (Proibida a Revenda)
APOSTILAS OPÇÃO

Objetivo estratégico V: c) (Revogado pelo Decreto nº 7.177, de 2010)


Garantia do respeito à livre orientação sexual e Responsável (Revogado pelo Decreto nº 7.177, de 2010)
identidade de gênero. d)Estabelecer o ensino da diversidade e história das
religiões, inclusive as derivadas de matriz africana, na rede
Ações programáticas: pública de ensino, com ênfase no reconhecimento das
a)Desenvolver políticas afirmativas e de promoção de diferenças culturais, promoção da tolerância e na afirmação da
cultura de respeito à livre orientação sexual e identidade de laicidade do Estado.
gênero, favorecendo a visibilidade e o reconhecimento social. Responsáveis: Ministério da Educação; Secretaria
Responsável: Secretaria Especial dos Direitos Humanos Especial dos Direitos Humanos da Presidência da República
da Presidência da República e)Realizar relatório sobre pesquisas populacionais
b)Apoiar projeto de lei que disponha sobre a união civil relativas a práticas religiosas, que contenha, entre outras,
entre pessoas do mesmo sexo. informações sobre número de religiões praticadas, proporção
Responsáveis: Secretaria Especial dos Direitos Humanos de pessoas distribuídas entre as religiões, proporção de
da Presidência da República; Ministério da Justiça pessoas que já trocaram de religião, número de pessoas
c)Promover ações voltadas à garantia do direito de adoção religiosas não praticantes e número de pessoas sem religião.
por casais homoafetivos. Responsável: Secretaria Especial dos Direitos Humanos
Responsáveis: Ministério da Justiça; Secretaria Especial da Presidência da República
dos Direitos Humanos da Presidência da República; Secretaria
Especial de Políticas para as Mulheres da Presidência da Eixo Orientador IV:
República Segurança Pública, Acesso à Justiça e Combate à
d)Reconhecer e incluir nos sistemas de informação do Violência
serviço público todas as configurações familiares constituídas
por lésbicas, gays, bissexuais, travestis e transexuais, com base Por muito tempo, alguns segmentos da militância em
na desconstrução da heteronormatividade. Direitos Humanos mantiveram-se distantes do debate sobre as
Responsável: Ministério do Planejamento, Orçamento e políticas públicas de segurança no Brasil. No processo de
Gestão consolidação da democracia, por diferentes razões,
e)Desenvolver meios para garantir o uso do nome social de movimentos sociais e entidades manifestaram dificuldade no
travestis e transexuais. tratamento do tema. Na base dessa dificuldade, estavam a
Responsável: Secretaria Especial dos Direitos Humanos memória dos enfrentamentos com o aparato repressivo ao
da Presidência da República longo de duas décadas de regime ditatorial, a postura violenta
f)Acrescentar campo para informações sobre a identidade vigente, muitas vezes, em órgãos de segurança pública, a
de gênero dos pacientes nos prontuários do sistema de saúde. percepção do crime e da violência como meros subprodutos de
Responsável: Ministério da Saúde uma ordem social injusta a ser transformada em seus próprios
g)Fomentar a criação de redes de proteção dos Direitos fundamentos.
Humanos de Lésbicas, Gays, Bissexuais, Travestis e Distanciamento análogo ocorreu nas universidades, que,
Transexuais (LGBT), principalmente a partir do apoio à com poucas exceções, não se debruçaram sobre o modelo de
implementação de Centros de Referência em Direitos polícia legado ou sobre os desafios da segurança pública. As
Humanos de Prevenção e Combate à Homofobia e de núcleos polícias brasileiras, nos termos de sua tradição institucional,
de pesquisa e promoção da cidadania daquele segmento em pouco aproveitaram da reflexão teórica e dos aportes
universidades públicas. oferecidos pela criminologia moderna e demais ciências
Responsável: Secretaria Especial dos Direitos Humanos sociais, já disponíveis há algumas décadas às polícias e aos
da Presidência da República gestores de países desenvolvidos. A cultura arraigada de
h)Realizar relatório periódico de acompanhamento das rejeitar as evidências acumuladas pela pesquisa e pela
políticas contra discriminação à população LGBT, que experiência de reforma das polícias no mundo era a mesma
contenha, entre outras, informações sobre inclusão no que expressava nostalgia de um passado de ausência de
mercado de trabalho, assistência à saúde integral, número de garantias individuais, e que identificava na idéia dos Direitos
violações registradas e apuradas, recorrências de violações, Humanos não a mais generosa entre as promessas construídas
dados populacionais, de renda e conjugais. pela modernidade, mas uma verdadeira ameaça.
Responsável: Secretaria Especial dos Direitos Humanos Estavam postas as condições históricas, políticas e
da Presidência da República culturais para que houvesse um fosso aparentemente
intransponível entre os temas da segurança pública e os
Objetivo estratégico VI: Direitos Humanos.
Respeito às diferentes crenças, liberdade de culto e Nos últimos anos, contudo, esse processo de
garantia da laicidade do Estado. estranhamento mútuo passou a ser questionado. De um lado,
articulações na sociedade civil assumiram o desafio de
Ações programáticas: repensar a segurança pública a partir de diálogos com
a)Instituir mecanismos que assegurem o livre exercício especialistas na área, policiais e gestores. De outro,
das diversas práticas religiosas, assegurando a proteção do começaram a ser implantadas as primeiras políticas públicas
seu espaço físico e coibindo manifestações de intolerância buscando caminhos alternativos de redução do crime e da
religiosa. violência, a partir de projetos centrados na prevenção e
Responsáveis: Ministério da Justiça; Ministério da influenciados pela cultura de paz.
Cultura; Secretaria Especial dos Direitos Humanos da A proposição do Sistema Único de Segurança Pública, a
Presidência da República modernização de parte das nossas estruturas policiais e a
b)Promover campanhas de divulgação sobre a diversidade aprovação de novos regimentos e leis orgânicas das polícias, a
religiosa para disseminar cultura da paz e de respeito às consciência crescente de que políticas de segurança pública
diferentes crenças. são realidades mais amplas e complexas do que as iniciativas
Responsáveis: Secretaria Especial dos Direitos Humanos possíveis às chamadas "forças da segurança", o surgimento de
da Presidência da República; Ministério da Cultura; Secretaria nova geração de policiais, disposta a repensar práticas e
Especial de Políticas de Promoção da Igualdade Racial da dogmas e, sobretudo, a cobrança da opinião pública e a maior
Presidência da República fiscalização sobre o Estado, resultante do processo de

Conhecimentos Específicos 73
Apostila Digital Licenciada para Alice Caroline Guarino dos Santos - alice.guarino@hotmail.com (Proibida a Revenda)
APOSTILAS OPÇÃO

democratização, têm tornado possível a construção de agenda respostas judiciais mais céleres e eficazes. Destacam-se, ainda,
de reformas na área. o direito de acesso à Justiça em matéria de conflitos agrários e
O Programa Nacional de Segurança Pública com Cidadania urbanos e o necessário estímulo aos meios de soluções
(Pronasci) e os investimentos já realizados pelo Governo pacíficas de controvérsias.
Federal na montagem de rede nacional de altos estudos em O PNDH-3 apresenta neste eixo, fundamentalmente,
segurança pública, que têm beneficiado milhares de policiais propostas para que o Poder Público se aperfeiçoe no
em cada Estado, simbolizam, ao lado do processo de debates desenvolvimento de políticas públicas de prevenção ao crime
da 1ª Conferência Nacional de Segurança Pública, acúmulos e à violência, reforçando a noção de acesso universal à Justiça
históricos significativos, que apontam para novas e mais como direito fundamental, e sustentando que a democracia, os
importantes mudanças. processos de participação e transparência, aliados ao uso de
As propostas elencadas neste eixo orientador do PNDH-3 ferramentas científicas e à profissionalização das instituições
articulam-se com tal processo histórico de transformação e e trabalhadores da segurança, assinalam os roteiros mais
exigem muito mais do que já foi alcançado. Para tanto, parte- promissores para que o Brasil possa avançar no caminho da
se do pressuposto de que a realidade brasileira segue sendo paz pública.
gravemente marcada pela violência e por severos impasses
estruturais na área da segurança pública. Diretriz 11: Democratização e modernização do
Problemas antigos, como a ausência de diagnósticos, de sistema de segurança pública
planejamento e de definição formal de metas, a desvalorização
profissional dos policiais e dos agentes penitenciários, o Objetivo estratégico I:
desperdício de recursos e a consagração de privilégios dentro Modernização do marco normativo do sistema de
das instituições, as práticas de abuso de autoridade e de segurança pública.
violência policial contra grupos vulneráveis e a corrupção dos
agentes de segurança pública, demandam reformas tão Ações programáticas:
urgentes quanto profundas. a)Propor alteração do texto constitucional, de modo a
As propostas sistematizadas no PNDH-3 agregam, nesse considerar as polícias militares não mais como forças
contexto, as contribuições oferecidas pelo processo da 11ª auxiliares do Exército, mantendo-as apenas como força
Conferência Nacional dos Direitos Humanos e avançam reserva.
também sobre temas que não foram objeto de debate, Responsável: Ministério da Justiça
trazendo para o PNDH-3 parte do acúmulo crítico que tem sido b)Propor a revisão da estrutura, treinamento, controle,
proposto ao País pelos especialistas e pesquisadores da área. emprego e regimentos disciplinares dos órgãos de segurança
Em linhas gerais, o PNDH-3 aponta para a necessidade de pública, de forma a potencializar as suas funções de combate
ampla reforma no modelo de polícia e propõe o ao crime e proteção dos direitos de cidadania, bem como
aprofundamento do debate sobre a implantação do ciclo garantir que seus órgãos corregedores disponham de carreira
completo de policiamento às corporações estaduais. Prioriza própria, sem subordinação à direção das instituições policiais.
transparência e participação popular, instando ao Responsável: Ministério da Justiça
aperfeiçoamento das estatísticas e à publicação de dados, c)Propor a criação obrigatória de ouvidorias de polícias
assim como à reformulação do Conselho Nacional de independentes nos Estados e no Distrito Federal, com
Segurança Pública. Contempla a prevenção da violência e da ouvidores protegidos por mandato e escolhidos com
criminalidade como diretriz, ampliando o controle sobre participação da sociedade.
armas de fogo e indicando a necessidade de profissionalização Responsável: Ministério da Justiça
da investigação criminal. d)Assegurar a autonomia funcional dos peritos e a
modernização dos órgãos periciais oficiais, como forma de
Com ênfase na erradicação da tortura e na redução da incrementar sua estruturação, assegurando a produção isenta
letalidade policial e carcerária, confere atenção especial ao e qualificada da prova material, bem como o princípio da
estabelecimento de procedimentos operacionais ampla defesa e do contraditório e o respeito aos Direitos
padronizados, que previnam as ocorrências de abuso de Humanos.
autoridade e de violência institucional, e confiram maior Responsável: Ministério da Justiça
segurança a policiais e agentes penitenciários. Reafirma a e)Promover o aprofundamento do debate sobre a
necessidade de criação de ouvidorias independentes em instituição do ciclo completo da atividade policial, com
âmbito federal e, inspirado em tendências mais modernas de competências repartidas pelas polícias, a partir da natureza e
policiamento, estimula as iniciativas orientadas por da gravidade dos delitos.
resultados, o desenvolvimento do policiamento comunitário e Responsável: Ministério da Justiça
voltado para a solução de problemas, elencando medidas que f)Apoiar a aprovação do Projeto de Lei no 1.937/2007, que
promovam a valorização dos trabalhadores em segurança dispõe sobre o Sistema Único de Segurança Pública.
pública. Contempla, ainda, a criação de sistema federal que Responsável: Ministério da Justiça
integre os atuais sistemas de proteção a vítimas e
testemunhas, defensores de Direitos Humanos e crianças e Objetivo estratégico II:
adolescentes ameaçados de morte. Modernização da gestão do sistema de segurança
Também como diretriz, o PNDH-3 propõe profunda pública.
reforma da Lei de Execução Penal que introduza garantias
fundamentais e novos regramentos para superar as práticas Ações programáticas:
abusivas, hoje comuns. E trata as penas privativas de liberdade a)Condicionar o repasse de verbas federais à elaboração e
como última alternativa, propondo a redução da demanda por revisão periódica de planos estaduais, distrital e municipais de
encarceramento e estimulando novas formas de tratamento segurança pública que se pautem pela integração e pela
dos conflitos, como as sugeridas pelo mecanismo da Justiça responsabilização territorial da gestão dos programas e ações.
Restaurativa. Responsável: Ministério da Justiça
Reafirma-se a centralidade do direito universal de acesso b)Criar base de dados unificada que permita o fluxo de
à Justiça, com a possibilidade de acesso aos tribunais por toda informações entre os diversos componentes do sistema de
a população, com o fortalecimento das defensorias públicas e segurança pública e a Justiça criminal.
a modernização da gestão judicial, de modo a garantir

Conhecimentos Específicos 74
Apostila Digital Licenciada para Alice Caroline Guarino dos Santos - alice.guarino@hotmail.com (Proibida a Revenda)
APOSTILAS OPÇÃO

Responsáveis: Ministério da Justiça; Secretaria Especial Responsável: Ministério da Justiça


dos Direitos Humanos da Presidência da República b)Fomentar mecanismos de gestão participativa das
c)Redefinir as competências e o funcionamento da políticas públicas de segurança, como conselhos e
Inspetoria-Geral das Polícias Militares e Corpos de Bombeiros conferências, ampliando a Conferência Nacional de Segurança
Militares. Pública.
Responsáveis: Ministério da Justiça; Ministério da Defesa Responsável: Ministério da Justiça

Objetivo estratégico III: Diretriz 13: Prevenção da violência e da criminalidade


Promoção dos Direitos Humanos dos profissionais do e profissionalização da investigação de atos criminosos.
sistema de segurança pública, assegurando sua formação
continuada e compatível com as atividades que exercem. Objetivo estratégico I:
Ampliação do controle de armas de fogo em circulação
Ações programáticas: no País.
a)Proporcionar equipamentos para proteção individual
efetiva para os profissionais do sistema federal de segurança Ações programáticas:
pública. a)Realizar ações permanentes de estímulo ao
Responsável: Ministério da Justiça desarmamento da população.
b)Condicionar o repasse de verbas federais aos Estados, ao Responsável: Ministério da Justiça
Distrito Federal e aos Municípios, à garantia da efetiva b)Propor reforma da legislação para ampliar as restrições
disponibilização de equipamentos de proteção individual aos e os requisitos para aquisição de armas de fogo por
profissionais do sistema nacional de segurança pública. particulares e empresas de segurança privada.
Responsável: Ministério da Justiça Responsável: Ministério da Justiça
c)Fomentar o acompanhamento permanente da saúde c)Propor alteração da legislação para garantir que as
mental dos profissionais do sistema de segurança pública, armas apreendidas em crimes que não envolvam disparo
mediante serviços especializados do sistema de saúde pública. sejam inutilizadas imediatamente após a perícia.
Responsáveis: Ministério da Justiça; Ministério da Saúde Responsável: Ministério da Justiça
d)Propor projeto de lei instituindo seguro para casos de d)Registrar no Sistema Nacional de Armas todas as armas
acidentes incapacitantes ou morte em serviço para os de fogo destruídas.
profissionais do sistema de segurança pública. Responsável: Ministério da Defesa
Responsável: Ministério da Justiça;
e)Garantir a reabilitação e reintegração ao trabalho dos Objetivo estratégico II:
profissionais do sistema de segurança pública federal, nos Qualificação da investigação criminal.
casos de deficiência adquirida no exercício da função.
Responsável: Ministério da Justiça; Ações programáticas:
a)Propor projeto de lei para alterar o procedimento do
Diretriz 12: Transparência e participação popular no inquérito policial, de modo a admitir procedimentos orais
sistema de segurança pública e justiça criminal. gravados e transformar em peça ágil e eficiente de
investigação criminal voltada à coleta de evidências.
Objetivo estratégico I: Responsável: Ministério da Justiça
Publicação de dados do sistema federal de segurança b)Fomentar o debate com o objetivo de unificar os meios
pública. de investigação e obtenção de provas e padronizar
procedimentos de investigação criminal.
Ação programática Responsável: Ministério da Justiça
a)Publicar trimestralmente estatísticas sobre: c)Promover a capacitação técnica em investigação criminal
- Crimes registrados, inquéritos instaurados e concluídos, para os profissionais dos sistemas estaduais de segurança
prisões efetuadas, flagrantes registrados, operações pública.
realizadas, armas e entorpecentes apreendidos pela Polícia Responsável: Ministério da Justiça
Federal em cada Estado da Federação; d)Realizar pesquisas para qualificação dos estudos sobre
- Veículos abordados, armas e entorpecentes apreendidos técnicas de investigação criminal.
e prisões efetuadas pela Polícia Rodoviária Federal em cada Responsável: Ministério da Justiça
Estado da Federação;
- Presos provisórios e condenados sob custódia do sistema Objetivo estratégico III:
penitenciário federal e quantidade de presos trabalhando e Produção de prova pericial com celeridade e
estudando por sexo, idade e raça ou etnia; procedimento padronizado.
- Vitimização de policiais federais, policiais rodoviários
federais, membros da Força Nacional de Segurança Pública e Ações programáticas:
agentes penitenciários federais; a)Propor regulamentação da perícia oficial.
- Quantidade e tipos de laudos produzidos pelos órgãos Responsável: Ministério da Justiça
federais de perícia oficial. b)Propor projeto de lei para proporcionar autonomia
Responsável: Ministério da Justiça administrativa e funcional dos órgãos periciais federais.
Responsável: Ministério da Justiça
Objetivo estratégico II: c)Propor padronização de procedimentos e equipamentos
Consolidação de mecanismos de participação popular a serem utilizados pelas unidades periciais oficiais em todos os
na elaboração das políticas públicas de segurança. exames periciais criminalísticos e médico-legais.
Responsáveis: Ministério da Justiça; Secretaria Especial
Ações programáticas: dos Direitos Humanos da Presidência da República
a)Reformular o Conselho Nacional de Segurança Pública, d)Desenvolver sistema de dados nacional informatizado
assegurando a participação da sociedade civil organizada em para monitoramento da produção e da qualidade dos laudos
sua composição e garantindo sua articulação com o Conselho produzidos nos órgãos periciais.
Nacional de Política Criminal e Penitenciária. Responsável: Ministério da Justiça

Conhecimentos Específicos 75
Apostila Digital Licenciada para Alice Caroline Guarino dos Santos - alice.guarino@hotmail.com (Proibida a Revenda)
APOSTILAS OPÇÃO

e)Fomentar parcerias com universidades para pesquisa e República; Secretaria Especial de Políticas de Promoção da
desenvolvimento de novas metodologias a serem implantadas Igualdade Racial da Presidência da República
nas unidades periciais. c)Desenvolver e implantar sistema nacional integrado das
Responsável: Ministério da Justiça redes de saúde, de assistência social e educação para a
f)Promover e apoiar a educação continuada dos notificação de violência.
profissionais da perícia oficial, em todas as áreas, para a Responsáveis: Secretaria Especial dos Direitos Humanos
formação técnica e em Direitos Humanos. da Presidência da República; Ministério da Saúde; Ministério
Responsáveis: Ministério da Justiça; Secretaria Especial do Desenvolvimento Social e Combate à Fome; Ministério da
dos Direitos Humanos da Presidência da República Educação; Secretaria Especial de Políticas de Promoção da
Igualdade Racial da Presidência da República
Objetivo estratégico IV: d)Promover campanhas educativas e pesquisas voltadas à
Fortalecimento dos instrumentos de prevenção à prevenção da violência contra pessoas com deficiência, idosos,
violência. mulheres, indígenas, negros, crianças, adolescentes, lésbicas,
gays, bissexuais, transexuais, travestis e pessoas em situação
Ações programáticas: de rua.
a)Elaborar diretrizes para as políticas de prevenção à Responsáveis: Secretaria Especial dos Direitos Humanos
violência com o objetivo de assegurar o reconhecimento das da Presidência da República; Ministério do Desenvolvimento
diferenças geracionais, de gênero, étnico-racial e de orientação Social e Combate à Fome; Secretaria Especial de Políticas de
sexual. Promoção da Igualdade Racial da Presidência da República;
Responsáveis: Ministério da Justiça; Secretaria Especial Secretaria Especial de Políticas para as Mulheres da
de Políticas para as Mulheres da Presidência da República; Presidência da República; Ministério da Justiça; Ministério do
Secretaria Especial de Políticas de Promoção da Igualdade Turismo; Ministério do Esporte
Racial da Presidência da República e)Fortalecer unidade especializada em conflitos indígenas
b)Realizar anualmente pesquisas nacionais de vitimização. na Polícia Federal e garantir sua atuação conjunta com a
Responsáveis: Ministério da Justiça; Secretaria Especial FUNAI, em especial nos processos conflituosos de demarcação.
dos Direitos Humanos da Presidência da República Responsável: Ministério da Justiça
c)Fortalecer mecanismos que possibilitem a efetiva f)Fomentar cursos de qualificação e capacitação sobre
fiscalização de empresas de segurança privada e a investigação aspectos da cultura tradicional dos povos indígenas e sobre
e responsabilização de policiais que delas participem de forma legislação indigenista para todas as corporações policiais,
direta ou indireta. principalmente para as polícias militares e civis especialmente
Responsáveis: Ministério da Justiça; Secretaria Especial nos Estados e Municípios em que as aldeias indígenas estejam
dos Direitos Humanos da Presidência da República localizadas nas proximidades dos centros urbanos.
d)Desenvolver normas de conduta e fiscalização dos Responsáveis: Ministério da Justiça; Secretaria Especial
serviços de segurança privados que atuam na área rural. dos Direitos Humanos da Presidência da República
Responsável: Ministério da Justiça g)Fortalecer mecanismos para combater a violência contra
e)Elaborar diretrizes para atividades de policiamento a população indígena, em especial para as mulheres indígenas
comunitário e policiamento orientado para a solução de vítimas de casos de violência psicológica, sexual e de assédio
problemas, bem como catalogar e divulgar boas práticas moral.
dessas atividades. Responsáveis: Ministério da Justiça; Ministério da Saúde;
Responsável: Ministério da Justiça Secretaria Especial de Políticas para as Mulheres da
f)Elaborar diretrizes para atuação conjunta entre os Presidência da República
órgãos de trânsito e os de segurança pública para reduzir a h)Apoiar a implementação do pacto nacional de
violência no trânsito. enfrentamento à violência contra as mulheres de forma
Responsáveis: Ministério da Justiça; Ministério das articulada com os planos estaduais de segurança pública e em
Cidades conformidade com a Lei Maria da Penha (Lei no 11.340/2006).
g)Realizar debate sobre o atual modelo de repressão e Responsáveis: Ministério da Justiça; Secretaria Especial
estimular a discussão sobre modelos alternativos de de Políticas para as Mulheres da Presidência da República;
tratamento do uso e tráfico de drogas, considerando o Ministério da Saúde; Secretaria Especial dos Direitos Humanos
paradigma da redução de danos. da Presidência da República
Responsáveis: Ministério da Justiça; Secretaria Especial i)Avaliar o cumprimento da Lei Maria da Penha com base
dos Direitos Humanos da Presidência da República; Gabinete nos dados sobre tipos de violência, agressor e vítima.
de Segurança Institucional; Ministério da Saúde Responsáveis: Ministério da Justiça; Secretaria Especial
de Políticas para as Mulheres da Presidência da República
Objetivo estratégico V: j)Fortalecer ações estratégicas de prevenção à violência
Redução da violência motivada por diferenças de contra jovens negros.
gênero, raça ou etnia, idade, orientação sexual e situação Responsáveis: Secretaria Especial de Políticas de
de vulnerabilidade. Promoção da Igualdade Racial da Presidência da República;
Ministério da Justiça
Ações programáticas: k)Estabelecer política de prevenção de violência contra a
a)Fortalecer a atuação da Polícia Federal no combate e na população em situação de rua, incluindo ações de capacitação
apuração de crimes contra os Direitos Humanos. de policiais em Direitos Humanos.
Responsáveis: Ministério da Justiça; Secretaria Especial Responsáveis: Ministério da Justiça; Secretaria Especial
dos Direitos Humanos da Presidência da República dos Direitos Humanos da Presidência da República
b)Garantir aos grupos em situação de vulnerabilidade o l)Promover a articulação institucional, em conjunto com a
conhecimento sobre serviços de atendimento, atividades sociedade civil, para implementar o Plano de Ação para o
desenvolvidas pelos órgãos e instituições de segurança e Enfrentamento da Violência contra a Pessoa Idosa.
mecanismos de denúncia, bem como a forma de acioná-los. Responsáveis: Secretaria Especial dos Direitos Humanos
Responsáveis: Ministério da Justiça; Secretaria Especial da Presidência da República; Ministério da Justiça; Ministério
dos Direitos Humanos da Presidência da República; Secretaria do Desenvolvimento Social e Combate à Fome; Ministério da
Especial de Políticas para as Mulheres da Presidência da Saúde

Conhecimentos Específicos 76
Apostila Digital Licenciada para Alice Caroline Guarino dos Santos - alice.guarino@hotmail.com (Proibida a Revenda)
APOSTILAS OPÇÃO

m)Fomentar a implantação do serviço de recebimento e Responsáveis: Secretaria Especial dos Direitos Humanos
encaminhamento de denúncias de violência contra a pessoa da Presidência da República; Ministério do Turismo;
idosa em todas as unidades da Federação. Ministério da Justiça; Secretaria Especial de Políticas para as
Responsável: Secretaria Especial dos Direitos Humanos Mulheres da Presidência da República
da Presidência da República
n)Capacitar profissionais de educação e saúde para Diretriz 14: Combate à violência institucional, com
identificar e notificar crimes e casos de violência contra a ênfase na erradicação da tortura e na redução da
pessoa idosa e contra a pessoa com deficiência. letalidade policial e carcerária.
Responsáveis: Secretaria Especial dos Direitos Humanos
da Presidência da República; Ministério da Saúde; Ministério Objetivo estratégico I:
da Educação Fortalecimento dos mecanismos de controle do
o)Implementar ações de promoção da cidadania e Direitos sistema de segurança pública.
Humanos das lésbicas, gays, bissexuais, transexuais e travestis,
com foco na prevenção à violência, garantindo redes Ações programáticas:
integradas de atenção. a)Criar ouvidoria de polícia com independência para
Responsáveis: Ministério da Justiça; Secretaria Especial exercer controle externo das atividades das Polícias Federais
dos Direitos Humanos da Presidência da República e da Força Nacional de Segurança Pública, coordenada por um
ouvidor com mandato.
Objetivo estratégico VI: Responsáveis: Ministério da Justiça; Secretaria Especial
Enfrentamento ao tráfico de pessoas. dos Direitos Humanos da Presidência da República
b)Fortalecer a Ouvidoria do Departamento Penitenciário
Ações programáticas: Nacional, dotando-a de recursos humanos e materiais
a)Desenvolver metodologia de monitoramento, necessários ao desempenho de suas atividades, propondo sua
disseminação e avaliação das metas do Plano Nacional de autonomia funcional.
Enfrentamento ao Tráfico de Pessoas, bem como construir e Responsável: Ministério da Justiça
implementar o II Plano Nacional de Enfrentamento ao Tráfico c)Condicionar a transferência voluntária de recursos
de Pessoas. federais aos Estados e ao Distrito Federal ao plano de
Responsáveis: Ministério da Justiça; Ministério do implementação ou à existência de ouvidorias de polícia e do
Turismo; Secretaria Especial de Políticas para as Mulheres da sistema penitenciário, que atendam aos requisitos de
Presidência da República; Ministério do Trabalho e Emprego; coordenação por ouvidor com mandato, escolhidos com
Secretaria Especial dos Direitos Humanos da Presidência da participação da sociedade civil e com independência para sua
República atuação.
b)Estruturar, a partir de serviços existentes, sistema Responsável: Ministério da Justiça
nacional de atendimento às vítimas do tráfico de pessoas, de d)Elaborar projeto de lei para aperfeiçoamento da
reintegração e diminuição da vulnerabilidade, especialmente legislação processual penal, visando padronizar os
de crianças, adolescentes, mulheres, transexuais e travestis. procedimentos da investigação de ações policiais com
Responsáveis: Secretaria Especial dos Direitos Humanos resultado letal.
da Presidência da República; Secretaria Especial de Políticas Responsável: Ministério da Justiça; Secretaria Especial
para as Mulheres da Presidência da República; Ministério da dos Direitos Humanos da Presidência da República
Justiça e)Dotar as Corregedorias da Polícia Federal, da Polícia
c)Implementar as ações referentes a crianças e Rodoviária Federal e do Departamento Penitenciário Nacional
adolescentes previstas na Política e no Plano Nacional de de recursos humanos e materiais suficientes para o
Enfrentamento ao Tráfico de Pessoas. desempenho de suas atividades, ampliando sua autonomia
Responsáveis: Ministério da Justiça; Secretaria Especial funcional.
dos Direitos Humanos da Presidência da República Responsável: Ministério da Justiça
d)Consolidar fluxos de encaminhamento e monitoramento f)Fortalecer a inspetoria da Força Nacional de Segurança
de denúncias de casos de tráfico de crianças e adolescentes. Pública e tornar obrigatória a publicação trimestral de
Responsáveis: Ministério da Justiça; Secretaria Especial estatísticas sobre procedimentos instaurados e concluídos e
dos Direitos Humanos da Presidência da República; Secretaria sobre o número de policiais desmobilizados.
Especial de Políticas para as Mulheres da Presidência da Responsável: Ministério da Justiça
República g)Publicar trimestralmente estatísticas sobre
e)Revisar e disseminar metodologia para atendimento de procedimentos instaurados e concluídos pelas Corregedorias
crianças e adolescentes vítimas de tráfico. da Polícia Federal e da Polícia Rodoviária Federal, e sobre a
Responsável: Secretaria Especial dos Direitos Humanos quantidade de policiais infratores e condenados, por cargo e
da Presidência da República tipo de punição aplicada.
f)Fomentar a capacitação de técnicos da gestão pública, Responsável: Ministério da Justiça
organizações não governamentais e representantes das h)Publicar trimestralmente informações sobre pessoas
cadeias produtivas para o enfrentamento ao tráfico de mortas e feridas em ações da Polícia Federal, da Polícia
pessoas. Rodoviária Federal e da Força Nacional de Segurança Pública.
Responsável: Ministério do Turismo Responsável: Ministério da Justiça
g)Desenvolver metodologia e material didático para i)Criar sistema de rastreamento de armas e de veículos
capacitar agentes públicos no enfrentamento ao tráfico de usados pela Polícia Federal, Polícia Rodoviária Federal e Força
pessoas. Nacional de Segurança Pública, e fomentar a criação de sistema
Responsáveis: Secretaria Especial dos Direitos Humanos semelhante nos Estados e no Distrito Federal.
da Presidência da República; Ministério do Turismo; Responsável: Ministério da Justiça
Ministério da Justiça; Secretaria Especial de Políticas para as
Mulheres da Presidência da República
h)Realizar estudos e pesquisas sobre o tráfico de pessoas,
inclusive sobre exploração sexual de crianças e adolescentes.

Conhecimentos Específicos 77
Apostila Digital Licenciada para Alice Caroline Guarino dos Santos - alice.guarino@hotmail.com (Proibida a Revenda)
APOSTILAS OPÇÃO

Objetivo estratégico II: e)Estabelecer protocolo para a padronização de


Padronização de procedimentos e equipamentos do procedimentos a serem realizados nas perícias destinadas a
sistema de segurança pública. averiguar alegações de tortura.
Responsáveis: Ministério da Justiça; Secretaria Especial
Ações programáticas: dos Direitos Humanos da Presidência da República
a)Elaborar procedimentos operacionais padronizados f)Elaborar matriz curricular e capacitar os operadores do
para as forças policiais federais, com respeito aos Direitos sistema de segurança pública e justiça criminal para o combate
Humanos. à tortura.
Responsáveis: Ministério da Justiça; Secretaria Especial Responsáveis: Ministério da Justiça; Secretaria Especial
dos Direitos Humanos da Presidência da República dos Direitos Humanos da Presidência da República
b)Elaborar procedimentos operacionais padronizados g)Capacitar e apoiar a qualificação dos agentes da perícia
sobre revistas aos visitantes de estabelecimentos prisionais, oficial, bem como de agentes públicos de saúde, para a
respeitando os preceitos dos Direitos Humanos. identificação de tortura.
Responsáveis: Ministério da Justiça; Secretaria Especial Responsáveis: Ministério da Justiça; Secretaria Especial
dos Direitos Humanos da Presidência da República; Secretaria dos Direitos Humanos da Presidência da República
Especial de Políticas para as Mulheres da Presidência da h)Incluir na formação de agentes penitenciários federais
República curso com conteúdos relativos ao combate à tortura e sobre a
c)Elaborar diretrizes nacionais sobre uso da força e de importância dos Direitos Humanos.
armas de fogo pelas instituições policiais e agentes do sistema Responsáveis: Ministério da Justiça; Secretaria Especial
penitenciário. dos Direitos Humanos da Presidência da República
Responsáveis: Ministério da Justiça; Secretaria Especial i)Realizar campanhas de prevenção e combate à tortura
dos Direitos Humanos da Presidência da República nos meios de comunicação para a população em geral, além de
d)Padronizar equipamentos, armas, munições e veículos campanhas específicas voltadas às forças de segurança
apropriados à atividade policial a serem utilizados pelas forças pública, bem como divulgar os parâmetros internacionais de
policiais da União, bem como aqueles financiados com combate às práticas de tortura.
recursos federais nos Estados, no Distrito Federal e nos Responsável: Secretaria Especial dos Direitos Humanos
Municípios. da Presidência da República
Responsáveis: Ministério da Justiça; Secretaria Especial j)Estabelecer procedimento para a produção de relatórios
dos Direitos Humanos da Presidência da República anuais, contendo informações sobre o número de casos de
e)Disponibilizar para a Polícia Federal, a Polícia torturas e de tratamentos desumanos ou degradantes levados
Rodoviária Federal e para a Força Nacional de Segurança às autoridades, número de perpetradores e de sentenças
Pública munição, tecnologias e armas de menor potencial judiciais.
ofensivo. Responsável: Secretaria Especial dos Direitos Humanos
Responsável: Ministério da Justiça da Presidência da República

Objetivo estratégico III: Objetivo estratégico IV:


Consolidação de política nacional visando à Combate às execuções extrajudiciais realizadas por
erradicação da tortura e de outros tratamentos ou penas agentes do Estado.
cruéis, desumanos ou degradantes.
Ações programáticas:
Ações programáticas: a)Fortalecer ações de combate às execuções extrajudiciais
a)Elaborar projeto de lei visando a instituir o Mecanismo realizadas por agentes do Estado, assegurando a investigação
Preventivo Nacional, sistema de inspeção aos locais de dessas violações.
detenção para o monitoramento regular e periódico dos Responsáveis: Ministério da Justiça; Secretaria Especial
centros de privação de liberdade, nos termos do protocolo dos Direitos Humanos da Presidência da República
facultativo à convenção da ONU contra a tortura e outros b)Desenvolver e apoiar ações específicas para investigação
tratamentos ou penas cruéis, desumanos ou degradantes. e combate à atuação de milícias e grupos de extermínio.
Responsáveis: Ministério da Justiça; Secretaria Especial Responsáveis: Ministério da Justiça; Secretaria Especial
dos Direitos Humanos da Presidência da República; Ministério dos Direitos Humanos da Presidência da República
das Relações Exteriores; Diretriz 15: Garantia dos direitos das vítimas de
b)Instituir grupo de trabalho para discutir e propor crimes e de proteção das pessoas ameaçadas.
atualização e aperfeiçoamento da Lei no 9.455/1997, que
define os crimes de tortura, de forma a atualizar os tipos Objetivo estratégico I:
penais, instituir sistema nacional de combate à tortura, Instituição de sistema federal que integre os
estipular marco legal para a definição de regras unificadas de programas de proteção.
exame médico-legal, bem como estipular ações preventivas
obrigatórias como formação específica das forças policiais e Ações programáticas:
capacitação de agentes para a identificação da tortura. a)Propor projeto de lei para integração, de forma
Responsável: Secretaria Especial dos Direitos Humanos sistêmica, dos programas de proteção a vítimas e testemunhas
da Presidência da República ameaçadas, defensores de Direitos Humanos e crianças e
c)Promover o fortalecimento, a criação e a reativação dos adolescentes ameaçados de morte.
comitês estaduais de combate à tortura. Responsável: Secretaria Especial dos Direitos Humanos
Responsável: Secretaria Especial dos Direitos Humanos da Presidência da República
da Presidência da República b)Desenvolver sistema nacional que integre as
d)Propor projeto de lei para tornar obrigatória a filmagem informações das ações de proteção às pessoas ameaçadas.
dos interrogatórios ou audiogravações realizadas durante as Responsável: Secretaria Especial dos Direitos Humanos
investigações policiais. da Presidência da República
Responsáveis: Ministério da Justiça; Secretaria Especial c)Ampliar os programas de proteção a vítimas e
dos Direitos Humanos da Presidência da República testemunhas ameaçadas, defensores dos Direitos Humanos e
crianças e adolescentes ameaçados de morte para os Estados

Conhecimentos Específicos 78
Apostila Digital Licenciada para Alice Caroline Guarino dos Santos - alice.guarino@hotmail.com (Proibida a Revenda)
APOSTILAS OPÇÃO

em que o índice de violência aponte a criação de programas c)Desenvolver e aperfeiçoar os indicadores de morte
locais. violenta de crianças e adolescentes, assegurando publicação
Responsável: Secretaria Especial dos Direitos Humanos anual dos dados.
da Presidência da República Responsáveis: Secretaria Especial dos Direitos Humanos
d)Garantir a formação de agentes da Polícia Federal para a da Presidência da República; Ministério da Saúde
proteção das pessoas incluídas nos programas de proteção de d)Desenvolver programas de enfrentamento da violência
pessoas ameaçadas, observadas suas diretrizes. letal contra crianças e adolescentes e divulgar as experiências
Responsáveis: Ministério da Justiça; Secretaria Especial bem sucedidas.
dos Direitos Humanos da Presidência da República Responsáveis: Secretaria Especial dos Direitos Humanos
e)Propor ampliação os recursos orçamentários para a da Presidência da República; Ministério da Justiça
realização das ações dos programas de proteção a vítimas e
testemunhas ameaçadas, defensores dos Direitos Humanos e Objetivo estratégico IV:
crianças e adolescentes ameaçados de morte. Garantia de proteção dos defensores dos Direitos
Responsável: Secretaria Especial dos Direitos Humanos Humanos e de suas atividades.
da Presidência da República
Ações programáticas:
Objetivo estratégico II: a)Fortalecer a execução do Programa Nacional de Proteção
Consolidação da política de assistência a vítimas e a aos Defensores dos Direitos Humanos, garantindo segurança
testemunhas ameaçadas. nos casos de violência, ameaça, retaliação, pressão ou ação
arbitrária, e a defesa em ações judiciais de má-fé, em
Ações programáticas: decorrência de suas atividades.
a)Propor projeto de lei para aperfeiçoar o marco legal do Responsável: Secretaria Especial dos Direitos Humanos
Programa Federal de Assistência a Vítimas e Testemunhas da Presidência da República
Ameaçadas, ampliando a proteção de escolta policial para as b)Articular com os órgãos de segurança pública de Direitos
equipes técnicas do programa, e criar sistema de apoio à Humanos nos Estados para garantir a segurança dos
reinserção social dos usuários do programa. defensores dos Direitos Humanos.
Responsáveis: Ministério da Justiça; Secretaria Especial Responsáveis: Ministério da Justiça; Secretaria Especial
dos Direitos Humanos da Presidência da República dos Direitos Humanos da Presidência da República
b)Regulamentar procedimentos e competências para a c)Capacitar os operadores do sistema de segurança pública
execução do Programa Federal de Assistência a Vítimas e e de justiça sobre o trabalho dos defensores dos Direitos
Testemunhas Ameaçadas, em especial para a realização de Humanos.
escolta de seus usuários. Responsável: Secretaria Especial dos Direitos Humanos
Responsáveis: Ministério da Justiça; Secretaria Especial da Presidência da República
dos Direitos Humanos da Presidência da República d)Fomentar parcerias com as Defensorias Públicas dos
c)Fomentar a criação de centros de atendimento a vítimas Estados e da União para a defesa judicial dos defensores dos
de crimes e a seus familiares, com estrutura adequada e capaz Direitos Humanos nos processos abertos contra eles.
de garantir o acompanhamento psicossocial e jurídico dos Responsável: Secretaria Especial dos Direitos Humanos
usuários, com especial atenção a grupos sociais mais da Presidência da República
vulneráveis, assegurando o exercício de seus direitos. e)Divulgar em âmbito nacional a atuação dos defensores e
Responsáveis: Secretaria Especial dos Direitos Humanos militantes dos Direitos Humanos, fomentando cultura de
da Presidência da República; Secretaria Especial de Políticas respeito e valorização de seus papéis na sociedade.
para as Mulheres da Presidência da República Responsável: Secretaria Especial dos Direitos Humanos
d)Incentivar a criação de unidades especializadas do da Presidência da República
Serviço de Proteção ao Depoente Especial da Polícia Federal
nos Estados e no Distrito Federal. Diretriz 16: Modernização da política de execução
Responsável: Ministério da Justiça; Secretaria Especial penal, priorizando a aplicação de penas e medidas
dos Direitos Humanos da Presidência da República alternativas à privação de liberdade e melhoria do
e)Garantir recursos orçamentários e de infraestrutura ao sistema penitenciário.
Serviço de Proteção ao Depoente Especial da Polícia Federal,
necessários ao atendimento pleno, imediato e de qualidade aos Objetivo estratégico I:
depoentes especiais e a seus familiares, bem como o Reestruturação do sistema penitenciário.
atendimento às demandas de inclusão provisória no programa
federal. Ações programáticas:
Responsável: Ministério da Justiça a)Elaborar projeto de reforma da Lei de Execução Penal
(Lei no 7.210/1984), com o propósito de:
Objetivo estratégico III: - Adotar mecanismos tecnológicos para coibir a entrada de
Garantia da proteção de crianças e adolescentes substâncias e materiais proibidos, eliminando a prática de
ameaçados de morte. revista íntima nos familiares de presos;
- Aplicar a Lei de Execução Penal também a presas e presos
Ações programáticas: provisórios e aos sentenciados pela Justiça Especial;
a)Ampliar a atuação federal no âmbito do Programa de - Vedar a divulgação pública de informações sobre perfil
Proteção a Crianças e Adolescentes Ameaçados de Morte nas psicológico do preso e eventuais diagnósticos psiquiátricos
unidades da Federação com maiores taxas de homicídio nessa feitos nos estabelecimentos prisionais;
faixa etária. - Instituir a obrigatoriedade da oferta de ensino pelos
Responsável: Secretaria Especial dos Direitos Humanos estabelecimentos penais e a remição de pena por estudo;
da Presidência da República - Estabelecer que a perda de direitos ou a redução de
b)Formular política nacional de enfrentamento da acesso a qualquer direito ocorrerá apenas como consequência
violência letal contra crianças e adolescentes. de faltas de natureza grave;
Responsável: Secretaria Especial dos Direitos Humanos - Estabelecer critérios objetivos para isolamento de presos
da Presidência da República e presas no regime disciplinar diferenciado;

Conhecimentos Específicos 79
Apostila Digital Licenciada para Alice Caroline Guarino dos Santos - alice.guarino@hotmail.com (Proibida a Revenda)
APOSTILAS OPÇÃO

- Configurar nulidade absoluta dos procedimentos


disciplinares quando não houver intimação do defensor do Objetivo estratégico II:
preso; Limitação do uso dos institutos de prisão cautelar.
- Estabelecer o regime de condenação como limite para
casos de regressão de regime; Ações programáticas:
- Assegurar e regulamentar as visitas íntimas para a a)Propor projeto de lei para alterar o Código de Processo
população carcerária LGBT. Penal, com o objetivo de:
Responsável: Ministério da Justiça - Estabelecer requisitos objetivos para decretação de
b)Elaborar decretos extraordinários de indulto a prisões preventivas que consagrem sua excepcionalidade;
condenados por crimes sem violência real, que reduzam - Vedar a decretação de prisão preventiva em casos que
substancialmente a população carcerária brasileira. envolvam crimes com pena máxima inferior a quatro anos,
Responsável: Ministério da Justiça excetuando crimes graves como formação de quadrilha e
c)Fomentar a realização de revisões periódicas peculato;
processuais dos processos de execução penal da população - Estabelecer o prazo máximo de oitenta e um dias para
carcerária. prisão provisória.
Responsável: Ministério da Justiça Responsável: Ministério da Justiça
d)Vincular o repasse de recursos federais para construção b)Alterar a legislação sobre abuso de autoridade,
de estabelecimentos prisionais nos Estados e no Distrito tipificando de modo específico as condutas puníveis.
Federal ao atendimento das diretrizes arquitetônicas que Responsáveis: Ministério da Justiça; Secretaria Especial
contemplem a existência de alas específicas para presas dos Direitos Humanos da Presidência da República
grávidas e requisitos de acessibilidade.
Responsáveis: Ministério da Justiça; Secretaria Especial Objetivo estratégico III:
de Políticas para as Mulheres da Presidência da República Tratamento adequado de pessoas com transtornos
e)Aplicar a Política Nacional de Saúde Mental e a Política mentais.
para a Atenção Integral a Usuários de Álcool e outras Drogas
no sistema penitenciário. Ações programáticas:
Responsáveis: Ministério da Justiça; Ministério da Saúde a)Estabelecer diretrizes que garantam tratamento
f)Aplicar a Política Nacional de Atenção Integral à Saúde da adequado às pessoas com transtornos mentais, em
Mulher no contexto prisional, regulamentando a assistência consonância com o princípio de desinstitucionalização.
pré-natal, a existência de celas específicas e período de Responsáveis: Ministério da Justiça; Ministério da Saúde
permanência com seus filhos para aleitamento. b)Propor projeto de lei para alterar o Código Penal,
Responsáveis: Ministério da Justiça; Ministério da Saúde; prevendo que o período de cumprimento de medidas de
Secretaria Especial de Políticas para as Mulheres da segurança não deve ultrapassar o da pena prevista para o
Presidência da República crime praticado, e estabelecendo a continuidade do
g)Implantar e implementar as ações de atenção integral tratamento fora do sistema penitenciário quando necessário.
aos presos previstas no Plano Nacional de Saúde no Sistema Responsáveis: Ministério da Justiça; Ministério da Saúde
Penitenciário. c)Estabelecer mecanismos para a reintegração social dos
Responsável: Ministério da Justiça; Ministério da Saúde internados em medida de segurança quando da extinção desta,
h)Promover estudo sobre a viabilidade de criação, em mediante aplicação dos benefícios sociais correspondentes.
âmbito federal, da carreira de oficial de condicional, trabalho Responsáveis: Ministério da Justiça; Ministério da Saúde;
externo e penas alternativas, para acompanhar os condenados Ministério do Desenvolvimento Social e Combate à Fome
em liberdade condicional, os presos em trabalho externo, em
qualquer regime de execução, e os condenados a penas Objetivo estratégico IV:
alternativas à prisão. Ampliação da aplicação de penas e medidas
Responsáveis: Ministério da Justiça; Ministério do alternativas.
Planejamento, Orçamento e Gestão
i)Avançar na implementação do Sistema de Informações Ações programáticas:
Penitenciárias (InfoPen), financiando a inclusão dos a)Desenvolver instrumentos de gestão que assegurem a
estabelecimentos prisionais dos Estados e do Distrito Federal sustentabilidade das políticas públicas de aplicação de penas e
e condicionando os repasses de recursos federais à sua efetiva medidas alternativas.
integração ao sistema. Responsáveis: Ministério da Justiça
Responsável: Ministério da Justiça b)Incentivar a criação de varas especializadas e de centrais
j)Ampliar campanhas de sensibilização para inclusão de monitoramento do cumprimento de penas e medidas
social de egressos do sistema prisional. alternativas.
Responsável: Ministério da Justiça Responsável: Ministério da Justiça
k)Estabelecer diretrizes na política penitenciária nacional c)Desenvolver modelos de penas e medidas alternativas
que fortaleçam o processo de reintegração social dos presos, que associem seu cumprimento ao ilícito praticado, com
internados e egressos, com sua efetiva inclusão nas políticas projetos temáticos que estimulem a capacitação do cumpridor,
públicas sociais. bem como penas de restrição de direitos com controle de
Responsáveis: Ministério da Justiça; Ministério do frequência.
Desenvolvimento Social e Combate à Fome; Ministério da Responsável: Ministério da Justiça
Saúde; Ministério da Educação; Ministério do Esporte d)Desenvolver programas-piloto com foco na educação,
l)Debater, por meio de grupo de trabalho interministerial, para aplicação da pena de limitação de final de semana.
ações e estratégias que visem assegurar o encaminhamento Responsáveis: Ministério da Justiça; Ministério da
para o presídio feminino de mulheres transexuais e travestis Educação
que estejam em regime de reclusão.
Responsáveis: Ministério da Justiça; Secretaria Especial Diretriz 17: Promoção de sistema de justiça mais
dos Direitos Humanos da Presidência da República; Secretaria acessível, ágil e efetivo, para o conhecimento, a garantia e
Especial de Políticas para as Mulheres da Presidência da a defesa dos direitos.
República

Conhecimentos Específicos 80
Apostila Digital Licenciada para Alice Caroline Guarino dos Santos - alice.guarino@hotmail.com (Proibida a Revenda)
APOSTILAS OPÇÃO

Objetivo estratégico I: Objetivo estratégico III:


Acesso da população à informação sobre seus direitos Utilização de modelos alternativos de solução de
e sobre como garanti-los. conflitos.

Ações programáticas: Ações programáticas:


a)Difundir o conhecimento sobre os Direitos Humanos e a)Fomentar iniciativas de mediação e conciliação,
sobre a legislação pertinente com publicações em linguagem e estimulando a resolução de conflitos por meios
formatos acessíveis. autocompositivos, voltados à maior pacificação social e menor
Responsáveis: Ministério da Justiça; Secretaria Especial judicialização.
dos Direitos Humanos da Presidência da República Responsáveis: Ministério da Justiça; Ministério do
b)Fortalecer as redes de canais de denúncia (disque- Desenvolvimento Agrário; Ministério das Cidades
denúncia) e sua articulação com instituições de Direitos b)Fortalecer a criação de núcleos de justiça comunitária,
Humanos. em articulação com os Estados, o Distrito Federal e os
Responsável: Secretaria Especial dos Direitos Humanos Municípios, e apoiar o financiamento de infraestrutura e de
da Presidência da República capacitação.
c)Incentivar a criação de centros integrados de serviços Responsável: Ministério da Justiça
públicos para prestação de atendimento ágil à população, c)Capacitar lideranças comunitárias sobre instrumentos e
inclusive com unidades itinerantes para obtenção de técnicas de mediação comunitária, incentivando a resolução
documentação básica. de conflitos nas próprias comunidades.
Responsáveis: Secretaria Especial dos Direitos Humanos Responsáveis: Ministério da Justiça; Secretaria Especial
da Presidência da República; Ministério da Justiça dos Direitos Humanos da Presidência da República
d)Fortalecer o governo eletrônico com a ampliação da d)Incentivar projetos pilotos de Justiça Restaurativa, como
disponibilização de informações e serviços para a população forma de analisar seu impacto e sua aplicabilidade no sistema
via Internet, em formato acessível. jurídico brasileiro.
Responsáveis: Secretaria Especial dos Direitos Humanos Responsável: Ministério da Justiça
da Presidência da República e)Estimular e ampliar experiências voltadas para a solução
de conflitos por meio da mediação comunitária e dos Centros
Objetivo estratégico II: de Referência em Direitos Humanos, especialmente em áreas
Garantia do aperfeiçoamento e monitoramento das de baixo Índice de Desenvolvimento Humano (IDH) e com
normas jurídicas para proteção dos Direitos Humanos. dificuldades de acesso a serviços públicos.
Responsáveis: Secretaria Especial dos Direitos Humanos
Ações programáticas: da Presidência da República; Ministério da Justiça
a)Implementar o Observatório da Justiça Brasileira, em
parceria com a sociedade civil. Objetivo estratégico IV:
Responsável: Ministério da Justiça Garantia de acesso universal ao sistema judiciário.
b)Aperfeiçoar o sistema de fiscalização de violações aos
Direitos Humanos, por meio do aprimoramento do arcabouço
Ações programáticas:
de sanções administrativas.
Responsáveis: Secretaria Especial dos Direitos Humanos a)Propor a ampliação da atuação da Defensoria Pública da
da Presidência da República; Ministério da Saúde; Ministério União.
da Justiça; Ministério do Trabalho e Emprego Responsável: Ministério da Justiça
c)Ampliar equipes de fiscalização sobre violações dos b)Fomentar parcerias entre Municípios e entidades de
Direitos Humanos, em parceria com a sociedade civil. proteção dos Direitos Humanos para atendimento da
Responsáveis: Ministério da Justiça; Secretaria Especial população com dificuldade de acesso ao sistema de justiça,
dos Direitos Humanos da Presidência da República
com base no mapeamento das principais demandas da
d)Propor projeto de lei buscando ampliar a utilização das
ações coletivas para proteção dos interesses difusos, coletivos população local e no estabelecimento de estratégias para
e individuais homogêneos, garantindo a consolidação de atendimento e ações educativas e informativas.
instrumentos coletivos de resolução de conflitos. Responsáveis: Ministério da Justiça; Secretaria Especial
Responsável: Ministério da Justiça dos Direitos Humanos da Presidência da República
e)Propor projetos de lei para simplificar o processamento c)Apoiar a capacitação periódica e constante dos
e julgamento das ações judiciais; coibir os atos protelatórios; operadores do Direito e servidores da Justiça na aplicação dos
restringir as hipóteses de recurso ex officio e reduzir recursos
Direitos Humanos voltada para a composição de conflitos.
e desjudicializar conflitos.
Responsável: Ministério da Justiça Responsáveis: Ministério da Justiça; Secretaria Especial
f)Aperfeiçoar a legislação trabalhista, visando ampliar dos Direitos Humanos da Presidência da República
novas tutelas de proteção das relações do trabalho e as d)Dialogar com o Poder Judiciário para assegurar o efetivo
medidas de combate à discriminação e ao abuso moral no acesso das pessoas com deficiência à justiça, em igualdade de
trabalho. condições com as demais pessoas.
Responsáveis: Ministério do Trabalho e Emprego;
Responsáveis: Ministério da Justiça; Secretaria Especial
Ministério da Justiça; Secretaria Especial de Políticas para as
Mulheres da Presidência da República dos Direitos Humanos da Presidência da República
g)Implementar mecanismos de monitoramento dos e)Apoiar os movimentos sociais e a Defensoria Pública na
serviços de atendimento ao aborto legalmente autorizado, obtenção da gratuidade das perícias para as demandas
garantindo seu cumprimento e facilidade de acesso. judiciais, individuais e coletivas, e relacionadas a violações de
Responsáveis: Ministério da Saúde; Secretaria Especial de Direitos Humanos.
Políticas para as Mulheres da Presidência da República Responsável: Ministério da Justiça

Conhecimentos Específicos 81
Apostila Digital Licenciada para Alice Caroline Guarino dos Santos - alice.guarino@hotmail.com (Proibida a Revenda)
APOSTILAS OPÇÃO

Objetivo estratégico V: defesa dos Direitos Humanos, bem como da reparação das
Modernização da gestão e agilização do violações.
funcionamento do sistema de justiça. O PNDH-3 dialoga com o Plano Nacional de Educação em
Direitos Humanos (PNEDH) como referência para a política
Ações programáticas:
a)Propor legislação de revisão e modernização dos nacional de Educação e Cultura em Direitos Humanos,
serviços notariais e de registro. estabelecendo os alicerces a serem adotados nos âmbitos
Responsável: Ministério da Justiça nacional, estadual, distrital e municipal.
b)Desenvolver sistema integrado de informações do Poder O PNEDH, refletido neste programa, se desdobra em cinco
Executivo e Judiciário e disponibilizar seu acesso à sociedade. grandes áreas:
Responsável: Ministério da Justiça Na educação básica, a ênfase do PNDH-3 é possibilitar,
desde a infância, a formação de sujeitos de direito, priorizando
Objetivo estratégico VI:
Acesso à Justiça no campo e na cidade. as populações historicamente vulnerabilizadas. A troca de
experiências entre crianças de diferentes raças e etnias,
Ações programáticas: imigrantes, com deficiência física ou mental, fortalece, desde
a)Assegurar a criação de marco legal para a prevenção e cedo, sentimento de convivência pacífica. Conhecer o
mediação de conflitos fundiários urbanos, garantindo o devido diferente, desde a mais tenra idade, é perder o medo do
processo legal e a função social da propriedade. desconhecido, formar opinião respeitosa e combater o
Responsáveis: Ministério da Justiça; Ministério das
preconceito, às vezes arraigado na própria família.
Cidades
b)Propor projeto de lei voltado a regulamentar o No PNDH-3, essa concepção se traduz em propostas de
cumprimento de mandados de reintegração de posse ou mudanças curriculares, incluindo a educação transversal e
correlatos, garantindo a observância do respeito aos Direitos permanente nos temas ligados aos Direitos Humanos e, mais
Humanos. especificamente, o estudo da temática de gênero e orientação
Responsáveis: Ministério da Justiça; Ministério das sexual, das culturas indígena e afro-brasileira entre as
Cidades; Ministério do Desenvolvimento Agrário
disciplinas do ensino fundamental e médio.
c)Promover o diálogo com o Poder Judiciário para a
elaboração de procedimento para o enfrentamento de casos de No ensino superior, as metas previstas visam a incluir os
conflitos fundiários coletivos urbanos e rurais. Direitos Humanos, por meio de diferentes modalidades como
Responsáveis: Ministério das Cidades; Ministério da disciplinas, linhas de pesquisa, áreas de concentração,
Justiça; Ministério do Desenvolvimento Agrário transversalização incluída nos projetos acadêmicos dos
d) Propor projeto de lei para institucionalizar a utilização diferentes cursos de graduação e pós-graduação, bem como
da mediação nas demandas de conflitos coletivos agrários e em programas e projetos de extensão.
urbanos, priorizando a oitiva do INCRA, institutos de terras
A educação não formal em Direitos Humanos é orientada
estaduais, Ministério Público e outros órgãos públicos
especializados, sem prejuízo de outros meios institucionais pelos princípios da emancipação e da autonomia,
para solução de conflitos. (Redação dada pelo Decreto nº configurando-se como processo de sensibilização e formação
7.177, de 2010) da consciência crítica. Desta forma, o PNDH-3 propõe inclusão
Responsáveis: Ministério do Desenvolvimento Agrário; da temática de Educação em Direitos Humanos nos programas
Ministério da Justiça de capacitação de lideranças comunitárias e nos programas de
qualificação profissional, alfabetização de jovens e adultos,
Eixo Orientador V:
entre outros. Volta-se, especialmente, para o estabelecimento
Educação e cultura em Direitos Humanos
de diálogo e parcerias permanentes como o vasto leque
brasileiro de movimentos populares, sindicatos, igrejas, ONGs,
A educação e a cultura em Direitos Humanos visam à
clubes, entidades empresariais e toda sorte de agrupamentos
formação de nova mentalidade coletiva para o exercício da
da sociedade civil que desenvolvem atividades formativas em
solidariedade, do respeito às diversidades e da tolerância.
seu cotidiano.
Como processo sistemático e multidimensional que orienta a
formação do sujeito de direitos, seu objetivo é combater o
A formação e a educação continuada em Direitos Humanos,
preconceito, a discriminação e a violência, promovendo a
com recortes de gênero, relações étnico-raciais e de orientação
adoção de novos valores de liberdade, justiça e igualdade.
sexual, em todo o serviço público, especialmente entre os
A educação em Direitos Humanos, como canal estratégico
agentes do sistema de Justiça de segurança pública, são
capaz de produzir uma sociedade igualitária, extrapola o
fundamentais para consolidar o Estado Democrático e a
direito à educação permanente e de qualidade. Trata-se de
proteção do direito à vida e à dignidade, garantindo
mecanismo que articula, entre outros elementos: a) a
tratamento igual a todas as pessoas e o funcionamento de
apreensão de conhecimentos historicamente construídos
sistemas de Justiça que promovam os Direitos Humanos.
sobre Direitos Humanos e a sua relação com os contextos
Por fim, aborda-se o papel estratégico dos meios de
internacional, regional, nacional e local; b) a afirmação de
comunicação de massa, no sentido de construir ou
valores, atitudes e práticas sociais que expressem a cultura dos
desconstruir ambiente nacional e cultura social de respeito e
Direitos Humanos em todos os espaços da sociedade; c) a
proteção aos Direitos Humanos. Daí a importância primordial
formação de consciência cidadã capaz de se fazer presente nos
de introduzir mudanças que assegurem ampla democratização
níveis cognitivo, social, ético e político; d) o desenvolvimento
desses meios, bem como de atuar permanentemente junto a
de processos metodológicos participativos e de construção
todos os profissionais e empresas do setor (seminários,
coletiva, utilizando linguagens e materiais didáticos
debates, reportagens, pesquisas e conferências), buscando
contextualizados; e) o fortalecimento de políticas que gerem
ações e instrumentos em favor da promoção, da proteção e da

Conhecimentos Específicos 82
Apostila Digital Licenciada para Alice Caroline Guarino dos Santos - alice.guarino@hotmail.com (Proibida a Revenda)
APOSTILAS OPÇÃO

sensibilizar e conquistar seu compromisso ético com a e)Garantir a continuidade da "Mostra Cinema e Direitos
afirmação histórica dos Direitos Humanos. Humanos na América do Sul" e o "Festival dos Direitos
Humanos" como atividades culturais para difusão dos Direitos
Diretriz 18: Efetivação das diretrizes e dos princípios Humanos.
da política nacional de educação em Direitos Humanos Responsável: Secretaria Especial dos Direitos Humanos
para fortalecer cultura de direitos. da Presidência da República
f)Consolidar a revista "Direitos Humanos" como
Objetivo estratégico I: instrumento de educação e cultura em Direitos Humanos,
Implementação do Plano Nacional de Educação em garantindo o caráter representativo e plural em seu conselho
Direitos Humanos - PNEDH editorial.
Responsável: Secretaria Especial dos Direitos Humanos
da Presidência da República
Ações programáticas: g)Produzir recursos pedagógicos e didáticos
a)Desenvolver ações programáticas e promover especializados e adquirir materiais e equipamentos em
articulação que viabilizem a implantação e a implementação formato acessível para a educação em Direitos Humanos, para
do PNEDH. todos os níveis de ensino.
Responsáveis: Secretaria Especial dos Direitos Humanos Responsáveis: Secretaria Especial dos Direitos Humanos
da Presidência da República; Ministério da Educação; da Presidência da República; Ministério da Educação
Ministério da Justiça h)Publicar materiais pedagógicos e didáticos para a
b)Implantar mecanismos e instrumentos de educação em Direitos Humanos em formato acessível para as
monitoramento, avaliação e atualização do PNEDH, em pessoas com deficiência, bem como promover o uso da Língua
processos articulados de mobilização nacional. Brasileira de Sinais (Libras) em eventos ou divulgação em
Responsáveis: Secretaria Especial dos Direitos Humanos mídia.
da Presidência da República; Ministério da Educação; Responsáveis: Secretaria Especial dos Direitos Humanos
Ministério da Justiça da Presidência da República; Ministério da Educação.
c)Fomentar e apoiar a elaboração de planos estaduais e i)Fomentar o acesso de estudantes, professores e demais
municipais de educação em Direitos Humanos. profissionais da educação às tecnologias da informação e
Responsáveis: Secretaria Especial dos Direitos Humanos comunicação.
da Presidência da República; Ministério da Educação; Responsáveis: Secretaria Especial dos Direitos Humanos
Ministério da Justiça da Presidência da República; Ministério da Educação
d)Apoiar técnica e financeiramente iniciativas em
educação em Direitos Humanos, que estejam em consonância Diretriz 19: Fortalecimento dos princípios da
com o PNEDH. democracia e dos Direitos Humanos nos sistemas de
Responsáveis: Secretaria Especial dos Direitos Humanos educação básica, nas instituições de ensino superior e
da Presidência da República; Ministério da Educação; outras instituições formadoras.
Ministério da Justiça
e)Incentivar a criação e investir no fortalecimento dos Objetivo Estratégico I:
comitês de educação em Direitos Humanos em todos os Inclusão da temática de Educação e Cultura em
Estados e no Distrito Federal, como órgãos consultivos e Direitos Humanos nas escolas de educação básica e em
propositivos da política de educação em Direitos Humanos. outras instituições formadoras.
Responsáveis: Secretaria Especial dos Direitos Humanos
da Presidência da República; Ministério da Justiça Ações Programáticas:
a)Estabelecer diretrizes curriculares para todos os níveis e
Objetivo Estratégico II: modalidades de ensino da educação básica para a inclusão da
Ampliação de mecanismos e produção de materiais temática de educação e cultura em Direitos Humanos,
pedagógicos e didáticos para Educação em Direitos promovendo o reconhecimento e o respeito das diversidades
Humanos. de gênero, orientação sexual, identidade de gênero, geracional,
étnico-racial, religiosa, com educação igualitária, não
Ações programáticas: discriminatória e democrática.
a)Incentivar a criação de programa nacional de formação Responsáveis: Secretaria Especial dos Direitos Humanos
em educação em Direitos Humanos. da Presidência da República; Ministério da Educação;
Responsáveis: Secretaria Especial dos Direitos Humanos Secretaria Especial de Políticas para as Mulheres da
da Presidência da República; Ministério da Educação; Presidência da República
Ministério da Justiça; Secretaria Especial de Políticas para as b)Promover a inserção da educação em Direitos Humanos
Mulheres da Presidência da República nos processos de formação inicial e continuada de todos os
b)Estimular a temática dos Direitos Humanos nos editais profissionais da educação, que atuam nas redes de ensino e nas
de avaliação e seleção de obras didáticas do sistema de ensino. unidades responsáveis por execução de medidas
Responsáveis: Secretaria Especial dos Direitos Humanos socioeducativas.
da Presidência da República; Ministério da Educação; Responsáveis: Secretaria Especial dos Direitos Humanos
c)Estabelecer critérios e indicadores de avaliação de da Presidência da República; Ministério da Educação
publicações na temática de Direitos Humanos para o c)Incluir, nos programas educativos, o direito ao meio
monitoramento da escolha de livros didáticos no sistema de ambiente como Direito Humano.
ensino. Responsáveis: Ministério do Meio Ambiente; Secretaria
Responsáveis: Secretaria Especial dos Direitos Humanos Especial dos Direitos Humanos da Presidência da República;
da Presidência da República; Ministério da Educação Ministério da Educação
d)Atribuir premiação anual de educação em Direitos d)Incluir conteúdos, recursos, metodologias e formas de
Humanos, como forma de incentivar a prática de ações e avaliação da educação em Direitos Humanos nos sistemas de
projetos de educação e cultura em Direitos Humanos. ensino da educação básica.
Responsáveis: Secretaria Especial dos Direitos Humanos Responsáveis: Secretaria Especial dos Direitos Humanos
da Presidência da República; Ministério da Educação da Presidência da República; Ministério da Educação

Conhecimentos Específicos 83
Apostila Digital Licenciada para Alice Caroline Guarino dos Santos - alice.guarino@hotmail.com (Proibida a Revenda)
APOSTILAS OPÇÃO

e)Desenvolver ações nacionais de elaboração de didático, a organização de acervos históricos e a criação de


estratégias de mediação de conflitos e de Justiça Restaurativa centros de referências.
nas escolas, e outras instituições formadoras e instituições de Responsáveis: Secretaria Especial dos Direitos Humanos
ensino superior, inclusive promovendo a capacitação de da Presidência da República; Ministério da Educação;
docentes para a identificação de violência e abusos contra Ministério da Justiça
crianças e adolescentes, seu encaminhamento adequado e a e)Incentivar a realização de estudos, pesquisas e produção
reconstrução das relações no âmbito escolar. bibliográfica sobre a história e a presença das populações
Responsáveis: Secretaria Especial dos Direitos Humanos tradicionais.
da Presidência da República; Ministério da Educação; Responsáveis: Secretaria Especial dos Direitos Humanos
Ministério da Justiça da Presidência da República; Ministério da Educação;
f)Publicar relatório periódico de acompanhamento da Secretaria Especial de Políticas de Promoção da Igualdade
inclusão da temática dos Direitos Humanos na educação Racial da Presidência da República; Ministério da Justiça
formal que contenha, pelo menos, as seguintes informações:
- Número de Estados e Municípios que possuem planos de Objetivo Estratégico III:
educação em Direitos Humanos; Incentivo à transdisciplinariedade e transversalidade
- Existência de normas que incorporam a temática de nas atividades acadêmicas em Direitos Humanos.
Direitos Humanos nos currículos escolares;
- Documentos que atestem a existência de comitês de Ações Programáticas:
educação em Direitos Humanos; a)Incentivar o desenvolvimento de cursos de graduação,
- Documentos que atestem a existência de órgãos de formação continuada e programas de pós-graduação em
governamentais especializados em educação em Direitos Direitos Humanos.
Humanos. Responsáveis: Secretaria Especial dos Direitos Humanos
Responsável: Secretaria Especial dos Direitos Humanos da Presidência da República; Ministério da Educação;
da Presidência da República Secretaria Especial de Políticas de Promoção da Igualdade
g)Desenvolver e estimular ações de enfrentamento Racial da Presidência da República; Secretaria Especial de
ao bullying e ao cyberbulling. Políticas para as Mulheres da Presidência da República
Responsáveis: Secretaria Especial dos Direitos Humanos b)Fomentar núcleos de pesquisa de educação em Direitos
da Presidência da República; Ministério da Educação Humanos em instituições de ensino superior e escolas públicas
h)Implementar e acompanhar a aplicação das leis que e privadas, estruturando-as com equipamentos e materiais
dispõem sobre a inclusão da história e cultura afro-brasileira didáticos.
e dos povos indígenas em todos os níveis e modalidades da Responsáveis: Secretaria Especial dos Direitos Humanos
educação básica. da Presidência da República; Ministério da Educação;
Responsáveis: Secretaria Especial dos Direitos Humanos Ministério da Ciência e Tecnologia
da Presidência da República; Ministério da Educação c)Fomentar e apoiar, no Conselho Nacional de
Desenvolvimento Científico e Tecnológico (CNPq) e na
Objetivo Estratégico II: Coordenação de Aperfeiçoamento de Pessoal de Nível
Inclusão da temática da Educação em Direitos Superior (Capes), a criação da área "Direitos Humanos" como
Humanos nos cursos das Instituições de Ensino Superior . campo de conhecimento transdisciplinar e recomendar às
agências de fomento que abram linhas de financiamento para
Ações Programáticas: atividades de ensino, pesquisa e extensão em Direitos
a)Propor a inclusão da temática da educação em Direitos Humanos.
Humanos nas diretrizes curriculares nacionais dos cursos de Responsáveis: Secretaria Especial dos Direitos Humanos
graduação. da Presidência da República; Ministério da Educação;
Responsáveis: Secretaria Especial dos Direitos Humanos Ministério da Fazenda
da Presidência da República; Ministério da Educação d)Implementar programas e ações de fomento à extensão
b)Incentivar a elaboração de metodologias pedagógicas de universitária em direitos humanos, para promoção e defesa
caráter transdisciplinar e interdisciplinar para a educação em dos Direitos Humanos e o desenvolvimento da cultura e
Direitos Humanos nas Instituições de Ensino Superior. educação em Direitos Humanos.
Responsáveis: Secretaria Especial dos Direitos Humanos Responsáveis: Secretaria Especial dos Direitos Humanos
da Presidência da República; Ministério da Educação da Presidência da República; Ministério da Educação
c)Elaborar relatórios sobre a inclusão da temática dos
Direitos Humanos no ensino superior, contendo informações Diretriz 20: Reconhecimento da educação não formal
sobre a existência de ouvidorias e sobre o número de: como espaço de defesa e promoção dos Direitos Humanos.
- cursos de pós-graduação com áreas de concentração em
Direitos Humanos; Objetivo Estratégico I:
- grupos de pesquisa em Direitos Humanos; Inclusão da temática da educação em Direitos
- cursos com a transversalização dos Direitos Humanos nos Humanos na educação não formal.
projetos políticos pedagógicos;
- disciplinas em Direitos Humanos; Ações programáticas:
- teses e dissertações defendidas; a)Fomentar a inclusão da temática de Direitos Humanos na
- associações e instituições dedicadas ao tema e com as educação não formal, nos programas de qualificação
quais os docentes e pesquisadores tenham vínculo; profissional, alfabetização de jovens e adultos, extensão rural,
- núcleos e comissões que atuam em Direitos Humanos; educação social comunitária e de cultura popular.
- educadores com ações no tema Direitos Humanos; Responsáveis: Secretaria Especial dos Direitos Humanos
- projetos de extensão em Direitos Humanos; da Presidência da República; Ministério do Desenvolvimento
Responsáveis: Ministério da Educação; Secretaria Agrário; Secretaria Especial de Políticas de Promoção da
Especial dos Direitos Humanos da Presidência da República Igualdade Racial da Presidência da República; Ministério da
d)Fomentar a realização de estudos, pesquisas e a Cultura; Secretaria Especial de Políticas para as Mulheres da
implementação de projetos de extensão sobre o período do Presidência da República
regime 1964-1985, bem como apoiar a produção de material

Conhecimentos Específicos 84
Apostila Digital Licenciada para Alice Caroline Guarino dos Santos - alice.guarino@hotmail.com (Proibida a Revenda)
APOSTILAS OPÇÃO

b)Apoiar iniciativas de educação popular em Direitos Responsáveis: Secretaria Especial dos Direitos Humanos
Humanos desenvolvidas por organizações comunitárias, da Presidência da República; Ministério da Educação;
movimentos sociais, organizações não governamentais e Ministério da Justiça; Ministério da Saúde; Ministério do
outros agentes organizados da sociedade civil.
Planejamento, Orçamento e Gestão; Ministério das Relações
Responsáveis: Secretaria Especial dos Direitos Humanos
da Presidência da República; Secretaria Especial de Políticas Exteriores
de Promoção da Igualdade Racial da Presidência da República; b)Incentivar a inserção da temática dos Direitos Humanos
Secretaria Especial de Políticas para as Mulheres da nos programas das escolas de formação de servidores
Presidência da República; Ministério da Cultura; Ministério da vinculados aos órgãos públicos federais.
Justiça Responsáveis: Secretaria Especial dos Direitos Humanos
c)Apoiar e promover a capacitação de agentes da Presidência da República; Secretaria Especial de Políticas
multiplicadores para atuarem em projetos de educação em
para as Mulheres da Presidência da República; Secretaria
Direitos Humanos.
Responsável: Secretaria Especial dos Direitos Humanos Especial de Políticas de Promoção da Igualdade Racial da
da Presidência da República Presidência da República
d)Apoiar e desenvolver programas de formação em c)Publicar materiais didático-pedagógicos sobre Direitos
comunicação e Direitos Humanos para comunicadores Humanos e função pública, desdobrando temas e aspectos
comunitários. adequados ao diálogo com as várias áreas de atuação dos
Responsáveis: Secretaria Especial dos Direitos Humanos servidores públicos.
da Presidência da República; Ministério das Comunicações;
Responsáveis: Secretaria Especial dos Direitos Humanos
Ministério da Cultura
e)Desenvolver iniciativas que levem a incorporar a da Presidência da República; Ministério do Planejamento,
temática da educação em Direitos Humanos nos programas de Orçamento e Gestão
inclusão digital e de educação à distância.
Responsáveis: Secretaria Especial dos Direitos Humanos Objetivo Estratégico II:
da Presidência da República; Ministério da Educação; Formação adequada e qualificada dos profissionais do
Ministério das Comunicações; Ministério de Ciência e
sistema de segurança pública.
Tecnologia
f)Apoiar a incorporação da temática da educação em
Direitos Humanos nos programas e projetos de esporte, lazer Ações programáticas:
e cultura como instrumentos de inclusão social. a)Oferecer, continuamente e permanentemente, cursos em
Responsáveis: Secretaria Especial dos Direitos Humanos Direitos Humanos para os profissionais do sistema de
da Presidência da República; Ministério da Educação; segurança pública e justiça criminal.
Ministério da Cultura; Ministério do Esporte Responsáveis: Ministério da Justiça; Secretaria Especial
g)Fortalecer experiências alternativas de educação para os
dos Direitos Humanos da Presidência da República; Secretaria
adolescentes, bem como para monitores e profissionais do
sistema de execução de medidas socioeducativas. Especial de Políticas para as Mulheres da Presidência da
Responsáveis: Secretaria Especial dos Direitos Humanos República; Secretaria Especial de Políticas de Promoção da
da Presidência da República; Ministério da Educação; Igualdade Racial da Presidência da República
Ministério da Justiça b)Oferecer permanentemente cursos de especialização aos
gestores, policiais e demais profissionais do sistema de
Objetivo estratégico II: segurança pública.
Resgate da memória por meio da reconstrução da
Responsável: Ministério da Justiça
história dos movimentos sociais.
c)Publicar materiais didático-pedagógicos sobre
Ações programáticas: segurança pública e Direitos Humanos.
a)Promover campanhas e pesquisas sobre a história dos Responsáveis: Ministério da Justiça; Secretaria Especial
movimentos de grupos historicamente vulnerabilizados, tais dos Direitos Humanos da Presidência da República
como o segmento LGBT, movimentos de mulheres, d)Incentivar a inserção da temática dos Direitos Humanos
quebradeiras de coco, castanheiras, ciganos, entre outros.
nos programas das escolas de formação inicial e continuada
Responsáveis: Secretaria Especial dos Direitos Humanos
da Presidência da República; Secretaria Especial de Políticas dos membros das Forças Armadas.
para as Mulheres da Presidência da República Responsáveis: Secretaria Especial dos Direitos Humanos
b)Apoiar iniciativas para a criação de museus voltados ao da Presidência da República; Ministério da Defesa
resgate da cultura e da história dos movimentos sociais. e)Criar escola nacional de polícia para educação
Responsáveis: Ministério da Cultura; Secretaria Especial continuada dos profissionais do sistema de segurança pública,
dos Direitos Humanos da Presidência da República com enfoque prático.
Responsável: Ministério da Justiça
Diretriz 21: Promoção da Educação em Direitos
Humanos no serviço público. f)Apoiar a capacitação de policiais em direitos das
Objetivo Estratégico I: crianças, em aspectos básicos do desenvolvimento infantil e
em maneiras de lidar com grupos em situação de
Formação e capacitação continuada dos servidores vulnerabilidade, como crianças e adolescentes em situação de
públicos em Direitos Humanos, em todas as esferas de rua, vítimas de exploração sexual e em conflito com a lei.
governo. Responsáveis: Ministério da Justiça; Secretaria Especial
dos Direitos Humanos da Presidência da República
Ações programáticas:
a) Apoiar e desenvolver atividades de formação e
capacitação continuadas interdisciplinares em Direitos
Humanos para servidores públicos.

Conhecimentos Específicos 85
Apostila Digital Licenciada para Alice Caroline Guarino dos Santos - alice.guarino@hotmail.com (Proibida a Revenda)
APOSTILAS OPÇÃO

Diretriz 22: Garantia do direito à comunicação Responsáveis: Ministério das Comunicações; Secretaria
democrática e ao acesso à informação para consolidação Especial dos Direitos Humanos da Presidência da República
de uma cultura em Direitos Humanos. c)Incentivar a produção de filmes, vídeos, áudios e
similares, voltada para a educação em Direitos Humanos e que
reconstrua a história recente do autoritarismo no Brasil, bem
Objetivo Estratégico I: como as iniciativas populares de organização e de resistência.
Promover o respeito aos Direitos Humanos nos meios Responsáveis: Ministério das Comunicações; Secretaria
de comunicação e o cumprimento de seu papel na Especial dos Direitos Humanos da Presidência da República;
promoção da cultura em Direitos Humanos. Ministério da Cultura; Ministério da Justiça

Ações Programáticas: Eixo Orientador VI:


a) Propor a criação de marco legal, nos termos do art. 221 Direito à Memória e à Verdade
da Constituição, estabelecendo o respeito aos Direitos
Humanos nos serviços de radiodifusão (rádio e televisão) A investigação do passado é fundamental para a
concedidos, permitidos ou autorizados. (Redação dada pelo construção da cidadania. Estudar o passado, resgatar sua
decreto nº 7.177, de 2010) verdade e trazer à tona seus acontecimentos caracterizam
Responsáveis: Ministério das Comunicações; Secretaria forma de transmissão de experiência histórica, que é essencial
Especial dos Direitos Humanos da Presidência da República; para a constituição da memória individual e coletiva.
Ministério da Justiça; Ministério da Cultura O Brasil ainda processa com dificuldades o resgate da
b)Promover diálogo com o Ministério Público para memória e da verdade sobre o que ocorreu com as vítimas
proposição de ações objetivando a suspensão de programação atingidas pela repressão política durante o regime de 1964. A
e publicidade atentatórias aos Direitos Humanos. impossibilidade de acesso a todas as informações oficiais
Responsáveis: Ministério da Justiça; Secretaria Especial impede que familiares de mortos e desaparecidos possam
dos Direitos Humanos da Presidência da República conhecer os fatos relacionados aos crimes praticados e não
c)Suspender patrocínio e publicidade oficial em meios que permite à sociedade elaborar seus próprios conceitos sobre
veiculam programações atentatórias aos Direitos Humanos. aquele período.
Responsáveis: Ministério das Comunicações; Secretaria A história que não é transmitida de geração a geração
Especial dos Direitos Humanos da Presidência da República; torna-se esquecida e silenciada. O silêncio e o esquecimento
Ministério da Justiça das barbáries geram graves lacunas na experiência coletiva de
d) (Revogado pelo decreto nº 7.177, de 2010) construção da identidade nacional. Resgatando a memória e a
Responsáveis: (Revogado pelo decreto nº 7.177, de verdade, o País adquire consciência superior sobre sua própria
2010) identidade, a democracia se fortalece. As tentações totalitárias
e)Desenvolver programas de formação nos meios de são neutralizadas e crescem as possibilidades de erradicação
comunicação públicos como instrumento de informação e definitiva de alguns resquícios daquele período sombrio, como
transparência das políticas públicas, de inclusão digital e de a tortura, por exemplo, ainda persistente no cotidiano
acessibilidade. brasileiro.
Responsáveis: Ministério das Comunicações; Secretaria O trabalho de reconstituir a memória exige revisitar o
Especial dos Direitos Humanos da Presidência da República; passado e compartilhar experiências de dor, violência e
Ministério da Cultura; Ministério da Justiça mortes. Somente depois de lembrá-las e fazer seu luto, será
f)Avançar na regularização das rádios comunitárias e possível superar o trauma histórico e seguir adiante. A
promover incentivos para que se afirmem como instrumentos vivência do sofrimento e das perdas não pode ser reduzida a
permanentes de diálogo com as comunidades locais. conflito privado e subjetivo, uma vez que se inscreveu num
Responsáveis: Ministério das Comunicações; Secretaria contexto social, e não individual.
Especial dos Direitos Humanos da Presidência da República; A compreensão do passado por intermédio da narrativa da
Ministério da Cultura; Ministério da Justiça herança histórica e pelo reconhecimento oficial dos
g)Promover a eliminação das barreiras que impedem o acontecimentos possibilita aos cidadãos construírem os
acesso de pessoas com deficiência sensorial à programação em valores que indicarão sua atuação no presente. O acesso a
todos os meios de comunicação e informação, em todos os arquivos e documentos produzidos durante o regime
conformidade com o Decreto no 5.296/2004, bem como acesso militar é fundamental no âmbito das políticas de proteção dos
a novos sistemas e tecnologias, incluindo Internet. Direitos Humanos.
Responsáveis: Ministério das Comunicações; Secretaria Desde os anos 1990, a persistência de familiares de mortos
Especial dos Direitos Humanos da Presidência da República; e desaparecidos vem obtendo vitórias significativas nessa luta,
Ministério da Justiça com abertura de importantes arquivos estaduais sobre a
repressão política do regime ditatorial. Em dezembro de 1995,
Estratégico II: coroando difícil e delicado processo de discussão entre esses
Garantia do direito à comunicação democrática e ao familiares, o Ministério da Justiça e o Poder Legislativo
acesso à informação. Federal, foi aprovada a Lei no 9.140/95, que reconheceu a
responsabilidade do Estado brasileiro pela morte de
Ações Programáticas: opositores ao regime de 1964.
a)Promover parcerias com entidades associativas de Essa Lei instituiu Comissão Especial com poderes para
mídia, profissionais de comunicação, entidades sindicais e deferir pedidos de indenização das famílias de uma lista inicial
populares para a produção e divulgação de materiais sobre de 136 pessoas e julgar outros casos apresentados para seu
Direitos Humanos. exame. No art. 4o, inciso II, a Lei conferiu à Comissão Especial
Responsáveis: Secretaria Especial dos Direitos Humanos também a incumbência de envidar esforços para a localização
da Presidência da República; Ministério da Cultura; Ministério dos corpos de pessoas desaparecidas no caso de existência de
das Comunicações indícios quanto ao local em que possam estar depositados.
b)Incentivar pesquisas regulares que possam identificar
formas, circunstâncias e características de violações dos Em 24 de agosto de 2001, foi criada, pela Medida
Direitos Humanos na mídia. Provisória no 2151-3, a Comissão de Anistia do Ministério da
Justiça. Esse marco legal foi reeditado pela Medida Provisória

Conhecimentos Específicos 86
Apostila Digital Licenciada para Alice Caroline Guarino dos Santos - alice.guarino@hotmail.com (Proibida a Revenda)
APOSTILAS OPÇÃO

no 65, de 28 de agosto de 2002, e finalmente convertido na Lei Humanos (CIDH), denunciando o desaparecimento de
no 10.559, de 13 de novembro de 2002. Essa norma integrantes da "Guerrilha do Araguaia". Em 31 de outubro de
regulamentou o art. 8o do Ato das Disposições Constitucionais 2008, a CIDH expediu o Relatório de Mérito no 91/08, onde fez
Transitórias (ADCT) da Constituição de 1988, que previa a recomendações ao Estado brasileiro. Em 26 de março de 2009,
concessão de anistia aos que foram perseguidos em a CIDH submeteu o caso à Corte Interamericana de Direitos
decorrência de sua oposição política. Em dezembro de 2005, o Humanos, requerendo declaração de responsabilidade do
Governo Federal determinou que os três arquivos da Agência Estado brasileiro sobre violações de direitos humanos
Brasileira de Inteligência (ABIN) fossem entregues ao Arquivo ocorridas durante as operações de repressão àquele
Nacional, subordinado à Casa Civil, onde passaram a ser movimento.
organizados e digitalizados. Em 2005 e 2008, duas famílias iniciaram, na Justiça Civil,
Em agosto de 2007, em ato oficial coordenado pelo ações declaratórias para o reconhecimento das torturas
Presidente da República, foi lançado, pela Secretaria Especial sofridas por seus membros, indicando o responsável pelas
dos Direitos Humanos da Presidência da República e pela sevícias. Ainda em 2008, o Ministério Público Federal em São
Comissão Especial sobre Mortos e Desaparecidos Políticos, o Paulo propôs Ação Civil Pública contra dois oficiais do exército
livro-relatório "Direito à Memória e à Verdade", registrando os acusados de determinarem prisão ilegal, tortura, homicídio e
onze anos de trabalho daquela Comissão e resumindo a desaparecimento forçado de dezenas de cidadãos.
história das vítimas da ditadura no Brasil.
A trajetória de estudantes, profissionais liberais, Tramita também, no âmbito do Supremo Tribunal Federal,
trabalhadores e camponeses que se engajaram no combate ao Arguição de Descumprimento de Preceito Fundamental,
regime militar aparece como documento oficial do Estado proposta pelo Conselho Federal da Ordem dos Advogados do
brasileiro. O Ministério da Educação e a Secretaria Especial dos Brasil, que solicita a mais alta corte brasileira posicionamento
Direitos Humanos formularam parceria para criar portal que formal para saber se, em 1979, houve ou não anistia dos
incluirá o livro-relatório, ampliado com abordagem que agentes públicos responsáveis pela prática de tortura,
apresenta o ambiente político, econômico, social e homicídio, desaparecimento forçado, abuso de autoridade,
principalmente os aspectos culturais do período. Serão lesões corporais e estupro contra opositores políticos,
distribuídas milhares de cópias desse material em mídia considerando, sobretudo, os compromissos internacionais
digital para estudantes de todo o País. assumidos pelo Brasil e a insuscetibilidade de graça ou anistia
Em julho de 2008, o Ministério da Justiça e a Comissão de do crime de tortura.
Anistia promoveram audiência pública sobre "Limites e Em abril de 2009, o Ministério da Defesa, no contexto da
Possibilidades para a Responsabilização Jurídica dos Agentes decisão transitada em julgado da referida ação judicial de
Violadores de Direitos Humanos durante o Estado de Exceção 1982, criou Grupo de Trabalho para realizar buscas de restos
no Brasil", que discutiu a interpretação da Lei de Anistia de mortais na região do Araguaia, sendo que, por ordem expressa
1979 no que se refere à controvérsia jurídica e política, do Presidente da República, foi instituído Comitê
envolvendo a prescrição ou imprescritibilidade dos crimes de Interinstitucional de Supervisão, com representação dos
tortura. familiares de mortos e desaparecidos políticos, para o
A Comissão de Anistia já realizou setecentas sessões de acompanhamento e orientação dos trabalhos. Após três meses
julgamento e promoveu, desde 2008, trinta caravanas, de buscas intensas, sem que tenham sido encontrados restos
possibilitando a participação da sociedade nas discussões, e mortais, os trabalhos foram temporariamente suspensos
contribuindo para a divulgação do tema no País. Até 1o de devido às chuvas na região, prevendo-se sua retomada ao final
novembro de 2009, já haviam sido apreciados por essa do primeiro trimestre de 2010.
Comissão mais de cinquenta e dois mil pedidos de concessão Em maio de 2009, o Presidente da República coordenou o
de anistia, dos quais quase trinta e cinco mil foram deferidos e ato de lançamento do projeto Memórias Reveladas, sob
cerca de dezessete mil, indeferidos. Outros doze mil pedidos responsabilidade da Casa Civil, que interliga digitalmente o
aguardavam julgamento, sendo possível, ainda, a apresentação acervo recolhido ao Arquivo Nacional após dezembro de 2005,
de novas solicitações. Em julho de 2009, em Belo Horizonte, o com vários outros arquivos federais sobre a repressão política
Ministro de Estado da Justiça realizou audiência pública de e com arquivos estaduais de quinze unidades da federação,
apresentação do projeto Memorial da Anistia Política do Brasil, superando cinco milhões de páginas de documentos
envolvendo a remodelação e construção de novo edifício junto (www.memoriasreveladas.arquivonacional.gov.br).
ao antigo "Coleginho" da Universidade Federal de Minas Gerais Cabe, agora, completar esse processo mediante
(UFMG), onde estará disponível para pesquisas todo o acervo recolhimento ao Arquivo Nacional de todo e qualquer
da Comissão de Anistia. documento indevidamente retido ou ocultado, nos termos da
Portaria Interministerial assinada na mesma data daquele
No âmbito da sociedade civil, foram levadas ao Poder lançamento. Cabe também sensibilizar o Legislativo pela
Judiciário importantes ações que provocaram debate sobre a aprovação do Projeto de Lei no 5.228/2009, assinado pelo
interpretação das leis e a apuração de responsabilidades. Em Presidente da República, que introduz avanços
1982, um grupo de familiares entrou com ação na Justiça democratizantes nas normas reguladoras do direito de acesso
Federal para a abertura de arquivos e localização dos restos à informação.
mortais dos mortos e desaparecidos políticos no episódio Iimportância superior nesse resgate da história nacional
conhecido como "Guerrilha do Araguaia". Em 2003, foi está no imperativo de localizar os restos mortais de pelo
proferida sentença condenando a União, que recorreu e, menos cento e quarenta brasileiros e brasileiras que foram
posteriormente, criou Comissão Interministerial pelo Decreto mortos pelo aparelho de repressão do regime ditatorial. A
no4.850, de 2 de outubro de 2003, com a finalidade de obter partir de junho de 2009, a Secretaria de Comunicação Social da
informações que levassem à localização dos restos mortais de Presidência da República planejou, concebeu e veiculou
participantes da "Guerrilha do Araguaia". Os trabalhos da abrangente campanha publicitária de televisão, internet,
Comissão Interministerial encerraram-se em março de 2007, rádio, jornais e revistas de todo o Brasil buscando sensibilizar
com a divulgação de seu relatório final. os cidadãos sobre essa questão. As mensagens solicitavam que
Em agosto de 1995, o Centro de Estudos para a Justiça e o informações sobre a localização de restos mortais ou sobre
Direito Internacional (CEJIL) e a Human Rights qualquer documento e arquivos envolvendo assuntos da
Watch/América (HRWA), em nome de um grupo de familiares, repressão política entre 1964 e 1985 sejam encaminhados ao
apresentaram petição à Comissão Interamericana de Direitos Memórias Reveladas. Seu propósito é assegurar às famílias o

Conhecimentos Específicos 87
Apostila Digital Licenciada para Alice Caroline Guarino dos Santos - alice.guarino@hotmail.com (Proibida a Revenda)
APOSTILAS OPÇÃO

exercício do direito sagrado de prantear seus entes queridos e - No exercício de suas atribuições, a Comissão Nacional da
promover os ritos funerais, sem os quais desaparece a certeza Verdade poderá realizar as seguintes atividades:
da morte e se perpetua angústia que equivale a nova forma de - requisitar documentos públicos, com a colaboração das
tortura. respectivas autoridades, bem como requerer ao Judiciário o
acesso a documentos privados;
As violações sistemáticas dos Direitos Humanos pelo - colaborar com todas as instâncias do Poder Público para
Estado durante o regime ditatorial são desconhecidas pela a apuração de violações de Direitos Humanos, observadas as
maioria da população, em especial pelos jovens. A radiografia disposições da Lei no 6.683, de 28 de agosto de 1979;
dos atingidos pela repressão política ainda está longe de ser - promover, com base em seus informes, a reconstrução da
concluída, mas calcula-se que pelo menos cinquenta mil história dos casos de violação de Direitos Humanos, bem como
pessoas foram presas somente nos primeiros meses de 1964; a assistência às vítimas de tais violações;
cerca de vinte mil brasileiros foram submetidos a torturas e - promover, com base no acesso às informações, os meios
cerca de quatrocentos cidadãos foram mortos ou estão e recursos necessários para a localização e identificação de
desaparecidos. Ocorreram milhares de prisões políticas não corpos e restos mortais de desaparecidos políticos;
registradas, cento e trinta banimentos, quatro mil, oitocentos - identificar e tornar públicas as estruturas utilizadas para
e sessenta e duas cassações de mandatos políticos, uma cifra a prática de violações de Direitos Humanos, suas ramificações
incalculável de exílios e refugiados políticos. nos diversos aparelhos do Estado e em outras instâncias da
As ações programáticas deste eixo orientador têm como sociedade;
finalidade assegurar o processamento democrático e - registrar e divulgar seus procedimentos oficiais, a fim de
republicano de todo esse período da história brasileira, para garantir o esclarecimento circunstanciado de torturas, mortes
que se viabilize o desejável sentimento de reconciliação e desaparecimentos, devendo-se discriminá-los e encaminhá-
nacional. E para se construir consenso amplo no sentido de los aos órgãos competentes;
que as violações sistemáticas de Direitos Humanos registradas - apresentar recomendações para promover a efetiva
entre 1964 e 1985, bem como no período do Estado Novo, não reconciliação nacional e prevenir no sentido da não repetição
voltem a ocorrer em nosso País, nunca mais. de violações de Direitos Humanos.
- A Comissão Nacional da Verdade deverá apresentar,
Diretriz 23: Reconhecimento da memória e da verdade anualmente, relatório circunstanciado que exponha as
como Direito Humano da cidadania e dever do Estado. atividades realizadas e as respectivas conclusões, com base em
Objetivo Estratégico I: informações colhidas ou recebidas em decorrência do
exercício de suas atribuições.
Promover a apuração e o esclarecimento público das
violações de Direitos Humanos praticadas no contexto da Diretriz 24: Preservação da memória histórica e
repressão política ocorrida no Brasil no período fixado construção pública da verdade.
pelo art. 8o do ADCT da Constituição, a fim de efetivar o Objetivo Estratégico I:
direito à memória e à verdade histórica e promover a
reconciliação nacional. Incentivar iniciativas de preservação da memória
histórica e de construção pública da verdade sobre
Ação Programática: períodos autoritários.
a)Designar grupo de trabalho composto por
representantes da Casa Civil, do Ministério da Justiça, do Ações programáticas:
Ministério da Defesa e da Secretaria Especial dos Direitos a)Disponibilizar linhas de financiamento para a criação de
Humanos da Presidência da República, para elaborar, até abril centros de memória sobre a repressão política, em todos os
de 2010, projeto de lei que institua Comissão Nacional da Estados, com projetos de valorização da história cultural e de
Verdade, composta de forma plural e suprapartidária, com socialização do conhecimento por diversos meios de difusão.
mandato e prazo definidos, para examinar as violações de Responsáveis: Secretaria Especial dos Direitos Humanos
Direitos Humanos praticadas no contexto da repressão da Presidência da República; Ministério da Justiça; Ministério
política no período mencionado, observado o seguinte: da Cultura; Ministério da Educação
- O grupo de trabalho será formado por representantes da b)Criar comissão específica, em conjunto com
Casa Civil da Presidência da República, que o presidirá, do departamentos de História e centros de pesquisa, para
Ministério da Justiça, do Ministério da Defesa, da Secretaria reconstituir a história da repressão ilegal relacionada ao
Especial dos Direitos Humanos da Presidência da República, Estado Novo (1937-1945). Essa comissão deverá publicar
do presidente da Comissão Especial sobre Mortos e relatório contendo os documentos que fundamentaram essa
Desaparecidos Políticos, criada pela Lei no 9.140/95 e de repressão, a descrição do funcionamento da justiça de exceção,
representante da sociedade civil, indicado por esta Comissão os responsáveis diretos no governo ditatorial, registros das
Especial; violações, bem como dos autores e das vítimas.
- Com o objetivo de promover o maior intercâmbio de Responsáveis: Secretaria Especial dos Direitos Humanos
informações e a proteção mais eficiente dos Direitos Humanos, da Presidência da República; Ministério da Educação;
a Comissão Nacional da Verdade estabelecerá coordenação Ministério da Justiça; Ministério da Cultura
com as atividades desenvolvidas pelos seguintes órgãos: c) Identificar e tornar públicos as estruturas, os locais, as
- Arquivo Nacional, vinculado à Casa Civil da Presidência instituições e as circunstâncias relacionados à prática de
da República; violações de direitos humanos, suas eventuais ramificações
- Comissão de Anistia, vinculada ao Ministério da Justiça; nos diversos aparelhos estatais e na sociedade, bem como
- Comissão Especial criada pela Lei no 9.140/95, vinculada promover, com base no acesso às informações, os meios e
à Secretaria Especial dos Direitos Humanos da Presidência da recursos necessários para a localização e identificação de
República; corpos e restos mortais de desaparecidos políticos. (Redação
- Comitê Interinstitucional de Supervisão instituído pelo dada pelo Decreto nº 7.177, de 2010)
Decreto Presidencial de 17 de julho de 2009; Responsáveis: Secretaria Especial dos Direitos Humanos
- Grupo de Trabalho instituído pela Portaria no 567/MD, de da Presidência da República; Casa Civil da Presidência da
29 de abril de 2009, do Ministro de Estado da Defesa; República; Ministério da Justiça; Secretaria de Relações
Institucionais da Presidência da República

Conhecimentos Específicos 88
Apostila Digital Licenciada para Alice Caroline Guarino dos Santos - alice.guarino@hotmail.com (Proibida a Revenda)
APOSTILAS OPÇÃO

d)Criar e manter museus, memoriais e centros de Constitucionais Transitórias da Constituição de


documentação sobre a resistência à ditadura. 1988. (Redação dada pelo Decreto nº 7.177, de 2010)
Responsáveis: Secretaria Especial dos Direitos Humanos Responsáveis: Secretaria Especial dos Direitos Humanos
da Presidência da República; Ministério da Justiça; Ministério da Presidência da República; Ministério da Justiça
da Cultura; Secretaria de Relações Institucionais da
Presidência da República
e)Apoiar técnica e financeiramente a criação de 9. Famílias - transformações e
observatórios do Direito à Memória e à Verdade nas configurações;
universidades e em organizações da sociedade civil.
Responsáveis: Secretaria Especial dos Direitos Humanos
da Presidência da República; Ministério da Educação Família, Gênero e Assistência Social14
f) Desenvolver programas e ações educativas, inclusive a
produção de material didático-pedagógico para ser utilizado A família é, como ressalta Mioto, talvez o objeto de
pelos sistemas de educação básica e superior sobre graves intervenção profissional mais antigo do Serviço Social. Por
violações de direitos humanos ocorridas no período fixado no isso, e porque tem a ver com a nossa vivência e não apenas com
art. 8º do Ato das Disposições Constitucionais Transitórias da o nosso trabalho profissional, tendemos a dá-la como
Constituição de 1988. (Redação dada pelo Decreto nº 7.177, de facilmente compreendida, sem problematiza-la no sentido das
2010) múltiplas concepções ou práticas existentes. Isso contribui,
Responsáveis: Secretaria Especial dos Direitos Humanos muitas vezes, para a naturalização e funcionalização da família
da Presidência da República; Ministério da Educação; - forma peculiar de organização social e categoria analítica
Ministério da Justiça; Ministério da Cultura; Ministério de fundamental para o Serviço Social pensar sua intervenção.
Ciência e Tecnologia Esse processo pode ser observado de maneira tão ou mais
intensa, quando se trata das relações de gênero e, entendo,
Diretriz 25: Modernização da legislação relacionada assim como argumenta Grassi, que essa não é uma discussão
com promoção do direito à memória e à verdade, periférica, mas essencial para compreendermos a
fortalecendo a democracia. especificidade do Serviço Social como profissão.
O uso da linguagem inclusiva para demarcar a diferença
Objetivo Estratégico I: entre os homens e as mulheres é um bom exemplo para
Suprimir do ordenamento jurídico brasileiro observar esse processo, mais recentemente. Estamos, de
eventuais normas remanescentes de períodos de exceção alguma maneira, nos acostumando a ouvir a saudação “bom
que afrontem os compromissos internacionais e os dia a todos e todas”, gostaria de saudar “às” e “os” presentes e
preceitos constitucionais sobre Direitos Humanos. muitas vezes não nos damos conta da complexidade e
profundidade do que está por trás dessa sinalização – ou seja,
Ações Programáticas: do longo processo de invisibilização das mulheres e de seu
a)Criar grupo de trabalho para acompanhar, discutir e encapsulamento pelo masculino, dado como universal.
articular, com o Congresso Nacional, iniciativas de legislação
propondo: No caso do Serviço Social e da Assistência Social, também
- revogação de leis remanescentes do período 1964-1985 não é casual que aqui sejamos mulheres todas – quase - e
que sejam contrárias à garantia dos Direitos Humanos ou alguns homens. Como demonstram as reflexões de Estela
tenham dado sustentação a graves violações; Grassi e Mirla Cisne, não é casual que a Assistência Social tenha
- revisão de propostas legislativas envolvendo retrocessos como público principal as mulheres e que sejam também elas
na garantia dos Direitos Humanos em geral e no direito à a serem demandadas a arcar com responsabilidades ou suprir
memória e à verdade. necessidades sociais básicas, que são de responsabilidade da
Responsáveis: Secretaria Especial dos Direitos Humanos sociedade e dos poderes públicos. Isso tem a ver com uma
da Presidência da República; Ministério da Justiça; Secretaria construção social das relações sociais de gênero,
de Relações Institucionais da Presidência da República conservadoras e patriarcais, extremamente arraigadas na
b)Propor e articular o reconhecimento nossa sociedade e que se revelam de modo bastante claro na
do status constitucional de instrumentos internacionais de divisão sexual do trabalho.
Direitos Humanos novos ou já existentes ainda não ratificados. A noção de patriarcado nos remete à forma de organização
Responsáveis: Secretaria Especial dos Direitos Humanos das relações sociais onde o homem (e todos os atributos
da Presidência da República; Ministério da Justiça; Secretaria associados ao masculino) assume o lugar de referente
de Relações Institucionais da Presidência da República; universal e, nessa condição de único representante legítimo de
Ministério das Relações Exteriores todas as necessidades sociais, exerce a dominação e o poder
c) Fomentar debates e divulgar informações no sentido de sobre mulheres, crianças e velhos. Nesse contexto, a família
que logradouros, atos e próprios nacionais ou prédios públicos patriarcal e a ideia de “família” que se instala como universal,
não recebam nomes de pessoas identificadas estabelece modelos, legitima papéis e regula comportamentos
reconhecidamente como torturadores. (Redação dada pelo específicos a mulheres e homens. Garante a prevalência dessa
Decreto nº 7.177, de 2010) lógica subsumindo mulheres, crianças e velhos à autoridade e
Responsáveis: Ministério da Justiça; Secretaria Especial necessidades dos homens. Esse modelo não incorpora,
dos Direitos Humanos da Presidência da República; Casa Civil representa, nem diz todavia das necessidades das mulheres
da Presidência da República; Secretaria de Relações enquanto mulheres ou de crianças e velhos enquanto tais.
Institucionais da Presidência da República Esse é o primeiro ponto. Então, problematizar a família,
d) Acompanhar e monitorar a tramitação judicial dos abrir esse conceito, remete à necessidade de pensar a família
processos de responsabilização civil sobre casos que também pela ótica do patriarcado e do gênero: há que
envolvam graves violações de direitos humanos praticadas no examinar as políticas, as legislações, as práticas e dinâmicas,
período fixado no art. 8º do Ato das Disposições entre outros processos, levando em conta tais variáveis.

14 Texto de RODRIGUES, M. T. Assistente social, doutora em Pesquisas sobre a Mulher (NEPEM/UnB) e coordenadora do Grupo de
Sociologia pela Universidade de Brasília (2003), professora adjunta da Estudos em Gênero, Política Social e Serviços Sociais (GENPOSS/UnB).
Universidade de Brasília, pesquisadora do Núcleo de Estudos e

Conhecimentos Específicos 89
Apostila Digital Licenciada para Alice Caroline Guarino dos Santos - alice.guarino@hotmail.com (Proibida a Revenda)
APOSTILAS OPÇÃO

Vejamos por exemplo como a família está colocada nas pesquisa, pelo planejamento, e o desenvolvimento,
políticas e legislações nacionais recentes e qual a concepção implementação e monitoramento da política.
que prevalece nessas. Na maioria desses textos, a família é A partir desse marco, o desafio aqui proposto é pensar,
mencionada como direito e lugar da proteção, do afeto e da então, a incorporação da transversalidade de gênero nas ações
interação. Algumas políticas específicas formuladas tem, ainda e programas que integram a Assistência Social e
que perifericamente, apontado a família como lugar também particularmente os programas de transferência de renda, que
da construção ou da reiteração da desigualdade, da opressão e constituem hoje a grande iniciativa da política social
da violência, principalmente contra as crianças e as próprias relacionada à Assistência Social?
mulheres. Esse foi o principal argumento a alimentar a luta dos Esse desafio não é de pouca monta, haja vista, inclusive,
movimentos feminista e de mulheres, de defesa dos direitos que uma crítica que o movimento feminista formulou, ainda na
humanos entre outros e que, na década de 1990, logra ser década de 1970, em relação à política social – por seu papel
incorporada na agenda das políticas públicas. ativo como produtora e reprodutora de relações desiguais e
hierárquicas de gênero e da concepção de família associada a
No caso da Assistência Social, se formos dar uma olhada na essa ideia - só muito recentemente foi incorporada à
Política Nacional de Assistência Social de 2004, vamos Assistência Social. Segundo Esping-Andersen, a crítica
identificar esse lugar da família como um lugar de proteção e feminista, o declínio salarial da sociedade salarial e a crise do
também da opressão. Welfare State permitiram criar um caldo de cultura propício
Diz o texto da PNAS em sua análise situacional que, para a para a recolocação da família e a sua redescoberta como uma
“nova concepção de Assistência Social como direito à proteção questão central da discussão da política.
social”, a família, como o “núcleo de apoio primeiro” da pessoa, A incorporação tardia dessa reflexão feita por estudiosas
é uma das “três vertentes da proteção social” que, ao lado feministas, no campo da política social e no da Assistência
ainda das circunstâncias em que esta se encontra, devem ser Social, teve como consequência a prevalência, em larga escala,
consideradas “para a construção da política pública”. da perspectiva familista, como marca característica dessas
As transformações que atingem essa esfera da organização políticas. O que significa a atribuição às famílias da
social são assinaladas também no documento da PNAS que responsabilidade principal pelo bem-estar dos seus membros.
destaca, na seção dedicada à família e aos indivíduos, dentre as As reflexões feministas sobre família e trabalho mostra que
alterações mais relevantes a serem observadas, o crescimento é importante lembrar que, embora estejamos falando das
da participação feminina como pessoa de referência da família. famílias o tempo todo, a prevalência da perspectiva familista,
Não obstante, o que aparece e prevalece como a lógica da eivada pela lógica patriarcal, resulta que, ao se dizer “famílias”,
política não é isso; ao contrário, como expresso nas diretrizes se está referindo às mulheres, porque não são os homens que
da referida política, a família deve ser considerada central na têm que cuidar das famílias: levar as crianças ao posto de
“concepção e implementação dos benefícios, serviços, saúde para vacinar, cuidar da casa, supervisionar as tarefas da
programas e projetos”. Ou seja, mais que alvo da proteção, a escola, enfim.
família é demandada e considerada como sujeito ativo e Por outro lado, esse desafio é colocado pela própria
imprescindível para a proteção. realidade, em função de que essa situação de desigualdade
Considerando as iniciativas desenvolvidas pela política de relacionada ao gênero faz com que nós mulheres sejamos o
assistência e os dados sobre pobreza e violência doméstica no grupo populacional mais afetado pela pobreza. Segundo Melo
Brasil, por exemplo, fica claro que tais diretrizes parecem não (2005), a penúria feminina é revelada em sua crueza por meio
dar conta porém das condições concretas e reais em que se dos dados. Tanto a renda média como a mediana feminina são
encontram parte substantiva dos sujeitos sociais atendidos inferiores à masculina para todos os tipos de família. A
pela Assistência Social, originários majoritariamente das desigualdade é uma realidade para todas as mulheres e pode-
classes populares e submetidos a toda sorte de privação de se afirmar que ser mulher é quase sempre sinônimo de ser
serviços e direitos e, como enfatiza Fonseca (2004), com pobre, sobretudo se for negra ou parda.
práticas sociais referentes ao parentesco e à família bastante Ou seja, levar em consideração a desigualdade vivida pelas
diversificados. Ademais, como assinala Di Marco, no cotidiano, mulheres diante dos homens, e incorporar o enfoque de
essas diferentes dinâmicas familiares se revelam gênero no estudo da Assistência Social são decisivos no âmbito
simultaneamente espaços de afeto, acolhida, enfrentamento, da política social. É uma estratégia que torna possível
dominação e disputa de poder. demarcar a discriminação tanto pública como familiar,
Vou procurar, portanto, discutir brevemente a identificando-se a relação de poder e a distribuição desigual de
centralidade das relações de gênero na construção da recursos entre mulheres e homens nas duas esferas e
desigualdade social, como essa desigualdade se materializa na desenvolvendo estratégias para seu enfrentamento. Como
família e a importância da incorporação da transversalidade destaca Graciela Di Marco, é necessário decodificar as
de gênero pela política social de modo geral e pela Assistência situações dentro da família, pois, nesse espaço de coabitação,
Social, em particular. Com esse propósito, apresento alguns as pessoas mantêm relações assimétricas, em que prevalece o
dados de pesquisas realizadas pelo Fundo de sistema de autoridade.
Desenvolvimento das Nações Unidas para a Mulher (UNIFEM A Política de Assistência Social tem, assim, grandes
Brasil), em conjunto com o Instituto de Pesquisa Econômica desafios para contribuir com a promoção da igualdade entre
Aplicada (IPEA) e a Secretaria Especial das Mulheres. mulheres e homens e o combate à situação de pobreza
Finalmente, para tematizar a perspectiva prevalente na feminina.
política social brasileira contemporânea em relação à família, Na perspectiva da garantia de equidade de gênero, esse
e a título de exemplo, teço algumas considerações em torno de desafio implica superar padrões desiguais e hierárquicos de
algumas questões relacionadas ao Programa Bolsa Família, gênero, que permeiam a política. Só assim ela poderá se
que considero estratégias para problematizar o debate constituir efetivamente numa política de gênero, que
proposto. considere a diversidade dos processos de socialização de
É importante começar dizendo que se entende como homens e mulheres e suas consequências nas relações
transversalidade de gênero a consideração das desigualdades individual e coletiva, a longo de suas vidas, e garanta acesso
de gênero e desenvolvimento de ações de enfrentamento mais equitativo aos bens e recursos produzidos.
específicas para estas, em todas as normas e padrões legais e Uma observação geral dos dados colhidos pelas pesquisas
sociais, em todas as políticas de desenvolvimento, pela levadas a cabo pelo IPEA junto com o UNIFEM e, mais
recentemente com a participação da Secretaria Especial da

Conhecimentos Específicos 90
Apostila Digital Licenciada para Alice Caroline Guarino dos Santos - alice.guarino@hotmail.com (Proibida a Revenda)
APOSTILAS OPÇÃO

Mulheres, mostra claramente como as diferenças entre os essa situação não é bem assim. O corte racial mais uma vez
sexos é transformada em desigualdade a partir de uma aprofunda a desigualdade e desfavorece duplamente as
construção social do valor do feminino e do masculino, e como mulheres. A análise de dados agregados por raça e cor aponta
essa desigualdade está organicamente articulada na dimensão que, sejam homens ou mulheres, a população de não brancos
econômica à lógica capitalista. apresenta os piores indicadores de educação. Os níveis
educacionais mais baixos são observados entre essa
No caso da relação com a pobreza, esses dados mostram, população, enquanto os níveis educacionais mais altos são
primeiro, que a pobreza não afeta do mesmo modo e com a encontrados entre os brancos.
mesma intensidade homens e mulheres, brancos e negros. Quanto ao corte de sexo, constata-se uma faixa maior de
Como assinala Melo (2005) em outro estudo, as mulheres analfabetismo entre as mulheres do que entre os homens,
negras têm sido o grupo da população mais afetado pela ainda que, em comparação a estes, elas apresentem níveis
pobreza e pela falta de acesso aos bens e serviços no Brasil. educacionais mais elevados, em educação secundária ou
Além disso, as famílias que têm mulheres como referência, sem superior.
cônjuge e filhos pequenos, são as que apresentam os menores Em relação ao mercado de trabalho, é importante também
rendimentos mensais de toda a estrutura educacional assinalar que, embora seja visível e quantificável uma maior
nacional. Não é por acaso que ainda existem a discriminação presença das mulheres, elas não só permanecem com os
racial e a discriminação de classes. maiores níveis de desocupação como somos nós que
Dentre cerca de 90 milhões de pessoas negras, conforme ocupamos os piores postos de trabalho, os serviços mais
dados de 2000 do CENSO do IBGE, as mulheres negras precarizados e mais mal pagos. Nesse sentido, é fundamental
totalizavam um pouco mais de 43 milhões, o que representa lembrarmos de assinalar o trabalho doméstico remunerado,
24,1% da população total e 50% da população negra do Brasil. via de regra precário e não coberto por direitos. Embora a
Em relação à localização dessa população pobre, ela vai se legislação trabalhista exista, o percentual de empregadas
concentrar principalmente na área rural. Segundo esses dados, domésticas com carteira assinada no Brasil representa um
do estudo do IPEA com a UNIFEM, os pobres na área rural índice ínfimo. Segundo esses dados, somente 4,7% das
representam o triplo da população percebida na área urbana: trabalhadoras domésticas têm seus direitos trabalhistas
ou seja, 26% da população rural contra 8,4% da população reconhecidos.
urbana. Em consequência dessa desproporção, se pensarmos
em termos de Brasil, o que vamos ver é que a proporção de Além dessa situação no mercado de trabalho, as mulheres
negros pobres no Nordeste será mais que o dobro da ainda estão em situação desfavorável, na medida em que são
proporção de negros pobres no Sudeste. Considerando o que elas também o grupo majoritário na produção para consumo
essa condição de mulher negra e pobre representa em termos próprio. Por causa dos trabalhos domésticos, os dados
de discriminação, podemos dizer que há mais pessoas mostram que, no caso do trabalho doméstico, apenas 3% das
discrimináveis no Nordeste do que no Sudeste. mulheres são empregadoras, 25% têm registro na carteira de
O percentual de domicílios cujas responsáveis são trabalho. No caso dos homens, essa proporção sobe para 5%
mulheres reitera essa desproporção entre as regiões e ajuda a como empregadores e 35% com registro em carteira. Vou
explicar a situação de desvantagem geral experimentada pelo parar com esses dados, mas eles estão disponíveis na página
Nordeste em relação ao Sudeste. Isso porque, no Nordeste, há da Secretaria Especial das Mulheres.
uma maior proporção de domicílios cujas pessoas de Esse quadro revela, portanto, a importância do gênero ou
referência são mulheres, em relação, por exemplo, ao Sudeste. da construção social que se faz sobre o feminino e masculino,
Nesse sentido, é importante também assinalar que esse estudo e a desigualdade que se constrói em cima dessa diferença, o
já está na terceira edição e um dos achados que ele traz em que é um elemento fundamental para se entender e avaliar a
2008 é, por um lado, a tendência a uma manutenção desse Política de Assistência Social na perspectiva de melhoria das
crescimento das famílias com mulheres como pessoas de condições de vida e no aumento da autonomia das mulheres.
referência. O outro dado que o estudo mais recente traz e que, Há ainda que considerar a questão da divisão sexual do
embora quantitativamente não seja relevante, mas o é do trabalho e da organização do mercado e das profissões. É
ponto de vista qualitativo, é a emergência de famílias necessário percebermos também como nós mulheres somos
monoparentais, cujas pessoas de referência são homens. centrais na política, como mão-de-obra para a política
Apontam, assim, para uma luz no fim do túnel, de que estamos acontecer, mas não como objeto dessa política. Temos que
com muita dificuldade, realizando transformações no universo cuidar das crianças, dos idosos, assim como das pessoas
das relações de gênero. Essa desigualdade social pautada no portadoras de deficiência. Todavia, quem cuida de nós? As
gênero ainda se expressa nos campos econômico, do trabalho políticas de Assistência Social, por exemplo, consideram as
e da educação. Os dados mostram, ademais, que o fato de ser mulheres, via de regra como recurso e insumo para
mulher implica em desigualdade e hierarquia. implementação de suas ações. Raramente como objeto da
No caso da educação é interessante notar - porque talvez proteção social. Nas ocasiões em que tal ocorre, são ações que
seja a única área em que os indicadores para as mulheres costumam estar orientadas também para outro “tipo de
muitas vezes são mais positivos do que os homens, tanto no vulnerabilidade”, por exemplo a velhice, questões específicas
âmbito do estudo quanto no grau de escolaridade –, em que de saúde/doença.
pese a maior escolaridade, maior quantidade de anos de Essa reflexão pode ser estendida a nós assistentes sociais
estudo e qualificação das mulheres, que essa realidade não se também e às nossas condições de trabalho. Somos
traduz em melhor entrada no mercado de trabalho. Muito pelo demandadas, dentro do paradigma da “ideologia do amor”,
contrário. As mulheres continuam ganhando menos do que os como bem mostra Grassi (1989) a “cuidar’ de todo mundo,
homens em todas as funções, inclusive naquelas em que porém raramente podemos encontrar, nas estruturas das
exercemos as mesmas atividades e funções. A nossa trincheira instituições que nos emprega, quem cuide de nós e nos dê
nesse caso tem sido o serviço público, onde o assédio pode ser suporte institucional e profissional nos trabalhos de alta
denunciado e a discriminação pode ser contestada. Nos demais tensão e imenso desgaste mental e afetivo, que
espaços de trabalho, o que rege é o assédio moral, o assédio desenvolvemos. Sem contar com as dimensões da
sexual e a discriminação muitas vezes aberta. desigualdade de trabalho que se materializam no universo
‘doméstico”, que neste momento não temos condições de nos
No caso da população que chamamos “não branca”, porque estender, mas que é importante considerar, em vista desse
o IBGE trata pardos e indígenas como categoria “não branca”, perfil feminino do Serviço Social.

Conhecimentos Específicos 91
Apostila Digital Licenciada para Alice Caroline Guarino dos Santos - alice.guarino@hotmail.com (Proibida a Revenda)
APOSTILAS OPÇÃO

No caso do Serviço Social como profissão, parece-me que a Essas possibilidades não previstas ou fomentadas pelo
construção do nosso estatuto científico e acadêmico veio Bolsa Família são insuficientes, porém, para mudar seu caráter
acompanhado de uma assexualização do contingente de benefício temporário e não direito de cidadania. A
profissional e da profissão. Porque o processo de rejeição e repercussões dessa peculiaridade do programa não pode ser
superação da identidade historicamente construída da desconsiderada, principalmente levando-se em conta a cultura
assistentes sociais como “freiras sem hábito” ou “moças política clientelista e patrimonialista hegemônicas no Brasil.
boazinhas”, tem se feito acompanhar de uma invisibilização do Ou seja, o que prevalece entre nós não é cultura de direito; o
ponto de vista da diferença sexual, como se, enquanto sujeitos que temos, nem nós nem os nossos usuários, porque não
sociais e históricos, não fossemos também constituídas pela estamos fora dessa sociedade. Isso é uma cultura política
diferença de sexo/gênero e a desigualdade que sob esta se nacional.
constrói. Esse processo de assexualização aparece na E no caso do acesso das mulheres a essa renda, a
linguagem amplamente hegemônica dentro do serviço social. contradição está aí. As mulheres estão recebendo e foram
Embora numericamente sejamos hegemonicamente mulheres, colocadas no programa como receptoras privilegiadas do
nós somos comumente saudadas como “eles’, “os benefício e não do direito. O que significa que não é porque
companheiros assistentes sociais”, e assim por diante. Ou seja, pensam que tem que ser sujeitas dos direitos ou autônomas,
os outros. Ora, eu Marlene, não me identifico com esse mas porque são “boas mães”, e ai daquelas que não o forem.
universal, aí eu me coloco como o outro, a outra, a alteridade. Constrói-se, assim, um controle a partir da maternidade
No caso da Bolsa Família, esse processo é muito como mecanismo de exclusão e de discriminação, visto que
emblemático também. As demandas postas às mulheres aquelas mães que não se enquadram nesse modelo ideal de
beneficiárias lhes atribui lugar de mão-de-obra imprescindível maternidade, as chamadas “mães desnaturadas” e suas
à materialização do Programa: para controlar a frequência dos “famílias desestruturadas”, não merecem o benefício e o
meninos, para levá-los para tomar vacina, para serem acesso a política. Aquelas mulheres que não cuidam, dentro
cobradas e responsabilizadas, porque os meninos estão indo das expectativas maternalistas, são assim mantidas alheias aos
sujos para a escola ou porque não a estão frequentando. espaços dos serviços, sendo reiterada e frequentemente
Recentemente começou a ser implantado nas unidades básicas punidas por sua incapacidade e incompetência de prover os
de saúde a disponibilização obrigatória de exame preventivo cuidados associados à maternidade.
para as mulheres. Iniciativa simples e garantidora de direitos
básicos das mulheres; isso, por exemplo, não havia, até então, Nesse sentido, nós assistentes sociais, como profissionais
se colocado como questão de interesse das mulheres da Bolsa centrais na implantação de diferentes ações dessa política,
Família, para a equipe de gestão do Programa. temos um papel fundamental. Como têm demonstrado nossas
Como regra, porém, no caso da Assistência Social e das pesquisas recentes, quem chama essas mulheres às unidades
transferências de renda, isso não tem sido pensado. O lugar socioassistenciais, seja para doutriná-las e normatizá-las ou
pensado para as mulheres, é somente como cuidadoras e prover-lhes condições de acesso e usufruto de direitos e
responsáveis pela gestão da reprodução. Espécie de “vacas”, serviços, lá na ponta dos serviços, somos nós, não é a política
aquelas que têm que dar leite para o bezerro crescer.... ainda abstratamente, não. Somos nós que a operamos.
que o Programa mencione o desafio de enfrentar a pobreza Assim, embora a diretriz demanda da política,
intergeracional – o que caberá a nós, mulheres, em termos de institucionalmente, aponta para a necessidade de “capacitá-las
direitos e serviços? Vamos continuar sendo consideradas só para a autonomia”, treinando as famílias (leia-se as mulheres)
como recurso para cuidar de crianças, velhos e incapazes? E de modo que possam exercitar seu lugar “núcleo de apoio
sem tais funções, deixadas à própria sorte para morrer? Como primeiro” da pessoa a cuidar bem de sua prole. O que pode ser
será com as próximas gerações, os próximos? E as meninas que exemplificado pelas falas reproduzidas por beneficiárias
estão sendo cuidadas por esses programas? Que perspectivas entrevistadas, dando conta das orientações passadas em
encontram? A maternidade como destino inescapável e reuniões ocorridas em Centros de Referência de Assistência
totalizante é uma diretriz que transparece e marca fortemente Social, para a não utilização do benefício para a compra de
a política. Mas não é a única a se fazer presente; a contradição batom ou perfume, porque o dinheiro do Bolsa Família não
é parte da vida e da política. deve ser usado para isso, mas para comprar uniforme e
comida. Produtos considerados supérfluos, os desejos das
Em se tratando da Política de Assistência e do Programa mulheres e demandas são bobagens, supérfluos. Esse
Bolsa Família, cabe assinalar que a preponderância do viés raciocínio se estende também na forma de organizar e gerir o
maternalista e familista não exclui a existência se segmentos cotidiano integralmente. Qual é a autonomia que se acredita
atuantes no âmbito da política, no entanto, felizmente, há que essas mulheres podem ter para gerir o seu cotidiano e o
pessoas que estão preocupadas com isso. Exemplo disso são seu dinheiro?
esses dados que trago, resultado de uma pesquisa que teve
financiamento do próprio MDS para poder pensar o lugar das Como ouvi de uma beneficiária durante realização da
mulheres no programa. Não chegou lá ainda, mas tem gente pesquisa de campo, em Belo Horizonte, em 2006:
pensando a respeito, e isso é muito importante. Temos palestras com as assistentes sociais, que falam que
Além disso, a Secretaria Especial das Mulheres tem temos que gastar o dinheiro assim, e não de outro jeito, temos
procurado normatizar esse tipo de procedimento e fazer o que fazer assim com os meninos. Vamos lá na escola e a
monitoramento das iniciativas em andamento. diretora também puxa nossa orelha, chama a nossa atenção.
O Programa estabelece que as mulheres sejam as Penso que nós, assistentes sociais, nosso horizonte de
receptoras privilegiadas do benefício, determinação que tem compromisso ético-político, não nos permite mais engajarmo-
um potencial transformador imenso, apesar de sua nos nesse tipo de prática. A questão do uso dos recursos do
justificativa original ser de cunho maternalista e familista, Programa e das demandas de consumo das mulheres foi talvez
como já destacado antes. Não só porque o acesso à renda, nele uma das coisas mais impactantes para mim nos dados obtidos
concretizado, é um direito humano básico, mas porque as na pesquisa.
mulheres, extrapolando o reconhecimento dessa cidadania, de “Não, essas mulheres, agora que estão com esse dinheiro, é
consumo, dado por sua condição de beneficiária da Bolsa uma coisa, é um tal de tomar pinga, comprar batom, comprar
Família, têm demandado direitos e participação efetiva em perfume, e agora ainda cismam que têm que ter televisão de
espaços de decisão, seja no espaço chamado privado, seja no 29, o que elas estão achando?”, escutei em uma oportunidade.
denominado espaço público. Em uma entrevista uma das mulheres beneficiárias expressou

Conhecimentos Específicos 92
Apostila Digital Licenciada para Alice Caroline Guarino dos Santos - alice.guarino@hotmail.com (Proibida a Revenda)
APOSTILAS OPÇÃO

assim seu contentamento com o recebimento do benefício:


“Ah, que bacana eu ter esse dinheiro e poder comprar uma TV 10. Relações de Gênero e
de 29’’ lá nas Casas Bahia em 52 prestações!”. Intergeracionais;
Nesse mesmo sentido, chamo para o nosso campo de
responsabilidade e da relativa autonomia profissional de que Uma imagem idealizada de família influencia o debate
nos fala Iamamoto (2001) e Guerra (2004), sem nenhum sobre o “conflito entre gerações”15 e a solidariedade
messianismo de superdimensionar, enfatizando a capacidade intrafamiliar. A solidariedade entre os membros da família é
e a autonomia profissional do serviço social no contexto tida como dada, desconsiderando-se as contradições e
institucional. É imprescindível que modifiquemos nossa conflitos em nível de tensões emocionais e de condições do
perspectiva enquanto assistentes sociais e mulheres, cotidiano. Existe um consenso de que as famílias representam
alterando e reorientando nossas ações, repensando nossos a arena onde as relações se desenvolvem, e os mecanismos de
referenciais e categorias teóricas, no sentido de tornar apoio são moldados pelos valores e pelas experiências que
convergente nossa prática profissional com nosso projeto envolvem ou são modificados ao longo de todo o ciclo de vida.
ético-político. A desnaturalização de categorias analíticas, No entanto, o que parece difícil de ser incorporado ao debate é
como a família, e a problematização de atividades o importante conceito de família proposto por Amartya Sen, de
tradicionalmente desenvolvidas pelo serviço social, como que “família é um espaço de conflito cooperativo”, bem como o
reuniões, visitas domiciliares e entrevistas, a partir daquele, é fato de que existe uma diversidade de arranjos sexuais, nos
por isso urgente e imprescindível. Não se trata daquela ideia quais o gênero e a idade definem, em grande medida, a
de metodologismo e tecnicismo que tivemos lá atrás, não. Mas trajetória de vida individual e o estágio do ciclo de vida
temos que recuperar isso, porque senão vamos voltar para o familiar.
empirismo da pior espécie. As famílias estão mudando tanto em termos de forma
As atividades socioeducativas e também socioassistenciais quanto de significado, expandindo-se para além da estrutura
nesse universo são extremamente relevantes, porque, muito das famílias nucleares, de forma a abarcar uma rede de
longe da nossa realidade, essas mulheres estão absolutamente relações de parentesco e de não-parentes gerada pelos
enclausuradas no espaço doméstico. São mulheres casamentos, divórcios e recasamentos. As fontes de dados
desprovidas de renda e sem condição nenhuma de circulação. secundários sobre as famílias são cada vez menos capazes de
Elas circulam na vizinhança, na igreja e na televisão. E só. oferecer um bom quadro do que ocorre dentro destas. As
Oportunizar espaços de interação e articulação torna-se assim informações sobre a diversidade dos arranjos sexuais, a
questão de relevância para essas pessoas e deve ser uma organização das relações de gênero e dos serviços domésticos
questão pensada nesse quadro. e de responsabilidades pelo cuidado dos filhos raramente
estão disponíveis no Brasil. Tudo isso tem servido de
Ou seja, o Programa tem a possibilidade de fomentar, de justificativa para que se continue a utilizar o casal
modo importante, a possibilidade de entrada na esfera pública heterossexual com (poucos) filhos como sinônimo de família-
e de constituição das mulheres beneficiárias como sujeitas. modelo para a formulação de políticas.
Embora as dimensões inscritas no Bolsa Família se limitem a As mudanças nos domicílios brasileiros são visíveis,
garantir o acesso ao dinheiro e à cidadania, fundamentalmente mesmo utilizando as fontes de dados tradicionais, e estas
mostram que o tipo de família e o estágio do ciclo de vida da
pela via do como consumo. Ou seja, o acesso das mulheres aos
família são aspectos importantes para a determinação da
serviços não se pauta fundamentalmente porque elas têm qualidade de vida de seus membros. Com uma taxa de
direitos. Essa cidadania tem se construído porque, como fecundidade de aproximadamente 2 ,4 em 2001 e expressivos
receptora dos recursos financeiros, ela se torna consumidora, segmentos da população urbana sob um regime de
abre crédito e paga. Ela abre conta no botequim, negocia as fecundidade abaixo do nível de reposição, é difícil atribuir a
contas. Constrói a casa pagando infindáveis prestações nas pobreza das famílias brasileiras a um grande número de filhos.
pequenas lojas do comércio local, tendo que comprar meia Esse argumento é reforçado pelas evidências de que as
desigualdades salariais representam a principal explicação
dúzia de tijolos por mês. Puxadinho para cá, puxadinho para lá
para as diferenças de renda entre as famílias brasileiras ricas
e, quando você vê, ela tem uma casa. Com 170 reais, 220 reais? e não-ricas.
Como? Faz? Faz! Claro que faz. Então, na condição de
consumidora, ela vira cidadã. Cidadania de consumo, porém. Em 2001, ao redor de 80% da população brasileira viviam
Pensar e tensionar tais iniciativas é, pois, o desafio que nossos em áreas urbanas e em domicílios organizados em torno do
compromissos ético-políticos nos impõem. grupo de parentesco. Dos 39,6 milhões de domicílios urbanos,
aproximadamente 86% estavam organizados em torno de
Trazer esse debate aqui é, para mim, parte desse esforço e,
diferentes tipos de famílias e os outros 14% eram do tipo “não-
como tal, de suma importância. Mas temos que sair do gueto. famílias”, ou seja, formados por indivíduos que viviam
Problematizar e incorporar as discussões a partir das sozinhos ou por grupos de indivíduos sem relações de
desigualdades de gênero é imprescindível, não apenas com as parentesco e onde não há um núcleo conjugal. O crescimento
amigas, companheiras e queridas assistentes sociais no número dos domicílios nas últimas décadas se deve,
feministas. É importante que travemos esse debate de modo principalmente, ao aumento dos não-familiares e
mais amplo no serviço social, porque não é uma questão particularmente das pessoas que vivem sozinhas.
No período 1990-2001 os domicílios com famílias
particular. Isso não é uma questão periférica, mas central para
aumentaram ao redor de 45 %, enquanto os arranjos “não-
a Política de Assistência Social e o Serviço Social. familiares” aumentaram em cerca de 75%. Entre os domicílios
familiares, predominam aqueles arranjos do tipo nuclear
(81% e 80%), comparativamente às famílias extensas (18% e
19%) e às famílias complexas (1,3% e 0,9%). Em termos dos

15 Texto adaptado de GOLDANI, A. M. Relações intergeracionais e

reconstrução do estado de bem-estar. Por que se deve repensar essa


relação para o Brasil?

Conhecimentos Específicos 93
Apostila Digital Licenciada para Alice Caroline Guarino dos Santos - alice.guarino@hotmail.com (Proibida a Revenda)
APOSTILAS OPÇÃO

estágios do ciclo de vida familiar, embora mais da metade das a) o aumento da longevidade, que vem acompanhado de
famílias esteja no estágio de expansão e/ou de consolidação uma probabilidade maior de os indivíduos enfrentarem
(55% em 1990 e 52% em 2001), uma grande proporção de incapacidades físicas ou mentais por períodos mais longos;
famílias brasileiras é “madura”, ou seja, aproximadamente b) a maior instabilidade das relações e os divórcios nas
23% e 29% das famílias são constituídas por um ou por ambos últimas décadas, que contribuem para enfraquecer a habilidade
os pais sem filhos com menos de 19 anos de idade ou por um das famílias nucleares em prover o apoio aos seus dependentes;
casal sem filhos em que a mulher (esposa ou chefe da família) e
tinha mais de 40 anos. Por outro lado, somente 22% e 19% das c) a revolução reprodutiva, através da qual a reprodução foi
famílias encontram-se na fase inicial de sua formação, separada não apenas dos casamentos mas também do gênero,
respectivamente em 1990 e 2001. da idade e do próprio ato sexual, desafia as tradicionais
Dados os propósitos deste capítulo, destacamos as definições de direitos e responsabilidades de pais e filhos e a
tendências e a diversidade dos domicílios familiares com noção tradicional de parentesco. Tudo isso redefine o “contrato
filhos. As famílias com filhos representavam implícito” nas famílias, pelo qual os pais cuidam dos filhos e
aproximadamente 77% e 74% do total de domicílios em 1990 esperam ser cuidados por eles nas idades avançadas. As relações
e em 2001, respectivamente. De acordo com as Pesquisas de cuidado dentro das famílias são cada vez mais importantes
Nacionais por Amostra de Domicílios (PNAD) de 1990 e de para as mulheres, principais cuidadoras, na maioria das
2001, algumas das principais tendências e mudanças nas sociedades. Estas aumentaram sua participação no mercado de
famílias com filhos são: trabalho e a intensidade de suas atividades de cuidar enquanto
1. A maioria das crianças vive em famílias com os pais os serviços públicos de apoio diminuíram.
presentes, embora haja um crescente aumento de crianças
vivendo em famílias monoparentais. A distribuição das No Brasil, a Constituição de 1988 considera
famílias com crianças (20.500 em 1990 e 29.209 em 2001) responsabilidade da família, da sociedade e do Estado dar
mostra que o tipo biparental nuclear representa 67% e 63% e suporte aos idosos, assegurar sua participação na comunidade,
o monoparental nuclear, 12 ,4% e 15,6%. Outros dois tipos de defender sua dignidade e bem-estar, bem como garantir o seu
famílias com crianças são as famílias extensas (19,3 % e direito à vida (artigo 230). No primeiro parágrafo desse artigo
20,3%) e “complexas” (1,3% e 1,0%), respectivamente, em constitucional é dito também que os programas de apoio para
1990 e 2001. os idosos devem ser realizados, preferencialmente, dentro de
2. As famílias nucleares biparentais com filhos apresentam seus domicílios. Assim que, na atribuição de
um número médio de filhos de 2,4 e 2,1, respectivamente, em responsabilidades, a família vem primeiro. Novas leis e
1990 e 2001, e encontram-se entre as famílias mais pobres. diversas medidas práticas foram empreendidas pelo Estado
Mais de 1/3 destas eram pobres ou indigentes em 1990, 38 % visando proteger a população idosa contra a discriminação, a
em relação a 33% em 2001. Essa situação de pobreza foi ainda violência e as dificuldades econômicas. A Política Nacional do
maior para famílias em certos estágios de desenvolvimento do Idoso, de 1994, e o Estatuto do Idoso de 2003, são alguns
ciclo familiar. É na fase de “expansão” do seu ciclo de vida que exemplos dessas medidas legais.
as famílias nucleares biparentais experimentam suas maiores O alargamento das fronteiras do grupo de parentesco é um
dificuldades econômicas. Aproximadamente 43%, em ambos outro elemento a considerar no debate sobre as relações
os anos, eram pobres. intergeracionais e as demandas por recursos e cuidados entre
3. As menores proporções de famílias nucleares os membros das famílias. Resultado das transformações
biparentais pobres e indigentes em 2001 sugerem alguma demográficas, particularmente do aumento da esperança de
melhoria em suas condições na última década. Entretanto, vale vida e dos novos padrões de nupcialidade, casamentos,
destacar que as desigualdades internas não se alteraram, divórcios e recasamentos, os limites do grupo de parentesco se
tendo mesmo se acentuado. Por exemplo, aumentou a ampliaram de forma a incluir novos membros nas famílias.
concentração dessas famílias nos dois quintis mais baixos da Isso dá origem a uma rede complexa de relacionamentos ou a
distribuição de renda em 2001 (24% e 23%) na comparação uma “matriz latente de parentesco” que Riley e Riley Jr.
com o ano de 1990 (20% e 21%). definiram como sendo uma rede flexível potencial que se
4. A desigualdade de renda entre famílias biparentais poderia ativar para intensificar relações pessoais. Essa “matriz
reflete, também, as desigualdades raciais e regionais. Um terço latente de parentesco” pode ser vista como uma rede de
das famílias brancas com crianças estava concentrado no proteção com significativas conexões para escolher em caso de
quintil superior em 2001, enquanto menos de 10% de famílias necessidade. No futuro, ela pode conter estruturas relacionais
de negros e pardos encontravam-se nessa situação. As inimagináveis em conseqüência da revolução reprodutiva. Por
comparações regionais mostram que enquanto 1/4 das exemplo, os filhos in vitro podem ter até cinco pais, não
famílias biparentais em São Paulo encontra-se nos quintis contando com mudanças de possíveis novos casamentos: uma
mais elevados da distribuição de renda, a proporção mãe doadora, uma mãe de nascimento, uma mãe social
comparável para as famílias da região Nordeste é de apenas (responsável pela criação da criança), um pai doador e um pai
3%. social.
5. As desigualdades de raça e entre as regiões no que se Paradoxalmente, as múltiplas e potenciais novas conexões
refere à distribuição de renda piorou entre as famílias dentro de um grupo de parentesco ampliado, não
nucleares biparentais com filhos entre 1990 e 2001. Ainda que necessariamente contribuiriam para exacerbar conflitos entre
tenha se verificado uma crescente concentração de todas as as gerações. Ao contrário, Riley e Riley Jr. sugerem que, pelo
famílias no quintil inferior, esta foi maior para os pardos e para fato de essas relações serem opcionais, mais do que
a região Nordeste. Assim que, ao mesmo tempo, que se contratuais e obrigatórias, apresentariam grandes
acentuaram as desigualdades internas de renda entre as possibilidades de servir de elemento moderador de conflitos
famílias, também se acentuaram as disparidades por região e intergeracionais e mesmo de desigualdades percebidas.
raça. Conceitos e Abordagens Teóricas
A crescente diversidade e fluidez nos arranjos familiares
aumenta as possibilidades de trocas intergeracionais mais No debate sobre as relações intergeracionais e a
igualitárias e democráticas, mas também pode provocar reconstrução do Estado de Bem Estar os conceitos e as
inseguranças e incertezas. Alguns exemplos a respeito seriam: abordagens teóricas são cruciais para entender os diferentes
discursos e posições. Um primeiro aspecto conceitual que se
deve considerar refere-se à definição de “geração”. Thomson

Conhecimentos Específicos 94
Apostila Digital Licenciada para Alice Caroline Guarino dos Santos - alice.guarino@hotmail.com (Proibida a Revenda)
APOSTILAS OPÇÃO

nota que geração pode significar tudo ou nada para um A economia política do envelhecimento surgiu, em parte,
conjunto de conceitos que se esconde por trás de um única como uma crítica às limitações analíticas da teoria da
palavra. Cobre aspectos tão vagos como a dimensão de tempo, estratificação por idade. Uma crítica central é de que se presta
que pode ser de uma extensão aproximada de 25 anos, um uma atenção excessiva para a idade cronológica e para o ano
ponto particular no passado, ou todos aqueles vivos em algum do nascimento dos indivíduos. Fato que desviaria a atenção,
momento do tempo. Esse mesmo autor sustenta que o debate tanto das experiências individuais de envelhecimento como
sobre gerações nos Estados Unidos está demasiadamente das diferenças internas das coortes por idade, resultantes de
focalizado na previdência social e na distribuição dos gastos fatores macroestruturais tais como classe, gênero e raça. Essa
públicos. Dessa forma, o conceito tem uma forte conotação perspectiva afirma que o status social e econômico dos idosos
política porque, para muitos, o único interesse nas geração é definido não por sua idade biológica, mas sim pelas
reside na possibilidade de ela ajudar a explicar a atual política. instituições organizadas completamente ou, em parte, na
Se isso não acontece, o conceito perde sua importância. produção. Em outras palavras, as muitas experiências que
Portanto, é preciso ter claro o conceito de geração com o afetam os idosos seriam “o produto de uma específica divisão
qual se trabalha. As noções mais comuns remetem as relações de trabalho e da estrutura de desigualdades mais do que uma
familiares aos grupos de idade e às coortes, definidas como conseqüência do processo natural de envelhecimento”.
sendo os indivíduos que compartilharam uma experiência Interpretações do Conflito Geracional
comum em um certo período. Entretanto, uma leitura mais
atenta dos conceitos de geração sugere a necessidade de Grande parte dos estudos sobre mudanças nos contratos
distinção entre coortes ou grupo etário (aqueles indivíduos entre as gerações reconhece a presença de elementos de
que nasceram aproximadamente no mesmo momento do conflito entre elas, mas estes variam em termos do
tempo cronológico) do termo geração (ordenação entendimento da natureza desses conflitos. Essas
descendente dos membros familiares). O mesmo estudo controvérsias serão exemplificadas através do modelo de
sugere que há quatro termos principais em uso na análise equidade intergeracional e o da economia política do
social e de políticas para definir geração: envelhecimento.
O modelo de “eqüidade intergeracional” assume que as
a) geração como uma coorte de idade, operacionalizada divergências entre os grupos de idade são marcantes e que os
como um grupo nascido entre um intervalo de cinco ou dez anos. mais jovens, os adultos e os idosos competem por recursos
Critério utilizado para análises em nível macrossocial; escassos. Essa visão está estreitamente relacionada com a
b) geração como uma linha descendente de parentesco, preocupação econômica em torno de um modelo de Estado de
operacionalizada como a sucessão social/biológica e utilizada Bem-Estar em que predomina a “responsabilidade pública”.
para análises em nível micro; Preocupação esta, que integra o cerne da economia neoclássica
c) geração como geração histórica ou subgrupos de coortes e seus pressupostos em termos da contribuição dos setores
de idade (elites), operacionalizada como um movimento social público e privado. Esse modelo assume que o setor público é
conduzido por subgrupos etários em níveis macrossociais de uma estrutura improdutiva e que sobrecarrega o setor
análise; e privado. Ao mesmo tempo, concebe os idosos como um
d) geração como um grupo de idade operacionalizado por subgrupo homogêneo, dependente e improdutivo. O
coortes e em nível macrossocial da análise. entendimento de que os idosos representam, apenas, um gasto
para o setor público nesses modelos econômicos ortodoxos se
Entre a variedade de abordagens teóricas presente no deve, em grande parte, à subordinação das políticas sociais à
debate das relações intergeracionais destacamos as duas mais política econômica, e a importância dessa última na esfera
recorrentes na literatura brasileira: a sociologia da política. Nessa visão fragmentada do envelhecimento, as
estratificação por idades e a economia política do aposentadorias e pensões representam pouco mais do que
envelhecimento. Essas duas abordagens compartilham da uma transferência dos recursos da população ativa para os
premissa de que o contrato formal das políticas sociais idosos. A razão de dependência, medida freqüentemente
encontra-se em processo de renegociação e modificação na usada por economistas e demógrafos, é utilizada para
maioria das sociedades ocidentais, mas que a extensão das descrever as potenciais demandas que afetariam as
mudanças difere substancialmente entre países. O mesmo articulações políticas e econômicas entre o mercado e o Estado
ocorre com os contratos familiares informais entre as diante do envelhecimento da população.
gerações, em que as tendências demográficas e Tendo como referência o modelo de equidade
comportamentais produziram mudanças dramáticas. Essas intergeracional, diversos estudos para países industrializados
perspectivas teóricas, no entanto, divergem em termos da começaram a chamar a atenção para as mudanças
interpretação da natureza das mudanças. demográficas e de expectativas na sucessão das gerações como
sendo o “novo problema social” das últimas décadas do século
A teoria sociológica da estratificação por idade tem sido XX nesses países. O debate começou em torno das obrigações
tradicionalmente utilizada nas análises do vínculo entre as econômicas, morais e sociais dos adultos e jovens para com um
relações intergeracionais e o bem-estar. Essa abordagem grupo cada vez maior de idosos e das obrigações deles para
oferece os fundamentos para o modelo de análise mais com os mais jovens. Largamente utilizado nos Estados Unidos,
utilizado, ou seja, o modelo de “equidade intergeracional”. A esse tipo de análise sugeriu a ocorrência de uma “guerra de
teoria da estratificação por idade ganhou visibilidade nos gerações” e a hipótese do conflito foi amplamente aceita e
Estados Unidos, quando Matilda Riley e seus colaboradores institucionalmente estimulada por interesses políticos e
destacaram a importância dos conceitos de estratificação e lobbies dos idosos, tais como a Associação Americana para as
estrutura nas pesquisas sobre o envelhecimento. Essa Pessoas Aposentadas (AARP), a Associação Americana de
abordagem propõe que a coorte etária (indivíduos nascidos no Boomers (AAB), e a dos Americanos para a Eqüidade
mesmo período de tempo) e o tempo histórico, através do qual Geracional (Idade).
essas coortes se movem, representam os principais
componentes de um sistema estratificado por idade. A perspectiva da economia política do envelhecimento se
Enfatizam, também, que a sucessão das coortes é uma fonte opõe ao modelo da equidade e argumenta que esse último
histórica de mudança porque “suas experiências históricas seria, de fato, uma construção sociopolítica. O interesse maior
particulares contribuem de forma única para as estruturas dos formuladores de políticas, inspirados neste modelo, reside
sociais”. na sobrecarga que as aposentadorias e pensões representam

Conhecimentos Específicos 95
Apostila Digital Licenciada para Alice Caroline Guarino dos Santos - alice.guarino@hotmail.com (Proibida a Revenda)
APOSTILAS OPÇÃO

para os gastos públicos e não na equidade intergeracional. Da Estados Unidos confirmam isso e sugerem que, mesmo que
mesma forma, o imperativo econômico-demográfico, não se possa considerar a solidariedade intrafamiliar como
freqüentemente apontado no modelo de equidade, seria a dada, pode-se falar de uma solidariedade crescente em nível
justificativa para facilitar a reestruturação do Estado de Bem- de família, da mesma forma que haveria uma disparidade
Estar e que isso foi influenciado, em alguns países, pelas crescente entre as gerações em nível macrossocial.
agências de cooperação econômica internacionais.
Comparando a experiência da União Europeia (UE) com a A interpretação alternativa de como se dão as relações
dos Estados Unidos, Walker reforça seu argumento de que intergeracionais dentro da família é baseada na economia
mais importantes do que a pressão derivada das mudanças política do envelhecimento. Essa abordagem insiste que a
demográficas, são os deslocamentos ideológicos, solidariedade entre pais e filhos não é dada, mas tem de ser
particularmente para a ortodoxia econômica, que alteram os construída com base no reconhecimento das diferenças entre
pressupostos sobre o papel do Estado no que diz respeito ao coortes e gênero e suas consequências para a afinidade entre
bem-estar e incentivaram alguns países a reestruturar seus eles. Em claro contraste com o modelo de equidade
sistemas previdenciários, reduzir o papel dos regimes públicos intergeracional, a economia política do envelhecimento não
e aumentar a participação do setor privado nos sistemas. separa as políticas sociais da natureza das relações de
Na abordagem da economia política, a concepção de cuidados intrafamiliares. Nessa perspectiva, os domínios
Estado de Bem-Estar é de que este seria um contrato públicos e privados interagem, revelando com isso que as
intergeracional, ou seja, um acordo negociado entre os políticas afetam as relações familiares. Por exemplo, quando o
trabalhadores ativos e os aposentados para a manutenção da Estado tenta minimizar seu compromisso financeiro para com
qualidade de vida na idade avançada. Assim, as políticas que a família, um maior peso recai sobre os mais vulneráveis,
pretendem criar novos contratos previdenciários devem ser particularmente as mulheres, que terão de arcar com as
analisadas como um elemento do esforço muito mais amplo responsabilidades do cuidado.
por parte de alguns Estados para reestruturar seus Um desacordo ainda maior entre o modelo da equidade e o
mecanismos de provisão de bem-estar, ou, do contrário da economia política do envelhecimento diz respeito à
poderão ser interpretadas, simplesmente, como medidas natureza e à prevalência da solidariedade e das tensões dentro
restritas a questões de idade. da família. Enquanto o primeiro idealiza o modelo de família
utilizado no debate sobre a solidariedade intergeracional
Entendendo as Relações Intergeracionais na Família familiar, a economia política do envelhecimento descreve a
solidariedade familiar como um fenômeno multidimensional,
Uma interpretação convencional sustenta que o contrato com complexas e às vezes contraditórias relações entre pais e
intergeracional entre os membros da família é mantido por um filhos adultos, e entre avós e netos. Um exemplo dessa última
conjunto de fatores, como laços de afeto, sentimentos de linha de análise refere-se às bem documentadas diferenças no
reciprocidade sobre a vida, poderosos incentivos econômicos papel das mulheres e dos homens nas relações
ou sanções negativas, e amplos valores culturais. Os intergeracionais, particularmente a diferença nos papéis de
sentimentos de afeto e a obrigação ou a promessa de gênero nas relações entre a primeira e a terceira geração. Uma
benefícios econômicos são fatores que asseguram o contrato vez que as mães têm normalmente a custódia de seus filhos
familiar informal entre as gerações, mas as outras variáveis após um divórcio, as avós maternas provavelmente poderão
também contribuem. Recursos limitados e a ausência de filhos desempenhar as obrigações e os deveres familiares, que
são, talvez, as razões mais importantes para explicar por que aumentam inclusive o contato com os parentes das mães
as pessoas idosas com algum tipo de deficiência podem ser comparado ao contato com os pais. Os resultados para o Brasil
negligenciadas, esquecidas, ou mesmo abandonadas. Então, mostram que 82% dos domicílios com co-residência de avós,
sob algumas circunstâncias, o contrato pode falhar, como, por pais ou sogros envolvem as mulheres.
exemplo, na ausência de filhos. Nas sociedades onde os
direitos de posse existem, a ausência da propriedade também Algumas considerações sobre a ocorrência da pobreza
pode conduzir à negligência. Foner ressalta, porém, que o que intergeracional
pode parecer negligência ou abandono na perspectiva
ocidental pode, preferivelmente, ser um complemento do O debate sobre as relações intergeracionais e a
contrato social dentro de um dado contexto cultural. reconstrução do Estado de Bem-Estar no Brasil nos remete à
Cabe lembrar, ainda, que os sentimentos de obrigação que velha questão do conflito na distribuição de recursos. O
atravessam as gerações são influenciados não só pelas aumento dos mecanismos de apoio por parte do setor público
experiências únicas dos indivíduos de uma família particular — tais como os cuidados de longa permanência para os idosos,
mas, também, por circunstâncias históricas específicas. Os os sistemas de saúde universais, a educação pública e
caminhos trilhados para a idade avançada determinam a programas mais generosos de assistência a famílias pobres —
adaptação dos indivíduos e de suas famílias às circunstâncias redistribui renda dos homens para as mulheres, dos adultos
sociais e econômicas que enfrentam na última fase da vida. A para os idosos e, muito provavelmente, dos ricos para os
migração, o sucesso econômico ou as dificuldades da economia pobres. Ou seja, beneficiariam grupos menos favorecidos da
local podem afetar os padrões de apoio e suporte e as sociedade brasileira à custa dos mais poderosos.
expectativas para receber e fornecer o auxílio na idade Esses importantes aspectos são obscurecidos pela
avançada. Assim, as condições de bem-estar em um estágio do ausência de um diagnóstico coerente dos custos diferenciados
ciclo de vida são, freqüentemente, relacionadas aos eventos do cuidado dos dependentes para as famílias, o Estado e o
ocorridos ao longo da vida e podem afetar o bem-estar em sua mercado. As análises econômicas convencionais consideram
última fase, tal como foi sugerido para as idosas brasileiras de elevados os gastos sociais com os idosos pelo Estado, mas
diferentes coortes. nunca se preocupam em comparar esses custos com os
Estudos com base no modelo da equidade intergeracional incorridos pelas famílias. Isso se deve, em grande parte, à
sugerem que a novidade sobre o conflito das gerações está em suposição implícita de que as famílias, especialmente as
nível macro, onde a reciprocidade e a equidade entre grupos mulheres, proveem serviços sociais gratuitos.
etários aparecem como fundamentais para as políticas sociais. Analistas e formuladores da política social brasileira pouco
Já em nível micro, as obrigações e as transferências têm a dizer sobre quem deve pagar pelos custos de cuidado
intergeracionais intrafamiliares teriam se mantido inalteradas dos idosos e muito menos sobre quem deve pagar pelos custos
na maioria das sociedades ocidentais. Resultados para os que envolvem o bem-estar das gerações mais jovens. O tempo

Conhecimentos Específicos 96
Apostila Digital Licenciada para Alice Caroline Guarino dos Santos - alice.guarino@hotmail.com (Proibida a Revenda)
APOSTILAS OPÇÃO

e o dinheiro que as famílias brasileiras dispensam com o redistribuição de recursos para alimentar, proteger e cuidar
cuidado para seus dependentes — ajudando a reprodução da saúde dos indivíduos, deve-se ampliar a noção de bem-estar
biológica e social da população — são ignorados nas individual. Nesse sentido, parece útil recuperar a ideia da
discussões do PIB e na formulação da política. Além disso, a existência de múltiplas esferas na qual o bem-estar está
família é, freqüentemente, responsabilizada por problemas distribuído, incluindo o bem-estar dos indivíduos, dos outros
sociais, inclusive do déficit público. Criar bons cidadãos e e da comunidade como um. Em outras palavras, que ao longo
manter uma sociedade coesa parece ser a obrigação das do dia nos movemos diversas vezes entre esferas da vida
famílias. A família nuclear idealizada, composta por um casal igualmente cruciais para o nosso bem-estar. Essa concepção
heterossexual estável, com poucos filhos e bem-sucedida de partilha de bem-estar nos leva a um entendimento mais
economicamente, pareceria ser a solução de problemas amplo de política social. Uma política que integra as formas de
nacionais como o desemprego, a violência e a pobreza. bem-estar propiciadas em nível da família e do mercado com
aqueles serviços oferecidos pelo Estado. Um passo necessário
O debate atual sobre a crise fiscal do sistema nessa direção seria ampliar a discussão sobre o modelo de
previdenciário polariza e paralisa os esforços para avançar o proteção social ou do futuro do welfare state, para além dos
sistema de proteção social universalista brasileiro, proposto argumentos econômicos e dos gastos públicos.
pela Constituição Federal de 1988. As evidências de um
aumento no fluxo inverso de transferências intergeracionais
mostram que mais e mais idosos proveem recursos para seus 11. casamento e separação;
filhos adultos e netos. Tampouco essas famílias estão
suficientemente bem para continuar com seu papel de
12. Noções de Direito de Família
principal cuidadora de seus dependentes. Nessa mesma linha, e Sucessões sobre as áreas de
as melhorias nas condições de vida das mulheres idosas atuação de Serviço Social;
brasileiras apenas começam a diminuir as desigualdades de
gênero entre os idosos, porém não significam que estas
estejam resolvidas. Equivocadamente interpretada como
Prezado candidato, estes conteúdos já foram estudado
resultado de um privilégio, essa situação poderia estimular
na matéria de Legislação, tópico: CÓDIGO CIVIL
cortes em benefícios, os quais são vitais para a estratégia de
- Lei 10.406/2002 Arts: 1511 a 1638; 1694 a 1727 e
sobrevivência das famílias brasileiras. Todas essas tendências
1728 a 1783.
de transferências de recursos dos idosos para os mais jovens
não devem ser vistas como uma solução para o problema da
redistribuição de recursos, mas como uma advertência sobre
as crescentes dificuldades das gerações mais jovens para se 13. direitos fundamentais da
reproduzirem. Também, parece claro que o emergente criança e do adolescente;
“conflito entre gerações” por escassos recursos deve ser visto
muito mais como parte de um contexto de múltiplas carências
e desigualdades do que um privilégio dos idosos.
Os direitos fundamentais da criança e do adolescente
As políticas sociais, em termos práticos, estão orientadas estão dispostos no ECA, que seguem em seguida:
tanto por uma lógica técnica como pela capacidade de pressão
dos diferentes grupos sociais sobre o Estado. Assim, como ( )
seria possível distinguir entre as boas e as más interpretações Título II
das necessidades da população? Ao tratar de responder a essa Dos Direitos Fundamentais
questão, concordamos que a justificativa para que algumas Capítulo I
interpretações sejam vistas como melhores que outras Do Direito à Vida e à Saúde
envolve a consideração de um balanço entre procedimentos e
consequências. Em outras palavras, um balanceamento entre Art. 7º A criança e o adolescente têm direito a proteção à
democracia, igualdade e justiça. vida e à saúde, mediante a efetivação de políticas sociais
Consistente com um discurso de “resolução dos problemas públicas que permitam o nascimento e o desenvolvimento
sociais”, sugerimos que se deva dar continuidade ao processo sadio e harmonioso, em condições dignas de existência.
de construção de um sistema universalista de proteção social,
enfatizando a complexidade de um sistema de políticas Art. 8º É assegurado à gestante, através do Sistema Único
integrado tendo presente o contexto das grandes de Saúde, o atendimento pré e perinatal.
desigualdades, ao invés de um sistema segmentado. Políticas § 1º A gestante será encaminhada aos diferentes níveis de
em nível macro, que estabeleçam uma responsabilidade atendimento, segundo critérios médicos específicos,
conjunta das famílias e do Estado no cuidado dos idosos, são obedecendo-se aos princípios de regionalização e
vitais para aliviar a pressão imposta às famílias, hierarquização do Sistema.
particularmente sobre as mulheres. Estas devem responder às § 2º A parturiente será atendida preferencialmente pelo
mudanças fundamentais na concepção de família e parentesco, mesmo médico que a acompanhou na fase pré-natal.
decorrentes das mudanças sociodemográficas e tecnológicas. § 3º Incumbe ao poder público propiciar apoio alimentar à
Está claro, também, que não se pode repensar as políticas de gestante e à nutriz que dele necessitem.
bem-estar separadamente das políticas de emprego. Em § 4º Incumbe ao poder público proporcionar assistência
outras palavras, o debate sobre o papel do Estado deve psicológica à gestante e à mãe, no período pré e pós-natal,
considerar, simultaneamente, suas consequências para o inclusive como forma de prevenir ou minorar as
mercado e para as famílias. consequências do estado puerperal.
§ 5º A assistência referida no § 4º deste artigo deverá ser
Ao concluir, chamamos a atenção para a necessidade de também prestada a gestantes ou mães que manifestem
redefinir a noção de bem-estar. O debate sobre o bem-estar e interesse em entregar seus filhos para adoção.
as políticas públicas tem se pautado pela idéia de provisão de
serviços básicos e por uma noção restrita de bem-estar Art. 9º O poder público, as instituições e os empregadores
individual. A sugestão é que, paralelamente à necessidade de propiciarão condições adequadas ao aleitamento materno,

Conhecimentos Específicos 97
Apostila Digital Licenciada para Alice Caroline Guarino dos Santos - alice.guarino@hotmail.com (Proibida a Revenda)
APOSTILAS OPÇÃO

inclusive aos filhos de mães submetidas a medida privativa de VI - participar da vida política, na forma da lei;
liberdade. VII - buscar refúgio, auxílio e orientação.

Art. 10. Os hospitais e demais estabelecimentos de atenção Art. 17. O direito ao respeito consiste na inviolabilidade da
à saúde de gestantes, públicos e particulares, são obrigados a: integridade física, psíquica e moral da criança e do
I - manter registro das atividades desenvolvidas, através adolescente, abrangendo a preservação da imagem, da
de prontuários individuais, pelo prazo de dezoito anos; identidade, da autonomia, dos valores, ideias e crenças, dos
II - identificar o recém-nascido mediante o registro de sua espaços e objetos pessoais.
impressão plantar e digital e da impressão digital da mãe, sem
prejuízo de outras formas normatizadas pela autoridade Art. 18. É dever de todos velar pela dignidade da criança e
administrativa competente; do adolescente, pondo-os a salvo de qualquer tratamento
III - proceder a exames visando ao diagnóstico e desumano, violento, aterrorizante, vexatório ou
terapêutica de anormalidades no metabolismo do recém- constrangedor.
nascido, bem como prestar orientação aos pais;
IV - fornecer declaração de nascimento onde constem Art. 18-A. A criança e o adolescente têm o direito de ser
necessariamente as intercorrências do parto e do educados e cuidados sem o uso de castigo físico ou de
desenvolvimento do neonato; tratamento cruel ou degradante, como formas de correção,
V - manter alojamento conjunto, possibilitando ao neonato disciplina, educação ou qualquer outro pretexto, pelos pais,
a permanência junto à mãe. pelos integrantes da família ampliada, pelos responsáveis,
pelos agentes públicos executores de medidas socioeducativas
Art. 11. É assegurado atendimento integral à saúde da ou por qualquer pessoa encarregada de cuidar deles, tratá-los,
criança e do adolescente, por intermédio do Sistema Único de educá-los ou protegê-los.
Saúde, garantido o acesso universal e igualitário às ações e Parágrafo único. Para os fins desta Lei, considera-se:
serviços para promoção, proteção e recuperação da saúde. I - castigo físico: ação de natureza disciplinar ou punitiva
§ 1º A criança e o adolescente portadores de deficiência aplicada com o uso da força física sobre a criança ou o
receberão atendimento especializado. adolescente que resulte em:
§ 2º Incumbe ao poder público fornecer gratuitamente a) sofrimento físico; ou
àqueles que necessitarem os medicamentos, próteses e outros b) lesão;
recursos relativos ao tratamento, habilitação ou reabilitação. II - tratamento cruel ou degradante: conduta ou forma
cruel de tratamento em relação à criança ou ao adolescente
Art. 12. Os estabelecimentos de atendimento à saúde que:
deverão proporcionar condições para a permanência em a) humilhe; ou
tempo integral de um dos pais ou responsável, nos casos de b) ameace gravemente; ou
internação de criança ou adolescente. c) ridicularize.

Art. 13. Os casos de suspeita ou confirmação de castigo Art. 18-B. Os pais, os integrantes da família ampliada, os
físico, de tratamento cruel ou degradante e de maus-tratos responsáveis, os agentes públicos executores de medidas
contra criança ou adolescente serão obrigatoriamente socioeducativas ou qualquer pessoa encarregada de cuidar de
comunicados ao Conselho Tutelar da respectiva localidade, crianças e de adolescentes, tratá-los, educá-los ou protegê-los
sem prejuízo de outras providências legais. que utilizarem castigo físico ou tratamento cruel ou
Parágrafo único. As gestantes ou mães que manifestem degradante como formas de correção, disciplina, educação ou
interesse em entregar seus filhos para adoção serão qualquer outro pretexto estarão sujeitos, sem prejuízo de
obrigatoriamente encaminhadas à Justiça da Infância e da outras sanções cabíveis, às seguintes medidas, que serão
Juventude. aplicadas de acordo com a gravidade do caso:
I - encaminhamento a programa oficial ou comunitário de
Art. 14. O Sistema Único de Saúde promoverá programas proteção à família;
de assistência médica e odontológica para a prevenção das II - encaminhamento a tratamento psicológico ou
enfermidades que ordinariamente afetam a população infantil, psiquiátrico;
e campanhas de educação sanitária para pais, educadores e III - encaminhamento a cursos ou programas de
alunos. orientação;
Parágrafo único. É obrigatória a vacinação das crianças nos IV - obrigação de encaminhar a criança a tratamento
casos recomendados pelas autoridades sanitárias. especializado;
V - advertência.
Capítulo II Parágrafo único. As medidas previstas neste artigo serão
Do Direito à Liberdade, ao Respeito e à Dignidade aplicadas pelo Conselho Tutelar, sem prejuízo de outras
providências legais.
Art. 15. A criança e o adolescente têm direito à liberdade,
ao respeito e à dignidade como pessoas humanas em processo Capítulo III
de desenvolvimento e como sujeitos de direitos civis, humanos Do Direito à Convivência Familiar e Comunitária
e sociais garantidos na Constituição e nas leis. Seção I
Disposições Gerais
Art. 16. O direito à liberdade compreende os seguintes
aspectos: Art. 19. É direito da criança e do adolescente ser criado e
I - ir, vir e estar nos logradouros públicos e espaços educado no seio de sua família e, excepcionalmente, em
comunitários, ressalvadas as restrições legais; família substituta, assegurada a convivência familiar e
II - opinião e expressão; comunitária, em ambiente que garanta seu desenvolvimento
III - crença e culto religioso; integral. (Redação dada pela Lei nº 13.257, de 2016)
IV - brincar, praticar esportes e divertir-se; § 1o Toda criança ou adolescente que estiver inserido em
V - participar da vida familiar e comunitária, sem programa de acolhimento familiar ou institucional terá sua
discriminação; situação reavaliada, no máximo, a cada 6 (seis) meses,

Conhecimentos Específicos 98
Apostila Digital Licenciada para Alice Caroline Guarino dos Santos - alice.guarino@hotmail.com (Proibida a Revenda)
APOSTILAS OPÇÃO

devendo a autoridade judiciária competente, com base em


relatório elaborado por equipe interprofissional ou 14. medidas específicas de
multidisciplinar, decidir de forma fundamentada pela proteção à criança e ao
possibilidade de reintegração familiar ou colocação em
família substituta, em quaisquer das modalidades previstas adolescente;
no art. 28 desta Lei.
§ 2o A permanência da criança e do adolescente em (ECA)
programa de acolhimento institucional não se prolongará por Das Medidas de Proteção
mais de 2 (dois) anos, salvo comprovada necessidade que Capítulo I
atenda ao seu superior interesse, devidamente fundamentada Disposições Gerais
pela autoridade judiciária.
§ 3oA manutenção ou a reintegração de criança ou Art. 98. As medidas de proteção à criança e ao adolescente
adolescente à sua família terá preferência em relação a são aplicáveis sempre que os direitos reconhecidos nesta Lei
qualquer outra providência, caso em que será esta incluída forem ameaçados ou violados:
em serviços e programas de proteção, apoio e promoção, nos I - por ação ou omissão da sociedade ou do Estado;
termos do § 1o do art. 23, dos incisos I e IV do caput do art. II - por falta, omissão ou abuso dos pais ou responsável;
101 e dos incisos I a IV do caput do art. 129 desta III - em razão de sua conduta.
Lei. (Redação dada pela Lei nº 13.257, de 2016)
§ 4o Será garantida a convivência da criança e do adolescente Capítulo II
com a mãe ou o pai privado de liberdade, por meio de visitas Das Medidas Específicas de Proteção
periódicas promovidas pelo responsável ou, nas hipóteses de
acolhimento institucional, pela entidade responsável, Art. 99. As medidas previstas neste Capítulo poderão ser
independentemente de autorização judicial. (Incluído pela Lei aplicadas isolada ou cumulativamente, bem como substituídas
nº 12.962, de 2014) a qualquer tempo.
Art. 20. Os filhos, havidos ou não da relação do casamento, Art. 100. Na aplicação das medidas levar-se-ão em conta as
ou por adoção, terão os mesmos direitos e qualificações, necessidades pedagógicas, preferindo-se aquelas que visem ao
proibidas quaisquer designações discriminatórias relativas à fortalecimento dos vínculos familiares e comunitários.
filiação. Parágrafo único. São também princípios que regem a
aplicação das medidas:
Art. 21. O poder familiar será exercido, em igualdade de I - condição da criança e do adolescente como sujeitos de
condições, pelo pai e pela mãe, na forma do que dispuser a direitos: crianças e adolescentes são os titulares dos direitos
legislação civil, assegurado a qualquer deles o direito de, em previstos nesta e em outras Leis, bem como na Constituição
caso de discordância, recorrer à autoridade judiciária Federal;
competente para a solução da divergência. II - proteção integral e prioritária: a interpretação e
aplicação de toda e qualquer norma contida nesta Lei deve ser
Art. 22. Aos pais incumbe o dever de sustento, guarda e voltada à proteção integral e prioritária dos direitos de que
educação dos filhos menores, cabendo-lhes ainda, no crianças e adolescentes são titulares;
interesse destes, a obrigação de cumprir e fazer cumprir as III - responsabilidade primária e solidária do poder
determinações judiciais. público: a plena efetivação dos direitos assegurados a crianças
Parágrafo único. A mãe e o pai, ou os responsáveis, têm e a adolescentes por esta Lei e pela Constituição Federal, salvo
direitos iguais e deveres e responsabilidades compartilhados nos casos por esta expressamente ressalvados, é de
no cuidado e na educação da criança, devendo ser responsabilidade primária e solidária das 3 (três) esferas de
resguardado o direito de transmissão familiar de suas governo, sem prejuízo da municipalização do atendimento e
crenças e culturas, assegurados os direitos da criança da possibilidade da execução de programas por entidades não
estabelecidos nesta Lei. (Incluído pela Lei nº 13.257, de governamentais;
2016) IV - interesse superior da criança e do adolescente: a
intervenção deve atender prioritariamente aos interesses e
Art. 23. A falta ou a carência de recursos materiais não direitos da criança e do adolescente, sem prejuízo da
constitui motivo suficiente para a perda ou a suspensão consideração que for devida a outros interesses legítimos no
do poder familiar. âmbito da pluralidade dos interesses presentes no caso
§ 1o Não existindo outro motivo que por si só autorize a concreto;
decretação da medida, a criança ou o adolescente será V - privacidade: a promoção dos direitos e proteção da
mantido em sua família de origem, a qual deverá criança e do adolescente deve ser efetuada no respeito pela
obrigatoriamente ser incluída em serviços e programas intimidade, direito à imagem e reserva da sua vida privada;
oficiais de proteção, apoio e promoção. (Redação dada pela VI - intervenção precoce: a intervenção das autoridades
Lei nº 13.257, de 2016) competentes deve ser efetuada logo que a situação de perigo
§ 2o A condenação criminal do pai ou da mãe não seja conhecida;
implicará a destituição do poder familiar, exceto na hipótese VII - intervenção mínima: a intervenção deve ser exercida
de condenação por crime doloso, sujeito à pena de reclusão, exclusivamente pelas autoridades e instituições cuja ação seja
contra o próprio filho ou filha. (Incluído pela Lei nº 12.962, de indispensável à efetiva promoção dos direitos e à proteção da
2014) criança e do adolescente;
VIII - proporcionalidade e atualidade: a intervenção deve
Art. 24. A perda e a suspensão do poder familiar serão ser a necessária e adequada à situação de perigo em que a
decretadas judicialmente, em procedimento contraditório, criança ou o adolescente se encontram no momento em que a
nos casos previstos na legislação civil, bem como na hipótese decisão é tomada;
de descumprimento injustificado dos deveres e obrigações a IX - responsabilidade parental: a intervenção deve ser
que alude o art. 22 efetuada de modo que os pais assumam os seus deveres para
com a criança e o adolescente;

Conhecimentos Específicos 99
Apostila Digital Licenciada para Alice Caroline Guarino dos Santos - alice.guarino@hotmail.com (Proibida a Revenda)
APOSTILAS OPÇÃO

X - prevalência da família: na promoção de direitos e na contrário de autoridade judiciária competente, caso em que
proteção da criança e do adolescente deve ser dada também deverá contemplar sua colocação em família
prevalência às medidas que os mantenham ou reintegrem na substituta, observadas as regras e princípios desta Lei.
sua família natural ou extensa ou, se isto não for possível, que § 5º O plano individual será elaborado sob a
promovam a sua integração em família substituta; responsabilidade da equipe técnica do respectivo programa de
XI - obrigatoriedade da informação: a criança e o atendimento e levará em consideração a opinião da criança ou
adolescente, respeitado seu estágio de desenvolvimento e do adolescente e a oitiva dos pais ou do responsável.
capacidade de compreensão, seus pais ou responsável devem § 6º Constarão do plano individual, dentre outros:
ser informados dos seus direitos, dos motivos que I - os resultados da avaliação interdisciplinar;
determinaram a intervenção e da forma como esta se processa; II - os compromissos assumidos pelos pais ou responsável;
XII - oitiva obrigatória e participação: a criança e o e
adolescente, em separado ou na companhia dos pais, de III - a previsão das atividades a serem desenvolvidas com
responsável ou de pessoa por si indicada, bem como os seus a criança ou com o adolescente acolhido e seus pais ou
pais ou responsável, têm direito a ser ouvidos e a participar responsável, com vista na reintegração familiar ou, caso seja
nos atos e na definição da medida de promoção dos direitos e esta vedada por expressa e fundamentada determinação
de proteção, sendo sua opinião devidamente considerada pela judicial, as providências a serem tomadas para sua colocação
autoridade judiciária competente, observado o disposto nos §§ em família substituta, sob direta supervisão da autoridade
1º e 2º do art. 28 desta Lei. judiciária.
§ 7º O acolhimento familiar ou institucional ocorrerá no
Art. 101. Verificada qualquer das hipóteses previstas no local mais próximo à residência dos pais ou do responsável e,
art. 98, a autoridade competente poderá determinar, dentre como parte do processo de reintegração familiar, sempre que
outras, as seguintes medidas: identificada a necessidade, a família de origem será incluída
I - encaminhamento aos pais ou responsável, mediante em programas oficiais de orientação, de apoio e de promoção
termo de responsabilidade; social, sendo facilitado e estimulado o contato com a criança
II - orientação, apoio e acompanhamento temporários; ou com o adolescente acolhido.
III - matrícula e frequência obrigatórias em § 8º Verificada a possibilidade de reintegração familiar, o
estabelecimento oficial de ensino fundamental; responsável pelo programa de acolhimento familiar ou
IV - inclusão em serviços e programas oficiais ou institucional fará imediata comunicação à autoridade
comunitários de proteção, apoio e promoção da família, da judiciária, que dará vista ao Ministério Público, pelo prazo de
criança e do adolescente; (Redação dada pela Lei nº 13.257, de 5 (cinco) dias, decidindo em igual prazo.
2016) § 9º Em sendo constatada a impossibilidade de
V - requisição de tratamento médico, psicológico ou reintegração da criança ou do adolescente à família de origem,
psiquiátrico, em regime hospitalar ou ambulatorial; após seu encaminhamento a programas oficiais ou
VI - inclusão em programa oficial ou comunitário de comunitários de orientação, apoio e promoção social, será
auxílio, orientação e tratamento a alcoólatras e toxicômanos; enviado relatório fundamentado ao Ministério Público, no qual
VII - acolhimento institucional; conste a descrição pormenorizada das providências tomadas e
VIII - inclusão em programa de acolhimento familiar; a expressa recomendação, subscrita pelos técnicos da entidade
IX - colocação em família substituta. ou responsáveis pela execução da política municipal de
§ 1º O acolhimento institucional e o acolhimento familiar garantia do direito à convivência familiar, para a destituição
são medidas provisórias e excepcionais, utilizáveis como do poder familiar, ou destituição de tutela ou guarda.
forma de transição para reintegração familiar ou, não sendo § 10. Recebido o relatório, o Ministério Público terá o
esta possível, para colocação em família substituta, não prazo de 30 (trinta) dias para o ingresso com a ação de
implicando privação de liberdade. destituição do poder familiar, salvo se entender necessária a
§ 2º Sem prejuízo da tomada de medidas emergenciais realização de estudos complementares ou outras providências
para proteção de vítimas de violência ou abuso sexual e das que entender indispensáveis ao ajuizamento da demanda.
providências a que alude o art. 130 desta Lei, o afastamento da § 11. A autoridade judiciária manterá, em cada comarca ou
criança ou adolescente do convívio familiar é de competência foro regional, um cadastro contendo informações atualizadas
exclusiva da autoridade judiciária e importará na deflagração, sobre as crianças e adolescentes em regime de acolhimento
a pedido do Ministério Público ou de quem tenha legítimo familiar e institucional sob sua responsabilidade, com
interesse, de procedimento judicial contencioso, no qual se informações pormenorizadas sobre a situação jurídica de cada
garanta aos pais ou ao responsável legal o exercício do um, bem como as providências tomadas para sua reintegração
contraditório e da ampla defesa. familiar ou colocação em família substituta, em qualquer das
§ 3º Crianças e adolescentes somente poderão ser modalidades previstas no art. 28 desta Lei.
encaminhados às instituições que executam programas de § 12. Terão acesso ao cadastro o Ministério Público, o
acolhimento institucional, governamentais ou não, por meio Conselho Tutelar, o órgão gestor da Assistência Social e os
de uma Guia de Acolhimento, expedida pela autoridade Conselhos Municipais dos Direitos da Criança e do Adolescente
judiciária, na qual obrigatoriamente constará, dentre outros: e da Assistência Social, aos quais incumbe deliberar sobre a
I - sua identificação e a qualificação completa de seus pais implementação de políticas públicas que permitam reduzir o
ou de seu responsável, se conhecidos; número de crianças e adolescentes afastados do convívio
II - o endereço de residência dos pais ou do responsável, familiar e abreviar o período de permanência em programa de
com pontos de referência; acolhimento.
III - os nomes de parentes ou de terceiros interessados em
tê-los sob sua guarda; Art. 102. As medidas de proteção de que trata este Capítulo
IV - os motivos da retirada ou da não reintegração ao serão acompanhadas da regularização do registro civil.
convívio familiar. § 1º Verificada a inexistência de registro anterior, o
§ 4º Imediatamente após o acolhimento da criança ou do assento de nascimento da criança ou adolescente será feito à
adolescente, a entidade responsável pelo programa de vista dos elementos disponíveis, mediante requisição da
acolhimento institucional ou familiar elaborará um plano autoridade judiciária.
individual de atendimento, visando à reintegração familiar,
ressalvada a existência de ordem escrita e fundamentada em

Conhecimentos Específicos 100


Apostila Digital Licenciada para Alice Caroline Guarino dos Santos - alice.guarino@hotmail.com (Proibida a Revenda)
APOSTILAS OPÇÃO

§ 2º Os registros e certidões necessários à regularização de IV - indicação do cartório onde foi inscrito nascimento,
que trata este artigo são isentos de multas, custas e anexando, se possível, uma cópia da respectiva certidão;
emolumentos, gozando de absoluta prioridade. V - declaração sobre a existência de bens, direitos ou
§ 3º Caso ainda não definida a paternidade, será deflagrado rendimentos relativos à criança ou ao adolescente.
procedimento específico destinado à sua averiguação, Parágrafo único. Em se tratando de adoção, observar-se-ão
conforme previsto pela Lei no 8.560, de 29 de dezembro de também os requisitos específicos.
1992.
§ 4º Nas hipóteses previstas no § 3o deste artigo, é Guarda
dispensável o ajuizamento de ação de investigação de
paternidade pelo Ministério Público se, após o não O instituto da guarda tem como principal referencia o
comparecimento ou a recusa do suposto pai em assumir a exercício do poder familiar. Acreditava-se que aquele que
paternidade a ele atribuída, a criança for encaminhada para exercia o poder familiar teria, obrigatoriamente, o direito de
adoção. guarda. Contudo, tal afirmação se mostra incorreta, uma vez
§ 5º Os registros e certidões necessários à inclusão, a que a guarda pode ser exercida por pessoas diversas, que não
qualquer tempo, do nome do pai no assento de nascimento são aquelas detentoras do poder familiar.
isentos de multas, custas e emolumentos, gozando de absoluta Assim sendo, a guarda é um dos atributos do poder
prioridade. (Incluído dada pela Lei nº 13.257, de 2016) familiar, mas não se exaure nele nem com ele se confunde; em
§ 6º São gratuitas, a qualquer tempo, a averbação condições tais, a guarda pode existir sem o poder familiar,
requerida do reconhecimento de paternidade no assento de como, reciprocamente, este pode ser exercido sem a guarda. 16
nascimento e a certidão correspondente. (Incluído dada pela
Lei nº 13.257, de 2016) A guarda, como forma de colocação em família substituta,
mostra-se bastante flexível, oferecendo alternativa segura e
prática para garantir o direito da criança e adolescente de
15. colocação em família convivência familiar e comunitária. Vale lembrar que a
substituta - guarda-tutela- guarda, ainda que de maneira informal, é prática bastante
difundida, nascida da solidariedade e apoio entre um grupo
adoção; social ou familiar, tal como ocorre quando a vizinha toma conta
das crianças enquanto a mãe está no hospital ou a tia que
Nos casos em que a criança e/ou o adolescente perdem sua permanece com as crianças enquanto os pais viajam, entre
família natural, uma outra família deve substituí-la, para outras situações.
proporcionar ao menor o desenvolvimento pleno de suas
capacidades cognitivas, recebendo a assistência necessária Tendo isto em vista, o parágrafo 1º do artigo buscou dar
para desempenhar seu papel na sociedade ao alcançar o respaldo jurídico para que estas situações possam ser
período da vida adulta. regularizadas, ou seja, nos casos em que a guarda de fato já é
Com a premissa de que a assistência que é devida ao menor exercida, possibilita-se a concessão da guarda de direito.
somente será satisfatória e em sua amplitude se este estiver
em ambiente familiar, o Estatuto estabelece neste artigo os O legislador, parágrafo 2º, informa que a guarda que não se
requisitos para a concessão de pedidos de colocação do menor relacionar com os casos de tutela e adoção, é medida
em família substituta. excepcional. Contudo, é possível notar que existem inúmeras
No pedido deverá constar a qualificação completa do circunstancias em que não se faz necessária a aplicação de
requerente e de seu eventual cônjuge, ou companheiro, com medida tão drástica quanto a adoção, sendo suficiente para as
expressa anuência deste; a indicação de eventual parentesco partes a concessão da guarda.
do requerente e de seu cônjuge, ou companheiro, com a
criança ou adolescente, especificando se tem ou não parente Muito importante as disposições do parágrafo 3º, que
vivo; a qualificação completa da criança ou adolescente e de confere à criança ou adolescente que esteja sob guarda, todos
seus pais, se conhecidos, a indicação do cartório onde foi os direitos relativos ao dependente. Isto porque, ao ser
inscrito nascimento, anexando, se possível, uma cópia da caracterizado como dependente, o infante poderá gozar de
respectiva certidão e a declaração sobre a existência de bens, alguns benefícios, como a inclusão em planos de saúde, em
direitos ou rendimentos relativos à criança ou ao adolescente. serviços familiares e, como bem descreve o legislador,
Estes requisito devem constar na petição inicial ou instruí- benefícios previdenciários.
la para os casos de guarda e tutela, visto que ao pedido de
adoção deve-se observar os requisitos específicos. De modo geral, a guarda não retira o poder familiar dos
pais, sendo que o parágrafo 4º do artigo bem descreve a
(ECA) possibilidade de visitação da criança por parte dos pais.
Seção IV Obviamente que quando a guarda é deferida em razão da
Da Colocação em Família Substituta adoção, os vínculos paternos já estão bastante desgastados,
quando não até destituídos, e para o bem da própria criança, a
Art. 165. São requisitos para a concessão de pedidos de visitação não será permitida.
colocação em família substituta:
I - qualificação completa do requerente e de seu eventual (ECA)
cônjuge, ou companheiro, com expressa anuência deste; Subseção II
II - indicação de eventual parentesco do requerente e de seu Da Guarda
cônjuge, ou companheiro, com a criança ou adolescente,
especificando se tem ou não parente vivo; Art. 33. A guarda obriga a prestação de assistência material,
III - qualificação completa da criança ou adolescente e de moral e educacional à criança ou adolescente, conferindo a seu
seus pais, se conhecidos; detentor o direito de opor-se a terceiros, inclusive aos pais.

16 CAHALI, Yussef Said. Estatuto da Criança e do Adolescente http://www.promenino.org.br/noticias/arquivo/eca-comentado-


(íntegra e comentários técnicos), Disponível em: artigo-33livro-1---tema-guarda. Acesso em: 07/05/2015.

Conhecimentos Específicos 101


Apostila Digital Licenciada para Alice Caroline Guarino dos Santos - alice.guarino@hotmail.com (Proibida a Revenda)
APOSTILAS OPÇÃO

§ 1º A guarda destina-se a regularizar a posse de fato,


podendo ser deferida, liminar ou incidentalmente, nos Embora o dispositivo traga recomendações em que há
procedimentos de tutela e adoção, exceto no de adoção por possibilidade de revogação da guarda à qualquer tempo, este
estrangeiros. deve ser conjugado com outros dispositivos do próprio
§ 2º Excepcionalmente, deferir-se-á a guarda, fora dos casos estatuto e da lei em geral, visto que para a revogação da guarda
de tutela e adoção, para atender a situações peculiares ou suprir é indispensável que se realize a oitiva dos guardiães e
a falta eventual dos pais ou responsável, podendo ser deferido o permitir a estes a oportunidade de manifestação e defesa
direito de representação para a prática de atos determinados. (princípios do contraditório e da ampla defesa).
§ 3º A União apoiará a implementação de serviços de
acolhimento em família acolhedora como política pública, os De outra feita, deve-se ter sempre em primeiro plano os
quais deverão dispor de equipe que organize o acolhimento direitos e as necessidades da criança e adolescente envolvidos,
temporário de crianças e de adolescentes em residências de pois em geral, formam-se laços afetivos entre as partes, que
famílias selecionadas, capacitadas e acompanhadas que não não devem ser rompidos de forma abrupta.
estejam no cadastro de adoção. (Incluído pela Lei nº 13.257, de
2016) Tutela:
§ 4º Poderão ser utilizados recursos federais, estaduais,
distritais e municipais para a manutenção dos serviços de Nas palavras de Roberto João Elias, podemos definir a
acolhimento em família acolhedora, facultando-se o repasse de tutela como o poder conferido a uma pessoa capaz, para reger
recursos para a própria família acolhedora. (Incluído pela Lei nº a pessoa de um incapaz e administrar seus bens.18
13.257, de 2016)
A tutela é um sucedâneo do poder familiar, o significa dizer
Art. 34. O poder público estimulará, por meio de assistência que, na falta dos pais, seja por qual motivo, faz-se necessária a
jurídica, incentivos fiscais e subsídios, o acolhimento, sob a substituição, uma vez que o tutelado (criança, adolescente ou
forma de guarda, de criança ou adolescente afastado do incapaz) ainda não possui condições de gerir sua vida por
convívio familiar. conta própria.
§ 1º A inclusão da criança ou adolescente em programas de
acolhimento familiar terá preferência a seu acolhimento IMPORTANTE
institucional, observado, em qualquer caso, o caráter
temporário e excepcional da medida, nos termos desta Lei. Não há que se falar em nomeação de tutor enquanto um
§ 2º Na hipótese do § 1o deste artigo a pessoa ou casal dos pais estiver vivo!!!
cadastrado no programa de acolhimento familiar poderá
receber a criança ou adolescente mediante guarda, observado o A adoção, nos moldes da atualidade, comporta diversos
disposto nos arts. 28 a 33 desta Lei. tipos de conceituação. Pontes de Miranda conceitua da
§ 3º A União apoiará a implementação de serviços de seguinte forma: a adoção é ato solene pelo qual se cria, entre
acolhimento em família acolhedora como política pública, os o adotante e o adotado, relação de paternidade e filiação19 Já
quais deverão dispor de equipe que organize o acolhimento para Maria Berenice Dias, esta define como modalidade de
temporário de crianças e de adolescentes em residências de filiação constituída no amor, gerando vínculo de parentesco
famílias selecionadas, capacitadas e acompanhadas que não por opção20
estejam no cadastro de adoção. (Incluído pela Lei nº 13.257, de
2016) Em síntese, adoção é uma das formas de colocação em
§ 4º Poderão ser utilizados recursos federais, estaduais, família substituta, sendo a mais ampla e mais abrangente, pois
distritais e municipais para a manutenção dos serviços de gera novos vínculos de parentesco, passando os adotantes à
acolhimento em família acolhedora, facultando-se o repasse de figura de pais e o adotado à figura de filho.
recursos para a própria família acolhedora. (Incluído pela Lei nº
13.257, de 2016) O legislador frisa que se trata de medida irrevogável, o
que significa dizer que, uma vez constituído o vínculo da
Nas palavras de Maria Josefina Becker, “a guarda adoção, este não poderá ser anulado ou desfeito.
subsidiada é medida destinada, principalmente, a crianças Corroborando neste sentido, a adoção somente poderá ser
cuja adoção seja inviável, em razão de ausência de procedida pessoalmente, impedindo o uso de procuração,
candidatos adequados disponíveis. É o caso, muitas vezes, de mesmo que esta tenha poderes específicos.
crianças portadoras de enfermidades ou deficiências, ou de
adolescentes abandonados, que necessitam proteção especial. O nobre doutrinador Murillo José Digiácomo leciona que a
Um programa de lares remunerados exige boa seleção e destituição da tutela é medida aplicável ao tutor (art. 129,
assistência técnica constante, para que cumpra suas inciso IX, do ECA), que somente pode ser decretada pela
finalidades e evite a utilização de instituições de abrigo, ainda autoridade judiciária, em procedimento contencioso, no qual
excessiva em nossa realidade.”17 seja assegurado o contraditório e a ampla defesa, conforme
disposição expressa do art. 24, do ECA, a que se faz remissão.
Art. 35. A guarda poderá ser revogada a qualquer tempo, O procedimento para destituição de tutela é o previsto nos
mediante ato judicial fundamentado, ouvido o Ministério arts. 761 a 763 do CPC de 2015 (ao qual se reporta o art. 164,
Público. do ECA).21

17 BECKER, Maria Josefina. Estatuto da Criança e do Adolescente juridico.com.br/site/index.php?n_link=revista_artigos_leitura&artigo


(íntegra e comentários técnicos), Disponível em: _id=10121 Acesso em: 07/05/2015
http://www.promenino.org.br/noticias/arquivo/comentando-o-eca, 20 DIAS, Maria Berenice apud BRUM, Amanda Netto. Dos vínculos

Acesso em: 07/05/2015 por adoção, Disponível em: http://www.ambito-


18 ELIAS, Roberto João. Estatuto da Criança e do Adolescente juridico.com.br/site/index.php?n_link=revista_artigos_leitura&artigo
(íntegra e comentários técnicos), Disponível em: _id=10121 Acesso em: 07/05/2015
http://www.promenino.org.br/noticias/arquivo/eca-comentado-
artigo-36livro-1---tema-tutela, Acesso em: 07/05/2015. 21 DIGIÁCOMO, Murillo José; DIGIÁCOMO, Ildeara de Amorim.
19 MIRANDA, Pontes apud BRUM, Amanda Netto. Dos vínculos por Estatuto da Criança e do Adolescente Anotado e Interpretado - Lei nº
adoção, Disponível em: http://www.ambito- 8.069, de 13 de julho de 1990 (atualizado até a Lei nº 12.796/2013, de

Conhecimentos Específicos 102


Apostila Digital Licenciada para Alice Caroline Guarino dos Santos - alice.guarino@hotmail.com (Proibida a Revenda)
APOSTILAS OPÇÃO

As questões que envolvem a nomeação de tutor por adoção, este não poderá ser anulado ou desfeito.
testamento devem ser conjugadas com o Código Civil, Corroborando neste sentido, a adoção somente poderá ser
quanto às disposições à respeito da sucessão e do inventário. procedida pessoalmente, impedindo o uso de procuração,
Bem como o próprio Estatuto da Criança e do Adolescente traz mesmo que esta tenha poderes específicos.
mais disposições neste sentido (Arts. 165 a 170).
A adoção prevista pelo Estatuto da Criança e do
Embora da leitura do artigo tem-se a impressão que haverá Adolescente se destina apenas à crianças (de 0 a 12 anos) e
necessidade de propositura de ação para deflagração do adolescentes (de 12 a 18 anos), sendo que adoções de pessoas
procedimento específico para a nomeação do tutor, esta não é maiores de dezoito anos deverão seguir o instituído no Código
imprescindível. O Ministério Público também poderá Civil.
requisitar a abertura de procedimento para nomeação de tutor
testamentário, nos termos do Art. 201, III, desta mesma lei. Quando se trata de adoção, está se tratando também do
rompimento do vínculo de filiação original, pela destituição do
Andou bem o legislador ao requisitar que seja avaliado o poder familiar, para poder constituir novo vínculo de filiação
tutor nomeado de forma testamentária, informando que através da adoção. Assim sendo, note-se que a adoção é o único
mesmo que esta tenha sido a última disposição de vontade dos instituto que altera a condição de filho.
pais, o tutor deverá ser idôneo e estar qualificado para exercer Uma vez que a adoção se complete, a relação de
tal função. parentesco original é extinta e uma nova relação de
parentesco é consolidada, com todos os direitos e deveres
(ECA) inerentes à ela, sem distinções entre filhos biológicos e filhos
Subseção III adotivos, vedada a discriminação entre uns e outros, conforme
Da Tutela disposto no Art. 227, §6º, da Constituição Federal.

Art. 36. A tutela será deferida, nos termos da lei civil, a O parágrafo 1º trata de um tipo excepcional de adoção,
pessoa de até 18 (dezoito) anos incompletos. conhecida como adoção unilateral, em que um dos cônjuges
Parágrafo único. O deferimento da tutela pressupõe a prévia é pai/mãe biológico da criança e o outro cônjuge o adota. Neste
decretação da perda ou suspensão do poder familiar e implica caso, não há destituição do poder familiar do cônjuge que
necessariamente o dever de guarda. possui parentesco biológico, mas cria-se o parentesco
completo com o cônjuge adotante.
Art. 37. O tutor nomeado por testamento ou qualquer
documento autêntico, conforme previsto no parágrafo único do O texto do parágrafo 2º se mostra redundante, vez que o
art. 1.729 da Lei no 10.406, de 10 de janeiro de 2002 - Código caput do artigo já traz a orientação para que os direitos e
Civil, deverá, no prazo de 30 (trinta) dias após a abertura da deveres, inclusive sucessórios, são formados entre os
sucessão, ingressar com pedido destinado ao controle judicial do adotantes. Mesmo assim, o legislador buscou deixar claro que
ato, observando o procedimento previsto nos arts. 165 a 170 a sucessão faz parte dos direitos que surgem entre adotante e
desta Lei. adotado.
Parágrafo único. Na apreciação do pedido, serão
observados os requisitos previstos nos arts. 28 e 29 desta Lei, O estágio de convivência é parte importante do processo
somente sendo deferida a tutela à pessoa indicada na disposição de adoção, uma vez que sua função é criar um período
de última vontade, se restar comprovado que a medida é necessário para a avaliação da adaptação da criança ou
vantajosa ao tutelando e que não existe outra pessoa em adolescente à sua nova família. Assim sendo, tendo em vista a
melhores condições de assumi-la. diversidade de situações que se apresentam, o legislador agiu
corretamente ao flexibilizar o prazo do referido estágio,
Art. 38. Aplica-se à destituição da tutela o disposto no art. devendo este ser observado de acordo com a situação fática
24. que se apresenta.

Adoção: No ECA, a adoção vem tratada nos artigos 39 a 52-D

A adoção, nos moldes da atualidade, comporta diversos É fundamental que haja compatibilidade entre a
tipos de conceituação. Pontes de Miranda conceitua da legislação brasileira e a legislação do país em que residem os
seguinte forma: a adoção é ato solene pelo qual se cria, entre adotantes, assim, pessoas que estão impedidas de adotar em
o adotante e o adotado, relação de paternidade e filiação22 Já seu país de origem, não serão autorizadas à adotar aqui no
para Maria Berenice Dias, esta define como modalidade de Brasil, sob pena desta adoção não ser reconhecida.
filiação constituída no amor, gerando vínculo de parentesco
por opção23 O laudo de habilitação à adoção internacional, que deve ser
expedido pelo país de origem, constitui-se em documento à
Em síntese, adoção é uma das formas de colocação em partir do qual o candidato à adoção estrangeiro será
família substituta, sendo a mais ampla e mais abrangente, pois considerado apto a adotar aqui no Brasil, sendo competência
gera novos vínculos de parentesco, passando os adotantes à da Autoridade Central estadual realizar o cadastro de tais
figura de pais e o adotado à figura de filho. candidatos.

O legislador frisa que se trata de medida irrevogável, o


que significa dizer que, uma vez constituído o vínculo da

04 de abril de 2013), 6ª Ed., Curitiba: Ministério Público do Estado do 23 DIAS, Maria Berenice apud BRUM, Amanda Netto. Dos vínculos

Paraná. Centro de Apoio Operacional das Promotorias da Criança e do por adoção, Disponível em: http://www.ambito-
Adolescente, 2013, p. 42 juridico.com.br/site/index.php?n_link=revista_artigos_leitura&artigo
22 MIRANDA, Pontes apud BRUM, Amanda Netto. Dos vínculos por _id=10121 Acesso em: 07/05/2015
adoção, Disponível em: http://www.ambito-
juridico.com.br/site/index.php?n_link=revista_artigos_leitura&artigo
_id=10121 Acesso em: 07/05/2015

Conhecimentos Específicos 103


Apostila Digital Licenciada para Alice Caroline Guarino dos Santos - alice.guarino@hotmail.com (Proibida a Revenda)
APOSTILAS OPÇÃO

sequência, a aprovação da NOB/SUAS estabeleceu parâmetros


16. Convivência Familiar e para a operacionalização do Sistema Único em todo o território
nacional. Em 2006, foi aprovada a NOB-RH do SUAS que,
Comunitária; Acolhimento dentre outros aspectos, estabeleceu parâmetros nacionais
Institucional e Familiar; para a composição das equipes que devem atuar nos serviços
de acolhimento.
A organização do SUAS como um sistema pressupõe a
Em conformidade com as disposições do ECA, deve-se articulação da rede socioassistencial com as demais políticas
recorrer ao encaminhamento da criança e do adolescente a públicas e com o Sistema de Garantia de Direitos (SGD) e elege
serviços de acolhimento apenas quando esgotados todos os a família como foco central de atenção. A previsão de serviços
recursos para sua manutenção na família de origem, extensa de caráter preventivo e de fortalecimento de vínculos
ou comunidade. A história brasileira revela, todavia, que, familiares e comunitários10, de atendimento especializado a
frente à situação de pobreza, vulnerabilidade ou risco, a indivíduos e famílias em situação de ameaça ou violação de
primeira resposta à qual durante muitos anos se recorreu foi o direitos11 e de serviços de acolhimento para crianças e
afastamento da criança e do adolescente do convívio familiar. adolescentes12 tem importância basilar no que diz respeito à
A promulgação do ECA veio romper com essa cultura, ao concretização do direito à convivência familiar e comunitária.
garantir a excepcionalidade da medida, estabelecendo, ainda, Um grande desafio que é colocado para a implementação
que a situação de pobreza da família não constitui motivo do SUAS no país é o de reordenar os serviços de acolhimento e
suficiente para o afastamento da criança e do adolescente do romper com práticas incompatíveis com os marcos
convívio familiar (Art. 23). regulatórios vigentes. De acordo com a PNAS (2004),
Para garantir a excepcionalidade do afastamento do A ênfase da proteção social especial deve priorizar a
convívio familiar, o Art.130 do ECA estabelece que, nos casos reestruturação dos serviços de abrigamento - dos indivíduos
de violência praticada por familiar ou responsável com o qual que, por uma série de fatores, não contam mais com a proteção
a criança ou adolescente resida, a autoridade judiciária poderá e o cuidado de suas famílias - para as novas modalidades de
determinar o afastamento do agressor da moradia comum. De atendimento. A história dos abrigos e asilos é antiga no Brasil.
forma a promover a qualidade dos serviços de acolhimento, o A colocação de crianças, adolescentes, pessoas com
ECA prevê, ainda, ações de fiscalização e controle social, ao deficiências e idosos em instituições para protegê-los ou
exigir a inscrição das entidades que ofertam “programas de afastá-los do convívio social e familiar foi, durante muito
abrigo” no Conselho Municipal dos Direitos da Criança e do tempo, materializada em grandes instituições de longa
Adolescente (Art. 90) e estabelecer princípios para sua permanência, ou seja, espaços que atendiam a um grande
organização (Art. 92). Do mesmo modo, como constituem número de pessoas, que lá permaneciam por longo período –
serviços que compõem a rede socioassistencial, os serviços de às vezes a vida toda. São os chamados, popularmente, como
acolhimento devem também possuir registro no Conselho orfanatos, internatos, educandários, asilos, entre outros.
Municipal de Assistência Social e submeter-se também à sua Nesse sentido, o presente documento visa estabelecer
fiscalização. parâmetros de funcionamento e oferecer orientações
metodológicas para que os serviços de acolhimento de
A despeito dos direitos assegurados no ECA, o crianças e adolescentes possam cumprir sua função protetiva
Levantamento Nacional de Abrigos para Crianças e e de restabelecimento de direitos, compondo uma rede de
Adolescentes, realizado pelo Instituto de Pesquisa Econômica proteção que favoreça o fortalecimento dos vínculos familiares
Aplicada (IPEA), em 2003, identificou o descompasso e comunitários, o desenvolvimento de potencialidades das
existente entre a legislação e a realidade dos serviços de crianças e adolescentes atendidos e o empoderamento de suas
acolhimento para crianças e adolescentes no Brasil. A pesquisa famílias.
foi realizada em 584 serviços de acolhimento para crianças e A elaboração dos parâmetros apresentados neste
adolescentes co-financiadas com recursos do MDS e documento dá continuidade ao compromisso do MDS, da
representou um marco na mobilização nacional para a SEDH, do CONANDA, do CNAS e da sociedade civil organizada
discussão acerca do direito de crianças e adolescentes à com a afirmação, no estado brasileiro, do direito de crianças e
convivência familiar e comunitária. Esse processo iniciado em adolescentes à convivência familiar e comunitária.
2002, com a Caravana da Comissão de Direitos Humanos da
Câmara dos Deputados , ganhou impulso, em 2004, após a
publicação da pesquisa, quando o Ministro Chefe da SEDH e o ORIENTAÇÕES METODOLÓGICAS
Ministro de Estado do MDS, com o apoio do UNICEF,
convocaram outros Ministérios e atores para a formação de Enquanto o acolhimento for necessário, é fundamental
uma Comissão Intersetorial que apresentou os subsídios ao ofertar à criança e ao adolescente um ambiente e cuidados
CONANDA e ao CNAS para elaboração do Plano Nacional de facilitadores do desenvolvimento, de modo a favorecer, dentre
Promoção, Proteção e Defesa do Direito de Crianças e outros aspectos: i. Seu desenvolvimento integral; ii. A
Adolescentes à Convivência Familiar e Comunitária. Os superação de vivências de separação e violência; iii. A
Conselhos analisaram e aprimoraram o documento, aprovado apropriação e ressignificação de sua história de vida; e iv. O
em assembleia conjunta em 2006, após incorporação das fortalecimento da cidadania, autonomia e a inserção social. Em
sugestões apresentadas na consulta pública. Ressalte-se que, consonância com diretrizes nacionais e internacionais de
além dos dois Conselhos e do Poder Público, a sociedade civil cuidados a crianças e adolescentes em serviços de
teve também uma importante participação na construção acolhimento, as orientações abordadas neste capítulo visam
coletiva do Plano Nacional. contribuir para a melhoria dos atendimentos prestados nestes
serviços.
Paralelo a esse processo, em 2004, foi aprovada, pelo
CNAS, a Política Nacional de Assistência Social (PNAS), com o Estudo Diagnóstico
objetivo de concretizar direitos assegurados na Constituição
Federal (1988) e na Lei Orgânica de Assistência Social (1993). O estudo diagnóstico tem como objetivo subsidiar a
A PNAS organiza a matriz de funcionamento do Sistema Único decisão acerca do afastamento da criança ou adolescente do
de Assistência Social (SUAS), inaugurando no país um novo convívio familiar. Salvo em situações de caráter emergencial
paradigma de defesa dos direitos socioassistenciais. Na e/ou de urgência, esta medida deve ser aplicada por

Conhecimentos Específicos 104


Apostila Digital Licenciada para Alice Caroline Guarino dos Santos - alice.guarino@hotmail.com (Proibida a Revenda)
APOSTILAS OPÇÃO

autoridade competente (Conselho Tutelar ou Justiça da de violência intrafamiliar, se há possibilidade de afastamento


Infância e da Juventude), com base em uma recomendação do agressor da moradia comum para a manutenção da criança
técnica, a partir de um estudo diagnóstico, caso a caso, ou adolescente na moradia em condições de proteção e
realizado por equipe interprofissional do órgão aplicador da segurança (Art. 130 do ECA); grau de risco e desproteção ao
medida ou por equipe formalmente designada para este fim. qual a criança ou adolescente estará exposto se não for
Em todos os casos, a realização deste estudo diagnóstico deve afastada do ambiente familiar; verificação da existência de
ser realizada sob supervisão e estreita articulação com pessoas significativas da comunidade para a criança ou
Conselho Tutelar, Justiça da Infância e da Juventude e equipe adolescente que possam vir a acolhê-los, de forma segura, no
de referência do órgão gestor da Assistência Social. Sempre caso de necessidade de afastamento da família de origem.
que necessário, o órgão aplicador da medida poderá requisitar, O processo de avaliação diagnóstica deve incluir uma
ainda, avaliação da situação por parte de outros serviços da escuta qualificada de todos os envolvidos: integrantes da
rede como, por exemplo, da Delegacia de Proteção da Criança família, inclusive a criança e o adolescente, pessoas da
e do Adolescente e de serviços de saúde. comunidade com vínculos significativos com a família, a
Os fluxos e responsabilidades referentes à realização do criança e o adolescente e profissionais que estejam prestando-
estudo diagnóstico deverão ser definidos a partir de acordos lhes atendimento, dentre outros.
formais firmados entre os órgãos envolvidos, considerando a Além da avaliação dos riscos aos quais porventura a
realidade, os recursos existentes e o respeito às competências criança ou adolescente estejam expostos no ambiente familiar,
legais de cada órgão da rede de atendimento e do Sistema de antes de considerar o encaminhamento para serviço de
Garantia de Direitos. acolhimento como uma alternativa para garantir sua proteção,
O estudo diagnóstico deve incluir uma criteriosa avaliação é preciso observar se na família extensa ou comunidade há
dos riscos a que estão submetidos à criança ou o adolescente e pessoas significativas que possam e aceitem se responsabilizar
as condições da família para superação das violações de por seus cuidados.
direitos observadas e o provimento de proteção e cuidados. No estudo diagnóstico deve-se buscar identificar, ainda, se
Com a devida fundamentação teórica, o estudo deve levar em a situação de risco à qual está exposta a criança ou adolescente
conta a proteção e a segurança imediata da criança e do decorre exclusivamente do contexto social, histórico e
adolescente, bem como seu cuidado e desenvolvimento em econômico de vida da família e se a garantia de apoio,
longo prazo. orientação e acesso às diversas políticas públicas seria
Sem a pretensão de abarcar todas as questões relevantes suficiente para reduzir os riscos e possibilitar a manutenção
que deverão ser levantadas em um diagnóstico, sugere-se que do convívio familiar.
o mesmo possibilite identificar: composição familiar, história Além de avaliar se há necessidade ou não de afastamento
e dinâmica de relacionamento entre seus membros; valores e do convívio familiar, o estudo diagnóstico deve analisar o
crenças da família; demandas e estratégias desenvolvidas para perfil e as demandas específicas da criança ou adolescente, de
o enfrentamento de situações adversas; e situações de forma a subsidiar a decisão pelo encaminhamento para o
vulnerabilidade e risco às quais estão expostos os integrantes serviço de acolhimento que melhor atenda às suas
do grupo familiar. Nessa perspectiva, recomenda-se que o peculiaridades. Para a identificação do serviço mais adequado,
estudo diagnóstico contemple, dentre outros, os seguintes deve-se considerar, ainda, sua estrutura física, recursos
aspectos: humanos e Projeto Político-Pedagógico, além da possibilidade
Composição familiar e contexto socioeconômico e cultural de manutenção de vínculos comunitários da criança ou
no qual a família está inserida; mapeamento dos vínculos adolescente e a continuidade da frequência à mesma escola e
significativos na família extensa e análise da rede social de aos equipamentos comunitários aos quais estejam vinculados.
apoio da criança ou adolescente e de sua família (família Salvo nos casos em que o afastamento de sua comunidade de
extensa, amigos, vizinhos, padrinhos, instituições, etc.); origem for essencial para garantir-lhes segurança, deve-se
valores e costumes da comunidade da qual a família faça parte, evitar que a inclusão em um serviço de acolhimento resulte no
especialmente, no caso de minorias étnicas ou comunidades rompimento ou na fragilização dos vínculos comunitários e de
tradicionais; condições de acesso da família a serviços, pertencimento preexistentes.
programas e projetos das diversas políticas públicas que
possam responder às suas necessidades; situações de Plano de Atendimento Individual e Familiar
vulnerabilidade e risco vivenciadas pela família que
repercutam sobre sua capacidade de prover cuidados; Assim que a criança ou adolescente chegar ao serviço de
situação atual da criança ou adolescente e de sua família, acolhimento, a equipe técnica do serviço, que, onde houver,
inclusive motivação, potencial e dificuldades da família para poderá contar com a contribuição da equipe responsável pela
exercer seu papel de cuidado e proteção; história familiar e se supervisão dos serviços de acolhimento (ligada ao órgão
há padrões transgeracionais de relacionamento com violação gestor da Assistência Social) para elaborar um Plano de
de direitos; situações atuais e pregressas de violência Atendimento Individual e Familiar, no qual constem objetivos,
intrafamiliar contra a criança e o adolescente, gravidade e estratégias e ações a serem desenvolvidos tendo em vista a
postura de cada membro da família em relação à mesma; nos superação dos motivos que levaram ao afastamento do
casos de violência intrafamiliar, se há consciência da convívio e o atendimento das necessidades específicas de cada
inadequação e das consequências negativas destas práticas situação. A elaboração deste Plano de Atendimento deve ser
para a criança e o adolescente e se há movimento em direção à realizada em parceria com o Conselho Tutelar e, sempre que
mudança e à construção de novas possibilidades de possível, com a equipe interprofissional da Justiça da Infância
relacionamento; análise da intensidade e qualidade dos e da Juventude. Tal Plano deverá partir das situações
vínculos entre os membros da família (vinculação afetiva, identificadas no estudo diagnóstico inicial que embasou o
interação, interesse e participação na vida da criança e do afastamento do convívio familiar.
adolescente); percepção da criança ou adolescente em relação Quando o acolhimento tiver sido realizado em caráter
à possibilidade de afastamento do convívio familiar – se emergencial e/ou de urgência, sem estudo diagnóstico prévio,
demonstra, por exemplo, medo de permanecer na família ou recomenda-se que este estudo seja realizado em até vinte dias
tristeza por afastar-se da mesma; possibilidade de intervenção após o acolhimento, a fim de avaliar a real necessidade da
profissional e encaminhamentos que visem à superação da medida ou a possibilidade imediata de retorno da criança ou
situação de violação de direitos, sem a necessidade de adolescente ao convívio familiar. Se o acolhimento
afastamento da criança e do adolescente da família; nos casos emergencial tiver sido realizado sem prévia determinação da

Conhecimentos Específicos 105


Apostila Digital Licenciada para Alice Caroline Guarino dos Santos - alice.guarino@hotmail.com (Proibida a Revenda)
APOSTILAS OPÇÃO

autoridade competente, esta deverá ser comunicada até o 2º A partir deste levantamento inicial devem ser definidas
dia útil imediato, conforme o Art. 93 do ECA. estratégias de atuação que contribuam para a superação dos
No caso de crianças e adolescentes acolhidos sem motivos que levaram ao acolhimento. Tais estratégias devem
referência familiar, o fato deve ser comunicado à Delegacia de primar pelo fortalecimento dos recursos e das potencialidades
Proteção à Criança e ao Adolescente ou, quando não houver, à da família (nuclear ou extensa), da criança, do adolescente, da
delegacia mais próxima. O serviço de acolhimento, em parceria comunidade e da rede local, a fim de possibilitar o
com a referida delegacia, deve consultar o Cadastro Nacional desenvolvimento de um trabalho que possa conduzir a
de Crianças e Adolescentes Desaparecidos e outros Cadastros soluções de caráter mais definitivo, como a reintegração
similares existentes nos Estados, a fim de verificar se não se familiar, a colocação sob cuidados de pessoa significativa da
trata de criança ou adolescente desaparecido. Nestes casos, comunidade ou, quando essa se mostrar a alternativa que
tendo em vista a localização de pais ou responsáveis, deve-se, melhor atenda ao superior interesse da criança e do
ainda, por meio de métodos adequados, buscar informações adolescente, o encaminhamento para adoção. Quando se tratar
com a própria criança ou adolescente como, por exemplo: seu de adolescente com idade próxima à maioridade com remotas
nome completo, idade, nome de pessoas da família, escola perspectivas de colocação em família substituta devem ser
onde estudava, bairro ou pontos de referência de sua moradia, viabilizadas ações destinadas à preparação para a vida
município de procedência, etc. Recomenda-se que estas autônoma.
informações sejam levantadas de modo gradativo no período O Plano de Atendimento Individual e Familiar deve
inicial do acolhimento, pois, dependendo da idade, a criança orientar as intervenções a serem desenvolvidas para o
pode não se lembrar de tais informações após certo período. acompanhamento de cada caso, devendo contemplar, dentre
Os cadastros de crianças e adolescentes desaparecidos outras, estratégias para:
devem também ser consultados no caso de crianças e - desenvolvimento saudável da criança e do adolescente
adolescentes acolhidos que estejam em processo de saída da durante o período de acolhimento: encaminhamentos
situação de rua. Nestas situações deve-se buscar analisar junto necessários para serviços da rede (saúde, educação,
à criança ou ao adolescente sua situação familiar, os motivos assistência social, esporte, cultura e outros); atividades para o
que levaram à saída de casa e o desejo de retomada do contato desenvolvimento da autonomia; acompanhamento da situação
com familiares, sobretudo com aqueles que a estiverem escolar; preservação e fortalecimento da convivência
procurando. comunitária e das redes sociais de apoio; construção de
A situação de todas aquelas crianças e adolescentes já projetos de vida; relacionamentos e interação no serviço de
acolhidos deve também ser revista, de modo a garantir que acolhimento – educadores/cuidadores, demais profissionais e
todos estejam em acompanhamento. Para estas situações colegas; preparação para ingresso no mundo do trabalho, etc;
deve-se também, na elaboração do Plano de Atendimento investimento nas possibilidades de reintegração familiar:
considerar os motivos do afastamento e as intervenções fortalecimento dos vínculos familiares e das redes sociais de
realizadas até o momento, a partir dos quais devem ser apoio; acompanhamento da família, em parceria com a rede,
delineadas outras intervenções necessárias tendo em vista o visando à superação dos motivos que levaram ao acolhimento;
alcance de soluções de caráter mais definitivo para a criança e potencialização de sua capacidade para o desempenho do
o adolescente. papel de cuidado e proteção; gradativa participação nas
O Plano de Atendimento tem como objetivo orientar o atividades que envolvam a criança e o adolescente; etc. Nos
trabalho de intervenção durante o período de acolhimento, casos de crianças e adolescentes em processo de saída da rua
visando à superação das situações que ensejaram a aplicação deve-se, ainda, buscar a identificação dos familiares, dos
da medida. Deve basear-se em um levantamento das motivos que conduziram à situação de rua e se há motivação e
particularidades, potencialidades e necessidades específicas possibilidades para a retomada da convivência familiar;
de cada caso e delinear estratégias para o seu atendimento. Tal acesso da família, da criança ou adolescente a serviços,
levantamento constitui um estudo da situação que deve programas e ações das diversas políticas públicas e do terceiro
contemplar, dentre outros aspectos: setor que contribuam para o alcance de condições favoráveis
ao retorno ao convívio familiar;
Motivos que levaram ao acolhimento e se já esteve - investimento nos vínculos afetivos com a família extensa
acolhido neste ou em outro serviço anteriormente, dentre e de pessoas significativas da comunidade: fortalecimento das
outros; vinculações afetivas e do papel na vida da criança e do
Configuração e dinâmica familiar, relacionamentos adolescente; apoio aos cuidados com a criança ou adolescente
afetivos na família nuclear e extensa, período do ciclo de vida no caso de reintegração familiar ou até mesmo
familiar, dificuldades e potencialidades da família no exercício - responsabilização por seu acolhimento;
de seu papel. - encaminhamento para adoção quando esgotadas as
Condições socioeconômicas, acesso a recursos, possibilidades de retorno ao convívio familiar: articulação
informações e serviços das diversas políticas públicas; com o Poder Judiciário e o Ministério Público para viabilizar,
Demandas específicas da criança, do adolescente e de sua nestes casos, o cadastramento para adoção. Desde que haja
família que requeiram encaminhamentos imediatos para a supervisão do Poder Judiciário, uma estratégia que pode ser
rede (sofrimento psíquico, abuso ou dependência de álcool e empreendida também pelos serviços de acolhimento, em
outras drogas, etc.), bem como potencialidades que possam parceria com Grupos de Apoio à Adoção ou similares, diz
ser estimuladas e desenvolvidas; respeito à busca ativa de famílias para a adoção de crianças e
Rede de relacionamentos sociais e vínculos institucionais adolescentes com perfil de difícil colocação familiar.
da criança, do adolescente e da família, composta por pessoas
significativas30 na comunidade, colegas, grupos de A elaboração do Plano de Atendimento Individual e
pertencimento, atividades coletivas que frequentam na Familiar deve envolver uma escuta qualificada da criança, do
comunidade, escola, instituições religiosas, etc.; adolescente e de sua família, bem como de pessoas que lhes
Violência e outras formas de violação de direitos na família, sejam significativas em seu convívio, de modo a compreender
seus significados e possível transgeracionalidade; a dinâmica familiar e as relações estabelecidas com o contexto.
Significado do afastamento do convívio e do serviço de É necessário que a criança, o adolescente e as famílias tenham
acolhimento para a criança, o adolescente e a família; papel ativo nesse processo e possam, junto aos técnicos e
demais integrantes da rede, pensar nos caminhos possíveis
para a superação das situações de risco e de violação de

Conhecimentos Específicos 106


Apostila Digital Licenciada para Alice Caroline Guarino dos Santos - alice.guarino@hotmail.com (Proibida a Revenda)
APOSTILAS OPÇÃO

direitos, participando da definição dos encaminhamentos, importante, ainda, para que a equipe técnica possa, no menor
intervenções e procedimentos que possam contribuir para o tempo possível, fazer sua análise quanto a real necessidade do
atendimento de suas demandas. Também devem ser ouvidos acolhimento. Caso conclua que a manutenção do afastamento
outros profissionais que porventura estejam atendendo ou da criança ou adolescente do convívio familiar não é
tenham atendido a criança, o adolescente ou a família, como necessária, a equipe técnica responsável pelo
nos casos de acompanhamento por equipes de saúde mental, acompanhamento deve proceder aos encaminhamentos para
de outros serviços da rede socioassistencial e da escola, dentre viabilizar a imediata reintegração. Nestas situações a família
outros. também deverá ser informada do seu direito a questionar o
Os Planos de Atendimento Individual e Familiar deverão afastamento e requerer, junto à Justiça, por intermédio de
ser encaminhados para conhecimento do Sistema de Justiça e advogado nomeado ou Defensor Público, a reintegração da
do Conselho Tutelar, em prazo previamente acordado. Tais criança ou adolescente (ECA, Art. 141).
órgãos devem acompanhar as intervenções realizadas com a Nos serviços onde porventura haja crianças e adolescentes
família, sendo acionados quando necessária a aplicação de já acolhidos cuja situação familiar não esteja sendo
outras medidas protetivas para assegurar o acesso da criança, acompanhada, a equipe técnica deve também iniciar estes
do adolescente ou da família aos serviços disponíveis na rede acompanhamentos, buscando soluções que contribuam para
(ECA, Art. 101, Inciso I a VI). assegurar a excepcionalidade e a provisoriedade do
O desenvolvimento das ações do Plano de Atendimento acolhimento. O acompanhamento da situação familiar, nestas
deve ser realizado de modo articulado com os demais órgãos e situações, é fundamental para se identificar:
serviços que estejam acompanhando a família, a criança ou o - crianças e adolescentes que permanecem acolhidos
adolescente (escola, Unidade Básica de Saúde, Estratégia de unicamente pela situação de pobreza de suas famílias: situação
Saúde da Família, CAPS, CREAS, CRAS, programas de geração que exige o investimento imediato em estratégias para seu
de trabalho e renda, etc.), a fim de que o trabalho conduza, no retorno ao convívio familiar;
menor tempo necessário, a uma resposta definitiva para a - crianças e adolescente, ou membros da família, com
criança e o adolescente, que não seja re-vitimizadora ou direito, mas sem acesso ao BPC. Nestes casos deve-se proceder
precipitada. Para tanto, deverão ser realizadas reuniões ao encaminhamento para o CRAS - ou órgão gestor
periódicas para estudo de cada caso pelos profissionais responsável - para seu cadastramento no Benefício, devendo
envolvidos, para acompanhamento da evolução do tal medida ser acompanhada do investimento nas
atendimento, verificação do alcance dos objetivos acordados, possibilidades de reintegração;
avaliação da necessidade de revisão do Plano de Atendimento - crianças e adolescentes sem possibilidades de
e elaboração de estratégias de ação que possam responder às reintegração ao convívio familiar (família nuclear ou extensa):
novas situações surgidas durante o atendimento. As situação que exige imediatos encaminhamentos para o
conclusões resultantes de tais reuniões servirão, inclusive, de Sistema de Justiça, para o cadastramento para adoção e a
subsídio para a elaboração de relatórios semestrais a serem Destituição do Poder Familiar.
enviados à autoridade judiciária e ao Ministério Público. A intervenção profissional na etapa inicial do
acompanhamento deve proporcionar, de modo construtivo, a
Os serviços de acolhimento devem construir uma conscientização por parte da família de origem dos motivos
sistemática de atendimento que possibilite o início da que levaram ao afastamento da criança e/ou do adolescente e
elaboração do Plano de Atendimento Individual e Familiar das consequências que podem advir do fato. Esta
imediatamente após o acolhimento da criança ou adolescente, conscientização é fundamental para que as próximas etapas
para que se alcance, no menor tempo necessário, soluções de possam ser planejadas, com acordos firmados entre serviço e
caráter mais definitivo. Cabe ressaltar, finalmente, que a família, com vistas ao desenvolvimento de ações proativas que
ênfase do Plano de Atendimento deve ser na construção de contribuam para a superação de situações adversas ou
estratégias para o atendimento, de modo a não transformá-lo padrões violadores que possam ter levado ao afastamento. A
em mera formalidade. Um registro sintético do Plano de equipe técnica do serviço de acolhimento deve, ainda,
Atendimento não deve, ainda, significar sua limitação às acompanhar o trabalho desenvolvido com a família na rede
estratégias inicialmente elaboradas, devendo-se garantir que local, mantendo-a informada, inclusive, a respeito de possíveis
seja sempre dinâmico e aberto a mudanças, reformulações e decisões por parte da Justiça.
aprimoramento, baseado nas intervenções realizadas e em Devem ser firmados acordos entre o serviço de
seus resultados. acolhimento, a equipe de supervisão e apoio aos serviços de
acolhimento - ligada ao órgão gestor da Assistência Social - a
Acompanhamento da Família de Origem equipe técnica do Poder Judiciário e os demais serviços da
rede das diversas políticas públicas, incluindo os não-
A implementação de uma sistemática de acompanhamento governamentais, a fim de promover a articulação das ações de
da situação familiar, iniciada imediatamente após o acompanhamento à família, além de reuniões periódicas para
acolhimento, é fundamental, pois, com o passar do tempo, discussão e acompanhamento dos casos.
tanto as possibilidades de reintegração familiar, quanto de Durante o período de acolhimento, o serviço deverá
adoção podem tornar-se mais difíceis. O prolongamento do encaminhar relatórios para a Justiça da Infância e da
afastamento da criança ou adolescente pode provocar Juventude com periodicidade mínima semestral, de modo a
enfraquecimento dos vínculos com a família, perda de subsidiar o acompanhamento da situação jurídico-familiar de
referências do contexto e de valores familiares e comunitários, cada criança/adolescente e a avaliação por parte da Justiça da
exigindo preparação ainda mais cuidadosa no caso de possibilidade de reintegração familiar ou necessidade de
reintegração familiar. Do mesmo modo, o prolongamento encaminhamento para família substituta, sobretudo nos casos
desnecessário do encaminhamento para adoção, nas situações em que o prognóstico de permanência da criança e do
em que esta constitui a melhor medida para a criança ou adolescente no serviço de acolhimento for de mais de dois
adolescente, pode reduzir consideravelmente as anos.
possibilidades de colocação familiar, em virtude das No trabalho com a família devem ser consideradas tanto as
dificuldades observadas no contexto brasileiro para a questões objetivas, quanto subjetivas. Considerar essas duas
realização de adoções de crianças maiores e de adolescentes. dimensões, objetiva e subjetiva, é importante para apoiar a
Dar início ao acompanhamento da situação familiar família no processo de reintegração e prevenir novos
imediatamente após a chegada da criança ou adolescente é afastamentos. Ressalte-se que lidar com questões objetivas é

Conhecimentos Específicos 107


Apostila Digital Licenciada para Alice Caroline Guarino dos Santos - alice.guarino@hotmail.com (Proibida a Revenda)
APOSTILAS OPÇÃO

fundamental para prevenir situações que possam gerar uma - A reflexão por parte da família acerca de suas
tensão excessiva na família, em função do processo de exclusão responsabilidades, de sua dinâmica de relacionamento
social ou das dificuldades reais para conciliar o cuidado com a intrafamiliar e de padrões de relacionamentos que violem
criança e o adolescente e a sobrevivência do núcleo familiar. direitos;
Tais situações podem dificultar, inclusive, o desenvolvimento - O desenvolvimento de novas estratégias para a resolução
de relações afetivas que contribuam para o exercício de seu de conflitos;
papel de proteção e cuidados. - O fortalecimento da autoestima e das competências da
Diversas técnicas podem ser utilizadas no família, de modo a estimular sua resiliência, ou seja, o
acompanhamento às famílias, como, por exemplo: aprendizado com a experiência e a possibilidade de superação
Estudo de caso: reflexão coletiva que deve partir das dos desafios;
informações disponíveis sobre a família e incluir resultados - O fortalecimento da autonomia, tanto do ponto de vista
das intervenções realizadas. Na medida do possível deve ser socioeconômico, quanto do ponto de vista emocional, para a
realizado com a participação dos profissionais do serviço de construção de possibilidades que viabilizem a retomada do
acolhimento, da equipe de supervisão do órgão gestor, da convívio com a criança e o adolescente.
Justiça da Infância e da Juventude e de outros serviços da rede - O fortalecimento das redes sociais de apoio da família;
que acompanhem a família; - O fortalecimento das alternativas para gerar renda e para
Entrevista individual e familiar: estratégia importante, garantir a sobrevivência da família.
particularmente nos primeiros contatos com a família e seus
membros, que permite avaliar a expectativa da família quanto Além das questões da metodologia do trabalho, as crenças
à reintegração familiar e elaborar conjuntamente o Plano de dos profissionais acerca das famílias e o modo como se
Atendimento. Esse instrumento também pode ser utilizado relacionam com as mesmas, também influenciam os
para abordar outras questões específicas, para aprofundar o resultados das intervenções. Assim, é preciso estar atento à:
conhecimento sobre a família e para fortalecer a relação de Postura de respeito à diversidade, aos diferentes arranjos
confiança com o serviço. Nas entrevistas podem ser realizados, familiares e às mais distintas estratégias às quais as famílias
ainda, o genograma, o mapa de rede social, dentre outras podem recorrer para lidar com situações adversas. Diversas
técnicas. experiências têm demonstrado que o trabalho bem-sucedido
Grupo com famílias: dentre outros aspectos, favorece a de reintegração familiar está fortemente associado à
comunicação com a família, a troca de experiências entre possibilidade de construção de um vínculo de referência
famílias e a aprendizagem e o apoio mútuos. Possibilita a significativo da família com profissionais que a acompanhem,
reflexão sobre as relações familiares e responsabilidades da aos quais possa recorrer, inclusive, em momentos de crise no
família na garantia dos direitos de seus membros e sobre os período pós-reintegração. O fortalecimento dos recursos da
aspectos concernentes ao acolhimento. Constitui importante família para cuidar da criança e do adolescente está
estratégia para potencialização dos recursos da família para o fortemente associado às possibilidades de sentir-se também
engajamento nas ações necessárias para retomada do convívio acolhida e cuidada.
familiar com a criança ou adolescente; Grupo Multifamiliar: A crença por parte dos profissionais nas possibilidades de
espaço importante para trocas de experiências, reflexões e reconstrução das relações, também representa aspecto que
discussão com as famílias, incluindo a participação de crianças pode contribuir para o fortalecimento da confiança da família
e adolescentes acolhidos. O Grupo Multifamiliar permite a em sua capacidade de se responsabilizar novamente pelos
compreensão de diferentes pontos de vista dos cuidados à criança e ao adolescente e superar os motivos que
relacionamento familiares e das diferenças entre gerações. conduziram ao afastamento.
Visita Domiciliar: importante recurso para conhecer o O acompanhamento da família deve ser sistemático para
contexto e a dinâmica familiar e identificar demandas, que, em um prazo de até dois anos, seja possível viabilizar o
necessidades, vulnerabilidades e riscos. Referenciada no retorno da criança ou adolescente ao convívio com sua família
princípio do respeito à privacidade, a visita possibilita uma ou, na sua impossibilidade, o encaminhamento para família
aproximação com a família e a construção de um vínculo de substituta. A questão do tempo de permanência no serviço de
confiança, necessário para o desenvolvimento do trabalho. acolhimento deve ser um dos eixos principais da avaliação
Orientação individual, grupal e familiar: intervenções que acerca da medida que melhor atende ao interesse da criança e
têm como objetivo informar, esclarecer e orientar pais e do adolescente. Nesse sentido, há que se considerar os
responsáveis sobre diversos aspectos, como a medida de prejuízos ao desenvolvimento da criança e do adolescente que
proteção aplicada e os procedimentos dela decorrentes. Deve possam advir tanto da permanência prolongada quanto de um
pautar-se em uma metodologia participativa que possibilite a rompimento definitivo dos vínculos com a família de origem. A
participação ativa da família; questão do tempo deve levar em consideração, sobretudo, a
Encaminhamento e acompanhamento de integrantes da idade da criança e do adolescente e o significado da privação
família à rede local, de acordo com demandas identificadas: do convívio familiar na etapa do ciclo de vida na qual se
psicoterapia, tratamento de uso, abuso ou dependência de encontram.
álcool e outras drogas, outros tratamentos na área de saúde, Embora o tempo deva ser um eixo central, as decisões não
geração de trabalho e renda, educação de jovens e adultos, etc. podem ser prematuras ou tomadas de modo precipitado.
Como resultado dessas atividades, de forma geral o Quando isso ocorre sem uma preparação adequada da criança,
acompanhamento familiar deve contribuir para: do adolescente e da família, a reintegração familiar pode ser
- A acolhida da família, a compreensão de sua dinâmica de conflituosa e acabar resultando em um novo afastamento do
funcionamento, valores e cultura; lar de origem. Nos casos de colocação em família substituta
A conscientização por parte da família de sua importância (adoção, guarda ou tutela), a falta de uma preparação
para a criança e o adolescente e das decisões definitivas que adequada de todos os envolvidos pode conduzir também a
podem vir a ser tomadas por parte da Justiça, baseadas no fato situações que dificultem a construção da vinculação afetiva,
da criança e do adolescente serem destinatários de direitos; resultando em retorno ao serviço de acolhimento. É
- A compreensão das estratégias de sobrevivência importante, portanto, que um acompanhamento sistemático
adotadas pela família e das dificuldades encontradas para possibilite a inserção familiar no menor tempo necessário, mas
prestar cuidados à criança e ao adolescente e para ter acesso com preparação adequada de todos os envolvidos.
às políticas públicas; Quando o desenvolvimento do trabalho de
acompanhamento familiar indicar a possibilidade de

Conhecimentos Específicos 108


Apostila Digital Licenciada para Alice Caroline Guarino dos Santos - alice.guarino@hotmail.com (Proibida a Revenda)
APOSTILAS OPÇÃO

reintegração da criança ou adolescente à sua família, nuclear Após a reintegração familiar é importante que o período de
ou extensa, a equipe técnica do serviço de acolhimento, em adaptação mútua entre criança/adolescente e família seja
parceria com a equipe responsável pela supervisão dos acompanhado por pelo menos seis meses, após os quais
serviços de acolhimento (ligada ao órgão gestor da Assistência deverá avaliar-se a necessidade de sua continuidade. Nesse
Social) e, sempre que possível, a equipe interprofissional da momento, devido às mudanças ocorridas em cada um dos
Justiça da Infância e da Juventude, deverá elaborar estratégias membros durante o período do afastamento e o descompasso
para a reintegração familiar que contemplem os entre expectativas construídas e realidade, podem ocorrer
encaminhamentos necessários para viabilizar o retorno entre os integrantes da família insegurança e conflitos na
seguro da criança ou adolescente à sua família e o relação devido à necessidade de readaptação da rotina e regas
fortalecimento desta para o exercício de seu papel de cuidado familiares. Por isso é preciso que sejam desenvolvidas
e proteção. estratégias para conciliar os cuidados com a criança e o
Para decidir se a reintegração é a melhor medida também adolescente com as demais responsabilidades da família.
devem ser ouvidos, a criança e ao adolescente, por meios O acompanhamento psicossocial nesse momento é
adequados, bem como suas famílias e os fundamental para auxiliar a família, a criança e o adolescente
educadores/cuidadores ou famílias acolhedoras. Para o a construírem novas possibilidades para estarem juntos,
trabalho com a reintegração familiar e decisão acerca da apesar da separação vivida. O apoio profissional será
melhor medida para a criança ou adolescente, é importante fundamental, ainda, para que a família se aproprie de novos
considerar, ainda, dentre outros, os seguintes aspectos: padrões de relacionamento mais saudáveis e favorecedores do
- as reações da criança, do adolescente e da família ao desenvolvimento. A definição quanto ao órgão responsável
afastamento e ao acolhimento no serviço; pelo acompanhamento no período após a reintegração
- a vinculação afetiva e a motivação mútua da família, da familiar deverá ser objeto de acordo formal entre os serviços
criança e do adolescente pela continuidade da relação afetiva de acolhimento, o órgão gestor da Assistência Social e a Justiça
e retomada do convívio; da Infância e da Juventude. Tal definição deve levar em
- se os encaminhamentos realizados foram viabilizados e consideração a estrutura e a capacidade técnica dos serviços
qual tem sido a resposta da família, da criança e do adolescente da rede local, podendo ser designada para esse fim a equipe
aos mesmos; técnica dos serviços de acolhimento, a equipe responsável pela
- se há, por parte da família, conscientização dos motivos supervisão dos serviços de acolhimento, o CREAS, ou até
que levaram ao afastamento da criança ou adolescente e mesmo o CRAS ou outro serviço de atendimento sociofamiliar
motivação para superá-los; existente no Município.
- se há movimento de mudança nos padrões de Nos casos em que forem esgotadas as possibilidades de
relacionamento entre os membros da família e desta com a reintegração familiar (família nuclear ou extensa), a equipe
comunidade e instituições sociais; técnica do serviço de acolhimento deverá elaborar e enviar à
- se existem membros da família (nuclear ou extensa) que autoridade judiciária relatório circunstanciado onde sejam
possam se responsabilizar e compartilhar os cuidados com a relatadas a situação familiar da criança ou adolescente, as
criança e o adolescente; intervenções realizadas com vistas à reintegração familiar e os
- se a família possui redes sociais de apoio da família: resultados obtidos, sugerindo a Destituição do Poder Familiar
vinculações significativas da família com pessoas da e a inserção da criança ou adolescente no cadastro para
comunidade e serviços que possam apoiar os cuidados à adoção.
criança e ao adolescente. Nos casos de encaminhamento para adoção, deve ser
Quando a reintegração familiar for considerada a melhor realizado um planejamento por parte da equipe do serviço de
medida, a preparação para o retorno deverá incluir uma acolhimento, da Justiça da Infância e da Juventude e, onde
crescente participação da família na vida da criança e do houver, do Grupo de Apoio à Adoção, com vistas à preparação
adolescente, inclusive no cumprimento das responsabilidades prévia de todos os envolvidos e a aproximação gradativa dos
parentais. Nesse sentido, deve ser propiciada a inserção da adotantes e da criança/adolescente. Além da preparação dos
família em atividades que envolvam a criança e o adolescente adotantes e da criança/adolescente, o educador/cuidador ou a
como, por exemplo, reuniões escolares, consultas de saúde, família acolhedora deverá também ser incluído no processo,
comemoração do aniversário, atividades na comunidade, sendo, inclusive, orientado quanto à preparação da
escola, etc. Do mesmo modo, a criança e o adolescente devem criança/adolescente para a adoção.
ter a oportunidade de uma reinserção gradual no contexto de Particularmente em relação ao encaminhamento para
origem, passando finais de semana ou datas comemorativas na família substituta, constitui papel, ainda, a ser desenvolvido
casa da família, por exemplo. Nesse momento é importante, em parceria entre o serviço de acolhimento, Poder Judiciário e
ainda, que sejam fortalecidas as redes sociais de apoio da Grupos de Apoio à Adoção, a busca ativa de famílias para
família, fundamentais para o exercício de seu papel de aquelas crianças e adolescentes com perfil de difícil colocação
cuidadora. em adoção.
É importante destacar que a reintegração com familiares É de fundamental importância que o órgão gestor da
com os quais a criança e o adolescente não possuam vínculo Política de Assistência Social, o Poder Judiciário e o Ministério
afetivo deve ser cuidadosamente avaliada, não devendo ser Público estabeleçam, de forma pactuada com os demais
conduzida meramente com base em uma “supervalorização operadores da rede de acolhida, fluxos, prazos e
dos laços consanguíneos”. Nesses casos, deve-se avaliar a procedimentos que viabilizem no menor tempo possível, tão
possibilidade de construção de vinculações significativas e de logo haja recomendação técnica, a reintegração familiar ou, na
aceitação mútua do convívio, para se decidir quanto à melhor sua impossibilidade, o encaminhamento para família
alternativa a ser recomendada à autoridade judiciária: substituta.
reintegração com família extensa, colocação com pessoa É importante que sejam implementadas as medidas
significativa da comunidade ou adoção. Quando a colocação necessárias em cada localidade para o fortalecimento das
com familiar sem vinculação prévia representar a melhor condições para o acompanhamento sistemático da situação
medida, deve-se preparar previamente todos os envolvidos, familiar, de modo a prevenir que a fragilidade na rede local
por meio de uma gradativa aproximação que possibilite a conduza à frequente insegurança por parte dos profissionais
construção da vinculação afetiva, fundamental para prevenir para tomar decisões definitivas sobre o futuro da criança e do
situações futuras de rejeição e, até mesmo, de retorno da adolescente. Devem-se criar condições e conscientização por
criança ou adolescente ao serviço de acolhimento. parte dos envolvidos acerca da importância destas decisões

Conhecimentos Específicos 109


Apostila Digital Licenciada para Alice Caroline Guarino dos Santos - alice.guarino@hotmail.com (Proibida a Revenda)
APOSTILAS OPÇÃO

delicadas, mas necessárias. A criança e o adolescente não CRAS: sempre que se identificar a necessidade de ações de
podem permanecer por tempo indefinido no serviço de proteção social básica para criança e adolescente atendido em
acolhimento em razão da dificuldade de se construir condições serviços de acolhimento ou para suas famílias, deverá ser
locais favoráveis para essa tomada de decisão. articulada sua inclusão em tais atividades por meio da equipe
do CRAS do território de moradia da família. Para dar agilidade
Articulação Intersetorial a tais procedimentos, recomenda-se que sejam definidos, de
forma conjunta, fluxos de encaminhamento e canais de
Os Serviços de Acolhimento integram o Sistema Único de comunicação entre os serviços de acolhimento e o(s) CRAS,
Assistência Social (SUAS), tendo interface com outros serviços além de encontros periódicos, que possibilitem o
da rede socioassistencial, quanto com demais órgãos do acompanhamento das ações.
Sistema de Garantia de Direitos. Sua atuação deve basear-se O CRAS de referência do território de moradia da família,
no princípio da incompletude institucional, não devendo sempre que necessário, deverá ser acionado para participar do
ofertar em seu interior atividades que sejam da competência processo de reintegração familiar de crianças e adolescentes
de outros serviços. A proteção integral a que têm direito as atendidas em serviços de acolhimento. Sua atuação se faz
crianças e os adolescentes acolhidos deve ser viabilizada por necessária para a inclusão da criança ou do adolescente que
meio da utilização de equipamentos comunitários e da rede de estiver sendo reintegrado à família, e de seus familiares ou
serviços local. responsáveis, em serviços, programas e ações de
Dessa forma, para que as intervenções realizadas junto às fortalecimento dos vínculos familiares e comunitários, bem
crianças e aos adolescentes acolhidos e suas famílias sejam como para fazer os encaminhamentos que se mostrarem
efetivas, é necessário que haja uma estreita articulação entre necessários com a retomada do convívio familiar, de modo a
os diversos órgãos envolvidos no seu atendimento. Assim, facilitar sua inclusão social e comunitária nesse período de
para fortalecer a complementaridade das ações e evitar vulnerabilidade.
sobreposições, é importante que esta articulação proporcione CREAS: Nos municípios que possuam CREAS e naqueles
o planejamento e o desenvolvimento conjunto de estratégias atendidos por CREAS regionais, quando o motivo do
de intervenção, sendo definido o papel de cada instância que afastamento do convívio familiar envolver violência
compõe a rede de serviços local e o Sistema de Garantia de intrafamiliar (física, psicológica, sexual, negligência grave),
Direitos, na busca de um objetivo comum. exploração sexual ou outras situações de violação de direitos
que estejam sob o escopo de ação dos serviços desenvolvidos
Articulação no âmbito do Sistema Único de Assistência no CREAS, as crianças e adolescentes acolhidos e seus
Social - SUAS familiares devem ser inseridos em seus serviços. Nesse caso, é
de suma importância que as equipes técnicas do serviço de
Para a garantia de um atendimento de qualidade às acolhimento e do CREAS atuem de forma articulada – com
crianças e aos adolescentes acolhidos e às suas famílias, os planejamento conjunto de estratégias de ação e reuniões
serviços de acolhimento devem funcionar de forma articulada periódicas para o acompanhamento dos casos – de modo a
com os demais serviços da rede socioassistencial local. Tal garantir uma atuação complementar e sinérgica, evitando
articulação possibilitará a inserção dos usuários nos demais sobreposições e ações contraditórias.
serviços, programas e ações que integram o SUAS, que se Equipe de Supervisão e Apoio aos Serviços de
fizerem necessários ao atendimento às demandas específicas Acolhimento: Em municípios de médio e grande porte e nas
de cada caso, favorecendo a integração comunitária e social metrópoles - e nos demais quando a demanda justificar - o
dos usuários. órgão gestor da Assistência Social deverá manter equipe
Como exemplo, pode-se citar a inserção de crianças e profissional especializada de referência, para supervisão e
adolescentes acolhidos – ou reintegrados ao convívio familiar apoio aos serviços de Acolhimento. De acordo com a realidade
- em atividades socioeducativas e de convivência ofertadas e as definições locais, tal equipe poderá compor um serviço
pela rede de proteção social básica, que pode ser-lhes especificamente voltado a esta função ou, ainda, estar
particularmente benéfica. Nessa direção, cabe destacar vinculada ao CREAS ou diretamente ao órgão gestor. Em todos
também que os adolescentes em serviços de acolhimento ou os casos, terá como atribuições mínimas:
egressos destes serviços constituem um dos públicos i. mapear a rede existente e fortalecer a articulação dos
prioritários de diversos programas como, por exemplo, o serviços de acolhimento com os demais serviços da rede
Projovem Adolescente. O encaminhamento para este socioassistencial, das demais políticas públicas e do SGD;
programa pode, inclusive, favorecer o processo de ii. monitorar as vagas na rede de acolhimento,
fortalecimento da autonomia, socialização e preparação dos indicando o serviço que melhor atenda às necessidades
adolescentes para o desligamento do serviço de acolhimento. específicas de cada caso encaminhado; iii. prestar supervisão
Da mesma forma, a participação em programas de inclusão e suporte técnico aos serviços de acolhimento;
produtiva, dos adolescentes maiores de 16 anos e dos iv. apoiar as equipes técnicas dos serviços de
familiares das crianças e adolescentes acolhidos ou acolhimento no acompanhamento psicossocial das famílias de
reintegrados, pode contribuir significativamente para o origem das crianças e adolescentes acolhidos;
desenvolvimento da autonomia e acesso à renda. A inclusão v. efetivar os encaminhamentos necessários, em
das famílias em atividades de apoio sociofamiliar pode articulação com os demais serviços da Rede Socioassistencial,
contribuir tanto para a construção de condições favoráveis à das demais Políticas Públicas e do SGD, monitorando,
reintegração familiar quanto para o desenvolvimento de posteriormente, seus desdobramentos;
relações saudáveis no período pós-reintegração. Ressalte-se, vi. Monitorar a situação de todas as crianças e
ainda, a necessidade de encaminhamento aos órgãos adolescentes que estejam em serviços de acolhimento no
responsáveis pela viabilização da inserção em programas de município, e de suas famílias, organizando, inclusive, cadastro
transferência de renda – como o PBF e o BPC - daqueles que permanentemente atualizado contendo o registro de todas as
preencherem os critérios de elegibilidade de tais programas e crianças e adolescentes atendidos nesses serviços;
benefícios.
Destaca-se a seguir, alguns dos principais equipamentos Articulação com o Sistema Único de Saúde - SUS
de referência do Sistema Único de Assistência Social, bem
como a equipe de supervisão e apoio aos serviços de O atendimento humanizado de crianças e adolescentes em
acolhimento, ligada ao órgão gestor: serviços de acolhimento requer uma estreita articulação entre

Conhecimentos Específicos 110


Apostila Digital Licenciada para Alice Caroline Guarino dos Santos - alice.guarino@hotmail.com (Proibida a Revenda)
APOSTILAS OPÇÃO

o Sistema Único de Saúde - SUS e o Sistema Único de acolhidos, incentivando, inclusive, sua participação ativa nas
Assistência Social - SUAS. Desse modo, orienta-se que os reuniões e comemorações escolares.
órgãos gestores dessas duas políticas desenvolvam estratégias Sempre que possível, deve-se procurar manter a criança ou
conjuntas e elaborem protocolos de atenção integral à saúde adolescente na mesma escola em que estudava antes da
de crianças e adolescentes que se encontram em Serviços de aplicação da medida protetiva, de modo a evitar rompimentos
Acolhimento, bem como de seus familiares. desnecessários de vínculos de amizade e de pertencimento e
As ações de promoção da saúde, ações educativas e de modificações radicais em sua rotina, além de prejuízos
prevenção de agravos devem ser articuladas com a Rede de acadêmicos. Constituem exceções a tal recomendação as
Atenção Básica/Primária, que é composta por Unidades Básica situações com determinação judicial em contrário ou
de Saúde da Família e Postos de Saúde. Esses serviços também recomendação técnica de mudança de escola por questões
devem ser procurados caso haja necessidade de solicitar relativas à preservação da segurança e proteção da criança ou
requisição de exames, medicamentos básicos e adolescente.
acompanhamento do quadro de situação de saúde de crianças É Importante, ainda, promover a inclusão de crianças e
e adolescentes em situação de acolhimento. Caso haja adolescentes que estão em serviços de acolhimento nas
necessidade, a equipe desses serviços fará também atividades propostas pelo Programa Mais Educação, em
encaminhamento para unidades de atenção especializada - drogas e todos aqueles que, por sua condição psíquica, estão
que inclui Hospitais, Maternidades, Unidades de Urgências impossibilitados de manter ou estabelecer laços sociais. A
e/ou Emergências e Serviços/Unidade de Referências - experiência acumulada em serviços que já funcionavam
capacitados para atenderem casos que demandem atenção segundo a lógica da atenção diária indica que se ampliam as
especializada, como adolescentes grávidas, crianças e possibilidades do tratamento para crianças e adolescentes
adolescentes com deficiência, com distúrbios de crescimento, quando o atendimento tem início o mais cedo possível,
com doenças infectocontagiosas ou imunodepressoras, dentre devendo, portanto, os CAPSi estabelecerem as parcerias
outros. necessárias com a rede de saúde, educação e assistência social
Nos casos de crianças e adolescentes com transtornos ligadas ao cuidado da população infanto-juvenil.
mentais e/ou que apresentam problemas devido ao uso A articulação com o sistema educacional permite, ainda,
abusivo ou dependência de álcool e outras drogas, deve ser desenvolver ações de conscientização e sensibilização de
acionada a rede de saúde mental, por meio das ações de saúde professores e demais profissionais da escola, de modo a que
mental na Atenção Básica, do Centro de Atenção Psicossocial estes atuem como agentes facilitadores da integração das
(CAPS) ou, onde houver, o Centro de Atenção Psicossocial crianças e adolescentes no ambiente escolar, evitando ou
Infanto-juvenil (CAPSi) , especializado no atendimento de superando possíveis situações de preconceito ou
crianças e adolescentes com transtornos mentais graves discriminação. Essas ações de capacitação podem ser
(autismo, psicoses, neuroses graves, abuso ou dependência de fortalecidas por meio do Projeto Escola que Protege, em cuja
álcool e outras drogas). proposta de formação continuada de profissionais da
Na articulação com o órgão gestor da saúde, devem ainda educação básica e da Rede de Proteção Integral, pode ser
estar previstas, ainda, ações de capacitação e inserida a temática da criança e do adolescente afastados do
acompanhamento dos educadores/cuidadores, além de convívio familiar.
profissionais dos serviços de acolhimento, bem como das Articulação com outras políticas públicas e demais órgãos
famílias acolhedoras, em relação a: cuidados diferenciados que do Sistema de Além da articulação com os serviços
crianças e adolescentes com deficiência, transtorno mental ou socioassistenciais, da saúde e da educação, é necessária a
outras necessidades específicas de saúde; amamentação, articulação com equipamentos comunitários, organizações
vacinação, crescimento e desenvolvimento de crianças e não-governamentais e serviços públicos responsáveis pela
adolescentes; saúde sexual e saúde reprodutiva e orientação execução de programas, projetos, serviços e ações nas áreas de
quanto aos direitos sexuais e direitos reprodutivos de cultura, esporte, lazer, geração de trabalho e renda, habitação,
adolescentes e prevenção do uso de álcool e outras drogas. transporte e capacitação profissional, garantindo o acesso de
Especial atenção deve ser dada no sentido de garantir à crianças e adolescentes acolhidos e de suas famílias.
criança e ao adolescente com deficiência ou necessidades Recomenda-se que, sempre que for viável, procure-se inserir
específicas de saúde, acolhidos no serviço, reintegrados à as crianças e adolescentes acolhidas em atividades localizadas
família de origem ou encaminhados à família substituta, o nas proximidades de sua comunidade de origem, de forma a
acesso a tratamentos, medicamentos, serviços especializados fortalecer sua inserção comunitária, o que contribuirá para o
e equipamentos de saúde, bem como o apoio necessário à processo de reintegração familiar ao evitar a futura
família para o atendimento a suas necessidades específicas. interrupção de suas atividades e dos vínculos de amizade
construídos nesses espaços.
Articulação com o Sistema Educacional Finalmente, destaca-se a importância da articulação e da
construção de fluxos locais entre os Serviços de Acolhimento,
A articulação dos serviços de acolhimento com o sistema o Serviço de Acolhimento em Família Acolhedora e os órgãos
educacional é fundamental, pois a escola constitui importante abaixo elencados, a fim de facilitar a comunicação, o
instrumento para assegurar o direito à convivência planejamento e o desenvolvimento de ações coordenadas. Tais
comunitária de crianças e adolescentes. Essa articulação pode órgãos desempenham funções fundamentais para a garantia
ser feita por meio da elaboração conjunta de protocolo de ação da excepcionalidade e provisoriedade do afastamento do
entre o órgão gestor da assistência social e da educação, convívio familiar, bem como da reparação de possíveis
garantindo a permanente comunicação entre os serviços, e o violações de direito vivenciadas. Abaixo são mencionados os
acesso das crianças, adolescentes acolhidos e seus familiares à principais aspectos que exigem uma articulação eficiente entre
rede de local de Educação. os serviços de acolhimento e os órgãos elencados:
Assim, os serviços de acolhimento devem manter canais de - Sistema de Justiça (Poder Judiciário, Ministério Público,
comunicação permanentes com as escolas onde estejam Defensoria Pública): apoio na implementação do Plano de
matriculadas as crianças e os adolescentes acolhidos, de modo Atendimento Individual e Familiar, por meio da aplicação de
a possibilitar o acompanhamento de seu desempenho escolar. outras medidas protetivas quando necessário;
Sempre que possível e recomendável, deve-se favorecer, acompanhamento do processo de reintegração familiar;
ainda, o envolvimento da família de origem ou extensa no investigação e responsabilização dos agressores nos casos de
acompanhamento escolar das crianças e adolescentes violência contra a criança ou adolescente; investigação de

Conhecimentos Específicos 111


Apostila Digital Licenciada para Alice Caroline Guarino dos Santos - alice.guarino@hotmail.com (Proibida a Revenda)
APOSTILAS OPÇÃO

paternidade e pensão alimentícia, quando for o caso; - Fortalecimento da autonomia da criança, do adolescente
destituição do Poder Familiar e cadastramento de crianças e e do jovem e preparação para desligamento do serviço;
adolescentes para adoção, nos casos em que não for possível a - Monitoramento e avaliação do atendimento (métodos de
reintegração familiar; preparação de todos os envolvidos para monitoramento e avaliação do serviço que incluam a
colocação em família substituta e deferimento da guarda, participação de funcionários, voluntários, famílias e atendidos
tutela ou adoção; fiscalização do atendimento prestado nos durante o acolhimento e após o desligamento);
serviços de acolhimento; acesso gratuito a serviços - Regras de convivência (direitos, deveres e sanções);
advocatícios para defesa de direitos, dentre outros;
- Conselho Tutelar: apoio na implementação do Plano de Atitude receptiva e acolhedora no momento da chegada
Atendimento Individual e Familiar; acompanhamento da da criança/adolescente e durante o período de
situação familiar de crianças e adolescentes acolhidos; acolhimento
aplicação de outras medidas protetivas quando necessário;
apoio na reintegração familiar; dentre outros; Muitas crianças e adolescentes desconhecem ou não
- Segurança Pública: investigação e responsabilização nos compreendem o motivo pelo qual foram afastadas do convívio
casos de violência contra a criança ou adolescente; localização familiar, o que pode levá-los a encarar a medida como uma
de familiares; acompanhamento da situação de pais ou espécie de punição e despertar sentimentos de insegurança,
responsáveis que estejam no sistema prisional, inclusive para rejeição, agressividade, revolta, abandono e outros. Diante
viabilizar a manutenção de contato destes com as crianças e disso, deve-se dar especial atenção ao momento de acolhida
adolescentes acolhidos; dentre outros. inicial da criança/adolescente, no qual deve ser dado
- Conselhos de Direitos: elaboração, aprovação e tratamento respeitoso e afetuoso, apresenta-lhes, inclusive, o
acompanhamento das ações do Plano de Nacional de espaço físico, as crianças e os adolescentes que lá se
Promoção, Proteção e Defesa de Direito de Crianças e encontram, seu educador/cuidador de referência - ou
Adolescentes à Convivência Familiar e Comunitária, no âmbito membros da família acolhedora - e seu espaço privado (cama,
nacional, estadual e municipal; elaboração e aprovação de armário, etc.). Tanto no acolhimento institucional quanto no
resoluções estaduais e municipais; inscrição de programas acolhimento familiar, é importante que as regras de convívio
governamentais e não-governamentais; registro de entidades no novo ambiente sejam explicadas para a criança ou
que executam serviços de acolhimento conforme Art. 90 do adolescente acolhido. Não é necessário que isso ocorra num
ECA; deliberação de políticas de atendimento para atender os primeiro momento do acolhimento, podendo estas regras ser
direitos humanos de crianças e adolescentes que se encontram gradativamente explicitadas. Tais normas têm como objetivo
atendidos nos serviços de acolhimento. organizar um ambiente seguro e previsível, porém com
flexibilidade e espaço para o lúdico, o coletivo e para a
Projeto Político-Pedagógico construção ou reconstrução de regras que incluam a
Para garantir a oferta de atendimento adequado às participação das crianças e adolescentes, de modo a facilitar
crianças e aos adolescentes, os serviços de acolhimento seu desenvolvimento. As famílias acolhedoras devem ser
deverão elaborar um Projeto Político-Pedagógico (PPP), que particularmente orientadas quanto ao acolhimento inicial da
deve orientar a proposta de funcionamento do serviço como criança/adolescente e sobre a importância de facilitar o
um todo, tanto no que se refere ao seu funcionamento interno, processo de conhecimento mútuo e integração da
quanto seu relacionamento com a rede local, as famílias e a criança/adolescente ao ambiente familiar e comunitário de
comunidade. Sua elaboração é uma tarefa que deve ser acolhimento.
realizada coletivamente, de modo a envolver toda a equipe do Nos serviços de acolhimento, para que a acolhida inicial
serviço, as crianças, adolescentes e suas famílias. Após a seja afetuosa e não represente uma re-vitimização de crianças
elaboração, o Projeto deve ser implantado, sendo avaliado e e adolescentes é importante que o serviço disponha de:
aprimorado a partir da prática do dia- a dia. - equipe técnica, educadores/cuidadores ou famílias
Sem a intenção de propor um modelo de PPP, destacam-se acolhedoras disponíveis e capacitados para a realização de
abaixo alguns tópicos a serem considerados para elaboração acolhida afetuosa e segura, capazes de compreender as
do mesmo: manifestações da criança ou adolescente no momento de
- Apresentação (histórico, atual composição da diretoria, chegada que envolve ruptura, incerteza, insegurança e
os principais momentos do serviço, as principais mudanças e transição (silêncio, choro ou agressividade, por exemplo);
melhorias realizadas, em especial se sua instalação for - espaço físico destinado à acolhida inicial daqueles que
anterior ao ECA,); estão chegando, adequado, inclusive, para a acomodação
- Valores do serviço de acolhimento (valores que daqueles que chegarem durante o período noturno;
permeiam o trabalho e ação de todos os que trabalham e - fluxos de comunicação eficiente e ágil dos órgãos
encontram-se acolhidos no serviço); encaminhadores (Conselho Tutelar, Justiça da Infância e da
- Justificativa (razão de ser do serviço de acolhimento Juventude ou outros, no caso de acolhida emergencial) com os
dentro do contexto social); Objetivos do Serviço de serviços de acolhimento. Estes fluxos são fundamentais para
Acolhimento; que os profissionais do serviço de acolhimento sejam
- Organização do serviço de acolhimento (espaço físico, comunicados previamente acerca de cada novo acolhimento e,
atividades, responsabilidades, etc.); em tempo hábil, possam preparar o ambiente e aqueles que já
- Organograma e quadro de pessoal (recursos humanos, se encontram acolhidos para a chegada do novo colega.
cargos, funções, turnos, funcionários, competências e No caso específico de crianças e adolescentes que estejam
habilidades necessárias para o exercício da função; modo de em situação de rua, a acolhida inicial deve fazer parte de uma
contratação; estratégias para capacitação e supervisão); estratégia de sensibilização para o acolhimento no serviço e
- Atividades psicossociais (com as crianças e adolescentes, construção de vínculo de confiança com o mesmo. Ao longo do
visando trabalhar questões pedagógicas complementares, processo de trabalho pela saída da rua, além dos aspectos aqui
autoestima, resiliência, autonomia; com as famílias de origem, mencionados, deve-se trabalhar também o significado do
visando a preservação e fortalecimento de vínculos e “estar e não-estar na rua”, expectativas, desejos e temores
reintegração familiar); quanto à retomada do convívio familiar e social, dentre outros
- Fluxo de atendimento e articulação com outros serviços aspectos.
que compõe o Sistema de Garantia de Direitos; Durante o período de acolhimento deve-se favorecer a
construção da vinculação de afeto e confiança com a equipe

Conhecimentos Específicos 112


Apostila Digital Licenciada para Alice Caroline Guarino dos Santos - alice.guarino@hotmail.com (Proibida a Revenda)
APOSTILAS OPÇÃO

técnica, educador/cuidador ou família acolhedora e colegas. É Crianças e adolescentes com deficiência, transtornos mentais
importante, ainda, que ao longo do acolhimento a criança e o e necessidades específicas de saúde devem ter registros e
adolescente tenham a possibilidade de dialogar com a equipe informações que favoreçam a prestação de cuidados
técnica e com educador/cuidador de referência (ou família adequados, inclusive, relativos à sua saúde. Devem ser
acolhedora) sobre suas impressões e sentimentos organizados registros semanais de cada criança e adolescente,
relacionados ao fato de estar afastado do convívio com a nos quais conste relato sintético sobre a rotina, progressos
família. Nessas conversas é importante que o interlocutor observados no desenvolvimento, vida escolar, socialização,
possibilite uma expressão livre da criança ou do adolescente, necessidades emergenciais, mudanças, encontro com
oportunizando-lhes espaço no qual possam falar sobre sua familiares, dados de saúde, etc.
história de vida, sentimentos, desejos, angústias e dúvidas Tais registros devem conter, ainda, informações sobre a
quanto às vivências pregressas, ao afastamento da família de família de origem, o trabalho desenvolvido com vistas à
origem e sua situação familiar. reintegração familiar (visitas, encaminhamentos,
Essas conversas não precisam ocorrer imediatamente acompanhamento em grupo, encontros da família com a
após a chegada da criança ou do adolescente, mas em criança ou adolescente, preparação para a reintegração, etc.) e
momentos de contato afetivo, nos quais a criança/adolescente o acompanhamento da família acolhedora, se for o caso. Esses
possa se expressar e ser ouvido de uma forma sensível e registros devem ser consultados apenas por profissionais
acolhedora. O interlocutor deve mostrar-se disponível e devidamente autorizados, devendo os serviços de acolhimento
manter postura acolhedora e respeitosa, sem culpabilizar ou ter uma política clara de confidencialidade desses dados,
julgar a família de origem ou a criança/adolescente. Nessas observada por todos os profissionais. A transmissão pelos
oportunidades, deve-se esclarecer também que o serviço de técnicos aos educadores/cuidadores ou família acolhedora de
acolhimento é organizado para a sua proteção e constitui um informações necessárias ao atendimento das crianças e
direito seu. adolescentes deve estar pautada em princípios éticos, os quais
também devem pautar a postura dos educadores/cuidadores.
Não-desmembramento de grupos de crianças/adolescentes Os registros devem ser acessíveis à equipe, caso a criança ou
com vínculos de parentesco e fortalecimento de sua vinculação adolescente seja novamente acolhida.
afetiva Sempre que possível, a fim de promover um sentido de
identidade própria, a criança e o adolescente - com o apoio de
Crianças e adolescentes com vínculos de parentesco um educador/cuidador, família acolhedora ou pessoa
(irmãos, primos, etc.), não devem ser separados ao serem previamente preparada - devem ter a oportunidade de
encaminhados para serviço de acolhimento, salvo se isso for organizar um livro de sua história de vida que reúna
contrário a seu desejo ou interesses ou se houver claro risco informações, fotografias e lembranças referentes a cada fase
de abuso, tendo em vista o melhor interesse da criança e do de sua vida, ao qual poderão ter acesso ao longo do ciclo vital.
adolescente. Para estas crianças e adolescentes que já se Este livro deve ser uma produção da própria criança ou
encontram afastados do convívio familiar, é particularmente adolescente, com fotos e outras criações de sua autoria. No
importante preservar e fortalecer seus vínculos fraternos e de momento do desligamento esse registro deve fazer parte dos
parentesco, o que pode contribuir para a formação de suas objetos pessoais que a criança ou adolescente levará consigo.
identidades, preservação da história de vida e referência Definição do papel e valorização dos
familiar. Por esse motivo, é importante que os serviços de educadores/cuidadores e da família acolhedora
acolhimento estejam organizados de modo a possibilitar
atendimento conjunto a grupos de irmãos ou de crianças e A postura dos educadores/cuidadores e das famílias
adolescentes com outros vínculos de parentesco, que podem acolhedoras e a qualidade da interação estabelecida com a
ter faixas etárias distintas e ambos os sexos. O PPP deve, criança e do adolescente representam importantes
portanto, contemplar estratégias para a preservação do referenciais para seu desenvolvimento. Para tanto, o PPP deve
convívio e o fortalecimento da vinculação afetiva de tais prever estratégias para sua seleção, capacitação e
crianças e adolescentes. acompanhamento/supervisão.
No caso de adolescentes acolhidos que possuam filhos, o Em função de sua importância, o educador/cuidador e a
atendimento deve fortalecer a vinculação afetiva, contribuir família acolhedora devem ter clareza quanto a seu papel:
para o desenvolvimento de habilidades para o cuidado, a vincular-se afetivamente às crianças/adolescentes atendidos e
construção de um projeto de vida e o desenvolvimento da contribuir para a construção de um ambiente familiar,
autonomia, de modo a garantir a proteção à(ao) adolescente e evitando, porém, “se apossar” da criança ou do adolescente e
a seu(s) filho(s). Esse cuidado pode contribuir para prevenir a competir ou desvalorizar a família de origem ou substituta. O
perpetuação de ciclos transgeracionais de ruptura de vínculos, serviço de acolhimento, não deve ter a pretensão de ocupar o
abandono, negligência ou violência, representando lugar da família da criança ou adolescente, mas contribuir para
importante recurso para garantir o direito à convivência o fortalecimento dos vínculos familiares, favorecendo o
familiar da(o) adolescente e da criança. Nesses casos é processo de reintegração familiar ou o encaminhamento para
importante que sejam viabilizadas condições para assegurar família substituta, quando for o caso.
às mães e aos pais adolescentes os direitos inerentes aos dois Para exercer sua função o educador/cuidador ou a família
momentos de vida: maternidade/paternidade e adolescência. acolhedora deve ter capacitação adequada para desempenhar
O PPP dos serviços de acolhimento deve também prever seu papel com autonomia e ser reconhecido como figura de
estratégias para o atendimento a estas situações. autoridade para a criança e o adolescente e, como tal, não ser
desautorizado pelos outros profissionais do serviço (técnicos,
Organização de registros sobre a história de vida e coordenadores), sobretudo na presença da criança e do
desenvolvimento de cada criança e adolescente adolescente. Além disso, devem de apoio e orientação
permanente por parte da equipe técnica do serviço, bem como
A equipe técnica do serviço de acolhimento deverá de espaço para trocas, nos quais possam compartilhar entre si
organizar prontuários individuais com registros sistemáticos experiências e angústias decorrentes da atuação, buscando a
que incluam: histórico de vida, motivo do acolhimento, data de construção coletiva de estratégias para o enfrentamento de
entrada e desligamento, documentação pessoal, informações desafios.
sobre o desenvolvimento (físico, psicológico e intelectual), Visando o constante aprimoramento do cuidado prestado,
condições de saúde, informações sobre a vida escolar, etc. devem ser realizados, periodicamente, estudos de caso com a

Conhecimentos Específicos 113


Apostila Digital Licenciada para Alice Caroline Guarino dos Santos - alice.guarino@hotmail.com (Proibida a Revenda)
APOSTILAS OPÇÃO

participação da equipe técnica e educadores/cuidadores, nos motivos graves, a criança e o adolescente devem ter sua
quais se possa refletir sobre o trabalho desenvolvido com cada origem – família, comunidade, cultura - tratada com respeito.
criança/adolescente e as dificuldades encontradas. Esses - O PPP deve prever no funcionamento do serviço e no
estudos devem propiciar também planejamentos de trabalho com as famílias de origem ações que promovam o
intervenções que tenham como objetivo a melhoria do fortalecimento dos vínculos das crianças e adolescentes com
atendimento no serviço e da relação entre educador/cuidador suas famílias:
e criança/adolescente, bem como a potencialização de - Preparação dos serviços de acolhimento institucional e da
aspectos favorecedores de seu processo de desenvolvimento, família acolhedora para aceitação e acolhimento dos
autoestima e autonomia. Tais aspectos devem ser igualmente familiares;
contemplados no acompanhamento às famílias acolhedoras. - Flexibilidade nos horários de visitas. Devem ser
É importante que a equipe técnica do serviço de acordados com a família de origem horários e periodicidade
acolhimento auxilie os educadores/ cuidadores ou as famílias das visitas à criança e ao adolescente. O esquema de visitação
acolhedoras na oferta de um cuidado individualizado para deve ser flexível e baseado na observação da realidade familiar
cada criança e adolescente, baseado na avaliação de suas e das dificuldades de acesso da família ao serviço (horários de
condições emocionais, história de vida, impacto da violência trabalho, distância, transporte, etc.). Podem ser organizadas,
ou do afastamento do convívio familiar, situação familiar, ainda, atividades que incluam a participação da família, como
vinculações significativas e interações estabelecidas. Estes almoço dominical com e para os familiares;
profissionais devem apoiar os educadores/cuidadores ou as - Participação da família na organização e comemoração de
famílias acolhedoras no exercício de seu papel, contribuindo aniversários e outras datas comemorativas, sempre que
para uma construção conjunta de estratégias que colaborem possível realizadas no domicílio da família;
para o desenvolvimento de um ambiente estruturante para a - Saídas das crianças e adolescentes para finais de semana
criança e o adolescente. com os familiares;
Finalmente, o educador/cuidador ou a família acolhedora - O serviço deve também apoiar as visitas da criança e do
devem participar e ter sua opinião ouvida pela equipe técnica adolescente à família;
do serviço na tomada de decisões sobre a vida da criança e do - Telefonemas para a família de origem e destas para as
adolescente, como, por exemplo, nas ocasiões em que se crianças e adolescentes que se encontrem acolhidos;
mostrar necessária a elaboração de relatório para a - Realizações de atividades recreativas e culturais com as
Autoridade Judiciária com recomendação de reintegração famílias, crianças, adolescentes e profissionais do serviço;
familiar ou adoção. Nesses casos, deve ser priorizada a - Realização de “Oficinas de talentos” nas quais as famílias
participação da família acolhedora ou daquele de origem, a criança ou adolescente difundam seus saberes e
educador/cuidador com o qual a criança/adolescente habilidades específicas (artesanato, brincadeiras, pequenos
mantenha vinculação afetiva mais significativa e que conheça consertos, aproveitamento de alimentos e materiais, etc.);
seus desejos e interesses. - Rodas de conversas para pais e filhos, abordando temas
levantados pela família, crianças e adolescentes;
Relação do Serviço com a família de origem
Participação dos familiares nas reuniões da escola do filho
Trabalhar com as famílias das crianças e dos adolescentes e consultas de saúde.
acolhidos em abrigos ou nas famílias acolhedoras implica Além de favorecer a aproximação entre as famílias,
compreender sua configuração, buscar suas competências e crianças e adolescentes, estas atividades podem favorecer,
entender sua inserção na comunidade. O trabalho com essas ainda, a aproximação das famílias entre si, de modo a construir
famílias precisa favorecer a superação das questões, por vezes uma rede de apoio mútuo, identificação e trocas de
bastante complexas, que contribuíram para o afastamento da experiência.
criança ou adolescente do convívio familiar. É importante No caso de acolhimento em Famílias Acolhedoras, é
compreender como as famílias estão vivenciando a situação de importante que estas possam contar com a orientação da
afastamento de seus filhos e potencializá-las para a retomada equipe técnica acerca do relacionamento com a família de
do convívio e exercício de seu papel de proteção e cuidados. origem, na perspectiva do fortalecimento de vínculos com a
As crenças e significados construídos pelas famílias criança e o adolescente. Nestes casos é igualmente importante
acolhedoras e pelos profissionais do serviço de acolhimento que o papel das famílias acolhedoras fique claro tanto para
acerca das famílias de origem e vice-versa influenciam a estas, quanto para as famílias de origem, de modo a evitar
relação entre estas e o serviço de acolhimento. Nesse sentido, rivalidades. Para favorecer uma interação positiva entre
é importante atentar para a forma como as famílias das família acolhedora e família de origem e a aproximação
crianças e adolescentes são significadas pelos profissionais crescente desta com a criança/adolescente, ao longo do
dos serviços de acolhimento e pelas famílias acolhedoras. Se acompanhamento podem ser promovidas pelo serviço
são consideradas “capazes” ou “incapazes”, “estruturadas” ou atividades que reúnam grupos com famílias de origem,
“desestruturadas”, “parte do problema” ou “agente acolhedoras e crianças e adolescentes acolhidos. Outras
transformador”. Por outro lado, os serviços de acolhimento ou atividades, espontâneas ou programadas, podem, ainda, ser
a família acolhedora também podem ser percebidos pela realizadas por iniciativas da família acolhedora, da família de
família como “aliados” ou “raptores de seus filhos”. origem ou da própria criança/adolescente acolhido. As
O educador/cuidador ou a família acolhedora e todos os famílias acolhedoras devem, ainda, ser orientadas pela equipe
profissionais do serviço de acolhimento devem receber técnica nas situações em que o contato da criança/adolescente
orientações para, nos momentos de visitas da família ao com a família de origem tiver impedimento judicial.
serviço e contato com a criança e o adolescente, atuarem, se O PPP deve prever também a elaboração do Plano de
necessário como mediadores dessa relação, proporcionando, Atendimento Individual e Familiar, de forma a assegurar o
ainda, momentos nos quais a família possa estar a sós com a acompanhamento da criança e/ou adolescente no período em
criança e adolescente. Os profissionais do serviço de que estiver acolhido e ações necessárias para que o retorno à
acolhimento, famílias acolhedoras e pessoas com as quais a família de origem, ou, na sua impossibilidade, a colocação em
criança ou o adolescente venham a ter contato em razão do família substituta, seja realizado da melhor maneira possível.
acolhimento não devem se referir de modo pejorativo à família
de origem. Ainda que o afastamento tenha ocorrido por

Conhecimentos Específicos 114


Apostila Digital Licenciada para Alice Caroline Guarino dos Santos - alice.guarino@hotmail.com (Proibida a Revenda)
APOSTILAS OPÇÃO

Preservação e fortalecimento da convivência comunitária a situação assim exigir. No convívio com a comunidade deve
ser oportunizado que crianças e adolescentes possam tanto
Os serviços de acolhimento devem estar localizados em receber seus colegas nas dependências do serviço como
áreas residenciais, sem distanciar-se excessivamente, do participar, por exemplo, de festas de aniversário de colegas da
ponto de vista geográfico e socioeconômico, do contexto de escola. Em síntese, no convívio comunitário devem ser
origem das crianças e adolescentes. Salvo determinação proporcionadas também experiências individualizadas.
judicial em contrário, quando necessário afastamento do O contato direto de pessoas da comunidade com crianças e
convívio familiar e encaminhamento para serviço de adolescentes em serviços de acolhimento, nas dependências
acolhimento, esforços devem ser empreendidos para manter a do mesmo, deverá ser precedido de preparação, visando
criança e o adolescente o mais próximo possível de seu assegurar que este contato será benéfico às crianças e aos
contexto de origem, a fim de facilitar o contato com a família e adolescentes. Nesse sentido, é importante destacar que visitas
o trabalho pela reintegração familiar. A proximidade com o esporádicas daqueles que não mantêm vínculo significativo e
contexto de origem tem como objetivo, ainda, preservar os frequentemente sequer retornam uma segunda vez ao serviço
vínculos comunitários já existentes e evitar que, além do de acolhimento, expõem as crianças e os adolescentes à
afastamento da família, o acolhimento implique o afastamento permanência de vínculos superficiais. Estes podem, inclusive,
da criança e do adolescente de seus colegas, vizinhos, escola, contribuir para que não aprendam a diferenciar conhecidos de
atividades realizadas na comunidade, etc. desconhecidos e tenham dificuldades para construir vínculos
Sempre que possível a criança e o adolescente devem estáveis e duradouros, essenciais para seu desenvolvimento.
frequentar a mesma escola em que estudavam antes do Por esse motivo, Programas de Apadrinhamento Afetivo ou
acolhimento, de modo a preservar vínculos pré-existentes – similares devem ser estabelecidos apenas quando dispuserem
salvo mudança necessária para sua proteção. As crianças e os de metodologia com previsão de cadastramento, seleção,
adolescentes devem ter, ainda, a possibilidade de continuar a preparação e acompanhamento de padrinhos e afilhados por
frequentar atividades que realizavam antes do acolhimento uma equipe interprofissional, em parceria com a Justiça da
(atividades esportivas, culturais, religiosas, entre outras). Infância e Juventude e Ministério Público.
O acolhimento não deve significar, ainda, privação do Nos Programas de Apadrinhamento Afetivo devem ser
direito à convivência comunitária. Nesse sentido, o serviço de incluídos, prioritariamente, crianças e adolescentes com
acolhimento, em parceria com a rede local e a comunidade, previsão de longa permanência no serviço de acolhimento,
deverá empreender esforços para favorecer a construção de com remotas perspectivas de retorno ao convívio familiar ou
vínculos significativos entre crianças, adolescentes e adoção, para os quais vínculos significativos com pessoas da
comunidade. Para evitar prejuízo ao convívio com a comunidade serão essenciais, sobretudo, no desligamento do
comunidade, espaços públicos e instituições, os serviços de serviço de acolhimento. Para estes casos, a construção de
acolhimento não deverão concentrar em suas dependências vínculos afetivos significativos na comunidade pode ser
equipamentos destinados à oferta de serviços de outra particularmente favorecedora, devendo ser estimulada,
natureza, como, por exemplo, de atendimento médico, observando os critérios anteriormente citados.
odontológico, educação infantil (“creche”), etc. Do mesmo
modo, não deverão concentrar espaços de lazer geralmente Fortalecimento da autonomia da criança, do adolescente e
não disponibilizados em unidades residenciais, como quadras do jovem
poliesportivas, etc.
A criança e o adolescente devem participar da vida diária Todas as decisões a respeito de crianças e adolescentes
da comunidade e ter a oportunidade de construir laços de atendidos em serviços de acolhimento devem garantir o
afetividade significativos com a mesma. Deve-se propiciar sua direito de ter sua opinião considerada. Pode meio de métodos
participação nas festividades e demais eventos da condizentes com o grau de desenvolvimento da
comunidade, além da utilização da rede socioassistencial, de criança/adolescente, deve-se assegurar o direito à escuta nas
educação, saúde, cultura, esporte e lazer disponíveis na rede diversas decisões que puderem repercutir sobre o
pública ou comunitária. No acesso a atividades culturais, desenvolvimento e a trajetória de vida da criança e do
esportivas e de lazer deve-se observar o interesse, as adolescente, envolvendo desde a identificação de seu interesse
habilidades e grau de desenvolvimento da criança e do pela participação em atividades na comunidade, até mudanças
adolescente. Sendo possível, deve-se propiciar que esse acesso relativas à sua situação familiar ou desligamento do serviço de
não seja realizado sempre de modo coletivo, ou seja, com acolhimento ou família acolhedora.
várias crianças e adolescentes do serviço frequentando as Além de participar da elaboração de projetos que versem
mesmas atividades nos mesmos horários, a fim de favorecer sobre sua trajetória futura, as crianças e os adolescentes
também a interação com outras crianças/adolescentes da devem ter acesso a informações sobre sua história de vida,
comunidade. situação familiar e motivos do acolhimento. A comunicação
Além de oportunizar o contato de crianças e adolescentes dessas informações deverá pautar-se na consideração do seu
acolhidos com crianças e adolescentes da comunidade, essas grau de desenvolvimento e na avaliação dos benefícios ou
medidas têm como objetivo propiciar o desenvolvimento da prejuízos que poderão resultar deste conhecimento. O acesso
autonomia e da socialização dos mesmos. O acesso aos a essas informações deverá respeitar o processo individual de
serviços na rede local tem como objetivo, ainda, inserir a apropriação da história de vida – devendo ser conduzido por
criança e o adolescente em atividades que possam continuar a profissionais orientados e preparados, com os quais a criança
frequentar após a reintegração familiar. e o adolescente mantenham vinculação afetiva significativa.
Esforços também devem ser empreendidos no sentido de Ações devem ser desenvolvidas visando o fortalecimento
evitar a estigmatização da criança e do adolescente durante e de habilidades, aptidões, capacidades e competências das
após o acolhimento. Nesse sentido, não devem ser utilizados crianças e adolescentes, de modo a fortalecer gradativamente
uniformes e, sempre que possível, o transporte em veículo com sua autonomia. Assim, a própria organização do ambiente de
identificação. Garantidas restrições essenciais à sua acolhimento deverá proporcionar o fortalecimento gradativo
segurança, crianças e adolescentes devem circular pela da autonomia, de modo condizente com o processo de
comunidade de modo semelhante àqueles de sua mesma faixa desenvolvimento e aquisição de habilidades nas diferentes
etária – caminhando, usando o transporte público ou bicicletas faixas etárias. Nessa direção deve-se considerar, por exemplo,
– contando com a companhia de educadores/cuidadores ou desde as condições para estimular a exploração do ambiente e
outros responsáveis quando o seu grau de desenvolvimento ou desenvolvimento psicomotor das crianças nos primeiros anos

Conhecimentos Específicos 115


Apostila Digital Licenciada para Alice Caroline Guarino dos Santos - alice.guarino@hotmail.com (Proibida a Revenda)
APOSTILAS OPÇÃO

de vida, até a aquisição da autonomia para o autocuidado, uma forma de transição entre o serviço de acolhimento para
preservação de objetos pessoais e cumprimento de crianças e adolescentes e a aquisição da autonomia.
responsabilidades decorrentes de atividades desenvolvidas na Sempre que possível, deverá também ser oportunizada a
comunidade – lazer, esporte, cultura, saúde, educação, participação das crianças, adolescentes e jovens em serviços
qualificação profissional, trabalho, etc. de acolhimento - ou adultos com histórico de atendimento
Os serviços de acolhimento devem propiciar a organização nesses serviços durante a infância, adolescência ou juventude
de espaços de escuta e construção de soluções coletivas com a - nas instâncias de formulação de políticas públicas, que
participação das crianças e adolescentes. Nesse sentido, constituem importantes espaços para estimular a participação
podem ser organizados, por exemplo, espaço para a realização social e o protagonismo, como as Conferências da Assistência
das chamadas “assembleias” nas quais crianças e adolescentes Social, do Direito de Criança e do Adolescente, etc.
sob cuidados em serviços de acolhimento possam
desempenhar um papel participativo, discutindo e Desligamento gradativo
construindo alternativas para a melhoria do serviço, para a
ampliação das estratégias para viabilizar o contato com a Tanto nos casos de reintegração à família de origem
família de origem, etc. No caso de crianças/adolescentes quanto nos de encaminhamento para família substituta o
acolhidos em Famílias Acolhedoras, às regras e à rotina diária serviço de acolhimento deve promover um processo de
da família é importante que sejam incorporados elementos desligamento gradativo, com o preparo da
significativos para a criança e o adolescente, propiciando a criança/adolescente, oportunizando-lhe a despedida
estes a oportunidade de sugerir alterações que promovam um necessária do ambiente, dos colegas, dos
senso de familiaridade com o novo ambiente. educadores/cuidadores e dos demais profissionais. Além da
As crianças e os adolescentes devem ter a oportunidade de criança e do adolescente, devem ser previamente preparados
realizar pequenas mudanças nos espaços privativos, fazer também os educadores/cuidadores e demais
escolhas e participar da organização do ambiente de crianças/adolescentes com as quais tenham mantido contato
acolhimento, segundo seu grau de desenvolvimento e em razão do acolhimento, assim com todos os membros da
capacidades. Assim, de modo gradativo e estritamente com famílias acolhedoras. Nesse sentido, podem ser viabilizados
função pedagógica, devem participar da organização da rotina rituais de despedida, atividades em grupo com as crianças e os
diária da instituição e assumir responsabilidade pelo cuidado adolescentes para tratar do desligamento, etc. É importante
com seus objetos pessoais, com seu autocuidado e que a família de origem (natural ou extensa) ou a família
cumprimento de compromissos (escola, atividades na adotiva sejam acompanhadas após a saída da
comunidade, trabalho, etc). É importante ressaltar que a criança/adolescente do serviço.
participação das crianças e adolescentes nas tarefas A criança e o adolescente em processo de desligamento
relacionadas aos cuidados domésticos devem ser estimuladas, devem ter a oportunidade de conversar, ainda, sobre suas
sem detrimento, todavia, de qualquer outra atividade lúdica ou expectativas e inseguranças quanto ao retorno ao convívio
educativa. familiar, bem como sobre o sentimento de saudade do
Na frequência a atividades realizadas na comunidade - ambiente de acolhimento, da família acolhedora, dos
escola, rede de saúde, atividades culturais, esportivas, de lazer profissionais do serviço e dos colegas. Os
e outras - considerar-se-á o estímulo gradativo à autonomia. educadores/cuidadores ou famílias acolhedoras,
Nesse sentido, não devem ser impostas restrições particularmente aqueles que mantêm vinculação afetiva mais
injustificáveis à liberdade e conduta, em comparação com significativa com a criança e o adolescente, devem ser
crianças e adolescentes da mesma idade e comunidade. Tais preparados e receber especial apoio nesse momento. Nesse
restrições devem ser condizentes com o grau de sentido, é importante que no serviço de acolhimento seja
desenvolvimento e capacidade da criança e do adolescente e viabilizado um espaço de acompanhamento contínuo, no qual
restritas apenas àquelas necessárias para viabilizar sua possam expressar, inclusive, a dor pela separação da criança
segurança e proteção. ou do adolescente.
Para ampliar a iniciativa, autonomia e o senso de Atenção especial deve ser dada à preparação nos casos de
responsabilidade é importante que as crianças e adolescentes desligamento de crianças/adolescentes que permaneceram no
acolhidos possam participar, ainda, de atividades rotineiras serviço de acolhimento por um longo período. Uma articulação
como ir à padaria ou ao supermercado, recebendo instruções permanente com a Justiça deve garantir um planejamento
sobre como lidar com o dinheiro. Geralmente os serviços de conjunto do processo de desligamento, de modo a prevenir
acolhimento pouco propiciam o contato de crianças e separações abruptas e permitir a avaliação do momento mais
adolescentes com esse tipo de conhecimento, aspecto que será adequado para a ocorrência do desligamento. Sempre que
fundamental para a construção de projetos de vida ligados ao possível e positivo para a criança e o adolescente, devem ser
trabalho e aquisição futura de autonomia financeira. viabilizados contatos posteriores ao desligamento com
Atenção especial deve ser dada aos adolescentes atendidos colegas, educadores/cuidadores, famílias acolhedoras e
em serviços de acolhimento, sobretudo àqueles cujas outros profissionais do serviço. Em casos de encaminhamento
possibilidades de reintegração à família de origem foram para adoção, é importante planejar o encontro da criança ou
esgotadas e têm reduzidas possibilidades de colocação em adolescente com a família substituta, com formas adequadas
família substituta, face às dificuldades de se encontrar famílias de aproximação e estratégias de apresentação que considerem
para os mesmos na realidade brasileira. Para estes casos, o PPP as características específicas do caso. Nesse trabalho, é
deve prever metodologia voltada à construção e fundamental a parceria efetiva entre a equipe interprofissional
fortalecimento de vínculos comunitários significativos, à da Justiça da Infância e da Juventude e do serviço de
ampliação do acesso à educação, à qualificação profissional e à acolhimento, que inclua também educadores/cuidadores e
progressiva autonomia do adolescente para o cuidado consigo demais profissionais, famílias acolhedoras e Grupos de Apoio
mesmo e o cumprimento de suas responsabilidades. O à Adoção, onde houver. Finalmente, é importante destacar que,
atendimento deve favorecer a construção de projetos de vida em conformidade com o ECA, decisão de quais crianças e
e o fortalecimento do protagonismo, desenvolvendo adolescentes serão colocados em quais famílias substitutas
gradativamente a capacidade do adolescente responsabilizar- compete, exclusivamente, à autoridade judiciária, ouvido o
se por suas ações e escolhas. Visando apoiar os adolescentes Ministério Público.
acolhidos após o alcance da maioridade, devem ser É importante que as crianças/adolescentes sejam
organizados serviços de acolhimento em Repúblicas66, como informados sobre sua real situação, sendo-lhes explicada a

Conhecimentos Específicos 116


Apostila Digital Licenciada para Alice Caroline Guarino dos Santos - alice.guarino@hotmail.com (Proibida a Revenda)
APOSTILAS OPÇÃO

possibilidade de adoção e oportunizando-lhes espaço para capacidade de escuta; estabilidade emocional, dentre outras.
expressarem o que pensam e sentem a respeito dessa Para os coordenadores, é ainda desejável capacidade de
possibilidade. Além disso, nos casos de encaminhamento para liderança e gestão de equipes.
adoção é preciso assegurar tempo suficiente para as crianças e No caso de educadores/cuidadores residentes, para
adolescentes se desligarem e se despedirem gradualmente das atendimento em casas-lares, também deverá ser verificado
pessoas com as quais construíram vinculações afetivas ao disponibilidade para residir, grau de independência pessoal e
longo do período de acolhimento. O desligamento não deve ser familiar que permita dedicação afetiva e profissional e
visto como um momento apenas, mas como resultado de um capacidade para administrar a rotina doméstica.
processo contínuo de desenvolvimento da autonomia e como No caso do coordenador, equipe técnica e
resultado de um investimento no acompanhamento da educador/cuidador, constituem habilidades e conhecimentos
situação de cada criança e adolescente. Particularmente no técnicos desejáveis:
que diz respeito aos adolescentes, a preparação para o Coordenador: gestão; trabalho em rede; crianças e
desligamento deve incluir o acesso a programas de adolescentes em situação de risco; conhecimentos sobre
qualificação profissional e inserção no mercado de trabalho, seleção e desenvolvimento de Recursos Humanos;
como aprendiz ou trabalhador – observadas as devidas conhecimento aprofundado do ECA, SUAS, Sistema de Justiça e
limitações e determinações da Lei nesse sentido, visando sua PNCFC.
preparação para uma vida autônoma. Sempre que possível, Equipe Técnica: violência e exclusão social, crianças e
ainda, o serviço manterá parceria com Repúblicas, utilizáveis adolescentes em situação de risco, separações, vinculações,
como uma forma de transição entre o abrigo e a aquisição de dependência química; desenvolvimento infanto-juvenil;
autonomia e independência. seleção e desenvolvimento de Recursos Humanos;
No caso de desligamento pela maioridade da adolescente atendimento a criança, adolescente e família; atendimento em
grávida ou com filhos pequenos, deve ser viabilizado, sempre grupo; trabalho em rede; acesso a serviços, programas e
que possível e necessário, seu encaminhamento para serviços benefícios; ECA; SUAS; Sistema de Justiça e PNCFC.
destinados ao atendimento a mulheres acompanhadas de seus Educador/Cuidador: cuidados com crianças e
filhos. adolescentes; noções sobre desenvolvimento infanto-juvenil;
noções sobre ECA; SUAS; Sistema de Justiça e PNCFC.
Gestão do Trabalho e Educação Permanente Quando se tratar de serviços de acolhimento
governamentais, particular atenção deverá ser dada à
Na história do nosso país, os serviços de acolhimentos elaboração de editais de concursos públicos para o
foram geridos e tinham o quadro de pessoal composto provimento dos cargos. Além da previsão de formação mínima
principalmente por pessoas voluntárias, religiosos ou leigos. exigida para a função72, da exigência de conteúdos específicos
Aos poucos essa realidade tem se modificado, mas ainda hoje para as provas de seleção73 (vide sugestões no item acima,
há a prevalência da concepção de que “basta o bom coração” “Habilidades e conhecimentos técnicos desejáveis”) e da
para se trabalhar nesses serviços. O reconhecimento de que previsão de provas de títulos que pontuem tanto titulação
todos os profissionais que atuam em serviços de acolhimento acadêmica em áreas relacionadas ao desempenho da função,
desempenham o papel de educador, impõe a necessidade de quanto experiência profissional no atendimento a crianças,
seleção, capacitação e acompanhamento de todos aqueles adolescentes e famílias. Recomenda-se que também sejam
responsáveis pelo cuidado direto e cotidiano das crianças e previstos nos editais, como etapas eliminatórias do concurso,
adolescentes acolhidos. a avaliação psicológica; a análise de vida pregressa e curso de
Para isso, em consonância com o que já está disposto na formação.
Norma Operacional Básica de Recursos Humanos do SUAS
(NOB-RH/SUAS), seguem algumas orientações para gestão do Capacitação
trabalho e educação permanente, que devem ser adequadas às
necessidades de cada município, considerando suas Investir na capacitação e acompanhamento dos
particularidades. educadores/cuidadores, assim como de toda a equipe que atua
nos serviços de acolhimento – incluindo coordenador, equipe
Seleção técnica e equipe de apoio - é indispensável para se alcançar
qualidade no atendimento, visto se tratar de uma tarefa
Um processo de seleção criterioso dos profissionais que complexa, que exige não apenas “espírito de solidariedade”,
atuarão nos Serviços de Acolhimento é essencial para a “afeto” e “boa vontade”, mas uma equipe com conhecimento
garantia de contratação de pessoal qualificado e com perfil técnico adequado. Para tanto, é importante que seja oferecida
adequado ao desenvolvimento de suas funções, possibilitando capacitação inicial de qualidade, e formação continuada a tais
a oferta de um serviço de qualidade aos usuários. Para tanto, profissionais, especialmente aqueles que têm contato direto
deve-se prever, minimamente, os seguintes passos: com as crianças e adolescentes e suas famílias.
Ampla divulgação, com informações claras sobre o A seguir, serão apresentados temas gerais a serem
serviço, o perfil dos usuários, as atribuições e exigências do abordados na capacitação dos profissionais que atuarão nos
cargo a ser ocupado, salário e carga horária, dentre outros; serviços de acolhimento, os quais devem ser adaptados às
Processo seletivo, com atenção à exigência da formação necessidades e demandas específicas de cada serviço e ser
mínima para cada função e experiência profissional; adequados de acordo com o nível de aprofundamento
Avaliação de documentação mínima a ser exigida: necessário ao desenvolvimento da função específica de cada
documentos pessoais, certidão negativa de antecedentes profissional.
criminais, atestado de saúde física e mental; Destaca-se que a realização dessa capacitação deve contar
Avaliação psicológica e social: análise da vida pregressa, com o apoio e parceria de profissionais que detenham
entrevista individual e atividade de grupo; conhecimento reconhecido no assunto, oriundos de possam
Constituem características desejáveis aos candidatos(a): falar da atividade e da experiência e, tenha retorno do
motivação para a função; aptidão para o cuidado com crianças facilitador sobre sua participação. Muitas vezes o próprio
e adolescentes; capacidade de lidar com frustração e candidato se depara com dificuldades em determinados
separação; habilidade para trabalhar em grupo; aspectos enfatizados na atividade de grupo que são
disponibilidade afetiva; empatia; capacidade de lidar com fundamentais para a função e declina do processo de seleção.
conflitos; criatividade; flexibilidade; tolerância; proatividade;

Conhecimentos Específicos 117


Apostila Digital Licenciada para Alice Caroline Guarino dos Santos - alice.guarino@hotmail.com (Proibida a Revenda)
APOSTILAS OPÇÃO

Capacitação Introdutória As demandas de um serviço de acolhimento exigem


resolutividade, rapidez e mobilidade, pois, com o passar do
A capacitação introdutória tem como objetivo inserir o tempo, pode-se gerar um automatismo de respostas dos
profissional no serviço e na equipe já existente, permitindo profissionais. Ou seja, há grande probabilidade de se cair na
ainda que acompanhe, como observador, os diferentes rotina, agindo sem refletir sobre o atendimento que está sendo
momentos da rotina e a possibilidade de posterior discussão realizado. Além disso, os casos atendidos nesses serviços
sobre as observações realizadas. O nível de experiência do acabam afetando de alguma forma emocionalmente os
profissional norteará o repasse e o conteúdo das informações profissionais. Por toda esta realidade, algumas atividades de
na etapa inicial de adaptação à rotina do serviço. acompanhamento são extremamente importantes no sentido
Abaixo são elencados temas relevantes a serem de melhorar o desempenho do profissional, a qualidade do
trabalhados em uma capacitação inicial: atendimento institucional e o bem-estar das crianças e dos
- Apresentação do serviço, suas especificidades e regras de adolescentes acolhidos. São elas:
funcionamento; Reuniões periódicas de equipe (discussão e fechamento
- Apresentação e discussão do Projeto Político-Pedagógico de casos; reavaliação de Planos de atendimento individual e
do serviço; familiar, construção de consensos, revisão e melhoria da
- Legislação pertinente (SUAS, PNCFC, ECA, dentre outros, metodologia)
além do presente documento); Formação continuada sobre temas recorrentes do
- SGD e rede de políticas públicas - com o intuito de que o cotidiano, assim como sobre temas já trabalhados na fase de
profissional compreenda as medidas protetivas, competências capacitação inicial, orientada pelas necessidades
e limites de atuação de cada órgão / entidade e articulação institucionais (promovida pela própria instituição e/ou cursos
entre as instâncias envolvidas; externos):
- Etapas do desenvolvimento da criança e do adolescente - Estudos de caso
(características, desafios, comportamentos típicos, - Supervisão institucional com profissional externo
fortalecimento da autonomia, desenvolvimento da - Encontros diários de 15-20 minutos entre os
sexualidade); brincadeiras e jogos adequados para cada faixa profissionais dos diferentes turnos para troca de informações
etária, exploração do ambiente, formas de lidar com conflitos, - Grupo de escuta mútua
colocação de limites, etc.; - Espaço de escuta individual
Comportamentos frequentemente observados entre - Avaliação, orientação e apoio periódicos pela equipe
crianças/adolescentes separados da família de origem, que técnica
sofreram abandono, violência, etc.; A seguir, serão apresentados parâmetros de
- Práticas educativas como ajudar a criança/adolescente a funcionamento para os serviços de acolhimento, no qual serão
conhecer e a lidar com sentimentos, fortalecer a autoestima e detalhadas, dentre outros aspectos, a formação da equipe
contribuir para a construção da identidade; mínima para cada serviço: i. Abrigos Institucionais; ii. Casas-
- Cuidados específicos com crianças e adolescentes com Lares; iii. Famílias Acolhedoras; iv. Repúblicas.
deficiência ou necessidades específicas de saúde (doença
infectocontagiosa ou imunodepressora; transtorno mental; Serviço de Acolhimento em Família Acolhedora
dependência química; etc);
- Novas configurações familiares e realidade das famílias Definição
em situação de vulnerabilidade e risco;
- Metodologia de trabalho com famílias; Serviço que organiza o acolhimento, em residências de
- Diversidade cultural e sexual, étnicas e religiosas; famílias acolhedoras cadastradas, de crianças e adolescentes
- Trabalho em rede. afastados do convívio familiar por meio de medida protetiva
(ECA, Art. 101), em função de abandono ou cujas famílias ou
Capacitação Prática responsáveis encontrem-se temporariamente
Antes de assumir suas funções, é importante que todos os impossibilitados de cumprir sua função de cuidado e proteção,
profissionais acompanhem como auxiliar a rotina da até que seja viabilizado o retorno ao convívio com a família de
instituição, para poder gradativamente se apropriar da função origem ou, na sua impossibilidade, encaminhamento para
que lhe é devida. adoção. Propicia o atendimento em ambiente familiar,
O educador/cuidador deverá passar por um período garantindo atenção individualizada e convivência
mínimo de 80 horas acompanhando, como auxiliar, os comunitária, permitindo a continuidade da socialização da
diferentes momentos da rotina institucional, sempre sob criança/adolescente.
supervisão de um educador/cuidador experiente e da equipe Embora ainda pouco difundida no País, esse serviço
técnica. encontra-se consolidado em outros países, especialmente nos
No caso de educador/cuidador residente, este período europeus e da América do Norte, além de contar com
deverá ser de, no mínimo, 30 dias de acompanhamento, como experiências exitosas no Brasil e América Latina. Tal serviço
auxiliar, dos diferentes momentos da rotina da casalar, sempre encontra-se contemplado, expressamente, na Política Nacional
sob supervisão de um(a) educador/cuidador residente de Assistência Social (2004), como um dos serviços de
experiente e da equipe técnica. proteção social especial de alta complexidade e no Plano
Nacional de Promoção, Proteção e Defesa de Direitos de
Formação continuada Crianças e Adolescentes à Convivência Familiar e Comunitária
(2006).
Para garantir qualidade ao Projeto Político-Pedagógico dos Do ponto de vista legal, assim como os serviços de
serviços de acolhimento, os horários para que os acolhimento institucional, o Serviços de Acolhimento em
educadores/cuidadores, equipe técnica e demais funcionários Família Acolhedora deve organizar-se segundo os princípios e
possam participar de cursos, reuniões de formação, diretrizes do Estatuto da Criança e do Adolescente,
seminários e leituras devem ter lugar no planejamento da especialmente no que se refere à excepcionalidade e à
organização e das escalas de trabalho. provisoriedade do acolhimento; ao investimento na
Depois da contratação, para adaptação à rotina reintegração à família de origem, nuclear ou extensa; à
institucional é fundamental o acompanhamento sistemático preservação da convivência e do vínculo afetivo entre grupos
do profissional, incrementado com capacitações continuadas.

Conhecimentos Específicos 118


Apostila Digital Licenciada para Alice Caroline Guarino dos Santos - alice.guarino@hotmail.com (Proibida a Revenda)
APOSTILAS OPÇÃO

de irmãos; a permanente articulação com a Justiça da Infância Um processo de seleção e capacitação criterioso é essencial
e da Juventude e a rede de serviços. para a obtenção de famílias acolhedoras com perfil adequado
Trata-se de um serviço de acolhimento provisório, até que ao desenvolvimento de suas funções, possibilitando a oferta de
seja viabilizada uma solução de caráter permanente para a um serviço de qualidade aos usuários. Para tanto, deve-se
criança ou adolescente – reintegração familiar ou, prever, minimamente, os seguintes passos:
excepcionalmente, adoção. É uma modalidade de acolhimento - Ampla Divulgação: com informações precisas sobre os
diferenciada, que não se enquadra no conceito de abrigo em objetivos e a operacionalização do Serviço, perfil dos usuários,
entidade, nem no de colocação em família substituta, no critérios mínimos para se tornar família acolhedora, dentre
sentido estrito, porém podendo ser entendido como regime de outros. A sensibilização de famílias para a participação do
colocação familiar preconizado no artigo 90 do Estatuto da serviço como famílias acolhedoras requer uma estratégia de
Criança e do Adolescente. divulgação permanente, realizada, em conjunto pelo executor
e pelo órgão do Governo Municipal competente, que privilegie
Público alvo a clareza dos objetivos dessa modalidade de atendimento, que
não deve ser confundida com adoção. O processo de
Geral divulgação também envolve a sensibilização de outros atores
do Sistema de Garantia de Direitos para que possam se
Crianças e adolescentes de 0 a 18 anos, que estão em estabelecer parcerias de trabalho.
medida protetiva. - Acolhida e avaliação inicial: Deve ser realizada por
equipe técnica multidisciplinar, qualificada e disponível para
Especificidades prestar os esclarecimentos necessários às famílias
interessadas, de modo individual e/ou em grupos de
Este serviço de acolhimento é particularmente adequado familiares. Este primeiro momento de interlocução possibilita,
ao atendimento de crianças e adolescentes cuja avaliação da inclusive, a identificação de possíveis motivações equivocadas
equipe técnica do programa e dos serviços da rede de – como interesse em adoção. Esse é o momento em que as
atendimento indique possibilidade de retorno à família de informações devem ser claras e objetivas, de modo a evitar
origem, ampliada ou extensa, salvo casos emergenciais, nos mal-entendidos e poupar tempo e envolvimento emocional da
quais inexistam alternativas de acolhimento e proteção. equipe e dos pretendentes ao acolhimento. Deve também ser
Para as crianças pequenas que vivenciam situações de verificado se as famílias atendem aos critérios mínimos
violação de direitos, o acolhimento familiar tem se mostrado exigidos para a função, inclusive em relação ao desejo,
uma forma de atendimento adequada a suas especificidades. disponibilidade e concordância de todos os membros do
núcleo familiar em acolher e participar dos encontros de
Número Máximo de Crianças e Adolescentes Acolhidos seleção, capacitação e acompanhamento.
- Avaliação Documental: Documentação mínima a ser
Cada família acolhedora deverá acolher uma exigida constitui em documentos pessoais (RG, CPF),
criança/adolescente por vez, exceto quando se tratar de grupo comprovante de residência, comprovante de rendimentos,
de irmãos, quando esse número poderá ser ampliado. Neste certidão negativa de antecedentes criminais, atestado de
último caso, em se tratando de grupo de mais de dois irmãos, saúde física e mental. Os documentos devem ser solicitados a
deverá haver uma avaliação técnica para verificar se o todos os membros maiores de idade do núcleo familiar. Em se
acolhimento em família acolhedora é a melhor alternativa para tratando de casal, é indicado que o termo de guarda seja
o caso, ou se seria mais adequado o acolhimento em outra expedido em nome de ambos. Os responsáveis pelo
modalidade de serviço, como Casa–lar, por exemplo. A acolhimento não devem ter qualquer problema em sua
decisão fica a critério da avaliação da equipe técnica do documentação. Quanto aos outros membros da família, a
programa, como também da disponibilidade da família em equipe técnica do programa deverá avaliar cada situação.
acolher. - Seleção: Após a avaliação inicial, as famílias inscritas
como potenciais acolhedoras deverão passar por um estudo
Aspectos jurídico-administrativos psicossocial, com o objetivo de identificar os aspectos
subjetivos que qualificam ou não a família para sua
As famílias acolhedoras são selecionadas, capacitadas e participação. Essa etapa deverá envolver entrevistas
acompanhadas pela equipe técnica do Serviço de Acolhimento individuais e coletivas, dinâmicas de grupo e visitas
para que possam acolher crianças ou adolescentes em medida domiciliares, sempre utilizando metodologias que privilegiem
de proteção aplicada por autoridade competente, a qual a coparticipação das famílias, em um processo que inclua a
encaminha a criança/adolescente para inclusão nesse serviço, reflexão e autoavaliação das mesmas. É essencial que todo o
competindo ao mesmo a indicação da família que esteja grupo familiar participe do processo de avaliação e seleção,
disponível e em condições para acolhê-lo. uma vez que todos os componentes do núcleo familiar devem
Dentro da sistemática jurídica, este tipo de acolhimento é estar de acordo e serem compatíveis com a proposta. Algumas
feito por meio de um termo de guarda provisória, solicitado características a serem observadas são:
pelo serviço de acolhimento e emitido pela autoridade - disponibilidade afetiva e emocional;
judiciária para a família acolhedora previamente cadastrada. A - padrão saudável das relações de apego e desapego;
guarda será deferida para a família acolhedora indicada pelo - relações familiares e comunitárias;
serviço, terá sempre o caráter provisório e sua manutenção - rotina familiar;
deve estar vinculada à permanência da família acolhedora no - não envolvimento de nenhum membro da família com
serviço. O termo de guarda deve ser expedido imediatamente dependência química; espaço e condições gerais da residência;
à aplicação da medida protetiva e início do acolhimento. motivação para a função; aptidão para o cuidado com crianças
e adolescentes; capacidade de lidar com separação;
Funcionamento do serviço de acolhimento em família - flexibilidade; tolerância; proatividade; capacidade de
acolhedora escuta; estabilidade emocional;
- capacidade de pedir ajuda e de colaborar com a equipe
Divulgação, Seleção, Preparação e Acompanhamento das técnica, dentre outras.
Famílias Acolhedoras Além da avaliação quanto à compatibilidade com a função
de acolhimento, o estudo psicossocial realizado pela equipe

Conhecimentos Específicos 119


Apostila Digital Licenciada para Alice Caroline Guarino dos Santos - alice.guarino@hotmail.com (Proibida a Revenda)
APOSTILAS OPÇÃO

técnica deverá indicar, também, o perfil de criança e/ou Com a criança/adolescente:


adolescente que cada família está habilitada a acolher. É Preparação da criança/adolescente para a entrada no
importante nesse processo, ouvir a opinião da família quanto programa, buscando-se estabelecer um vínculo de confiança,
a este aspecto, ainda que durante o processo de capacitação fornecendo explicação da situação e esclarecimentos quanto
essa avaliação possa modificar-se. ao acolhimento familiar. Essa ação deve ser partilhada com o
- Capacitação: as famílias selecionadas deverão participar órgão que encaminhou a criança ou adolescente.
de processo de capacitação. Tal processo deve ser Aproximação supervisionada entre a criança/adolescente
desenvolvido com metodologia participativa, de modo e a família acolhedora.
dinâmico, por meio de oficinas e seminários, que podem ser Escuta individual da criança/adolescente, com foco na
conduzidos pelos profissionais da equipe do Serviço e por adaptação à família acolhedora.
especialistas convidados (outros profissionais da rede, do Acompanhamento do desempenho escolar da criança e
Sistema de Justiça, etc). Também é bastante recomendável sua situação de saúde.
que, durante o processo de capacitação, sejam feitas Viabilização de encontro semanal entre a família de
apresentações de experiências de famílias acolhedoras que já origem e a criança e/ou adolescente, o qual deverá ser
vivenciaram o acolhimento, assim como de famílias de origem acompanhado pela equipe técnica.
cujas crianças/adolescentes foram acolhidos pelo serviço e já
retornaram ao lar, de modo a dar concretude à proposta. Com a família acolhedora:
Alguns temas relevantes a serem trabalhados em uma Preparação da família acolhedora para a recepção da
capacitação inicial são: criança/adolescente, inclusive informando a situação
- Operacionalização jurídico-administrativa do serviço e sociojurídica do caso e, quando possível, previsão inicial do
particularidades do mesmo; tempo de acolhimento.
- Direitos da criança e do adolescente; Aproximação supervisionada entre a criança/adolescente
- Novas configurações familiares e realidade das famílias e a família acolhedora.
em situação de vulnerabilidade social; Construção de um plano de acompanhamento da família
- Etapas do desenvolvimento da criança e do adolescente acolhedora, em conformidade com as necessidades do
(características, desafios, comportamentos típicos, acolhimento de cada criança/adolescente, respeitando-se as
fortalecimento da autonomia, desenvolvimento da características das famílias e do acolhido.
sexualidade); brincadeiras e jogos adequados para cada faixa Acompanhamento da família acolhedora, com entrevistas
etária, exploração do ambiente, formas de lidar com conflitos, e visitas domiciliares com foco na adaptação e
colocação de limites, etc.); desenvolvimento do acolhimento, com frequência mínima
- Comportamentos frequentemente observados entre quinzenal ou de acordo com a avaliação do caso.
crianças/ adolescentes separados da família de origem, que Construção de espaço para troca de experiências entre
sofreram abandono, violência, etc; famílias acolhedoras (Ex.: grupos de apoio, de escuta mútua).
- Práticas educativas; como ajudar a criança/adolescente a
conhecer e a lidar com sentimentos, fortalecer a autoestima e Com a família de origem:
contribuir para a construção da identidade; Contato inicial com a família de origem (salvo em
- Políticas públicas, direitos humanos e de cidadania; situações de restrição judicial) para esclarecimento do que é o
- Papel da família acolhedora, da equipe técnica do acolhimento familiar, seus termos e regras, assim como para
programa e da família de origem. convidá-la a participar do processo de adaptação da
Cadastramento: As famílias que forem consideradas aptas criança/adolescente na família acolhedora, fornecendo
a serem acolhedoras deverão formalizar sua inscrição no informações sobre seus hábitos e costumes. Se possível,
Serviço, com o preenchimento da ficha de cadastro, onde possibilitar o encontro da família de origem com seu filho(a).
constam os documentos necessários (já citados no item Acompanhamento da família de origem, com entrevistas e
Avaliação Documental), informações sobre toda a família e visitas domiciliares periódicas, articuladas com o
indicação quanto ao perfil de criança/ adolescente que se julga planejamento realizado para superação das vulnerabilidades
capaz de acolher. A documentação necessária deverá ser da família.
encaminhada pela coordenação do Serviço à Justiça da Infância Construção de espaço para troca de experiências entre
e Juventude, para que possa ser emitido, com presteza, o termo famílias de origem (Ex.: grupos de apoio, de escuta mútua).
de guarda e responsabilidade quando ocorrer o acolhimento
de uma criança/adolescente pela família cadastrada. Outras atribuições da equipe técnica do programa:
Acompanhamento: Os serviços da rede de proteção à
infância e juventude, especialmente o requerente do ingresso Construir com a participação da família de origem e
da criança no programa (Justiça da Infância e Juventude, serviços da rede de proteção um plano de acompanhamento
Conselho Tutelar, Equipe de Supervisão e Apoio aos Serviços da família de origem, nuclear ou extensa, que objetive a
de Acolhimento, etc.), devem iniciar discussão do caso com a superação dos motivos que levaram à necessidade do
equipe técnica, antes que a modalidade acolhimento familiar afastamento da criança/adolescente e conseqüente
seja a opção de proteção decidida. Objetiva-se com isso traçar reintegração familiar.
um trabalho em rede e de continuidade ao atendimento à Providenciar encaminhamentos jurídico-administrativos
criança e sua família. e junto à rede de educação, saúde, dentre outros que se fizerem
necessários.
Preparação para o Acolhimento e Acompanhamento Possibilitar situações de escuta individual, ao longo de
todo o tempo de acolhimento, de qualquer dos envolvidos
A partir do momento em que uma criança/adolescente for (família de origem, família acolhedora e acolhido).
encaminhada para o serviço, a equipe técnica deve iniciar a
preparação e acompanhamento psicossocial da Atribuições das Famílias Acolhedoras:
criança/adolescente, da família acolhedora, da família de Preservar o vínculo e convivência entre irmãos e parentes
origem e da rede social de apoio. Isso poderá ocorrer por meio (primos, sobrinhos) quando o acolhimento for realizado por
de ações específicas tais como: famílias diferentes.
Responsabilizar-se pelas atividades cotidianas e
rotineiras dos acolhidos (levar à escola, atendimentos de

Conhecimentos Específicos 120


Apostila Digital Licenciada para Alice Caroline Guarino dos Santos - alice.guarino@hotmail.com (Proibida a Revenda)
APOSTILAS OPÇÃO

saúde etc), cabendo à equipe técnica auxiliar as famílias Recursos humanos


acolhedoras na obtenção destes atendimentos,
preferencialmente na rede pública. Em se tratando de serviços de acolhimento desenvolvidos
Comunicação à equipe do serviço todas as situações de por organizações não governamentais, a equipe técnica deverá
enfrentamento de dificuldades que observem durante o pertencer ao quadro de pessoal da entidade ou ser cedida pelo
acolhimento, seja sobre a criança, seja sobre a própria família órgão gestor da Assistência Social ou por outro órgão público
acolhedora e a família de origem. ou privado, exclusivamente para esse fim. Em ambos os casos,
deverá ser respeitado o número mínimo de profissionais
Desligamento da criança/adolescente necessários, a carga horária mínima e o cumprimento das
atribuições abaixo elencadas.
O desligamento do programa ocorrerá quando for avaliado
pela equipe de profissionais do serviço, em diálogo com a República
Justiça da Infância e Juventude, com o Ministério Público,
Conselho Tutelar e rede envolvida - a possibilidade de retorno Definição
familiar (à família de origem, nuclear ou extensa); a Serviço de acolhimento que oferece apoio e moradia
necessidade de acolhimento em outro espaço de proteção, ou subsidiada a grupos de jovens em situação de vulnerabilidade
o encaminhamento para adoção. A esta avaliação deve suceder e risco pessoal e social; com vínculos familiares rompidos ou
a preparação e o apoio específico por parte da equipe técnica, extremamente fragilizados; em processo de desligamento de
com ações: instituições de acolhimento, que não tenham possibilidade de
retorno à família de origem ou de colocação em família
Com a criança/adolescente: substituta e que não possuam meios para autossustentação.
Escuta individual e apoio emocional à criança/adolescente, Com a estrutura de uma residência privada, deve receber
com foco no retorno à família de origem e separação da família supervisão técnica e localizar-se em áreas residenciais da
acolhedora. cidade, seguindo o padrão socioeconômico da comunidade
onde estiverem inseridas, sem distanciar-se excessivamente,
Com a família de origem: do ponto de vista socioeconômico, da comunidade de origem
Intensificar e ampliar, de forma progressiva, os encontros dos usuários.
entre a criança/adolescente e sua família - que gradativamente A república oferece atendimento durante o processo de
deverão deixar de ser acompanhados pela equipe, a construção de autonomia pessoal e possibilita o
permanência com a família nos finais de semana e, por fim, o desenvolvimento de auto-gestão, autossustentação e
retorno definitivo. independência. Possui tempo de permanência limitado,
Dar continuidade ao acompanhamento à família de origem podendo ser reavaliado e prorrogado em função do projeto
após a reintegração da criança/adolescente, por um período individual formulado em conjunto com o profissional de
mínimo de seis meses, de forma a lhe dar suporte para o referência.
cumprimento de suas funções de cuidado e proteção,
buscando sua autonomia e visando evitar a reincidência da Público alvo
necessidade de acolhimento. Conforme a estrutura local, tal
acompanhamento poderá ser feito pela equipe técnica do Geral
serviço de famílias acolhedoras que acompanhou o
acolhimento ou por outro serviço socioassistencial (CRAS, Jovens entre 18 e 21 anos em situação de vulnerabilidade
CREAS) em articulação com a rede local. e risco pessoal e social, com vínculos familiares rompidos ou
extremamente fragilizados e que não possuam meios para
Com a família acolhedora: autossustentação. Tal serviço é particularmente indicado para
Orientar a família acolhedora para intensificar a o acolhimento de jovens em processo de desligamento de
preparação da criança/adolescente para o retorno à família de serviços de acolhimento para crianças e adolescentes por
origem. terem completado a maioridade, porém que ainda não tenham
Realizar encontros com a família acolhedora (entrevistas conquistado a autonomia, podendo também destinar-se a
individuais e com o grupo familiar), com foco na saída da outros jovens que necessitem do serviço.
criança/adolescente e na experiência de separação,
oferecendo apoio psicossocial após a saída do(a) acolhido(a) Especificidades
manutenção das atividades em grupo com outras famílias
acolhedoras e do contato regular com a equipe técnica. As repúblicas devem ser organizadas em unidades
Intermediar e orientar a família acolhedora com relação à femininas e unidades masculinas, garantindo-se, na rede, o
manutenção de vínculos com a criança/adolescente e sua atendimento a ambos os sexos, conforme demanda local,
família após a reintegração familiar, o que também amplia a devendo ser dada a devida atenção à perspectiva de gênero no
proteção da criança/adolescente acolhido. Entretanto, deve planejamento políticopedagógico do serviço, inclusive no que
ser respeitado o desejo de todos os envolvidos, além de serem tange aos direitos sexuais e reprodutivos e à proteção à
consideradas as características de cada caso, avaliando-se a maternidade.
pertinência ou não da manutenção desde contato. Especial atenção deve ser dada à escolha dos componentes
No caso em que forem esgotadas todas as possibilidades de de cada república, a qual deverá ser feita por equipe técnica
reintegração familiar e a criança/adolescente for capacitada, devendo ser levados em consideração aspectos
encaminhada para adoção, a família acolhedora deverá como perfil, demandas específicas e grau de autonomia de
contribuir para essa transição e, em conjunto com a equipe cada usuário, bem como o grau de afinidade entre os mesmos.
técnica do serviço, preparar esta criança para a colocação em Quando um novo jovem vier a integrar uma república, a equipe
uma família definitiva. técnica do serviço deverá prepará-lo e aos demais jovens da
O desligamento do programa deve ocorrer mediante república, de modo a facilitar sua inserção e integração ao
conhecimento e autorização da Justiça da Infância e Juventude, ambiente.
que deve estar devidamente informado das ações do serviço e Sempre que possível e recomendável, os jovens deverão
atuar em conjunto com estas. ter participação ativa na escolha dos colegas de república, de

Conhecimentos Específicos 121


Apostila Digital Licenciada para Alice Caroline Guarino dos Santos - alice.guarino@hotmail.com (Proibida a Revenda)
APOSTILAS OPÇÃO

modo a que, na composição dos grupos, sejam respeitadas (como estágios, programas de adolescente aprendiz, etc.),
afinidades e vínculos previamente construídos. sempre se respeitando seus interesses e habilidades.
Deverá ser disponibilizado na rede número suficiente de
repúblicas localizadas em edificações que respeitem as Aspectos físicos
normas de acessibilidade103, de maneira a possibilitar o
atendimento integrado a jovens com deficiência. Localização

Número Máximo de Usuários por Equipamento Áreas residenciais, sem distanciar-se excessivamente, do
ponto de vista socioeconômico, da realidade de origem dos
6 (seis) jovens usuários.

4.4.3 Características Fachada e aspectos gerais da construção

Os custos da locação do imóvel - no caso de imóvel alugado O serviço deve funcionar em construção destinada ao uso
- e tarifas podem ser subsidiados e gradativamente assumidos residencial, seguindo o padrão arquitetônico das demais
pelos jovens. As demais despesas podem ser cotizadas entre os residências da comunidade na qual estiver inserida. Deve-se
moradores, com subsídio quando necessário. evitar a instalação de placas indicativas da natureza
O grupo deve contar com supervisão técnico-profissional institucional do equipamento.
para a gestão coletiva da moradia (regras de convívio,
atividades domésticas cotidianas, gerenciamento de despesas, Recursos humanos
etc.), orientação e encaminhamento para outros serviços,
programas ou benefícios da rede socioassistencial e das Em se tratando de serviços de acolhimento desenvolvidos
demais políticas públicas, em especial programas de por organizações não governamentais, a equipe técnica deverá
profissionalização, inserção no mercado de trabalho, pertencer ao quadro de pessoal da entidade ou ser cedida pelo
habitação e inclusão produtiva. O apoio técnico também é órgão gestor da Assistência Social ou por outro órgão público
essencial na organização de espaços de escuta e construção de ou privado, exclusivamente para esse fim. Em ambos os casos,
soluções coletivas por parte dos(as) jovens para as questões deverá ser respeitado o número mínimo de profissionais
que lhes são próprias, na construção de projetos de vida, no necessários, a carga horária mínima e o cumprimento das
incentivo ao estabelecimento de vínculos comunitários fortes atribuições elencadas neste documento.
e na participação nas instâncias de controle social e espaços de
participação social. Regionalização do Atendimento nos Serviços de
Caso desejem, os(as) jovens devem ter acesso a todas as Acolhimento
informações que lhes digam respeito que estiverem
disponíveis nas instituições que lhes prestaram atendimento Excepcionalmente pode ser necessária e justificável a
durante a infância e adolescência (Ex.: prontuários e regionalização do atendimento nos serviços de acolhimento de
documentos contendo informações sobre sua história de vida, crianças e adolescentes afastados do convívio familiar ou de
possíveis familiares, situação familiar e motivos do Repúblicas para jovens. Esta situação pode ser observada
abrigamento). O acesso a essas informações deverá respeitar tanto no caso de municípios de pequeno porte - cuja demanda
o processo individual de apropriação da história de vida, e condições de gestão dificultem a implementação de serviços
devendo ser conduzido por profissionais orientados e locais - quanto no caso do atendimento a crianças e
preparados. adolescentes ameaçados de morte, situação na qual o
O processo de transição do jovem do serviço de atendimento em serviço de acolhimento localizado próximo ao
acolhimento para crianças e adolescentes para o serviço de contexto familiar e comunitário de origem pode representar
acolhimento em república deve desenvolver-se de modo risco à segurança da criança ou adolescente ameaçado.
gradativo, com a participação ativa do mesmo no Serão abordados neste item estratégias e orientações para
planejamento das fases subsequentes. Transições dessa a organização de serviços de acolhimento nas situações acima
natureza devem sempre ser planejadas o mais cedo possível. mencionadas.
Atenção especial deve ser dada a adolescentes atendidos
em serviços de acolhimento, sobretudo àqueles cujas Serviços de acolhimento para municípios de pequeno porte
possibilidades de reintegração à família de origem foram com compartilhamento de equipe
esgotadas e têm reduzidas possibilidades de colocação em
família substituta. O atendimento, nesses casos, deve No caso de municípios de pequeno porte, as alternativas
perseverar no apoio ao fortalecimento dos vínculos para a implementação especificamente de serviços de
comunitários, na qualificação profissional e na construção do acolhimento para crianças e adolescentes devem ser
projeto de vida, bem como estar fundamentado em consideradas na seguinte ordem:
metodologia participativa que favoreça o exercício de seu
protagonismo. Implementação de serviço local
Ações devem ser desenvolvidas visando o fortalecimento
de habilidades, aptidões, capacidades e competências dos O município deve avaliar inicialmente as possibilidades de
adolescentes, que promovam gradativamente sua autonomia, implementar, na localidade, um Serviço de Acolhimento em
de forma a que, preferencialmente, já estejam exercendo Família Acolhedora ou uma Casa-lar, em conformidade com os
alguma atividade remunerada quando da sua transferência parâmetros contidos nos Parâmetros de Funcionamento deste
para uma república. documento. Nestes casos, pode-se, inclusive, avaliar a
Para tanto, deve-se viabilizar o acesso a: i. Programas, necessidade de redução da carga horária mínima indicada à
projetos e serviços nos quais possam desenvolver atividades jornada diária de trabalho da equipe técnica108 e,
culturais, artísticas e esportivas que propiciem a vivência de excepcionalmente, a redução da equipe técnica para um
experiências positivas e favorecedoras de sua autoestima; ii. profissional, desde que este possa contar com apoio regular de
Programas de aceleração da aprendizagem, para os casos de outro profissional da rede local, inclusive com papel e
grande distorção série – idade; e iii. Cursos profissionalizantes atribuições definidas em relação ao serviço.
e programas de inserção gradativa no mercado de trabalho

Conhecimentos Específicos 122


Apostila Digital Licenciada para Alice Caroline Guarino dos Santos - alice.guarino@hotmail.com (Proibida a Revenda)
APOSTILAS OPÇÃO

Implementação de serviço com compartilhamento de equipe Serviço de Família Acolhedora: Neste caso, o serviço deve
ter famílias cadastradas em todos os municípios atendidos, de
Esgotadas as possibilidades de implementação de serviço modo a viabilizar o acolhimento da criança ou adolescente no
de acolhimento conforme recomendação do item acima, deve- seu próprio município de origem. A equipe compartilhada
se considerar, nesta ordem, a implementação de Serviço de entre os municípios – coordenador e equipe técnica - deverá
Acolhimento em Família Acolhedora ou Casa-lar, com atender a todas as atribuições que lhe são próprias,
compartilhamento de equipe (coordenador e equipe técnica). destacando-se a articulação intersetorial necessária à
No caso de municípios de pequeno porte que, apesar da implementação do serviço, a divulgação do mesmo, a
necessidade, apresentem dificuldades para implementar e mobilização, seleção, capacitação e acompanhamento das
manter serviços de acolhimento para crianças e adolescentes - famílias acolhedoras, a elaboração dos Planos de Atendimento
em virtude da pequena demanda e das condições de gestão - Individual e Familiar e dos relatórios a serem enviados à
pode-se recorrer ao compartilhamento dos seguintes autoridade judicial, o acompanhamento das crianças e
profissionais: coordenador e equipe técnica. O adolescentes acolhidos e de suas famílias e os
compartilhamento dessa equipe constitui estratégia para encaminhamentos necessários para possibilitar a reintegração
assegurar o atendimento da criança e do adolescente próximo familiar ou, na sua impossibilidade, o encaminhamento para
à sua comunidade de origem, de modo a evitar seu família substituta.
acolhimento em serviços localizados nas capitais dos estados Casa-Lar: Neste caso, pode-se compartilhar a equipe
ou em municípios muito distantes de seu contexto de moradia compartilhada entre os municípios – coordenador e equipe
e de sua família. Em hipótese alguma a utilização de equipes técnica - a qual deverá atender a todas as atribuições que lhe
compartilhadas poderá implicar a precarização do serviço são próprias, destacando-se a articulação intersetorial, a
oferecido, o qual deverá atender aos demais parâmetros seleção, a capacitação e acompanhamento dos
contidos neste documento. educadores/cuidadores residentes e seus auxiliares, o
Os serviços de acolhimento com compartilhamento de acompanhamento das crianças e adolescentes acolhidos e de
equipe podem ser implementados sob gestão estadual ou suas famílias, os encaminhamentos necessários para viabilizar
como consórcios entre municípios, desde que disponham de a reintegração familiar ou, na sua impossibilidade, o
coordenação e equipe técnica suficiente para o atendimento a encaminhamento para família substituta. Ressalte-se que cada
mais de um município. Destaca-se que, a despeito da um dos municípios atendidos deverá dispor de uma casa-lar
possibilidade de compartilhamento da equipe entre implantada em seu território, de modo a garantir o
municípios, o ambiente de acolhimento (casa-lar ou residência acolhimento das crianças e adolescentes em seu próprio
da família acolhedora) deverá estar localizado em cada um município. O número de educadores/cuidadores por criança e
deles. Exceção a esta exigência poderá ser feita, apenas, no adolescente, assim como os demais aspectos do serviço,
caso de Repúblicas que, por atenderem jovens com mais de 18 deverão obedecer aos parâmetros deste documento que trata
anos e, consequentemente, com maior autonomia de dessa modalidade de serviço. No caso do município possuir
locomoção, poderão estar localizadas no município-sede e uma demanda por acolhimento muito pequena, permite-se a
atender jovens dos municípios circunvizinhos. implementação de uma casalar com reduzida capacidade de
Cabe ressaltar que a implementação de serviços de atendimento – de até 6 crianças e adolescentes.
acolhimento com compartilhamento de equipe, deve ser Excepcionalmente neste caso, o serviço poderá contar com a
acompanhada do investimento efetivo, em cada um dos presença de apenas um educador/cuidador residente,
municípios, em estratégias preventivas ao afastamento do dispensando-se a presença permanente do auxiliar de
convívio familiar, fortalecimento dos serviços necessários educador/cuidador.
para o acompanhamento das famílias de origem e para a Repúblicas: Neste caso, a equipe compartilhada entre os
reintegração familiar ou, na sua impossibilidade, municípios – coordenador e equipe técnica - deverá atender a
encaminhamento para família substituta. todas as atribuições que lhe são próprias, destacando-se a
Destaca-se que, nos casos de compartilhamento de equipe, articulação intersetorial, o preparo dos(as) jovens para a
tanto a coordenação quanto a equipe técnica deverá ser inserção no serviço e seu posterior acompanhamento
destinada exclusivamente para esta finalidade, devendo-se psicossocial, o apoio à gestão coletiva da moradia (regras de
atender aos parâmetros deste documento no que diz respeito convívio, atividades domésticas cotidianas, gerenciamento de
ao quantitativo de profissionais em relação ao número de despesas, etc.) e o encaminhamento dos(as) jovens para os
crianças/adolescentes ou jovens atendidos, perfil, carga serviços da rede. De acordo com a avaliação técnica que indicar
horária mínima recomendada e ao cumprimento das a melhor opção, levando-se em consideração a realidade,
atribuições elencadas neste documento. demandas e recursos locais, as unidades de República poderão
A estratégia de compartilhamento de equipe exigirá a estar localizadas em cada um dos municípios que
previsão de veículos e combustível suficientes, de modo a compartilharem a equipe, ou no município-sede do serviço.
permitir o deslocamento da equipe técnica do município-sede Neste último caso, deverão ser assegurados meios necessários
para os demais os municípios atendidos, com periodicidade à locomoção do(a) jovem ao seu município de origem,
mínima semanal, de modo a possibilitar o desenvolvimento de periodicamente, de modo a manter laços de convivência ali
suas ações no que diz respeito ao apoio, capacitação e estabelecidos.
acompanhamento dos educadores/cuidadores e/ou famílias
acolhedoras; acompanhamento psicossocial das crianças e Implementação de casa-lar regionalizada
adolescentes atendidos e de suas famílias de origem;
articulação com a rede se serviços e o SGD; e o exercício das Deve-se, excepcionalmente, recorrer a esta alternativa
demais atribuições que lhe sejam próprias. Também devem quando nenhuma das anteriores for de possível
ser previstos meios suficientes ao deslocamento eventual dos implementação. Neste caso, devem ser observados,
educadores/cuidadores e/ou famílias acolhedoras, para obrigatoriamente, os critérios estabelecidos para o
eventos de capacitação e formação continuada, que compartilhamento de equipe (coordenador e equipe técnica),
porventura sejam desenvolvidos fora do seu município. além de asseguradas as condições para o deslocamento
De acordo com os aspectos acima mencionados, devem ser semanal, tanto das famílias para o município onde se localizar
observadas, ainda, as seguintes orientações na implementação a Casalar, quanto das crianças e adolescentes para o município
de serviços de acolhimento com equipes compartilhadas de residência da família de origem, de modo a favorecer o
(coordenador e equipe técnica): processo de reintegração familiar.

Conhecimentos Específicos 123


Apostila Digital Licenciada para Alice Caroline Guarino dos Santos - alice.guarino@hotmail.com (Proibida a Revenda)
APOSTILAS OPÇÃO

Serviços de acolhimento para crianças e adolescentes


ameaçados de morte 17. Adolescente em Conflito
com a Lei;
Nos casos de crianças e adolescentes ameaçados de morte,
sua manutenção no contexto familiar e comunitário de origem
pode representar sério risco a sua segurança. Trata-se de uma Violência no meio social24
situação particularmente delicada, na qual pode ser necessário
o encaminhamento para serviço de acolhimento em localidade Gomide (2000), afirma que a Teoria da Aprendizagem
distinta do município de residência habitual. Nestes casos, é Social de Bandura auxilia na compreensão do efeito de filmes
preciso considerar que a proximidade do serviço de violentos para o desenvolvimento de comportamentos
acolhimento com a comunidade de origem, a manutenção das agressivos. Os espectadores aprendem a se comportar
atividades rotineiramente desenvolvidas e o convívio com sua agressivamente a partir de observações de um modelo que é
rede social local – parâmetros que devem orientar os serviços reforçado pelo seu comportamento agressivo. A maioria dos
de acolhimento em geral – não são aconselháveis, por heróis de filmes violentos justifica seu comportamento
colocarem em risco a segurança da criança ou adolescente violento por estar em defesa de valores ligados à família,
ameaçado podendo também representar risco para as demais governo, território etc. Gomide (2000) menciona dois
crianças e adolescentes atendidos no mesmo serviço. trabalhos de Bandura, Ross e Ross afirmando que essa
Dessa forma, sugere-se, para a operacionalização destes justificativa permite que, após assistir muitas horas de
serviços, as seguintes estratégias: programas violentos, os espectadores deixem de considerar
Em estados / regiões onde houver número significativo de aqueles comportamentos agressivos como sendo desviantes,
crianças e adolescentes ameaçados de morte: podem ser passando a aceitá-los como maneira apropriada para resolver
organizados abrigos regionais ou estaduais destinados problemas reais. É importante ressaltar que a informação
especificamente a esta finalidade, os quais, em virtude das exibida pela mídia é mediada por fatores como família, escola
ameaças vividas pelas crianças e adolescentes atendidos, e a percepção individual, e a mera exposição não é suficiente
devem ser implementados em locais que garantam o sigilo para desenvolver comportamentos agressivos (GOMIDE,
quanto à sua existência, necessário à proteção dos acolhidos. 2000).
Tais serviços deverão atender os parâmetros referentes a
recursos humanos e infraestrutura física constantes neste É possível identificar, por meio de discriminações
documento, devendo ser avaliada, ainda, de acordo com a sucessivas ao longo da história de vida, pistas sociais
realidade local, a necessidade de disponibilização de associadas à manifestação de comportamentos hostis. Estudos
profissional específico, que possa se responsabilizar por uma etológicos sugerem que indivíduos de um mesmo grupo social
articulação mais próxima do serviço com o Sistema de Justiça, aprendem a discriminar uma linguagem não-verbal, que pode
o Sistema de Segurança Pública e programas de proteção aos sinalizar aceitação ou desagrado. Nesse caso, identificar as
quais as crianças e adolescentes atendidos estejam vinculados. pistas de agressividade como estímulos associados à
Em virtude das particularidades desta situação recomenda-se manifestação de episódios de raiva ou agressão vivenciados
o atendimento de crianças e adolescentes ameaçados de morte anteriormente pode ser um estímulo discriminativo para
em serviços regionalizados, organizados especificamente para comportamentos de esquiva do indivíduo (RENFREW, 1997).
esta finalidade e com maiores condições de garantir proteção Além disso, os comportamentos agressivos podem ser
nestes casos. mantidos por reforçamento intermitente. Quando uma criança
Em estados / regiões onde o número de casos de crianças pacífica é agredida por um colega e tem a possibilidade de
e adolescentes ameaçados de morte que necessitem de contra-atacar e fugir de outras agressões futuras, o
acolhimento não justificar a implementação de serviços comportamento de contra-atacar é reforçado negativamente
estaduais / regionais: podem ser firmados acordos formais (pelo fato de remover a agressão), aumentando assim a
entre municípios de diferentes regiões, a fim de viabilizar a probabilidade de comportamentos agressivos futuros. Nos
transferência da crianças ou adolescente ameaçado para outro crimes, quando o autor obtém dinheiro, prestígio dentre os
município, de modo a possibilitar seu acolhimento em serviços colegas "assaltantes", bens ou até território, o comportamento
distantes de sua comunidade de origem e, assim, facilitar a sua de assaltar é reforçado positivamente, aumentando assim a
proteção. Nestas situações o serviço deve também manter probabilidade de crimes futuros. Em situações diárias,
articulação com programas de proteção aos quais as crianças indivíduos obtêm vários reforçadores por meio da agressão.
e adolescentes atendidos estejam vinculados, além do Sistema Motoristas "presos" no trânsito de grandes cidades, ao gritar e
de Justiça e do Sistema de Segurança Pública, de modo a buzinar, podem conseguir que o carro da frente lhes dê
propiciar condições de segurança tanto para a criança ou passagem; ao comprar uma mercadoria, que por descuido da
adolescente ameaçado quanto para os demais ali acolhidos. loja ou do fabricante estava com defeito, os consumidores
Em todos os casos, recomenda-se que os serviços de podem "fazer um escândalo" no estabelecimento comercial,
acolhimento que atendam crianças e adolescentes ameaçados humilhando assim o vendedor e conseguir que a mercadoria
de morte atuem em articulação com programas específicos de seja trocada o mais rápido possível.
proteção, como o Programa de Proteção à Criança e ao
Adolescente Ameaçado de Morte – PPCAM. Instituído pela Resolução-CNJ nº 77, de 26 de maio de
Finalmente, ressalta-se que o encaminhamento da criança 2009, o Cadastro Nacional de Adolescentes em Conflito com a
ou adolescente ameaçado de morte para serviço de Lei (CNACL) reúne dados fornecidos pelas Varas de Infância e
acolhimento deve ser considerado apenas quando esgotadas Juventude de todo o país sobre os adolescentes em conflito
outras alternativas que preservem seus vínculos familiares, com a lei. Informações sobre o histórico das infrações
como, por exemplo, a mudança de contexto ou cidade cometidas e as medidas socioeducativas que já foram
acompanhado da família, de familiar ou responsável. aplicadas aos jovens integram o sistema. Juízes e
representantes do Ministério Público têm acesso ao CNACL.

24

http://pepsic.bvsalud.org/scielo.php?script=sci_arttext&pid=S1516-
36872005000100007

Conhecimentos Específicos 124


Apostila Digital Licenciada para Alice Caroline Guarino dos Santos - alice.guarino@hotmail.com (Proibida a Revenda)
APOSTILAS OPÇÃO

RESOLUÇÃO Nº 77, DE 26 DE MAIO DE 2009 funcionamento se dê na forma prevista em lei. (Alterado pela
Resolução nº 188, de 28 de fevereiro de 2014)
Dispõe sobre a inspeção nos estabelecimentos e entidades §3º Constatada qualquer irregularidade na entidade de
de atendimento ao adolescente e sobre a implantação do atendimento ao adolescente, o juiz tomará as providências
cadastro nacional de adolescentes em conflito com a lei. necessárias para a apuração dos fatos e de eventual
responsabilidade, comunicando as medidas tomadas à
O PRESIDENTE DO CONSELHO NACIONAL DE JUSTIÇA, no Corregedoria-Geral e ao magistrado Coordenador da Infância
uso de suas atribuições conferidas pela Constituição da e Juventude do respectivo Tribunal. (Alterado pela Resolução
República, especialmente o disposto no inciso I, §4º, art. 103- nº 188, de 28 de fevereiro de 2014)
B;
Art. 3º Os Tribunais de Justiça poderão expedir
CONSIDERANDO a prioridade das políticas de regulamentos suplementares, considerando as peculiaridades
atendimento à infância e à juventude, preconizada pelo artigo locais.
227 da Constituição Federal;
Art. 4º Os Tribunais devem assegurar a seus respectivos
CONSIDERANDO as normas referentes aos adolescentes juízes condições objetivas para a realização de inspeções
contidas no Estatuto da Criança e do Adolescente, dentre as bimestrais nas Unidades de internação e semiliberdade, sem
quais a obrigatoriedade de efetivação dos direitos referentes à prejuízo das disposições da Resolução CNJ n. 176/2013.
vida, ao respeito e à dignidade, que consistem na (Alterado pela Resolução nº 188, de 28 de fevereiro de 2014)
inviolabilidade da integridade física, psíquica e moral e na §1º O magistrado responsável pela fiscalização bimestral
proibição de tratamento desumano; de mais de 4 (quatro) Unidades, poderá requisitar apoio à
Coordenadoria da Infância e Juventude a fim de que
CONSIDERANDO a peculiar condição do adolescente como encaminhe, no prazo de 48 (quarenta e oito horas), pedido ao
pessoa em processo de desenvolvimento; órgão competente, no sentido de designar, em até 5 (cinco)
dias úteis a partir da comunicação da Coordenadoria da
CONSIDERANDO a obrigatoriedade, prevista no artigo 95 Infância e Juventude, juiz(es) auxiliar(es), com o fim específico
do Estatuto da Criança e do Adolescente, de fiscalização das de atuar(em) na inspeção bimestral das Unidades, com
entidades governamentais e não-governamentais de prioridade sobre demais solicitações, em razão da matéria.
atendimento ao adolescente, elencadas no artigo 90 da mesma (Incluído pela Resolução nº 188, de 28 de fevereiro de 2014)
norma, pelo Poder Judiciário; §2º Os Tribunais devem disponibilizar, em até 10 (dez)
dias, a contar da comunicação da Coordenadoria da Infância e
CONSIDERANDO a aprovação, na 73ª Sessão Plenária, Juventude, a segurança pessoal ao magistrado e sua equipe,
realizada em 04 de novembro de 2008, da proposta nacional para a realização de inspeções nas Unidades, se houver
de promoção de medidas de proteção à infância e à juventude parecer positivo daquele órgão. (Incluído pela Resolução nº
e de reinserção social do adolescente em conflito com a lei; 188, de 28 de fevereiro de 2014)

RESOLVE: Art. 5º Para auxiliar os juízes no controle da aplicação das


medidas socioeducativas, o Conselho Nacional de Justiça
Art. 1º Determinar aos juízes das Varas da Infância e da implanta, neste ato, o cadastro nacional de adolescentes em
Juventude com competência para a matéria referente à conflito com a lei, que tem por finalidade consolidar dados de
execução das medidas socioeducativas sobre os adolescentes todas as comarcas das unidades da federação referentes aos
em conflito com a lei que realizem pessoalmente inspeção envolvidos na prática de atos infracionais, estejam ou não em
bimestral nas Unidades de Internação e de Semiliberdade sob cumprimento das referidas medidas.
sua responsabilidade e adotem as providências necessárias
para o seu adequado funcionamento. (Alterado pela Resolução Art. 6º O cadastro nacional de adolescentes em conflito
nº 188, de 28 de fevereiro de 2014) com a lei ficará hospedado no sítio eletrônico do Conselho
§1º Igual procedimento deve ser adotado pelos juízes que Nacional de Justiça, assegurado o acesso exclusivamente aos
atuam em outros juízos, inclusive juízo único, com órgãos por ele autorizados.
competência concorrente para a matéria de adolescentes em
conflito com a lei. Art. 7º As Corregedorias dos Tribunais de Justiça
§2º No caso de existirem na Comarca entidades ou funcionarão como administradoras do sistema no respectivo
programas para cumprimento de medidas socioeducativas em Estado, e terão acesso integral aos dados, com a atribuição de
meio aberto, a inspeção judicial de que cuida o caput deste cadastrar e liberar o acesso ao juiz competente de cada uma
artigo deverá ser realizada pelo menos uma vez a cada das varas das comarcas, bem como zelar pela correta inserção
semestre. (Incluído pela Resolução nº 157, de 08.08.12) das informações, que deverá se ultimar no prazo de 180 (cento
e oitenta) dias da publicação desta Resolução.
Art. 2º Nas inspeções bimestrais, deverá o juiz preencher
formulário eletrônico do CNJ, disponível no Cadastro Nacional Art. 8º As Corregedorias-Gerais de Justiça e os juízes
de Inspeções em Unidades de Internação e Semiliberdade competentes encaminharão os dados por meio eletrônico ao
(CNIUIS), até o dia 10 do mês seguinte ao bimestre em cadastro nacional dos adolescentes em conflito com a lei.
referência. (Alterado pela Resolução nº 188, de 28 de fevereiro Parágrafo único. Compete às Corregedorias-Gerais dos
de 2014) tribunais organizarem, com o auxílio das Coordenadorias da
§1º Os bimestres serão necessariamente os períodos de Infância e Juventude, curso de capacitação anual para
janeiro e fevereiro; março e abril; maio e junho; julho e agosto; magistrados e servidores acerca do Cadastro Nacional de
setembro e outubro; e novembro e dezembro. (Alterado pela Adolescentes em Conflito com a Lei (CNACL) e do Cadastro
Resolução nº 188, de 28 de fevereiro de 2014) Nacional de Inspeções em unidades de Internação e
§2º Caberá às Corregedorias-Gerais comunicar à Semiliberdade (CNIUIS). (Incluído pela Resolução nº 188, de
Corregedoria Nacional de Justiça o não cumprimento da 28 de fevereiro de 2014)
inspeção bimestral pelo juiz titular ou substituto em exercício,
sem prejuízo das imediatas providências para que o seu

Conhecimentos Específicos 125


Apostila Digital Licenciada para Alice Caroline Guarino dos Santos - alice.guarino@hotmail.com (Proibida a Revenda)
APOSTILAS OPÇÃO

Art. 9º O Conselho Nacional de Justiça prestará o apoio Vamos acompanhar o que o ECA traz a respeito:
técnico necessário aos Tribunais de Justiça dos Estados e do
Distrito Federal para a inserção dos dados no cadastro Capítulo IV
nacional. Das Medidas Socioeducativas
Seção I
Parágrafo único. Os Tribunais poderão manter os atuais
Disposições Gerais
sistemas de dados em utilização no respectivo Estado, ou
substituí-los por outros que entendam mais adequados, desde Art. 112. Verificada a prática de ato infracional, a
que assegurada a migração dos dados ao cadastro nacional. autoridade competente poderá aplicar ao adolescente as
seguintes medidas:
Art. 10. Compete ao Comitê Gestor promover a I - advertência;
implantação, o acompanhamento e o desenvolvimento do II - obrigação de reparar o dano;
III - prestação de serviços à comunidade;
cadastro nacional de adolescentes em conflito com a lei e
IV - liberdade assistida;
efetuar o detalhamento dos procedimentos para o V - inserção em regime de semiliberdade;
cumprimento desta resolução. VI - internação em estabelecimento educacional;
VII - qualquer uma das previstas no art. 101, I a VI.
Art. 11. O Cadastro Nacional de Adolescentes em Conflito § 1º A medida aplicada ao adolescente levará em conta a sua
com a Lei será gerido e fiscalizado pelo Departamento de capacidade de cumpri-la, as circunstâncias e a gravidade da
Monitoramento e Fiscalização do Sistema Carcerário e do infração.
§ 2º Em hipótese alguma e sob pretexto algum, será
Sistema de Execução de Medidas Socioeducativas (DMF), do
admitida a prestação de trabalho forçado.
Conselho Nacional de Justiça. (Alterado pela Resolução nº 188, § 3º Os adolescentes portadores de doença ou deficiência
de 28 de fevereiro de 2014) mental receberão tratamento individual e especializado, em
Parágrafo único. Os demais cadastros do sistema da local adequado às suas condições.
infância e da juventude continuarão a ser geridos e fiscalizados
pela Corregedoria Nacional de Justiça. (Incluído pela Art. 113. Aplica-se a este Capítulo o disposto nos arts. 99 e
100.
Resolução nº 188, de 28 de fevereiro de 2014)
Art. 114. A imposição das medidas previstas nos incisos II a
Art. 12. Esta Resolução entrará em vigor na data de sua VI do art. 112 pressupõe a existência de provas suficientes da
publicação. autoria e da materialidade da infração, ressalvada a hipótese de
remissão, nos termos do art. 127.
Ministro GILMAR MENDES Parágrafo único. A advertência poderá ser aplicada sempre
que houver prova da materialidade e indícios suficientes da
autoria.

Seção II
18. Medidas Socioeducativas; Da Advertência

Art. 115. A advertência consistirá em admoestação verbal,


Durante a execução das medidas sócio educativas, deverá que será reduzida a termo e assinada.
ser observados os seguintes princípio:
- legalidade, não podendo o adolescente receber Seção III
tratamento mais gravoso do que o conferido ao adulto; Da Obrigação de Reparar o Dano
- excepcionalidade da intervenção judicial e da imposição
Art. 116. Em se tratando de ato infracional com reflexos
de medidas, favorecendo-se meios de autocomposição de
patrimoniais, a autoridade poderá determinar, se for o caso, que
conflitos; o adolescente restitua a coisa, promova o ressarcimento do
- prioridade a práticas ou medidas que sejam restaurativas dano, ou, por outra forma, compense o prejuízo da vítima.
e, sempre que possível, atendam às necessidades das vítimas; Parágrafo único. Havendo manifesta impossibilidade, a
- proporcionalidade da medida aplicada em relação à medida poderá ser substituída por outra adequada.
ofensa cometida;
Seção IV
- brevidade da medida em resposta ao ato cometido, em
Da Prestação de Serviços à Comunidade
especial o respeito ao que dispõe o art. 122 do Estatuto da
Criança e do Adolescente; Art. 117. A prestação de serviços comunitários consiste na
- individualização, considerando-se a idade, capacidades e realização de tarefas gratuitas de interesse geral, por período
circunstâncias pessoais do adolescente; não excedente a seis meses, junto a entidades assistenciais,
- mínima intervenção, restrita ao necessário para a hospitais, escolas e outros estabelecimentos congêneres, bem
realização dos objetivos da medida; como em programas comunitários ou governamentais.
Parágrafo único. As tarefas serão atribuídas conforme as
- não discriminação do adolescente, notadamente em
aptidões do adolescente, devendo ser cumpridas durante
razão de etnia, gênero, nacionalidade, classe social, orientação jornada máxima de oito horas semanais, aos sábados, domingos
religiosa, política ou sexual, ou associação ou pertencimento a e feriados ou em dias úteis, de modo a não prejudicar a
qualquer minoria ou status; e frequência à escola ou à jornada normal de trabalho.
- fortalecimento dos vínculos familiares e comunitários no
processo socioeducativo.

Conhecimentos Específicos 126


Apostila Digital Licenciada para Alice Caroline Guarino dos Santos - alice.guarino@hotmail.com (Proibida a Revenda)
APOSTILAS OPÇÃO

Seção V III - por descumprimento reiterado e injustificável da


Da Liberdade Assistida medida anteriormente imposta.
§ 1º O prazo de internação na hipótese do inciso III deste
Art. 118. A liberdade assistida será adotada sempre que se artigo não poderá ser superior a 3 (três) meses, devendo ser
afigurar a medida mais adequada para o fim de acompanhar, decretada judicialmente após o devido processo legal.
auxiliar e orientar o adolescente. § 2º. Em nenhuma hipótese será aplicada a internação,
§ 1º A autoridade designará pessoa capacitada para havendo outra medida adequada.
acompanhar o caso, a qual poderá ser recomendada por
entidade ou programa de atendimento. Art. 123. A internação deverá ser cumprida em entidade
§ 2º A liberdade assistida será fixada pelo prazo mínimo de exclusiva para adolescentes, em local distinto daquele destinado
seis meses, podendo a qualquer tempo ser prorrogada, revogada ao abrigo, obedecida rigorosa separação por critérios de idade,
ou substituída por outra medida, ouvido o orientador, o compleição física e gravidade da infração.
Ministério Público e o defensor. Parágrafo único. Durante o período de internação, inclusive
provisória, serão obrigatórias atividades pedagógicas.
Art. 119. Incumbe ao orientador, com o apoio e a supervisão
da autoridade competente, a realização dos seguintes encargos, Art. 124. São direitos do adolescente privado de liberdade,
entre outros: entre outros, os seguintes:
I - promover socialmente o adolescente e sua família, I - entrevistar-se pessoalmente com o representante do
fornecendo-lhes orientação e inserindo-os, se necessário, em Ministério Público;
programa oficial ou comunitário de auxílio e assistência social; II - peticionar diretamente a qualquer autoridade;
II - supervisionar a frequência e o aproveitamento escolar do III - avistar-se reservadamente com seu defensor;
adolescente, promovendo, inclusive, sua matrícula; IV - ser informado de sua situação processual, sempre que
III - diligenciar no sentido da profissionalização do solicitada;
adolescente e de sua inserção no mercado de trabalho; V - ser tratado com respeito e dignidade;
IV - apresentar relatório do caso. VI - permanecer internado na mesma localidade ou naquela
mais próxima ao domicílio de seus pais ou responsável;
Seção VI VII - receber visitas, ao menos, semanalmente;
Do Regime de Semiliberdade VIII - corresponder-se com seus familiares e amigos;
IX - ter acesso aos objetos necessários à higiene e asseio
Art. 120. O regime de semiliberdade pode ser determinado pessoal;
desde o início, ou como forma de transição para o meio aberto, X - habitar alojamento em condições adequadas de higiene e
possibilitada a realização de atividades externas, salubridade;
independentemente de autorização judicial. XI - receber escolarização e profissionalização;
§ 1º São obrigatórias a escolarização e a profissionalização, XII - realizar atividades culturais, esportivas e de lazer:
devendo, sempre que possível, ser utilizados os recursos XIII - ter acesso aos meios de comunicação social;
existentes na comunidade. XIV - receber assistência religiosa, segundo a sua crença, e
§ 2º A medida não comporta prazo determinado aplicando- desde que assim o deseje;
se, no que couber, as disposições relativas à internação. XV - manter a posse de seus objetos pessoais e dispor de local
seguro para guardá-los, recebendo comprovante daqueles
Seção VII porventura depositados em poder da entidade;
Da Internação XVI - receber, quando de sua desinternação, os documentos
pessoais indispensáveis à vida em sociedade.
Art. 121. A internação constitui medida privativa da § 1º Em nenhum caso haverá incomunicabilidade.
liberdade, sujeita aos princípios de brevidade, excepcionalidade § 2º A autoridade judiciária poderá suspender
e respeito à condição peculiar de pessoa em desenvolvimento. temporariamente a visita, inclusive de pais ou responsável, se
§ 1º Será permitida a realização de atividades externas, a existirem motivos sérios e fundados de sua prejudicialidade aos
critério da equipe técnica da entidade, salvo expressa interesses do adolescente.
determinação judicial em contrário.
§ 2º A medida não comporta prazo determinado, devendo Art. 125. É dever do Estado zelar pela integridade física e
sua manutenção ser reavaliada, mediante decisão mental dos internos, cabendo-lhe adotar as medidas adequadas
fundamentada, no máximo a cada seis meses. de contenção e segurança.
§ 3º Em nenhuma hipótese o período máximo de internação
excederá a três anos.
§ 4º Atingido o limite estabelecido no parágrafo anterior, o 19. Criança e Adolescente
adolescente deverá ser liberado, colocado em regime de vitimizados;
semiliberdade ou de liberdade assistida.
§ 5º A liberação será compulsória aos vinte e um anos de
idade. A manifestação da violência contra crianças e adolescentes
§ 6º Em qualquer hipótese a desinternação será precedida pode ser difícil de ser identificada. Essa dificuldade decorre, na
de autorização judicial, ouvido o Ministério Público. maioria das vezes, quando a violência é empregada no
§ 7º A determinação judicial mencionada no § 1º poderá ser ambiente familiar e a criança é, ainda, muito pequena. Aliada à
revista a qualquer tempo pela autoridade judiciária. proposta de “educar” pelo castigo, essa identificação torna-se,
ainda, mais difícil, pois os pais não reconhecem - ou não
Art. 122. A medida de internação só poderá ser aplicada querem reconhecer - que usaram a violência contra seus filhos.
quando: A violência manifestada no ambiente familiar,
I - tratar-se de ato infracional cometido mediante grave principalmente a sexual, é, muitas vezes, dissimulada através
ameaça ou violência a pessoa; de carícias, toques, “passadas de mão”, chegando ao ponto de
II - por reiteração no cometimento de outras infrações ser manifestada, até mesmo pela conjunção carnal. A
graves; conivência entre os pais pode ser fruto da destruição de seus
próprios valores constituídos na juventude. A agressão sexual

Conhecimentos Específicos 127


Apostila Digital Licenciada para Alice Caroline Guarino dos Santos - alice.guarino@hotmail.com (Proibida a Revenda)
APOSTILAS OPÇÃO

de pais contra filhos revela uma continuação daquele ato Sem medo de errar, acreditamos que a família deve ser
agressivo que sofreram quando crianças ou jovens. O que os reestruturada, sob o aspecto de sua própria composição, da
pais tentam fazer é reproduzir esse tipo de violência, como se representação das figuras paterna e materna, sua
o ato que estão praticando fosse o modelo a ser seguido. responsabilidade frente aos novos desafios da educação
familiar, a aceitação para mudança de comportamentos e de
Quando esse fato ocorre, geralmente, os mecanismos de valores, o repensar sobre as funções e importância de cada um
defesa da criança e do adolescente (governamentais ou não) na família celular, a interferência de parentes, além do diálogo.
entram em ação. Gera-se, então, um conflito entre aqueles Esses são alguns passos que podem ser desenvolvidos para a
órgãos e os pais que não permitem “intromissão” em “assuntos inibição da violência doméstica, principalmente aquela que
de família”; “esses assuntos devem ser resolvidos em família”, tem como vítimas crianças e adolescentes.
“a educação dos filhos compete aos pais”; “os pais educam os
filhos como eles querem”. Essas respostas dadas ao problema O apoio às famílias e seus familiares deve partir de uma
inferem que a comunidade familiar está seguindo modelos premissa de que todos devem mudar seus parâmetros sobre a
propostos por outros núcleos familiares, mormente aqueles de educação de filhos; de que todos devem mudar o “modo de
seus antepassados. Esses modelos, diga-se de passagem, não olhar” o problema. Encará-lo como uma porta que traz
estão errados, podem não estar adequados à realidade soluções e amenizar o sofrimento de um passado desgastado
hodierna, mas o foram em um determinado momento da pela violência, pela agressão, pelo abuso. A mudança de foco
história de seus protagonistas. Contudo, essa resposta sobre o problema possibilita, também, um enriquecimento
simplista não satisfaz mais. As pessoas não querem mais estar interior e um amadurecimento propícios à aceitação das
apegadas e sujeitas a tradições comportamentais, conferindo- mudanças que, certamente, virão após esse exercício de
lhes a responsabilidade pelos erros e acertos. cidadania.
Além disso, há uma certa disseminação de mitos sobre o Esse suporte às famílias é oferecido por Convenções
episódio “violência doméstica”, que, somente para Internacionais de Proteção dos Direitos da Criança e do
exemplificar, poderíamos elencar alguns deles: a) a Adolescente, cuja materialização, entre nós, deu-se através do
vitimização sexual é rara; b) as crianças menores de 10 anos Estatuto da Criança e do Adolescente, que procurou sintetizar,
estão a salvo dessas agressões; c) os agressores são homens na doutrina da proteção integral, uma nova condição jurídica
velhos, violentos, alcoólatras e desempregados, depravados para a população infanto-juvenil: a condição de cidadãos,
sexualmente, retardados ou loucos; d) se uma criança sujeitos de direitos e detentores da especial proteção do
“consente” é porque deve ter gostado, se ela não diz “não” é Estado e de todos em virtude de seu grau de desenvolvimento.
porque não é abuso; e) as crianças inventam histórias de Essa doutrina foi desenvolvida pela Organização das Nações
vitimização sexual. Unidas a pedido dos próprios Estados participantes, que
necessitavam de uma “receita” mais moderna e atual para
Essas alegações, por serem inverídicas, produzem uma tentar reverter o modo de analisar ou oferecer uma nova
sensação de impunidade, ora de indignação, ora de perspectiva entre as relações dos adultos para com as crianças
conformismo diante do episódio violência doméstica e adolescentes.
materializada como aquela ligada ao abuso sexual. Esse
“sintoma” entreguista ou de aceitação da violência contra
crianças e adolescentes dificulta sua apuração e sua 20. Violências e Violência
erradicação. Pais, mães e educadores são, ao seu modo e com Doméstica;
sua bagagem de valores, cúmplices ou coautores desse ato
criminoso.
Como impedir, então, que essa violência se manifeste se os O conceito de violência intrafamiliar inclui toda e qualquer
próprios pais e educadores se protegem em modelos forma de agressão causada entre pessoas que tenham vínculos
anacrônicos de comportamento? A tarefa não é fácil. A couraça familiares ou afetivos entre si e, também, vínculos decorrentes
comportamental, construída ao longo de sua existência, da convivência próxima. A violência intrafamiliar pode ser
impede ou dificulta a transformação ou a aceitação das praticada contra o gênero feminino e masculino. É um tipo de
mudanças na educação de filhos. violência que ocorre dentro de casa, nas relações entre as
A diminuição da violência contra crianças e adolescentes é pessoas da família, entre homens e mulheres, pais, mães e
tarefa que a todos deve interessar. A repressão da violência filhos, entre jovens e idosos, além de eventuais cuidadores que
pode ser o marco inicial de um conjunto de medidas podem estabelecer, mesmo que momentaneamente, um
impeditivas do continuísmo das práticas comportamentais vínculo doméstico.
que envolvem agressões físicas, psíquicas e morais. A tarefa é Exemplo de definição para este conceito, dado por
cidadã. Sem essa conotação é impossível transformar uma Machado e Gonçalves (2003): qualquer ato, conduta ou
civilização, tampouco um conjunto de valores morais omissão que sirva para infligir, deliberadamente e com
familiares, neles incluído, a “severa” educação que inclui o intensidade, sofrimentos físicos, sexuais, mentais ou econômicos,
castigo físico. A transformação dos modelos dantes utilizados de modo direto ou indireto (por meio de ameaças, enganos,
requer, basicamente, educação para a cidadania, incluindo, coação ou qualquer outro meio) a qualquer pessoa que habite
aqui, aquele conjunto de atitudes preservadoras dos direitos no mesmo agregado doméstico privado (crianças, jovens,
da população infanto-juvenil, e, numa visão mais avançada, da mulheres, homens ou idosos) ou que, não habitando no mesmo
preservação até da própria existência humana. agregado doméstico privado que o agente da violência seja
cônjuge ou companheiro marital ou ex-cônjuge ou ex-
Apoio às vítimas e seus familiares companheiro marital.

Ao lado da identificação da violência, dos seus mecanismos Esta definição implica a referência a vários crimes:
inibitórios e restritivos, perquire-se qual o caminho para violência doméstica, ameaça, coação, difamação, injúria,
amparar as vítimas e indicar novos procedimentos e subtração de menor, violação de obrigação de alimentos,
comportamentos aos seus familiares, muitas vezes, abuso sexual, homicídio e outros.
agressores. A tarefa continua sendo árdua, pois o terreno a ser
trabalhado é o da formação dos valores, ou propriamente, da
recomposição dos valores assentados em suas vidas.

Conhecimentos Específicos 128


Apostila Digital Licenciada para Alice Caroline Guarino dos Santos - alice.guarino@hotmail.com (Proibida a Revenda)
APOSTILAS OPÇÃO

Tipos de violência pessoal, vestimenta, educação, habitação e sustentação


emocional e, quando tal falha não é o resultado das condições
- Violência física de vida além do seu controle.
Os diferentes tipos de violência intrafamiliar, em sua
Violência física é entendida como qualquer conduta que maioria não acontecem separadamente, estão
ofenda a integridade ou saúde corporal, como por exemplo, intrinsecamente relacionados e podem ser concretizados em
lesões corporais e tortura. É o uso da força com o objetivo de um único ato de violência. Alguns tipos são mais facilmente
ferir, deixando ou não marcas evidentes. São comuns “murros” detectados do que outros. Contudo, as consequências de todos
e “tapas”, agressões com diversos objetos e queimaduras por eles deixam marcas muitas vezes indeléveis na vida de suas
objetos ou líquidos quentes. vítimas e familiares.

- Violência psicológica Violência intrafamiliar contra a mulher

Violência psicológica consiste na conduta que cause dano A violência contra a mulher é estrutural e está presente em
emocional, diminuição da autoestima, prejudique e perturbe o nossa sociedade desde os primórdios da civilização. Este tipo
pleno desenvolvimento ou que vise degradar ou controlar de violência é inerente aos sistemas patriarcal e capitalista,
ações, comportamentos, crenças e decisões. São exemplos de sendo usado como uma ferramenta de controle da vida, corpo
violência psicológica: ameaça, constrangimento, humilhação, e sexualidade de mulheres.
manipulação, isolamento, vigilância constante, perseguição As mulheres sempre resistiram, e seguem resistindo, a este
contumaz, insulto, chantagem, ridicularização, exploração e tipo de violência, tanto no âmbito individual quanto no
limitação do direito de ir e vir ou qualquer outro meio que lhe coletivo. Sempre que uma mulher age dessa forma, desafiando
cause prejuízo à saúde psicológica e à autodeterminação. ou denunciando a violência contra ela mesma ou contra
A violência psicológica ou “agressão emocional”, às vezes mulheres de sua comunidade, está rompendo com o
tão ou mais prejudicial que a física, é caracterizada por paradigma machista dominante e com o ciclo de violência
rejeição, depreciação, discriminação, humilhação, desrespeito contra a mulher.
e punições exageradas. Trata-se de uma agressão que não A defesa da igualdade de gênero tem o amparo legal na
deixa marcas corporais visíveis, mas emocionalmente pode Constituição Brasileira, promulgada em 1988, que estabelece
causar cicatrizes permanentes para toda a vida. no seu artigo 5º que “todos são iguais perante a lei, sem
distinção de qualquer natureza”.
- Violência verbal A alta incidência da violência contra a mulher cometida no
espaço doméstico e familiar e a luta das mulheres alcançou um
A violência verbal normalmente se dá concomitante à marco jurídico fundamental que é a Lei 11.340/2006 - Lei
violência psicológica, sendo considerada um subtítulo Maria da Penha.
daquela. Alguns agressores verbais dirigem sua “artilharia” Através desse instrumento, configura-se como Violência
(xingamentos, palavrões, ofensas) contra outros membros da Doméstica e Familiar Contra a Mulher toda e qualquer ação ou
família, inclusive nos momentos em que estes estão na omissão que cause morte, lesão, sofrimento físico, sexual ou
presença de outras pessoas estranhas ao lar. psicológico e dano moral ou patrimonial, praticado por pessoa
(homem ou mulher) com que a ofendida (somente a mulher)
- Violência sexual conviva no âmbito doméstico, ou que faça parte do seu âmbito
familiar ou de qualquer relacionamento íntimo de afeto atual
A violência sexual é entendida como qualquer conduta que ou já encerrado.
constranja a vítima a presenciar, a manter ou a participar de A mulher em situação de violência doméstica deve ser
relação sexual não desejada, mediante intimidação, ameaça, orientada para comparecer à DDM (Delegacia de Defesa da
coação ou uso da força; que induza a comercializar ou a Mulher) ou à delegacia comum para registrar o boletim de
utilizar, de qualquer modo, a sexualidade; que impeça de usar ocorrência e para efetuar a representação contra o agressor. A
qualquer método contraceptivo ou que force ao matrimônio, à representação é a confirmação do interesse em processar
gravidez, ao aborto ou à prostituição, mediante coação, criminalmente o agressor e é exigida em alguns crimes. O
chantagem, suborno ou manipulação; ou que limite ou anule o prazo para efetuar a representação é de seis meses a partir da
exercício de seus direitos sexuais e reprodutivos. Exemplos: data da agressão. Lavrado o boletim de ocorrência e feita a
estupro e exploração sexual comercial. representação perante a autoridade policial, a ação judicial
somente será arquivada a pedido da mulher perante o(a)
- Violência patrimonial juiz(a), após ouvido o(a) promotor(a) de justiça.

A violência patrimonial é entendida como qualquer Violência intrafamiliar contra a pessoa com
conduta que configure retenção, subtração, destruição parcial deficiência
ou total de objetos, instrumentos de trabalho, documentos
pessoais, bens, valores e direitos ou recursos econômicos da A segregação das pessoas com deficiência fez parte da
vítima, incluindo os destinados a satisfazer suas necessidades. cultura das sociedades registradas historicamente, o que já
caracteriza a presença de cenários graves de preconceito e
- Violência moral violência contra esse segmento da população. Nos dias atuais
ainda é muito difícil lidar com as minorias e com as diferenças
A violência moral é aquela entendida como qualquer existentes, fazendo parte dos movimentos sociais a luta pela
conduta que configure calúnia, difamação ou injúria. Também equidade social (garantir a igualdade de oportunidades e
se dá de forma verbal, porém, o conteúdo das ofensas se dá direitos, mas considerando positivamente a existência das
prioritariamente em falsas acusações contra a vítima. diferenças).
A falta de debate público e políticas sociais efetivas
- Negligência contribuem para que as pessoas com deficiência estejam
potencialmente mais expostas a situações de violência -
A negligência configura-se quando os pais ou responsáveis inclusive no âmbito intrafamiliar - tendo maior dificuldade em
falham em prover cuidados de saúde, nutrição, higiene denunciar os abusos sofridos. Outro fator importante que

Conhecimentos Específicos 129


Apostila Digital Licenciada para Alice Caroline Guarino dos Santos - alice.guarino@hotmail.com (Proibida a Revenda)
APOSTILAS OPÇÃO

devemos considerar é que, ainda nos dias de hoje, a falta de aos problemas identificados, tanto de ordem física, sexual
acessibilidade provoca a perda de autonomia das pessoas com como emocional - procurando a sua correlação com o relato da
deficiência, dificultando o rompimento com a situação de possível vítima, dos familiares ou pessoas de sua convivência.
violência.
Podemos dizer que alguns fatores potencializam a Violência intrafamiliar contra idosos
vitimização da pessoa com deficiência, como a infância, o
gênero feminino e a condição de pessoa idosa, que são fatores O número de idosos no Brasil está cada vez maior, mas
que por si só também caracterizam as possíveis vítimas, algumas instituições e sujeitos ainda estão despreparados
levando em consideração que o agressor procura estar em para lidar com questões próprias do envelhecimento, assim
posição de poder em relação ao outro (poder de autoridade, como condições psicológicas e sociais, o que contribui para o
social, econômico, etc.). crescimento da violência contra idosos.
A informação contribui substantivamente para o Fatores sociais como isolamento, casas de longa
empoderamento da população, por isso, faz-se necessário que permanência, empobrecimento da população, dificuldades dos
as pessoas com deficiência e seus familiares conheçam os seus cuidadores ou dependência financeira dos membros da
direitos e que a Defensoria Pública participe desse processo, família, ampliam a possibilidade de ocorrência de violência,
garantindo tanto com ações judiciais quanto uma vez que torna o idoso mais vulnerável.
extrajudicialmente que esse público tenha direito à Em relação aos aspectos legais, a Constituição Federal
acessibilidade e demais direitos que compõem sua cidadania e impede qualquer forma de discriminação por idade e atribui à
os protegem da violência. família, a sociedade e ao Estado o dever de amparar o idoso,
assegurar sua participação na comunidade, defender sua
Violência intrafamiliar contra crianças e adolescentes cidadania e bem-estar e garantir seu direito à vida. Além disso,
o crime cometido contra idosos é agravante da pena nos
A violência intrafamiliar ou doméstica contra crianças e delitos previstos pelo Código Penal.
adolescentes é uma das várias expressões da questão social,
considerada pelo Ministério da Saúde como um sério O Estatuto do Idoso transforma em lei o que deveria ser
problema de saúde pública. Suas raízes estão associadas ao obrigação da sociedade. Estabelece que, ao chegar aos 60 anos,
contexto histórico, social, cultural, econômico e político em todos têm o direito de envelhecer com dignidade, e tanto o
que se inserem vítimas e agressores. Assim, este tipo de Estado, como a sociedade e a família têm obrigação de
violência não pode ser compreendido somente como uma assegurar aos idosos do país liberdade e respeito, como
questão de conflitos interpessoais entre pais e filhos. pessoas humanas e sujeitos de direitos civis, políticos,
Há alguns anos, pensava-se que a violência intrafamiliar individuais e sociais garantidos pela Constituição.
contra crianças e adolescentes era consequência de A maioria dos agressores de idosos é um familiar ou algum
transtornos individuais, alcoolismo, toxicomania, fragilidades conhecido. Uma das queixas mais registradas é a violência
socioeconômicas ou educacionais, dentre outras de caráter patrimonial, que decorre da apropriação indevida dos bens do
individual. Contudo, segundo a literatura recente, passa a ser idoso através do uso da violência. Os idosos acabam
compreendida como uma questão multifacetada, permitindo que os agressores se apropriem de seus bens, pelo
caracterizada principalmente por aspectos sociais e culturais. medo da solidão, da vida muitas vezes vulnerável em casas de
Tanto no Brasil como em várias partes do mundo, em longa permanência e clínicas geriátricas.
diferentes culturas e classes sociais, independente de sexo ou Os idosos maltratados, sendo mais vulneráveis que as
etnia, crianças e adolescentes são vítimas cotidianas da demais faixas etárias, encontram-se muitas vezes inseguros e
violência doméstica. Os casos registrados em delegacias, fragilizados e, por essa razão não procuram em primeira
conselhos tutelares, hospitais e institutos médico-legais são instância os serviços especializados. Sem o apoio de familiares
apenas um alerta; não revelam a verdadeira dimensão do e sem amizades, torna-se difícil e constrangedor procurar,
problema. A cultura do silêncio e da omissão ainda é muito sozinho, os serviços de segurança pública especializados.
forte em nossa sociedade. Alguns idosos são, ainda, destituídos do poder de decisão,
De acordo com as professoras Maria Amélia Azevedo e privados de ocupar um espaço físico próprio, do direito de
Viviane Guerra (USP/SP, 1989) a questão da violência escolha, da liberdade de expressão e do direito de envelhecer
doméstica contra crianças e adolescentes pode ser com saúde e plenitude de cidadania.
considerada A presença ativa na vida social deve ser valorizada e
estimulada junto aos idosos, mediante formas de convívio e
[...] todo ato ou omissão praticado por pais, parentes ou expressão, em centros de comunidade, clubes de convivência,
responsáveis contra crianças e/ou adolescentes que - sendo associações culturais, de lazer e esportivas, atividades de
capaz de causar dano físico, sexual e/ou psicológico a vítima - recuperação da história e habilidades de uma comunidade,
implica de um lado, numa transgressão do direito que crianças valorizando-se a transmissão aos mais jovens de suas
e adolescentes têm de ser tratados como sujeitos e pessoas em experiências e histórias de vida.
condição peculiar de desenvolvimento. (AZEVEDO e GUERRA, Para isso a rede de atendimento à pessoa idosa é
1989). fundamental e precisa estar articulada numa diversidade de
serviços, como a viabilização de formas alternativas de
Segundo Day et. al. (2003) a Constituição Federal e o participação, ocupação e convívio do idoso.
Estatuto da Criança e do Adolescente passam a ser os novos
paradigmas para o sistema de Justiça, para a sociedade e para O tratamento dispensado às pessoas em situação de
o Brasil como um todo. A nova legislação, signatária da violência intrafamiliar
Doutrina da Proteção Integral, reconhece direitos à criança e
ao adolescente, respeitando seu estágio de desenvolvimento. É Quando recebemos uma pessoa tentando romper o ciclo de
uma das legislações mais avançadas do mundo, que pouco a violência, cabe a nós participar desse processo, realizando um
pouco começa a ser implementada. atendimento de qualidade e comprometimento diante da sua
A violência contra crianças e adolescentes apresenta-se situação.
sob diversas formas, tanto que um sintoma ou sinal isolado Algumas vezes vamos perceber que uma pessoa pode dar
não permite afirmar sua existência. Por isso, é fundamental o início a um processo judicial para se desvincular da situação
olhar atento e crítico dos profissionais e da sociedade frente de violência, mas que tempos depois, desiste de levá-lo

Conhecimentos Específicos 130


Apostila Digital Licenciada para Alice Caroline Guarino dos Santos - alice.guarino@hotmail.com (Proibida a Revenda)
APOSTILAS OPÇÃO

adiante. Esta situação é comum nos casos que envolvem O que oferece? Assistência integral à mulher que tenha
violência e laços afetivos, sendo caracterizada, segundo necessidade de acolhimento residencial, em virtude de risco
estudos científicos, como um momento do perverso ciclo da de morte.
violência.
Nas situações de violência intrafamiliar, devemos estar - Delegacia de Defesa da Mulher – DDM
atentos para não fazer julgamentos baseados no senso comum, O que é? Delegacia especializada no atendimento a
mas sim colaborar no processo de fortalecimento de cidadania mulheres vítimas de violência.
da vítima para que se consiga estabelecer um cotidiano sem O que oferece? Registro de denúncias (ocorrências),
violência. investigação criminal, solicitação de medidas protetivas ao
A vítima da violência intrafamiliar muitas vezes é juízo competente, encaminhamento para outros órgãos de
conduzida a acreditar que é ela mesma a responsável pela proteção.
violência sofrida, o que contribui para a construção de
sentimentos de vergonha e culpa. Neste sentido, um - Disque 180
atendimento inadequado pode colaborar para que esses O que é? Este serviço oferecido pela Secretaria Especial de
sentimentos e condições éticas se cristalizem, colocando mais Políticas para as Mulheres da Presidência da República que
obstáculos para o rompimento do ciclo de violência. Num esclarece as vítimas sobre seus direitos e mostra onde e como
atendimento a uma vítima de violência, seja ela qual for, é obter ajuda. Funciona 24 horas, todos dias da semana,
importante, antes de tudo, que se constitua um ambiente inclusive aos finais de semana.
sigiloso e acolhedor, que poderá indicar à vítima o respaldo à O que oferece? Informações gratuitas via telefone
sua necessidade, sem julgamento, e sem que ela se sinta o
“assunto do dia” na instituição. Violência contra crianças e adolescentes
Portanto, o atendimento deve ser realizado em um local
que garanta o sigilo das informações que estão sendo - Conselho Tutelar
reveladas, cumprindo o princípio da Psicologia e do Serviço O que é? Órgão encarregado de zelar pelo cumprimento
Social no Art. 1, Item IX, da Deliberação CSDP 187, de dos direitos da criança e do adolescente.
12/08/2010, bem como dos respectivos códigos de ética O que oferece? Acolhimento e averiguação de denúncias,
profissionais e resoluções dos conselhos federais de profissão. aplicação de medidas de proteção a crianças e adolescentes e
A violência intrafamiliar é constantemente tratada como medidas aos pais, requisição de serviços públicos
assunto da esfera privada das relações sociais, cabendo
exclusivamente à vítima se desvencilhar dessa situação. - Delegacia Especializada
Porém, é responsabilidade também da sociedade e do Estado O que é? Delegacia especializada no atendimento a criança
trazer essa temática ao campo público para garantirmos que e adolescentes que sofreram violência.
todo cidadão tenha o direito a uma vida sem violência. O que oferece? Registro de denúncias (ocorrências),
investigação criminal, encaminhamento para outros órgãos de
Serviços da Rede de Proteção que podem ser proteção.
acessados
- Disque 100
- CREAS (diversos públicos) Violência contra pessoas com deficiência
O que é? O Centro de Referência Especializado de
Assistência Social (Creas) configura-se como uma unidade - Delegacia de Polícia
pública e estatal, que oferta serviços especializados e O que é? Atendimento a cidadãos vítimas de violência.
continuados a famílias e indivíduos em situação de ameaça ou O que oferece? Registro de denúncias (ocorrências),
violação de direitos (violência física, psicológica, sexual, tráfico investigação criminal, encaminhamento para outros órgãos de
de pessoas, cumprimento de medidas socioeducativas em proteção.
meio aberto, etc.).
O que oferece? Atendimento psicológico, social e jurídico - Violência contra idosos
especializado a pessoas com direitos violados e suas famílias,
inclusive vítimas de violência. - Delegacia Especializada
O que é? Delegacia especializada no atendimento a idosos
Violência contra mulher vítimas de violência.
O que oferece? Registro de denúncias (ocorrências),
- CRM – Centro de Referência da Mulher investigação criminal, solicitação de medidas protetivas ao
O que é? O Centro de Referência é o espaço estratégico da juízo competente, encaminhamento para outros órgãos de
política de enfrentamento à violência contra as mulheres, proteção.
exercendo o papel de articulador dos serviços, organismos
governamentais e não governamentais que integram a rede de - Centro de Referência do Idoso
atendimento às mulheres e garantindo o acesso a outros O que é? Serviço de referência, proteção e defesa de
serviços para as que se encontram em situação de direitos da pessoa idosa.
vulnerabilidade social, em função da violência de gênero. O que oferece? Atendimento de modo individual e coletivo
O que oferece? Atendimento psicológico, social e jurídico estimula a participação social. É espaço difusor de
a mulheres vítimas de violência, tendo ou não registrado conhecimento e intercâmbio de experiências inovadoras,
boletim de ocorrência. objetivando fortalecer as políticas públicas e disseminar
práticas qualificadas para os demais parceiros da rede de
- Casa Abrigo proteção ao idoso. Além da atenção direta aos idosos; é
O que é? As Casas Abrigo oferecem atendimento referência para qualificação institucional e defesa dos direitos,
temporário para mulheres em risco de morte. Os endereços mantendo estratégias de trabalho articulado com a rede de
são sigilosos para garantir a segurança e integridade física da proteção social ao idoso.
mulher.

Conhecimentos Específicos 131


Apostila Digital Licenciada para Alice Caroline Guarino dos Santos - alice.guarino@hotmail.com (Proibida a Revenda)
APOSTILAS OPÇÃO

Autor: Marco Antônio de Oliveira Branco III – requisição para tratamento de sua saúde, em regime
Organização: Assessoria Técnica Psicossocial e Comissão de ambulatorial, hospitalar ou domiciliar;
Estudos Interdisciplinares IV – inclusão em programa oficial ou comunitário de
Revisão: Assessorias Criminal e Cível auxílio, orientação e tratamento a usuários dependentes de
http://www.defensoria.sp.gov.br/dpesp/repositorio/0/doc drogas lícitas ou ilícitas, ao próprio idoso ou à pessoa de sua
umentos/cam/Violencia.pdf convivência que lhe cause perturbação;
V – abrigo em entidade;
VI – abrigo temporário.
21. Medidas de Proteção e a
Tutela de idosos em situação de
risco; 22. Tutela e Curatela;

O Estatuto do Idoso, assim como o ECA (Estatuto da Prezado candidato, este assunto já foi estudado na
Criança e do Adolescente), é mais um instrumento para a matéria de Legislação, tópico: - Lei 10.406/2002 Arts:
realização da cidadania. 1511 a 1638; 1694 a 1727 e 1728 a 1783, mais
O idoso possui direito à liberdade, à dignidade, especificamente entre os artigos 1728-1783.
à integridade, à educação, à saúde, a um meio ambiente de
qualidade, entre outros direitos fundamentais (individuais,
sociais, difusos e coletivos), cabendo ao Estado, à Sociedade 23. Guarda;
e à família a responsabilidade pela proteção e garantia 24. Guarda Compartilhada;
desses direitos.
Pode-se afirmar que o cerne do Estatuto está nas normas
gerais que referem sobre a PROTEÇÃO INTEGRAL. Guarda: é uma das modalidades de colocação em família
A natureza e essência encontram-se no Artigo 2º, quando substituta.
estabelece a sucessão de direitos do idoso e visualiza sua Trata-se de medida destinada a regularizar uma situação
condição como ser constituído de corpo, mente e espírito – já de fato e pode ser deferida em liminar ou incidentalmente em
prevê a preservação de seu bem-estar físico, mental e processo de tutela ou adoção, salvo nos casos de adoção de
espiritual – e identifica a existência de instrumentos que estrangeiros (não é possível liminar para adoção
assegurem seu bem-estar, o qual na lei seria: internacional), não implicando necessariamente na perda ou
Art. 2º - O idoso goza de todos os direitos fundamentais suspensão do poder familiar.
inerentes à pessoa humana, sem prejuízo da proteção integral É conferida nos casos em que os pais não possuam
de que trata esta Lei, assegurando-se lhe, por lei ou por outros condições de exercer adequadamente o poder familiar. Ainda
meios, todas as oportunidades e facilidades, para preservação que esta incapacidade seja temporária, poderá ser colocada a
de sua saúde, em condições de liberdade e dignidade. criança ou adolescente sob guarda de outrem.
O grande desafio diário é construir uma consciência
coletiva de forma a que tenhamos “uma sociedade para Modalidades de guarda no ECA:
todas as idades”, com justiça e garantia plena de direitos.
A guarda, como forma de colocação em família substituta,
(Estatuto do Idoso) divide-se em de fato e de direito.
TÍTULO III A guarda de fato é aquela que não decorrente de decisão
DAS MEDIDAS DE PROTEÇÃO judicial, isto é, o menor é simplesmente acolhido por terceiros
CAPÍTULO I e passa a conviver com eles. Lembre-se que o ECA não prevê
DAS DISPOSIÇÕES GERAIS essa modalidade de guarda, porém descreve-a, em seu art. 33,
§ 1º, como uma das finalidade da guarda.
Art. 43. As medidas de proteção ao idoso são aplicáveis Por outro lado, a guarda de direito vem descrita no ECA e
sempre que os direitos reconhecidos nesta Lei forem pode ser:
ameaçados ou violados:
I – por ação ou omissão da sociedade ou do Estado; - Provisória (art. 33, § 1°, do ECA), que pode ser liminar ou
II – por falta, omissão ou abuso da família, curador ou incidental;
entidade de atendimento; É aquela que ocorre nos processos de tutela ou adoção,
III – em razão de sua condição pessoal. podendo ser deferida como como medida preparatória para a
ação de adoção ou tutela, caso em que será deferida por meio
CAPÍTULO II de liminar ou no curso da ação, ocasião em que será incidental
DAS MEDIDAS ESPECÍFICAS DE PROTEÇÃO (art. 33, § 1°, parte final)

Art. 44. As medidas de proteção ao idoso previstas nesta - Permanente (art. 33, § 2°, do ECA);
Lei poderão ser aplicadas, isolada ou cumulativamente, e Em regra, a guarda tem caráter transitório. Contudo, em
levarão em conta os fins sociais a que se destinam e o situações peculiares, é possível que se defira guarda
fortalecimento dos vínculos familiares e comunitários. permanente, como nas hipóteses de ausência eventual dos pais
ou responsável. (art. 33).
Art. 45. Verificada qualquer das hipóteses previstas no art. Vale lembrar que, embora vigore por prazo indeterminado,
43, o Ministério Público ou o Poder Judiciário, a requerimento esta guarda pode ser revogada a qualquer tempo, por meio ato
daquele, poderá determinar, dentre outras, as seguintes judicial fundamentado, ouvido o Ministério Público (art. 35).
medidas:
I – encaminhamento à família ou curador, mediante termo - Previdenciária (art. 33, § 3°, do ECA);
de responsabilidade; Esta modalidade de guarda consiste na transferência, para
II – orientação, apoio e acompanhamento temporários; a criança ou adolescente, da condição de dependente, para

Conhecimentos Específicos 132


Apostila Digital Licenciada para Alice Caroline Guarino dos Santos - alice.guarino@hotmail.com (Proibida a Revenda)
APOSTILAS OPÇÃO

todos os fins e efeitos de direito, inclusive previdenciários (art. II - (revogado)


33, §3°). III - (revogado)
§ 3º Na guarda compartilhada, a cidade considerada base
Apenas a título de conhecimento, existe certa divergência de moradia dos filhos será aquela que melhor atender aos
de entendimentos acerca do tema: interesses dos filhos. (Redação dada pela Lei nº 13.058, de 2014)
O STJ entende que em se tratando de ação para fins de § 4º (VETADO).
inclusão de menor sob guarda como dependente de segurado § 5º A guarda unilateral obriga o pai ou a mãe que não a
abrangido pelo Regime Geral da Previdência Social - RGPS, não detenha a supervisionar os interesses dos filhos, e, para
prevalece o disposto no art. 33, § 3°, do ECA em face da possibilitar tal supervisão, qualquer dos genitores sempre será
alteração introduzida pela Lei 9.528/97 (REsp 503.019/RS, parte legítima para solicitar informações e/ou prestação de
rei. Min. Paulo Gallotti, DJ 30.10.2006). contas, objetivas ou subjetivas, em assuntos ou situações que
Por outro lado, existem autores que entendem não ser direta ou indiretamente afetem a saúde física e psicológica e a
possível admitir a concessão de guarda única e exclusivamente educação de seus filhos. (Incluído pela Lei nº 13.058, de 2014)
para fins previdenciários, pois configura desvio de finalidade
da guarda. Entretanto, deve-se observar que, havendo situação Art. 1.584. A guarda, unilateral ou compartilhada, poderá
real de guarda, deve ser concedido o benefício previdenciário ser:
para a criança ou adolescente, em decorrência do art. 33, § 3°, I – requerida, por consenso, pelo pai e pela mãe, ou por
do ECA. qualquer deles, em ação autônoma de separação, de divórcio, de
dissolução de união estável ou em medida cautelar;
- Especial (art. 34 do ECA) II – decretada pelo juiz, em atenção a necessidades
Cuida-se da colocação, sob o regime de guarda, de criança específicas do filho, ou em razão da distribuição de tempo
ou de adolescente que tenha sido afastado do convívio necessário ao convívio deste com o pai e com a mãe.
familiar, devendo o Poder Público incentivar essa forma de § 1º Na audiência de conciliação, o juiz informará ao pai e à
guarda. mãe o significado da guarda compartilhada, a sua importância,
a similitude de deveres e direitos atribuídos aos genitores e as
Procedimento para a Guarda no ECA sanções pelo descumprimento de suas cláusulas.
O procedimento da guarda judicial está previsto nos § 2º Quando não houver acordo entre a mãe e o pai quanto
arts.165 a 170 do ECA. à guarda do filho, encontrando-se ambos os genitores aptos a
O procedimento, que se dá mediante contraditório, é exercer o poder familiar, será aplicada a guarda compartilhada,
utilizado quando não há concordância de um dos genitores ou salvo se um dos genitores declarar ao magistrado que não deseja
de ambos. No caso da guarda em que há consenso dos a guarda do menor. (Redação dada pela Lei nº 13.058, de 2014)
genitores, ou caso eles tenham sido destituídos do poder § 3º Para estabelecer as atribuições do pai e da mãe e os
familiar ou tenham falecido, não é necessário procedimento períodos de convivência sob guarda compartilhada, o juiz, de
contraditório, sendo possível seu pedido diretamente em ofício ou a requerimento do Ministério Público, poderá basear-
cartório. Na hipótese de concordância dos pais, serão eles se em orientação técnico-profissional ou de equipe
ouvidos pelo juiz e pelo Ministério Público (art. 166 do ECA). interdisciplinar, que deverá visar à divisão equilibrada do tempo
com o pai e com a mãe. (Redação dada pela Lei nº 13.058, de
Nos termos do art. 35, poderá ocorrer a revogação ou 2014)
modificação da guarda, mediante ato fundamentado do juiz, § 4º A alteração não autorizada ou o descumprimento
ouvido o Ministério Público. imotivado de cláusula de guarda unilateral ou compartilhada
poderá implicar a redução de prerrogativas atribuídas ao seu
Guarda Compartilhada: é aquela definida por consenso detentor. (Redação dada pela Lei nº 13.058, de 2014)
entre o pai e mãe, se não houver acordo enre ambos, será § 5º Se o juiz verificar que o filho não deve permanecer sob
decretado pelo juiz. a guarda do pai ou da mãe, deferirá a guarda a pessoa que
Quando o magistrado for fixar a guarda, levará em revele compatibilidade com a natureza da medida,
consideração as necessidades específicas dos filhos e considerados, de preferência, o grau de parentesco e as relações
distribuição necessária do convívio com os pais. de afinidade e afetividade. (Redação dada pela Lei nº 13.058, de
2014)
O juiz não determinará a guarda compartilhada quando: § 6º Qualquer estabelecimento público ou privado é
- um dos genitores não estiver apto a exercer o poder obrigado a prestar informações a qualquer dos genitores sobre
familiar, oi os filhos destes, sob pena de multa de R$ 200,00 (duzentos reais)
- um dos genitores declarar ao magistrado que não deseja a R$ 500,00 (quinhentos reais) por dia pelo não atendimento da
a guarda do menor. solicitação. (Incluído pela Lei nº 13.058, de 2014)

Disposições do Código Civil sobre o tema: Art. 1.585. Em sede de medida cautelar de separação de
corpos, em sede de medida cautelar de guarda ou em outra sede
CAPÍTULO XI de fixação liminar de guarda, a decisão sobre guarda de filhos,
Da Proteção da Pessoa dos Filhos mesmo que provisória, será proferida preferencialmente após a
oitiva de ambas as partes perante o juiz, salvo se a proteção aos
Art. 1.583. A guarda será unilateral ou compartilhada. interesses dos filhos exigir a concessão de liminar sem a oitiva
§ 1º Compreende-se por guarda unilateral a atribuída a um da outra parte, aplicando-se as disposições do art. 1.584.
só dos genitores ou a alguém que o substitua (art. 1.584, § 5º) e, (Redação dada pela Lei nº 13.058, de 2014)
por guarda compartilhada a responsabilização conjunta e o
exercício de direitos e deveres do pai e da mãe que não vivam sob Art. 1.586. Havendo motivos graves, poderá o juiz, em
o mesmo teto, concernentes ao poder familiar dos filhos comuns. qualquer caso, a bem dos filhos, regular de maneira diferente da
§ 2º Na guarda compartilhada, o tempo de convívio com os estabelecida nos artigos antecedentes a situação deles para com
filhos deve ser dividido de forma equilibrada com a mãe e com o os pais.
pai, sempre tendo em vista as condições fáticas e os interesses
dos filhos. (Redação dada pela Lei nº 13.058, de 2014) Art. 1.587. No caso de invalidade do casamento, havendo
I - (revogado) filhos comuns, observar-se-á o disposto nos arts. 1.584 e 1.586.

Conhecimentos Específicos 133


Apostila Digital Licenciada para Alice Caroline Guarino dos Santos - alice.guarino@hotmail.com (Proibida a Revenda)
APOSTILAS OPÇÃO

Art. 1.588. O pai ou a mãe que contrair novas núpcias não eventos e publicações bem como de informações veiculadas
perde o direito de ter consigo os filhos, que só lhe poderão ser pelos diferentes meios de comunicação sobre a SAP. A
retirados por mandado judicial, provado que não são tratados mobilização da opinião pública e a comoção gerada em torno
convenientemente. do sofrimento de crianças que supostamente seriam vítimas
da SAP culminou, naquele mesmo ano, na elaboração do
Art. 1.589. O pai ou a mãe, em cuja guarda não estejam os Projeto de Lei nº. 4853/08, que teria como objetivo identificar
filhos, poderá visitá-los e tê-los em sua companhia, segundo o e punir os genitores responsáveis pela alienação parental dos
que acordar com o outro cônjuge, ou for fixado pelo juiz, bem filhos2. Tal projeto, com célere trâmite legislativo, foi
como fiscalizar sua manutenção e educação. sancionado pelo Presidente da República, em agosto de 2010,
Parágrafo único. O direito de visita estende-se a qualquer como Lei nº 12.318/10.
dos avós, a critério do juiz, observados os interesses da criança Embora as justificativas para a criação da nova lei
ou do adolescente. (Incluído pela Lei nº 12.398, de 2011) mencionem aspectos ligados ao campo da Psicologia e de a lei
dispor sobre a maneira como devem atuar os profissionais que
Art. 1.590. As disposições relativas à guarda e prestação de irão avaliar possíveis casos de alienação parental, o assunto,
alimentos aos filhos menores estendem-se aos maiores com efeito, parece não ter sido motivo de análise detalhada
incapazes. pelos profissionais da área. No presente artigo, tem-se como
proposta examinar os argumentos que envolvem o campo da
Psicologia que deram respaldo à criação da Lei nº.12.318/10,
explanando-se, também, as decisões judiciais proferidas em
25. Alienação Parental. diferentes países que tomaram por base o conceito síndrome
de alienação parental.

SÍNDROME DE ALIENAÇÃO PARENTAL25 Avaliando os motivos e as proposições da Lei

A despeito das polêmicas e controvérsias que envolvem o No presente item, serão abordados alguns argumentos que
assunto (Escudero, Aguilar, & Cruz, 2008), a proposta de se encontram na exposição de motivos do projeto de lei sobre
Gardner difundiu-se rapidamente no Brasil e em outros países, alienação parental, que serão examinados, especialmente, à
levando alguns a pensar que a suposta síndrome havia se luz de conhecimentos da Psicologia. No entanto, faz-se
tornado uma epidemia em todo o mundo (Álvarez, n.d). No necessário esclarecer, inicialmente, que o conceito de SAP se
Brasil, como evidenciado em pesquisa empreendida por Sousa acha ligado a uma corrente da psiquiatria norte-americana que
(2010) sobre o tema, a escassez de debates e estudos acerca tem como um dos seus representantes Richard Gardner, que
do conceito de SAP, bem como a ausência de questionamentos se baseou antes em elementos supostamente lógicos na defesa
sobre a ideia de um distúrbio infantil ligado às situações de de sua teoria do que na realização de estudos sobre o
disputa entre pais separados, vêm contribuindo para a fenômeno que tentava apreender (Escudero et al., 2008).
naturalização do assunto de forma acrítica. Tal cenário Interessa notar, ainda, a expectativa – presente nos escritos de
colabora, ainda, com a visão de que muitos casos de litígio Gardner – de que a denominada SAP fosse incluída na próxima
conjugal têm como consequência o surgimento da denominada revisão do Manual Diagnóstico e Estatístico de Transtornos
síndrome. Mentais, DSM-V, pela Associação Americana de Psiquiatria.
Sousa (2010), em estudo desenvolvido sobre a temática, Dessa maneira, tal síndrome seria somada ao rol de
notou que associações de pais separados tiveram papel de categorias diagnósticas ou transtornos mentais infantis
destaque na promoção das ideias do psiquiatra norte- incluídas no DSM, como, por exemplo, o distúrbio do déficit de
americano sobre a SAP. Cabe mencionar que, no Brasil, essas atenção com hiperatividade (DDAH), comumente associado a
associações inicialmente se dedicaram a promover a igualdade crianças muito agitadas em sala de aula. Vale mencionar que
de direitos e deveres de pais separados, gerando, com isso, diversas categorias diagnósticas listadas no referido manual
uma série de debates acerca da importância da modalidade de têm contribuído para o incremento de pesquisas com vistas a
guarda compartilhada como forma de preservar a convivência que se disponibilizem novos medicamentos no mercado
familiar após o rompimento conjugal. Contudo, apesar das (Martins, 2008). O diagnóstico do DDAH, por exemplo, vem
contrariedades e dos dissensos que envolvem o entendimento justificando a medicalização de milhares de crianças em todo
e a aplicação desse modelo de guarda no país (Brito & o mundo.
Gonsalves, 2009), muitas associações de pais separados, nos Quanto à revisão do DSM, com previsão para ser publicada
últimos tempos, privilegiaram a divulgação da SAP. em 2013, importa salientar que diversos profissionais
Verifica-se que essa mudança de foco do tema igualdade estrangeiros vêm expondo suas preocupações acerca da nova
parental para a temática da SAP teve início no ano 2006, edição do Manual, conforme notícia veiculada no jornal O
quando da tramitação do projeto de lei sobre a guarda Estado de São Paulo, em 27 de julho de 2010. Na matéria,
compartilhada. Como justificativa para tanto, destaca-se a alguns psiquiatras alertam para o fato de que haverá uma
afirmação publicada na página eletrônica de uma associação considerável expansão de comportamentos que passarão a ser
de pais separados de que, "em decorrência da celeridade com vistos como transtornos psiquiátricos, incluindo-se nessa
que o projeto de lei (sobre guarda compartilhada) está listagem a síndrome de risco de psicose e o transtorno do
tramitando, (e) do novo artifício usado pelos genitores temperamento irregular. Na citada notícia, alguns
guardiães em não aceitar a participação do genitor não especialistas chegam a concluir que "em breve ninguém mais
guardião no desenvolvimento dos filhos (...)", a associação vai ser classificado como normal", motivo pelo qual a todos
estabelece como prioridade em suas ações a difusão do tema será recomendado o uso de algum tipo de medicamento.
SAP1. No que tange às dificuldades que atingem as relações
parentais quando há exacerbado conflito conjugal, cabe
Ainda nessa esteira, nota-se que, especialmente a partir da destacar que, de certa forma, estas já teriam sido abarcadas
aprovação da lei sobre guarda compartilhada (Lei n.º por aquele manual. No DSM-IV-TR (2002), encontra-se o
11698/08), em fins de 2008, houve acréscimo do número de capítulo referente a categorias de problemas de

25 SOUSA, A. M. de; BRITO, L. M. T de. Psicol. cienc. prof. vol.31 no.2 http://dx.doi.org/10.1590/S1414-98932011000200006
Brasília, 2011

Conhecimentos Específicos 134


Apostila Digital Licenciada para Alice Caroline Guarino dos Santos - alice.guarino@hotmail.com (Proibida a Revenda)
APOSTILAS OPÇÃO

relacionamento que podem merecer cuidado clínico. Dentre escolha profissional, futebol e namoros – com o pai que não
aquelas, merece destaque a de problemas de relacionamento permaneceu com a guarda. Os jovens ressaltaram que não
entre pai/mãe-criança que, segundo o Manual, "(...) deve ser havia naturalidade no relacionamento com este, não existindo,
usada quando o foco de atenção clínica é um padrão de por exemplo, o hábito de fazer ligações telefônicas para
interação entre pai/mãe-criança (...), associado com prejuízo conversar ou comentar a respeito de qualquer assunto na
significativo individual ou familiar, ou desenvolvimento de medida em que, com a guarda unilateral, sentiam que o genitor
sintomas clinicamente significativos no pai, na mãe ou na não guardião deixava de acompanhar seu cotidiano. Não havia
criança" (DSM-IV-TR, p. 688). Compreende-se que essa clareza, por parte dos filhos entrevistados, de que, embora
categoria pode ser empregada em diversas situações, inclusive separados, tanto o pai quanto a mãe continuavam
nas de elevado conflito entre pais separados, em que a criança responsáveis por sua educação. Não se pode desconsiderar o
exibe forte vinculação com um dos genitores e extrema fato de que, por vezes, a guarda é atribuída às mães devido ao
rejeição ao outro. A atual versão do manual diagnóstico, entendimento, ainda presente nos dias de hoje, de que estas
portanto, não teria descartado a possibilidade de existência de seriam portadoras de instinto materno, ou de que mulheres
formas de interação que seriam problemáticas e que poderiam seriam mais aptas e dedicadas nos cuidados dos filhos.
merecer atenção e intervenção clínica. Com isso, indaga-se Investigações revelam também que o divórcio pode ser
sobre a obstinação de alguns em defender a inclusão da vivido como um período de grande instabilidade na família,
denominada SAP ou alienação parental na próxima edição do levando adultos e crianças a se voltarem intensamente para as
DSM, bem como sobre as consequências que podem daí advir. relações parentais (Rapizo, Falcão, Costamilan, Scodro, &
O rótulo de síndrome ou enfermidade mental, em realidade, Moritz, 1998; Wallerstein & Kelly, 1996/1998). Em tais
pode ser uma forma de aprisionar os indivíduos em um circunstâncias, pode vir a se estabelecer uma forte aliança
diagnóstico, quando os seus comportamentos passam a ser entre o genitor guardião e os filhos, o que contribuiria para que
vistos exclusivamente como resultado de uma patologia. estes rejeitassem o outro pai e recusassem suas visitas
Entende-se que a diversidade e a complexidade dos (Wallerstein & Kelly, 1996/1998). Diante desse cenário, tais
comportamentos humanos não podem ser contidas pesquisas apontam, ainda, diferentes fatores que permeiam o
inteiramente na descrição de um transtorno ou doença. contexto da separação e que podem contribuir para o
No Brasil, verifica-se que a SAP não foi objeto de estudo da desenvolvimento das alianças parentais. Nota-se, dessa forma,
psiquiatria, haja vista a ausência de pesquisas e publicações que essas investigações não se fixam em aspectos psicológicos
científicas dessa área sobre o assunto (Sousa, 2010). individuais, como ocorre na teoria de Gardner sobre a SAP.
Possivelmente, isso ocorreu devido ao fato de esse ser um Outros trabalhos assinalam, também, que diferentes fatores
tema relativamente recente no país, difundido especialmente como idade, sexo, desenvolvimento cognitivo (Souza, 2000),
entre os profissionais que atuam nos juízos de família. De bem como os vínculos afetivos que crianças possuem com os
forma semelhante, não se identificam, no contexto nacional, pais, podem influenciar suas vivências e sentimentos acerca do
estudos na área da Psicologia que deem sustentação ao divórcio de seus responsáveis (Ramires, 2004).
conceito de SAP, ou ao de alienação parental, bem como a No Brasil, no entanto, os resultados de pesquisas sobre
programação ou lavagem cerebral de crianças como descreve separação conjugal – como os acima mencionados – parecem
a teoria de Gardner (1991, 2002b). ser desconsiderados quando o assunto é a SAP. Observa-se que
no país tem sido corrente o argumento de que a dita alienação
Destaca-se que a justificativa do projeto de lei nacional da criança seria motivada por sentimento de vingança por
sobre a matéria faz referência a aspectos emocionais e parte do genitor guardião (Ullmann, 2008), quando este, por
psicológicos encontrados em crianças que seriam vítimas da ter sido abandonado, traído, ou por razões diversas,
alienação parental, dispondo também sobre comportamentos desqualificaria o ex-cônjuge, além de impedir ou dificultar ao
e distúrbios psicológicos que a mesma acarretaria, ou seja, máximo a visitação da criança. Outros autores acrescentam a
comprometimentos à saúde mental na idade adulta. Quanto a possibilidade de o alienador ser portador de "moléstia mental
esses aspectos, verifica-se que tais proposições ou comportamental" (Lagrasta Neto, 2009, p. 39) ou ainda de
desconsideram estudos recentes, na área da Psicologia, sobre este se valer da prática de "tortura psicológica" (Barbosa,
crianças e jovens em famílias após o divórcio. Tais estudos 2010).
chamam a atenção para a diversidade de respostas no modo
como crianças e adolescentes vivenciam a separação dos pais Como já mencionado, estudos que discorrem sobre a
(Wallerstein & Kelly, 1996/1998; Brito, 2007) e para a formação de alianças ou de alinhamentos entre a criança e o
possibilidade de a guarda única contribuir com o guardião apontam os diversos motivos que contribuem para o
estreitamento de vínculos entre os filhos e o guardião, estabelecimento dessa situação, contrapondo-se à visão de
conduzindo ao afastamento daquele pai que não permaneceu que filhos de pais separados seriam portadores de transtornos
com a guarda. Como comprovam levantamentos nacionais ou de distúrbios psicológicos, como defendem alguns autores
(Instituto Brasileiro de Geografia e Estatística, 2008), as mães (Vallejo Orellana, Sanchez-Barranco Vallejo, Sanchez-
obtêm a guarda dos filhos na maior parte dos casos de divórcio Barranco Vallejo, 2004). Nesse sentido, vale lembrar o
e de disputas de guarda. Diante de desavenças entre os ex- entendimento de Hurstel (1999), quando a autora destaca que
parceiros e da atribuição de guarda unilateral, os filhos podem possíveis consequências da separação conjugal na vida dos
ter a convivência com o genitor não guardião restrita a visitas membros do grupo familiar devem ser percebidas na
esparsas, que acarretam, por vezes, o esgarçamento parcial ou interseção de fatores pessoais e sociais.
total da relação desse pai com a criança (Wallerstein, Lewis, & A partir dessa perspectiva, alguns países vêm solicitando
Blakeslee, 2000/2002). estudos aprofundados sobre possíveis desdobramentos da
separação conjugal para pais e filhos, objetivando maior
Brito (2008), em investigação que desenvolveu com filhos clareza a respeito de artigos a serem modificados nas
de pais separados, relata que, ao serem questionados a respectivas legislações para que seja possível assegurar a
respeito de como era o contato com o genitor que não convivência familiar entre pais e filhos após o divórcio.
permaneceu com a guarda, foi expressiva a parcela dos Mostra-se, dessa forma, compreensão sobre a necessidade de
entrevistados que considerou o contato insuficiente, com o ordenamento jurídico ser um fator de suporte ao exercício
prejuízos para o relacionamento. Nas entrevistas realizadas, a da paternidade e da maternidade. Nessa perspectiva, destaca-
autora observou que muitos filhos demonstravam não se se o estudo coordenado pela socióloga Irène Théry (1998) – a
sentir à vontade para abordar uma série de questões – como pedido do governo francês – que visava a avaliar se a legislação

Conhecimentos Específicos 135


Apostila Digital Licenciada para Alice Caroline Guarino dos Santos - alice.guarino@hotmail.com (Proibida a Revenda)
APOSTILAS OPÇÃO

daquele país estaria apropriada à realidade das famílias Retornando à justificação do PL nacional, não se
contemporâneas e, em caso contrário, quais seriam as encontram dados e/ou critérios que embasem o registro sobre
mudanças legislativas necessárias. No material elaborado a a proporção similar entre homens e mulheres alienadores,
partir de consulta e da colaboração de diversos profissionais, como também ocorre na teoria de Gardner. Diante disso, pode-
Théry chama a atenção para a crescente tendência de se indagar se a justificação teria objetivo semelhante ao
psicologização de questões que surgem no debate sobre as daquele autor, ou seja, o de evitar polêmicas, não chamando a
famílias contemporâneas, com interpretações carregadas de atenção para o assunto.
conteúdo moral e que desconsideram o contexto social, Outro item que se pode ressaltar na exposição de motivos
político, econômico e cultural que afetam as questões do Projeto de Lei é a importância dada à "definição legal da
familiares (1998, p. 20). alienação parental", ou "reconhecimento jurídico da conduta
alienação parental", ou ainda, "previsão legal do que seja
Missão semelhante à que foi endereçada a Théry recebeu a alienação parental ou síndrome de alienação parental".
professora de Direito da Universidade Lille II, Dekeuwer- Partindo do entendimento de que esses conceitos se referem a
Défossez (1999), responsável pela coordenação de um grupo aspectos emocionais e psicológicos – como mencionado
de trabalho que deveria apresentar propostas de alterações na inicialmente na justificação do texto legal –, entende-se que,
legislação do Direito de família francês, a partir de um com a criação da nova lei, diferentes comportamentos no
conjunto de reflexões e de questionamentos que lhes foram âmbito das relações familiares após o divórcio passam a ser
encaminhados. alocados sob a tipificação jurídica de alienação parental, sendo
Retomando a exposição de motivos que acompanhou o passíveis de reprimenda estatal, como se encontra na
projeto de lei nacional, encontra-se a afirmação de que este foi justificação, ou seja, em última análise, a família em litígio se
elaborado a partir de livro sobre a síndrome de alienação tornará objeto de controle e intervenção por parte do Estado,
parental editado por uma associação brasileira de pais e aos pais caberá não só se defender da acusação de alienação
separados, bem como de informações e textos traduzidos, parental como também comprovar sua sanidade, o que
disponíveis no site desta e de outras associações, e, ainda, de certamente contribuirá para fomentar disputas.
sugestões de membros participantes das mesmas. Não se
encontra, entretanto, qualquer menção aos diversos Como foi demonstrado na pesquisa desenvolvida por
questionamentos e polêmicas presentes na literatura Sousa (2010), apesar de a problemática sobre as alianças
internacional sobre o tema em apreço. Concebe-se que, no parentais não ser algo novo no cenário do rompimento
contexto nacional, a ausência dessas discussões sobre a teoria conjugal, o assunto só se tornou objeto de preocupação e
proposta por Gardner veio prejudicar o surgimento de intervenção por parte do Estado quando passou a ser
possíveis reflexões e debates sociais, contribuindo para que o percebido como expressão de algum distúrbio psicológico
assunto fosse difundido como verdade inconteste. relativo ao genitor guardião, o qual não mediria esforços no
sentido de afastar o(s) filho(s) do ex-parceiro. No entanto,
Observa-se que a justificação para a nova lei, ao se referir apesar da referência – por autores já citados – quanto ao
à denominada alienação parental como resultado de conduta suposto desequilíbrio emocional do genitor alienador, na
hostil por parte de um genitor e da manipulação que este exposição de motivos da nova lei brasileira, enfatiza-se a
exerceria sobre a criança, reduz a problemática que envolve as obrigação que o Judiciário teria de, comprovada a alienação,
relações parentais no divórcio a disposições pessoais, punir o alienador.
especialmente no que se refere ao genitor guardião. Cabe
salientar que, na visão de Gardner (1991), a alienação, em Chama a atenção, ainda, o fato de que a nova lei engendra
alguns casos, estaria integrada à estrutura psíquica do dito uma perspectiva de vitimação dos que integram famílias que
genitor alienador; assim, o desfecho do casamento, aliado a vivenciam situações de intenso conflito. Em outras palavras, o
disputas judiciais, poderia dar sequência à irrupção de guardião seria vítima de sua doença, a criança – que sofreria
transtornos psiquiátricos no mesmo. Diante disso, pode-se da síndrome de alienação parental – seria vítima do alienador,
pensar que, a partir da criação da nova lei brasileira sobre enquanto o pai alienado seria vítima da situação e do
alienação parental, não só as crianças, mas também os afastamento que lhe fora imposto. Tal constatação remete à
genitores guardiães, ou seja, as mães, na maioria dos casos, citação de Birman (2010, p. 28) de que "a vitimação, inclusive
passam a ser percebidas como possíveis portadoras de nos seus desdobramentos jurídicos (Eliacheff & Soulez, 2007),
distúrbios psicológicos, como descrevem os autores nacionais se transformou numa das marcas da sociedade
citados anteriormente. contemporânea, (...)". Com isso, compreende-se que os
componentes do grupo familiar tanto se veem quanto são
Merece também destaque o seguinte trecho da justificação percebidos no campo social como vítimas, desconsiderando-se
do PL: "a proporção de homens e mulheres que induzem a possibilidade de um exame aprofundado acerca das
distúrbios psicológicos relacionados à alienação parental nos implicações e das responsabilidades em sua história familiar.
filhos tende atualmente ao equilíbrio"( ). Tal informação faz Além das questões que se encontram na exposição de
lembrar o que registrou Gardner (2001) sobre a porcentagem motivos, a nova lei, no §2º do art. 5º, dispõe sobre a atuação de
de homens e mulheres que teriam compelido os filhos a SAP, profissionais que compõem as equipes que assessoram os
nos casos por ele avaliados. Inicialmente, esse autor assegurou juízos, exigindo aptidão profissional ou acadêmica
que a síndrome ocorreria especialmente devido ao comprovada "para diagnosticar atos de alienação parental", o
comportamento materno, tendo constatado que, em mais de que sugere a existência de um especialista em SAP ou
80% dos casos por ele examinados, as mães seriam as alienação parental. Na verdade, parece se esperar que, por
alienadoras. (Gardner, 2001). Após receber uma série de meio de perícias, os profissionais de Psicologia associem
críticas por parte do movimento de mulheres nos Estados atitudes e conflitos relacionais observados aos sintomas ou
Unidos, o autor passou a afirmar que a proporção de pais e comportamentos que compõem a lista de situações que seriam
mães alienadores seria de 50% (Gardner, 2002a). Interessa identificadas como SAP ou alienação parental. Entende-se,
notar que essa foi uma estratégia bastante utilizada pelo dessa forma, porque alguns reiteram observar, com
psiquiatra norte-americano, ou seja, a alteração de suas frequência, a existência da SAP em seus atendimentos. Não se
proposições frente às críticas recebidas, pois, como ele mesmo pode esquecer que os discursos produzidos por profissionais
reconhecia polêmicas e censuras que envolvessem a SAP detêm status de ciência, e, portanto, valor de verdade acerca
poderiam dificultar a inclusão desta no DSM-V (Sousa, 2010). dos indivíduos avaliados.

Conhecimentos Específicos 136


Apostila Digital Licenciada para Alice Caroline Guarino dos Santos - alice.guarino@hotmail.com (Proibida a Revenda)
APOSTILAS OPÇÃO

Direito, ao equipararem a SAP à tortura psicológica,


A lei sobre a alienação, além de exigir especialistas no interpretam que
assunto, traz também determinações quanto à elaboração de
laudo pericial, afirmando, no §1º do art. 5º, que este deverá se A prisão do recalcitrante (em práticas de alienação) não
basear, dentre outras coisas, "em exame de documentos dos está impedida pelos princípios constitucionais (...), uma vez que
autos, histórico do relacionamento do casal e da separação, existe previsão de punição àquele que sob qualquer pretexto ou
cronologia de incidentes, avaliação da personalidade dos utilizando-se de quaisquer meios promova a tortura e suas
envolvidos e exame da forma como a criança ou adolescente se respectivas sequelas (Lagrasta Neto, 2009, p. 48).
manifesta acerca de eventual acusação contra genitor". No que
diz respeito à atuação de profissionais psicólogos, esse texto Apesar de a lei sobre alienação parental não prever sanção
legal, ao que parece, estaria confundindo a prática de penal, a interpretação de que o alienador cometeria tortura
psicólogos com a de advogados ou mesmo com a de psicológica poderia dar ensejo à aplicação de penalidades.
investigadores, divergindo, claramente, das diretrizes
emitidas pelo Conselho Federal de Psicologia (CFP). Cabe Medidas judiciais e o diagnóstico de SAP
lembrar que, por meio da Resolução nº 007/2003, o CFP
especifica critérios e orienta psicólogos sobre a elaboração de Medidas punitivas com relação à denominada síndrome de
laudos e pareceres. Nesse rumo, essa Resolução destaca que o alienação parental ou à alienação parental semelhantes às
psicólogo deve levar em conta os condicionantes históricos e previstas na lei brasileira vêm sendo adotadas por Tribunais
sociais nas avaliações realizadas, bem como basear suas de Justiça em outros países. Na Espanha, por exemplo, no ano
informações na observância dos princípios e dispositivos do 2007, teve repercussão na mídia daquele país (Elmundo,
Código de Ética Profissional do Psicólogo (2005). Consta 2007) processo judicial no qual uma juíza retirou a guarda da
também da Resolução nº 007/2003 indicação de que os mãe de uma menina de oito anos, concedendo-a em favor do
psicólogos, ao produzirem documentos escritos, devem se pai, ao mesmo tempo em que proibiu qualquer forma de
basear exclusivamente (grifo nosso) nos instrumentais contato entre a criança e a genitora, bem como da menina com
técnicos (entrevistas, testes, observações, dinâmicas de grupo, a família materna, durante o período de seis meses. De acordo
escuta, intervenções verbais) que se configuram como com notícias veiculadas na época, a medida teria sido adotada
métodos e técnicas psicológicas para a coleta de dados (...). a partir de informes periciais que indicavam que a criança
seria vítima da síndrome de alienação parental, empreendida
Nesse sentido, estranha-se a postura do legislador ao pela mãe, com o objetivo de afastá-la do pai. Na sentença
determinar, na lei sobre a alienação parental, que a perícia judicial, houve determinação que a menina deveria receber
psicológica se fundamente, dentre outros aspectos, em exame assistência psicológica por parte dos especialistas que fariam
de documentos dos autos. Questiona-se, assim, se, na avaliação o acompanhamento e a avaliação da síndrome.
psicológica, o profissional deveria basear-se em informações e Outro caso que ganhou vulto nos meios de comunicação,
dados coletados e interpretados por outras áreas de em meados do ano 2009, ocorreu em Portugal, onde um juiz
conhecimento em contextos que, por vezes, o psicólogo decidiu pela entrega da guarda provisória de uma menina de
desconhece. Nota-se ainda que, ao seguir o modelo de trabalho sete anos a uma instituição de acolhimento. À semelhança do
disposto na referida lei, o psicólogo corre sério risco de deixar caso espanhol citado acima, a sentença judicial portuguesa foi
de lado as determinações históricas, sociais, econômicas e proferida com base em parecer técnico que concluiu que a
políticas que estariam contribuindo, ao longo do tempo, com o pequena estava sendo vítima da SAP. Acrescenta-se que, no
afastamento da criança em relação a um dos genitores. caso português, o magistrado teria determinado, além da
institucionalização da criança, o impedimento de qualquer
A nova lei estabelece no art. 6º que, identificada a alienação forma de contato entre esta e ambos os genitores (Correio da
parental, diferentes medidas podem ser imputadas aos Manhã, 2010).
denominados genitores alienadores, sem prejuízo da Em Portugal, também veio a público, por meio de matéria
decorrente responsabilidade civil ou criminal. Dentre as veiculada no jornal Diário de Notícias Portugal (2010), a
medidas listadas, destacam-se a fixação cautelar do domicílio preocupação de juízes com relação às disputas entre pais
da criança ou adolescente, o pagamento de multas, a inversão separados, que vêm sendo percebidas como casos de síndrome
da guarda, a determinação da guarda compartilhada e a de alienação parental. Alguns magistrados admitem não ser
suspensão da autoridade parental, dentre outras. Em relação simples proferir decisões judiciais acerca de tais situações,
às sanções que podem ser aplicadas ao chamado pai alienador, devido às dificuldades em averiguar quem estaria dizendo a
por vezes se tem a impressão de que a criança acaba sendo verdade: a mãe, o pai ou a criança. Como concluiu um juiz: "Isso
relegada a segundo plano, quando a preocupação parece porque os mecanismos de segurança que o tribunal tem de
voltada para a medida exemplar que será determinada para aplicar podem apenas servir para penalizar as crianças e os
um dos genitores. Não se pode desconsiderar que, em casos pais com um afastamento desnecessário (...)".
nos quais haja forte ligação com um dos genitores, a decisão de
inverter a guarda, ou de proibir esse genitor de ver a criança No Brasil, no entanto, alguns acreditam que, com a recém-
durante período de tempo estipulado em sentença judicial, ou criada lei, não haverá dificuldades para se identificar supostos
mesmo de lhe retirar o poder familiar, pode trazer intensos casos de alienação parental e se aplicar sentenças judiciais,
sofrimentos para a criança. como exalta Cherulli (2010) no artigo Lei da alienação
A lista de medidas que podem ser adotadas parece sugerir parental: tão simples assim. Tal entendimento se justifica pelo
que, agora, o Estado é quem possui o direito de alienar um dos fato de a nova lei listar, no parágrafo único do art. 2º, formas
pais da vida da criança. Nesse sentido, questiona-se se teorias de alienação parental, como, por exemplo, "realizar campanha
psicológicas dariam respaldo a tais medidas. Estar-se-ia de desqualificação da conduta do genitor no exercício da
desconsiderando os prejuízos emocionais causados à criança, paternidade ou maternidade". Além dos comportamentos
que bruscamente será afastada do genitor com quem convive exemplificados no artigo 2°, outros atos vistos como práticas
e com quem mantém fortes ligações? de alienação parental podem ser declarados pelo juiz bem
como podem ser constatados por peritos. Em outras palavras,
Por último, cabe assinalar que, apesar do veto presidencial com a nova lei, uma gama de comportamentos exibidos em
ao art.10 do Projeto de Lei, que previa sanção penal ao genitor meio a conflitos familiares pode ser identificada como indício
que apresentasse falsas denúncias, alguns operadores do da conjecturada alienação parental. Sem dúvida, tal avaliação

Conhecimentos Específicos 137


Apostila Digital Licenciada para Alice Caroline Guarino dos Santos - alice.guarino@hotmail.com (Proibida a Revenda)
APOSTILAS OPÇÃO

pode levar a um aumento exponencial de sentenças judiciais guarda compartilhada ou sua inversão (...) (Cherulli, 2010,
que preveem medidas punitivas contra genitores em litígio grifo nosso).
conjugal e, em última análise, ao sofrimento dos filhos.
No cenário nacional, disputas pela guarda de filhos de pais Nota-se que o espírito da lei da guarda compartilhada visa,
separados com frequência são noticiadas pela mídia devido dentre outros aspectos, a alterar o entendimento – que vigorou
aos inusitados, e por vezes trágicos, rumos e desfechos que se por muito tempo – de que a guarda deveria ser
observam. Apesar da recente promulgação da lei sobre preferencialmente materna. Dessa maneira, em face da
alienação parental, já se identificam não só sentenças judiciais compreensão atual sobre a importância de a criança manter
como também jurisprudência baseadas em processos uma convivência com ambos os responsáveis e suas
litigiosos avaliados como situações de SAP (Barbosa, 2010). respectivas famílias, a guarda deve ser prioritariamente
Em julho de 2010, alcançou grande destaque na mídia o caso compartilhada. A lei da guarda compartilhada possibilita,
de uma menina de 5 anos de idade que teve a guarda invertida portanto, um apoio legal para a manutenção dos vínculos entre
em favor do pai, ao mesmo tempo em que foi impedido pais e filhos após uma separação conjugal, distanciando-se da
qualquer contato entre mãe e filha pelo período de noventa ideia de um dispositivo punitivo como parece sugerir a lei
dias. Embora fosse uma criança saudável, como garantiu seu sobre alienação parental. Com a lei da guarda compartilhada,
pediatra, a menina veio a falecer após sucessivas internações desenha-se outra coerência para a manutenção do
hospitalares ao longo do primeiro mês em que esteve sob a relacionamento entre pais e filhos após uma separação
guarda do pai. Ao serem identificados ferimentos e luxações no conjugal, construindo-se, também, uma ancoragem social para
corpo da criança, foi levantada a suspeita de maus-tratos por que pais e mães mantenham seus respectivos lugares junto aos
parte do pai guardião (Lima, 2010). filhos.
Não se pode deixar de assinalar que a tramitação da lei
Conforme matéria publicada em revista de grande sobre guarda compartilhada foi acompanhada por diversos
circulação nacional (Lobato, 2010), a juíza responsável pelo eventos, debates e matérias divulgadas pela mídia acerca do
caso teria baseado sua decisão em laudo psicológico, o qual exercício da maternidade e da paternidade nos dias de hoje,
concluíra que a criança estaria sendo vítima de alienação que contribuíram com o amadurecimento da concepção de que
parental. divórcios e separações, quando ocorrem, se dão no contexto da
O caso relatado permaneceu sob os holofotes da mídia por conjugalidade, e não em referência à parentalidade. Na época,
alguns dias, não faltando declarações, indagações e também ocorreu uma série de questionamentos sobre as
questionamentos a respeito de quem seria culpado pelo que atribuições que caberiam aos psicólogos com a nova
ocorreu com a criança. A culpa seria dos pais, por não modalidade de guarda. A guarda compartilhada requer que se
priorizarem o entendimento mútuo a fim de preservar a filha? deixe de lado a procura do genitor que apresente melhores
Seria do guardião, por ter cometido violência contra a criança? condições para deter a guarda dos filhos, como se encontrava
Do suposto médico que atendeu a menina? A responsabilidade disposto no artigo 1.584 do Código Civil Brasileiro de 2002,
seria dos psicólogos que avaliaram a situação como um caso para se pensar em uma atuação que auxilie aos pais no
inequívoco da chamada SAP? Seria do juízo que sentenciou a compartilhamento da guarda de filhos comuns. Configura-se
inversão da guarda e o impedimento de qualquer contato para o psicólogo, portanto, um distanciamento do papel de
entre mãe e filha pelo período de 90 dias? Seria de todos ou de avaliador ou daquele que apontaria o melhor genitor,
ninguém? Diante desse cenário, resta indagar, tendo a criança buscando-se uma aproximação com a figura de um
realmente sofrido agressões, a quem deveria ter se queixado, profissional que irá facilitar, colaborar para que os pais
já que seu comportamento e suas queixas poderiam ser entendam a importância de os dois estarem presentes e
classificadas como sintomas de SAP? participativos na educação dos filhos.
Entretanto, no contexto nacional, o clamor por punição dos
Preocupados com as consequências de sentenças como as denominados genitores alienadores logo fez desvanecer essa
relatadas acima, profissionais criaram, na Espanha, um grupo expectativa quanto à atuação dos psicólogos nos juízos de
de trabalho junto ao Observatório contra la Violência família, realocando-se esses profissionais no lugar daqueles
Doméstica y de Género, que funciona no Consejo General del que devem privilegiar a elaboração de avaliações, visando,
Poder Judicial (CGPJ), com o objetivo de estudar o tema SAP. agora, à apuração da existência, ou não, da alienação parental.
Com isso, em 2008, o CGPJ publicou o Guía de Criterios de Nesse rumo, novos indicadores vão sendo apontados para
Actuación Judicial frente a la Violência de Género, que guiar essa atuação que, como dispõe a lei sobre alienação
recomenda aos magistrados não utilizarem o tema SAP em parental, invariavelmente fornecerão elementos para as
suas sentenças (Consejo General del Poder Judicial, 2008). medidas e punições a serem aplicadas ao identificado
Polêmicas que envolvem o assunto motivaram, ainda, a alienador. Assim, dá ênfase na participação de ambos os pais
elaboração, no ano 2010, de declaração por parte da na educação dos filhos, independentemente de sua situação
Asociación Española de Neuropsiquiatria- Profesionales de conjugal, fixam-se as lentes no suposto perigo que um dos pais
Salud Mental repudiando, de forma terminante, o uso clínico e pode representar para a criança.
legal da denominada síndrome de alienação parental.
É digno de destaque também que, apesar de a lei brasileira Questões
relativa à alienação parental dispor sobre a possibilidade de se
determinar a guarda compartilhada, essa é uma aplicação com 01. De acordo com o Estatuto do Idoso (Lei n. 10.471/03):
propósito que soa como diferenciado do que se pretendia (A) O Ministério Público tem legitimidade para a
contemplar na lei promulgada em 2008, que instituiu essa promoção da tutela coletiva dos direitos de pessoas com idade
modalidade de guarda como de uso preferencial. Em certos igual ou superior a sessenta anos, mas não poderá atuar na
textos que vêm sendo publicados sobre o tema, os autores esfera individual de direitos dessa parcela da população, uma
sublinham que os comportamentos listados no art. 6º da lei vez que a senilidade não induz incapacidade para os atos da
sobre a alienação parental passaram a ser práticas passíveis vida civil.
com as seguintes punições: declarar a ocorrência de alienação (B) O idoso, que necessite de alimentos, deverá acionar
parental e advertir o alienador; ampliar o regime de simultaneamente os filhos, cobrando de cada qual, na medida
convivência familiar em favor do genitor alienado; estipular de suas possibilidades.
multa ao alienador; determinar acompanhamento psicológico (C) O Poder Judiciário, a requerimento do Ministério
e/ou biopsicossocial; determinar a alteração da guarda para Público, poderá determinar medidas protetivas em favor de

Conhecimentos Específicos 138


Apostila Digital Licenciada para Alice Caroline Guarino dos Santos - alice.guarino@hotmail.com (Proibida a Revenda)
APOSTILAS OPÇÃO

idoso em situação de risco, tais como: requisição de (A) O direito à liberdade compreende o aspecto de ir, vir e
tratamento de saúde, em regime ambulatorial, hospitalar ou estar nos logradouros públicos e espaços comunitários,
domiciliar; encaminhamento à família ou curador, mediante ressalvadas as restrições legais.
termo de responsabilidade; abrigamento em entidade. (B) O direito ao respeito consiste na inviolabilidade da
(D) O Poder Público tem responsabilidade residual e, no integridade física, psíquica e moral da criança e do
âmbito da assistência social, estará obrigado a assegurar os adolescente, abrangendo a preservação da imagem, da
direitos fundamentais de pessoa idosa, em caso de inexistência identidade, da autonomia, dos valores, ideias e crenças, dos
de parentes na linha reta ou colateral até o 3º grau. espaços e objetos pessoais.
(C) É dever exclusivo do Estado velar pela dignidade da
02. Com a edição da Lei n. 12.962/14, a alterar o art. 23 da criança e do adolescente, pondo-os a salvo de qualquer
Lei n. 8.069/90, a condenação criminal do pai ou da mãe traz tratamento desumano, violento, aterrorizante, vexatório ou
como efeito específico a destituição do poder familiar se constrangedor.
aplicada pena privativa de liberdade, detenção ou reclusão, (D) A criança e o adolescente têm o direito de ser educado
por delito doloso contra o próprio filho ou filha. e cuidado sem o uso de castigo físico ou de tratamento cruel ou
degradante, como formas de correção, disciplina, educação ou
( ) Certo ( ) Errado qualquer outro pretexto, pelos pais, pelos integrantes da
família ampliada, pelos responsáveis, pelos agentes públicos
03. Conforme a Lei nº 8.069/1990, que dispõe sobre o executores de medidas socioeducativas ou por qualquer
Estatuto da Criança e do Adolescente, assinale a alternativa pessoa encarregada de cuidar deles, tratá-los, educá-los ou
incorreta. protegê-los
(A) É assegurado à gestante, através do acompanhamento (E) A criança e o adolescente têm direito à liberdade, ao
médico, o atendimento pré e perinatal. respeito e à dignidade como pessoas humanas em processo de
(B) A gestante será encaminhada aos diferentes níveis de desenvolvimento e como sujeitos de direitos civis, humanos e
atendimento, segundo critérios médicos específicos, sociais garantidos na Constituição e nas leis.
obedecendo-se aos princípios de regionalização e
hierarquização do Sistema. 08. Para o Estatuto da Criança e Adolescente considera-se
(C) A parturiente será atendida preferencialmente pelo criança a pessoa até:
mesmo médico que a acompanhou na fase pré-natal. (A) Doze anos de idade incompletos.
(D) Incumbe ao poder público propiciar apoio alimentar à (B) Quatorze anos de idade incompletos.
gestante e à nutriz que dele necessitem. (C) Oito anos de idade incompletos.
(D) Dez anos de idade incompletos.
04. Nos termos preconizados pela Lei no 8.069, de 13 de
julho de 1990, a criança e o adolescente têm direito à 09. Dispõe o Estatuto da Criança e do Adolescente acerca
liberdade, ao respeito e à dignidade como pessoas humanas de viagem de criança ou adolescente para o exterior:
em processo de desenvolvimento e como sujeitos de direitos (A) o detentor da guarda poderá opor-se a saída de criança
civis, humanos e sociais garantidos na Constituição e nas leis. ou adolescente na companhia de seus pais.
E, ainda, estabelece que o direito ao respeito consiste (B) a autorização para viagem internacional concedida por
(A) em buscar refúgio, auxílio e orientação, bem como ambos os pais é exigível para criança acompanhada por
crença e culto religioso. terceiros e dispensável em caso de viagem de adolescente
(B) na inviolabilidade da integridade física, psíquica e sozinho ao exterior.
moral da criança e do adolescente. (C) a saída de criança ou de adolescente nascido no Brasil,
(C) na participação da vida política, na forma da lei, como na companhia de estrangeiro residente ou domiciliado no
também da vida familiar e comunitária, sem discriminação. exterior, requer declaração de ambos os pais com firma
(D) em ir, vir e estar nos logradouros públicos e espaços reconhecida.
comunitários, ressalvadas as restrições legais. (D) requer-se autorização judicial para criança ou
(E) em ser criado e educado no seio de sua família e, adolescente viajar ao exterior apenas quando for realizada na
excepcionalmente, em família substituta. companhia do responsável.

05. De acordo com o Estatuto da Criança e do Adolescente, 10. Considerando o Estatuto da Criança e do Adolescente,
considera-se tratamento cruel ou degradante dispensado à assinale a alternativa correta.
criança aquele que a (A) Os filhos, havidos ou não da relação do casamento, ou
(A) submete ao aleitamento materno no interior de por adoção, não terão os mesmos direitos e qualificações.
presídio onde a mãe cumpre pena. (B) O poder familiar será exercido apenas pelo pai, na
(B) submeta a tratamento a toxicômanos. forma do que dispuser a legislação civil.
(C) proporcione castigo e sofrimento físico desnecessário. (C) A falta ou a carência de recursos materiais não constitui
(D) humilhe, ameace gravemente ou a ridicularize. motivo suficiente para a perda ou a suspensão do poder
(E) prive da frequência ao ensino fundamental. familiar.
(D) A guarda confere à criança ou adolescente a condição
06. Como projeção do direito à liberdade, a criança e o de dependente apenas para fins de imposto de renda.
adolescente não têm direito a
(A) opinião e expressão Resposta
(B) crença e culto religioso.
(C) brincadeiras, prática de esportes e diversões. 01. C / 02. Errado / 03. A / 04. B / 05. D /06. E / 07. C /
(D) participação na vida familiar e comunitária, sem 08. A / 09. A / 10. C
discriminação.
(E) inscrição em partido político.

07. Tendo por base o Estatuto da Criança e do Adolescente


(ECA), no que concerne ao Direito à Liberdade, ao Respeito e à
Dignidade, assinale a opção incorreta:

Conhecimentos Específicos 139


Apostila Digital Licenciada para Alice Caroline Guarino dos Santos - alice.guarino@hotmail.com (Proibida a Revenda)
APOSTILAS OPÇÃO

Anotações

Conhecimentos Específicos 140


Apostila Digital Licenciada para Alice Caroline Guarino dos Santos - alice.guarino@hotmail.com (Proibida a Revenda)
LEGISLAÇÃO

Apostila Digital Licenciada para Alice Caroline Guarino dos Santos - alice.guarino@hotmail.com (Proibida a Revenda)
Apostila Digital Licenciada para Alice Caroline Guarino dos Santos - alice.guarino@hotmail.com (Proibida a Revenda)
APOSTILAS OPÇÃO

IV - é livre a manifestação do pensamento, sendo vedado o


anonimato;
V - é assegurado o direito de resposta, proporcional ao
agravo, além da indenização por dano material, moral ou à
imagem;
VI - é inviolável a liberdade de consciência e de crença,
sendo assegurado o livre exercício dos cultos religiosos e
garantida, na forma da lei, a proteção aos locais de culto e a
suas liturgias;
VII - é assegurada, nos termos da lei, a prestação de
assistência religiosa nas entidades civis e militares de
CONSTITUIÇÃO DA REPÚBLICA internação coletiva;
FEDERATIVA DO BRASIL TÍTULO VIII - ninguém será privado de direitos por motivo de
II Dos Direitos e Garantias crença religiosa ou de convicção filosófica ou política, salvo se
Fundamentais CAPÍTULO I Dos as invocar para eximir-se de obrigação legal a todos imposta e
recusar-se a cumprir prestação alternativa, fixada em lei;
Direitos e Deveres Individuais e
IX - é livre a expressão da atividade intelectual, artística,
Coletivos CAPÍTULO II Dos científica e de comunicação, independentemente de censura
Direitos Sociais TÍTULO VIII Do ou licença;
Ordem Social CAPÍTULO II Da X - são invioláveis a intimidade, a vida privada, a honra e a
Seguridade Social CAPÍTULO VII imagem das pessoas, assegurado o direito a indenização pelo
dano material ou moral decorrente de sua violação;
Da Família, da Criança, do
Adolescente, do Jovem e do
Súmula 403 do STJ: “Independe de prova do prejuízo a
Idoso indenização pela publicação não autorizada da imagem de
pessoa com fins econômicos ou comerciais”.

TÍTULO II Dos Direitos e Garantias Fundamentais XI - a casa é asilo inviolável do indivíduo, ninguém nela
CAPÍTULO I Dos Direitos e Deveres Individuais e Coletivos podendo penetrar sem consentimento do morador, salvo em
CAPÍTULO II Dos Direitos Sociais caso de flagrante delito ou desastre, ou para prestar socorro,
ou, durante o dia, por determinação judicial;
Dos Direitos e Garantias Fundamentais
Durante o dia Durante a noite
A Constituição Federal de 1988 trouxe em seu Título II os
direitos e garantias fundamentais, subdividindo-os em cinco Consentimento do Consentimento do
capítulos: direitos individuais e coletivos, direitos sociais, morador morador
nacionalidade, direitos políticos e partidos políticos.
Caso de flagrante delito Caso de flagrante delito
É necessária maior atenção na leitura do artigo 5º, Desastre ou prestar Desastre ou prestar
tendo em vista que este é um dos mais exigidos em socorro socorro
concurso público!
Determinação judicial --

Dos Direitos e Deveres Individuais e Coletivos


XII - é inviolável o sigilo da correspondência e das
comunicações telegráficas, de dados e das comunicações
A Constituição de 1988 foi a primeira a estabelecer direitos
telefônicas, salvo, no último caso, por ordem judicial, nas
não só de indivíduos, mas também de grupos sociais, os
hipóteses e na forma que a lei estabelecer para fins de
denominados direitos coletivos. As pessoas passaram a ser
investigação criminal ou instrução processual penal; (Vide Lei
coletivamente consideradas. Por outro lado, pela primeira vez,
nº 9.296, de 1996).
junto com direitos foram estabelecidos expressamente
deveres fundamentais. Tanto os agentes públicos como os
indivíduos têm obrigações específicas, inclusive a de respeitar A interceptação só pode ocorrer com ordem judicial,
os direitos das demais pessoas que vivem na ordem social. para fins de investigação criminal ou instrução processual
penal, sob pena de constituir prova ilícita.
TÍTULO II
DOS DIREITOS E GARANTIAS FUNDAMENTAIS XIII - é livre o exercício de qualquer trabalho, ofício ou
CAPÍTULO I profissão, atendidas as qualificações profissionais que a lei
DOS DIREITOS E DEVERES INDIVIDUAIS E COLETIVOS estabelecer;

Art. 5º Todos são iguais perante a lei, sem distinção de Um exemplo muito utilizado pela doutrina para explicar
qualquer natureza, garantindo-se aos brasileiros e aos esse inciso é o do Exame aplicado pela Ordem dos
estrangeiros residentes no País a inviolabilidade do direito à Advogados do Brasil aos bacharéis em Direito, para que
vida, à liberdade, à igualdade, à segurança e à propriedade, nos estes obtenham habilitação para exercer a profissão de
termos seguintes: advogados. Como é notório, a lei garante a liberdade de
I - homens e mulheres são iguais em direitos e obrigações, trabalho, sendo, no entanto, que a lei posterior, ou seja, o
nos termos desta Constituição; Estatuto da OAB, prevê a realização do exame para que seja
II - ninguém será obrigado a fazer ou deixar de fazer possível o exercício da profissão de advogado.
alguma coisa senão em virtude de lei;
III - ninguém será submetido a tortura nem a tratamento
desumano ou degradante;

Legislação 1
Apostila Digital Licenciada para Alice Caroline Guarino dos Santos - alice.guarino@hotmail.com (Proibida a Revenda)
APOSTILAS OPÇÃO

XIV - é assegurado a todos o acesso à informação e aos intérpretes e às respectivas representações sindicais e
resguardado o sigilo da fonte, quando necessário ao exercício associativas;
profissional; XXIX - a lei assegurará aos autores de inventos industriais
XV - é livre a locomoção no território nacional em tempo privilégio temporário para sua utilização, bem como proteção
de paz, podendo qualquer pessoa, nos termos da lei, nele às criações industriais, à propriedade das marcas, aos nomes
entrar, permanecer ou dele sair com seus bens; de empresas e a outros signos distintivos, tendo em vista o
XVI - todos podem reunir-se pacificamente, sem armas, interesse social e o desenvolvimento tecnológico e econômico
em locais abertos ao público, independentemente de do País;
autorização, desde que não frustrem outra reunião XXX - é garantido o direito de herança;
anteriormente convocada para o mesmo local, sendo apenas XXXI - a sucessão de bens de estrangeiros situados no País
exigido prévio aviso à autoridade competente; será regulada pela lei brasileira em benefício do cônjuge ou
XVII - é plena a liberdade de associação para fins lícitos, dos filhos brasileiros, sempre que não lhes seja mais favorável
vedada a de caráter paramilitar; a lei pessoal do "de cujus";
XVIII - a criação de associações e, na forma da lei, a de XXXII - o Estado promoverá, na forma da lei, a defesa do
cooperativas independem de autorização, sendo vedada a consumidor;
interferência estatal em seu funcionamento; XXXIII - todos têm direito a receber dos órgãos públicos
XIX - as associações só poderão ser compulsoriamente informações de seu interesse particular, ou de interesse
dissolvidas ou ter suas atividades suspensas por decisão coletivo ou geral, que serão prestadas no prazo da lei, sob pena
judicial, exigindo-se, no primeiro caso, o trânsito em julgado; de responsabilidade, ressalvadas aquelas cujo sigilo seja
XX - ninguém poderá ser compelido a associar-se ou a imprescindível à segurança da sociedade e do
permanecer associado; Estado; (Regulamento) (Vide Lei nº 12.527, de 2011).
XXI - as entidades associativas, quando expressamente XXXIV - são a todos assegurados, independentemente do
autorizadas, têm legitimidade para representar seus filiados pagamento de taxas:
judicial ou extrajudicialmente; a) o direito de petição aos Poderes Públicos em defesa de
XXII - é garantido o direito de propriedade; direitos ou contra ilegalidade ou abuso de poder;
XXIII - a propriedade atenderá a sua função social; b) a obtenção de certidões em repartições públicas, para
XXIV - a lei estabelecerá o procedimento para defesa de direitos e esclarecimento de situações de interesse
desapropriação por necessidade ou utilidade pública, ou por pessoal;
interesse social, mediante justa e prévia indenização em XXXV - a lei não excluirá da apreciação do Poder Judiciário
dinheiro, ressalvados os casos previstos nesta Constituição; lesão ou ameaça a direito;
XXXVI - a lei não prejudicará o direito adquirido, o ato
Desde que sejam obedecidos alguns requisitos o jurídico perfeito e a coisa julgada;
proprietário poderá ter subtraída a coisa de sua
propriedade. São eles: - Direito adquirido: Direito que o seu titular, ou alguém
- Necessidade pública; por ele, possa exercer, como aqueles cujo começo do
- Utilidade pública; exercício tenha termo prefixo ou condição preestabelecida
- Interesse social; inalterável, a arbítrio de outrem;
- Justa e prévia indenização; e - Ato jurídico perfeito: Ato já consumado segundo a lei
- Indenização em dinheiro. vigente ao tempo em que se efetuou;
- Coisa julgada: Decisão judicial de que não caiba mais
XXV - no caso de iminente perigo público, a autoridade recurso.
competente poderá usar de propriedade particular,
assegurada ao proprietário indenização ulterior, se houver
XXXVII - não haverá juízo ou tribunal de exceção;
dano;
XXXVIII - é reconhecida a instituição do júri, com a
XXVI - a pequena propriedade rural, assim definida em lei,
organização que lhe der a lei, assegurados:
desde que trabalhada pela família, não será objeto de
a) a plenitude de defesa;
penhora para pagamento de débitos decorrentes de sua
b) o sigilo das votações;
atividade produtiva, dispondo a lei sobre os meios de
c) a soberania dos veredictos;
financiar o seu desenvolvimento;
d) a competência para o julgamento dos crimes dolosos
contra a vida;
Súmula 364-STJ: O conceito de impenhorabilidade de XXXIX - não há crime sem lei anterior que o defina, nem
bem de família abrange também o imóvel pertencente a pena sem prévia cominação legal;
pessoas solteiras, separadas e viúvas. XL - a lei penal não retroagirá, salvo para beneficiar o réu;

Súmula 486-STJ: É impenhorável o único imóvel A lei penal produz efeitos a partir de sua entrada em
residencial do devedor que esteja locado a terceiros, desde vigor, não se admitindo sua retroatividade maléfica. Não
que a renda obtida com a locação seja revertida para a pode retroagir, salvo se beneficiar o réu.
subsistência ou a moradia da sua família.
XLI - a lei punirá qualquer discriminação atentatória dos
XXVII - aos autores pertence o direito exclusivo de direitos e liberdades fundamentais;
utilização, publicação ou reprodução de suas obras, XLII - a prática do racismo constitui crime inafiançável e
transmissível aos herdeiros pelo tempo que a lei fixar; imprescritível, sujeito à pena de reclusão, nos termos da lei;
XXVIII - são assegurados, nos termos da lei: XLIII - a lei considerará crimes inafiançáveis e insuscetíveis
a) a proteção às participações individuais em obras de graça ou anistia a prática da tortura, o tráfico ilícito de
coletivas e à reprodução da imagem e voz humanas, inclusive entorpecentes e drogas afins, o terrorismo e os definidos como
nas atividades desportivas; crimes hediondos, por eles respondendo os mandantes, os
b) o direito de fiscalização do aproveitamento econômico executores e os que, podendo evitá-los, se omitirem;
das obras que criarem ou de que participarem aos criadores,

Legislação 2
Apostila Digital Licenciada para Alice Caroline Guarino dos Santos - alice.guarino@hotmail.com (Proibida a Revenda)
APOSTILAS OPÇÃO

XLIV - constitui crime inafiançável e imprescritível a ação delito ou por ordem fundamentada do juiz (preventiva ou
de grupos armados, civis ou militares, contra a ordem temporária).
constitucional e o Estado Democrático;
XLV - nenhuma pena passará da pessoa do condenado, LVIII - o civilmente identificado não será submetido a
podendo a obrigação de reparar o dano e a decretação do identificação criminal, salvo nas hipóteses previstas em lei;
perdimento de bens ser, nos termos da lei, estendidas aos
sucessores e contra eles executadas, até o limite do valor do Atualmente, a Lei nº 12.037/2009, traz em seu artigo
patrimônio transferido; 3º, as hipóteses em que o civilmente identificado deverá
XLVI - a lei regulará a individualização da pena e adotará, proceder à identificação criminal. São elas:
entre outras, as seguintes: – o documento apresentar rasura ou tiver indício de
a) privação ou restrição da liberdade; falsificação;
b) perda de bens; – o documento apresentado for insuficiente para
c) multa; identificar cabalmente o indiciado;
d) prestação social alternativa; – o indiciado portar documentos de identidade
e) suspensão ou interdição de direitos; distintos, com informações conflitantes entre si;
XLVII - não haverá penas: – a identificação criminal for essencial às investigações
a) de morte, salvo em caso de guerra declarada, nos termos policiais, segundo despacho da autoridade judiciária
do art. 84, XIX; competente, que decidirá de ofício ou mediante
b) de caráter perpétuo; representação da autoridade policial, do Ministério Público
c) de trabalhos forçados; ou da defesa;
d) de banimento; – constar de registros policiais o uso de outros nomes
e) cruéis; ou diferentes qualificações;
XLVIII - a pena será cumprida em estabelecimentos – o estado de conservação ou a distância temporal ou da
distintos, de acordo com a natureza do delito, a idade e o sexo localidade da expedição do documento apresentado
do apenado; impossibilite a completa identificação dos caracteres
XLIX - é assegurado aos presos o respeito à integridade essenciais.
física e moral;
L - às presidiárias serão asseguradas condições para que LIX - será admitida ação privada nos crimes de ação
possam permanecer com seus filhos durante o período de pública, se esta não for intentada no prazo legal;
amamentação; LX - a lei só poderá restringir a publicidade dos atos
LI - nenhum brasileiro será extraditado, salvo o processuais quando a defesa da intimidade ou o interesse
naturalizado, em caso de crime comum, praticado antes da social o exigirem;
naturalização, ou de comprovado envolvimento em tráfico LXI - ninguém será preso senão em flagrante delito ou por
ilícito de entorpecentes e drogas afins, na forma da lei; ordem escrita e fundamentada de autoridade judiciária
competente, salvo nos casos de transgressão militar ou crime
propriamente militar, definidos em lei;
Antes da Depois da
LXII - a prisão de qualquer pessoa e o local onde se
naturalização naturalização
encontre serão comunicados imediatamente ao juiz
- prática de crime competente e à família do preso ou à pessoa por ele indicada;
-- LXIII - o preso será informado de seus direitos, entre os
comum
quais o de permanecer calado, sendo-lhe assegurada a
-comprovado -comprovado assistência da família e de advogado;
envolvimento em tráfico envolvimento em tráfico LXIV - o preso tem direito à identificação dos responsáveis
ilícito de entorpecentes e ilícito de entorpecentes e por sua prisão ou por seu interrogatório policial;
drogas afins drogas afins. LXV - a prisão ilegal será imediatamente relaxada pela
autoridade judiciária;
LII - não será concedida extradição de estrangeiro por LXVI - ninguém será levado à prisão ou nela mantido,
crime político ou de opinião; quando a lei admitir a liberdade provisória, com ou sem fiança;
LIII - ninguém será processado nem sentenciado senão LXVII - não haverá prisão civil por dívida, salvo a do
pela autoridade competente; responsável pelo inadimplemento voluntário e inescusável de
LIV - ninguém será privado da liberdade ou de seus bens obrigação alimentícia e a do depositário infiel;
sem o devido processo legal;
LV - aos litigantes, em processo judicial ou administrativo, Súmula Vinculante Nº 25 do STF: "É ilícito a prisão
e aos acusados em geral são assegurados o contraditório e civil de depositário infiel, qualquer que seja a modalidade
ampla defesa, com os meios e recursos a ela inerentes; do depósito".

Súmula Vinculante 5 do STF: A falta de defesa técnica LXVIII - conceder-se-á "habeas-corpus" sempre que
por advogado no processo administrativo disciplinar não alguém sofrer ou se achar ameaçado de sofrer violência ou
ofende a Constituição. coação em sua liberdade de locomoção, por ilegalidade ou
abuso de poder;
LVI - são inadmissíveis, no processo, as provas obtidas por LXIX - conceder-se-á mandado de segurança para proteger
meios ilícitos; direito líquido e certo, não amparado por "habeas-corpus"
LVII - ninguém será considerado culpado até o trânsito em ou "habeas-data", quando o responsável pela ilegalidade ou
julgado de sentença penal condenatória; abuso de poder for autoridade pública ou agente de pessoa
jurídica no exercício de atribuições do Poder Público;
Até o trânsito em julgado da sentença penal
condenatória, o acusado não pode ser considerado culpado. LXX - o mandado de segurança coletivo pode ser impetrado
Cabe à acusação provar a sua culpa. A prisão, antes da por:
condenação definitiva, só é possível em casos de flagrante a) partido político com representação no Congresso
Nacional;

Legislação 3
Apostila Digital Licenciada para Alice Caroline Guarino dos Santos - alice.guarino@hotmail.com (Proibida a Revenda)
APOSTILAS OPÇÃO

b) organização sindical, entidade de classe ou associação O Brasil se submete à jurisdição do TPI (Tribunal Penal
legalmente constituída e em funcionamento há pelo menos um Internacional), criado pelo Estatuto de Roma em 17 de
ano, em defesa dos interesses de seus membros ou associados; julho de 1998, o qual foi subscrito pelo Brasil e aprovado
LXXI - conceder-se-á mandado de injunção sempre que a pelo Decreto Legislativo nº 112/2002, tendo sua vigência
falta de norma regulamentadora torne inviável o exercício dos apartes desde ano. Trata-se de instituição permanente, com
direitos e liberdades constitucionais e das prerrogativas jurisdição para julgar genocídio, crimes de guerra, contra a
inerentes à nacionalidade, à soberania e à cidadania; humanidade e de agressão, e cuja sede se encontra em Haia,
na Holanda. Os crimes de competência desse Tribunal são
Durante muitos anos não houve uma lei imprescritíveis, dado que atentam contra a humanidade
regulamentando o procedimento do mandado de injunção, como um todo.
e por tal razão, aplicava-se, por analogia, as regras
procedimentais do mandado de segurança. Dos Direitos Sociais
Contudo, após longa espera foi editada a Lei nº
13.300/2016, que disciplina o processo e o julgamento dos Segundo José Afonso da Silva, os direitos sociais “são
mandados de injunção individual e coletivo. prestações positivas proporcionadas pelo Estado, direta ou
A grande consequência do mandado de injunção indiretamente, enunciadas em normas constitucionais, que
consiste na comunicação ao Poder Legislativo para que possibilitam melhores condições de vida aos mais fracos,
elabore a lei necessária ao exercício dos direitos e direitos que tendem a realizar a igualização de situações
liberdades constitucionais. sociais desiguais. São, portanto, direitos que se ligam ao direito
de igualdade”.
LXXII - conceder-se-á "habeas-data": Os direitos sociais exigem a intermediação dos entes
a) para assegurar o conhecimento de informações relativas estatais para sua concretização; consideram o homem para
à pessoa do impetrante, constantes de registros ou bancos de além de sua condição individualista, e guardam íntima relação
dados de entidades governamentais ou de caráter público; com o cidadão e a sociedade, porquanto abrangem a pessoa
b) para a retificação de dados, quando não se prefira fazê- humana na perspectiva de que ela necessita de condições
lo por processo sigiloso, judicial ou administrativo; mínimas de subsistência.
LXXIII - qualquer cidadão é parte legítima para propor
ação popular que vise a anular ato lesivo ao patrimônio CAPÍTULO II
público ou de entidade de que o Estado participe, à moralidade DOS DIREITOS SOCIAIS
administrativa, ao meio ambiente e ao patrimônio histórico e
cultural, ficando o autor, salvo comprovada má-fé, isento de Art. 6º São direitos sociais a educação, a saúde, a
custas judiciais e do ônus da sucumbência; alimentação, o trabalho, a moradia, o transporte, o lazer, a
LXXIV - o Estado prestará assistência jurídica integral e segurança, a previdência social, a proteção à maternidade e à
gratuita aos que comprovarem insuficiência de recursos; infância, a assistência aos desamparados, na forma desta
LXXV - o Estado indenizará o condenado por erro Constituição. (Redação dada pela Emenda Constitucional nº
judiciário, assim como o que ficar preso além do tempo fixado 90, de 2015)
na sentença;
LXXVI - são gratuitos para os reconhecidamente pobres, na Art. 7º São direitos dos trabalhadores urbanos e rurais,
forma da lei: além de outros que visem à melhoria de sua condição social:
a) o registro civil de nascimento; I - relação de emprego protegida contra despedida
b) a certidão de óbito; arbitrária ou sem justa causa, nos termos de lei complementar,
LXXVII - são gratuitas as ações de "habeas-corpus" e que preverá indenização compensatória, dentre outros
"habeas-data", e, na forma da lei, os atos necessários ao direitos;
exercício da cidadania. (Regulamento). II - seguro-desemprego, em caso de desemprego
LXXVIII a todos, no âmbito judicial e administrativo, são involuntário;
assegurados a razoável duração do processo e os meios que III - fundo de garantia do tempo de serviço;
garantam a celeridade de sua tramitação. IV - salário mínimo, fixado em lei, nacionalmente unificado,
§ 1º - As normas definidoras dos direitos e garantias capaz de atender às suas necessidades vitais básicas e às de
fundamentais têm aplicação imediata. sua família com moradia, alimentação, educação, saúde, lazer,
§ 2º - Os direitos e garantias expressos nesta Constituição vestuário, higiene, transporte e previdência social, com
não excluem outros decorrentes do regime e dos princípios reajustes periódicos que lhe preservem o poder aquisitivo,
por ela adotados, ou dos tratados internacionais em que a sendo vedada sua vinculação para qualquer fim;
República Federativa do Brasil seja parte. V - piso salarial proporcional à extensão e à complexidade
§ 3º Os tratados e convenções internacionais sobre direitos do trabalho;
humanos que forem aprovados, em cada Casa do Congresso VI - irredutibilidade do salário, salvo o disposto em
Nacional, em dois turnos, por três quintos dos votos dos convenção ou acordo coletivo;
respectivos membros, serão equivalentes às emendas VII - garantia de salário, nunca inferior ao mínimo, para os
constitucionais. que percebem remuneração variável;
VIII - décimo terceiro salário com base na remuneração
Cabe ressaltar que este parágrafo somente abrange os integral ou no valor da aposentadoria;
tratados e convenções internacionais sobre direitos IX - remuneração do trabalho noturno superior à do
humanos. Assim, os demais tratados serão recepcionados diurno;
pelo ordenamento jurídico brasileiro com o caráter de lei X - proteção do salário na forma da lei, constituindo crime
ordinária. sua retenção dolosa;
XI - participação nos lucros, ou resultados, desvinculada da
remuneração, e, excepcionalmente, participação na gestão da
§ 4º O Brasil se submete à jurisdição de Tribunal Penal empresa, conforme definido em lei;
Internacional a cuja criação tenha manifestado adesão. XII - salário-família pago em razão do dependente do
trabalhador de baixa renda nos termos da lei;

Legislação 4
Apostila Digital Licenciada para Alice Caroline Guarino dos Santos - alice.guarino@hotmail.com (Proibida a Revenda)
APOSTILAS OPÇÃO

XIII - duração do trabalho normal não superior a oito menores de dezesseis anos, salvo na condição de
horas diárias e quarenta e quatro semanais, facultada a aprendiz, a partir de quatorze anos;
compensação de horários e a redução da jornada, mediante XXXIV - igualdade de direitos entre o trabalhador com
acordo ou convenção coletiva de trabalho; vínculo empregatício permanente e o trabalhador avulso
XIV - jornada de seis horas para o trabalho realizado em Parágrafo único. São assegurados à categoria dos
turnos ininterruptos de revezamento, salvo negociação trabalhadores domésticos os direitos previstos nos incisos IV,
coletiva; VI, VII, VIII, X, XIII, XV, XVI, XVII, XVIII, XIX, XXI, XXII, XXIV,
XV - repouso semanal remunerado, preferencialmente aos XXVI, XXX, XXXI e XXXIII e, atendidas as condições
domingos; estabelecidas em lei e observada a simplificação do
XVI - remuneração do serviço extraordinário superior, no cumprimento das obrigações tributárias, principais e
mínimo, em cinquenta por cento à do normal; (Vide Del acessórias, decorrentes da relação de trabalho e suas
5.452, art. 59 § 1º). peculiaridades, os previstos nos incisos I, II, III, IX, XII, XXV e
XVII - gozo de férias anuais remuneradas com, pelo menos, XXVIII, bem como a sua integração à previdência
um terço a mais do que o salário normal; social. (Redação dada pela Emenda Constitucional nº 72, de
XVIII - licença à gestante, sem prejuízo do emprego e do 2013).
salário, com a duração de cento e vinte dias;
XIX - licença-paternidade, nos termos fixados em lei; Art. 8º É livre a associação profissional ou sindical,
observado o seguinte:
Observação: A Lei nº 11.770/2008 instituiu o programa I - a lei não poderá exigir autorização do Estado para a
empresa Cidadã, que permite que seja prorrogada a licença fundação de sindicato, ressalvado o registro no órgão
à gestante por mais 60 (sessenta) dias, ampliando, com isso competente, vedadas ao Poder Público a interferência e a
o prazo de 120 (cento e vinte) para 180 (cento e oitenta) intervenção na organização sindical;
dias. Contudo, não é obrigatória a adesão a este programa. II - é vedada a criação de mais de uma organização sindical,
Assim, a prorrogação é uma faculdade para as empresas em qualquer grau, representativa de categoria profissional ou
privadas (que ao aderirem o programa recebem incentivos econômica, na mesma base territorial, que será definida pelos
fiscais) e para a Administração Pública direita, indireta e trabalhadores ou empregadores interessados, não podendo
fundacional. ser inferior à área de um Município;
Cabe destacar, ainda, que esta lei foi recentemente III - ao sindicato cabe a defesa dos direitos e interesses
alterada pela lei nº 13.257/2016, sendo instituída a coletivos ou individuais da categoria, inclusive em questões
possibilidade de prorrogação da licença-paternidade por judiciais ou administrativas;
mais 15 (quinze) dias, além dos 5 (cinco) já assegurados IV - a Assembleia geral fixará a contribuição que, em se
constitucionalmente as empresas que fazem parte do tratando de categoria profissional, será descontada em folha,
programa. Porém, para isso, o empregado tem que requerer para custeio do sistema confederativo da representação
o benefício no prazo de 2 (dois) dias úteis após o parto e sindical respectiva, independentemente da contribuição
comprovar a participação em programa ou atividade de prevista em lei;
orientação sobre paternidade responsável. V - ninguém será obrigado a filiar-se ou a manter-se filiado
a sindicato;
VI - é obrigatória a participação dos sindicatos nas
XX - proteção do mercado de trabalho da mulher, mediante negociações coletivas de trabalho;
incentivos específicos, nos termos da lei; VII - o aposentado filiado tem direito a votar e ser votado
XXI - aviso prévio proporcional ao tempo de serviço, sendo nas organizações sindicais;
no mínimo de trinta dias, nos termos da lei; VIII - é vedada a dispensa do empregado sindicalizado a
XXII - redução dos riscos inerentes ao trabalho, por meio partir do registro da candidatura a cargo de direção ou
de normas de saúde, higiene e segurança; representação sindical e, se eleito, ainda que suplente, até um
XXIII - adicional de remuneração para as atividades ano após o final do mandato, salvo se cometer falta grave nos
penosas, insalubres ou perigosas, na forma da lei; termos da lei.
XXIV - aposentadoria; Parágrafo único. As disposições deste artigo aplicam-se à
XXV - assistência gratuita aos filhos e dependentes desde o organização de sindicatos rurais e de colônias de pescadores,
nascimento até 5 (cinco) anos de idade em creches e pré- atendidas as condições que a lei estabelecer.
escolas;
XXVI - reconhecimento das convenções e acordos coletivos Art. 9º É assegurado o direito de greve, competindo aos
de trabalho; trabalhadores decidir sobre a oportunidade de exercê-lo e
XXVII - proteção em face da automação, na forma da lei; sobre os interesses que devam por meio dele defender.
XXVIII - seguro contra acidentes de trabalho, a cargo do § 1º - A lei definirá os serviços ou atividades essenciais e
empregador, sem excluir a indenização a que este está disporá sobre o atendimento das necessidades inadiáveis da
obrigado, quando incorrer em dolo ou culpa; comunidade.
XXIX - ação, quanto aos créditos resultantes das relações § 2º - Os abusos cometidos sujeitam os responsáveis às
de trabalho, com prazo prescricional de cinco anos para os penas da lei.
trabalhadores urbanos e rurais, até o limite de dois anos após
a extinção do contrato de trabalho; Art. 10. É assegurada a participação dos trabalhadores e
XXX - proibição de diferença de salários, de exercício de empregadores nos colegiados dos órgãos públicos em que seus
funções e de critério de admissão por motivo de sexo, idade, interesses profissionais ou previdenciários sejam objeto de
cor ou estado civil; discussão e deliberação.
XXXI - proibição de qualquer discriminação no tocante a
salário e critérios de admissão do trabalhador portador de Art. 11. Nas empresas de mais de duzentos empregados, é
deficiência; assegurada a eleição de um representante destes com a
XXXII - proibição de distinção entre trabalho manual, finalidade exclusiva de promover-lhes o entendimento direto
técnico e intelectual ou entre os profissionais respectivos; com os empregadores.
XXXIII - proibição de trabalho noturno, perigoso ou
insalubre a menores de dezoito e de qualquer trabalho a

Legislação 5
Apostila Digital Licenciada para Alice Caroline Guarino dos Santos - alice.guarino@hotmail.com (Proibida a Revenda)
APOSTILAS OPÇÃO

Título VIII Da Ordem Social Capítulo II Da Seguridade Já os direitos dos idosos são regulados no art. 230.
Social Capítulo VII Da Família, da Criança, do A família, a sociedade e o Estado têm o dever de amparar
Adolescente, do Jovem e do Idoso as pessoas idosas, assegurando sua participação na
comunidade, defendendo sua dignidade e bem-estar e
Capítulo VII - Da Família, da Criança, do Adolescente, garantindo-lhes o direito à vida.
do Jovem e do Idoso
O § 2º garante a gratuidade dos transportes coletivos
A Constituição Federal assegura à família uma proteção urbanos aos maiores de sessenta e cinco anos. Daí o erro da
especial ao reconhece-la como base da sociedade civil. questão.

Os §§ 3º e 4º do art. 226 tratam o que a CF reconhece como Além disso, a Constituição Federal estabelece ainda que os
entidade familiar. Assim, a entidade familiar é a união entre o programas de amparo aos idosos sejam executados
homem e a mulher e a comunidade formada por qualquer dos preferencialmente em seus lares.
pais e seus descendentes.
Veja os dispositivos constitucionais acerca do assunto:
Por ser a família base da sociedade caberá ao Estado
assegurá-la assistência na pessoa de cada um dos que a CAPÍTULO VII
integram, criando mecanismos para coibir a violência no Da Família, da Criança, do Adolescente, do Jovem e do
âmbito de suas relações (CF, art. 226, § 8°). Idoso
(Redação dada Pela Emenda Constitucional nº 65, de
Atenção! o STF aplicando os princípios da dignidade da 2010)
pessoa humana, da liberdade, da autodeterminação, da
igualdade, do pluralismo, da intimidade, da não discriminação Art. 226. A família, base da sociedade, tem especial
e da busca da felicidade estendeu às uniões homoafetivas o proteção do Estado.
mesmo regime jurídico aplicável à união estável entre pessoas § 1º O casamento é civil e gratuita a celebração.
de sexo distinto. § 2º O casamento religioso tem efeito civil, nos termos da
lei.
Tome nota: Em março de 2017, a Comissão de § 3º Para efeito da proteção do Estado, é reconhecida a
Constituição e Justiça do Senado aprovou projeto para união estável entre o homem e a mulher como entidade
permitir a união estável entre pessoas do mesmo sexo e familiar, devendo a lei facilitar sua conversão em casamento.
posterior conversão dessa união em casamento. Esse projeto § 4º Entende-se, também, como entidade familiar a
de lei ainda tem que passar pelo quórum de aprovação da comunidade formada por qualquer dos pais e seus
Câmara de Deputados. descendentes.
§ 5º Os direitos e deveres referentes à sociedade conjugal
O casamento poderá ser dissolvido pelo divórcio (art. 226, são exercidos igualmente pelo homem e pela mulher.
§6º). § 6º O casamento civil pode ser dissolvido pelo divórcio.
(Redação dada Pela Emenda Constitucional nº 66, de 2010)
Os direitos e deveres inerentes à entidade familiar serão § 7º Fundado nos princípios da dignidade da pessoa
exercidos igualmente pelo homem e pela mulher. humana e da paternidade responsável, o planejamento
familiar é livre decisão do casal, competindo ao Estado
O §7º do art. 226 trata sobre o planejamento familiar e propiciar recursos educacionais e científicos para o exercício
dispõe que será livre decisão do casal, competindo ao Estado desse direito, vedada qualquer forma coercitiva por parte de
propiciar recursos educacionais e científicos para o exercício instituições oficiais ou privadas.
desse direito, vedada qualquer forma coercitiva por parte de § 8º O Estado assegurará a assistência à família na pessoa
instituições oficiais ou privadas. de cada um dos que a integram, criando mecanismos para
coibir a violência no âmbito de suas relações.
Com relação a proteção destinada à criança e ao
adolescente a Carta Magna adota a doutrina da proteção Art. 227. É dever da família, da sociedade e do Estado
integral. O menor passa a ser sujeito de direitos, tendo acesso assegurar à criança, ao adolescente e ao jovem, com absoluta
irrestrito e privilegiado à Justiça. prioridade, o direito à vida, à saúde, à alimentação, à educação,
ao lazer, à profissionalização, à cultura, à dignidade, ao
O Estado e a sociedade também se tornam responsáveis respeito, à liberdade e à convivência familiar e comunitária,
pela criança e adolescente (art. 227). além de colocá-los a salvo de toda forma de negligência,
discriminação, exploração, violência, crueldade e opressão.
Caberá ao Estado promoverá programas de assistência (Redação dada Pela Emenda Constitucional nº 65, de 2010)
integral à saúde da criança, do adolescente e do jovem, § 1º O Estado promoverá programas de assistência
admitida a participação de entidades não governamentais, integral à saúde da criança, do adolescente e do jovem,
mediante políticas específicas e obedecendo aos seguintes admitida a participação de entidades não governamentais,
preceitos: mediante políticas específicas e obedecendo aos seguintes
- aplicação de percentual dos recursos públicos destinados preceitos: (Redação dada Pela Emenda Constitucional nº 65,
à saúde na assistência materno-infantil; de 2010)
- criação de programas de prevenção e atendimento I - aplicação de percentual dos recursos públicos
especializado para as pessoas portadoras de deficiência física, destinados à saúde na assistência materno-infantil;
sensorial ou mental, bem como de integração social do II - criação de programas de prevenção e atendimento
adolescente e do jovem portador de deficiência, mediante o especializado para as pessoas portadoras de deficiência física,
treinamento para o trabalho e a convivência, e a facilitação do sensorial ou mental, bem como de integração social do
acesso aos bens e serviços coletivos, com a eliminação de adolescente e do jovem portador de deficiência, mediante o
obstáculos arquitetônicos e de todas as formas de treinamento para o trabalho e a convivência, e a facilitação do
discriminação. acesso aos bens e serviços coletivos, com a eliminação de
obstáculos arquitetônicos e de todas as formas de

Legislação 6
Apostila Digital Licenciada para Alice Caroline Guarino dos Santos - alice.guarino@hotmail.com (Proibida a Revenda)
APOSTILAS OPÇÃO

discriminação. (Redação dada Pela Emenda Constitucional nº Questões


65, de 2010)
§ 2º A lei disporá sobre normas de construção dos 01. (UFCG - Auxiliar em Administração – UFCG/2016)
logradouros e dos edifícios de uso público e de fabricação de Não fazem parte dos direitos sociais assegurados
veículos de transporte coletivo, a fim de garantir acesso constitucionalmente:
adequado às pessoas portadoras de deficiência. (A) A assistência aos desamparados.
§ 3º O direito a proteção especial abrangerá os seguintes (B) O lazer.
aspectos: (C) A segurança.
I - idade mínima de quatorze anos para admissão ao (D) A vida.
trabalho, observado o disposto no art. 7º, XXXIII; (E) A previdência social.
II - garantia de direitos previdenciários e trabalhistas;
III - garantia de acesso do trabalhador adolescente e jovem 02. (Prefeitura de Paracambi – RJ - Advogado – BIO-
à escola; (Redação dada Pela Emenda Constitucional nº 65, de RIO/2016) Sobre os direitos sociais, assinale a opção
2010) CORRETA:
IV - garantia de pleno e formal conhecimento da atribuição (A) É proibido trabalho noturno, perigoso ou insalubre a
de ato infracional, igualdade na relação processual e defesa menores de dezoito e de qualquer trabalho a menores de
técnica por profissional habilitado, segundo dispuser a dezesseis anos, salvo na condição de aprendiz, a partir de doze
legislação tutelar específica; anos.
V - obediência aos princípios de brevidade, (B) É proibido trabalho noturno, perigoso ou insalubre a
excepcionalidade e respeito à condição peculiar de pessoa em menores de dezoito e de qualquer trabalho a menores de
desenvolvimento, quando da aplicação de qualquer medida dezesseis anos, salvo na condição de aprendiz, a partir de
privativa da liberdade; quatorze anos.
VI - estímulo do Poder Público, através de assistência (C) É proibido trabalho noturno e perigoso, sendo
jurídica, incentivos fiscais e subsídios, nos termos da lei, ao permitido o insalubre, a menores de dezoito e de qualquer
acolhimento, sob a forma de guarda, de criança ou adolescente trabalho a menores de dezesseis anos, salvo na condição de
órfão ou abandonado; aprendiz, a partir de quatorze anos.
VII - programas de prevenção e atendimento especializado (D) É proibido trabalho noturno, perigoso ou insalubre a
à criança, ao adolescente e ao jovem dependente de menores de dezesseis e de qualquer trabalho a menores de
entorpecentes e drogas afins. (Redação dada Pela Emenda quatorze anos, salvo na condição de aprendiz, a partir de doze
Constitucional nº 65, de 2010) anos.
§ 4º A lei punirá severamente o abuso, a violência e a
exploração sexual da criança e do adolescente. 03. (TRF - 3ª REGIÃO - Analista Judiciário – Contadoria
§ 5º A adoção será assistida pelo Poder Público, na forma – FCC/2016) Entre os direitos sociais, a Constituição Federal
da lei, que estabelecerá casos e condições de sua efetivação por garante os direitos dos trabalhadores, EXCETO:
parte de estrangeiros. (A) Relação de emprego protegida contra despedida
§ 6º Os filhos, havidos ou não da relação do casamento, ou arbitrária ou sem justa causa, nos termos de lei complementar,
por adoção, terão os mesmos direitos e qualificações, que preverá indenização compensatória, dentre outros
proibidas quaisquer designações discriminatórias relativas à direitos.
filiação. (B) Salário-família pago em razão do dependente do
§ 7º No atendimento dos direitos da criança e do trabalhador de baixa renda, nos termos da lei.
adolescente levar-se- á em consideração o disposto no art. 204. (C) Seguro contra acidentes do trabalho, a cargo do
§ 8º A lei estabelecerá: (Incluído Pela Emenda empregador, sem excluir a indenização a que este está
Constitucional nº 65, de 2010) obrigado quando incorrer em dolo ou culpa.
I - o estatuto da juventude, destinado a regular os direitos (D) Participação nos lucros ou resultados da empresa,
dos jovens; (Incluído Pela Emenda Constitucional nº 65, de vinculada à remuneração, nos termos da lei.
2010) (E) Duração do trabalho normal não superior a oito horas
II - o plano nacional de juventude, de duração decenal, diárias e quarenta e quatro semanais, facultadas a
visando à articulação das várias esferas do poder público para compensação de horários e a redução da jornada, mediante
a execução de políticas públicas. (Incluído Pela Emenda acordo ou convenção coletiva de trabalho
Constitucional nº 65, de 2010)
04. (UFMA - Pedagogo – UFMA/2016) Os direitos e
Art. 228. São penalmente inimputáveis os menores de garantias fundamentais
dezoito anos, sujeitos às normas da legislação especial. (A) Estão taxativamente previstos na Constituição de
1988.
Art. 229. Os pais têm o dever de assistir, criar e educar os (B) De caráter prestacional não são exigíveis do Estado.
filhos menores, e os filhos maiores têm o dever de ajudar e (C) Não se aplicam às relações privadas.
amparar os pais na velhice, carência ou enfermidade. (D) São inalienáveis e indisponíveis.
(E) Podem sofrer limitações que atinjam seu núcleo
Art. 230. A família, a sociedade e o Estado têm o dever de essencial.
amparar as pessoas idosas, assegurando sua participação na
comunidade, defendendo sua dignidade e bem-estar e 05. (TRE-PI - Técnico Judiciário – Administrativa –
garantindo-lhes o direito à vida. CESPE/2016) A respeito dos direitos e das garantias
§ 1º Os programas de amparo aos idosos serão executados fundamentais, assinale a opção correta.
preferencialmente em seus lares. (A) Os direitos sociais, econômicos e culturais são,
§ 2º Aos maiores de sessenta e cinco anos é garantida a atualmente, classificados como direitos fundamentais de
gratuidade dos transportes coletivos urbanos. terceira geração.
(B) O direito ao meio ambiente equilibrado e o direito à
autodeterminação dos povos são exemplos de direitos
classificados como de segunda geração.

Legislação 7
Apostila Digital Licenciada para Alice Caroline Guarino dos Santos - alice.guarino@hotmail.com (Proibida a Revenda)
APOSTILAS OPÇÃO

(C) A comissão parlamentar de inquérito tem autonomia 09. (FUB - Conhecimentos Básicos - Somente para os
para determinar a busca e a apreensão em domicílio alheio, cargos 10 e 13 – CESPE/2016) À luz das disposições
com o objetivo de coletar provas que interessem ao poder constitucionais relativas aos direitos sociais, julgue o item a
público. seguir.
(D) A entrada em domicílio, sem o consentimento do
morador, é permitida durante o dia e a noite, desde que haja Aos trabalhadores compete decidir sobre os interesses que
autorização judicial. devam defender por meio do exercício do direito de greve.
(E) A doutrina moderna classifica os direitos civis e (...) Certo
políticos como direitos fundamentais de primeira geração. (...) Errado

06. (PC-PE - Escrivão de Polícia – CESPE/2016) No que 10. (PGE-MT - Técnico - Técnico Adminstrativo –
se refere aos direitos e às garantias fundamentais, assinale a FCC/2016) O trabalho infantil é proibido pela Constituição
opção correta. Federal, que estabelece a idade inicial e as condições em que é
(A) O direito fundamental ao contraditório não se aplica permitido trabalhar no Brasil. O dispositivo constitucional
aos inquéritos policiais. estabelece a proibição de trabalho noturno, perigoso ou
(B) O início de execução da pena criminal condenatória insalubre a menores de
após a confirmação da sentença em segundo grau ofende o (A) dezesseis anos e de qualquer trabalho a menores de
princípio constitucional de presunção da inocência. quatorze anos, salvo na condição de aprendiz, a partir de doze
(C) Os direitos e as garantias individuais não são anos.
assegurados às pessoas jurídicas, uma vez que elas possuem (B) vinte e um anos e de qualquer trabalho a menores de
dimensão coletiva. dezoito anos, salvo na condição de aprendiz, a partir de
(D) O sigilo de correspondência e o sigilo das dezesseis anos
comunicações telefônicas são invioláveis ressalvadas as (C) dezoito anos e de qualquer trabalho a menores de
hipóteses legais, por ordem judicial ou administrativa dezesseis anos, salvo na condição de aprendiz, a partir de
devidamente motivada. quatorze anos.
(E) O tribunal do júri tem competência para o julgamento (D) vinte e um anos e de qualquer trabalho a menores de
dos crimes culposos e dolosos contra a vida. dezesseis anos, salvo na condição de aprendiz, a partir de doze
anos.
07. (Prefeitura de Teresina – PI - Técnico de Nível (E) dezoito anos e de qualquer trabalho a menores de
Superior – Contador – FCC/2016) Ao tratar dos direitos e quatorze anos, salvo na condição de aprendiz, a partir de doze
garantias fundamentais a Constituição Federal dispõe que anos.
(A) será admitida a ação privada nos crimes de ação
pública desde que a ação seja proposta no prazo legal. 11. (Câmara de Maria Helena – PR - Advogado –
(B) o Estado indenizará o condenado por erro judiciário, FAUEL/2017) Acerca dos direitos e garantias fundamentais,
assim como o que ficar preso além do tempo fixado na assinale a alternativa correta:
sentença. (A) A falta de defesa técnica por advogado no processo
(C) os tratados e convenções internacionais sobre direitos administrativo disciplinar ofende a Constituição.
humanos que forem aprovados, em cada Casa do Congresso (B) É inconstitucional a exigência de depósito ou
Nacional, em dois turnos, por três quintos dos votos dos arrolamento prévios de dinheiro ou bens para admissibilidade
respectivos membros, serão equivalentes às leis ordinárias. de recurso administrativo.
(D) os brasileiros naturalizados não poderão ser (C) Só é lícita a prisão civil de depositário infiel quando se
extraditados. tratar de depositário nomeado pelo juízo.
(E) sempre que solicitada, poderá ser concedida a (D) É constitucional a exigência de depósito prévio como
extradição de estrangeiro pela prática de qualquer crime requisito de admissibilidade de ação judicial na qual se
cometido no país de origem. pretenda discutir a exigibilidade de crédito tributário.

08. (EBSERH - Advogado (HUGG-UNIRIO) – IBFC/2017) 12. (TRE-SP - Técnico Judiciário – Área Administrativa
Considere as normas da Constituição Federal sobre a – FCC/2017) Seria incompatível com as normas
liberdade de associação profissional ou sindical e assinale a constitucionais garantidoras de direitos e garantias
alternativa correta. fundamentais
(A) A lei poderá exigir autorização do Estado para a (A) o estabelecimento de restrições, por lei, à entrada ou
fundação de sindicato, bem como o registro no órgão permanência de pessoas com seus bens no território nacional.
competente, vedadas ao Poder Público a interferência e a (B) a reunião pacífica, sem armas, em local aberto ao
intervenção na organização sindical público, independentemente de autorização, mediante aviso
(B) É vedada a criação de mais de uma organização prévio à autoridade competente.
sindical, em qualquer grau, representativa de categoria (C) a suspensão das atividades de associação por decisão
profissional ou econômica, na mesma base territorial, que será judicial não transitada em julgado.
definida pelos trabalhadores ou empregadores interessados, (D) a interceptação de comunicações telefônicas, para fins
não podendo ser inferior à área de um Município de investigação criminal, por determinação da autoridade
(C) Ao sindicato cabe a defesa dos direitos e interesses policial competente.
coletivos ou individuais da categoria, inclusive apenas em (E) a entrada na casa, sem consentimento do morador, em
questões judiciais caso de flagrante delito, durante a noite.
(D) A assembleia geral fixará a contribuição que, em se
tratando de categoria profissional, será descontada em folha, 13. (EBSERH - Advogado (HUGG-UNIRIO) – IBFC/2017)
para custeio do sistema confederativo da representação Considere as normas da Constituição Federal sobre direitos e
sindical respectiva, limitada até o máximo independentemente garantias fundamentais e assinale a alternativa INCORRETA.
da contribuição prevista em lei (A) São assegurados, nos termos da lei, o direito de
(E) É facultativa a participação dos sindicatos nas fiscalização do aproveitamento econômico das obras que
negociações coletivas de trabalho criarem ou de que participarem aos criadores, aos intérpretes
e às respectivas representações associativas

Legislação 8
Apostila Digital Licenciada para Alice Caroline Guarino dos Santos - alice.guarino@hotmail.com (Proibida a Revenda)
APOSTILAS OPÇÃO

(B) São assegurados, nos termos da lei, a proteção às (B) somente poderá ser extraditado se ficar caracterizado
participações individuais em obras coletivas e à reprodução da crime político ou de opinião, casos em que a Constituição veda
imagem e voz humanas, inclusive nas atividades desportivas expressamente a extradição apenas de estrangeiro.
(D) São assegurados, nos termos da lei, o direito de (C) somente poderá ser extraditado se, antes, for cancelada
fiscalização do aproveitamento econômico das obras que a naturalização, por ato da autoridade administrativa
criarem ou de que participarem aos criadores, aos intérpretes competente, em virtude de atividade nociva ao interesse
e às respectivas representações sindicais e associativas nacional.
(D) São assegurados, nos termos da lei, a proteção às (D) poderá ser extraditado, desde que haja condenação
participações individuais em obras coletivas, excluídas as pelo cometimento de crime comum praticado anteriormente à
atividades desportivas naturalização.
(E) São assegurados, nos termos da lei, o direito de (E) poderá ser extraditado, ainda que o envolvimento com
fiscalização do aproveitamento econômico das obras que o tráfico ilícito de entorpecentes seja posterior à naturalização.
criarem ou de que participarem aos criadores, aos intérpretes
e às respectivas representações sindicais 18. (TRT - 11ª Região (AM e RR) - Analista Judiciário -
Área Administrativa – FCC/2017) Durval foi alvo de racismo
14. (SEDF - Professor – Direito – QUADRIX/2017) em seu trabalho. Ao consultar a Constituição Federal,
Julgue o próximo item com relação ao Direito Constitucional. descobriu que a prática de racismo constitui crime
A garantia constitucional quanto à impossibilidade de (A) inafiançável, apenas, sujeito à pena de detenção, nos
utilização, nos processos, de prova ilícita, mantém estreito termos da lei.
vínculo com outros direitos e outras garantias também (B) inafiançável, apenas, sujeito à pena de reclusão, nos
constitucionais, como o direito à intimidade e à privacidade. termos da lei.
(...) Certo (C) imprescritível, apenas, sujeito à pena de reclusão, nos
(...) Errado termos da lei.
(D) inafiançável e imprescritível, sujeito à pena de
15. (MPE-RS - Secretário de Diligências - MPE- detenção, nos termos da lei.
RS/2017) De acordo com o tratamento constitucional (E) inafiançável e imprescritível, sujeito à pena de
dispensado aos direitos políticos e à nacionalidade, assinale a reclusão, nos termos da lei.
alternativa INCORRETA.
(A) O alistamento eleitoral e o voto são obrigatórios para 19. (TRE-SP - Técnico Judiciário – Área Administrativa
os maiores de dezoito anos. – FCC/2017) Os direitos ao décimo terceiro salário com base
(B) Os analfabetos são inalistáveis e inelegíveis. na remuneração integral ou no valor da aposentadoria, à
(C) Não podem alistar-se como eleitores, os estrangeiros e, remuneração do serviço extraordinário superior, no mínimo,
durante o período do serviço militar obrigatório, os conscritos. em cinquenta por cento à do normal e à redução dos riscos
(D) São brasileiros natos os nascidos no estrangeiro de pai inerentes ao trabalho, por meio de normas de saúde, higiene e
brasileiro ou de mãe brasileira, desde que registrados em segurança, são todos assegurados, na Constituição Federal, aos
repartição brasileira competente ou venham a residir na (A) trabalhadores urbanos e rurais, mas não aos
República Federativa do Brasil e optem, em qualquer tempo, domésticos, nem aos servidores ocupantes de cargo público.
depois de atingida a maioridade, pela nacionalidade brasileira. (B) trabalhadores urbanos e rurais, bem como aos
(E) São privativos de brasileiro nato os cargos de Ministro domésticos e aos servidores ocupantes de cargo público.
do Supremo Tribunal Federal e de Oficial das Forças Armadas. (C) trabalhadores urbanos e rurais, bem como aos
domésticos, mas não aos servidores ocupantes de cargo
16. (EBSERH - Advogado (HUJB – UFCG) – INSTITUTO público.
AOCP/2017) São direitos dos trabalhadores urbanos e rurais, (D) trabalhadores domésticos, mas não aos urbanos e
nos moldes da Constituição Federal de 1988: rurais, nem aos servidores ocupantes de cargo público.
(A) piso salarial proporcional à extensão e à complexidade (E) servidores ocupantes de cargo público, mas não aos
do trabalho; remuneração do trabalho noturno em dobro à do trabalhadores urbanos e rurais, nem aos domésticos.
diurno e jornada de sete horas para turnos ininterruptos de
revezamento, ainda que sem negociação coletiva. 20. (TJM-SP - Escrevente Técnico Judiciário –
(B) décimo terceiro salário com base na remuneração VUNESP/2017) Quanto ao habeas corpus, assinale a
integral ou no valor da aposentadoria; salário-família pago em alternativa correta.
razão do dependente do trabalhador de baixa renda e gozo de (A) É gratuito.
férias anuais remuneradas com no mínimo um terço a mais do (B) É cabível em relação a qualquer punição disciplinar
que o salário normal. militar.
(C) repouso semanal remunerado aos sábados e domingos; (C) Concede-se para proteger direito líquido e certo.
licença à gestante de no mínimo 180 dias e hora extraordinária (D) Assegura o conhecimento de informações pessoais.
de pelo menos cinquenta por cento da hora normal. (E) Exige sigilo processual.
(D) licença-paternidade; adicional de remuneração para
atividades mais trabalhosas; e aviso prévio proporcional ao 21. (PC-DF - Perito Criminal - Ciências Contábeis –
tempo de serviço, sendo no mínimo trinta dias. IADES/2016) A família, base da sociedade, tem especial
(E) participação nos lucros e resultados; garantia de proteção do Estado e está preconizada pela Constituição
salário, nunca inferior a oitenta por cento do mínimo nacional, Federal. Em relação ao texto constitucional, assinale a
para quem tem remuneração variável; e seguro contra alternativa correta.
acidentes de trabalho, a cargo do empregador. (A) Como entidade familiar entende-se, também, a
comunidade formada por duas pessoas do mesmo gênero.
17. (TRE-SP - Analista Judiciário - Área Administrativa (B) Como entidade familiar entende-se, também, a
– FCC/2017) À luz da Constituição da República, brasileiro comunidade formada por qualquer dos pais e seus
naturalizado que, comprovadamente, esteja envolvido em descendentes.
tráfico ilícito de entorpecentes, na forma da lei, (C) O casamento religioso deixa de ter efeito civil.
(A) não poderá ser extraditado, pois é expressamente (D) Para efeito da proteção do Estado, não deve a lei
vedada a extradição de brasileiro. facilitar a conversão da união estável em casamento.

Legislação 9
Apostila Digital Licenciada para Alice Caroline Guarino dos Santos - alice.guarino@hotmail.com (Proibida a Revenda)
APOSTILAS OPÇÃO

(E) A união estável pode ser dissolvida pelo divórcio. (D) Os pais têm o dever de assistir, criar e educar os filhos
menores, e os filhos maiores têm o dever de ajudar e amparar
22. (IF-AP - Técnico em Assuntos Educacionais – os pais na velhice, carência ou enfermidade.
FUNIVERSA/2016) De acordo com a CF, é correto afirmar que (E) A adoção será assistida pelo Poder Público, na forma da
(A) são penalmente inimputáveis os menores de dezoito lei, que estabelecerá casos e condições de sua efetivação por
anos de idade. parte de brasileiros natos e naturalizados, vedada a adoção por
(B) os programas de amparo aos idosos serão executados estrangeiros.
preferencialmente fora de seus lares.
(C) é garantida a gratuidade dos transportes coletivos Respostas
urbanos aos maiores de cinquenta e cinco anos de idade.
(D) os estados e o Distrito Federal são obrigados a vincular 01. D. / 02. B. / 03. D. / 04. D. / 05. E.
parcela de sua receita orçamentária a entidades públicas de 06. A. / 07. B. / 08. B / 09. Certo / 10. C
fomento ao ensino e à pesquisa científica e tecnológica. 11. B / 12. D / 13. D / 14. Certo / 15. B
(E) os recursos públicos serão destinados exclusivamente 16. B / 17. E / 18. E / 19. B / 20. A
às escolas públicas. 21. B. / 22. A. / 23. C. / 24. B. / 25. D.

23. Considerando o estabelecido na Constituição Federal


de 1988, indique a alternativa INCORRETA.
ESTATUTO DA CRIANÇA E DO
(A) O Estado assegurará a assistência à família na pessoa
de cada um dos que a integram, criando mecanismos para ADOLESCENTE Atualizado
coibir a violência no âmbito de suas relações. com a Lei 12.010.
(B) O Estado promoverá programas de assistência integral
à saúde da criança, do adolescente e do jovem, admitida a
participação de entidades não governamentais, mediante
LEI Nº 8.069, DE 13 DE JULHO DE 1990.
políticas específicas e obedecendo, entre outro, ao preceito de
aplicação de percentual dos recursos públicos destinados à
Dispõe sobre o Estatuto da Criança e do Adolescente e dá
saúde na assistência materno-infantil.
outras providências.
(C) Aos maiores de sessenta e cinco anos é garantida a
gratuidade dos transportes coletivos intermunicipais e
O PRESIDENTE DA REPÚBLICA: Faço saber que o
interestaduais.
Congresso Nacional decreta e eu sanciono a seguinte Lei:
(D) A lei estabelecerá o estatuto da juventude, destinado a
regular os direitos dos jovens e o Plano Nacional de Juventude,
Título I
de duração decenal, visando à articulação das várias esferas do
Das Disposições Preliminares
poder público para a execução de políticas públicas.
Art. 1º Esta Lei dispõe sobre a proteção integral à criança
24. (Prefeitura de Pelotas – RS - Conselheiro Tutelar -
e ao adolescente.
MGA) Em relação aos parágrafos do Art. 226, da Constituição
Federal de 1988, analise as afirmativas.
Art. 2º Considera-se criança, para os efeitos desta Lei, a
I - O casamento é civil e gratuita a celebração.
pessoa até doze anos de idade incompletos, e adolescente
II - O casamento religioso não tem efeito civil, nos termos
aquela entre doze e dezoito anos de idade.
da lei.
Parágrafo único. Nos casos expressos em lei, aplica-se
III - Para efeito da proteção do Estado, não é reconhecida a
excepcionalmente este Estatuto às pessoas entre dezoito e
união estável entre o homem e a mulher como entidade
vinte e um anos de idade.
familiar, devendo a lei restringir sua conversão em casamento.
IV - Fundado nos princípios da dignidade da pessoa
Art. 3º A criança e o adolescente gozam de todos os direitos
humana e da paternidade responsável, o planejamento
fundamentais inerentes à pessoa humana, sem prejuízo da
familiar é livre decisão do casal, competindo ao Estado
proteção integral de que trata esta Lei, assegurando-se-lhes,
propiciar recursos educacionais e científicos para o exercício
por lei ou por outros meios, todas as oportunidades e
desse direito, vedada qualquer forma coercitiva por parte de
facilidades, a fim de lhes facultar o desenvolvimento físico,
instituições oficiais ou privadas.
mental, moral, espiritual e social, em condições de liberdade e
de dignidade.
Assinale a alternativa CORRETA.
Parágrafo único. Os direitos enunciados nesta Lei aplicam-
(A) Somente I está correta.
se a todas as crianças e adolescentes, sem discriminação de
(B) I e IV estão corretas.
nascimento, situação familiar, idade, sexo, raça, etnia ou cor,
(C) II, III e IV estão corretas.
religião ou crença, deficiência, condição pessoal de
(D) Todas as afirmativas estão corretas.
desenvolvimento e aprendizagem, condição econômica,
ambiente social, região e local de moradia ou outra condição
25. (PC-CE - Escrivão de Polícia Civil de 1ª Classe -
que diferencie as pessoas, as famílias ou a comunidade em que
VUNESP) Assinale a alternativa que está de acordo com o
vivem. (Incluído pela Lei nº 13.257, de 2016)
disposto na Constituição Federal a respeito da Família, da
Criança, do Adolescente, do Jovem e do Idoso.
Art. 4º É dever da família, da comunidade, da sociedade em
(A) São civil e penalmente inimputáveis os menores de
geral e do poder público assegurar, com absoluta prioridade, a
vinte e um anos, sujeitos às normas da legislação especial.
efetivação dos direitos referentes à vida, à saúde, à
(B) O casamento é civil e gratuita a celebração, mas o
alimentação, à educação, ao esporte, ao lazer, à
casamento religioso não terá efeito civil
profissionalização, à cultura, à dignidade, ao respeito, à
(C) Os índios, suas comunidades e organizações são partes
liberdade e à convivência familiar e comunitária.
legítimas para ingressar em juízo em defesa de seus direitos e
Parágrafo único. A garantia de prioridade compreende:
interesses, intervindo o Poder Executivo em todos os atos do
a) primazia de receber proteção e socorro em quaisquer
processo.
circunstâncias;

Legislação 10
Apostila Digital Licenciada para Alice Caroline Guarino dos Santos - alice.guarino@hotmail.com (Proibida a Revenda)
APOSTILAS OPÇÃO

b) precedência de atendimento nos serviços públicos ou de § 8º A gestante tem direito a acompanhamento saudável
relevância pública; durante toda a gestação e a parto natural cuidadoso,
c) preferência na formulação e na execução das políticas estabelecendo-se a aplicação de cesariana e outras
sociais públicas; intervenções cirúrgicas por motivos médicos. (Incluído pela
d) destinação privilegiada de recursos públicos nas áreas Lei nº 13.257, de 2016)
relacionadas com a proteção à infância e à juventude. § 9º A atenção primária à saúde fará a busca ativa da
gestante que não iniciar ou que abandonar as consultas de pré-
Art. 5º Nenhuma criança ou adolescente será objeto de natal, bem como da puérpera que não comparecer às consultas
qualquer forma de negligência, discriminação, exploração, pós-parto. (Incluído pela Lei nº 13.257, de 2016)
violência, crueldade e opressão, punido na forma da lei § 10. Incumbe ao poder público garantir, à gestante e à
qualquer atentado, por ação ou omissão, aos seus direitos mulher com filho na primeira infância que se encontrem sob
fundamentais. custódia em unidade de privação de liberdade, ambiência que
atenda às normas sanitárias e assistenciais do Sistema Único
Art. 6º Na interpretação desta Lei levar-se-ão em conta os de Saúde para o acolhimento do filho, em articulação com o
fins sociais a que ela se dirige, as exigências do bem comum, os sistema de ensino competente, visando ao desenvolvimento
direitos e deveres individuais e coletivos, e a condição peculiar integral da criança. (Incluído pela Lei nº 13.257, de 2016)
da criança e do adolescente como pessoas em
desenvolvimento. Art. 9º O poder público, as instituições e os empregadores
propiciarão condições adequadas ao aleitamento materno,
Título II inclusive aos filhos de mães submetidas a medida privativa de
Dos Direitos Fundamentais liberdade.
Capítulo I § 1º Os profissionais das unidades primárias de saúde
Do Direito à Vida e à Saúde desenvolverão ações sistemáticas, individuais ou coletivas,
visando ao planejamento, à implementação e à avaliação de
Art. 7º A criança e o adolescente têm direito a proteção à ações de promoção, proteção e apoio ao aleitamento materno
vida e à saúde, mediante a efetivação de políticas sociais e à alimentação complementar saudável, de forma contínua.
públicas que permitam o nascimento e o desenvolvimento (Incluído pela Lei nº 13.257, de 2016)
sadio e harmonioso, em condições dignas de existência. § 2º Os serviços de unidades de terapia intensiva neonatal
deverão dispor de banco de leite humano ou unidade de coleta
Art. 8º É assegurado a todas as mulheres o acesso aos de leite humano. (Incluído pela Lei nº 13.257, de 2016)
programas e às políticas de saúde da mulher e de
planejamento reprodutivo e, às gestantes, nutrição adequada, Art. 10. Os hospitais e demais estabelecimentos de atenção
atenção humanizada à gravidez, ao parto e ao puerpério e à saúde de gestantes, públicos e particulares, são obrigados a:
atendimento pré-natal, perinatal e pós-natal integral no I - manter registro das atividades desenvolvidas, através
âmbito do Sistema Único de Saúde. (Redação dada pela Lei nº de prontuários individuais, pelo prazo de dezoito anos;
13.257, de 2016) II - identificar o recém-nascido mediante o registro de sua
§ 1º O atendimento pré-natal será realizado por impressão plantar e digital e da impressão digital da mãe, sem
profissionais da atenção primária. (Redação dada pela Lei nº prejuízo de outras formas normatizadas pela autoridade
13.257, de 2016) administrativa competente;
§ 2º Os profissionais de saúde de referência da gestante III - proceder a exames visando ao diagnóstico e
garantirão sua vinculação, no último trimestre da gestação, ao terapêutica de anormalidades no metabolismo do recém-
estabelecimento em que será realizado o parto, garantido o nascido, bem como prestar orientação aos pais;
direito de opção da mulher. (Redação dada pela Lei nº IV - fornecer declaração de nascimento onde constem
13.257, de 2016) necessariamente as intercorrências do parto e do
§ 3º Os serviços de saúde onde o parto for realizado desenvolvimento do neonato;
assegurarão às mulheres e aos seus filhos recém-nascidos alta V - manter alojamento conjunto, possibilitando ao neonato
hospitalar responsável e contrarreferência na atenção a permanência junto à mãe.
primária, bem como o acesso a outros serviços e a grupos de Art. 11. É assegurado acesso integral às linhas de cuidado
apoio à amamentação. (Redação dada pela Lei nº 13.257, de voltadas à saúde da criança e do adolescente, por intermédio
2016) do Sistema Único de Saúde, observado o princípio da equidade
§ 4º Incumbe ao poder público proporcionar assistência no acesso a ações e serviços para promoção, proteção e
psicológica à gestante e à mãe, no período pré e pós-natal, recuperação da saúde. (Redação dada pela Lei nº 13.257, de
inclusive como forma de prevenir ou minorar as 2016)
consequências do estado puerperal. § 1º A criança e o adolescente com deficiência serão
§ 5º A assistência referida no § 4o deste artigo deverá ser atendidos, sem discriminação ou segregação, em suas
prestada também a gestantes e mães que manifestem necessidades gerais de saúde e específicas de habilitação e
interesse em entregar seus filhos para adoção, bem como a reabilitação. (Redação dada pela Lei nº 13.257, de 2016)
gestantes e mães que se encontrem em situação de privação de § 2º Incumbe ao poder público fornecer gratuitamente,
liberdade. (Redação dada pela Lei nº 13.257, de 2016) àqueles que necessitarem, medicamentos, órteses, próteses e
§ 6º A gestante e a parturiente têm direito a 1 (um) outras tecnologias assistivas relativas ao tratamento,
acompanhante de sua preferência durante o período do pré- habilitação ou reabilitação para crianças e adolescentes, de
natal, do trabalho de parto e do pós-parto imediato. (Incluído acordo com as linhas de cuidado voltadas às suas necessidades
pela Lei nº 13.257, de 2016) específicas. (Redação dada pela Lei nº 13.257, de 2016)
§ 7º A gestante deverá receber orientação sobre § 3º Os profissionais que atuam no cuidado diário ou
aleitamento materno, alimentação complementar saudável e frequente de crianças na primeira infância receberão
crescimento e desenvolvimento infantil, bem como sobre formação específica e permanente para a detecção de sinais de
formas de favorecer a criação de vínculos afetivos e de risco para o desenvolvimento psíquico, bem como para o
estimular o desenvolvimento integral da criança. (Incluído acompanhamento que se fizer necessário. (Incluído pela Lei nº
pela Lei nº 13.257, de 2016) 13.257, de 2016)

Legislação 11
Apostila Digital Licenciada para Alice Caroline Guarino dos Santos - alice.guarino@hotmail.com (Proibida a Revenda)
APOSTILAS OPÇÃO

Art. 12. Os estabelecimentos de atendimento à saúde, VII - buscar refúgio, auxílio e orientação.
inclusive as unidades neonatais, de terapia intensiva e de
cuidados intermediários, deverão proporcionar condições Art. 17. O direito ao respeito consiste na inviolabilidade da
para a permanência em tempo integral de um dos pais ou integridade física, psíquica e moral da criança e do
responsável, nos casos de internação de criança ou adolescente, abrangendo a preservação da imagem, da
adolescente. (Redação dada pela Lei nº 13.257, de 2016) identidade, da autonomia, dos valores, ideias e crenças, dos
espaços e objetos pessoais.
Art. 13. Os casos de suspeita ou confirmação de castigo
físico, de tratamento cruel ou degradante e de maus-tratos Art. 18. É dever de todos velar pela dignidade da criança e
contra criança ou adolescente serão obrigatoriamente do adolescente, pondo-os a salvo de qualquer tratamento
comunicados ao Conselho Tutelar da respectiva localidade, desumano, violento, aterrorizante, vexatório ou
sem prejuízo de outras providências legais. (Redação dada constrangedor.
pela Lei nº 13.010, de 2014)
§ 1º As gestantes ou mães que manifestem interesse em Art. 18-A. A criança e o adolescente têm o direito de ser
entregar seus filhos para adoção serão obrigatoriamente educados e cuidados sem o uso de castigo físico ou de
encaminhadas, sem constrangimento, à Justiça da Infância e da tratamento cruel ou degradante, como formas de correção,
Juventude. (Incluído pela Lei nº 13.257, de 2016) disciplina, educação ou qualquer outro pretexto, pelos pais,
§ 2º Os serviços de saúde em suas diferentes portas de pelos integrantes da família ampliada, pelos responsáveis,
entrada, os serviços de assistência social em seu componente pelos agentes públicos executores de medidas socioeducativas
especializado, o Centro de Referência Especializado de ou por qualquer pessoa encarregada de cuidar deles, tratá-los,
Assistência Social (Creas) e os demais órgãos do Sistema de educá-los ou protegê-los. (Incluído pela Lei nº 13.010, de
Garantia de Direitos da Criança e do Adolescente deverão 2014)
conferir máxima prioridade ao atendimento das crianças na Parágrafo único. Para os fins desta Lei, considera-se:
faixa etária da primeira infância com suspeita ou confirmação (Incluído pela Lei nº 13.010, de 2014)
de violência de qualquer natureza, formulando projeto I - castigo físico: ação de natureza disciplinar ou punitiva
terapêutico singular que inclua intervenção em rede e, se aplicada com o uso da força física sobre a criança ou o
necessário, acompanhamento domiciliar. (Incluído pela Lei nº adolescente que resulte em: (Incluído pela Lei nº 13.010, de
13.257, de 2016) 2014)
a) sofrimento físico; ou (Incluído pela Lei nº 13.010, de
Art. 14. O Sistema Único de Saúde promoverá programas 2014)
de assistência médica e odontológica para a prevenção das b) lesão; (Incluído pela Lei nº 13.010, de 2014)
enfermidades que ordinariamente afetam a população infantil, II - tratamento cruel ou degradante: conduta ou forma
e campanhas de educação sanitária para pais, educadores e cruel de tratamento em relação à criança ou ao adolescente
alunos. que: (Incluído pela Lei nº 13.010, de 2014)
§ 1º É obrigatória a vacinação das crianças nos casos a) humilhe; ou (Incluído pela Lei nº 13.010, de 2014)
recomendados pelas autoridades sanitárias. (Renumerado b) ameace gravemente; ou (Incluído pela Lei nº 13.010, de
do parágrafo único pela Lei nº 13.257, de 2016) 2014)
§ 2º O Sistema Único de Saúde promoverá a atenção à c) ridicularize. (Incluído pela Lei nº 13.010, de 2014)
saúde bucal das crianças e das gestantes, de forma transversal,
integral e intersetorial com as demais linhas de cuidado Art. 18-B. Os pais, os integrantes da família ampliada, os
direcionadas à mulher e à criança. (Incluído pela Lei nº 13.257, responsáveis, os agentes públicos executores de medidas
de 2016) socioeducativas ou qualquer pessoa encarregada de cuidar de
§ 3º A atenção odontológica à criança terá função crianças e de adolescentes, tratá-los, educá-los ou protegê-los
educativa protetiva e será prestada, inicialmente, antes de o que utilizarem castigo físico ou tratamento cruel ou
bebê nascer, por meio de aconselhamento pré-natal, e, degradante como formas de correção, disciplina, educação ou
posteriormente, no sexto e no décimo segundo anos de vida, qualquer outro pretexto estarão sujeitos, sem prejuízo de
com orientações sobre saúde bucal. (Incluído pela Lei nº outras sanções cabíveis, às seguintes medidas, que serão
13.257, de 2016) aplicadas de acordo com a gravidade do caso: (Incluído
§ 4º A criança com necessidade de cuidados odontológicos pela Lei nº 13.010, de 2014)
especiais será atendida pelo Sistema Único de Saúde. (Incluído I - encaminhamento a programa oficial ou comunitário de
pela Lei nº 13.257, de 2016) proteção à família; Incluído pela Lei nº 13.010, de 2014)
II - encaminhamento a tratamento psicológico ou
Capítulo II psiquiátrico; (Incluído pela Lei nº 13.010, de 2014)
Do Direito à Liberdade, ao Respeito e à Dignidade III - encaminhamento a cursos ou programas de
orientação; (Incluído pela Lei nº 13.010, de 2014)
Art. 15. A criança e o adolescente têm direito à liberdade, IV - obrigação de encaminhar a criança a tratamento
ao respeito e à dignidade como pessoas humanas em processo especializado; (Incluído pela Lei nº 13.010, de 2014)
de desenvolvimento e como sujeitos de direitos civis, humanos V - advertência. (Incluído pela Lei nº 13.010, de 2014)
e sociais garantidos na Constituição e nas leis.
Parágrafo único. As medidas previstas neste artigo serão
Art. 16. O direito à liberdade compreende os seguintes aplicadas pelo Conselho Tutelar, sem prejuízo de outras
aspectos: providências legais. (Incluído pela Lei nº 13.010, de 2014)
I - ir, vir e estar nos logradouros públicos e espaços
comunitários, ressalvadas as restrições legais; Capítulo III
II - opinião e expressão; Do Direito à Convivência Familiar e Comunitária
III - crença e culto religioso; Seção I
IV - brincar, praticar esportes e divertir-se; Disposições Gerais
V - participar da vida familiar e comunitária, sem
discriminação; Art. 19. É direito da criança e do adolescente ser criado e
VI - participar da vida política, na forma da lei; educado no seio de sua família e, excepcionalmente, em família

Legislação 12
Apostila Digital Licenciada para Alice Caroline Guarino dos Santos - alice.guarino@hotmail.com (Proibida a Revenda)
APOSTILAS OPÇÃO

substituta, assegurada a convivência familiar e comunitária, descumprimento injustificado dos deveres e obrigações a que
em ambiente que garanta seu desenvolvimento integral. alude o art. 22.
(Redação dada pela Lei nº 13.257, de 2016)
§ 1º Toda criança ou adolescente que estiver inserido em Seção II
programa de acolhimento familiar ou institucional terá sua Da Família Natural
situação reavaliada, no máximo, a cada 6 (seis) meses,
devendo a autoridade judiciária competente, com base em Art. 25. Entende-se por família natural a comunidade
relatório elaborado por equipe interprofissional ou formada pelos pais ou qualquer deles e seus descendentes.
multidisciplinar, decidir de forma fundamentada pela Parágrafo único. Entende-se por família extensa ou
possibilidade de reintegração familiar ou colocação em família ampliada aquela que se estende para além da unidade pais e
substituta, em quaisquer das modalidades previstas no art. 28 filhos ou da unidade do casal, formada por parentes próximos
desta Lei. com os quais a criança ou adolescente convive e mantém
§ 2º A permanência da criança e do adolescente em vínculos de afinidade e afetividade.
programa de acolhimento institucional não se prolongará por
mais de 2 (dois) anos, salvo comprovada necessidade que Art. 26. Os filhos havidos fora do casamento poderão ser
atenda ao seu superior interesse, devidamente fundamentada reconhecidos pelos pais, conjunta ou separadamente, no
pela autoridade judiciária. próprio termo de nascimento, por testamento, mediante
§ 3º A manutenção ou a reintegração de criança ou escritura ou outro documento público, qualquer que seja a
adolescente à sua família terá preferência em relação a origem da filiação.
qualquer outra providência, caso em que será esta incluída em Parágrafo único. O reconhecimento pode preceder o
serviços e programas de proteção, apoio e promoção, nos nascimento do filho ou suceder-lhe ao falecimento, se deixar
termos do § 1º do art. 23, dos incisos I e IV do caput do art. 101 descendentes.
e dos incisos I a IV do caput do art. 129 desta Lei. (Redação
dada pela Lei nº 13.257, de 2016) Art. 27. O reconhecimento do estado de filiação é direito
§ 4º Será garantida a convivência da criança e do personalíssimo, indisponível e imprescritível, podendo ser
adolescente com a mãe ou o pai privado de liberdade, por meio exercitado contra os pais ou seus herdeiros, sem qualquer
de visitas periódicas promovidas pelo responsável ou, nas restrição, observado o segredo de Justiça.
hipóteses de acolhimento institucional, pela entidade
responsável, independentemente de autorização judicial. Seção III
(Incluído pela Lei nº 12.962, de 2014) Da Família Substituta
Subseção I
Art. 20. Os filhos, havidos ou não da relação do casamento, Disposições Gerais
ou por adoção, terão os mesmos direitos e qualificações,
proibidas quaisquer designações discriminatórias relativas à Art. 28. A colocação em família substituta far-se-á
filiação. mediante guarda, tutela ou adoção, independentemente da
situação jurídica da criança ou adolescente, nos termos desta
Art. 21. O poder familiar será exercido, em igualdade de Lei.
condições, pelo pai e pela mãe, na forma do que dispuser a § 1º Sempre que possível, a criança ou o adolescente será
legislação civil, assegurado a qualquer deles o direito de, em previamente ouvido por equipe interprofissional, respeitado
caso de discordância, recorrer à autoridade judiciária seu estágio de desenvolvimento e grau de compreensão sobre
competente para a solução da divergência. as implicações da medida, e terá sua opinião devidamente
considerada.
Art. 22. Aos pais incumbe o dever de sustento, guarda e § 2º Tratando-se de maior de 12 (doze) anos de idade, será
educação dos filhos menores, cabendo-lhes ainda, no interesse necessário seu consentimento, colhido em audiência.
destes, a obrigação de cumprir e fazer cumprir as § 3º Na apreciação do pedido levar-se-á em conta o grau de
determinações judiciais. parentesco e a relação de afinidade ou de afetividade, a fim de
Parágrafo único. A mãe e o pai, ou os responsáveis, têm evitar ou minorar as consequências decorrentes da medida.
direitos iguais e deveres e responsabilidades compartilhados § 4º Os grupos de irmãos serão colocados sob adoção,
no cuidado e na educação da criança, devendo ser resguardado tutela ou guarda da mesma família substituta, ressalvada a
o direito de transmissão familiar de suas crenças e culturas, comprovada existência de risco de abuso ou outra situação que
assegurados os direitos da criança estabelecidos nesta Lei. justifique plenamente a excepcionalidade de solução diversa,
(Incluído pela Lei nº 13.257, de 2016) procurando-se, em qualquer caso, evitar o rompimento
definitivo dos vínculos fraternais.
Art. 23. A falta ou a carência de recursos materiais não § 5º A colocação da criança ou adolescente em família
constitui motivo suficiente para a perda ou a suspensão do substituta será precedida de sua preparação gradativa e
poder familiar. acompanhamento posterior, realizados pela equipe
§ 1º Não existindo outro motivo que por si só autorize a interprofissional a serviço da Justiça da Infância e da
decretação da medida, a criança ou o adolescente será mantido Juventude, preferencialmente com o apoio dos técnicos
em sua família de origem, a qual deverá obrigatoriamente ser responsáveis pela execução da política municipal de garantia
incluída em serviços e programas oficiais de proteção, apoio e do direito à convivência familiar.
promoção. (Redação dada pela Lei nº 13.257, de 2016) § 6º Em se tratando de criança ou adolescente indígena ou
§ 2º A condenação criminal do pai ou da mãe não implicará proveniente de comunidade remanescente de quilombo, é
a destituição do poder familiar, exceto na hipótese de ainda obrigatório:
condenação por crime doloso, sujeito à pena de reclusão, I - que sejam consideradas e respeitadas sua identidade
contra o próprio filho ou filha. (Incluído pela Lei nº 12.962, social e cultural, os seus costumes e tradições, bem como suas
de 2014) instituições, desde que não sejam incompatíveis com os
direitos fundamentais reconhecidos por esta Lei e pela
Art. 24. A perda e a suspensão do poder familiar serão Constituição Federal;
decretadas judicialmente, em procedimento contraditório, nos II - que a colocação familiar ocorra prioritariamente no
casos previstos na legislação civil, bem como na hipótese de seio de sua comunidade ou junto a membros da mesma etnia;

Legislação 13
Apostila Digital Licenciada para Alice Caroline Guarino dos Santos - alice.guarino@hotmail.com (Proibida a Revenda)
APOSTILAS OPÇÃO

III - a intervenção e oitiva de representantes do órgão estejam no cadastro de adoção. (Incluído pela Lei nº 13.257, de
federal responsável pela política indigenista, no caso de 2016)
crianças e adolescentes indígenas, e de antropólogos, perante § 4º Poderão ser utilizados recursos federais, estaduais,
a equipe interprofissional ou multidisciplinar que irá distritais e municipais para a manutenção dos serviços de
acompanhar o caso. acolhimento em família acolhedora, facultando-se o repasse de
recursos para a própria família acolhedora. (Incluído pela Lei
Art. 29. Não se deferirá colocação em família substituta a nº 13.257, de 2016)
pessoa que revele, por qualquer modo, incompatibilidade com
a natureza da medida ou não ofereça ambiente familiar Art. 35. A guarda poderá ser revogada a qualquer tempo,
adequado. mediante ato judicial fundamentado, ouvido o Ministério
Público.
Art. 30. A colocação em família substituta não admitirá
transferência da criança ou adolescente a terceiros ou a Subseção III
entidades governamentais ou não-governamentais, sem Da Tutela
autorização judicial.
Art. 36. A tutela será deferida, nos termos da lei civil, a
Art. 31. A colocação em família substituta estrangeira pessoa de até 18 (dezoito) anos incompletos.
constitui medida excepcional, somente admissível na Parágrafo único. O deferimento da tutela pressupõe a
modalidade de adoção. prévia decretação da perda ou suspensão do poder familiar e
implica necessariamente o dever de guarda.
Art. 32. Ao assumir a guarda ou a tutela, o responsável
prestará compromisso de bem e fielmente desempenhar o Art. 37. O tutor nomeado por testamento ou qualquer
encargo, mediante termo nos autos. documento autêntico, conforme previsto no parágrafo único
do art. 1.729 da Lei no 10.406, de 10 de janeiro de 2002 -
Subseção II Código Civil, deverá, no prazo de 30 (trinta) dias após a
Da Guarda abertura da sucessão, ingressar com pedido destinado ao
controle judicial do ato, observando o procedimento previsto
Art. 33. A guarda obriga a prestação de assistência nos arts. 165 a 170 desta Lei.
material, moral e educacional à criança ou adolescente, Parágrafo único. Na apreciação do pedido, serão
conferindo a seu detentor o direito de opor-se a terceiros, observados os requisitos previstos nos arts. 28 e 29 desta Lei,
inclusive aos pais. somente sendo deferida a tutela à pessoa indicada na
§ 1º A guarda destina-se a regularizar a posse de fato, disposição de última vontade, se restar comprovado que a
podendo ser deferida, liminar ou incidentalmente, nos medida é vantajosa ao tutelando e que não existe outra pessoa
procedimentos de tutela e adoção, exceto no de adoção por em melhores condições de assumi-la.
estrangeiros.
§ 2º Excepcionalmente, deferir-se-á a guarda, fora dos Art. 38. Aplica-se à destituição da tutela o disposto no art.
casos de tutela e adoção, para atender a situações peculiares 24.
ou suprir a falta eventual dos pais ou responsável, podendo ser
deferido o direito de representação para a prática de atos Subseção IV
determinados. Da Adoção
§ 3º A guarda confere à criança ou adolescente a condição
de dependente, para todos os fins e efeitos de direito, inclusive Art. 39. A adoção de criança e de adolescente reger-se-á
previdenciários. segundo o disposto nesta Lei.
§ 4º Salvo expressa e fundamentada determinação em § 1º A adoção é medida excepcional e irrevogável, à qual se
contrário, da autoridade judiciária competente, ou quando a deve recorrer apenas quando esgotados os recursos de
medida for aplicada em preparação para adoção, o manutenção da criança ou adolescente na família natural ou
deferimento da guarda de criança ou adolescente a terceiros extensa, na forma do parágrafo único do art. 25 desta Lei.
não impede o exercício do direito de visitas pelos pais, assim § 2º É vedada a adoção por procuração.
como o dever de prestar alimentos, que serão objeto de
regulamentação específica, a pedido do interessado ou do Art. 40. O adotando deve contar com, no máximo, dezoito
Ministério Público. anos à data do pedido, salvo se já estiver sob a guarda ou tutela
dos adotantes.
Art. 34. O poder público estimulará, por meio de
assistência jurídica, incentivos fiscais e subsídios, o Art. 41. A adoção atribui a condição de filho ao adotado,
acolhimento, sob a forma de guarda, de criança ou adolescente com os mesmos direitos e deveres, inclusive sucessórios,
afastado do convívio familiar. desligando-o de qualquer vínculo com pais e parentes, salvo os
§ 1º A inclusão da criança ou adolescente em programas de impedimentos matrimoniais.
acolhimento familiar terá preferência a seu acolhimento § 1º Se um dos cônjuges ou concubinos adota o filho do
institucional, observado, em qualquer caso, o caráter outro, mantêm-se os vínculos de filiação entre o adotado e o
temporário e excepcional da medida, nos termos desta Lei. cônjuge ou concubino do adotante e os respectivos parentes.
§ 2º Na hipótese do § 1o deste artigo a pessoa ou casal § 2º É recíproco o direito sucessório entre o adotado, seus
cadastrado no programa de acolhimento familiar poderá descendentes, o adotante, seus ascendentes, descendentes e
receber a criança ou adolescente mediante guarda, observado colaterais até o 4º grau, observada a ordem de vocação
o disposto nos arts. 28 a 33 desta Lei. hereditária.
§ 3º A União apoiará a implementação de serviços de
acolhimento em família acolhedora como política pública, os Art. 42. Podem adotar os maiores de 18 (dezoito) anos,
quais deverão dispor de equipe que organize o acolhimento independentemente do estado civil.
temporário de crianças e de adolescentes em residências de § 1º Não podem adotar os ascendentes e os irmãos do
famílias selecionadas, capacitadas e acompanhadas que não adotando.

Legislação 14
Apostila Digital Licenciada para Alice Caroline Guarino dos Santos - alice.guarino@hotmail.com (Proibida a Revenda)
APOSTILAS OPÇÃO

§ 2º Para adoção conjunta, é indispensável que os § 6º Caso a modificação de prenome seja requerida pelo
adotantes sejam casados civilmente ou mantenham união adotante, é obrigatória a oitiva do adotando, observado o
estável, comprovada a estabilidade da família. disposto nos §§ 1o e 2o do art. 28 desta Lei.
§ 3º O adotante há de ser, pelo menos, dezesseis anos mais § 7º A adoção produz seus efeitos a partir do trânsito em
velho do que o adotando. julgado da sentença constitutiva, exceto na hipótese prevista
§ 4º Os divorciados, os judicialmente separados e os ex- no § 6o do art. 42 desta Lei, caso em que terá força retroativa
companheiros podem adotar conjuntamente, contanto que à data do óbito.
acordem sobre a guarda e o regime de visitas e desde que o § 8º O processo relativo à adoção assim como outros a ele
estágio de convivência tenha sido iniciado na constância do relacionados serão mantidos em arquivo, admitindo-se seu
período de convivência e que seja comprovada a existência de armazenamento em microfilme ou por outros meios, garantida
vínculos de afinidade e afetividade com aquele não detentor da a sua conservação para consulta a qualquer tempo.
guarda, que justifiquem a excepcionalidade da concessão. § 9º Terão prioridade de tramitação os processos de
§ 5º Nos casos do § 4o deste artigo, desde que adoção em que o adotando for criança ou adolescente com
demonstrado efetivo benefício ao adotando, será assegurada a deficiência ou com doença crônica. (Incluído pela Lei nº
guarda compartilhada, conforme previsto no art. 1.584 da Lei 12.955, de 2014)
no 10.406, de 10 de janeiro de 2002 - Código Civil.
§ 6º A adoção poderá ser deferida ao adotante que, após Art. 48. O adotado tem direito de conhecer sua origem
inequívoca manifestação de vontade, vier a falecer no curso do biológica, bem como de obter acesso irrestrito ao processo no
procedimento, antes de prolatada a sentença. qual a medida foi aplicada e seus eventuais incidentes, após
completar 18 (dezoito) anos.
Art. 43. A adoção será deferida quando apresentar reais Parágrafo único. O acesso ao processo de adoção poderá
vantagens para o adotando e fundar-se em motivos legítimos. ser também deferido ao adotado menor de 18 (dezoito) anos,
a seu pedido, assegurada orientação e assistência jurídica e
Art. 44. Enquanto não der conta de sua administração e psicológica.
saldar o seu alcance, não pode o tutor ou o curador adotar o
pupilo ou o curatelado. Art. 49. A morte dos adotantes não restabelece o poder
familiar dos pais naturais.
Art. 45. A adoção depende do consentimento dos pais ou
do representante legal do adotando. Art. 50. A autoridade judiciária manterá, em cada comarca
§ 1º. O consentimento será dispensado em relação à ou foro regional, um registro de crianças e adolescentes em
criança ou adolescente cujos pais sejam desconhecidos ou condições de serem adotados e outro de pessoas interessadas
tenham sido destituídos do poder familiar. na adoção.
§ 2º. Em se tratando de adotando maior de doze anos de § 1º O deferimento da inscrição dar-se-á após prévia
idade, será também necessário o seu consentimento. consulta aos órgãos técnicos do juizado, ouvido o Ministério
Público.
Art. 46. A adoção será precedida de estágio de convivência § 2º Não será deferida a inscrição se o interessado não
com a criança ou adolescente, pelo prazo que a autoridade satisfazer os requisitos legais, ou verificada qualquer das
judiciária fixar, observadas as peculiaridades do caso. hipóteses previstas no art. 29.
§ 1º O estágio de convivência poderá ser dispensado se o § 3º A inscrição de postulantes à adoção será precedida de
adotando já estiver sob a tutela ou guarda legal do adotante um período de preparação psicossocial e jurídica, orientado
durante tempo suficiente para que seja possível avaliar a pela equipe técnica da Justiça da Infância e da Juventude,
conveniência da constituição do vínculo. preferencialmente com apoio dos técnicos responsáveis pela
§ 2º A simples guarda de fato não autoriza, por si só, a execução da política municipal de garantia do direito à
dispensa da realização do estágio de convivência. convivência familiar.
§ 3º Em caso de adoção por pessoa ou casal residente ou § 4º Sempre que possível e recomendável, a preparação
domiciliado fora do País, o estágio de convivência, cumprido referida no § 3o deste artigo incluirá o contato com crianças e
no território nacional, será de, no mínimo, 30 (trinta) dias. adolescentes em acolhimento familiar ou institucional em
§ 4º O estágio de convivência será acompanhado pela condições de serem adotados, a ser realizado sob a orientação,
equipe interprofissional a serviço da Justiça da Infância e da supervisão e avaliação da equipe técnica da Justiça da Infância
Juventude, preferencialmente com apoio dos técnicos e da Juventude, com apoio dos técnicos responsáveis pelo
responsáveis pela execução da política de garantia do direito à programa de acolhimento e pela execução da política
convivência familiar, que apresentarão relatório minucioso municipal de garantia do direito à convivência familiar.
acerca da conveniência do deferimento da medida. § 5º Serão criados e implementados cadastros estaduais e
nacional de crianças e adolescentes em condições de serem
Art. 47. O vínculo da adoção constitui-se por sentença adotados e de pessoas ou casais habilitados à adoção.
judicial, que será inscrita no registro civil mediante mandado § 6º Haverá cadastros distintos para pessoas ou casais
do qual não se fornecerá certidão. residentes fora do País, que somente serão consultados na
§ 1º A inscrição consignará o nome dos adotantes como inexistência de postulantes nacionais habilitados nos
pais, bem como o nome de seus ascendentes. cadastros mencionados no § 5o deste artigo.
§ 2º O mandado judicial, que será arquivado, cancelará o § 7º As autoridades estaduais e federais em matéria de
registro original do adotado. adoção terão acesso integral aos cadastros, incumbindo-lhes a
§ 3º A pedido do adotante, o novo registro poderá ser troca de informações e a cooperação mútua, para melhoria do
lavrado no Cartório do Registro Civil do Município de sua sistema.
residência. § 8º A autoridade judiciária providenciará, no prazo de 48
§ 4º Nenhuma observação sobre a origem do ato poderá (quarenta e oito) horas, a inscrição das crianças e adolescentes
constar nas certidões do registro. em condições de serem adotados que não tiveram colocação
§ 5º A sentença conferirá ao adotado o nome do adotante familiar na comarca de origem, e das pessoas ou casais que
e, a pedido de qualquer deles, poderá determinar a tiveram deferida sua habilitação à adoção nos cadastros
modificação do prenome. estadual e nacional referidos no § 5o deste artigo, sob pena de
responsabilidade.

Legislação 15
Apostila Digital Licenciada para Alice Caroline Guarino dos Santos - alice.guarino@hotmail.com (Proibida a Revenda)
APOSTILAS OPÇÃO

§ 9º Compete à Autoridade Central Estadual zelar pela II - se a Autoridade Central do país de acolhida considerar
manutenção e correta alimentação dos cadastros, com que os solicitantes estão habilitados e aptos para adotar,
posterior comunicação à Autoridade Central Federal emitirá um relatório que contenha informações sobre a
Brasileira. identidade, a capacidade jurídica e adequação dos solicitantes
§ 10. A adoção internacional somente será deferida se, para adotar, sua situação pessoal, familiar e médica, seu meio
após consulta ao cadastro de pessoas ou casais habilitados à social, os motivos que os animam e sua aptidão para assumir
adoção, mantido pela Justiça da Infância e da Juventude na uma adoção internacional;
comarca, bem como aos cadastros estadual e nacional III - a Autoridade Central do país de acolhida enviará o
referidos no § 5o deste artigo, não for encontrado interessado relatório à Autoridade Central Estadual, com cópia para a
com residência permanente no Brasil. Autoridade Central Federal Brasileira;
§ 11. Enquanto não localizada pessoa ou casal interessado IV - o relatório será instruído com toda a documentação
em sua adoção, a criança ou o adolescente, sempre que necessária, incluindo estudo psicossocial elaborado por
possível e recomendável, será colocado sob guarda de família equipe interprofissional habilitada e cópia autenticada da
cadastrada em programa de acolhimento familiar. legislação pertinente, acompanhada da respectiva prova de
§ 12. A alimentação do cadastro e a convocação criteriosa vigência;
dos postulantes à adoção serão fiscalizadas pelo Ministério V - os documentos em língua estrangeira serão
Público. devidamente autenticados pela autoridade consular,
§ 13. Somente poderá ser deferida adoção em favor de observados os tratados e convenções internacionais, e
candidato domiciliado no Brasil não cadastrado previamente acompanhados da respectiva tradução, por tradutor público
nos termos desta Lei quando: juramentado;
I - se tratar de pedido de adoção unilateral; VI - a Autoridade Central Estadual poderá fazer exigências
II - for formulada por parente com o qual a criança ou e solicitar complementação sobre o estudo psicossocial do
adolescente mantenha vínculos de afinidade e afetividade; postulante estrangeiro à adoção, já realizado no país de
III - oriundo o pedido de quem detém a tutela ou guarda acolhida;
legal de criança maior de 3 (três) anos ou adolescente, desde VII - verificada, após estudo realizado pela Autoridade
que o lapso de tempo de convivência comprove a fixação de Central Estadual, a compatibilidade da legislação estrangeira
laços de afinidade e afetividade, e não seja constatada a com a nacional, além do preenchimento por parte dos
ocorrência de má-fé ou qualquer das situações previstas nos postulantes à medida dos requisitos objetivos e subjetivos
arts. 237 ou 238 desta Lei. necessários ao seu deferimento, tanto à luz do que dispõe esta
§ 14. Nas hipóteses previstas no § 13 deste artigo, o Lei como da legislação do país de acolhida, será expedido laudo
candidato deverá comprovar, no curso do procedimento, que de habilitação à adoção internacional, que terá validade por,
preenche os requisitos necessários à adoção, conforme no máximo, 1 (um) ano;
previsto nesta Lei. VIII - de posse do laudo de habilitação, o interessado será
autorizado a formalizar pedido de adoção perante o Juízo da
Art. 51. Considera-se adoção internacional aquela na qual Infância e da Juventude do local em que se encontra a criança
a pessoa ou casal postulante é residente ou domiciliado fora do ou adolescente, conforme indicação efetuada pela Autoridade
Brasil, conforme previsto no Artigo 2 da Convenção de Haia, de Central Estadual.
29 de maio de 1993, Relativa à Proteção das Crianças e à § 1º Se a legislação do país de acolhida assim o autorizar,
Cooperação em Matéria de Adoção Internacional, aprovada admite-se que os pedidos de habilitação à adoção
pelo Decreto Legislativo no 1, de 14 de janeiro de 1999, e internacional sejam intermediados por organismos
promulgada pelo Decreto no 3.087, de 21 de junho de 1999. credenciados.
§ 1º A adoção internacional de criança ou adolescente § 2º Incumbe à Autoridade Central Federal Brasileira o
brasileiro ou domiciliado no Brasil somente terá lugar quando credenciamento de organismos nacionais e estrangeiros
restar comprovado: encarregados de intermediar pedidos de habilitação à adoção
I - que a colocação em família substituta é a solução internacional, com posterior comunicação às Autoridades
adequada ao caso concreto; Centrais Estaduais e publicação nos órgãos oficiais de
II - que foram esgotadas todas as possibilidades de imprensa e em sítio próprio da internet.
colocação da criança ou adolescente em família substituta § 3º Somente será admissível o credenciamento de
brasileira, após consulta aos cadastros mencionados no art. 50 organismos que:
desta Lei; I - sejam oriundos de países que ratificaram a Convenção
III - que, em se tratando de adoção de adolescente, este foi de Haia e estejam devidamente credenciados pela Autoridade
consultado, por meios adequados ao seu estágio de Central do país onde estiverem sediados e no país de acolhida
desenvolvimento, e que se encontra preparado para a medida, do adotando para atuar em adoção internacional no Brasil;
mediante parecer elaborado por equipe interprofissional, II - satisfizerem as condições de integridade moral,
observado o disposto nos §§ 1o e 2o do art. 28 desta Lei. competência profissional, experiência e responsabilidade
§ 2º Os brasileiros residentes no exterior terão preferência exigidas pelos países respectivos e pela Autoridade Central
aos estrangeiros, nos casos de adoção internacional de criança Federal Brasileira;
ou adolescente brasileiro. III - forem qualificados por seus padrões éticos e sua
§ 3º A adoção internacional pressupõe a intervenção das formação e experiência para atuar na área de adoção
Autoridades Centrais Estaduais e Federal em matéria de internacional;
adoção internacional. IV - cumprirem os requisitos exigidos pelo ordenamento
jurídico brasileiro e pelas normas estabelecidas pela
Art. 52. A adoção internacional observará o procedimento Autoridade Central Federal Brasileira.
previsto nos arts. 165 a 170 desta Lei, com as seguintes § 4º Os organismos credenciados deverão ainda:
adaptações: I - perseguir unicamente fins não lucrativos, nas condições
I - a pessoa ou casal estrangeiro, interessado em adotar e dentro dos limites fixados pelas autoridades competentes do
criança ou adolescente brasileiro, deverá formular pedido de país onde estiverem sediados, do país de acolhida e pela
habilitação à adoção perante a Autoridade Central em matéria Autoridade Central Federal Brasileira;
de adoção internacional no país de acolhida, assim entendido II - ser dirigidos e administrados por pessoas qualificadas
aquele onde está situada sua residência habitual; e de reconhecida idoneidade moral, com comprovada

Legislação 16
Apostila Digital Licenciada para Alice Caroline Guarino dos Santos - alice.guarino@hotmail.com (Proibida a Revenda)
APOSTILAS OPÇÃO

formação ou experiência para atuar na área de adoção Art. 52-A. É vedado, sob pena de responsabilidade e
internacional, cadastradas pelo Departamento de Polícia descredenciamento, o repasse de recursos provenientes de
Federal e aprovadas pela Autoridade Central Federal organismos estrangeiros encarregados de intermediar
Brasileira, mediante publicação de portaria do órgão federal pedidos de adoção internacional a organismos nacionais ou a
competente; pessoas físicas.
III - estar submetidos à supervisão das autoridades Parágrafo único. Eventuais repasses somente poderão ser
competentes do país onde estiverem sediados e no país de efetuados via Fundo dos Direitos da Criança e do Adolescente
acolhida, inclusive quanto à sua composição, funcionamento e e estarão sujeitos às deliberações do respectivo Conselho de
situação financeira; Direitos da Criança e do Adolescente.
IV - apresentar à Autoridade Central Federal Brasileira, a
cada ano, relatório geral das atividades desenvolvidas, bem Art. 52-B. A adoção por brasileiro residente no exterior em
como relatório de acompanhamento das adoções país ratificante da Convenção de Haia, cujo processo de adoção
internacionais efetuadas no período, cuja cópia será tenha sido processado em conformidade com a legislação
encaminhada ao Departamento de Polícia Federal; vigente no país de residência e atendido o disposto na Alínea
V - enviar relatório pós-adotivo semestral para a “c” do Artigo 17 da referida Convenção, será automaticamente
Autoridade Central Estadual, com cópia para a Autoridade recepcionada com o reingresso no Brasil.
Central Federal Brasileira, pelo período mínimo de 2 (dois) § 1º Caso não tenha sido atendido o disposto na Alínea “c”
anos. O envio do relatório será mantido até a juntada de cópia do Artigo 17 da Convenção de Haia, deverá a sentença ser
autenticada do registro civil, estabelecendo a cidadania do país homologada pelo Superior Tribunal de Justiça.
de acolhida para o adotado; § 2º O pretendente brasileiro residente no exterior em país
VI - tomar as medidas necessárias para garantir que os não ratificante da Convenção de Haia, uma vez reingressado no
adotantes encaminhem à Autoridade Central Federal Brasil, deverá requerer a homologação da sentença
Brasileira cópia da certidão de registro de nascimento estrangeira pelo Superior Tribunal de Justiça.
estrangeira e do certificado de nacionalidade tão logo lhes
sejam concedidos. Art. 52-C. Nas adoções internacionais, quando o Brasil for
§ 5º A não apresentação dos relatórios referidos no § 4o o país de acolhida, a decisão da autoridade competente do país
deste artigo pelo organismo credenciado poderá acarretar a de origem da criança ou do adolescente será conhecida pela
suspensão de seu credenciamento. Autoridade Central Estadual que tiver processado o pedido de
§ 6º O credenciamento de organismo nacional ou habilitação dos pais adotivos, que comunicará o fato à
estrangeiro encarregado de intermediar pedidos de adoção Autoridade Central Federal e determinará as providências
internacional terá validade de 2 (dois) anos. necessárias à expedição do Certificado de Naturalização
§ 7º A renovação do credenciamento poderá ser concedida Provisório.
mediante requerimento protocolado na Autoridade Central § 1º A Autoridade Central Estadual, ouvido o Ministério
Federal Brasileira nos 60 (sessenta) dias anteriores ao Público, somente deixará de reconhecer os efeitos daquela
término do respectivo prazo de validade. decisão se restar demonstrado que a adoção é manifestamente
§ 8º Antes de transitada em julgado a decisão que contrária à ordem pública ou não atende ao interesse superior
concedeu a adoção internacional, não será permitida a saída da criança ou do adolescente.
do adotando do território nacional. § 2º Na hipótese de não reconhecimento da adoção,
§ 9º Transitada em julgado a decisão, a autoridade prevista no § 1o deste artigo, o Ministério Público deverá
judiciária determinará a expedição de alvará com autorização imediatamente requerer o que for de direito para resguardar
de viagem, bem como para obtenção de passaporte, constando, os interesses da criança ou do adolescente, comunicando-se as
obrigatoriamente, as características da criança ou adolescente providências à Autoridade Central Estadual, que fará a
adotado, como idade, cor, sexo, eventuais sinais ou traços comunicação à Autoridade Central Federal Brasileira e à
peculiares, assim como foto recente e a aposição da impressão Autoridade Central do país de origem.
digital do seu polegar direito, instruindo o documento com
cópia autenticada da decisão e certidão de trânsito em julgado. Art. 52-D. Nas adoções internacionais, quando o Brasil for
§ 10. A Autoridade Central Federal Brasileira poderá, a o país de acolhida e a adoção não tenha sido deferida no país
qualquer momento, solicitar informações sobre a situação das de origem porque a sua legislação a delega ao país de acolhida,
crianças e adolescentes adotados. ou, ainda, na hipótese de, mesmo com decisão, a criança ou o
§ 11. A cobrança de valores por parte dos organismos adolescente ser oriundo de país que não tenha aderido à
credenciados, que sejam considerados abusivos pela Convenção referida, o processo de adoção seguirá as regras da
Autoridade Central Federal Brasileira e que não estejam adoção nacional.
devidamente comprovados, é causa de seu
descredenciamento. Capítulo IV
§ 12. Uma mesma pessoa ou seu cônjuge não podem ser Do Direito à Educação, à Cultura, ao Esporte e ao
representados por mais de uma entidade credenciada para Lazer
atuar na cooperação em adoção internacional.
§ 13. A habilitação de postulante estrangeiro ou Art. 53. A criança e o adolescente têm direito à educação,
domiciliado fora do Brasil terá validade máxima de 1 (um) ano, visando ao pleno desenvolvimento de sua pessoa, preparo
podendo ser renovada. para o exercício da cidadania e qualificação para o trabalho,
§ 14. É vedado o contato direto de representantes de assegurando-se-lhes:
organismos de adoção, nacionais ou estrangeiros, com I - igualdade de condições para o acesso e permanência na
dirigentes de programas de acolhimento institucional ou escola;
familiar, assim como com crianças e adolescentes em II - direito de ser respeitado por seus educadores;
condições de serem adotados, sem a devida autorização III - direito de contestar critérios avaliativos, podendo
judicial. recorrer às instâncias escolares superiores;
§ 15. A Autoridade Central Federal Brasileira poderá IV - direito de organização e participação em entidades
limitar ou suspender a concessão de novos credenciamentos estudantis;
sempre que julgar necessário, mediante ato administrativo V - acesso à escola pública e gratuita próxima de sua
fundamentado. residência.

Legislação 17
Apostila Digital Licenciada para Alice Caroline Guarino dos Santos - alice.guarino@hotmail.com (Proibida a Revenda)
APOSTILAS OPÇÃO

Parágrafo único. É direito dos pais ou responsáveis ter Art. 62. Considera-se aprendizagem a formação técnico-
ciência do processo pedagógico, bem como participar da profissional ministrada segundo as diretrizes e bases da
definição das propostas educacionais. legislação de educação em vigor.

Art. 54. É dever do Estado assegurar à criança e ao Art. 63. A formação técnico-profissional obedecerá aos
adolescente: seguintes princípios:
I - ensino fundamental, obrigatório e gratuito, inclusive I - garantia de acesso e frequência obrigatória ao ensino
para os que a ele não tiveram acesso na idade própria; regular;
II - progressiva extensão da obrigatoriedade e gratuidade II - atividade compatível com o desenvolvimento do
ao ensino médio; adolescente;
III - atendimento educacional especializado aos portadores III - horário especial para o exercício das atividades.
de deficiência, preferencialmente na rede regular de ensino;
IV – atendimento em creche e pré-escola às crianças de Art. 64. Ao adolescente até quatorze anos de idade é
zero a cinco anos de idade; (Redação dada pela Lei nº 13.306, assegurada bolsa de aprendizagem.
de 2016)
V - acesso aos níveis mais elevados do ensino, da pesquisa Art. 65. Ao adolescente aprendiz, maior de quatorze anos,
e da criação artística, segundo a capacidade de cada um; são assegurados os direitos trabalhistas e previdenciários.
VI - oferta de ensino noturno regular, adequado às
condições do adolescente trabalhador; Art. 66. Ao adolescente portador de deficiência é
VII - atendimento no ensino fundamental, através de assegurado trabalho protegido.
programas suplementares de material didático-escolar,
transporte, alimentação e assistência à saúde. Art. 67. Ao adolescente empregado, aprendiz, em regime
§ 1º O acesso ao ensino obrigatório e gratuito é direito familiar de trabalho, aluno de escola técnica, assistido em
público subjetivo. entidade governamental ou não-governamental, é vedado
§ 2º O não oferecimento do ensino obrigatório pelo poder trabalho:
público ou sua oferta irregular importa responsabilidade da I - noturno, realizado entre as vinte e duas horas de um dia
autoridade competente. e as cinco horas do dia seguinte;
§ 3º Compete ao poder público recensear os educandos no II - perigoso, insalubre ou penoso;
ensino fundamental, fazer-lhes a chamada e zelar, junto aos III - realizado em locais prejudiciais à sua formação e ao
pais ou responsável, pela frequência à escola. seu desenvolvimento físico, psíquico, moral e social;
IV - realizado em horários e locais que não permitam a
Art. 55. Os pais ou responsável têm a obrigação de frequência à escola.
matricular seus filhos ou pupilos na rede regular de ensino. Art. 68. O programa social que tenha por base o trabalho
educativo, sob responsabilidade de entidade governamental
Art. 56. Os dirigentes de estabelecimentos de ensino ou não-governamental sem fins lucrativos, deverá assegurar
fundamental comunicarão ao Conselho Tutelar os casos de: ao adolescente que dele participe condições de capacitação
I - maus-tratos envolvendo seus alunos; para o exercício de atividade regular remunerada.
II - reiteração de faltas injustificadas e de evasão escolar, § 1º Entende-se por trabalho educativo a atividade laboral
esgotados os recursos escolares; em que as exigências pedagógicas relativas ao
III - elevados níveis de repetência. desenvolvimento pessoal e social do educando prevalecem
sobre o aspecto produtivo.
Art. 57. O poder público estimulará pesquisas, experiências § 2º A remuneração que o adolescente recebe pelo
e novas propostas relativas a calendário, seriação, currículo, trabalho efetuado ou a participação na venda dos produtos de
metodologia, didática e avaliação, com vistas à inserção de seu trabalho não desfigura o caráter educativo.
crianças e adolescentes excluídos do ensino fundamental
obrigatório. Art. 69. O adolescente tem direito à profissionalização e à
proteção no trabalho, observados os seguintes aspectos, entre
Art. 58. No processo educacional respeitar-se-ão os valores outros:
culturais, artísticos e históricos próprios do contexto social da I - respeito à condição peculiar de pessoa em
criança e do adolescente, garantindo-se a estes a liberdade da desenvolvimento;
criação e o acesso às fontes de cultura. II - capacitação profissional adequada ao mercado de
trabalho.
Art. 59. Os municípios, com apoio dos estados e da União,
estimularão e facilitarão a destinação de recursos e espaços Título III
para programações culturais, esportivas e de lazer voltadas Da Prevenção
para a infância e a juventude. Capítulo I
Disposições Gerais
Capítulo V
Do Direito à Profissionalização e à Proteção no Art. 70. É dever de todos prevenir a ocorrência de ameaça
Trabalho ou violação dos direitos da criança e do adolescente.

Art. 60. É proibido qualquer trabalho a menores de Art. 70-A. A União, os Estados, o Distrito Federal e os
quatorze anos de idade, salvo na condição de aprendiz. (Vide Municípios deverão atuar de forma articulada na elaboração
Constituição Federal) de políticas públicas e na execução de ações destinadas a coibir
o uso de castigo físico ou de tratamento cruel ou degradante e
Art. 61. A proteção ao trabalho dos adolescentes é regulada difundir formas não violentas de educação de crianças e de
por legislação especial, sem prejuízo do disposto nesta Lei. adolescentes, tendo como principais ações: (Incluído pela Lei
nº 13.010, de 2014)
I - a promoção de campanhas educativas permanentes
para a divulgação do direito da criança e do adolescente de

Legislação 18
Apostila Digital Licenciada para Alice Caroline Guarino dos Santos - alice.guarino@hotmail.com (Proibida a Revenda)
APOSTILAS OPÇÃO

serem educados e cuidados sem o uso de castigo físico ou de Capítulo II


tratamento cruel ou degradante e dos instrumentos de Da Prevenção Especial
proteção aos direitos humanos; (Incluído pela Lei nº 13.010, Seção I
de 2014) Da informação, Cultura, Lazer, Esportes, Diversões e
II - a integração com os órgãos do Poder Judiciário, do Espetáculos
Ministério Público e da Defensoria Pública, com o Conselho
Tutelar, com os Conselhos de Direitos da Criança e do Art. 74. O poder público, através do órgão competente,
Adolescente e com as entidades não governamentais que regulará as diversões e espetáculos públicos, informando
atuam na promoção, proteção e defesa dos direitos da criança sobre a natureza deles, as faixas etárias a que não se
e do adolescente; (Incluído pela Lei nº 13.010, de 2014) recomendem, locais e horários em que sua apresentação se
III - a formação continuada e a capacitação dos mostre inadequada.
profissionais de saúde, educação e assistência social e dos Parágrafo único. Os responsáveis pelas diversões e
demais agentes que atuam na promoção, proteção e defesa dos espetáculos públicos deverão afixar, em lugar visível e de fácil
direitos da criança e do adolescente para o desenvolvimento acesso, à entrada do local de exibição, informação destacada
das competências necessárias à prevenção, à identificação de sobre a natureza do espetáculo e a faixa etária especificada no
evidências, ao diagnóstico e ao enfrentamento de todas as certificado de classificação.
formas de violência contra a criança e o adolescente; (Incluído
pela Lei nº 13.010, de 2014) Art. 75. Toda criança ou adolescente terá acesso às
IV - o apoio e o incentivo às práticas de resolução pacífica diversões e espetáculos públicos classificados como
de conflitos que envolvam violência contra a criança e o adequados à sua faixa etária.
adolescente; (Incluído pela Lei nº 13.010, de 2014) Parágrafo único. As crianças menores de dez anos somente
V - a inclusão, nas políticas públicas, de ações que visem a poderão ingressar e permanecer nos locais de apresentação ou
garantir os direitos da criança e do adolescente, desde a exibição quando acompanhadas dos pais ou responsável.
atenção pré-natal, e de atividades junto aos pais e
responsáveis com o objetivo de promover a informação, a Art. 76. As emissoras de rádio e televisão somente exibirão,
reflexão, o debate e a orientação sobre alternativas ao uso de no horário recomendado para o público infanto juvenil,
castigo físico ou de tratamento cruel ou degradante no programas com finalidades educativas, artísticas, culturais e
processo educativo; (Incluído pela Lei nº 13.010, de 2014) informativas.
VI - a promoção de espaços intersetoriais locais para a Parágrafo único. Nenhum espetáculo será apresentado ou
articulação de ações e a elaboração de planos de atuação anunciado sem aviso de sua classificação, antes de sua
conjunta focados nas famílias em situação de violência, com transmissão, apresentação ou exibição.
participação de profissionais de saúde, de assistência social e
de educação e de órgãos de promoção, proteção e defesa dos Art. 77. Os proprietários, diretores, gerentes e funcionários
direitos da criança e do adolescente. (Incluído pela Lei nº de empresas que explorem a venda ou aluguel de fitas de
13.010, de 2014) programação em vídeo cuidarão para que não haja venda ou
Parágrafo único. As famílias com crianças e adolescentes locação em desacordo com a classificação atribuída pelo órgão
com deficiência terão prioridade de atendimento nas ações e competente.
políticas públicas de prevenção e proteção. (Incluído pela Lei Parágrafo único. As fitas a que alude este artigo deverão
nº 13.010, de 2014) exibir, no invólucro, informação sobre a natureza da obra e a
faixa etária a que se destinam.
Art. 70-B. As entidades, públicas e privadas, que atuem nas
áreas a que se refere o art. 71, dentre outras, devem contar, em Art. 78. As revistas e publicações contendo material
seus quadros, com pessoas capacitadas a reconhecer e impróprio ou inadequado a crianças e adolescentes deverão
comunicar ao Conselho Tutelar suspeitas ou casos de maus- ser comercializadas em embalagem lacrada, com a advertência
tratos praticados contra crianças e adolescentes. (Incluído de seu conteúdo.
pela Lei nº 13.046, de 2014) Parágrafo único. As editoras cuidarão para que as capas
Parágrafo único. São igualmente responsáveis pela que contenham mensagens pornográficas ou obscenas sejam
comunicação de que trata este artigo, as pessoas encarregadas, protegidas com embalagem opaca.
por razão de cargo, função, ofício, ministério, profissão ou
ocupação, do cuidado, assistência ou guarda de crianças e Art. 79. As revistas e publicações destinadas ao público
adolescentes, punível, na forma deste Estatuto, o injustificado infanto-juvenil não poderão conter ilustrações, fotografias,
retardamento ou omissão, culposos ou dolosos. (Incluído pela legendas, crônicas ou anúncios de bebidas alcoólicas, tabaco,
Lei nº 13.046, de 2014) armas e munições, e deverão respeitar os valores éticos e
sociais da pessoa e da família.
Art. 71. A criança e o adolescente têm direito a informação,
cultura, lazer, esportes, diversões, espetáculos e produtos e Art. 80. Os responsáveis por estabelecimentos que
serviços que respeitem sua condição peculiar de pessoa em explorem comercialmente bilhar, sinuca ou congênere ou por
desenvolvimento. casas de jogos, assim entendidas as que realizem apostas,
ainda que eventualmente, cuidarão para que não seja
Art. 72. As obrigações previstas nesta Lei não excluem da permitida a entrada e a permanência de crianças e
prevenção especial outras decorrentes dos princípios por ela adolescentes no local, afixando aviso para orientação do
adotados. público.

Art. 73. A inobservância das normas de prevenção Seção II


importará em responsabilidade da pessoa física ou jurídica, Dos Produtos e Serviços
nos termos desta Lei.
Art. 81. É proibida a venda à criança ou ao adolescente de:
I - armas, munições e explosivos;
II - bebidas alcoólicas;

Legislação 19
Apostila Digital Licenciada para Alice Caroline Guarino dos Santos - alice.guarino@hotmail.com (Proibida a Revenda)
APOSTILAS OPÇÃO

III - produtos cujos componentes possam causar VI - políticas e programas destinados a prevenir ou
dependência física ou psíquica ainda que por utilização abreviar o período de afastamento do convívio familiar e a
indevida; garantir o efetivo exercício do direito à convivência familiar de
IV - fogos de estampido e de artifício, exceto aqueles que crianças e adolescentes;
pelo seu reduzido potencial sejam incapazes de provocar VII - campanhas de estímulo ao acolhimento sob forma de
qualquer dano físico em caso de utilização indevida; guarda de crianças e adolescentes afastados do convívio
V - revistas e publicações a que alude o art. 78; familiar e à adoção, especificamente inter-racial, de crianças
VI - bilhetes lotéricos e equivalentes. maiores ou de adolescentes, com necessidades específicas de
saúde ou com deficiências e de grupos de irmãos.
Art. 82. É proibida a hospedagem de criança ou
adolescente em hotel, motel, pensão ou estabelecimento Art. 88. São diretrizes da política de atendimento:
congênere, salvo se autorizado ou acompanhado pelos pais ou I - municipalização do atendimento;
responsável. II - criação de conselhos municipais, estaduais e nacional
dos direitos da criança e do adolescente, órgãos deliberativos
Seção III e controladores das ações em todos os níveis, assegurada a
Da Autorização para Viajar participação popular paritária por meio de organizações
representativas, segundo leis federal, estaduais e municipais;
Art. 83. Nenhuma criança poderá viajar para fora da III - criação e manutenção de programas específicos,
comarca onde reside, desacompanhada dos pais ou observada a descentralização político-administrativa;
responsável, sem expressa autorização judicial. IV - manutenção de fundos nacional, estaduais e
§ 1º A autorização não será exigida quando: municipais vinculados aos respectivos conselhos dos direitos
a) tratar-se de comarca contígua à da residência da criança, da criança e do adolescente;
se na mesma unidade da Federação, ou incluída na mesma V - integração operacional de órgãos do Judiciário,
região metropolitana; Ministério Público, Defensoria, Segurança Pública e
b) a criança estiver acompanhada: Assistência Social, preferencialmente em um mesmo local,
1) de ascendente ou colateral maior, até o terceiro grau, para efeito de agilização do atendimento inicial a adolescente
comprovado documentalmente o parentesco; a quem se atribua autoria de ato infracional;
2) de pessoa maior, expressamente autorizada pelo pai, VI - integração operacional de órgãos do Judiciário,
mãe ou responsável. Ministério Público, Defensoria, Conselho Tutelar e
§ 2º A autoridade judiciária poderá, a pedido dos pais ou encarregados da execução das políticas sociais básicas e de
responsável, conceder autorização válida por dois anos. assistência social, para efeito de agilização do atendimento de
crianças e de adolescentes inseridos em programas de
Art. 84. Quando se tratar de viagem ao exterior, a acolhimento familiar ou institucional, com vista na sua rápida
autorização é dispensável, se a criança ou adolescente: reintegração à família de origem ou, se tal solução se mostrar
I - estiver acompanhado de ambos os pais ou responsável; comprovadamente inviável, sua colocação em família
II - viajar na companhia de um dos pais, autorizado substituta, em quaisquer das modalidades previstas no art. 28
expressamente pelo outro através de documento com firma desta Lei;
reconhecida. VII - mobilização da opinião pública para a indispensável
participação dos diversos segmentos da sociedade.
Art. 85. Sem prévia e expressa autorização judicial, VIII - especialização e formação continuada dos
nenhuma criança ou adolescente nascido em território profissionais que trabalham nas diferentes áreas da atenção à
nacional poderá sair do País em companhia de estrangeiro primeira infância, incluindo os conhecimentos sobre direitos
residente ou domiciliado no exterior. da criança e sobre desenvolvimento infantil; (Incluído pela Lei
nº 13.257, de 2016)
PARTE ESPECIAL IX - formação profissional com abrangência dos diversos
TÍTULO I direitos da criança e do adolescente que favoreça a
intersetorialidade no atendimento da criança e do adolescente
Da Política de Atendimento e seu desenvolvimento integral; (Incluído pela Lei nº 13.257,
Capítulo I de 2016)
Disposições Gerais X - realização e divulgação de pesquisas sobre
desenvolvimento infantil e sobre prevenção da violência.
Art. 86. A política de atendimento dos direitos da criança e (Incluído pela Lei nº 13.257, de 2016)
do adolescente far-se-á através de um conjunto articulado de
ações governamentais e não-governamentais, da União, dos Art. 89. A função de membro do conselho nacional e dos
estados, do Distrito Federal e dos municípios. conselhos estaduais e municipais dos direitos da criança e do
Art. 87. São linhas de ação da política de atendimento: adolescente é considerada de interesse público relevante e não
I - políticas sociais básicas; será remunerada.
II - serviços, programas, projetos e benefícios de
assistência social de garantia de proteção social e de Capítulo II
prevenção e redução de violações de direitos, seus Das Entidades de Atendimento
agravamentos ou reincidências; (Redação dada pela Lei nº Seção I
13.257, de 2016) Disposições Gerais
III - serviços especiais de prevenção e atendimento médico
e psicossocial às vítimas de negligência, maus-tratos, Art. 90. As entidades de atendimento são responsáveis pela
exploração, abuso, crueldade e opressão; manutenção das próprias unidades, assim como pelo
IV - serviço de identificação e localização de pais, planejamento e execução de programas de proteção e
responsável, crianças e adolescentes desaparecidos; socioeducativos destinados a crianças e adolescentes, em
V - proteção jurídico-social por entidades de defesa dos regime de:
direitos da criança e do adolescente. I - orientação e apoio sócio-familiar;
II - apoio socioeducativo em meio aberto;

Legislação 20
Apostila Digital Licenciada para Alice Caroline Guarino dos Santos - alice.guarino@hotmail.com (Proibida a Revenda)
APOSTILAS OPÇÃO

III - colocação familiar; VI - evitar, sempre que possível, a transferência para


IV - acolhimento institucional; outras entidades de crianças e adolescentes abrigados;
V - prestação de serviços à comunidade; VII - participação na vida da comunidade local;
VI - liberdade assistida; VIII - preparação gradativa para o desligamento;
VII - semiliberdade; e IX - participação de pessoas da comunidade no processo
VIII - internação. educativo.
§ 1º As entidades governamentais e não governamentais § 1º O dirigente de entidade que desenvolve programa de
deverão proceder à inscrição de seus programas, acolhimento institucional é equiparado ao guardião, para
especificando os regimes de atendimento, na forma definida todos os efeitos de direito.
neste artigo, no Conselho Municipal dos Direitos da Criança e § 2º Os dirigentes de entidades que desenvolvem
do Adolescente, o qual manterá registro das inscrições e de programas de acolhimento familiar ou institucional remeterão
suas alterações, do que fará comunicação ao Conselho Tutelar à autoridade judiciária, no máximo a cada 6 (seis) meses,
e à autoridade judiciária. relatório circunstanciado acerca da situação de cada criança
§ 2º Os recursos destinados à implementação e ou adolescente acolhido e sua família, para fins da reavaliação
manutenção dos programas relacionados neste artigo serão prevista no § 1o do art. 19 desta Lei.
previstos nas dotações orçamentárias dos órgãos públicos § 3º Os entes federados, por intermédio dos Poderes
encarregados das áreas de Educação, Saúde e Assistência Executivo e Judiciário, promoverão conjuntamente a
Social, dentre outros, observando-se o princípio da prioridade permanente qualificação dos profissionais que atuam direta
absoluta à criança e ao adolescente preconizado pelo caput do ou indiretamente em programas de acolhimento institucional
art. 227 da Constituição Federal e pelo caput e parágrafo único e destinados à colocação familiar de crianças e adolescentes,
do art. 4o desta Lei. incluindo membros do Poder Judiciário, Ministério Público e
§ 3º Os programas em execução serão reavaliados pelo Conselho Tutelar.
Conselho Municipal dos Direitos da Criança e do Adolescente, § 4º Salvo determinação em contrário da autoridade
no máximo, a cada 2 (dois) anos, constituindo-se critérios para judiciária competente, as entidades que desenvolvem
renovação da autorização de funcionamento: programas de acolhimento familiar ou institucional, se
I - o efetivo respeito às regras e princípios desta Lei, bem necessário com o auxílio do Conselho Tutelar e dos órgãos de
como às resoluções relativas à modalidade de atendimento assistência social, estimularão o contato da criança ou
prestado expedidas pelos Conselhos de Direitos da Criança e adolescente com seus pais e parentes, em cumprimento ao
do Adolescente, em todos os níveis; disposto nos incisos I e VIII do caput deste artigo.
II - a qualidade e eficiência do trabalho desenvolvido, § 5º As entidades que desenvolvem programas de
atestadas pelo Conselho Tutelar, pelo Ministério Público e pela acolhimento familiar ou institucional somente poderão
Justiça da Infância e da Juventude; receber recursos públicos se comprovado o atendimento dos
III - em se tratando de programas de acolhimento princípios, exigências e finalidades desta Lei.
institucional ou familiar, serão considerados os índices de § 6º O descumprimento das disposições desta Lei pelo
sucesso na reintegração familiar ou de adaptação à família dirigente de entidade que desenvolva programas de
substituta, conforme o caso. acolhimento familiar ou institucional é causa de sua
destituição, sem prejuízo da apuração de sua responsabilidade
Art. 91. As entidades não-governamentais somente administrativa, civil e criminal.
poderão funcionar depois de registradas no Conselho § 7º Quando se tratar de criança de 0 (zero) a 3 (três) anos
Municipal dos Direitos da Criança e do Adolescente, o qual em acolhimento institucional, dar-se-á especial atenção à
comunicará o registro ao Conselho Tutelar e à autoridade atuação de educadores de referência estáveis e
judiciária da respectiva localidade. qualitativamente significativos, às rotinas específicas e ao
§ 1º Será negado o registro à entidade que: atendimento das necessidades básicas, incluindo as de afeto
a) não ofereça instalações físicas em condições adequadas como prioritárias. (Incluído pela Lei nº 13.257, de 2016)
de habitabilidade, higiene, salubridade e segurança;
b) não apresente plano de trabalho compatível com os Art. 93. As entidades que mantenham programa de
princípios desta Lei; acolhimento institucional poderão, em caráter excepcional e
c) esteja irregularmente constituída; de urgência, acolher crianças e adolescentes sem prévia
d) tenha em seus quadros pessoas inidôneas. determinação da autoridade competente, fazendo
e) não se adequar ou deixar de cumprir as resoluções e comunicação do fato em até 24 (vinte e quatro) horas ao Juiz
deliberações relativas à modalidade de atendimento prestado da Infância e da Juventude, sob pena de responsabilidade.
expedidas pelos Conselhos de Direitos da Criança e do Parágrafo único. Recebida a comunicação, a autoridade
Adolescente, em todos os níveis. judiciária, ouvido o Ministério Público e se necessário com o
§ 2º O registro terá validade máxima de 4 (quatro) anos, apoio do Conselho Tutelar local, tomará as medidas
cabendo ao Conselho Municipal dos Direitos da Criança e do necessárias para promover a imediata reintegração familiar da
Adolescente, periodicamente, reavaliar o cabimento de sua criança ou do adolescente ou, se por qualquer razão não for
renovação, observado o disposto no § 1o deste artigo. isso possível ou recomendável, para seu encaminhamento a
programa de acolhimento familiar, institucional ou a família
Art. 92. As entidades que desenvolvam programas de substituta, observado o disposto no § 2o do art. 101 desta Lei.
acolhimento familiar ou institucional deverão adotar os
seguintes princípios: Art. 94. As entidades que desenvolvem programas de
I - preservação dos vínculos familiares e promoção da internação têm as seguintes obrigações, entre outras:
reintegração familiar; I - observar os direitos e garantias de que são titulares os
II - integração em família substituta, quando esgotados os adolescentes;
recursos de manutenção na família natural ou extensa; II - não restringir nenhum direito que não tenha sido
III - atendimento personalizado e em pequenos grupos; objeto de restrição na decisão de internação;
IV - desenvolvimento de atividades em regime de III - oferecer atendimento personalizado, em pequenas
coeducação; unidades e grupos reduzidos;
V - não desmembramento de grupos de irmãos; IV - preservar a identidade e oferecer ambiente de respeito
e dignidade ao adolescente;

Legislação 21
Apostila Digital Licenciada para Alice Caroline Guarino dos Santos - alice.guarino@hotmail.com (Proibida a Revenda)
APOSTILAS OPÇÃO

V - diligenciar no sentido do restabelecimento e da d) fechamento de unidade ou interdição de programa.


preservação dos vínculos familiares; II - às entidades não-governamentais:
VI - comunicar à autoridade judiciária, periodicamente, os a) advertência;
casos em que se mostre inviável ou impossível o reatamento b) suspensão total ou parcial do repasse de verbas
dos vínculos familiares; públicas;
VII - oferecer instalações físicas em condições adequadas c) interdição de unidades ou suspensão de programa;
de habitabilidade, higiene, salubridade e segurança e os d) cassação do registro.
objetos necessários à higiene pessoal; § 1º Em caso de reiteradas infrações cometidas por
VIII - oferecer vestuário e alimentação suficientes e entidades de atendimento, que coloquem em risco os direitos
adequados à faixa etária dos adolescentes atendidos; assegurados nesta Lei, deverá ser o fato comunicado ao
IX - oferecer cuidados médicos, psicológicos, Ministério Público ou representado perante autoridade
odontológicos e farmacêuticos; judiciária competente para as providências cabíveis, inclusive
X - propiciar escolarização e profissionalização; suspensão das atividades ou dissolução da entidade.
XI - propiciar atividades culturais, esportivas e de lazer; § 2º As pessoas jurídicas de direito público e as
XII - propiciar assistência religiosa àqueles que desejarem, organizações não governamentais responderão pelos danos
de acordo com suas crenças; que seus agentes causarem às crianças e aos adolescentes,
XIII - proceder a estudo social e pessoal de cada caso; caracterizado o descumprimento dos princípios norteadores
XIV - reavaliar periodicamente cada caso, com intervalo das atividades de proteção específica.
máximo de seis meses, dando ciência dos resultados à
autoridade competente; Título II
XV - informar, periodicamente, o adolescente internado Das Medidas de Proteção
sobre sua situação processual; Capítulo I
XVI - comunicar às autoridades competentes todos os Disposições Gerais
casos de adolescentes portadores de moléstias
infectocontagiosas; Art. 98. As medidas de proteção à criança e ao adolescente
XVII - fornecer comprovante de depósito dos pertences dos são aplicáveis sempre que os direitos reconhecidos nesta Lei
adolescentes; forem ameaçados ou violados:
XVIII - manter programas destinados ao apoio e I - por ação ou omissão da sociedade ou do Estado;
acompanhamento de egressos; II - por falta, omissão ou abuso dos pais ou responsável;
XIX - providenciar os documentos necessários ao exercício III - em razão de sua conduta.
da cidadania àqueles que não os tiverem;
XX - manter arquivo de anotações onde constem data e Capítulo II
circunstâncias do atendimento, nome do adolescente, seus Das Medidas Específicas de Proteção
pais ou responsável, parentes, endereços, sexo, idade,
acompanhamento da sua formação, relação de seus pertences Art. 99. As medidas previstas neste Capítulo poderão ser
e demais dados que possibilitem sua identificação e a aplicadas isolada ou cumulativamente, bem como substituídas
individualização do atendimento. a qualquer tempo.
§ 1º Aplicam-se, no que couber, as obrigações constantes
deste artigo às entidades que mantêm programas de Art. 100. Na aplicação das medidas levar-se-ão em conta as
acolhimento institucional e familiar. necessidades pedagógicas, preferindo-se aquelas que visem ao
§ 2º No cumprimento das obrigações a que alude este fortalecimento dos vínculos familiares e comunitários.
artigo as entidades utilizarão preferencialmente os recursos Parágrafo único. São também princípios que regem a
da comunidade. aplicação das medidas:
I - condição da criança e do adolescente como sujeitos de
Art. 94-A. As entidades, públicas ou privadas, que direitos: crianças e adolescentes são os titulares dos direitos
abriguem ou recepcionem crianças e adolescentes, ainda que previstos nesta e em outras Leis, bem como na Constituição
em caráter temporário, devem ter, em seus quadros, Federal;
profissionais capacitados a reconhecer e reportar ao Conselho II - proteção integral e prioritária: a interpretação e
Tutelar suspeitas ou ocorrências de maus-tratos. (Incluído aplicação de toda e qualquer norma contida nesta Lei deve ser
pela Lei nº 13.046, de 2014) voltada à proteção integral e prioritária dos direitos de que
crianças e adolescentes são titulares;
Seção II III - responsabilidade primária e solidária do poder
Da Fiscalização das Entidades público: a plena efetivação dos direitos assegurados a crianças
e a adolescentes por esta Lei e pela Constituição Federal, salvo
Art. 95. As entidades governamentais e não- nos casos por esta expressamente ressalvados, é de
governamentais referidas no art. 90 serão fiscalizadas pelo responsabilidade primária e solidária das 3 (três) esferas de
Judiciário, pelo Ministério Público e pelos Conselhos Tutelares. governo, sem prejuízo da municipalização do atendimento e
da possibilidade da execução de programas por entidades não
Art. 96. Os planos de aplicação e as prestações de contas governamentais;
serão apresentados ao estado ou ao município, conforme a IV - interesse superior da criança e do adolescente: a
origem das dotações orçamentárias. intervenção deve atender prioritariamente aos interesses e
direitos da criança e do adolescente, sem prejuízo da
Art. 97. São medidas aplicáveis às entidades de consideração que for devida a outros interesses legítimos no
atendimento que descumprirem obrigação constante do art. âmbito da pluralidade dos interesses presentes no caso
94, sem prejuízo da responsabilidade civil e criminal de seus concreto;
dirigentes ou prepostos: V - privacidade: a promoção dos direitos e proteção da
I - às entidades governamentais: criança e do adolescente deve ser efetuada no respeito pela
a) advertência; intimidade, direito à imagem e reserva da sua vida privada;
b) afastamento provisório de seus dirigentes;
c) afastamento definitivo de seus dirigentes;

Legislação 22
Apostila Digital Licenciada para Alice Caroline Guarino dos Santos - alice.guarino@hotmail.com (Proibida a Revenda)
APOSTILAS OPÇÃO

VI - intervenção precoce: a intervenção das autoridades de uma Guia de Acolhimento, expedida pela autoridade
competentes deve ser efetuada logo que a situação de perigo judiciária, na qual obrigatoriamente constará, dentre outros:
seja conhecida; I - sua identificação e a qualificação completa de seus pais
VII - intervenção mínima: a intervenção deve ser exercida ou de seu responsável, se conhecidos;
exclusivamente pelas autoridades e instituições cuja ação seja II - o endereço de residência dos pais ou do responsável,
indispensável à efetiva promoção dos direitos e à proteção da com pontos de referência;
criança e do adolescente; III - os nomes de parentes ou de terceiros interessados em
VIII - proporcionalidade e atualidade: a intervenção deve tê-los sob sua guarda;
ser a necessária e adequada à situação de perigo em que a IV - os motivos da retirada ou da não reintegração ao
criança ou o adolescente se encontram no momento em que a convívio familiar.
decisão é tomada; § 4º Imediatamente após o acolhimento da criança ou do
IX - responsabilidade parental: a intervenção deve ser adolescente, a entidade responsável pelo programa de
efetuada de modo que os pais assumam os seus deveres para acolhimento institucional ou familiar elaborará um plano
com a criança e o adolescente; individual de atendimento, visando à reintegração familiar,
X - prevalência da família: na promoção de direitos e na ressalvada a existência de ordem escrita e fundamentada em
proteção da criança e do adolescente deve ser dada contrário de autoridade judiciária competente, caso em que
prevalência às medidas que os mantenham ou reintegrem na também deverá contemplar sua colocação em família
sua família natural ou extensa ou, se isto não for possível, que substituta, observadas as regras e princípios desta Lei.
promovam a sua integração em família substituta; § 5º O plano individual será elaborado sob a
XI - obrigatoriedade da informação: a criança e o responsabilidade da equipe técnica do respectivo programa de
adolescente, respeitado seu estágio de desenvolvimento e atendimento e levará em consideração a opinião da criança ou
capacidade de compreensão, seus pais ou responsável devem do adolescente e a oitiva dos pais ou do responsável.
ser informados dos seus direitos, dos motivos que § 6º Constarão do plano individual, dentre outros:
determinaram a intervenção e da forma como esta se processa; I - os resultados da avaliação interdisciplinar;
XII - oitiva obrigatória e participação: a criança e o II - os compromissos assumidos pelos pais ou responsável;
adolescente, em separado ou na companhia dos pais, de e
responsável ou de pessoa por si indicada, bem como os seus III - a previsão das atividades a serem desenvolvidas com
pais ou responsável, têm direito a ser ouvidos e a participar a criança ou com o adolescente acolhido e seus pais ou
nos atos e na definição da medida de promoção dos direitos e responsável, com vista na reintegração familiar ou, caso seja
de proteção, sendo sua opinião devidamente considerada pela esta vedada por expressa e fundamentada determinação
autoridade judiciária competente, observado o disposto nos §§ judicial, as providências a serem tomadas para sua colocação
1o e 2o do art. 28 desta Lei. em família substituta, sob direta supervisão da autoridade
judiciária.
Art. 101. Verificada qualquer das hipóteses previstas no § 7º O acolhimento familiar ou institucional ocorrerá no
art. 98, a autoridade competente poderá determinar, dentre local mais próximo à residência dos pais ou do responsável e,
outras, as seguintes medidas: como parte do processo de reintegração familiar, sempre que
I - encaminhamento aos pais ou responsável, mediante identificada a necessidade, a família de origem será incluída
termo de responsabilidade; em programas oficiais de orientação, de apoio e de promoção
II - orientação, apoio e acompanhamento temporários; social, sendo facilitado e estimulado o contato com a criança
III - matrícula e frequência obrigatórias em ou com o adolescente acolhido.
estabelecimento oficial de ensino fundamental; § 8º Verificada a possibilidade de reintegração familiar, o
IV - inclusão em serviços e programas oficiais ou responsável pelo programa de acolhimento familiar ou
comunitários de proteção, apoio e promoção da família, da institucional fará imediata comunicação à autoridade
criança e do adolescente; (Redação dada pela Lei nº 13.257, de judiciária, que dará vista ao Ministério Público, pelo prazo de
2016) 5 (cinco) dias, decidindo em igual prazo.
V - requisição de tratamento médico, psicológico ou § 9º Em sendo constatada a impossibilidade de
psiquiátrico, em regime hospitalar ou ambulatorial; reintegração da criança ou do adolescente à família de origem,
VI - inclusão em programa oficial ou comunitário de após seu encaminhamento a programas oficiais ou
auxílio, orientação e tratamento a alcoólatras e toxicômanos; comunitários de orientação, apoio e promoção social, será
VII - acolhimento institucional; enviado relatório fundamentado ao Ministério Público, no qual
VIII - inclusão em programa de acolhimento familiar; conste a descrição pormenorizada das providências tomadas e
IX - colocação em família substituta. a expressa recomendação, subscrita pelos técnicos da entidade
§ 1º O acolhimento institucional e o acolhimento familiar ou responsáveis pela execução da política municipal de
são medidas provisórias e excepcionais, utilizáveis como garantia do direito à convivência familiar, para a destituição
forma de transição para reintegração familiar ou, não sendo do poder familiar, ou destituição de tutela ou guarda.
esta possível, para colocação em família substituta, não § 10. Recebido o relatório, o Ministério Público terá o
implicando privação de liberdade. prazo de 30 (trinta) dias para o ingresso com a ação de
§ 2º Sem prejuízo da tomada de medidas emergenciais destituição do poder familiar, salvo se entender necessária a
para proteção de vítimas de violência ou abuso sexual e das realização de estudos complementares ou outras providências
providências a que alude o art. 130 desta Lei, o afastamento da que entender indispensáveis ao ajuizamento da demanda.
criança ou adolescente do convívio familiar é de competência § 11. A autoridade judiciária manterá, em cada comarca ou
exclusiva da autoridade judiciária e importará na deflagração, foro regional, um cadastro contendo informações atualizadas
a pedido do Ministério Público ou de quem tenha legítimo sobre as crianças e adolescentes em regime de acolhimento
interesse, de procedimento judicial contencioso, no qual se familiar e institucional sob sua responsabilidade, com
garanta aos pais ou ao responsável legal o exercício do informações pormenorizadas sobre a situação jurídica de cada
contraditório e da ampla defesa. um, bem como as providências tomadas para sua reintegração
§ 3º Crianças e adolescentes somente poderão ser familiar ou colocação em família substituta, em qualquer das
encaminhados às instituições que executam programas de modalidades previstas no art. 28 desta Lei.
acolhimento institucional, governamentais ou não, por meio § 12. Terão acesso ao cadastro o Ministério Público, o
Conselho Tutelar, o órgão gestor da Assistência Social e os

Legislação 23
Apostila Digital Licenciada para Alice Caroline Guarino dos Santos - alice.guarino@hotmail.com (Proibida a Revenda)
APOSTILAS OPÇÃO

Conselhos Municipais dos Direitos da Criança e do Adolescente Parágrafo único. A decisão deverá ser fundamentada e
e da Assistência Social, aos quais incumbe deliberar sobre a basear-se em indícios suficientes de autoria e materialidade,
implementação de políticas públicas que permitam reduzir o demonstrada a necessidade imperiosa da medida.
número de crianças e adolescentes afastados do convívio
familiar e abreviar o período de permanência em programa de Art. 109. O adolescente civilmente identificado não será
acolhimento. submetido a identificação compulsória pelos órgãos policiais,
de proteção e judiciais, salvo para efeito de confrontação,
Art. 102. As medidas de proteção de que trata este Capítulo havendo dúvida fundada.
serão acompanhadas da regularização do registro civil.
§ 1º Verificada a inexistência de registro anterior, o Capítulo III
assento de nascimento da criança ou adolescente será feito à Das Garantias Processuais
vista dos elementos disponíveis, mediante requisição da
autoridade judiciária. Art. 110. Nenhum adolescente será privado de sua
§ 2º Os registros e certidões necessários à regularização de liberdade sem o devido processo legal.
que trata este artigo são isentos de multas, custas e
emolumentos, gozando de absoluta prioridade. Art. 111. São asseguradas ao adolescente, entre outras, as
§ 3º Caso ainda não definida a paternidade, será deflagrado seguintes garantias:
procedimento específico destinado à sua averiguação, I - pleno e formal conhecimento da atribuição de ato
conforme previsto pela Lei no 8.560, de 29 de dezembro de infracional, mediante citação ou meio equivalente;
1992. II - igualdade na relação processual, podendo confrontar-
§ 4º Nas hipóteses previstas no § 3o deste artigo, é se com vítimas e testemunhas e produzir todas as provas
dispensável o ajuizamento de ação de investigação de necessárias à sua defesa;
paternidade pelo Ministério Público se, após o não III - defesa técnica por advogado;
comparecimento ou a recusa do suposto pai em assumir a IV - assistência judiciária gratuita e integral aos
paternidade a ele atribuída, a criança for encaminhada para necessitados, na forma da lei;
adoção. V - direito de ser ouvido pessoalmente pela autoridade
§ 5º Os registros e certidões necessários à inclusão, a competente;
qualquer tempo, do nome do pai no assento de nascimento são VI - direito de solicitar a presença de seus pais ou
isentos de multas, custas e emolumentos, gozando de absoluta responsável em qualquer fase do procedimento.
prioridade. (Incluído dada pela Lei nº 13.257, de 2016)
§ 6º São gratuitas, a qualquer tempo, a averbação Capítulo IV
requerida do reconhecimento de paternidade no assento de Das Medidas Socioeducativas
nascimento e a certidão correspondente. (Incluído dada pela Seção I
Lei nº 13.257, de 2016) Disposições Gerais

Título III Art. 112. Verificada a prática de ato infracional, a


Da Prática de Ato Infracional autoridade competente poderá aplicar ao adolescente as
Capítulo I seguintes medidas:
Disposições Gerais I - advertência;
II - obrigação de reparar o dano;
Art. 103. Considera-se ato infracional a conduta descrita III - prestação de serviços à comunidade;
como crime ou contravenção penal. IV - liberdade assistida;
V - inserção em regime de semiliberdade;
Art. 104. São penalmente inimputáveis os menores de VI - internação em estabelecimento educacional;
dezoito anos, sujeitos às medidas previstas nesta Lei. VII - qualquer uma das previstas no art. 101, I a VI.
Parágrafo único. Para os efeitos desta Lei, deve ser § 1º A medida aplicada ao adolescente levará em conta a
considerada a idade do adolescente à data do fato. sua capacidade de cumpri-la, as circunstâncias e a gravidade
da infração.
Art. 105. Ao ato infracional praticado por criança § 2º Em hipótese alguma e sob pretexto algum, será
corresponderão as medidas previstas no art. 101. admitida a prestação de trabalho forçado.
§ 3º Os adolescentes portadores de doença ou deficiência
Capítulo II mental receberão tratamento individual e especializado, em
Dos Direitos Individuais local adequado às suas condições.

Art. 106. Nenhum adolescente será privado de sua Art. 113. Aplica-se a este Capítulo o disposto nos arts. 99 e
liberdade senão em flagrante de ato infracional ou por ordem 100.
escrita e fundamentada da autoridade judiciária competente.
Parágrafo único. O adolescente tem direito à identificação Art. 114. A imposição das medidas previstas nos incisos II
dos responsáveis pela sua apreensão, devendo ser informado a VI do art. 112 pressupõe a existência de provas suficientes
acerca de seus direitos. da autoria e da materialidade da infração, ressalvada a
hipótese de remissão, nos termos do art. 127.
Art. 107. A apreensão de qualquer adolescente e o local Parágrafo único. A advertência poderá ser aplicada sempre
onde se encontra recolhido serão incontinenti comunicados à que houver prova da materialidade e indícios suficientes da
autoridade judiciária competente e à família do apreendido ou autoria.
à pessoa por ele indicada.
Parágrafo único. Examinar-se-á, desde logo e sob pena de Seção II
responsabilidade, a possibilidade de liberação imediata. Da Advertência

Art. 108. A internação, antes da sentença, pode ser Art. 115. A advertência consistirá em admoestação verbal,
determinada pelo prazo máximo de quarenta e cinco dias. que será reduzida a termo e assinada.

Legislação 24
Apostila Digital Licenciada para Alice Caroline Guarino dos Santos - alice.guarino@hotmail.com (Proibida a Revenda)
APOSTILAS OPÇÃO

Seção III Seção VII


Da Obrigação de Reparar o Dano Da Internação

Art. 116. Em se tratando de ato infracional com reflexos Art. 121. A internação constitui medida privativa da
patrimoniais, a autoridade poderá determinar, se for o caso, liberdade, sujeita aos princípios de brevidade,
que o adolescente restitua a coisa, promova o ressarcimento excepcionalidade e respeito à condição peculiar de pessoa em
do dano, ou, por outra forma, compense o prejuízo da vítima. desenvolvimento.
Parágrafo único. Havendo manifesta impossibilidade, a § 1º Será permitida a realização de atividades externas, a
medida poderá ser substituída por outra adequada. critério da equipe técnica da entidade, salvo expressa
determinação judicial em contrário.
Seção IV § 2º A medida não comporta prazo determinado, devendo
Da Prestação de Serviços à Comunidade sua manutenção ser reavaliada, mediante decisão
fundamentada, no máximo a cada seis meses.
Art. 117. A prestação de serviços comunitários consiste na § 3º Em nenhuma hipótese o período máximo de
realização de tarefas gratuitas de interesse geral, por período internação excederá a três anos.
não excedente a seis meses, junto a entidades assistenciais, § 4º Atingido o limite estabelecido no parágrafo anterior, o
hospitais, escolas e outros estabelecimentos congêneres, bem adolescente deverá ser liberado, colocado em regime de
como em programas comunitários ou governamentais. semiliberdade ou de liberdade assistida.
Parágrafo único. As tarefas serão atribuídas conforme as § 5º A liberação será compulsória aos vinte e um anos de
aptidões do adolescente, devendo ser cumpridas durante idade.
jornada máxima de oito horas semanais, aos sábados, § 6º Em qualquer hipótese a desinternação será precedida
domingos e feriados ou em dias úteis, de modo a não de autorização judicial, ouvido o Ministério Público.
prejudicar a frequência à escola ou à jornada normal de § 7º A determinação judicial mencionada no § 1o poderá
trabalho. ser revista a qualquer tempo pela autoridade judiciária.
(Incluído pela Lei nº 12.594, de 2012) (Vide)
Seção V
Da Liberdade Assistida Art. 122. A medida de internação só poderá ser aplicada
quando:
Art. 118. A liberdade assistida será adotada sempre que se I - tratar-se de ato infracional cometido mediante grave
afigurar a medida mais adequada para o fim de acompanhar, ameaça ou violência a pessoa;
auxiliar e orientar o adolescente. II - por reiteração no cometimento de outras infrações
§ 1º A autoridade designará pessoa capacitada para graves;
acompanhar o caso, a qual poderá ser recomendada por III - por descumprimento reiterado e injustificável da
entidade ou programa de atendimento. medida anteriormente imposta.
§ 2º A liberdade assistida será fixada pelo prazo mínimo de § 1º O prazo de internação na hipótese do inciso III deste
seis meses, podendo a qualquer tempo ser prorrogada, artigo não poderá ser superior a 3 (três) meses, devendo ser
revogada ou substituída por outra medida, ouvido o decretada judicialmente após o devido processo legal.
orientador, o Ministério Público e o defensor. (Redação dada pela Lei nº 12.594, de 2012) (Vide)
§ 2º. Em nenhuma hipótese será aplicada a internação,
Art. 119. Incumbe ao orientador, com o apoio e a havendo outra medida adequada.
supervisão da autoridade competente, a realização dos
seguintes encargos, entre outros: Art. 123. A internação deverá ser cumprida em entidade
I - promover socialmente o adolescente e sua família, exclusiva para adolescentes, em local distinto daquele
fornecendo-lhes orientação e inserindo-os, se necessário, em destinado ao abrigo, obedecida rigorosa separação por
programa oficial ou comunitário de auxílio e assistência social; critérios de idade, compleição física e gravidade da infração.
II - supervisionar a frequência e o aproveitamento escolar Parágrafo único. Durante o período de internação,
do adolescente, promovendo, inclusive, sua matrícula; inclusive provisória, serão obrigatórias atividades
III - diligenciar no sentido da profissionalização do pedagógicas.
adolescente e de sua inserção no mercado de trabalho;
IV - apresentar relatório do caso. Art. 124. São direitos do adolescente privado de liberdade,
entre outros, os seguintes:
Seção VI I - entrevistar-se pessoalmente com o representante do
Ministério Público;
Do Regime de Semiliberdade II - peticionar diretamente a qualquer autoridade;
III - avistar-se reservadamente com seu defensor;
Art. 120. O regime de semiliberdade pode ser determinado IV - ser informado de sua situação processual, sempre que
desde o início, ou como forma de transição para o meio aberto, solicitada;
possibilitada a realização de atividades externas, V - ser tratado com respeito e dignidade;
independentemente de autorização judicial. VI - permanecer internado na mesma localidade ou
§ 1º São obrigatórias a escolarização e a profissionalização, naquela mais próxima ao domicílio de seus pais ou
devendo, sempre que possível, ser utilizados os recursos responsável;
existentes na comunidade. VII - receber visitas, ao menos, semanalmente;
§ 2º A medida não comporta prazo determinado VIII - corresponder-se com seus familiares e amigos;
aplicando-se, no que couber, as disposições relativas à IX - ter acesso aos objetos necessários à higiene e asseio
internação. pessoal;
X - habitar alojamento em condições adequadas de higiene
e salubridade;
XI - receber escolarização e profissionalização;
XII - realizar atividades culturais, esportivas e de lazer:
XIII - ter acesso aos meios de comunicação social;

Legislação 25
Apostila Digital Licenciada para Alice Caroline Guarino dos Santos - alice.guarino@hotmail.com (Proibida a Revenda)
APOSTILAS OPÇÃO

XIV - receber assistência religiosa, segundo a sua crença, e judiciária poderá determinar, como medida cautelar, o
desde que assim o deseje; afastamento do agressor da moradia comum.
XV - manter a posse de seus objetos pessoais e dispor de Parágrafo único. Da medida cautelar constará, ainda, a
local seguro para guardá-los, recebendo comprovante fixação provisória dos alimentos de que necessitem a criança
daqueles porventura depositados em poder da entidade; ou o adolescente dependentes do agressor. (Incluído pela Lei
XVI - receber, quando de sua desinternação, os nº 12.415, de 2011)
documentos pessoais indispensáveis à vida em sociedade.
§ 1º Em nenhum caso haverá incomunicabilidade. Título V
§ 2º A autoridade judiciária poderá suspender Do Conselho Tutelar
temporariamente a visita, inclusive de pais ou responsável, se Capítulo I
existirem motivos sérios e fundados de sua prejudicialidade Disposições Gerais
aos interesses do adolescente.
Art. 131. O Conselho Tutelar é órgão permanente e
Art. 125. É dever do Estado zelar pela integridade física e autônomo, não jurisdicional, encarregado pela sociedade de
mental dos internos, cabendo-lhe adotar as medidas zelar pelo cumprimento dos direitos da criança e do
adequadas de contenção e segurança. adolescente, definidos nesta Lei.

Capítulo V Art. 132. Em cada Município e em cada Região


Da Remissão Administrativa do Distrito Federal haverá, no mínimo, 1 (um)
Conselho Tutelar como órgão integrante da administração
Art. 126. Antes de iniciado o procedimento judicial para pública local, composto de 5 (cinco) membros, escolhidos pela
apuração de ato infracional, o representante do Ministério população local para mandato de 4 (quatro) anos, permitida 1
Público poderá conceder a remissão, como forma de exclusão (uma) recondução, mediante novo processo de escolha.
do processo, atendendo às circunstâncias e consequências do (Redação dada pela Lei nº 12.696, de 2012)
fato, ao contexto social, bem como à personalidade do
adolescente e sua maior ou menor participação no ato Art. 133. Para a candidatura a membro do Conselho
infracional. Tutelar, serão exigidos os seguintes requisitos:
Parágrafo único. Iniciado o procedimento, a concessão da I - reconhecida idoneidade moral;
remissão pela autoridade judiciária importará na suspensão II - idade superior a vinte e um anos;
ou extinção do processo. III - residir no município.

Art. 127. A remissão não implica necessariamente o Art. 134. Lei municipal ou distrital disporá sobre o local,
reconhecimento ou comprovação da responsabilidade, nem dia e horário de funcionamento do Conselho Tutelar, inclusive
prevalece para efeito de antecedentes, podendo incluir quanto à remuneração dos respectivos membros, aos quais é
eventualmente a aplicação de qualquer das medidas previstas assegurado o direito a: (Redação dada pela Lei nº 12.696, de
em lei, exceto a colocação em regime de semiliberdade e a 2012)
internação. I - cobertura previdenciária; (Incluído pela Lei nº 12.696,
de 2012)
Art. 128. A medida aplicada por força da remissão poderá II - gozo de férias anuais remuneradas, acrescidas de 1/3
ser revista judicialmente, a qualquer tempo, mediante pedido (um terço) do valor da remuneração mensal; (Incluído pela Lei
expresso do adolescente ou de seu representante legal, ou do nº 12.696, de 2012)
Ministério Público. III - licença-maternidade; (Incluído pela Lei nº 12.696, de
2012)
Título IV IV - licença-paternidade; (Incluído pela Lei nº 12.696, de
Das Medidas Pertinentes aos Pais ou Responsável 2012)
V - gratificação natalina. (Incluído pela Lei nº 12.696, de
Art. 129. São medidas aplicáveis aos pais ou responsável: 2012)
I - encaminhamento a serviços e programas oficiais ou Parágrafo único. Constará da lei orçamentária municipal e
comunitários de proteção, apoio e promoção da família; da do Distrito Federal previsão dos recursos necessários ao
(Redação dada pela Lei nº 13.257, de 2016) funcionamento do Conselho Tutelar e à remuneração e
II - inclusão em programa oficial ou comunitário de auxílio, formação continuada dos conselheiros tutelares. (Redação
orientação e tratamento a alcoólatras e toxicômanos; dada pela Lei nº 12.696, de 2012)
III - encaminhamento a tratamento psicológico ou
psiquiátrico; Art. 135. O exercício efetivo da função de conselheiro
IV - encaminhamento a cursos ou programas de constituirá serviço público relevante e estabelecerá presunção
orientação; de idoneidade moral. (Redação dada pela Lei nº 12.696, de
V - obrigação de matricular o filho ou pupilo e acompanhar 2012)
sua frequência e aproveitamento escolar;
VI - obrigação de encaminhar a criança ou adolescente a Capítulo II
tratamento especializado; Das Atribuições do Conselho
VII - advertência;
VIII - perda da guarda; Art. 136. São atribuições do Conselho Tutelar:
IX - destituição da tutela; I - atender as crianças e adolescentes nas hipóteses
X - suspensão ou destituição do poder familiar. previstas nos arts. 98 e 105, aplicando as medidas previstas no
Parágrafo único. Na aplicação das medidas previstas nos art. 101, I a VII;
incisos IX e X deste artigo, observar-se-á o disposto nos arts. II - atender e aconselhar os pais ou responsável, aplicando
23 e 24. as medidas previstas no art. 129, I a VII;
III - promover a execução de suas decisões, podendo para
Art. 130. Verificada a hipótese de maus-tratos, opressão ou tanto:
abuso sexual impostos pelos pais ou responsável, a autoridade

Legislação 26
Apostila Digital Licenciada para Alice Caroline Guarino dos Santos - alice.guarino@hotmail.com (Proibida a Revenda)
APOSTILAS OPÇÃO

a) requisitar serviços públicos nas áreas de saúde, Capítulo V


educação, serviço social, previdência, trabalho e segurança; Dos Impedimentos
b) representar junto à autoridade judiciária nos casos de
descumprimento injustificado de suas deliberações. Art. 140. São impedidos de servir no mesmo Conselho
IV - encaminhar ao Ministério Público notícia de fato que marido e mulher, ascendentes e descendentes, sogro e genro
constitua infração administrativa ou penal contra os direitos ou nora, irmãos, cunhados, durante o cunhadio, tio e sobrinho,
da criança ou adolescente; padrasto ou madrasta e enteado.
V - encaminhar à autoridade judiciária os casos de sua Parágrafo único. Estende-se o impedimento do
competência; conselheiro, na forma deste artigo, em relação à autoridade
VI - providenciar a medida estabelecida pela autoridade judiciária e ao representante do Ministério Público com
judiciária, dentre as previstas no art. 101, de I a VI, para o atuação na Justiça da Infância e da Juventude, em exercício na
adolescente autor de ato infracional; comarca, foro regional ou distrital.
VII - expedir notificações;
VIII - requisitar certidões de nascimento e de óbito de Título VI
criança ou adolescente quando necessário; Do Acesso à Justiça
IX - assessorar o Poder Executivo local na elaboração da Capítulo I
proposta orçamentária para planos e programas de Disposições Gerais
atendimento dos direitos da criança e do adolescente;
X - representar, em nome da pessoa e da família, contra a Art. 141. É garantido o acesso de toda criança ou
violação dos direitos previstos no art. 220, § 3º, inciso II, da adolescente à Defensoria Pública, ao Ministério Público e ao
Constituição Federal; Poder Judiciário, por qualquer de seus órgãos.
XI - representar ao Ministério Público para efeito das ações § 1º. A assistência judiciária gratuita será prestada aos que
de perda ou suspensão do poder familiar, após esgotadas as dela necessitarem, através de defensor público ou advogado
possibilidades de manutenção da criança ou do adolescente nomeado.
junto à família natural. § 2º As ações judiciais da competência da Justiça da
XII - promover e incentivar, na comunidade e nos grupos Infância e da Juventude são isentas de custas e emolumentos,
profissionais, ações de divulgação e treinamento para o ressalvada a hipótese de litigância de má-fé.
reconhecimento de sintomas de maus-tratos em crianças e
adolescentes. (Incluído pela Lei nº 13.046, de 2014) Art. 142. Os menores de dezesseis anos serão
Parágrafo único. Se, no exercício de suas atribuições, o representados e os maiores de dezesseis e menores de vinte e
Conselho Tutelar entender necessário o afastamento do um anos assistidos por seus pais, tutores ou curadores, na
convívio familiar, comunicará incontinenti o fato ao Ministério forma da legislação civil ou processual.
Público, prestando-lhe informações sobre os motivos de tal Parágrafo único. A autoridade judiciária dará curador
entendimento e as providências tomadas para a orientação, o especial à criança ou adolescente, sempre que os interesses
apoio e a promoção social da família. destes colidirem com os de seus pais ou responsável, ou
quando carecer de representação ou assistência legal ainda
Art. 137. As decisões do Conselho Tutelar somente que eventual.
poderão ser revistas pela autoridade judiciária a pedido de
quem tenha legítimo interesse. Art. 143. E vedada a divulgação de atos judiciais, policiais e
administrativos que digam respeito a crianças e adolescentes
Capítulo III a que se atribua autoria de ato infracional.
Da Competência Parágrafo único. Qualquer notícia a respeito do fato não
poderá identificar a criança ou adolescente, vedando-se
Art. 138. Aplica-se ao Conselho Tutelar a regra de fotografia, referência a nome, apelido, filiação, parentesco,
competência constante do art. 147. residência e, inclusive, iniciais do nome e sobrenome.

Capítulo IV Art. 144. A expedição de cópia ou certidão de atos a que se


Da Escolha dos Conselheiros refere o artigo anterior somente será deferida pela autoridade
judiciária competente, se demonstrado o interesse e
Art. 139. O processo para a escolha dos membros do justificada a finalidade.
Conselho Tutelar será estabelecido em lei municipal e
realizado sob a responsabilidade do Conselho Municipal dos Capítulo II
Direitos da Criança e do Adolescente, e a fiscalização do Da Justiça da Infância e da Juventude
Ministério Público. (Redação dada pela Lei nº 8.242, de Seção I
12.10.1991) Disposições Gerais
§ 1º O processo de escolha dos membros do Conselho
Tutelar ocorrerá em data unificada em todo o território Art. 145. Os estados e o Distrito Federal poderão criar
nacional a cada 4 (quatro) anos, no primeiro domingo do mês varas especializadas e exclusivas da infância e da juventude,
de outubro do ano subsequente ao da eleição presidencial. cabendo ao Poder Judiciário estabelecer sua
(Incluído pela Lei nº 12.696, de 2012) proporcionalidade por número de habitantes, dotá-las de
§ 2º A posse dos conselheiros tutelares ocorrerá no dia 10 infraestrutura e dispor sobre o atendimento, inclusive em
de janeiro do ano subsequente ao processo de escolha. plantões.
(Incluído pela Lei nº 12.696, de 2012)
§ 3º No processo de escolha dos membros do Conselho Seção II
Tutelar, é vedado ao candidato doar, oferecer, prometer ou Do Juiz
entregar ao eleitor bem ou vantagem pessoal de qualquer
natureza, inclusive brindes de pequeno valor. (Incluído pela Art. 146. A autoridade a que se refere esta Lei é o Juiz da
Lei nº 12.696, de 2012) Infância e da Juventude, ou o juiz que exerce essa função, na
forma da lei de organização judiciária local.

Legislação 27
Apostila Digital Licenciada para Alice Caroline Guarino dos Santos - alice.guarino@hotmail.com (Proibida a Revenda)
APOSTILAS OPÇÃO

Art. 147. A competência será determinada: b) as peculiaridades locais;


I - pelo domicílio dos pais ou responsável; c) a existência de instalações adequadas;
II - pelo lugar onde se encontre a criança ou adolescente, à d) o tipo de frequência habitual ao local;
falta dos pais ou responsável. e) a adequação do ambiente a eventual participação ou
§ 1º. Nos casos de ato infracional, será competente a frequência de crianças e adolescentes;
autoridade do lugar da ação ou omissão, observadas as regras f) a natureza do espetáculo.
de conexão, continência e prevenção. § 2º As medidas adotadas na conformidade deste artigo
§ 2º A execução das medidas poderá ser delegada à deverão ser fundamentadas, caso a caso, vedadas as
autoridade competente da residência dos pais ou responsável, determinações de caráter geral.
ou do local onde sediar-se a entidade que abrigar a criança ou
adolescente. Seção III
§ 3º Em caso de infração cometida através de transmissão Dos Serviços Auxiliares
simultânea de rádio ou televisão, que atinja mais de uma
comarca, será competente, para aplicação da penalidade, a Art. 150. Cabe ao Poder Judiciário, na elaboração de sua
autoridade judiciária do local da sede estadual da emissora ou proposta orçamentária, prever recursos para manutenção de
rede, tendo a sentença eficácia para todas as transmissoras ou equipe interprofissional, destinada a assessorar a Justiça da
retransmissoras do respectivo estado. Infância e da Juventude.

Art. 148. A Justiça da Infância e da Juventude é competente Art. 151. Compete à equipe interprofissional dentre outras
para: atribuições que lhe forem reservadas pela legislação local,
I - conhecer de representações promovidas pelo Ministério fornecer subsídios por escrito, mediante laudos, ou
Público, para apuração de ato infracional atribuído a verbalmente, na audiência, e bem assim desenvolver trabalhos
adolescente, aplicando as medidas cabíveis; de aconselhamento, orientação, encaminhamento, prevenção
II - conceder a remissão, como forma de suspensão ou e outros, tudo sob a imediata subordinação à autoridade
extinção do processo; judiciária, assegurada a livre manifestação do ponto de vista
III - conhecer de pedidos de adoção e seus incidentes; técnico.
IV - conhecer de ações civis fundadas em interesses
individuais, difusos ou coletivos afetos à criança e ao Capítulo III
adolescente, observado o disposto no art. 209; Dos Procedimentos
V - conhecer de ações decorrentes de irregularidades em Seção I
entidades de atendimento, aplicando as medidas cabíveis; Disposições Gerais
VI - aplicar penalidades administrativas nos casos de
infrações contra norma de proteção à criança ou adolescente; Art. 152. Aos procedimentos regulados nesta Lei aplicam-
VII - conhecer de casos encaminhados pelo Conselho se subsidiariamente as normas gerais previstas na legislação
Tutelar, aplicando as medidas cabíveis. processual pertinente.
Parágrafo único. Quando se tratar de criança ou Parágrafo único. É assegurada, sob pena de
adolescente nas hipóteses do art. 98, é também competente a responsabilidade, prioridade absoluta na tramitação dos
Justiça da Infância e da Juventude para o fim de: processos e procedimentos previstos nesta Lei, assim como na
a) conhecer de pedidos de guarda e tutela; execução dos atos e diligências judiciais a eles referentes.
b) conhecer de ações de destituição do poder familiar,
perda ou modificação da tutela ou guarda; Art. 153. Se a medida judicial a ser adotada não
c) suprir a capacidade ou o consentimento para o corresponder a procedimento previsto nesta ou em outra lei, a
casamento; autoridade judiciária poderá investigar os fatos e ordenar de
d) conhecer de pedidos baseados em discordância paterna ofício as providências necessárias, ouvido o Ministério
ou materna, em relação ao exercício do poder familiar; Público.
e) conceder a emancipação, nos termos da lei civil, quando Parágrafo único. O disposto neste artigo não se aplica para
faltarem os pais; o fim de afastamento da criança ou do adolescente de sua
f) designar curador especial em casos de apresentação de família de origem e em outros procedimentos
queixa ou representação, ou de outros procedimentos judiciais necessariamente contenciosos.
ou extrajudiciais em que haja interesses de criança ou
adolescente; Art. 154. Aplica-se às multas o disposto no art. 214.
g) conhecer de ações de alimentos;
h) determinar o cancelamento, a retificação e o suprimento Seção II
dos registros de nascimento e óbito. Da Perda e da Suspensão do Poder Familiar

Art. 149. Compete à autoridade judiciária disciplinar, Art. 155. O procedimento para a perda ou a suspensão do
através de portaria, ou autorizar, mediante alvará: poder familiar terá início por provocação do Ministério
I - a entrada e permanência de criança ou adolescente, Público ou de quem tenha legítimo interesse.
desacompanhado dos pais ou responsável, em:
a) estádio, ginásio e campo desportivo; Art. 156. A petição inicial indicará:
b) bailes ou promoções dançantes; I - a autoridade judiciária a que for dirigida;
c) boate ou congêneres; II - o nome, o estado civil, a profissão e a residência do
d) casa que explore comercialmente diversões eletrônicas; requerente e do requerido, dispensada a qualificação em se
e) estúdios cinematográficos, de teatro, rádio e televisão. tratando de pedido formulado por representante do
II - a participação de criança e adolescente em: Ministério Público;
a) espetáculos públicos e seus ensaios; III - a exposição sumária do fato e o pedido;
b) certames de beleza. IV - as provas que serão produzidas, oferecendo, desde
§ 1º Para os fins do disposto neste artigo, a autoridade logo, o rol de testemunhas e documentos.
judiciária levará em conta, dentre outros fatores:
a) os princípios desta Lei;

Legislação 28
Apostila Digital Licenciada para Alice Caroline Guarino dos Santos - alice.guarino@hotmail.com (Proibida a Revenda)
APOSTILAS OPÇÃO

Art. 157. Havendo motivo grave, poderá a autoridade o parecer técnico, salvo quando apresentado por escrito,
judiciária, ouvido o Ministério Público, decretar a suspensão manifestando-se sucessivamente o requerente, o requerido e
do poder familiar, liminar ou incidentalmente, até o o Ministério Público, pelo tempo de vinte minutos cada um,
julgamento definitivo da causa, ficando a criança ou prorrogável por mais dez. A decisão será proferida na
adolescente confiado a pessoa idônea, mediante termo de audiência, podendo a autoridade judiciária, excepcionalmente,
responsabilidade. designar data para sua leitura no prazo máximo de cinco dias.

Art. 158. O requerido será citado para, no prazo de dez Art. 163. O prazo máximo para conclusão do procedimento
dias, oferecer resposta escrita, indicando as provas a serem será de 120 (cento e vinte) dias.
produzidas e oferecendo desde logo o rol de testemunhas e Parágrafo único. A sentença que decretar a perda ou a
documentos. suspensão do poder familiar será averbada à margem do
§ 1º A citação será pessoal, salvo se esgotados todos os registro de nascimento da criança ou do adolescente.
meios para sua realização. (Incluído pela Lei nº 12.962, de
2014) Seção III
§ 2º O requerido privado de liberdade deverá ser citado Da Destituição da Tutela
pessoalmente. (Incluído pela Lei nº 12.962, de 2014)
Art. 164. Na destituição da tutela, observar-se-á o
Art. 159. Se o requerido não tiver possibilidade de procedimento para a remoção de tutor previsto na lei
constituir advogado, sem prejuízo do próprio sustento e de sua processual civil e, no que couber, o disposto na seção anterior.
família, poderá requerer, em cartório, que lhe seja nomeado
dativo, ao qual incumbirá a apresentação de resposta, Seção IV
contando-se o prazo a partir da intimação do despacho de Da Colocação em Família Substituta
nomeação.
Parágrafo único. Na hipótese de requerido privado de Art. 165. São requisitos para a concessão de pedidos de
liberdade, o oficial de justiça deverá perguntar, no momento colocação em família substituta:
da citação pessoal, se deseja que lhe seja nomeado defensor. I - qualificação completa do requerente e de seu eventual
(Incluído pela Lei nº 12.962, de 2014) cônjuge, ou companheiro, com expressa anuência deste;
II - indicação de eventual parentesco do requerente e de
Art. 160. Sendo necessário, a autoridade judiciária seu cônjuge, ou companheiro, com a criança ou adolescente,
requisitará de qualquer repartição ou órgão público a especificando se tem ou não parente vivo;
apresentação de documento que interesse à causa, de ofício ou III - qualificação completa da criança ou adolescente e de
a requerimento das partes ou do Ministério Público. seus pais, se conhecidos;
IV - indicação do cartório onde foi inscrito nascimento,
Art. 161. Não sendo contestado o pedido, a autoridade anexando, se possível, uma cópia da respectiva certidão;
judiciária dará vista dos autos ao Ministério Público, por cinco V - declaração sobre a existência de bens, direitos ou
dias, salvo quando este for o requerente, decidindo em igual rendimentos relativos à criança ou ao adolescente.
prazo. Parágrafo único. Em se tratando de adoção, observar-se-ão
§ 1º A autoridade judiciária, de ofício ou a requerimento também os requisitos específicos.
das partes ou do Ministério Público, determinará a realização
de estudo social ou perícia por equipe interprofissional ou Art. 166. Se os pais forem falecidos, tiverem sido
multidisciplinar, bem como a oitiva de testemunhas que destituídos ou suspensos do poder familiar, ou houverem
comprovem a presença de uma das causas de suspensão ou aderido expressamente ao pedido de colocação em família
destituição do poder familiar previstas nos arts. 1.637 e 1.638 substituta, este poderá ser formulado diretamente em
da Lei no 10.406, de 10 de janeiro de 2002 - Código Civil, ou no cartório, em petição assinada pelos próprios requerentes,
art. 24 desta Lei. dispensada a assistência de advogado.
§ 2º Em sendo os pais oriundos de comunidades indígenas, § 1º Na hipótese de concordância dos pais, esses serão
é ainda obrigatória a intervenção, junto à equipe profissional ouvidos pela autoridade judiciária e pelo representante do
ou multidisciplinar referida no § 1o deste artigo, de Ministério Público, tomando-se por termo as declarações.
representantes do órgão federal responsável pela política § 2º O consentimento dos titulares do poder familiar será
indigenista, observado o disposto no § 6o do art. 28 desta Lei. precedido de orientações e esclarecimentos prestados pela
§ 3º Se o pedido importar em modificação de guarda, será equipe interprofissional da Justiça da Infância e da Juventude,
obrigatória, desde que possível e razoável, a oitiva da criança em especial, no caso de adoção, sobre a irrevogabilidade da
ou adolescente, respeitado seu estágio de desenvolvimento e medida.
grau de compreensão sobre as implicações da medida. § 3º O consentimento dos titulares do poder familiar será
§ 4º É obrigatória a oitiva dos pais sempre que esses forem colhido pela autoridade judiciária competente em audiência,
identificados e estiverem em local conhecido. presente o Ministério Público, garantida a livre manifestação
§ 5º Se o pai ou a mãe estiverem privados de liberdade, a de vontade e esgotados os esforços para manutenção da
autoridade judicial requisitará sua apresentação para a oitiva. criança ou do adolescente na família natural ou extensa.
(Incluído pela Lei nº 12.962, de 2014) § 4º O consentimento prestado por escrito não terá
validade se não for ratificado na audiência a que se refere o §
Art. 162. Apresentada a resposta, a autoridade judiciária 3o deste artigo.
dará vista dos autos ao Ministério Público, por cinco dias, salvo § 5º O consentimento é retratável até a data da publicação
quando este for o requerente, designando, desde logo, da sentença constitutiva da adoção.
audiência de instrução e julgamento. § 6º O consentimento somente terá valor se for dado após
§ 1º A requerimento de qualquer das partes, do Ministério o nascimento da criança.
Público, ou de ofício, a autoridade judiciária poderá § 7º A família substituta receberá a devida orientação por
determinar a realização de estudo social ou, se possível, de intermédio de equipe técnica interprofissional a serviço do
perícia por equipe interprofissional. Poder Judiciário, preferencialmente com apoio dos técnicos
§ 2º Na audiência, presentes as partes e o Ministério responsáveis pela execução da política municipal de garantia
Público, serão ouvidas as testemunhas, colhendo-se oralmente do direito à convivência familiar.

Legislação 29
Apostila Digital Licenciada para Alice Caroline Guarino dos Santos - alice.guarino@hotmail.com (Proibida a Revenda)
APOSTILAS OPÇÃO

Art. 167. A autoridade judiciária, de ofício ou a internação para garantia de sua segurança pessoal ou
requerimento das partes ou do Ministério Público, manutenção da ordem pública.
determinará a realização de estudo social ou, se possível,
perícia por equipe interprofissional, decidindo sobre a Art. 175. Em caso de não liberação, a autoridade policial
concessão de guarda provisória, bem como, no caso de adoção, encaminhará, desde logo, o adolescente ao representante do
sobre o estágio de convivência. Ministério Público, juntamente com cópia do auto de
Parágrafo único. Deferida a concessão da guarda apreensão ou boletim de ocorrência.
provisória ou do estágio de convivência, a criança ou o § 1º Sendo impossível a apresentação imediata, a
adolescente será entregue ao interessado, mediante termo de autoridade policial encaminhará o adolescente à entidade de
responsabilidade. atendimento, que fará a apresentação ao representante do
Ministério Público no prazo de vinte e quatro horas.
Art. 168. Apresentado o relatório social ou o laudo pericial, § 2º Nas localidades onde não houver entidade de
e ouvida, sempre que possível, a criança ou o adolescente, dar- atendimento, a apresentação far-se-á pela autoridade policial.
se-á vista dos autos ao Ministério Público, pelo prazo de cinco À falta de repartição policial especializada, o adolescente
dias, decidindo a autoridade judiciária em igual prazo. aguardará a apresentação em dependência separada da
destinada a maiores, não podendo, em qualquer hipótese,
Art. 169. Nas hipóteses em que a destituição da tutela, a exceder o prazo referido no parágrafo anterior.
perda ou a suspensão do poder familiar constituir pressuposto
lógico da medida principal de colocação em família substituta, Art. 176. Sendo o adolescente liberado, a autoridade
será observado o procedimento contraditório previsto nas policial encaminhará imediatamente ao representante do
Seções II e III deste Capítulo. Ministério Público cópia do auto de apreensão ou boletim de
Parágrafo único. A perda ou a modificação da guarda ocorrência.
poderá ser decretada nos mesmos autos do procedimento,
observado o disposto no art. 35. Art. 177. Se, afastada a hipótese de flagrante, houver
indícios de participação de adolescente na prática de ato
Art. 170. Concedida a guarda ou a tutela, observar-se-á o infracional, a autoridade policial encaminhará ao
disposto no art. 32, e, quanto à adoção, o contido no art. 47. representante do Ministério Público relatório das
Parágrafo único. A colocação de criança ou adolescente investigações e demais documentos.
sob a guarda de pessoa inscrita em programa de acolhimento
familiar será comunicada pela autoridade judiciária à entidade Art. 178. O adolescente a quem se atribua autoria de ato
por este responsável no prazo máximo de 5 (cinco) dias. infracional não poderá ser conduzido ou transportado em
compartimento fechado de veículo policial, em condições
Seção V atentatórias à sua dignidade, ou que impliquem risco à sua
Da Apuração de Ato Infracional Atribuído a integridade física ou mental, sob pena de responsabilidade.
Adolescente
Art. 179. Apresentado o adolescente, o representante do
Art. 171. O adolescente apreendido por força de ordem Ministério Público, no mesmo dia e à vista do auto de
judicial será, desde logo, encaminhado à autoridade judiciária. apreensão, boletim de ocorrência ou relatório policial,
devidamente autuados pelo cartório judicial e com informação
Art. 172. O adolescente apreendido em flagrante de ato sobre os antecedentes do adolescente, procederá imediata e
infracional será, desde logo, encaminhado à autoridade informalmente à sua oitiva e, em sendo possível, de seus pais
policial competente. ou responsável, vítima e testemunhas.
Parágrafo único. Havendo repartição policial especializada Parágrafo único. Em caso de não apresentação, o
para atendimento de adolescente e em se tratando de ato representante do Ministério Público notificará os pais ou
infracional praticado em coautoria com maior, prevalecerá a responsável para apresentação do adolescente, podendo
atribuição da repartição especializada, que, após as requisitar o concurso das polícias civil e militar.
providências necessárias e conforme o caso, encaminhará o
adulto à repartição policial própria. Art. 180. Adotadas as providências a que alude o artigo
anterior, o representante do Ministério Público poderá:
Art. 173. Em caso de flagrante de ato infracional cometido I - promover o arquivamento dos autos;
mediante violência ou grave ameaça a pessoa, a autoridade II - conceder a remissão;
policial, sem prejuízo do disposto nos arts. 106, parágrafo III - representar à autoridade judiciária para aplicação de
único, e 107, deverá: medida sócio-educativa.
I - lavrar auto de apreensão, ouvidos as testemunhas e o
adolescente; Art. 181. Promovido o arquivamento dos autos ou
II - apreender o produto e os instrumentos da infração; concedida a remissão pelo representante do Ministério
III - requisitar os exames ou perícias necessários à Público, mediante termo fundamentado, que conterá o resumo
comprovação da materialidade e autoria da infração. dos fatos, os autos serão conclusos à autoridade judiciária para
Parágrafo único. Nas demais hipóteses de flagrante, a homologação.
lavratura do auto poderá ser substituída por boletim de § 1º Homologado o arquivamento ou a remissão, a
ocorrência circunstanciada. autoridade judiciária determinará, conforme o caso, o
cumprimento da medida.
Art. 174. Comparecendo qualquer dos pais ou responsável, § 2º Discordando, a autoridade judiciária fará remessa dos
o adolescente será prontamente liberado pela autoridade autos ao Procurador-Geral de Justiça, mediante despacho
policial, sob termo de compromisso e responsabilidade de sua fundamentado, e este oferecerá representação, designará
apresentação ao representante do Ministério Público, no outro membro do Ministério Público para apresentá-la, ou
mesmo dia ou, sendo impossível, no primeiro dia útil imediato, ratificará o arquivamento ou a remissão, que só então estará a
exceto quando, pela gravidade do ato infracional e sua autoridade judiciária obrigada a homologar.
repercussão social, deva o adolescente permanecer sob

Legislação 30
Apostila Digital Licenciada para Alice Caroline Guarino dos Santos - alice.guarino@hotmail.com (Proibida a Revenda)
APOSTILAS OPÇÃO

Art. 182. Se, por qualquer razão, o representante do Art. 187. Se o adolescente, devidamente notificado, não
Ministério Público não promover o arquivamento ou conceder comparecer, injustificadamente à audiência de apresentação, a
a remissão, oferecerá representação à autoridade judiciária, autoridade judiciária designará nova data, determinando sua
propondo a instauração de procedimento para aplicação da condução coercitiva.
medida sócio-educativa que se afigurar a mais adequada.
§ 1º A representação será oferecida por petição, que Art. 188. A remissão, como forma de extinção ou suspensão
conterá o breve resumo dos fatos e a classificação do ato do processo, poderá ser aplicada em qualquer fase do
infracional e, quando necessário, o rol de testemunhas, procedimento, antes da sentença.
podendo ser deduzida oralmente, em sessão diária instalada
pela autoridade judiciária. Art. 189. A autoridade judiciária não aplicará qualquer
§ 2º A representação independe de prova pré-constituída medida, desde que reconheça na sentença:
da autoria e materialidade. I - estar provada a inexistência do fato;
II - não haver prova da existência do fato;
Art. 183. O prazo máximo e improrrogável para a III - não constituir o fato ato infracional;
conclusão do procedimento, estando o adolescente internado IV - não existir prova de ter o adolescente concorrido para
provisoriamente, será de quarenta e cinco dias. o ato infracional.
Parágrafo único. Na hipótese deste artigo, estando o
Art. 184. Oferecida a representação, a autoridade judiciária adolescente internado, será imediatamente colocado em
designará audiência de apresentação do adolescente, liberdade.
decidindo, desde logo, sobre a decretação ou manutenção da
internação, observado o disposto no art. 108 e parágrafo. Art. 190. A intimação da sentença que aplicar medida de
§ 1º O adolescente e seus pais ou responsável serão internação ou regime de semiliberdade será feita:
cientificados do teor da representação, e notificados a I - ao adolescente e ao seu defensor;
comparecer à audiência, acompanhados de advogado. II - quando não for encontrado o adolescente, a seus pais
§ 2º Se os pais ou responsável não forem localizados, a ou responsável, sem prejuízo do defensor.
autoridade judiciária dará curador especial ao adolescente. § 1º Sendo outra a medida aplicada, a intimação far-se-á
§ 3º Não sendo localizado o adolescente, a autoridade unicamente na pessoa do defensor.
judiciária expedirá mandado de busca e apreensão, § 2º Recaindo a intimação na pessoa do adolescente,
determinando o sobrestamento do feito, até a efetiva deverá este manifestar se deseja ou não recorrer da sentença.
apresentação.
§ 4º Estando o adolescente internado, será requisitada a Seção VI
sua apresentação, sem prejuízo da notificação dos pais ou Da Apuração de Irregularidades em Entidade de
responsável. Atendimento

Art. 185. A internação, decretada ou mantida pela Art. 191. O procedimento de apuração de irregularidades
autoridade judiciária, não poderá ser cumprida em em entidade governamental e não-governamental terá início
estabelecimento prisional. mediante portaria da autoridade judiciária ou representação
§ 1º Inexistindo na comarca entidade com as do Ministério Público ou do Conselho Tutelar, onde conste,
características definidas no art. 123, o adolescente deverá ser necessariamente, resumo dos fatos.
imediatamente transferido para a localidade mais próxima. Parágrafo único. Havendo motivo grave, poderá a
§ 2º Sendo impossível a pronta transferência, o autoridade judiciária, ouvido o Ministério Público, decretar
adolescente aguardará sua remoção em repartição policial, liminarmente o afastamento provisório do dirigente da
desde que em seção isolada dos adultos e com instalações entidade, mediante decisão fundamentada.
apropriadas, não podendo ultrapassar o prazo máximo de
cinco dias, sob pena de responsabilidade. Art. 192. O dirigente da entidade será citado para, no prazo
Art. 186. Comparecendo o adolescente, seus pais ou de dez dias, oferecer resposta escrita, podendo juntar
responsável, a autoridade judiciária procederá à oitiva dos documentos e indicar as provas a produzir.
mesmos, podendo solicitar opinião de profissional qualificado.
§ 1º Se a autoridade judiciária entender adequada a Art. 193. Apresentada ou não a resposta, e sendo
remissão, ouvirá o representante do Ministério Público, necessário, a autoridade judiciária designará audiência de
proferindo decisão. instrução e julgamento, intimando as partes.
§ 2º Sendo o fato grave, passível de aplicação de medida de § 1º Salvo manifestação em audiência, as partes e o
internação ou colocação em regime de semiliberdade, a Ministério Público terão cinco dias para oferecer alegações
autoridade judiciária, verificando que o adolescente não finais, decidindo a autoridade judiciária em igual prazo.
possui advogado constituído, nomeará defensor, designando, § 2º Em se tratando de afastamento provisório ou
desde logo, audiência em continuação, podendo determinar a definitivo de dirigente de entidade governamental, a
realização de diligências e estudo do caso. autoridade judiciária oficiará à autoridade administrativa
§ 3º O advogado constituído ou o defensor nomeado, no imediatamente superior ao afastado, marcando prazo para a
prazo de três dias contado da audiência de apresentação, substituição.
oferecerá defesa prévia e rol de testemunhas. § 3º Antes de aplicar qualquer das medidas, a autoridade
§ 4º Na audiência em continuação, ouvidas as testemunhas judiciária poderá fixar prazo para a remoção das
arroladas na representação e na defesa prévia, cumpridas as irregularidades verificadas. Satisfeitas as exigências, o
diligências e juntado o relatório da equipe interprofissional, processo será extinto, sem julgamento de mérito.
será dada a palavra ao representante do Ministério Público e § 4º A multa e a advertência serão impostas ao dirigente da
ao defensor, sucessivamente, pelo tempo de vinte minutos entidade ou programa de atendimento.
para cada um, prorrogável por mais dez, a critério da
autoridade judiciária, que em seguida proferirá decisão.

Legislação 31
Apostila Digital Licenciada para Alice Caroline Guarino dos Santos - alice.guarino@hotmail.com (Proibida a Revenda)
APOSTILAS OPÇÃO

Seção VII Art. 197-C. Intervirá no feito, obrigatoriamente, equipe


Da Apuração de Infração Administrativa às Normas interprofissional a serviço da Justiça da Infância e da
de Proteção à Criança e ao Adolescente Juventude, que deverá elaborar estudo psicossocial, que
conterá subsídios que permitam aferir a capacidade e o
Art. 194. O procedimento para imposição de penalidade preparo dos postulantes para o exercício de uma paternidade
administrativa por infração às normas de proteção à criança e ou maternidade responsável, à luz dos requisitos e princípios
ao adolescente terá início por representação do Ministério desta Lei.
Público, ou do Conselho Tutelar, ou auto de infração elaborado § 1º É obrigatória a participação dos postulantes em
por servidor efetivo ou voluntário credenciado, e assinado por programa oferecido pela Justiça da Infância e da Juventude
duas testemunhas, se possível. preferencialmente com apoio dos técnicos responsáveis pela
§ 1º No procedimento iniciado com o auto de infração, execução da política municipal de garantia do direito à
poderão ser usadas fórmulas impressas, especificando-se a convivência familiar, que inclua preparação psicológica,
natureza e as circunstâncias da infração. orientação e estímulo à adoção inter-racial, de crianças
§ 2º Sempre que possível, à verificação da infração seguir- maiores ou de adolescentes, com necessidades específicas de
se-á a lavratura do auto, certificando-se, em caso contrário, dos saúde ou com deficiências e de grupos de irmãos.
motivos do retardamento. § 2º Sempre que possível e recomendável, a etapa
obrigatória da preparação referida no § 1o deste artigo
Art. 195. O requerido terá prazo de dez dias para incluirá o contato com crianças e adolescentes em regime de
apresentação de defesa, contado da data da intimação, que acolhimento familiar ou institucional em condições de serem
será feita: adotados, a ser realizado sob a orientação, supervisão e
I - pelo autuante, no próprio auto, quando este for lavrado avaliação da equipe técnica da Justiça da Infância e da
na presença do requerido; Juventude, com o apoio dos técnicos responsáveis pelo
II - por oficial de justiça ou funcionário legalmente programa de acolhimento familiar ou institucional e pela
habilitado, que entregará cópia do auto ou da representação execução da política municipal de garantia do direito à
ao requerido, ou a seu representante legal, lavrando certidão; convivência familiar.
III - por via postal, com aviso de recebimento, se não for
encontrado o requerido ou seu representante legal; Art. 197-D. Certificada nos autos a conclusão da
IV - por edital, com prazo de trinta dias, se incerto ou não participação no programa referido no art. 197-C desta Lei, a
sabido o paradeiro do requerido ou de seu representante legal. autoridade judiciária, no prazo de 48 (quarenta e oito) horas,
Art. 196. Não sendo apresentada a defesa no prazo legal, a decidirá acerca das diligências requeridas pelo Ministério
autoridade judiciária dará vista dos autos do Ministério Público e determinará a juntada do estudo psicossocial,
Público, por cinco dias, decidindo em igual prazo. designando, conforme o caso, audiência de instrução e
julgamento.
Art. 197. Apresentada a defesa, a autoridade judiciária Parágrafo único. Caso não sejam requeridas diligências, ou
procederá na conformidade do artigo anterior, ou, sendo sendo essas indeferidas, a autoridade judiciária determinará a
necessário, designará audiência de instrução e julgamento. juntada do estudo psicossocial, abrindo a seguir vista dos
Parágrafo único. Colhida a prova oral, manifestar-se-ão autos ao Ministério Público, por 5 (cinco) dias, decidindo em
sucessivamente o Ministério Público e o procurador do igual prazo.
requerido, pelo tempo de vinte minutos para cada um,
prorrogável por mais dez, a critério da autoridade judiciária, Art. 197-E. Deferida a habilitação, o postulante será
que em seguida proferirá sentença. inscrito nos cadastros referidos no art. 50 desta Lei, sendo a
sua convocação para a adoção feita de acordo com ordem
Seção VIII cronológica de habilitação e conforme a disponibilidade de
Da Habilitação de Pretendentes à Adoção crianças ou adolescentes adotáveis.
§ 1º A ordem cronológica das habilitações somente poderá
Art. 197-A. Os postulantes à adoção, domiciliados no Brasil, deixar de ser observada pela autoridade judiciária nas
apresentarão petição inicial na qual conste: hipóteses previstas no § 13 do art. 50 desta Lei, quando
I - qualificação completa; comprovado ser essa a melhor solução no interesse do
II - dados familiares; adotando.
III - cópias autenticadas de certidão de nascimento ou § 2º A recusa sistemática na adoção das crianças ou
casamento, ou declaração relativa ao período de união estável; adolescentes indicados importará na reavaliação da
IV - cópias da cédula de identidade e inscrição no Cadastro habilitação concedida.
de Pessoas Físicas;
V - comprovante de renda e domicílio; Capítulo IV
VI - atestados de sanidade física e mental; Dos Recursos
VII - certidão de antecedentes criminais;
VIII - certidão negativa de distribuição cível. Art. 198. Nos procedimentos afetos à Justiça da Infância e
da Juventude, inclusive os relativos à execução das medidas
Art. 197-B. A autoridade judiciária, no prazo de 48 socioeducativas, adotar-se-á o sistema recursal da Lei no
(quarenta e oito) horas, dará vista dos autos ao Ministério 5.869, de 11 de janeiro de 1973 (Código de Processo Civil),
Público, que no prazo de 5 (cinco) dias poderá: com as seguintes adaptações:
I - apresentar quesitos a serem respondidos pela equipe I - os recursos serão interpostos independentemente de
interprofissional encarregada de elaborar o estudo técnico a preparo;
que se refere o art. 197-C desta Lei; II - em todos os recursos, salvo nos embargos de
II - requerer a designação de audiência para oitiva dos declaração, o prazo para o Ministério Público e para a defesa
postulantes em juízo e testemunhas; será sempre de 10 (dez) dias;
III - requerer a juntada de documentos complementares e III - os recursos terão preferência de julgamento e
a realização de outras diligências que entender necessárias. dispensarão revisor;
IV - Revogado
V - Revogado

Legislação 32
Apostila Digital Licenciada para Alice Caroline Guarino dos Santos - alice.guarino@hotmail.com (Proibida a Revenda)
APOSTILAS OPÇÃO

VI - Revogado relativos à infância e à adolescência, inclusive os definidos no


VII - antes de determinar a remessa dos autos à superior art. 220, § 3º inciso II, da Constituição Federal;
instância, no caso de apelação, ou do instrumento, no caso de VI - instaurar procedimentos administrativos e, para
agravo, a autoridade judiciária proferirá despacho instruí-los:
fundamentado, mantendo ou reformando a decisão, no prazo a) expedir notificações para colher depoimentos ou
de cinco dias; esclarecimentos e, em caso de não comparecimento
VIII - mantida a decisão apelada ou agravada, o escrivão injustificado, requisitar condução coercitiva, inclusive pela
remeterá os autos ou o instrumento à superior instância polícia civil ou militar;
dentro de vinte e quatro horas, independentemente de novo b) requisitar informações, exames, perícias e documentos
pedido do recorrente; se a reformar, a remessa dos autos de autoridades municipais, estaduais e federais, da
dependerá de pedido expresso da parte interessada ou do administração direta ou indireta, bem como promover
Ministério Público, no prazo de cinco dias, contados da inspeções e diligências investigatórias;
intimação. c) requisitar informações e documentos a particulares e
instituições privadas;
Art. 199. Contra as decisões proferidas com base no art. VII - instaurar sindicâncias, requisitar diligências
149 caberá recurso de apelação. investigatórias e determinar a instauração de inquérito
policial, para apuração de ilícitos ou infrações às normas de
Art. 199-A. A sentença que deferir a adoção produz efeito proteção à infância e à juventude;
desde logo, embora sujeita a apelação, que será recebida VIII - zelar pelo efetivo respeito aos direitos e garantias
exclusivamente no efeito devolutivo, salvo se se tratar de legais assegurados às crianças e adolescentes, promovendo as
adoção internacional ou se houver perigo de dano irreparável medidas judiciais e extrajudiciais cabíveis;
ou de difícil reparação ao adotando IX - impetrar mandado de segurança, de injunção e habeas
corpus, em qualquer juízo, instância ou tribunal, na defesa dos
Art. 199-B. A sentença que destituir ambos ou qualquer interesses sociais e individuais indisponíveis afetos à criança e
dos genitores do poder familiar fica sujeita a apelação, que ao adolescente;
deverá ser recebida apenas no efeito devolutivo. X - representar ao juízo visando à aplicação de penalidade
por infrações cometidas contra as normas de proteção à
Art. 199-C. Os recursos nos procedimentos de adoção e de infância e à juventude, sem prejuízo da promoção da
destituição de poder familiar, em face da relevância das responsabilidade civil e penal do infrator, quando cabível;
questões, serão processados com prioridade absoluta, XI - inspecionar as entidades públicas e particulares de
devendo ser imediatamente distribuídos, ficando vedado que atendimento e os programas de que trata esta Lei, adotando de
aguardem, em qualquer situação, oportuna distribuição, e pronto as medidas administrativas ou judiciais necessárias à
serão colocados em mesa para julgamento sem revisão e com remoção de irregularidades porventura verificadas;
parecer urgente do Ministério Público. XII - requisitar força policial, bem como a colaboração dos
serviços médicos, hospitalares, educacionais e de assistência
Art. 199-D. O relator deverá colocar o processo em mesa social, públicos ou privados, para o desempenho de suas
para julgamento no prazo máximo de 60 (sessenta) dias, atribuições.
contado da sua conclusão. § 1º A legitimação do Ministério Público para as ações
Parágrafo único. O Ministério Público será intimado da cíveis previstas neste artigo não impede a de terceiros, nas
data do julgamento e poderá na sessão, se entender mesmas hipóteses, segundo dispuserem a Constituição e esta
necessário, apresentar oralmente seu parecer. Lei.
§ 2º As atribuições constantes deste artigo não excluem
Art. 199-E. O Ministério Público poderá requerer a outras, desde que compatíveis com a finalidade do Ministério
instauração de procedimento para apuração de Público.
responsabilidades se constatar o descumprimento das § 3º O representante do Ministério Público, no exercício de
providências e do prazo previstos nos artigos anteriores. suas funções, terá livre acesso a todo local onde se encontre
criança ou adolescente.
Capítulo V § 4º O representante do Ministério Público será
Do Ministério Público responsável pelo uso indevido das informações e documentos
que requisitar, nas hipóteses legais de sigilo.
Art. 200. As funções do Ministério Público previstas nesta § 5º Para o exercício da atribuição de que trata o inciso VIII
Lei serão exercidas nos termos da respectiva lei orgânica. deste artigo, poderá o representante do Ministério Público:
a) reduzir a termo as declarações do reclamante,
Art. 201. Compete ao Ministério Público: instaurando o competente procedimento, sob sua presidência;
I - conceder a remissão como forma de exclusão do b) entender-se diretamente com a pessoa ou autoridade
processo; reclamada, em dia, local e horário previamente notificados ou
II - promover e acompanhar os procedimentos relativos às acertados;
infrações atribuídas a adolescentes; c) efetuar recomendações visando à melhoria dos serviços
III - promover e acompanhar as ações de alimentos e os públicos e de relevância pública afetos à criança e ao
procedimentos de suspensão e destituição do poder familiar, adolescente, fixando prazo razoável para sua perfeita
nomeação e remoção de tutores, curadores e guardiães, bem adequação.
como oficiar em todos os demais procedimentos da
competência da Justiça da Infância e da Juventude; Art. 202. Nos processos e procedimentos em que não for
IV - promover, de ofício ou por solicitação dos parte, atuará obrigatoriamente o Ministério Público na defesa
interessados, a especialização e a inscrição de hipoteca legal e dos direitos e interesses de que cuida esta Lei, hipótese em que
a prestação de contas dos tutores, curadores e quaisquer terá vista dos autos depois das partes, podendo juntar
administradores de bens de crianças e adolescentes nas documentos e requerer diligências, usando os recursos
hipóteses do art. 98; cabíveis.
V - promover o inquérito civil e a ação civil pública para a
proteção dos interesses individuais, difusos ou coletivos

Legislação 33
Apostila Digital Licenciada para Alice Caroline Guarino dos Santos - alice.guarino@hotmail.com (Proibida a Revenda)
APOSTILAS OPÇÃO

Art. 203. A intimação do Ministério Público, em qualquer § 2º A investigação do desaparecimento de crianças ou


caso, será feita pessoalmente. adolescentes será realizada imediatamente após notificação
aos órgãos competentes, que deverão comunicar o fato aos
Art. 204. A falta de intervenção do Ministério Público portos, aeroportos, Polícia Rodoviária e companhias de
acarreta a nulidade do feito, que será declarada de ofício pelo transporte interestaduais e internacionais, fornecendo-lhes
juiz ou a requerimento de qualquer interessado. todos os dados necessários à identificação do desaparecido.

Art. 205. As manifestações processuais do representante Art. 209. As ações previstas neste Capítulo serão propostas
do Ministério Público deverão ser fundamentadas. no foro do local onde ocorreu ou deva ocorrer a ação ou
omissão, cujo juízo terá competência absoluta para processar
Capítulo VI a causa, ressalvadas a competência da Justiça Federal e a
Do Advogado competência originária dos tribunais superiores.

Art. 206. A criança ou o adolescente, seus pais ou Art. 210. Para as ações cíveis fundadas em interesses
responsável, e qualquer pessoa que tenha legítimo interesse coletivos ou difusos, consideram-se legitimados
na solução da lide poderão intervir nos procedimentos de que concorrentemente:
trata esta Lei, através de advogado, o qual será intimado para I - o Ministério Público;
todos os atos, pessoalmente ou por publicação oficial, II - a União, os estados, os municípios, o Distrito Federal e
respeitado o segredo de justiça. os territórios;
Parágrafo único. Será prestada assistência judiciária III - as associações legalmente constituídas há pelo menos
integral e gratuita àqueles que dela necessitarem. um ano e que incluam entre seus fins institucionais a defesa
dos interesses e direitos protegidos por esta Lei, dispensada a
Art. 207. Nenhum adolescente a quem se atribua a prática autorização da assembleia, se houver prévia autorização
de ato infracional, ainda que ausente ou foragido, será estatutária.
processado sem defensor. § 1º Admitir-se-á litisconsórcio facultativo entre os
§ 1º Se o adolescente não tiver defensor, ser-lhe-á Ministérios Públicos da União e dos estados na defesa dos
nomeado pelo juiz, ressalvado o direito de, a todo tempo, interesses e direitos de que cuida esta Lei.
constituir outro de sua preferência. § 2º Em caso de desistência ou abandono da ação por
§ 2º A ausência do defensor não determinará o adiamento associação legitimada, o Ministério Público ou outro
de nenhum ato do processo, devendo o juiz nomear substituto, legitimado poderá assumir a titularidade ativa.
ainda que provisoriamente, ou para o só efeito do ato.
§ 3º Será dispensada a outorga de mandato, quando se Art. 211. Os órgãos públicos legitimados poderão tomar
tratar de defensor nomeado ou, sido constituído, tiver sido dos interessados compromisso de ajustamento de sua conduta
indicado por ocasião de ato formal com a presença da às exigências legais, o qual terá eficácia de título executivo
autoridade judiciária. extrajudicial.
Art. 212. Para defesa dos direitos e interesses protegidos
Capítulo VII por esta Lei, são admissíveis todas as espécies de ações
Da Proteção Judicial dos Interesses Individuais, pertinentes.
Difusos e Coletivos § 1º Aplicam-se às ações previstas neste Capítulo as
normas do Código de Processo Civil.
Art. 208. Regem-se pelas disposições desta Lei as ações de § 2º Contra atos ilegais ou abusivos de autoridade pública
responsabilidade por ofensa aos direitos assegurados à ou agente de pessoa jurídica no exercício de atribuições do
criança e ao adolescente, referentes ao não oferecimento ou poder público, que lesem direito líquido e certo previsto nesta
oferta irregular: Lei, caberá ação mandamental, que se regerá pelas normas da
I - do ensino obrigatório; lei do mandado de segurança.
II - de atendimento educacional especializado aos
portadores de deficiência; Art. 213. Na ação que tenha por objeto o cumprimento de
III – de atendimento em creche e pré-escola às crianças de obrigação de fazer ou não fazer, o juiz concederá a tutela
zero a cinco anos de idade; (Redação dada pela Lei nº 13.306, específica da obrigação ou determinará providências que
de 2016) assegurem o resultado prático equivalente ao do
IV - de ensino noturno regular, adequado às condições do adimplemento.
educando; § 1º Sendo relevante o fundamento da demanda e havendo
V - de programas suplementares de oferta de material justificado receio de ineficácia do provimento final, é lícito ao
didático-escolar, transporte e assistência à saúde do juiz conceder a tutela liminarmente ou após justificação
educando do ensino fundamental; prévia, citando o réu.
VI - de serviço de assistência social visando à proteção à § 2º O juiz poderá, na hipótese do parágrafo anterior ou na
família, à maternidade, à infância e à adolescência, bem como sentença, impor multa diária ao réu, independentemente de
ao amparo às crianças e adolescentes que dele necessitem; pedido do autor, se for suficiente ou compatível com a
VII - de acesso às ações e serviços de saúde; obrigação, fixando prazo razoável para o cumprimento do
VIII - de escolarização e profissionalização dos preceito.
adolescentes privados de liberdade. § 3º A multa só será exigível do réu após o trânsito em
IX - de ações, serviços e programas de orientação, apoio e julgado da sentença favorável ao autor, mas será devida desde
promoção social de famílias e destinados ao pleno exercício do o dia em que se houver configurado o descumprimento.
direito à convivência familiar por crianças e adolescentes.
X - de programas de atendimento para a execução das Art. 214. Os valores das multas reverterão ao fundo gerido
medidas socioeducativas e aplicação de medidas de proteção. pelo Conselho dos Direitos da Criança e do Adolescente do
§ 1º As hipóteses previstas neste artigo não excluem da respectivo município.
proteção judicial outros interesses individuais, difusos ou § 1º As multas não recolhidas até trinta dias após o trânsito
coletivos, próprios da infância e da adolescência, protegidos em julgado da decisão serão exigidas através de execução
pela Constituição e pela Lei.

Legislação 34
Apostila Digital Licenciada para Alice Caroline Guarino dos Santos - alice.guarino@hotmail.com (Proibida a Revenda)
APOSTILAS OPÇÃO

promovida pelo Ministério Público, nos mesmos autos, § 4º A promoção de arquivamento será submetida a exame
facultada igual iniciativa aos demais legitimados. e deliberação do Conselho Superior do Ministério Público,
§ 2º Enquanto o fundo não for regulamentado, o dinheiro conforme dispuser o seu regimento.
ficará depositado em estabelecimento oficial de crédito, em § 5º Deixando o Conselho Superior de homologar a
conta com correção monetária. promoção de arquivamento, designará, desde logo, outro
órgão do Ministério Público para o ajuizamento da ação.
Art. 215. O juiz poderá conferir efeito suspensivo aos
recursos, para evitar dano irreparável à parte. Art. 224. Aplicam-se subsidiariamente, no que couber, as
disposições da Lei n.º 7.347, de 24 de julho de 1985.
Art. 216. Transitada em julgado a sentença que impuser
condenação ao poder público, o juiz determinará a remessa de Título VII
peças à autoridade competente, para apuração da Dos Crimes e Das Infrações Administrativas
responsabilidade civil e administrativa do agente a que se Capítulo I
atribua a ação ou omissão. Dos Crimes
Seção I
Art. 217. Decorridos sessenta dias do trânsito em julgado Disposições Gerais
da sentença condenatória sem que a associação autora lhe
promova a execução, deverá fazê-lo o Ministério Público, Art. 225. Este Capítulo dispõe sobre crimes praticados
facultada igual iniciativa aos demais legitimados. contra a criança e o adolescente, por ação ou omissão, sem
prejuízo do disposto na legislação penal.
Art. 218. O juiz condenará a associação autora a pagar ao
réu os honorários advocatícios arbitrados na conformidade do Art. 226. Aplicam-se aos crimes definidos nesta Lei as
§ 4º do art. 20 da Lei n.º 5.869, de 11 de janeiro de 1973 normas da Parte Geral do Código Penal e, quanto ao processo,
(Código de Processo Civil), quando reconhecer que a as pertinentes ao Código de Processo Penal.
pretensão é manifestamente infundada.
Parágrafo único. Em caso de litigância de má-fé, a Art. 227. Os crimes definidos nesta Lei são de ação pública
associação autora e os diretores responsáveis pela incondicionada
propositura da ação serão solidariamente condenados ao
décuplo das custas, sem prejuízo de responsabilidade por Seção II
perdas e danos. Dos Crimes em Espécie

Art. 219. Nas ações de que trata este Capítulo, não haverá Art. 228. Deixar o encarregado de serviço ou o dirigente de
adiantamento de custas, emolumentos, honorários periciais e estabelecimento de atenção à saúde de gestante de manter
quaisquer outras despesas. registro das atividades desenvolvidas, na forma e prazo
referidos no art. 10 desta Lei, bem como de fornecer à
Art. 220. Qualquer pessoa poderá e o servidor público parturiente ou a seu responsável, por ocasião da alta médica,
deverá provocar a iniciativa do Ministério Público, prestando- declaração de nascimento, onde constem as intercorrências do
lhe informações sobre fatos que constituam objeto de ação parto e do desenvolvimento do neonato:
civil, e indicando-lhe os elementos de convicção. Pena - detenção de seis meses a dois anos.
Parágrafo único. Se o crime é culposo:
Art. 221. Se, no exercício de suas funções, os juízos e Pena - detenção de dois a seis meses, ou multa.
tribunais tiverem conhecimento de fatos que possam ensejar a
propositura de ação civil, remeterão peças ao Ministério Art. 229. Deixar o médico, enfermeiro ou dirigente de
Público para as providências cabíveis. estabelecimento de atenção à saúde de gestante de identificar
corretamente o neonato e a parturiente, por ocasião do parto,
Art. 222. Para instruir a petição inicial, o interessado bem como deixar de proceder aos exames referidos no art. 10
poderá requerer às autoridades competentes as certidões e desta Lei:
informações que julgar necessárias, que serão fornecidas no Pena - detenção de seis meses a dois anos.
prazo de quinze dias. Parágrafo único. Se o crime é culposo:
Pena - detenção de dois a seis meses, ou multa.
Art. 223. O Ministério Público poderá instaurar, sob sua Art. 230. Privar a criança ou o adolescente de sua
presidência, inquérito civil, ou requisitar, de qualquer pessoa, liberdade, procedendo à sua apreensão sem estar em flagrante
organismo público ou particular, certidões, informações, de ato infracional ou inexistindo ordem escrita da autoridade
exames ou perícias, no prazo que assinalar, o qual não poderá judiciária competente:
ser inferior a dez dias úteis. Pena - detenção de seis meses a dois anos.
§ 1º Se o órgão do Ministério Público, esgotadas todas as Parágrafo único. Incide na mesma pena aquele que
diligências, se convencer da inexistência de fundamento para procede à apreensão sem observância das formalidades legais.
a propositura da ação cível, promoverá o arquivamento dos
autos do inquérito civil ou das peças informativas, fazendo-o Art. 231. Deixar a autoridade policial responsável pela
fundamentadamente. apreensão de criança ou adolescente de fazer imediata
§ 2º Os autos do inquérito civil ou as peças de informação comunicação à autoridade judiciária competente e à família do
arquivados serão remetidos, sob pena de se incorrer em falta apreendido ou à pessoa por ele indicada:
grave, no prazo de três dias, ao Conselho Superior do Pena - detenção de seis meses a dois anos.
Ministério Público.
§ 3º Até que seja homologada ou rejeitada a promoção de Art. 232. Submeter criança ou adolescente sob sua
arquivamento, em sessão do Conselho Superior do Ministério autoridade, guarda ou vigilância a vexame ou a
público, poderão as associações legitimadas apresentar razões constrangimento:
escritas ou documentos, que serão juntados aos autos do Pena - detenção de seis meses a dois anos.
inquérito ou anexados às peças de informação.
Art. 233. Revogado

Legislação 35
Apostila Digital Licenciada para Alice Caroline Guarino dos Santos - alice.guarino@hotmail.com (Proibida a Revenda)
APOSTILAS OPÇÃO

Art. 234. Deixar a autoridade competente, sem justa causa, II – assegura, por qualquer meio, o acesso por rede de
de ordenar a imediata liberação de criança ou adolescente, tão computadores às fotografias, cenas ou imagens de que trata o
logo tenha conhecimento da ilegalidade da apreensão: caput deste artigo.
Pena - detenção de seis meses a dois anos. § 2º As condutas tipificadas nos incisos I e II do § 1o deste
artigo são puníveis quando o responsável legal pela prestação
Art. 235. Descumprir, injustificadamente, prazo fixado do serviço, oficialmente notificado, deixa de desabilitar o
nesta Lei em benefício de adolescente privado de liberdade: acesso ao conteúdo ilícito de que trata o caput deste artigo.
Pena - detenção de seis meses a dois anos.
Art. 241-B. Adquirir, possuir ou armazenar, por qualquer
Art. 236. Impedir ou embaraçar a ação de autoridade meio, fotografia, vídeo ou outra forma de registro que
judiciária, membro do Conselho Tutelar ou representante do contenha cena de sexo explícito ou pornográfica envolvendo
Ministério Público no exercício de função prevista nesta Lei: criança ou adolescente:
Pena - detenção de seis meses a dois anos. Pena – reclusão, de 1 (um) a 4 (quatro) anos, e multa.
§ 1º A pena é diminuída de 1 (um) a 2/3 (dois terços) se
Art. 237. Subtrair criança ou adolescente ao poder de quem de pequena quantidade o material a que se refere o caput deste
o tem sob sua guarda em virtude de lei ou ordem judicial, com artigo.
o fim de colocação em lar substituto: § 2º Não há crime se a posse ou o armazenamento tem a
Pena - reclusão de dois a seis anos, e multa. finalidade de comunicar às autoridades competentes a
ocorrência das condutas descritas nos arts. 240, 241, 241-A e
Art. 238. Prometer ou efetivar a entrega de filho ou pupilo 241-C desta Lei, quando a comunicação for feita por:
a terceiro, mediante paga ou recompensa: I – agente público no exercício de suas funções;
Pena - reclusão de um a quatro anos, e multa. II – membro de entidade, legalmente constituída, que
Parágrafo único. Incide nas mesmas penas quem oferece inclua, entre suas finalidades institucionais, o recebimento, o
ou efetiva a paga ou recompensa. processamento e o encaminhamento de notícia dos crimes
referidos neste parágrafo;
Art. 239. Promover ou auxiliar a efetivação de ato III – representante legal e funcionários responsáveis de
destinado ao envio de criança ou adolescente para o exterior provedor de acesso ou serviço prestado por meio de rede de
com inobservância das formalidades legais ou com o fito de computadores, até o recebimento do material relativo à notícia
obter lucro: feita à autoridade policial, ao Ministério Público ou ao Poder
Pena - reclusão de quatro a seis anos, e multa. Judiciário.
Parágrafo único. Se há emprego de violência, grave ameaça § 3º As pessoas referidas no § 2o deste artigo deverão
ou fraude: manter sob sigilo o material ilícito referido.
Pena - reclusão, de 6 (seis) a 8 (oito) anos, além da pena
correspondente à violência. Art. 241-C. Simular a participação de criança ou
adolescente em cena de sexo explícito ou pornográfica por
Art. 240. Produzir, reproduzir, dirigir, fotografar, filmar ou meio de adulteração, montagem ou modificação de fotografia,
registrar, por qualquer meio, cena de sexo explícito ou vídeo ou qualquer outra forma de representação visual:
pornográfica, envolvendo criança ou adolescente: Pena – reclusão, de 1 (um) a 3 (três) anos, e multa.
Pena – reclusão, de 4 (quatro) a 8 (oito) anos, e multa. Parágrafo único. Incorre nas mesmas penas quem vende,
§ 1º Incorre nas mesmas penas quem agencia, facilita, expõe à venda, disponibiliza, distribui, publica ou divulga por
recruta, coage, ou de qualquer modo intermedeia a qualquer meio, adquire, possui ou armazena o material
participação de criança ou adolescente nas cenas referidas no produzido na forma do caput deste artigo.
caput deste artigo, ou ainda quem com esses contracena.
§ 2º Aumenta-se a pena de 1/3 (um terço) se o agente Art. 241-D. Aliciar, assediar, instigar ou constranger, por
comete o crime: qualquer meio de comunicação, criança, com o fim de com ela
I – no exercício de cargo ou função pública ou a pretexto de praticar ato libidinoso:
exercê-la; Pena – reclusão, de 1 (um) a 3 (três) anos, e multa.
II – prevalecendo-se de relações domésticas, de coabitação Parágrafo único. Nas mesmas penas incorre quem:
ou de hospitalidade; ou I – facilita ou induz o acesso à criança de material contendo
III – prevalecendo-se de relações de parentesco cena de sexo explícito ou pornográfica com o fim de com ela
consanguíneo ou afim até o terceiro grau, ou por adoção, de praticar ato libidinoso;
tutor, curador, preceptor, empregador da vítima ou de quem, II – pratica as condutas descritas no caput deste artigo com
a qualquer outro título, tenha autoridade sobre ela, ou com seu o fim de induzir criança a se exibir de forma pornográfica ou
consentimento. sexualmente explícita.

Art. 241. Vender ou expor à venda fotografia, vídeo ou Art. 241-E. Para efeito dos crimes previstos nesta Lei, a
outro registro que contenha cena de sexo explícito ou expressão “cena de sexo explícito ou pornográfica”
pornográfica envolvendo criança ou adolescente: compreende qualquer situação que envolva criança ou
Pena – reclusão, de 4 (quatro) a 8 (oito) anos, e multa. adolescente em atividades sexuais explícitas, reais ou
simuladas, ou exibição dos órgãos genitais de uma criança ou
Art. 241-A. Oferecer, trocar, disponibilizar, transmitir, adolescente para fins primordialmente sexuais
distribuir, publicar ou divulgar por qualquer meio, inclusive
por meio de sistema de informática ou telemático, fotografia, Art. 242. Vender, fornecer ainda que gratuitamente ou
vídeo ou outro registro que contenha cena de sexo explícito ou entregar, de qualquer forma, a criança ou adolescente arma,
pornográfica envolvendo criança ou adolescente: munição ou explosivo:
Pena – reclusão, de 3 (três) a 6 (seis) anos, e multa. Pena - reclusão, de 3 (três) a 6 (seis) anos.
§ 1º Nas mesmas penas incorre quem:
I – assegura os meios ou serviços para o armazenamento Art. 243. Vender, fornecer, servir, ministrar ou entregar,
das fotografias, cenas ou imagens de que trata o caput deste ainda que gratuitamente, de qualquer forma, a criança ou a
artigo; adolescente, bebida alcoólica ou, sem justa causa, outros

Legislação 36
Apostila Digital Licenciada para Alice Caroline Guarino dos Santos - alice.guarino@hotmail.com (Proibida a Revenda)
APOSTILAS OPÇÃO

produtos cujos componentes possam causar dependência por dois dias, bem como da publicação do periódico até por
física ou psíquica: (Redação dada pela Lei nº 13.106, de 2015) dois números. (Expressão declara inconstitucional pela ADIN
Pena - detenção de 2 (dois) a 4 (quatro) anos, e multa, se o 869-2)
fato não constitui crime mais grave. (Redação dada pela
Lei nº 13.106, de 2015) Art. 248. Deixar de apresentar à autoridade judiciária de
seu domicílio, no prazo de cinco dias, com o fim de regularizar
Art. 244. Vender, fornecer ainda que gratuitamente ou a guarda, adolescente trazido de outra comarca para a
entregar, de qualquer forma, a criança ou adolescente fogos de prestação de serviço doméstico, mesmo que autorizado pelos
estampido ou de artifício, exceto aqueles que, pelo seu pais ou responsável:
reduzido potencial, sejam incapazes de provocar qualquer Pena - multa de três a vinte salários de referência,
dano físico em caso de utilização indevida: aplicando-se o dobro em caso de reincidência,
Pena - detenção de seis meses a dois anos, e multa. independentemente das despesas de retorno do adolescente,
se for o caso.
Art. 244-A. Submeter criança ou adolescente, como tais
definidos no caput do art. 2o desta Lei, à prostituição ou à Art. 249. Descumprir, dolosa ou culposamente, os deveres
exploração sexual: inerentes ao poder familiar ou decorrente de tutela ou guarda,
Pena – reclusão de quatro a dez anos, e multa. bem assim determinação da autoridade judiciária ou Conselho
§ 1º Incorrem nas mesmas penas o proprietário, o gerente Tutelar:
ou o responsável pelo local em que se verifique a submissão de Pena - multa de três a vinte salários de referência,
criança ou adolescente às práticas referidas no caput deste aplicando-se o dobro em caso de reincidência.
artigo.
§ 2º Constitui efeito obrigatório da condenação a cassação Art. 250. Hospedar criança ou adolescente
da licença de localização e de funcionamento do desacompanhado dos pais ou responsável, ou sem autorização
estabelecimento. escrita desses ou da autoridade judiciária, em hotel, pensão,
motel ou congênere:
Art. 244-B. Corromper ou facilitar a corrupção de menor Pena – multa.
de 18 (dezoito) anos, com ele praticando infração penal ou § 1º Em caso de reincidência, sem prejuízo da pena de
induzindo-o a praticá-la: multa, a autoridade judiciária poderá determinar o
Pena - reclusão, de 1 (um) a 4 (quatro) anos. fechamento do estabelecimento por até 15 (quinze) dias.
§ 1º Incorre nas penas previstas no caput deste artigo § 2º Se comprovada a reincidência em período inferior a
quem pratica as condutas ali tipificadas utilizando-se de 30 (trinta) dias, o estabelecimento será definitivamente
quaisquer meios eletrônicos, inclusive salas de bate-papo da fechado e terá sua licença cassada.
internet.
§ 2º As penas previstas no caput deste artigo são Art. 251. Transportar criança ou adolescente, por qualquer
aumentadas de um terço no caso de a infração cometida ou meio, com inobservância do disposto nos arts. 83, 84 e 85
induzida estar incluída no rol do art. 1o da Lei no 8.072, de 25 desta Lei:
de julho de 1990. Capítulo II Pena - multa de três a vinte salários de referência,
aplicando-se o dobro em caso de reincidência.
Das Infrações Administrativas
Art. 252. Deixar o responsável por diversão ou espetáculo
Art. 245. Deixar o médico, professor ou responsável por público de afixar, em lugar visível e de fácil acesso, à entrada
estabelecimento de atenção à saúde e de ensino fundamental, do local de exibição, informação destacada sobre a natureza da
pré-escola ou creche, de comunicar à autoridade competente diversão ou espetáculo e a faixa etária especificada no
os casos de que tenha conhecimento, envolvendo suspeita ou certificado de classificação:
confirmação de maus-tratos contra criança ou adolescente: Pena - multa de três a vinte salários de referência,
Pena - multa de três a vinte salários de referência, aplicando-se o dobro em caso de reincidência.
aplicando-se o dobro em caso de reincidência.
Art. 253. Anunciar peças teatrais, filmes ou quaisquer
Art. 246. Impedir o responsável ou funcionário de entidade representações ou espetáculos, sem indicar os limites de idade
de atendimento o exercício dos direitos constantes nos incisos a que não se recomendem:
II, III, VII, VIII e XI do art. 124 desta Lei: Pena - multa de três a vinte salários de referência,
Pena - multa de três a vinte salários de referência, duplicada em caso de reincidência, aplicável, separadamente,
aplicando-se o dobro em caso de reincidência. à casa de espetáculo e aos órgãos de divulgação ou
publicidade.
Art. 247. Divulgar, total ou parcialmente, sem autorização
devida, por qualquer meio de comunicação, nome, ato ou Art. 254. Transmitir, através de rádio ou televisão,
documento de procedimento policial, administrativo ou espetáculo em horário diverso do autorizado ou sem aviso de
judicial relativo a criança ou adolescente a que se atribua ato sua classificação:
infracional: Pena - multa de vinte a cem salários de referência;
Pena - multa de três a vinte salários de referência, duplicada em caso de reincidência a autoridade judiciária
aplicando-se o dobro em caso de reincidência. poderá determinar a suspensão da programação da emissora
§ 1º Incorre na mesma pena quem exibe, total ou por até dois dias.
parcialmente, fotografia de criança ou adolescente envolvido
em ato infracional, ou qualquer ilustração que lhe diga Art. 255. Exibir filme, trailer, peça, amostra ou congênere
respeito ou se refira a atos que lhe sejam atribuídos, de forma classificado pelo órgão competente como inadequado às
a permitir sua identificação, direta ou indiretamente. crianças ou adolescentes admitidos ao espetáculo:
§ 2º Se o fato for praticado por órgão de imprensa ou Pena - multa de vinte a cem salários de referência; na
emissora de rádio ou televisão, além da pena prevista neste reincidência, a autoridade poderá determinar a suspensão do
artigo, a autoridade judiciária poderá determinar a apreensão espetáculo ou o fechamento do estabelecimento por até quinze
da publicação ou a suspensão da programação da emissora até dias.

Legislação 37
Apostila Digital Licenciada para Alice Caroline Guarino dos Santos - alice.guarino@hotmail.com (Proibida a Revenda)
APOSTILAS OPÇÃO

Art. 256. Vender ou locar a criança ou adolescente fita de I - 1% (um por cento) do imposto sobre a renda devido
programação em vídeo, em desacordo com a classificação apurado pelas pessoas jurídicas tributadas com base no lucro
atribuída pelo órgão competente: real; e
Pena - multa de três a vinte salários de referência; em caso II - 6% (seis por cento) do imposto sobre a renda apurado
de reincidência, a autoridade judiciária poderá determinar o pelas pessoas físicas na Declaração de Ajuste Anual, observado
fechamento do estabelecimento por até quinze dias. o disposto no art. 22 da Lei no 9.532, de 10 de dezembro de
1997.
Art. 257. Descumprir obrigação constante dos arts. 78 e 79 § 1º - Revogado
desta Lei: § 1º-A. Na definição das prioridades a serem atendidas
Pena - multa de três a vinte salários de referência, com os recursos captados pelos fundos nacional, estaduais e
duplicando-se a pena em caso de reincidência, sem prejuízo de municipais dos direitos da criança e do adolescente, serão
apreensão da revista ou publicação. consideradas as disposições do Plano Nacional de Promoção,
Proteção e Defesa do Direito de Crianças e Adolescentes à
Art. 258. Deixar o responsável pelo estabelecimento ou o Convivência Familiar e Comunitária e as do Plano Nacional
empresário de observar o que dispõe esta Lei sobre o acesso pela Primeira Infância. (Redação dada pela Lei nº 13.257, de
de criança ou adolescente aos locais de diversão, ou sobre sua 2016)
participação no espetáculo: § 2º Os conselhos nacional, estaduais e municipais dos
Pena - multa de três a vinte salários de referência; em caso direitos da criança e do adolescente fixarão critérios de
de reincidência, a autoridade judiciária poderá determinar o utilização, por meio de planos de aplicação, das dotações
fechamento do estabelecimento por até quinze dias. subsidiadas e demais receitas, aplicando necessariamente
percentual para incentivo ao acolhimento, sob a forma de
Art. 258-A. Deixar a autoridade competente de guarda, de crianças e adolescentes e para programas de
providenciar a instalação e operacionalização dos cadastros atenção integral à primeira infância em áreas de maior
previstos no art. 50 e no § 11 do art. 101 desta Lei: carência socioeconômica e em situações de calamidade.
Pena - multa de R$ 1.000,00 (mil reais) a R$ 3.000,00 (três (Redação dada pela Lei nº 13.257, de 2016)
mil reais). § 3º O Departamento da Receita Federal, do Ministério da
Parágrafo único. Incorre nas mesmas penas a autoridade Economia, Fazenda e Planejamento, regulamentará a
que deixa de efetuar o cadastramento de crianças e de comprovação das doações feitas aos fundos, nos termos deste
adolescentes em condições de serem adotadas, de pessoas ou artigo.
casais habilitados à adoção e de crianças e adolescentes em § 4º O Ministério Público determinará em cada comarca a
regime de acolhimento institucional ou familiar. forma de fiscalização da aplicação, pelo Fundo Municipal dos
Direitos da Criança e do Adolescente, dos incentivos fiscais
Art. 258-B. Deixar o médico, enfermeiro ou dirigente de referidos neste artigo.
estabelecimento de atenção à saúde de gestante de efetuar § 5º Observado o disposto no § 4º do art. 3º da Lei no 9.249,
imediato encaminhamento à autoridade judiciária de caso de de 26 de dezembro de 1995, a dedução de que trata o inciso I
que tenha conhecimento de mãe ou gestante interessada em do caput:
entregar seu filho para adoção: I - será considerada isoladamente, não se submetendo a
Pena - multa de R$ 1.000,00 (mil reais) a R$ 3.000,00 (três limite em conjunto com outras deduções do imposto; e
mil reais). II - não poderá ser computada como despesa operacional
Parágrafo único. Incorre na mesma pena o funcionário de na apuração do lucro real.
programa oficial ou comunitário destinado à garantia do
direito à convivência familiar que deixa de efetuar a Art. 260-A. A partir do exercício de 2010, ano-calendário
comunicação referida no caput deste artigo. de 2009, a pessoa física poderá optar pela doação de que trata
o inciso II do caput do art. 260 diretamente em sua Declaração
Art. 258-C. Descumprir a proibição estabelecida no inciso de Ajuste Anual.
II do art. 81: (Redação dada pela Lei nº 13.106, de 2015) § 1º A doação de que trata o caput poderá ser deduzida até
Pena - multa de R$ 3.000,00 (três mil reais) a R$ 10.000,00 os seguintes percentuais aplicados sobre o imposto apurado
(dez mil reais); Redação dada pela Lei nº 13.106, de 2015) na declaração:
Medida Administrativa - interdição do estabelecimento I - (VETADO);
comercial até o recolhimento da multa aplicada. (Redação II - (VETADO);
dada pela Lei nº 13.106, de 2015) III - 3% (três por cento) a partir do exercício de 2012.
§ 2º A dedução de que trata o caput:
Disposições Finais e Transitórias I - está sujeita ao limite de 6% (seis por cento) do imposto
sobre a renda apurado na declaração de que trata o inciso II do
Art. 259. A União, no prazo de noventa dias contados da caput do art. 260;
publicação deste Estatuto, elaborará projeto de lei dispondo II - não se aplica à pessoa física que:
sobre a criação ou adaptação de seus órgãos às diretrizes da a) utilizar o desconto simplificado;
política de atendimento fixadas no art. 88 e ao que estabelece b) apresentar declaração em formulário; ou
o Título V do Livro II. c) entregar a declaração fora do prazo;
Parágrafo único. Compete aos estados e municípios III - só se aplica às doações em espécie; e
promoverem a adaptação de seus órgãos e programas às IV - não exclui ou reduz outros benefícios ou deduções em
diretrizes e princípios estabelecidos nesta Lei. vigor.
§ 3º O pagamento da doação deve ser efetuado até a data
Art. 260. Os contribuintes poderão efetuar doações aos de vencimento da primeira quota ou quota única do imposto,
Fundos dos Direitos da Criança e do Adolescente nacional, observadas instruções específicas da Secretaria da Receita
distrital, estaduais ou municipais, devidamente comprovadas, Federal do Brasil.
sendo essas integralmente deduzidas do imposto de renda, § 4º O não pagamento da doação no prazo estabelecido no
obedecidos os seguintes limites: § 3o implica a glosa definitiva desta parcela de dedução,
ficando a pessoa física obrigada ao recolhimento da diferença

Legislação 38
Apostila Digital Licenciada para Alice Caroline Guarino dos Santos - alice.guarino@hotmail.com (Proibida a Revenda)
APOSTILAS OPÇÃO

de imposto devido apurado na Declaração de Ajuste Anual com I - manter conta bancária específica destinada
os acréscimos legais previstos na legislação. exclusivamente a gerir os recursos do Fundo;
§ 5º A pessoa física poderá deduzir do imposto apurado na II - manter controle das doações recebidas; e
Declaração de Ajuste Anual as doações feitas, no respectivo III - informar anualmente à Secretaria da Receita Federal
ano-calendário, aos fundos controlados pelos Conselhos dos do Brasil as doações recebidas mês a mês, identificando os
Direitos da Criança e do Adolescente municipais, distrital, seguintes dados por doador:
estaduais e nacional concomitantemente com a opção de que a) nome, CNPJ ou CPF;
trata o caput, respeitado o limite previsto no inciso II do art. b) valor doado, especificando se a doação foi em espécie ou
260. em bens.

Art. 260-B. A doação de que trata o inciso I do art. 260 Art. 260-H. Em caso de descumprimento das obrigações
poderá ser deduzida: previstas no art. 260-G, a Secretaria da Receita Federal do
I - do imposto devido no trimestre, para as pessoas Brasil dará conhecimento do fato ao Ministério Público.
jurídicas que apuram o imposto trimestralmente; e
II - do imposto devido mensalmente e no ajuste anual, para Art. 260-I. Os Conselhos dos Direitos da Criança e do
as pessoas jurídicas que apuram o imposto anualmente. Adolescente nacional, estaduais, distrital e municipais
Parágrafo único. A doação deverá ser efetuada dentro do divulgarão amplamente à comunidade:
período a que se refere a apuração do imposto. I - o calendário de suas reuniões;
II - as ações prioritárias para aplicação das políticas de
Art. 260-C. As doações de que trata o art. 260 desta Lei atendimento à criança e ao adolescente;
podem ser efetuadas em espécie ou em bens. III - os requisitos para a apresentação de projetos a serem
Parágrafo único. As doações efetuadas em espécie devem beneficiados com recursos dos Fundos dos Direitos da Criança
ser depositadas em conta específica, em instituição financeira e do Adolescente nacional, estaduais, distrital ou municipais;
pública, vinculadas aos respectivos fundos de que trata o art. IV - a relação dos projetos aprovados em cada ano-
260. calendário e o valor dos recursos previstos para
implementação das ações, por projeto;
Art. 260-D. Os órgãos responsáveis pela administração das V - o total dos recursos recebidos e a respectiva destinação,
contas dos Fundos dos Direitos da Criança e do Adolescente por projeto atendido, inclusive com cadastramento na base de
nacional, estaduais, distrital e municipais devem emitir recibo dados do Sistema de Informações sobre a Infância e a
em favor do doador, assinado por pessoa competente e pelo Adolescência; e
presidente do Conselho correspondente, especificando: VI - a avaliação dos resultados dos projetos beneficiados
I - número de ordem; com recursos dos Fundos dos Direitos da Criança e do
II - nome, Cadastro Nacional da Pessoa Jurídica (CNPJ) e Adolescente nacional, estaduais, distrital e municipais.
endereço do emitente;
III - nome, CNPJ ou Cadastro de Pessoas Físicas (CPF) do Art. 260-J. O Ministério Público determinará, em cada
doador; Comarca, a forma de fiscalização da aplicação dos incentivos
IV - data da doação e valor efetivamente recebido; e fiscais referidos no art. 260 desta Lei.
V - ano-calendário a que se refere a doação. Parágrafo único. O descumprimento do disposto nos arts.
§ 1º O comprovante de que trata o caput deste artigo pode 260-G e 260-I sujeitará os infratores a responder por ação
ser emitido anualmente, desde que discrimine os valores judicial proposta pelo Ministério Público, que poderá atuar de
doados mês a mês. ofício, a requerimento ou representação de qualquer cidadão.
§ 2º No caso de doação em bens, o comprovante deve Art. 260-K. A Secretaria de Direitos Humanos da
conter a identificação dos bens, mediante descrição em campo Presidência da República (SDH/PR) encaminhará à Secretaria
próprio ou em relação anexa ao comprovante, informando da Receita Federal do Brasil, até 31 de outubro de cada ano,
também se houve avaliação, o nome, CPF ou CNPJ e endereço arquivo eletrônico contendo a relação atualizada dos Fundos
dos avaliadores. dos Direitos da Criança e do Adolescente nacional, distrital,
estaduais e municipais, com a indicação dos respectivos
Art. 260-E. Na hipótese da doação em bens, o doador números de inscrição no CNPJ e das contas bancárias
deverá: específicas mantidas em instituições financeiras públicas,
I - comprovar a propriedade dos bens, mediante destinadas exclusivamente a gerir os recursos dos Fundos.
documentação hábil;
II - baixar os bens doados na declaração de bens e direitos, Art. 260-L. A Secretaria da Receita Federal do Brasil
quando se tratar de pessoa física, e na escrituração, no caso de expedirá as instruções necessárias à aplicação do disposto nos
pessoa jurídica; e arts. 260 a 260-K.
III - considerar como valor dos bens doados:
a) para as pessoas físicas, o valor constante da última Art. 261. A falta dos conselhos municipais dos direitos da
declaração do imposto de renda, desde que não exceda o valor criança e do adolescente, os registros, inscrições e alterações a
de mercado; que se referem os arts. 90, parágrafo único, e 91 desta Lei serão
b) para as pessoas jurídicas, o valor contábil dos bens. efetuados perante a autoridade judiciária da comarca a que
Parágrafo único. O preço obtido em caso de leilão não será pertencer a entidade.
considerado na determinação do valor dos bens doados, Parágrafo único. A União fica autorizada a repassar aos
exceto se o leilão for determinado por autoridade judiciária. estados e municípios, e os estados aos municípios, os recursos
referentes aos programas e atividades previstos nesta Lei, tão
Art. 260-F. Os documentos a que se referem os arts. 260-D logo estejam criados os conselhos dos direitos da criança e do
e 260-E devem ser mantidos pelo contribuinte por um prazo adolescente nos seus respectivos níveis.
de 5 (cinco) anos para fins de comprovação da dedução
perante a Receita Federal do Brasil. Art. 262. Enquanto não instalados os Conselhos Tutelares,
Art. 260-G. Os órgãos responsáveis pela administração das as atribuições a eles conferidas serão exercidas pela
contas dos Fundos dos Direitos da Criança e do Adolescente autoridade judiciária.
nacional, estaduais, distrital e municipais devem:

Legislação 39
Apostila Digital Licenciada para Alice Caroline Guarino dos Santos - alice.guarino@hotmail.com (Proibida a Revenda)
APOSTILAS OPÇÃO

Art. 263. O Decreto-Lei n.º 2.848, de 7 de dezembro de 02. O trabalho educativo visa, primordialmente, a partir da
1940 (Código Penal), passa a vigorar com as seguintes edição do Estatuto da Criança e do Adolescente,
alterações: (A) o desenvolvimento pessoal e humano dos
1) Art. 121 ............................................................ adolescentes, acoplado à sua profissionalização.
§ 4º No homicídio culposo, a pena é aumentada de um (B) a capacitação do adolescente, não gerando
terço, se o crime resulta de inobservância de regra técnica de remuneração a quem o executa, diante de sua natureza
profissão, arte ou ofício, ou se o agente deixa de prestar específica.
imediato socorro à vítima, não procura diminuir as (C) a conscientização do adolescente para a importância no
consequências do seu ato, ou foge para evitar prisão em desenvolvimento de qualquer atividade, diante do efeito
flagrante. Sendo doloso o homicídio, a pena é aumentada de dignificante do trabalho.
um terço, se o crime é praticado contra pessoa menor de (D) a possibilidade de iniciação profissional, via
catorze anos. aprendizagem, a partir dos 16 anos de idade
2) Art. 129 ............................................................... (E) a garantia da cidadania ao adolescente, haja vista que o
§ 7º Aumenta-se a pena de um terço, se ocorrer qualquer trabalho é dever social
das hipóteses do art. 121, § 4º.
§ 8º Aplica-se à lesão culposa o disposto no § 5º do art. 121. 03. No exercício da fiscalização e controle das entidades de
3) Art. 136................................................................. atendimento,
§ 3º Aumenta-se a pena de um terço, se o crime é praticado (A) a irregularidade a ser verificada sempre será referente
contra pessoa menor de catorze anos. ao programa de atendimento, e não à entidade não
4) Art. 213 .................................................................. governamental.
Parágrafo único. Se a ofendida é menor de catorze anos: (B) o procedimento de apuração de irregularidades na
Pena - reclusão de quatro a dez anos. entidade de atendimento, segundo o Estatuto da Criança e do
5) Art. 214................................................................... Adolescente, pode ser instaurado mediante representação do
Parágrafo único. Se o ofendido é menor de catorze anos: Conselho Tutelar, do Ministério Público ou por portaria da
Pena - reclusão de três a nove anos própria autoridade judiciária que julgará o feito.
(C) é possível, em caráter liminar, o pedido e concessão do
Art. 264. O art. 102 da Lei n.º 6.015, de 31 de dezembro de afastamento do dirigente, desde que de entidade não
1973, fica acrescido do seguinte item: governamental.
"Art. 102 .................................................................... (D) o procedimento específico de apuração de
6º) a perda e a suspensão do pátrio poder. " irregularidades busca, ao final, a cessação do programa de
atendimento, não sendo alternativa para que a entidade faça
Art. 265. A Imprensa Nacional e demais gráficas da União, ajustes em seu programa, removendo as irregularidades, o
da administração direta ou indireta, inclusive fundações que, neste caso, seria próprio do mero expediente de apuração
instituídas e mantidas pelo poder público federal promoverão de denúncias de irregularidades.
edição popular do texto integral deste Estatuto, que será posto (E) a Justiça da Infância e Juventude deve apurar se as
à disposição das escolas e das entidades de atendimento e de entidades governamentais ou não governamentais,
defesa dos direitos da criança e do adolescente. irregularmente, não possuem registro deferido pelo Conselho
de Direitos.
Art. 265-A. O poder público fará periodicamente ampla
divulgação dos direitos da criança e do adolescente nos meios 04. Considera-se ato infracional
de comunicação social. (Redação dada pela Lei nº 13.257, de (A) a sentença que aplica medida socioeducativa ao
2016) adolescente.
Parágrafo único. A divulgação a que se refere o caput será (B) a extinção da medida socioeducativa pela realização de
veiculada em linguagem clara, compreensível e adequada a sua finalidade.
crianças e adolescentes, especialmente às crianças com idade (C) a medida aplicável aos pais ou responsável pelo
inferior a 6 (seis) anos. (Incluído dada pela Lei nº 13.257, de descumprimento de seus deveres acerca da criança e do
2016) adolescente, em relação aos quais exerçam seu poder.
(D) o descumprimento de medida socioeducativa aplicada
Art. 266. Esta Lei entra em vigor noventa dias após sua ao adolescente.
publicação. (E) a conduta praticada por adolescente, descrita como
Parágrafo único. Durante o período de vacância deverão crime ou contravenção penal.
ser promovidas atividades e campanhas de divulgação e
esclarecimentos acerca do disposto nesta Lei. 05. De acordo com a Lei 8.069/90 (Estatuto da Criança e
do Adolescente), assinale a alternativa correta a respeito da
Art. 267. Revogam-se as Leis n.º 4.513, de 1964, e 6.697, de adoção:
10 de outubro de 1979 (Código de Menores), e as demais (A) A adoção atribui condição de filho ao adotado, com os
disposições em contrário. mesmos direitos e deveres, inclusive sucessórios, desligando-
o de todo e qualquer vínculo e impedimento com os pais e
Questões parentes
(B) O adotando deve contar com no máximo 16 anos à data
01. De acordo com o Estatuto da Criança e do Adolescente, do pedido, salvo se já estiver sob a guarda ou tutela dos
considera-se tratamento cruel ou degradante dispensado à adotantes.
criança aquele que a (C) É vedada a adoção por procuração.
(A) submete ao aleitamento materno no interior de (D) A adoção não poderá ser deferida ao adotante que,
presídio onde a mãe cumpre pena. após inequívoca manifestação de vontade, vier a falecer no
(B) submeta a tratamento a toxicômanos. curso do procedimento, antes de prolatada a sentença.
(C) proporcione castigo e sofrimento físico desnecessário. (E) Podem adotar os ascendentes e os irmãos do adotando
(D) humilhe, ameace gravemente ou a ridicularize.
(E) prive da frequência ao ensino fundamental.

Legislação 40
Apostila Digital Licenciada para Alice Caroline Guarino dos Santos - alice.guarino@hotmail.com (Proibida a Revenda)
APOSTILAS OPÇÃO

06. Amanda, 14 anos, foi entregue diretamente pela família (C) descumprir, dolosa ou culposamente, os deveres
biológica para adoção, tendo sido legalmente adotada pela Sra. inerentes ao poder familiar ou decorrentes de tutela ou
Angélica quando tinha 2 anos de idade. Como a relação entre guarda.
mãe e filha vem se deteriorando, a Sra. Angélica buscou a (D) hospedar crianças ou adolescentes desacompanhados
família biológica da adolescente, visando desistir da adoção. dos pais ou dos responsáveis, ou sem autorização escrita
Segundo estabelecido no Estatuto da Criança e do desses ou da autoridade judiciária, em hotel, pensão, motel ou
Adolescente, congênere.
(A) a devolução é possível, sendo a adoção revogável na (E) exibir filmes, trailers, peças, amostras ou congêneres
hipótese de avaliação de fragilidade do vínculo emocional classificados pelo órgão competente como inadequados a
entre adotante e adotando pela equipe técnica do Juízo e o crianças ou adolescentes admitidos no espetáculo.
melhor interesse da adotando.
(B) a devolução é possível, uma vez que comprovado o Respostas
risco à estabilidade emocional da adotante por psicoterapeuta
que atenda à família 01. D. / 02. A. / 03. B / 04. E / 05. C
(C) a devolução é possível, desde que identificado risco à 06. D / 07. A. / 08. A. / 09. C. / 10. B
saúde emocional da adotanda por psicólogo que acompanhe
terapeuticamente a menina.
(D) a devolução não é possível, pois a adoção é irrevogável CÓDIGO CIVIL - Lei
e atribui a condição de filho ao adotando, desligando-o de
quaisquer vínculo com pais e parentes. 10.406/2002 Arts: 1511 a
(E) a devolução não é possível, pois a adotanda já atingiu a 1638; 1694 a 1727 e 1728 a
adolescência, só sendo possível durante o período infantil, 1783.
quando as chances de outra colocação em família substituta
são maiores.

07. Para o Estatuto da Criança e Adolescente considera-se Código Civil – arts. 1.511 a 1.638
criança a pessoa até:
(A) Doze anos de idade incompletos. Esses artigos do Código Civil tratam sobre o Direito de
(B) Quatorze anos de idade incompletos. Família, o casamento, o poder familiar, enfim acerca das
(C) Oito anos de idade incompletos. relações familiares.
(D) Dez anos de idade incompletos.
1- Do Casamento.
08. Dispõe o Estatuto da Criança e do Adolescente acerca O casamento tem natureza jurídica negocial, já que é ato
de viagem de criança ou adolescente para o exterior: que decorre da vontade das partes, fundado, basicamente, no
(A) o detentor da guarda poderá opor-se a saída de criança consentimento. Trata-se de um negócio jurídico complexo,
ou adolescente na companhia de seus pais. pois se perfaz com o consentimento dos interessados.
(B) a autorização para viagem internacional concedida por De acordo com o STJ, o casamento é espécie de contrato
ambos os pais é exigível para criança acompanhada por sui generis.
terceiros e dispensável em caso de viagem de adolescente
sozinho ao exterior. Características
(C) a saída de criança ou de adolescente nascido no Brasil, a) Caráter personalíssimo;
na companhia de estrangeiro residente ou domiciliado no b) Celebração solene;
exterior, requer declaração de ambos os pais com firma c) Diversidade de sexos.
reconhecida. Atenção! Atualmente o Brasil admite o casamento entre
(D) requer-se autorização judicial para criança ou pessoas do mesmo sexo.
adolescente viajar ao exterior apenas quando for realizada na d) Inadmissibilidade de submissão a termo ou condição: o
companhia do responsável. casamento deve necessariamente ser incondicionado;
e) Estabelecimento de uma comunhão de vida;
09. Considerando o Estatuto da Criança e do Adolescente, f) Natureza jurídica das normas que o regulamentam;
assinale a alternativa correta. g) Estrutura monogâmica;
(A) Os filhos, havidos ou não da relação do casamento, ou h) Dissolubilidade.
por adoção, não terão os mesmos direitos e qualificações.
(B) O poder familiar será exercido apenas pelo pai, na Prova do Casamento
forma do que dispuser a legislação civil. O casamento deve ser provado por certidão do registro
(C) A falta ou a carência de recursos materiais não constitui cível expedida pelo cartório competente, na forma do art.
motivo suficiente para a perda ou a suspensão do poder 1.536. Trata-se, aqui, da consagração de provas pré-
familiar. constituídas e tarifadas, que advém do próprio caráter formal
(D) A guarda confere à criança ou adolescente a condição e solene do negócio.
de dependente apenas para fins de imposto de renda. Porém, o p. único do art. 1.543 reconhece a possibilidade
de prová-lo por outros meios.
10. É crime previsto no ECA. A prova indireta ou supletiva somente será utilizada
(A) deixar o médico de comunicar à autoridade quando o próprio assento, em cartório, vier a perecer.
competente os casos de seu conhecimento que envolvam Para tanto, deverá ser promovido o procedimento
suspeita de maus-tratos contra criança ou adolescente. voluntário de justificação judicial de casamento. Nesse
(B) deixar a autoridade policial responsável pela procedimento, de acordo com o art. 1.546, haverá efeitos retro
apreensão de criança ou adolescente de fazer imediata operantes no que toca aos cônjuges e aos filhos, desde a data
comunicação à autoridade judiciária competente e à família do em que se provou realizado o casamento.
apreendido ou à pessoa por ele indicada. De acordo com o art. 1.547, havendo dúvida entre as
provas favoráveis e as contrárias à existência do casamento,
deve-se admiti-lo como existente (in dubio pro casamento).

Legislação 41
Apostila Digital Licenciada para Alice Caroline Guarino dos Santos - alice.guarino@hotmail.com (Proibida a Revenda)
APOSTILAS OPÇÃO

Quando o casamento é realizado no exterior (entre Oposição das Causas Suspensivas


brasileiros) perante autoridades estrangeiras ou consulares, Elas não podem ser opostas pelo oficial do CRCPF, nem
deverá ser registrado no prazo de 180 dias, a contar do retorno reconhecida de ofício pelo juiz. Somente podem ser arguidas
definitivo de um ou de ambos os consortes ao Brasil. Tal pelos parentes em linha reta de um dos nubentes, sejam
registro é meramente declaratório, e não constitutivo da consanguíneos ou afins, e pelos colaterais até o segundo grau,
sociedade conjugal, servindo para dar publicidade e provar o também consanguíneos ou afins.
casamento. Nem mesmo o MP poderá provocar o reconhecimento das
causas suspensivas.
Esponsais
É equivalente ao noivado. a) O viúvo ou a viúva que tiver filho do cônjuge falecido,
É o ato pelo qual as partes interessadas prometem, enquanto não fizer inventário dos bens do casal e der partilha
recíproca e livremente, casar, assumindo obrigações aos herdeiros;
recíprocas, como o pagamento de despesas com habilitação, b) O divorciado, enquanto não houver sido homologada ou
enxoval etc. decidida a partilha dos bens do casal;
A simples desistência do matrimônio é apenas c) A viúva ou a mulher cujo casamento se desfez por ser
manifestação da liberdade, garantia individual constitucional, nulo ou ter sido anulado, até 10 meses depois do começo da
razão pela qual não se pode falar em obrigação de reparar os viuvez ou da dissolução da sociedade conjugal;
danos. d) O tutor ou curador e os seus descendentes, ascendentes,
Somente poderá se falar em indenização quando a ruptura irmãos, cunhados ou sobrinhos, com a pessoa tutelada ou
da promessa estiver caracterizada como um ato ilícito, curatelada, enquanto não cessar a tutela ou curatela, e não
descabendo indenização por danos morais se não houver estiverem saldadas as respectivas contas.
violência física ou moral ou ofensa contra a honra e a
dignidade da pessoa. Habilitação para o Casamento
Trata-se do procedimento administrativo, de iniciativa dos
O legislador brasileiro reconheceu que tanto o casamento nubentes, que tramita perante o oficial do Cartório do
quanto seus efeitos são civis (art. 1.512), não havendo validade domicílio de qualquer deles, com o propósito de demonstrar a
e eficácia jurídica no casamento eclesiástico. Porém, em capacidade para casar e a inexistência de impedimentos
relação à cerimônia, esta poderá ser civil ou religiosa. matrimoniais e de causas suspensivas.
O p. único do art. 1.512, CC/02, possibilitou a gratuidade
da habilitação para o casamento, do registro e da primeira Fases do Procedimento de Habilitação
certidão para as pessoas que se declararem pobres, na forma a) Fase de requerimento e apresentação da documentação
da Lei nº 1.060/50. (art. 1.525);
b) Fase dos editais de proclamas (art. 1.527): dura 15 dias
Impedimentos Matrimoniais a fixação do edital;
Impedimentos são proibições decorrentes da lei de que c) Fase de registro: trata-se do registro de habilitação para
determinadas pessoas possam contrair casamento. É a o casamento oficial, que expedirá a certidão habilitatória. A
proibição de casar dirigida a uma pessoa em relação a outra certidão de habilitação tem eficácia por 90 dias (art. 1.532).
predeterminada.
Os impedimentos estão taxativamente previstos no CC/02, Existência, Validade e Eficácia do Casamento
não comportando interpretação ampliativa. Por se tratarem de São os seguintes os pressupostos existenciais do
questão de ordem pública, podem ser alegados por qualquer casamento:
pessoa a qualquer momento, implicando nulidade do
casamento. a) Diversidade de sexos;
Os impedimentos são aplicáveis à união estável, pois (*) Nota: a menção aos sexos distintos foi retirada do
somente pode ser reputada como união estável a convivência conceito de casamento. Atualmente o Brasil admite o
que puder ser convertida em casamento, de acordo com a casamento entre pessoas de mesmo sexo. Veja o enunciado
vontade das partes. 601 da VII Jornada de Direito Civil:

Oposição dos Impedimentos “É existente e válido o casamento entre pessoas do mesmo


As causas impeditivas do casamento devem ser opostas até sexo”.
o momento da celebração, por qualquer pessoa capaz. Por se
tratar de matérias de ordem pública, o juiz quando tenha b) Existência de consentimento expresso;
conhecimento poderá reconhece-las de ofício. Se porventura, c) Celebração do matrimônio com a presença da
o casamento chegar a ser celebrado ele será nulo de pleno autoridade (a falta da autoridade implica em inexistência do
direito (nulidade absoluta). casamento, enquanto a sua incompetência implica em
anulabilidade).
Causas Suspensivas
Trata-se de recomendação legal para que o casamento não Validade do Casamento
ocorra. Por não ser regra de interesse público, não impede que Como qualquer negócio jurídico, o casamento será válido
o casamento ocorra e nem é causa de invalidade. desde que atenda aos requisitos legais e que não contrarie o
A única consequência da inobservância de causa ordenamento jurídico.
suspensiva é a imposição do regime de separação obrigatória
de bens, afastando a vontade das partes. Assim, diz-se que são São hipóteses de nulidade do casamento (art. 1.548):
causas que geram a necessária proteção do patrimônio da
pessoa. Art. 1.548. É nulo o casamento contraído:
As causas suspensivas não são aplicáveis às uniões estáveis I - pelo enfermo mental sem o necessário discernimento para
(art. 1.723, § 2º). os atos da vida civil;
I - (Revogado); (Redação dada pela Lei nº 13.146, de 2015).
II - por infringência de impedimento.

Legislação 42
Apostila Digital Licenciada para Alice Caroline Guarino dos Santos - alice.guarino@hotmail.com (Proibida a Revenda)
APOSTILAS OPÇÃO

Já as hipóteses de anulabilidade apresentam as seguintes Conversão da União Estável em Casamento


características: A conversão da união estável em casamento será feita
a) Não invalidam quaisquer efeitos do casamento até que através de pedido dirigido ao juiz e assento no Registro Civil,
sobrevenha sua invalidação; obrigando a se seguir o procedimento comum de habilitação,
b) Somente poderá propor a ação anulatória a parte para que se apure eventual existência de impedimento
juridicamente interessada; matrimoniais. É imprescindível a intervenção do MP nesse
c) Admitem retificação; procedimento.
d) A ação anulatória se submete a prazos decadenciais
previstos em lei; 1.1- Regime de Bens do Casamento.
e) O MP poderá suscitá-las;
f) O juiz não pode conhecê-las de ofício. O casamento estabelece comunhão plena de vida. A
plenitude, evidentemente, inclui os aspectos materiais, motivo
São hipóteses de anulabilidade (art. 1.550), de acordo com pelo qual a Lei Civil regula o estatuto patrimonial do
o prazo decadencial de propositura: casamento, caracterizado pelo regime de bens. Porém, essas
consequências patrimoniais têm de estar alinhadas à proteção
Prazo de 180 dias da dignidade humana e de seus valores essenciais.
Não ter idade mínima para casar
Menor em idade núbil, quando não autorizado pelo 1.1.1- Regime de Bens na União Estável.
representante legal O regime de bens da união estável, de acordo com o art.
Do incapaz de consentir ou manifestar inequivocamente 1.725 do CC, inexistindo regulamentação particular em
seu consentimento contrário, é regulado pelo regime supletivo, qual seja,
Realizado pelo mandatário, sem que ele ou o outro comunhão parcial de bens.
contraente soubessem da revogação do mandato Os companheiros poderão eleger livremente o regime de
bens a que pretendem submeter-se, por meio de negócio
Prazo de 02 anos Incompetência da autoridade jurídico escrito, independentemente de instrumento público,
celebrante dispensado também o registro no Cartório de Imóveis.
Comunicam-se os bens adquiridos a título oneroso na
Prazo de 03 anos Quando houver erro essencial quanto constância do casamento, presumindo-se o esforço comum de
ao outro consorte: em relação à identidade, honra, boa fama, maneira absoluta. Já os bens adquiridos gratuitamente ou cuja
tornando insuportável a vida em comum do cônjuge enganado, causa aquisitiva seja anterior ao início da união estável não
ignorância, anterior ao casamento, de doença mental, defeito entram na comunhão, pois ausente o elemento da colaboração
físico irremediável ou moléstia grave e transmissível, além da recíproca.
prática de crime. A mudança de regime de bens na união estável pode ser
operada por simples contrato escrito, por instrumento público
Prazo de 04 anos Mediante coação: ameaça à vida, saúde ou particular, independentemente de publicação de editais,
e honra, sua ou de seus familiares intervenção do MP e homologação do juiz.
A união estável é uma união de fato, sem a necessidade de
- Casamento Putativo. registros públicos, não sendo possível vincular terceiros,
Trata-se do casamento inválido (nulo ou anulável) que foi motivo pelo qual a outorga não pode ser exigida, em nome da
contraído de boa fé subjetiva, por um ou ambos os consortes, proteção do adquirente de boa- fé. Didier defende, entretanto,
incorrendo em erro de fato ou de direito, permitindo o que se o terceiro conhecer da condição em que vive o
aproveitamento dos efeitos jurídicos decorrentes do contratante, será imprescindível a outorga (mesmo porque
matrimônio, que serão emprestados pelo juiz. restará descaracterizada a boa- fé).
Ou seja, não haverá convalidação do casamento inválido,
tão-somente o aproveito de sua eficácia para alguma 1.1.2- Regime de Comunhão Universal de Bens.
finalidade específica e somente para o cônjuge de boa fé. Neste regime, forma-se uma massa patrimonial única,
A putatividade, que deve ser reconhecida judicialmente, atingindo débitos e créditos, comunicando-se os bens
pode ser declarada na própria sentença que declara nulo o pretéritos e futuros, independentemente de terem sido
casamento ou que o desconstitui, se foi alegada pelas partes adquiridos antes ou depois das núpcias, a título gratuito ou
durante o processo. oneroso.
No entanto, a comunhão universal não é absoluta, havendo
- Celebração do Casamento. algumas exceções à comunicação estabelecidas no art. 1.668:
O casamento é ato solene; logo, está cercado de rigorismos
formais, exigindo uma celebração cheia de requisitos. Uma vez a) Os bens doados ou herdados com a cláusula de
concluído o procedimento de habilitação para o casamento, os incomunicabilidade e os sub-rogados em seu lugar;
nubentes, de posse da certidão e dentro do prazo de 90 dias de b) Os bens gravados de fideicomisso e o direito do herdeiro
sua expedição, deverão peticionar à autoridade competente, fideicomissário, antes de realizada a condição suspensiva;
que presidirá o ato requerendo a designação de dia, hora e c) As dívidas anteriores ao casamento, salvo se provierem
local para a realização da cerimônia (art. 1.533). de despesas com seus aprestos, ou reverterem em proveito
comum;
O Casamento Consular d) As doações antenupciais feitas por um dos cônjuges ao
É o casamento entre brasileiros no estrangeiro perante outro com a cláusula de incomunicabilidade;
autoridade ou cônsules brasileiros, o qual deverá ser e) Os bens de uso pessoal, os livros e instrumentos de
registrado em 180 dias, a contar da volta definitiva de um ou profissão;
ambos ao Brasil, no Cartório do respectivo domicílio ou, na f) Os proventos do trabalho pessoal de cada cônjuge;
falta, no 1º Ofício da Capital do Estado em que passarem a g) As pensões, meios-soldos, montepios e outras rendas
residir. semelhantes.

Legislação 43
Apostila Digital Licenciada para Alice Caroline Guarino dos Santos - alice.guarino@hotmail.com (Proibida a Revenda)
APOSTILAS OPÇÃO

1.1.3- Separação Convencional ou Absoluta de Bens. transijam, ouvindo-as em separado e em conjunto. A melhor
Cada cônjuge manterá um patrimônio particular, doutrina defende que, apesar de ser determinação legal, a
inexistindo qualquer ponto de interseção de bens, sendo livres supressão da tentativa de reconciliação não causa nulidade
para dispor, onerar, prestar fiança e aval, contrair dívidas e absoluta do feito. Existe corrente em sentido contrário, por
administrar seus bens. A penhora determinada pelo juiz sobre entender que se trata de norma de ordem pública.
os bens do outro cônjuge será nula, se para garantir dívida e) Foro privilegiado da mulher: as ações deverão ser
alheia. ajuizadas no domicílio da mulher; entretanto, trata-se de regra
Porém, é obrigatório o rateio das despesas com a de competência relativa, não podendo ser conhecida de ofício
manutenção do lar, proporcionalmente aos ganhos de cada pelo juiz, requerida pelo MP e alegada pela parte após a
um, salvo disposição em contrário no pacto antenupcial. preclusão.
Somente haverá comunicação dos bens adquiridos f) Possibilidade de não divisão do patrimônio comum (art.
conjuntamente pelo casal; além disso, o CC/02 atribui a quem 1.581, manutenção dos bens em condomínio):
é casado nesse regime o direito à herança de seu cônjuge. g) Tratamento jurídico da revelia diferenciado: nessas
ações, a revelia não causa a presunção de veracidade dos fatos
1.1.4- Regime de Participação Final nos Aquestos. afirmado pelo autor; seu único efeito é a desnecessidade de
Neste regime, durante a convivência conjugal, o casamento intimação dos atos subsequentes do processo, o que não
fica submetido às regras da separação convencional de bens; impede o réu de nele intervir em qualquer fase.
porém, no instante da dissolução, incidirão as normas da
comunhão parcial, comunicando-se os bens adquiridos Separação
onerosamente por cada um durante a constância das núpcias.
Durante o casamento, os consortes têm a livre Trata-se de modo de extinção dos deveres do casamento
administração, podendo até mesmo ter a livre oneração ou que põe fim à obrigação de coabitação, de fidelidade recíproca
alienação de bens imóveis, se previsto no pacto antenupcial. e de respeito comum, além de pôr fim ao regime de bens.
Cada um deles terá direito à meação sobre os bens que o A separação não põe fim ao dever de mútua assistência
outro adquiriu a título oneroso, na constância da convivência; entre os cônjuges, motivo que justifica a prestação de
além, é claro, dos adquiridos conjuntamente (art. 1.679). alimentos.
Os esposos ficam associados nos ganhos e dissociados nas Como o vínculo jurídico matrimonial permanece, os
perdas. Os bens a serem apurados e calculados somente o cônjuges apenas separados não poderão contrair novo
serão após a dissolução, com uma reconstituição contábil dos casamento, apesar de poderem constituir união estável. A
aquestos. separação pode ocorrer sob duas formas:

1.2- Dissolução do Casamento. a) Separação consensual: por vontade das partes,


Dissolução do Casamento pela Morte perante o juiz ou mediante escritura pública (art. 1.574);
Com a morte, desaparecem os deveres recíprocos de
ordem pessoal e patrimonial e modifica-se o estado civil. b) Separação litigiosa: por iniciativa de apenas um dos
Porém, mesmo após o óbito, alguns efeitos decorrentes do cônjuges. Essa separação perdeu sentido com a vigência da
matrimônio permanecem. emenda constitucional nº 66/2010, já que foi dispensado
qualquer pré-requisito para o divórcio. A separação
a) O parentesco por afinidade em linha reta e o consensual, por sua vez, ainda possui utilidade, pois os
impedimento matrimonial dele decorrente; cônjuges podem resolver se separar, mas deixando aberta a
b) O direito de continuar utilizando o sobrenome de possibilidade de futuro restabelecimento da sociedade
casado, já que é um direito da personalidade; conjugal. Esse tema será tratado em tópico específico.
c) O cônjuge supérstite poderá titularizar direito
sucessório e direito real de habitação, de acordo com o regime 2- Relações de parentesco.
de bens. É o vínculo que existe entre as pessoas que descendem
umas das outras ou de um mesmo tronco conjugal e também a
Características Comuns à Separação e ao Divórcio relação entre o cônjuge ou companheiro e os parentes do
a) Natureza personalíssima: ambos são medidas jurídicas adotante e do adotado. Em poucas palavras, podemos
de natureza personalíssima, somente competindo aos considerar como oriundos de consaguinidade, afinidade e
próprios cônjuges promovê-los. Por isso que não se admite socioafetividade.
substituição processual. Quando um dos cônjuges for incapaz
ou quando houver filho menor de idade, não se admitirá o uso O parentesco é dividido em linhas: reta e colateral, e a
da via administrativa para a separação ou o divórcio, impondo- contagem faz-se por graus1.
se a utilização da via judicial (art. 1.582, p. único). Parentes em linha reta são as pessoas que descendem
É plenamente possível a utilização da separação ou umas das outras: bisavô, avô, pai, filho, neto e bisneto. A linha
divórcio consensuais, mesmo se um dos interessados for reta é ascendente quando se sobe de determinada pessoa para
incapaz, desde que esteja representado e que o acordo seja os seus antepassados (do pai para o avô etc.). É descendente
homologado judicialmente, com a intervenção do MP. quando se desce dessa pessoa para os seus descendentes (CC,
art. 1.591). Os parentes em linha colateral, transversal ou
b) Impossibilidade de intervenção de terceiros na ação: oblíqua as pessoas que provêm de um tronco comum, sem
c) Obrigatoriedade de intervenção do MP: esta descenderem uma da outra (CC, art. 1.592). É o caso de irmãos,
obrigatoriedade é mitigada, já que será desnecessária nas tios, sobrinhos e primos. Na linha reta não há limite de
dissoluções amigáveis, seja via judicial ou por escritura parentesco; na colateral, este estende-se somente até o quarto
pública. grau.
d) Obrigatoriedade de tentativa de reconciliação: de
acordo com o art. 3º, § 2º, da Lei nº 6.515/77, o juiz deverá
promover todos os meios para que as partes se reconciliem ou

1 Carlos Roberto Gonçalves. Sinopses Jurídicas. Direito Civil. Direito de

Família. Saraiva.

Legislação 44
Apostila Digital Licenciada para Alice Caroline Guarino dos Santos - alice.guarino@hotmail.com (Proibida a Revenda)
APOSTILAS OPÇÃO

2.1- Filiação. Já o reconhecimento judicial ocorre por meio da ação


Nos ensinamentos de Flávio Tartuce, é uma relação investigatória de paternidade ou de maternidade. Essa ação
jurídica entre pais e filhos regida pelo princípio da igualdade por ter natureza declaratória e envolver estado de pessoas e
entre os filhos. dignidade humana não se sujeita a qualquer prazo. O foro
competente para apreciação será o do foro do domicílio do réu.
O art. 1.597 traz hipóteses em que se presumirá a Em regra, a legitimidade ativa para propositura é do filho. No
paternidade decorrente do casamento: polo contrário da ação a legitimidade passiva será do suposto
pai ou da suposta mãe, sendo falecidos será proposta contra os
Art. 1.597. Presumem-se concebidos na constância do herdeiros. No caso de ausência de herdeiros deverá ser
casamento os filhos: proposta contra o Estado. Essa ação também pode ser
I - nascidos cento e oitenta dias, pelo menos, depois de proposta em face dos avós.
estabelecida a convivência conjugal;
II - nascidos nos trezentos dias subsequentes à dissolução da 2.3- Adoção.
sociedade conjugal, por morte, separação judicial, nulidade e É forma tradicional de parentesco civil.
anulação do casamento;
III - havidos por fecundação artificial homóloga, mesmo que Segundo Flávio Tartuce apud Maria Berenice Dias: “A
falecido o marido; adoção é um ato jurídico em sentido estrito, cuja eficácia está
IV - havidos, a qualquer tempo, quando se tratar de embriões condicionada à chancela judicial. Cria um vínculo fictício de
excedentários, decorrentes de concepção artificial homóloga; paternidade-maternidade-filiação entre pessoas estranhas,
V - havidos por inseminação artificial heteróloga, desde que análogo ao que resulta da filiação biológica”.2
tenha prévia autorização do marido.
A adoção está disciplinada na Lei 12.010/09 e no Estatuto
Exceção à regra do art. 1.597 é o art. 1.599 que dispõe que da Criança do Adolescente.
sendo o marido impotente a presunção de paternidade está
afastada. É considerada medida excepcional, a ser aplicada apenas
quando esgotados todos os meios para manutenção da criança
Questão que vem ganhando força acerca da filiação é sobre ou adolescente na família natural ou extensa.
a socioafetividade e multiparentalidade. Mas, o que vem a ser
socioafetividade e multiparentalidade? Filiação socioafetiva é Somente pessoa maior de 18 anos tem capacidade para
aquela que foge aos laços consanguíneos, a relação pai e filho adotar. A adoção pode ser unilateral (única pessoa adota) ou
é construída por meio da afetividade do convívio. Já a bilateral (duas pessoas adotam). Na adoção bilateral ou
multiparentalidade é a possibilidade de se fazer constar no conjunta, os adotantes devem ser casados civilmente ou
registro concomitante o nome de dois pais ou duas mães. mantenham união estável.
O STF tem decidido que havendo divergência entre a
parentalidade socioafetiva e a biológica, uma não excluirá a É possível que pessoas divorciadas ou ex-companheiros
outra, portanto, a regra será a multiparentalidade. adotem conjuntamente, entretanto, é preciso que estejam de
acordo com relação a guarda, regime de visitas e que o estágio
2.2- Reconhecimento dos filhos. de convivência tenha se iniciado na constância do casamento
O reconhecimento dos filhos pode se dar em duas formas: ou da união estável.
a- voluntária: situações do art. 1.609 do CC.
b- judicial: situações que estão afora do art. 1.609 do CC. Veja os dispositivos acerca do assunto:
SUBTÍTULO II
O reconhecimento voluntário dos filhos pode preceder o Das Relações de Parentesco
nascimento ou ser posterior ao falecimento. CAPÍTULO I
Veja as hipóteses que o art. 1.609 do CC traz acerca do Disposições Gerais
reconhecimento voluntário dos filhos:
Art. 1.591. São parentes em linha reta as pessoas que estão
Art. 1.609. O reconhecimento dos filhos havidos fora do umas para com as outras na relação de ascendentes e
casamento é irrevogável e será feito: descendentes.
I - no registro do nascimento;
II - por escritura pública ou escrito particular, a ser Art. 1.592. São parentes em linha colateral ou transversal,
arquivado em cartório; até o quarto grau, as pessoas provenientes de um só tronco, sem
III - por testamento, ainda que incidentalmente descenderem uma da outra.
manifestado;
IV - por manifestação direta e expressa perante o juiz, ainda Art. 1.593. O parentesco é natural ou civil, conforme resulte
que o reconhecimento não haja sido o objeto único e principal de consanguinidade ou outra origem.
do ato que o contém.
Parágrafo único. O reconhecimento pode preceder o Art. 1.594. Contam-se, na linha reta, os graus de parentesco
nascimento do filho ou ser posterior ao seu falecimento, se ele pelo número de gerações, e, na colateral, também pelo número
deixar descendentes. delas, subindo de um dos parentes até ao ascendente comum, e
descendo até encontrar o outro parente.
O reconhecimento dos filhos é ato jurídico stricto sensu, ou
seja, seus efeitos decorrem da lei. Art. 1.595. Cada cônjuge ou companheiro é aliado aos
parentes do outro pelo vínculo da afinidade.
Atenção! segundo o art. 1.614 do CC o reconhecimento de § 1º O parentesco por afinidade limita-se aos ascendentes,
filho maior depende de seu consentimento. aos descendentes e aos irmãos do cônjuge ou companheiro.
A exigência desse consentimento não torna o § 2º Na linha reta, a afinidade não se extingue com a
reconhecimento um ato bilateral. dissolução do casamento ou da união estável.

2 Manual de Direito Civil. Volume Único. Método. 2017.

Legislação 45
Apostila Digital Licenciada para Alice Caroline Guarino dos Santos - alice.guarino@hotmail.com (Proibida a Revenda)
APOSTILAS OPÇÃO

CAPÍTULO II CAPÍTULO III


Da Filiação Do Reconhecimento dos Filhos

Art. 1.596. Os filhos, havidos ou não da relação de Art. 1.607. O filho havido fora do casamento pode ser
casamento, ou por adoção, terão os mesmos direitos e reconhecido pelos pais, conjunta ou separadamente.
qualificações, proibidas quaisquer designações discriminatórias
relativas à filiação. Art. 1.608. Quando a maternidade constar do termo do
nascimento do filho, a mãe só poderá contestá-la, provando a
Art. 1.597. Presumem-se concebidos na constância do falsidade do termo, ou das declarações nele contidas.
casamento os filhos:
I - nascidos cento e oitenta dias, pelo menos, depois de Art. 1.609. O reconhecimento dos filhos havidos fora do
estabelecida a convivência conjugal; casamento é irrevogável e será feito:
II - nascidos nos trezentos dias subsequentes à dissolução da I - no registro do nascimento;
sociedade conjugal, por morte, separação judicial, nulidade e II - por escritura pública ou escrito particular, a ser
anulação do casamento; arquivado em cartório;
III - havidos por fecundação artificial homóloga, mesmo que III - por testamento, ainda que incidentalmente
falecido o marido; manifestado;
IV - havidos, a qualquer tempo, quando se tratar de embriões IV - por manifestação direta e expressa perante o juiz, ainda
excedentários, decorrentes de concepção artificial homóloga; que o reconhecimento não haja sido o objeto único e principal
V - havidos por inseminação artificial heteróloga, desde que do ato que o contém.
tenha prévia autorização do marido. Parágrafo único. O reconhecimento pode preceder o
nascimento do filho ou ser posterior ao seu falecimento, se ele
Art. 1.598. Salvo prova em contrário, se, antes de decorrido deixar descendentes.
o prazo previsto no inciso II do art. 1.523, a mulher contrair
novas núpcias e lhe nascer algum filho, este se presume do Art. 1.610. O reconhecimento não pode ser revogado, nem
primeiro marido, se nascido dentro dos trezentos dias a contar mesmo quando feito em testamento.
da data do falecimento deste e, do segundo, se o nascimento
ocorrer após esse período e já decorrido o prazo a que se refere Art. 1.611. O filho havido fora do casamento, reconhecido
o inciso I do art. 1597. por um dos cônjuges, não poderá residir no lar conjugal sem o
consentimento do outro.
Art. 1.599. A prova da impotência do cônjuge para gerar, à
época da concepção, ilide a presunção da paternidade. Art. 1.612. O filho reconhecido, enquanto menor, ficará sob a
guarda do genitor que o reconheceu, e, se ambos o
Art. 1.600. Não basta o adultério da mulher, ainda que reconheceram e não houver acordo, sob a de quem melhor
confessado, para ilidir a presunção legal da paternidade. atender aos interesses do menor.

Art. 1.601. Cabe ao marido o direito de contestar a Art. 1.613. São ineficazes a condição e o termo apostos ao
paternidade dos filhos nascidos de sua mulher, sendo tal ação ato de reconhecimento do filho.
imprescritível.
Parágrafo único. Contestada a filiação, os herdeiros do Art. 1.614. O filho maior não pode ser reconhecido sem o seu
impugnante têm direito de prosseguir na ação. consentimento, e o menor pode impugnar o reconhecimento, nos
quatro anos que se seguirem à maioridade, ou à emancipação.
Art. 1.602. Não basta a confissão materna para excluir a
paternidade. Art. 1.615. Qualquer pessoa, que justo interesse tenha, pode
contestar a ação de investigação de paternidade, ou
Art. 1.603. A filiação prova-se pela certidão do termo de maternidade.
nascimento registrada no Registro Civil.
Art. 1.616. A sentença que julgar procedente a ação de
Art. 1.604. Ninguém pode vindicar estado contrário ao que investigação produzirá os mesmos efeitos do reconhecimento;
resulta do registro de nascimento, salvo provando-se erro ou mas poderá ordenar que o filho se crie e eduque fora da
falsidade do registro. companhia dos pais ou daquele que lhe contestou essa
qualidade.
Art. 1.605. Na falta, ou defeito, do termo de nascimento,
poderá provar-se a filiação por qualquer modo admissível em Art. 1.617. A filiação materna ou paterna pode resultar de
direito: casamento declarado nulo, ainda mesmo sem as condições do
I - quando houver começo de prova por escrito, proveniente putativo.
dos pais, conjunta ou separadamente;
II - quando existirem veementes presunções resultantes de CAPÍTULO IV
fatos já certos. Da Adoção

Art. 1.606. A ação de prova de filiação compete ao filho, Art. 1.618. Só a pessoa maior de dezoito anos pode adotar.
enquanto viver, passando aos herdeiros, se ele morrer menor ou Parágrafo único. A adoção por ambos os cônjuges ou
incapaz. companheiros poderá ser formalizada, desde que um deles tenha
Parágrafo único. Se iniciada a ação pelo filho, os herdeiros completado dezoito anos de idade, comprovada a estabilidade
poderão continuá-la, salvo se julgado extinto o processo. da família.

Art. 1.618. A adoção de crianças e adolescentes será deferida


na forma prevista pela Lei no 8.069, de 13 de julho de 1990 -
Estatuto da Criança e do Adolescente. (Redação dada pela Lei nº
12.010, de 2009)

Legislação 46
Apostila Digital Licenciada para Alice Caroline Guarino dos Santos - alice.guarino@hotmail.com (Proibida a Revenda)
APOSTILAS OPÇÃO

Art. 1.619. O adotante há de ser pelo menos dezesseis anos VII - exigir que lhes prestem obediência, respeito e os serviços
mais velho que o adotado. próprios de sua idade e condição.

Art. 1.619. A adoção de maiores de 18 (dezoito) anos Art. 1.634. Compete a ambos os pais, qualquer que seja a sua
dependerá da assistência efetiva do poder público e de sentença situação conjugal, o pleno exercício do poder familiar, que
constitutiva, aplicando-se, no que couber, as regras gerais da Lei consiste em, quanto aos filhos: (Redação dada pela Lei nº 13.058,
nº 8.069, de 13 de julho de 1990 - Estatuto da Criança e do de 2014)
Adolescente. (Redação dada pela Lei nº 12.010, de 2009) I - dirigir-lhes a criação e a educação; (Redação dada pela
Lei nº 13.058, de 2014)
Art. 1.620. (Revogado pela Lei nº 12.010, de 2009) II- exercer a guarda unilateral ou compartilhada nos termos
do art. 1.584; (Redação dada pela Lei nº 13.058, de 2014)
Art. 1.621. (Revogado pela Lei nº 12.010, de 2009) III - conceder-lhes ou negar-lhes consentimento para
casarem; (Redação dada pela Lei nº 13.058, de 2014)
Art. 1.622. (Revogado pela Lei nº 12.010, de 2009) IV - conceder-lhes ou negar-lhes consentimento para
viajarem ao exterior; (Redação dada pela Lei nº 13.058, de
Art. 1.623. (Revogado pela Lei nº 12.010, de 2009) 2014)
V - conceder-lhes ou negar-lhes consentimento para
Art. 1.624. (Revogado pela Lei nº 12.010, de 2009) mudarem sua residência permanente para outro Município;
(Redação dada pela Lei nº 13.058, de 2014)
Art. 1.625. (Revogado pela Lei nº 12.010, de 2009) VI - nomear-lhes tutor por testamento ou documento
autêntico, se o outro dos pais não lhe sobreviver, ou o sobrevivo
Art. 1.626. (Revogado pela Lei nº 12.010, de 2009) não puder exercer o poder familiar; (Redação dada pela Lei nº
13.058, de 2014)
Art. 1.627. (Revogado pela Lei nº 12.010, de 2009) VII - representá-los judicial e extrajudicialmente até os 16
(dezesseis) anos, nos atos da vida civil, e assisti-los, após essa
Art. 1.628. (Revogado pela Lei nº 12.010, de 2009) idade, nos atos em que forem partes, suprindo-lhes o
consentimento; (Redação dada pela Lei nº 13.058, de 2014)
Art. 1.629. (Revogado pela Lei nº 12.010, de 2009) VIII - reclamá-los de quem ilegalmente os detenha; (Incluído
pela Lei nº 13.058, de 2014)
CAPÍTULO V IX - exigir que lhes prestem obediência, respeito e os serviços
Do Poder FAMILIAR próprios de sua idade e condição. (Incluído pela Lei nº 13.058,
Seção I de 2014)
Disposições Gerais
Seção III
Art. 1.630. Os filhos estão sujeitos ao poder familiar, Da Suspensão e Extinção do Poder Familiar
enquanto menores.
Art. 1.635. Extingue-se o poder familiar:
Art. 1.631. Durante o casamento e a união estável, compete I -pela morte dos pais ou do filho;
o poder familiar aos pais; na falta ou impedimento de um deles, II - pela emancipação, nos termos do art. 5o, parágrafo
o outro o exercerá com exclusividade. único;
Parágrafo único. Divergindo os pais quanto ao exercício do III- pela maioridade;
poder familiar, é assegurado a qualquer deles recorrer ao juiz IV - pela adoção;
para solução do desacordo. V - por decisão judicial, na forma do artigo 1.638.

Art. 1.632. A separação judicial, o divórcio e a dissolução da Art. 1.636. O pai ou a mãe que contrai novas núpcias, ou
união estável não alteram as relações entre pais e filhos senão estabelece união estável, não perde, quanto aos filhos do
quanto ao direito, que aos primeiros cabe, de terem em sua relacionamento anterior, os direitos ao poder familiar,
companhia os segundos. exercendo-os sem qualquer interferência do novo cônjuge ou
companheiro.
Art. 1.633. O filho, não reconhecido pelo pai, fica sob poder Parágrafo único. Igual preceito ao estabelecido neste artigo
familiar exclusivo da mãe; se a mãe não for conhecida ou capaz aplica-se ao pai ou à mãe solteiros que casarem ou
de exercê-lo, dar-se-á tutor ao menor. estabelecerem união estável.

Seção II Art. 1.637. Se o pai, ou a mãe, abusar de sua autoridade,


Do Exercício do Poder Familiar faltando aos deveres a eles inerentes ou arruinando os bens dos
filhos, cabe ao juiz, requerendo algum parente, ou o Ministério
Art. 1.634. Compete aos pais, quanto à pessoa dos filhos Público, adotar a medida que lhe pareça reclamada pela
menores: segurança do menor e seus haveres, até suspendendo o poder
I - dirigir-lhes a criação e educação; familiar, quando convenha.
II - tê-los em sua companhia e guarda; Parágrafo único. Suspende-se igualmente o exercício do
III - conceder-lhes ou negar-lhes consentimento para poder familiar ao pai ou à mãe condenados por sentença
casarem; irrecorrível, em virtude de crime cuja pena exceda a dois anos de
IV - nomear-lhes tutor por testamento ou documento prisão.
autêntico, se o outro dos pais não lhe sobreviver, ou o sobrevivo
não puder exercer o poder familiar; Art. 1.638. Perderá por ato judicial o poder familiar o pai ou
V - representá-los, até aos dezesseis anos, nos atos da vida a mãe que:
civil, e assisti-los, após essa idade, nos atos em que forem partes, I - castigar imoderadamente o filho;
suprindo-lhes o consentimento; II - deixar o filho em abandono;
VI - reclamá-los de quem ilegalmente os detenha; III - praticar atos contrários à moral e aos bons costumes;

Legislação 47
Apostila Digital Licenciada para Alice Caroline Guarino dos Santos - alice.guarino@hotmail.com (Proibida a Revenda)
APOSTILAS OPÇÃO

IV - incidir, reiteradamente, nas faltas previstas no artigo deste binômio, desaparecerá também a obrigação alimentar;
antecedente. havendo significativa mudança na situação financeira de quem
os supre, ou na de quem os recebe, justificada estará a
Código Civil – arts. 1.694 a 1.727 possibilidade de modificação do montante alimentar devido,
nos termos do artigo 1.699 do Código Civil.
Alimentos.
Os alimentos devem suprir todas as necessidades do Execução da Obrigação Alimentar
alimentado, não se restringindo meramente à subsistência do A obrigação alimentar é executável a partir de seu
corpo físico, englobando moradia, o vestuário, saúde, inadimplemento. Ocorrendo a mora no cumprimento da
educação, inclusive lazer, visando portanto de um modo geral obrigação, nasce ao credor o direito de executar a prestação
não somente a sobrevivência do indivíduo, mas a manutenção devida, compelindo através de ação judicial o devedor, sob
deda dignidade e vida do alimentado. pena de expropriação de bens ou de prisão civil. Não se faz
A obrigação de prestar alimentos pode ter origem necessário aguardar o acúmulo de três ou mais parcelas
decorrente da relação de parentesco ou pelo vínculo vencidas para só então buscar a execução do crédito. As
matrimonial ou de união estável. Entre parentes, especifica a prestações alimentares são independentes entre si e permitem
lei que a obrigação de prestar alimentos é recíproca entre pais a forma coativa de cobrança a partir da primeira parcela
e filhos, e extensivo a todos os ascendentes, recaindo a efetivamente vencida.
obrigação nos mais próximos em grau, uns em falta de outros.
Assim, se estabelece a legitimidade ativa e passiva da Formas de execução dos alimentos
obrigação alimentar. A norma processual civil elegeu duas formas que permitem
Importante salientar que há reciprocidade no que tange a ao credor buscar o cumprimento da obrigação alimentar. São
alimentos, alcançando a todos parentes em linha reta, ou seja, elas:
ascendentes e descendentes, podendo assim os alimentos - Execução de alimentos sob pena de expropriação de bens
serem prestados ou exigidos entre pais e filhos, avós e netos, do devedor e
bisavós e bisnetos, etc. - Execução sob pena de prisão civil.

Os alimentos podem também ser buscados na linha Execução de alimentos sob pena de expropriação de
colateral, respeitando os limites impostos pelo artigo 1.697 bens do devedor, encontra-se no artigo 732 do Código de
que transfere a obrigação alimentar aos irmãos germanos ou Processo Civil.
unilaterais do alimentando. Com relação aos colaterais de A expropriação de bens do devedor para cumprimento de
maior grau (3º e 4º graus), a lei silencia acerca do tema. Deste obrigação alimentar é aplicável a todas as prestações, as quais
modo, ante a omissão da lei, exclui-se a responsabilidade não dizem respeito às três últimas parcelas vencidas, não
alimentar destes parentes, consoante o entendimento sendo constituídas com a urgência alimentar, ficando,
majoritário da jurisprudência, que tem afastado a obrigação de portanto, sujeitas à regra geral da execução por quantia certa
prestar alimentos entre tios e sobrinhos, face ao parentesco de contra devedor solvente, disposto pelo artigo 732, conforme já
terceiro grau e ausência de imposição legal para tanto. citado, ou ainda do cumprimento de sentença do artigo 475-I,
ambos do CPC.
Podem também os cônjuges ou companheiros pleitear
alimentos de seus consortes ou conviventes. Nestes casos, a Execução sujeita à pena de prisão civil, denominada
obrigação alimentar decorre do dever de mútua assistência, como coação pessoal, encontra-se regulamentada no artigo
previsto nos artigos 1.566, inciso III, e 1724 do Código Civil. 733 do Código de Processo Civil, com raiz na Constituição
Federal, artigo 5º, inciso LXVII, que permite no sistema
Sejam decorrentes do vínculo de parentesco, do casamento processual pátrio a aplicação de pena de prisão civil ao
ou da união estável, os alimentos deverão ser prestados em inadimplente, voluntário e inescusável, de obrigação
montante suficiente a garantir ao alimentando subsistência de alimentícia.
modo compatível com a sua condição social, inclusive A urgência no cumprimento da prestação alimentar se
educação, atendendo dignamente às suas necessidades. justifica pela própria necessidade do indivíduo. Os alimentos
representam garantia de sobrevivência digna do homem,
Para que os alimentos sejam efetivamente devidos é sendo portanto necessário impor a esta espécie obrigacional
necessário respeitar o instituto denominado binômio da maior coercibilidade na busca de seu adimplemento,
necessidade e a possibilidade. autorizando assim que o devedor seja compelido ao
pagamento da obrigação alimentícia sob pena de prisão civil.
A necessidade daquele que pleiteia os alimentos é o Frise-se que a aplicação do artigo 733 do CPC diz respeito ao
primeiro e imprescindível requisito da obrigação alimentar. inadimplemento das três últimas parcelas da prestação
Somente diante da demonstração de necessidade é que se alimentar, e ainda aquelas que vencerem no curso do processo,
apresenta a possibilidade jurídica de postular os alimentos. Os vez que mantêm estas a urgência e relevância do encargo.
alimentos portanto, serão devidos quando aquele que Deste modo, após o pleiteio da demanda o devedor, será
pretende recebê-los não dispõe de bens suficientes, nem pode citado e terá ele o prazo de três dias para efetuar o pagamento
prover, pelo seu trabalho, a própria mantença. Por outro lado, das prestações em atraso. Não ocorrendo o pagamento, poderá
a possibilidade é a contrapartida lógica da obrigação, ou seja, o devedor justificar a impossibilidade de fazê-lo ou, ainda,
aquele de quem se cobram os alimentos deve dispor de provar que a dívida executada já foi regularmente quitada.
condições financeiras suficientes a provê-los, sem que com A prisão civil será decretada pelo juiz da execução quando
isso venha a comprometer o valor necessário ao seu próprio o devedor deixar de efetivar o pagamento, de provar que já o
sustento e ao de seus dependentes. fez ou de justificar a impossibilidade de fazê-lo. A prisão
poderá se prolongar por até três meses, caso não haja o
O binômio "necessidade-possibilidade" norteia a prestação pagamento da dívida. Ocorrendo pagamento ou transação
alimentar. Por sua verificação o juiz disporá de bases antes da prisão, o mandado será revogado; se posterior, será o
necessárias a formar seu convencimento acerca da devedor imediatamente posto em liberdade. Sobre o tema, o
procedência ou não da ação alimentar, bem como do montante parágrafo 2º do artigo 733 menciona que o cumprimento da
devido pela prestação. Desaparecendo, portanto, um dos polos

Legislação 48
Apostila Digital Licenciada para Alice Caroline Guarino dos Santos - alice.guarino@hotmail.com (Proibida a Revenda)
APOSTILAS OPÇÃO

pena não exime o devedor do pagamento das prestações alimentos pagos por ele em favor da criança que, depois se
vencidas e vincendas. soube, era filha de outro homem (REsp 412.684-SP, Quarta
Importante salientar que permanecem intactas as Turma, DJ 25/11/2002). De mais a mais, quaisquer valores que
disposições do artigo 733 e seus parágrafos, em relação à sejam porventura apurados em favor do alimentante estarão
possibilidade de coação pessoal do devedor, submetendo-o à cobertos pelo princípio da irrepetibilidade dos alimentos já
pena de prisão civil, entretanto, neste caso não se aplica a pagos, justificado pelo dever de solidariedade entre os seres
multa de 10% apresentada pelo artigo 475-J do CPC humanos, uma vez que, em última análise, os alimentos
garantem a própria existência do alimentando. (REsp n.
Alimentos Gravídicos 922.462-SP, rel. Ministro Ricardo Villas Bôas Cueva, j. em
A lei nº 11.804/08 trata sobre os alimentos gravídicos. 04.04.2013).
Dessa maneira, a teoria concepcionista ganha força,
garantindo-se o direito à vida antes do nascimento. A lei Alimentos Provisionais x Alimentos Provisórios
assegura a mulher grávida o direito a alimentos a lhe serem
alcançados por quem afirma ser o pai do seu filho. Com a Os alimentos provisionais possuem natureza jurídica de
criação desta lei, supriu-se a falha na legislação referente com ação cautelar, visando garantir a eficácia de uma ação principal
a inexistência de regulamentação dos alimentos devidos aos em que se pleiteiam alimentos. São os casos de divórcio,
nascituros. dissolução de união estável, ação de alimentos, etc. Desta
maneira, exige a presença de dois pressupostos legais, fumus
Acompanhe o que traz o artigo 2º da Lei nº boni iuri e o periculum in mora. São concedidos
11.804/2008: provisoriamente no correr de uma lide, onde se pleiteiam os
Art. 2º Os alimentos de que trata esta Lei compreenderão os alimentos definitivos.
valores suficientes para cobrir as despesas adicionais do período
de gravidez e que sejam dela decorrentes, da concepção ao Como a intenção é a satisfação, entende-se que se trata de
parto, inclusive as referentes a alimentação especial, assistência hipótese de tutela antecipada. Portanto, devem ser requeridos
médica e psicológica, exames complementares, internações, em processo apartado do principal, com fundamentação no
parto, medicamentos e demais prescrições preventivas e Código de Processo Civil.
terapêuticas indispensáveis, a juízo do médico, além de outras
que o juiz considere pertinentes. Os alimentos provisórios, por sua vez, são conferidos
Parágrafo único. Os alimentos de que trata este artigo dentro da demanda principal em que se pleiteiam alimentos
referem-se à parte das despesas que deverá ser custeada pelo definitivos e que respeita o regime jurídico especial da Lei
futuro pai, considerando-se a contribuição que também deverá 5478/68. O juiz também poderá conceder liminarmente os
ser dada pela mulher grávida, na proporção dos recursos de alimentos definitivos ao alimentando sem que precise esperar
ambos. que o processo chegue ao fim. Trata-se de uma tutela.
Nos alimentos provisórios exige-se apenas a prova pré-
Os alimentos gravídicos perduram até o nascimento, sendo constituída da obrigação alimentar (certidão de nascimento) e
o réu citado para apresentar resposta em 5 (cinco) dias. não inclui verba de custeio da demanda.
Acompanhe a Lei nº 11.804/2008:
Conclusão
Art. 6º Convencido da existência de indícios da paternidade, Sintetizando estas questões, a execução da obrigação
o juiz fixará alimentos gravídicos que perdurarão até o alimentar inadimplida não se ajusta mais ao antigo método
nascimento da criança, sopesando as necessidades da parte apresentado pelo Título II, Capítulo V do Livro II do CPC (em
autora e as possibilidades da parte ré. relação específica ao artigo 732), sendo imprescindível
Parágrafo único. Após o nascimento com vida, os alimentos reconhecer-se que a inovação processual trazida pela Lei
gravídicos ficam convertidos em pensão alimentícia em favor do 11.232/05 alcançou todas as formas de execução pautada em
menor até que uma das partes solicite a sua revisão. decisão judicial, dentre elas, a execução de alimentos. O
descumprimento de prestação alimentícia com origem em
Art. 7ºO réu será citado para apresentar resposta em 5 decisão judicial condenatória ou homologatória, enseja
(cinco) dias. procedimento de cumprimento de sentença, de acordo com o
artigo 475-I, passível de aplicação da multa de 10%, conforme
Sobre o dever de alimentos e relação socioafetiva, artigo 475-J, ambos do CPC. Não é justificável a aplicação do
citamos recente julgado: artigo 732 do CPC, que remete à execução por quantia certa
Direito Civil. Alimentos na hipótese de formação de vínculo contra devedor solvente, vez que o cumprimento de sentença,
socioafetivo. A esposa infiel não tem o dever de restituir ao nos moldes atuais, é mais célere e eficaz ao credor e não
marido traído os alimentos pagos por ele em favor de filho importa em nenhum prejuízo ao devedor.
criado com estreitos laços de afeto pelo casal, ainda que a
adúltera tenha ocultado do marido o fato de que a referida Referências
criança seria filha biológica sua e de seu “cúmplice”. Isso porque,
se o marido, ainda que enganado por sua esposa, cria como seu RIBAS, Claudia Aparecida Colla Taques. Execução da
o filho biológico de outrem, tem-se por configurada verdadeira obrigação alimentar. Revista Jus Navigandi, Teresina, ano
relação de paternidade socioafetiva, a qual, por si mesma, 16, n. 2869, 10 maio 2011. Disponível
impede a repetição da verba alimentar, haja vista que, a fim de em: <http://jus.com.br/artigos/19069>. Acesso em: 5 maio
preservar o elo da afetividade, deve-se considerar secundária a 2015.
verdade biológica, porquanto a CF e o próprio CC garantem a
igualdade absoluta dos filhos de qualquer origem (biológica ou Código Civil – arts. 1.728 a 1.783
não biológica). Além do mais, o dever de fidelidade recíproca dos
cônjuges, atributo básico do casamento, em nada se comunica TUTELA E CURATELA
com a relação paternal gerada, mostrando-se desarrazoado
transferir o ônus por suposto insucesso da relação à criança A tutela nada mais é do que instituto de proteção aos
alimentada. Ademais, o STJ já firmou o entendimento de que a menores que não estão sob o poder familiar dos pais.
mulher não está obrigada a restituir ao marido o valor dos

Legislação 49
Apostila Digital Licenciada para Alice Caroline Guarino dos Santos - alice.guarino@hotmail.com (Proibida a Revenda)
APOSTILAS OPÇÃO

Trata-se de conjunto de direitos e obrigações conferidos Resumindo, instituir um imóvel como bem de família dará
pela lei a um terceiro, de forma que proteja a pessoa de um uma garantia aos cônjuges ou a entidade familiar.
menor não emancipado que não esteja sob o poder familiar,
administrando seus bens, representando e assistindo-os na Essa garantia pode ser instituída voluntariamente (bem de
vida civil. família voluntário ou convencional) ou em razão da lei (bem de
família legal).
Sua natureza jurídica é de múnus público (encargo), que
deve ser imposto pelo Estado, devendo atender um interesse - Voluntário: também chamado de convencional é o
público. instituído por membro ou entidade familiar. Trata-se de
A tutela é personalíssima. prédio ou parcela do patrimônio que os cônjuges ou
conviventes destinam para abrigo e domicílio, com cláusula de
A tutela pode ser: ficar isento da execução por dívidas futuras.
- testamentária;
- legítima; Esse instituto tem por finalidade assegurar um lar à
- dativa. família.

Será concedida tutela para proteção de menor não Os Tribunais têm admitido o bem de família de pessoa
emancipado e de seus bens, em caso de falecimento de seus solteira. Sua instituição ocorrerá por meio de escritura pública
pais que forem declarados ausentes pelo juiz ou que sofrerem ou testamento.
suspensão ou destituição do poder familiar.
A Tutela se extingue com o alcance da maioridade ou O bem de família voluntário só será penhorado nos
emancipação do tutelado e quando este é adotado. seguintes casos:
a) por dívidas de tributos relacionados ao prédio;
A curatela é instituto de proteção à pessoa e ao patrimônio b) por dívidas relativas ao condomínio do prédio.
dos maiores incapazes.
- Legal: é regulado pela Lei nº 8009/90, que estabelece a
O Estatuto da Pessoa com Deficiência, assegura às pessoas impenhorabilidade geral de todas as moradias familiares
com deficiência mental ou intelectual o direito ao exercício da próprias, uma para cada família, independente de ato do
capacidade legal em condições de igualdade com as demais interessado, para isso, se a família tiver mais de um imóvel, o
pessoas. bem de família será aquele de menor valor.
A penhora desse imóvel se dará nas seguintes hipóteses:
O instituto da curatela é uma medida extraordinária, a) crédito de natureza trabalhista, previdenciário;
proporcional as necessidades e circunstâncias e tem como b) créditos referentes a aquisição/ construção de bem;
objetivo a menor duração do processo. Tem como objetivo a c) pensão alimentícia;
proteção dos elementos negociais e patrimoniais, não d) taxas, imposto de imóvel;
atingindo o direito das pessoas. e) hipoteca do bem, dado em garantia;
f) dívida de fiança concedida em contrato de locação;
Podemos citar a autocuratela que ocorre quando o g) valores decorrentes da aquisição do imóvel com
curatelado, plenamente capaz, elege seu curador. Já na produto de crime.
curatela compartilhada, pode ocorrer do deficiente escolher
mais que um curador, para que possa gerir e administrar seu A Súmula 364 do STJ ampliou o conceito de família,
patrimônio. Competirá também ao juiz escolher mais de um podendo ser protegidas as pessoas solteiras, viúvas e
curador. separadas:

Tutela Curatela Súmula 364 STJ. O conceito de impenhorabilidade de bem


Visa a proteção da Visa a proteção aos de família abrange também o imóvel pertencente a pessoas
criança e do adolescente, maiores incapazes para solteiras, separadas e viúvas.
em razão da menoridade, autodeterminação.
não dispõem de plena Quanto à destinação do bem de família, os valores
capacidade e seus genitores mobiliários, abrangidos no bem de família deverá ser aplicada
estão afastados do poder para conservação do imóvel, que abriga os instituidores e sua
familiar. prole, para sustento da família.
Os valores mobiliários, destinados à instituição do bem de
OBS: Quanto ao nascituro, ele será submetido à curatela e família, não podem exceder ao valor do prédio, que constitui o
não à tutela. domicílio familiar, visto que se destinam à conservação do
imóvel e ao sustento da família, que ali reside.
BEM DE FAMÍLIA
Os valores mobiliários devem ser individualizados no
Conceito: esse termo é usado para denominar o imóvel de instrumento, seja por escritura pública, testamento, que visa
um casal ou de uma entidade familiar, que em razão de instituir o bem de família. Caso se trate de títulos nominativos,
proteção legal, não poderá ser penhorado. deverão, por sua vez, constar dos respectivos livros de
registro, conferindo-se publicidade ao ato, isentando os
O requisito objetivo do bem de família consiste em imóvel referidos títulos da satisfação de débitos futuros.
residencial rural e urbano, destinado para abrigo familiar e
inclui as pertenças e acessórios, valores mobiliários, cuja Quanto à requisição formal da instituição do bem de
renda deverá ser aplicada na conservação de prédio e no família, esta deverá ocorrer por testamento ou por escritura
sustento da família. pública, com a individuação do prédio e declaração de sua
destinação, devidamente registrada. Não é qualquer imóvel
que pode ser objeto de bem de família, mas sim prédio

Legislação 50
Apostila Digital Licenciada para Alice Caroline Guarino dos Santos - alice.guarino@hotmail.com (Proibida a Revenda)
APOSTILAS OPÇÃO

residencial, urbano ou rural, com o limite de valor estipulado Art. 1.703. Para a manutenção dos filhos, os cônjuges
no artigo 1.711 do Código Civil. separados judicialmente contribuirão na proporção de seus
recursos.
Lavrada a escritura pública, os instituidores a entregarão
ao oficial do Registro Imobiliário da situação do bem, para que Art. 1.704. Se um dos cônjuges separados judicialmente
seja providenciado os editais para sua publicação na imprensa vier a necessitar de alimentos, será o outro obrigado a prestá-
local, de forma que torne ciente os interessados que deverão los mediante pensão a ser fixada pelo juiz, caso não tenha sido
apresentar impugnação dentro de 30 dias, contados da declarado culpado na ação de separação judicial.
publicação. Parágrafo único. Se o cônjuge declarado culpado vier a
necessitar de alimentos, e não tiver parentes em condições de
Veja os dispositivos correspondentes: prestá-los, nem aptidão para o trabalho, o outro cônjuge será
obrigado a assegurá-los, fixando o juiz o valor indispensável à
SUBTÍTULO III sobrevivência.
Dos Alimentos
Art. 1.705. Para obter alimentos, o filho havido fora do
Art. 1.694. Podem os parentes, os cônjuges ou casamento pode acionar o genitor, sendo facultado ao juiz
companheiros pedir uns aos outros os alimentos de que determinar, a pedido de qualquer das partes, que a ação se
necessitem para viver de modo compatível com a sua condição processe em segredo de justiça.
social, inclusive para atender às necessidades de sua educação.
§ 1º Os alimentos devem ser fixados na proporção das Art. 1.706. Os alimentos provisionais serão fixados pelo
necessidades do reclamante e dos recursos da pessoa juiz, nos termos da lei processual.
obrigada.
§ 2º Os alimentos serão apenas os indispensáveis à Art. 1.707. Pode o credor não exercer, porém lhe é vedado
subsistência, quando a situação de necessidade resultar de renunciar o direito a alimentos, sendo o respectivo crédito
culpa de quem os pleiteia. insuscetível de cessão, compensação ou penhora.

Art. 1.695. São devidos os alimentos quando quem os Art. 1.708. Com o casamento, a união estável ou o
pretende não tem bens suficientes, nem pode prover, pelo seu concubinato do credor, cessa o dever de prestar alimentos.
trabalho, à própria mantença, e aquele, de quem se reclamam, Parágrafo único. Com relação ao credor cessa, também, o
pode fornecê-los, sem desfalque do necessário ao seu sustento. direito a alimentos, se tiver procedimento indigno em relação
ao devedor.
Art. 1.696. O direito à prestação de alimentos é recíproco
entre pais e filhos, e extensivo a todos os ascendentes, Art. 1.709. O novo casamento do cônjuge devedor não
recaindo a obrigação nos mais próximos em grau, uns em falta extingue a obrigação constante da sentença de divórcio.
de outros.
Art. 1.710. As prestações alimentícias, de qualquer
Art. 1.697. Na falta dos ascendentes cabe a obrigação aos natureza, serão atualizadas segundo índice oficial
descendentes, guardada a ordem de sucessão e, faltando estes, regularmente estabelecido.
aos irmãos, assim germanos como unilaterais.
SUBTÍTULO IV
Art. 1.698. Se o parente, que deve alimentos em primeiro Do Bem de Família
lugar, não estiver em condições de suportar totalmente o
encargo, serão chamados a concorrer os de grau imediato; Art. 1.711. Podem os cônjuges, ou a entidade familiar,
sendo várias as pessoas obrigadas a prestar alimentos, todas mediante escritura pública ou testamento, destinar parte de
devem concorrer na proporção dos respectivos recursos, e, seu patrimônio para instituir bem de família, desde que não
intentada ação contra uma delas, poderão as demais ser ultrapasse um terço do patrimônio líquido existente ao tempo
chamadas a integrar a lide. da instituição, mantidas as regras sobre a impenhorabilidade
do imóvel residencial estabelecida em lei especial.
Art. 1.699. Se, fixados os alimentos, sobrevier mudança na Parágrafo único. O terceiro poderá igualmente instituir
situação financeira de quem os supre, ou na de quem os recebe, bem de família por testamento ou doação, dependendo a
poderá o interessado reclamar ao juiz, conforme as eficácia do ato da aceitação expressa de ambos os cônjuges
circunstâncias, exoneração, redução ou majoração do encargo. beneficiados ou da entidade familiar beneficiada.

Art. 1.700. A obrigação de prestar alimentos transmite-se Art. 1.712. O bem de família consistirá em prédio
aos herdeiros do devedor, na forma do art. 1.694. residencial urbano ou rural, com suas pertenças e acessórios,
destinando-se em ambos os casos a domicílio familiar, e
Art. 1.701. A pessoa obrigada a suprir alimentos poderá poderá abranger valores mobiliários, cuja renda será aplicada
pensionar o alimentando, ou dar-lhe hospedagem e sustento, na conservação do imóvel e no sustento da família.
sem prejuízo do dever de prestar o necessário à sua educação,
quando menor. Art. 1.713. Os valores mobiliários, destinados aos fins
Parágrafo único. Compete ao juiz, se as circunstâncias o previstos no artigo antecedente, não poderão exceder o valor
exigirem, fixar a forma do cumprimento da prestação. do prédio instituído em bem de família, à época de sua
instituição.
Art. 1.702. Na separação judicial litigiosa, sendo um dos § 1º Deverão os valores mobiliários ser devidamente
cônjuges inocente e desprovido de recursos, prestar-lhe-á o individualizados no instrumento de instituição do bem de
outro a pensão alimentícia que o juiz fixar, obedecidos os família.
critérios estabelecidos no art. 1.694. § 2º Se se tratar de títulos nominativos, a sua instituição
como bem de família deverá constar dos respectivos livros de
registro.

Legislação 51
Apostila Digital Licenciada para Alice Caroline Guarino dos Santos - alice.guarino@hotmail.com (Proibida a Revenda)
APOSTILAS OPÇÃO

§ 3º O instituidor poderá determinar que a administração § 2º As causas suspensivas do art. 1.523 não impedirão a
dos valores mobiliários seja confiada a instituição financeira, caracterização da união estável.
bem como disciplinar a forma de pagamento da respectiva
renda aos beneficiários, caso em que a responsabilidade dos Art. 1.724. As relações pessoais entre os companheiros
administradores obedecerá às regras do contrato de depósito. obedecerão aos deveres de lealdade, respeito e assistência, e
de guarda, sustento e educação dos filhos.
Art. 1.714. O bem de família, quer instituído pelos cônjuges
ou por terceiro, constitui-se pelo registro de seu título no Art. 1.725. Na união estável, salvo contrato escrito entre os
Registro de Imóveis. companheiros, aplica-se às relações patrimoniais, no que
couber, o regime da comunhão parcial de bens.
Art. 1.715. O bem de família é isento de execução por
dívidas posteriores à sua instituição, salvo as que provierem Art. 1.726. A união estável poderá converter-se em
de tributos relativos ao prédio, ou de despesas de condomínio. casamento, mediante pedido dos companheiros ao juiz e
Parágrafo único. No caso de execução pelas dívidas assento no Registro Civil.
referidas neste artigo, o saldo existente será aplicado em outro
prédio, como bem de família, ou em títulos da dívida pública, Art. 1.727. As relações não eventuais entre o homem e a
para sustento familiar, salvo se motivos relevantes mulher, impedidos de casar, constituem concubinato.
aconselharem outra solução, a critério do juiz.
TÍTULO IV
Art. 1.716. A isenção de que trata o artigo antecedente Da Tutela e da Curatela
durará enquanto viver um dos cônjuges, ou, na falta destes, até TÍTULO IV
que os filhos completem a maioridade. Da Tutela, da Curatela e da Tomada de Decisão
Apoiada
Art. 1.717. O prédio e os valores mobiliários, constituídos (Redação dada pela Lei nº 13.146, de 2015)
como bem da família, não podem ter destino diverso do CAPÍTULO I
previsto no art. 1.712 ou serem alienados sem o Da Tutela
consentimento dos interessados e seus representantes legais, Seção I
ouvido o Ministério Público. Dos Tutores

Art. 1.718. Qualquer forma de liquidação da entidade Art. 1.728. Os filhos menores são postos em tutela:
administradora, a que se refere o § 3o do art. 1.713, não I - com o falecimento dos pais, ou sendo estes julgados
atingirá os valores a ela confiados, ordenando o juiz a sua ausentes;
transferência para outra instituição semelhante, obedecendo- II - em caso de os pais decaírem do poder familiar.
se, no caso de falência, ao disposto sobre pedido de restituição.
Art. 1.729. O direito de nomear tutor compete aos pais, em
Art. 1.719. Comprovada a impossibilidade da manutenção conjunto.
do bem de família nas condições em que foi instituído, poderá Parágrafo único. A nomeação deve constar de testamento
o juiz, a requerimento dos interessados, extingui-lo ou ou de qualquer outro documento autêntico.
autorizar a sub-rogação dos bens que o constituem em outros,
ouvidos o instituidor e o Ministério Público. Art. 1.730. É nula a nomeação de tutor pelo pai ou pela mãe
que, ao tempo de sua morte, não tinha o poder familiar.
Art. 1.720. Salvo disposição em contrário do ato de
instituição, a administração do bem de família compete a Art. 1.731. Em falta de tutor nomeado pelos pais incumbe
ambos os cônjuges, resolvendo o juiz em caso de divergência. a tutela aos parentes consanguíneos do menor, por esta
Parágrafo único. Com o falecimento de ambos os cônjuges, ordem:
a administração passará ao filho mais velho, se for maior, e, do I - aos ascendentes, preferindo o de grau mais próximo ao
contrário, a seu tutor. mais remoto;
II - aos colaterais até o terceiro grau, preferindo os mais
Art. 1.721. A dissolução da sociedade conjugal não próximos aos mais remotos, e, no mesmo grau, os mais velhos
extingue o bem de família. aos mais moços; em qualquer dos casos, o juiz escolherá entre
Parágrafo único. Dissolvida a sociedade conjugal pela eles o mais apto a exercer a tutela em benefício do menor.
morte de um dos cônjuges, o sobrevivente poderá pedir a
extinção do bem de família, se for o único bem do casal. Art. 1.732. O juiz nomeará tutor idôneo e residente no
domicílio do menor:
Art. 1.722. Extingue-se, igualmente, o bem de família com I - na falta de tutor testamentário ou legítimo;
a morte de ambos os cônjuges e a maioridade dos filhos, desde II - quando estes forem excluídos ou escusados da tutela;
que não sujeitos a curatela. III - quando removidos por não idôneos o tutor legítimo e
o testamentário.
TÍTULO III
DA UNIÃO ESTÁVEL Art. 1.733. Aos irmãos órfãos dar-se-á um só tutor.
§ 1º No caso de ser nomeado mais de um tutor por
Art. 1.723. É reconhecida como entidade familiar a união disposição testamentária sem indicação de precedência,
estável entre o homem e a mulher, configurada na convivência entende-se que a tutela foi cometida ao primeiro, e que os
pública, contínua e duradoura e estabelecida com o objetivo de outros lhe sucederão pela ordem de nomeação, se ocorrer
constituição de família. morte, incapacidade, escusa ou qualquer outro impedimento.
§ 1º A união estável não se constituirá se ocorrerem os § 2º Quem institui um menor herdeiro, ou legatário seu,
impedimentos do art. 1.521; não se aplicando a incidência do poderá nomear-lhe curador especial para os bens deixados,
inciso VI no caso de a pessoa casada se achar separada de fato ainda que o beneficiário se encontre sob o poder familiar, ou
ou judicialmente. tutela.

Legislação 52
Apostila Digital Licenciada para Alice Caroline Guarino dos Santos - alice.guarino@hotmail.com (Proibida a Revenda)
APOSTILAS OPÇÃO

Art. 1.734. Os menores abandonados terão tutores II - reclamar do juiz que providencie, como houver por
nomeados pelo juiz, ou serão recolhidos a estabelecimento bem, quando o menor haja mister correção;
público para este fim destinado, e, na falta desse III - adimplir os demais deveres que normalmente cabem
estabelecimento, ficam sob a tutela das pessoas que, aos pais, ouvida a opinião do menor, se este já contar doze anos
voluntária e gratuitamente, se encarregarem da sua criação. de idade.

Art. 1.734. As crianças e os adolescentes cujos pais forem Art. 1.741. Incumbe ao tutor, sob a inspeção do juiz,
desconhecidos, falecidos ou que tiverem sido suspensos ou administrar os bens do tutelado, em proveito deste,
destituídos do poder familiar terão tutores nomeados pelo Juiz cumprindo seus deveres com zelo e boa-fé.
ou serão incluídos em programa de colocação familiar, na
forma prevista pela Lei nº 8.069, de 13 de julho de 1990 - Art. 1.742. Para fiscalização dos atos do tutor, pode o juiz
Estatuto da Criança e do Adolescente. (Redação dada pela Lei nomear um protutor.
nº 12.010, de 2009)
Art. 1.743. Se os bens e interesses administrativos
Seção II exigirem conhecimentos técnicos, forem complexos, ou
Dos Incapazes de Exercer a Tutela realizados em lugares distantes do domicílio do tutor, poderá
este, mediante aprovação judicial, delegar a outras pessoas
Art. 1.735. Não podem ser tutores e serão exonerados da físicas ou jurídicas o exercício parcial da tutela.
tutela, caso a exerçam:
I - aqueles que não tiverem a livre administração de seus Art. 1.744. A responsabilidade do juiz será:
bens; I - direta e pessoal, quando não tiver nomeado o tutor, ou
II - aqueles que, no momento de lhes ser deferida a tutela, não o houver feito oportunamente;
se acharem constituídos em obrigação para com o menor, ou II - subsidiária, quando não tiver exigido garantia legal do
tiverem que fazer valer direitos contra este, e aqueles cujos tutor, nem o removido, tanto que se tornou suspeito.
pais, filhos ou cônjuges tiverem demanda contra o menor;
III - os inimigos do menor, ou de seus pais, ou que tiverem Art. 1.745. Os bens do menor serão entregues ao tutor
sido por estes expressamente excluídos da tutela; mediante termo especificado deles e seus valores, ainda que os
IV - os condenados por crime de furto, roubo, estelionato, pais o tenham dispensado.
falsidade, contra a família ou os costumes, tenham ou não Parágrafo único. Se o patrimônio do menor for de valor
cumprido pena; considerável, poderá o juiz condicionar o exercício da tutela à
V - as pessoas de mau procedimento, ou falhas em prestação de caução bastante, podendo dispensá-la se o tutor
probidade, e as culpadas de abuso em tutorias anteriores; for de reconhecida idoneidade.
VI - aqueles que exercerem função pública incompatível
com a boa administração da tutela. Art. 1.746. Se o menor possuir bens, será sustentado e
educado a expensas deles, arbitrando o juiz para tal fim as
Seção III quantias que lhe pareçam necessárias, considerado o
Da Escusa dos Tutores rendimento da fortuna do pupilo quando o pai ou a mãe não as
houver fixado.
Art. 1.736. Podem escusar-se da tutela:
I - mulheres casadas; Art. 1.747. Compete mais ao tutor:
II - maiores de sessenta anos; I - representar o menor, até os dezesseis anos, nos atos da
III - aqueles que tiverem sob sua autoridade mais de três vida civil, e assisti-lo, após essa idade, nos atos em que for
filhos; parte;
IV - os impossibilitados por enfermidade; II - receber as rendas e pensões do menor, e as quantias a
V - aqueles que habitarem longe do lugar onde se haja de ele devidas;
exercer a tutela; III - fazer-lhe as despesas de subsistência e educação, bem
VI - aqueles que já exercerem tutela ou curatela; como as de administração, conservação e melhoramentos de
VII - militares em serviço. seus bens;
IV - alienar os bens do menor destinados a venda;
Art. 1.737. Quem não for parente do menor não poderá ser V - promover-lhe, mediante preço conveniente, o
obrigado a aceitar a tutela, se houver no lugar parente idôneo, arrendamento de bens de raiz.
consanguíneo ou afim, em condições de exercê-la.
Art. 1.748. Compete também ao tutor, com autorização do
Art. 1.738. A escusa apresentar-se-á nos dez dias juiz:
subsequentes à designação, sob pena de entender-se I - pagar as dívidas do menor;
renunciado o direito de alegá-la; se o motivo escusatório II - aceitar por ele heranças, legados ou doações, ainda que
ocorrer depois de aceita a tutela, os dez dias contar-se-ão do com encargos;
em que ele sobrevier. III - transigir;
IV - vender-lhe os bens móveis, cuja conservação não
Art. 1.739. Se o juiz não admitir a escusa, exercerá o convier, e os imóveis nos casos em que for permitido;
nomeado a tutela, enquanto o recurso interposto não tiver V - propor em juízo as ações, ou nelas assistir o menor, e
provimento, e responderá desde logo pelas perdas e danos que promover todas as diligências a bem deste, assim como
o menor venha a sofrer. defendê-lo nos pleitos contra ele movidos.
Parágrafo único. No caso de falta de autorização, a eficácia
Seção IV de ato do tutor depende da aprovação ulterior do juiz.
Do Exercício da Tutela
Art. 1.749. Ainda com a autorização judicial, não pode o
Art. 1.740. Incumbe ao tutor, quanto à pessoa do menor: tutor, sob pena de nulidade:
I - dirigir-lhe a educação, defendê-lo e prestar-lhe
alimentos, conforme os seus haveres e condição;

Legislação 53
Apostila Digital Licenciada para Alice Caroline Guarino dos Santos - alice.guarino@hotmail.com (Proibida a Revenda)
APOSTILAS OPÇÃO

I - adquirir por si, ou por interposta pessoa, mediante Art. 1.756. No fim de cada ano de administração, os tutores
contrato particular, bens móveis ou imóveis pertencentes ao submeterão ao juiz o balanço respectivo, que, depois de
menor; aprovado, se anexará aos autos do inventário.
II - dispor dos bens do menor a título gratuito;
III - constituir-se cessionário de crédito ou de direito, Art. 1.757. Os tutores prestarão contas de dois em dois
contra o menor. anos, e também quando, por qualquer motivo, deixarem o
exercício da tutela ou toda vez que o juiz achar conveniente.
Art. 1.750. Os imóveis pertencentes aos menores sob tutela Parágrafo único. As contas serão prestadas em juízo, e
somente podem ser vendidos quando houver manifesta julgadas depois da audiência dos interessados, recolhendo o
vantagem, mediante prévia avaliação judicial e aprovação do tutor imediatamente a estabelecimento bancário oficial os
juiz. saldos, ou adquirindo bens imóveis, ou títulos, obrigações ou
letras, na forma do § 1o do art. 1.753.
Art. 1.751. Antes de assumir a tutela, o tutor declarará tudo
o que o menor lhe deva, sob pena de não lhe poder cobrar, Art. 1.758. Finda a tutela pela emancipação ou maioridade,
enquanto exerça a tutoria, salvo provando que não conhecia o a quitação do menor não produzirá efeito antes de aprovadas
débito quando a assumiu. as contas pelo juiz, subsistindo inteira, até então, a
responsabilidade do tutor.
Art. 1.752. O tutor responde pelos prejuízos que, por culpa,
ou dolo, causar ao tutelado; mas tem direito a ser pago pelo Art. 1.759. Nos casos de morte, ausência, ou interdição do
que realmente despender no exercício da tutela, salvo no caso tutor, as contas serão prestadas por seus herdeiros ou
do art. 1.734, e a perceber remuneração proporcional à representantes.
importância dos bens administrados.
§ 1º Ao protutor será arbitrada uma gratificação módica Art. 1.760. Serão levadas a crédito do tutor todas as
pela fiscalização efetuada. despesas justificadas e reconhecidamente proveitosas ao
§ 2º São solidariamente responsáveis pelos prejuízos as menor.
pessoas às quais competia fiscalizar a atividade do tutor, e as
que concorreram para o dano. Art. 1.761. As despesas com a prestação das contas serão
pagas pelo tutelado.
Seção V
Dos Bens do Tutelado Art. 1.762. O alcance do tutor, bem como o saldo contra o
tutelado, são dívidas de valor e vencem juros desde o
Art. 1.753. Os tutores não podem conservar em seu poder julgamento definitivo das contas.
dinheiro dos tutelados, além do necessário para as despesas
ordinárias com o seu sustento, a sua educação e a Seção VII
administração de seus bens. Da Cessação da Tutela
§ 1º Se houver necessidade, os objetos de ouro e prata,
pedras preciosas e móveis serão avaliados por pessoa idônea Art. 1.763. Cessa a condição de tutelado:
e, após autorização judicial, alienados, e o seu produto I - com a maioridade ou a emancipação do menor;
convertido em títulos, obrigações e letras de responsabilidade II - ao cair o menor sob o poder familiar, no caso de
direta ou indireta da União ou dos Estados, atendendo-se reconhecimento ou adoção.
preferentemente à rentabilidade, e recolhidos ao
estabelecimento bancário oficial ou aplicado na aquisição de Art. 1.764. Cessam as funções do tutor:
imóveis, conforme for determinado pelo juiz. I - ao expirar o termo, em que era obrigado a servir;
§ 2º O mesmo destino previsto no parágrafo antecedente II - ao sobrevir escusa legítima;
terá o dinheiro proveniente de qualquer outra procedência. III - ao ser removido.
§ 3º Os tutores respondem pela demora na aplicação dos
valores acima referidos, pagando os juros legais desde o dia em Art. 1.765. O tutor é obrigado a servir por espaço de dois
que deveriam dar esse destino, o que não os exime da anos.
obrigação, que o juiz fará efetiva, da referida aplicação. Parágrafo único. Pode o tutor continuar no exercício da
tutela, além do prazo previsto neste artigo, se o quiser e o juiz
Art. 1.754. Os valores que existirem em estabelecimento julgar conveniente ao menor.
bancário oficial, na forma do artigo antecedente, não se
poderão retirar, senão mediante ordem do juiz, e somente: Art. 1.766. Será destituído o tutor, quando negligente,
I - para as despesas com o sustento e educação do tutelado, prevaricador ou incurso em incapacidade.
ou a administração de seus bens;
II - para se comprarem bens imóveis e títulos, obrigações CAPÍTULO II
ou letras, nas condições previstas no § 1o do artigo Da Curatela
antecedente; Seção I
III - para se empregarem em conformidade com o disposto Dos Interditos
por quem os houver doado, ou deixado;
IV - para se entregarem aos órfãos, quando emancipados, Art. 1.767. Estão sujeitos a curatela:
ou maiores, ou, mortos eles, aos seus herdeiros. I - aqueles que, por causa transitória ou permanente, não
puderem exprimir sua vontade; (Redação dada pela Lei nº
Seção VI 13.146, de 2015)
Da Prestação de Contas II - (Revogado); (Redação dada pela Lei nº 13.146, de
2015)
Art. 1.755. Os tutores, embora o contrário tivessem III - os ébrios habituais e os viciados em tóxico; (Redação
disposto os pais dos tutelados, são obrigados a prestar contas dada pela Lei nº 13.146, de 2015)
da sua administração. IV - (Revogado); (Redação dada pela Lei nº 13.146, de
2015)

Legislação 54
Apostila Digital Licenciada para Alice Caroline Guarino dos Santos - alice.guarino@hotmail.com (Proibida a Revenda)
APOSTILAS OPÇÃO

V - os pródigos. Art. 1.778. A autoridade do curador estende-se à pessoa e


aos bens dos filhos do curatelado, observado o art. 5o.
Art. 1.768. O processo que define os termos da curatela
deve ser promovido: (Redação dada pela Lei nº 13.146, de Seção II
2015) Da Curatela do Nascituro e do Enfermo ou Portador
I - (Revogado pela Lei n º 13.105, de 2015) de Deficiência Física
II - (Revogado pela Lei n º 13.105, de 2015)
III - (Revogado pela Lei n º 13.105, de 2015) Art. 1.779. Dar-se-á curador ao nascituro, se o pai falecer
IV - (Incluído pela Lei nº 13.146, de 2015) estando grávida a mulher, e não tendo o poder familiar.
Parágrafo único. Se a mulher estiver interdita, seu curador
Art. 1.769. O Ministério Público somente promoverá o será o do nascituro.
processo que define os termos da curatela: (Redação dada pela
Lei nº 13.146, de 2015) Art. 1.780. (Revogado pela Lei nº 13.146, de 2015)
I - nos casos de deficiência mental ou intelectual; (Redação
dada pela Lei nº 13.146, de 2015) Seção III
II - (Revogado pela Lei n º 13.105, de 2015) Do Exercício da Curatela
III - se, existindo, forem menores ou incapazes as pessoas
mencionadas no inciso II. (Redação dada pela Lei nº 13.146, de Art. 1.781. As regras a respeito do exercício da tutela
2015) aplicam-se ao da curatela, com a restrição do art. 1.772 e as
desta Seção.
Art. 1.770. (Revogado pela Lei n º 13.105, de 2015)
Art. 1.782. A interdição do pródigo só o privará de, sem
Art. 1.771. Antes de se pronunciar acerca dos termos da curador, emprestar, transigir, dar quitação, alienar, hipotecar,
curatela, o juiz, que deverá ser assistido por equipe demandar ou ser demandado, e praticar, em geral, os atos que
multidisciplinar, entrevistará pessoalmente o interditando. não sejam de mera administração.
(Redação dada pela Lei nº 13.146, de 2015)
Art. 1.783. Quando o curador for o cônjuge e o regime de
Art. 1.772. O juiz determinará, segundo as potencialidades bens do casamento for de comunhão universal, não será
da pessoa, os limites da curatela, circunscritos às restrições obrigado à prestação de contas, salvo determinação judicial.
constantes do art. 1.782, e indicará curador. (Redação dada
pela Lei nº 13.146, de 2015) Questões
Parágrafo único. Para a escolha do curador, o juiz levará
em conta a vontade e as preferências do interditando, a 01. (TRE/MT - Analista Judiciário – Judiciária – CESPE).
ausência de conflito de interesses e de influência indevida, a A respeito do direito de família, assinale a opção correta à luz
proporcionalidade e a adequação às circunstâncias da pessoa. da jurisprudência do STJ.
(Incluído pela Lei nº 13.146, de 2015) (A) Se a união estável for formalizada por escritura pública,
a fiança prestada por um dos conviventes sem a autorização
Art. 1.773. (Revogado pela Lei n º 13.105, de 2015) do outro é inválida.
(B) Em regra, não se mostra cabível a fixação, por tempo
Art. 1.774. Aplicam-se à curatela as disposições determinado, de pensão alimentícia para ex-cônjuge.
concernentes à tutela, com as modificações dos artigos (C) É incabível o pedido de alimentos decorrente do
seguintes. rompimento de união estável homoafetiva.
(D) A invalidação da venda de imóvel comum fundada na
Art. 1.775. O cônjuge ou companheiro, não separado falta de consentimento do companheiro dependerá da
judicialmente ou de fato, é, de direito, curador do outro, publicidade conferida à união estável ou da demonstração de
quando interdito. má-fé do adquirente.
§1º Na falta do cônjuge ou companheiro, é curador legítimo (E) É suficiente para a caracterização da união estável o
o pai ou a mãe; na falta destes, o descendente que se fato de um casal de namorados que moram juntos projetar
demonstrar mais apto. constituir família no futuro.
§ 2º Entre os descendentes, os mais próximos precedem
aos mais remotos. 02. (: MANAUSPREV -Técnico Previdenciário –
§ 3º Na falta das pessoas mencionadas neste artigo, Administrativa – FCC). De acordo com o ordenamento
compete ao juiz a escolha do curador. jurídico vigente, a equiparação do casamento religioso ao
casamento civil é
Art. 1.775-A. Na nomeação de curador para a pessoa com (A) incabível, salvo por decisão judicial.
deficiência, o juiz poderá estabelecer curatela compartilhada a (B) admitida, desde que atendidas as exigências da lei para
mais de uma pessoa. (Incluído pela Lei nº 13.146, de 2015) validade do casamento civil e registrado no registro próprio.
(C) incabível, dado o caráter laico do Estado.
Art. 1.776. (Revogado pela Lei nº 13.146, de 2015) (D) possível, apenas para evitar o cumprimento de pena
criminal ou no caso de gravidez
Art. 1.777. Os interditos referidos nos incisos I, III e IV do (E) exceção absoluta, apenas sendo admissível na hipótese
art. 1.767 serão recolhidos em estabelecimentos adequados, de um dos nubentes encontrar-se em risco iminente de vida.
quando não se adaptarem ao convívio doméstico.
03. (TJ/PE - Juiz Substituto – FCC). Na habilitação para o
Art. 1.777. As pessoas referidas no inciso I do art. 1.767 casamento, se houver oposição de impedimento, o oficial
receberão todo o apoio necessário para ter preservado o (A) indeferirá o pedido de habilitação e remeterá o
direito à convivência familiar e comunitária, sendo evitado o oponente e os nubentes às vias ordinárias em juízo, para
seu recolhimento em estabelecimento que os afaste desse decisão do magistrado
convívio. (Redação dada pela Lei nº 13.146, de 2015) (B) encaminhará a oposição ao juiz, sem efeito suspensivo
do procedimento, que, depois de regular instrução e

Legislação 55
Apostila Digital Licenciada para Alice Caroline Guarino dos Santos - alice.guarino@hotmail.com (Proibida a Revenda)
APOSTILAS OPÇÃO

manifestação do Ministério Público, decidirá até a data do (C) No caso de falecimento de ambos os cônjuges, a
casamento. administração do bem de família instituído nos termos do
(C) encaminhará os autos, imediatamente, ao juiz, que Código Civil, passará ao filho mais velho, se for maior, e, do
intimará o oponente e os nubentes a indicarem provas, que contrário, a seu tutor.
serão produzidas e, ouvido o Ministério Público, decidirá. (D) A dissolução da sociedade conjugal extingue o bem de
(D) dará ciência do fato aos nubentes para que indiquem família.
provas que desejam produzir, colhendo-as e em seguida (E) Extingue-se o bem de família com a morte de ambos os
remeterá os autos ao juiz que, ouvido o Ministério Público, cônjuges e a maioridade dos filhos, desde que não sujeitos a
decidirá. curatela.
(E) dará ciência do fato aos nubentes, para que indiquem
provas que desejam produzir e remeterá os autos ao juiz que Respostas
decidirá depois da produção das provas pelo oponente e pelos
nubentes, com a participação do Ministério Público. 01. D / 02. B / 03. E / 04. C / 05. A / 06. D / 07. E

04. (SEFAZ/PI - Auditor Fiscal da Fazenda Estadual –


SEFAZ-PI). No regime da comunhão parcial de bens, LEI MARIA DA PENHA - LEI
(A) pode o cônjuge, sem autorização do outro, prestar aval,
porém não fiança.
11.340/2006.
(B) comunicam-se os bens adquiridos na constância do
casamento, ainda que por doação, porém não por sucessão.
LEI Nº 11.340, DE 7 DE AGOSTO DE 2006.
(C) não pode o cônjuge, sem autorização do outro, alienar
bens imóveis, ainda que adquiridos antes do casamento.
Cria mecanismos para coibir a violência doméstica e
(D) comunicam-se os bens adquiridos na constância do
familiar contra a mulher, nos termos do § 8o do art. 226 da
casamento, ainda que por doação ou sucessão.
Constituição Federal, da Convenção sobre a Eliminação de
(E) excluem-se da comunhão os bens adquiridos por fato
Todas as Formas de Discriminação contra as Mulheres e da
eventual, a exemplo dos prêmios de loteria.
Convenção Interamericana para Prevenir, Punir e Erradicar a
Violência contra a Mulher; dispõe sobre a criação dos
05. (TJ/MG - Titular de Serviços de Notas e de Registro
Juizados de Violência Doméstica e Familiar contra a Mulher;
– CONSULPLAN). Sobre o regime da comunhão parcial de
altera o Código de Processo Penal, o Código Penal e a Lei de
bens, é correto afirmar que estão incluídos na comunhão:
Execução Penal; e dá outras providências.
(A) Os bens adquiridos por fato eventual, com ou sem o
concurso de trabalho ou despesa anterior.
O PRESIDENTE DA REPÚBLICA Faço saber que o
(B) As pensões, meios-soldos, montepios e outras rendas
Congresso Nacional decreta e eu sanciono a seguinte Lei:
semelhantes.
(C) As obrigações provenientes de atos ilícitos, salvo
TÍTULO I
reversão em proveito do casal.
DISPOSIÇÕES PRELIMINARES
(D) Os bens adquiridos com valores exclusivamente
pertencentes a um dos cônjuges em sub-rogação dos bens
Art. 1o Esta Lei cria mecanismos para coibir e prevenir a
particulares.
violência doméstica e familiar contra a mulher, nos termos do
§ 8o do art. 226 da Constituição Federal, da Convenção sobre a
06. (IF/RS - Professor – Direito – IF-RS). No regime de
Eliminação de Todas as Formas de Violência contra a Mulher,
comunhão parcial, comunicam- se os bens que sobrevierem ao
da Convenção Interamericana para Prevenir, Punir e Erradicar
casal, na constância do casamento. Excluem-se da comunhão:
a Violência contra a Mulher e de outros tratados internacionais
I. as obrigações anteriores ao casamento.
ratificados pela República Federativa do Brasil; dispõe sobre a
II. os bens que cada cônjuge possuir ao casar, e os que lhe
criação dos Juizados de Violência Doméstica e Familiar contra
sobrevierem, na constância do casamento, por doação ou
a Mulher; e estabelece medidas de assistência e proteção às
sucessão, e os sub- rogados em seu lugar.
mulheres em situação de violência doméstica e familiar.
III. as benfeitorias em bens particulares de cada cônjuge.
IV. os bens adquiridos na constância do casamento por
Art. 2o Toda mulher, independentemente de classe, raça,
título oneroso, ainda que só em nome de um dos cônjuges.
etnia, orientação sexual, renda, cultura, nível educacional,
Assinale a alternativa em que toda(s) a(s) afirmativa(s)
idade e religião, goza dos direitos fundamentais inerentes à
está(ão) CORRETA(S):
pessoa humana, sendo-lhe asseguradas as oportunidades e
(A) Apenas II.
facilidades para viver sem violência, preservar sua saúde física
(B) Apenas III.
e mental e seu aperfeiçoamento moral, intelectual e social.
(C) Apenas III e IV.
(D) Apenas I e II.
Art. 3o Serão asseguradas às mulheres as condições para o
(E) I, II, III e IV.
exercício efetivo dos direitos à vida, à segurança, à saúde, à
alimentação, à educação, à cultura, à moradia, ao acesso à
07. (TRT - 2ª REGIÃO/SP - Juiz do Trabalho Substituto
justiça, ao esporte, ao lazer, ao trabalho, à cidadania, à
– TRT 2R (SP)/2016). Considerando-se o disposto no Código
liberdade, à dignidade, ao respeito e à convivência familiar e
Civil, quanto ao bem de família, é INCORRETO afirmar que:
comunitária.
(A) Comprovada a impossibilidade da manutenção do bem
§ 1o O poder público desenvolverá políticas que visem
de família nas condições em que foi instituído, poderá o juiz, a
garantir os direitos humanos das mulheres no âmbito das
requerimento dos interessados, extingui-lo ou autorizar a sub-
relações domésticas e familiares no sentido de resguardá-las
rogação dos bens que o constituem em outros, ouvidos o
de toda forma de negligência, discriminação, exploração,
instituidor e o Ministério Público.
violência, crueldade e opressão.
(B) Não havendo disposição em contrário do ato de
§ 2o Cabe à família, à sociedade e ao poder público criar as
instituição, a administração do bem de família compete a
condições necessárias para o efetivo exercício dos direitos
ambos os cônjuges, resolvendo o juiz em caso de divergência.
enunciados no caput.

Legislação 56
Apostila Digital Licenciada para Alice Caroline Guarino dos Santos - alice.guarino@hotmail.com (Proibida a Revenda)
APOSTILAS OPÇÃO

Art. 4o Na interpretação desta Lei, serão considerados os TÍTULO III


fins sociais a que ela se destina e, especialmente, as condições DA ASSISTÊNCIA À MULHER EM SITUAÇÃO DE
peculiares das mulheres em situação de violência doméstica e VIOLÊNCIA DOMÉSTICA E FAMILIAR
familiar. CAPÍTULO I
DAS MEDIDAS INTEGRADAS DE PREVENÇÃO
TÍTULO II
DA VIOLÊNCIA DOMÉSTICA E FAMILIAR CONTRA A Art. 8o A política pública que visa coibir a violência
MULHER doméstica e familiar contra a mulher far-se-á por meio de um
CAPÍTULO I conjunto articulado de ações da União, dos Estados, do Distrito
DISPOSIÇÕES GERAIS Federal e dos Municípios e de ações não-governamentais,
tendo por diretrizes:
Art. 5o Para os efeitos desta Lei, configura violência I - a integração operacional do Poder Judiciário, do
doméstica e familiar contra a mulher qualquer ação ou Ministério Público e da Defensoria Pública com as áreas de
omissão baseada no gênero que lhe cause morte, lesão, segurança pública, assistência social, saúde, educação,
sofrimento físico, sexual ou psicológico e dano moral ou trabalho e habitação;
patrimonial: II - a promoção de estudos e pesquisas, estatísticas e outras
I - no âmbito da unidade doméstica, compreendida como o informações relevantes, com a perspectiva de gênero e de raça
espaço de convívio permanente de pessoas, com ou sem ou etnia, concernentes às causas, às consequências e à
vínculo familiar, inclusive as esporadicamente agregadas; frequência da violência doméstica e familiar contra a mulher,
II - no âmbito da família, compreendida como a para a sistematização de dados, a serem unificados
comunidade formada por indivíduos que são ou se consideram nacionalmente, e a avaliação periódica dos resultados das
aparentados, unidos por laços naturais, por afinidade ou por medidas adotadas;
vontade expressa; III - o respeito, nos meios de comunicação social, dos
III - em qualquer relação íntima de afeto, na qual o agressor valores éticos e sociais da pessoa e da família, de forma a coibir
conviva ou tenha convivido com a ofendida, os papéis estereotipados que legitimem ou exacerbem a
independentemente de coabitação. violência doméstica e familiar, de acordo com o estabelecido
Parágrafo único. As relações pessoais enunciadas neste no inciso III do art. 1o, no inciso IV do art. 3o e no inciso IV do
artigo independem de orientação sexual. art. 221 da Constituição Federal;
IV - a implementação de atendimento policial
Art. 6o A violência doméstica e familiar contra a mulher especializado para as mulheres, em particular nas Delegacias
constitui uma das formas de violação dos direitos humanos. de Atendimento à Mulher;
V - a promoção e a realização de campanhas educativas de
CAPÍTULO II prevenção da violência doméstica e familiar contra a mulher,
DAS FORMAS DE VIOLÊNCIA DOMÉSTICA E FAMILIAR voltadas ao público escolar e à sociedade em geral, e a difusão
CONTRA A MULHER desta Lei e dos instrumentos de proteção aos direitos humanos
das mulheres;
Art. 7o São formas de violência doméstica e familiar contra VI - a celebração de convênios, protocolos, ajustes, termos
a mulher, entre outras: ou outros instrumentos de promoção de parceria entre órgãos
I - a violência física, entendida como qualquer conduta que governamentais ou entre estes e entidades não-
ofenda sua integridade ou saúde corporal; governamentais, tendo por objetivo a implementação de
II - a violência psicológica, entendida como qualquer programas de erradicação da violência doméstica e familiar
conduta que lhe cause dano emocional e diminuição da contra a mulher;
autoestima ou que lhe prejudique e perturbe o pleno VII - a capacitação permanente das Polícias Civil e Militar,
desenvolvimento ou que vise degradar ou controlar suas da Guarda Municipal, do Corpo de Bombeiros e dos
ações, comportamentos, crenças e decisões, mediante ameaça, profissionais pertencentes aos órgãos e às áreas enunciados
constrangimento, humilhação, manipulação, isolamento, no inciso I quanto às questões de gênero e de raça ou etnia;
vigilância constante, perseguição contumaz, insulto, VIII - a promoção de programas educacionais que
chantagem, ridicularização, exploração e limitação do direito disseminem valores éticos de irrestrito respeito à dignidade
de ir e vir ou qualquer outro meio que lhe cause prejuízo à da pessoa humana com a perspectiva de gênero e de raça ou
saúde psicológica e à autodeterminação; etnia;
III - a violência sexual, entendida como qualquer conduta IX - o destaque, nos currículos escolares de todos os níveis
que a constranja a presenciar, a manter ou a participar de de ensino, para os conteúdos relativos aos direitos humanos, à
relação sexual não desejada, mediante intimidação, ameaça, equidade de gênero e de raça ou etnia e ao problema da
coação ou uso da força; que a induza a comercializar ou a violência doméstica e familiar contra a mulher.
utilizar, de qualquer modo, a sua sexualidade, que a impeça de
usar qualquer método contraceptivo ou que a force ao CAPÍTULO II
matrimônio, à gravidez, ao aborto ou à prostituição, mediante DA ASSISTÊNCIA À MULHER EM SITUAÇÃO DE
coação, chantagem, suborno ou manipulação; ou que limite ou VIOLÊNCIA DOMÉSTICA E FAMILIAR
anule o exercício de seus direitos sexuais e reprodutivos;
IV - a violência patrimonial, entendida como qualquer Art. 9o A assistência à mulher em situação de violência
conduta que configure retenção, subtração, destruição parcial doméstica e familiar será prestada de forma articulada e
ou total de seus objetos, instrumentos de trabalho, conforme os princípios e as diretrizes previstos na Lei
documentos pessoais, bens, valores e direitos ou recursos Orgânica da Assistência Social, no Sistema Único de Saúde, no
econômicos, incluindo os destinados a satisfazer suas Sistema Único de Segurança Pública, entre outras normas e
necessidades; políticas públicas de proteção, e emergencialmente quando for
V - a violência moral, entendida como qualquer conduta o caso.
que configure calúnia, difamação ou injúria. § 1o O juiz determinará, por prazo certo, a inclusão da
mulher em situação de violência doméstica e familiar no
cadastro de programas assistenciais do governo federal,
estadual e municipal.

Legislação 57
Apostila Digital Licenciada para Alice Caroline Guarino dos Santos - alice.guarino@hotmail.com (Proibida a Revenda)
APOSTILAS OPÇÃO

§ 2o O juiz assegurará à mulher em situação de violência § 2o A autoridade policial deverá anexar ao documento
doméstica e familiar, para preservar sua integridade física e referido no § 1o o boletim de ocorrência e cópia de todos os
psicológica: documentos disponíveis em posse da ofendida.
I - acesso prioritário à remoção quando servidora pública, § 3o Serão admitidos como meios de prova os laudos ou
integrante da administração direta ou indireta; prontuários médicos fornecidos por hospitais e postos de
II - manutenção do vínculo trabalhista, quando necessário saúde.
o afastamento do local de trabalho, por até seis meses.
§ 3o A assistência à mulher em situação de violência TÍTULO IV
doméstica e familiar compreenderá o acesso aos benefícios DOS PROCEDIMENTOS
decorrentes do desenvolvimento científico e tecnológico, CAPÍTULO I
incluindo os serviços de contracepção de emergência, a DISPOSIÇÕES GERAIS
profilaxia das Doenças Sexualmente Transmissíveis (DST) e da
Síndrome da Imunodeficiência Adquirida (AIDS) e outros Art. 13. Ao processo, ao julgamento e à execução das
procedimentos médicos necessários e cabíveis nos casos de causas cíveis e criminais decorrentes da prática de violência
violência sexual. doméstica e familiar contra a mulher aplicar-se-ão as normas
dos Códigos de Processo Penal e Processo Civil e da legislação
CAPÍTULO III específica relativa à criança, ao adolescente e ao idoso que não
DO ATENDIMENTO PELA AUTORIDADE POLICIAL conflitarem com o estabelecido nesta Lei.

Art. 10. Na hipótese da iminência ou da prática de violência Art. 14. Os Juizados de Violência Doméstica e Familiar
doméstica e familiar contra a mulher, a autoridade policial que contra a Mulher, órgãos da Justiça Ordinária com competência
tomar conhecimento da ocorrência adotará, de imediato, as cível e criminal, poderão ser criados pela União, no Distrito
providências legais cabíveis. Federal e nos Territórios, e pelos Estados, para o processo, o
Parágrafo único. Aplica-se o disposto no caput deste artigo julgamento e a execução das causas decorrentes da prática de
ao descumprimento de medida protetiva de urgência deferida. violência doméstica e familiar contra a mulher.
Parágrafo único. Os atos processuais poderão realizar-se
Art. 11. No atendimento à mulher em situação de violência em horário noturno, conforme dispuserem as normas de
doméstica e familiar, a autoridade policial deverá, entre outras organização judiciária.
providências:
I - garantir proteção policial, quando necessário, Art. 15. É competente, por opção da ofendida, para os
comunicando de imediato ao Ministério Público e ao Poder processos cíveis regidos por esta Lei, o Juizado:
Judiciário; I - do seu domicílio ou de sua residência;
II - encaminhar a ofendida ao hospital ou posto de saúde e II - do lugar do fato em que se baseou a demanda;
ao Instituto Médico Legal; III - do domicílio do agressor.
III - fornecer transporte para a ofendida e seus
dependentes para abrigo ou local seguro, quando houver risco Art. 16. Nas ações penais públicas condicionadas à
de vida; representação da ofendida de que trata esta Lei, só será
IV - se necessário, acompanhar a ofendida para assegurar admitida a renúncia à representação perante o juiz, em
a retirada de seus pertences do local da ocorrência ou do audiência especialmente designada com tal finalidade, antes
domicílio familiar; do recebimento da denúncia e ouvido o Ministério Público.
V - informar à ofendida os direitos a ela conferidos nesta
Lei e os serviços disponíveis. Art. 17. É vedada a aplicação, nos casos de violência
doméstica e familiar contra a mulher, de penas de cesta básica
Art. 12. Em todos os casos de violência doméstica e ou outras de prestação pecuniária, bem como a substituição de
familiar contra a mulher, feito o registro da ocorrência, deverá pena que implique o pagamento isolado de multa.
a autoridade policial adotar, de imediato, os seguintes
procedimentos, sem prejuízo daqueles previstos no Código de CAPÍTULO II
Processo Penal: DAS MEDIDAS PROTETIVAS DE URGÊNCIA
I - ouvir a ofendida, lavrar o boletim de ocorrência e tomar Seção I
a representação a termo, se apresentada; Disposições Gerais
II - colher todas as provas que servirem para o
esclarecimento do fato e de suas circunstâncias; Art. 18. Recebido o expediente com o pedido da ofendida,
III - remeter, no prazo de 48 (quarenta e oito) horas, caberá ao juiz, no prazo de 48 (quarenta e oito) horas:
expediente apartado ao juiz com o pedido da ofendida, para a I - conhecer do expediente e do pedido e decidir sobre as
concessão de medidas protetivas de urgência; medidas protetivas de urgência;
IV - determinar que se proceda ao exame de corpo de delito II - determinar o encaminhamento da ofendida ao órgão de
da ofendida e requisitar outros exames periciais necessários; assistência judiciária, quando for o caso;
V - ouvir o agressor e as testemunhas; III - comunicar ao Ministério Público para que adote as
VI - ordenar a identificação do agressor e fazer juntar aos providências cabíveis.
autos sua folha de antecedentes criminais, indicando a
existência de mandado de prisão ou registro de outras Art. 19. As medidas protetivas de urgência poderão ser
ocorrências policiais contra ele; concedidas pelo juiz, a requerimento do Ministério Público ou
VII - remeter, no prazo legal, os autos do inquérito policial a pedido da ofendida.
ao juiz e ao Ministério Público. § 1o As medidas protetivas de urgência poderão ser
§ 1o O pedido da ofendida será tomado a termo pela concedidas de imediato, independentemente de audiência das
autoridade policial e deverá conter: partes e de manifestação do Ministério Público, devendo este
I - qualificação da ofendida e do agressor; ser prontamente comunicado.
II - nome e idade dos dependentes; § 2o As medidas protetivas de urgência serão aplicadas
III - descrição sucinta do fato e das medidas protetivas isolada ou cumulativamente, e poderão ser substituídas a
solicitadas pela ofendida.

Legislação 58
Apostila Digital Licenciada para Alice Caroline Guarino dos Santos - alice.guarino@hotmail.com (Proibida a Revenda)
APOSTILAS OPÇÃO

qualquer tempo por outras de maior eficácia, sempre que os Seção III
direitos reconhecidos nesta Lei forem ameaçados ou violados. Das Medidas Protetivas de Urgência à Ofendida
§ 3o Poderá o juiz, a requerimento do Ministério Público
ou a pedido da ofendida, conceder novas medidas protetivas Art. 23. Poderá o juiz, quando necessário, sem prejuízo de
de urgência ou rever aquelas já concedidas, se entender outras medidas:
necessário à proteção da ofendida, de seus familiares e de seu I - encaminhar a ofendida e seus dependentes a programa
patrimônio, ouvido o Ministério Público. oficial ou comunitário de proteção ou de atendimento;
II - determinar a recondução da ofendida e a de seus
Art. 20. Em qualquer fase do inquérito policial ou da dependentes ao respectivo domicílio, após afastamento do
instrução criminal, caberá a prisão preventiva do agressor, agressor;
decretada pelo juiz, de ofício, a requerimento do Ministério III - determinar o afastamento da ofendida do lar, sem
Público ou mediante representação da autoridade policial. prejuízo dos direitos relativos a bens, guarda dos filhos e
Parágrafo único. O juiz poderá revogar a prisão preventiva alimentos;
se, no curso do processo, verificar a falta de motivo para que IV - determinar a separação de corpos.
subsista, bem como de novo decretá-la, se sobrevierem razões
que a justifiquem. Art. 24. Para a proteção patrimonial dos bens da sociedade
conjugal ou daqueles de propriedade particular da mulher, o
Art. 21. A ofendida deverá ser notificada dos atos juiz poderá determinar, liminarmente, as seguintes medidas,
processuais relativos ao agressor, especialmente dos entre outras:
pertinentes ao ingresso e à saída da prisão, sem prejuízo da I - restituição de bens indevidamente subtraídos pelo
intimação do advogado constituído ou do defensor público. agressor à ofendida;
Parágrafo único. A ofendida não poderá entregar II - proibição temporária para a celebração de atos e
intimação ou notificação ao agressor. contratos de compra, venda e locação de propriedade em
comum, salvo expressa autorização judicial;
Seção II III - suspensão das procurações conferidas pela ofendida
Das Medidas Protetivas de Urgência que Obrigam o ao agressor;
Agressor IV - prestação de caução provisória, mediante depósito
judicial, por perdas e danos materiais decorrentes da prática
Art. 22. Constatada a prática de violência doméstica e de violência doméstica e familiar contra a ofendida.
familiar contra a mulher, nos termos desta Lei, o juiz poderá Parágrafo único. Deverá o juiz oficiar ao cartório
aplicar, de imediato, ao agressor, em conjunto ou competente para os fins previstos nos incisos II e III deste
separadamente, as seguintes medidas protetivas de urgência, artigo.
entre outras:
I - suspensão da posse ou restrição do porte de armas, com CAPÍTULO III
comunicação ao órgão competente, nos termos da Lei DA ATUAÇÃO DO MINISTÉRIO PÚBLICO
no 10.826, de 22 de dezembro de 2003;
II - afastamento do lar, domicílio ou local de convivência Art. 25. O Ministério Público intervirá, quando não for
com a ofendida; parte, nas causas cíveis e criminais decorrentes da violência
III - proibição de determinadas condutas, entre as quais: doméstica e familiar contra a mulher.
a) aproximação da ofendida, de seus familiares e das
testemunhas, fixando o limite mínimo de distância entre estes Art. 26. Caberá ao Ministério Público, sem prejuízo de
e o agressor; outras atribuições, nos casos de violência doméstica e familiar
b) contato com a ofendida, seus familiares e testemunhas contra a mulher, quando necessário:
por qualquer meio de comunicação; I - requisitar força policial e serviços públicos de saúde, de
c) frequentação de determinados lugares a fim de educação, de assistência social e de segurança, entre outros;
preservar a integridade física e psicológica da ofendida; II - fiscalizar os estabelecimentos públicos e particulares
IV - restrição ou suspensão de visitas aos dependentes de atendimento à mulher em situação de violência doméstica
menores, ouvida a equipe de atendimento multidisciplinar ou e familiar, e adotar, de imediato, as medidas administrativas
serviço similar; ou judiciais cabíveis no tocante a quaisquer irregularidades
V - prestação de alimentos provisionais ou provisórios. constatadas;
§ 1o As medidas referidas neste artigo não impedem a III - cadastrar os casos de violência doméstica e familiar
aplicação de outras previstas na legislação em vigor, sempre contra a mulher.
que a segurança da ofendida ou as circunstâncias o exigirem,
devendo a providência ser comunicada ao Ministério Público. CAPÍTULO IV
§ 2o Na hipótese de aplicação do inciso I, encontrando-se o DA ASSISTÊNCIA JUDICIÁRIA
agressor nas condições mencionadas no caput e incisos do art.
6o da Lei no 10.826, de 22 de dezembro de 2003, o juiz Art. 27. Em todos os atos processuais, cíveis e criminais, a
comunicará ao respectivo órgão, corporação ou instituição as mulher em situação de violência doméstica e familiar deverá
medidas protetivas de urgência concedidas e determinará a estar acompanhada de advogado, ressalvado o previsto no art.
restrição do porte de armas, ficando o superior imediato do 19 desta Lei.
agressor responsável pelo cumprimento da determinação
judicial, sob pena de incorrer nos crimes de prevaricação ou de Art. 28. É garantido a toda mulher em situação de violência
desobediência, conforme o caso. doméstica e familiar o acesso aos serviços de Defensoria
§ 3o Para garantir a efetividade das medidas protetivas de Pública ou de Assistência Judiciária Gratuita, nos termos da lei,
urgência, poderá o juiz requisitar, a qualquer momento, auxílio em sede policial e judicial, mediante atendimento específico e
da força policial. humanizado.
§ 4o Aplica-se às hipóteses previstas neste artigo, no que
couber, o disposto no caput e nos §§ 5o e 6º do art. 461 da Lei
no 5.869, de 11 de janeiro de 1973 (Código de Processo Civil).

Legislação 59
Apostila Digital Licenciada para Alice Caroline Guarino dos Santos - alice.guarino@hotmail.com (Proibida a Revenda)
APOSTILAS OPÇÃO

TÍTULO V Art. 36. A União, os Estados, o Distrito Federal e os


DA EQUIPE DE ATENDIMENTO MULTIDISCIPLINAR Municípios promoverão a adaptação de seus órgãos e de seus
programas às diretrizes e aos princípios desta Lei.
Art. 29. Os Juizados de Violência Doméstica e Familiar
contra a Mulher que vierem a ser criados poderão contar com Art. 37. A defesa dos interesses e direitos transindividuais
uma equipe de atendimento multidisciplinar, a ser integrada previstos nesta Lei poderá ser exercida, concorrentemente,
por profissionais especializados nas áreas psicossocial, pelo Ministério Público e por associação de atuação na área,
jurídica e de saúde. regularmente constituída há pelo menos um ano, nos termos
da legislação civil.
Art. 30. Compete à equipe de atendimento Parágrafo único. O requisito da pré-constituição poderá
multidisciplinar, entre outras atribuições que lhe forem ser dispensado pelo juiz quando entender que não há outra
reservadas pela legislação local, fornecer subsídios por escrito entidade com representatividade adequada para o
ao juiz, ao Ministério Público e à Defensoria Pública, mediante ajuizamento da demanda coletiva.
laudos ou verbalmente em audiência, e desenvolver trabalhos
de orientação, encaminhamento, prevenção e outras medidas, Art. 38. As estatísticas sobre a violência doméstica e
voltados para a ofendida, o agressor e os familiares, com familiar contra a mulher serão incluídas nas bases de dados
especial atenção às crianças e aos adolescentes. dos órgãos oficiais do Sistema de Justiça e Segurança a fim de
subsidiar o sistema nacional de dados e informações relativo
Art. 31. Quando a complexidade do caso exigir avaliação às mulheres.
mais aprofundada, o juiz poderá determinar a manifestação de Parágrafo único. As Secretarias de Segurança Pública dos
profissional especializado, mediante a indicação da equipe de Estados e do Distrito Federal poderão remeter suas
atendimento multidisciplinar. informações criminais para a base de dados do Ministério da
Justiça.
Art. 32. O Poder Judiciário, na elaboração de sua proposta
orçamentária, poderá prever recursos para a criação e Art. 39. A União, os Estados, o Distrito Federal e os
manutenção da equipe de atendimento multidisciplinar, nos Municípios, no limite de suas competências e nos termos das
termos da Lei de Diretrizes Orçamentárias. respectivas leis de diretrizes orçamentárias, poderão
estabelecer dotações orçamentárias específicas, em cada
TÍTULO VI exercício financeiro, para a implementação das medidas
DISPOSIÇÕES TRANSITÓRIAS estabelecidas nesta Lei.

Art. 33. Enquanto não estruturados os Juizados de Art. 40. As obrigações previstas nesta Lei não excluem
Violência Doméstica e Familiar contra a Mulher, as varas outras decorrentes dos princípios por ela adotados.
criminais acumularão as competências cível e criminal para
conhecer e julgar as causas decorrentes da prática de violência Art. 41. Aos crimes praticados com violência doméstica e
doméstica e familiar contra a mulher, observadas as previsões familiar contra a mulher, independentemente da pena
do Título IV desta Lei, subsidiada pela legislação processual prevista, não se aplica a Lei no 9.099, de 26 de setembro de
pertinente. 1995.
Parágrafo único. Será garantido o direito de preferência,
nas varas criminais, para o processo e o julgamento das causas Art. 42. O art. 313 do Decreto-Lei no 3.689, de 3 de outubro
referidas no caput. de 1941 (Código de Processo Penal), passa a vigorar acrescido
do seguinte inciso IV:
TÍTULO VII “Art. 313. ...
DISPOSIÇÕES FINAIS ...
IV - se o crime envolver violência doméstica e familiar
Art. 34. A instituição dos Juizados de Violência Doméstica contra a mulher, nos termos da lei específica, para garantir a
e Familiar contra a Mulher poderá ser acompanhada pela execução das medidas protetivas de urgência.” (NR)
implantação das curadorias necessárias e do serviço de
assistência judiciária. Art. 43. A alínea f do inciso II do art. 61 do Decreto-Lei
no 2.848, de 7 de dezembro de 1940 (Código Penal), passa a
Art. 35. A União, o Distrito Federal, os Estados e os vigorar com a seguinte redação:
Municípios poderão criar e promover, no limite das “Art. 61. ...
respectivas competências: ...
I - centros de atendimento integral e multidisciplinar para II - ...
mulheres e respectivos dependentes em situação de violência ...
doméstica e familiar; f) com abuso de autoridade ou prevalecendo-se de relações
II - casas-abrigos para mulheres e respectivos dependentes domésticas, de coabitação ou de hospitalidade, ou com
menores em situação de violência doméstica e familiar; violência contra a mulher na forma da lei específica;
III - delegacias, núcleos de defensoria pública, serviços de
saúde e centros de perícia médico-legal especializados no Art. 44. O art. 129 do Decreto-Lei nº 2.848, de 7 de
atendimento à mulher em situação de violência doméstica e dezembro de 1940 (Código Penal), passa a vigorar com as
familiar; seguintes alterações:
IV - programas e campanhas de enfrentamento da “Art. 129. ...
violência doméstica e familiar; § 9o Se a lesão for praticada contra ascendente,
V - centros de educação e de reabilitação para os descendente, irmão, cônjuge ou companheiro, ou com quem
agressores. conviva ou tenha convivido, ou, ainda, prevalecendo-se o
agente das relações domésticas, de coabitação ou de
hospitalidade:
Pena - detenção, de 3 (três) meses a 3 (três) anos.
...

Legislação 60
Apostila Digital Licenciada para Alice Caroline Guarino dos Santos - alice.guarino@hotmail.com (Proibida a Revenda)
APOSTILAS OPÇÃO

§ 11. Na hipótese do § 9o deste artigo, a pena será (C) Violência sexual.


aumentada de um terço se o crime for cometido contra pessoa (D) Violência patrimonial.
portadora de deficiência.” (NR) (E) Violência moral.

Art. 45. O art. 152 da Lei no 7.210, de 11 de julho de 1984 04. (SAEB/BA - Técnico de Registro de Comércio -
(Lei de Execução Penal), passa a vigorar com a seguinte IBFC/2015) Considere as disposições da Lei Federal n°
redação: 11.340, de 7 de agosto de 2006 (Lei Maria da Penha) e assinale
“Art. 152. ... a alternativa correta sobre o rol exato das áreas que, segundo
Parágrafo único. Nos casos de violência doméstica contra a referida lei, são indicadas para integrar as equipes de
a mulher, o juiz poderá determinar o comparecimento atendimento multidisciplinar ligadas aos Juizados de Violência
obrigatório do agressor a programas de recuperação e Doméstica e Familiar contra a Mulher que vierem a ser criados.
reeducação.” (NR) (A) Jurídica e de Saúde.
(B) Religiosa e jurídica.
Art. 46. Esta Lei entra em vigor 45 (quarenta e cinco) dias (C) Psicossocial, jurídica e de saúde.
após sua publicação. (D) Religiosa, jurídica e psicossocial.
(E) Psicossocial, religiosa e de saúde.
Questões
05. (PC/CE - Inspetor de Polícia Civil de 1ª Classe -
01. (TJ/BA - Analista Judiciário – Psicologia - VUNESP/2015) A Lei nº 11.340/2006, conhecida como “Lei
FGV/2015) A Lei nº 11.340/2006 (Lei Maria da Penha) Maria da Penha”, estabelece que
configura como violência doméstica e familiar contra a (A) em qualquer fase do inquérito policial ou da ação penal
mulher: cabe prisão preventiva contra o agressor.
(A) qualquer ação que lhe cause morte, lesão, sofrimento (B) em nenhuma hipótese o sujeito ativo dos crimes
físico, sexual ou psicológico e danos morais; previstos nessa Lei poderá ser uma mulher.
(B) qualquer omissão que lhe cause morte, lesão, (C) para efeito dessa Lei são formas de violência a física e a
sofrimento sexual ou psicológico e danos morais; sexual.
(C) qualquer ação e omissão que lhe cause morte, lesão, (D) na ocorrência de uma briga (com agressão física) entre
sofrimento sexual e danos morais; vizinhos, desde que envolvendo um homem e uma mulher e
(D) qualquer ação ou omissão, independentemente da com residências próximas, aplicam-se as disposições da Lei
relação de gênero, que lhe cause morte, lesão, sofrimento Maria da Penha.
sexual e dano patrimonial ou moral; (E) se o agressor comprovar que é o proprietário da
(E) qualquer ação ou omissão baseada no gênero que lhe residência conjugal, não poderá ser determinado o seu
cause morte, lesão, sofrimento físico, sexual ou psicológico e afastamento do lar.
dano moral ou patrimonial.
06. (MPE/PA - Promotor de Justiça - FCC/2014) Em
02. (PC/AC - Perito Criminal - FUNCAB/2015) Com base relação às medidas protetivas de urgência previstas na Lei nº
na Lei n° 11.340/2006 (Lei Maria da Penha), assinale a 11.340/06, conhecida como Lei Maria da Penha, é correto
alternativa correta. afirmar:
(A) Na hipótese da prática de violência doméstica contra a (A) O juiz pode aplicar o afastamento do agressor, mas não
mulher, somente a autoridade judiciária poderá autorizar o da ofendida, do lar.
encaminhamento da ofendida ao Instituto Médico Legal, para (B) A suspensão do porte de arma é medida protetiva de
realização do regular exame de corpo de delito. urgência que o juiz pode aplicar contra o agressor caso
(B) A competência para o processo e julgamento dos constatada a prática de violência doméstica e familiar contra a
delitos decorrentes de violência doméstica é determinada mulher.
exclusivamente pelo domicílio ou pela residência da ofendida (C) As medidas protetivas de urgência poderão ser
por economia processual e objetivando facilitar a prática dos concedidas de imediato, desde que ouvido previamente o
atos processuais. Ministério Público.
(C) Após registrar a ocorrência de violência doméstica e (D) A própria ofendida poderá entregar intimação ou
familiar em uma Unidade de Polícia Judiciária e, em notificação ao agressor.
consequência, ter sido instaurado inquérito policial, a vítima, (E) A prestação de alimentos provisionais ou provisórios
desejando impedir o prosseguimento da investigação criminal, não está no rol de medidas protetivas de urgência que o juiz
deve manifestar expressamente o seu desejo de renúncia pode aplicar contra o agressor caso constatada a prática de
diretamente à autoridade policial. violência doméstica e familiar contra a mulher.
(D) Para concessão de medida protetiva de urgência
prevista na Lei Maria da Penha, o juiz deverá colher prévia 07. (PC/SP - Desenhista Técnico-Pericial -
manifestação do Ministério Público, sob pena de nulidade VUNESP/2014) À luz da Lei n.º 11.340/2006 – Lei Maria da
absoluta do ato. Penha, é correto afirmar que
(E) Em todos os atos processuais, cíveis e criminais, a (A) a violência doméstica e familiar contra a mulher
mulher em situação de violência doméstica e familiar deverá constitui uma das formas de violação dos direitos humanos.
estar acompanhada de advogado, contudo, poderá, (B) tal norma não é aplicável aos crimes praticados com
excepcionalmente, sem assistência de advogado, pedir ao juiz violência doméstica e familiar contra crianças e adolescentes
a concessão de medida protetiva de urgência. de sexo feminino.
(C) não caracteriza violência moral a conduta que
03. (SAEB/BA - Técnico de Registro de Comércio - configure calúnia, difamação ou injúria contra a mulher.
IBFC/2015) Assinale a alternativa correta sobre o que a Lei (D) é permitida a aplicação, nos casos de violência
Federal n° 11.340, de 7 de agosto de 2006 (Lei Maria da Penha) doméstica e familiar contra a mulher, de penas de cesta básica
estabelece como qualquer conduta que configure destruição ou outras de prestação pecuniária, bem como a substituição de
parcial ou total de objetos da mulher pelo seu agressor. pena que implique o pagamento isolado de multa.
(A) Violência física.
(B) Violência psicológica.

Legislação 61
Apostila Digital Licenciada para Alice Caroline Guarino dos Santos - alice.guarino@hotmail.com (Proibida a Revenda)
APOSTILAS OPÇÃO

(E) aplica-se a Lei n.º 9.099/1995 – Juizados Especiais (A) Fulano, pela sua conduta, poderá ser submetido à pena
Cíveis e Criminais – aos crimes praticados com violência de pagamento de cestas básicas em favor de entidades
doméstica e familiar contra a mulher. assistenciais.
(B) Fulano não se sujeitará às penas da Lei Maria da Penha,
08. (OAB - Exame de Ordem Unificado - XIII - Primeira pois a sua conduta ocorreu apenas dentro do ambiente
Fase - FGV/2014) Fernanda, durante uma discussão com seu familiar.
marido Renato, levou vários socos e chutes. Inconformada com (C) Fulano estará sujeito à prisão preventiva, a ser
a agressão, dirigiu-se à Delegacia de Polícia mais próxima e decretada pelo juiz, de ofício, a requerimento do Ministério
narrou todo o ocorrido. Após a realização do exame de corpo Público ou mediante representação da autoridade policial.
de delito, foi constatada a prática de lesão corporal leve por (D) Fulano não poderá ser processado pela Lei Maria da
parte de Renato. O Delegado de Polícia registrou a ocorrência Penha, tendo em vista que esta se destina a proteger a mulher
e requereu as medidas cautelares constantes no Artigo 23 da contra agressões físicas, psicológicas ou morais, mas não
Lei nº 11.340/2006. Após alguns dias e com objetivo de patrimoniais.
reconciliação com o marido, Fernanda foi novamente à (E) Ciclana terá direito a obter medida judicial protetiva de
Delegacia de Polícia requerendo a cessação das investigações urgência contra Fulano, podendo entregar pessoalmente a
para que não fosse ajuizada a ação penal respectiva. Diante do intimação da respectiva medida ao seu marido.
caso narrado, de acordo com o recente entendimento do
Supremo Tribunal Federal, assinale a afirmativa correta. 11. (PC-GO - Delegado de Polícia Substituto –
(A) No âmbito da Lei Maria da Penha, nos crimes de lesão CESPE/2017) À luz do posicionamento jurisprudencial e
corporal leve, a ação penal é condicionada à representação. doutrinário dominantes acerca das disposições da Lei n.º
Desta forma, é possível a sua retratação, pois não houve o 11.340/2006 (Lei Maria da Penha), assinale a opção correta.
oferecimento da denúncia. (A) Caracteriza o crime de desobediência o reiterado
(B) No âmbito da Lei Maria da Penha, nos crimes de lesão descumprimento, pelo agressor, de medida protetiva
corporal leve, a ação penal é pública incondicionada, sendo decretada no âmbito das disposições da Lei Maria da Penha.
impossível interromper as investigações e obstar o (B) Em se tratando dos crimes de lesão corporal leve e
prosseguimento da ação penal. ameaça, pode o Ministério Público dar início a ação penal sem
(C) No âmbito da Lei Maria da Penha, nos crimes de lesão necessidade de representação da vítima de violência
corporal leve, a ação penal é pública incondicionada, mas é doméstica.
possível a retratação da representação antes do oferecimento (C) No caso de condenação à pena de detenção em regime
da denúncia. aberto pela prática do crime de ameaça no âmbito doméstico
(D) No âmbito da Lei Maria da Penha, nos crimes de lesão e familiar, é possível a substituição da pena pelo pagamento
corporal leve, a ação penal é pública condicionada à isolado de multa.
representação, mas como os fatos já foram levados ao (D) No âmbito de aplicação da referida lei, as medidas
conhecimento da autoridade policial será impossível impedir protetivas de urgência poderão ser concedidas
o prosseguimento das investigações e o ajuizamento da ação independentemente de audiência das partes e de manifestação
penal. do Ministério Público, o qual deverá ser prontamente
comunicado.
09. (PC/SP - Escrivão de Polícia Civil - VUNESP) Assinale (E) Afasta-se a incidência da Lei Maria da Penha na
a alternativa que está de acordo com o disposto na Lei Maria violência havida em relações homoafetivas se o sujeito ativo é
da Penha (Lei n.º 11.340/2006). uma mulher.
(A) Em qualquer fase do inquérito policial ou da instrução
criminal, caberá a prisão preventiva do agressor, a ser 12. (AGERBA - Técnico em Regulação – IBFC/2017)
decretada pela autoridade policial competente, desde que esta Assinale a alternativa INCORRETA considerando as
entenda urgente e indispensável a sua aplicação. disposições da Lei Federal nº 11.340, de 7 de agosto de 2006
(B) Nos casos de violência doméstica e familiar contra a (Lei Maria da Penha), sobre a assistência à mulher em situação
mulher, poderão ser aplicadas ao réu as penas de detenção, de violência doméstica e familiar.
reclusão, de pagamento de cesta básica ou outras de prestação (A) A assistência à mulher em situação de violência
pecuniária, bem como a imposição de multa. doméstica e familiar será prestada de forma articulada e
(C) Constatada a prática de violência doméstica e familiar conforme os princípios e as diretrizes previstos na Lei
contra a mulher, poderá ser aplicada ao agressor, entre outras, Orgânica da Assistência Social, no Sistema Único de Saúde, no
a medida protetiva de urgência de afastamento do lar, Sistema Único de Segurança Pública, entre outras normas e
domicílio ou local de convivência com a ofendida, podendo a políticas públicas de proteção, e emergencialmente quando for
intimação ser entregue pela ofendida diretamente ao agressor. o caso
(D) No atendimento à mulher em situação de violência (B) O juiz determinará, por prazo incerto, a inclusão da
doméstica e familiar, a autoridade policial deverá, entre outras mulher em situação de violência doméstica e familiar no
providências, conceder-lhe as medidas protetivas de urgência cadastro de programas assistenciais do governo federal,
cabíveis no caso. estadual e municipal
(E) O juiz assegurará à mulher em situação de violência (C) O juiz assegurará à mulher em situação de violência
doméstica e familiar, para preservar sua integridade física e doméstica e familiar, para preservar sua integridade física e
psicológica, a manutenção do vínculo trabalhista, quando psicológica, acesso prioritário à remoção quando servidora
necessário o afastamento do local de trabalho, por até seis pública, integrante da administração direta ou indireta
meses. (D) O juiz assegurará à mulher em situação de violência
doméstica e familiar, para preservar sua integridade física e
10. (PC/SP - Investigador de Polícia - VUNESP) Fulano, psicológica manutenção do vínculo trabalhista, quando
casado com Ciclana, num momento de discussão no lar, necessário o afastamento do local de trabalho, por até seis
destruiu parte dos instrumentos de trabalho de sua esposa. meses
Considerando a conduta de Fulano em face do disposto na Lei (E) A assistência à mulher em situação de violência
Maria da Penha, pode-se afirmar que doméstica e familiar compreenderá o acesso aos benefícios
decorrentes do desenvolvimento científco e tecnológico,
incluindo os serviços de contracepção de emergência, a

Legislação 62
Apostila Digital Licenciada para Alice Caroline Guarino dos Santos - alice.guarino@hotmail.com (Proibida a Revenda)
APOSTILAS OPÇÃO

proflaxia das Doenças Sexualmente Transmissíveis (DST) e da Sendo acolhida a alegação, o tribunal condenará o juiz nas
Síndrome da Imunodeficiência Adquirida (AIDS) e outros custas e remeterá os autos ao substituto legal do juiz, poderá o
procedimentos médicos necessários e cabíveis nos casos de magistrado recorrer da decisão.
violência sexual.
Ao julgar a exceção o tribunal fixará o momento a partir do
13. (AGERBA - Técnico em Regulação – IBFC/2017) qual o juiz não poderia ter atuado, bem como decretará a
Assinale a alternativa correta sobre a espécie de violência que nulidade dos atos do juiz, se praticados quando já presente o
a Lei Federal nº 11.340, de 7 de agosto de 2006 (Lei Maria da motivo de impedimento ou de suspeição.
Penha) indica, em termos expressos e precisos, como qualquer
conduta contra a mulher que lhe cause dano emocional e É preciso destacar que caso a exceção seja alegada contra
diminuição da autoestima, que lhe prejudique e perturbe o os demais sujeitos que não o juiz, a parte interessada deverá
pleno desenvolvimento, que vise degradar ou controlar suas arguir o impedimento ou a suspeição, em petição
ações, comportamentos, crenças e decisões, mediante ameaça. fundamentada e devidamente instruída, na primeira
(A) Violência psicológica oportunidade em que lhe couber falar nos autos e o juiz
(B) Violência moral mandará processar o incidente em separado e sem suspensão
(C) Violência imaterial do processo, ouvindo o arguido no prazo de 15 dias e
(D) Violência uxória facultando a produção de prova, quando necessária.
(E) Violência extra corporal
Dos auxiliares da justiça.
Respostas
Conceito de auxiliar da justiça3
01. E / 02. E. / 03. D. / 04. C. / 05. A O aumento da complexidade social, com um número cada
06. B / 07. A / 08. B / 09. E / 10. C vez maior de tipos de demandas, bem como de simples
11. D. / 12. B. / 13. A. quantidade de processos, torna necessária a ampliação da
estrutura judicial de modo a acompanhar tal realidade.
Ademais, a demanda por maior acesso à Justiça, mote de
diversas iniciativas nas três décadas finais do século XX,
CÓDIGO DE PROCESSO CIVIL – produziu seus resultados mais latentes nos anos 2000. Trata-
LEI 13.105/2015 Arts: 144 a se de um aumento exponencial nos processos trazidos ao
149, 156 a 158, 464 a 480, 693 Judiciário e nos novos atores que passaram a buscar a
a 699, 747 a 765. efetivação dos seus direitos. Cidadãos que estavam à margem
da esfera jurisdicional foram nela incluídos.
Exatamente nesse sentido, a estrutural de pessoal dos
tribunais também precisa ser não só aumentada, detalhada, de
Os artigos 144 a 149 do Código de Processo Civil tratam modo a conter um número crescente de profissionais com
sobre as hipóteses de suspeição e impedimento. Já os artigos diferentes habilidades e aptos para atender às questões
156 a 158 dispõe acerca dos auxiliares da justiça, mais apresentadas pelas demandas propostas pelos cidadãos.
precisamente o perito. Não por acaso, o rol enumerado de “auxiliares da justiça”
existentes cresceu de 6, conforme o art. 139, caput, do
As hipóteses de impedimento e suspeição estão descritas, CPC/1973, para 13, conforme o art. 149, caput, do CPC/2015,
respectivamente, nos artigos 144 e 145 do Novo CPC. As respectivamente.
causas de impedimento acarretam presunção absoluta de A expressão “além de outros”, para indicar ainda que
parcialidade do juiz e podem gerar nulidade absoluta do feito. outros profissionais são “auxiliares da justiça”, existe nas duas
Já as causas de suspeição acarretam presunção relativa de redações.
parcialidade do juiz e poderá conduzir a nulidade relativa do Não houve alteração, contudo, ao caráter teleológico da
processo. função de “auxiliares da justiça”. Sob o regime do CPC/1973 ou
do CPC/2015, o fato é que todos esses profissionais têm como
Lembre-se que tais causas também se aplicam aos função principal participarem da movimentação do processo
membros do Ministério Público, aos auxiliares da justiça e aos de algum modo e auxiliarem o magistrado nessa condução e,
demais sujeitos do processo. ao final, permitir que essa confira a prestação jurisdicional.
Tradicionalmente, os “Auxiliares da Justiça” não são
Processamento da Exceção: mencionados como sujeitos do processo (tríade entre autor,
A parte tem o prazo de 15 dias, contados do juiz e réu), mas indubitavelmente permitem que o juiz
conhecimento do fato para alegar o impedimento ou participe de diversas relações processuais ao mesmo tempo e,
suspeição. ainda, conferindo a prestação jurisdicional (STJ, 4ª T., REsp nº
A alegação deve ser feita em peça apartada dirigida ao juiz 213799/SP, Rel. Min. Sálvio de Figueiredo Teixeira, unânime,
do processo, na qual indicará o fundamento da recusa, j. em 24/6/2003, DJ de 29/9/2003, p. 253, e RT vol. 820, p.
podendo ser instruída com documentos a basear a alegação e 188).
rol de testemunhas. Como se verá a seguir, há uma grande heterogeneidade
entre as funções que podem ser exercidas decorrente da
Reconhecendo o impedimento ou a suspeição, o juiz grande heterogeneidade de demandas que podem ser
ordenará imediatamente a remessa dos autos a seu substituto apresentadas no Judiciário.
legal, caso contrário determinará a atuação em apartado da Alguns desses auxiliares são permanentes (“esquema fixo
petição e, no prazo de 15 dias, apresentará suas razões, do tribunal”), isto é, pertencem aos quadros do Poder
também poderá ser acompanhada de documentos e rol de Judiciário enquanto outros são eventuais, chamados a
testemunhas, com o que ordenará a remessa do incidente ao participar apenas em processos específicos e para exercer
tribunal para apreciação. funções específicas.

3 Luis Fernando Guerreo.

Legislação 63
Apostila Digital Licenciada para Alice Caroline Guarino dos Santos - alice.guarino@hotmail.com (Proibida a Revenda)
APOSTILAS OPÇÃO

Perito § 1º Poderá o juiz declarar-se suspeito por motivo de foro


Perito é o profissional cuja função é fornecer íntimo, sem necessidade de declarar suas razões.
conhecimento ao magistrado acerca de questões técnicas e § 2º Será ilegítima a alegação de suspeição quando:
específicas. É muito comum sua atuação em processos que I - houver sido provocada por quem a alega;
demandem conhecimento profundo nas áreas de II - a parte que a alega houver praticado ato que signifique
Contabilidade, Economia, Engenharia, Medicina, por exemplo. manifesta aceitação do arguido.
Além de uma análise do caso, geralmente elabora um laudo
técnico com o escopo de esclarecer ou elucidar uma Art. 146. No prazo de 15 (quinze) dias, a contar do
determinada questão específica ao magistrado. Não pertence conhecimento do fato, a parte alegará o impedimento ou a
ao esquema fixo do Tribunal e é indicado por ter a confiança suspeição, em petição específica dirigida ao juiz do processo, na
do magistrado. qual indicará o fundamento da recusa, podendo instruí-la com
documentos em que se fundar a alegação e com rol de
Segue abaixo os dispositivos que trazem as hipóteses de testemunhas.
suspeição e impedimento: § 1º Se reconhecer o impedimento ou a suspeição ao receber
a petição, o juiz ordenará imediatamente a remessa dos autos a
CAPÍTULO II seu substituto legal, caso contrário, determinará a autuação em
DOS IMPEDIMENTOS E DA SUSPEIÇÃO apartado da petição e, no prazo de 15 (quinze) dias, apresentará
suas razões, acompanhadas de documentos e de rol de
Art. 144. Há impedimento do juiz, sendo-lhe vedado exercer testemunhas, se houver, ordenando a remessa do incidente ao
suas funções no processo: tribunal.
I - em que interveio como mandatário da parte, oficiou como § 2º Distribuído o incidente, o relator deverá declarar os seus
perito, funcionou como membro do Ministério Público ou efeitos, sendo que, se o incidente for recebido:
prestou depoimento como testemunha; I - sem efeito suspensivo, o processo voltará a correr;
II - de que conheceu em outro grau de jurisdição, tendo II - com efeito suspensivo, o processo permanecerá suspenso
proferido decisão; até o julgamento do incidente.
III - quando nele estiver postulando, como defensor público, § 3º Enquanto não for declarado o efeito em que é recebido
advogado ou membro do Ministério Público, seu cônjuge ou o incidente ou quando este for recebido com efeito suspensivo, a
companheiro, ou qualquer parente, consanguíneo ou afim, em tutela de urgência será requerida ao substituto legal.
linha reta ou colateral, até o terceiro grau, inclusive; § 4º Verificando que a alegação de impedimento ou de
IV - quando for parte no processo ele próprio, seu cônjuge ou suspeição é improcedente, o tribunal rejeitá-la-á.
companheiro, ou parente, consanguíneo ou afim, em linha reta § 5º Acolhida a alegação, tratando-se de impedimento ou de
ou colateral, até o terceiro grau, inclusive; manifesta suspeição, o tribunal condenará o juiz nas custas e
V - quando for sócio ou membro de direção ou de remeterá os autos ao seu substituto legal, podendo o juiz
administração de pessoa jurídica parte no processo; recorrer da decisão.
VI - quando for herdeiro presuntivo, donatário ou § 6º Reconhecido o impedimento ou a suspeição, o tribunal
empregador de qualquer das partes; fixará o momento a partir do qual o juiz não poderia ter atuado.
VII - em que figure como parte instituição de ensino com a § 7º O tribunal decretará a nulidade dos atos do juiz, se
qual tenha relação de emprego ou decorrente de contrato de praticados quando já presente o motivo de impedimento ou de
prestação de serviços; suspeição.
VIII - em que figure como parte cliente do escritório de
advocacia de seu cônjuge, companheiro ou parente, Art. 147. Quando 2 (dois) ou mais juízes forem parentes,
consanguíneo ou afim, em linha reta ou colateral, até o terceiro consanguíneos ou afins, em linha reta ou colateral, até o terceiro
grau, inclusive, mesmo que patrocinado por advogado de outro grau, inclusive, o primeiro que conhecer do processo impede que
escritório; o outro nele atue, caso em que o segundo se escusará, remetendo
IX - quando promover ação contra a parte ou seu advogado. os autos ao seu substituto legal.
§ 1º Na hipótese do inciso III, o impedimento só se verifica
quando o defensor público, o advogado ou o membro do Art. 148. Aplicam-se os motivos de impedimento e de
Ministério Público já integrava o processo antes do início da suspeição:
atividade judicante do juiz. I - ao membro do Ministério Público;
§ 2º É vedada a criação de fato superveniente a fim de II - aos auxiliares da justiça;
caracterizar impedimento do juiz. III - aos demais sujeitos imparciais do processo.
§ 3º O impedimento previsto no inciso III também se verifica § 1º A parte interessada deverá arguir o impedimento ou a
no caso de mandato conferido a membro de escritório de suspeição, em petição fundamentada e devidamente instruída,
advocacia que tenha em seus quadros advogado que na primeira oportunidade em que lhe couber falar nos autos.
individualmente ostente a condição nele prevista, mesmo que § 2º O juiz mandará processar o incidente em separado e sem
não intervenha diretamente no processo. suspensão do processo, ouvindo o arguido no prazo de 15
(quinze) dias e facultando a produção de prova, quando
Art. 145. Há suspeição do juiz: necessária.
I - amigo íntimo ou inimigo de qualquer das partes ou de § 3º Nos tribunais, a arguição a que se refere o § 1º será
seus advogados; disciplinada pelo regimento interno.
II - que receber presentes de pessoas que tiverem interesse § 4º O disposto nos §§ 1º e 2º não se aplica à arguição de
na causa antes ou depois de iniciado o processo, que aconselhar impedimento ou de suspeição de testemunha.
alguma das partes acerca do objeto da causa ou que
subministrar meios para atender às despesas do litígio; CAPÍTULO III
III - quando qualquer das partes for sua credora ou DOS AUXILIARES DA JUSTIÇA
devedora, de seu cônjuge ou companheiro ou de parentes destes,
em linha reta até o terceiro grau, inclusive; Art. 149. São auxiliares da Justiça, além de outros cujas
IV - interessado no julgamento do processo em favor de atribuições sejam determinadas pelas normas de organização
qualquer das partes. judiciária, o escrivão, o chefe de secretaria, o oficial de justiça,
o perito, o depositário, o administrador, o intérprete, o

Legislação 64
Apostila Digital Licenciada para Alice Caroline Guarino dos Santos - alice.guarino@hotmail.com (Proibida a Revenda)
APOSTILAS OPÇÃO

tradutor, o mediador, o conciliador judicial, o partidor, o incumbe provar os fatos impeditivos, modificativos e
distribuidor, o contabilista e o regulador de avarias. extintivos do direito do autor.
Esse regramento, no entanto, é relativizado pelo §1º, o qual
Seção II possibilita a distribuição diversa do ônus da prova conforme
Do Perito as peculiaridades do caso concreto, atribuindo à parte que
tenha melhores condições de suportá-lo. Trata-se da
Art. 156. O juiz será assistido por perito quando a prova distribuição dinâmica do ônus da prova, que se contrapõe à
do fato depender de conhecimento técnico ou científico. concepção estática prevista na legislação anterior (art. 333 do
§ 1º Os peritos serão nomeados entre os profissionais CPC/73).
legalmente habilitados e os órgãos técnicos ou científicos De acordo com o novo CPC, o encargo probatório deve ser
devidamente inscritos em cadastro mantido pelo tribunal ao atribuído casuisticamente, de modo dinâmico, concedendo-se
qual o juiz está vinculado. ao juiz, como gestor das provas, poderes para avaliar qual das
§ 2º Para formação do cadastro, os tribunais devem partes terá maiores facilidades na sua produção.
realizar consulta pública, por meio de divulgação na rede Evidentemente, a decisão deverá ser fundamentada,
mundial de computadores ou em jornais de grande circulação, justificando as razões que convenceram o juiz da
além de consulta direta a universidades, a conselhos de classe, impossibilidade de produção da prova por uma das partes.
ao Ministério Público, à Defensoria Pública e à Ordem dos Ademais, essencial ater-se ao dever do juiz de permitir que a
Advogados do Brasil, para a indicação de profissionais ou de parte possa se desincumbir do ônus probatório, conforme
órgãos técnicos interessados. disposto na parte final do §1º. Com efeito, a inversão do ônus
§ 3º Os tribunais realizarão avaliações e reavaliações da prova não pode violar o contraditório, impedindo que a
periódicas para manutenção do cadastro, considerando a parte sucumba em momento sentencial por não ter cumprido
formação profissional, a atualização do conhecimento e a ônus que não lhe era devido anteriormente. Situação como
experiência dos peritos interessados. essa configuraria decisão surpresa, violando o art. 10 do
§ 4º Para verificação de eventual impedimento ou motivo CPC/2015.
de suspeição, nos termos dos arts. 148 e 467, o órgão técnico A distribuição dinâmica do ônus da prova decorre dos
ou científico nomeado para realização da perícia informará ao princípios da igualdade, da lealdade, da boa-fé e do princípio
juiz os nomes e os dados de qualificação dos profissionais que da cooperação entre os sujeitos do processo e também com o
participarão da atividade. órgão jurisdicional. De todo modo, deve o juiz aplicar esta nova
§ 5º Na localidade onde não houver inscrito no cadastro regra com cautela, de forma a afastar injustiças, mas sem
disponibilizado pelo tribunal, a nomeação do perito é de livre prejudicar demasiadamente a produção probatória para uma
escolha pelo juiz e deverá recair sobre profissional ou órgão das partes.
técnico ou científico comprovadamente detentor do O Código também possibilita que as partes
conhecimento necessário à realização da perícia. convencionem de maneira diversa a distribuição do ônus
probatório, só não podendo fazê-lo quando se tratar de
Art. 157. O perito tem o dever de cumprir o ofício no prazo direito indisponível ou quando tal convenção tornar essa
que lhe designar o juiz, empregando toda sua diligência, prova excessivamente difícil. Isso significa que o magistrado,
podendo escusar-se do encargo alegando motivo legítimo. em uma dessas duas hipóteses e à luz do caso concreto, pode
§ 1º A escusa será apresentada no prazo de 15 (quinze) afastar a convenção anteriormente pactuada entre as partes.
dias, contado da intimação, da suspeição ou do impedimento A regra do §4º permite que o acordo que distribui o ônus
supervenientes, sob pena de renúncia ao direito a alegá-la. da prova seja formalizado antes no curso do processo. O juiz
§ 2º Será organizada lista de peritos na vara ou na deve velar para que esse acordo não seja formalizado com o
secretaria, com disponibilização dos documentos exigidos objetivo de prejudicar uma das partes. Cabe-lhe, pois, indeferir
para habilitação à consulta de interessados, para que a a convenção quando presentes as situações descritas no § 3º.
nomeação seja distribuída de modo equitativo, observadas a
capacidade técnica e a área de conhecimento. Importante:
Qual o momento para a inversão do ônus da prova?
Art. 158. O perito que, por dolo ou culpa, prestar Segundo o STJ, a inversão do ônus da prova é regra de
informações inverídicas responderá pelos prejuízos que instrução (ou de procedimento), devendo a decisão judicial
causar à parte e ficará inabilitado para atuar em outras que determina-la ser proferida preferencialmente na fase de
perícias no prazo de 2 (dois) a 5 (cinco) anos, saneamento do processo. Caso a decisão sobre a inversão seja
independentemente das demais sanções previstas em lei, posterior, deve-se assegurar à parte a quem não incumbia
devendo o juiz comunicar o fato ao respectivo órgão de classe inicialmente o encargo a reabertura de oportunidade para
para adoção das medidas que entender cabíveis. manifestar-se nos autos. O CPC/2015 adotou esse
posicionamento, conforme consta na parte final do §1º do art.
Arts. 464 a 480 373.

O CPC/73 adota a regra do livre convencimento motivado Prova Pericial.


dispondo que o juiz apreciará livremente a prova. O CPC de A prova pericial será necessária sempre que for essencial
2015 não exclui essa ideia. o conhecimento técnico ou científico específico para esclarecer
determinado fato.
Outra novidade legislativa trazida em 2015 é que o CPC
passa a adotar de modo expresso a possibilidade do uso da Poderá ser determinada de ofício ou mediante
prova emprestada, isto é, da prova produzida em outro requerimento das partes.
processo e que também afeta a causa em questão.
Esse meio de prova auxilia o juiz na solução do litígio. Será
Quanto ao Ônus da prova, da leitura do art. 373, pode-se indeferida quando:
visualizar que o Código estabelece, aprioristicamente, a quem a) não houver a necessidade de conhecimento especial de
compete a produção de determinada prova. Regra geral, ao técnico para prova do fato;
autor cabe provar os fatos constitutivos de seu direito e ao réu b) o fato já estiver comprovado por outros meios de prova;
e,

Legislação 65
Apostila Digital Licenciada para Alice Caroline Guarino dos Santos - alice.guarino@hotmail.com (Proibida a Revenda)
APOSTILAS OPÇÃO

c) a verificação for impraticável (art. 464, §1º, CPC). II - currículo, com comprovação de especialização;
III - contatos profissionais, em especial o endereço
A produção da prova pericial poderá ser dispensada eletrônico, para onde serão dirigidas as intimações pessoais.
quando as partes, na inicial e na contestação, apresentarem, § 3º As partes serão intimadas da proposta de honorários
sobre as questões de fato, pareceres técnicos ou documentos para, querendo, manifestar-se no prazo comum de 5 (cinco)
elucidativos que forem considerados suficientes pelo dias, após o que o juiz arbitrará o valor, intimando-se as partes
magistrado (art. 472, CPC). para os fins do art. 95.
§ 4º O juiz poderá autorizar o pagamento de até cinquenta
Uma novidade trazida pelo CPC/15 é o recurso da prova por cento dos honorários arbitrados a favor do perito no início
técnica simplificada, que consistem em substituição da dos trabalhos, devendo o remanescente ser pago apenas ao
perícia pela inquirição de especialista com formação final, depois de entregue o laudo e prestados todos os
acadêmica específica na área objeto do assunto (art. 464, §§ 2º esclarecimentos necessários.
a 4º). § 5º Quando a perícia for inconclusiva ou deficiente, o juiz
poderá reduzir a remuneração inicialmente arbitrada para o
Procedimento prova pericial: trabalho.
Após a intimação do despacho de nomeação do perito as § 6º Quando tiver de realizar-se por carta, poder-se-á
partes têm o prazo de 15 dias para indicar assistentes técnicos, proceder à nomeação de perito e à indicação de assistentes
apresentar quesitos e, se for o caso, arguir suspeição ou técnicos no juízo ao qual se requisitar a perícia.
impedimento.
O perito também será cientificado de sua nomeação e terá Art. 466. O perito cumprirá escrupulosamente o encargo
o prazo de 5 dias para apresentar proposta de honorários. As que lhe foi cometido, independentemente de termo de
partes terão 5 dias para se manifestarem acerca dos compromisso.
honorários. § 1º Os assistentes técnicos são de confiança da parte e não
Após a entrega do laudo as partes terão 15 dias para se estão sujeitos a impedimento ou suspeição.
manifestarem. No caso de litisconsórcio com procuradores § 2º O perito deve assegurar aos assistentes das partes o
distintos o prazo será dobrado. acesso e o acompanhamento das diligências e dos exames que
Havendo esclarecimentos a serem feitos pelo expert eles realizar, com prévia comunicação, comprovada nos autos, com
poderão ser feitos em laudo complementar ou na própria antecedência mínima de 5 (cinco) dias.
audiência.
Art. 467. O perito pode escusar-se ou ser recusado por
Veja os dispositivos: impedimento ou suspeição.
Parágrafo único. O juiz, ao aceitar a escusa ou ao julgar
CAPÍTULO XII procedente a impugnação, nomeará novo perito.
DAS PROVAS
Seção X Art. 468. O perito pode ser substituído quando:
Da Prova Pericial I - faltar-lhe conhecimento técnico ou científico;
II - sem motivo legítimo, deixar de cumprir o encargo no
Art. 464. A prova pericial consiste em exame, vistoria ou prazo que lhe foi assinado.
avaliação. § 1º No caso previsto no inciso II, o juiz comunicará a
§ 1º O juiz indeferirá a perícia quando: ocorrência à corporação profissional respectiva, podendo,
I - a prova do fato não depender de conhecimento especial ainda, impor multa ao perito, fixada tendo em vista o valor da
de técnico; causa e o possível prejuízo decorrente do atraso no processo.
II - for desnecessária em vista de outras provas § 2º O perito substituído restituirá, no prazo de 15 (quinze)
produzidas; dias, os valores recebidos pelo trabalho não realizado, sob
III - a verificação for impraticável. pena de ficar impedido de atuar como perito judicial pelo
§ 2º De ofício ou a requerimento das partes, o juiz poderá, prazo de 5 (cinco) anos.
em substituição à perícia, determinar a produção de prova § 3º Não ocorrendo a restituição voluntária de que trata o
técnica simplificada, quando o ponto controvertido for de § 2º, a parte que tiver realizado o adiantamento dos
menor complexidade. honorários poderá promover execução contra o perito, na
§ 3º A prova técnica simplificada consistirá apenas na forma dos arts. 513 e seguintes deste Código, com fundamento
inquirição de especialista, pelo juiz, sobre ponto controvertido na decisão que determinar a devolução do numerário.
da causa que demande especial conhecimento científico ou
técnico. Art. 469. As partes poderão apresentar quesitos
§ 4º Durante a arguição, o especialista, que deverá ter suplementares durante a diligência, que poderão ser
formação acadêmica específica na área objeto de seu respondidos pelo perito previamente ou na audiência de
depoimento, poderá valer-se de qualquer recurso tecnológico instrução e julgamento.
de transmissão de sons e imagens com o fim de esclarecer os Parágrafo único. O escrivão dará à parte contrária ciência
pontos controvertidos da causa. da juntada dos quesitos aos autos.

Art. 465. O juiz nomeará perito especializado no objeto da Art. 470. Incumbe ao juiz:
perícia e fixará de imediato o prazo para a entrega do laudo. I - indeferir quesitos impertinentes;
§ 1º Incumbe às partes, dentro de 15 (quinze) dias II - formular os quesitos que entender necessários ao
contados da intimação do despacho de nomeação do perito: esclarecimento da causa.
I - arguir o impedimento ou a suspeição do perito, se for o
caso; Art. 471. As partes podem, de comum acordo, escolher o
II - indicar assistente técnico; perito, indicando-o mediante requerimento, desde que:
III - apresentar quesitos. I - sejam plenamente capazes;
§ 2º Ciente da nomeação, o perito apresentará em 5 (cinco) II - a causa possa ser resolvida por autocomposição.
dias: § 1º As partes, ao escolher o perito, já devem indicar os
I - proposta de honorários; respectivos assistentes técnicos para acompanhar a realização

Legislação 66
Apostila Digital Licenciada para Alice Caroline Guarino dos Santos - alice.guarino@hotmail.com (Proibida a Revenda)
APOSTILAS OPÇÃO

da perícia, que se realizará em data e local previamente Art. 478. Quando o exame tiver por objeto a autenticidade
anunciados. ou a falsidade de documento ou for de natureza médico-legal,
§ 2º O perito e os assistentes técnicos devem entregar, o perito será escolhido, de preferência, entre os técnicos dos
respectivamente, laudo e pareceres em prazo fixado pelo juiz. estabelecimentos oficiais especializados, a cujos diretores o
§ 3º A perícia consensual substitui, para todos os efeitos, a juiz autorizará a remessa dos autos, bem como do material
que seria realizada por perito nomeado pelo juiz. sujeito a exame.
§ 1º Nas hipóteses de gratuidade de justiça, os órgãos e as
Art. 472. O juiz poderá dispensar prova pericial quando as repartições oficiais deverão cumprir a determinação judicial
partes, na inicial e na contestação, apresentarem, sobre as com preferência, no prazo estabelecido.
questões de fato, pareceres técnicos ou documentos § 2º A prorrogação do prazo referido no § 1º pode ser
elucidativos que considerar suficientes. requerida motivadamente.
§ 3º Quando o exame tiver por objeto a autenticidade da
Art. 473. O laudo pericial deverá conter: letra e da firma, o perito poderá requisitar, para efeito de
I - a exposição do objeto da perícia; comparação, documentos existentes em repartições públicas
II - a análise técnica ou científica realizada pelo perito; e, na falta destes, poderá requerer ao juiz que a pessoa a quem
III - a indicação do método utilizado, esclarecendo-o e se atribuir a autoria do documento lance em folha de papel, por
demonstrando ser predominantemente aceito pelos cópia ou sob ditado, dizeres diferentes, para fins de
especialistas da área do conhecimento da qual se originou; comparação.
IV - resposta conclusiva a todos os quesitos apresentados
pelo juiz, pelas partes e pelo órgão do Ministério Público. Art. 479. O juiz apreciará a prova pericial de acordo com o
§ 1º No laudo, o perito deve apresentar sua fundamentação disposto no art. 371, indicando na sentença os motivos que o
em linguagem simples e com coerência lógica, indicando como levaram a considerar ou a deixar de considerar as conclusões
alcançou suas conclusões. do laudo, levando em conta o método utilizado pelo perito.
§ 2º É vedado ao perito ultrapassar os limites de sua
designação, bem como emitir opiniões pessoais que excedam Art. 480. O juiz determinará, de ofício ou a requerimento
o exame técnico ou científico do objeto da perícia. da parte, a realização de nova perícia quando a matéria não
§ 3º Para o desempenho de sua função, o perito e os estiver suficientemente esclarecida.
assistentes técnicos podem valer-se de todos os meios § 1º A segunda perícia tem por objeto os mesmos fatos
necessários, ouvindo testemunhas, obtendo informações, sobre os quais recaiu a primeira e destina-se a corrigir
solicitando documentos que estejam em poder da parte, de eventual omissão ou inexatidão dos resultados a que esta
terceiros ou em repartições públicas, bem como instruir o conduziu.
laudo com planilhas, mapas, plantas, desenhos, fotografias ou § 2º A segunda perícia rege-se pelas disposições
outros elementos necessários ao esclarecimento do objeto da estabelecidas para a primeira.
perícia. § 3º A segunda perícia não substitui a primeira, cabendo ao
juiz apreciar o valor de uma e de outra.
Art. 474. As partes terão ciência da data e do local
designados pelo juiz ou indicados pelo perito para ter início a Ações de Família (arts. 693 a 699, CPC)
produção da prova.
O CPC em seu artigo 693 estabelece que processos
Art. 475. Tratando-se de perícia complexa que abranja contenciosos de divórcio, separação, reconhecimento e
mais de uma área de conhecimento especializado, o juiz extinção de união estável, guarda, visitação e filiação seguirão
poderá nomear mais de um perito, e a parte, indicar mais de um procedimento especial – as chamadas “ações de família”.
um assistente técnico.
O rol das ações de família é exemplificativo.
Art. 476. Se o perito, por motivo justificado, não puder
apresentar o laudo dentro do prazo, o juiz poderá conceder- Segundo dispõe o art. 695, recebida a inicial o juiz
lhe, por uma vez, prorrogação pela metade do prazo ordenará a citação do réu para comparecer à audiência de
originalmente fixado. mediação e conciliação, que ocorrerá antes da apresentação da
contestação.
Art. 477. O perito protocolará o laudo em juízo, no prazo
fixado pelo juiz, pelo menos 20 (vinte) dias antes da audiência A novidade legislativa está em determinar que o mandado
de instrução e julgamento. de citação deverá conter apenas os dados necessários à
§ 1º As partes serão intimadas para, querendo, manifestar- audiência, dispensando-se assim a cópia da inicial (art. 695,
se sobre o laudo do perito do juízo no prazo comum de 15 §1º).
(quinze) dias, podendo o assistente técnico de cada uma das
partes, em igual prazo, apresentar seu respectivo parecer. A citação deverá ocorrer com antecedência mínima de 15
§ 2º O perito do juízo tem o dever de, no prazo de 15 dias e será feita pessoalmente.
(quinze) dias, esclarecer ponto:
I - sobre o qual exista divergência ou dúvida de qualquer O Ministério Público intervirá quando houver interesse de
das partes, do juiz ou do órgão do Ministério Público; incapazes.
II - divergente apresentado no parecer do assistente
técnico da parte. Veja os dispositivos correspondentes:
§ 3º Se ainda houver necessidade de esclarecimentos, a
parte requererá ao juiz que mande intimar o perito ou o CAPÍTULO X
assistente técnico a comparecer à audiência de instrução e DAS AÇÕES DE FAMÍLIA
julgamento, formulando, desde logo, as perguntas, sob forma
de quesitos. Art. 693. As normas deste Capítulo aplicam-se aos
§ 4º O perito ou o assistente técnico será intimado por processos contenciosos de divórcio, separação,
meio eletrônico, com pelo menos 10 (dez) dias de reconhecimento e extinção de união estável, guarda, visitação
antecedência da audiência. e filiação.

Legislação 67
Apostila Digital Licenciada para Alice Caroline Guarino dos Santos - alice.guarino@hotmail.com (Proibida a Revenda)
APOSTILAS OPÇÃO

Parágrafo único. A ação de alimentos e a que versar sobre Legitimidade.


interesse de criança ou de adolescente observarão o Segundo o disposto no art. 747 do CPC a interdição poderá
procedimento previsto em legislação específica, aplicando-se, ser proposta por:
no que couber, as disposições deste Capítulo. I - pelo cônjuge ou companheiro;
II - pelos parentes ou tutores;
Art. 694. Nas ações de família, todos os esforços serão III - pelo representante da entidade em que se encontra
empreendidos para a solução consensual da controvérsia, abrigado o interditando;
devendo o juiz dispor do auxílio de profissionais de outras IV - pelo Ministério Público.
áreas de conhecimento para a mediação e conciliação.
Parágrafo único. A requerimento das partes, o juiz pode Aspectos gerais.
determinar a suspensão do processo enquanto os litigantes se O novo CPC estabelece que o juiz poderá entrevistar o
submetem a mediação extrajudicial ou a atendimento interditando e ainda prevê a possibilidade de o juiz se deslocar
multidisciplinar. até o interditando em casos de impossibilidade de se
apresentar em juízo. Haverá ainda a possibilidade do emprego
Art. 695. Recebida a petição inicial e, se for o caso, tomadas de meios tecnológicos para auxiliar o interditando a
as providências referentes à tutela provisória, o juiz ordenará manifestar sua vontade.
a citação do réu para comparecer à audiência de mediação e
conciliação, observado o disposto no art. 694. Também prevê a possibilidade de ouvir os parentes e
§ 1º O mandado de citação conterá apenas os dados pessoas próximas ao interditando.
necessários à audiência e deverá estar desacompanhado de
cópia da petição inicial, assegurado ao réu o direito de O prazo de impugnação do pedido, a contar da data da
examinar seu conteúdo a qualquer tempo. entrevista será de 15 (quinze) dias. O interditando poderá
§ 2º A citação ocorrerá com antecedência mínima de 15 constituir advogado, todavia, caso não o faça, seu cônjuge,
(quinze) dias da data designada para a audiência. companheiro ou algum parente do interditando poderá
§ 3º A citação será feita na pessoa do réu. intervir como assistente.
§ 4º Na audiência, as partes deverão estar acompanhadas
de seus advogados ou de defensores públicos. A sentença que decreta a interdição não faz coisa julgada.
A qualquer tempo a curatela poderá ser levantada.
Art. 696. A audiência de mediação e conciliação poderá
dividir-se em tantas sessões quantas sejam necessárias para É perfeitamente possível a interdição parcial, bem como o
viabilizar a solução consensual, sem prejuízo de providências levantamento parcial da curatela.
jurisdicionais para evitar o perecimento do direito.
Veja os dispositivos correspondentes.
Art. 697. Não realizado o acordo, passarão a incidir, a partir
de então, as normas do procedimento comum, observado o art. Seção IX
335. Da Interdição

Art. 698. Nas ações de família, o Ministério Público Art. 747. A interdição pode ser promovida:
somente intervirá quando houver interesse de incapaz e I - pelo cônjuge ou companheiro;
deverá ser ouvido previamente à homologação de acordo. II - pelos parentes ou tutores;
III - pelo representante da entidade em que se encontra
Art. 699. Quando o processo envolver discussão sobre fato abrigado o interditando;
relacionado a abuso ou a alienação parental, o juiz, ao tomar o IV - pelo Ministério Público.
depoimento do incapaz, deverá estar acompanhado por Parágrafo único. A legitimidade deverá ser comprovada
especialista. por documentação que acompanhe a petição inicial.

Arts. 747 a 765 Art. 748. O Ministério Público só promoverá interdição em


caso de doença mental grave:
Partindo da premissa que todas pessoas têm I - se as pessoas designadas nos incisos I, II e III do art. 747
personalidade civil (capacidade de direito), nem todas têm a não existirem ou não promoverem a interdição;
chamada capacidade de fato (a capacidade de exercer por si II - se, existindo, forem incapazes as pessoas mencionadas
própria os atos da vida civil). nos incisos I e II do art. 747.

É preciso alertarmos que o Código Civil sofreu relevante Art. 749. Incumbe ao autor, na petição inicial, especificar
alteração na teoria das incapacidades com o advento do os fatos que demonstram a incapacidade do interditando para
Estatuto da Pessoa com Deficiência (Lei nº 13.146/15). administrar seus bens e, se for o caso, para praticar atos da
Assim, atualmente temos apenas uma hipótese de vida civil, bem como o momento em que a incapacidade se
incapacidade absoluta – a do menor de 16 anos (art. 3º, CC). revelou.
Os relativamente incapazes são os seguintes: a- maior de 16 e Parágrafo único. Justificada a urgência, o juiz pode nomear
menor de 18 anos; b- ébrios habituais e viciados em curador provisório ao interditando para a prática de
tóxico; c- aqueles que, por causa transitória ou permanente, determinados atos.
não puderem exprimir sua vontade e d- os pródigos.
Art. 750. O requerente deverá juntar laudo médico para
A interdição servirá para privar a pessoa que está fazer prova de suas alegações ou informar a impossibilidade
impossibilidade de gerir, por si só, sua vida e seus negócios, de fazê-lo.
ficando ela dependente dos cuidados de pessoa legalmente
habilitada por meio de nomeação em processo judicial. Art. 751. O interditando será citado para, em dia
designado, comparecer perante o juiz, que o entrevistará
minuciosamente acerca de sua vida, negócios, bens, vontades,
preferências e laços familiares e afetivos e sobre o que mais lhe

Legislação 68
Apostila Digital Licenciada para Alice Caroline Guarino dos Santos - alice.guarino@hotmail.com (Proibida a Revenda)
APOSTILAS OPÇÃO

parecer necessário para convencimento quanto à sua vezes, com intervalo de 10 (dez) dias, seguindo-se a averbação
capacidade para praticar atos da vida civil, devendo ser no registro de pessoas naturais.
reduzidas a termo as perguntas e respostas. § 4º A interdição poderá ser levantada parcialmente
§ 1º Não podendo o interditando deslocar-se, o juiz o quando demonstrada a capacidade do interdito para praticar
ouvirá no local onde estiver. alguns atos da vida civil.
§ 2º A entrevista poderá ser acompanhada por especialista.
§ 3º Durante a entrevista, é assegurado o emprego de Art. 757. A autoridade do curador estende-se à pessoa e
recursos tecnológicos capazes de permitir ou de auxiliar o aos bens do incapaz que se encontrar sob a guarda e a
interditando a expressar suas vontades e preferências e a responsabilidade do curatelado ao tempo da interdição, salvo
responder às perguntas formuladas. se o juiz considerar outra solução como mais conveniente aos
§ 4º A critério do juiz, poderá ser requisitada a oitiva de interesses do incapaz.
parentes e de pessoas próximas.
Art. 758. O curador deverá buscar tratamento e apoio
Art. 752. Dentro do prazo de 15 (quinze) dias contado da apropriados à conquista da autonomia pelo interdito.
entrevista, o interditando poderá impugnar o pedido.
§ 1º O Ministério Público intervirá como fiscal da ordem Seção X
jurídica. Disposições Comuns à Tutela e à Curatela
§ 2º O interditando poderá constituir advogado, e, caso não
o faça, deverá ser nomeado curador especial. Art. 759. O tutor ou o curador será intimado a prestar
§ 3º Caso o interditando não constitua advogado, o seu compromisso no prazo de 5 (cinco) dias contado da:
cônjuge, companheiro ou qualquer parente sucessível poderá I - nomeação feita em conformidade com a lei;
intervir como assistente. II - intimação do despacho que mandar cumprir o
testamento ou o instrumento público que o houver instituído.
Art. 753. Decorrido o prazo previsto no art. 752, o juiz § 1º O tutor ou o curador prestará o compromisso por
determinará a produção de prova pericial para avaliação da termo em livro rubricado pelo juiz.
capacidade do interditando para praticar atos da vida civil. § 2º Prestado o compromisso, o tutor ou o curador assume
§ 1º A perícia pode ser realizada por equipe composta por a administração dos bens do tutelado ou do interditado.
expertos com formação multidisciplinar.
§ 2º O laudo pericial indicará especificadamente, se for o Art. 760. O tutor ou o curador poderá eximir-se do encargo
caso, os atos para os quais haverá necessidade de curatela. apresentando escusa ao juiz no prazo de 5 (cinco) dias
contado:
Art. 754. Apresentado o laudo, produzidas as demais I - antes de aceitar o encargo, da intimação para prestar
provas e ouvidos os interessados, o juiz proferirá sentença. compromisso;
II - depois de entrar em exercício, do dia em que sobrevier
Art. 755. Na sentença que decretar a interdição, o juiz: o motivo da escusa.
I - nomeará curador, que poderá ser o requerente da § 1º Não sendo requerida a escusa no prazo estabelecido
interdição, e fixará os limites da curatela, segundo o estado e o neste artigo, considerar-se-á renunciado o direito de alegá-la.
desenvolvimento mental do interdito; § 2º O juiz decidirá de plano o pedido de escusa, e, não o
II - considerará as características pessoais do interdito, admitindo, exercerá o nomeado a tutela ou a curatela
observando suas potencialidades, habilidades, vontades e enquanto não for dispensado por sentença transitada em
preferências. julgado.
§ 1º A curatela deve ser atribuída a quem melhor possa
atender aos interesses do curatelado. Art. 761. Incumbe ao Ministério Público ou a quem tenha
§ 2º Havendo, ao tempo da interdição, pessoa incapaz sob legítimo interesse requerer, nos casos previstos em lei, a
a guarda e a responsabilidade do interdito, o juiz atribuirá a remoção do tutor ou do curador.
curatela a quem melhor puder atender aos interesses do Parágrafo único. O tutor ou o curador será citado para
interdito e do incapaz. contestar a arguição no prazo de 5 (cinco) dias, findo o qual
§ 3º A sentença de interdição será inscrita no registro de observar-se-á o procedimento comum.
pessoas naturais e imediatamente publicada na rede mundial
de computadores, no sítio do tribunal a que estiver vinculado Art. 762. Em caso de extrema gravidade, o juiz poderá
o juízo e na plataforma de editais do Conselho Nacional de suspender o tutor ou o curador do exercício de suas funções,
Justiça, onde permanecerá por 6 (seis) meses, na imprensa nomeando substituto interino.
local, 1 (uma) vez, e no órgão oficial, por 3 (três) vezes, com
intervalo de 10 (dez) dias, constando do edital os nomes do Art. 763. Cessando as funções do tutor ou do curador pelo
interdito e do curador, a causa da interdição, os limites da decurso do prazo em que era obrigado a servir, ser-lhe-á lícito
curatela e, não sendo total a interdição, os atos que o interdito requerer a exoneração do encargo.
poderá praticar autonomamente. § 1º Caso o tutor ou o curador não requeira a exoneração
do encargo dentro dos 10 (dez) dias seguintes à expiração do
Art. 756. Levantar-se-á a curatela quando cessar a causa termo, entender-se-á reconduzido, salvo se o juiz o dispensar.
que a determinou. § 2º Cessada a tutela ou a curatela, é indispensável a
§ 1º O pedido de levantamento da curatela poderá ser feito prestação de contas pelo tutor ou pelo curador, na forma da lei
pelo interdito, pelo curador ou pelo Ministério Público e será civil.
apensado aos autos da interdição.
§ 2º O juiz nomeará perito ou equipe multidisciplinar para Seção XI
proceder ao exame do interdito e designará audiência de Da Organização e da Fiscalização das Fundações
instrução e julgamento após a apresentação do laudo.
§ 3º Acolhido o pedido, o juiz decretará o levantamento da Art. 764. O juiz decidirá sobre a aprovação do estatuto das
interdição e determinará a publicação da sentença, após o fundações e de suas alterações sempre que o requeira o
trânsito em julgado, na forma do art. 755, § 3o, ou, não sendo interessado, quando:
possível, na imprensa local e no órgão oficial, por 3 (três)

Legislação 69
Apostila Digital Licenciada para Alice Caroline Guarino dos Santos - alice.guarino@hotmail.com (Proibida a Revenda)
APOSTILAS OPÇÃO

I - ela for negada previamente pelo Ministério Público ou II. O juiz é impedido de exercer suas funções em processo
por este forem exigidas modificações com as quais o em que figure como parte instituição de ensino com a qual
interessado não concorde; tenha relação de emprego ou decorrente de contrato de
II - o interessado discordar do estatuto elaborado pelo prestação de serviços.
Ministério Público. III. É legítima a alegação de suspeição ainda que esta haja
§ 1º O estatuto das fundações deve observar o disposto na sido provocada por quem a alega.
Lei no 10.406, de 10 de janeiro de 2002 (Código Civil). IV. Declarando-se suspeito por motivo de foro íntimo,
§ 2º Antes de suprir a aprovação, o juiz poderá mandar deverá o juiz declinar suas razões, remetendo os autos a seu
fazer no estatuto modificações a fim de adaptá-lo ao objetivo substituto legal.
do instituidor.
Está correto o que se afirma APENAS em
Art. 765. Qualquer interessado ou o Ministério Público (A) I e II.
promoverá em juízo a extinção da fundação quando: (B) I, III e IV.
I - se tornar ilícito o seu objeto; (C) III e IV.
II - for impossível a sua manutenção; (D) II e III.
III - vencer o prazo de sua existência. (E) I, II e IV.

Questões 04. (TRF - 4ª REGIÃO - Juiz Federal Substituto - TRF - 4ª


REGIÃO/2016) Dadas as assertivas abaixo, assinale a
01. (TRE-SP - Analista Judiciário - Área Judiciária – alternativa correta.
FCC/2017) Acerca dos impedimentos e suspeições do juiz, Considerando as regras do Código de Processo Civil de
segundo o novo Código de Processo Civil, considere: 2015:
I. Há suspeição do juiz quando promover ação contra a I. A suspeição e o impedimento devem ser suscitados em
parte ou seu advogado. preliminar da contestação, e não por petição separada.
II. Há impedimento do juiz que for amigo íntimo ou inimigo II. Há suspeição do juiz que for amigo íntimo ou inimigo do
de qualquer das partes ou de seus advogados. advogado de qualquer das partes.
III. Há impedimento do juiz quando qualquer das partes for III. O benefício da gratuidade da justiça pode ser concedido
sua credora ou devedora, de seu cônjuge ou companheiro ou apenas parcialmente ou consistir na redução percentual das
de parentes destes, em linha reta até o terceiro grau, inclusive. despesas processuais iniciais ou ainda no parcelamento dessas
IV. Há impedimento do juiz no processo em que figure despesas e não afasta o dever de o beneficiário pagar as multas
como parte cliente do escritório de advocacia de seu cônjuge, processuais que lhe sejam impostas.
companheiro ou parente, consanguíneo ou afim, em linha reta IV. As espécies de intervenção de terceiros são a
ou colateral, até o terceiro grau, inclusive. assistência, o chamamento ao processo, o incidente de
V. Há suspeição do juiz interessado no julgamento do desconsideração de personalidade jurídica e a oposição.
processo em favor de qualquer das partes. (A) Estão corretas apenas as assertivas I e IV.
Está correto o que consta APENAS em (B) Estão corretas apenas as assertivas II e III.
(A) I e III. (C) Estão corretas apenas as assertivas I, II e III.
(B) I e II. (D) Estão corretas todas as assertivas.
(C) II e IV. (E) Nenhuma assertiva está correta.
(D) III e V.
(E) IV e V. Respostas

02. (TRT - 20ª REGIÃO (SE) - Analista Judiciário - Área 01. E / 02. C. / 03. A / 04. B
Judiciária Especialidade Oficial de Justiça Avaliador
Federal - FCC) Analise as proposições abaixo, acerca dos
impedimentos e da suspeição: GUARDA
I. Há impedimento quando o juiz promover ação contra a
parte ou seu advogado. COMPARTILHADA - LEI
II. Há impedimento quando o primo do juiz estiver 11.698/2014.
postulando como advogado.
III. Há suspeição quando o juiz for amigo íntimo ou inimigo
das partes ou seus advogados.
(*) O ano correto da lei que trata da guarda compartilhada
IV. Poderá o juiz declarar-se suspeito por motivo de foro
é 2008, conforme segue abaixo.
íntimo, sem necessidade de declarar suas razões.
LEI Nº 11.698, DE 13 DE JUNHO DE 2008.
Está correto o que se afirma APENAS em
(A) I, II e III.
Altera os arts. 1.583 e 1.584 da Lei nº 10.406, de 10 de
(B) III e IV.
janeiro de 2002 – Código Civil, para instituir e disciplinar a
(C) I, III e IV.
guarda compartilhada.
(D) I e II.
(E) II e IV.
O PRESIDENTE DA REPÚBLICA Faço saber que o
Congresso Nacional decreta e eu sanciono a seguinte Lei:
03. (AL-MS - Consultor de Processo Legislativo –
FCC/2016) Acerca do impedimento e da suspeição, considere:
Art. 1º Os arts. 1.583 e 1.584 da Lei no 10.406, de 10 de
I. Há impedimento do juiz quando figurar como parte
janeiro de 2002 – Código Civil, passam a vigorar com a
cliente do escritório de advocacia de seu cônjuge,
seguinte redação:
companheiro ou parente, consanguíneo ou afim, em linha reta
“Art. 1.583. A guarda será unilateral ou compartilhada.
ou colateral, até o terceiro grau, inclusive, mesmo que
§ 1º Compreende-se por guarda unilateral a atribuída a um
patrocinado por advogado de outro escritório.
só dos genitores ou a alguém que o substitua (art. 1.584, § 5o)
e, por guarda compartilhada a responsabilização conjunta e o

Legislação 70
Apostila Digital Licenciada para Alice Caroline Guarino dos Santos - alice.guarino@hotmail.com (Proibida a Revenda)
APOSTILAS OPÇÃO

exercício de direitos e deveres do pai e da mãe que não vivam Art. 2º A Lei nº 10.406, de 10 de janeiro de 2002 (Código
sob o mesmo teto, concernentes ao poder familiar dos filhos Civil), passa a vigorar com as seguintes alterações:
comuns. “Art. 1.583. ...
§ 2º A guarda unilateral será atribuída ao genitor que ...
revele melhores condições para exercê-la e, objetivamente, § 2º Na guarda compartilhada, o tempo de convívio com os
mais aptidão para propiciar aos filhos os seguintes fatores: filhos deve ser dividido de forma equilibrada com a mãe e com
I – afeto nas relações com o genitor e com o grupo familiar; o pai, sempre tendo em vista as condições fáticas e os
II – saúde e segurança; interesses dos filhos.
III – educação. I - (revogado);
§ 3º A guarda unilateral obriga o pai ou a mãe que não a II - (revogado);
detenha a supervisionar os interesses dos filhos. III - (revogado).
§ 4º (VETADO).” (NR) § 3º Na guarda compartilhada, a cidade considerada base
de moradia dos filhos será aquela que melhor atender aos
“Art. 1.584. A guarda, unilateral ou compartilhada, poderá interesses dos filhos.
ser: ...
I – requerida, por consenso, pelo pai e pela mãe, ou por § 5º A guarda unilateral obriga o pai ou a mãe que não a
qualquer deles, em ação autônoma de separação, de divórcio, detenha a supervisionar os interesses dos filhos, e, para
de dissolução de união estável ou em medida cautelar; possibilitar tal supervisão, qualquer dos genitores sempre
II – decretada pelo juiz, em atenção a necessidades será parte legítima para solicitar informações e/ou prestação
específicas do filho, ou em razão da distribuição de tempo de contas, objetivas ou subjetivas, em assuntos ou situações
necessário ao convívio deste com o pai e com a mãe. que direta ou indiretamente afetem a saúde física e psicológica
§ 1º Na audiência de conciliação, o juiz informará ao pai e e a educação de seus filhos.” (NR)
à mãe o significado da guarda compartilhada, a sua
importância, a similitude de deveres e direitos atribuídos aos “Art. 1.584. ...
genitores e as sanções pelo descumprimento de suas cláusulas. ...
§ 2º Quando não houver acordo entre a mãe e o pai quanto § 2º Quando não houver acordo entre a mãe e o pai quanto
à guarda do filho, será aplicada, sempre que possível, a guarda à guarda do filho, encontrando-se ambos os genitores aptos a
compartilhada. exercer o poder familiar, será aplicada a guarda
§ 3º Para estabelecer as atribuições do pai e da mãe e os compartilhada, salvo se um dos genitores declarar ao
períodos de convivência sob guarda compartilhada, o juiz, de magistrado que não deseja a guarda do menor.
ofício ou a requerimento do Ministério Público, poderá basear- § 3º Para estabelecer as atribuições do pai e da mãe e os
se em orientação técnico-profissional ou de equipe períodos de convivência sob guarda compartilhada, o juiz, de
interdisciplinar. ofício ou a requerimento do Ministério Público, poderá basear-
§ 4º A alteração não autorizada ou o descumprimento se em orientação técnico-profissional ou de equipe
imotivado de cláusula de guarda, unilateral ou compartilhada, interdisciplinar, que deverá visar à divisão equilibrada do
poderá implicar a redução de prerrogativas atribuídas ao seu tempo com o pai e com a mãe.
detentor, inclusive quanto ao número de horas de convivência § 4º A alteração não autorizada ou o descumprimento
com o filho. imotivado de cláusula de guarda unilateral ou compartilhada
§ 5º Se o juiz verificar que o filho não deve permanecer sob poderá implicar a redução de prerrogativas atribuídas ao seu
a guarda do pai ou da mãe, deferirá a guarda à pessoa que detentor.
revele compatibilidade com a natureza da medida, § 5º Se o juiz verificar que o filho não deve permanecer sob
considerados, de preferência, o grau de parentesco e as a guarda do pai ou da mãe, deferirá a guarda a pessoa que
relações de afinidade e afetividade.” (NR) revele compatibilidade com a natureza da medida,
considerados, de preferência, o grau de parentesco e as
Art. 2º Esta Lei entra em vigor após decorridos 60 relações de afinidade e afetividade.
(sessenta) dias de sua publicação. § 6º Qualquer estabelecimento público ou privado é
obrigado a prestar informações a qualquer dos genitores sobre
os filhos destes, sob pena de multa de R$ 200,00 (duzentos
NOVA GUARDA reais) a R$ 500,00 (quinhentos reais) por dia pelo não
atendimento da solicitação.” (NR)
COMPARTILHADA - LEI
13.058/2014. “Art. 1.585. Em sede de medida cautelar de separação de
corpos, em sede de medida cautelar de guarda ou em outra
sede de fixação liminar de guarda, a decisão sobre guarda de
filhos, mesmo que provisória, será proferida
LEI Nº 13.058, DE 22 DE DEZEMBRO DE 2014.
preferencialmente após a oitiva de ambas as partes perante o
juiz, salvo se a proteção aos interesses dos filhos exigir a
Altera os arts. 1.583, 1.584, 1.585 e 1.634 da Lei nº
concessão de liminar sem a oitiva da outra parte, aplicando-se
10.406, de 10 de janeiro de 2002 (Código Civil), para
as disposições do art. 1.584.” (NR)
estabelecer o significado da expressão “guarda
compartilhada” e dispor sobre sua aplicação.
“Art. 1.634. Compete a ambos os pais, qualquer que seja a
sua situação conjugal, o pleno exercício do poder familiar, que
A PRESIDENTA DA REPÚBLICA Faço saber que o
consiste em, quanto aos filhos:
Congresso Nacional decreta e eu sanciono a seguinte Lei:
I - dirigir-lhes a criação e a educação;
II - exercer a guarda unilateral ou compartilhada nos
Art. 1º Esta Lei estabelece o significado da expressão
termos do art. 1.584;
“guarda compartilhada” e dispõe sobre sua aplicação, para o
III - conceder-lhes ou negar-lhes consentimento para
que modifica os arts. 1.583, 1.584, 1.585 e 1.634 da Lei no
casarem;
10.406, de 10 de janeiro de 2002 (Código Civil).
IV - conceder-lhes ou negar-lhes consentimento para
viajarem ao exterior;

Legislação 71
Apostila Digital Licenciada para Alice Caroline Guarino dos Santos - alice.guarino@hotmail.com (Proibida a Revenda)
APOSTILAS OPÇÃO

V - conceder-lhes ou negar-lhes consentimento para que legitima pai e mãe a assumirem a criação dos filhos de
mudarem sua residência permanente para outro Município; forma corresponsável, qual seja:
VI - nomear-lhes tutor por testamento ou documento (A) guarda alternada;
autêntico, se o outro dos pais não lhe sobreviver, ou o (B) guarda exclusiva;
sobrevivo não puder exercer o poder familiar; (C) guarda compartilhada;
VII - representá-los judicial e extrajudicialmente até os 16 (D) tutela antecipada;
(dezesseis) anos, nos atos da vida civil, e assisti-los, após essa (E) visitação livre.
idade, nos atos em que forem partes, suprindo-lhes o
consentimento; 05. (FCC) Reconhecer que cada progenitor tem os mesmos
VIII - reclamá-los de quem ilegalmente os detenha; direitos e os mesmos deveres perante seu filho é valorizar a
IX - exigir que lhes prestem obediência, respeito e os (A) guarda compartilhada.
serviços próprios de sua idade e condição.” (NR) (B) síndrome da alienação parental.
(C) guarda alternada.
Art. 3º Esta Lei entra em vigor na data de sua publicação. (D) guarda unilateral.
(E) guarda judicial.
Questões
Respostas
01. (FGV/2015) A Lei nº 13.058/2014 regulamenta a
aplicação da guarda compartilhada de forma mais clara, 01. D. / 02. B. / 03. A. / 04. C. / 05. A.
corrigindo alguns pontos da lei anterior, Lei nº 11.698/2008.
De acordo com a nova lei:
(A) a guarda compartilhada deve ser aplicada sempre que
possível; ALIENAÇÃO PARENTAL –
(B) os filhos devem revezar a moradia entre os pais, salvo LEI 12.318/2010.
se a criança não demonstrar interesse;
(C) a guarda compartilhada não será aplicada se não
houver concordância de um ou ambos os pais;
(D) o juiz pode basear-se em orientação técnico-
LEI Nº 12.318, DE 26 DE AGOSTO DE 2010.
profissional para estabelecer atribuições parentais e períodos
de convivência sob guarda compartilhada;
Dispõe sobre a alienação parental e altera o art. 236 da
(E) uma vez decretada a guarda compartilhada, o
Lei nº 8.069, de 13 de julho de 1990.
pagamento de pensão, se houver, deverá ser dividido pela
metade.
O PRESIDENTE DA REPÚBLICA Faço saber que o
Congresso Nacional decreta e eu sanciono a seguinte Lei:
02. (FCC/2012) Caracteriza-se Guarda Compartilhada
quando
Art. 1º Esta Lei dispõe sobre a alienação parental.
(A) a criança não tem residência fixa, ora permanecendo
com a mãe, ora permanecendo com o pai.
Art. 2º Considera-se ato de alienação parental a
(B) não há ruptura na habitualidade, pois um dos genitores
interferência na formação psicológica da criança ou do
permanece residindo com a criança e o outro tem total
adolescente promovida ou induzida por um dos genitores,
flexibilidade para participar da sua vida.
pelos avós ou pelos que tenham a criança ou adolescente sob a
(C) há a quebra da parentalidade, pois a criança não
sua autoridade, guarda ou vigilância para que repudie genitor
vivencia a continuidade dos relacionamentos com os
ou que cause prejuízo ao estabelecimento ou à manutenção de
genitores.
vínculos com este.
(D) a criança fica fixa em uma determinada residência e
Parágrafo único. São formas exemplificativas de alienação
seus genitores se revezam para estar com ela periodicamente.
parental, além dos atos assim declarados pelo juiz ou
(E) há definição de horários fixos distribuídos entre um e
constatados por perícia, praticados diretamente ou com
outro genitor por sentença judicial renovável a cada dois anos.
auxílio de terceiros:
I - realizar campanha de desqualificação da conduta do
03. (FCC) O aumento das separações conjugais e dos
genitor no exercício da paternidade ou maternidade;
divórcios trouxe questões novas às famílias e aos profissionais
II - dificultar o exercício da autoridade parental;
que as assistem, gerando também novos arranjos na atribuição
III - dificultar contato de criança ou adolescente com
da guarda de filho(s). A modalidade na qual os pais dividem a
genitor;
guarda de um ou mais filhos e ambos têm responsabilidade
IV - dificultar o exercício do direito regulamentado de
constante pelos cuidados a eles denomina-se guarda
convivência familiar;
(A) compartilhada.
V - omitir deliberadamente a genitor informações pessoais
(B) dividida.
relevantes sobre a criança ou adolescente, inclusive escolares,
(C) física.
médicas e alterações de endereço;
(D) mesclada.
VI - apresentar falsa denúncia contra genitor, contra
(E) única.
familiares deste ou contra avós, para obstar ou dificultar a
convivência deles com a criança ou adolescente;
04. (FGV/2015) Já foram proferidas pela justiça brasileira
VII - mudar o domicílio para local distante, sem
sentenças determinando que pais pagassem indenizações por
justificativa, visando a dificultar a convivência da criança ou
danos morais aos seus filhos. Por um lado, se a justiça tem a
adolescente com o outro genitor, com familiares deste ou com
tarefa de chamar a atenção para a importância da convivência
avós.
familiar, por outro lado, deve-se levar em conta que a
sociedade, a família e a legislação contribuíram historicamente
Art. 3º A prática de ato de alienação parental fere direito
para o afastamento paterno em relação à prole. Fazendo frente
fundamental da criança ou do adolescente de convivência
a esse elemento histórico, temos em nossas leis um expediente
familiar saudável, prejudica a realização de afeto nas relações
com genitor e com o grupo familiar, constitui abuso moral

Legislação 72
Apostila Digital Licenciada para Alice Caroline Guarino dos Santos - alice.guarino@hotmail.com (Proibida a Revenda)
APOSTILAS OPÇÃO

contra a criança ou o adolescente e descumprimento dos Art. 8º A alteração de domicílio da criança ou adolescente
deveres inerentes à autoridade parental ou decorrentes de é irrelevante para a determinação da competência relacionada
tutela ou guarda. às ações fundadas em direito de convivência familiar, salvo se
decorrente de consenso entre os genitores ou de decisão
Art. 4º Declarado indício de ato de alienação parental, a judicial.
requerimento ou de ofício, em qualquer momento processual,
em ação autônoma ou incidentalmente, o processo terá Art. 9º (VETADO)
tramitação prioritária, e o juiz determinará, com urgência,
ouvido o Ministério Público, as medidas provisórias Art. 10. (VETADO)
necessárias para preservação da integridade psicológica da
criança ou do adolescente, inclusive para assegurar sua Art. 11. Esta Lei entra em vigor na data de sua publicação.
convivência com genitor ou viabilizar a efetiva reaproximação
entre ambos, se for o caso. Questões
Parágrafo único. Assegurar-se-á à criança ou adolescente e
ao genitor garantia mínima de visitação assistida, ressalvados 01. (FCC) A Lei nº 12.318/10 dispôs, definitivamente, e
os casos em que há iminente risco de prejuízo à integridade com grande importância, sobre a alienação parental, que já era
física ou psicológica da criança ou do adolescente, atestado por muito debatida na doutrina e jurisprudência em nosso país.
profissional eventualmente designado pelo juiz para Especificamente sobre a alienação parental, é INCORRETO
acompanhamento das visitas. afirmar:
(A) Caracterizados atos típicos de alienação parental, em
Art. 5º Havendo indício da prática de ato de alienação ação autônoma ou incidental, o juiz poderá aplicar uma série
parental, em ação autônoma ou incidental, o juiz, se de medidas, cumulativamente ou não, para prevenir e inibir a
necessário, determinará perícia psicológica ou biopsicossocial. prática de atos de alienação parental, ou tolher-lhes a eficácia,
§ 1º O laudo pericial terá base em ampla avaliação sem prejuízo da responsabilização civil e criminal, mas não
psicológica ou biopsicossocial, conforme o caso, poderá estipular multa ao alienador.
compreendendo, inclusive, entrevista pessoal com as partes, (B) A alteração de domicílio da criança ou adolescente é
exame de documentos dos autos, histórico do relacionamento irrelevante para a determinação da competência relacionada
do casal e da separação, cronologia de incidentes, avaliação da às ações fundadas em direito de convivência familiar, salvo se
personalidade dos envolvidos e exame da forma como a decorrente de consenso entre os genitores ou de decisão
criança ou adolescente se manifesta acerca de eventual judicial.
acusação contra genitor. (C) A atribuição ou alteração da guarda dar-se-á por
§ 2º A perícia será realizada por profissional ou equipe preferência ao genitor que viabiliza a efetiva convivência da
multidisciplinar habilitados, exigido, em qualquer caso, criança ou adolescente com o outro genitor nas hipóteses em
aptidão comprovada por histórico profissional ou acadêmico que seja inviável a guarda compartilhada.
para diagnosticar atos de alienação parental. (D) A omissão deliberada a genitor de informações
§ 3º O perito ou equipe multidisciplinar designada para pessoais relevantes sobre a criança ou adolescente, inclusive
verificar a ocorrência de alienação parental terá prazo de 90 escolares, médicas e alterações de endereço, caracteriza ato de
(noventa) dias para apresentação do laudo, prorrogável alienação parental.
exclusivamente por autorização judicial baseada em (E) Havendo indício da prática de ato de alienação
justificativa circunstanciada. parental, em ação autônoma ou incidental, o juiz, se
necessário, determinará perícia psicológica ou biopsicossocial.
Art. 6º Caracterizados atos típicos de alienação parental ou
qualquer conduta que dificulte a convivência de criança ou 02. (VUNESP/2014) De acordo com a Lei n.º
adolescente com genitor, em ação autônoma ou incidental, o 12.318/2010, que dispõe sobre a alienação parental, assinale
juiz poderá, cumulativamente ou não, sem prejuízo da a alternativa correta.
decorrente responsabilidade civil ou criminal e da ampla (A) Em caso de mudança abusiva de endereço, pode o juiz
utilização de instrumentos processuais aptos a inibir ou determinar a inversão da obrigação de levar e buscar a criança
atenuar seus efeitos, segundo a gravidade do caso: ou adolescente da residência do alienador.
I - declarar a ocorrência de alienação parental e advertir o (B) A alienação parental abrange não apenas as
alienador; interferências na formação psicológica, mas também a
II - ampliar o regime de convivência familiar em favor do violência física para que a criança ou adolescente cumpra as
genitor alienado; ordens do genitor.
III - estipular multa ao alienador; (C) A lei determina que a alienação parental sancionável é
IV - determinar acompanhamento psicológico e/ou aquela praticada pelo genitor, avós e outros familiares, não se
biopsicossocial; estendendo ao tutor e outras pessoas que tenham a criança ou
V - determinar a alteração da guarda para guarda adolescente sob a sua autoridade.
compartilhada ou sua inversão; (D) A sanção de alteração da guarda da criança ou
VI - determinar a fixação cautelar do domicílio da criança adolescente não pode ser cumulada com a sanção de multa ao
ou adolescente; alienador, em razão da vedação à dupla sanção.
VII - declarar a suspensão da autoridade parental.
Parágrafo único. Caracterizado mudança abusiva de 03. Assinale a alternativa que apresenta uma afirmação
endereço, inviabilização ou obstrução à convivência familiar, o INCORRETA sobre alienação parental, considerando o
juiz também poderá inverter a obrigação de levar para ou disposto na Lei nº 12.318/2010.
retirar a criança ou adolescente da residência do genitor, por (A) Considera-se ato de alienação parental a interferência,
ocasião das alternâncias dos períodos de convivência familiar. que pode ser promovida ou induzida pelos avós, na formação
psicológica da criança ou do adolescente para que repudie
Art. 7º A atribuição ou alteração da guarda dar-se-á por genitor ou que cause prejuízo ao estabelecimento ou à
preferência ao genitor que viabiliza a efetiva convivência da manutenção de vínculos com este.
criança ou adolescente com o outro genitor nas hipóteses em (B) A Lei nº 12.318/2010 prevê, taxativamente, as formas
que seja inviável a guarda compartilhada. de alienação parental, descrevendo, dentre outras, omitir

Legislação 73
Apostila Digital Licenciada para Alice Caroline Guarino dos Santos - alice.guarino@hotmail.com (Proibida a Revenda)
APOSTILAS OPÇÃO

deliberadamente a genitor informações pessoais relevantes e) a garantia de 1 (um) salário-mínimo de benefício mensal
sobre a criança ou adolescente, inclusive escolares, médicas e à pessoa com deficiência e ao idoso que comprovem não
alterações de endereço. possuir meios de prover a própria manutenção ou de tê-la
(C) Caracterizados atos típicos de alienação parental o juiz provida por sua família;
poderá, segundo a gravidade do caso, declarar a suspensão da II - a vigilância socioassistencial, que visa a analisar
autoridade parental. territorialmente a capacidade protetiva das famílias e nela a
(D) Havendo indício da prática de ato de alienação ocorrência de vulnerabilidades, de ameaças, de vitimizações e
parental, o juiz, se necessário, determinará perícia psicológica danos;
ou biopsicossocial, caso em que o laudo deverá ser III - a defesa de direitos, que visa a garantir o pleno acesso
apresentado no prazo de 90 (noventa) dias, prorrogado aos direitos no conjunto das provisões socioassistenciais.
exclusivamente por autorização judicial baseada em Parágrafo único. Para o enfrentamento da pobreza, a
justificativa circunstanciada. assistência social realiza-se de forma integrada às políticas
(E) Declarado indício de ato de alienação parental, a setoriais, garantindo mínimos sociais e provimento de
requerimento ou de ofício, em qualquer momento processual, condições para atender contingências sociais e promovendo a
em ação autônoma ou incidentalmente, o processo terá universalização dos direitos sociais.” (NR)
tramitação prioritária. “Art. 3o Consideram-se entidades e organizações de
assistência social aquelas sem fins lucrativos que, isolada ou
04. (CESPE/2015) Com relação à alienação parental, cumulativamente, prestam atendimento e assessoramento aos
julgue o item subsequente. beneficiários abrangidos por esta Lei, bem como as que atuam
na defesa e garantia de direitos.
Segundo a Lei n.º 12.318/2010, o laudo pericial em casos § 1o São de atendimento aquelas entidades que, de forma
de alienação parental deve ser produzido em até sessenta dias continuada, permanente e planejada, prestam serviços,
e será fundamentado em avaliação psicológica, realizada com executam programas ou projetos e concedem benefícios de
testes validados pelo SATEPSI, após entrevista com um prestação social básica ou especial, dirigidos às famílias e
assistente social. indivíduos em situações de vulnerabilidade ou risco social e
(...) Certo pessoal, nos termos desta Lei, e respeitadas as deliberações do
(...) Errado Conselho Nacional de Assistência Social (CNAS), de que tratam
os incisos I e II do art. 18.
05. Acerca da Lei n. 12.318/10, julgue o item a seguir: § 2o São de assessoramento aquelas que, de forma
continuada, permanente e planejada, prestam serviços e
Assegurar-se-á à criança ou adolescente e ao genitor executam programas ou projetos voltados prioritariamente
garantia mínima de visitação assistida, ressalvados os casos para o fortalecimento dos movimentos sociais e das
em que há iminente risco de prejuízo à integridade física ou organizações de usuários, formação e capacitação de
psicológica da criança ou do adolescente, atestado por lideranças, dirigidos ao público da política de assistência
profissional eventualmente designado pelo juiz para social, nos termos desta Lei, e respeitadas as deliberações do
acompanhamento das visitas. CNAS, de que tratam os incisos I e II do art. 18.
(...) Certo § 3o São de defesa e garantia de direitos aquelas que, de
(...) Errado forma continuada, permanente e planejada, prestam serviços
e executam programas e projetos voltados prioritariamente
Respostas para a defesa e efetivação dos direitos socioassistenciais,
construção de novos direitos, promoção da cidadania,
01. A. / 02. A. / 03. B. / 04. Errado. / 05. Certo. enfrentamento das desigualdades sociais, articulação com
órgãos públicos de defesa de direitos, dirigidos ao público da
política de assistência social, nos termos desta Lei, e
SISTEMA ÚNICO DE respeitadas as deliberações do CNAS, de que tratam os incisos
I e II do art. 18.” (NR)
ASSISTÊNCIA SOCIAL – “Art. 6o A gestão das ações na área de assistência social fica
SUAS – Lei 12.435/2011. organizada sob a forma de sistema descentralizado e
participativo, denominado Sistema Único de Assistência Social
(Suas), com os seguintes objetivos:
I - consolidar a gestão compartilhada, o cofinanciamento e
LEI Nº 12.435, DE 6 DE JULHO DE 2011.
a cooperação técnica entre os entes federativos que, de modo
articulado, operam a proteção social não contributiva;
Altera a Lei nº 8.742, de 7 de dezembro de 1993, que
II - integrar a rede pública e privada de serviços,
dispõe sobre a organização da Assistência Social
programas, projetos e benefícios de assistência social, na
forma do art. 6o-C;
A PRESIDENTA DA REPÚBLICA Faço saber que o
III - estabelecer as responsabilidades dos entes federativos
Congresso Nacional decreta e eu sanciono a seguinte Lei:
na organização, regulação, manutenção e expansão das ações
de assistência social;
Art. 1o Os arts. 2o, 3o, 6o, 12, 13, 14, 15, 16, 17, 20, 21, 22,
IV - definir os níveis de gestão, respeitadas as diversidades
23, 24, 28 e 36 da Lei no 8.742, de 7 de dezembro de 1993,
regionais e municipais;
passam a vigorar com a seguinte redação:
V - implementar a gestão do trabalho e a educação
“Art. 2o A assistência social tem por objetivos:
permanente na assistência social;
I - a proteção social, que visa à garantia da vida, à redução
VI - estabelecer a gestão integrada de serviços e benefícios;
de danos e à prevenção da incidência de riscos, especialmente:
e
a) a proteção à família, à maternidade, à infância, à
VII - afiançar a vigilância socioassistencial e a garantia de
adolescência e à velhice;
direitos.
b) o amparo às crianças e aos adolescentes carentes;
§ 1o As ações ofertadas no âmbito do Suas têm por objetivo
c) a promoção da integração ao mercado de trabalho;
a proteção à família, à maternidade, à infância, à adolescência
d) a habilitação e reabilitação das pessoas com deficiência
e à velhice e, como base de organização, o território.
e a promoção de sua integração à vida comunitária; e

Legislação 74
Apostila Digital Licenciada para Alice Caroline Guarino dos Santos - alice.guarino@hotmail.com (Proibida a Revenda)
APOSTILAS OPÇÃO

§ 2o O Suas é integrado pelos entes federativos, pelos idoso com 65 (sessenta e cinco) anos ou mais que comprovem
respectivos conselhos de assistência social e pelas entidades e não possuir meios de prover a própria manutenção nem de tê-
organizações de assistência social abrangidas por esta Lei. la provida por sua família.
§ 3o A instância coordenadora da Política Nacional de § 1o Para os efeitos do disposto no caput, a família é
Assistência Social é o Ministério do Desenvolvimento Social e composta pelo requerente, o cônjuge ou companheiro, os pais
Combate à Fome.” (NR) e, na ausência de um deles, a madrasta ou o padrasto, os
“Art. 12. ....................................................................... irmãos solteiros, os filhos e enteados solteiros e os menores
............................................................................................. tutelados, desde que vivam sob o mesmo teto.
II - cofinanciar, por meio de transferência automática, o § 2o Para efeito de concessão deste benefício, considera-se:
aprimoramento da gestão, os serviços, os programas e os I - pessoa com deficiência: aquela que tem impedimentos
projetos de assistência social em âmbito nacional; de longo prazo de natureza física, intelectual ou sensorial, os
............................................................................................. quais, em interação com diversas barreiras, podem obstruir
IV - realizar o monitoramento e a avaliação da política de sua participação plena e efetiva na sociedade com as demais
assistência social e assessorar Estados, Distrito Federal e pessoas;
Municípios para seu desenvolvimento.” (NR) II - impedimentos de longo prazo: aqueles que incapacitam
“Art. 13. .......................................................................... a pessoa com deficiência para a vida independente e para o
I - destinar recursos financeiros aos Municípios, a título de trabalho pelo prazo mínimo de 2 (dois) anos.
participação no custeio do pagamento dos benefícios § 3o Considera-se incapaz de prover a manutenção da
eventuais de que trata o art. 22, mediante critérios pessoa com deficiência ou idosa a família cuja renda mensal
estabelecidos pelos Conselhos Estaduais de Assistência Social; per capita seja inferior a 1/4 (um quarto) do salário-mínimo.
II - cofinanciar, por meio de transferência automática, o § 4o O benefício de que trata este artigo não pode ser
aprimoramento da gestão, os serviços, os programas e os acumulado pelo beneficiário com qualquer outro no âmbito da
projetos de assistência social em âmbito regional ou local; seguridade social ou de outro regime, salvo os da assistência
............................................................................................. médica e da pensão especial de natureza indenizatória.
VI - realizar o monitoramento e a avaliação da política de § 5o A condição de acolhimento em instituições de longa
assistência social e assessorar os Municípios para seu permanência não prejudica o direito do idoso ou da pessoa
desenvolvimento.” (NR) com deficiência ao benefício de prestação continuada.
“Art. 14. .......................................................................... § 6o A concessão do benefício ficará sujeita à avaliação da
I - destinar recursos financeiros para custeio do deficiência e do grau de incapacidade, composta por avaliação
pagamento dos benefícios eventuais de que trata o art. 22, médica e avaliação social realizadas por médicos peritos e por
mediante critérios estabelecidos pelos Conselhos de assistentes sociais do Instituto Nacional do Seguro Social
Assistência Social do Distrito Federal; (INSS).
............................................................................................. ...................................................................................” (NR)
VI - cofinanciar o aprimoramento da gestão, os serviços, os “Art. 21. ........................................................................
programas e os projetos de assistência social em âmbito local; .............................................................................................
VII - realizar o monitoramento e a avaliação da política de § 3o O desenvolvimento das capacidades cognitivas,
assistência social em seu âmbito.” (NR) motoras ou educacionais e a realização de atividades não
“Art. 15. ......................................................................... remuneradas de habilitação e reabilitação, entre outras, não
I - destinar recursos financeiros para custeio do constituem motivo de suspensão ou cessação do benefício da
pagamento dos benefícios eventuais de que trata o art. 22, pessoa com deficiência.
mediante critérios estabelecidos pelos Conselhos Municipais § 4o A cessação do benefício de prestação continuada
de Assistência Social; concedido à pessoa com deficiência, inclusive em razão do seu
............................................................................................. ingresso no mercado de trabalho, não impede nova concessão
VI - cofinanciar o aprimoramento da gestão, os serviços, os do benefício, desde que atendidos os requisitos definidos em
programas e os projetos de assistência social em âmbito local; regulamento.” (NR)
VII - realizar o monitoramento e a avaliação da política de “Art. 22. Entendem-se por benefícios eventuais as
assistência social em seu âmbito.” (NR) provisões suplementares e provisórias que integram
“Art. 16. As instâncias deliberativas do Suas, de caráter organicamente as garantias do Suas e são prestadas aos
permanente e composição paritária entre governo e sociedade cidadãos e às famílias em virtude de nascimento, morte,
civil, são: situações de vulnerabilidade temporária e de calamidade
............................................................................................. pública.
Parágrafo único. Os Conselhos de Assistência Social estão § 1o A concessão e o valor dos benefícios de que trata este
vinculados ao órgão gestor de assistência social, que deve artigo serão definidos pelos Estados, Distrito Federal e
prover a infraestrutura necessária ao seu funcionamento, Municípios e previstos nas respectivas leis orçamentárias
garantindo recursos materiais, humanos e financeiros, anuais, com base em critérios e prazos definidos pelos
inclusive com despesas referentes a passagens e diárias de respectivos Conselhos de Assistência Social.
conselheiros representantes do governo ou da sociedade civil, § 2o O CNAS, ouvidas as respectivas representações de
quando estiverem no exercício de suas atribuições.” (NR) Estados e Municípios dele participantes, poderá propor, na
“Art. 17. ....................................................................... medida das disponibilidades orçamentárias das 3 (três)
............................................................................................. esferas de governo, a instituição de benefícios subsidiários no
§ 4o Os Conselhos de que tratam os incisos II, III e IV do art. valor de até 25% (vinte e cinco por cento) do salário-mínimo
16, com competência para acompanhar a execução da política para cada criança de até 6 (seis) anos de idade.
de assistência social, apreciar e aprovar a proposta § 3o Os benefícios eventuais subsidiários não poderão ser
orçamentária, em consonância com as diretrizes das cumulados com aqueles instituídos pelas Leis no 10.954, de 29
conferências nacionais, estaduais, distrital e municipais, de de setembro de 2004, e no 10.458, de 14 de maio de 2002.”
acordo com seu âmbito de atuação, deverão ser instituídos, (NR)
respectivamente, pelos Estados, pelo Distrito Federal e pelos “Art. 23. Entendem-se por serviços socioassistenciais as
Municípios, mediante lei específica.” (NR) atividades continuadas que visem à melhoria de vida da
“Art. 20. O benefício de prestação continuada é a garantia população e cujas ações, voltadas para as necessidades
de um salário-mínimo mensal à pessoa com deficiência e ao

Legislação 75
Apostila Digital Licenciada para Alice Caroline Guarino dos Santos - alice.guarino@hotmail.com (Proibida a Revenda)
APOSTILAS OPÇÃO

básicas, observem os objetivos, princípios e diretrizes III - integrar o sistema de cadastro de entidades de que
estabelecidos nesta Lei. trata o inciso XI do art. 19.
§ 1o O regulamento instituirá os serviços § 3o As entidades e organizações de assistência social
socioassistenciais. vinculadas ao Suas celebrarão convênios, contratos, acordos
§ 2o Na organização dos serviços da assistência social serão ou ajustes com o poder público para a execução, garantido
criados programas de amparo, entre outros: financiamento integral, pelo Estado, de serviços, programas,
I - às crianças e adolescentes em situação de risco pessoal projetos e ações de assistência social, nos limites da
e social, em cumprimento ao disposto no art. 227 da capacidade instalada, aos beneficiários abrangidos por esta
Constituição Federal e na Lei no 8.069, de 13 de julho de 1990 Lei, observando-se as disponibilidades orçamentárias.
(Estatuto da Criança e do Adolescente); § 4o O cumprimento do disposto no § 3o será informado ao
II - às pessoas que vivem em situação de rua.” (NR) Ministério do Desenvolvimento Social e Combate à Fome pelo
“Art. 24. ........................................................................ órgão gestor local da assistência social.”
............................................................................................. “Art. 6º-C. As proteções sociais, básica e especial, serão
§ 2o Os programas voltados para o idoso e a integração da ofertadas precipuamente no Centro de Referência de
pessoa com deficiência serão devidamente articulados com o Assistência Social (Cras) e no Centro de Referência
benefício de prestação continuada estabelecido no art. 20 Especializado de Assistência Social (Creas), respectivamente,
desta Lei.” (NR) e pelas entidades sem fins lucrativos de assistência social de
“Art. 28. .......................................................................... que trata o art. 3o desta Lei.
§ 1o Cabe ao órgão da Administração Pública responsável § 1o O Cras é a unidade pública municipal, de base
pela coordenação da Política de Assistência Social nas 3 (três) territorial, localizada em áreas com maiores índices de
esferas de governo gerir o Fundo de Assistência Social, sob vulnerabilidade e risco social, destinada à articulação dos
orientação e controle dos respectivos Conselhos de Assistência serviços socioassistenciais no seu território de abrangência e
Social. à prestação de serviços, programas e projetos
............................................................................................. socioassistenciais de proteção social básica às famílias.
§ 3o O financiamento da assistência social no Suas deve ser § 2o O Creas é a unidade pública de abrangência e gestão
efetuado mediante cofinanciamento dos 3 (três) entes municipal, estadual ou regional, destinada à prestação de
federados, devendo os recursos alocados nos fundos de serviços a indivíduos e famílias que se encontram em situação
assistência social ser voltados à operacionalização, prestação, de risco pessoal ou social, por violação de direitos ou
aprimoramento e viabilização dos serviços, programas, contingência, que demandam intervenções especializadas da
projetos e benefícios desta política.” (NR) proteção social especial.
“Art. 36. As entidades e organizações de assistência social § 3o Os Cras e os Creas são unidades públicas estatais
que incorrerem em irregularidades na aplicação dos recursos instituídas no âmbito do Suas, que possuem interface com as
que lhes foram repassados pelos poderes públicos terão a sua demais políticas públicas e articulam, coordenam e ofertam os
vinculação ao Suas cancelada, sem prejuízo de serviços, programas, projetos e benefícios da assistência
responsabilidade civil e penal.” (NR) social.”
“Art. 6º-D. As instalações dos Cras e dos Creas devem ser
Art. 2o A Lei no 8.742, de 1993, passa a vigorar acrescida compatíveis com os serviços neles ofertados, com espaços
dos seguintes artigos: para trabalhos em grupo e ambientes específicos para
“Art. 6o-A. A assistência social organiza-se pelos seguintes recepção e atendimento reservado das famílias e indivíduos,
tipos de proteção: assegurada a acessibilidade às pessoas idosas e com
I - proteção social básica: conjunto de serviços, programas, deficiência.”
projetos e benefícios da assistência social que visa a prevenir “Art. 6º-E. Os recursos do cofinanciamento do Suas,
situações de vulnerabilidade e risco social por meio do destinados à execução das ações continuadas de assistência
desenvolvimento de potencialidades e aquisições e do social, poderão ser aplicados no pagamento dos profissionais
fortalecimento de vínculos familiares e comunitários; que integrarem as equipes de referência, responsáveis pela
II - proteção social especial: conjunto de serviços, organização e oferta daquelas ações, conforme percentual
programas e projetos que tem por objetivo contribuir para a apresentado pelo Ministério do Desenvolvimento Social e
reconstrução de vínculos familiares e comunitários, a defesa Combate à Fome e aprovado pelo CNAS.
de direito, o fortalecimento das potencialidades e aquisições e Parágrafo único. A formação das equipes de referência
a proteção de famílias e indivíduos para o enfrentamento das deverá considerar o número de famílias e indivíduos
situações de violação de direitos. referenciados, os tipos e modalidades de atendimento e as
Parágrafo único. A vigilância socioassistencial é um dos aquisições que devem ser garantidas aos usuários, conforme
instrumentos das proteções da assistência social que identifica deliberações do CNAS.”
e previne as situações de risco e vulnerabilidade social e seus “Art. 12-A. A União apoiará financeiramente o
agravos no território.” aprimoramento à gestão descentralizada dos serviços,
“Art. 6º-B. As proteções sociais básica e especial serão programas, projetos e benefícios de assistência social, por
ofertadas pela rede socioassistencial, de forma integrada, meio do Índice de Gestão Descentralizada (IGD) do Sistema
diretamente pelos entes públicos e/ou pelas entidades e Único de Assistência Social (Suas), para a utilização no âmbito
organizações de assistência social vinculadas ao Suas, dos Estados, dos Municípios e do Distrito Federal, destinado,
respeitadas as especificidades de cada ação. sem prejuízo de outras ações a serem definidas em
§ 1o A vinculação ao Suas é o reconhecimento pelo regulamento, a:
Ministério do Desenvolvimento Social e Combate à Fome de I - medir os resultados da gestão descentralizada do Suas,
que a entidade de assistência social integra a rede com base na atuação do gestor estadual, municipal e do
socioassistencial. Distrito Federal na implementação, execução e
§ 2o Para o reconhecimento referido no § 1o, a entidade monitoramento dos serviços, programas, projetos e benefícios
deverá cumprir os seguintes requisitos: de assistência social, bem como na articulação intersetorial;
I - constituir-se em conformidade com o disposto no art. 3o; II - incentivar a obtenção de resultados qualitativos na
II - inscrever-se em Conselho Municipal ou do Distrito gestão estadual, municipal e do Distrito Federal do Suas; e
Federal, na forma do art. 9o;

Legislação 76
Apostila Digital Licenciada para Alice Caroline Guarino dos Santos - alice.guarino@hotmail.com (Proibida a Revenda)
APOSTILAS OPÇÃO

III - calcular o montante de recursos a serem repassados pública com a seguridade social, na forma do art. 24 da Lei
aos entes federados a título de apoio financeiro à gestão do Complementar no 101, de 4 de maio de 2000.”
Suas. “Art. 30-B. Caberá ao ente federado responsável pela
§ 1o Os resultados alcançados pelo ente federado na gestão utilização dos recursos do respectivo Fundo de Assistência
do Suas, aferidos na forma de regulamento, serão Social o controle e o acompanhamento dos serviços,
considerados como prestação de contas dos recursos a serem programas, projetos e benefícios, por meio dos respectivos
transferidos a título de apoio financeiro. órgãos de controle, independentemente de ações do órgão
§ 2o As transferências para apoio à gestão descentralizada repassador dos recursos.”
do Suas adotarão a sistemática do Índice de Gestão “Art. 30-C. A utilização dos recursos federais
Descentralizada do Programa Bolsa Família, previsto no art. 8o descentralizados para os fundos de assistência social dos
da Lei no 10.836, de 9 de janeiro de 2004, e serão efetivadas Estados, dos Municípios e do Distrito Federal será declarada
por meio de procedimento integrado àquele índice. pelos entes recebedores ao ente transferidor, anualmente,
§ 3o (VETADO). mediante relatório de gestão submetido à apreciação do
§ 4o Para fins de fortalecimento dos Conselhos de respectivo Conselho de Assistência Social, que comprove a
Assistência Social dos Estados, Municípios e Distrito Federal, execução das ações na forma de regulamento.
percentual dos recursos transferidos deverá ser gasto com Parágrafo único. Os entes transferidores poderão
atividades de apoio técnico e operacional àqueles colegiados, requisitar informações referentes à aplicação dos recursos
na forma fixada pelo Ministério do Desenvolvimento Social e oriundos do seu fundo de assistência social, para fins de
Combate à Fome, sendo vedada a utilização dos recursos para análise e acompanhamento de sua boa e regular utilização.”
pagamento de pessoal efetivo e de gratificações de qualquer
natureza a servidor público estadual, municipal ou do Distrito Art. 3o Revoga-se o art. 38 da Lei nº 8.742, de 7 de
Federal.” dezembro de 1993.
“Art. 24-A. Fica instituído o Serviço de Proteção e
Atendimento Integral à Família (Paif), que integra a proteção Art. 4o Esta Lei entra em vigor na data de sua publicação.
social básica e consiste na oferta de ações e serviços
socioassistenciais de prestação continuada, nos Cras, por meio Questões
do trabalho social com famílias em situação de vulnerabilidade
social, com o objetivo de prevenir o rompimento dos vínculos 01. (TRT - 15ª Região - Analista Judiciário - Serviço
familiares e a violência no âmbito de suas relações, garantindo Social - FCC) O Programa Erradicação do Trabalho Infantil - -
o direito à convivência familiar e comunitária. PETI, instituído pela Lei nº 12.435/11, tem
Parágrafo único. Regulamento definirá as diretrizes e os (A) vinculação exclusiva à Secretaria Nacional de Direitos
procedimentos do Paif.” Humanos, da Presidência da República e opera uma bolsa
“Art. 24-B. Fica instituído o Serviço de Proteção e destinada a crianças e adolescentes com idade inferior a 16
Atendimento Especializado a Famílias e Indivíduos (Paefi), (dezesseis) anos em situação de trabalho, ressalvada a
que integra a proteção social especial e consiste no apoio, condição de aprendiz, a partir de 14 (quatorze) anos.
orientação e acompanhamento a famílias e indivíduos em (B) abrangência Nacional e será desenvolvido de forma
situação de ameaça ou violação de direitos, articulando os articulada pelos entes federados, com a participação da
serviços socioassistenciais com as diversas políticas públicas e sociedade civil, e tem como objetivo contribuir para a retirada
com órgãos do sistema de garantia de direitos. de crianças e adolescentes com idade inferior a 16 (dezesseis)
Parágrafo único. Regulamento definirá as diretrizes e os anos em situação de trabalho, ressalvada a condição de
procedimentos do Paefi.” aprendiz, a partir de 14 (quatorze) anos.
“Art. 24-C. Fica instituído o Programa de Erradicação do (C) abrangência Nacional e será desenvolvido de forma
Trabalho Infantil (Peti), de caráter intersetorial, integrante da articulada pelos entes federados, com a participação da
Política Nacional de Assistência Social, que, no âmbito do Suas, sociedade civil, e tem como objetivo contribuir para a retirada
compreende transferências de renda, trabalho social com de crianças e adolescentes com idade inferior a 14 (quatorze)
famílias e oferta de serviços socioeducativos para crianças e anos em situação de trabalho, ressalvada a condição de
adolescentes que se encontrem em situação de trabalho. aprendiz, a partir de 12 (doze) anos.
§ 1o O Peti tem abrangência nacional e será desenvolvido (D) transferência de renda coordenado pelas Secretarias
de forma articulada pelos entes federados, com a participação Estaduais de Assistência Social e pela Secretaria Estadual do
da sociedade civil, e tem como objetivo contribuir para a Distrito Federal, com o objetivo de superar a exploração do
retirada de crianças e adolescentes com idade inferior a 16 trabalho infantil nos casos de insalubridade.
(dezesseis) anos em situação de trabalho, ressalvada a (E) caráter intersetorial e integra benefícios e serviços
condição de aprendiz, a partir de 14 (quatorze) anos. destinados a crianças e adolescentes em situação de trabalho
§ 2o As crianças e os adolescentes em situação de trabalho com idade mínima de 18 anos; no entanto, com a criação do
deverão ser identificados e ter os seus dados inseridos no Programa Bolsa Família o mesmo foi extinto, pois o foco dado
Cadastro Único para Programas Sociais do Governo Federal às políticas sociais a partir de 2003 é na matricialidade
(CadÚnico), com a devida identificação das situações de sociofamiliar.
trabalho infantil.”
“Art. 30-A. O cofinanciamento dos serviços, programas, 02. (TRT - 15ª Região - Analista Judiciário - Serviço
projetos e benefícios eventuais, no que couber, e o Social - FCC) Atuando como Assistente Social foram
aprimoramento da gestão da política de assistência social no solicitadas, a você, informações referentes ao Benefício de
Suas se efetuam por meio de transferências automáticas entre Prestação Continuada, sobretudo de como se deve entender a
os fundos de assistência social e mediante alocação de composição familiar referente à renda. Conforme o disposto na
recursos próprios nesses fundos nas 3 (três) esferas de Lei nº 12.435/11, a família
governo. (A) é composta por todos aqueles que residem sob o
Parágrafo único. As transferências automáticas de mesmo teto, excetuando o requerente que pode ter outro
recursos entre os fundos de assistência social efetuadas à endereço residencial, uma vez que a requisição do benefício
conta do orçamento da seguridade social, conforme o art. 204 pode ser induzida por qualquer pessoa da família.
da Constituição Federal, caracterizam-se como despesa (B) é composta pelo requerente, o cônjuge ou
companheiro, os pais e, na ausência de um deles, a madrasta

Legislação 77
Apostila Digital Licenciada para Alice Caroline Guarino dos Santos - alice.guarino@hotmail.com (Proibida a Revenda)
APOSTILAS OPÇÃO

ou o padrasto, os irmãos solteiros, os filhos e enteados Respostas


solteiros e os menores tutelados, sem que necessariamente
vivam sob o mesmo teto. 01. B. / 02. D. / 03. E. / 04. A. / 05. E.
(C) está estabelecida no código civil - Lei nº 10.406/2002,
e é composta por pais e filhos reconhecidos por instrumento
legal perante a justiça (registros de casamento e nascimento SISTEMA NACIONAL DE
com paternidade comprovada).
(D) é composta pelo requerente, o cônjuge ou ATENDIMENTO
companheiro, os pais e, na ausência de um deles, a madrasta SOCIOEDUCATIVO – Lei
ou o padrasto, os irmãos solteiros, os filhos e enteados 12.594/2012.
solteiros e os menores tutelados, desde que vivam sob o
mesmo teto.
(E) é composta por todos os membros que vivem sob o
mesmo teto, podendo ser considerados também os irmãos LEI Nº 12.594, DE 18 DE JANEIRO DE 2012.
casados e, neste caso não há necessidade de incorporação da
renda dos mesmos no cálculo per capita. Institui o Sistema Nacional de Atendimento Socioeducativo
(Sinase), regulamenta a execução das medidas
03. (FUNDAÇÃO CASA - Analista Técnico – Assistente socioeducativas destinadas a adolescente que pratique ato
Social - VUNESP) Conforme previsões contidas na LOAS, infracional; e altera as Leis nos 8.069, de 13 de julho de 1990
artigo 3.º (redação dada pela Lei n.º 12.435/11), consideram- (Estatuto da Criança e do Adolescente); 7.560, de 19 de
se entidades e organizações de assistência social aquelas sem dezembro de 1986, 7.998, de 11 de janeiro de 1990, 5.537, de
fins lucrativos que, isolada ou cumulativamente, prestam 21 de novembro de 1968, 8.315, de 23 de dezembro de 1991,
atendimento e _______ aos beneficiários abrangidos por esta Lei, 8.706, de 14 de setembro de 1993, os Decretos-Leis nos 4.048,
bem como as que atuam na _________ e garantia de direitos. de 22 de janeiro de 1942, 8.621, de 10 de janeiro de 1946, e a
Consolidação das Leis do Trabalho (CLT), aprovada pelo
Assinale a alternativa que completa, correta e Decreto-Lei no 5.452, de 1o de maio de 1943.
respectivamente, as lacunas do texto
(A) consultoria … certificação A PRESIDENTA DA REPÚBLICA Faço saber que o
(B) acompanhamento … revogação Congresso Nacional decreta e eu sanciono a seguinte Lei:
(C) orientação … definição
(D) encaminhamento … revisão TÍTULO I
(E) assessoramento … defesa DO SISTEMA NACIONAL DE ATENDIMENTO
SOCIOEDUCATIVO (Sinase)
04. (TRT - 6ª Região (PE) - Analista Judiciário - Serviço CAPÍTULO I
Social - FCC) O benefício de prestação continuada é a garantia DISPOSIÇÕES GERAIS
de um salário-mínimo mensal à pessoa com deficiência e ao
idoso com 65 (sessenta e cinco) anos ou mais que comprovem Art. 1º Esta Lei institui o Sistema Nacional de Atendimento
não possuir meios de prover a própria manutenção nem de tê- Socioeducativo (Sinase) e regulamenta a execução das
la provida por sua família. Conforme art. 20 da Lei nº medidas destinadas a adolescente que pratique ato infracional.
12.435/11, devem ser considerados na composição familiar, § 1º Entende-se por Sinase o conjunto ordenado de
desde que vivam sob o mesmo teto, os princípios, regras e critérios que envolvem a execução de
(A) irmãos solteiros. medidas socioeducativas, incluindo-se nele, por adesão, os
(B) filhos casados. sistemas estaduais, distrital e municipais, bem como todos os
(C) enteados casados. planos, políticas e programas específicos de atendimento a
(D) irmãos casados. adolescente em conflito com a lei.
(E) cunhados casados. § 2º Entendem-se por medidas socioeducativas as
previstas no art. 112 da Lei no 8.069, de 13 de julho de 1990
05. (MPE-RO - Analista - Assistência Social - FUNCAB) (Estatuto da Criança e do Adolescente), as quais têm por
Para os efeitos do disposto no da Lei nº 12.435, de 06 de julho objetivos:
de 2011, a família: I - a responsabilização do adolescente quanto às
(A) é definida pelos membros que compõem o núcleo consequências lesivas do ato infracional, sempre que possível
residencial, restrito aos que possuam afinidade de incentivando a sua reparação;
consanguinidade e que convivam na mesma casa. II - a integração social do adolescente e a garantia de seus
(B) é restrita aos membros da família nuclear, pai, mãe e direitos individuais e sociais, por meio do cumprimento de seu
filhos, desde que residam sob o mesmo teto. plano individual de atendimento; e
(C) é formada pelos arranjos familiares que incluem a III - a desaprovação da conduta infracional, efetivando as
família nuclear e a extensa, como também o requerente e todos disposições da sentença como parâmetro máximo de privação
aqueles que estejam solicitando os benefícios no mesmo de liberdade ou restrição de direitos, observados os limites
processo. previstos em lei.
(D) é constituída por pai, mãe, filhos e avós, desde que § 3º Entendem-se por programa de atendimento a
residam na mesma casa, e enteados, caso estes estejam sob a organização e o funcionamento, por unidade, das condições
guarda legal de um dos requerentes. necessárias para o cumprimento das medidas socioeducativas.
(E) é composta pelo requerente, o cônjuge ou § 4º Entende-se por unidade a base física necessária para
companheiro, os pais e, na ausência de um deles, a madrasta a organização e o funcionamento de programa de
ou o padrasto, os irmãos solteiros, os filhos e enteados atendimento.
solteiros e os menores tutelados, desde que vivam sob o § 5º Entendem-se por entidade de atendimento a pessoa
mesmo teto. jurídica de direito público ou privado que instala e mantém a
unidade e os recursos humanos e materiais necessários ao
desenvolvimento de programas de atendimento.

Legislação 78
Apostila Digital Licenciada para Alice Caroline Guarino dos Santos - alice.guarino@hotmail.com (Proibida a Revenda)
APOSTILAS OPÇÃO

Art. 2º O Sinase será coordenado pela União e integrado da Lei no 8.069, de 13 de julho de 1990 (Estatuto da Criança e
pelos sistemas estaduais, distrital e municipais responsáveis do Adolescente);
pela implementação dos seus respectivos programas de VIII - garantir defesa técnica do adolescente a quem se
atendimento a adolescente ao qual seja aplicada medida atribua prática de ato infracional;
socioeducativa, com liberdade de organização e IX - cadastrar-se no Sistema Nacional de Informações
funcionamento, respeitados os termos desta Lei. sobre o Atendimento Socioeducativo e fornecer regularmente
os dados necessários ao povoamento e à atualização do
CAPÍTULO II Sistema; e
DAS COMPETÊNCIAS X - cofinanciar, com os demais entes federados, a execução
de programas e ações destinados ao atendimento inicial de
Art. 3º Compete à União: adolescente apreendido para apuração de ato infracional, bem
I - formular e coordenar a execução da política nacional de como aqueles destinados a adolescente a quem foi aplicada
atendimento socioeducativo; medida socioeducativa privativa de liberdade.
II - elaborar o Plano Nacional de Atendimento § 1º Ao Conselho Estadual dos Direitos da Criança e do
Socioeducativo, em parceria com os Estados, o Distrito Federal Adolescente competem as funções deliberativas e de controle
e os Municípios; do Sistema Estadual de Atendimento Socioeducativo, nos
III - prestar assistência técnica e suplementação financeira termos previstos no inciso II do art. 88 da Lei nº 8.069, de 13
aos Estados, ao Distrito Federal e aos Municípios para o de julho de 1990 (Estatuto da Criança e do Adolescente), bem
desenvolvimento de seus sistemas; como outras definidas na legislação estadual ou distrital.
IV - instituir e manter o Sistema Nacional de Informações § 2º O Plano de que trata o inciso II do caput deste artigo
sobre o Atendimento Socioeducativo, seu funcionamento, será submetido à deliberação do Conselho Estadual dos
entidades, programas, incluindo dados relativos a Direitos da Criança e do Adolescente.
financiamento e população atendida; § 3º Competem ao órgão a ser designado no Plano de que
V - contribuir para a qualificação e ação em rede dos trata o inciso II do caput deste artigo as funções executiva e de
Sistemas de Atendimento Socioeducativo; gestão do Sistema Estadual de Atendimento Socioeducativo.
VI - estabelecer diretrizes sobre a organização e
funcionamento das unidades e programas de atendimento e as Art. 5º Compete aos Municípios:
normas de referência destinadas ao cumprimento das medidas I - formular, instituir, coordenar e manter o Sistema
socioeducativas de internação e semiliberdade; Municipal de Atendimento Socioeducativo, respeitadas as
VII - instituir e manter processo de avaliação dos Sistemas diretrizes fixadas pela União e pelo respectivo Estado;
de Atendimento Socioeducativo, seus planos, entidades e II - elaborar o Plano Municipal de Atendimento
programas; Socioeducativo, em conformidade com o Plano Nacional e o
VIII - financiar, com os demais entes federados, a execução respectivo Plano Estadual;
de programas e serviços do Sinase; e III - criar e manter programas de atendimento para a
IX - garantir a publicidade de informações sobre repasses execução das medidas socioeducativas em meio aberto;
de recursos aos gestores estaduais, distrital e municipais, para IV - editar normas complementares para a organização e
financiamento de programas de atendimento socioeducativo. funcionamento dos programas do seu Sistema de Atendimento
§ 1º São vedados à União o desenvolvimento e a oferta de Socioeducativo;
programas próprios de atendimento. V - cadastrar-se no Sistema Nacional de Informações sobre
§ 2º Ao Conselho Nacional dos Direitos da Criança e do o Atendimento Socioeducativo e fornecer regularmente os
Adolescente (Conanda) competem as funções normativa, dados necessários ao povoamento e à atualização do Sistema;
deliberativa, de avaliação e de fiscalização do Sinase, nos e
termos previstos na Lei no 8.242, de 12 de outubro de 1991, VI - cofinanciar, conjuntamente com os demais entes
que cria o referido Conselho. federados, a execução de programas e ações destinados ao
§ 3º O Plano de que trata o inciso II do caput deste artigo atendimento inicial de adolescente apreendido para apuração
será submetido à deliberação do Conanda. de ato infracional, bem como aqueles destinados a adolescente
§ 4º À Secretaria de Direitos Humanos da Presidência da a quem foi aplicada medida socioeducativa em meio aberto.
República (SDH/PR) competem as funções executiva e de § 1º Para garantir a oferta de programa de atendimento
gestão do Sinase. socioeducativo de meio aberto, os Municípios podem instituir
os consórcios dos quais trata a Lei no 11.107, de 6 de abril de
Art. 4º Compete aos Estados: 2005, que dispõe sobre normas gerais de contratação de
I - formular, instituir, coordenar e manter Sistema Estadual consórcios públicos e dá outras providências, ou qualquer
de Atendimento Socioeducativo, respeitadas as diretrizes outro instrumento jurídico adequado, como forma de
fixadas pela União; compartilhar responsabilidades.
II - elaborar o Plano Estadual de Atendimento § 2º Ao Conselho Municipal dos Direitos da Criança e do
Socioeducativo em conformidade com o Plano Nacional; Adolescente competem as funções deliberativas e de controle
III - criar, desenvolver e manter programas para a do Sistema Municipal de Atendimento Socioeducativo, nos
execução das medidas socioeducativas de semiliberdade e termos previstos no inciso II do art. 88 da Lei nº 8.069, de 13
internação; de julho de 1990 (Estatuto da Criança e do Adolescente), bem
IV - editar normas complementares para a organização e como outras definidas na legislação municipal.
funcionamento do seu sistema de atendimento e dos sistemas § 3º O Plano de que trata o inciso II do caput deste artigo
municipais; será submetido à deliberação do Conselho Municipal dos
V - estabelecer com os Municípios formas de colaboração Direitos da Criança e do Adolescente.
para o atendimento socioeducativo em meio aberto; § 4º Competem ao órgão a ser designado no Plano de que
VI - prestar assessoria técnica e suplementação financeira trata o inciso II do caput deste artigo as funções executiva e de
aos Municípios para a oferta regular de programas de meio gestão do Sistema Municipal de Atendimento Socioeducativo.
aberto;
VII - garantir o pleno funcionamento do plantão Art. 6º Ao Distrito Federal cabem, cumulativamente, as
interinstitucional, nos termos previstos no inciso V do art. 88 competências dos Estados e dos Municípios.

Legislação 79
Apostila Digital Licenciada para Alice Caroline Guarino dos Santos - alice.guarino@hotmail.com (Proibida a Revenda)
APOSTILAS OPÇÃO

CAPÍTULO III VI - a indicação da equipe técnica, cuja quantidade e


DOS PLANOS DE ATENDIMENTO SOCIOEDUCATIVO formação devem estar em conformidade com as normas de
referência do sistema e dos conselhos profissionais e com o
Art. 7º O Plano de que trata o inciso II do art. 3o desta Lei atendimento socioeducativo a ser realizado; e
deverá incluir um diagnóstico da situação do Sinase, as VII - a adesão ao Sistema de Informações sobre o
diretrizes, os objetivos, as metas, as prioridades e as formas de Atendimento Socioeducativo, bem como sua operação efetiva.
financiamento e gestão das ações de atendimento para os 10 Parágrafo único. O não cumprimento do previsto neste
(dez) anos seguintes, em sintonia com os princípios elencados artigo sujeita as entidades de atendimento, os órgãos gestores,
na Lei nº 8.069, de 13 de julho de 1990 (Estatuto da Criança e seus dirigentes ou prepostos à aplicação das medidas
do Adolescente). previstas no art. 97 da Lei nº 8.069, de 13 de julho de 1990
§ 1º As normas nacionais de referência para o atendimento (Estatuto da Criança e do Adolescente).
socioeducativo devem constituir anexo ao Plano de que trata o
inciso II do art. 3o desta Lei. Art. 12. A composição da equipe técnica do programa de
§ 2º Os Estados, o Distrito Federal e os Municípios deverão, atendimento deverá ser interdisciplinar, compreendendo, no
com base no Plano Nacional de Atendimento Socioeducativo, mínimo, profissionais das áreas de saúde, educação e
elaborar seus planos decenais correspondentes, em até 360 assistência social, de acordo com as normas de referência.
(trezentos e sessenta) dias a partir da aprovação do Plano § 1º Outros profissionais podem ser acrescentados às
Nacional. equipes para atender necessidades específicas do programa.
§ 2º Regimento interno deve discriminar as atribuições de
Art. 8º Os Planos de Atendimento Socioeducativo deverão, cada profissional, sendo proibida a sobreposição dessas
obrigatoriamente, prever ações articuladas nas áreas de atribuições na entidade de atendimento.
educação, saúde, assistência social, cultura, capacitação para o § 3º O não cumprimento do previsto neste artigo sujeita as
trabalho e esporte, para os adolescentes atendidos, em entidades de atendimento, seus dirigentes ou prepostos à
conformidade com os princípios elencados na Lei nº 8.069, de aplicação das medidas previstas no art. 97 da Lei nº 8.069, de
13 de julho de 1990 (Estatuto da Criança e do Adolescente). 13 de julho de 1990 (Estatuto da Criança e do Adolescente).
Parágrafo único. Os Poderes Legislativos federal,
estaduais, distrital e municipais, por meio de suas comissões Seção II
temáticas pertinentes, acompanharão a execução dos Planos Dos Programas de Meio Aberto
de Atendimento Socioeducativo dos respectivos entes
federados. Art. 13. Compete à direção do programa de prestação de
serviços à comunidade ou de liberdade assistida:
CAPÍTULO IV I - selecionar e credenciar orientadores, designando-os,
DOS PROGRAMAS DE ATENDIMENTO caso a caso, para acompanhar e avaliar o cumprimento da
Seção I medida;
Disposições Gerais II - receber o adolescente e seus pais ou responsável e
orientá-los sobre a finalidade da medida e a organização e
Art. 9º Os Estados e o Distrito Federal inscreverão seus funcionamento do programa;
programas de atendimento e alterações no Conselho Estadual III - encaminhar o adolescente para o orientador
ou Distrital dos Direitos da Criança e do Adolescente, conforme credenciado;
o caso. IV - supervisionar o desenvolvimento da medida; e
V - avaliar, com o orientador, a evolução do cumprimento
Art. 10. Os Municípios inscreverão seus programas e da medida e, se necessário, propor à autoridade judiciária sua
alterações, bem como as entidades de atendimento substituição, suspensão ou extinção.
executoras, no Conselho Municipal dos Direitos da Criança e Parágrafo único. O rol de orientadores credenciados
do Adolescente. deverá ser comunicado, semestralmente, à autoridade
judiciária e ao Ministério Público.
Art. 11. Além da especificação do regime, são requisitos
obrigatórios para a inscrição de programa de atendimento: Art. 14. Incumbe ainda à direção do programa de medida
I - a exposição das linhas gerais dos métodos e técnicas de prestação de serviços à comunidade selecionar e credenciar
pedagógicas, com a especificação das atividades de natureza entidades assistenciais, hospitais, escolas ou outros
coletiva; estabelecimentos congêneres, bem como os programas
II - a indicação da estrutura material, dos recursos comunitários ou governamentais, de acordo com o perfil do
humanos e das estratégias de segurança compatíveis com as socioeducando e o ambiente no qual a medida será cumprida.
necessidades da respectiva unidade; Parágrafo único. Se o Ministério Público impugnar o
III - regimento interno que regule o funcionamento da credenciamento, ou a autoridade judiciária considerá-lo
entidade, no qual deverá constar, no mínimo: inadequado, instaurará incidente de impugnação, com a
a) o detalhamento das atribuições e responsabilidades do aplicação subsidiária do procedimento de apuração de
dirigente, de seus prepostos, dos membros da equipe técnica e irregularidade em entidade de atendimento regulamentado na
dos demais educadores; Lei no 8.069, de 13 de julho de 1990 (Estatuto da Criança e do
b) a previsão das condições do exercício da disciplina e Adolescente), devendo citar o dirigente do programa e a
concessão de benefícios e o respectivo procedimento de direção da entidade ou órgão credenciado.
aplicação; e
c) a previsão da concessão de benefícios extraordinários e Seção III
enaltecimento, tendo em vista tornar público o Dos Programas de Privação da Liberdade
reconhecimento ao adolescente pelo esforço realizado na
consecução dos objetivos do plano individual; Art. 15. São requisitos específicos para a inscrição de
IV - a política de formação dos recursos humanos; programas de regime de semiliberdade ou internação:
V - a previsão das ações de acompanhamento do I - a comprovação da existência de estabelecimento
adolescente após o cumprimento de medida socioeducativa; educacional com instalações adequadas e em conformidade
com as normas de referência;

Legislação 80
Apostila Digital Licenciada para Alice Caroline Guarino dos Santos - alice.guarino@hotmail.com (Proibida a Revenda)
APOSTILAS OPÇÃO

II - a previsão do processo e dos requisitos para a escolha § 3º O relatório da avaliação deverá ser encaminhado aos
do dirigente; respectivos Conselhos de Direitos, Conselhos Tutelares e ao
III - a apresentação das atividades de natureza coletiva; Ministério Público.
IV - a definição das estratégias para a gestão de conflitos, § 4º Os gestores e entidades têm o dever de colaborar com
vedada a previsão de isolamento cautelar, exceto nos casos o processo de avaliação, facilitando o acesso às suas
previstos no § 2o do art. 49 desta Lei; e instalações, à documentação e a todos os elementos
V - a previsão de regime disciplinar nos termos do art. 72 necessários ao seu efetivo cumprimento.
desta Lei. § 5º O acompanhamento tem por objetivo verificar o
cumprimento das metas dos Planos de Atendimento
Art. 16. A estrutura física da unidade deverá ser Socioeducativo.
compatível com as normas de referência do Sinase.
§ 1º É vedada a edificação de unidades socioeducacionais Art. 20. O Sistema Nacional de Avaliação e
em espaços contíguos, anexos, ou de qualquer outra forma Acompanhamento da Gestão do Atendimento Socioeducativo
integrados a estabelecimentos penais. assegurará, na metodologia a ser empregada:
§ 2º A direção da unidade adotará, em caráter excepcional, I - a realização da auto avaliação dos gestores e das
medidas para proteção do interno em casos de risco à sua instituições de atendimento;
integridade física, à sua vida, ou à de outrem, comunicando, de II - a avaliação institucional externa, contemplando a
imediato, seu defensor e o Ministério Público. análise global e integrada das instalações físicas, relações
institucionais, compromisso social, atividades e finalidades
Art. 17. Para o exercício da função de dirigente de das instituições de atendimento e seus programas;
programa de atendimento em regime de semiliberdade ou de III - o respeito à identidade e à diversidade de entidades e
internação, além dos requisitos específicos previstos no programas;
respectivo programa de atendimento, é necessário: IV - a participação do corpo de funcionários das entidades
I - formação de nível superior compatível com a natureza de atendimento e dos Conselhos Tutelares da área de atuação
da função; da entidade avaliada; e
II - comprovada experiência no trabalho com adolescentes V - o caráter público de todos os procedimentos, dados e
de, no mínimo, 2 (dois) anos; e resultados dos processos avaliativos.
III - reputação ilibada.
Art. 21. A avaliação será coordenada por uma comissão
CAPÍTULO V permanente e realizada por comissões temporárias, essas
DA AVALIAÇÃO E ACOMPANHAMENTO DA GESTÃO DO compostas, no mínimo, por 3 (três) especialistas com
ATENDIMENTO SOCIOEDUCATIVO reconhecida atuação na área temática e definidas na forma do
regulamento.
Art. 18. A União, em articulação com os Estados, o Distrito Parágrafo único. É vedado à comissão permanente
Federal e os Municípios, realizará avaliações periódicas da designar avaliadores:
implementação dos Planos de Atendimento Socioeducativo em I - que sejam titulares ou servidores dos órgãos gestores
intervalos não superiores a 3 (três) anos. avaliados ou funcionários das entidades avaliadas;
§ 1º O objetivo da avaliação é verificar o cumprimento das II - que tenham relação de parentesco até o 3o grau com
metas estabelecidas e elaborar recomendações aos gestores e titulares ou servidores dos órgãos gestores avaliados e/ou
operadores dos Sistemas. funcionários das entidades avaliadas; e
§ 2º O processo de avaliação deverá contar com a III - que estejam respondendo a processos criminais.
participação de representantes do Poder Judiciário, do
Ministério Público, da Defensoria Pública e dos Conselhos Art. 22. A avaliação da gestão terá por objetivo:
Tutelares, na forma a ser definida em regulamento. I - verificar se o planejamento orçamentário e sua execução
§ 3º A primeira avaliação do Plano Nacional de se processam de forma compatível com as necessidades do
Atendimento Socioeducativo realizar-se-á no terceiro ano de respectivo Sistema de Atendimento Socioeducativo;
vigência desta Lei, cabendo ao Poder Legislativo federal II - verificar a manutenção do fluxo financeiro,
acompanhar o trabalho por meio de suas comissões temáticas considerando as necessidades operacionais do atendimento
pertinentes. socioeducativo, as normas de referência e as condições
previstas nos instrumentos jurídicos celebrados entre os
Art. 19. É instituído o Sistema Nacional de Avaliação e órgãos gestores e as entidades de atendimento;
Acompanhamento do Atendimento Socioeducativo, com os III - verificar a implementação de todos os demais
seguintes objetivos: compromissos assumidos por ocasião da celebração dos
I - contribuir para a organização da rede de atendimento instrumentos jurídicos relativos ao atendimento
socioeducativo; socioeducativo; e
II - assegurar conhecimento rigoroso sobre as ações do IV - a articulação interinstitucional e intersetorial das
atendimento socioeducativo e seus resultados; políticas.
III - promover a melhora da qualidade da gestão e do
atendimento socioeducativo; e Art. 23. A avaliação das entidades terá por objetivo
IV - disponibilizar informações sobre o atendimento identificar o perfil e o impacto de sua atuação, por meio de suas
socioeducativo. atividades, programas e projetos, considerando as diferentes
§ 1º A avaliação abrangerá, no mínimo, a gestão, as dimensões institucionais e, entre elas, obrigatoriamente, as
entidades de atendimento, os programas e os resultados da seguintes:
execução das medidas socioeducativas. I - o plano de desenvolvimento institucional;
§ 2º Ao final da avaliação, será elaborado relatório II - a responsabilidade social, considerada especialmente
contendo histórico e diagnóstico da situação, as sua contribuição para a inclusão social e o desenvolvimento
recomendações e os prazos para que essas sejam cumpridas, socioeconômico do adolescente e de sua família;
além de outros elementos a serem definidos em regulamento. III - a comunicação e o intercâmbio com a sociedade;

Legislação 81
Apostila Digital Licenciada para Alice Caroline Guarino dos Santos - alice.guarino@hotmail.com (Proibida a Revenda)
APOSTILAS OPÇÃO

IV - as políticas de pessoal quanto à qualificação, Parágrafo único. A aplicação das medidas previstas neste
aperfeiçoamento, desenvolvimento profissional e condições artigo dar-se-á a partir da análise de relatório circunstanciado
de trabalho; elaborado após as avaliações, sem prejuízo do que
V - a adequação da infraestrutura física às normas de determinam os arts. 191 a 197, 225 a 227, 230 a236, 243 e 245
referência; a 247 da Lei nº 8.069, de 13 de julho de 1990 (Estatuto da
VI - o planejamento e a autoavaliação quanto aos Criança e do Adolescente).
processos, resultados, eficiência e eficácia do projeto
pedagógico e da proposta socioeducativa; Art. 29. Àqueles que, mesmo não sendo agentes públicos,
VII - as políticas de atendimento para os adolescentes e induzam ou concorram, sob qualquer forma, direta ou indireta,
suas famílias; para o não cumprimento desta Lei, aplicam-se, no que couber,
VIII - a atenção integral à saúde dos adolescentes em as penalidades dispostas na Lei nº 8.429, de 2 de junho de
conformidade com as diretrizes do art. 60 desta Lei; e 1992, que dispõe sobre as sanções aplicáveis aos agentes
IX - a sustentabilidade financeira. públicos nos casos de enriquecimento ilícito no exercício de
mandato, cargo, emprego ou função na administração pública
Art. 24. A avaliação dos programas terá por objetivo direta, indireta ou fundacional e dá outras providências (Lei de
verificar, no mínimo, o atendimento ao que determinam os Improbidade Administrativa).
arts. 94, 100, 117, 119, 120, 123 e 124 da Lei no 8.069, de 13
de julho de 1990 (Estatuto da Criança e do Adolescente). CAPÍTULO VII
DO FINANCIAMENTO E DAS PRIORIDADES
Art. 25. A avaliação dos resultados da execução de medida
socioeducativa terá por objetivo, no mínimo: Art. 30. O Sinase será cofinanciado com recursos dos
I - verificar a situação do adolescente após cumprimento orçamentos fiscal e da seguridade social, além de outras
da medida socioeducativa, tomando por base suas fontes.
perspectivas educacionais, sociais, profissionais e familiares; e § 1º (VETADO).
II - verificar reincidência de prática de ato infracional. § 2º Os entes federados que tenham instituído seus
sistemas de atendimento socioeducativo terão acesso aos
Art. 26. Os resultados da avaliação serão utilizados para: recursos na forma de transferência adotada pelos órgãos
I - planejamento de metas e eleição de prioridades do integrantes do Sinase.
Sistema de Atendimento Socioeducativo e seu financiamento; § 3º Os entes federados beneficiados com recursos dos
II - reestruturação e/ou ampliação da rede de atendimento orçamentos dos órgãos responsáveis pelas políticas
socioeducativo, de acordo com as necessidades integrantes do Sinase, ou de outras fontes, estão sujeitos às
diagnosticadas; normas e procedimentos de monitoramento estabelecidos
III - adequação dos objetivos e da natureza do atendimento pelas instâncias dos órgãos das políticas setoriais envolvidas,
socioeducativo prestado pelas entidades avaliadas; sem prejuízo do disposto nos incisos IX e X do art. 4º, nos
IV - celebração de instrumentos de cooperação com vistas incisos V e VI do art. 5º e no art. 6º desta Lei.
à correção de problemas diagnosticados na avaliação;
V - reforço de financiamento para fortalecer a rede de Art. 31. Os Conselhos de Direitos, nas 3 (três) esferas de
atendimento socioeducativo; governo, definirão, anualmente, o percentual de recursos dos
VI - melhorar e ampliar a capacitação dos operadores do Fundos dos Direitos da Criança e do Adolescente a serem
Sistema de Atendimento Socioeducativo; e aplicados no financiamento das ações previstas nesta Lei, em
VII - os efeitos do art. 95 da Lei no 8.069, de 13 de julho de especial para capacitação, sistemas de informação e de
1990 (Estatuto da Criança e do Adolescente). avaliação.
Parágrafo único. As recomendações originadas da Parágrafo único. Os entes federados beneficiados com
avaliação deverão indicar prazo para seu cumprimento por recursos do Fundo dos Direitos da Criança e do Adolescente
parte das entidades de atendimento e dos gestores avaliados, para ações de atendimento socioeducativo prestarão
ao fim do qual estarão sujeitos às medidas previstas no art. 28 informações sobre o desempenho dessas ações por meio do
desta Lei. Sistema de Informações sobre Atendimento Socioeducativo.

Art. 27. As informações produzidas a partir do Sistema Art. 32. A Lei nº 7.560, de 19 de dezembro de 1986, passa
Nacional de Informações sobre Atendimento Socioeducativo a vigorar com as seguintes alterações:
serão utilizadas para subsidiar a avaliação, o “Art. 5º Os recursos do Funad serão destinados:
acompanhamento, a gestão e o financiamento dos Sistemas .............................................................................................
Nacional, Distrital, Estaduais e Municipais de Atendimento X - às entidades governamentais e não governamentais
Socioeducativo. integrantes do Sistema Nacional de Atendimento
Socioeducativo (Sinase).
CAPÍTULO VI ...................................................................................” (NR)
DA RESPONSABILIZAÇÃO DOS GESTORES, “Art. 5º-A. A Secretaria Nacional de Políticas sobre Drogas
OPERADORES E ENTIDADES DE ATENDIMENTO (Senad), órgão gestor do Fundo Nacional Antidrogas (Funad),
poderá financiar projetos das entidades do Sinase desde que:
Art. 28. No caso do desrespeito, mesmo que parcial, ou do I - o ente federado de vinculação da entidade que solicita o
não cumprimento integral às diretrizes e determinações desta recurso possua o respectivo Plano de Atendimento
Lei, em todas as esferas, são sujeitos: Socioeducativo aprovado;
I - gestores, operadores e seus prepostos e entidades II - as entidades governamentais e não governamentais
governamentais às medidas previstas no inciso I e no § 1ºdo integrantes do Sinase que solicitem recursos tenham
art. 97 da Lei nº 8.069, de 13 de julho de 1990 (Estatuto da participado da avaliação nacional do atendimento
Criança e do Adolescente); e socioeducativo;
II - entidades não governamentais, seus gestores, III - o projeto apresentado esteja de acordo com os
operadores e prepostos às medidas previstas no inciso II e no pressupostos da Política Nacional sobre Drogas e legislação
§ 1º do art. 97 da Lei nº 8.069, de 13 de julho de 1990 (Estatuto específica.”
da Criança e do Adolescente).

Legislação 82
Apostila Digital Licenciada para Alice Caroline Guarino dos Santos - alice.guarino@hotmail.com (Proibida a Revenda)
APOSTILAS OPÇÃO

Art. 33. A Lei nº 7.998, de 11 de janeiro de 1990, passa a socioeducativa, asseguradas aos seus membros as
vigorar acrescida do seguinte art. 19-A: prerrogativas previstas na Lei nº 8.069, de 13 de julho de 1990
“Art. 19-A. O Codefat poderá priorizar projetos das (Estatuto da Criança e do Adolescente), podendo requerer as
entidades integrantes do Sistema Nacional de Atendimento providências necessárias para adequar a execução aos ditames
Socioeducativo (Sinase) desde que: legais e regulamentares.
I - o ente federado de vinculação da entidade que solicita o
recurso possua o respectivo Plano de Atendimento Art. 38. As medidas de proteção, de advertência e de
Socioeducativo aprovado; reparação do dano, quando aplicadas de forma isolada, serão
II - as entidades governamentais e não governamentais executadas nos próprios autos do processo de conhecimento,
integrantes do Sinase que solicitem recursos tenham se respeitado o disposto nos arts. 143 e 144 da Lei nº 8.069, de
submetido à avaliação nacional do atendimento 13 de julho de 1990 (Estatuto da Criança e do Adolescente).
socioeducativo.”
Art. 39. Para aplicação das medidas socioeducativas de
Art. 34. O art. 2º da Lei nº 5.537, de 21 de novembro de prestação de serviços à comunidade, liberdade assistida,
1968, passa a vigorar acrescido do seguinte § 3o: semiliberdade ou internação, será constituído processo de
“Art. 2º....................................................................... execução para cada adolescente, respeitado o disposto nos
............................................................................................. arts. 143 e 144 da Lei nº 8.069, de 13 de julho de 1990
§ 3º O fundo de que trata o art. 1o poderá financiar, na (Estatuto da Criança e do Adolescente), e com autuação das
forma das resoluções de seu conselho deliberativo, programas seguintes peças:
e projetos de educação básica relativos ao Sistema Nacional de I - documentos de caráter pessoal do adolescente
Atendimento Socioeducativo (Sinase) desde que: existentes no processo de conhecimento, especialmente os que
I - o ente federado que solicitar o recurso possua o comprovem sua idade; e
respectivo Plano de Atendimento Socioeducativo aprovado; II - as indicadas pela autoridade judiciária, sempre que
II - as entidades de atendimento vinculadas ao ente houver necessidade e, obrigatoriamente:
federado que solicitar o recurso tenham se submetido à a) cópia da representação;
avaliação nacional do atendimento socioeducativo; e b) cópia da certidão de antecedentes;
III - o ente federado tenha assinado o Plano de Metas c) cópia da sentença ou acórdão; e
Compromisso Todos pela Educação e elaborado o respectivo d) cópia de estudos técnicos realizados durante a fase de
Plano de Ações Articuladas (PAR).” (NR) conhecimento.
Parágrafo único. Procedimento idêntico será observado na
TÍTULO II hipótese de medida aplicada em sede de remissão, como forma
DA EXECUÇÃO DAS MEDIDAS SOCIOEDUCATIVAS de suspensão do processo.
CAPÍTULO I
DISPOSIÇÕES GERAIS Art. 40. Autuadas as peças, a autoridade judiciária
encaminhará, imediatamente, cópia integral do expediente ao
Art. 35. A execução das medidas socioeducativas reger-se- órgão gestor do atendimento socioeducativo, solicitando
á pelos seguintes princípios: designação do programa ou da unidade de cumprimento da
I - legalidade, não podendo o adolescente receber medida.
tratamento mais gravoso do que o conferido ao adulto;
II - excepcionalidade da intervenção judicial e da Art. 41. A autoridade judiciária dará vistas da proposta de
imposição de medidas, favorecendo-se meios de plano individual de que trata o art. 53 desta Lei ao defensor e
autocomposição de conflitos; ao Ministério Público pelo prazo sucessivo de 3 (três) dias,
III - prioridade a práticas ou medidas que sejam contados do recebimento da proposta encaminhada pela
restaurativas e, sempre que possível, atendam às necessidades direção do programa de atendimento.
das vítimas; § 1º O defensor e o Ministério Público poderão requerer, e
IV - proporcionalidade em relação à ofensa cometida; o Juiz da Execução poderá determinar, de ofício, a realização
V - brevidade da medida em resposta ao ato cometido, em de qualquer avaliação ou perícia que entenderem necessárias
especial o respeito ao que dispõe o art. 122 da Lei nº 8.069, de para complementação do plano individual.
13 de julho de 1990 (Estatuto da Criança e do Adolescente); § 2º A impugnação ou complementação do plano
VI - individualização, considerando-se a idade, capacidades individual, requerida pelo defensor ou pelo Ministério Público,
e circunstâncias pessoais do adolescente; deverá ser fundamentada, podendo a autoridade judiciária
VII - mínima intervenção, restrita ao necessário para a indeferi-la, se entender insuficiente a motivação.
realização dos objetivos da medida; § 3º Admitida a impugnação, ou se entender que o plano é
VIII - não discriminação do adolescente, notadamente em inadequado, a autoridade judiciária designará, se necessário,
razão de etnia, gênero, nacionalidade, classe social, orientação audiência da qual cientificará o defensor, o Ministério Público,
religiosa, política ou sexual, ou associação ou pertencimento a a direção do programa de atendimento, o adolescente e seus
qualquer minoria ou status; e pais ou responsável.
IX - fortalecimento dos vínculos familiares e comunitários § 4º A impugnação não suspenderá a execução do plano
no processo socioeducativo. individual, salvo determinação judicial em contrário.
§ 5º Findo o prazo sem impugnação, considerar-se-á o
CAPÍTULO II plano individual homologado.
DOS PROCEDIMENTOS
Art. 42. As medidas socioeducativas de liberdade assistida,
Art. 36. A competência para jurisdicionar a execução das de semiliberdade e de internação deverão ser reavaliadas no
medidas socioeducativas segue o determinado pelo art. 146 da máximo a cada 6 (seis) meses, podendo a autoridade
Lei no 8.069, de 13 de julho de 1990 (Estatuto da Criança e do judiciária, se necessário, designar audiência, no prazo máximo
Adolescente). de 10 (dez) dias, cientificando o defensor, o Ministério Público,
a direção do programa de atendimento, o adolescente e seus
Art. 37. A defesa e o Ministério Público intervirão, sob pena pais ou responsável.
de nulidade, no procedimento judicial de execução de medida

Legislação 83
Apostila Digital Licenciada para Alice Caroline Guarino dos Santos - alice.guarino@hotmail.com (Proibida a Revenda)
APOSTILAS OPÇÃO

§ 1º A audiência será instruída com o relatório da equipe Art. 46. A medida socioeducativa será declarada extinta:
técnica do programa de atendimento sobre a evolução do I - pela morte do adolescente;
plano de que trata o art. 52 desta Lei e com qualquer outro II - pela realização de sua finalidade;
parecer técnico requerido pelas partes e deferido pela III - pela aplicação de pena privativa de liberdade, a ser
autoridade judiciária. cumprida em regime fechado ou semiaberto, em execução
§ 2º A gravidade do ato infracional, os antecedentes e o provisória ou definitiva;
tempo de duração da medida não são fatores que, por si, IV - pela condição de doença grave, que torne o adolescente
justifiquem a não substituição da medida por outra menos incapaz de submeter-se ao cumprimento da medida; e
grave. V - nas demais hipóteses previstas em lei.
§ 3º Considera-se mais grave a internação, em relação a § 1º No caso de o maior de 18 (dezoito) anos, em
todas as demais medidas, e mais grave a semiliberdade, em cumprimento de medida socioeducativa, responder a
relação às medidas de meio aberto. processo-crime, caberá à autoridade judiciária decidir sobre
eventual extinção da execução, cientificando da decisão o juízo
Art. 43. A reavaliação da manutenção, da substituição ou criminal competente.
da suspensão das medidas de meio aberto ou de privação da § 2º Em qualquer caso, o tempo de prisão cautelar não
liberdade e do respectivo plano individual pode ser solicitada convertida em pena privativa de liberdade deve ser
a qualquer tempo, a pedido da direção do programa de descontado do prazo de cumprimento da medida
atendimento, do defensor, do Ministério Público, do socioeducativa.
adolescente, de seus pais ou responsável.
§ 1º Justifica o pedido de reavaliação, entre outros motivos: Art. 47. O mandado de busca e apreensão do adolescente
I - o desempenho adequado do adolescente com base no terá vigência máxima de 6 (seis) meses, a contar da data da
seu plano de atendimento individual, antes do prazo da expedição, podendo, se necessário, ser renovado,
reavaliação obrigatória; fundamentadamente.
II - a inadaptação do adolescente ao programa e o reiterado
descumprimento das atividades do plano individual; e Art. 48. O defensor, o Ministério Público, o adolescente e
III - a necessidade de modificação das atividades do plano seus pais ou responsável poderão postular revisão judicial de
individual que importem em maior restrição da liberdade do qualquer sanção disciplinar aplicada, podendo a autoridade
adolescente. judiciária suspender a execução da sanção até decisão final do
§ 2º A autoridade judiciária poderá indeferir o pedido, de incidente.
pronto, se entender insuficiente a motivação. § 1º Postulada a revisão após ouvida a autoridade
§ 3º Admitido o processamento do pedido, a autoridade colegiada que aplicou a sanção e havendo provas a produzir
judiciária, se necessário, designará audiência, observando o em audiência, procederá o magistrado na forma do § 1º do art.
princípio do § 1o do art. 42 desta Lei. 42 desta Lei.
§ 4º A substituição por medida mais gravosa somente § 2º É vedada a aplicação de sanção disciplinar de
ocorrerá em situações excepcionais, após o devido processo isolamento a adolescente interno, exceto seja essa
legal, inclusive na hipótese do inciso III do art. 122 da Lei nº imprescindível para garantia da segurança de outros internos
8.069, de 13 de julho de 1990 (Estatuto da Criança e do ou do próprio adolescente a quem seja imposta a sanção,
Adolescente), e deve ser: sendo necessária ainda comunicação ao defensor, ao
I - fundamentada em parecer técnico; Ministério Público e à autoridade judiciária em até 24 (vinte e
II - precedida de prévia audiência, e nos termos do § 1º do quatro) horas.
art. 42 desta Lei.
CAPÍTULO III
Art. 44. Na hipótese de substituição da medida ou DOS DIREITOS INDIVIDUAIS
modificação das atividades do plano individual, a autoridade
judiciária remeterá o inteiro teor da decisão à direção do Art. 49. São direitos do adolescente submetido ao
programa de atendimento, assim como as peças que entender cumprimento de medida socioeducativa, sem prejuízo de
relevantes à nova situação jurídica do adolescente. outros previstos em lei:
Parágrafo único. No caso de a substituição da medida I - ser acompanhado por seus pais ou responsável e por seu
importar em vinculação do adolescente a outro programa de defensor, em qualquer fase do procedimento administrativo
atendimento, o plano individual e o histórico do cumprimento ou judicial;
da medida deverão acompanhar a transferência. II - ser incluído em programa de meio aberto quando
inexistir vaga para o cumprimento de medida de privação da
Art. 45. Se, no transcurso da execução, sobrevier sentença liberdade, exceto nos casos de ato infracional cometido
de aplicação de nova medida, a autoridade judiciária mediante grave ameaça ou violência à pessoa, quando o
procederá à unificação, ouvidos, previamente, o Ministério adolescente deverá ser internado em Unidade mais próxima
Público e o defensor, no prazo de 3 (três) dias sucessivos, de seu local de residência;
decidindo-se em igual prazo. III - ser respeitado em sua personalidade, intimidade,
§ 1º É vedado à autoridade judiciária determinar reinício liberdade de pensamento e religião e em todos os direitos não
de cumprimento de medida socioeducativa, ou deixar de expressamente limitados na sentença;
considerar os prazos máximos, e de liberação compulsória IV - peticionar, por escrito ou verbalmente, diretamente a
previstos na Lei nº 8.069, de 13 de julho de 1990 (Estatuto da qualquer autoridade ou órgão público, devendo,
Criança e do Adolescente), excetuada a hipótese de medida obrigatoriamente, ser respondido em até 15 (quinze) dias;
aplicada por ato infracional praticado durante a execução. V - ser informado, inclusive por escrito, das normas de
§ 2º É vedado à autoridade judiciária aplicar nova medida organização e funcionamento do programa de atendimento e
de internação, por atos infracionais praticados anteriormente, também das previsões de natureza disciplinar;
a adolescente que já tenha concluído cumprimento de medida VI - receber, sempre que solicitar, informações sobre a
socioeducativa dessa natureza, ou que tenha sido transferido evolução de seu plano individual, participando,
para cumprimento de medida menos rigorosa, sendo tais atos obrigatoriamente, de sua elaboração e, se for o caso,
absorvidos por aqueles aos quais se impôs a medida reavaliação;
socioeducativa extrema.

Legislação 84
Apostila Digital Licenciada para Alice Caroline Guarino dos Santos - alice.guarino@hotmail.com (Proibida a Revenda)
APOSTILAS OPÇÃO

VII - receber assistência integral à sua saúde, conforme o Parágrafo único. O PIA será elaborado no prazo de até 45
disposto no art. 60 desta Lei; e (quarenta e cinco) dias da data do ingresso do adolescente no
VIII - ter atendimento garantido em creche e pré-escola aos programa de atendimento.
filhos de 0 (zero) a 5 (cinco) anos.
§ 1º As garantias processuais destinadas a adolescente Art. 56. Para o cumprimento das medidas de prestação de
autor de ato infracional previstas na Lei no 8.069, de 13 de serviços à comunidade e de liberdade assistida, o PIA será
julho de 1990 (Estatuto da Criança e do Adolescente), aplicam- elaborado no prazo de até 15 (quinze) dias do ingresso do
se integralmente na execução das medidas socioeducativas, adolescente no programa de atendimento.
inclusive no âmbito administrativo.
§ 2º A oferta irregular de programas de atendimento Art. 57. Para a elaboração do PIA, a direção do respectivo
socioeducativo em meio aberto não poderá ser invocada como programa de atendimento, pessoalmente ou por meio de
motivo para aplicação ou manutenção de medida de privação membro da equipe técnica, terá acesso aos autos do
da liberdade. procedimento de apuração do ato infracional e aos dos
procedimentos de apuração de outros atos infracionais
Art. 50. Sem prejuízo do disposto no § 1º do art. 121 da Lei atribuídos ao mesmo adolescente.
nº 8.069, de 13 de julho de 1990 (Estatuto da Criança e do § 1º O acesso aos documentos de que trata o caput deverá
Adolescente), a direção do programa de execução de medida ser realizado por funcionário da entidade de atendimento,
de privação da liberdade poderá autorizar a saída, monitorada, devidamente credenciado para tal atividade, ou por membro
do adolescente nos casos de tratamento médico, doença grave da direção, em conformidade com as normas a serem definidas
ou falecimento, devidamente comprovados, de pai, mãe, filho, pelo Poder Judiciário, de forma a preservar o que determinam
cônjuge, companheiro ou irmão, com imediata comunicação ao os arts. 143 e 144 da Lei nº 8.069, de 13 de julho de 1990
juízo competente. (Estatuto da Criança e do Adolescente).
§ 2º A direção poderá requisitar, ainda:
Art. 51. A decisão judicial relativa à execução de medida I - ao estabelecimento de ensino, o histórico escolar do
socioeducativa será proferida após manifestação do defensor adolescente e as anotações sobre o seu aproveitamento;
e do Ministério Público. II - os dados sobre o resultado de medida anteriormente
aplicada e cumprida em outro programa de atendimento; e
CAPÍTULO IV III - os resultados de acompanhamento especializado
DO PLANO INDIVIDUAL DE ATENDIMENTO (PIA) anterior.

Art. 52. O cumprimento das medidas socioeducativas, em Art. 58. Por ocasião da reavaliação da medida, é obrigatória
regime de prestação de serviços à comunidade, liberdade a apresentação pela direção do programa de atendimento de
assistida, semiliberdade ou internação, dependerá de Plano relatório da equipe técnica sobre a evolução do adolescente no
Individual de Atendimento (PIA), instrumento de previsão, cumprimento do plano individual.
registro e gestão das atividades a serem desenvolvidas com o
adolescente. Art. 59. O acesso ao plano individual será restrito aos
Parágrafo único. O PIA deverá contemplar a participação servidores do respectivo programa de atendimento, ao
dos pais ou responsáveis, os quais têm o dever de contribuir adolescente e a seus pais ou responsável, ao Ministério Público
com o processo ressocializador do adolescente, sendo esses e ao defensor, exceto expressa autorização judicial.
passíveis de responsabilização administrativa, nos termos do
art. 249 da Lei nº 8.069, de 13 de julho de 1990 (Estatuto da CAPÍTULO V
Criança e do Adolescente), civil e criminal. DA ATENÇÃO INTEGRAL À SAÚDE DE ADOLESCENTE
EM CUMPRIMENTO DE MEDIDA SOCIOEDUCATIVA
Art. 53. O PIA será elaborado sob a responsabilidade da Seção I
equipe técnica do respectivo programa de atendimento, com a Disposições Gerais
participação efetiva do adolescente e de sua família,
representada por seus pais ou responsável. Art. 60. A atenção integral à saúde do adolescente no
Sistema de Atendimento Socioeducativo seguirá as seguintes
Art. 54. Constarão do plano individual, no mínimo: diretrizes:
I - os resultados da avaliação interdisciplinar; I - previsão, nos planos de atendimento socioeducativo, em
II - os objetivos declarados pelo adolescente; todas as esferas, da implantação de ações de promoção da
III - a previsão de suas atividades de integração social e/ou saúde, com o objetivo de integrar as ações socioeducativas,
capacitação profissional; estimulando a autonomia, a melhoria das relações
IV - atividades de integração e apoio à família; interpessoais e o fortalecimento de redes de apoio aos
V - formas de participação da família para efetivo adolescentes e suas famílias;
cumprimento do plano individual; e II - inclusão de ações e serviços para a promoção, proteção,
VI - as medidas específicas de atenção à sua saúde. prevenção de agravos e doenças e recuperação da saúde;
III - cuidados especiais em saúde mental, incluindo os
Art. 55. Para o cumprimento das medidas de relacionados ao uso de álcool e outras substâncias psicoativas,
semiliberdade ou de internação, o plano individual conterá, e atenção aos adolescentes com deficiências;
ainda: IV - disponibilização de ações de atenção à saúde sexual e
I - a designação do programa de atendimento mais reprodutiva e à prevenção de doenças sexualmente
adequado para o cumprimento da medida; transmissíveis;
II - a definição das atividades internas e externas, V - garantia de acesso a todos os níveis de atenção à saúde,
individuais ou coletivas, das quais o adolescente poderá por meio de referência e contrarreferência, de acordo com as
participar; e normas do Sistema Único de Saúde (SUS);
III - a fixação das metas para o alcance de desenvolvimento VI - capacitação das equipes de saúde e dos profissionais
de atividades externas. das entidades de atendimento, bem como daqueles que atuam
nas unidades de saúde de referência voltadas às
especificidades de saúde dessa população e de suas famílias;

Legislação 85
Apostila Digital Licenciada para Alice Caroline Guarino dos Santos - alice.guarino@hotmail.com (Proibida a Revenda)
APOSTILAS OPÇÃO

VII - inclusão, nos Sistemas de Informação de Saúde do SUS, Art. 66. (VETADO).
bem como no Sistema de Informações sobre Atendimento
Socioeducativo, de dados e indicadores de saúde da população CAPÍTULO VI
de adolescentes em atendimento socioeducativo; e DAS VISITAS A ADOLESCENTE EM CUMPRIMENTO DE
VIII - estruturação das unidades de internação conforme as MEDIDA DE
normas de referência do SUS e do Sinase, visando ao INTERNAÇÃO
atendimento das necessidades de Atenção Básica.
Art. 67. A visita do cônjuge, companheiro, pais ou
Art. 61. As entidades que ofereçam programas de responsáveis, parentes e amigos a adolescente a quem foi
atendimento socioeducativo em meio aberto e de aplicada medida socioeducativa de internação observará dias
semiliberdade deverão prestar orientações aos e horários próprios definidos pela direção do programa de
socioeducandos sobre o acesso aos serviços e às unidades do atendimento.
SUS.
Art. 68. É assegurado ao adolescente casado ou que viva,
Art. 62. As entidades que ofereçam programas de privação comprovadamente, em união estável o direito à visita íntima.
de liberdade deverão contar com uma equipe mínima de Parágrafo único. O visitante será identificado e registrado
profissionais de saúde cuja composição esteja em pela direção do programa de atendimento, que emitirá
conformidade com as normas de referência do SUS. documento de identificação, pessoal e intransferível,
específico para a realização da visita íntima.
Art. 63. (VETADO).
§ 1º O filho de adolescente nascido nos estabelecimentos Art. 69. É garantido aos adolescentes em cumprimento de
referidos no caput deste artigo não terá tal informação lançada medida socioeducativa de internação o direito de receber
em seu registro de nascimento. visita dos filhos, independentemente da idade desses.
§ 2º Serão asseguradas as condições necessárias para que
a adolescente submetida à execução de medida socioeducativa Art. 70. O regulamento interno estabelecerá as hipóteses
de privação de liberdade permaneça com o seu filho durante o de proibição da entrada de objetos na unidade de internação,
período de amamentação. vedando o acesso aos seus portadores.

Seção II CAPÍTULO VII


Do Atendimento a Adolescente com Transtorno DOS REGIMES DISCIPLINARES
Mental e com Dependência de Álcool e de Substância
Psicoativa Art. 71. Todas as entidades de atendimento socioeducativo
deverão, em seus respectivos regimentos, realizar a previsão
Art. 64. O adolescente em cumprimento de medida de regime disciplinar que obedeça aos seguintes princípios:
socioeducativa que apresente indícios de transtorno mental, I - tipificação explícita das infrações como leves, médias e
de deficiência mental, ou associadas, deverá ser avaliado por graves e determinação das correspondentes sanções;
equipe técnica multidisciplinar e Multisetorial. II - exigência da instauração formal de processo disciplinar
§ 1º As competências, a composição e a atuação da equipe para a aplicação de qualquer sanção, garantidos a ampla
técnica de que trata o caput deverão seguir, conjuntamente, as defesa e o contraditório;
normas de referência do SUS e do Sinase, na forma do III - obrigatoriedade de audiência do socioeducando nos
regulamento. casos em que seja necessária a instauração de processo
§ 2º A avaliação de que trata o caput subsidiará a disciplinar;
elaboração e execução da terapêutica a ser adotada, a qual será IV - sanção de duração determinada;
incluída no PIA do adolescente, prevendo, se necessário, ações V - enumeração das causas ou circunstâncias que eximam,
voltadas para a família. atenuem ou agravem a sanção a ser imposta ao socioeducando,
§ 3º As informações produzidas na avaliação de que trata bem como os requisitos para a extinção dessa;
o caput são consideradas sigilosas. VI - enumeração explícita das garantias de defesa;
§ 4º Excepcionalmente, o juiz poderá suspender a VII - garantia de solicitação e rito de apreciação dos
execução da medida socioeducativa, ouvidos o defensor e o recursos cabíveis; e
Ministério Público, com vistas a incluir o adolescente em VIII - apuração da falta disciplinar por comissão composta
programa de atenção integral à saúde mental que melhor por, no mínimo, 3 (três) integrantes, sendo 1 (um),
atenda aos objetivos terapêuticos estabelecidos para o seu obrigatoriamente, oriundo da equipe técnica.
caso específico.
§ 5º Suspensa a execução da medida socioeducativa, o juiz Art. 72. O regime disciplinar é independente da
designará o responsável por acompanhar e informar sobre a responsabilidade civil ou penal que advenha do ato cometido.
evolução do atendimento ao adolescente.
§ 6º A suspensão da execução da medida socioeducativa Art. 73. Nenhum socioeducando poderá desempenhar
será avaliada, no mínimo, a cada 6 (seis) meses. função ou tarefa de apuração disciplinar ou aplicação de
§ 7º O tratamento a que se submeterá o adolescente deverá sanção nas entidades de atendimento socioeducativo.
observar o previsto na Lei nº 10.216, de 6 de abril de 2001, que
dispõe sobre a proteção e os direitos das pessoas portadoras Art. 74. Não será aplicada sanção disciplinar sem expressa
de transtornos mentais e redireciona o modelo assistencial em e anterior previsão legal ou regulamentar e o devido processo
saúde mental. administrativo.
§ 8º (VETADO).
Art. 75. Não será aplicada sanção disciplinar ao
Art. 65. Enquanto não cessada a jurisdição da Infância e socioeducando que tenha praticado a falta:
Juventude, a autoridade judiciária, nas hipóteses tratadas no I - por coação irresistível ou por motivo de força maior;
art. 64, poderá remeter cópia dos autos ao Ministério Público II - em legítima defesa, própria ou de outrem.
para eventual propositura de interdição e outras providências
pertinentes.

Legislação 86
Apostila Digital Licenciada para Alice Caroline Guarino dos Santos - alice.guarino@hotmail.com (Proibida a Revenda)
APOSTILAS OPÇÃO

CAPÍTULO VIII publicação desta Lei, garantir a inserção de adolescentes em


DA CAPACITAÇÃO PARA O TRABALHO cumprimento de medida socioeducativa na rede pública de
educação, em qualquer fase do período letivo, contemplando
Art. 76. O art. 2º do Decreto-Lei nº 4.048, de 22 de janeiro as diversas faixas etárias e níveis de instrução.
de 1942, passa a vigorar acrescido do seguinte § 1º,
renumerando-se o atual parágrafo único para § 2º: Art. 83. Os programas de atendimento socioeducativo sob
“Art. 2º ......................................................................... a responsabilidade do Poder Judiciário serão,
§ 1º As escolas do Senai poderão ofertar vagas aos usuários obrigatoriamente, transferidos ao Poder Executivo no prazo
do Sistema Nacional de Atendimento Socioeducativo (Sinase) máximo de 1 (um) ano a partir da publicação desta Lei e de
nas condições a serem dispostas em instrumentos de acordo com a política de oferta dos programas aqui definidos.
cooperação celebrados entre os operadores do Senai e os
gestores dos Sistemas de Atendimento Socioeducativo locais. Art. 84. Os programas de internação e semiliberdade sob a
§ 2º ...................................................................... (NR) responsabilidade dos Municípios serão, obrigatoriamente,
transferidos para o Poder Executivo do respectivo Estado no
Art. 77. O art. 3º do Decreto-Lei nº 8.621, de 10 de janeiro prazo máximo de 1 (um) ano a partir da publicação desta Lei e
de 1946, passa a vigorar acrescido do seguinte § 1º, de acordo com a política de oferta dos programas aqui
renumerando-se o atual parágrafo único para § 2º: definidos.
“Art. 3º .........................................................................
§ 1º As escolas do Senac poderão ofertar vagas aos Art. 85. A não transferência de programas de atendimento
usuários do Sistema Nacional de Atendimento Socioeducativo para os devidos entes responsáveis, no prazo determinado
(Sinase) nas condições a serem dispostas em instrumentos de nesta Lei, importará na interdição do programa e caracterizará
cooperação celebrados entre os operadores do Senac e os ato de improbidade administrativa do agente responsável,
gestores dos Sistemas de Atendimento Socioeducativo locais. vedada, ademais, ao Poder Judiciário e ao Poder Executivo
§ 2º. ....................................................................” (NR) municipal, ao final do referido prazo, a realização de despesas
para a sua manutenção.
Art. 78. O art. 1º da Lei nº 8.315, de 23 de dezembro de
1991, passa a vigorar acrescido do seguinte parágrafo único: Art. 86. Os arts. 90, 97, 121, 122, 198 e 208 da Lei nº 8.069,
“Art. 1º ......................................................................... de 13 de julho de 1990 (Estatuto da Criança e do Adolescente),
Parágrafo único. Os programas de formação profissional passam a vigorar com a seguinte redação:
rural do Senar poderão ofertar vagas aos usuários do Sistema “Art. 90. ......................................................................
Nacional de Atendimento Socioeducativo (Sinase) nas .............................................................................................
condições a serem dispostas em instrumentos de cooperação V - prestação de serviços à comunidade;
celebrados entre os operadores do Senar e os gestores dos VI - liberdade assistida;
Sistemas de Atendimento Socioeducativo locais.” (NR) VII - semiliberdade; e
VIII - internação.
Art. 79. O art. 3º da Lei nº 8.706, de 14 de setembro de ....................................................................................” (NR)
1993, passa a vigorar acrescido do seguinte parágrafo único: “Art. 97. (VETADO)”
“Art. 3º ......................................................................... “Art. 121. .................................…………………............
Parágrafo único. Os programas de formação profissional .............................................................................................
do Senat poderão ofertar vagas aos usuários do Sistema § 7º A determinação judicial mencionada no § 1º poderá
Nacional de Atendimento Socioeducativo (Sinase) nas ser revista a qualquer tempo pela autoridade judiciária.” (NR)
condições a serem dispostas em instrumentos de cooperação “Art. 122. .....................................................................
celebrados entre os operadores do Senat e os gestores dos .............................................................................................
Sistemas de Atendimento Socioeducativo locais.” (NR) § 1º O prazo de internação na hipótese do inciso III deste
artigo não poderá ser superior a 3 (três) meses, devendo ser
Art. 80. O art. 429 do Decreto-Lei nº 5.452, de 1o de maio decretada judicialmente após o devido processo legal.
de 1943, passa a vigorar acrescido do seguinte § 2º: ...................................................................................” (NR)
“Art. 429. ..................................................................... “Art. 198. Nos procedimentos afetos à Justiça da Infância e
............................................................................................. da Juventude, inclusive os relativos à execução das medidas
§ 2º estabelecimentos de que trata o caput ofertarão vagas socioeducativas, adotar-se-á o sistema recursal da Lei no
de aprendizes a adolescentes usuários do Sistema Nacional de 5.869, de 11 de janeiro de 1973 (Código de Processo Civil),
Atendimento Socioeducativo (Sinase) nas condições a serem com as seguintes adaptações:
dispostas em instrumentos de cooperação celebrados entre os .............................................................................................
estabelecimentos e os gestores dos Sistemas de Atendimento II - em todos os recursos, salvo nos embargos de
Socioeducativo locais.” (NR) declaração, o prazo para o Ministério Público e para a defesa
será sempre de 10 (dez) dias;
TÍTULO III ...................................................................................” (NR)
DISPOSIÇÕES FINAIS E TRANSITÓRIAS “Art. 208. .....................................................................
.............................................................................................
Art. 81. As entidades que mantenham programas de X - de programas de atendimento para a execução das
atendimento têm o prazo de até 6 (seis) meses após a medidas socioeducativas e aplicação de medidas de proteção.
publicação desta Lei para encaminhar ao respectivo Conselho ...................................................................................” (NR)
Estadual ou Municipal dos Direitos da Criança e do
Adolescente proposta de adequação da sua inscrição, sob pena Art. 87. A Lei nº 8.069, de 13 de julho de 1990 (Estatuto da
de interdição. Criança e do Adolescente), passa a vigorar com as seguintes
alterações:
Art. 82. Os Conselhos dos Direitos da Criança e do “Art. 260. Os contribuintes poderão efetuar doações aos
Adolescente, em todos os níveis federados, com os órgãos Fundos dos Direitos da Criança e do Adolescente nacional,
responsáveis pelo sistema de educação pública e as entidades distrital, estaduais ou municipais, devidamente comprovadas,
de atendimento, deverão, no prazo de 1 (um) ano a partir da

Legislação 87
Apostila Digital Licenciada para Alice Caroline Guarino dos Santos - alice.guarino@hotmail.com (Proibida a Revenda)
APOSTILAS OPÇÃO

sendo essas integralmente deduzidas do imposto de renda, Adolescente nacional, estaduais, distrital e municipais devem
obedecidos os seguintes limites: emitir recibo em favor do doador, assinado por pessoa
I - 1% (um por cento) do imposto sobre a renda devido competente e pelo presidente do Conselho correspondente,
apurado pelas pessoas jurídicas tributadas com base no lucro especificando:
real; e I - número de ordem;
II - 6% (seis por cento) do imposto sobre a renda apurado II - nome, Cadastro Nacional da Pessoa Jurídica (CNPJ) e
pelas pessoas físicas na Declaração de Ajuste Anual, observado endereço do emitente;
o disposto no art. 22 da Lei no 9.532, de 10 de dezembro de III - nome, CNPJ ou Cadastro de Pessoas Físicas (CPF) do
1997. doador;
............................................................................................. IV - data da doação e valor efetivamente recebido; e
§ 5º Observado o disposto no § 4o do art. 3o da Lei no V - ano-calendário a que se refere a doação.
9.249, de 26 de dezembro de 1995, a dedução de que trata o § 1º O comprovante de que trata o caput deste artigo pode
inciso I do caput: ser emitido anualmente, desde que discrimine os valores
I - será considerada isoladamente, não se submetendo a doados mês a mês.
limite em conjunto com outras deduções do imposto; e § 2º No caso de doação em bens, o comprovante deve
II - não poderá ser computada como despesa operacional conter a identificação dos bens, mediante descrição em campo
na apuração do lucro real.” (NR) próprio ou em relação anexa ao comprovante, informando
“Art. 260-A. A partir do exercício de 2010, ano-calendário também se houve avaliação, o nome, CPF ou CNPJ e endereço
de 2009, a pessoa física poderá optar pela doação de que trata dos avaliadores.”
o inciso II do caput do art. 260 diretamente em sua Declaração “Art. 260-E. Na hipótese da doação em bens, o doador
de Ajuste Anual. deverá:
§ 1º A doação de que trata o caput poderá ser deduzida até I - comprovar a propriedade dos bens, mediante
os seguintes percentuais aplicados sobre o imposto apurado documentação hábil;
na declaração: II - baixar os bens doados na declaração de bens e direitos,
I - (VETADO); quando se tratar de pessoa física, e na escrituração, no caso de
II - (VETADO); pessoa jurídica; e
III - 3% (três por cento) a partir do exercício de 2012. III - considerar como valor dos bens doados:
§ 2º A dedução de que trata o caput: a) para as pessoas físicas, o valor constante da última
I - está sujeita ao limite de 6% (seis por cento) do imposto declaração do imposto de renda, desde que não exceda o valor
sobre a renda apurado na declaração de que trata o inciso II do de mercado;
caput do art. 260; b) para as pessoas jurídicas, o valor contábil dos bens.
II - não se aplica à pessoa física que: Parágrafo único. O preço obtido em caso de leilão não será
a) utilizar o desconto simplificado; considerado na determinação do valor dos bens doados,
b) apresentar declaração em formulário; ou exceto se o leilão for determinado por autoridade judiciária.”
c) entregar a declaração fora do prazo; “Art. 260-F. Os documentos a que se referem os arts. 260-
III - só se aplica às doações em espécie; e D e 260-E devem ser mantidos pelo contribuinte por um prazo
IV - não exclui ou reduz outros benefícios ou deduções em de 5 (cinco) anos para fins de comprovação da dedução
vigor. perante a Receita Federal do Brasil.”
§ 3º O pagamento da doação deve ser efetuado até a data “Art. 260-G. Os órgãos responsáveis pela administração
de vencimento da primeira quota ou quota única do imposto, das contas dos Fundos dos Direitos da Criança e do
observadas instruções específicas da Secretaria da Receita Adolescente nacional, estaduais, distrital e municipais devem:
Federal do Brasil. I - manter conta bancária específica destinada
§ 4º O não pagamento da doação no prazo estabelecido no exclusivamente a gerir os recursos do Fundo;
§ 3o implica a glosa definitiva desta parcela de dedução, II - manter controle das doações recebidas; e
ficando a pessoa física obrigada ao recolhimento da diferença III - informar anualmente à Secretaria da Receita Federal
de imposto devido apurado na Declaração de Ajuste Anual com do Brasil as doações recebidas mês a mês, identificando os
os acréscimos legais previstos na legislação. seguintes dados por doador:
§ 5º A pessoa física poderá deduzir do imposto apurado na a) nome, CNPJ ou CPF;
Declaração de Ajuste Anual as doações feitas, no respectivo b) valor doado, especificando se a doação foi em espécie ou
ano-calendário, aos fundos controlados pelos Conselhos dos em bens.”
Direitos da Criança e do Adolescente municipais, distrital, “Art. 260-H. Em caso de descumprimento das obrigações
estaduais e nacional concomitantemente com a opção de que previstas no art. 260-G, a Secretaria da Receita Federal do
trata o caput, respeitado o limite previsto no inciso II do art. Brasil dará conhecimento do fato ao Ministério Público.”
260.” “Art. 260-I. Os Conselhos dos Direitos da Criança e do
“Art. 260-B. A doação de que trata o inciso I do art. 260 Adolescente nacional, estaduais, distrital e municipais
poderá ser deduzida: divulgarão amplamente à comunidade:
I - do imposto devido no trimestre, para as pessoas I - o calendário de suas reuniões;
jurídicas que apuram o imposto trimestralmente; e II - as ações prioritárias para aplicação das políticas de
II - do imposto devido mensalmente e no ajuste anual, para atendimento à criança e ao adolescente;
as pessoas jurídicas que apuram o imposto anualmente. III - os requisitos para a apresentação de projetos a serem
Parágrafo único. A doação deverá ser efetuada dentro do beneficiados com recursos dos Fundos dos Direitos da Criança
período a que se refere a apuração do imposto.” e do Adolescente nacional, estaduais, distrital ou municipais;
“Art. 260-C. As doações de que trata o art. 260 desta Lei IV - a relação dos projetos aprovados em cada ano-
podem ser efetuadas em espécie ou em bens. calendário e o valor dos recursos previstos para
Parágrafo único. As doações efetuadas em espécie devem implementação das ações, por projeto;
ser depositadas em conta específica, em instituição financeira V - o total dos recursos recebidos e a respectiva destinação,
pública, vinculadas aos respectivos fundos de que trata o art. por projeto atendido, inclusive com cadastramento na base de
260.” dados do Sistema de Informações sobre a Infância e a
“Art. 260-D. Os órgãos responsáveis pela administração Adolescência; e
das contas dos Fundos dos Direitos da Criança e do

Legislação 88
Apostila Digital Licenciada para Alice Caroline Guarino dos Santos - alice.guarino@hotmail.com (Proibida a Revenda)
APOSTILAS OPÇÃO

VI - a avaliação dos resultados dos projetos beneficiados 1. Legalidade, não podendo o adolescente receber
com recursos dos Fundos dos Direitos da Criança e do tratamento mais gravoso do que o conferido ao adulto.
Adolescente nacional, estaduais, distrital e municipais.” 2. Excepcionalidade da intervenção judicial e da imposição
“Art. 260-J. O Ministério Público determinará, em cada de medidas, favorecendo-se meios de autocomposição de
Comarca, a forma de fiscalização da aplicação dos incentivos conflitos, exclusivos aos atos infracionais de natureza leve e
fiscais referidos no art. 260 desta Lei. sem grave ameaça à pessoa.
Parágrafo único. O descumprimento do disposto nos arts. 3. Prioridade a práticas ou medidas que sejam
260-G e 260-I sujeitará os infratores a responder por ação restaurativas que atendam às necessidades das vítimas e
judicial proposta pelo Ministério Público, que poderá atuar de apenas para aqueles atos infracionais de natureza leve e sem
ofício, a requerimento ou representação de qualquer cidadão.” grave ameaça à pessoa.
“Art. 260-K. A Secretaria de Direitos Humanos da 4. Não discriminação do adolescente, notadamente em
Presidência da República (SDH/PR) encaminhará à Secretaria razão de etnia, gênero, nacionalidade, classe social, orientação
da Receita Federal do Brasil, até 31 de outubro de cada ano, religiosa, política ou sexual, ou associação ou pertencimento a
arquivo eletrônico contendo a relação atualizada dos Fundos qualquer minoria ou status.
dos Direitos da Criança e do Adolescente nacional, distrital,
estaduais e municipais, com a indicação dos respectivos Assinale a alternativa que indica todas as afirmativas
números de inscrição no CNPJ e das contas bancárias corretas.
específicas mantidas em instituições financeiras públicas, (A) São corretas apenas as afirmativas 1 e 4.
destinadas exclusivamente a gerir os recursos dos Fundos.” (B) São corretas apenas as afirmativas 2 e 4.
“Art. 260-L. A Secretaria da Receita Federal do Brasil (C) São corretas apenas as afirmativas 3 e 4.
expedirá as instruções necessárias à aplicação do disposto nos (D) São corretas apenas as afirmativas 1, 2 e 3.
arts. 260 a 260-K.” (E) São corretas apenas as afirmativas 2, 3 e 4.

Art. 88. O parágrafo único do art. 3o da Lei no 12.213, de 03. (MPE-SC - Promotor de Justiça – Vespertina –
20 de janeiro de 2010, passa a vigorar com a seguinte redação: MPESC/2016) As hipóteses de extinção da medida
“Art. 3º .......................................................................... socioeducativa vieram bem definidas na Lei n. 12.594/12. Será
Parágrafo único. A dedução a que se refere o caput deste automática a extinção do cumprimento de medida
artigo não poderá ultrapassar 1% (um por cento) do imposto socioeducativa em relação ao imputável que vier a responder
devido.” (NR) a processo-crime, frente à notória ausência de interesse em se
prosseguir no correspondente processo de educação e
Art. 89. (VETADO). integração sociofamiliar.
(...) Certo
Art. 90. Esta Lei entra em vigor após decorridos 90 (...) Errado
(noventa) dias de sua publicação oficial.
04. (Prefeitura de Betim/MG - Técnico de Informática
Questões - Prefeitura de Betim – MG/2015). De acordo com a Lei
12.594/2012, que institui o Sistema Nacional de Atendimento
01. (DPE-MT - Defensor Público – UFMT/2016) Sobre a Socioeducativo (SINASE), não compete ao município
Lei nº 12.594/2012, analise as assertivas abaixo. (A) elaborar o Plano Municipal de Atendimento
I - Compete _____________ estabelecer e desenvolver Socioeducativo
programa para a execução das medidas socioeducativas de (B) criar e manter programas de atendimento para a
semiliberdade. execução das medidas socioeducativas em meio aberto.
II - Compete ____________ estabelecer e manter programas de (C) formular, instituir, coordenar e manter o Sistema
atendimento para a execução das medidas socioeducativas em Municipal de Atendimento Socioeducativo.
meio aberto. (D) garantir defesa técnica (defensor público ou advogado)
III - Compete ____________ estabelecer e manter processo de para o adolescente a quem se atribua a prática de ato
avaliação dos Sistemas de Atendimento Socioeducativo, seus infracional.
planos, entidades e programas.
IV - Compete ____________ estabelecer as hipóteses de 05. (DPU - Assistente Social – CESPE/2016). Segundo as
proibição de entradas de objetos na unidade de internação, normas contidas na legislação social voltada para os direitos
vedando o acesso aos seus portadores. sociais e proteção de crianças e adolescentes, julgue o seguinte
item.
Assinale a sequência que preenche correta e Para o Sistema Nacional de Atendimento Socioeducativo, é
respectivamente as lacunas. prioritária a aplicação de medidas privativas ou restritivas de
(A) à União, ao Estado, ao Regulamento interno, ao liberdade em estabelecimento educacional, de modo a garantir
Município. a inclusão social dos egressos do sistema socioeducativo.
(B) à União, ao Município, ao Estado, ao Regulamento (...) Certo
Interno (...) Errado
(C) ao Estado, à União, ao Regulamento Interno, ao
Município. 06. (Prefeitura de Natal/RN - Administrador –
(D) ao Município, ao Estado, à União, ao Regulamento IDECAN/2016). Sobre a Lei nº 12.594, de 18 de janeiro de
Interno. 2012, que institui o Sistema Nacional de Atendimento
(E) ao Estado, ao Município, à União, ao Regulamento Socioeducativo (Sinase) e trata da execução das medidas
Interno. socioeducativas, assinale a afirmativa INCORRETA.
(A) O Plano Individual de Atendimento – PIA será
02. (SJC-SC - Agente de Segurança Socioeducativo – elaborado sob a responsabilidade e exclusiva participação da
FEPESE/2016) De acordo com a Lei do SINASE (Lei nº equipe técnica do respectivo programa de atendimento.
12.594/2012), a execução das medidas socioeducativas reger- (B) Os Municípios inscreverão seus programas e
se-á pelos seguintes princípios: alterações, bem como as entidades de atendimento

Legislação 89
Apostila Digital Licenciada para Alice Caroline Guarino dos Santos - alice.guarino@hotmail.com (Proibida a Revenda)
APOSTILAS OPÇÃO

executoras, no Conselho Municipal dos Direitos da Criança e (D) A estrutura física da unidade socioeducacional deverá
do Adolescente. ser compatível com as normas de referência do Sinase, sendo
(C) No caso de o maior de dezoito anos, em cumprimento que a direção da unidade deverá adotar medidas para a
de medida socioeducativa, responder a processo-crime, caberá proteção do interno em casos de risco à sua integridade física,
à autoridade judiciária decidir sobre eventual extinção da à sua vida, ou à de outrem, comunicando tais medidas, de
execução, cientificando da decisão o juízo criminal imediato, ao defensor e ao Ministério Público.
competente. (E) O Plano Individual de Atendimento deverá ser
(D) A gravidade do ato infracional, os antecedentes e o elaborado sob a responsabilidade da equipe técnica do
tempo de duração da medida não são fatores que, por si, respectivo programa de atendimento, sendo facultada a
justifiquem a não substituição da medida por outra menos participação do adolescente e de sua família.
grave. Considera-se mais grave a internação, em relação a
todas as demais medidas, e mais grave a semiliberdade, em 10. (Secretaria da Criança/DF - Especialista
relação às medidas de meio aberto. Socioeducativo - Direito e Legislação – FUNIVERSA/2015).
Em relação ao que estabelece o Sistema Nacional de
07. (Prefeitura de Natal/RN - Advogado – Atendimento Socioeducativo (Sinase), assinale a alternativa
IDECAN/2016). De acordo com a Lei nº 12.594, de 18 de correta.
janeiro de 2012, que institui o Sistema Nacional de (A) O Sinase será totalmente financiado com recursos dos
Atendimento Socioeducativo (Sinase) compete aos orçamentos tributário e da assistência social.
Municípios: (B) Para o exercício da função de dirigente de programa de
(A) Garantir defesa técnica do adolescente a quem se atendimento em regime de semiliberdade ou de internação,
atribua prática de ato infracional. além dos requisitos específicos previstos no respectivo
(B) Contribuir para a qualificação e ação em rede dos programa de atendimento, é necessário reputação ilibada,
Sistemas de Atendimento Socioeducativo. formação de nível superior em administração e comprovada
(C) Criar e manter programas de atendimento para a experiência no trabalho com adolescentes de, no mínimo, seis
execução das medidas socioeducativas em meio aberto. meses.
(D) Criar, desenvolver e manter programas para a (C) É vedado ao juiz aplicar nova medida de internação, por
execução das medidas socioeducativas de semiliberdade e atos infracionais praticados anteriormente, a adolescente que
internação. já tenha cumprido medida socioeducativa dessa natureza,
sendo tais atos absorvidos por aqueles aos quais se impôs a
08. (Prefeitura de Fortaleza/CE - Psicologia - medida socioeducativa extrema.
Prefeitura de Fortaleza-CE/2016). De acordo com a Lei nº (D) A gravidade do ato infracional, os antecedentes e o
12.594, de 18 de janeiro de 2012, que institui o Sistema tempo de duração da medida são fatores que justificam, por si,
Nacional de Atendimento Socioeducativo (Sinase) e a não substituição da medida por outra menos grave.
regulamenta a execução das medidas destinadas a adolescente (E) É assegurado ao adolescente em cumprimento de
que pratique ato infracional, assinale a alternativa correta medida socioeducativa de internação que seja casado ou que
quanto aos objetivos dessas medidas. viva, comprovadamente, em união estável o direito à visita
(A) A responsabilização do adolescente quanto às íntima e a receber visita dos filhos, desde que seja respeitada
consequências lesivas do ato infracional, sempre que possível a faixa etária estabelecida no regimento da unidade de
incentivando a sua punição. atendimento.
(B) A integração social do adolescente e a garantia de seus
direitos individuais e sociais, por meio do cumprimento de seu Respostas
plano individual de atendimento e de sua inserção qualificada
no mercado de trabalho após seu desligamento do sistema. 01. E. / 02. A. / 03. Errada. / 04. D / 05. Errado
(C) A desaprovação da conduta infracional, efetivando as 06. A / 07. C / 08. C / 09. D / 10. C
disposições da sentença como parâmetro máximo de privação
de liberdade ou restrição de direitos, observados os limites
previstos em lei.
PLANO NACIONAL DE
(D) A criação e desenvolvimento de programas para a
execução das medidas socioeducativas de semiliberdade e PROMOÇÃO, PROTEÇÃO E
internação, bem como apoio às famílias daqueles adolescentes DEFESA DO DIREITO DE
que cometeram infração devido a sua situação de CRIANÇAS E ADOLESCENTES
vulnerabilidade socioeconômica. À CONVIVÊNCIA FAMILIAR E
09. (Secretaria da Criança/DF - Especialista COMUNITÁRIA – 2006.
Socioeducativo - Artes Plásticas – FUNIVERSA/2015). Em
relação ao que estabelece o Sistema Nacional de Atendimento
Socioeducativo (Sinase), assinale a alternativa correta. Plano Nacional de Promoção, Proteção e Defesa do
(A) A equipe técnica do programa de atendimento deve ser Direito de Crianças e Adolescentes à Convivência
interdisciplinar, compreendendo, no mínimo, profissionais Familiar e Comunitária
das áreas de saúde e de assistência social.
(B) Compete à direção do programa de prestação de Introdução
serviços à comunidade ou de liberdade assistida avaliar, com
o orientador, a evolução do cumprimento de determinada O Plano Nacional de Promoção, Proteção e Defesa do
medida e, se necessário, substitui-la. Direito de Crianças e Adolescentes à Convivência Familiar e
(C) É direito do adolescente submetido ao cumprimento de Comunitária – PNCFC, foi aprovado em 2006 por resolução
medida socioeducativa ser incluído em programa de meio conjunta do Conselho Nacional dos Direitos da Criança e do
aberto quando inexistir vaga para o cumprimento de medida Adolescente – CONANDA e do Conselho Nacional de
de privação de liberdade, independentemente da natureza do Assistência Social – CNAS, destinado a nortear as ações de
ato infracional cometido. promoção, proteção e defesa do direito de crianças e
adolescentes à convivência familiar e comunitária.

Legislação 90
Apostila Digital Licenciada para Alice Caroline Guarino dos Santos - alice.guarino@hotmail.com (Proibida a Revenda)
APOSTILAS OPÇÃO

Tais ações podem ser entendidas como formulação e liberdade e à convivência familiar e comunitária, além de
implementação de políticas públicas que assegurem a garantia colocá-los a salvo de toda forma de negligencia, discriminação,
dos direitos das crianças e adolescentes, de forma integrada e exploração, violência, crueldade e opressão”.
articulada com os demais programas de governo. A CF/88 ainda acabou ainda com a diferença havida entre
O mencionado Plano Nacional foi criado em um contexto filhos havidos ou não da relação do casamento, ou por adoção,
onde as políticas públicas voltadas às crianças e adolescentes, estipulando em que todos teriam os mesmos direitos e
não priorizavam a convivência familiar e comunitária, e a qualificações, proibidas quaisquer designações
legislação era totalmente paternalista, visando apenas a discriminatórias relativas à filiação. E finalizou preconizando
contenção social, o que acabou gerando um aprofundamento em seu artigo 229 que “Os pais têm o dever de assistir, criar e
nas desigualdades sociais, sobretudo na população mais educar os filhos menores (...)".
pobre, que devido a sua situação social precária, eram vistos Posteriormente com a promulgação do Estatuto da Criança
como “incapazes” de cuidar de seus filho, dando sustentação a e do Adolescente – Lei Federal n. 8.069/90, de 13 de julho de
pratica recorrente da suspensão provisória do poder familiar 1990, em consonância com as diretrizes aprovadas na
ou da destituição dos pais e de seus deveres em relação aos Convenção Internacional dos Direitos da Criança e do
filhos. Adolescente, em 20 de novembro de 1989, pode se considerar
Tais consequências e o crescente número das que o país adotou uma nova concepção em relação à
desigualdades sociais, em especial no tocante as condições de formulação e implementação das políticas públicas dirigidas à
vida das crianças e adolescentes, levaram a sociedade a infância e à adolescência.
reavaliar os padrões da assistência social em geral, e a clamar A diferença da concepção adotada é clara. Enquanto
por políticas públicas e legislações que garantissem os direitos anteriormente a legislação tratava as crianças e adolescentes
dessas crianças e adolescentes, especialmente no que diz como objeto, sob regras coercitivas, em geral aplicada as
respeito à família. crianças em situação de vulnerabilidade social e financeira,
As mudanças então começaram a acontecer com a equiparando muitas vezes crianças e adolescentes pobres à
promulgação da Constituição Federal, em 1988, do Estatuto da delinquentes, o Estatuto da Criança e do Adolescente, propõe
Criança e do Adolescente (ECA), em 1990, da Lei Orgânica da atender a todas as crianças e adolescentes, respeitando-os
Assistência Social (LOAS), em 1993 e com a ratificação da como "sujeitos de direitos", dentro da chamada Doutrina da
Convenção sobre os Direitos da Criança em 1990, provocando Proteção Integral.
rupturas em relação às concepções e práticas assistencialistas Dentre as principais inovações do Estatuto da Criança e do
e institucionalizantes. Adolescente podemos destacar:
Essas mudanças passaram a olhar as crianças e - Todas as crianças passam a ter direitos iguais, sem
adolescentes como sujeitos de direitos, instituindo políticas discriminações;
públicas em um sistema conhecido como Sistema de Garantia - É previsto que as crianças e adolescentes são
de Direitos, onde essas crianças e adolescentes são vistas de responsabilidades da família, do Estado e da sociedade;
maneira indissociável do seu contexto sociofamiliar e - As crianças e adolescentes devem ser considerados como
comunitário. prioridade absoluta no conjunto das políticas públicas;
Este Plano constitui um marco nas políticas públicas no - Crianças e adolescentes devem ser tratados como
Brasil, ao romper com a cultura da institucionalização de "pessoas em condição peculiar de desenvolvimento".
crianças e adolescentes e ao fortalecer o paradigma da Já no tocante a questão do acolhimento institucional, mais
proteção integral e da preservação dos vínculos familiares e conhecido como “abrigamento”, podemos destacar que com a
comunitários preconizados pelo Estatuto da Criança e do promulgação do ECA, este tipo de serviço passou e ainda passa
Adolescente. O núcleo principal do Plano Nacional é promover por um processo de reordenamento, ou seja, os serviços estão
à cultura de valorização, respeito e promoção da convivência sendo adequados a realidade das novas políticas públicas, bem
familiar e comunitária. como das legislações.
O PNCFC vem propor ainda o rompimento com a cultura de A nova legislação (ECA) estabelece o caráter de
institucionalização de crianças e adolescentes e o provisoriedade e de excepcionalidade. Ou seja, essa medida
fortalecimento do paradigma da proteção integral e da deverá ser adotada como última alternativa, depois de
preservação dos vínculos familiares e comunitários. esgotadas todas as possibilidades.
Para tal, faz-se necessário o investimento nas políticas Além disso, no capítulo II do Estatuto, que trata do trabalho
públicas de atenção à família; das Entidades de Atendimento, artigos 90 a 94 e no artigo 100,
Os principais temas que orientaram a discussão e a há clareza sobre as responsabilidades das organizações e de
elaboração do PNCF foi a valorização da família, e neste seus dirigentes, no sentido de assegurar que sejam respeitados
contexto as políticas de apoio sociofamiliar e o reordenamento todos os direitos da criança e, sobretudo, da tarefa de
dos abrigos e a implementação de programas de famílias desenvolver ações para a preservação dos vínculos familiares.
acolhedoras. Neste contexto a adoção passa a ser centrada no O reordenamento do serviço de acolhimento institucional
interesse da criança e do adolescente. visa especialmente garantir que as crianças e adolescentes
acolhidos, permaneçam como titulares de direitos, integrados
Marco legal na rede de atendimento de saúde, cultura, educação, nos
serviços da comunidade propiciados pelas demais
Em se tratando dos princípios legais, norteadores e que organizações, envolvendo projetos de dança, teatro, esporte,
dão sustento ao PNCF, temos que a Constituição Federa de lazer, profissionalização e outros que venham frequentando
1988 foi o marco inicial da transformação da política nacional ou tenham necessidade. Além disso, todos os esforços devem
de atenção às crianças e adolescentes. ser realizados para manter juntos os grupos de irmãos.
Em seu artigo 227 a CF/88 estabelece que
Marco conceitual
Constituição Federal
Existem alguns conceitos básicos que precisam ser
Artigo 227. “É dever da família, da sociedade e do Estado apresentados para a devida compreensão do Plano Nacional
assegurar à criança e ao adolescente, com absoluta prioridade, de Convivência Familiar, quais sejam: o termo “Família”, a
o direito à vida, à saúde, à alimentação, à educação, ao lazer, à compreensão da Criança e do Adolescente como sujeitos de
profissionalização, à cultura, à dignidade, ao respeito, à

Legislação 91
Apostila Digital Licenciada para Alice Caroline Guarino dos Santos - alice.guarino@hotmail.com (Proibida a Revenda)
APOSTILAS OPÇÃO

direitos e a conceituação de “Convivência Familiar e Convivência Familiar e Comunitária


Comunitária”.
A convivência familiar e comunitária é um direito
“Família” reservado a toda criança e adolescente de ser criado e educado
no seio de sua família original, e excepcionalmente se
Indispensável, desta forma, citar o artigo 226, §4º da necessário, em família substituta, conforme artigo 19 do
CF/88 que explicita: "Entende-se, também, como entidade Estatuto da Criança e do Adolescente.
familiar a comunidade formada por qualquer dos pais ou O direito à convivência familiar e comunitária é tão
descendentes". Já o Estatuto da Criança e do Adolescente, em importante quanto o direito à vida, à saúde, à alimentação, à
seu art. 25, assim preconiza: "Entende-se por família natural a educação, ao lazer, à profissionalização, à cultura, à dignidade,
comunidade formada pelos pais ou qualquer deles e seus ao respeito e à liberdade.
descendentes". Nesse sentido, verifica-se que o fortalecimento de vínculos
Dessa forma, independente do tipo de arranjo familiar, não sociais e familiares, bem como a inclusão em ambiente onde as
importando se a família seja classificada como "nuclear, relações afetivas são preservadas, assegura a criança e o
monoparental ou reconstituída", a ênfase está na existência de adolescente um melhor desenvolvimento.
vínculos de filiação de origem natural ou adotiva, conferindo- Por esta razão, os Governos, tanto Federal, Estadual
lhe igualdade de direitos. Este formato, seguramente, propõe quanto o Municipal, devem desenvolver políticas públicas de
superar o modelo "ideal de família", que tem ênfase na modo a garantir os direitos de crianças e adolescentes que
"estrutura" para enfatizar a sua capacidade de exercer a estão em situação de vulnerabilidade social e pessoal, com
função de proteção, de socialização e de cuidados das suas vínculos fragilizados ou não e ainda, executá-las de forma
crianças e adolescentes, considerando também outras articulada com toda a rede de atendimento à criança e
possibilidades de arranjos. adolescente.
Desta realidade começa a emergir um modelo de família Dentre essas medidas necessariamente precisa estar o
que se estende para além da unidade casal (papai, mamãe e apoio sociofamiliar as famílias que não conseguem suprir as
criança), da unidade pais e filhos, vivendo ou não no mesmo necessidades básicas de seus filhos, acarretando o
domicílio, na concepção tradicional. Este novo modelo afastamento dessas crianças do seu lar, buscando assim,
considera uma família ampliada, a "família extensa". Em suma, garantir a reintegração desta criança ou adolescente na família
propõe a compreensão para além do espaço físico, para um de origem ou extensa. Caso isto não seja possível, outras
modelo que reconhece e valoriza a rede de vínculos. É preciso medidas devem ser tomadas, como por exemplo, o
compreender a diversidade de organizações familiares, a encaminhamento para família substituta.
complexidade e riqueza dos vínculos familiares e O afastamento da criança e do adolescente só ocorrerá
comunitários. Mais do que isso, é preciso superar o modelo quando não for possível uma intervenção junto à família
estático e reconhecer um modelo dinâmico, ainda que (origem ou extensa), neste caso, a medida terá por objetivo
convivam, concomitantemente, os diferentes modelos e que o garantir o melhor para criança e o adolescente, colocando a
tradicional carregue seu valor intrínseco. Não se trata aqui de salvo de qualquer prejuízo ao seu processo de
valorizar um novo modelo familiar e desconstruir o anterior. desenvolvimento.
Neste momento o Estatuto da Criança e do Adolescente Faz-se necessário destacar, que antes do afastamento da
passa, no artigo 23, preconiza que a pobreza material, a criança e do adolescente e de sua inserção nos serviços ou
falta/carência de recursos materiais não constitui mais motivo programas de acolhimento institucional, os órgãos
suficiente para a perda ou suspensão do poder familiar. Não competentes devem promover ações de fortalecimento e de
existindo outros motivos que levem o Poder Judiciário a inclusão social dessas famílias nas políticas sociais públicas,
afastar a criança ou adolescente da sua família de origem, como também em redes sociais e comunitárias, visando
compete então exigir que essa família seja obrigatoriamente potencializar o núcleo familiar para que este possa garantir a
inserida em programas oficiais de auxílio. Essa atribuição é da convivência saudável de crianças e adolescentes no seio da
responsabilidade dos gestores das políticas sociais no família e da comunidade.
município e pode ser requerida pelo Ministério Público, caso O Estatuto da Criança e do Adolescente busca em sua
não esteja sendo cumprida. essência garantir à criança e ao adolescente os direitos
fundamentais inerentes à pessoa humana, sem prejuízo da
A criança e o adolescente como “sujeitos de direitos” proteção integral, logo, estes também devem ser os princípios
norteadores dos serviços sociais de atendimento à criança e ao
Nesse sentido, a legislação passa garantir que as crianças e adolescente, em especial aquelas que estão com os vínculos
adolescentes sejam vistos como indivíduos autônomos e familiares e comunitários fragilizados e/ou rompidos.
íntegros, dotados de personalidade e vontade próprias que, na
sua relação com o adulto, não podem ser tratados como seres Diretrizes
passivos, subalternos ou meros “objetos”, devendo participar
das decisões que lhes dizem respeito, sendo ouvidos e São diretrizes básicas do Plano Nacional de Convivência
considerados em conformidade com suas capacidades e grau Familiar e Comunitária:
de desenvolvimento. - Centralidade da família nas políticas públicas: O direito
Significa ainda que a sociedade deve garantir a estas das crianças e adolescentes à convivência familiar e
crianças e adolescentes o pleno desenvolvimento, no seio de comunitária está relacionado à inclusão social de suas famílias.
uma família e de uma comunidade, ou prestar-lhes cuidados Significa dizer que a família é a matriz das relações sociais, ou
alternativos temporários, quando afastados do convívio com a seja, é o núcleo de acolhida, convívio, autonomia,
família de origem, o que passa a ser cumprimento de deveres sustentabilidade e protagonismo social e deve ser apoiada e
para com a criança e o adolescente e o exercício da ter acesso a condições para responder ao seu papel no
responsabilidade da família, da sociedade e do Estado. sustento, na guarda e na educação de suas crianças e
Todas essas garantias são condições peculiares da criança adolescentes, bem como na proteção de seus idosos e
e do adolescente como pessoa em desenvolvimento. portadores de deficiência.
- Primazia da responsabilidade do Estado no fomento de
políticas integradas de apoio à família: O Estado é o maior
responsável por garantir e oferecer as políticas públicas

Legislação 92
Apostila Digital Licenciada para Alice Caroline Guarino dos Santos - alice.guarino@hotmail.com (Proibida a Revenda)
APOSTILAS OPÇÃO

adequadas à prevenção e superação das situações de violação convivência familiar e comunitária. Trata-se sempre de uma
de direitos, possibilitando o fortalecimento dos vínculos medida excepcional, mas que garante a criança e ao
familiares e sócio-comunitários. Todo este apoio às famílias, adolescente o direito de crescer e se desenvolver em uma
deve ser realizado através da articulação de atendimento das família substituta. Na intenção de manutenção do conceito de
diversas políticas públicas ofertadas. família e que se deve preservar a união de grupos de irmão
- Reconhecimento das competências da família na sua num processo de adoção. A adoção é um processo que
organização interna e na superação das suas dificuldades: As necessariamente precisa ocorrer com a intervenção do Poder
políticas especiais para promoção, proteção e defesa do direito Judiciário, visando dar legalidade e transparência ao ato.
de crianças e adolescentes à convivência familiar e - Controle social das políticas públicas: A sociedade, na
comunitária não devem ser assistencialistas, ou seja, devem busca de garantir os direitos fundamentais e os princípios
reconhecer a família como um grupo social capaz de se democráticos, deve realizar o controle social das políticas
organizar e reorganizar dentro de seu contexto e a partir de públicas. Isso significa dizer que precisam existir espaços
suas demandas e necessidades, bem como rever e reconstruir democráticos para as discussões das instituições e sociedade
seus vínculos ameaçados, a partir do apoio recebido das civil organizada, com objetivo de fiscalizar e solucionar
políticas sociais. problemas que afligem a população infanto-juvenil.
Reconhece-se aqui uma metodologia de participativa, que
envolva a família, pactuando responsabilidades e Objetivos gerais
compromissos, metas e objetivos. As ações são desenvolvidas
com e pelas famílias. Dentre os objetivos gerais do Plano Nacional de
- Respeito à diversidade étnico-cultural, à identidade e Convivência Familiar e Comunitária, podemos citar:
orientação sexuais, à equidade de gênero e às particularidades a) Ampliar, articular e integrar as diversas políticas,
das condições físicas, sensoriais e mentais: Neste ponto o programas, projetos, serviços e ações de apoio sociofamiliar
objetivo é combater os estigmas sociais através da promoção para a promoção, proteção e defesa do direito de crianças e
dos direitos humanos. A meta também é desenvolver adolescentes à convivência familiar e comunitária;
programas sociais que incentivem os laços de solidariedade b) Difundir uma cultura de promoção, proteção e defesa do
social e respeito ao próximo. Os princípios que norteiam está direito à convivência familiar e comunitária, em suas mais
diretriz são o da discriminação e do respeito à diversidade. variadas formas, extensiva a todas as crianças e adolescentes,
- Fortalecimento da autonomia do adolescente e do jovem com ênfase no fortalecimento ou resgate de vínculos com suas
adulto na elaboração de seu projeto de vida. Nesta diretriz, famílias de origem;
busca-se garantir aos jovens e adolescentes direito de c) Proporcionar, por meio de apoio psicossocial adequado,
participar das formulações das políticas públicas que os a manutenção da criança ou adolescente em seu ambiente
afetam, com direito de voz e de participação. É preciso ouvir familiar e comunitário, considerando os recursos e
estes jovens e adolescentes, para reconhecer suas habilidades, potencialidades da família natural, da família extensa e da rede
competências, interesses e necessidades. Quando os jovens e social de apoio;
adolescentes adotam um modelo participativo na sociedade, d) Fomentar a implementação de Programas de Famílias
isso fortalece sua autonomia e lhes garante condições para Acolhedoras, como alternativa de acolhimento a crianças e
elaborar um projeto de vida mais justo e consciente. adolescentes que necessitam ser temporariamente afastados
- Garantia dos princípios de excepcionalidade e da família de origem, atendendo aos princípios de
provisoriedade dos PFA e de Programa de Acolhimento excepcionalidade e de provisoriedade, estabelecidos pelo
Institucional (PAI): As medidas de afastamento de crianças e Estatuto da Criança e do Adolescente, bem como assegurando
adolescentes dos seus lares, deve ser vista sempre como uma parâmetros técnicos de qualidade no atendimento e
medida temporária, com uma ruptura de vínculos temporária, acompanhamento às famílias acolhedoras, às famílias de
devendo imediatamente a Entidade de Acolhimento iniciar o origem, às crianças e aos adolescentes;
trabalho de restabelecimento do vínculo e das condições de e) Assegurar que o Acolhimento Institucional seja
autonomia desta família. efetivamente utilizado como medida de caráter excepcional e
Ou seja, a regra geral é pela permanência e a manutenção provisório, proporcionando atendimento individualizado, de
da família de origem e a exceção, o acolhimento institucional. qualidade e em pequenos grupos, bem como proceder ao
Importante ressaltar, que a decisão pela separação da família reordenamento institucional das entidades para que sejam
é sempre de um técnico competente e somente será tomada adequadas aos princípios, diretrizes e procedimentos
após o esgotamento de todas as possibilidades de manutenção estabelecidos no ECA;
da família de origem ou extensa. A simples precariedade f) Fomentar a implementação de programas para
financeira não constitui motivo para o acolhimento promoção da autonomia do adolescente e/ou jovem egressos
institucional. de programas de acolhimento, desenvolvendo parâmetros
- Reordenamento dos Programas de Acolhimento para a sua organização, monitoramento e avaliação;
Institucional: O Reordenamento Institucional surge para g) Aprimorar os procedimentos de adoção nacional e
reorientar os serviços de acolhimento, que antes eram vistos internacional, visando: a) estimular, no País, as adoções de
como abrigamento, mantendo crianças e adolescentes crianças e adolescentes que, por circunstâncias diversas, têm
isolados do convívio social, para se alinharem ao modelo de sido preteridos pelos adotantes – crianças maiores e
família como a unidade básica da ação social, não sendo mais adolescentes, com deficiência, com necessidades específicas
concebido este isolamento. Busca-se o desenvolvimento de de saúde, afrodescendentes ou pertencentes a minorias
programas para reorientar as redes públicas e privadas para étnicas, dentre outros; b) investir para que todos os processos
se alinharem à mudança de paradigma. Os serviços de de adoção no País ocorram em consonância com os
reordenamento deverão ter como base, as orientações procedimentos legais previstos no Estatuto da Criança e do
recomendadas pelo Conselho Nacional dos Direitos da Criança Adolescente; e c) garantir que a adoção internacional ocorra
e do Adolescente (Conanda) e pelo Conselho Nacional de somente quando esgotadas todas as tentativas de adoção em
Assistência Social. território nacional, sendo, nestes casos, priorizados os países
- Adoção centrada no interesse da criança e do que ratificaram a Convenção de Haia;
adolescente: esse tópico visa esclarecer que a adoção não se h) Assegurar estratégias e ações que favoreçam os
trata de “dar” um filho a quem ainda não tem, mas sim de uma mecanismos de controle social e a mobilização da opinião
forma de garantir a estas crianças e adolescentes o direito à pública na perspectiva da implementação do Plano Nacional

Legislação 93
Apostila Digital Licenciada para Alice Caroline Guarino dos Santos - alice.guarino@hotmail.com (Proibida a Revenda)
APOSTILAS OPÇÃO

de Promoção, Proteção e Defesa do Direito de Crianças e Eixo 2 – Atendimento


Adolescentes à Convivência Familiar e Comunitária;
i) Aprimorar e integrar mecanismos para o - Articulação e integração entre as políticas públicas;
cofinanciamento, pela União, Estados, Distrito Federal e - Sistematização e difusão de metodologias de trabalho
Municípios, das ações previstas no Plano Nacional de com famílias e comunidades;
Promoção, Proteção e Defesa do Direito de Crianças e - Ampliação e estruturação da oferta de serviços de ASF,
Adolescentes à Convivência Familiar e Comunitária, tendo que contribuam para o empoderamento das famílias;
como referência a absoluta prioridade definida no artigo 227 - Reordenamento dos serviços de Acolhimento
da Constituição Federal de 1988 e no artigo 4° do Estatuto da Institucional e Implementação de PFA;
Criança e do Adolescente. - Implantação, ampliação e implementação de Programas e
serviços de preparação de adolescentes e jovens em
Resultados programáticos Acolhimento Institucional, para a autonomia.
- Fortalecimento de vínculos familiares de adolescentes em
Como resultado da implantação do Plano Nacional, prevê- cumprimento de medida socioeducativa, sobretudo privativas
se a concretização do fortalecimento das políticas públicas de liberdade, bem como de filhos com pais privados de
especificadas de proteção à criança e adolescente, que liberdade;
garantam efetivamente o acesso a políticas, programas, - Articulação entre os serviços de AI e o SGD, em particular
projetos, serviços e ações intersetoriais, materializando dessa o judiciário, de modo a evitar o “esquecimento” de crianças e
forma o direito fundamental dessas crianças e adolescentes. adolescentes nestas instituições;
- Aprimoramento dos procedimentos de Adoção nacional e
Implementação, monitoramento e avaliação internacional, priorizando a adoção nacional;
- Capacitação e assessoramento aos municípios para a
Para a materialização deste direito será necessário: implementação das ações do PNCFC;
- Cumprimento integral do Plano nas três esferas do - Consolidação de uma rede nacional de identificação e
governo. O Plano Nacional precisa ser integralmente localização de crianças e adolescentes desaparecidos e de pais
desenvolvido na União, nos Estados e nos Municípios. e responsáveis.
- Constituição formal de Comissão Nacional Intersetorial
(Grupo de Trabalho). Necessário à constituição de comissão Eixo 3 – Marcos Normativos e Regulatórios
intersetorial, para acompanhar o desenvolvimento e a
implementação dos planos estaduais e municipais. - Parametrização e regulamentação dos PASF, PFA e PAI e
- Elaboração de Planos Estaduais e Municipais e de apadrinhamento;
constituição de Comissões Intersetoriais; - Regulamentação e aplicação dos conceitos de
- Conselhos dos Direitos da Criança e do Adolescente (três “excepcionalidade e provisoriedade”;
esferas) assumindo o Plano como prioridade, a partir de 2007 - Regulamentação dos programas e serviços de
– viabilizando recursos nos orçamentos; Acolhimento Familiar;
- Participação e integração entre os Conselhos de Direitos - Aprimoramento dos instrumentos legais de proteção
da Criança e Setoriais nas três esferas de governo; social que ofereçam alternativas e a possibilidade do
- Corresponsabilidade entre os entes federativos no contraditório à suspensão ou destituição do poder familiar.
financiamento para a implementação do Plano. Vale ressaltar
que o Plano Nacional já especificou responsabilidades que Eixo 4 – Mobilização, Articulação e Participação
devem ser compartilhadas pelas três esferas de governo.
- Estratégias de comunicação social para mobilização da
Plano de Ação sociedade e afirmação de novos valores;
- Mobilização e articulação para a garantia da
Foi estipulado, que o conjunto das ações será provisoriedade e excepcionalidade do Acolhimento
implementado e implantado no horizonte de 09 anos (2007- Institucional;
2015), ficando estabelecidos os seguintes intervalos: - Produção e divulgação de material de orientação e
- Curto Prazo: 2007-2008 capacitação;
- Médio Prazo: 2009-2011 - Articulação e integração de ações entre as três esferas de
- Longo Prazo: 2012-2015 Poder;
- Ações permanentes: 2007-2015 - Garantia de recursos para viabilização do Plano.
As propostas operacionais PNCFC estão organizadas em Neste momento, visando garantir uma visão mais
quatro eixos estratégicos: aprofundada e detalhada sobre o tema, necessários se faz a
leitura da íntegra do Plano Nacional de Promoção, Proteção e
Eixo 1 – Análise da Situação e Sistemas de Informação Defesa do Direito de Crianças e Adolescentes à Convivência
Familiar e Comunitária, o qual apresentamos a seguir:
- Levantamento de dados visando o conhecimento em
relação à situação familiar, identificando os fatores que Plano Nacional de Promoção, Defesa e Garantia do
favorecem ou ameaçam a CFC; Direito de Crianças e Adolescentes à Convivência
- Mapeamento e análise das iniciativas de PASF, PFA, PAI e Familiar e Comunitária.
Adoção e sua adequação aos marcos legais;
- Aprimoramento e valorização da comunicação entre os 1. Antecedentes
Sistemas de Informação sobre crianças, adolescentes e família.
A legislação brasileira vigente reconhece e preconiza a
família,4 enquanto estrutura vital, lugar essencial à

4 Com base no texto constitucional e infraconstitucional, define-se modelo familiar “normal”. Trata-se, portanto, de saber se a família é capaz
família como um grupo de pessoas, com laços de consanguinidade e/ou de de realizar as funções de proteção e de socialização das suas crianças e
aliança e/ou de afinidade, cujos vínculos circunscrevem obrigações adolescentes em uma diversidade de arranjos familiares e culturais, mas
recíprocas, organizadas em torno de relações de geração e de gênero. A já não mais de se perguntar pela forma ou estrutura da família.
amplitude desta definição derruba qualquer ideia preconcebida de

Legislação 94
Apostila Digital Licenciada para Alice Caroline Guarino dos Santos - alice.guarino@hotmail.com (Proibida a Revenda)
APOSTILAS OPÇÃO

humanização e à socialização da criança e do adolescente, Diante do desafio de garantir efetivamente o direito à


espaço ideal e privilegiado ao desenvolvimento integral dos convivência familiar e comunitária de crianças e adolescentes,
indivíduos. o então Departamento da Criança e do Adolescente (DCA) do
Contudo, a história social da criança, do adolescente e da Ministério de Justiça (MJ), a Secretaria de Estado de
família foi marcada pela dificuldade da família em proteger e Assistência Social (SEAS) do Ministério da Previdência e
educar seus filhos. O discurso de uma “incapacidade” da Assistência Social (MPAS) e o Fundo das Nações Unidas para
família foi assumido pelo Poder Público, que passou a Infância (UNICEF) reuniram-se, no primeiro semestre de
desenvolver políticas paternalistas voltadas para o controle e 2002, com a finalidade de discutir os dados apresentados pela
a contenção social, principalmente para a população mais Caravana da Comissão de Direitos Humanos da Câmara dos
pobre. Essa desqualificação das famílias pobres, tratadas como Deputados5 sobre os programas de abrigo. A partir dessa
incapazes, deu sustentação ideológica à prática recorrente da situação problema percebeu-se que para ampliar e qualificar o
suspensão provisória do poder familiar ou da destituição dos debate fazia-se necessário integrar novos atores sociais no
pais e de seus deveres em relação aos filhos. processo.
A engenharia construída com o sistema de proteção e Assim, em agosto de 2002 foi realizado o “Colóquio Técnico
assistência, sobretudo durante o século passado, permitiu que sobre Rede Nacional de Abrigos”, que contou com a
qualquer criança ou adolescente, por sua condição de pobreza, participação de Secretarias Estaduais de Assistência Social, e
estivesse sujeita a se enquadrar no raio da ação da justiça e da entidades não-governamentais dos diferentes estados
assistência, que sob o argumento de “prender para proteger” brasileiros envolvidos com a temática.
confinavam-nas em grandes instituições totais. Nesse evento foram identificadas ações a serem
Essas representações negativas sobre as famílias cujos priorizadas, entre elas: a realização de um censo nacional de
filhos formavam o público da assistência social e demais crianças e adolescentes em abrigos e práticas institucionais e
políticas sociais tornaram-se parte estratégica das políticas de a elaboração de um Plano de Ação para o reordenamento de
atendimento, principalmente da infância e da juventude, até abrigos. Para o encaminhamento das decisões deliberadas no
muito recentemente. Colóquio, constituiu-se o “Comitê Nacional para
O aprofundamento das desigualdades sociais, com todas as Reordenamento de Abrigos”, com objetivo de estimular
suas consequências, principalmente para as condições de vida mudanças nas políticas e práticas de atendimento, efetivando
das crianças e dos adolescentes, levou à revisão dos uma transição para o novo paradigma legal, Estatuto da
paradigmas assistenciais cristalizados na sociedade. O olhar Criança e do Adolescente, a respeito do direito de crianças e
multidisciplinar e intersetorial iluminou a complexidade e adolescentes á convivência familiar e comunitária. O Comitê
multiplicidade dos vínculos familiares. O coroamento destas foi composto pelos seguintes órgãos e organizações: DCA,
mudanças aconteceu com a promulgação da Constituição SEAS, FONSEAS, CNAS, CONANDA, Colegiado do Fórum
Federal, em 1988, do Estatuto da Criança e do Adolescente Nacional de Conselheiros Tutelares, RENIPAC, UNICEF e
(ECA), em 1990, da Lei Orgânica da Assistência Social (LOAS), Fundação ORSA. Este grupo realizou três encontros6 ainda em
em 1993 e com a ratificação da Convenção sobre os Direitos da 2002, e concluiu pela relevância do levantamento nacional de
Criança em 1990, provocando rupturas em relação às abrigos, porém, dado a limitações de recursos e tempo,
concepções e práticas assistencialistas e institucionalizantes. delimitando o universo da pesquisa para os programas de
Trata-se da mudança do olhar e do fazer, não apenas das abrigos que faziam parte da Rede de Serviço de Ação
políticas públicas focalizadas na infância e na juventude, mas Continuada (Rede SAC). 7 8No final de 2002 o CONANDA, e o
extensivos aos demais atores sociais do chamado Sistema de DCA do Ministério de Justiça, aprovaram/alocaram recursos
Garantia de Direitos, implicando a capacidade de ver essas para financiar esta pesquisa. Em 2003, o Instituto de Pesquisa
crianças e adolescentes de maneira indissociável do seu Econômica Aplicada (IPEA) iniciou a pesquisa.
contexto sociofamiliar e comunitário. No inicio de 2004, no seu Planejamento Estratégico para o
Crianças e adolescentes têm o direito a uma família, cujos exercício 2004-2005, o CONANDA elegeu como uma de suas
vínculos devem ser protegidos pela sociedade e pelo Estado. prioridades a promoção do direito à convivência familiar e
Nas situações de risco e enfraquecimento desses vínculos comunitária de crianças e adolescentes. Por parte do Poder
familiares, as estratégias de atendimento deverão esgotar as Executivo, o Ministro Chefe da Secretaria Especial dos Direitos
possibilidades de preservação de tais vínculos, aliando o apoio Humanos e o Ministro de Estado de Desenvolvimento Social e
socioeconômico à elaboração de novas formas de interação, Combate à Fome se articularam e propuseram a convocação de
referências morais e afetivas no grupo familiar. outros Ministérios e atores numa Comissão Intersetorial.
No caso de ruptura desses vínculos, o Estado é o Nesse novo momento, de maior integração intersetorial,
responsável pela proteção das crianças e dos adolescentes, ampliou-se o escopo temático para além da proposta inicial de
incluindo o desenvolvimento de programas e estratégias que reordenamento dos abrigos. A incorporação das questões
possam levar a constituição de novos vínculos familiares e sobre Família e Adoção tornou necessário redimensionar o
comunitários, mas sempre tendo em vista a possibilidade de grupo de trabalho, criando-se a Comissão Intersetorial que
resgate dos vínculos originais. teria, agora, como finalidade superior, construir subsídios
para a elaboração do “Plano Nacional de Promoção, Defesa e

5 A Caravana, realizada de setembro a dezembro de 2001, percorreu Cadastro existente na SEAS reduzia-se aos serviços de abrigos que
oito estados brasileiros com o objetivo de verificar a real situação dos recebiam subvenção do Fundo Nacional de Assistência Social, não se tendo
programas de abrigos para crianças e adolescentes. Os resultados da dimensionado a representação dessa rede diante do universo das
Caravana foram apresentados no Caderno Especial do jornal Correio organizações que executavam tal medida de proteção e iii) A realização de
Braziliense datado de 09/01/ 2002. um Censo implicaria na cobertura de toda a rede, necessitando para tanto
6 Os três encontros aconteceram em Brasília/DF nas seguintes datas: de um banco de dados que identificasse a localização de cada um dos
1ª reunião – 24/09/2002; 2ª reunião – 22/10/2002 e 3ª reunião – abrigos ou instituições que operavam o abrigamento de crianças e
22/11/2002. adolescentes.
7 A decisão pela proposta de Levantamento da Rede de Abrigos com 8 Ressalta-se que, com a aprovação da NOB/SUAS em julho de 2005 e

base na Rede SAC em detrimento de um Censo Nacional (proposta inicial e das portarias nº 440 e nº 442 do MDS, os recursos do cofinanciamento
mais completa) deu-se em razão de que a realização desse Censo Nacional federal das ações socioassistenciais passam a ser transferidos por “Pisos
seria uma tarefa praticamente impossível diante do quadro de ausência de de Proteção”, cujos recursos poderão ser utilizados conforme a
dados que possibilitassem a identificação de todas as instituições que necessidade local, dentro das ações passíveis de financiamento por cada
executavam serviços dessa natureza, destacando-se: i) Muitos municípios piso. Cabe ao gestor local e ao CMAS a definição da rede de atendimento. O
não possuíam Conselhos dos Direitos da Criança e do Adolescente (órgãos Piso de Alta Complexidade I pode ser utilizado para a manutenção dos
responsáveis pelos registros das entidades e serviços de abrigos); ii) O serviços da rede de acolhimento para crianças e adolescentes.

Legislação 95
Apostila Digital Licenciada para Alice Caroline Guarino dos Santos - alice.guarino@hotmail.com (Proibida a Revenda)
APOSTILAS OPÇÃO

Garantia do Direito de Crianças e Adolescentes à Convivência complexidade do tema e das múltiplas variáveis que interagem
Familiar e Comunitária”. Esta Comissão Intersetorial foi em cada dimensão da realidade focalizada.
nomeada por decreto presidencial de 19 de outubro de 2004 e Em 15 de abril de 2005 o documento contendo os
composta por cinco Ministérios, cada um com obrigação de “Subsídios para a elaboração do Plano de Promoção, Defesa e
orçar recursos para a nova política. Foram também convidadas Garantia do Direito de Crianças e Adolescentes à Convivência
representações dos três poderes e da sociedade civil. A Familiar e Comunitária” foi apresentado ao MDS e ao SEDH, em
Comissão Intersetorial teve noventa dias para a elaboração do cerimônia oficial, e contou com a presença dos Conselhos
documento, tendo seu prazo ampliado para abril de 2005. Nacionais dos Direitos da Criança e do Adolescente
A composição dessa Comissão, de acordo com o decreto, (CONANDA) e da Assistência Social (CNAS). Nesta solenidade
obedeceu à lógica da intersetorialidade. Articulou atores ficou definido o prazo de dois meses para que o Poder
institucionais dos três poderes da República, das três esferas Executivo Federal fizesse a readequação programática e
de poder, das diferentes políticas sociais básicas, da área de orçamentária e em seguida encaminhasse o documento ao
planejamento do Governo Federal, das instâncias de CONANDA e CNAS para a aprovação conjunta do Plano
participação e controle social que integram o Sistema de Nacional. Neste período, o Governo Federal desenvolveu um
Garantia de Direitos, das entidades de atendimento, bem como importante trabalho de análise das ações, dos programas e dos
do Conselho Nacional dos Direitos da Criança e do Adolescente respectivos orçamentos, acrescentando nas tabelas dos
(CONANDA), Fórum Colegiado Nacional dos Conselheiros quatros eixos do Plano as estratégias relevantes,
Tutelares, do Conselho Nacional de Assistência Social (CNAS) possibilitando a inserção da temática em programas bem
e do Conselho Nacional dos Direitos da Pessoa Portadora de como sua articulação. Conselheiros do CONANDA e do CNAS
Deficiência (CONADE), além de entidades civis de âmbito participaram como titulares da Comissão Intersetorial,
nacional, que militam pelo direito de crianças e adolescentes à contribuindo e acompanhando todo o processo e, sobretudo,
convivência familiar e comunitária. Participou também desse informando os respectivos Conselhos a respeito. Ao
esforço, o Fundo das Nações Unidas para a Infância (UNICEF), receberem o documento “Subsídios para a elaboração do Plano
apoiando tecnicamente os trabalhos da Comissão mediante a de Promoção, Defesa e Garantia do Direito de Crianças e
contratação de consultores, e pelo aporte ao debate do “estado Adolescentes à Convivência Familiar e Comunitária”, ambos os
da arte” da discussão internacional que se trava sobre o Conselhos nomearam Comissões (CNAS – a Comissão de
mesmo tema. Política e CONANDA – uma Subcomissão ad-hoc da Comissão
Além da participação dos seus membros, a Comissão de Políticas Públicas), para discussão e encaminhamento de
Intersetorial não prescindiu da valiosa contribuição de contribuições. Os Presidentes de ambos os Conselhos
colaboradores dos campos jurídico, técnico, acadêmico e lideraram este processo, facilitando reunião conjunta entre as
midiático, bem como dos diferentes atores sociais do sistema Comissões dos Conselhos para a consideração do documento.
de atendimento, entre eles as famílias que participaram deste Num momento de intensa parametrização de ambos – o
processo, que proferiram palestras ou deram seus Sistema Único de Assistência Social (SUAS) e o Sistema de
depoimentos durante as jornadas de trabalho, enriquecendo Garantia de Direitos da Criança e do Adolescente (SGD) – esse
sobremaneira a discussão. esforço de deliberação conjunta constitui-se como elemento
Entre novembro de 2004 e março de 2005, a Comissão estratégico. Uma política de promoção, defesa e garantia do
realizou quatro reuniões ordinárias.9 Em cada uma, foram direito da criança e do adolescente à convivência familiar e
discutidas questões referentes a um dos quatro Eixos comunitária perpassa ambos os sistemas e é fundamental para
Estratégicos propostos para o Plano de Ação, a saber: a) o aprimoramento da interface entre eles. Tanto CONANDA
Análise da situação e sistemas de informação; b) Atendimento; quanto CNAS são categóricos ao afirmarem que este direito só
c) Marcos normativos e regulatórios; d) Mobilização, será garantido com a interação de todas as políticas sociais,
articulação e participação. com centralidade na família para acesso a serviços de saúde, a
Para melhor organização do trabalho a Comissão educação de qualidade, geração de emprego e renda entre
Intersetorial optou por dividir-se em três Câmaras Técnicas, outros. Desta forma, as contribuições sobre o papel de cada
cada uma voltada ao aprofundamento de uma das três áreas setor no apoio e garantia do direito à convivência familiar e
temáticas que juntas abarcam as diferentes facetas do Direito comunitária será de grande relevância.
à Convivência Familiar e Comunitária. Em primeiro lugar, a O documento ora intitulado como “Plano Nacional de
família de origem e a comunidade na qual está inserida, a Promoção, Defesa e Garantia do Direito de Crianças e
importância da preservação dos vínculos familiares e Adolescentes à Convivência Familiar e Comunitária – Versão
comunitários e o papel das políticas públicas de apoio Preliminar” – é o produto histórico da elaboração de inúmeros
sociofamiliar. Em segundo lugar, a intervenção institucional atores sociais comprometidos com os direitos das crianças e
nas situações de rompimento ou ameaça de rompimento dos adolescentes brasileiros. O CONANDA e o CNAS, ao
vínculos familiares, os abrigos e as alternativas ao apresentarem o documento esperam contribuir para a
Acolhimento Institucional, com ênfase na excepcionalidade, na construção de um novo patamar conceitual que orientará a
brevidade e na provisoriedade destas medidas e, ainda, na formulação das políticas para que cada vez mais crianças e
restauração e na preservação dos vínculos familiares. Por fim, adolescentes tenham seus direitos assegurados e encontrem
a necessidade de uma nova família para a criança/adolescente na família os elementos necessários ao seu pleno
que perdeu a sua própria. desenvolvimento. Este processo acontece simultaneamente
As três Câmaras Técnicas que trataram de Políticas de com um processo de discussão internacional liderado pelo
Apoio à Família, Medidas de Abrigamento e Adoção foram Comitê dos Direitos da Criança da Organização das Nações
coordenados pelo MDS, o UNICEF e a SEDH respectivamente. Unidas (ONU) sobre a necessidade de aprimorar os
O UNICEF disponibilizou consultores técnicos para mecanismos de proteção integral dos direitos da criança
sistematizar as contribuições dos participantes da Comissão. privada dos cuidados parentais, com recomendações em 2004
Cada uma destas áreas foi objeto de discussões e 2005 da elaboração de nova normativa internacional a esse
aprofundadas e propositivas, abarcando também suas respeito.
interfaces e inter-relações, tentando dar conta da imensa Elaborar um “plano nacional” requer outro e importante
desafio: mobilizar ainda mais outros atores sociais para que se

9 Datas das reuniões: 19 e 20 de novembro de 2004 – Tema: Análise da regulatórios; 21 a 23 de março de 2005 – Tema: Mobilização, articulação e
situação e sistemas de informação; 16 e 17 de dezembro de 2004 – Tema: participação.
Atendimento; 02 a 04 de março de 2005 – Tema: Marcos normativos e

Legislação 96
Apostila Digital Licenciada para Alice Caroline Guarino dos Santos - alice.guarino@hotmail.com (Proibida a Revenda)
APOSTILAS OPÇÃO

integrem a esse movimento, que deve ser coletivo e articulado reforma de toda e qualquer norma reguladora, nesse campo da
na efetivação de direitos, tornando efetiva a participação social família e no embasamento de processos de reforma
e, sobretudo possibilitando o avanço na promoção, defesa e administrativa, de implantação e implementação de políticas,
garantia do direito à convivência familiar e comunitária. programas, serviços e ações públicas. A Convenção das Nações
A promoção, a defesa e a garantia dos direitos das crianças Unidas sobre os Direitos da Criança assegura as duas
e adolescentes à convivência familiar e comunitária envolvem prerrogativas maiores que a sociedade e o Estado devem
o esforço de toda a sociedade e o compromisso com uma conferir à criança e ao adolescente, para operacionalizar a
mudança cultural que atinge as relações familiares, as relações proteção dos seus Direitos Humanos: cuidados e
comunitárias e as relações do Estado com a sociedade. O responsabilidades. As crianças e os adolescentes têm direitos
respeito à diversidade cultural não é contraditório com esta subjetivos à liberdade, à dignidade, à integridade física,
mudança que atravessa os diversos grupos socioculturais, na psíquica e moral, à educação, à saúde, à proteção no trabalho,
defesa desses direitos. Pelo contrário, exige que se amplie a à assistência social, à cultura, ao lazer, ao desporto, à
concepção de cidadania para incluir as crianças e adolescentes habitação, a um meio ambiente de qualidade e outros direitos
e suas famílias, com suas necessidades próprias. individuais indisponíveis, sociais, difusos e coletivos. E
Desafio de dimensões estratégicas, sem dúvida, de cujo consequentemente se postam, como credores desses direitos,
enfrentamento eficaz depende a viabilidade de qualquer diante do Estado e da sociedade, devedores que devem
projeto de nação e de país que se deseje construir agora e no garantir esses direitos. Não apenas como atendimento de
futuro. Eis o nosso desafio! necessidades, desejos e interesses, mas como Direitos
Humanos indivisíveis, como os qualifica a normativa
Marco Legal internacional – como direito a um desenvolvimento humano,
econômico e social. São pessoas que precisam de alguém, de
A Constituição Federal estabelece que a “família é a base da grupos e instituições, responsáveis pela promoção e defesa da
sociedade” (artigo 226) e que, portanto compete a ela, sua “participação, proteção, desenvolvimento e
juntamente com o Estado, a sociedade em geral e as sobrevivência”, responsáveis por seu cuidado, em especial. Em
comunidades, “assegurar à criança e ao adolescente o exercício seu preâmbulo e em muitos dos seus artigos a Convenção
de seus direitos fundamentais” (artigo 227). Por sua vez, o define os direitos da criança realmente num sentido próximo
referido artigo, especifica os direitos fundamentais especiais da Declaração dos Direitos da Criança, da ONU, em 1959,
da criança e do adolescente, ampliando e aprofundando apenas como direito a uma proteção especial: “a criança tem
aqueles reconhecidos e garantidos para os cidadãos adultos no necessidade de uma proteção especial e de cuidados especiais,
seu artigo 5º. E dentre esses direitos fundamentais da notadamente de uma proteção jurídica, antes e depois de seu
cidadania está o direito à convivência familiar e comunitária. nascimento.” Todavia, em outros pontos, a Convenção avança
Em face desse papel de mecanismo de promoção e e acresce a esse “direito à proteção especial”, outros tipos de
proteção dos direitos humanos, no tocante às relações direitos que só podem ser exercidos pelos próprios
familiares, a Constituição Federal rompe com o anterior beneficiários: o direito à liberdade de opinião (artigo12), à
tratamento diferenciado e discriminatório dado aos filhos em liberdade de expressão (artigo 13), à liberdade de
razão da origem do nascimento ou das condições de pensamento, de consciência e de religião (artigo 14), à
convivência dos pais, determinando a equiparação de filhos liberdade de associação (artigo 15). Direitos que pressupõem
havidos ou não da relação do casamento ou por adoção (artigo certo grau de capacidade, de responsabilidade, isto é, que
227 §6º). A mesma Carta Constitucional, em seu artigo 226 §8º pressupõem sujeitos de direitos como titulares. As crianças e
estabelece que ao Estado compete assegurar a assistência à os adolescentes são seres essencialmente autônomos, mas
família na pessoa de cada um dos que a integram, criando com capacidade limitada de exercício da sua liberdade e dos
mecanismos para coibir violências no âmbito de suas relações. seus direitos.
Adiante, no artigo 229 determina que os pais têm o dever de Para efetivação da Convenção sobre os Direitos da Criança,
assistir, criar e educar os filhos menores e os filhos maiores no País é importante que sejam observados os seguintes
têm o dever de ajudar e amparar os pais na velhice, carência princípios:
ou enfermidade. Consequentemente, todo reordenamento
normativo e político-institucional que se pretenda fazer há de - Não discriminação;
partir das normas constitucionais, marco legal basilar para o - Interesse superior da criança;
presente Plano. - Direitos à sobrevivência e ao desenvolvimento;
Respeitando-se essa hierarquia normativa, quando se - Respeito à opinião da criança
tratar desta questão da convivência familiar e comunitária,
igualmente deve ser dada prevalência a toda normativa Regulamentando esses princípios constitucionais e tais
convencional internacional, reguladora da promoção e normas internacionais, a Lei Federal 8.069 de 13 de julho de
proteção dos direitos humanos, ratificada em caráter especial 1990 (Estatuto da Criança e do Adolescente – ECA) reforça o
pelo Brasil10 e àquela estabelecida por força de resoluções da papel da família na vida da criança e do adolescente como
Assembleia Geral das Nações Unidas. Assim sendo, é de se elemento imprescindível dentro do processo de proteção
destacar como marcos normativos a serem considerados as integral, e como um dos objetivos maiores do sistema de
Declarações sobre os Direitos da Criança (1924/1959), a promoção e defesa dos direitos da infância e adolescência, que
Declaração Universal dos Direitos Humanos (1948), a aquela lei propõe instituir, articulando e integrando todas as
Declaração Americana dos Direitos e Deveres do Homem políticas públicas (sociais, institucionais, econômicas e
(1948), o Pacto de São José da Costa Rica (1969), o Pacto infraestruturantes), no sentido da priorização do atendimento
Internacional dos Direitos Civis e Políticos e o Pacto direto desse segmento da população, como forma de garantia
Internacional dos Direitos Econômicos, Sociais e Culturais de direitos: fazer com que o atendimento das necessidades
(1966, ratificados em 1992). básicas das crianças e dos adolescentes seja realizado como
A Convenção sobre os Direitos da Criança, ratificada pelo direito do cidadão-criança e do cidadão-adolescente e ao
Brasil em 24 de setembro de 1990, em especial, tem um papel mesmo tempo dever do Estado, da sociedade e da família, com
superior e preponderante no embasamento da criação ou prioridade absoluta.

10 Para a Convenção a criança é considerada, no seu artigo 1º, menor

de 18 anos.

Legislação 97
Apostila Digital Licenciada para Alice Caroline Guarino dos Santos - alice.guarino@hotmail.com (Proibida a Revenda)
APOSTILAS OPÇÃO

No tocante ao direito à convivência familiar e comunitária, discurso da norma jamais poderá prescindir de qualquer uma
o Estatuto da Criança e do Adolescente estabeleceu no artigo das duas funções: justificadora e modificadora.
19 que toda criança ou adolescente tem direito de ser criado e Muito embora vivamos em um período de exigência de um
educado no seio de sua família e, excepcionalmente, em família processo de positivação do Direito, alguns institutos previstos
substituta, assegurada a convivência familiar e comunitária. na legislação e na doutrina sobre os direitos da criança e do
Esse dispositivo do Estatuto deve ser considerado, em seguida adolescente estão sendo consolidados e, outros ainda,
aos princípios constitucionais e convencionais, como o outro construídos.
marco legal basilar na construção do presente Plano. Em Sendo assim, para este Plano é necessário a verificação
função desse princípio o ECA estabelece a excepcionalidade e mínima do discurso dentro dos nossos marcos legal e
a provisoriedade do Acolhimento Institucional a exemplo do situacional, sem prejuízo das questões jurídicas conflituais que
acolhimento em regime de abrigo, obrigando que se assegure existam e que venham a existir.13
a “preservação dos vínculos familiares e a integração em Entende-se como família natural, nos termos do artigo 25
família substituta quando esgotados os recursos de do ECA, a comunidade formada pelos pais ou qualquer deles e
manutenção na família de origem” (artigos 92 e 100). Não seus antecedentes.
havendo possibilidade de preservar os vínculos com a sua Embora o Estatuto não se utilize expressamente do termo
família natural, o ECA estabelece que o acolhimento em família família extensa, entende-se neste documento como aquela que
substituta se dê provisoriamente via tutela ou guarda ou em inclui, além dos parentes e agregados, todas as demais pessoas
definitivo via adoção (artigos 28 a 52 do ECA), sempre por que tenham relação de afinidade ou de afetividade com o
decisão judicial, processando-se dentro dos princípios e núcleo familiar natural, como se depreende do texto do artigo
requisitos previstos na citada Lei 8.069/90, aplicando-se 28, parágrafo 2º (ECA)14 não se exigindo que as pessoas
quando for o caso, subsidiariamente, as regras do Código Civil. residam no mesmo domicílio.
Nesse ponto, essa regulação das diversas formas de É fato que o acolhimento informal de filhos de outra pessoa
acolhimento familiar citadas não foi alterado pelo novo Código diz respeito à prática secular e disseminada em todo o país,15
Civil (2002) e por nenhuma outra posterior ao ECA. ficando caracterizada esta situação quando os pais,
A colocação em família substituta dar-se-á meio de decisão voluntariamente, delegam seu papel parental a outro membro
judicial e somente tendo lugar quando comprovadamente da família extensa ou, simplesmente, por sua omissão,
representar para a criança e o adolescente a melhor medida permitem que haja esta transferência. É sabido que não são
para sua proteção e desenvolvimento. Essa nova família deve poucas as crianças e adolescentes que passam a maior parte de
proporcionar um ambiente familiar adequado (art. 29 do ECA) sua vida sendo criadas por parentes, padrinhos ou amigos
e devem ser excluídas de sua convivência pessoas próximos dos seus pais (família extensa). Esta prática antiga
dependentes de substâncias entorpecentes, pessoas que os que se denomina por processo de circulação de crianças e de
submetam a maus-tratos, ou lhes imponham tratamento adolescentes16 ainda persiste e persistirá por ser natural e
desumano, violento, aterrorizante, vexatório e constrangedor culturalmente legitimada. Nestes casos, a regularização da
ou que pratiquem exploração, abuso, crueldade e opressão situação da criança ou do adolescente vai exigir apenas uma
(artigos 5º, 18 e 19 do ECA). solução judicial, consistente na colocação em família pelos
Em respeito ao disposto nos artigos 226 e 227 da instrumentos jurídicos previstos no art. 28 do Estatuto da
Constituição Federal, no tocante ao direito à convivência Criança e do Adolescente.
familiar e comunitária, as leis orgânicas das políticas sociais Sendo assim, não há que se falar em acolhimento “formal”
foram sendo editadas e reformadas aprofundando esses na hipótese de simples colocação em guarda ou tutela, ou até
princípios constitucionais, regulamentados pelo Estatuto da mesmo de adoção de criança ou de adolescente, mantendo-se
Criança e do Adolescente, tornando-os operacionais, com a na sua família extensa, como forma de regularização jurídica
construção de sistemas de atendimento de direitos, de uma situação protetora ou para se efetivar a reintegração
especializados. Assim, se procedeu com a promulgação da Lei dessas em sua família.
Orgânica da Assistência Social, da Lei Orgânica da Saúde, da Lei O termo acolhimento “formal”, a que se refere este Plano,
de Diretrizes e Bases da Educação. corresponde à modalidade de atendimento de serviço de
proteção especial de alta complexidade17 que garanta proteção
Marco Conceitual11 integral, incluindo a moradia, alimentação, higienização, bem
como os demais cuidados para crianças e adolescentes que se
O Estatuto da Criança e do Adolescente (ECA) tem uma encontram sem referência e/ ou em situação de ameaça, que
redação que facilita a sua interpretação e reduz a possibilidade não possam permanecer em seu núcleo familiar ou
de divergências extremadas sobre as noções de seus institutos. comunitário.18 Ou seja, “trata-se de uma prática mediada por
Como toda norma esta possui o seu discurso. Este discurso é uma autoridade, com um plano de intervenção definido,
dotado de racionalidade, na medida em que oferece razões administrada por um serviço com recursos disponíveis,
para aquilo que é dito,12 e tudo isto decorre da verificação da conforme política pública estabelecida.”19
realidade e dos conceitos que todos temos desta. Entretanto, o Este “acolhimento formal”, definido como o ato de criar o
filho de uma pessoa, não pode ser realizado por pessoa da

11 ''Na Oficina de revisão jurídica do presente Plano Nacional, Violência de Gênero: Poder e Impotência. Rio de Janeiro, Revinter, 1995, p.
organizado pelo CONANDA com especialistas da área em 18 de abril de 71.
2006, surgiu a necessidade de introduzir maior detalhamento sobre Marco 15 SERRA, Márcia Milena Pivatto. O Brasil das muitas mães: aspectos

Conceitual, especialmente para dirimir dúvidas quanto ao conceito de demográficos da circulação de crianças. Tese de Doutorado no PPG
“Acolhimento Familiar” utilizado no texto, antes de submetê-lo à Consulta Antropologia – UNICAMP, 2003.
Pública. Ressalta-se que o conteúdo do Marco Conceitual não foi objeto de 16 FONSECA, Cláudia, TERTO, Veriano e ALVES, Caleb F. Antropologia,

apreciação do Conanda e do CNAS. Portanto, este capítulo, assim como os diversidade e direitos humanos: diálogos interdisciplinares. Porto Alegre:
demais, devem receber as críticas e sugestões pertinentes quando da Editora da UFRGS, 2004.
consulta pública. 17 Ministério do Desenvolvimento Social e Combate à Fome e
12 FERRAZ JR, Tércio Sampaio. Direito, retórica e comunicação. Saraiva: Secretaria Nacional de Assistência Social – Brasília, 2004, pág. 32.
São Paulo, 1973, p. 126. 18 Política Nacional de Assistência Social (2.5.2.) – Brasília, 2004.
13 Em se tratando de interpretação é perfeitamente natural que 19 CABRAL, Cláudia (Org.). “Perspectivas do Acolhimento Familiar no

aconteça. Assim, justifica-se a elaboração deste marco conceitual e de Brasil”. Acolhimento Familiar – experiências e perspectivas. Rio de
glossário neste Plano. Janeiro: Booklink, 2004, p. 11.
14 Sobre o conceito de família extensa ligada pela consanguinidade e

pela afinidade ver também Saffioti, Heleieth I. B. e Almeida, Suely de Souza.

Legislação 98
Apostila Digital Licenciada para Alice Caroline Guarino dos Santos - alice.guarino@hotmail.com (Proibida a Revenda)
APOSTILAS OPÇÃO

mesma família – natural ou extensa –, mas por uma pessoa, Este atendimento poderá ser efetivado por uma entidade
família ou instituição, que cuide transitoriamente da criança governamental ou não-governamental em regime de
ou do adolescente com seus direitos violados, até que este colocação familiar, nos termos do artigo 90, inciso III, do ECA.
possa ser reintegrado à sua família de origem. Nos casos em Dentro de nossa sistemática jurídica, este tipo de
que se inviabiliza a reintegração à família de origem outras acolhimento possui como pressuposto um mandato formal,
alternativas podem ser consideradas, como o uma guarda fixada judicialmente, a ser requerida pelo
encaminhamento para adoção ou eventualmente a indicação programa de atendimento ao Juízo, em favor da família
de um acolhimento permanente por parte da família acolhedora. A manutenção da guarda judicial, que é
acolhedora. instrumento judicial exigível para a regularização deste
A criança ou o adolescente que se encontra em situação de acolhimento, à obviedade, estará vinculada à permanência da
violação permanente de seus direitos deve receber as medidas família acolhedora no Programa. Nesta modalidade de
específicas de proteção, traduzidas no âmbito da assistência atendimento há supervisão e apoio para aqueles que estão
social pelos serviços de proteção especial, conforme acolhendo as crianças ou os adolescentes com quem possuem
estabelecidos no Estatuto da Criança e do Adolescente e na uma obrigação direta. O programa de atendimento deve ter
Política Nacional de Assistência Social, respectivamente. como objetivos não só o cuidado adequado e individualizado
da criança ou do adolescente acolhido, mas também a
São duas as espécies de Acolhimento: Acolhimento viabilização do retorno da criança à sua família de origem. 22
Institucional e o Acolhimento Familiar. Assim como as demais entidades previstas no artigo 90 do
O Acolhimento Institucional é a modalidade de ECA, os programas de “Famílias Acolhedoras”, denominadas
atendimento integral institucional, que oferece cuidado e também de “Família Guardiã”, “Família de Apoio”, “Família
espaço para socialização e desenvolvimento de crianças e Cuidadora”, “Família Solidária”,23 dentre outras, deverão se
adolescentes com medida de proteção, que necessitem de sujeitar ao regime previsto nos artigos 92 e 93 do Estatuto,
acolhimento fora da família de origem, até que seja possível mesmo porque ausente legislação federal específica.
sua reintegração familiar (natural ou extensa) ou As demais denominações se encontram, ad referendum,
encaminhamento para família substituta. As modalidades de devidamente descritas no glossário.
Acolhimento Institucional são: Casa Lar, República, Casa de
Passagem, Albergue, entre outros.20 Marco Situacional
Embora estas modalidades de acolhimento não estejam
especificadas no artigo 101 do ECA, todas elas se encaixam na É amplamente reconhecida a importância da família no
hipótese denominada no inciso VII, como entidade de abrigo, cuidado e no bem-estar de seus integrantes, uma vez que é ela
cujo regime jurídico básico está estabelecido nos artigos 90, 92 o âmbito privilegiado e primeiro a proporcionar a garantia de
e 93 da referida Lei. sua sobrevivência, o aporte afetivo fundamental para o seu
As entidades que desenvolvem programas de abrigo pleno desenvolvimento e para a sua saúde mental, a absorção
servem para acolher a criança e o adolescente e prestar-lhe de valores éticos e de conduta, e a sua introdução na cultura e
plena assistência. O abrigo é o lar coletivo, de pequenas na sociedade em que estão inseridas. Essa importância adquire
dimensões, onde o abrigado não está privado da liberdade e contornos ainda mais decisivos no caso dos indivíduos mais
deve obedecer aos princípios estabelecidos no artigo 92 do vulneráveis, como as crianças, os adolescentes, os idosos e os
ECA. As casas-lares, segundo a Lei no. 7.644, de 18 de doentes.
dezembro de 1987, são unidades residenciais sob O direito fundamental à convivência familiar está
responsabilidade de mãe social, que abrigue até dez crianças consagrado nas normas e instrumentos legislativos. No
e/ou adolescentes. As entidades de abrigo podem manter estas entanto, a garantia formal desse direito coloca problemas de
casas, nos termos do artigo 16 desta Lei, e possui como ordem prática para a sua implementação.
finalidade proporcionar tratamento especial às crianças e aos Ainda hoje, predomina no ideário social o modelo de
adolescentes, visando dar maior individualidade ao família tradicional e abstrato composto por pai, mãe e filhos
tratamento. pequenos. De fato, os laços de parentesco ainda mantêm
fortíssima influência na organização das famílias brasileiras.
O Acolhimento Familiar é a modalidade de atendimento Os dados que vão de 1977 a 1998, mostram que a grande
que oferece acolhimento na residência de famílias maioria dos arranjos domiciliares no Brasil está baseada em
cadastradas, selecionadas, capacitadas e acompanhadas para relações de parentesco entre pelo menos dois dos moradores,
receber crianças e/ou adolescentes com medida de proteção, ressaltando que, em 1998, apenas 9,3% dos arranjos
que necessitem de acolhimento fora da família de origem, até domiciliares não eram familiares – maior índice do período.24
que seja possível sua reintegração familiar ou salvo exceções A família brasileira está desde há muito em pleno processo
encaminhamento para família substituta. de mudança, especialmente no que se refere a sua composição.
Embora se constitua um instituto novo no país, esta Movido por novas práticas e valores sociais, esse processo
experiência já se encontra consolidada em outros países, muitas vezes passa ao largo da legislação e das políticas
principalmente nos europeus,21 e se encontra contemplada públicas que foram desenhadas para a família modelar.
expressamente na Política Nacional de Assistência Social, ao Neste sentido, a discussão sobre o direito à convivência
dispor que dentro dos serviços de proteção social especial de familiar das crianças e dos adolescentes brasileiros em
alta complexidade está a “Família Acolhedora”. situação de risco envolve questões ainda mais específicas,

20 Vide rol da Política Nacional de Assistência Social, “2.5.2.” – Brasília, cabe aos governos, particularmente às autoridades das políticas sociais.
2004. Sendo um recurso que pode adotar diferentes formas na sua
21 conforme estudos divulgados pela International Foster Care implementação, cada país escolhe a sua segundo algumas variáveis (...)”.
Organisation ( IFCO): GEORGE, Shanti & OUDENHOVEN, Nico Van. Trad. In, “Menores em riesgo y acogimiento familiar. Compartir el compromiso”.
Maria Soledad Franco. Apostando al Acogimiento Familiar - Um estúdio Buenos Aires. Ed. Humanitas, 1994.
comparativo internacional.Bélgica: Garant, 2003. 23 Vide diversas experiências pelo país, como os projetos do Rio de
22 Na definição de Matilde Luna, Presidente do Instituto Mercosul Janeiro; SAPECA, de Campinas/SP; São Bento do Sul/SC, etc:, in, CABRAL,
Social ( IMS), Buenos Aires, Argentina, “O Acolhimento Familiar se define Cláudia (Org.). “Perspectivas do Acolhimento Familiar no Brasil”.
como, entre outras questões, um dos recursos técnicos utilizados pelos Acolhimento Familiar – experiências e perspectivas. Rio de Janeiro:
governos na instrumentação de medidas que resguardem às crianças e aos Booklink, 2004, p. 11.
adolescentes na situação de risco psicossocial. Isto implica que no menu de 24 MEDEIROS, M. e OSÓRIO, R. Arranjos domiciliares e arranjos

programas assistenciais se incluam os programas de acolhimento como nucleares no Brasil: classificação e evolução de 1977 a 1998. Texto para
resposta às demandas sociais e no cumprimento da responsabilidade que Discussão no 788. Brasília, IPEA, abril de 2001.

Legislação 99
Apostila Digital Licenciada para Alice Caroline Guarino dos Santos - alice.guarino@hotmail.com (Proibida a Revenda)
APOSTILAS OPÇÃO

relacionadas aos diferentes aspectos dos problemas direitos não ocorre em todas as famílias que são pobres, assim
enfrentados por eles e suas famílias. Antes de tudo, há que se como não é verdade que crianças e adolescentes oriundos de
considerar que a família, enquanto unidade essencial de famílias de classes de renda mais elevadas estão livres da
organização da sociedade brasileira sofre as influências do vivência de maus-tratos e da violação de direitos cometidos
desenvolvimento socioeconômico e da ação estatal por meio por seus próprios familiares.
das políticas públicas. E são as famílias pobres as mais Existem outros fatores explicativos para a incidência da
negativamente afetadas pelas consequências destes violência contra crianças e adolescentes no âmbito familiar,
processos, quer se considere o impacto das transformações entre eles destaca-se: a história familiar passada ou presente
sociais e econômicas de longo prazo que vêm redesenhando o de violência doméstica; a ocorrência de perturbações
país desde a década de 1950, com a industrialização e a psicológicas entre os membros das famílias; o despreparo para
urbanização crescente da população brasileira, quer se a maternidade e/ou paternidade de pais jovens, inexperientes
considere as medidas de política mais recentes, tomadas a ou sujeitos a uma gravidez indesejada; a adoção de práticas
partir dos anos de 1990 e caracterizadas pelo ajuste educativas muito rígidas e autoritárias; o isolamento social das
econômico e pela restrição das políticas sociais. famílias que evitam desenvolver intimidade com pessoas de
Essa família empobrecida, embora tenha peculiaridades na fora do pequeno círculo familiar; a ocorrência de práticas
sua forma de organização que lhe possibilitam apoiar-se hostis, desprotetoras ou negligentes em relação às crianças, e
fortemente nas relações de solidariedade parental ampliada e fatores situacionais diversos que colocam as famílias frente a
conterrânea,25 tem experimentado uma crescente diminuição circunstâncias não antecipadas. 30
da sua capacidade de proteger os seus membros.26 Criar e A relação entre pobreza e vitimização de crianças e
educar os filhos, garantindo-lhes o usufruto de todos os adolescentes por parte de seus responsáveis não é, portanto,
direitos de que são titulares como pessoas humanas em direta, pois existem outras mediações que refutam o caráter
situação peculiar de desenvolvimento, tem sido uma tarefa natural e fatalista com frequência atribuído a essa associação.
muitas vezes impossível de ser cumprida pelas famílias Entretanto, não é possível dissociar o padrão de convivência
submetidas a condições de vida precárias, sem garantia de familiar das questões mais amplas de frustração, humilhação,
alimento, de moradia, de trabalho, de assistência à saúde e de redução dos direitos sociais e privações causadas pelo
todos os serviços que definem uma vida minimamente digna desemprego e pela diminuição do papel do Estado na garantia
no mundo contemporâneo. Além disso, a dinâmica familiar, da sobrevivência das famílias por meio da provisão de
naturalmente marcada pela ocorrência de entradas e saídas de políticas sociais.31
integrantes, registra, no caso das famílias pobres, movimentos A condição socioeconômica precária das famílias, ao impor
ainda mais traumáticos, determinados pelas condições maiores dificuldades para a sobrevivência digna do grupo
socioeconômicas e pela luta pela sobrevivência: migrações em familiar, pode funcionar como um elemento agravante e
busca de novas oportunidades; institucionalização de crianças, desencadeador de outros fatores de risco preexistentes.
adolescentes, adultos e idosos; afastamento dos responsáveis Portanto, tratar do direito à convivência familiar e comunitária
por longos períodos em função da ocupação exercida, como o de crianças e adolescentes em situação de risco é falar das
trabalho doméstico, por exemplo, entre inúmeras outras políticas de atenção às suas famílias, majoritariamente pobres.
situações.27 O Brasil é um país com tradição de atendimento
As consequências da desigualdade social e da pobreza, que institucional às crianças e adolescentes em situação de
tem como resultado a “produção social de crianças vitimadas vulnerabilidade, tradição essa historicamente forjada na
pela fome, por ausência de abrigo ou por morar em habitações desvalorização social da parcela da população a que
precárias, por falta de escolas, por doenças contagiosas, por pertencem, em sua grande maioria pobre e procedente de
inexistência de saneamento básico”,28 refletem diretamente na etnias não-brancas e na adaptação dessa população aos
relação entre criança, adolescente e violência no cotidiano de padrões considerados aceitáveis.
famílias brasileiras. Essa situação de vulnerabilidade, A colocação de crianças e adolescentes em instituições
denominada vitimação, pode desencadear a agressão física como medida de proteção contra os desvios causados pelas
e/ou sexual contra crianças e adolescentes, haja vista que a condições sociais, econômicas e morais das famílias pobres ou
cronificação da pobreza da família contribui para a como medida corretiva de desvios, ao longo da história social
precarização e deterioração de suas relações afetivas e da criança, do adolescente e da família, cristalizou as
parentais. Nesse sentido, pequenos espaços, pouca ou experiências das chamadas instituições totais, onde crianças e
nenhuma privacidade, falta de alimentos e problemas adolescentes viviam sob rígida disciplina e afastados da
econômicos acabam gerando situações estressantes que, convivência familiar e comunitária, visto que quase todas as
direta ou indiretamente, acarretam danos ao desenvolvimento atividades pertinentes a suas vidas eram realizadas
infantil”.29 intramuros.32
Contudo, a pobreza ou carência de recursos materiais não O Levantamento Nacional de Abrigos para Crianças e
constitui motivo suficiente para explicar o fenômeno da Adolescentes da Rede SAC33 do Ministério do Desenvolvimento
violação de direitos da criança e do adolescente. A violação de Social realizado pelo IPEA/CONANDA34 mostrou que a

25 CARVALHO, M. C. B. A priorização da família na agenda da política 32 RIZZINI, Irma. Assistência à infância no Brasil: uma análise de sua

social. In: KALOUSTIAN, op. cit. construção. Rio de Janeiro, Ed. Universitária Santa Úrsula, 1993.
26 CAMPOS, M. S. e MIOTO, R.C.T. Política de Assistência Social e a 33 Ressalta-se que, com a aprovação da NOB/SUAS em julho de 2005 e

posição da Família na Política Social Brasileira. In: Ser Social: Revista do das portarias Nº 440 e Nº 442 do MDS, os recursos do cofinanciamento
Programa de Pós-Graduação em Política Social/Universidade de Brasília. federal das ações socioassistenciais passam a ser transferidos por “Pisos
Departamento de Serviço Social – v.1, n.1 (1º semestre, 1998). Brasília, SER de Proteção”, cujos recursos poderão ser utilizados conforme a
Social UnB, 1998. necessidade local, dentro das ações passíveis de financiamento por cada
27 FERRARI, Mário; KALOUSTIAN, Silvio M. Introdução. In: piso. Cabe ao gestor local e ao CNAS a definição da rede de atendimento. O
KALOUSTIAN, Silvio M. (Org.). Família Brasileira: a base de tudo São Paulo: Piso de Alta Complexidade I pode ser utilizado para a manutenção dos
Cortez; Brasília, DF: UNICEF, 1994.. serviços da rede de acolhimento para crianças e adolescentes.
28 Azevedo e Guerra apud AMARO, Sarita. Crianças vítimas de violência: 34 Levantamento realizado pelo IPEA em 2003 e promovido pela

das sombras do sofrimento à genealogia da resistência – uma nova teoria Secretaria Especial dos Direitos Humanos (SEDH) da Presidência da
científica. Porto Alegre: AGE/EDIPURS, 2003. República, por meio da Subsecretaria de Promoção dos Direitos da Criança
29 Idem. do Adolescente (SPDCA) e do Conselho Nacional de Direitos da Criança e do
30 Amaro, op. cit. Adolescente (Conanda). Das cerca de 670 instituições de abrigo que eram
31 FALEIROS, Vicente de Paula. A questão da violência. In: SOUSA JR., beneficiadas, naquele ano, por recursos da Rede de Serviços de Ação
José Geraldo de [et al.] organizadores. Educando para Direitos Humanos: Continuada (Rede SAC) do Ministério do Desenvolvimento Social e
pautas pedagógicas para a cidadania na universidade. Porto Alegre, 2004. Combate à Fome, foram investigados 589 abrigos, ou seja, 88% do total.

Legislação 100
Apostila Digital Licenciada para Alice Caroline Guarino dos Santos - alice.guarino@hotmail.com (Proibida a Revenda)
APOSTILAS OPÇÃO

institucionalização se mantém, ainda nos dias atuais, como viviam, portanto, a paradoxal situação de estarem
caminho utilizado indiscriminadamente – e, muitas vezes, juridicamente vinculados a uma família que, na prática, havia
considerado o único possível – para a proteção de infância e algum tempo, não exerce a responsabilidade de cuidar deles,
adolescência, demonstrando que o princípio da principalmente por causa da pobreza (Gráfico 3).
excepcionalidade da medida de abrigo não vem sendo
respeitado. Gráfico 3
Existem cerca de vinte mil crianças e adolescentes
atendidos nas 589 instituições de abrigos beneficiados com Brasil – Crianças e adolescentes abrigados, segundo
recursos do Governo Federal repassado por meio da Rede de situação familiar
Serviços de Ação Continuada (Rede SAC). Os dados levantados
mostram características típicas de exclusão social, apontando
que os abrigos no Brasil são o locus da pobreza e da
menoridade (lugar de menor valor). Ressalta-se ainda que o
perfil de meninos e meninas encontrados nessas instituições
em nada corresponde às expectativas da sociedade para
adoção, cuja preferência recai nos bebês da cor branca e do
sexo feminino. Vivendo nos abrigos do país encontram-se, na
maioria, meninos (58,5%), afrodescendentes (63%) e mais
velhos, isto é, com idade entre 7 e 15 anos (61,3%) (Gráfico 1
e 2).

Gráfico 1

Brasil – Crianças e adolescentes abrigados por faixa


etária, segundo cor Fonte: IPEA/DISOC (2003). Levantamento Nacional de
Abrigos para Crianças e Adolescentes da Rede SAC

Embora a carência de recursos materiais, de acordo com o


ECA, não constitua motivo para a perda ou suspensão do poder
familiar, o Levantamento Nacional identificou que as causas
que motivaram o abrigamento da maioria das crianças e
adolescentes encontradas nas instituições de abrigos estavam
relacionadas à pobreza, consequência da falha ou inexistência
das políticas complementares de apoio aos que delas
necessitam. Entre os principais motivos: a pobreza das
famílias (24,1%), o abandono (18,8%), a violência doméstica
(11,6%), a dependência química dos pais ou responsáveis
Fonte: IPEA/DISOC (2003). Levantamento Nacional de incluindo alcoolismo (11,3%), a vivência de rua (7,0%) e a
Abrigos para Crianças e Adolescentes da Rede SAC orfandade (5,2%).

Gráfico 2 Gráfico 4

Brasil – Crianças e adolescentes abrigados por faixa Brasil – Motivos do ingresso de crianças e adolescentes
etária, segundo sexo em abrigo, segundo a frequência

Carência de recursos materias da família/


24,2%
responsável (pobreza)

Abandono pelos pais ou responsáveis 18,9%

Violência doméstica (maus-tratos físicos


e/ou psicológicos praticados pelos pais ou 11,7%
responsáveis)

Pais ou responsáveis dependentes químicos/


11,4%
alcoolistas

Vivência de rua 7,0%

Fonte: IPEA/DISOC (2003). Levantamento Nacional de Órfão (morte dos pais ou responsáveis) 5,2%

Abrigos para Crianças e Adolescentes da Rede SAC


Outros motivos 21,6%

Contrariando o senso comum que imaginava serem órfãos


as crianças e adolescentes que viviam nos abrigos, o
Levantamento Nacional também mostrou que a grande
Fonte: IPEA/DISOC (2003). Levantamento Nacional de
maioria desses meninos e meninas (86,7%) tinha família,
Abrigos para Crianças e Adolescentes da Rede SAC.
sendo que 58,2% mantinham vínculos com os familiares.
Apenas 5,8% estavam impedidos judicialmente desse contato
com eles e somente 5% eram órfãos. Esses meninos e meninas

Essas instituições acolhiam, no momento da realização da Pesquisa, 19.373


crianças e adolescentes.

Legislação 101
Apostila Digital Licenciada para Alice Caroline Guarino dos Santos - alice.guarino@hotmail.com (Proibida a Revenda)
APOSTILAS OPÇÃO

Se de um lado tem havido por parte das autoridades Quadro 1 – Perfil das entidades de abrigo
competentes – Conselho Tutelar e Judiciário – uma aplicação
indiscriminada da medida de abrigo, de outro lado, à saída do
abrigo permanece sendo um desafio. O Levantamento Nacional Não-
apontou que o princípio da brevidade da medida do abrigo governamentais..........................................................................................
estabelecido pelo ECA também não vem sendo cumprido, uma ....................68,3%
vez que mais da metade das crianças e dos adolescentes Públicas
abrigados viviam nas instituições há mais de dois anos, ....................................................................................................................30,
enquanto 32,9% estavam nos abrigos por um período entre 0%
dois e cinco anos, 13,3%, entre seis e dez anos, e 6,4%, por Têm orientação/vínculo
mais de dez anos. religioso..........................................................................................................
Em relação à possibilidade de adoção a situação também é ......67,2%
dramática, uma vez que dentre as crianças e adolescentes Católicos............................62,1%
abrigadas nas instituições pesquisadas, apenas 10,7% estavam Evangélicos.......................22,5%
judicialmente em condições de serem encaminhados para a Espíritas.............................12,6%
adoção.35 Além disso, apenas metade desses meninos e  Anteriores a 1990
meninas (54%) abrigados tinha processo judicial. A outra ............................................................................................................................
metade, por certo, lá estava sem o conhecimento do judiciário, .........41,4%
já que muitas crianças e adolescentes foram encaminhadas aos Posteriores a
abrigos pelas próprias famílias (11,1%), pela polícia (5,5%), 1990.................................................................................................................
dentre outras instituições que, judicialmente, não teriam tal ....................58,6%
prerrogativa.36  Dirigidas por voluntários
Embora a legislação tenha como regra geral a convivência ............................................................................................................................
de crianças e adolescentes com suas famílias naturais – e, 59,3%
excepcionalmente, com famílias substitutas –, para muitos dos Dirigidas por profissionais
meninos e meninas brasileiros esse direito permanece negado, remunerados................................................................................................
passando um período significativo da sua infância e ....33,4%
adolescência institucionalizadas e afastadas do convívio com  Profissionais do quadro próprio do
suas famílias e suas comunidades. abrigo..................................................................................................59,2
É preciso considerar sempre a prioridade a ser dada à %
manutenção de crianças e adolescentes no arranjo familiar de Profissionais voluntários
origem, seja ele qual for, evitando-se a separação e suas ............................................................................................................................
implicações e, sobretudo é necessário pensar em como manter ..25,3%
a vivência familiar e comunitária quando o afastamento é  Funcionam sob regime de permanência
inevitável. integral.........................................................................................78,4%
As sequelas para crianças e adolescentes de um período de Funcionam sob outros regimes de
institucionalização prolongado serão tanto maiores quanto permanência........................................................................................19,7
maior for o tempo de espera, que interfere não só na adaptação %
em caso de retorno à família de origem, mas também nos casos  Recursos próprios e privados no financiamento das
de inserção definitiva em outra família.37 entidades não-governamentais.................................61,7%
Neste sentido, considerando-se que o Acolhimento  Recursos públicos no financiamento das entidades
Institucional ainda cumpre um papel muito importante no não-governamentais.....................................................32,3%
cuidado com crianças e adolescentes em situação de risco no
Brasil, é crucial reconhecer a obrigatoriedade de promoção do Fonte: IPEA/Conanda. O Direito à Convivência Familiar e
direito à convivência familiar e comunitária também recai Comunitária: os abrigos para crianças e adolescentes no
sobre as entidades que oferecem programas de abrigo. Muito Brasil. Brasília, 2004.
embora, essa atribuição seja compartilhada por toda a rede de
atendimento à criança e ao adolescente, que inclui ainda o Ao analisar com base nos princípios do ECA os aspectos do
Judiciário, o Ministério Público, os Conselhos Tutelares e de atendimento realizado pelos abrigos quanto à convivência
Direitos da Criança e do Adolescente, as organizações civis de familiar, o Levantamento Nacional observou que, em relação
defesa de direitos humanos e o próprio Poder Executivo nos às ações de incentivo à convivência das crianças e dos
níveis federal, estadual e municipal. adolescentes com suas famílias de origem, a maioria dos
O perfil institucional dos 589 abrigos identificado no programas realiza visitas das crianças e adolescentes aos seus
Levantamento Nacional aponta que majoritariamente essas lares, mas a minoria permite visitas livres dos familiares aos
instituições são não-governamentais, orientadas por valores abrigos Entretanto, somente 31,2% realizavam as duas ações
religiosos, dirigidas por voluntários e que dependem conjuntamente. Quanto às ações de não-desmembramento de
fundamentalmente de recursos próprios e privadas para o seu grupos de irmãos, a maioria dos programas priorizava a
funcionamento (Quadro 1). manutenção ou a reconstituição de grupos de irmãos, adotava
o modelo de “agrupamento vertical”, possibilitando o
acolhimento de irmãos em diferentes idades e recebia tanto

35 Observados os dispostos nos artigos 166 e 169 do ECA, a adoção permanência indefinida de crianças e adolescentes nas instituições sem
requer a destituição do poder familiar e implica no afastamento definitivo qualquer perspectiva de convivência.
da criança e do adolescente de suas famílias de origem. Assim, para não 36 Cumpre esclarecer que o ECA estabelece em seu artigo 93: “As

incorrer em injustiças, é da maior importância que essas famílias recebam entidades que mantenham programas de abrigo poderão, em caráter
apoio e suporte necessários para sua reestruturação. Em muitos casos, a excepcional e de urgência, abrigar crianças e adolescentes sem prévia
inadequação dos processos de destituição do poder familiar pode provocar determinação da autoridade competente, fazendo comunicação do fato até
injustiças com famílias que sequer receberam apoio e/ou tiveram tempo o segundo dia útil imediato.
para reintegração de seus filhos. No entanto, é fundamental chamar 37 SILVA, Roberto, apud MINISTÉRIO DO DESENVOLVIMENTO SOCIAL,

atenção para o fato de que o próprio Estatuto estabelece que o abrigo é Comitê Nacional para o Reordenamento dos Abrigos. Subsídios para
uma “medida provisória e excepcional, utilizável como forma de transição reflexão na aplicação da medida e o funcionamento de programas em
para a colocação em família substituta”, não sendo aceitável a regime de abrigo. Brasília: 2003, p. 13 (não publicado).

Legislação 102
Apostila Digital Licenciada para Alice Caroline Guarino dos Santos - alice.guarino@hotmail.com (Proibida a Revenda)
APOSTILAS OPÇÃO

meninos quanto meninas. Contudo, somente 27,8% do total tradicional prática brasileira de institucionalização
das instituições que desenvolvem programas de abrigo prolongada de crianças e adolescentes em situação de risco,
atendiam todas as três ações (Quadro 2). condenados a viver grande parte de suas vidas privados de
qualquer vivência familiar.38
Quadro 2 – Preservação dos vínculos familiares Mesmo que a colocação em família substituta não
dependa exclusivamente do trabalho das instituições de
CRITÉRIOS CONSIDERADOS ABRIGOS QUE abrigo, elas podem desempenhar um papel fundamental
ATENDEM (%) nesse processo, incentivando a convivência de crianças e
1. INCENTIVO À CONVIVÊNCIA COM A adolescentes abrigados com outras famílias por meio de
FAMÍLIA DE ORIGEM ações como: o incentivo à integração em família substituta
1.1. Promovem visitas de crianças 65,9% sob as formas de guarda, tutela ou adoção, o envio de
e adolescentes aos lares de suas relatórios periódicos sobre a situação dos abrigados e de suas
famílias famílias para as Varas da Infância e da Juventude e a
1.2. Permitem visitas livres dos 41,4% manutenção de programas de apadrinhamento afetivo.39 Das
familiares ao abrigo 589 instituições pesquisadas, apenas 22,1% desenvolviam
 Atendem a todos os 31,2% todos esses tipos de ação de incentivo à convivência de
critérios crianças e adolescentes abrigados com outras famílias
2. NÃO-DESMEMBRAMENTO DE GRUPOS (Quadro 4).
DE IRMÃOS ABRIGADOS
3.1. Priorizam a manutenção ou 66,4% Quadro 4 – Incentivo à convivência familiar com
reconstituição de grupos de irmãos outras famílias
3.2. Organizam-se sob 62,1%
agrupamento vertical (intervalo entre CRITÉRIOS CONSIDERADOS ABRIGOS QUE
idades mínima e máxima maior do que ATENDEM (%)
10 anos) 1. Incentivam a integração em 67,5%
3.3. Atuam em regime de 62,3% família substituta (guarda, tutela ou
coeducação (meninos e meninas) adoção)
 Atendem a todos os critérios 27,8% 2. Mantêm programas de 55,3%
1+2 6,6% apadrinhamento
1+2 22,1%
Fonte: IPEA/DISOC, Levantamento Nacional de Abrigos
para Crianças e Adolescentes da Rede SAC (2003) Fonte: IPEA/DISOC, Levantamento Nacional de Abrigos
para Crianças e Adolescentes da Rede SAC (2003).
Além do fortalecimento e da manutenção dos vínculos
afetivos entre as crianças e adolescentes em abrigos e seus Nos aspectos do atendimento realizado pelos abrigos
familiares, o apoio à reestruturação das famílias constitui-se quanto à convivência comunitária, o Levantamento Nacional
em ação importante e complexa. Muito embora a maioria dos identificou um quadro preocupante em relação às ações de
programas realize atividades de visitas às famílias e estímulo à participação das crianças e adolescentes na vida da
acompanhamento social, a minoria realiza reuniões ou grupos comunidade local, pois apenas 6,6% dos abrigos pesquisados
de discussão e encaminha famílias para inserção em utilizavam todos os serviços necessários que estavam
programas de proteção social. E ainda, somente 14,1% do total disponíveis na comunidade, tais como: creche; ensino regular;
de abrigos pesquisados realizavam todas as quatro ações de profissionalização para adolescentes; assistência médica e
apoio à reestruturação familiar (Quadro 3). odontológica; atividades culturais, esportivas e de lazer; e
assistência jurídica. A maioria das instituições (80,3%) ainda
Quadro 3 – Apoio à reestruturação familiar oferecia pelo menos um desses serviços diretamente, ou seja,
de forma exclusiva dentro do abrigo (Quadro 5).
CRITÉRIOS CONSIDERADOS ABRIGOS QUE
ATENDEM (%) Quadro 5 – Participação na vida da comunidade local
1. Realizam visitas domiciliares 78,1%
2. Oferecem acompanhamento 65,5% CRITÉRIO CONSIDERADO ABRIGOS QUE
ATENDEM (%)
social
3. Organizam reuniões ou 34,5% 1. Utilizam serviços 6,6 %
especializados existentes na
grupos de discussão e apoio
comunidade
4. Encaminham para inserção 31,6%
2. Oferecem pelo menos um dos 80,3%
em programas de auxílio/proteção à
serviços de forma exclusiva dentro
família
dos abrigos
1+2+3+4 14,1%
Fonte: IPEA/DISOC, Levantamento Nacional de Abrigos
Fonte: IPEA/DISOC, Levantamento Nacional de Abrigos
para Crianças e Adolescentes da Rede SAC (2003)
para Crianças e Adolescentes da Rede SAC (2003)
Muitas das instituições investigadas – a maioria delas –
O relacionamento de crianças e adolescentes abrigados
como se viu, surgida durante a vigência do ECA já introduziram
com outras famílias é outra forma de garantir o direito à
condutas diferentes e programas mais condizentes com as
convivência familiar cujas chances de retorno para suas
diretrizes legais, ampliando-se no país o elenco de
famílias de origem foram esgotadas. Assim, a colocação em
experiências pautadas pelos princípios da proteção integral e
família substituta configura-se como uma opção frente à

38 O Estatuto estabelece como princípio a ser seguido pelas entidades 39 Os programas de apadrinhamento se constituem em alternativa de

de abrigo “a colocação em família substituta, quando esgotados os recursos referência familiar para as crianças e os adolescentes abrigados.
de manutenção na família de origem” (Lei 8.069/90, Art. 92, Inc. II).

Legislação 103
Apostila Digital Licenciada para Alice Caroline Guarino dos Santos - alice.guarino@hotmail.com (Proibida a Revenda)
APOSTILAS OPÇÃO

do atendimento individualizado. No entanto, há ainda crianças, aos adolescentes e às famílias a revisão e a mudança
inúmeras instituições que mantêm práticas que privam quase de suas práticas, no sentido de se implantar alternativas que
que totalmente crianças e adolescentes da convivência social. contemplem ações emancipatórias e, sobretudo que garantam
Sobretudo, ainda falta estratégia de coordenação das várias os direitos das crianças e dos adolescentes com prioridade
atividades desenvolvidas e que poderiam contribuir para a absoluta nas políticas públicas, com decisões fundadas na
promoção efetiva da convivência familiar e comunitária avaliação do seu melhor interesse, considerando sua voz e
daqueles que vivem nesses abrigos. opinião.
As questões mais decisivas talvez estejam relacionadas
com a falta de integração entre essas instituições e os demais Diretrizes
atores da rede de atendimento, o que dificulta em muito a
realização de suas atividades em consonância com os A mudança no paradigma do atendimento à criança e
princípios do ECA. adolescente, sobretudo na efetivação do seu direito à
No tocante às alternativas ao Acolhimento Institucional é convivência familiar e comunitária apresentada na forma
importante considerar o acolhimento familiar provisório de operacional deste Plano fundamenta-se nas seguintes
crianças e adolescentes em situação de risco. Com efeito, na diretrizes:
busca de garantir o direito à convivência familiar e
comunitária às crianças e adolescentes privados do convívio - Centralidade da família nas políticas públicas
com seus pais, uma primeira opção que poderia ser O direito das crianças e adolescentes à convivência
considerada é o acolhimento por outros membros da família familiar e comunitária está relacionada à inclusão social de
da criança ou adolescente em risco, a chamada família suas famílias. O reconhecimento da importância da família no
extensiva. Outra forma de propiciar vivência em família para contexto da vida social está explicito no artigo 226 da
esta população seriam as experiências de acolhimento por Constituição Federal do Brasil, na Convenção sobre os Direitos
famílias, que têm surgido em vários lugares do mundo – da Criança, no Estatuto da Criança e do Adolescente, na Lei
sobretudo na Europa, e, mais recentemente no Brasil – sob as Orgânica da Assistência Social e na Declaração dos Direitos
denominações de famílias acolhedoras, guardiãs, madrinhas, Humanos.
entre outras. A família é compreendida como um grupo de pessoas com
É preciso ser destacado, entretanto, que o acolhimento laços de consanguinidade, de aliança, de afinidade ou de
familiar não se apresenta como substituto ao atendimento solidariedade, cujos vínculos circunscrevem obrigações
institucional ou às políticas de adoção. Ao contrário, deve ser recíprocas, organizadas em torno de relações de geração e de
mais uma opção, na busca da melhor medida para cada criança gênero. Arranjos familiares diversos devem ser respeitados e
ou adolescente que teve um ou mais de seus direitos violados. reconhecidos como potencialmente capazes de realizar as
Na construção de que o atendimento institucional e o funções de proteção e de socialização de suas crianças e
acolhimento familiar sejam opções alternativas em vez de adolescentes.
excludentes, busca-se humanizar o cuidado institucional, com Sendo assim, “a família, independente de seu formato, é a
a mudança da postura institucional e do quadro de recursos mediadora das relações entre os sujeitos e a coletividade e
humanos em relação aos abrigados e a suas famílias; a geradora de modalidades comunitárias de vida”.40 Portanto,
organização de atendimentos complementares, como as diante de situações de risco social e vulnerabilidades vividas
experiências de crianças e adolescentes que vivem em pelas famílias brasileiras, principalmente por pressões
instituições e visitam famílias voluntárias em finais de semana geradas pelos processos de exclusão social e cultural, essas
e férias; ou, ainda, a aproximação do ambiente institucional famílias precisam ser apoiadas pelo Estado e pela sociedade,
aos padrões familiares e residenciais, conforme recomendado para cumprir suas responsabilidades. Esse apoio visa a
pelo ECA. superação de vulnerabilidades e riscos vividos por cada
Da mesma forma que temos tradição de atendimento família, favorecendo e ampliando os recursos socioculturais,
institucional como caminho usual na atenção às crianças e materiais, simbólicos e afetivos que contribuem para o
adolescentes em situação de risco, a colocação em família fortalecimento desses vínculos. Diante disso, a centralidade da
substituta no Brasil é muito mais conhecida na forma de família no âmbito das políticas públicas se constitui em
adoção. importante mecanismo para a efetiva garantia do direito à
No Brasil, ainda não existe a tradição do acolhimento convivência familiar e comunitária de crianças e adolescentes.
formal por famílias voluntárias e o caminho mais usual de - Primazia da responsabilidade do Estado no fomento
colocação em família substituta acaba sendo a adoção. de políticas integradas de apoio à família
Entretanto, como se trata de uma medida definitiva, a adoção No cumprimento do princípio da prioridade absoluta à
não deve ser vista como a única solução para os problemas das garantia dos direitos da criança e do adolescente, o Estado
crianças pobres brasileiras, inclusive para o problema da deve se responsabilizar por oferecer serviços adequados e
institucionalização indiscriminada, como com frequência se suficientes à prevenção e superação das situações de violação
apresenta. Antes, a adoção deve ser encarada como uma entre de direitos, possibilitando o fortalecimento dos vínculos
várias opções, a ser aplicada apenas quando as chances de familiares e sócio-comunitários. O apoio às famílias e seus
manutenção ou recuperação dos vínculos com a família de membros deve ser concretizado na articulação eficiente da
origem, incluindo-se a ampliada, não existem mais. rede de atendimento das diferentes políticas públicas,
É preciso superar a aplicação indiscriminada de medidas garantindo o acesso a serviços de educação, de saúde, de
que acarretam no afastamento de crianças e de adolescentes geração de trabalho e renda, de cultura, de esporte, de
de suas famílias de origem, o que, na prática, tem representado assistência social, dentre outros. Nas situações de risco e
uma forma de vitimizar famílias, crianças e adolescentes em enfraquecimento dos vínculos familiares, as estratégias de
situação de pobreza. atendimento deverão favorecer a reestruturação do grupo
Desde a consagração da doutrina da proteção integral de familiar e a elaboração de novas referências morais e afetivas.
crianças e adolescentes com a Constituição de 1988, em Estas estratégias visam potencializar a família para o exercício
seguida com o ECA e posteriormente com a LOAS, vem-se de suas funções de proteção e socialização e o
exigindo da rede de atendimento, das instituições, dos desenvolvimento de sua autonomia, incluindo o
programas e dos serviços que prestam atendimento às desenvolvimento de ações que possam levar à constituição de

40 Política Nacional de Assistência Social – PNAS (2004).

Legislação 104
Apostila Digital Licenciada para Alice Caroline Guarino dos Santos - alice.guarino@hotmail.com (Proibida a Revenda)
APOSTILAS OPÇÃO

novos vínculos familiares e comunitários em caso de ruptura integridade do desenvolvimento da criança e do adolescente é
dos vínculos originais. Para garantir a qualidade das políticas que se deve pensar no seu afastamento da família de origem. A
de apoio às famílias, o Estado tem a responsabilidade de decisão sobre a separação é de grande responsabilidade, por
capacitar seus agentes e de fiscalizar, monitorar e avaliar esses parte dos agentes sociais e deve estar baseada em
serviços na articulação dos níveis municipal, estadual e fundamentação teórica sobre o desenvolvimento infantil, as
federal. etapas do ciclo de vida individual e familiar e a teoria dos
- Reconhecimento das competências da família na sua vínculos; e deve ter como prioridades a comunicação na
organização interna e na superação de suas dificuldades família e o investimento na reorganização dos laços familiares.
As políticas especiais para promoção, defesa e garantia do A análise da situação evita danos ao desenvolvimento da
direito de crianças e adolescentes à convivência familiar e criança e do adolescente causados por separações bruscas,
comunitária devem reconhecer a família como um grupo social longas e desnecessárias. Deve, ainda, considerar a qualidade
capaz de se organizar e reorganizar dentro de seu contexto e a das relações, a atitude proativa de seus membros na
partir de suas demandas e necessidades. Em sua relação com requalificação dos vínculos e construção de sua autonomia.
a sociedade e em sua rede de relações internas, a família A decisão por um afastamento definitivo, ou seja, a
apresenta capacidade de criar soluções para seus problemas, destituição do poder familiar, só deve ocorrer após um
e de rever e reconstruir seus vínculos ameaçados, a partir do investimento eficiente na busca de recursos na família de
apoio recebido das políticas sociais. origem, nuclear ou extensa, com acompanhamento
Reconhecendo a complexidade desse processo, é preciso profissional sistemático e aprofundado de cada caso e
escutar e respeitar as famílias, seus valores e crenças, criando considerando o tempo de afastamento, a idade da criança e do
com elas soluções que possam ser adequadas ao contexto, adolescente e a qualidade das relações.
coerentes com os direitos dos seus membros e consistentes Cabe esclarecer que a expressão “Acolhimento” utilizada
com as políticas sociais. neste Plano refere-se aos Programas de Proteção Social
- Respeito à diversidade étnico-cultural, à identidade Especial de “Acolhimento Institucional” ou “Acolhimento
sexual e à equidade de gênero Familiar”. Por Acolhimento familiar entende-se a modalidade
O apoio às famílias deve se pautar pelo respeito à de atendimento que oferece acolhimento na residência de
diversidade dos arranjos familiares, às diferenças étnico- famílias previamente cadastradas, selecionadas, capacitadas e
raciais e culturais bem como à equidade de gênero, consoante acompanhadas para receber crianças e/ou adolescentes com
com a Constituição Federal. A defesa dos direitos de cidadania medida de proteção, que necessitem de acolhimento fora da
deve ter cunho universalista, considerando todos os atores família de origem até que seja possível sua reintegração
sociais desenvolvidos no complexo das relações familiares e familiar ou encaminhamento para família substituta. Por
sociais e tendo impacto emancipatório nas desigualdades Acolhimento Institucional entende-se a modalidade de
sociais. Dessa forma, o respeito à diversidade não pode ser atendimento integral institucional que oferece acolhimento,
contraditório com uma ética dos direitos que incentive cuidado e espaço para socialização e desenvolvimento de
mudanças culturais, por meio do resgate das tradições de crianças e adolescentes com medida de proteção, que
cuidado e afeto nos vínculos familiares e comunitários, em necessitem de acolhimento fora da família de origem, até que
suas bases de identidade cultural, nem com a construção seja possível sua reintegração familiar ou encaminhamento
participativa de novas práticas. O respeito à diversidade está para família substituta. Recebem atualmente várias
associado à reflexão das famílias sobre suas bases culturais, ao denominações, tais como: “abrigos”, “casas lares”, “casas de
combate aos estigmas sociais, à promoção dos direitos passagem”, entre outros.
humanos e ao incentivo aos laços de solidariedade social. - Reordenamento institucional dos programas de
- Fortalecimento da autonomia do adolescente e do Acolhimento Institucional em consonância com as
jovem adulto na elaboração do seu projeto de vida normativas nacionais, priorizando o desenvolvimento de
Sendo a criança e o adolescente sujeitos de direitos, é ações sustentadas nos princípios dos direitos humanos
necessário reconhecer suas habilidades, competências, O reordenamento institucional se constitui em um novo
interesses e necessidades específicas, incentivando-os, paradigma na política social que deve ser incorporado por
inclusive por meio de espaços de participação nas políticas toda a rede de atendimento social do país. Reordenar o
públicas, à busca compartilhada de soluções para as questões atendimento significa reorientar as redes pública e privada,
que lhes são próprias. que historicamente praticaram o regime de abrigamento, para
Atenção especial deve ser dada aos adolescentes em afinarem-se com a mudança de paradigma proposto, de eleger
regime de Acolhimento Institucional, ou sem possibilidades de a família como a unidade básica da ação social, e não mais a
reatar os vínculos familiares e que requerem soluções criança, o adolescente, o deficiente físico ou o idoso,
participativas e negociadas para a elaboração de seus projetos individualmente e deslocado de seu contexto familiar.
de vida. Os espaços públicos frequentados por crianças e Conselhos Municipais dos Direitos da Criança e do
adolescentes e as instâncias de formulação de políticas Adolescente, Conselhos Municipais de Assistência Social e
públicas constituem importantes instrumentos para exercício órgãos públicos repassadores de recursos podem sugerir
dos direitos de cidadania, sob a perspectiva tanto de incentivar adequações, tanto nos estatutos quanto nos projetos
a criatividade no campo das ciências, das artes, da cultura e dos pedagógicos das entidades, como condição para o registro,
esportes quanto na formação de lideranças infanto-juvenis. para aprovação de projetos e/ou para liberação de recursos.
- Garantia dos princípios de excepcionalidade, Esta diretriz requer ações como: 1) mudança na sistemática de
brevidade e provisoriedade nos programas de financiamento das entidades de abrigo, eliminando-se formas
Acolhimento Familiar e Acolhimento Institucional41 de que incentivem a manutenção indefinida das crianças e
crianças e de adolescentes adolescentes na instituição – como o financiamento por
Toda medida de proteção que indique o afastamento da criança e adolescente atendido; 2) qualificação dos
criança e do adolescente de seu contexto familiar, podendo profissionais que trabalham na entidade; 3) estabelecimento
ocasionar suspensão temporária ou ruptura dos vínculos de indicadores qualitativos e quantitativos de avaliação para a
atuais, deve ser uma medida rara, excepcional. Apenas em entidade; 4) desenvolvimento ou incorporação de
casos onde a situação de risco e desproteção afetam a metodologias para trabalho com famílias; 5) ênfase na

41 Medidas de proteção especial, excepcional e temporária que visam

atender crianças e adolescentes que precisam ser afastados


provisoriamente de suas famílias de origem.

Legislação 105
Apostila Digital Licenciada para Alice Caroline Guarino dos Santos - alice.guarino@hotmail.com (Proibida a Revenda)
APOSTILAS OPÇÃO

prevenção ao abandono e na recuperação das competências da A consolidação de novas representações e práticas das
família; 6) reconhecimento da autonomia e da competência da famílias e da sociedade acerca dos direitos das crianças e
família para bem criar e educar os seus filhos, 7) adequação do adolescentes reside na sustentabilidade de uma mudança
espaço físico e do número de crianças e adolescentes cultural. A legitimidade desta mudança cultural apoia-se nos
atendidos em cada unidade, de forma a garantir o atendimento processos participativos e no exercício do controle social, por
individualizado e em pequenos grupos, 8) articulação com a meio das instituições da sociedade, sobre a política social e na
rede de serviços e o SGD. As instituições que aceitarem ética da defesa dos direitos.
reordenarem-se podem continuar atendendo em regime de
abrigamento, em situações de provisoriedade, brevidade e de Objetivos Gerais
excepcionalidade, desde que incluam em seus objetivos o
atendimento: 1) das famílias das crianças e dos adolescentes - Ampliar, articular e integrar as políticas, os programas, os
abrigados; 2) famílias das crianças e adolescentes projetos, os serviços e as ações de apoio sociofamiliar para a
desabrigadas; 3) famílias da comunidade em situação de promoção, defesa e garantia do direito a convivência familiar
vulnerabilidade social e/ou que sejam encaminhados por e comunitária;
agentes do sistema de garantia de direitos. - Difundir uma cultura de promoção, defesa e garantia do
- Adoção centrada no interesse da criança e do direito de crianças e adolescentes à convivência familiar e
adolescente comunitária.
De acordo com o ECA, a colocação em família substituta, - Parametrizar o Acolhimento Institucional como medida
concebida nas formas de guarda, tutela e adoção, é uma de caráter excepcional e provisório, assegurando atendimento
medida de proteção que visa garantir o direito fundamental individualizado de qualidade e em pequenos grupos;
das crianças e adolescentes à convivência familiar e - Fomentar e implementar alternativas à
comunitária. Entretanto, tradicional e culturalmente a adoção institucionalização, na forma de programas de Acolhimento
foi e ainda é bastante aplicada no Brasil com a finalidade Familiar e de programas para promoção da autonomia do
precípua de dar filhos a quem não os tem, estando, portanto, adolescente e do jovem adulto;
centrado no interesse dos adultos. - Aprimorar os procedimentos de adoção nacional e
O direito de toda criança e adolescente cujos pais foram internacional.
destituídos do poder familiar deve prevalecer sobre o desejo - Fomentar a implementação de programas para promoção
dos pretendentes a adoção. A orientação deve seguir a ideia de da autonomia do adolescente e/ou jovem egressos de
“uma família para uma criança” e não de “uma criança para programas de acolhimento, desenvolvendo parâmetros para a
uma família”. Isso pressupõe a busca de famílias disponíveis a sua organização, monitoramento e avaliação;
acolherem crianças e adolescentes hoje privados do direito à - Aprimorar os procedimentos de adoção nacional e
convivência familiar e comunitária. Não se trata mais de internacional, visando: a) estimular, no País, as adoções de
procurar crianças para preencher o perfil desejado pelos crianças e adolescentes que, por circunstâncias diversas, têm
pretendentes, mas sim de informá-los quanto ao perfil dessas sido preteridos pelos adotantes – crianças maiores e
crianças e adolescentes efetivamente disponíveis para adoção. adolescentes, com deficiência, com necessidades específicas
Este é o sentido da proposta de uma nova cultura para a de saúde, afrodescendentes ou pertencentes a minorias
adoção, que não mais se atém à semelhança biológica, étnicas, dentre outros; b) investir para que todos os processos
tradicionalmente procurada na adoção, mas que entende a de adoção no País ocorram em consonância com os
adoção como alternativa excepcional e extraordinária para procedimentos legais previstos no Estatuto da Criança e do
assegurar o direito à convivência familiar e comunitária. Adolescente; e c) garantir que a adoção internacional ocorra
A nova cultura da adoção visa estimular, sobretudo, as somente quando esgotadas todas as tentativas de adoção em
adoções inter-raciais, as adoções tardias42 a adoção de território nacional, sendo, nestes casos, priorizados os países
crianças e adolescentes com deficiências físicas ou mentais e a que ratificaram a Convenção de Haia;
adoção de crianças e adolescentes com doenças congênitas e - Assegurar estratégias e ações que favoreçam os
afetados pelo vírus HIV/AIDS, para assegurar a todos o mecanismos de controle social e a mobilização da opinião
respeito ao seu direito à convivência familiar e comunitária. pública na perspectiva da implementação do Plano Nacional
- Controle social das políticas públicas de Promoção, Proteção e Defesa do Direito de Crianças e
Efetivada nas normativas constitucional e Adolescentes à Convivência Familiar e Comunitária;
infraconstitucionais (Constituição Federal, Convenção sobre - Aprimorar e integrar mecanismos para o
os Direitos da Criança, ECA, LOAS, LDB e LOS) a participação cofinanciamento, pela União, Estados, Distrito Federal e
popular, com caráter democrático e descentralizado se dá em Municípios, das ações previstas no Plano Nacional de
cada esfera do governo, abrangendo o processo de gestão Promoção, Proteção e Defesa do Direito de Crianças e
político-administrativa-financeira e técnico-operativa. O Adolescentes à Convivência Familiar e Comunitária, tendo
controle do Estado deve ser exercido pela sociedade na busca como referência a absoluta prioridade definida no artigo 227
de garantir os direitos fundamentais e os princípios da Constituição Federal de 1988 e no artigo 4° do Estatuto da
democráticos. Criança e do Adolescente.
Os Conselhos e as Conferências são espaços privilegiados
para esta participação, mas, também existem outros como a Resultados Programáticos
mídia e os conselhos profissionais. As Conferências avaliam a O direito à convivência familiar e comunitária, assegurado
situação das políticas públicas e da garantia de direitos, como fundamental na Carta Constitucional e na legislação
definem diretrizes e avaliam os seus avanços. Os Conselhos infraconstitucional, garantido a todas as crianças e
têm, dentre outras, a responsabilidade de formular, deliberar adolescentes, demanda iniciativas de diferentes políticas
e fiscalizar a política de atendimento, normatizar, disciplinar, públicas e sociais.
acompanhar e avaliar os serviços. Avanços na organização e Essa articulação e intersetorialidade entre as políticas, no
fortalecimento da participação da população são necessários, Sistema de Garantia de Direitos e respaldados pelos seus
buscando a integração das políticas sociais nos níveis federal, mecanismos de exigibilidade de direitos, é condição
estadual e municipal. fundamental para que a família, a comunidade, o poder público

42 São consideradas adoções tardias as adoções de crianças acima de

dois anos de idade.

Legislação 106
Apostila Digital Licenciada para Alice Caroline Guarino dos Santos - alice.guarino@hotmail.com (Proibida a Revenda)
APOSTILAS OPÇÃO

e a sociedade em geral assegurem a efetivação dos direitos Municipal dos Direitos da Criança e do Adolescente (CMDCA),
descritos nos artigos 227 da Constituição Federal e 4º do ECA. atendendo as diretrizes do artigo 92 do ECA e no Conselho
O fortalecimento, a efetivação e a consolidação desses Municipal de Assistência Social (CMAS);
direitos passam necessariamente pela concretização de - Programas de Acolhimento Institucional e Acolhimento
políticas, programas, projetos, serviços e ações que assegurem Familiar em constante articulação com o Conselho Tutelar e a
aquilo o que antes se constituía em expectativa de direito. Vara da Infância e Juventude para o acompanhamento;
O Plano Nacional de Promoção, Defesa e Garantia do - Programas de Acolhimento Institucional e Acolhimento
Direito de Crianças e Adolescentes à Convivência Familiar e Familiar assegurando os princípios de excepcionalidade,
Comunitária pretende, com sua execução, materializar esse provisoriedade e transitoriedade no atendimento, priorizando
direito fundamental, alcançando resultados programáticos a o enfoque nas relações afetivas da criança e do adolescente
seguir descritos: com suas famílias de origem;
- Parâmetros nacionais de atendimento para programas de
Família de origem/comunidade Acolhimento Familiar elaborados conjuntamente pelos
Conselhos Nacionais dos Direitos da Criança e do Adolescente
- Famílias incluídas, principalmente aquelas em maior e da Assistência Social e critérios de operacionalização
vulnerabilidade social, nas políticas sociais de educação, definidos pelos Conselhos Municipais dos Direitos da Criança
saúde, assistência social, esporte cultura e lazer e tendo acesso e do Adolescente e da Assistência Social em cumprimento
a: habitação digna; creches; atividades lúdicas, esportivas e pelos programas;
culturais que respeitem a diversidade étnico-racial e de - Programas de Acolhimento Familiar funcionando como
gênero; escolarização formal e reforço escolar no contra turno alternativa ao abrigamento em instituições e reconhecidos
escolar; tratamento preventivo e curativo da dependência de como medida de proteção social e defesa do direito à
álcool e outras drogas na rede de saúde, convivência familiar e comunitária e, portanto incorporados
informações/orientações quanto ao pré-natal tanto no na política de atendimento à infância e juventude e as demais
atendimento ao aspecto físico quanto no psicoafetivo, com políticas públicas municipais;
destaque para mães adolescentes; oferta de métodos - Famílias dos programas de Acolhimento Familiar
contraceptivos aos que desejarem, apoio às crianças e capacitadas para o atendimento, para facilitar a reconstrução
adolescentes com deficiência e afetados pelo vírus HIV/AIDS e e/ou preservação do vínculo com a família de origem e para
suas famílias; apoio sociofamiliar; atividades socioeducativas; manter grupos de irmãos em um mesmo programa;
atendimento psicossocial; transferência de renda; qualificação - Crianças e adolescentes de programas de Acolhimento
profissional; geração de renda e inclusão no mundo do Institucional sem perspectivas de adoção, colocadas em
trabalho; programas de Acolhimento Familiar ou em programas que
- Famílias estimuladas a buscar e participar em sua estimulem a passagem para a sua autonomia;
comunidade de diferentes espaços de integração e mobilização - Projeto políticopedagógicos dos programas de
social, assegurando por meio do controle social a qualidade Acolhimento Institucional prevendo: a incompletude
dos serviços e, sobretudo favorecendo o dinamismo a institucional, o atendimento personalizado e orientado pelo
diversidade cultural e a sua participação política; Plano Individual de Atendimento da criança e do adolescente,
- Equipamentos e serviços públicos disponibilizados em o atendimento em pequenos grupos, a manutenção de grupos
quantidade e qualidade suficientes e prontos para atender às de irmãos num mesmo programa, o registro de dados de cada
demandas da população em situação de vulnerabilidade social, criança e adolescente constantemente atualizado, a
com programas e ações preventivos à fragilização e/ou preparação para o desligamento e a participação na
rompimento de vínculos; comunidade local;
- Família participando ativamente nos projetos - Profissionais do Acolhimento Institucional capacitados
políticopedagógicos dos programas de atendimento permanentemente no trabalho social de famílias e atuando
governamental e não-governamental de apoio socioeducativo sistematicamente no reforço aos vínculos familiares,
às crianças e adolescentes, incluindo também os programas priorizando o investimento na família de origem e na defesa
que atendem adolescentes em conflito com a lei; do direito à convivência familiar e comunitária das crianças e
- Equipamentos, programas e serviços públicos e sociais adolescentes que vivem em instituições;
em permanente articulação entre si e com os Conselhos - Crianças e adolescentes pretendidos à adoção atendidos
Tutelares, Vara da Infância e Juventude, Ministério Público, por programas de Acolhimento Institucional sendo
Conselhos de Direitos e Setoriais de políticas públicas, previamente preparados;
mantendo uma rede de informações que assessore o - Aumento dos encaminhamentos de crianças e
atendimento e acompanhamento das famílias; adolescentes aos programas de Acolhimento Familiar (como
- Família com vínculos fragilizados incluídas em transição para a volta á família de origem), aos programas que
programas de superação de violação de direitos e estimulem a passagem para a autonomia ou encaminhadas à
fortalecimento de vínculos familiares; Adoção reduzindo a permanência no Acolhimento
- Famílias em situação de vulnerabilidade incluídas em Institucional;
ações de fortalecimento da autonomia, da independência, da - Destituição do poder familiar proposto com segurança
autoestima e da identidade, tendo reconhecidas as diferenças pelos profissionais dos programas quando esgotados todos os
culturais, favorecendo a existência de um contexto positivo investimentos na capacidade de reorganização do contexto
para a criação dos filhos e o desenvolvimento de seus projetos que gerou o afastamento da criança e do adolescente da família
de vida. de origem, nuclear ou extensa.

Acolhimento Institucional Programas de Famílias


Acolhedoras.

- Modalidades de Acolhimento Institucional (Casa de


Passagem, Abrigo de pequeno porte, Casa Lar e República)
oferecidos na rede de atendimento municipal;
- Todos os programas de Acolhimento Institucional e
Acolhimento Familiar devidamente registrados no Conselho

Legislação 107
Apostila Digital Licenciada para Alice Caroline Guarino dos Santos - alice.guarino@hotmail.com (Proibida a Revenda)
APOSTILAS OPÇÃO

Adoção nacional e internacional - Conselho Tutelar desempenhando suas prerrogativas


legais, tendo sua decisão de abrigamento respaldada num
- Aumento do número de famílias pretendentes à adoção consciencioso diagnóstico da excepcionalidade da medida e
disponíveis em acolher criança maiores de cinco anos conselheiros tutelares com boa estrutura logística mantendo
independente da raça/etnia,43 deficiência ou estado de saúde; estreito contato com toda a rede de serviços e sobretudo
- Famílias adotivas devidamente preparadas e respaldado pelo Poder Executivo local;
acompanhadas pela equipe técnica da Vara da Infância e - Poder Executivo desempenhando suas prerrogativas
Juventude (VIJ) da sua comarca e por grupos de apoio à adoção legais, sendo responsável pelo atendimento à população e
(GAA); contando com equipe profissional em estreita parceria com o
- Famílias pretendentes à adoção assessoradas com Conselho Tutelar, realizando o diagnóstico e o
eficiência pela Defensoria Pública e/ou advogado particular acompanhamento às famílias de forma preventiva e protetiva
durante o processo de adoção; por meio medidas de Acolhimento (Institucional ou Familiar),
- Metodologia desenvolvida e consensuada entre a VIJ e o promovendo a proteção social básica e especial da criança e do
GAA para apresentação da família pretendente à criança e ao adolescente em situação de risco;
adolescente a serem adotados respeitando o tempo e o - Organizações não-governamentais oferecendo
entrosamento gradual entre as partes; complementação ao atendimento oferecido pelo Poder
- Crianças e adolescentes com os devidos dados e registros Executivo e requisitando a participação ativa da comunidade
da sua história de vida de forma que a família pretendente na solução de seus problemas; atuando de forma integrada
tenha condições de adotar suas histórias; com as demais organizações da rede de atendimento, de
- Estágio de convivência da família adotiva com a criança e acordo com a sua missão institucional e as necessidades locais
adolescente autorizado pela VIJ da comarca de origem de trabalho especializado;
devidamente respaldado pelo acompanhamento técnico; - Sociedade civil organizada participando ativamente nos
- Busca ativa de pais para crianças e adolescentes Conselhos de Direitos e Setoriais, deliberando e monitorando
priorizando a adoção nacional; as políticas municipais, oferecendo programas de Apoio
- Famílias adotivas frequentando grupos de pais adotivos, Sociofamiliar, atividades socioeducativas, geração de trabalho
recebendo atendimento individualizado com frequência e renda, Abrigo, Casa lar, Repúblicas e Acolhimento Familiar;
sistemática durante o período previamente determinado - Poder Judiciário desempenhando suas prerrogativas
(aproximadamente um ano, podendo estender-se caso legais, aplicando as medidas legais de proteção; contando com
necessário), recorrendo sempre que necessário à equipe equipe técnica interdisciplinar própria, eficientemente
técnica da VIJ, da sua região, todas as vezes que necessitar articulada com todos os atores sociais da região, monitorando
durante o período de adaptação da criança e do adolescente; a aplicação das medidas legais deliberadas em juízo; em
- Seminários e trocas de experiências auxiliando no estreita articulação com o Conselho Tutelar, o Poder Executivo
aprimoramento de metodologias de acompanhamento; e a sociedade civil organizada, promovendo a proteção das
- Sociedade brasileira informada sobre adoções inter- crianças e adolescentes e prestando atendimento efetivo a
raciais e tardias reduzindo o preconceito às famílias adotivas. suas famílias;
- Encaminhamento processual da adoção agilizado, depois - Poder Legislativo desempenhando suas prerrogativas
de esgotadas todas as possibilidades de reintegração à família legais, promovendo a revisão das leis; monitorando o
de origem, evitando a longa permanência de crianças e orçamento público, determinando à Prefeitura a aplicação da
adolescentes nos programas de Acolhimento Institucional; política municipal deliberada no Conselho de Direitos e
- Adoções nacionais bem sucedidas e país sendo Setoriais promovendo por meio de audiências públicas espaço
reconhecido pela qualidade com que promove a adoção aberto para o controle social com participação de todos os
nacional; atores sociais estratégicos;
- Crianças e adolescentes encaminhados para adoção - Conselhos de Direitos e Setoriais desempenhando suas
internacional somente nos casos em que estejam esgotadas prerrogativas legais, sendo responsáveis pela discussão
todas as tentativas de adoção em território nacional, democrática e elaboração de políticas públicas, envolvendo
respeitando a Convenção de Haia de 1993; crianças, adolescentes e suas famílias; controlando as ações do
- Cadastro nacional de adoção em rede informatizada e em Poder Executivo visando à implementação das políticas e
funcionamento organizado sob responsabilidade da Secretaria programas de atendimento, acionando a participação da
Especial dos Direitos Humanos, denominado sociedade civil organizada;
SIPIA/INFOADOTE, favorecendo a comunicação fluente entre - Fundo dos Direitos da Criança e do Adolescente (FDCA) e
diversas Autoridades Centrais Estaduais e o agrupamento de Fundo Municipal da Assistência Social (FMAS)
informações relativas às crianças abrigadas e aos desempenhando uma gestão ágil e autônoma e contando com
pretendentes nacionais e estrangeiros à adoção. a participação dos diversos setores da sociedade; contando,
também, com recursos suficientes para implementar suas
Sistema de Garantia de Direitos da criança e do adolescente propostas;
- Famílias participando ativamente da rede de
- Políticas públicas e, principalmente, sociais, entre elas: atendimento, sendo lideranças protagonistas na defesa dos
educação, saúde, assistência social, cultura, esporte, lazer, direitos de sua comunidade;
trabalho, previdência social, segurança pública, executando - Sociedade em geral, sendo mobilizada por meio de
suas ações intersetorialmente com qualidade e chegando campanhas de divulgação pressionando os Poderes Executivo,
efetivamente aos seus destinatários; Legislativo e Judiciário, de forma a garantir a implementação e
- Conselhos Tutelares, Judiciário, Poder Executivo, a continuidade das políticas públicas;
Organizações Não-Governamentais, Poder Legislativo, - Controle social sobre a execução dos programas e dos
Conselhos de Direitos e Setoriais e sociedade em geral orçamentos contando com a participação popular, além do
desempenhando ativamente suas tarefas e responsabilidades Estado e da sociedade civil organizada;
na rede de atendimento às crianças e adolescentes afastados - Agilidade no fluxo de informações e troca entre atores
ou em vias de afastarem-se do convívio familiar; sociais estratégicos garantindo a otimização dos resultados no

43 Considera-se que o termo raça, longe de possuir na atualidade as diferenças e desigualdades sociais determinadas pela cor e, portanto,
conotações biológicas que tinha nos séculos XIX e começo do XX, é um serve para entender as discriminações raciais existentes no Brasil.
conceito socialmente construído. Utilizado como indicador específico das

Legislação 108
Apostila Digital Licenciada para Alice Caroline Guarino dos Santos - alice.guarino@hotmail.com (Proibida a Revenda)
APOSTILAS OPÇÃO

atendimento às crianças e adolescentes e famílias em situação • Proporcionar informações necessárias e contribuir para
de risco; a tomada de decisões por parte dos responsáveis pela
- Sistema de registro e de tratamento de dados para cada execução dos objetivos e ações do Plano;
caso de criança e adolescente afastado de sua família, por • Acompanhar o desenvolvimento das ações e tarefas
intermédio do SIPIA – Módulo de acompanhamento de referentes à execução do Plano;
crianças e adolescentes em programas de Acolhimento • Controlar as ações, as atividades e os resultados
Familiar e Institucional –, estabelecido e alimentado por todos propostos no Plano assegurando o cronograma previsto;
os atores do Sistema e programado de forma a obter • Socializar informações periodicamente aos diferentes
informações que orientem no diagnóstico, acompanhamento atores do Sistema de Garantia de Direitos e aos Conselhos de
de cada caso e prognóstico; Direitos da Criança e do Adolescente e da Assistência Social;
- Conselho Municipal de Direitos, de Assistência Social e a • Avaliar continuamente a implementação do Plano, nas
Câmara de Vereadores, por meio de suas assembleias e diferentes esferas ajustando as condições operacionais e
audiências públicas, se constituindo em espaços privilegiados correção de rumos durante o processo de execução;
para articulação dos atores sociais locais e participação • Realizar bianualmente a revisão do Plano, de forma a
conjunta na elaboração e monitoramento de políticas públicas adequá-lo às deliberações das Conferências Nacionais dos
de proteção social e de garantia de direitos. Direitos da Criança e do Adolescente e da Assistência Social.

Implementação, Monitoramento e Avaliação Específicas à esfera Federal

O Presente Plano tem como desafio garantir efetivamente • Articular com as Comissões das esferas estadual e
o direito de crianças e adolescentes à convivência familiar e municipal para ampliar o diálogo e acompanhar o
comunitária, principalmente àquelas que se encontram em desenvolvimento das tarefas e ações dos referidos Planos;
situação de vulnerabilidade. • Produzir informações consolidadas sobre a
Sendo assim, sua implementação integral é condição implementação do Plano;
fundamental para uma real mudança do olhar e do fazer que • Socializar as informações consolidadas;
possibilite a concreta experiência e vivência singular da • Cofinanciar as ações necessárias à implementação do
convivência familiar e comunitária para toda criança e presente Plano, bem como dos Planos Estaduais e Municipais;
adolescente no Brasil. Para a materialização deste direito será • O Governo Federal deverá apresentar anualmente
necessário: Relatório de Implementação do Plano Nacional de Promoção,
1) Cumprimento integral deste Plano nas três esferas de Proteção e Defesa do Direito de Crianças e Adolescentes à
governo; Convivência Familiar e Comunitária, inclusive com
2) Constituição formal de Comissão Nacional Intersetorial informações sobre orçamento.
para acompanhamento da implementação do Plano;
3) Elaboração de Planos Estaduais e Municipais em Específicas à esfera Estadual
consonância com o Plano Nacional e constituição de Comissões
Intersetoriais de acompanhamento do Plano nas esferas • Dialogar permanentemente com a Comissão Nacional e
estaduais e municipais; com os municípios, visando o cumprimento deste Plano;
4) Conselhos dos Direitos da Criança e do Adolescente nas • Apoiar os municípios no cumprimento deste Plano,
três esferas públicas assumindo o presente Plano como inclusive na produção de informações a serem consolidadas;
prioridade, a partir de 2007, viabilizando recursos nos • Produzir informações consolidadas sobre a
orçamentos, de um modo geral, e, em particular, nos Fundos implementação do Plano;
da Infância e Adolescência para a sua implementação; • Socializar as informações consolidadas;
5) Participação e integração entre os Conselhos de Direitos • Encaminhar informações sobre monitoramento e as
da Criança e Setoriais nas três esferas de governo; avaliações referentes à implementação do Plano nas esferas
6) Corresponsabilidade entre os entes federativos no Estadual e Municipal em períodos previamente acordados
financiamento para implementação dos objetivos e ações para a Comissão Nacional;
propostos no presente Plano. • Cofinanciar as ações necessárias à implementação do
presente Plano, bem como dos Planos Estaduais e Municipais.
Atribuições e Competências dos entes federativos
Específicas à esfera Municipal
Guardadas as competências e atribuições específicas nas
disposições contidas na Constituição Federal, a realização do • Dialogar permanentemente com a Comissão Nacional e
presente Plano somente será possível se for assumido pelas Estadual;
três esferas públicas (União, Estados e Municípios). Assim, os • Produzir informações consolidadas sobre a
objetivos e ações propostos no presente Plano terão as implementação do Plano;
responsabilidades compartilhadas pelas três esferas de • Socializar as informações consolidadas;
governo. • Encaminhar informações sobre monitoramento e as
avaliações referentes à implementação do Plano na esfera
Competências e atribuições da Comissão de Municipal em períodos previamente acordados para a
Acompanhamento e Implementação do Plano, comuns às três Comissão Nacional;
esferas de governo • Cofinanciar as ações necessárias à implementação do
presente Plano, bem como do Plano Municipal.
• Articular os atores envolvidos na implementação para a
consecução dos objetivos propostos nos eixos: Indicadores de eficácia e monitoramento
a) análise da situação e sistemas de informação; b)
atendimento; c) marcos normativos e regulatórios; d) Para implementação, monitoramento e avaliação do Plano
mobilização, articulação e participação do presente Plano; Nacional de Promoção, Proteção e Defesa do Direito de
• Identificar e mensurar os resultados, efeitos e impactos Crianças e Adolescentes à Convivência Familiar e Comunitária
dos objetivos e ações propostas antes, durante e depois de sua faz-se necessária a coleta de informações que possibilitem o
implementação; acompanhamento da implementação do Plano. Assim, os

Legislação 109
Apostila Digital Licenciada para Alice Caroline Guarino dos Santos - alice.guarino@hotmail.com (Proibida a Revenda)
APOSTILAS OPÇÃO

indicadores abaixo relacionados permitirão o levantamento de Diagnóstico da situação de crianças e adolescentes em


informações e dados que auxiliarão no monitoramento e situação de adoção nacional e internacional.
avaliação do Plano em execução. Cabe ressaltar que os dados a
serem coletados, em sua maioria, devem ser obtidos no • Número de adoções de crianças maiores e adolescentes,
município que é o executor das políticas públicas e a coleta dos afrodescendentes e pertencentes a minorias étnicas, vivendo e
dados deve ser anual. convivendo com HIV/AIDS/AIDS em relação ao número de
crianças e adolescentes que esperam por adoção nas mesmas
Diagnóstico da situação de famílias com crianças e condições, por município, por ano;
adolescentes em Programas de Acolhimento Institucional, • Número de crianças e adolescentes que esperam por
em situação de rua e em medida socioeducativa. adoção: a) por gênero, b) etnia, c) condição de saúde, d) grupo
de irmãos e) idade;
• Número de famílias com crianças/adolescentes em: a) • Número de adoções que tramitaram pelo cadastro das
acolhimento institucional b) situação de rua c) em medida Varas da Infância e Juventude em relação ao universo de
socioeducativa, e outras, comparado com o número de famílias crianças e adolescentes “prontas para adoção”, por município,
da população brasileira, observadas as variações de renda, por ano;
arranjo familiar, meio rural ou urbano e pertencimento étnico; • Número de entrega de bebês às autoridades judiciárias
• Número e perfil das famílias abrangidas pelas diferentes para adoção, por município, por ano; observadas as variações
políticas protetivas, por região ou território, ao ano, inclusive de arranjos familiares, meio rural ou urbano e pertencimento
comparando-se as médias nacionais e regionais; étnico, inclusive comparando-se as médias nacionais e
• Número e perfil de crianças e adolescentes fora do regionais;
convívio familiar devido a: a) por questões de pobreza; b) por • Considerando o universo de crianças entregues às
questões de uso e ou abuso de drogas (lícitas e ilícitas); c) por autoridades judiciárias: a) quantas foram encaminhadas para
violência doméstica; d) por abuso sexual; e) por exploração acolhimento institucional; b) quantas foram encaminhadas
sexual; inclusive comparado com o número de crianças e para acolhimento em família extensiva; c) quantas foram
adolescentes na população brasileira, observadas as variações encaminhadas para programas de famílias acolhedoras; c)
de renda, gênero, meio rural ou urbano e pertencimento quantas retornaram às suas famílias de origem; por município,
étnico; por ano, observadas as variações de arranjos familiares, meio
• Número de famílias das crianças e adolescentes em rural ou urbano e pertencimento étnico, inclusive
acolhimento institucional, em programas de famílias comparando-se as médias nacionais e regionais.
acolhedoras, em situação de rua, medida socioeducativa, e
outras, em programas de transferência de renda, em relação Diagnóstico de situação de operação de políticas
ao total de famílias inseridas neste programa por território públicas (municipais, estaduais) de proteção às crianças,
ano a ano, inclusive observadas as variações de arranjo aos adolescentes e a suas famílias (assistência social; saúde
familiar, meio rural ou urbano e pertencimento étnico, nas – física e mental; tratamentos ao uso e abuso de drogas –
médias nacionais e regionais; lícitas e ilícitas; etc.)
• Número de famílias inseridas em programas de
assistência, saúde, e outros que perderam a guarda temporária • Identificação e perfil das políticas em operação, por
dos filhos, inclusive comparado com o número de famílias da município, incluindo as alternativas previstas para as várias
população brasileira observadas as variações de renda, condições de vulnerabilidade da criança e do adolescente e de
arranjo familiar, meio rural ou urbano e pertencimento étnico; suas famílias;
• Número de famílias atendidas em programas • Metodologia assumida, no nível municipal, para garantir
socioeducativos da proteção social básica, em relação à a integração intersetorial dessas políticas;
totalidade de famílias na mesma faixa de renda no mesmo • Informações sobre a quantidade e qualidade dos
território, ano a ano, observadas as variações de arranjos trabalhadores das instituições acolhedoras de crianças e
familiares, meio rural ou urbano e pertencimento étnico; adolescentes e de seu processo de formação (incluindo se
• Número de famílias inseridas em programas de receberam formação em direitos humanos); por questões de
atendimento para prevenção da violência doméstica, em abandono, maus tratos, violência doméstica, abuso e
relação ao total de demanda e às famílias na mesma faixa de exploração sexual, entre outros, ano a ano;
renda, no território, ano a ano, observadas as variações de • Tempo médio de permanência de crianças e adolescentes
arranjos familiares, meio rural ou urbano e pertencimento em acolhimento institucional, por município, por ano,
étnico; inclusive comparando-se por idade, gênero, etnia, condição de
• Causas geradoras do rompimento dos vínculos saúde e, quando possível, pela renda familiar;
familiares, em relação à população com a mesma faixa de • Tempo médio de permanência de crianças e adolescentes
renda, por município, por ano, observadas as variações de em programas de famílias acolhedoras, família substituta, por
arranjos familiares, meio rural ou urbano e pertencimento município, por ano, inclusive comparando-se por idade,
étnico, inclusive comparando-se as médias nacionais e gênero, etnia, condição de saúde e, quando possível, pela renda
regionais; familiar;
• Causas motivadores da retirada de crianças e • Número de crianças e adolescentes reintegrados à família
adolescentes do convívio familiar e comunitário, em relação à de origem por município, por ano, em relação ao total de
população com a mesma faixa de renda, por ordem do a) crianças e adolescentes em acolhimento institucional e ou em
Juizado b) Conselho tutelar c) própria família, etc. por Programas de Famílias Acolhedoras, inclusive comparando-se
município, por ano, observadas as variações de arranjos por idade, gênero, etnia, condição de saúde e, quando possível,
familiares, meio rural ou urbano e pertencimento étnico, pela renda familiar;
inclusive comparando-se as médias nacionais e regionais. • Número de crianças e adolescentes reintegradas à família
• Número de crianças e adolescentes fora do convívio de origem que retornaram para acolhimento institucional ou
familiar por questões de pobreza, em relação à quantidade de aos Programas de Famílias Acolhedoras, por município, por
crianças que vivem em família na mesma faixa de renda, por ano, em relação ao total de crianças e adolescentes
ano, observadas as variações de arranjos familiares, meio rural reintegradas à família de origem, inclusive comparando-se por
ou urbano e pertencimento étnico, inclusive comparando-se as idade, gênero, etnia, condição de saúde e, quando possível,
médias nacionais e regionais. pela renda familiar;

Legislação 110
Apostila Digital Licenciada para Alice Caroline Guarino dos Santos - alice.guarino@hotmail.com (Proibida a Revenda)
APOSTILAS OPÇÃO

• Considerando a demanda, qual a oferta de políticas - Sistematização e socialização de boas práticas no


públicas para crianças e adolescentes, em especial àquelas que atendimento às famílias, no acolhimento familiar, no
estão privadas do direito à convivência familiar e comunitária, Acolhimento Institucional e na adoção;
por município, por ano. - Fortalecimento e integração das ações governamentais
(intersetorialidade) entre as diferentes políticas e programas
Orçamento Público sociais;
- Fortalecimento e integração entre os diferentes
Valor destinado nos orçamentos do Município, do Estado e Conselhos Setoriais;
da União, por ano, para implantação e implementação das - Aprimoramento dos procedimentos de adoção, incluindo
ações do Plano Nacional de Proteção, Defesa e Garantia do a implementação de cadastro nacional de pretendentes à
Direito da Criança e do Adolescente à Convivência Familiar e adoção e de crianças e adolescentes passíveis de serem
Comunitária. adotados;
- Formação continuada em todos os níveis da federação,
Plano de Ação com metodologias participativas, que promovam a mudança
de paradigma necessária à transformação da cultura que apoia
As propostas operacionais deste Plano estão organizadas o direito a convivência familiar e comunitária.
em quatro eixos estratégicos articulados entre si: 1) Análise da
situação e sistemas de informação, 2) Atendimento, 3) Marcos Eixo 3 – Marcos Normativos e Regulatórios
normativos e regulatórios e 4) Mobilização, articulação e
participação. Os quadros a seguir são resultados de um esforço São propostos objetivos, ações e estratégias que enfatizam:
para propor ações em curto, médio e longo prazo,44 - Aperfeiçoamento dos marcos normativos e regulatórios,
almejando caminhar na direção de uma sociedade que de fato para a efetivação da promoção, defesa e garantia do direito à
respeite o direito à convivência familiar e comunitária. convivência familiar e comunitária.
- Regulamentação e aplicação dos conceitos de
Eixo 1 – Análise da Situação e Sistemas de Informação provisoriedade e excepcionalidade nos programas de Abrigo;
- Regulamentação dos procedimentos necessários ao
São propostos objetivos, ações e estratégias que enfatizam: reordenamento das instituições e seus programas junto aos
- Mapeamento quantitativo e qualitativo sobre família, Conselhos Municipais dos Direitos da Criança e do Adolescente
programas de atendimento e adoção; e de Assistência Social;
- Realização e socialização de pesquisas nas esferas - Elaboração de parâmetros para definição do papel e
estaduais e municipais sobre convivência familiar e função de educador social;
comunitária; - Estabelecimento de parâmetros para as diferentes
- Levantamento de pesquisas existentes que auxiliem na modalidades de programas de Acolhimento Institucional e
análise e indicação de critérios de qualidade do atendimento; para programas de Acolhimento Familiar;
- Identificação de lacunas na oferta de dados dos Sistemas - Regulamentação dos programas de Acolhimento
de Informação; Familiar;
- Implementação nacional do SIPIA nos seus módulos: ( I) - Garantia de igualdade, equidade e inclusão em programas
Registro de violações de direitos, (II) Medidas socioeducativas; de Acolhimento Institucional, Acolhimento Familiar e Adoção;
( III) Cadastro de Adoções /InfoAdote e (IV) Cadastro de - Aprimoramento de instrumentos legais de proteção
Conselhos de Direitos e Tutelares; social que oferecem alternativas e a possibilidade do
- Implantação e implementação de Conselhos Tutelares contraditório à suspensão ou destituição do poder familiar;
que faltam no Brasil; - Regulamentação da legislação referente à Adoção;
- Elaboração de indicadores de monitoramento e - Estabelecimento de parâmetros que garantam a
avaliação; legalidade dos procedimentos de Adoção nacional e
- Aperfeiçoamento, articulação e integração entre os internacional.
sistemas de informação existentes;
- Capacitação de atores estratégicos para Eixo 4 – Mobilização, Articulação e Participação
operacionalização dos sistemas propostos.
São propostos objetivos que enfatizam:
Eixo 2 – Atendimento - Estabelecimento de estratégias de comunicação social
para mobilização da sociedade e afirmação de novos valores;
São propostos objetivos, ações e estratégias que enfatizam: - Qualificação do trabalho da imprensa;
- Ampliação e estruturação de programas de atendimento - Identificação de pontos de contato e interfaces entre o
sociofamiliar; Plano Nacional de Promoção, Defesa e Garantia do Direito de
- Sistematização de metodologias participativas de Crianças e Adolescentes à Convivência Familiar e Comunitária,
trabalho com famílias e comunidade; e os Parâmetros para a institucionalização e fortalecimento do
- Reordenamento institucional do sistema de atendimento Sistema de Garantia de Direitos (Resolução 105, CONANDA) e
com ênfase para os programas de abrigos e instrumentalização do Sistema Nacional de Atendimento Socioeducativo (SINASE)
para sua regulamentação; e com as demais políticas públicas e programas
- Implementação de políticas e programas de acolhimento governamentais, visando à integração de ações;
familiar; - Articulação e integração de ações entre as três esferas de
- Ampliação de programas de emancipação para Poder;
adolescentes e/ou jovens abrigados; - Ampliação da participação da sociedade e do controle
- Construção de parâmetros de atendimento para social;
programas de Acolhimento Institucional, e suas diferentes - Mobilização e articulação para a garantia da
modalidades, e Acolhimento Familiar; provisoriedade, excepcionalidade do Acolhimento
Institucional e para o reordenamento dos Abrigos;

44 Para efeito deste documento considerou-se a seguinte


temporalidade: curto prazo – até 2007; médio prazo – de 2007 a 2010 e
longo prazo – de 2010 a 2016.

Legislação 111
Apostila Digital Licenciada para Alice Caroline Guarino dos Santos - alice.guarino@hotmail.com (Proibida a Revenda)
APOSTILAS OPÇÃO

- Inclusão da temática da convivência familiar e socioeconômico à elaboração de novas formas de interação e


comunitária em cursos de formação de educadores, referências afetivas no grupo familiar.
operadores do SGD, lideranças comunitárias e religiosas, (D) O direito ao vínculo familiar engloba o conceito de
dentre outros atores sociais; família extensa ou ampliada, a qual se constitui pelos pais e
- Inclusão da disciplina “direitos da criança e do irmãos ou por parentes próximos com os quais a criança e o
adolescente” em programas de concursos públicos; adolescente tenham vínculos sanguíneos.
- Garantia de recursos financeiros e orçamentários para a (E) O direito à assistência psicológica gratuita, por
realização deste Plano. intermédio do Sistema Único de Saúde, é exclusivo da mãe,
após o parto, a fim de minimizar os efeitos do estado
Indicadores de Eficácia e Monitoramento puerperal, se comprovada por junta médica a ocorrência de
depressão.
- Número de famílias atendidas em programas
socioeducativos da proteção social básica e em relação ao Respostas
número de famílias na mesma faixa de renda na população
brasileira; 01. A / 02. C
- Proporção entre eventos de ruptura de vínculos
familiares nos territórios atendidos pelos programas de
proteção social básica e no total da população na mesma faixa ESTATUTO DO IDOSO – Lei
de renda;
- Número de crianças e adolescentes reintegrados à família
10.741/2003.
de origem por município, por ano;
- Proporção entre o número de crianças e adolescentes em
LEI Nº 10.741, DE 1º DE OUTUBRO DE 2003
Acolhimento Institucional e o número de crianças e
adolescentes em Acolhimento Familiar por município, por ano;
Dispõe sobre o Estatuto do Idoso e dá outras providências.
- Tempo médio de permanência de crianças e adolescentes
no Acolhimento Institucional e Familiar por município, por
O PRESIDENTE DA REPÚBLICA Faço saber que o
ano;
Congresso Nacional decreta e eu sanciono a seguinte Lei:
- Número de adoções que tramitam regularmente no
cadastro nacional;
TÍTULO I
- Número de adoções tardias, inter-raciais, com deficiência
DISPOSIÇÕES PRELIMINARES
e afetados pelo vírus HIV/AIDS por estado e por ano.
Art. 1º É instituído o Estatuto do Idoso, destinado a regular
Questões
os direitos assegurados às pessoas com idade igual ou superior
a 60 (sessenta) anos.
01. (MPE-SE - Analista do Ministério Público –
Especialidade Serviço Social – FCC/2014) O Plano Nacional
Art. 2º O idoso goza de todos os direitos fundamentais
de Promoção, Proteção e Defesa do Direito de Crianças e
inerentes à pessoa humana, sem prejuízo da proteção integral
Adolescentes à convivência familiar e comunitária constitui
de que trata esta Lei, assegurando-se lhe, por lei ou por outros
um marco nas políticas públicas no Brasil ao romper com a
meios, todas as oportunidades e facilidades, para preservação
cultura da institucionalização de crianças e adolescentes. As
de sua saúde física e mental e seu aperfeiçoamento moral,
estratégias, objetivos e diretrizes deste Plano estão
intelectual, espiritual e social, em condições de liberdade e
fundamentados na:
dignidade.
(A) prevenção do rompimento dos vínculos familiares.
(B) qualificação profissional dos profissionais que
Art. 3º É obrigação da família, da comunidade, da
trabalham em instituições de abrigo.
sociedade e do Poder Público assegurar ao idoso, com absoluta
(C) reordenação dos programas de atendimento
prioridade, a efetivação do direito à vida, à saúde, à
institucional.
alimentação, à educação, à cultura, ao esporte, ao lazer, ao
(D) divulgação de informações e sensibilização da
trabalho, à cidadania, à liberdade, à dignidade, ao respeito e à
sociedade brasileira em relação à adoção.
convivência familiar e comunitária.
(E) concretização, sensibilização e desmistificação para a
Parágrafo único. A garantia de prioridade compreende:
adoção.
I – atendimento preferencial imediato e individualizado
junto aos órgãos públicos e privados prestadores de serviços à
02. (SEAP-DF - Conselheiro Tutelar –
população;
FUNIVERSA/2014) O Plano Nacional de Promoção, Proteção
II – preferência na formulação e na execução de políticas
e Defesa do Direito de Crianças e Adolescentes à Convivência
sociais públicas específicas;
Familiar e Comunitária está fundamentado nas disposições do
III – destinação privilegiada de recursos públicos nas áreas
Estatuto da Criança e do Adolescente, visando garantir à mãe,
relacionadas com a proteção ao idoso;
à criança e ao adolescente o cumprimento de alguns direitos.
IV – viabilização de formas alternativas de participação,
Assinale a alternativa correta acerca desses direitos.
ocupação e convívio do idoso com as demais gerações;
(A) A mãe tem liberdade de entregar seu filho para a
V – priorização do atendimento do idoso por sua própria
adoção, desde que comprove que o interessado seja pessoa
família, em detrimento do atendimento asilar, exceto dos que
idônea e atenda aos requisitos legais de idade mínima.
não a possuam ou careçam de condições de manutenção da
(B) A criança e o adolescente terão acolhimento garantido
própria sobrevivência;
em instituição ou família substituta, e a situação em
VI – capacitação e reciclagem dos recursos humanos nas
atendimento ao convívio familiar, obrigatoriamente, não
áreas de geriatria e gerontologia e na prestação de serviços aos
ultrapassará o prazo de dois anos.
idosos;
(C) Nas situações de risco e enfraquecimento dos vínculos
VII – estabelecimento de mecanismos que favoreçam a
familiares, as estratégias de atendimento deverão esgotar as
divulgação de informações de caráter educativo sobre os
possibilidades de preservação desses vínculos, aliando o apoio
aspectos biopsicossociais de envelhecimento;

Legislação 112
Apostila Digital Licenciada para Alice Caroline Guarino dos Santos - alice.guarino@hotmail.com (Proibida a Revenda)
APOSTILAS OPÇÃO

VIII – garantia de acesso à rede de serviços de saúde e de CAPÍTULO III


assistência social locais. DOS ALIMENTOS
IX – prioridade no recebimento da restituição do Imposto
de Renda. Art. 11. Os alimentos serão prestados ao idoso na forma da
lei civil.
Art. 4º Nenhum idoso será objeto de qualquer tipo de
negligência, discriminação, violência, crueldade ou opressão, e Art. 12. A obrigação alimentar é solidária, podendo o idoso
todo atentado aos seus direitos, por ação ou omissão, será optar entre os prestadores.
punido na forma da lei.
§ 1º É dever de todos prevenir a ameaça ou violação aos Art. 13. As transações relativas a alimentos poderão ser
direitos do idoso. celebradas perante o Promotor de Justiça ou Defensor Público,
§ 2º As obrigações previstas nesta Lei não excluem da que as referendará, e passarão a ter efeito de título executivo
prevenção outras decorrentes dos princípios por ela adotados. extrajudicial nos termos da lei processual civil.

Art. 5º A inobservância das normas de prevenção Art. 14. Se o idoso ou seus familiares não possuírem
importará em responsabilidade à pessoa física ou jurídica nos condições econômicas de prover o seu sustento, impõe-se ao
termos da lei. Poder Público esse provimento, no âmbito da assistência
social.
Art. 6º Todo cidadão tem o dever de comunicar à
autoridade competente qualquer forma de violação a esta Lei CAPÍTULO IV
que tenha testemunhado ou de que tenha conhecimento. DO DIREITO À SAÚDE

Art. 7º Os Conselhos Nacional, Estaduais, do Distrito Art. 15. É assegurada a atenção integral à saúde do idoso,
Federal e Municipais do Idoso, previstos na Lei no 8.842, de 4 por intermédio do Sistema Único de Saúde – SUS, garantindo-
de janeiro de 1994, zelarão pelo cumprimento dos direitos do lhe o acesso universal e igualitário, em conjunto articulado e
idoso, definidos nesta Lei. contínuo das ações e serviços, para a prevenção, promoção,
proteção e recuperação da saúde, incluindo a atenção especial
TÍTULO II às doenças que afetam preferencialmente os idosos.
DOS DIREITOS FUNDAMENTAIS § 1º A prevenção e a manutenção da saúde do idoso serão
CAPÍTULO I efetivadas por meio de:
DO DIREITO À VIDA I – cadastramento da população idosa em base territorial;
II – atendimento geriátrico e gerontológico em
Art. 8º O envelhecimento é um direito personalíssimo e a ambulatórios;
sua proteção um direito social, nos termos desta Lei e da III – unidades geriátricas de referência, com pessoal
legislação vigente. especializado nas áreas de geriatria e gerontologia social;
IV – atendimento domiciliar, incluindo a internação, para a
Art. 9º É obrigação do Estado, garantir à pessoa idosa a população que dele necessitar e esteja impossibilitada de se
proteção à vida e à saúde, mediante efetivação de políticas locomover, inclusive para idosos abrigados e acolhidos por
sociais públicas que permitam um envelhecimento saudável e instituições públicas, filantrópicas ou sem fins lucrativos e
em condições de dignidade. eventualmente conveniadas com o Poder Público, nos meios
urbano e rural;
CAPÍTULO II V – reabilitação orientada pela geriatria e gerontologia,
DO DIREITO À LIBERDADE, AO RESPEITO E À para redução das sequelas decorrentes do agravo da saúde.
DIGNIDADE § 2º Incumbe ao Poder Público fornecer aos idosos,
gratuitamente, medicamentos, especialmente os de uso
Art. 10. É obrigação do Estado e da sociedade, assegurar à continuado, assim como próteses, órteses e outros recursos
pessoa idosa a liberdade, o respeito e a dignidade, como pessoa relativos ao tratamento, habilitação ou reabilitação.
humana e sujeito de direitos civis, políticos, individuais e § 3º É vedada a discriminação do idoso nos planos de saúde
sociais, garantidos na Constituição e nas leis. pela cobrança de valores diferenciados em razão da idade.
§ 1º O direito à liberdade compreende, entre outros, os § 4º Os idosos portadores de deficiência ou com limitação
seguintes aspectos: incapacitante terão atendimento especializado, nos termos da
I – faculdade de ir, vir e estar nos logradouros públicos e lei.
espaços comunitários, ressalvadas as restrições legais; § 5º É vedado exigir o comparecimento do idoso enfermo
II – opinião e expressão; perante os órgãos públicos, hipótese na qual será admitido o
III – crença e culto religioso; seguinte procedimento:
IV – prática de esportes e de diversões; I - quando de interesse do poder público, o agente
V – participação na vida familiar e comunitária; promoverá o contato necessário com o idoso em sua
VI – participação na vida política, na forma da lei; residência; ou
VII – faculdade de buscar refúgio, auxílio e orientação. II - quando de interesse do próprio idoso, este se fará
§ 2º O direito ao respeito consiste na inviolabilidade da representar por procurador legalmente constituído.
integridade física, psíquica e moral, abrangendo a preservação § 6º É assegurado ao idoso enfermo o atendimento
da imagem, da identidade, da autonomia, de valores, ideias e domiciliar pela perícia médica do Instituto Nacional do Seguro
crenças, dos espaços e dos objetos pessoais. Social - INSS, pelo serviço público de saúde ou pelo serviço
§ 3º É dever de todos zelar pela dignidade do idoso, privado de saúde, contratado ou conveniado, que integre o
colocando-o a salvo de qualquer tratamento desumano, Sistema Único de Saúde - SUS, para expedição do laudo de
violento, aterrorizante, vexatório ou constrangedor. saúde necessário ao exercício de seus direitos sociais e de
isenção tributária.

Legislação 113
Apostila Digital Licenciada para Alice Caroline Guarino dos Santos - alice.guarino@hotmail.com (Proibida a Revenda)
APOSTILAS OPÇÃO

Art. 16. Ao idoso internado ou em observação é assegurado Art. 22. Nos currículos mínimos dos diversos níveis de
o direito a acompanhante, devendo o órgão de saúde ensino formal serão inseridos conteúdos voltados ao processo
proporcionar as condições adequadas para a sua permanência de envelhecimento, ao respeito e à valorização do idoso, de
em tempo integral, segundo o critério médico. forma a eliminar o preconceito e a produzir conhecimentos
Parágrafo único. Caberá ao profissional de saúde sobre a matéria.
responsável pelo tratamento conceder autorização para o
acompanhamento do idoso ou, no caso de impossibilidade, Art. 23. A participação dos idosos em atividades culturais e
justificá-la por escrito. de lazer será proporcionada mediante descontos de pelo
menos 50% (cinquenta por cento) nos ingressos para eventos
Art. 17. Ao idoso que esteja no domínio de suas faculdades artísticos, culturais, esportivos e de lazer, bem como o acesso
mentais é assegurado o direito de optar pelo tratamento de preferencial aos respectivos locais.
saúde que lhe for reputado mais favorável.
Parágrafo único. Não estando o idoso em condições de Art. 24. Os meios de comunicação manterão espaços ou
proceder à opção, esta será feita: horários especiais voltados aos idosos, com finalidade
I – pelo curador, quando o idoso for interditado; informativa, educativa, artística e cultural, e ao público sobre
II – pelos familiares, quando o idoso não tiver curador ou o processo de envelhecimento.
este não puder ser contatado em tempo hábil;
III – pelo médico, quando ocorrer iminente risco de vida e Art. 25. O Poder Público apoiará a criação de universidade
não houver tempo hábil para consulta a curador ou familiar; aberta para as pessoas idosas e incentivará a publicação de
IV – pelo próprio médico, quando não houver curador ou livros e periódicos, de conteúdo e padrão editorial adequados
familiar conhecido, caso em que deverá comunicar o fato ao ao idoso, que facilitem a leitura, considerada a natural redução
Ministério Público. da capacidade visual.

Art. 18. As instituições de saúde devem atender aos CAPÍTULO VI


critérios mínimos para o atendimento às necessidades do DA PROFISSIONALIZAÇÃO E DO TRABALHO
idoso, promovendo o treinamento e a capacitação dos
profissionais, assim como orientação a cuidadores familiares e Art. 26. O idoso tem direito ao exercício de atividade
grupos de autoajuda. profissional, respeitadas suas condições físicas, intelectuais e
psíquicas.
Art. 19. Os casos de suspeita ou confirmação de violência
praticada contra idosos serão objeto de notificação Art. 27. Na admissão do idoso em qualquer trabalho ou
compulsória pelos serviços de saúde públicos e privados à emprego, é vedada a discriminação e a fixação de limite
autoridade sanitária, bem como serão obrigatoriamente máximo de idade, inclusive para concursos, ressalvados os
comunicados por eles a quaisquer dos seguintes órgãos: casos em que a natureza do cargo o exigir.
I – autoridade policial; Parágrafo único. O primeiro critério de desempate em
II – Ministério Público; concurso público será a idade, dando-se preferência ao de
III – Conselho Municipal do Idoso; idade mais elevada.
IV – Conselho Estadual do Idoso;
V – Conselho Nacional do Idoso. Art. 28. O Poder Público criará e estimulará programas de:
§ 1º Para os efeitos desta Lei, considera-se violência contra I – profissionalização especializada para os idosos,
o idoso qualquer ação ou omissão praticada em local público aproveitando seus potenciais e habilidades para atividades
ou privado que lhe cause morte, dano ou sofrimento físico ou regulares e remuneradas;
psicológico. II – preparação dos trabalhadores para a aposentadoria,
§ 2º Aplica-se, no que couber, à notificação compulsória com antecedência mínima de 1 (um) ano, por meio de estímulo
prevista no caput deste artigo, o disposto na Lei no 6.259, de a novos projetos sociais, conforme seus interesses, e de
30 de outubro de 1975. esclarecimento sobre os direitos sociais e de cidadania;
III – estímulo às empresas privadas para admissão de
CAPÍTULO V idosos ao trabalho.
DA EDUCAÇÃO, CULTURA, ESPORTE E LAZER
CAPÍTULO VII
Art. 20. O idoso tem direito a educação, cultura, esporte, DA PREVIDÊNCIA SOCIAL
lazer, diversões, espetáculos, produtos e serviços que
respeitem sua peculiar condição de idade. Art. 29. Os benefícios de aposentadoria e pensão do
Regime Geral da Previdência Social observarão, na sua
Art. 21. O Poder Público criará oportunidades de acesso do concessão, critérios de cálculo que preservem o valor real dos
idoso à educação, adequando currículos, metodologias e salários sobre os quais incidiram contribuição, nos termos da
material didático aos programas educacionais a ele legislação vigente.
destinados. Parágrafo único. Os valores dos benefícios em manutenção
§ 1º Os cursos especiais para idosos incluirão conteúdo serão reajustados na mesma data de reajuste do salário-
relativo às técnicas de comunicação, computação e demais mínimo, pro rata, de acordo com suas respectivas datas de
avanços tecnológicos, para sua integração à vida moderna. início ou do seu último reajustamento, com base em
§ 2º Os idosos participarão das comemorações de caráter percentual definido em regulamento, observados os critérios
cívico ou cultural, para transmissão de conhecimentos e estabelecidos pela Lei no 8.213, de 24 de julho de 1991.
vivências às demais gerações, no sentido da preservação da
memória e da identidade culturais. Art. 30. A perda da condição de segurado não será
considerada para a concessão da aposentadoria por idade,
desde que a pessoa conte com, no mínimo, o tempo de
contribuição correspondente ao exigido para efeito de
carência na data de requerimento do benefício.

Legislação 114
Apostila Digital Licenciada para Alice Caroline Guarino dos Santos - alice.guarino@hotmail.com (Proibida a Revenda)
APOSTILAS OPÇÃO

Parágrafo único. O cálculo do valor do benefício previsto necessidades deles, bem como provê-los com alimentação
no caput observará o disposto no caput e § 2º do art. 3º da Lei regular e higiene indispensáveis às normas sanitárias e com
no 9.876, de 26 de novembro de 1999, ou, não havendo estas condizentes, sob as penas da lei.
salários-de-contribuição recolhidos a partir da competência
de julho de 1994, o disposto no art. 35 da Lei no 8.213, de 1991. Art. 38. Nos programas habitacionais, públicos ou
subsidiados com recursos públicos, o idoso goza de prioridade
Art. 31. O pagamento de parcelas relativas a benefícios, na aquisição de imóvel para moradia própria, observado o
efetuado com atraso por responsabilidade da Previdência seguinte:
Social, será atualizado pelo mesmo índice utilizado para os I - reserva de pelo menos 3% (três por cento) das unidades
reajustamentos dos benefícios do Regime Geral de Previdência habitacionais residenciais para atendimento aos idosos;
Social, verificado no período compreendido entre o mês que II – implantação de equipamentos urbanos comunitários
deveria ter sido pago e o mês do efetivo pagamento. voltados ao idoso;
III – eliminação de barreiras arquitetônicas e urbanísticas,
Art. 32. O Dia Mundial do Trabalho, 1º de Maio, é a data- para garantia de acessibilidade ao idoso;
base dos aposentados e pensionistas. IV – critérios de financiamento compatíveis com os
rendimentos de aposentadoria e pensão.
CAPÍTULO VIII Parágrafo único. As unidades residenciais reservadas para
DA ASSISTÊNCIA SOCIAL atendimento a idosos devem situar-se, preferencialmente, no
pavimento térreo.
Art. 33. A assistência social aos idosos será prestada, de
forma articulada, conforme os princípios e diretrizes previstos CAPÍTULO X
na Lei Orgânica da Assistência Social, na Política Nacional do DO TRANSPORTE
Idoso, no Sistema Único de Saúde e demais normas
pertinentes. Art. 39. Aos maiores de 65 (sessenta e cinco) anos fica
assegurada a gratuidade dos transportes coletivos públicos
Art. 34. Aos idosos, a partir de 65 (sessenta e cinco) anos, urbanos e semiurbanos, exceto nos serviços seletivos e
que não possuam meios para prover sua subsistência, nem de especiais, quando prestados paralelamente aos serviços
tê-la provida por sua família, é assegurado o benefício mensal regulares.
de 1 (um) salário-mínimo, nos termos da Lei Orgânica da § 1º Para ter acesso à gratuidade, basta que o idoso
Assistência Social – Loas. apresente qualquer documento pessoal que faça prova de sua
Parágrafo único. O benefício já concedido a qualquer idade.
membro da família nos termos do caput não será computado § 2º Nos veículos de transporte coletivo de que trata este
para os fins do cálculo da renda familiar per capita a que se artigo, serão reservados 10% (dez por cento) dos assentos
refere a Loas. para os idosos, devidamente identificados com a placa de
reservado preferencialmente para idosos.
Art. 35. Todas as entidades de longa permanência, ou casa- § 3º No caso das pessoas compreendidas na faixa etária
lar, são obrigadas a firmar contrato de prestação de serviços entre 60 (sessenta) e 65 (sessenta e cinco) anos, ficará a
com a pessoa idosa abrigada. critério da legislação local dispor sobre as condições para
§ 1º No caso de entidades filantrópicas, ou casa-lar, é exercício da gratuidade nos meios de transporte previstos no
facultada a cobrança de participação do idoso no custeio da caput deste artigo.
entidade.
§ 2º O Conselho Municipal do Idoso ou o Conselho Art. 40. No sistema de transporte coletivo interestadual
Municipal da Assistência Social estabelecerá a forma de observar-se-á, nos termos da legislação específica:
participação prevista no § 1º, que não poderá exceder a 70% I – a reserva de 2 (duas) vagas gratuitas por veículo para
(setenta por cento) de qualquer benefício previdenciário ou de idosos com renda igual ou inferior a 2 (dois) salários-mínimos;
assistência social percebido pelo idoso. II – desconto de 50% (cinquenta por cento), no mínimo, no
§ 3º Se a pessoa idosa for incapaz, caberá a seu valor das passagens, para os idosos que excederem as vagas
representante legal firmar o contrato a que se refere o caput gratuitas, com renda igual ou inferior a 2 (dois) salários-
deste artigo. mínimos.
Parágrafo único. Caberá aos órgãos competentes definir os
Art. 36. O acolhimento de idosos em situação de risco mecanismos e os critérios para o exercício dos direitos
social, por adulto ou núcleo familiar, caracteriza a dependência previstos nos incisos I e II.
econômica, para os efeitos legais.
Art. 41. É assegurada a reserva, para os idosos, nos termos
CAPÍTULO IX da lei local, de 5% (cinco por cento) das vagas nos
DA HABITAÇÃO estacionamentos públicos e privados, as quais deverão ser
posicionadas de forma a garantir a melhor comodidade ao
Art. 37. O idoso tem direito a moradia digna, no seio da idoso.
família natural ou substituta, ou desacompanhado de seus
familiares, quando assim o desejar, ou, ainda, em instituição Art. 42. São asseguradas a prioridade e a segurança do
pública ou privada. idoso nos procedimentos de embarque e desembarque nos
§ 1º A assistência integral na modalidade de entidade de veículos do sistema de transporte coletivo.
longa permanência será prestada quando verificada
inexistência de grupo familiar, casa-lar, abandono ou carência
de recursos financeiros próprios ou da família.
§ 2º Toda instituição dedicada ao atendimento ao idoso
fica obrigada a manter identificação externa visível, sob pena
de interdição, além de atender toda a legislação pertinente.
§ 3º As instituições que abrigarem idosos são obrigadas a
manter padrões de habitação compatíveis com as

Legislação 115
Apostila Digital Licenciada para Alice Caroline Guarino dos Santos - alice.guarino@hotmail.com (Proibida a Revenda)
APOSTILAS OPÇÃO

TÍTULO III planejamento e execução emanadas do órgão competente da


DAS MEDIDAS DE PROTEÇÃO Política Nacional do Idoso, conforme a Lei no 8.842, de 1994.
CAPÍTULO I Parágrafo único. As entidades governamentais e não-
DAS DISPOSIÇÕES GERAIS governamentais de assistência ao idoso ficam sujeitas à
inscrição de seus programas, junto ao órgão competente da
Art. 43. As medidas de proteção ao idoso são aplicáveis Vigilância Sanitária e Conselho Municipal da Pessoa Idosa, e
sempre que os direitos reconhecidos nesta Lei forem em sua falta, junto ao Conselho Estadual ou Nacional da Pessoa
ameaçados ou violados: Idosa, especificando os regimes de atendimento, observados
I – por ação ou omissão da sociedade ou do Estado; os seguintes requisitos:
II – por falta, omissão ou abuso da família, curador ou I – oferecer instalações físicas em condições adequadas de
entidade de atendimento; habitabilidade, higiene, salubridade e segurança;
III – em razão de sua condição pessoal. II – apresentar objetivos estatutários e plano de trabalho
compatíveis com os princípios desta Lei;
CAPÍTULO II III – estar regularmente constituída;
DAS MEDIDAS ESPECÍFICAS DE PROTEÇÃO IV – demonstrar a idoneidade de seus dirigentes.

Art. 44. As medidas de proteção ao idoso previstas nesta Art. 49. As entidades que desenvolvam programas de
Lei poderão ser aplicadas, isolada ou cumulativamente, e institucionalização de longa permanência adotarão os
levarão em conta os fins sociais a que se destinam e o seguintes princípios:
fortalecimento dos vínculos familiares e comunitários. I – preservação dos vínculos familiares;
II – atendimento personalizado e em pequenos grupos;
Art. 45. Verificada qualquer das hipóteses previstas no art. III – manutenção do idoso na mesma instituição, salvo em
43, o Ministério Público ou o Poder Judiciário, a requerimento caso de força maior;
daquele, poderá determinar, dentre outras, as seguintes IV – participação do idoso nas atividades comunitárias, de
medidas: caráter interno e externo;
I – encaminhamento à família ou curador, mediante termo V – observância dos direitos e garantias dos idosos;
de responsabilidade; VI – preservação da identidade do idoso e oferecimento de
II – orientação, apoio e acompanhamento temporários; ambiente de respeito e dignidade.
III – requisição para tratamento de sua saúde, em regime Parágrafo único. O dirigente de instituição prestadora de
ambulatorial, hospitalar ou domiciliar; atendimento ao idoso responderá civil e criminalmente pelos
IV – inclusão em programa oficial ou comunitário de atos que praticar em detrimento do idoso, sem prejuízo das
auxílio, orientação e tratamento a usuários dependentes de sanções administrativas.
drogas lícitas ou ilícitas, ao próprio idoso ou à pessoa de sua
convivência que lhe cause perturbação; Art. 50. Constituem obrigações das entidades de
V – abrigo em entidade; atendimento:
VI – abrigo temporário. I – celebrar contrato escrito de prestação de serviço com o
idoso, especificando o tipo de atendimento, as obrigações da
TÍTULO IV entidade e prestações decorrentes do contrato, com os
DA POLÍTICA DE ATENDIMENTO AO IDOSO respectivos preços, se for o caso;
CAPÍTULO I II – observar os direitos e as garantias de que são titulares
DISPOSIÇÕES GERAIS os idosos;
III – fornecer vestuário adequado, se for pública, e
Art. 46. A política de atendimento ao idoso far-se-á por alimentação suficiente;
meio do conjunto articulado de ações governamentais e não- IV – oferecer instalações físicas em condições adequadas
governamentais da União, dos Estados, do Distrito Federal e de habitabilidade;
dos Municípios. V – oferecer atendimento personalizado;
VI – diligenciar no sentido da preservação dos vínculos
Art. 47. São linhas de ação da política de atendimento: familiares;
I – políticas sociais básicas, previstas na Lei no 8.842, de 4 VII – oferecer acomodações apropriadas para recebimento
de janeiro de 1994; de visitas;
II – políticas e programas de assistência social, em caráter VIII – proporcionar cuidados à saúde, conforme a
supletivo, para aqueles que necessitarem; necessidade do idoso;
III – serviços especiais de prevenção e atendimento às IX – promover atividades educacionais, esportivas,
vítimas de negligência, maus-tratos, exploração, abuso, culturais e de lazer;
crueldade e opressão; X – propiciar assistência religiosa àqueles que desejarem,
IV – serviço de identificação e localização de parentes ou de acordo com suas crenças;
responsáveis por idosos abandonados em hospitais e XI – proceder a estudo social e pessoal de cada caso;
instituições de longa permanência; XII – comunicar à autoridade competente de saúde toda
V – proteção jurídico-social por entidades de defesa dos ocorrência de idoso portador de doenças infectocontagiosas;
direitos dos idosos; XIII – providenciar ou solicitar que o Ministério Público
VI – mobilização da opinião pública no sentido da requisite os documentos necessários ao exercício da cidadania
participação dos diversos segmentos da sociedade no àqueles que não os tiverem, na forma da lei;
atendimento do idoso. XIV – fornecer comprovante de depósito dos bens móveis
que receberem dos idosos;
CAPÍTULO II XV – manter arquivo de anotações onde constem data e
DAS ENTIDADES DE ATENDIMENTO AO IDOSO circunstâncias do atendimento, nome do idoso, responsável,
parentes, endereços, cidade, relação de seus pertences, bem
Art. 48. As entidades de atendimento são responsáveis pela como o valor de contribuições, e suas alterações, se houver, e
manutenção das próprias unidades, observadas as normas de demais dados que possibilitem sua identificação e a
individualização do atendimento;

Legislação 116
Apostila Digital Licenciada para Alice Caroline Guarino dos Santos - alice.guarino@hotmail.com (Proibida a Revenda)
APOSTILAS OPÇÃO

XVI – comunicar ao Ministério Público, para as CAPÍTULO IV


providências cabíveis, a situação de abandono moral ou DAS INFRAÇÕES ADMINISTRATIVAS
material por parte dos familiares;
XVII – manter no quadro de pessoal profissionais com Art. 56. Deixar a entidade de atendimento de cumprir as
formação específica. determinações do art. 50 desta Lei:
Pena – multa de R$ 500,00 (quinhentos reais) a R$
Art. 51. As instituições filantrópicas ou sem fins lucrativos 3.000,00 (três mil reais), se o fato não for caracterizado como
prestadoras de serviço ao idoso terão direito à assistência crime, podendo haver a interdição do estabelecimento até que
judiciária gratuita. sejam cumpridas as exigências legais.
Parágrafo único. No caso de interdição do estabelecimento
CAPÍTULO III de longa permanência, os idosos abrigados serão transferidos
DA FISCALIZAÇÃO DAS ENTIDADES DE ATENDIMENTO para outra instituição, a expensas do estabelecimento
interditado, enquanto durar a interdição.
Art. 52. As entidades governamentais e não-
governamentais de atendimento ao idoso serão fiscalizadas Art. 57. Deixar o profissional de saúde ou o responsável por
pelos Conselhos do Idoso, Ministério Público, Vigilância estabelecimento de saúde ou instituição de longa permanência
Sanitária e outros previstos em lei. de comunicar à autoridade competente os casos de crimes
contra idoso de que tiver conhecimento:
Art. 53. O art. 7º da Lei no 8.842, de 1994, passa a vigorar Pena – multa de R$ 500,00 (quinhentos reais) a R$
com a seguinte redação: 3.000,00 (três mil reais), aplicada em dobro no caso de
reincidência.
"Art. 7º Compete aos Conselhos de que trata o art. 6º desta
Lei a supervisão, o acompanhamento, a fiscalização e a Art. 58. Deixar de cumprir as determinações desta Lei
avaliação da política nacional do idoso, no âmbito das sobre a prioridade no atendimento ao idoso:
respectivas instâncias político-administrativas." (NR) Pena – multa de R$ 500,00 (quinhentos reais) a R$
1.000,00 (um mil reais) e multa civil a ser estipulada pelo juiz,
Art. 54. Será dada publicidade das prestações de contas dos conforme o dano sofrido pelo idoso.
recursos públicos e privados recebidos pelas entidades de
atendimento. CAPÍTULO V
DA APURAÇÃO ADMINISTRATIVA DE INFRAÇÃO ÀS
Art. 55. As entidades de atendimento que descumprirem as NORMAS DE PROTEÇÃO AO IDOSO
determinações desta Lei ficarão sujeitas, sem prejuízo da
responsabilidade civil e criminal de seus dirigentes ou Art. 59. Os valores monetários expressos no Capítulo IV
prepostos, às seguintes penalidades, observado o devido serão atualizados anualmente, na forma da lei.
processo legal:
I – as entidades governamentais: Art. 60. O procedimento para a imposição de penalidade
a) advertência; administrativa por infração às normas de proteção ao idoso
b) afastamento provisório de seus dirigentes; terá início com requisição do Ministério Público ou auto de
c) afastamento definitivo de seus dirigentes; infração elaborado por servidor efetivo e assinado, se possível,
d) fechamento de unidade ou interdição de programa; por duas testemunhas.
II – as entidades não-governamentais: § 1º No procedimento iniciado com o auto de infração
a) advertência; poderão ser usadas fórmulas impressas, especificando-se a
b) multa; natureza e as circunstâncias da infração.
c) suspensão parcial ou total do repasse de verbas § 2º Sempre que possível, à verificação da infração seguir-
públicas; se-á a lavratura do auto, ou este será lavrado dentro de 24
d) interdição de unidade ou suspensão de programa; (vinte e quatro) horas, por motivo justificado.
e) proibição de atendimento a idosos a bem do interesse
público. Art. 61. O autuado terá prazo de 10 (dez) dias para a
§ 1º Havendo danos aos idosos abrigados ou qualquer tipo apresentação da defesa, contado da data da intimação, que
de fraude em relação ao programa, caberá o afastamento será feita:
provisório dos dirigentes ou a interdição da unidade e a I – pelo autuante, no instrumento de autuação, quando for
suspensão do programa. lavrado na presença do infrator;
§ 2º A suspensão parcial ou total do repasse de verbas II – por via postal, com aviso de recebimento.
públicas ocorrerá quando verificada a má aplicação ou desvio
de finalidade dos recursos. Art. 62. Havendo risco para a vida ou à saúde do idoso, a
§ 3º Na ocorrência de infração por entidade de autoridade competente aplicará à entidade de atendimento as
atendimento, que coloque em risco os direitos assegurados sanções regulamentares, sem prejuízo da iniciativa e das
nesta Lei, será o fato comunicado ao Ministério Público, para providências que vierem a ser adotadas pelo Ministério
as providências cabíveis, inclusive para promover a suspensão Público ou pelas demais instituições legitimadas para a
das atividades ou dissolução da entidade, com a proibição de fiscalização.
atendimento a idosos a bem do interesse público, sem prejuízo
das providências a serem tomadas pela Vigilância Sanitária. Art. 63. Nos casos em que não houver risco para a vida ou
§ 4º Na aplicação das penalidades, serão consideradas a a saúde da pessoa idosa abrigada, a autoridade competente
natureza e a gravidade da infração cometida, os danos que dela aplicará à entidade de atendimento as sanções
provierem para o idoso, as circunstâncias agravantes ou regulamentares, sem prejuízo da iniciativa e das providências
atenuantes e os antecedentes da entidade. que vierem a ser adotadas pelo Ministério Público ou pelas
demais instituições legitimadas para a fiscalização.

Legislação 117
Apostila Digital Licenciada para Alice Caroline Guarino dos Santos - alice.guarino@hotmail.com (Proibida a Revenda)
APOSTILAS OPÇÃO

CAPÍTULO VI § 3º A prioridade se estende aos processos e


DA APURAÇÃO JUDICIAL DE IRREGULARIDADES EM procedimentos na Administração Pública, empresas
ENTIDADE DE ATENDIMENTO prestadoras de serviços públicos e instituições financeiras, ao
atendimento preferencial junto à Defensoria Pública da União,
Art. 64. Aplicam-se, subsidiariamente, ao procedimento dos Estados e do Distrito Federal em relação aos Serviços de
administrativo de que trata este Capítulo as disposições das Assistência Judiciária.
Leis nos 6.437, de 20 de agosto de 1977, e 9.784, de 29 de § 4º Para o atendimento prioritário será garantido ao idoso
janeiro de 1999. o fácil acesso aos assentos e caixas, identificados com a
destinação a idosos em local visível e caracteres legíveis.
Art. 65. O procedimento de apuração de irregularidade em
entidade governamental e não-governamental de atendimento CAPÍTULO II
ao idoso terá início mediante petição fundamentada de pessoa DO MINISTÉRIO PÚBLICO
interessada ou iniciativa do Ministério Público.
Art. 72. (VETADO)
Art. 66. Havendo motivo grave, poderá a autoridade
judiciária, ouvido o Ministério Público, decretar liminarmente Art. 73. As funções do Ministério Público, previstas nesta
o afastamento provisório do dirigente da entidade ou outras Lei, serão exercidas nos termos da respectiva Lei Orgânica.
medidas que julgar adequadas, para evitar lesão aos direitos
do idoso, mediante decisão fundamentada. Art. 74. Compete ao Ministério Público:
I – instaurar o inquérito civil e a ação civil pública para a
Art. 67. O dirigente da entidade será citado para, no prazo proteção dos direitos e interesses difusos ou coletivos,
de 10 (dez) dias, oferecer resposta escrita, podendo juntar individuais indisponíveis e individuais homogêneos do idoso;
documentos e indicar as provas a produzir. II – promover e acompanhar as ações de alimentos, de
interdição total ou parcial, de designação de curador especial,
Art. 68. Apresentada a defesa, o juiz procederá na em circunstâncias que justifiquem a medida e oficiar em todos
conformidade do art. 69 ou, se necessário, designará audiência os feitos em que se discutam os direitos de idosos em
de instrução e julgamento, deliberando sobre a necessidade de condições de risco;
produção de outras provas. III – atuar como substituto processual do idoso em situação
§ 1º Salvo manifestação em audiência, as partes e o de risco, conforme o disposto no art. 43 desta Lei;
Ministério Público terão 5 (cinco) dias para oferecer alegações IV – promover a revogação de instrumento procuratório
finais, decidindo a autoridade judiciária em igual prazo. do idoso, nas hipóteses previstas no art. 43 desta Lei, quando
§ 2º Em se tratando de afastamento provisório ou necessário ou o interesse público justificar;
definitivo de dirigente de entidade governamental, a V – instaurar procedimento administrativo e, para instruí-
autoridade judiciária oficiará a autoridade administrativa lo:
imediatamente superior ao afastado, fixando-lhe prazo de 24 a) expedir notificações, colher depoimentos ou
(vinte e quatro) horas para proceder à substituição. esclarecimentos e, em caso de não comparecimento
§ 3º Antes de aplicar qualquer das medidas, a autoridade injustificado da pessoa notificada, requisitar condução
judiciária poderá fixar prazo para a remoção das coercitiva, inclusive pela Polícia Civil ou Militar;
irregularidades verificadas. Satisfeitas as exigências, o b) requisitar informações, exames, perícias e documentos
processo será extinto, sem julgamento do mérito. de autoridades municipais, estaduais e federais, da
§ 4º A multa e a advertência serão impostas ao dirigente da administração direta e indireta, bem como promover
entidade ou ao responsável pelo programa de atendimento. inspeções e diligências investigatórias;
c) requisitar informações e documentos particulares de
TÍTULO V instituições privadas;
DO ACESSO À JUSTIÇA VI – instaurar sindicâncias, requisitar diligências
CAPÍTULO I investigatórias e a instauração de inquérito policial, para a
DISPOSIÇÕES GERAIS apuração de ilícitos ou infrações às normas de proteção ao
idoso;
Art. 69. Aplica-se, subsidiariamente, às disposições deste VII – zelar pelo efetivo respeito aos direitos e garantias
Capítulo, o procedimento sumário previsto no Código de legais assegurados ao idoso, promovendo as medidas judiciais
Processo Civil, naquilo que não contrarie os prazos previstos e extrajudiciais cabíveis;
nesta Lei. VIII – inspecionar as entidades públicas e particulares de
atendimento e os programas de que trata esta Lei, adotando de
Art. 70. O Poder Público poderá criar varas especializadas pronto as medidas administrativas ou judiciais necessárias à
e exclusivas do idoso. remoção de irregularidades porventura verificadas;
IX – requisitar força policial, bem como a colaboração dos
Art. 71. É assegurada prioridade na tramitação dos serviços de saúde, educacionais e de assistência social,
processos e procedimentos e na execução dos atos e públicos, para o desempenho de suas atribuições;
diligências judiciais em que figure como parte ou interveniente X – referendar transações envolvendo interesses e direitos
pessoa com idade igual ou superior a 60 (sessenta) anos, em dos idosos previstos nesta Lei.
qualquer instância. § 1º A legitimação do Ministério Público para as ações
§ 1º O interessado na obtenção da prioridade a que alude cíveis previstas neste artigo não impede a de terceiros, nas
este artigo, fazendo prova de sua idade, requererá o benefício mesmas hipóteses, segundo dispuser a lei.
à autoridade judiciária competente para decidir o feito, que § 2º As atribuições constantes deste artigo não excluem
determinará as providências a serem cumpridas, anotando-se outras, desde que compatíveis com a finalidade e atribuições
essa circunstância em local visível nos autos do processo. do Ministério Público.
§ 2º A prioridade não cessará com a morte do beneficiado, § 3º O representante do Ministério Público, no exercício de
estendendo-se em favor do cônjuge supérstite, companheiro suas funções, terá livre acesso a toda entidade de atendimento
ou companheira, com união estável, maior de 60 (sessenta) ao idoso.
anos.

Legislação 118
Apostila Digital Licenciada para Alice Caroline Guarino dos Santos - alice.guarino@hotmail.com (Proibida a Revenda)
APOSTILAS OPÇÃO

Art. 75. Nos processos e procedimentos em que não for Art. 83. Na ação que tenha por objeto o cumprimento de
parte, atuará obrigatoriamente o Ministério Público na defesa obrigação de fazer ou não-fazer, o juiz concederá a tutela
dos direitos e interesses de que cuida esta Lei, hipóteses em específica da obrigação ou determinará providências que
que terá vista dos autos depois das partes, podendo juntar assegurem o resultado prático equivalente ao adimplemento.
documentos, requerer diligências e produção de outras § 1º Sendo relevante o fundamento da demanda e havendo
provas, usando os recursos cabíveis. justificado receio de ineficácia do provimento final, é lícito ao
juiz conceder a tutela liminarmente ou após justificação
Art. 76. A intimação do Ministério Público, em qualquer prévia, na forma do art. 273 do Código de Processo Civil.
caso, será feita pessoalmente. § 2º O juiz poderá, na hipótese do § 1º ou na sentença,
impor multa diária ao réu, independentemente do pedido do
Art. 77. A falta de intervenção do Ministério Público autor, se for suficiente ou compatível com a obrigação, fixando
acarreta a nulidade do feito, que será declarada de ofício pelo prazo razoável para o cumprimento do preceito.
juiz ou a requerimento de qualquer interessado. § 3º A multa só será exigível do réu após o trânsito em
julgado da sentença favorável ao autor, mas será devida desde
CAPÍTULO III o dia em que se houver configurado.
DA PROTEÇÃO JUDICIAL DOS INTERESSES DIFUSOS,
COLETIVOS E INDIVIDUAIS INDISPONÍVEIS OU Art. 84. Os valores das multas previstas nesta Lei
HOMOGÊNEOS reverterão ao Fundo do Idoso, onde houver, ou na falta deste,
ao Fundo Municipal de Assistência Social, ficando vinculados
Art. 78. As manifestações processuais do representante do ao atendimento ao idoso.
Ministério Público deverão ser fundamentadas. Parágrafo único. As multas não recolhidas até 30 (trinta)
dias após o trânsito em julgado da decisão serão exigidas por
Art. 79. Regem-se pelas disposições desta Lei as ações de meio de execução promovida pelo Ministério Público, nos
responsabilidade por ofensa aos direitos assegurados ao mesmos autos, facultada igual iniciativa aos demais
idoso, referentes à omissão ou ao oferecimento insatisfatório legitimados em caso de inércia daquele.
de:
I – acesso às ações e serviços de saúde; Art. 85. O juiz poderá conferir efeito suspensivo aos
II – atendimento especializado ao idoso portador de recursos, para evitar dano irreparável à parte.
deficiência ou com limitação incapacitante;
III – atendimento especializado ao idoso portador de Art. 86. Transitada em julgado a sentença que impuser
doença infectocontagiosa; condenação ao Poder Público, o juiz determinará a remessa de
IV – serviço de assistência social visando ao amparo do peças à autoridade competente, para apuração da
idoso. responsabilidade civil e administrativa do agente a que se
Parágrafo único. As hipóteses previstas neste artigo não atribua a ação ou omissão.
excluem da proteção judicial outros interesses difusos,
coletivos, individuais indisponíveis ou homogêneos, próprios Art. 87. Decorridos 60 (sessenta) dias do trânsito em
do idoso, protegidos em lei. julgado da sentença condenatória favorável ao idoso sem que
o autor lhe promova a execução, deverá fazê-lo o Ministério
Art. 80. As ações previstas neste Capítulo serão propostas Público, facultada, igual iniciativa aos demais legitimados,
no foro do domicílio do idoso, cujo juízo terá competência como assistentes ou assumindo o polo ativo, em caso de
absoluta para processar a causa, ressalvadas as competências inércia desse órgão.
da Justiça Federal e a competência originária dos Tribunais
Superiores. Art. 88. Nas ações de que trata este Capítulo, não haverá
adiantamento de custas, emolumentos, honorários periciais e
Art. 81. Para as ações cíveis fundadas em interesses quaisquer outras despesas.
difusos, coletivos, individuais indisponíveis ou homogêneos, Parágrafo único. Não se imporá sucumbência ao Ministério
consideram-se legitimados, concorrentemente: Público.
I – o Ministério Público;
II – a União, os Estados, o Distrito Federal e os Municípios; Art. 89. Qualquer pessoa poderá, e o servidor deverá,
III – a Ordem dos Advogados do Brasil; provocar a iniciativa do Ministério Público, prestando-lhe
IV – as associações legalmente constituídas há pelo menos informações sobre os fatos que constituam objeto de ação civil
1 (um) ano e que incluam entre os fins institucionais a defesa e indicando-lhe os elementos de convicção.
dos interesses e direitos da pessoa idosa, dispensada a
autorização da assembleia, se houver prévia autorização Art. 90. Os agentes públicos em geral, os juízes e tribunais,
estatutária. no exercício de suas funções, quando tiverem conhecimento de
§ 1º Admitir-se-á litisconsórcio facultativo entre os fatos que possam configurar crime de ação pública contra
Ministérios Públicos da União e dos Estados na defesa dos idoso ou ensejar a propositura de ação para sua defesa, devem
interesses e direitos de que cuida esta Lei. encaminhar as peças pertinentes ao Ministério Público, para
§ 2º Em caso de desistência ou abandono da ação por as providências cabíveis.
associação legitimada, o Ministério Público ou outro
legitimado deverá assumir a titularidade ativa. Art. 91. Para instruir a petição inicial, o interessado poderá
requerer às autoridades competentes as certidões e
Art. 82. Para defesa dos interesses e direitos protegidos informações que julgar necessárias, que serão fornecidas no
por esta Lei, são admissíveis todas as espécies de ação prazo de 10 (dez) dias.
pertinentes.
Parágrafo único. Contra atos ilegais ou abusivos de Art. 92. O Ministério Público poderá instaurar sob sua
autoridade pública ou agente de pessoa jurídica no exercício presidência, inquérito civil, ou requisitar, de qualquer pessoa,
de atribuições de Poder Público, que lesem direito líquido e organismo público ou particular, certidões, informações,
certo previsto nesta Lei, caberá ação mandamental, que se exames ou perícias, no prazo que assinalar, o qual não poderá
regerá pelas normas da lei do mandado de segurança. ser inferior a 10 (dez) dias.

Legislação 119
Apostila Digital Licenciada para Alice Caroline Guarino dos Santos - alice.guarino@hotmail.com (Proibida a Revenda)
APOSTILAS OPÇÃO

§ 1º Se o órgão do Ministério Público, esgotadas todas as Art. 98. Abandonar o idoso em hospitais, casas de saúde,
diligências, se convencer da inexistência de fundamento para entidades de longa permanência, ou congêneres, ou não
a propositura da ação civil ou de peças informativas, prover suas necessidades básicas, quando obrigado por lei ou
determinará o seu arquivamento, fazendo-o mandado:
fundamentadamente. Pena – detenção de 6 (seis) meses a 3 (três) anos e multa.
§ 2º Os autos do inquérito civil ou as peças de informação
arquivados serão remetidos, sob pena de se incorrer em falta Art. 99. Expor a perigo a integridade e a saúde, física ou
grave, no prazo de 3 (três) dias, ao Conselho Superior do psíquica, do idoso, submetendo-o a condições desumanas ou
Ministério Público ou à Câmara de Coordenação e Revisão do degradantes ou privando-o de alimentos e cuidados
Ministério Público. indispensáveis, quando obrigado a fazê-lo, ou sujeitando-o a
§ 3º Até que seja homologado ou rejeitado o arquivamento, trabalho excessivo ou inadequado:
pelo Conselho Superior do Ministério Público ou por Câmara Pena – detenção de 2 (dois) meses a 1 (um) ano e multa.
de Coordenação e Revisão do Ministério Público, as § 1º Se do fato resulta lesão corporal de natureza grave:
associações legitimadas poderão apresentar razões escritas ou Pena – reclusão de 1 (um) a 4 (quatro) anos.
documentos, que serão juntados ou anexados às peças de § 2º Se resulta a morte:
informação. Pena – reclusão de 4 (quatro) a 12 (doze) anos.
§ 4º Deixando o Conselho Superior ou a Câmara de
Coordenação e Revisão do Ministério Público de homologar a Art. 100. Constitui crime punível com reclusão de 6 (seis)
promoção de arquivamento, será designado outro membro do meses a 1 (um) ano e multa:
Ministério Público para o ajuizamento da ação. I – obstar o acesso de alguém a qualquer cargo público por
motivo de idade;
O Estatuto do Idoso traz em seu Título VI importantes II – negar a alguém, por motivo de idade, emprego ou
disposições acerca da tutela penal ao idoso. Tal proteção tem trabalho;
como bem jurídico a dignidade da pessoa humana. III – recusar, retardar ou dificultar atendimento ou deixar
de prestar assistência à saúde, sem justa causa, a pessoa idosa;
TÍTULO VI IV – deixar de cumprir, retardar ou frustrar, sem justo
DOS CRIMES motivo, a execução de ordem judicial expedida na ação civil a
CAPÍTULO I que alude esta Lei;
DISPOSIÇÕES GERAIS V – recusar, retardar ou omitir dados técnicos
indispensáveis à propositura da ação civil objeto desta Lei,
Art. 93. Aplicam-se subsidiariamente, no que couber, as quando requisitados pelo Ministério Público.
disposições da Lei nº 7.347, de 24 de julho de 1985.
Art. 101. Deixar de cumprir, retardar ou frustrar, sem justo
Art. 94. Aos crimes previstos nesta Lei, cuja pena máxima motivo, a execução de ordem judicial expedida nas ações em
privativa de liberdade não ultrapasse 4 (quatro) anos, aplica- que for parte ou interveniente o idoso:
se o procedimento previsto na Lei nº 9.099, de 26 de setembro Pena – detenção de 6 (seis) meses a 1 (um) ano e multa.
de 1995, e, subsidiariamente, no que couber, as disposições do
Código Penal e do Código de Processo Penal. Art. 102. Apropriar-se de ou desviar bens, proventos,
pensão ou qualquer outro rendimento do idoso, dando-lhes
CAPÍTULO II aplicação diversa da de sua finalidade:
DOS CRIMES EM ESPÉCIE Pena – reclusão de 1 (um) a 4 (quatro) anos e multa.

Art. 95. Os crimes definidos nesta Lei são de ação penal Art. 103. Negar o acolhimento ou a permanência do idoso,
pública incondicionada, não se lhes aplicando os arts. 181 e como abrigado, por recusa deste em outorgar procuração à
182 do Código Penal. entidade de atendimento:
Pena – detenção de 6 (seis) meses a 1 (um) ano e multa.
Art. 96. Discriminar pessoa idosa, impedindo ou
dificultando seu acesso a operações bancárias, aos meios de Art. 104. Reter o cartão magnético de conta bancária
transporte, ao direito de contratar ou por qualquer outro meio relativa a benefícios, proventos ou pensão do idoso, bem como
ou instrumento necessário ao exercício da cidadania, por qualquer outro documento com objetivo de assegurar
motivo de idade: recebimento ou ressarcimento de dívida:
Pena – reclusão de 6 (seis) meses a 1 (um) ano e multa. Pena – detenção de 6 (seis) meses a 2 (dois) anos e multa.
§ 1º Na mesma pena incorre quem desdenhar, humilhar,
menosprezar ou discriminar pessoa idosa, por qualquer Art. 105. Exibir ou veicular, por qualquer meio de
motivo. comunicação, informações ou imagens depreciativas ou
§ 2º A pena será aumentada de 1/3 (um terço) se a vítima injuriosas à pessoa do idoso:
se encontrar sob os cuidados ou responsabilidade do agente. Pena – detenção de 1 (um) a 3 (três) anos e multa.

Art. 97. Deixar de prestar assistência ao idoso, quando Art. 106. Induzir pessoa idosa sem discernimento de seus
possível fazê-lo sem risco pessoal, em situação de iminente atos a outorgar procuração para fins de administração de bens
perigo, ou recusar, retardar ou dificultar sua assistência à ou deles dispor livremente:
saúde, sem justa causa, ou não pedir, nesses casos, o socorro Pena – reclusão de 2 (dois) a 4 (quatro) anos.
de autoridade pública:
Pena – detenção de 6 (seis) meses a 1 (um) ano e multa. Art. 107. Coagir, de qualquer modo, o idoso a doar,
Parágrafo único. A pena é aumentada de metade, se da contratar, testar ou outorgar procuração:
omissão resulta lesão corporal de natureza grave, e triplicada, Pena – reclusão de 2 (dois) a 5 (cinco) anos.
se resulta a morte.
Art. 108. Lavrar ato notarial que envolva pessoa idosa sem
discernimento de seus atos, sem a devida representação legal:
Pena – reclusão de 2 (dois) a 4 (quatro) anos.

Legislação 120
Apostila Digital Licenciada para Alice Caroline Guarino dos Santos - alice.guarino@hotmail.com (Proibida a Revenda)
APOSTILAS OPÇÃO

TÍTULO VII Art. 112. O inciso II do § 4º do art. 1º da Lei nº 9.455, de 7


DISPOSIÇÕES FINAIS E TRANSITÓRIAS de abril de 1997, passa a vigorar com a seguinte redação:

Art. 109. Impedir ou embaraçar ato do representante do "Art. 1º ...


Ministério Público ou de qualquer outro agente fiscalizador: § 4º ...
Pena – reclusão de 6 (seis) meses a 1 (um) ano e multa. II – se o crime é cometido contra criança, gestante,
portador de deficiência, adolescente ou maior de 60 (sessenta)
Art. 110. O Decreto-Lei no 2.848, de 7 de dezembro de anos; (NR)
1940, Código Penal, passa a vigorar com as seguintes
alterações: Art. 113. O inciso III do art. 18 da Lei no 6.368, de 21 de
"Art. 61. ... outubro de 1976, passa a vigorar com a seguinte redação:
II - ...
h) contra criança, maior de 60 (sessenta) anos, enfermo ou "Art. 18...
mulher grávida;" (NR) III – se qualquer deles decorrer de associação ou visar a
menores de 21 (vinte e um) anos ou a pessoa com idade igual
"Art. 121. ... ou superior a 60 (sessenta) anos ou a quem tenha, por
§ 4º No homicídio culposo, a pena é aumentada de 1/3 (um qualquer causa, diminuída ou suprimida a capacidade de
terço), se o crime resulta de inobservância de regra técnica de discernimento ou de autodeterminação:
profissão, arte ou ofício, ou se o agente deixa de prestar
imediato socorro à vítima, não procura diminuir as Art. 114. O art. 1º da Lei no 10.048, de 8 de novembro de
consequências do seu ato, ou foge para evitar prisão em 2000, passa a vigorar com a seguinte redação:
flagrante. Sendo doloso o homicídio, a pena é aumentada de
1/3 (um terço) se o crime é praticado contra pessoa menor de "Art. 1º As pessoas portadoras de deficiência, os idosos
14 (quatorze) ou maior de 60 (sessenta) anos. (NR) com idade igual ou superior a 60 (sessenta) anos, as gestantes,
as lactantes e as pessoas acompanhadas por crianças de colo
"Art. 133. ... terão atendimento prioritário, nos termos desta Lei." (NR)
§ 3º ...
III – se a vítima é maior de 60 (sessenta) anos." (NR) Art. 115. O Orçamento da Seguridade Social destinará ao
Fundo Nacional de Assistência Social, até que o Fundo
"Art. 140. ... Nacional do Idoso seja criado, os recursos necessários, em
§ 3º Se a injúria consiste na utilização de elementos cada exercício financeiro, para aplicação em programas e
referentes a raça, cor, etnia, religião, origem ou a condição de ações relativos ao idoso.
pessoa idosa ou portadora de deficiência: (NR)
Art. 116. Serão incluídos nos censos demográficos dados
"Art. 141. ... relativos à população idosa do País.
IV – contra pessoa maior de 60 (sessenta) anos ou
portadora de deficiência, exceto no caso de injúria." (NR) Art. 117. O Poder Executivo encaminhará ao Congresso
Nacional projeto de lei revendo os critérios de concessão do
"Art. 148. ... Benefício de Prestação Continuada previsto na Lei Orgânica da
§ 1º... Assistência Social, de forma a garantir que o acesso ao direito
I – se a vítima é ascendente, descendente, cônjuge do seja condizente com o estágio de desenvolvimento
agente ou maior de 60 (sessenta) anos. " (NR) socioeconômico alcançado pelo País.

"Art. 159... Art. 118. Esta Lei entra em vigor decorridos 90 (noventa)
§ 1º Se o sequestro dura mais de 24 (vinte e quatro) horas, dias da sua publicação, ressalvado o disposto no caput do art.
se o sequestrado é menor de 18 (dezoito) ou maior de 60 36, que vigorará a partir de 1º de janeiro de 2004.
(sessenta) anos, ou se o crime é cometido por bando ou
quadrilha. " (NR) Questões

"Art. 183... 01. (Prefeitura de Natal/RN - Advogado –


III – se o crime é praticado contra pessoa com idade igual IDECAN/2016). Estabelece o Estatuto do Idoso que NÃO
ou superior a 60 (sessenta) anos." (NR) constitui obrigação das entidades de atendimento ao idoso:
(A) Oferecer instalações físicas em condições adequadas
"Art. 244. Deixar, sem justa causa, de prover a subsistência de habitabilidade.
do cônjuge, ou de filho menor de 18 (dezoito) anos ou inapto (B) Comunicar ao Ministério Público, para as providências
para o trabalho, ou de ascendente inválido ou maior de 60 cabíveis, a situação de abandono moral ou material por parte
(sessenta) anos, não lhes proporcionando os recursos dos familiares.
necessários ou faltando ao pagamento de pensão alimentícia (C) Fornecer vestuário adequado, se for privada, e
judicialmente acordada, fixada ou majorada; deixar, sem justa assistência religiosa mesmo àqueles que não desejarem e de
causa, de socorrer descendente ou ascendente, gravemente acordo com suas crenças.
enfermo: (NR) (D) Celebrar contrato escrito de prestação de serviço com
o idoso, especificando o tipo de atendimento, as obrigações da
Art. 111. O art. 21 do Decreto-Lei nº 3.688, de 3 de outubro entidade e prestações decorrentes do contrato, com os
de 1941, Lei das Contravenções Penais, passa a vigorar respectivos preços, se for o caso.
acrescido do seguinte parágrafo único:
02. (MPE/GO – Promotor de Justiça- MPE/GO/2016) De
"Art. 21... acordo com o Estatuto do Idoso (Lei n. 10.471/03):
Parágrafo único. Aumenta-se a pena de 1/3 (um terço) até (A) O Ministério Público tem legitimidade para a
a metade se a vítima é maior de 60 (sessenta) anos." (NR) promoção da tutela coletiva dos direitos de pessoas com idade
igual ou superior a sessenta anos, mas não poderá atuar na

Legislação 121
Apostila Digital Licenciada para Alice Caroline Guarino dos Santos - alice.guarino@hotmail.com (Proibida a Revenda)
APOSTILAS OPÇÃO

esfera individual de direitos dessa parcela da população, uma (B) não é considerada como crime, uma vez que Pomona,
vez que a senilidade não induz incapacidade para os atos da embora abandonada, foi deixada sob cuidados médicos.
vida civil. (C) não é considerada crime, por se tratar de hospital
(B) O idoso, que necessite de alimentos, deverá acionar público, que tem a obrigação legal de cuidar de Pomona.
simultaneamente os filhos, cobrando de cada qual, na medida (D) seria considerada crime pelo Estatuto do Idoso apenas
de suas possibilidades. se Pomona fosse maior de 65 anos de idade.
(C) O Poder Judiciário, a requerimento do Ministério (E) é considerada um crime pelo Estatuto do Idoso.
Público, poderá determinar medidas protetivas em favor de
idoso em situação de risco, tais como: requisição de 07. (DPE/PB - Defensor Público – FCC) O Estatuto do
tratamento de saúde, em regime ambulatorial, hospitalar ou Idoso define o idoso como aquele com idade igual ou superior
domiciliar; encaminhamento à família ou curador, mediante a
termo de responsabilidade; abrigamento em entidade. (A) 60 (sessenta) anos, garantindo a ele todos os direitos
(D) O Poder Público tem responsabilidade residual e, no previstos no respectivo diploma legal.
âmbito da assistência social, estará obrigado a assegurar os (B) 65 (sessenta e cinco) anos, garantindo a ele todos os
direitos fundamentais de pessoa idosa, em caso de inexistência direitos previstos no respectivo diploma legal.
de parentes na linha reta ou colateral até o 3º grau. (C) 70 (setenta) anos, garantindo a ele todos os direitos
previstos no respectivo diploma legal.
03. (Prefeitura de Xaxim/SC – Assistente Social – (D) 60 (sessenta) anos, mas estabelecendo idades e
ASSCONPP/2015) Assinale a alternativa correta a respeito do circunstâncias diferenciadas para o exercício pleno de todos os
estatuto do idoso. direitos previstos no respectivo diploma legal.
(A) Priorização do atendimento do idoso por sua própria (E) 65 (sessenta e cinco) anos, mas estabelecendo idades e
família, em detrimento do atendimento asilar, exceto dos que circunstâncias diferenciadas para o exercício pleno de todos os
não a possuam ou careçam de condições de manutenção da direitos previstos no respectivo diploma legal.
própria sobrevivência.
(B) Atendimento preferencial imediato e individualizado 08. (DPE/AM - Defensor Público – FCC) O Estatuto do
junto aos órgãos públicos e privados prestadores de serviços à Idoso define a violência contra o idoso como sendo
população. (A) o atentado contra a pessoa do idoso, nos termos da lei
(C) Garantia de acesso à rede de serviços de saúde e de penal.
assistência social locais. (B) a prática dos crimes contra a vida, de lesões corporais,
(D) Todas as alternativas estão corretas de periclitação da vida e da saúde e contra a liberdade
individual do idoso.
04. (PC-SC - Delegado de Polícia - ACAFE/2014) Analise (C) o crime que envolver violência doméstica e familiar
as afirmações a seguir, identifique o que constitui crime contra o idoso.
praticado contra o idoso e assinale a alternativa correta. (D) o atentado contra os direitos fundamentais do idoso.
l Obstar o acesso de alguém a qualquer cargo público por (E) a ação ou omissão praticada em local público ou
motivo de idade. privado que lhe cause morte, dano ou sofrimento físico ou
ll Recusar, retardar ou dificultar atendimento ou deixar de psicológico.
prestar assistência à saúde, sem justa causa, a pessoa com mais
de 55 anos. 09. (DPE/SC - Técnico Administrativo – FEPESE)
lll Deixar de cumprir, retardar ou frustrar, sem justo Assinale a alternativa correta de acordo com o Estatuto do
motivo, a execução de ordem judicial expedida nas ações em Idoso.
que for parte ou interveniente pessoa com mais de 65 anos. (A) É dever exclusivo dos familiares prevenir as ameaças
lV Reter o cartão magnético de conta bancária relativa a ou violação aos direitos do idoso.
benefícios, proventos ou pensão de pessoa com mais de 70 (B) A proteção ao idoso deve ser restrita à preservação de
anos, bem como qualquer outro documento, com objetivo de sua saúde física e mental.
assegurar recebimento ou ressarcimento de dívida. (C) Somente as pessoas físicas poderão ser
(A) Todas as afirmações estão corretas. responsabilizadas pela inobservância das normas de
(B) Apenas II e III estão corretas. prevenção e proteção ao idoso.
(C) Apenas I, II e III estão corretas (D) O Estatuto do Idoso, por ser norma especial em razão
(D) Apenas I, III e IV estão corretas. da matéria, impede a aplicação de qualquer outra legislação.
(E) Apenas III e IV estão corretas. (E) Todo cidadão tem o dever de comunicar à autoridade
competente qualquer forma de violação a esta Lei que tenha
05. (MPE/SC - Promotor de Justiça - MPE-SC) Analise os testemunhado ou de que tenha conhecimento.
enunciados das questões abaixo e assinale se ele é Certo ou
Errado. 10. (TJ/RJ - Comissário da Infância e da Juventude –
A Lei n. 10.741/03 (Estatuto do Idoso) possui tipo penal FCC) Segundo prevê o Estatuto do Idoso, é obrigação da
específico para punir tabelião que lavrar ato notarial que entidade de atendimento ao idoso
envolva pessoa idosa sem discernimento de seus atos e sem a (A) comunicar ao juiz as situações de abandono moral ou
devida representação legal. material por parte dos familiares.
(A) Certo (B) celebrar contrato escrito ou verbal de prestação de
(B) Errado serviço com o idoso.
(C) elaborar e remeter ao Ministério Público plano
06. (PC-SP - Investigador de Polícia - VUNESP) Minerva, individual de atendimento para cada caso com vistas à
45 anos de idade, é filha de Pomona, 62 anos de idade. Ambas reintegração familiar.
vivem juntas. Quando Pomona veio a adoecer gravemente, (D) administrar os rendimentos financeiros de seus
Minerva a levou para um hospital público e lá a abandonou sob usuários.
os cuidados médicos do estabelecimento, não mais retornando (E) proporcionar cuidados à saúde, conforme a
para buscá-la. Essa conduta de Minerva. necessidade do idoso.
(A) é considerada um crime de preconceito punível pelo
Estatuto do Idoso

Legislação 122
Apostila Digital Licenciada para Alice Caroline Guarino dos Santos - alice.guarino@hotmail.com (Proibida a Revenda)
APOSTILAS OPÇÃO

Respostas II - desenho universal: concepção de produtos, ambientes,


programas e serviços a serem usados por todas as pessoas,
01. C / 02 C. / 03. D. / 04. D / 05. A sem necessidade de adaptação ou de projeto específico,
06. E / 07. D / 08. E / 09. E / 10. E incluindo os recursos de tecnologia assistiva;
III - tecnologia assistiva ou ajuda técnica: produtos,
equipamentos, dispositivos, recursos, metodologias,
LEI BRASILEIRA DE estratégias, práticas e serviços que objetivem promover a
funcionalidade, relacionada à atividade e à participação da
INCLUSÃO DA PESSOA COM pessoa com deficiência ou com mobilidade reduzida, visando à
DEFICIÊNCIA - LEI sua autonomia, independência, qualidade de vida e inclusão
13.146/2015. social;
IV - barreiras: qualquer entrave, obstáculo, atitude ou
comportamento que limite ou impeça a participação social da
pessoa, bem como o gozo, a fruição e o exercício de seus
LEI Nº 13.146, DE 6 DE JULHO DE 201545 direitos à acessibilidade, à liberdade de movimento e de
expressão, à comunicação, ao acesso à informação, à
Institui a Lei Brasileira de Inclusão da Pessoa com compreensão, à circulação com segurança, entre outros,
Deficiência (Estatuto da Pessoa com Deficiência). classificadas em:
a) barreiras urbanísticas: as existentes nas vias e nos
LIVRO I espaços públicos e privados abertos ao público ou de uso
PARTE GERAL coletivo;
TÍTULO I b) barreiras arquitetônicas: as existentes nos edifícios
DISPOSIÇÕES PRELIMINARES públicos e privados;
CAPÍTULO I c) barreiras nos transportes: as existentes nos sistemas e
DISPOSIÇÕES GERAIS meios de transportes;
d) barreiras nas comunicações e na informação: qualquer
Art. 1º É instituída a Lei Brasileira de Inclusão da Pessoa entrave, obstáculo, atitude ou comportamento que dificulte ou
com Deficiência (Estatuto da Pessoa com Deficiência), impossibilite a expressão ou o recebimento de mensagens e de
destinada a assegurar e a promover, em condições de informações por intermédio de sistemas de comunicação e de
igualdade, o exercício dos direitos e das liberdades tecnologia da informação;
fundamentais por pessoa com deficiência, visando à sua e) barreiras atitudinais: atitudes ou comportamentos que
inclusão social e cidadania. impeçam ou prejudiquem a participação social da pessoa com
Parágrafo único. Esta Lei tem como base a Convenção deficiência em igualdade de condições e oportunidades com as
sobre os Direitos das Pessoas com Deficiência e seu Protocolo demais pessoas;
Facultativo, ratificados pelo Congresso Nacional por meio do f) barreiras tecnológicas: as que dificultam ou impedem o
Decreto Legislativo no 186, de 9 de julho de 2008, em acesso da pessoa com deficiência às tecnologias;
conformidade com o procedimento previsto no § 3º do art. 5º V - comunicação: forma de interação dos cidadãos que
da Constituição da República Federativa do Brasil, em vigor abrange, entre outras opções, as línguas, inclusive a Língua
para o Brasil, no plano jurídico externo, desde 31 de agosto de Brasileira de Sinais (Libras), a visualização de textos, o Braille,
2008, e promulgados pelo Decreto nº 6.949, de 25 de agosto o sistema de sinalização ou de comunicação tátil, os caracteres
de 2009, data de início de sua vigência no plano interno. ampliados, os dispositivos multimídia, assim como a
linguagem simples, escrita e oral, os sistemas auditivos e os
Art. 2º Considera-se pessoa com deficiência aquela que meios de voz digitalizados e os modos, meios e formatos
tem impedimento de longo prazo de natureza física, mental, aumentativos e alternativos de comunicação, incluindo as
intelectual ou sensorial, o qual, em interação com uma ou mais tecnologias da informação e das comunicações;
barreiras, pode obstruir sua participação plena e efetiva na VI - adaptações razoáveis: adaptações, modificações e
sociedade em igualdade de condições com as demais pessoas. ajustes necessários e adequados que não acarretem ônus
§ 1º A avaliação da deficiência, quando necessária, será desproporcional e indevido, quando requeridos em cada caso,
biopsicossocial, realizada por equipe multiprofissional e a fim de assegurar que a pessoa com deficiência possa gozar
interdisciplinar e considerará: ou exercer, em igualdade de condições e oportunidades com as
I - os impedimentos nas funções e nas estruturas do corpo; demais pessoas, todos os direitos e liberdades fundamentais;
II - os fatores socioambientais, psicológicos e pessoais; VII - elemento de urbanização: quaisquer componentes de
III - a limitação no desempenho de atividades; e obras de urbanização, tais como os referentes a pavimentação,
IV - a restrição de participação. saneamento, encanamento para esgotos, distribuição de
§ 2º O Poder Executivo criará instrumentos para avaliação energia elétrica e de gás, iluminação pública, serviços de
da deficiência. comunicação, abastecimento e distribuição de água,
paisagismo e os que materializam as indicações do
Art. 3º Para fins de aplicação desta Lei, consideram-se: planejamento urbanístico;
I - acessibilidade: possibilidade e condição de alcance para VIII - mobiliário urbano: conjunto de objetos existentes nas
utilização, com segurança e autonomia, de espaços, vias e nos espaços públicos, superpostos ou adicionados aos
mobiliários, equipamentos urbanos, edificações, transportes, elementos de urbanização ou de edificação, de forma que sua
informação e comunicação, inclusive seus sistemas e modificação ou seu traslado não provoque alterações
tecnologias, bem como de outros serviços e instalações substanciais nesses elementos, tais como semáforos, postes de
abertos ao público, de uso público ou privados de uso coletivo, sinalização e similares, terminais e pontos de acesso coletivo
tanto na zona urbana como na rural, por pessoa com às telecomunicações, fontes de água, lixeiras, toldos,
deficiência ou com mobilidade reduzida; marquises, bancos, quiosques e quaisquer outros de natureza
análoga;

45 Disponível em: http://www.planalto.gov.br/ccivil_03/_Ato2015-


2018/2015/Lei/L13146.htm.

Legislação 123
Apostila Digital Licenciada para Alice Caroline Guarino dos Santos - alice.guarino@hotmail.com (Proibida a Revenda)
APOSTILAS OPÇÃO

IX - pessoa com mobilidade reduzida: aquela que tenha, IV - conservar sua fertilidade, sendo vedada a esterilização
por qualquer motivo, dificuldade de movimentação, compulsória;
permanente ou temporária, gerando redução efetiva da V - exercer o direito à família e à convivência familiar e
mobilidade, da flexibilidade, da coordenação motora ou da comunitária; e
percepção, incluindo idoso, gestante, lactante, pessoa com VI - exercer o direito à guarda, à tutela, à curatela e à
criança de colo e obeso; adoção, como adotante ou adotando, em igualdade de
X - residências inclusivas: unidades de oferta do Serviço de oportunidades com as demais pessoas.
Acolhimento do Sistema Único de Assistência Social (Suas)
localizadas em áreas residenciais da comunidade, com Art. 7º É dever de todos comunicar à autoridade
estruturas adequadas, que possam contar com apoio competente qualquer forma de ameaça ou de violação aos
psicossocial para o atendimento das necessidades da pessoa direitos da pessoa com deficiência.
acolhida, destinadas a jovens e adultos com deficiência, em Parágrafo único. Se, no exercício de suas funções, os juízes
situação de dependência, que não dispõem de condições de e os tribunais tiverem conhecimento de fatos que caracterizem
autossustentabilidade e com vínculos familiares fragilizados as violações previstas nesta Lei, devem remeter peças ao
ou rompidos; Ministério Público para as providências cabíveis.
XI - moradia para a vida independente da pessoa com
deficiência: moradia com estruturas adequadas capazes de Art. 8º É dever do Estado, da sociedade e da família
proporcionar serviços de apoio coletivos e individualizados assegurar à pessoa com deficiência, com prioridade, a
que respeitem e ampliem o grau de autonomia de jovens e efetivação dos direitos referentes à vida, à saúde, à
adultos com deficiência; sexualidade, à paternidade e à maternidade, à alimentação, à
XII - atendente pessoal: pessoa, membro ou não da família, habitação, à educação, à profissionalização, ao trabalho, à
que, com ou sem remuneração, assiste ou presta cuidados previdência social, à habilitação e à reabilitação, ao transporte,
básicos e essenciais à pessoa com deficiência no exercício de à acessibilidade, à cultura, ao desporto, ao turismo, ao lazer, à
suas atividades diárias, excluídas as técnicas ou os informação, à comunicação, aos avanços científicos e
procedimentos identificados com profissões legalmente tecnológicos, à dignidade, ao respeito, à liberdade, à
estabelecidas; convivência familiar e comunitária, entre outros decorrentes
XIII - profissional de apoio escolar: pessoa que exerce da Constituição Federal, da Convenção sobre os Direitos das
atividades de alimentação, higiene e locomoção do estudante Pessoas com Deficiência e seu Protocolo Facultativo e das leis
com deficiência e atua em todas as atividades escolares nas e de outras normas que garantam seu bem-estar pessoal,
quais se fizer necessária, em todos os níveis e modalidades de social e econômico.
ensino, em instituições públicas e privadas, excluídas as
técnicas ou os procedimentos identificados com profissões Seção Única
legalmente estabelecidas; Do Atendimento Prioritário
XIV - acompanhante: aquele que acompanha a pessoa com
deficiência, podendo ou não desempenhar as funções de Art. 9º A pessoa com deficiência tem direito a receber
atendente pessoal. atendimento prioritário, sobretudo com a finalidade de:
I - proteção e socorro em quaisquer circunstâncias;
CAPÍTULO II II - atendimento em todas as instituições e serviços de
DA IGUALDADE E DA NÃO DISCRIMINAÇÃO atendimento ao público;
III - disponibilização de recursos, tanto humanos quanto
Art. 4º Toda pessoa com deficiência tem direito à igualdade tecnológicos, que garantam atendimento em igualdade de
de oportunidades com as demais pessoas e não sofrerá condições com as demais pessoas;
nenhuma espécie de discriminação. IV - disponibilização de pontos de parada, estações e
§ 1º Considera-se discriminação em razão da deficiência terminais acessíveis de transporte coletivo de passageiros e
toda forma de distinção, restrição ou exclusão, por ação ou garantia de segurança no embarque e no desembarque;
omissão, que tenha o propósito ou o efeito de prejudicar, V - acesso a informações e disponibilização de recursos de
impedir ou anular o reconhecimento ou o exercício dos comunicação acessíveis;
direitos e das liberdades fundamentais de pessoa com VI - recebimento de restituição de imposto de renda;
deficiência, incluindo a recusa de adaptações razoáveis e de VII - tramitação processual e procedimentos judiciais e
fornecimento de tecnologias assistivas. administrativos em que for parte ou interessada, em todos os
§ 2º A pessoa com deficiência não está obrigada à fruição atos e diligências.
de benefícios decorrentes de ação afirmativa.
§ 1º Os direitos previstos neste artigo são extensivos ao
Art. 5º A pessoa com deficiência será protegida de toda acompanhante da pessoa com deficiência ou ao seu atendente
forma de negligência, discriminação, exploração, violência, pessoal, exceto quanto ao disposto nos incisos VI e VII deste
tortura, crueldade, opressão e tratamento desumano ou artigo.
degradante. § 2º Nos serviços de emergência públicos e privados, a
Parágrafo único. Para os fins da proteção mencionada no prioridade conferida por esta Lei é condicionada aos
caput deste artigo, são considerados especialmente protocolos de atendimento médico.
vulneráveis a criança, o adolescente, a mulher e o idoso, com
deficiência. TÍTULO II
DOS DIREITOS FUNDAMENTAIS
Art. 6º A deficiência não afeta a plena capacidade civil da CAPÍTULO I
pessoa, inclusive para: DO DIREITO À VIDA
I - casar-se e constituir união estável;
II - exercer direitos sexuais e reprodutivos; Art. 10. Compete ao poder público garantir a dignidade da
III - exercer o direito de decidir sobre o número de filhos e pessoa com deficiência ao longo de toda a vida.
de ter acesso a informações adequadas sobre reprodução e Parágrafo único. Em situações de risco, emergência ou
planejamento familiar; estado de calamidade pública, a pessoa com deficiência será

Legislação 124
Apostila Digital Licenciada para Alice Caroline Guarino dos Santos - alice.guarino@hotmail.com (Proibida a Revenda)
APOSTILAS OPÇÃO

considerada vulnerável, devendo o poder público adotar profissional, de acordo com as especificidades de cada pessoa
medidas para sua proteção e segurança. com deficiência;
IV - capacitação continuada de todos os profissionais que
Art. 11. A pessoa com deficiência não poderá ser obrigada participem dos programas e serviços.
a se submeter a intervenção clínica ou cirúrgica, a tratamento
ou a institucionalização forçada. Art. 17. Os serviços do SUS e do Suas deverão promover
Parágrafo único. O consentimento da pessoa com ações articuladas para garantir à pessoa com deficiência e sua
deficiência em situação de curatela poderá ser suprido, na família a aquisição de informações, orientações e formas de
forma da lei. acesso às políticas públicas disponíveis, com a finalidade de
propiciar sua plena participação social.
Art. 12. O consentimento prévio, livre e esclarecido da Parágrafo único. Os serviços de que trata o caput deste
pessoa com deficiência é indispensável para a realização de artigo podem fornecer informações e orientações nas áreas de
tratamento, procedimento, hospitalização e pesquisa saúde, de educação, de cultura, de esporte, de lazer, de
científica. transporte, de previdência social, de assistência social, de
§ 1º Em caso de pessoa com deficiência em situação de habitação, de trabalho, de empreendedorismo, de acesso ao
curatela, deve ser assegurada sua participação, no maior grau crédito, de promoção, proteção e defesa de direitos e nas
possível, para a obtenção de consentimento. demais áreas que possibilitem à pessoa com deficiência
§ 2º A pesquisa científica envolvendo pessoa com exercer sua cidadania.
deficiência em situação de tutela ou de curatela deve ser
realizada, em caráter excepcional, apenas quando houver CAPÍTULO III
indícios de benefício direto para sua saúde ou para a saúde de DO DIREITO À SAÚDE
outras pessoas com deficiência e desde que não haja outra
opção de pesquisa de eficácia comparável com participantes Art. 18. É assegurada atenção integral à saúde da pessoa
não tutelados ou curatelados. com deficiência em todos os níveis de complexidade, por
intermédio do SUS, garantido acesso universal e igualitário.
Art. 13. A pessoa com deficiência somente será atendida § 1º É assegurada a participação da pessoa com deficiência
sem seu consentimento prévio, livre e esclarecido em casos de na elaboração das políticas de saúde a ela destinadas.
risco de morte e de emergência em saúde, resguardado seu § 2º É assegurado atendimento segundo normas éticas e
superior interesse e adotadas as salvaguardas legais cabíveis. técnicas, que regulamentarão a atuação dos profissionais de
saúde e contemplarão aspectos relacionados aos direitos e às
CAPÍTULO II especificidades da pessoa com deficiência, incluindo temas
DO DIREITO À HABILITAÇÃO E À REABILITAÇÃO como sua dignidade e autonomia.
§ 3º Aos profissionais que prestam assistência à pessoa
Art. 14. O processo de habilitação e de reabilitação é um com deficiência, especialmente em serviços de habilitação e de
direito da pessoa com deficiência. reabilitação, deve ser garantida capacitação inicial e
Parágrafo único. O processo de habilitação e de continuada.
reabilitação tem por objetivo o desenvolvimento de § 4º As ações e os serviços de saúde pública destinados à
potencialidades, talentos, habilidades e aptidões físicas, pessoa com deficiência devem assegurar:
cognitivas, sensoriais, psicossociais, atitudinais, profissionais I - diagnóstico e intervenção precoces, realizados por
e artísticas que contribuam para a conquista da autonomia da equipe multidisciplinar;
pessoa com deficiência e de sua participação social em II - serviços de habilitação e de reabilitação sempre que
igualdade de condições e oportunidades com as demais necessários, para qualquer tipo de deficiência, inclusive para a
pessoas. manutenção da melhor condição de saúde e qualidade de vida;
III - atendimento domiciliar multidisciplinar, tratamento
Art. 15. O processo mencionado no art. 14 desta Lei baseia- ambulatorial e internação;
se em avaliação multidisciplinar das necessidades, habilidades IV - campanhas de vacinação;
e potencialidades de cada pessoa, observadas as seguintes V - atendimento psicológico, inclusive para seus familiares
diretrizes: e atendentes pessoais;
I - diagnóstico e intervenção precoces; VI - respeito à especificidade, à identidade de gênero e à
II - adoção de medidas para compensar perda ou limitação orientação sexual da pessoa com deficiência;
funcional, buscando o desenvolvimento de aptidões; VII - atenção sexual e reprodutiva, incluindo o direito à
III - atuação permanente, integrada e articulada de fertilização assistida;
políticas públicas que possibilitem a plena participação social VIII - informação adequada e acessível à pessoa com
da pessoa com deficiência; deficiência e a seus familiares sobre sua condição de saúde;
IV - oferta de rede de serviços articulados, com atuação IX - serviços projetados para prevenir a ocorrência e o
intersetorial, nos diferentes níveis de complexidade, para desenvolvimento de deficiências e agravos adicionais;
atender às necessidades específicas da pessoa com deficiência; X - promoção de estratégias de capacitação permanente
V - prestação de serviços próximo ao domicílio da pessoa das equipes que atuam no SUS, em todos os níveis de atenção,
com deficiência, inclusive na zona rural, respeitadas a no atendimento à pessoa com deficiência, bem como
organização das Redes de Atenção à Saúde (RAS) nos orientação a seus atendentes pessoais;
territórios locais e as normas do Sistema Único de Saúde (SUS). XI - oferta de órteses, próteses, meios auxiliares de
locomoção, medicamentos, insumos e fórmulas nutricionais,
Art. 16. Nos programas e serviços de habilitação e de conforme as normas vigentes do Ministério da Saúde.
reabilitação para a pessoa com deficiência, são garantidos: § 5º As diretrizes deste artigo aplicam-se também às
instituições privadas que participem de forma complementar
I - organização, serviços, métodos, técnicas e recursos para do SUS ou que recebam recursos públicos para sua
atender às características de cada pessoa com deficiência; manutenção.
II - acessibilidade em todos os ambientes e serviços;
III - tecnologia assistiva, tecnologia de reabilitação,
materiais e equipamentos adequados e apoio técnico

Legislação 125
Apostila Digital Licenciada para Alice Caroline Guarino dos Santos - alice.guarino@hotmail.com (Proibida a Revenda)
APOSTILAS OPÇÃO

Art. 19. Compete ao SUS desenvolver ações destinadas à CAPÍTULO IV


prevenção de deficiências por causas evitáveis, inclusive por DO DIREITO À EDUCAÇÃO
meio de:
I - acompanhamento da gravidez, do parto e do puerpério, Art. 27. A educação constitui direito da pessoa com
com garantia de parto humanizado e seguro; deficiência, assegurados sistema educacional inclusivo em
II - promoção de práticas alimentares adequadas e todos os níveis e aprendizado ao longo de toda a vida, de forma
saudáveis, vigilância alimentar e nutricional, prevenção e a alcançar o máximo desenvolvimento possível de seus
cuidado integral dos agravos relacionados à alimentação e talentos e habilidades físicas, sensoriais, intelectuais e sociais,
nutrição da mulher e da criança; segundo suas características, interesses e necessidades de
III - aprimoramento e expansão dos programas de aprendizagem.
imunização e de triagem neonatal; Parágrafo único. É dever do Estado, da família, da
IV - identificação e controle da gestante de alto risco. comunidade escolar e da sociedade assegurar educação de
qualidade à pessoa com deficiência, colocando-a a salvo de
Art. 20. As operadoras de planos e seguros privados de toda forma de violência, negligência e discriminação.
saúde são obrigadas a garantir à pessoa com deficiência, no
mínimo, todos os serviços e produtos ofertados aos demais Art. 28. Incumbe ao poder público assegurar, criar,
clientes. desenvolver, implementar, incentivar, acompanhar e avaliar:
Art. 21. Quando esgotados os meios de atenção à saúde da I - sistema educacional inclusivo em todos os níveis e
pessoa com deficiência no local de residência, será prestado modalidades, bem como o aprendizado ao longo de toda a vida;
atendimento fora de domicílio, para fins de diagnóstico e de II - aprimoramento dos sistemas educacionais, visando a
tratamento, garantidos o transporte e a acomodação da pessoa garantir condições de acesso, permanência, participação e
com deficiência e de seu acompanhante. aprendizagem, por meio da oferta de serviços e de recursos de
acessibilidade que eliminem as barreiras e promovam a
Art. 22. À pessoa com deficiência internada ou em inclusão plena;
observação é assegurado o direito a acompanhante ou a III - projeto pedagógico que institucionalize o atendimento
atendente pessoal, devendo o órgão ou a instituição de saúde educacional especializado, assim como os demais serviços e
proporcionar condições adequadas para sua permanência em adaptações razoáveis, para atender às características dos
tempo integral. estudantes com deficiência e garantir o seu pleno acesso ao
§ 1º Na impossibilidade de permanência do acompanhante currículo em condições de igualdade, promovendo a conquista
ou do atendente pessoal junto à pessoa com deficiência, cabe e o exercício de sua autonomia;
ao profissional de saúde responsável pelo tratamento justificá- IV - oferta de educação bilíngue, em Libras como primeira
la por escrito. língua e na modalidade escrita da língua portuguesa como
§ 2º Na ocorrência da impossibilidade prevista no § 1º segunda língua, em escolas e classes bilíngues e em escolas
deste artigo, o órgão ou a instituição de saúde deve adotar as inclusivas;
providências cabíveis para suprir a ausência do acompanhante
ou do atendente pessoal. V - adoção de medidas individualizadas e coletivas em
ambientes que maximizem o desenvolvimento acadêmico e
Art. 23. São vedadas todas as formas de discriminação social dos estudantes com deficiência, favorecendo o acesso, a
contra a pessoa com deficiência, inclusive por meio de permanência, a participação e a aprendizagem em instituições
cobrança de valores diferenciados por planos e seguros de ensino;
privados de saúde, em razão de sua condição. VI - pesquisas voltadas para o desenvolvimento de novos
métodos e técnicas pedagógicas, de materiais didáticos, de
Art. 24. É assegurado à pessoa com deficiência o acesso aos equipamentos e de recursos de tecnologia assistiva;
serviços de saúde, tanto públicos como privados, e às VII - planejamento de estudo de caso, de elaboração de
informações prestadas e recebidas, por meio de recursos de plano de atendimento educacional especializado, de
tecnologia assistiva e de todas as formas de comunicação organização de recursos e serviços de acessibilidade e de
previstas no inciso V do art. 3º desta Lei. disponibilização e usabilidade pedagógica de recursos de
tecnologia assistiva;
Art. 25. Os espaços dos serviços de saúde, tanto públicos VIII - participação dos estudantes com deficiência e de suas
quanto privados, devem assegurar o acesso da pessoa com famílias nas diversas instâncias de atuação da comunidade
deficiência, em conformidade com a legislação em vigor, escolar;
mediante a remoção de barreiras, por meio de projetos IX - adoção de medidas de apoio que favoreçam o
arquitetônico, de ambientação de interior e de comunicação desenvolvimento dos aspectos linguísticos, culturais,
que atendam às especificidades das pessoas com deficiência vocacionais e profissionais, levando-se em conta o talento, a
física, sensorial, intelectual e mental. criatividade, as habilidades e os interesses do estudante com
deficiência;
Art. 26. Os casos de suspeita ou de confirmação de X - adoção de práticas pedagógicas inclusivas pelos
violência praticada contra a pessoa com deficiência serão programas de formação inicial e continuada de professores e
objeto de notificação compulsória pelos serviços de saúde oferta de formação continuada para o atendimento
públicos e privados à autoridade policial e ao Ministério educacional especializado;
Público, além dos Conselhos dos Direitos da Pessoa com XI - formação e disponibilização de professores para o
Deficiência. atendimento educacional especializado, de tradutores e
Parágrafo único. Para os efeitos desta Lei, considera-se intérpretes da Libras, de guias intérpretes e de profissionais
violência contra a pessoa com deficiência qualquer ação ou de apoio;
omissão, praticada em local público ou privado, que lhe cause XII - oferta de ensino da Libras, do Sistema Braille e de uso
morte ou dano ou sofrimento físico ou psicológico. de recursos de tecnologia assistiva, de forma a ampliar
habilidades funcionais dos estudantes, promovendo sua
autonomia e participação;

Legislação 126
Apostila Digital Licenciada para Alice Caroline Guarino dos Santos - alice.guarino@hotmail.com (Proibida a Revenda)
APOSTILAS OPÇÃO

XIII - acesso à educação superior e à educação profissional CAPÍTULO V


e tecnológica em igualdade de oportunidades e condições com DO DIREITO À MORADIA
as demais pessoas;
XIV - inclusão em conteúdos curriculares, em cursos de Art. 31. A pessoa com deficiência tem direito à moradia
nível superior e de educação profissional técnica e tecnológica, digna, no seio da família natural ou substituta, com seu cônjuge
de temas relacionados à pessoa com deficiência nos ou companheiro ou desacompanhada, ou em moradia para a
respectivos campos de conhecimento; vida independente da pessoa com deficiência, ou, ainda, em
XV - acesso da pessoa com deficiência, em igualdade de residência inclusiva.
condições, a jogos e a atividades recreativas, esportivas e de § 1º O poder público adotará programas e ações
lazer, no sistema escolar; estratégicas para apoiar a criação e a manutenção de moradia
XVI - acessibilidade para todos os estudantes, para a vida independente da pessoa com deficiência.
trabalhadores da educação e demais integrantes da § 2º A proteção integral na modalidade de residência
comunidade escolar às edificações, aos ambientes e às inclusiva será prestada no âmbito do Suas à pessoa com
atividades concernentes a todas as modalidades, etapas e deficiência em situação de dependência que não disponha de
níveis de ensino; condições de autossustentabilidade, com vínculos familiares
XVII - oferta de profissionais de apoio escolar; fragilizados ou rompidos.
XVIII - articulação intersetorial na implementação de
políticas públicas. Art. 32. Nos programas habitacionais, públicos ou
§ 1º Às instituições privadas, de qualquer nível e subsidiados com recursos públicos, a pessoa com deficiência
modalidade de ensino, aplica-se obrigatoriamente o disposto ou o seu responsável goza de prioridade na aquisição de
nos incisos I, II, III, V, VII, VIII, IX, X, XI, XII, XIII, XIV, XV, XVI, imóvel para moradia própria, observado o seguinte:
XVII e XVIII do caput deste artigo, sendo vedada a cobrança de I - reserva de, no mínimo, 3% (três por cento) das unidades
valores adicionais de qualquer natureza em suas habitacionais para pessoa com deficiência;
mensalidades, anuidades e matrículas no cumprimento dessas II - (VETADO);
determinações. III - em caso de edificação multifamiliar, garantia de
§ 2º Na disponibilização de tradutores e intérpretes da acessibilidade nas áreas de uso comum e nas unidades
Libras a que se refere o inciso XI do caput deste artigo, deve-se habitacionais no piso térreo e de acessibilidade ou de
observar o seguinte: adaptação razoável nos demais pisos;
I - os tradutores e intérpretes da Libras atuantes na IV - disponibilização de equipamentos urbanos
educação básica devem, no mínimo, possuir ensino médio comunitários acessíveis;
completo e certificado de proficiência na Libras; V - elaboração de especificações técnicas no projeto que
II - os tradutores e intérpretes da Libras, quando permitam a instalação de elevadores.
direcionados à tarefa de interpretar nas salas de aula dos § 1º O direito à prioridade, previsto no caput deste artigo,
cursos de graduação e pós-graduação, devem possuir nível será reconhecido à pessoa com deficiência beneficiária apenas
superior, com habilitação, prioritariamente, em Tradução e uma vez.
Interpretação em Libras. § 2º Nos programas habitacionais públicos, os critérios de
financiamento devem ser compatíveis com os rendimentos da
Art. 29. (VETADO). pessoa com deficiência ou de sua família.
§ 3º Caso não haja pessoa com deficiência interessada nas
Art. 30. Nos processos seletivos para ingresso e unidades habitacionais reservadas por força do disposto no
permanência nos cursos oferecidos pelas instituições de inciso I do caput deste artigo, as unidades não utilizadas serão
ensino superior e de educação profissional e tecnológica, disponibilizadas às demais pessoas.
públicas e privadas, devem ser adotadas as seguintes medidas:
I - atendimento preferencial à pessoa com deficiência nas Art. 33. Ao poder público compete:
dependências das Instituições de Ensino Superior (IES) e nos I - adotar as providências necessárias para o cumprimento
serviços; do disposto nos arts. 31 e 32 desta Lei; e
II - disponibilização de formulário de inscrição de exames II - divulgar, para os agentes interessados e beneficiários, a
com campos específicos para que o candidato com deficiência política habitacional prevista nas legislações federal,
informe os recursos de acessibilidade e de tecnologia assistiva estaduais, distrital e municipais, com ênfase nos dispositivos
necessários para sua participação; sobre acessibilidade.
III - disponibilização de provas em formatos acessíveis
para atendimento às necessidades específicas do candidato CAPÍTULO VI
com deficiência; DO DIREITO AO TRABALHO
IV - disponibilização de recursos de acessibilidade e de Seção I
tecnologia assistiva adequados, previamente solicitados e Disposições Gerais
escolhidos pelo candidato com deficiência;
Art. 34. A pessoa com deficiência tem direito ao trabalho
V - dilação de tempo, conforme demanda apresentada pelo de sua livre escolha e aceitação, em ambiente acessível e
candidato com deficiência, tanto na realização de exame para inclusivo, em igualdade de oportunidades com as demais
seleção quanto nas atividades acadêmicas, mediante prévia pessoas.
solicitação e comprovação da necessidade; § 1º As pessoas jurídicas de direito público, privado ou de
VI - adoção de critérios de avaliação das provas escritas, qualquer natureza são obrigadas a garantir ambientes de
discursivas ou de redação que considerem a singularidade trabalho acessíveis e inclusivos.
linguística da pessoa com deficiência, no domínio da § 2º A pessoa com deficiência tem direito, em igualdade de
modalidade escrita da língua portuguesa; oportunidades com as demais pessoas, a condições justas e
VII - tradução completa do edital e de suas retificações em favoráveis de trabalho, incluindo igual remuneração por
Libras. trabalho de igual valor.
§ 3º É vedada restrição ao trabalho da pessoa com
deficiência e qualquer discriminação em razão de sua
condição, inclusive nas etapas de recrutamento, seleção,

Legislação 127
Apostila Digital Licenciada para Alice Caroline Guarino dos Santos - alice.guarino@hotmail.com (Proibida a Revenda)
APOSTILAS OPÇÃO

contratação, admissão, exames admissional e periódico, Seção III


permanência no emprego, ascensão profissional e reabilitação Da Inclusão da Pessoa com Deficiência no Trabalho
profissional, bem como exigência de aptidão plena.
Art. 37. Constitui modo de inclusão da pessoa com
§ 4º A pessoa com deficiência tem direito à participação e deficiência no trabalho a colocação competitiva, em igualdade
ao acesso a cursos, treinamentos, educação continuada, planos de oportunidades com as demais pessoas, nos termos da
de carreira, promoções, bonificações e incentivos profissionais legislação trabalhista e previdenciária, na qual devem ser
oferecidos pelo empregador, em igualdade de oportunidades atendidas as regras de acessibilidade, o fornecimento de
com os demais empregados. recursos de tecnologia assistiva e a adaptação razoável no
§ 5º É garantida aos trabalhadores com deficiência ambiente de trabalho.
acessibilidade em cursos de formação e de capacitação. Parágrafo único. A colocação competitiva da pessoa com
deficiência pode ocorrer por meio de trabalho com apoio,
Art. 35. É finalidade primordial das políticas públicas de observadas as seguintes diretrizes:
trabalho e emprego promover e garantir condições de acesso I - prioridade no atendimento à pessoa com deficiência
e de permanência da pessoa com deficiência no campo de com maior dificuldade de inserção no campo de trabalho;
trabalho. II - provisão de suportes individualizados que atendam a
Parágrafo único. Os programas de estímulo ao necessidades específicas da pessoa com deficiência, inclusive
empreendedorismo e ao trabalho autônomo, incluídos o a disponibilização de recursos de tecnologia assistiva, de
cooperativismo e o associativismo, devem prever a agente facilitador e de apoio no ambiente de trabalho;
participação da pessoa com deficiência e a disponibilização de III - respeito ao perfil vocacional e ao interesse da pessoa
linhas de crédito, quando necessárias. com deficiência apoiada;
IV - oferta de aconselhamento e de apoio aos
Seção II empregadores, com vistas à definição de estratégias de
Da Habilitação Profissional e Reabilitação inclusão e de superação de barreiras, inclusive atitudinais;
Profissional V - realização de avaliações periódicas;
VI - articulação intersetorial das políticas públicas;
Art. 36. O poder público deve implementar serviços e
programas completos de habilitação profissional e de VII - possibilidade de participação de organizações da
reabilitação profissional para que a pessoa com deficiência sociedade civil.
possa ingressar, continuar ou retornar ao campo do trabalho,
respeitados sua livre escolha, sua vocação e seu interesse. Art. 38. A entidade contratada para a realização de
§ 1º Equipe multidisciplinar indicará, com base em processo seletivo público ou privado para cargo, função ou
critérios previstos no § 1º do art. 2º desta Lei, programa de emprego está obrigada à observância do disposto nesta Lei e
habilitação ou de reabilitação que possibilite à pessoa com em outras normas de acessibilidade vigentes.
deficiência restaurar sua capacidade e habilidade profissional
ou adquirir novas capacidades e habilidades de trabalho. CAPÍTULO VII
§ 2º A habilitação profissional corresponde ao processo DO DIREITO À ASSISTÊNCIA SOCIAL
destinado a propiciar à pessoa com deficiência aquisição de
conhecimentos, habilidades e aptidões para exercício de Art. 39. Os serviços, os programas, os projetos e os
profissão ou de ocupação, permitindo nível suficiente de benefícios no âmbito da política pública de assistência social à
desenvolvimento profissional para ingresso no campo de pessoa com deficiência e sua família têm como objetivo a
trabalho. garantia da segurança de renda, da acolhida, da habilitação e
§ 3º Os serviços de habilitação profissional, de reabilitação da reabilitação, do desenvolvimento da autonomia e da
profissional e de educação profissional devem ser dotados de convivência familiar e comunitária, para a promoção do acesso
recursos necessários para atender a toda pessoa com a direitos e da plena participação social.
deficiência, independentemente de sua característica § 1º A assistência social à pessoa com deficiência, nos
específica, a fim de que ela possa ser capacitada para trabalho termos do caput deste artigo, deve envolver conjunto
que lhe seja adequado e ter perspectivas de obtê-lo, de articulado de serviços do âmbito da Proteção Social Básica e
conservá-lo e de nele progredir. da Proteção Social Especial, ofertados pelo Suas, para a
§ 4º Os serviços de habilitação profissional, de reabilitação garantia de seguranças fundamentais no enfrentamento de
profissional e de educação profissional deverão ser oferecidos situações de vulnerabilidade e de risco, por fragilização de
em ambientes acessíveis e inclusivos. vínculos e ameaça ou violação de direitos.
§ 5º A habilitação profissional e a reabilitação profissional § 2º Os serviços socioassistenciais destinados à pessoa
devem ocorrer articuladas com as redes públicas e privadas, com deficiência em situação de dependência deverão contar
especialmente de saúde, de ensino e de assistência social, em com cuidadores sociais para prestar-lhe cuidados básicos e
todos os níveis e modalidades, em entidades de formação instrumentais.
profissional ou diretamente com o empregador.
§ 6º A habilitação profissional pode ocorrer em empresas Art. 40. É assegurado à pessoa com deficiência que não
por meio de prévia formalização do contrato de emprego da possua meios para prover sua subsistência nem de tê-la
pessoa com deficiência, que será considerada para o provida por sua família o benefício mensal de 1 (um) salário-
cumprimento da reserva de vagas prevista em lei, desde que mínimo, nos termos da Lei no 8.742, de 7 de dezembro de
por tempo determinado e concomitante com a inclusão 1993.
profissional na empresa, observado o disposto em
regulamento. CAPÍTULO VIII
§ 7º A habilitação profissional e a reabilitação profissional DO DIREITO À PREVIDÊNCIA SOCIAL
atenderão à pessoa com deficiência.
Art. 41. A pessoa com deficiência segurada do Regime Geral
de Previdência Social (RGPS) tem direito à aposentadoria nos
termos da Lei Complementar no 142, de 8 de maio de 2013.

Legislação 128
Apostila Digital Licenciada para Alice Caroline Guarino dos Santos - alice.guarino@hotmail.com (Proibida a Revenda)
APOSTILAS OPÇÃO

CAPÍTULO IX § 7º O valor do ingresso da pessoa com deficiência não


DO DIREITO À CULTURA, AO ESPORTE, AO TURISMO E poderá ser superior ao valor cobrado das demais pessoas.
AO LAZER
Art. 45. Os hotéis, pousadas e similares devem ser
Art. 42. A pessoa com deficiência tem direito à cultura, ao construídos observando-se os princípios do desenho
esporte, ao turismo e ao lazer em igualdade de oportunidades universal, além de adotar todos os meios de acessibilidade,
com as demais pessoas, sendo-lhe garantido o acesso: conforme legislação em vigor. (Vigência)
I - a bens culturais em formato acessível; § 1º Os estabelecimentos já existentes deverão
II - a programas de televisão, cinema, teatro e outras disponibilizar, pelo menos, 10% (dez por cento) de seus
atividades culturais e desportivas em formato acessível; e dormitórios acessíveis, garantida, no mínimo, 1 (uma) unidade
III - a monumentos e locais de importância cultural e a acessível.
espaços que ofereçam serviços ou eventos culturais e § 2º Os dormitórios mencionados no § 1º deste artigo
esportivos. deverão ser localizados em rotas acessíveis.
§ 1º É vedada a recusa de oferta de obra intelectual em
formato acessível à pessoa com deficiência, sob qualquer CAPÍTULO X
argumento, inclusive sob a alegação de proteção dos direitos DO DIREITO AO TRANSPORTE E À MOBILIDADE
de propriedade intelectual.
§ 2º O poder público deve adotar soluções destinadas à Art. 46. O direito ao transporte e à mobilidade da pessoa
eliminação, à redução ou à superação de barreiras para a com deficiência ou com mobilidade reduzida será assegurado
promoção do acesso a todo patrimônio cultural, observadas as em igualdade de oportunidades com as demais pessoas, por
normas de acessibilidade, ambientais e de proteção do meio de identificação e de eliminação de todos os obstáculos e
patrimônio histórico e artístico nacional. barreiras ao seu acesso.
§ 1º Para fins de acessibilidade aos serviços de transporte
Art. 43. O poder público deve promover a participação da coletivo terrestre, aquaviário e aéreo, em todas as jurisdições,
pessoa com deficiência em atividades artísticas, intelectuais, consideram-se como integrantes desses serviços os veículos,
culturais, esportivas e recreativas, com vistas ao seu os terminais, as estações, os pontos de parada, o sistema viário
protagonismo, devendo: e a prestação do serviço.
I - incentivar a provisão de instrução, de treinamento e de § 2º São sujeitas ao cumprimento das disposições desta
recursos adequados, em igualdade de oportunidades com as Lei, sempre que houver interação com a matéria nela regulada,
demais pessoas; a outorga, a concessão, a permissão, a autorização, a renovação
II - assegurar acessibilidade nos locais de eventos e nos ou a habilitação de linhas e de serviços de transporte coletivo.
serviços prestados por pessoa ou entidade envolvida na § 3º Para colocação do símbolo internacional de acesso nos
organização das atividades de que trata este artigo; e veículos, as empresas de transporte coletivo de passageiros
III - assegurar a participação da pessoa com deficiência em dependem da certificação de acessibilidade emitida pelo
jogos e atividades recreativas, esportivas, de lazer, culturais e gestor público responsável pela prestação do serviço.
artísticas, inclusive no sistema escolar, em igualdade de
condições com as demais pessoas. Art. 47. Em todas as áreas de estacionamento aberto ao
público, de uso público ou privado de uso coletivo e em vias
Art. 44. Nos teatros, cinemas, auditórios, estádios, ginásios públicas, devem ser reservadas vagas próximas aos acessos de
de esporte, locais de espetáculos e de conferências e similares, circulação de pedestres, devidamente sinalizadas, para
serão reservados espaços livres e assentos para a pessoa com veículos que transportem pessoa com deficiência com
deficiência, de acordo com a capacidade de lotação da comprometimento de mobilidade, desde que devidamente
edificação, observado o disposto em regulamento. identificados.
§ 1º Os espaços e assentos a que se refere este artigo § 1º As vagas a que se refere o caput deste artigo devem
devem ser distribuídos pelo recinto em locais diversos, de boa equivaler a 2% (dois por cento) do total, garantida, no mínimo,
visibilidade, em todos os setores, próximos aos corredores, 1 (uma) vaga devidamente sinalizada e com as especificações
devidamente sinalizados, evitando-se áreas segregadas de de desenho e traçado de acordo com as normas técnicas
público e obstrução das saídas, em conformidade com as vigentes de acessibilidade.
normas de acessibilidade. § 2º Os veículos estacionados nas vagas reservadas devem
§ 2º No caso de não haver comprovada procura pelos exibir, em local de ampla visibilidade, a credencial de
assentos reservados, esses podem, excepcionalmente, ser beneficiário, a ser confeccionada e fornecida pelos órgãos de
ocupados por pessoas sem deficiência ou que não tenham trânsito, que disciplinarão suas características e condições de
mobilidade reduzida, observado o disposto em regulamento. uso.
§ 3º Os espaços e assentos a que se refere este artigo § 3º A utilização indevida das vagas de que trata este artigo
devem situar-se em locais que garantam a acomodação de, no sujeita os infratores às sanções previstas no inciso XX do art.
mínimo, 1 (um) acompanhante da pessoa com deficiência ou 181 da Lei nº 9.503, de 23 de setembro de 1997 (Código de
com mobilidade reduzida, resguardado o direito de se Trânsito Brasileiro). (Redação dada pela Lei nº 13.281, de
acomodar proximamente a grupo familiar e comunitário. 2016)
§ 4º Nos locais referidos no caput deste artigo, deve haver, § 4º A credencial a que se refere o § 2º deste artigo é
obrigatoriamente, rotas de fuga e saídas de emergência vinculada à pessoa com deficiência que possui
acessíveis, conforme padrões das normas de acessibilidade, a comprometimento de mobilidade e é válida em todo o
fim de permitir a saída segura da pessoa com deficiência ou território nacional.
com mobilidade reduzida, em caso de emergência.
§ 5º Todos os espaços das edificações previstas no caput Art. 48. Os veículos de transporte coletivo terrestre,
deste artigo devem atender às normas de acessibilidade em aquaviário e aéreo, as instalações, as estações, os portos e os
vigor. terminais em operação no País devem ser acessíveis, de forma
§ 6º As salas de cinema devem oferecer, em todas as a garantir o seu uso por todas as pessoas.
sessões, recursos de acessibilidade para a pessoa com § 1º Os veículos e as estruturas de que trata o caput deste
deficiência. artigo devem dispor de sistema de comunicação acessível que
disponibilize informações sobre todos os pontos do itinerário.

Legislação 129
Apostila Digital Licenciada para Alice Caroline Guarino dos Santos - alice.guarino@hotmail.com (Proibida a Revenda)
APOSTILAS OPÇÃO

§ 2º São asseguradas à pessoa com deficiência prioridade rural, devem atender aos princípios do desenho universal,
e segurança nos procedimentos de embarque e de tendo como referência as normas de acessibilidade.
desembarque nos veículos de transporte coletivo, de acordo § 1º O desenho universal será sempre tomado como regra
com as normas técnicas. de caráter geral.
§ 3º Para colocação do símbolo internacional de acesso nos § 2º Nas hipóteses em que comprovadamente o desenho
veículos, as empresas de transporte coletivo de passageiros universal não possa ser empreendido, deve ser adotada
dependem da certificação de acessibilidade emitida pelo adaptação razoável.
gestor público responsável pela prestação do serviço. § 3º Caberá ao poder público promover a inclusão de
conteúdos temáticos referentes ao desenho universal nas
Art. 49. As empresas de transporte de fretamento e de diretrizes curriculares da educação profissional e tecnológica
turismo, na renovação de suas frotas, são obrigadas ao e do ensino superior e na formação das carreiras de Estado.
cumprimento do disposto nos arts. 46 e 48 desta Lei. § 4º Os programas, os projetos e as linhas de pesquisa a
(Vigência) serem desenvolvidos com o apoio de organismos públicos de
auxílio à pesquisa e de agências de fomento deverão incluir
Art. 50. O poder público incentivará a fabricação de temas voltados para o desenho universal.
veículos acessíveis e a sua utilização como táxis e vans, de § 5º Desde a etapa de concepção, as políticas públicas
forma a garantir o seu uso por todas as pessoas. deverão considerar a adoção do desenho universal.

Art. 51. As frotas de empresas de táxi devem reservar 10% Art. 56. A construção, a reforma, a ampliação ou a mudança
(dez por cento) de seus veículos acessíveis à pessoa com de uso de edificações abertas ao público, de uso público ou
deficiência. privadas de uso coletivo deverão ser executadas de modo a
§ 1º É proibida a cobrança diferenciada de tarifas ou de serem acessíveis.
valores adicionais pelo serviço de táxi prestado à pessoa com § 1º As entidades de fiscalização profissional das
deficiência. atividades de Engenharia, de Arquitetura e correlatas, ao
§ 2º O poder público é autorizado a instituir incentivos anotarem a responsabilidade técnica de projetos, devem exigir
fiscais com vistas a possibilitar a acessibilidade dos veículos a a responsabilidade profissional declarada de atendimento às
que se refere o caput deste artigo. regras de acessibilidade previstas em legislação e em normas
técnicas pertinentes.
Art. 52. As locadoras de veículos são obrigadas a oferecer § 2º Para a aprovação, o licenciamento ou a emissão de
1 (um) veículo adaptado para uso de pessoa com deficiência, a certificado de projeto executivo arquitetônico, urbanístico e de
cada conjunto de 20 (vinte) veículos de sua frota. instalações e equipamentos temporários ou permanentes e
Parágrafo único. O veículo adaptado deverá ter, no mínimo, para o licenciamento ou a emissão de certificado de conclusão
câmbio automático, direção hidráulica, vidros elétricos e de obra ou de serviço, deve ser atestado o atendimento às
comandos manuais de freio e de embreagem. regras de acessibilidade.
§ 3º O poder público, após certificar a acessibilidade de
TÍTULO III edificação ou de serviço, determinará a colocação, em espaços
DA ACESSIBILIDADE ou em locais de ampla visibilidade, do símbolo internacional
CAPÍTULO I de acesso, na forma prevista em legislação e em normas
DISPOSIÇÕES GERAIS técnicas correlatas.

Art. 53. A acessibilidade é direito que garante à pessoa com Art. 57. As edificações públicas e privadas de uso coletivo
deficiência ou com mobilidade reduzida viver de forma já existentes devem garantir acessibilidade à pessoa com
independente e exercer seus direitos de cidadania e de deficiência em todas as suas dependências e serviços, tendo
participação social. como referência as normas de acessibilidade vigentes.

Art. 54. São sujeitas ao cumprimento das disposições desta Art. 58. O projeto e a construção de edificação de uso
Lei e de outras normas relativas à acessibilidade, sempre que privado multifamiliar devem atender aos preceitos de
houver interação com a matéria nela regulada: acessibilidade, na forma regulamentar.
I - a aprovação de projeto arquitetônico e urbanístico ou de § 1º As construtoras e incorporadoras responsáveis pelo
comunicação e informação, a fabricação de veículos de projeto e pela construção das edificações a que se refere o
transporte coletivo, a prestação do respectivo serviço e a caput deste artigo devem assegurar percentual mínimo de
execução de qualquer tipo de obra, quando tenham destinação suas unidades internamente acessíveis, na forma
pública ou coletiva; regulamentar.
II - a outorga ou a renovação de concessão, permissão, § 2º É vedada a cobrança de valores adicionais para a
autorização ou habilitação de qualquer natureza; aquisição de unidades internamente acessíveis a que se refere
III - a aprovação de financiamento de projeto com o § 1º deste artigo.
utilização de recursos públicos, por meio de renúncia ou de
incentivo fiscal, contrato, convênio ou instrumento congênere; Art. 59. Em qualquer intervenção nas vias e nos espaços
e públicos, o poder público e as empresas concessionárias
IV - a concessão de aval da União para obtenção de responsáveis pela execução das obras e dos serviços devem
empréstimo e de financiamento internacionais por entes garantir, de forma segura, a fluidez do trânsito e a livre
públicos ou privados. circulação e acessibilidade das pessoas, durante e após sua
execução.
Art. 55. A concepção e a implantação de projetos que
tratem do meio físico, de transporte, de informação e Art. 60. Orientam-se, no que couber, pelas regras de
comunicação, inclusive de sistemas e tecnologias da acessibilidade previstas em legislação e em normas técnicas,
informação e comunicação, e de outros serviços, observado o disposto na Lei no 10.098, de 19 de dezembro de
equipamentos e instalações abertos ao público, de uso público 2000, no 10.257, de 10 de julho de 2001, e no 12.587, de 3 de
ou privado de uso coletivo, tanto na zona urbana como na janeiro de 2012:

Legislação 130
Apostila Digital Licenciada para Alice Caroline Guarino dos Santos - alice.guarino@hotmail.com (Proibida a Revenda)
APOSTILAS OPÇÃO

I - os planos diretores municipais, os planos diretores de II - janela com intérprete da Libras;


transporte e trânsito, os planos de mobilidade urbana e os III - audiodescrição.
planos de preservação de sítios históricos elaborados ou
atualizados a partir da publicação desta Lei; Art. 68. O poder público deve adotar mecanismos de
II - os códigos de obras, os códigos de postura, as leis de incentivo à produção, à edição, à difusão, à distribuição e à
uso e ocupação do solo e as leis do sistema viário; comercialização de livros em formatos acessíveis, inclusive em
III - os estudos prévios de impacto de vizinhança; publicações da administração pública ou financiadas com
IV - as atividades de fiscalização e a imposição de sanções; recursos públicos, com vistas a garantir à pessoa com
e deficiência o direito de acesso à leitura, à informação e à
V - a legislação referente à prevenção contra incêndio e comunicação.
pânico. § 1º Nos editais de compras de livros, inclusive para o
§ 1º A concessão e a renovação de alvará de funcionamento abastecimento ou a atualização de acervos de bibliotecas em
para qualquer atividade são condicionadas à observação e à todos os níveis e modalidades de educação e de bibliotecas
certificação das regras de acessibilidade. públicas, o poder público deverá adotar cláusulas de
§ 2º A emissão de carta de habite-se ou de habilitação impedimento à participação de editoras que não ofertem sua
equivalente e sua renovação, quando esta tiver sido emitida produção também em formatos acessíveis.
anteriormente às exigências de acessibilidade, é condicionada § 2º Consideram-se formatos acessíveis os arquivos
à observação e à certificação das regras de acessibilidade. digitais que possam ser reconhecidos e acessados por
softwares leitores de telas ou outras tecnologias assistivas que
Art. 61. A formulação, a implementação e a manutenção vierem a substituí-los, permitindo leitura com voz sintetizada,
das ações de acessibilidade atenderão às seguintes premissas ampliação de caracteres, diferentes contrastes e impressão em
básicas: Braille.
§ 3º O poder público deve estimular e apoiar a adaptação e
I - eleição de prioridades, elaboração de cronograma e a produção de artigos científicos em formato acessível,
reserva de recursos para implementação das ações; e inclusive em Libras.
II - planejamento contínuo e articulado entre os setores
envolvidos. Art. 69. O poder público deve assegurar a disponibilidade
de informações corretas e claras sobre os diferentes produtos
Art. 62. É assegurado à pessoa com deficiência, mediante e serviços ofertados, por quaisquer meios de comunicação
solicitação, o recebimento de contas, boletos, recibos, extratos empregados, inclusive em ambiente virtual, contendo a
e cobranças de tributos em formato acessível. especificação correta de quantidade, qualidade,
características, composição e preço, bem como sobre os
CAPÍTULO II eventuais riscos à saúde e à segurança do consumidor com
DO ACESSO À INFORMAÇÃO E À COMUNICAÇÃO deficiência, em caso de sua utilização, aplicando-se, no que
couber, os arts. 30 a 41 da Lei no 8.078, de 11 de setembro de
Art. 63. É obrigatória a acessibilidade nos sítios da internet 1990.
mantidos por empresas com sede ou representação comercial § 1º Os canais de comercialização virtual e os anúncios
no País ou por órgãos de governo, para uso da pessoa com publicitários veiculados na imprensa escrita, na internet, no
deficiência, garantindo-lhe acesso às informações disponíveis, rádio, na televisão e nos demais veículos de comunicação
conforme as melhores práticas e diretrizes de acessibilidade abertos ou por assinatura devem disponibilizar, conforme a
adotadas internacionalmente. compatibilidade do meio, os recursos de acessibilidade de que
§ 1º Os sítios devem conter símbolo de acessibilidade em trata o art. 67 desta Lei, a expensas do fornecedor do produto
destaque. ou do serviço, sem prejuízo da observância do disposto nos
§ 2º Telecentros comunitários que receberem recursos arts. 36 a 38 da Lei no 8.078, de 11 de setembro de 1990.
públicos federais para seu custeio ou sua instalação e lan § 2º Os fornecedores devem disponibilizar, mediante
houses devem possuir equipamentos e instalações acessíveis. solicitação, exemplares de bulas, prospectos, textos ou
§ 3º Os telecentros e as lan houses de que trata o § 2º deste qualquer outro tipo de material de divulgação em formato
artigo devem garantir, no mínimo, 10% (dez por cento) de acessível.
seus computadores com recursos de acessibilidade para
pessoa com deficiência visual, sendo assegurado pelo menos 1 Art. 70. As instituições promotoras de congressos,
(um) equipamento, quando o resultado percentual for inferior seminários, oficinas e demais eventos de natureza científico-
a 1 (um). cultural devem oferecer à pessoa com deficiência, no mínimo,
os recursos de tecnologia assistiva previstos no art. 67 desta
Art. 64. A acessibilidade nos sítios da internet de que trata Lei.
o art. 63 desta Lei deve ser observada para obtenção do
financiamento de que trata o inciso III do art. 54 desta Lei. Art. 71. Os congressos, os seminários, as oficinas e os
demais eventos de natureza científico-cultural promovidos ou
Art. 65. As empresas prestadoras de serviços de financiados pelo poder público devem garantir as condições de
telecomunicações deverão garantir pleno acesso à pessoa com acessibilidade e os recursos de tecnologia assistiva.
deficiência, conforme regulamentação específica.
Art. 72. Os programas, as linhas de pesquisa e os projetos a
Art. 66. Cabe ao poder público incentivar a oferta de serem desenvolvidos com o apoio de agências de
aparelhos de telefonia fixa e móvel celular com acessibilidade financiamento e de órgãos e entidades integrantes da
que, entre outras tecnologias assistivas, possuam administração pública que atuem no auxílio à pesquisa devem
possibilidade de indicação e de ampliação sonoras de todas as contemplar temas voltados à tecnologia assistiva.
operações e funções disponíveis.
Art. 73. Caberá ao poder público, diretamente ou em
Art. 67. Os serviços de radiodifusão de sons e imagens parceria com organizações da sociedade civil, promover a
devem permitir o uso dos seguintes recursos, entre outros: capacitação de tradutores e intérpretes da Libras, de guias
I - subtitulação por meio de legenda oculta;

Legislação 131
Apostila Digital Licenciada para Alice Caroline Guarino dos Santos - alice.guarino@hotmail.com (Proibida a Revenda)
APOSTILAS OPÇÃO

intérpretes e de profissionais habilitados em Braille, II - formação de organizações para representar a pessoa


audiodescrição, estenotipia e legendagem. com deficiência em todos os níveis;
III - participação da pessoa com deficiência em
CAPÍTULO III organizações que a representem.
DA TECNOLOGIA ASSISTIVA
TÍTULO IV
Art. 74. É garantido à pessoa com deficiência acesso a DA CIÊNCIA E TECNOLOGIA
produtos, recursos, estratégias, práticas, processos, métodos e
serviços de tecnologia assistiva que maximizem sua Art. 77. O poder público deve fomentar o desenvolvimento
autonomia, mobilidade pessoal e qualidade de vida. científico, a pesquisa e a inovação e a capacitação tecnológicas,
voltados à melhoria da qualidade de vida e ao trabalho da
Art. 75. O poder público desenvolverá plano específico de pessoa com deficiência e sua inclusão social.
medidas, a ser renovado em cada período de 4 (quatro) anos, § 1º O fomento pelo poder público deve priorizar a geração
com a finalidade de: de conhecimentos e técnicas que visem à prevenção e ao
I - facilitar o acesso a crédito especializado, inclusive com tratamento de deficiências e ao desenvolvimento de
oferta de linhas de crédito subsidiadas, específicas para tecnologias assistiva e social.
aquisição de tecnologia assistiva; § 2º A acessibilidade e as tecnologias assistiva e social
II - agilizar, simplificar e priorizar procedimentos de devem ser fomentadas mediante a criação de cursos de pós-
importação de tecnologia assistiva, especialmente as questões graduação, a formação de recursos humanos e a inclusão do
atinentes a procedimentos alfandegários e sanitários; tema nas diretrizes de áreas do conhecimento.
III - criar mecanismos de fomento à pesquisa e à produção § 3º Deve ser fomentada a capacitação tecnológica de
nacional de tecnologia assistiva, inclusive por meio de instituições públicas e privadas para o desenvolvimento de
concessão de linhas de crédito subsidiado e de parcerias com tecnologias assistiva e social que sejam voltadas para melhoria
institutos de pesquisa oficiais; da funcionalidade e da participação social da pessoa com
IV - eliminar ou reduzir a tributação da cadeia produtiva e deficiência.
de importação de tecnologia assistiva; § 4º As medidas previstas neste artigo devem ser
V - facilitar e agilizar o processo de inclusão de novos reavaliadas periodicamente pelo poder público, com vistas ao
recursos de tecnologia assistiva no rol de produtos seu aperfeiçoamento.
distribuídos no âmbito do SUS e por outros órgãos
governamentais. Art. 78. Devem ser estimulados a pesquisa, o
Parágrafo único. Para fazer cumprir o disposto neste desenvolvimento, a inovação e a difusão de tecnologias
artigo, os procedimentos constantes do plano específico de voltadas para ampliar o acesso da pessoa com deficiência às
medidas deverão ser avaliados, pelo menos, a cada 2 (dois) tecnologias da informação e comunicação e às tecnologias
anos. sociais.
Parágrafo único. Serão estimulados, em especial:
CAPÍTULO IV I - o emprego de tecnologias da informação e comunicação
DO DIREITO À PARTICIPAÇÃO NA VIDA PÚBLICA E como instrumento de superação de limitações funcionais e de
POLÍTICA barreiras à comunicação, à informação, à educação e ao
entretenimento da pessoa com deficiência;
Art. 76. O poder público deve garantir à pessoa com II - a adoção de soluções e a difusão de normas que visem
deficiência todos os direitos políticos e a oportunidade de a ampliar a acessibilidade da pessoa com deficiência à
exercê-los em igualdade de condições com as demais pessoas. computação e aos sítios da internet, em especial aos serviços
§ 1º À pessoa com deficiência será assegurado o direito de de governo eletrônico.
votar e de ser votada, inclusive por meio das seguintes ações:
I - garantia de que os procedimentos, as instalações, os LIVRO II
materiais e os equipamentos para votação sejam apropriados, PARTE ESPECIAL
acessíveis a todas as pessoas e de fácil compreensão e uso, TÍTULO I
sendo vedada a instalação de seções eleitorais exclusivas para DO ACESSO À JUSTIÇA
a pessoa com deficiência; CAPÍTULO I
II - incentivo à pessoa com deficiência a candidatar-se e a DISPOSIÇÕES GERAIS
desempenhar quaisquer funções públicas em todos os níveis
de governo, inclusive por meio do uso de novas tecnologias Art. 79. O poder público deve assegurar o acesso da pessoa
assistivas, quando apropriado; com deficiência à justiça, em igualdade de oportunidades com
as demais pessoas, garantindo, sempre que requeridos,
III - garantia de que os pronunciamentos oficiais, a adaptações e recursos de tecnologia assistiva.
propaganda eleitoral obrigatória e os debates transmitidos § 1º A fim de garantir a atuação da pessoa com deficiência
pelas emissoras de televisão possuam, pelo menos, os recursos em todo o processo judicial, o poder público deve capacitar os
elencados no art. 67 desta Lei; membros e os servidores que atuam no Poder Judiciário, no
IV - garantia do livre exercício do direito ao voto e, para Ministério Público, na Defensoria Pública, nos órgãos de
tanto, sempre que necessário e a seu pedido, permissão para segurança pública e no sistema penitenciário quanto aos
que a pessoa com deficiência seja auxiliada na votação por direitos da pessoa com deficiência.
pessoa de sua escolha. § 2º Devem ser assegurados à pessoa com deficiência
§ 2º O poder público promoverá a participação da pessoa submetida a medida restritiva de liberdade todos os direitos e
com deficiência, inclusive quando institucionalizada, na garantias a que fazem jus os apenados sem deficiência,
condução das questões públicas, sem discriminação e em garantida a acessibilidade.
igualdade de oportunidades, observado o seguinte: § 3º A Defensoria Pública e o Ministério Público tomarão
I - participação em organizações não governamentais as medidas necessárias à garantia dos direitos previstos nesta
relacionadas à vida pública e à política do País e em atividades Lei.
e administração de partidos políticos;

Legislação 132
Apostila Digital Licenciada para Alice Caroline Guarino dos Santos - alice.guarino@hotmail.com (Proibida a Revenda)
APOSTILAS OPÇÃO

Art. 80. Devem ser oferecidos todos os recursos de § 1º Aumenta-se a pena em 1/3 (um terço) se a vítima
tecnologia assistiva disponíveis para que a pessoa com encontrar-se sob cuidado e responsabilidade do agente.
deficiência tenha garantido o acesso à justiça, sempre que § 2º Se qualquer dos crimes previstos no caput deste artigo
figure em um dos polos da ação ou atue como testemunha, é cometido por intermédio de meios de comunicação social ou
partícipe da lide posta em juízo, advogado, defensor público, de publicação de qualquer natureza:
magistrado ou membro do Ministério Público. Pena - reclusão, de 2 (dois) a 5 (cinco) anos, e multa.
Parágrafo único. A pessoa com deficiência tem garantido o
acesso ao conteúdo de todos os atos processuais de seu § 3º Na hipótese do § 2º deste artigo, o juiz poderá
interesse, inclusive no exercício da advocacia. determinar, ouvido o Ministério Público ou a pedido deste,
ainda antes do inquérito policial, sob pena de desobediência:
Art. 81. Os direitos da pessoa com deficiência serão I - recolhimento ou busca e apreensão dos exemplares do
garantidos por ocasião da aplicação de sanções penais. material discriminatório;
II - interdição das respectivas mensagens ou páginas de
Art. 82. (VETADO). informação na internet.
§ 4º Na hipótese do § 2º deste artigo, constitui efeito da
Art. 83. Os serviços notariais e de registro não podem condenação, após o trânsito em julgado da decisão, a
negar ou criar óbices ou condições diferenciadas à prestação destruição do material apreendido.
de seus serviços em razão de deficiência do solicitante,
devendo reconhecer sua capacidade legal plena, garantida a Art. 89. Apropriar-se de ou desviar bens, proventos,
acessibilidade. pensão, benefícios, remuneração ou qualquer outro
Parágrafo único. O descumprimento do disposto no caput rendimento de pessoa com deficiência:
deste artigo constitui discriminação em razão de deficiência. Pena - reclusão, de 1 (um) a 4 (quatro) anos, e multa.
Parágrafo único. Aumenta-se a pena em 1/3 (um terço) se
CAPÍTULO II o crime é cometido:
DO RECONHECIMENTO IGUAL PERANTE A LEI I - por tutor, curador, síndico, liquidatário, inventariante,
testamenteiro ou depositário judicial; ou
Art. 84. A pessoa com deficiência tem assegurado o direito II - por aquele que se apropriou em razão de ofício ou de
ao exercício de sua capacidade legal em igualdade de profissão.
condições com as demais pessoas.
§ 1º Quando necessário, a pessoa com deficiência será Art. 90. Abandonar pessoa com deficiência em hospitais,
submetida à curatela, conforme a lei. casas de saúde, entidades de abrigamento ou congêneres:
§ 2º É facultado à pessoa com deficiência a adoção de Pena - reclusão, de 6 (seis) meses a 3 (três) anos, e multa.
processo de tomada de decisão apoiada. Parágrafo único. Na mesma pena incorre quem não prover
§ 3º A definição de curatela de pessoa com deficiência as necessidades básicas de pessoa com deficiência quando
constitui medida protetiva extraordinária, proporcional às obrigado por lei ou mandado.
necessidades e às circunstâncias de cada caso, e durará o
menor tempo possível. Art. 91. Reter ou utilizar cartão magnético, qualquer meio
§ 4º Os curadores são obrigados a prestar, anualmente, eletrônico ou documento de pessoa com deficiência
contas de sua administração ao juiz, apresentando o balanço destinados ao recebimento de benefícios, proventos, pensões
do respectivo ano. ou remuneração ou à realização de operações financeiras, com
o fim de obter vantagem indevida para si ou para outrem:
Art. 85. A curatela afetará tão somente os atos relacionados Pena - detenção, de 6 (seis) meses a 2 (dois) anos, e multa.
aos direitos de natureza patrimonial e negocial. Parágrafo único. Aumenta-se a pena em 1/3 (um terço) se
§ 1º A definição da curatela não alcança o direito ao o crime é cometido por tutor ou curador.
próprio corpo, à sexualidade, ao matrimônio, à privacidade, à
educação, à saúde, ao trabalho e ao voto. TÍTULO III
§ 2º A curatela constitui medida extraordinária, devendo DISPOSIÇÕES FINAIS E TRANSITÓRIAS
constar da sentença as razões e motivações de sua definição,
preservados os interesses do curatelado. Art. 92. É criado o Cadastro Nacional de Inclusão da Pessoa
§ 3º No caso de pessoa em situação de institucionalização, com Deficiência (Cadastro-Inclusão), registro público
ao nomear curador, o juiz deve dar preferência a pessoa que eletrônico com a finalidade de coletar, processar, sistematizar
tenha vínculo de natureza familiar, afetiva ou comunitária com e disseminar informações georreferenciadas que permitam a
o curatelado. identificação e a caracterização socioeconômica da pessoa com
deficiência, bem como das barreiras que impedem a realização
Art. 86. Para emissão de documentos oficiais, não será de seus direitos.
exigida a situação de curatela da pessoa com deficiência. § 1º O Cadastro-Inclusão será administrado pelo Poder
Executivo federal e constituído por base de dados,
Art. 87. Em casos de relevância e urgência e a fim de instrumentos, procedimentos e sistemas eletrônicos.
proteger os interesses da pessoa com deficiência em situação § 2º Os dados constituintes do Cadastro-Inclusão serão
de curatela, será lícito ao juiz, ouvido o Ministério Público, de obtidos pela integração dos sistemas de informação e da base
oficio ou a requerimento do interessado, nomear, desde logo, de dados de todas as políticas públicas relacionadas aos
curador provisório, o qual estará sujeito, no que couber, às direitos da pessoa com deficiência, bem como por informações
disposições do Código de Processo Civil. coletadas, inclusive em censos nacionais e nas demais
pesquisas realizadas no País, de acordo com os parâmetros
TÍTULO II estabelecidos pela Convenção sobre os Direitos das Pessoas
DOS CRIMES E DAS INFRAÇÕES ADMINISTRATIVAS com Deficiência e seu Protocolo Facultativo.
§ 3º Para coleta, transmissão e sistematização de dados, é
Art. 88. Praticar, induzir ou incitar discriminação de pessoa facultada a celebração de convênios, acordos, termos de
em razão de sua deficiência: parceria ou contratos com instituições públicas e privadas,
Pena - reclusão, de 1 (um) a 3 (três) anos, e multa.

Legislação 133
Apostila Digital Licenciada para Alice Caroline Guarino dos Santos - alice.guarino@hotmail.com (Proibida a Revenda)
APOSTILAS OPÇÃO

observados os requisitos e procedimentos previstos em Art. 97. A Consolidação das Leis do Trabalho (CLT),
legislação específica. aprovada pelo Decreto-Lei no 5.452, de 1º de maio de 1943,
§ 4º Para assegurar a confidencialidade, a privacidade e as passa a vigorar com as seguintes alterações:
liberdades fundamentais da pessoa com deficiência e os “Art. 428. ..................................................................
princípios éticos que regem a utilização de informações, ...........................................................................................
devem ser observadas as salvaguardas estabelecidas em lei. § 6º Para os fins do contrato de aprendizagem, a
§ 5º Os dados do Cadastro-Inclusão somente poderão ser comprovação da escolaridade de aprendiz com deficiência
utilizados para as seguintes finalidades: deve considerar, sobretudo, as habilidades e competências
I - formulação, gestão, monitoramento e avaliação das relacionadas com a profissionalização.
políticas públicas para a pessoa com deficiência e para ...........................................................................................
identificar as barreiras que impedem a realização de seus § 8º Para o aprendiz com deficiência com 18 (dezoito) anos
direitos; ou mais, a validade do contrato de aprendizagem pressupõe
II - realização de estudos e pesquisas. anotação na CTPS e matrícula e frequência em programa de
§ 6º As informações a que se refere este artigo devem ser aprendizagem desenvolvido sob orientação de entidade
disseminadas em formatos acessíveis. qualificada em formação técnico-profissional metódica.” (NR)
“Art. 433. ..................................................................
Art. 93. Na realização de inspeções e de auditorias pelos ...........................................................................................
órgãos de controle interno e externo, deve ser observado o I - desempenho insuficiente ou inadaptação do aprendiz,
cumprimento da legislação relativa à pessoa com deficiência e salvo para o aprendiz com deficiência quando desprovido de
das normas de acessibilidade vigentes. recursos de acessibilidade, de tecnologias assistivas e de apoio
necessário ao desempenho de suas atividades;
Art. 94. Terá direito a auxílio-inclusão, nos termos da lei, a ..................................................................................”
pessoa com deficiência moderada ou grave que:
Art. 98. A Lei no 7.853, de 24 de outubro de 1989, passa a
I - receba o benefício de prestação continuada previsto no vigorar com as seguintes alterações:
art. 20 da Lei no 8.742, de 7 de dezembro de 1993, e que passe “Art. 3º As medidas judiciais destinadas à proteção de
a exercer atividade remunerada que a enquadre como interesses coletivos, difusos, individuais homogêneos e
segurado obrigatório do RGPS; individuais indisponíveis da pessoa com deficiência poderão
II - tenha recebido, nos últimos 5 (cinco) anos, o benefício ser propostas pelo Ministério Público, pela Defensoria Pública,
de prestação continuada previsto no art. 20 da Lei no 8.742, de pela União, pelos Estados, pelos Municípios, pelo Distrito
7 de dezembro de 1993, e que exerça atividade remunerada Federal, por associação constituída há mais de 1 (um) ano, nos
que a enquadre como segurado obrigatório do RGPS. termos da lei civil, por autarquia, por empresa pública e por
fundação ou sociedade de economia mista que inclua, entre
Art. 95. É vedado exigir o comparecimento de pessoa com suas finalidades institucionais, a proteção dos interesses e a
deficiência perante os órgãos públicos quando seu promoção de direitos da pessoa com deficiência.
deslocamento, em razão de sua limitação funcional e de .................................................................................” (NR)
condições de acessibilidade, imponha-lhe ônus “Art. 8º Constitui crime punível com reclusão de 2 (dois) a
desproporcional e indevido, hipótese na qual serão 5 (cinco) anos e multa:
observados os seguintes procedimentos:
I - quando for de interesse do poder público, o agente I - recusar, cobrar valores adicionais, suspender,
promoverá o contato necessário com a pessoa com deficiência procrastinar, cancelar ou fazer cessar inscrição de aluno em
em sua residência; estabelecimento de ensino de qualquer curso ou grau, público
II - quando for de interesse da pessoa com deficiência, ela ou privado, em razão de sua deficiência;
apresentará solicitação de atendimento domiciliar ou fará II - obstar inscrição em concurso público ou acesso de
representar-se por procurador constituído para essa alguém a qualquer cargo ou emprego público, em razão de sua
finalidade. deficiência;
Parágrafo único. É assegurado à pessoa com deficiência III - negar ou obstar emprego, trabalho ou promoção à
atendimento domiciliar pela perícia médica e social do pessoa em razão de sua deficiência;
Instituto Nacional do Seguro Social (INSS), pelo serviço IV - recusar, retardar ou dificultar internação ou deixar de
público de saúde ou pelo serviço privado de saúde, contratado prestar assistência médico-hospitalar e ambulatorial à pessoa
ou conveniado, que integre o SUS e pelas entidades da rede com deficiência;
socioassistencial integrantes do Suas, quando seu V - deixar de cumprir, retardar ou frustrar execução de
deslocamento, em razão de sua limitação funcional e de ordem judicial expedida na ação civil a que alude esta Lei;
condições de acessibilidade, imponha-lhe ônus VI - recusar, retardar ou omitir dados técnicos
desproporcional e indevido. indispensáveis à propositura da ação civil pública objeto desta
Lei, quando requisitados.
Art. 96. O § 6º-A do art. 135 da Lei no 4.737, de 15 de julho § 1º Se o crime for praticado contra pessoa com deficiência
de 1965 (Código Eleitoral), passa a vigorar com a seguinte menor de 18 (dezoito) anos, a pena é agravada em 1/3 (um
redação: terço).
“Art. 135. ................................................................. § 2º A pena pela adoção deliberada de critérios subjetivos
........................................................................................ para indeferimento de inscrição, de aprovação e de
§ 6º-A. Os Tribunais Regionais Eleitorais deverão, a cada cumprimento de estágio probatório em concursos públicos
eleição, expedir instruções aos Juízes Eleitorais para orientá- não exclui a responsabilidade patrimonial pessoal do
los na escolha dos locais de votação, de maneira a garantir administrador público pelos danos causados.
acessibilidade para o eleitor com deficiência ou com § 3º Incorre nas mesmas penas quem impede ou dificulta
mobilidade reduzida, inclusive em seu entorno e nos sistemas o ingresso de pessoa com deficiência em planos privados de
de transporte que lhe dão acesso. assistência à saúde, inclusive com cobrança de valores
....................................................................................” diferenciados.
§ 4º Se o crime for praticado em atendimento de urgência
e emergência, a pena é agravada em 1/3 (um terço).”

Legislação 134
Apostila Digital Licenciada para Alice Caroline Guarino dos Santos - alice.guarino@hotmail.com (Proibida a Revenda)
APOSTILAS OPÇÃO

Art. 99. O art. 20 da Lei no 8.036, de 11 de maio de 1990, solicitados, aos sindicatos, às entidades representativas dos
passa a vigorar acrescido do seguinte inciso XVIII: empregados ou aos cidadãos interessados.
“Art. 20. ...................................................................... § 3º Para a reserva de cargos será considerada somente a
.............................................................................................. contratação direta de pessoa com deficiência, excluído o
XVIII - quando o trabalhador com deficiência, por aprendiz com deficiência de que trata a Consolidação das Leis
prescrição, necessite adquirir órtese ou prótese para do Trabalho (CLT), aprovada pelo Decreto-Lei no 5.452, de 1º
promoção de acessibilidade e de inclusão social. de maio de 1943.
..................................................................................” § 4º (VETADO).”
“Art. 110-A. No ato de requerimento de benefícios
Art. 100. A Lei no 8.078, de 11 de setembro de 1990 operacionalizados pelo INSS, não será exigida apresentação de
(Código de Defesa do Consumidor), passa a vigorar com as termo de curatela de titular ou de beneficiário com deficiência,
seguintes alterações: observados os procedimentos a serem estabelecidos em
“Art. 6º ....................................................................... regulamento.”

............................................................................................ Art. 102. O art. 2º da Lei no 8.313, de 23 de dezembro de


Parágrafo único. A informação de que trata o inciso III do 1991, passa a vigorar acrescido do seguinte § 3º:
caput deste artigo deve ser acessível à pessoa com deficiência, “Art. 2º .........................................................................
observado o disposto em regulamento.” .............................................................................................
“Art. 43. ...................................................................... § 3º Os incentivos criados por esta Lei somente serão
............................................................................................ concedidos a projetos culturais que forem disponibilizados,
sempre que tecnicamente possível, também em formato
§ 6º Todas as informações de que trata o caput deste artigo acessível à pessoa com deficiência, observado o disposto em
devem ser disponibilizadas em formatos acessíveis, inclusive regulamento.”
para a pessoa com deficiência, mediante solicitação do
consumidor.” (NR) Art. 103. O art. 11 da Lei no 8.429, de 2 de junho de 1992,
passa a vigorar acrescido do seguinte inciso IX:
Art. 101. A Lei no 8.213, de 24 de julho de 1991, passa a “Art. 11. .....................................................................
vigorar com as seguintes alterações: ............................................................................................
“Art. 16. ...................................................................... IX - deixar de cumprir a exigência de requisitos de
I - o cônjuge, a companheira, o companheiro e o filho não acessibilidade previstos na legislação.”
emancipado, de qualquer condição, menor de 21 (vinte e um)
anos ou inválido ou que tenha deficiência intelectual ou mental Art. 104. A Lei no 8.666, de 21 de junho de 1993, passa a
ou deficiência grave; vigorar com as seguintes alterações:
............................................................................................ “Art. 3º .....................................................................
III - o irmão não emancipado, de qualquer condição, menor ..........................................................................................
de 21 (vinte e um) anos ou inválido ou que tenha deficiência § 2º ...........................................................................
intelectual ou mental ou deficiência grave;
.................................................................................” ..........................................................................................
“Art. 77. ..................................................................... V - produzidos ou prestados por empresas que comprovem
............................................................................................ cumprimento de reserva de cargos prevista em lei para pessoa
§ 2º .............................................................................. com deficiência ou para reabilitado da Previdência Social e que
atendam às regras de acessibilidade previstas na legislação.
............................................................................................ ...........................................................................................
§ 5º Nos processos de licitação, poderá ser estabelecida
II - para o filho, a pessoa a ele equiparada ou o irmão, de margem de preferência para:
ambos os sexos, pela emancipação ou ao completar 21 (vinte e I - produtos manufaturados e para serviços nacionais que
um) anos de idade, salvo se for inválido ou tiver deficiência atendam a normas técnicas brasileiras; e
intelectual ou mental ou deficiência grave; II - bens e serviços produzidos ou prestados por empresas
................................................................................... que comprovem cumprimento de reserva de cargos prevista
§ 4º (VETADO). em lei para pessoa com deficiência ou para reabilitado da
...................................................................................” Previdência Social e que atendam às regras de acessibilidade
“Art. 93. (VETADO): previstas na legislação.
I - (VETADO); ...................................................................................”
II - (VETADO); “Art. 66-A. As empresas enquadradas no inciso V do § 2º e
III - (VETADO); no inciso II do § 5º do art. 3º desta Lei deverão cumprir,
IV - (VETADO); durante todo o período de execução do contrato, a reserva de
V - (VETADO). cargos prevista em lei para pessoa com deficiência ou para
§ 1º A dispensa de pessoa com deficiência ou de reabilitado da Previdência Social, bem como as regras de
beneficiário reabilitado da Previdência Social ao final de acessibilidade previstas na legislação.
contrato por prazo determinado de mais de 90 (noventa) dias Parágrafo único. Cabe à administração fiscalizar o
e a dispensa imotivada em contrato por prazo indeterminado cumprimento dos requisitos de acessibilidade nos serviços e
somente poderão ocorrer após a contratação de outro nos ambientes de trabalho.”
trabalhador com deficiência ou beneficiário reabilitado da
Previdência Social. Art. 105. O art. 20 da Lei no 8.742, de 7 de dezembro de
§ 2º Ao Ministério do Trabalho e Emprego incumbe 1993, passa a vigorar com as seguintes alterações:
estabelecer a sistemática de fiscalização, bem como gerar “Art. 20. ......................................................................
dados e estatísticas sobre o total de empregados e as vagas .............................................................................................
preenchidas por pessoas com deficiência e por beneficiários § 2º Para efeito de concessão do benefício de prestação
reabilitados da Previdência Social, fornecendo-os, quando continuada, considera-se pessoa com deficiência aquela que
tem impedimento de longo prazo de natureza física, mental,

Legislação 135
Apostila Digital Licenciada para Alice Caroline Guarino dos Santos - alice.guarino@hotmail.com (Proibida a Revenda)
APOSTILAS OPÇÃO

intelectual ou sensorial, o qual, em interação com uma ou mais 147 desta Lei deve ser acessível, por meio de subtitulação com
barreiras, pode obstruir sua participação plena e efetiva na legenda oculta associada à tradução simultânea em Libras.
sociedade em igualdade de condições com as demais pessoas. § 2º É assegurado também ao candidato com deficiência
............................................................................................ auditiva requerer, no ato de sua inscrição, os serviços de
§ 9º Os rendimentos decorrentes de estágio intérprete da Libras, para acompanhamento em aulas práticas
supervisionado e de aprendizagem não serão computados e teóricas.”
para os fins de cálculo da renda familiar per capita a que se “Art. 154. (VETADO).”
refere o § 3º deste artigo. “Art. 181. ...................................................................
............................................................................................. ..........................................................................................
§ 11. Para concessão do benefício de que trata o caput XVII - .........................................................................
deste artigo, poderão ser utilizados outros elementos Infração - grave;
probatórios da condição de miserabilidade do grupo familiar e .................................................................................”
da situação de vulnerabilidade, conforme regulamento.”
Art. 110. O inciso VI e o § 1º do art. 56 da Lei no 9.615, de
Art. 106. (VETADO). 24 de março de 1998, passam a vigorar com a seguinte
redação:
Art. 107. A Lei no 9.029, de 13 de abril de 1995, passa a “Art. 56. ....................................................................
vigorar com as seguintes alterações: ...........................................................................................
“Art. 1º É proibida a adoção de qualquer prática VI - 2,7% (dois inteiros e sete décimos por cento) da
discriminatória e limitativa para efeito de acesso à relação de arrecadação bruta dos concursos de prognósticos e loterias
trabalho, ou de sua manutenção, por motivo de sexo, origem, federais e similares cuja realização estiver sujeita a
raça, cor, estado civil, situação familiar, deficiência, autorização federal, deduzindo-se esse valor do montante
reabilitação profissional, idade, entre outros, ressalvadas, destinado aos prêmios;
nesse caso, as hipóteses de proteção à criança e ao adolescente .............................................................................................
previstas no inciso XXXIII do art. 7º da Constituição Federal.” § 1º Do total de recursos financeiros resultantes do
“Art. 3º Sem prejuízo do prescrito no art. 2º desta Lei e nos percentual de que trata o inciso VI do caput, 62,96% (sessenta
dispositivos legais que tipificam os crimes resultantes de e dois inteiros e noventa e seis centésimos por cento) serão
preconceito de etnia, raça, cor ou deficiência, as infrações ao destinados ao Comitê Olímpico Brasileiro (COB) e 37,04%
disposto nesta Lei são passíveis das seguintes cominações: (trinta e sete inteiros e quatro centésimos por cento) ao
..................................................................................” Comitê Paralímpico Brasileiro (CPB), devendo ser observado,
“Art. 4º ........................................................................ em ambos os casos, o conjunto de normas aplicáveis à
I - a reintegração com ressarcimento integral de todo o celebração de convênios pela União.
período de afastamento, mediante pagamento das ..................................................................................”
remunerações devidas, corrigidas monetariamente e
acrescidas de juros legais; Art. 111. O art. 1º da Lei no 10.048, de 8 de novembro de
....................................................................................” 2000, passa a vigorar com a seguinte redação:
“Art. 1º As pessoas com deficiência, os idosos com idade
Art. 108. O art. 35 da Lei no 9.250, de 26 de dezembro de igual ou superior a 60 (sessenta) anos, as gestantes, as
1995, passa a vigorar acrescido do seguinte § 5º: lactantes, as pessoas com crianças de colo e os obesos terão
“Art. 35. ...................................................................... atendimento prioritário, nos termos desta Lei.” (NR)
.............................................................................................
§ 5º Sem prejuízo do disposto no inciso IX do parágrafo Art. 112. A Lei no 10.098, de 19 de dezembro de 2000,
único do art. 3º da Lei no 10.741, de 1º de outubro de 2003, a passa a vigorar com as seguintes alterações:
pessoa com deficiência, ou o contribuinte que tenha “Art. 2º .......................................................................
dependente nessa condição, tem preferência na restituição I - acessibilidade: possibilidade e condição de alcance para
referida no inciso III do art. 4º e na alínea “c” do inciso II do art. utilização, com segurança e autonomia, de espaços,
8º.” (NR) mobiliários, equipamentos urbanos, edificações, transportes,
informação e comunicação, inclusive seus sistemas e
Art. 109. A Lei no 9.503, de 23 de setembro de 1997 tecnologias, bem como de outros serviços e instalações
(Código de Trânsito Brasileiro), passa a vigorar com as abertos ao público, de uso público ou privados de uso coletivo,
seguintes alterações: tanto na zona urbana como na rural, por pessoa com
“Art. 2º ........................................................... deficiência ou com mobilidade reduzida;
Parágrafo único. Para os efeitos deste Código, são II - barreiras: qualquer entrave, obstáculo, atitude ou
consideradas vias terrestres as praias abertas à circulação comportamento que limite ou impeça a participação social da
pública, as vias internas pertencentes aos condomínios pessoa, bem como o gozo, a fruição e o exercício de seus
constituídos por unidades autônomas e as vias e áreas de direitos à acessibilidade, à liberdade de movimento e de
estacionamento de estabelecimentos privados de uso expressão, à comunicação, ao acesso à informação, à
coletivo.” (NR) compreensão, à circulação com segurança, entre outros,
“Art. 86-A. As vagas de estacionamento regulamentado de classificadas em:
que trata o inciso XVII do art. 181 desta Lei deverão ser a) barreiras urbanísticas: as existentes nas vias e nos
sinalizadas com as respectivas placas indicativas de espaços públicos e privados abertos ao público ou de uso
destinação e com placas informando os dados sobre a infração coletivo;
por estacionamento indevido.” b) barreiras arquitetônicas: as existentes nos edifícios
“Art. 147-A. Ao candidato com deficiência auditiva é públicos e privados;
assegurada acessibilidade de comunicação, mediante emprego c) barreiras nos transportes: as existentes nos sistemas e
de tecnologias assistivas ou de ajudas técnicas em todas as meios de transportes;
etapas do processo de habilitação. d) barreiras nas comunicações e na informação: qualquer
§ 1º O material didático audiovisual utilizado em aulas entrave, obstáculo, atitude ou comportamento que dificulte ou
teóricas dos cursos que precedem os exames previstos no art. impossibilite a expressão ou o recebimento de mensagens e de

Legislação 136
Apostila Digital Licenciada para Alice Caroline Guarino dos Santos - alice.guarino@hotmail.com (Proibida a Revenda)
APOSTILAS OPÇÃO

informações por intermédio de sistemas de comunicação e de “Art. 10-A. A instalação de qualquer mobiliário urbano em
tecnologia da informação; área de circulação comum para pedestre que ofereça risco de
III - pessoa com deficiência: aquela que tem impedimento acidente à pessoa com deficiência deverá ser indicada
de longo prazo de natureza física, mental, intelectual ou mediante sinalização tátil de alerta no piso, de acordo com as
sensorial, o qual, em interação com uma ou mais barreiras, normas técnicas pertinentes.”
pode obstruir sua participação plena e efetiva na sociedade em “Art. 12-A. Os centros comerciais e os estabelecimentos
igualdade de condições com as demais pessoas; congêneres devem fornecer carros e cadeiras de rodas,
IV - pessoa com mobilidade reduzida: aquela que tenha, motorizados ou não, para o atendimento da pessoa com
por qualquer motivo, dificuldade de movimentação, deficiência ou com mobilidade reduzida.”
permanente ou temporária, gerando redução efetiva da
mobilidade, da flexibilidade, da coordenação motora ou da Art. 113. A Lei no 10.257, de 10 de julho de 2001 (Estatuto
percepção, incluindo idoso, gestante, lactante, pessoa com da Cidade), passa a vigorar com as seguintes alterações:
criança de colo e obeso; “Art. 3º ......................................................................
............................................................................................
V - acompanhante: aquele que acompanha a pessoa com III - promover, por iniciativa própria e em conjunto com os
deficiência, podendo ou não desempenhar as funções de Estados, o Distrito Federal e os Municípios, programas de
atendente pessoal; construção de moradias e melhoria das condições
VI - elemento de urbanização: quaisquer componentes de habitacionais, de saneamento básico, das calçadas, dos
obras de urbanização, tais como os referentes a pavimentação, passeios públicos, do mobiliário urbano e dos demais espaços
saneamento, encanamento para esgotos, distribuição de de uso público;
energia elétrica e de gás, iluminação pública, serviços de IV - instituir diretrizes para desenvolvimento urbano,
comunicação, abastecimento e distribuição de água, inclusive habitação, saneamento básico, transporte e
paisagismo e os que materializam as indicações do mobilidade urbana, que incluam regras de acessibilidade aos
planejamento urbanístico; locais de uso público;
VII - mobiliário urbano: conjunto de objetos existentes nas .................................................................................”
vias e nos espaços públicos, superpostos ou adicionados aos “Art. 41. ....................................................................
elementos de urbanização ou de edificação, de forma que sua ...........................................................................................
modificação ou seu traslado não provoque alterações § 3º As cidades de que trata o caput deste artigo devem
substanciais nesses elementos, tais como semáforos, postes de elaborar plano de rotas acessíveis, compatível com o plano
sinalização e similares, terminais e pontos de acesso coletivo diretor no qual está inserido, que disponha sobre os passeios
às telecomunicações, fontes de água, lixeiras, toldos, públicos a serem implantados ou reformados pelo poder
marquises, bancos, quiosques e quaisquer outros de natureza público, com vistas a garantir acessibilidade da pessoa com
análoga; deficiência ou com mobilidade reduzida a todas as rotas e vias
VIII - tecnologia assistiva ou ajuda técnica: produtos, existentes, inclusive as que concentrem os focos geradores de
equipamentos, dispositivos, recursos, metodologias, maior circulação de pedestres, como os órgãos públicos e os
estratégias, práticas e serviços que objetivem promover a locais de prestação de serviços públicos e privados de saúde,
funcionalidade, relacionada à atividade e à participação da educação, assistência social, esporte, cultura, correios e
pessoa com deficiência ou com mobilidade reduzida, visando à telégrafos, bancos, entre outros, sempre que possível de
sua autonomia, independência, qualidade de vida e inclusão maneira integrada com os sistemas de transporte coletivo de
social; passageiros.” (NR)
IX - comunicação: forma de interação dos cidadãos que
abrange, entre outras opções, as línguas, inclusive a Língua Art. 114. A Lei no 10.406, de 10 de janeiro de 2002 (Código
Brasileira de Sinais (Libras), a visualização de textos, o Braille, Civil), passa a vigorar com as seguintes alterações:
o sistema de sinalização ou de comunicação tátil, os caracteres “Art. 3º São absolutamente incapazes de exercer
ampliados, os dispositivos multimídia, assim como a pessoalmente os atos da vida civil os menores de 16
linguagem simples, escrita e oral, os sistemas auditivos e os (dezesseis) anos.
meios de voz digitalizados e os modos, meios e formatos I - (Revogado);
aumentativos e alternativos de comunicação, incluindo as II - (Revogado);
tecnologias da informação e das comunicações; III - (Revogado).”
X - desenho universal: concepção de produtos, ambientes, “Art. 4º São incapazes, relativamente a certos atos ou à
programas e serviços a serem usados por todas as pessoas, maneira de os exercer:
sem necessidade de adaptação ou de projeto específico, .....................................................................................
incluindo os recursos de tecnologia assistiva.” II - os ébrios habituais e os viciados em tóxico;
“Art. 3º O planejamento e a urbanização das vias públicas, III - aqueles que, por causa transitória ou permanente, não
dos parques e dos demais espaços de uso público deverão ser puderem exprimir sua vontade;
concebidos e executados de forma a torná-los acessíveis para .............................................................................................
todas as pessoas, inclusive para aquelas com deficiência ou Parágrafo único. A capacidade dos indígenas será regulada
com mobilidade reduzida. por legislação especial.”
Parágrafo único. O passeio público, elemento obrigatório “Art. 228. .....................................................................
de urbanização e parte da via pública, normalmente segregado .............................................................................................
e em nível diferente, destina-se somente à circulação de II - (Revogado);
pedestres e, quando possível, à implantação de mobiliário III - (Revogado);
urbano e de vegetação.” .............................................................................................
“Art. 9º ........................................................................ § 1º ..............................................................................
Parágrafo único. Os semáforos para pedestres instalados § 2º A pessoa com deficiência poderá testemunhar em
em vias públicas de grande circulação, ou que deem acesso aos igualdade de condições com as demais pessoas, sendo-lhe
serviços de reabilitação, devem obrigatoriamente estar assegurados todos os recursos de tecnologia assistiva.”
equipados com mecanismo que emita sinal sonoro suave para “Art. 1.518. Até a celebração do casamento podem os pais
orientação do pedestre.” ou tutores revogar a autorização.”
“Art. 1.548. ...................................................................

Legislação 137
Apostila Digital Licenciada para Alice Caroline Guarino dos Santos - alice.guarino@hotmail.com (Proibida a Revenda)
APOSTILAS OPÇÃO

I - (Revogado); de sua confiança, para prestar-lhe apoio na tomada de decisão


....................................................................................” sobre atos da vida civil, fornecendo-lhes os elementos e
“Art. 1.550. .................................................................. informações necessários para que possa exercer sua
............................................................................................. capacidade.
§ 1º .............................................................................. § 1º Para formular pedido de tomada de decisão apoiada, a
§ 2º A pessoa com deficiência mental ou intelectual em pessoa com deficiencia e os apoiadores devem apresentar
idade núbia poderá contrair matrimônio, expressando sua termo em que constem os limites do apoio a ser oferecido e os
vontade diretamente ou por meio de seu responsável ou compromissos dos apoiadores, inclusive o prazo de vigencia
curador.” (NR) do acordo e o respeito à vontade, aos direitos e aos interesses
“Art. 1.557. ................................................................ da pessoa que devem apoiar.
............................................................................................ § 2º O pedido de tomada de decisão apoiada será requerido
III - a ignorância, anterior ao casamento, de defeito físico pela pessoa a ser apoiada, com indicação expressa das pessoas
irremediável que não caracterize deficiência ou de moléstia aptas a prestarem o apoio previsto no caput deste artigo.
grave e transmissível, por contágio ou por herança, capaz de § 3º Antes de se pronunciar sobre o pedido de tomada de
pôr em risco a saúde do outro cônjuge ou de sua descendência; decisão apoiada, o juiz, assistido por equipe multidisciplinar,
IV - (Revogado).” após oitiva do Ministério Público, ouvirá pessoalmente o
“Art. 1.767. .................................................................. requerente e as pessoas que lhe prestarão apoio.
I - aqueles que, por causa transitória ou permanente, não § 4º A decisão tomada por pessoa apoiada terá validade e
puderem exprimir sua vontade; efeitos sobre terceiros, sem restrições, desde que esteja
II - (Revogado); inserida nos limites do apoio acordado.
III - os ébrios habituais e os viciados em tóxico; § 5º Terceiro com quem a pessoa apoiada mantenha
IV - (Revogado); relação negocial pode solicitar que os apoiadores contra
....................................................................................” assinem o contrato ou acordo, especificando, por escrito, sua
“Art. 1.768. O processo que define os termos da curatela função em relação ao apoiado.
deve ser promovido: § 6º Em caso de negócio jurídico que possa trazer risco ou
............................................................................................. prejuízo relevante, havendo divergência de opiniões entre a
IV - pela própria pessoa.” pessoa apoiada e um dos apoiadores, deverá o juiz, ouvido o
“Art. 1.769. O Ministério Público somente promoverá o Ministério Público, decidir sobre a questão.
processo que define os termos da curatela: § 7º Se o apoiador agir com negligência, exercer pressão
I - nos casos de deficiência mental ou intelectual; indevida ou não adimplir as obrigações assumidas, poderá a
............................................................................................ pessoa apoiada ou qualquer pessoa apresentar denúncia ao
III - se, existindo, forem menores ou incapazes as pessoas Ministério Público ou ao juiz.
mencionadas no inciso II.” § 8º Se procedente a denúncia, o juiz destituirá o apoiador
“Art. 1.771. Antes de se pronunciar acerca dos termos da e nomeará, ouvida a pessoa apoiada e se for de seu interesse,
curatela, o juiz, que deverá ser assistido por equipe outra pessoa para prestação de apoio.
multidisciplinar, entrevistará pessoalmente o interditando.” § 9º A pessoa apoiada pode, a qualquer tempo, solicitar o
“Art. 1.772. O juiz determinará, segundo as potencialidades término de acordo firmado em processo de tomada de decisão
da pessoa, os limites da curatela, circunscritos às restrições apoiada.
constantes do art. 1.782, e indicará curador. § 10. O apoiador pode solicitar ao juiz a exclusao de sua
participaçao do processo de tomada de decisao apoiada, sendo
Parágrafo único. Para a escolha do curador, o juiz levará em seu desligamento condicionado à manifestaçao do juiz sobre a
conta a vontade e as preferências do interditando, a ausência matéria.
de conflito de interesses e de influência indevida, a § 11. Aplicam-se à tomada de decisao apoiada, no que
proporcionalidade e a adequação às circunstâncias da pessoa.” couber, as disposiçoes referentes à prestaçao de contas na
“Art. 1.775-A. Na nomeação de curador para a pessoa com curatela.”
deficiência, o juiz poderá estabelecer curatela compartilhada a
mais de uma pessoa.” Art. 117. O art. 1º da Lei no 11.126, de 27 de junho de 2005,
“Art. 1.777. As pessoas referidas no inciso I do art. 1.767 passa a vigorar com a seguinte redação:
receberão todo o apoio necessário para ter preservado o “Art. 1º E assegurado à pessoa com deficiencia visual
direito à convivência familiar e comunitária, sendo evitado o acompanhada de cão-guia o direito de ingressar e de
seu recolhimento em estabelecimento que os afaste desse permanecer com o animal em todos os meios de transporte e
convívio.” em estabelecimentos abertos ao público, de uso público e
Art. 115. O Título IV do Livro IV da Parte Especial da Lei no privados de uso coletivo, desde que observadas as condições
10.406, de 10 de janeiro de 2002 (Código Civil), passa a vigorar impostas por esta Lei.
com a seguinte redação: .............................................................................................
§ 2º O disposto no caput deste artigo aplica-se a todas as
“TÍTULO IV modalidades e jurisdições do serviço de transporte coletivo de
Da Tutela, da Curatela e da Tomada de Decisão passageiros, inclusive em esfera internacional com origem no
Apoiada” território brasileiro.” (NR)

Art. 116. O Título IV do Livro IV da Parte Especial da Lei no Art. 118. O inciso IV do art. 46 da Lei no 11.904, de 14 de
10.406, de 10 de janeiro de 2002 (Código Civil), passa a vigorar janeiro de 2009, passa a vigorar acrescido da seguinte alínea
acrescido do seguinte Capítulo III: “k”:
“Art. 46. ......................................................................
“CAPÍTULO III ...........................................................................................
Da Tomada de Decisão Apoiada IV - ..............................................................................
...........................................................................................
Art. 1.783-A. A tomada de decisão apoiada é o processo k) de acessibilidade a todas as pessoas.
pelo qual a pessoa com deficiência elege pelo menos 2 (duas) .................................................................................”
pessoas idôneas, com as quais mantenha vínculos e que gozem

Legislação 138
Apostila Digital Licenciada para Alice Caroline Guarino dos Santos - alice.guarino@hotmail.com (Proibida a Revenda)
APOSTILAS OPÇÃO

Art. 119. A Lei no 12.587, de 3 de janeiro de 2012, passa a Art. 126. Prorroga-se até 31 de dezembro de 2021 a
vigorar acrescida do seguinte art. 12-B: vigência da Lei no 8.989, de 24 de fevereiro de 1995.
“Art. 12-B. Na outorga de exploração de serviço de táxi,
reservar-se-ão 10% (dez por cento) das vagas para condutores Art. 127. Esta Lei entra em vigor após decorridos 180
com deficiência. (cento e oitenta) dias de sua publicação oficial.
§ 1º Para concorrer às vagas reservadas na forma do caput
deste artigo, o condutor com deficiência deverá observar os Questões
seguintes requisitos quanto ao veículo utilizado:
I - ser de sua propriedade e por ele conduzido; e 01. (Prefeitura de Natal/RN - Advogado –
II - estar adaptado às suas necessidades, nos termos da IDECAN/2016). De acordo com o Estatuto da Pessoa com
legislação vigente. Deficiência, as ações e os serviços de saúde pública destinados
§ 2º No caso de não preenchimento das vagas na forma à pessoa com deficiência devem assegurar:
estabelecida no caput deste artigo, as remanescentes devem (A) Atenção sexual e reprodutiva, incluindo o direito à
ser disponibilizadas para os demais concorrentes.” fertilização assistida.
(B) Atendimento psicológico, inclusive para seus
Art. 120. Cabe aos órgãos competentes, em cada esfera de familiares, sendo vedado aos atendentes pessoais.
governo, a elaboração de relatórios circunstanciados sobre o (C) Promoção de estratégias de capacitação permanente
cumprimento dos prazos estabelecidos por força das Leis no das equipes que atuam no SUS, em todos os níveis de atenção,
10.048, de 8 de novembro de 2000, e no 10.098, de 19 de no atendimento à pessoa com deficiência, vedada a orientação
dezembro de 2000, bem como o seu encaminhamento ao a seus atendentes pessoais.
Ministério Público e aos órgãos de regulação para adoção das (D) Serviços de habilitação e de reabilitação sempre que
providências cabíveis. necessários, para qualquer tipo de deficiência, exclusivamente,
Parágrafo único. Os relatórios a que se refere o caput deste quando houver possibilidade de recuperação da capacidade
artigo deverão ser apresentados no prazo de 1 (um) ano a produtiva, sendo vedada apenas para a manutenção da melhor
contar da entrada em vigor desta Lei. condição de saúde e qualidade de vida.

Art. 121. Os direitos, os prazos e as obrigações previstos 02. (Prefeitura de Fortaleza/CE - Psicologia -
nesta Lei não excluem os já estabelecidos em outras Prefeitura de Fortaleza – CE/2016). De acordo com a Lei nº
legislações, inclusive em pactos, tratados, convenções e 13.146, de 6 de julho de 2015 – Estatuto da Pessoa com
declarações internacionais aprovados e promulgados pelo Deficiência, quanto ao Direito ao Trabalho é correto afirmar.
Congresso Nacional, e devem ser aplicados em conformidade (A) A pessoa com deficiência tem direito, em igualdade de
com as demais normas internas e acordos internacionais oportunidades com as demais pessoas, às condições justas e
vinculantes sobre a matéria. favoráveis de trabalho, exceto quanto à igual remuneração por
Parágrafo único. Prevalecerá a norma mais benéfica à trabalho de igual valor.
pessoa com deficiência. (B) As pessoas jurídicas de direito público são obrigadas a
garantir ambientes de trabalho acessíveis e inclusivos e as de
Art. 122. Regulamento disporá sobre a adequação do direito privado ficam com a responsabilidade exclusiva de
disposto nesta Lei ao tratamento diferenciado, simplificado e manter ações de promoção valorativa as pessoas com
favorecido a ser dispensado às microempresas e às empresas deficiência.
de pequeno porte, previsto no § 3º do art. 1º da Lei (C) É garantida aos trabalhadores com deficiência que
Complementar no 123, de 14 de dezembro de 2006. demonstrem bom desempenho cognitivo e de aprendizagem a
acessibilidade em cursos de formação e de capacitação.
Art. 123. Revogam-se os seguintes dispositivos: (D) É vedada restrição ao trabalho da pessoa com
I - o inciso II do § 2º do art. 1º da Lei no 9.008, de 21 de deficiência e qualquer discriminação em razão de sua
março de 1995; condição, inclusive nas etapas de recrutamento, seleção,
II - os incisos I, II e III do art. 3º da Lei no 10.406, de 10 de contratação, admissão, exames admissional e periódico,
janeiro de 2002 (Código Civil); permanência no emprego, ascensão profissional e reabilitação
III - os incisos II e III do art. 228 da Lei no 10.406, de 10 de profissional, bem como exigência de aptidão plena.
janeiro de 2002 (Código Civil);
IV - o inciso I do art. 1.548 da Lei no 10.406, de 10 de 03. (Prefeitura de Campinas/SP - Procurador –
janeiro de 2002 (Código Civil); FCC/2016). Acerca das inovações introduzidas pela Lei
V - o inciso IV do art. 1.557 da Lei no 10.406, de 10 de Brasileira de Inclusão da Pessoa com Deficiência (Estatuto da
janeiro de 2002 (Código Civil); Pessoa com Deficiência, Lei n°13.146, de 06 de julho de 2015),
VI - os incisos II e IV do art. 1.767 da Lei no 10.406, de 10 é correto afirmar:
de janeiro de 2002 (Código Civil); (A) O pedido de tomada de decisão apoiada será formulado
VII - os arts. 1.776 e 1.780 da Lei no 10.406, de 10 de por pelo menos dois apoiadores idôneos, devendo constar os
janeiro de 2002 (Código Civil). limites do apoio a ser oferecido e o prazo de vigência do
acordo.
Art. 124. O § 1º do art. 2º desta Lei deverá entrar em vigor (B) A interdição da pessoa com deficiência não mais afeta
em até 2 (dois) anos, contados da entrada em vigor desta Lei. os atos relacionados aos direitos de natureza patrimonial e de
gestão negocial, que poderão ser realizados com a adoção de
Art. 125. Devem ser observados os prazos a seguir processo de tomada de decisão apoiada.
discriminados, a partir da entrada em vigor desta Lei, para o (C) A declaração de incapacidade absoluta da pessoa com
cumprimento dos seguintes dispositivos: deficiência está condicionada à prévia avaliação por equipe
I - incisos I e II do § 2º do art. 28, 48 (quarenta e oito) multi-profissional e interdisciplinar.
meses; (D) Para emissão de documentos oficiais, não será exigida
II - § 6º do art. 44, 48 (quarenta e oito) meses; a situação de curatela da pessoa com deficiência.
III - art. 45, 24 (vinte e quatro) meses; (E) O Estatuto instituiu em favor da pessoa com deficiência
IV - art. 49, 48 (quarenta e oito) meses. o benefício da meia entrada em espetáculos artístico-culturais
e esportivos.

Legislação 139
Apostila Digital Licenciada para Alice Caroline Guarino dos Santos - alice.guarino@hotmail.com (Proibida a Revenda)
APOSTILAS OPÇÃO

04. (Prefeitura de Nova Ponte/MG - Advogado – figure em um dos polos da ação ou participe da lide posta em
IBGP/2016). A Lei Brasileira de Inclusão da Pessoa com Juízo, salvo na condição de testemunha.
Deficiência (Estatuto da Pessoa com Deficiência) busca (...) Certo
assegurar e a promover, em condições de igualdade, o (...) Errado
exercício dos direitos e das liberdades fundamentais para
pessoa com deficiência, visando à sua inclusão social e 08. (Questão adaptada – 2015) Consoante o que dispõe
cidadania. a Lei 13.146/2015:
Nos termos da Lei em referência, assinale a alternativa (A) A pessoa com deficiência não poderá ser obrigada a se
INCORRETA. submeter a intervenção clínica ou cirúrgica, a tratamento ou a
(A) Toda pessoa com deficiência tem direito à igualdade de institucionalização forçada.
oportunidades com as demais pessoas e não sofrerá nenhuma (B) A pessoa com deficiência poderá ser obrigada a se
espécie de discriminação. submeter a intervenção clínica ou cirúrgica, a tratamento ou a
(B) É dever de todos comunicar à autoridade competente institucionalização forçada.
qualquer forma de ameaça ou de violação aos direitos da (C) A pessoa com deficiência poderá ser obrigada a se
pessoa com deficiência. submeter somente a intervenção clínica.
(C) A deficiência afeta a plena capacidade civil da pessoa, (D) A pessoa com deficiência não poderá ser obrigada a se
limitando o exercício do direito à família e à convivência submeter a intervenção clínica ou cirúrgica, podendo ser
familiar e comunitária. obrigada todavia a tratamento ou a institucionalização
(D) A pessoa com deficiência tem direito a receber forçada.
atendimento prioritário, sobretudo com a finalidade de
atendimento em todas as instituições e serviços de 09. (Questão adaptada – 2015) A pessoa com deficiência
atendimento ao público. tem direito à cultura, ao esporte, ao turismo e ao lazer em
igualdade de oportunidades com as demais pessoas, sendo-lhe
05. (MPE/RJ - Analista do Ministério Público – garantido o acesso:
Processual – FGV/2016). Promotoria de Tutela Coletiva I - a bens culturais em formato acessível;
especializada na Proteção à Pessoa com Deficiência instaurou II - a programas de televisão, cinema, teatro e outras
inquérito civil público para apurar eventual desatendimento atividades culturais e desportivas em formato acessível; e
das disposições do Estatuto da Pessoa com Deficiência, no que III - a monumentos e locais de importância cultural e a
se refere ao direito ao transporte e à mobilidade da pessoa com espaços que ofereçam serviços ou eventos culturais e
deficiência ou com mobilidade reduzida. Identificada esportivos.
irregularidade cometida pelo investigado, com base na Lei nº IV - a oferta de obra intelectual em formato acessível,
13.146/2015, o Promotor expediu recomendação: exceto sob a alegação de proteção dos direitos de propriedade
(A) à sociedade empresária que opera frota de táxi para intelectual.
reservar 50% (cinquenta por cento) de seus veículos
acessíveis à pessoa com deficiência, que terá prioridade sobre Está CORRETO o que se afirma apenas em:
os demais passageiros nas filas para embarque nos táxis; (A) I, II e IV
(B) à locadora de veículos para oferecer pelo menos 1 (um) (B) I, II e III.
veículo adaptado para uso de pessoa com deficiência, (C) II, III e IV
independentemente da quantidade total de veículos que (D) II e IIV.
compõem sua frota;
(C) à concessionária de serviço público de transporte 10. (Questão adaptada – 2015) Consoante o que prevê a
coletivo municipal de passageiros para que assegure à pessoa Lei 13.146/2015:
com deficiência prioridade e segurança nos procedimentos de (A) É assegurado à pessoa com deficiência que não possua
embarque e de desembarque, de acordo com as normas meios para prover sua subsistência nem de tê-la provida por
técnicas; sua família o benefício mensal de meio salário-mínimo, nos
(D) ao shopping center, para garantir ao menos 10 (dez) termos da Lei no 8.742, de 7 de dezembro de 1993.
vagas no estacionamento, independentemente de sua (B) É assegurado à pessoa com deficiência que não possua
capacidade total, próximas aos acessos de circulação de meios para prover sua subsistência nem de tê-la provida por
pedestres, devidamente sinalizadas, para veículos que sua família o benefício mensal de 2 (dois) salários-mínimos,
transportem pessoa com deficiência; nos termos da Lei no 8.742, de 7 de dezembro de 1993.
(E) ao Prefeito Municipal, para reservar ao menos 2 (duas) (C) É assegurado à pessoa com deficiência que não possua
vagas em cada via pública que ofereça estacionamento ao meios para prover sua subsistência nem de tê-la provida por
público, independentemente do total de vagas na rua, para sua família o benefício mensal de 1 (um) salário-mínimo, nos
pessoa com deficiência ou com comprometimento de termos da Lei no 8.742, de 7 de dezembro de 1993.
mobilidade, desde que devidamente identificada. (D) É assegurado à pessoa com deficiência que não possua
meios para prover sua subsistência nem de tê-la provida por
06. (INSS - Analista do Seguro Social - Serviço Social – sua família o benefício mensal de um salário-mínimo e meio,
CESPE/2016). No que se refere ao Estatuto da Pessoa com nos termos da Lei no 8.742, de 7 de dezembro de 1993.
Deficiência, julgue o seguinte item.
A pessoa com deficiência tem o direito de receber Respostas
cobranças de tributos de forma acessível, independentemente
de solicitação. 01. A / 02. D / 03. D / 04. C / 05. C
(...) Certo 06. Errado / 07. Errado / 08. A. / 09. B. / 10. C.
(...) Errado

07. (MPE/SC - Promotor de Justiça – Vespertina – MPE-


SC/2016). A Lei n. 13.146/2015 (Estatuto da Pessoa com
Deficiência) determina o oferecimento de todos os recursos de
tecnologia assistida disponíveis para que a pessoa com
deficiência tenha garantido o acesso à justiça, sempre que

Legislação 140
Apostila Digital Licenciada para Alice Caroline Guarino dos Santos - alice.guarino@hotmail.com (Proibida a Revenda)
APOSTILAS OPÇÃO

XI - realizar estudos socioeconômicos com os usuários


CÓDIGO DE ÉTICA DO/A para fins de benefícios e serviços sociais junto a órgãos da
administração pública direta e indireta, empresas privadas e
ASSISTENTE SOCIAL – LEI outras entidades.
8.662/93.
Art. 5º Constituem atribuições privativas do Assistente
Social:
I - coordenar, elaborar, executar, supervisionar e avaliar
LEI Nº 8.662, DE 7 DE JUNHO DE 1993
estudos, pesquisas, planos, programas e projetos na área de
Serviço Social;
Dispõe sobre a profissão de Assistente Social e dá outras
II - planejar, organizar e administrar programas e projetos
providências
em Unidade de Serviço Social;
III - assessoria e consultoria e órgãos da Administração
O PRESIDENTE DA REPÚBLICA Faço saber que o
Pública direta e indireta, empresas privadas e outras
Congresso Nacional decreta e eu sanciono a seguinte lei:
entidades, em matéria de Serviço Social;
IV - realizar vistorias, perícias técnicas, laudos periciais,
Art. 1º É livre o exercício da profissão de Assistente Social
informações e pareceres sobre a matéria de Serviço Social;
em todo o território nacional, observadas as condições
V - assumir, no magistério de Serviço Social tanto a nível de
estabelecidas nesta lei.
graduação como pós-graduação, disciplinas e funções que
exijam conhecimentos próprios e adquiridos em curso de
Art. 2º Somente poderão exercer a profissão de Assistente
formação regular;
Social:
VI - treinamento, avaliação e supervisão direta de
I - Os possuidores de diploma em curso de graduação em
estagiários de Serviço Social;
Serviço Social, oficialmente reconhecido, expedido por
VII - dirigir e coordenar Unidades de Ensino e Cursos de
estabelecimento de ensino superior existente no País,
Serviço Social, de graduação e pós-graduação;
devidamente registrado no órgão competente;
VIII - dirigir e coordenar associações, núcleos, centros de
II - os possuidores de diploma de curso superior em
estudo e de pesquisa em Serviço Social;
Serviço Social, em nível de graduação ou equivalente, expedido
IX - elaborar provas, presidir e compor bancas de exames
por estabelecimento de ensino sediado em países
e comissões julgadoras de concursos ou outras formas de
estrangeiros, conveniado ou não com o governo brasileiro,
seleção para Assistentes Sociais, ou onde sejam aferidos
desde que devidamente revalidado e registrado em órgão
conhecimentos inerentes ao Serviço Social;
competente no Brasil;
X - coordenar seminários, encontros, congressos e eventos
III - os agentes sociais, qualquer que seja sua denominação
assemelhados sobre assuntos de Serviço Social;
com funções nos vários órgãos públicos, segundo o disposto no
XI - fiscalizar o exercício profissional através dos
art. 14 e seu parágrafo único da Lei nº 1.889, de 13 de junho de
Conselhos Federal e Regionais;
1953.
XII - dirigir serviços técnicos de Serviço Social em
Parágrafo único. O exercício da profissão de Assistente
entidades públicas ou privadas;
Social requer prévio registro nos Conselhos Regionais que
XIII - ocupar cargos e funções de direção e fiscalização da
tenham jurisdição sobre a área de atuação do interessado nos
gestão financeira em órgãos e entidades representativas da
termos desta lei.
categoria profissional.
Art. 3º A designação profissional de Assistente Social é
Art. 5º-A. A duração do trabalho do Assistente Social é de
privativa dos habilitados na forma da legislação vigente.
30 (trinta) horas semanais. (Incluído pela Lei nº 12.317, de
2010)
Art. 4º Constituem competências do Assistente Social:
I - elaborar, implementar, executar e avaliar políticas
Art. 6º São alteradas as denominações do atual Conselho
sociais junto a órgãos da administração pública, direta ou
Federal de Assistentes Sociais (CFAS) e dos Conselhos
indireta, empresas, entidades e organizações populares;
Regionais de Assistentes Sociais (CRAS), para,
II - elaborar, coordenar, executar e avaliar planos,
respectivamente, Conselho Federal de Serviço Social (CFESS)
programas e projetos que sejam do âmbito de atuação do
e Conselhos Regionais de Serviço Social (CRESS).
Serviço Social com participação da sociedade civil;
III - encaminhar providências, e prestar orientação social a
Art. 7º O Conselho Federal de Serviço Social (CFESS) e os
indivíduos, grupos e à população;
Conselhos Regionais de Serviço Social (CRESS) constituem, em
IV - (Vetado);
seu conjunto, uma entidade com personalidade jurídica e
V - orientar indivíduos e grupos de diferentes segmentos
forma federativa, com o objetivo básico de disciplinar e
sociais no sentido de identificar recursos e de fazer uso dos
defender o exercício da profissão de Assistente Social em todo
mesmos no atendimento e na defesa de seus direitos;
o território nacional.
VI - planejar, organizar e administrar benefícios e Serviços
1º Os Conselhos Regionais de Serviço Social (CRESS) são
Sociais;
dotados de autonomia administrativa e financeira, sem
VII - planejar, executar e avaliar pesquisas que possam
prejuízo de sua vinculação ao Conselho Federal, nos termos da
contribuir para a análise da realidade social e para subsidiar
legislação em vigor.
ações profissionais;
2º Cabe ao Conselho Federal de Serviço Social (CFESS) e
VIII - prestar assessoria e consultoria a órgãos da
aos Conselhos Regionais de Serviço Social (CRESS),
administração pública direta e indireta, empresas privadas e
representar, em juízo e fora dele, os interesses gerais e
outras entidades, com relação às matérias relacionadas no
individuais dos Assistentes Sociais, no cumprimento desta lei.
inciso II deste artigo;
IX - prestar assessoria e apoio aos movimentos sociais em
Art. 8º Compete ao Conselho Federal de Serviço Social
matéria relacionada às políticas sociais, no exercício e na
(CFESS), na qualidade de órgão normativo de grau superior, o
defesa dos direitos civis, políticos e sociais da coletividade;
exercício das seguintes atribuições:
X - planejamento, organização e administração de Serviços
Sociais e de Unidade de Serviço Social;

Legislação 141
Apostila Digital Licenciada para Alice Caroline Guarino dos Santos - alice.guarino@hotmail.com (Proibida a Revenda)
APOSTILAS OPÇÃO

I - orientar, disciplinar, normatizar, fiscalizar e defender o Art. 14. Cabe às Unidades de Ensino credenciar e
exercício da profissão de Assistente Social, em conjunto com o comunicar aos Conselhos Regionais de sua jurisdição os
CRESS; campos de estágio de seus alunos e designar os Assistentes
II - assessorar os CRESS sempre que se fizer necessário; Sociais responsáveis por sua supervisão.
III - aprovar os Regimentos Internos dos CRESS no fórum Parágrafo único. Somente os estudantes de Serviço Social,
máximo de deliberação do conjunto CFESS/CRESS; sob supervisão direta de Assistente Social em pleno gozo de
IV - aprovar o Código de Ética Profissional dos Assistentes seus direitos profissionais, poderão realizar estágio de Serviço
Sociais juntamente com os CRESS, no fórum máximo de Social.
deliberação do conjunto CFESS/CRESS;
V - funcionar como Tribunal Superior de Ética Profissional; Art. 15. É vedado o uso da expressão Serviço Social por
VI - julgar, em última instância, os recursos contra as quaisquer pessoas de direito público ou privado que não
sanções impostas pelos CRESS; desenvolvam atividades previstas nos arts. 4º e 5º desta lei.
VII - estabelecer os sistemas de registro dos profissionais Parágrafo único. As pessoas de direito público ou privado
habilitados; que se encontrem na situação mencionada neste artigo terão o
VIII - prestar assessoria técnico-consultiva aos organismos prazo de noventa dias, a contar da data da vigência desta lei,
públicos ou privados, em matéria de Serviço Social; para processarem as modificações que se fizerem necessárias
IX - (Vetado). a seu integral cumprimento, sob pena das medidas judiciais
cabíveis.
Art. 9º O fórum máximo de deliberação da profissão para
os fins desta lei dar-se-á nas reuniões conjuntas dos Conselhos Art. 16. Os CRESS aplicarão as seguintes penalidades aos
Federal e Regionais, que inclusive fixarão os limites de sua infratores dos dispositivos desta Lei:
competência e sua forma de convocação. I - multa no valor de uma a cinco vezes a anuidade vigente;
II - suspensão de um a dois anos de exercício da profissão
Art. 10. Compete aos CRESS, em suas respectivas áreas de ao Assistente Social que, no âmbito de sua
jurisdição, na qualidade de órgão executivo e de primeira III - cancelamento definitivo do registro, nos casos de
instância, o exercício das seguintes atribuições: extrema gravidade ou de reincidência contumaz.
I - organizar e manter o registro profissional dos 1º Provada a participação ativa ou conivência de empresas,
Assistentes Sociais e o cadastro das instituições e obras sociais entidades, instituições ou firmas individuais nas infrações a
públicas e privadas, ou de fins filantrópicos; dispositivos desta lei pelos profissionais delas dependentes,
II - fiscalizar e disciplinar o exercício da profissão de serão estas também passíveis das multas aqui estabelecidas,
Assistente Social na respectiva região; na proporção de sua responsabilidade, sob pena das medidas
III - expedir carteiras profissionais de Assistentes Sociais, judiciais cabíveis.
fixando a respectiva taxa; 2º No caso de reincidência na mesma infração no prazo de
IV - zelar pela observância do Código de Ética Profissional, dois anos, a multa cabível será elevada ao dobro.
funcionando como Tribunais Regionais de Ética Profissional;
V - aplicar as sanções previstas no Código de Ética Art. 17. A Carteira de Identificação Profissional expedida
Profissional; pelos Conselhos Regionais de Serviço Social (CRESS), servirá
VI - fixar, em assembleia da categoria, as anuidades que de prova para fins de exercício profissional e de Carteira de
devem ser pagas pelos Assistentes Sociais; Identidade Pessoal, e terá fé pública em todo o território
VII - elaborar o respectivo Regimento Interno e submetê- nacional.
lo a exame e aprovação do fórum máximo de deliberação do
conjunto CFESS/CRESS. Art. 18. As organizações que se registrarem nos CRESS
Art. 11. O Conselho Federal de Serviço Social (CFESS) terá receberão um certificado que as habilitará a atuar na área de
sede e foro no Distrito Federal. Serviço Social.

Art. 12. Em cada capital de Estado, de Território e no Art. 19. O Conselho Federal de Serviço Social (CFESS) será
Distrito Federal, haverá um Conselho Regional de Serviço mantido:
Social (CRESS) denominado segundo a sua jurisdição, a qual I - por contribuições, taxas e emolumentos arrecadados
alcançará, respectivamente, a do Estado, a do Território e a do pelos CRESS, em percentual a ser definido pelo fórum máximo
Distrito Federal. instituído pelo art. 9º desta lei;
1º Nos Estados ou Territórios em que os profissionais que II - por doações e legados;
neles atuam não tenham possibilidade de instalar um III - por outras rendas.
Conselho Regional, deverá ser constituída uma delegacia
subordinada ao Conselho Regional que oferecer melhores Art. 20. O Conselho Federal de Serviço Social (CFESS) e os
condições de comunicação, fiscalização e orientação, ouvido o Conselhos Regionais de Serviço Social (CRESS) contarão cada
órgão regional e com homologação do Conselho Federal. um com nove membros efetivos: Presidente, Vice-Presidente,
2º Os Conselhos Regionais poderão constituir, dentro de dois Secretários, dois Tesoureiros e três membros do Conselho
sua própria área de jurisdição, delegacias seccionais para Fiscal, e nove suplentes, eleitos dentre os Assistentes Sociais,
desempenho de suas atribuições executivas e de primeira por via direta, para um mandato de três anos, de acordo com
instância nas regiões em que forem instalados, desde que a as normas estabelecidas em Código Eleitoral aprovado pelo
arrecadação proveniente dos profissionais nelas atuantes seja fórum instituído pelo art. 9º desta lei.
suficiente para sua própria manutenção. Parágrafo único. As delegacias seccionais contarão com
três membros efetivos: um Delegado, um Secretário e um
Art. 13. A inscrição nos Conselhos Regionais sujeita os Tesoureiro, e três suplentes, eleitos dentre os Assistentes
Assistentes Sociais ao pagamento das contribuições Sociais da área de sua jurisdição, nas condições previstas neste
compulsórias (anuidades), taxas e demais emolumentos que artigo.
forem estabelecidos em regulamentação baixada pelo
Conselho Federal, em deliberação conjunta com os Conselhos Art. 21. (Vetado).
Regionais.

Legislação 142
Apostila Digital Licenciada para Alice Caroline Guarino dos Santos - alice.guarino@hotmail.com (Proibida a Revenda)
APOSTILAS OPÇÃO

Art. 22. O Conselho Federal e os Conselhos Regionais terão (A) Coordenar, elaborar, executar, supervisionar e avaliar
legitimidade para agir contra qualquer pessoa que infringir as estudos, pesquisas, planos, programas e projetos na área de
disposições que digam respeito às prerrogativas, à dignidade Serviço Social.
e ao prestígio da profissão de Assistente Social. (B) Assumir, no magistério de Serviço Social tanto a nível
de graduação como pós-graduação, disciplinas e funções que
Art. 23. Esta lei entra em vigor na data de sua publicação. exijam conhecimentos próprios e adquiridos em curso de
formação regular.
Art. 24. Revogam-se as disposições em contrário e, em (C) Fiscalizar o exercício profissional através dos
especial, a Lei nº 3.252, de 27 de agosto de 1957. Conselhos Federal e Regionais.
(D) Realizar vistorias, perícias técnicas, laudos periciais,
Questões informações e pareceres sobre a matéria de Serviço Social.
(E) Planejamento, organização e administração de
01. (Prefeitura de Fortaleza – CE - Assistente Social - Serviços Sociais e de Unidade de Serviço Social.
Prefeitura de Fortaleza – CE/2016) De acordo com a Lei
Federal nº. 8.662/93, assinale a alternativa que se refere a uma Respostas
competência dos assistentes sociais.
(A) Assessoria e consultoria e órgãos das administrações 01. C. / 02. E. / 03. A. / 04. E.
públicas direta e indireta, empresas privadas e outras
entidades, em matéria de serviço social.
(B) Treinamento, avaliação e supervisão direta de CÓDIGO DE ÉTICA DO
estagiários de serviço social.
(C) Planejamento, organização e administração de serviços
PSICÓLOGO.
sociais e de unidade de serviço social.
(D) Realização de vistorias, perícias técnicas, laudos
Código de Ética Profissional do Psicólogo
periciais, informações e pareceres sobre a matéria de serviço
social.
RESOLUÇÃO CFP Nº 010/05
02. (UFMA - Assistente Social – UFMA/2016) A Lei
Aprova o Código de Ética Profissional do Psicólogo.
8.662/93, que regulamenta a profissão, estabelece,
respectivamente nos seus artigos 4º e 5º as competências e
O CONSELHO FEDERAL DE PSICOLOGIA, no uso de suas
atribuições privativas do Assistente Social. Marque apenas o
atribuições legais e regimentais, que lhe são conferidas pela
que se refere a atribuições privativas do Assistente Social.
Lei no 5.766, de 20 de dezembro de 1971;
(A) Coordenar, elaborar, executar e avaliar planos,
programas e projetos, que sejam de âmbito de atuação do
CONSIDERANDO o disposto no Art. 6º, letra “e”, da Lei no
Serviço Social com a participação da sociedade civil.
5.766 de 20/12/1971, e o Art. 6º, inciso VII, do Decreto nº
(B) Elaborar, implementar, executar e avaliar políticas
79.822 de 17/6/1977;
sociais junto a órgãos da administração direta ou indireta,
empresas, entidades e organizações populares.
CONSIDERANDO o disposto na Constituição Federal de
(C) Planejar, executar e avaliar pesquisas que possam
1988, conhecida como Constituição Cidadã, que consolida o
contribuir para análise da realidade social e para subsidiar
Estado Democrático de Direito e legislações dela decorrentes;
ações profissionais.
(D) Planejar, organizar e administrar benefícios sociais.
CONSIDERANDO decisão deste Plenário em reunião
(E) Coordenar, elaborar, executar, supervisionar e avaliar
realizada no dia 21 de julho de 2005;
estudos, pesquisas, planos, programas e projetos na área do
serviço social
RESOLVE:
03. (EBSERH - Assistente Social (HUAP-UFF) –
Art. 1º - Aprovar o Código de Ética Profissional do
IBFC/2016) Os itens:
Psicólogo.
VI. treinamento, avaliação e supervisão direta de
estagiários de Serviço Social;
Art. 2º - A presente Resolução entrará em vigor no dia 27
VII. dirigir e coordenar Unidades de Ensino e Cursos de
de agosto de 2005.
Serviço Social, de graduação e pós-graduação (BRASIL, 1993).
Art. 3º - Revogam-se as disposições em contrário, em
Foram extraídos da Lei 8.662/93, e constituem:
especial a Resolução CFP n º 002/87.
(A) Direitos do Assistente Social
(B) Deveres do Assistente Social
Apresentação
(C) Responsabilidades do Assistente Social
(D) Atribuições privativas do Assistente Social
Toda profissão define-se a partir de um corpo de práticas
(E) Competências do Assistente Social
que busca atender demandas sociais, norteado por elevados
padrões técnicos e pela existência de normas éticas que
04. (IF-BA - Assistente Social – FUNRIO/2016) De
garantam a adequada relação de cada profissional com seus
acordo com a Lei nº 8.662/93, é livre o exercício da profissão
pares e com a sociedade como um todo.
de assistente social aos possuidores de diploma em Serviço
Um Código de Ética profissional, ao estabelecer padrões
Social reconhecido oficialmente, sendo que o exercício da
esperados quanto às práticas referendadas pela respectiva
profissão de assistente social requer prévio registro nos
categoria profissional e pela sociedade, procura fomentar a
Conselhos Regionais. De acordo com a referida lei, são
autorreflexão exigida de cada indivíduo acerca da sua práxis,
atribuídas competências e atribuições privativas a estes
de modo a responsabilizá-lo, pessoal e coletivamente, por
profissionais, as quais podemos elencar como uma de suas
ações e suas consequências no exercício profissional. A missão
competências profissionais:
primordial de um código de ética profissional não é de
normatizar a natureza técnica do trabalho, e, sim, a de

Legislação 143
Apostila Digital Licenciada para Alice Caroline Guarino dos Santos - alice.guarino@hotmail.com (Proibida a Revenda)
APOSTILAS OPÇÃO

assegurar, dentro de valores relevantes para a sociedade e III. O psicólogo atuará com responsabilidade social,
para as práticas desenvolvidas, um padrão de conduta que analisando crítica e historicamente a realidade política,
fortaleça o reconhecimento social daquela categoria. Códigos econômica, social e cultural.
de Ética expressam sempre uma concepção de homem e de IV. O psicólogo atuará com responsabilidade, por meio do
sociedade que determina a direção das relações entre os contínuo aprimoramento profissional, contribuindo para o
indivíduos. Traduzem-se em princípios e normas que devem desenvolvimento da Psicologia como campo científico de
se pautar pelo respeito ao sujeito humano e seus direitos conhecimento e de prática.
fundamentais. Por constituir a expressão de valores V. O psicólogo contribuirá para promover a
universais, tais como os constantes na Declaração Universal universalização do acesso da população às informações, ao
dos Direitos Humanos; socioculturais, que refletem a realidade conhecimento da ciência psicológica, aos serviços e aos
do país; e de valores que estruturam uma profissão, um código padrões éticos da profissão.
de ética não pode ser visto como um conjunto fixo de normas VI. O psicólogo zelará para que o exercício profissional seja
e imutável no tempo. As sociedades mudam, as profissões efetuado com dignidade, rejeitando situações em que a
transformam-se e isso exige, também, uma reflexão contínua Psicologia esteja sendo aviltada.
sobre o próprio código de ética que nos orienta. VII. O psicólogo considerará as relações de poder nos
A formulação deste Código de Ética, o terceiro da profissão contextos em que atua e os impactos dessas relações sobre as
de psicólogo no Brasil, responde ao contexto organizativo dos suas atividades profissionais, posicionando-se de forma crítica
psicólogos, ao momento do país e ao estágio de e em consonância com os demais princípios deste Código.
desenvolvimento da Psicologia enquanto campo científico e
profissional. Este Código de Ética dos Psicólogos é reflexo da Das Responsabilidades do Psicólogo
necessidade, sentida pela categoria e suas entidades
representativas, de atender à evolução do contexto Art. 1º – São deveres fundamentais dos psicólogos:
institucional-legal do país, marcadamente a partir da
promulgação da denominada Constituição Cidadã, em 1988, e a) Conhecer, divulgar, cumprir e fazer cumprir este Código;
das legislações dela decorrentes. b) Assumir responsabilidades profissionais somente por
Consoante com a conjuntura democrática vigente, o atividades para as quais esteja capacitado pessoal, teórica e
presente Código foi construído a partir de múltiplos espaços tecnicamente;
de discussão sobre a ética da profissão, suas responsabilidades c) Prestar serviços psicológicos de qualidade, em
e compromissos com a promoção da cidadania. O processo condições de trabalho dignas e apropriadas à natureza desses
ocorreu ao longo de três anos, em todo o país, com a serviços, utilizando princípios, conhecimentos e técnicas
participação direta dos psicólogos e aberto à sociedade. reconhecidamente fundamentados na ciência psicológica, na
Este Código de Ética pautou-se pelo princípio geral de ética e na legislação profissional;
aproximar-se mais de um instrumento de reflexão do que de d) Prestar serviços profissionais em situações de
um conjunto de normas a serem seguidas pelo psicólogo. Para calamidade pública ou de emergência, sem visar benefício
tanto, na sua construção buscou-se: pessoal;
a. Valorizar os princípios fundamentais como grandes e) Estabelecer acordos de prestação de serviços que
eixos que devem orientar a relação do psicólogo com a respeitem os direitos do usuário ou beneficiário de serviços de
sociedade, a profissão, as entidades profissionais e a ciência, Psicologia;
pois esses eixos atravessam todas as práticas e estas f) Fornecer, a quem de direito, na prestação de serviços
demandam uma contínua reflexão sobre o contexto social e psicológicos, informações concernentes ao trabalho a ser
institucional. realizado e ao seu objetivo profissional;
b. Abrir espaço para a discussão, pelo psicólogo, dos limites g) Informar, a quem de direito, os resultados decorrentes
e interseções relativos aos direitos individuais e coletivos, da prestação de serviços psicológicos, transmitindo somente o
questão crucial para as relações que estabelece com a que for necessário para a tomada de decisões que afetem o
sociedade, os colegas de profissão e os usuários ou usuário ou beneficiário;
beneficiários dos seus serviços. h) Orientar a quem de direito sobre os encaminhamentos
c. Contemplar a diversidade que configura o exercício da apropriados, a partir da prestação de serviços psicológicos, e
profissão e a crescente inserção do psicólogo em contextos fornecer, sempre que solicitado, os documentos pertinentes ao
institucionais e em equipes multiprofissionais. bom termo do trabalho;
d. Estimular reflexões que considerem a profissão como i) Zelar para que a comercialização, aquisição, doação,
um todo e não em suas práticas particulares, uma vez que os empréstimo, guarda e forma de divulgação do material
principais dilemas éticos não se restringem a práticas privativo do psicólogo sejam feitas conforme os princípios
específicas e surgem em quaisquer contextos de atuação. deste Código;
Ao aprovar e divulgar o Código de Ética Profissional do j) Ter, para com o trabalho dos psicólogos e de outros
Psicólogo, a expectativa é de que ele seja um instrumento profissionais, respeito, consideração e solidariedade, e,
capaz de delinear para a sociedade as responsabilidades e quando solicitado, colaborar com estes, salvo impedimento
deveres do psicólogo, oferecer diretrizes para a sua formação por motivo relevante;
e balizar os julgamentos das suas ações, contribuindo para o k) Sugerir serviços de outros psicólogos, sempre que, por
fortalecimento e ampliação do significado social da profissão. motivos justificáveis, não puderem ser continuados pelo
profissional que os assumiu inicialmente, fornecendo ao seu
Princípios Fundamentais substituto as informações necessárias à continuidade do
trabalho;
I. O psicólogo baseará o seu trabalho no respeito e na l) Levar ao conhecimento das instâncias competentes o
promoção da liberdade, da dignidade, da igualdade e da exercício ilegal ou irregular da profissão, transgressões a
integridade do ser humano, apoiado nos valores que embasam princípios e diretrizes deste Código ou da legislação
a Declaração Universal dos Direitos Humanos. profissional.
II. O psicólogo trabalhará visando promover a saúde e a
qualidade de vida das pessoas e das coletividades e contribuirá
para a eliminação de quaisquer formas de negligência,
discriminação, exploração, violência, crueldade e opressão.

Legislação 144
Apostila Digital Licenciada para Alice Caroline Guarino dos Santos - alice.guarino@hotmail.com (Proibida a Revenda)
APOSTILAS OPÇÃO

Art. 2º – Ao psicólogo é vedado: b) Estipulará o valor de acordo com as características da


atividade e o comunicará ao usuário ou beneficiário antes do
a) Praticar ou ser conivente com quaisquer atos que início do trabalho a ser realizado;
caracterizem negligência, discriminação, exploração, c) Assegurará a qualidade dos serviços oferecidos
violência, crueldade ou opressão; independentemente do valor acordado.
b) Induzir a convicções políticas, filosóficas, morais,
ideológicas, religiosas, de orientação sexual ou a qualquer tipo Art. 5º – O psicólogo, quando participar de greves ou
de preconceito, quando do exercício de suas funções paralisações, garantirá que:
profissionais; a) As atividades de emergência não sejam interrompidas;
c) Utilizar ou favorecer o uso de conhecimento e a b) Haja prévia comunicação da paralisação aos usuários ou
utilização de práticas psicológicas como instrumentos de beneficiários dos serviços atingidos pela mesma.
castigo, tortura ou qualquer forma de violência;
d) Acumpliciar-se com pessoas ou organizações que Art. 6º – O psicólogo, no relacionamento com profissionais
exerçam ou favoreçam o exercício ilegal da profissão de não psicólogos:
psicólogo ou de qualquer outra atividade profissional; a) Encaminhará a profissionais ou entidades habilitados e
e) Ser conivente com erros, faltas éticas, violação de qualificados demandas que extrapolem seu campo de atuação;
direitos, crimes ou contravenções penais praticados por b) Compartilhará somente informações relevantes para
psicólogos na prestação de serviços profissionais; qualificar o serviço prestado, resguardando o caráter
f) Prestar serviços ou vincular o título de psicólogo a confidencial das comunicações, assinalando a
serviços de atendimento psicológico cujos procedimentos, responsabilidade, de quem as receber, de preservar o sigilo.
técnicas e meios não estejam regulamentados ou reconhecidos
pela profissão; Art. 7º – O psicólogo poderá intervir na prestação de
g) Emitir documentos sem fundamentação e qualidade serviços psicológicos que estejam sendo efetuados por outro
técnico-científica; profissional, nas seguintes situações:
h) Interferir na validade e fidedignidade de instrumentos e
técnicas psicológicas, adulterar seus resultados ou fazer a) A pedido do profissional responsável pelo serviço;
declarações falsas; b) Em caso de emergência ou risco ao beneficiário ou
i) Induzir qualquer pessoa ou organização a recorrer a seus usuário do serviço, quando dará imediata ciência ao
serviços; profissional;
j) Estabelecer com a pessoa atendida, familiar ou terceiro, c) Quando informado expressamente, por qualquer uma
que tenha vínculo com o atendido, relação que possa interferir das partes, da interrupção voluntária e definitiva do serviço;
negativamente nos objetivos do serviço prestado; d) Quando se tratar de trabalho multiprofissional e a
k) Ser perito, avaliador ou parecerista em situações nas intervenção fizer parte da metodologia adotada.
quais seus vínculos pessoais ou profissionais, atuais ou
anteriores, possam afetar a qualidade do trabalho a ser Art. 8º – Para realizar atendimento não eventual de
realizado ou a fidelidade aos resultados da avaliação; criança, adolescente ou interdito, o psicólogo deverá obter
l) Desviar para serviço particular ou de outra instituição, autorização de ao menos um de seus responsáveis, observadas
visando benefício próprio, pessoas ou organizações atendidas as determinações da legislação vigente:
por instituição com a qual mantenha qualquer tipo de vínculo §1° – No caso de não se apresentar um responsável legal, o
profissional; atendimento deverá ser efetuado e comunicado às autoridades
m) Prestar serviços profissionais a organizações competentes;
concorrentes de modo que possam resultar em prejuízo para §2° – O psicólogo responsabilizar-se-á pelos
as partes envolvidas, decorrentes de informações encaminhamentos que se fizerem necessários para garantir a
privilegiadas; proteção integral do atendido.
n) Prolongar, desnecessariamente, a prestação de serviços
profissionais; Art. 9º – É dever do psicólogo respeitar o sigilo profissional
o) Pleitear ou receber comissões, empréstimos, doações ou a fim de proteger, por meio da confidencialidade, a intimidade
vantagens outras de qualquer espécie, além dos honorários das pessoas, grupos ou organizações, a que tenha acesso no
contratados, assim como intermediar transações financeiras; exercício profissional.
p) Receber, pagar remuneração ou porcentagem por
encaminhamento de serviços; Art. 10° – Nas situações em que se configure conflito entre
q) Realizar diagnósticos, divulgar procedimentos ou as exigências decorrentes do disposto no Art. 9º e as
apresentar resultados de serviços psicológicos em meios de afirmações dos princípios fundamentais deste Código,
comunicação, de forma a expor pessoas, grupos ou excetuando-se os casos previstos em lei, o psicólogo poderá
organizações. decidir pela quebra de sigilo, baseando sua decisão na busca
do menor prejuízo.
Art. 3º – O psicólogo, para ingressar, associar-se ou Parágrafo único – Em caso de quebra do sigilo previsto no
permanecer em uma organização, considerará a missão, a caput deste artigo, o psicólogo deverá restringir-se a prestar
filosofia, as políticas, as normas e as práticas nela vigentes e as informações estritamente necessárias.
sua compatibilidade com os princípios e regras deste Código.
Parágrafo único: Existindo incompatibilidade, cabe ao Art. 11° – Quando requisitado a depor em juízo, o psicólogo
psicólogo recusar-se a prestar serviços e, se pertinente, poderá prestar informações, considerando o previsto neste
apresentar denúncia ao órgão competente. Código.

Art. 4º – Ao fixar a remuneração pelo seu trabalho, o Art. 12° – Nos documentos que embasam as atividades em
psicólogo: equipe multiprofissional, o psicólogo registrará apenas as
informações necessárias para o cumprimento dos objetivos do
a) Levará em conta a justa retribuição aos serviços trabalho.
prestados e as condições do usuário ou beneficiário;

Legislação 145
Apostila Digital Licenciada para Alice Caroline Guarino dos Santos - alice.guarino@hotmail.com (Proibida a Revenda)
APOSTILAS OPÇÃO

Art. 13° – No atendimento à criança, ao adolescente ou ao h) Não fará divulgação sensacionalista das atividades
interdito, deve ser comunicado aos responsáveis o profissionais.
estritamente essencial para se promoverem medidas em seu
benefício. Das Disposições Gerais

Art. 14° – A utilização de quaisquer meios de registro e Art. 21° – As transgressões dos preceitos deste Código
observação da prática psicológica obedecerá às normas deste constituem infração disciplinar com a aplicação das seguintes
Código e a legislação profissional vigente, devendo o usuário penalidades, na forma dos dispositivos legais ou regimentais:
ou beneficiário, desde o início, ser informado.
a) Advertência;
Art. 15° – Em caso de interrupção do trabalho do psicólogo, b) Multa;
por quaisquer motivos, ele deverá zelar pelo destino dos seus c) Censura pública;
arquivos confidenciais. d) Suspensão do exercício profissional, por até 30 (trinta)
§ 1° – Em caso de demissão ou exoneração, o psicólogo dias, ad referendum do Conselho Federal de Psicologia;
deverá repassar todo o material ao psicólogo que vier a e) Cassação do exercício profissional, ad referendum do
substituí-lo, ou lacrá-lo para posterior utilização pelo Conselho Federal de Psicologia.
psicólogo substituto.
§ 2° – Em caso de extinção do serviço de Psicologia, o Art. 22° – As dúvidas na observância deste Código e os
psicólogo responsável informará ao Conselho Regional de casos omissos serão resolvidos pelos Conselhos Regionais de
Psicologia, que providenciará a destinação dos arquivos Psicologia, ad referendum do Conselho Federal de Psicologia.
confidenciais.
Art. 23° – Competirá ao Conselho Federal de Psicologia
Art. 16° – O psicólogo, na realização de estudos, pesquisas firmar jurisprudência quanto aos casos omissos e fazê-la
e atividades voltadas para a produção de conhecimento e incorporar a este Código.
desenvolvimento de tecnologias:
Art. 24° – O presente Código poderá ser alterado pelo
a) Avaliará os riscos envolvidos, tanto pelos Conselho Federal de Psicologia, por iniciativa própria ou da
procedimentos, como pela divulgação dos resultados, com o categoria, ouvidos os Conselhos Regionais de Psicologia.
objetivo de proteger as pessoas, grupos, organizações e
comunidades envolvidas; Art. 25°– Este Código entra em vigor em 27 de agosto de
b) Garantirá o caráter voluntário da participação dos 2005.
envolvidos, mediante consentimento livre e esclarecido, salvo
nas situações previstas em legislação específica e respeitando Referencia
os princípios deste Código;
c) Garantirá o anonimato das pessoas, grupos ou Código de Ética Profissional do Psicólogo produzido pelo
organizações, salvo interesse manifesto destes; Conselho Federal de Psicologia. Disponível em
d) Garantirá o acesso das pessoas, grupos ou organizações http://site.cfp.org.br/wp-
aos resultados das pesquisas ou estudos, após seu content/uploads/2012/07/Co%CC%81digo-de-
encerramento, sempre que assim o desejarem. %C3%89tica.pdf

Art. 17° – Caberá aos psicólogos docentes ou supervisores Questões


esclarecer, informar, orientar e exigir dos estudantes a
observância dos princípios e normas contidas neste Código. 01. (SESA-ES-Psicologia- CESPE) Assinale a opção
correta referente ao Código de Ética Profissional dos
Art. 18° – O psicólogo não divulgará, ensinará, cederá, Psicólogos.
emprestará ou venderá a leigos instrumentos e técnicas (A) Constitui princípio fundamental estabelecido pelo
psicológicas que permitam ou facilitem o exercício ilegal da Código de Ética Profissional dos Psicólogos a promoção de
profissão. saúde e de qualidade de vida dos indivíduos e das
coletividades, devendo o psicólogo ser neutro em quaisquer
Art. 19° – O psicólogo, ao participar de atividade em formas de violência e de discriminação.
veículos de comunicação, zelará para que as informações (B) O profissional da psicologia deverá atuar com
prestadas disseminem o conhecimento a respeito das responsabilidade social e econômica, estando a par dos
atribuições, da base científica e do papel social da profissão. contextos históricos e sociais. Cabe ao profissional ainda a
análise crítica das realidades social, cultural e individual na
Art. 20° – O psicólogo, ao promover publicamente seus tomada de decisão.
serviços, por quaisquer meios, individual ou coletivamente: (C) O psicólogo deverá atuar com responsabilidade no
a) Informará o seu nome completo, o CRP e seu número de desenvolvimento da psicologia como campo científico de
registro; conhecimento e de prática.
b) Fará referência apenas a títulos ou qualificações (D) O profissional da psicologia deverá promover a
profissionais que possua; universalização do acesso da população às informações e
c) Divulgará somente qualificações, atividades e recursos aos serviços, não cabendo sua contribuição no que tange ao
relativos a técnicas e práticas que estejam reconhecidas ou conhecimento das especificidades da ciência psicológica.
regulamentadas pela profissão; (E) Constitui princípio fundamental estabelecido pelo
d) Não utilizará o preço do serviço como forma de Código de Ética Profissional dos Psicólogos o trabalho
propaganda; fundamentado no respeito e na promoção da liberdade, da
e) Não fará previsão taxativa de resultados; dignidade, da igualdade e da integridade do ser humano,
f) Não fará autopromoção em detrimento de outros apoiado nos valores que embasam o artigo 5o da
profissionais; Constituição Federal.
g) Não proporá atividades que sejam atribuições privativas
de outras categorias profissionais;

Legislação 146
Apostila Digital Licenciada para Alice Caroline Guarino dos Santos - alice.guarino@hotmail.com (Proibida a Revenda)
APOSTILAS OPÇÃO

02. (AL-MT- Psicólogo-FGV) De acordo com o Código de (B) levar ao conhecimento das instâncias competentes o
Ética do Psicólogo, indique a conduta adequada. exercício ilegal ou irregular da profissão, transgressões a
(A) Após a entrevista de triagem, é permitido ao psicólogo princípios e diretrizes do Código de Ética ou da legislação
sugerir o encaminhamento de paciente para outra instituição profissional;
em que trabalhe, desde que de comum acordo com o paciente. (C) pleitear ou receber comissões, empréstimos, doações
(B) Um psicólogo iniciou o trabalho, acertando um valor ou vantagens outras de qualquer espécie, além dos honorários
que considerou justo e que acordou com o paciente. Ao ter contratados, assim como intermediar transações financeiras;
mais detalhes sobre a situação financeira do paciente, decidiu (D) estipular o valor da remuneração de acordo com as
cobrar mais pelas sessões do que o previamente acordado. características de sua atividade e comunicá-lo ao usuário ou
(C) Durante uma greve dos funcionários, os profissionais beneficiário antes do início do trabalho a ser realizado;
de psicologia decidiram manter os atendimentos emergenciais (E) intervir na prestação de serviços psicológicos que
e avisar aos outros pacientes da interrupção do atendimento estejam sendo efetuados por outro profissional, numa situação
por um determinado período. de trabalho multiprofissional em que a intervenção faz parte
(D) Numa situação emergencial, os psicólogos convocados da metodologia adotada.
para ajudar os moradores que perderam suas casas, se
recusaram a trabalhar ou disseram que só trabalhariam se Respostas
fosse pago um adicional pelos serviços prestados.
(E) Um psicólogo foi solicitado pelo gerente de uma 01. C / 02. C / 03. B / 04. C / 05. C
empresa a administrar um curso de capacitação para
funcionários administrativos que iriam aplicar testes em um
processo seletivo.

03. (TRT 12° região-Analista Judiciário- FCC) Acerca do Anotações


Código de Ética Profissional do Psicólogo, é INCORRETO
afirmar que o psicólogo
(A) contribuirá para promover a universalização do acesso
da população às informações, ao conhecimento da ciência
psicológica, aos serviços e aos padrões éticos da profissão.
(B) zelará para que o exercício profissional seja efetuado
com austeridade, mesmo quando levado a tolerar e aceitar
situações em que a Psicologia esteja sendo aviltada.
(C) atuará com responsabilidade, por meio do contínuo
aprimoramento profissional, contribuindo para o
desenvolvimento da Psicologia como campo científico de
conhecimento e de prática.
(D) trabalhará visando a promover a saúde e a qualidade
de vida das pessoas e das coletividades e contribuirá para a
eliminação de quaisquer formas de negligência, discriminação,
exploração, violência, crueldade e opressão.
(E) atuará com responsabilidade social, analisando crítica
e historicamente a realidade política, econômica, social e
cultural.

04. (Prefeitura de Paulista- PE- Psicólogo-


UPENET/2014) Segundo o Código de Ética do Psicólogo, no
que se refere aos seus deveres fundamentais, assinale a
alternativa INCORRETA.
(A) Ter, para com o trabalho dos psicólogos e de outros
profissionais, respeito, consideração e solidariedade, e,
quando solicitado, colaborar com estes, salvo impedimento
por motivo relevante
(B) Sugerir serviços de outros psicólogos, sempre que, por
motivos justificáveis, não puderem ser continuados pelo
profissional que os assumiu inicialmente, fornecendo ao seu
substituto as informações necessárias à continuidade do
trabalho.
(C) Prestar serviços profissionais em qualquer situação de
solicitação por órgão público – Municipal, Estadual ou Federal
- sem visar benefício pessoal.
(D) Estabelecer acordos de prestação de serviços que
respeitem os direitos do usuário ou beneficiário de serviços de
Psicologia.
(E) Assumir responsabilidades profissionais somente por
atividades para as quais esteja capacitado pessoal, teórica e
tecnicamente.

05. (FUNARTE-Psicólogo- FGV/2014) O psicólogo


comete infração ao Código de Ética na seguinte situação:
(A) prestar serviço profissional em situação de calamidade
pública ou emergência, sem visar benefício pessoal;

Legislação 147
Apostila Digital Licenciada para Alice Caroline Guarino dos Santos - alice.guarino@hotmail.com (Proibida a Revenda)
APOSTILAS OPÇÃO

Legislação 148
Apostila Digital Licenciada para Alice Caroline Guarino dos Santos - alice.guarino@hotmail.com (Proibida a Revenda)

Você também pode gostar